Download as pdf or txt
Download as pdf or txt
You are on page 1of 677

TEXTBOOK~OF

BIOCHEM STRY
for Me dic al Students
..

SECTION A

Chemical Basis of Life 1

Chapter 1 Biochemical Perspective to Medicine


Chapter 2 Subcellular Organelles and Cell Membranes
Chapter 3 Amino Acids: Structure and Properties
Chapter4 Proteins: Structure and Function
Chapter 5 Enzymology: General Concepts and Enzyme Kinetics
Chapter 6 Clinical Enzymology
Chapter 7 Chemistry of Carbohydrates
Chapter 8 Chemistry of Lipids

_ _ _ _ _Chapter 1

Biochemical
Perspective to Medicine

Chapter at a Glance
The learner will be able to answer questions on the following topics:
0 History of biochemistry 0 Hydrophobic interactions
0 Ionic bonds 0 Principles of thermodynamics
0 Hydrogen bonding 0 Donnan membrane equilibrium

Biochemistry is the language of biology. The tools for


research in all the branches of medical science are
mainly biochemical in nature. The study of biochemistry
is essential to understand basic functions of the body.
This study will give information regarding the functioning
of cells at the molecular level. How the food that we eat
Hippocrates Charaka Sushruta
is digested , absorbed, and used to make ingredients 460-377 BC 400BC 500ac
of the body? How does the body derive energy for the
normal day-to-day work? How are the various metabolic Charaka, the great master of Indian Medicine, in his
processes interrelated? What is the function of genes? treatise (circa 400 BC) observed that madhumeha
What is the molecular basis for immunological resistance (diabetes mellitus) is produced by the alterations in the
against invading organisms? Answer for such basic metabolism of carbohydrates and fats; the statement
questions can only be derived by a systematic study of still holds good.
biochemistry. Biochemistry has developed as an offshoot of
It is estimated that about 70% of clinical diagnosis is organic chemistry, and this branch was often referred as

based on the laboratory analysis of body fluids, especially "physiological chemistry". The term "Biochemistry" was
the blood . The disease manifestations are reflected in coined by Neuberg in 1903 from Greek words, bios (= life)
the composition of blood and other tissues. Hence, the and chymos (= juice). One of the earliest treatises in
demarcation of abnormal from normal constituents of biochemistry was the "Book of Oq~anic Chemistry and
the body is another aim of the study of biochemistry. its Applications to Physiology and Pathology", published
The word chemistry is derived from the Greek word in 1842 by Justus von Liebig (1803-- 73), who introduced
"chemi" (the black land}, the ancient name of Egypt. the concept of metabolism. The "Textbook of Physio-
Indian medical science, even from ancient times, had logical Chemistry" was published in 1877 by Felix Hoppe-
identified the metabolic and genetic basis of diseases. Seyler (1825-95), who was Professor of Physiological
4 Section A: Chemical Basis of Life

TABLE 1.1: Milestones in history of B1ochem1stry


Scientists Year Landmark discoveries
Rouelle 1773 Isolated urea from urine
Lavoisier 1785 Oxidation of food stuffs
Wohler 1828 Synthesis of urea
Berzelius 1835 Enzyme catalysis theory Lavoisier Berzelius Friedrich Justus von
Louis Pasteur 1860 Fermentation process 1743-1794 1779-1848 Wohler Liebig
Edward Buchner 1897 Extracted enzymes 1800-1882 1803-1873
Fiske and Subbarow 1926 Isolated ATP from muscle
Lohmann 1932 Creatine phosphate
Hans Krebs 1937 Citric acid cycle
Avery and Macleod 1944 DNA is genetic material
lehninger 1950 TCA cycle in mitochondria
Watson and Crick 1953 Structure of DNA
Louis Pasteur Johannes van Albert
Frederick
Nirenberg 1961 Genetic code in mRNA G Donnan 1822-1895 derWaa/s Lehninger
Holley 1963 Sequenced gene for tRNA 1870-1956 NP 1910, 1917-1 986
1965 Synthesized the gene
1837-1923
Khorana
Paul Berg 1972 Recombinant DNA technology therapy. Many genes, (e.g. human insulin gene) have
Kary Mullis 1985 Polymerase chain reaction already been transferred to microorganisms for large-
1990 Human genome project started scale production of human insulin. Advances in genomics
2000 Draft human genome like RNA interference for silencing of genes and crea- -;
2003 Human genome project completed tion of transgenic animals by gene targeting of embry-
ENCODE 2012 ENCyclopedia of DNA Elements, onic stem cells are opening up new vistas in therapy of
diseases like cancer and AIDS. It is hoped that in futu re,
Chemistry at Strasbourg University, France. Some of the physician will be able to treat the patient, after under-
the milestones in the development of the science of standing his genetic basis, so that very efficient "designer
medicine" could cure the diseases. The large amount of
biochemistry are given in Table 1.1.
data, especially with regard to single nucleotide polymor-
The practice of medicine is both an art and a
phisms (SNPs) that are available, could be harnessed by
science. The word "doctor" is derived from the Latin root,
"Bioinformatics". Computers are already helping in drug
"docere", which means "to teach". Knowledge devoid
designing process. Studies on oncogenes have identi-
of ethical background may sometimes be disastrous!
fied molecular mechanisms of control of normal and
Hippocrates (460 BC to 377 BC), the father of modern
abnormal cells. Medical practice is now depending more
medicine articulated "the Oath". About one century earlier,
on the science of Medical Biochemistry. With the help of
Sushruta (?500 BC), the great Indian surgeon, enuncia-
Human genome project (HGP) the sequences of whole
ted a code of conduct for the medical practitioners, which
human genes are now available; it has already made great
is still valid. He proclaims: "You must speak only truth;
t
, rJ. ca~ l or the good of all living beings; devote yourself to
impact on medicine and related health sciences.

d. ¥°healing of the sick even if your life be lost by your


ork; be simply clothed and drink no intoxicant; always BIOMOLECULES
seek to grow in knowledge; in face of God, you can take More than 99~ ,ff the hum~ odyri}~...qomP.R_~d of
upon yourself these vows." 6elem~ i.e.'o~YQen9?a~ on, hydrogeri?ni\ro ge~ i& m
Biochemistry is perhaps the most rapidly develop- and pJt\5sphorus. uman bod is composed of about
j '-' 07
ing discipline in medicine. No wonder, the major share of 60% w :ter,J.J,% p teins, 15% 1pids, 2% carbohydrates
Nobel Prizes in medicine has gone to research workers and 8% mirWrals. Molecular structures in organisms are
engaged in biochemistry. Thanks to the advent of DNA built from 30 small precursors, sometimes called the
recombinant technology, genes can now be transferre? alphabets of biochemist%_ These ar I o amino acids,
from one person to another, so that many of the genet1- purine '3 pyrimidine~ ugars (glucose and ribose),
cally determined diseases are now amenable to gene@ palmitat lycerol an@:holine.
Chapter 1: Biochemical Perspective to Medicine 5

0~G)
TABLE 1.2: Bacterial and mammalian cells
Prokaryotic cell Eukaryotif_ ceJL __
.,
Size Small Large; 1doo to 10,000 times
Cell wall
!
Rigid Membrane of lipid bilayer V Transf13r of
Nucleus Not defined Well defined electrons

•.
- •------..
l
Organelles Nil Several; including mitochondria
and ~ samP<

0 Atom
,0 Positive ion

Fig. 1.2: Covalent bond


Negativ,e ion
-

Sharingof
electrons
Y is called digestion or primary metab~ . After absorp-
tion, the small mole~s are further broken down and
oxidized to carbon dioxide. In this process, NADH or
FADH2 are generated. This is named as secondary or
intermediary metabolism. Finally, these reducing
equivalents enter the electron trans1Port chain in the mi-
Molecule
tochondria, where they are oxidized to water; in this pro-
Fig. 1.1: ~
cess energy is trapped as ATP. This is termed tertiary

In living organisms, biomolecules are ordered into


metabolism. Metabolism is the sum of all chemical
'
changes of a cornp_ound inside the body, which includes
-
a hierarchy of increasing molecular complexity. These
syn~ is (anabolism) and breajltJown (catabolism).
biomolecules are covalently linked to each other to form (Greek word , kata = down; balleirY'= change).
macromolecules of the cell, e.g. glucose to glycogen,
amino ac· s to prot~ ns, etc. Major c~ plex bio~ lecules STABl[IZfNG FORCES
are proteins, polRaccharides, lipidsand nuc1efd acids.
IN MOLECUlES
The macromolecules associate with each other by
noncovalent forces to form (iipramolecular syst@ s, ovalent Bonds
e.g. ribosomes, lipoproteins. Molecules are formed by sharing o1f electrons between
Finally at the highest level of organization in the atoms (Fig. 1.1).
hierarchy of cell structure, various supramolecular comp-
lexes are further assembled into@ U organ~ . In pr~
Ionic Bonds or Electrosta1tic Bonds
karyotes (e.g. bacteria; Greek word "pro"= before; karyon Ionic bonds result from the electrostatic attraction
= nucleus), these macromolecules are seen in a~ e- between two ionized groups of opposite charges
n~us matrix: but in eukaryotic cells (e.g. higher organisms; (Fig.1.2). They are formed by transfer of one or more
Greek word "eu" = true), the cytoplasm contains various electrons from the outermost orbit of an electropositive
subcellular organelles. Comparison of prokaryotes and atom to the outermost orbit of an el1ectronegative atom.
eukaryotes are shown in Table 1.2. This transfer results in the formation of a 'cation' and an
'anion', which get consequently bound by an ionic bond.
STUD OF METABOLIC Common examples of such compounds include NaCl,
KBr and NaF.
PROCESSES
With regard to protein chemistry,(posjtjye charges)
Our food contains carbohydrates, fats and proteins as are produced by ei:,silon amino gI.Qup of lysine, guani-rt-tf\).__l
principal ingredients. These macromolecules are to dinium group of arginine and imidazolium group off.. l j
be first broken down to small unit~ rbohydrates to histidine.~ egative charge~ are provided by aillL
monosaccharides an roteins to amino acids. This gamma carboxyl grouP§_ of aspartic acid and glutamic
process is taking place in the gastrointestinal tract and acid (Fig.1.3).
6 Section A: Chemical Basis of Life

Substrate
{A) Two hydrophobic
molecules surrounded
by water molecules

\
I
Enzyme (

----
Fig. 1.3: Ionic bonds used in p: o~ teractions
{B) When two
hydrophobic molecules
come together,
surrounding water
molecules are minimal
@ Hydrogen Bonds
These are formed by sharing of a hydrogen between
two electron donors. Hydrogen bonds result from elec- g . ·.4: Hydrophobic interaction
F~
trostatic attraction between an electronegative a\om and 1. . h . . t t
. attr ct1ve forces between c emIca1 groups in con ac .
a hydrogen atom that is anded covalently to a secon~ This force will drastically reduce, when the distance
electrol ~ive atom. a hydrogen atom forms . .
u-\.-\ 1..-c h H
a "CQ\Tal t 1ond with on Iy one ot er atom. owever,
between atoms Is increased. Although very weak, van der
. . .
--,,- Waals forces colh~ct1vely contribute maximum towards
hydrogen atom coyale~tl)wndedto a donor atom, mly
';/:==~
- =~~~r~~~=-=~;f.=~~~=,=:::;::: the stability of proltein structure, especially in preserving
_...... form an o ti , the hydrogen bo d
1 the nonpolar interior structure of proteins.
with an acceptor atom. In biological systems, both donors
\ and acceptors are usually n~ gen or oxJ,ife n atf m~ .
especially those atoms in amino (NH 2) and ht oxyl
WATER: THE UNIVERSAL SOLVENT
(OH) groups. Water constitutes :about 70 to 80 percent of the weight of
With regard to protein chemistry, hydrogen rel asing most cells. The hydrogen atom in one water molecule is
groups are -NH (imidazole, in dole, peptide); -OH ~ erine, attracted to a pair oIfelectrons in the outer shell of an oxygen
threonine) and -NH2 (arginine, lysine). Hydrogen accep- atom in an adjacent molecule. The structure of liquid water
ting groups are COO-(aspartic, glutamic) C=O (peptide); contains hydrogen-bonded networks (Fig . 1.5).
and S- S (disulphide). The DNA structure js roaiotaioed water is a polar molecule. Molecules with polar bonds
the
by hydrogen bonding between
residues. J
purine a1pyrimidine that can easily fornn hydrogen bonds wit~~ ~~r and can
dissolve in water are termed "hJ'.drophJ!$"ift11as immense

© Hydrophobic Interactions
Nonpolar groups have a tendency to associate with
hydrogen bondin~, capacity both with other molecules
and also the adjacent water molecules. This contributes
to cohesiveness o1'water. Water favors hydrophobic inter-
each other in an aqueous environment; this is referred actions and provides a basis for metabolism of insoluble
to as hydrophobic interaction . These are formed by substances.
interactions between QOnpolar hydrophobic side chains
by eliminating water mo lecules. The force that causes PRINCIPLES OF
hydrophobic molecules or nonpolar portions of molecules
to aggregate together rather than to dissolve in water
THERMODYNAMICS
is called the [ hydrophobicboiio} (Fig.1.4). This serves Thermodynamics is concerned with the flow of heat and
to hold lipophilic side chains of amino acids together. it deals with the relationship between heat and work.
Thus nonpolar molecules will have minimum exposure Bloenergetics, o r biochemical thermodynamics, is the
to water molecules. study of the energy changes accompanying biochemical
reactions. Biologi1cal systems use chemical energy to
Van der Waals Forces ...L
0 (, drive processes within living cells.
These are very weak forces of attraction between all
atoms, due to oscillating dipoles, described by the Dutch First Law of Thermodynamics
physicist Johannes van der Waals (1837- 1923). He was The total energy of a system, including its surround-
awarded Nobel Prize in 1910. These are short range ings, remains constant. Or, t.E = Q - W, where Q is the
Chapter 1: Biochemical Perspiective to Medicine 7

kj.~,k'
denotes a portion of the tbtal energy change in a
system that is available for doing work.

y_., vi·i'rr
- Hydrogen bond
For most biochemical reactions ,. \is seen that t.H is
nearly equal to t.E. So,ft.G = t.E - T ~YHence, t.G or free
energy of a system depends on the change in internal

J "'"' '"
energy and change in entropy of a system.
A{:Ai;J;?
Standard Free Energy Change
It is the free energy change under standard conditions.
It is designated as t.G0 • The standard conditions are
defined for biochemical reactions at a pH of 7 and 1 M
Fig. 1.5: Water molecules hydrogen bonded conce~OAraAEkl#ferentiated-by-;!!t"~ ming sign t.G0 '.
·11 is d ~eA,1{ f latec!J'.ia,'ijie.,,equitlbr~yn__ponstant. Actual
heat absorbed by the system and W is the work done. free e ~ b aA9es dei,ene-o~ ~tant and product.
Thi s is also called the law of conservation of energy. Most of the reversible metabolic reactions are near-
If heat is transformed into work, there is proportionality equilibrium reactions and therefore their t.G is nearly zero.
between the work obtained and the heat dissipated. The net rate of near-equilibrium reactions are effectively
A system is an object or a quantity of matter, chosen for regulated by the relative concentr,ation of substrates
observation. All other parts of the universe, outside the and products. The metabolic reactions that function far
boundary of the system, are called the surrounding. from equilibrium are irreversible. The velocities of these
reactions are altered by changes in1 enzyme activity. A
Second Law of Ttiermodynamics highly exer!;lonic reaction is irreversible and goes to com-
The total entropy of a system must increase if a pletion. Such a reaction that is part of a metabolic path-
process is to occur spontaneously. A reaction occurs way, confe f direction to the pathway and makes the entire
spontaneously if t.E is negative, or if the entropy of the pathwa irreversible
system increases. Entropy (S) is a measure of the degree Laws of thermodynamic have many applications in
of randomness or disorder of a system. Entropy becomes biology and biochemistry, such as study ~ TP hydro-
maximum in a system as it approaches true equilibrium. lysis@,~m9rane diffusion zyme catalysis as well as
Enthalpy is the heat content of a system and entropy A binding a@ !:9tein stabi!ity'. These laws have been
~ is that fraction of enthalpy w hich is not available to do used to explain hypothesis of origin of life.
\.!Y useful work. I
, - [ ~ l;-:o:-:s:-:e-::;
d;-s::'.y:--:-s:-:t~e-=
m=-=a--=p--=p-ro
_a_c
- ;h~e- s_ a_s
_ t,...a_,.e
- o-,-e_q_u
...,i"", Three Trpes of Reactions
~ Any system can spontaneously proceed from a A. A reaction can occur spontanHously when t.G is
state of low probability (ordered state) to a state of high negativa.. Then the- reactionJ xergonic. If t.G is
probability (disordered state). The entropy of a system of great magnitude, the reaction goes to completion
may decrease with an increase in that of the surround- and is essentially irreversible.
ings . The second law may be expressed in simple terms B. When t.G is zero, the system is at equilibrium.
as Q = T x t.S , where Q is the heat absorbed, Tis the C. For reactions where t.G is positive, an input of energy
absolute temperature and t.S is the change in entropy. is required to drive the reaction. Tl~e reaction is termed
as endergonic (Examples are gi1ven in Chapter 5).
Gibbs Free Energy Concept Similarly a reaction may be exot1 ennic (t.H is nega-
tive), isothennic (t.H is zero) or endothe ic (t.H is positive).
The term free energy is used to get an equation combin- Energetically unfavorable reaction may be driven
ing the first and second laws of thermodynamics. Thus, forward by coupling it with a favorablle reaction.
t.G = t.H - Tt.S, where t.G is the change in free energy, Glucose + Pi --. Glucose-6-phosphate (reaction 1)
t.H is the change in enthalpy or heat content of the system ATP + H O --. ADP + Pi (reaction 2)
and t.S is the change in entropy. The term free energy Glucose+ ATP-. Glucose-6-phosphate+ADP (3)

l 9\ b_~
8 Section A: Chemical Basis of Life

Reaction 1 cannot proceed spontaneously. But the


Left Right Left Right I
2nd reaction is coupled in the body, so that the reaction
becomes possible. For the first reaction, 6 G 0 is +13.8 (5)Na• (10)Na• (9)NaI• (6)Na'
kJ/mole; for the second reaction, 6 G0 is - 30.5 kJ/mole. (5)R-
(5)R- (10)CI. (6)c1·
When the two reactions are coupled in the reaction 3, (4)Cf"
the 6G 0 becomes -16.7 kJ/mole, and hence the reaction
A B
becomes possible. Details on ATP and other high-energy
Figs. 1.6A and B: Donnan membrane equilibrium
phosphate bonds are described in Chapter 21 .
Reactions of catabolic pathways (degradation or oxida- In left: Na•= R-+ Cl-; substituting: 9 = 5 + 4
tion of fuel molecules) are .usually exergoni . On the other In right: Na• = Cl-; substituting: 6 = 15
hand, anabolic ~athways (synthetic reactions or building The equation should also sati sfy that the number
up of compounds) are endergonic. Metabolism constitutes of sodium ions before and after the equilibrium are the
anabolic and catabolic processes that are well co-ordinated. same; in our example, initial Na• in the two compartments
together is 5 + 10 = 15; after equilibrium also, the value
DONNAN MEMBRANE is 9 + 6 = 15. In the case of chloride ions, initial value is
] EQUILIBRIUM 10 and final value is also 4 + 6 = 10.
In summary, Donnan's equations satisfy the follow-
When two solutions are separated by a membrane perme-
ing results:
able to both water and small ions, but when one of the
1. The products of~ e electro~rtes in both com-
compartments contains impermeable ions like proteins,
partments are equal.
distribution of permeable ions occurs according to the
2. The electrical neutrality of each compartment is
calculations of Donnan.
In Figures 1.6A and B, the left compartment contains maintained.
NaR, which will dissociate into Na• and R . Then Na• can 3. The total number of a particular type of ions before
diffuse freely, but R- having high molecular weight cannot and after the equilibrium is the same.
diffuse. The right compartment contains NaCl, which disso- 4. As a result, when there is nondlfliuslble anion on
ciates into Na • and Cl-, in which case, both ions can side of a membrane, the d~ !i!9_l&cations are
diffuse freely. <more, and diffoatble amonscite lflf , on,thar.side.
Thus, if a salt of NaR is placed in one side of a mem-
Clinical Applications of the !Equation
brane, at equilibrium
Na• x R- x H' x OH- = Na• x OH. x H· 1. The total concentration of solutes n plasma will be
To convey the meaning of the mathematical values, more than that of a solution of same ionic streng th
a hypothetical quantity of each of the ion is also incorpo- containing only diffusible ions; this provides the net
rated in brackets. Initially 5 molecules of NaR are added osmotic gradient (see under Albumin, in Chapter 26).
to the left compartment and 10 molecules of NaCl in 2. The lower pH values within tilssue cellg than
the right compartment and both of them are ionized in the surrounding fluids are partly due to the
(Fig.1.6A). When equilibrium is reached , the distributions concentrations of negative protejs1 ions within the
of ions are shown in Figure 1.6B. According to Donnan's cells being higher than in surrounding fluids.
equilibrium, the products of diffusible electrolytes in both 3. The pH within red cells is lower than that of the
the compartments will be equal, so that surrounding plasma is due, in part, to the very high
[Na•] L " [Cl ] L = [Na·] R x [Cr] R concentration of negative nondlfluJi2Lble hemogl2bin>
If we substitute the actual numbers of ions, the ~ - This will cause unequal distribution of W ions
formula becomes with a higher concentration w ithin t he cell.
9 ,c 4 in left = 6 x 6 in right 4 . The chroride shift in erythrocytes as well as higher
concentration of chloride in CSFj,are also due to
Donnan's effect (see under Hemoglobin, in Chapter 23).
In other words the number of cations should be equal to 5. Osmolarity of body fluid compartments and
the number of anions, such that sodium concentratiqr will follow Donnan equation.
,, Chapter 1: Biochemical Perspective to Medicine 9
Ro -\t) \' P'Jj)t
6. Different steps of water purification employ the non-covalent bonds are hydrogen bonds, ionic or
same principle and may be cited as an example of electrostatic bonds, hydrogen bonds, hydrophobic
industrial application of the equation. interactions and van der Waal's forces.
4. 70-80% of the cell is composed of water. Water
ILEARNING POINTS, CHAPTER 1 soluble compounds remain in solution, others are
solubilized by binding to water soluble compounds
1. Almost 99% of biomolecules of human tissues are
like proteins.
made up of only 6 elements; Carbon (C), Hydrogen 5. Energy changes in cells are governed by the laws of
(H), Oxygen (0 ), Nitrogen (N), Calcium (Ca) and thermodynamics and the term free energy denotes
Phosphorus (P). a portion of the total energy change in a system
2. Living organisms are ordered into(f)hierarc~ f available for doing work (~G).
increasing m~ ular complexity; Elements > Com- 6. All reactions can be classified into two; a) exergonic
pounds> Macromolecules > Supra molecular systems. or energy releasing reactions (degradative reac-
3. Bonds responsible for stabilizing these molecules tions) and b) endergonic reactions requiring energy
are covalent bonds and noncovalent bonds. The input (synthetic reactions).
_ _ _ _ _Chapter 2
Subcellular Organelles

and Cell Membranes

Chapter at a Glance
The learner will be able to answer questions on the foUowing topics:
, I
Nucleus
Transport mechanisms
Endoplasmic reticulum
Simple and facilitated diffusion
Golgi apparatus
Ion cha nnels
Lysosomes , p~-,.\~
Active transport
Mitochondria
Uni port, symport and antiport
Plasma membrane

CT) I SUBCELLULAR ORGANELLES TABLE 2.1: Separation of subcellular organelles


Subcellular Pellet formed at the
Cells contain various organized structures, collectively
organelle centrifugal force of Marker enzyme
called as cell organelles (Fig. 2. 1). When the cell mem-
Nucleus 600-750xg, 10min
brane is...disrupted, either by mechanical means or by
Mitochondria 10,000-15,000 x g, 10 min Inner membrane:
lysing the membrane by Tween-20 (a lipid solvent), the ATP synthase
organized particles inside the cell are homogenized. Lysosome 18,000-25,000 x g, 10 min Cathepsin
The organelles could then be separated by applying
Golgi 35,000- 40,000 x g, 30 min Galact~ylhans-
differential centrifugal forces (Table 2.1 ). Albert Claude complex ferase '
got Nobel Prize in 1974 for fractionating subcellular Glucose-6-
organelles. phosphatase

Ribosome
Plasma
membrane
Mitochondrion - - -- -
Rough /\ Marker Enzymes
endoplasmic
reticulum Some enzymes are present in certain organelles only;
Nucleus such specific enzymes are called as marker enzymes ·
Chromatin
(Table 2.1 ). After centrifugation, the separated organelles
Nuclear pore
are identified by detection of marker enzymes in the
sample.

I NUCLEUS
It is the most prominen organelle of the cell. All cells
Fig. 2.1: A typical cell in the body contain nucleus, except mature RBCs in

r
Chapter 2: Subcellular Organelles and Cell Membranes 11

- - - Endoplasmic reticulum Albert Camillo Christian George


Claude Golgi de Duve Pa/ade
NP 1974 NP 1906 NP ,f974 NP 1974
1899-1983 1843-1926 1917--2013 1912-2008
Fig. 2.2: Nucleus l~1'adfm 4f1114-<1h\t Ge.
circulation. The uppermost layer of skin also may not pos- proteins are being synthesized. Smooth endoplasmic
sess a readily identifiable nucleus. In some cells, nucleus reticulum is tubular and the site qf complex lipid ans
occupies most of the available space, e.g. small lympho- steroid synthesis. When cells are fractionated, the complex
cytes and spe.rmatozoa. ER is disrupted in many places. T~ey are automatically~
Nucleus ts surrounded by two membranes-the reassembled to form microsomes. \..!Y
inner one is called perinuclear membrane with numerou~
p~res (Fig. 2.2) and the outer membrane is continuous'-[_GOLGI APPARATUS C'frq..n)
with membrane of endoplasmic reticulum. --
Nucleus contains the DNA, the chemical basis of Camillo Golgi described the structure in 1898 (Nobel
; genes, which governs all the functions of the cell. The very Prize 1906). The Golgi organelle is a netw.ork of flattened
long DNA molecules are complexed with proteins to form smooth membranes and vesicles. It may be considered
chromatin and are further organized into chromosomes. as the converging area of eodoolasmjc retiQUW (see
DNA replication and RNA synthesis (transcription) are Fig. 2.1). While moving through ER, carbohydrate groups
taking place inside the nucleus. are successively added to the nascent proteins. These
In some cells, a portion of the nucleus may be seen glycoproteins reach the Golgi area1JGolgi apparatus is
as lighter shaded area; this is called nucleolus (Fig. composed of cis, medial and trans cisternae. Glyco-
2.2).This is the area for RNA processing and ribosome ~~?.~.i.~ ~ g~ ally transported from ER to cis Golgi
synthesis. The nucleolus is very prominent in cells Wro~ I...cIsterna), then to medial ~ i. Jl~W ~ qiate
actively synthesizing proteins. Gabriel Valentine in 1836 cisterna) and finally to trans Golgi (~ emaT for
described the n~ leolus. temporary storage. Main function o·f Golgi apparatus is
f,,\ l 'fr'\P' protein sorting, packaging and secmtion.
\g/~ NDOPLASMIC RETICULUM (ER) The finished products may have any one of the
It is a network of interconnecting membranes enclosing following destinations;
<;l)aonels or cisterr:iae They are continuous from 1. They may pass through plasma membrane to the
surrounding medium. This forms continuous secretion, !Ii
nuclear envelope to ~ ter plasma membrane. Under
electron microscope, the reticular arrangements will have
railway track appearance (see Fig. 2.1 (george Palaia)
was awarded Nobel Prize in 1974, who identified the ER.
e.g. secretion of immunoglobulins by plasma
2. They reach plasma membrane and form an integral
part of it, but not secreted.
cells.
-I
.
This will be very prominent in cells actively synthesizing 3. They form a secretory vesicle, where these products
proteins, e.~ mmunoglobulin secreting plasma cells. The are stored for a longer time. UndHr appropriate stimuli,
@ proteins, glycoproteins and lipoproteins are synthesized the contents are secretec:QRelea1se of trypsinogen by
in the ERI.J)e~ ification of various drugs is an important pancreatic acinar cells and re,lease of insulin by
function of EfftMicrosomal cytochrome P-450 hydro- beta cells of pancreas are cited as examples.
xylates drugs, such as benzpyrene, aminopyrine, aniline, 4. The synthesized materials may also reach lysosomal
morphine, phenobarbitone, etc. bags.
According to the electron microscopic appearance, Golgi bodies are fragmented during mitosis, but get
the ER is generally classified into rough and smooth reorganized by interaction with micrcitubules. Connective
varieties. The rough appearance is due to ribosomes tissue disorders like Sjogren's syndrome are found to
attached to cytoplasmic side of membrane where the be associated with anti-golgi antibodies.
12 Section A: Chemical Basis of Life

BOX 2.2: Perox1somal def1c1ency diseases


1. In gout, urate crystals are deposited around knee joints 1. Deficiency of peroxisomal matrix proteins can lead to aclreno-
(see Chapter 38). These crystals when phagocytosed, cause leukodY.illQQhy (ALQ) (8rown-Schilder's disease) characterized
physical damage to lysosomes and release of enzymes. by progressive degeneration of liver, kidney and brain. It is
Inflammation and arthritis result. a rare autosomal recessive condition. The defect is due to
2. Following cell death, the lysosomes rupture releasing the insufficient oxidation of very long chain fatty acids (VLCFA)
hydrolytic enzymes which bring about postmortem autolysis. by peroxisomes (see Chapter 16).
3. Lysosomal proteases, cathepsins are implicated in tumor 2. In Zellweger syndrome, proteins are not transported into the
metastasis. Cathepsins are normally restricted to the interior peroxisomes. This leads to formation of empty peroxisomes
of lysosomes, but certain cancer cells liberate the cathepsins or peroxisomal ghosts inside the cells. Protein targeting
out of the cells. Then c~ e g r g t b~ a.ssLJwiaa bv defects are described in Chapter 41 .
liydrglV1iog c~llagen and e(astin, so that other tumor cells 3. Primary hyperoxaluria is due to the defective peroxisomal
can travel out to form distant metastasis. ~ y ~~(seeChapter 18).
4. There are a few genetic diseases, where lysosomal enzymes
are deficient or absent. This leads to accumulation of lipids or
polysaccharides (see Chapters 12 and 14). embrane is intact, the encapsulated ,enzymes can act
5. Silkosis results from inhalation of silica particles into the
lungs which are taken up by phagocytes. Lysosomal mem- only locally. But when the membrane is disrupted, the
brane ruptures, releasing the enzymes. This stimulates fibro- released enzymes can hydrolyze ext,ernal substrates,
blast to proliferate and deposit collagen fibers, resulting in leading to tissue damage. ·
fibrosis and decreased lung elasticity.
6. Inclusion ce ll (I-cell) disease is a rare condition in w hich
lysosomes lack in enzymes, but they are seen in blood. This PEROXISOMES f-\n (!'< q ~-
means that the enzymes are synthesized, but are not able to
reach the correct site. It is s!'own that ~peso 6 pl;io,plu•e The peroxisomes have a granular matrix. They are of
is tbe rnarker to target the nascent enzymes to lysosomes In 0.3-1 .5 tnm in diameter. They contain peroxidases and
these persons, the carbohydrate units are not a"aded to the
enzyme. Mannose-6-phosphate deficient enzymes cannot
catalase. They are prominent in leukocytes and platelets.
reach their destination (protein targeting defect). Peroxidation of polyunsaturated fatty acids in vivo
may lead to hydroperoxide formation, Ft-OOH -+ R-O0,.
The free radicals damage molecules, cell membranes,
tissues and genes. (see Chapter 30). Catalase and
peroxidase are the enzymes present. in peroxisomes,
which will destroy the unwanted peroxides and other free
radicals. A few minor metabolic pathways operate in the
peroxisomes. Clinical applications of peroxisomes are
shown in Box 2_ : j-
l 'hCV"·)
MITOCHONDRIA
They are spherical, oval or rod-like bodies, about 0.5-1 tnm
in diameter and up to 7 tnm in length (see Fig. 2.1 )'.

Fig. 2.3: M
tW~2, Erythrocytes do not contain mitochm1dria. The tail of
spermatozoa is fully packed with mitochondria. Mito-
chondria are the powerhouse of the cell, where energy
l.!:._YSOSOMES released from oxidation of food stuffs is trapped as
Discovered in 1950 by Christian de Duve (Nobel Prize chemical energy in the form of ATP (see Chapter 21 ). Meta-

.
1974), lysosomes are tiny organelles. Solid wastes of a bolic functions of mitochondria are shown in Table 2.2.
township are usually decomposed in incinerators. Inside Mitochondria have two membranes. The in~ mem-
a cell, such a process is taking place within the lysosomes. brane convolutes into folds or cristae (Fig. 2.3)~ inner
They arG gs of enzymp. Clinical applications of lyso- mitochondrial membrane contains the e~ es of el~ -
somes are shown in Box 2.1. tron transport chain (see Chapter 21 ). fluid matrilf
Endocytic vesicles and phagosomes are fused with contains the enzymes of citric acid cycle, urea cycle and
lysosome (primary) to form the secondary lysosome heme synthesis.
or digestive vacuole. Foreign particles are progressively Cytochrome P-450 system present in mitochondrial
digested inside these vacuoles. Completely hydrolyzed inner membrane is involved in stenoidogenesis (see
products are utilized by the cell. As long as the lysosomal Chapter 45). Superoxide dismutase is present in cytosol
Chapter 2: Subcellular Organelles and Cell Membranes 13

TABLE 2.2: Metabolic functions of subcellular organelles


Nucleus DNA replication, transcription
Endop lasmic Biosynthesis of proteins, glycoproteins, lipoproteins, drug metabolism, ethanol oxidation, synthesis of cholesterol (partial)
reticulum
Golgi body Maturation of synthesized proteins

.£1- ~oasome Degradation of proteins, carbohydrates, lipids and nu<;leotides . .0 r, A,_

'll d itochondria Electron transport chain, ATP generation, TCA cycle, oefa oxidati'6n oH;li'.•y-a'/!id s, ke'tone body production, urea synthesis
(part), heme synthesis (part), gluconeogenesis (part), pyrimidine synthesis (part)
Cytosol Protein synthesis, glycolysis, glycogen metabolism, HMP shunt pathway, transaminations, fatty acid synthesis, cholesterol
synthesis, heme synthesis (part), urea synthesis (part), pyrimidine synthesis (part), purine synthesis

n
Fence with gates; gates open when
message is received
Nucleus Manager's office
Endoplasmic reticulum Conveyer belt of p roduct ion units Fluid Mosaic Model
Golgi apparatus Packing units
Lysosomes Incinerators The lipid bilayer was originally proposed by Davson and
Vacuoles Lorries carrying finished products
Danielle in 1935. Later, the structure of the biomem-
Mitochondria Power generating units
branes was described as a fluid mosaic model (Singer
and Nicolson, 1972).
and mitochondria (see Chapter 30). Mitochondria also The phospholipids are arranged in bilayers with the
contain specific DNA. The integral inner membrane pro- polar head groups oriented towards the extracellular side
teins, are made by mitochondrial protein synthesizing an~ e cytoplasmic side with a hydrophobic core (Fig.
machinery. However, the majority of proteins, especially 2.4~ / Choline co~ ining phospholipids are mainly in the
of outer membrane are synthesized under the control of external layer ar@e~3-01i~ e containing
cellular DNA. The division of mitochondria is under the phospholipids in the inner layer. Gerd Binnig and Heinrich
command of mitochondrial DNA. Mitochondrial ribo- Rohrer introduced the scanning electron microscopy
somes are different from cellular ribosomes. Antibiotics in 1981 by which the outer and inner layers of membranes
inhibiting bacterial protein synthesis do not affect cellular could be visualized separately. They were awarded
processes, but do inhibit mitochondrial protein biosyn- Nobel Priz~ 86l Eac~ leaflet is 25 A thick2with the
thesis (see Chapter 40).(Mitochondria play a role in · h.¥9 portjon 19 A and tail 15 A lbick The total thickness
tnggenng apoptosi~ (see Chapter 42). is about 50 to 80 A.
A summary of functions of organelles is given in The lipid bilayer shows free lateral movement of its
~ able 2.2 and Box 2.3. components, hence the membrane is said to be fluid in
nature. Fluidity enables the memb~ane to perform endo-
LASMA MEMBRANE cytosis and exocytosis. However, ttie cprnponents go
The plasma membrane separates the cell from the not freely move from inner to outer layer, or outer to inner
external environment. It has highly selective permeability layer (flip-flop movement is restrict d). During apoptosis
properties so that the entry and exit of compounds (programed cell death), flip-flop movement occurs. The
are regulated. The cellular metabolism is in turn influ- cholesterol content of the membrane alters the fluid-
enced and probably regulated by the membrane. The ity of the membrane. When cholesterol concentration
membrane is metabolically very active. The enzyme, increases, the membrane becomes less fluid on the outer
nucleotide phosphatase (5' nucleotidase) and alkaline surface, but more fluid in the hydrophobic core. The
phosphatase are seen on the outer art of cell mem- nature of the fatty acids also affects the fluid ity of the
brane; they are therefore calle ecto-en s membrane. Unsaturated cis fatty acids increase the
Membranes are mainly made up of lipids, proteins fluidity w hereas trans fatty acids decrease the fluidity.
and small amount of carbohydrates. The contents of these Sphingolipids and glycerophospholipids constitute the
compounds vary according to the nature of the mem- largest percentage of the lipid weight of biological mem-
brane. The carbohydrates are present as glycoproteins branes.
14 Section A: Chemical Basis of Life

Glycoprotein Anchor with


glycolipid

. I
SOA

]2sA ]

Transmembrane
protein
Fig. 2.4A: The fluid mosaic model of membrane Fig. 2.4B: Proteins are anchored in membrane by different mecha-
nisms

@ Membrane Proteins @ 1SPECIALIZED MEMBRANE


The peripheral proteins exist on the surfaces of the · h TRUCTURES
bilayer (Fig. 2.4B). They are attached by ionic and polar.
bonds to polar heads of the lipids.
ight Junction
Microdomains on membranes: Proteins are often When two cells are in close approximation, in certain
attached to the external surface of plasma membrane areas, instead of 4 layers, only 3 layers of plasma
at microdomains called lipid rafts. They are areas on membranes a ~n. TfiTs tight junction permits calcium
the membrane having predominantly glycosphingolipids and other small molecules to pass through from one cell
and cholesterol. These lipid rafts are implicated in to another through narrow hydrophilic pores. Some sort
endocytosis, G protein signaling and binding of viral of communication between cells thus results. Absence
pathogens.
of tight junction is implicated in loss of contact inhibition
The Integral membrane proteins are deeply embed-
in cancer cells.
ded in the bilayer and are attached by hydrophobic
Most eukaryotic cells are in contact with their neigh-
bonds or van der Waals forces. Some of the integral
boring cells and these interactions are the basis of
membrane proteins span the whole bilayer and they
formation of organs. Cells .that abut one another are in
are called transmembrane proteins (Fig. 2.4.A:). The
metabolic contact, which is brought about by specialized
hydrophobic side chains of the amino acids are embed-
particles called gap junctions. Gap junctions are inter-
ded in the hydrophobic central core of the membrane.
cellular channels and their presence allows whole organs
The transmembrane proteins can serve as receptors
(for hormones, growth factors, neurotransmitters), tissue to be continuous from within. Gap junctions are formed
from a type of protein called connexin. -.---
specific antigens, ion channels, membrane-based enzy-
mes, etc.
icrovilli
© Bacterial Cell Wall Microvilli of intestinal epithelial cells and pseudopodia of
Prokaryotic (bacterial) cells as well as plant cells have macrophages are produced by membrane evagination.
a cell wall surrounding. th.e plasma membrane; this cell
This is due to the fluid nature of membranes.
wall provides mechanical ·stre~th to withstand high .
osmotic pressure. Animal cells are devoid of the cell wall·
embranes of Organelle
they have only plasma membrane. Major constituent of
bacterial cell wall is a heteropolysaccharide, consisting Membranes of endoplasmic reticulum, nucleus, lyso-
of repeating units of N-acetyl muramic acid (NAM) and somes and outer layer of mitochondria may be considered
N-acetyl.glucosamine (NAG). Synthesis of this complex as variants of plasma membrane. Percentage of protein
polysaccharide is blocked by penicillin. This inhibition is euntent varies from 20% in myelin sheath to over 70%
responsible for the bactericidal action of penicillin. in the inner membranes of mitochondria.
Chapter 2: Subcellular Organelles and Cell Membranes 15

@ cytoskeleton
Human body is supported by the skeletal system;
• • •
similarly the structure of a cell is maintained by the cyto-
skeleton present underneath the plasma membrane.
The cytoskeleton is responsible for the sbape pf the cell,
its nJ,QlilitY and chromosomal rnmt.em.ents during cell
division. ®
(I(') The cytoskeleton is ~.p,posed of m icrofilaments, Fig . 2.5: Facilitated diffusion. The carrier molecule exists in two
~ ntermediate filaments an~ microtubules, formi ng a net- con Formations
work within the cell. Microtubules contain alpha and betam D'!ae.i'ii~"'Tii;;iric'
tubulin with a diameter of 25 nm. They are essential fo sive ransport
formation of mitotic spindle and participate in exocytosis Simple Diffusion
and endocytosis. @.nca Blkalo1°5°sed as anticancer
drugs. iobibi11be farmatiori:ol..mi~ @ioole-byJD.tecfer- Solutes and gases enter into the cells passively. They are
iQ.Q-Witb..tb.e-assem~ rotu bu les and thus in hi bit ,-driven by the concentration gradient. The rate of entry
ell division. 'is proportional to the solubility of that solute in the hydro-
phobic core of the-membrane. Simple diffusion occurs
TRANSPORT MECHANISM
- --- from higher to lower concentration. This does not require
Thc(@rmeabilit,y of substaoc:d across cell membrane is · any energy. However, it is a very slow process.
degendeot theic sol! 1bility io lipid§ and not on their Diffusion of gases such a~ a[ CO2) NO ahd Cq:Occurs
molecular siz~-~ compounds are generally at a rate that is solely dependent upon c;~mcentration
impermeable ~ eq~ e carrier mediated transport. Qradients. Lipophilic molecules will also diffuse across
An important function of the membrane is to withhold membranes at a rate th at is directly proportional to tbe
unwanted molecules, while permitting entry of molecules SQlubjJjty pf the coropo1,od io th e n:ien:ibrane.
necessary for cellular metabolism. Transport mecha- F .. d D ·u .
nisms are classified into: { BC/11tate 111us1on
t ive transport This is a carrier mediated process. (Fig . 2.5). Important
A. Simple diffusion features of facilitated diffusion are:
B. Facilitated diff~~ ormones regulate the a. The carrier mechanism c~ ld b§Cg) turated which
number of carrier moleqfes_For example, glucose tr.aos- is simi.!slr...t~ of eQ2l!JleS.
port across membrane is by f~ involv- b. StructI1ra1ty similar sal! 1te !': can competjtjyel£lohjbjt r,....
ing a family of @llCbse fraosporters. Glucose transport the entry of the solutes. p1D~
is described in detail in Chapter 10. c. Facilitated diffusion can operate bi,YirectjonaUy.
C. Ion channels are s~cjaljzed carrier systoo:is. d. This mechanism does not reguirea;qergv but the
They allow pass! ; i ;of molecules in accordance with the rate of transport is more rapid than si_mple diffusion
@ ncentratioo gr,a_:_~ _ process.
2 transpolt e. The carriE!r molecules__Gfn exist in two conforma-
mps can drive molecules against the gradient tions, RI r and ~ fsf'ates. In ~ Rong st~. the
using energy. active sites are ex os d t · r, when the

- solutes bind to the

sites are facin · io~


Then there is a
conformational change. In t~ 1ng state)the active
the cell, where the
solute is minimal. This will
cause the release at the soil 1te rnolec1 lies and the
protein molecule reverts to the pong state. By this
Ernst Ruska Gerd Heinrich Theodor mechanism the inward flow js facilitated, but the
NP 1986 Binnig Rohrer Schwann outward flow is in bibited (Fig. 2 .5). Hormones regu-
1906-1988 NP 1986 NP 1986 1810-1882
b.1947
late the number of carrier molecules. For example,
b. 1933
glucose transport across membrane is by facilitated

16 Section A: Chemical Basis of Life

BOX 2.5: Clinical applications of channels


1. Closing of potassium channels and opening of calcium
channels are important event s leading to insulin secretion.
.
2. Calcium channel blockers are used in treatment of hyperten-
sion. Potassium channel blockers used in cardiac arrhythmias
and potassium channel openers as smooth m uscle relaxants.
Cardiac glycosides like ouabain and digitalis are inhibitors of
V sodium-potassium ATPase, an effect therapeutically used in
olden days.
Sodium channels: Local anesthetics such as p rocaine act on
sodium channels both as blockers and on gating mechanisms
to hold the channel in an inactivated state.
5. Potassium channel mutations in "Long QT syndrome" leads
to inherited cardiac arrhythmia, resulting in prolonged QT
intervals in ECG. ·
6. Chloride channels: The role of GABA and glycine as inhibitory
neurotransmitters is attributed to their ability to open t he
chloride channels at the postsynaptic membranes.
Fig. 2.6: Water channel or aquaporin 7. Cystic fibrosis is due to certain mutations in the CFTR gene
(cystic fibrosis transmembrane regulator protein), w hich is a
chloride transport ing ABC protein.
BOX 2.4: Salient features of 10n channels 8. Retina: Light induced hyperpolarization is the major event in
1. They are transmembrane proteins visual excitation (see Chapter 32).
2. Selective for one particular ion
3. Regulation of a~tivity is done by 11pltage-gated, li_g_and-gated
or mechanically-gated mechanisms
4. Different channels are available for~•- K•, Ca.. and o- syndrome (sodium chann_el) periodic paralysis (potas-
5. Transport through the channel is ver sium channel) and some types of deafness.

fcu-tli~
E diffusion involving a family of ~ cose transe,ortfil:s.
Ion
Glucose transport is described in detail in Chapter 10. [ Membranes have special devices called ion channels
'-.:>~ c..~~ (see Fig. 2.9). Ion channels are tra11smembrane proteins
A UaP.orms · that allow the selective entry of various ions. Salient
They are water channels (Fig. 2.6) throug h which water features are enumerated in Box 2.4. These channels are
crosses the plasma membranes of cells. They QQ!l1wl the for qu.ic~ transport of electrolytes, such as~ ..$.
water cnnteoJ of cells. 'Agre.and_MacKinnof.l w~ war- and....Qr-. These are selective ion conductive
~G Nobel Prize for Chemistry in 2003 for their contribu- channels are specialized protein molecules that span~tie r&._
tions on aquaporins and ion channels. Diseases, such membranes. The channels generally remain closed, but in
as (oe hro enic diabetes insipid us due to impaired response to stimulus, they o~ allowing rapid flux of ions
fu nction of these chann Is. down the gradient. This may be compared to openjpg Q!
->
'ff t • ~ d -b d . th e gate of a cioeroa bo••se, when people rysb to enter
lien d I eren a ua onn are aescnoea in ru.iwar,s . . . . 11 .. •
d AQP pg) L ff t· -f in. Hence, this regulation Is named as gated . Such 10n
rme . oss o unc I0n o rena I . . .
_ . . . . channels are important f erve impulse pmpagation,
~quaponns is a~soci d wit~ sever~! dise~se ates; (i}ynaptic transmission a l: ecretion of biologically active
st
r;.e_du.ced ex,12ress,on, . 9 ~is associated with nephro- substances from the cells. Ion channels are different
enic diabetes insipidus NOi . :-.ira.m ion transport pumps described below.
Channe apathies are a group of disorders that
result fro a norm liti i r t · orming thej,Q[l A)..igand-Gated Channels
chan I to r ins. Channelopathies may be Ligand gated channels are o~ e-
acgu!!.;d or c~genital. ~ onge ital channelopathi~s may ~ - The bJnding o! a ligand to a receptor site O!l the
occur due to genetic · s in s~ . pot~fum, charme~..results in the.opening (or closi_ng) of the channel.
ch©rid e and ca~ channels. A few examples are The ligand may ~ ~tracellular sigoa ling molecule
Bartter sxndrome, myasthenia gravis, lo_!!9 and short OJ or an iotracellulrr: messJ nger. Clinical applications of
syndromes, cystic fibrosis {chloride channel), Liddle's channels are shown in Box 2.5.
Chapter 2: Subcellu/ar Organelles ancf Cell Membranes 17

Acetylcholine binding site

a
Exterior
Peter Agre Roderick Jens Skou
Plasma
NP2003 MacKinnon NP 1997
membrane
b. 1949 NP 2003 b. 1956 b. 1918

Cytosol Ion channels allow passage of molecules in accordance


Gate with the concentration gradient. Ion pumps can transport
Fig. 2.7 : Acetylcholine receptor
molecules against the gradient.

Acetylcholine receptor (Fig. 2.7) is the best example qlonophores .c~\\-e..l


Jor ligand gated ion channel. It is Rresent i os na tic They are membrane for spec.ific ions. They trans-
membrane. Acetylcholine released from the na tic port antibiotics. lonophores per,meability
region binds with the receptors o~ ynaptic region, oi,mem.brane to jons by acting as channel formers. The
which triggers OP.~ 9 of the channel and influx of Na·. two types of ionophores are; mobille ion carriers (e.g.
This generates a;:;ii;ction potential in the postsynaptic Valinomycin) and channel for~ers (e.g. Gramicidin).
nerve. The c h a n n e l ~ because They are P.roduced by certain mLC[ciQ.rg~ s and are
the acetylcholine is rapidly@egraded by acetylchohne- used a§ antibiat~ s. When cells of higher organisms are


sfeJase:) 40 - o -
Calcium channels:~ppropriate ti
Q ~_9alcium
·
exposed to ionophores, the ion gradient is dissipated.
- The ~ on..oLJtle.ion.grac:fum.U~es tb.fl proton
coannels are opened I sarcoplasmic reticulum gradient. The result is uncoupling ~of...oxidaiiv.e ph~p
membrane, leading to an ~ ~ ~ Q J be 1 ·on (see Chapter 21 ). '])\ :1'-
cy,to~QJ..oLtrius.ci~ ls. (~Alcilim e aooel 617iclffirs are f?J cD ~t\,el
1 l .,.._..
- therefore wid~ Y. ,u,sed in the 111F~ementotb.yp,;r1eD;!_Of1 ,. • e Transport \) .
Amelogeni~ m.ieio present in enamel of>teeth . . N, 1
U '-'->
has hydrophobic residues on the outside. A 27 amino The salient features of active transport are:
acid portion of amelog_e~ raJciuro cha~ ~ a. This form of transport requires energy. About {o~
Phosphorylation of a s@ resid the prot · of the total energy expenditure in a cell is useWr
the calcium channel. Calcium ions zoo rough and are the active transport system.
ot"tunneled to the mineralization front. e amelogenin is b. The active transport is unidirec1LI.orl.al
used for the formation of calcium hydroxy apatite crystals. c. It requires specialized integral proteins called trans,:
po
B)Voltage-Gated Channels d. The transport system is §.aiuralf~ at higher concen-
trations of solutes.
7 Voltage-gated channels (see Fig . 2.9) are opened by wem- e. The transporters are s.1isceQ!jble to iphibitioo by
brane depolarization. The channel is usually ~ in the specific organic or inorganic c,ompounds. General
gr~upd sti;!te. The membrane potential change (voltag! reaction is depicted in Figures :2.8 and 2.9 .

--
di!t,eceo.ce) switches the ion channel to open, lasting less
than ~5 !!!llli~G.Qn9s.
Sodium Pump I
In voltage-gated channels, the channels open or
close in response to changes in membrane potential. It is the best example for active transport. Cell has low
0'oJ1age-gated sodium channei~ and Qgitage-gated intracellular sodium; but concentration of gotassium
_eotassium chann~ are the common examples. These inside the cell is v~ 1intained by sodium-
are seen in gerye s!!,§..and are involved in the conduction potassium activated ATPase, £Jenerally called as
of nerve impulses. sodium pump. The ATPase is an integral protein of the
r

18 Section A: Chemical Basis of Life

• 5

Cytoplasm

4 2
3

Extracellular fluid

Fig. 2.8: The sodium potassium pump. It brings sodium ions out of the cells and potassium ions into the cells. Black circle = sodium
ion; green square = potassium ion; pink circle = phosphate. (1) Cytoplasmic sodium ions (3 numbers) bind to the clhannel protein. This
favors phosphorylation of the protein along with hydrolysis of ATP; (2) Phosphorylation causes the protein to change conformation,
expelling the sodium ions across the membrane; (3) Simultaneously, extracellular potassium ions (2 numbers) bind to the carrier
protein. Potassium binding leads to release of phosphate group; (4) So, original conformation is restored~ (5) !Potassium ions are
released into the cytoplasm. The steps 1, 2, 3, 4 and 5 are repeated .

Extracellular
Simple Ion/water ATP-coupled •
diffusion channel Gated-ion channel Symporter Antiporter active-transport
DOD
DD
Mil M6 Mil M Oc°

DOD
DD
Intracellular
6 -Ions

~ } Macromolecules

Fig. 2.9: Different types of transport systems

It has binding sites for ATP and sodiium on the inner


BOX 2.6 : Clinical applications of sodium pump
side and the potassium binding site is located outside
The use of cardiotonic drugs like digoxin and ouabain was
prompted by the use of t he leaves of t he plant foxglove by
the membrane. Details are shown in Figure 2.8. Clinical
natives. They bind to the alpha-subunit and act as competitive applications of sodium pump are s hown in Box 2.6. •
inhibitor of potassium ion binding to the pump. Inhibition of
the pump leads to an increase in Na' level inside the cell and Calcium Pump
extrusion of Ca" from the myocardial cell. This would enhance
the contractility of the cardiac muscle and so improve the An ATP dependent calcium pump also !'unctions to regu-
function of the heart. These drugs are now rarely used. late muscle contra~ ~ A specialized membrane system
called sarcoplasmic reticulum is found in skeletal mus-
membrane (Fig. 2.8). Jens Skou was awarded Nobel cles, which regulates the Catt concentration around
Prize in 1997 for his work on Sodium-Potassium-ATPase. muscle fibers.
Chapter 2: Subcellular Organelles anci Cell Membranes 19

In resting muscle the concentration of Ca .. around


I I External environment
muscle fibers is ~ - But §1irDulation~Ne impulse
results in a sudden release of large amounts of Ca++.
This would trigger muscle contraction. The function of
calcium pump is to remove cytosolic calcium and main-
tain low cytosolic concentration, so that muscle can Cell membrane

receive the next signal. For fil!Ch ATP b¥cim.btzed 2 C,a:+


!,Q_ns are transported.

{) Uniport, Symport and Antipo,t Cytosol


Transport systems are classified as uniport, symport
and antiport systems (Fig. 2.9).
Uniport system carriesrs·-,n-g...l_e_s_o_lu_t_ID across the
me mbrane, e.g. gl~se..tcaosporter in most of the cells. Fig. 2.10: Endocytosis
Calcium.pump is another example.
If the transfer of one molecule depends on simul- Clinical Applications
taneous or sequential transfer of another molecule, it is
In artnyp disea~ transport mechanism fo r amino
called cotransport system. The active transport may be
acids are defective in intestine and renal tubules (see
coupled with e nergy indirectly. Here, movement of the
Chapter 19). In ~ tinurra, renal realbsorption of cystine
substance against a concentration gradient is coupled
is abnormal (see Chapter 18). ReInal reabsorption of
with movement of a second substance down the con-
phosphate is decreased in fu.1amii1 D resistant nck_§ts
centration gradient; the second molecule being already
(see Chapter 32).
concentrated within the cell by an energy requiring process
The cotransport system may either be a symport or an ndocytosis
antiport. In symport, (Fig. 2.9) the transporter carries two
solutes in the same direction across the membrane, e.g. Endocytosis is the mechanism by wt1ich cells internalize
sodium dependent glucose transport (see Chapter 10). extracellula r macromolecules, to 1form an endocytic
Phlorhizin, an inhibitor of sodium-dependent cotransport vesicle. This requires energy in the form of ATP as well as
of glucose, especially in the proximal convoluted tubules calcium ions in the extracellular fluict Cytoplasmic con-
of kidney, produces renal damage and resu lts in renal tractile elements take part in this movement. In general,
glucosuria. Amino acid transport is another example plasma membrane is invaginated, enclosing the matter.
for symport. The antiport systerm (Fig. 2.9) carries two This forms the endocytic vesicle (Fig. 2.10). The endo-
solutes or ions in opposite direction, e.g. sodium pump cytosis may either be pinocytosis or phagocytosis.
(Fig. 2.8) or chloride-bicarbonate exchange in RBC (see.~ -.-
Chapter 23). Features of different types inoc_ytosis
modalities are summarized in Table 2.3. Pinocytosis literally means 'drinking by the cell'. Cells take
up fluid by this method. The fluid p,hase pinocytosis is
a nonselective process.

Carrier Examples
ndOC1YtOsis
1) Simple no no Water
diffusion The selective or adsorptive pinocytosis is receptor-
i.) Facilitated yes no nil Glucose to RBCs mediated; also called as abso rptiv,e pinocytosis. Low
diffusion
density lipoprotein (LDL) is a good 13xample. LDL binds
:1) Primary yes yes directly Sodium pump
to the LDL receptor and the complex is later internalized.
active
The cytoplasmic side of these vesicles are coated with
4) Secondary
active
yes yes indi~ect"
'-._J
Glucose to intestine
filaments; mainly composed of ClathIrin. These are called

s) Ion
channels
yes no no Sodium channel Clathrin coated pits. Absorption of cholesterol by clathrin
coated pit is shown in Chapter 14. After the LDL-recepto r
22 Section A: Chemical Basis of Life
2-3. Which of the following cellular organelle can cause B. Membrane proteins are fixed and cannot move
autodigestion? C. Membrane contains bimolecular leaflet of lipids
A. Golgi bodies B. Lysosomes D. Permeability of drugs across cell membrane is
C. Microsomes D. Peroxisomes dependent on their solubility in water
2-4. All the following activities are taking place inside 2-15. Galactosyltransferase is a marke,r enzyme of:
mitochondria, except:
A. Cell membrane B. Mitochondria
A. Citric acid cycle
C. Lysosomes D. Golgi complex
B. Electron transport chain
2-16. ATP synthase is a marker enzyme present in:
C. Beta oxidation of fatty acid
D . De novo fatty acid synthesis A. Cytoplasm B. Mitochondria
2-5. All the following statements are true with regard to C . Lysosomes D. Golgi complex
biomembranes, except 2-17. All the following enzymes are p,resent in lysoso-
A. They are selectively permeable mes, except:
B. Have interior hydrophilic and exterior hydrophobic A. Alpha fucosidase
groups B . Arylsulfatase
C . Lipid bilayers contain phospholipids and cholesterol C . Beta glucuronidase
D. Carbohydrate residues are located on their surfaces D. Superoxide dismutase
2-6. The fluid mosaic model of membrane includes: 2-18. Golgi bodies are:
A. Cholesterol molecules in the membrane increase A. Involved in phagocytosis
stability of membrane B. Temporary storage sites for secretory products
B. Membrane proteins are fixed and cannot move C . Concerned with disposal of waste products
C . Membrane contains bimolecular leaflet of lipids D. Engaged in transport of molec1ules into mitochon-
D. Permeability of drugs across cell membrane is dria
dependent on their solubility in water 2-19. Which of the following is not syn1thesized in endo-
2-7. All the following lipids are present in cell membra- plasmic reticulum?
nes , except: A. Lipoproteins B. Glycoproteins
A. Lecithin B. Cholesterol C . RNA D . Ganglioside
C . Sphingomyelin D. Triacylglycerol 2-20. Which of the diseases listed bel<lw is a lysosomal
2-8. Fluidity of membranes depends on: storage disease?
A. Nature of fatty acids A. Zellweger syndrome
B. Concentration of proteins B. Inclusion cell disease
C. Glycosylation of proteins C. Pompe's disease
D. Membrane pumps D. Leber's neuropathy
2-9. Sodium-potassium pump is inhibited by: 2-21. Peroxisomes are:
A. Phlorizin B. Digoxin A . The site of biological oxidation
C . Valinomycin D. Dicoumarol B. The site of post-translational modifications
2-10. Which of the statements regarding active transport is C . Concerned with drug detoxification
incorrect? D. Involved in the production of free radicals
A. Requires carrier protein 2-22. Which are specialized membrane, structures?
B. Energy dependent A. Peroxisomes B. Desmosomes
C. Against concentration gradient C. Lysosomes D. Microsomes
D . Transport of water is an example 2-23. All the following are channel formers, except:
2-11 . All the following are channel formers, except: A. Gramicidin B. Amelogenin
A. Gramicidin B. Amelogenin C . Valinomycin D. Glutamate
C. Valinomycin D. Glutamate 2-24. Phagocytosis involves:
2-12. Symport system operates in the transport of: A. Respiratory burst
A. Bicarbonate B. Chloride B. Lipid peroxidation
C. Calcium D. Glucose C. Alpha oxidation
2-13. Glucose-6-phosphatase is a marker enzyme pre- D. Endosmosis
sent in: 2-25. Which of the following is a featur,e of endocytosis?
A . Cytoplasm B. Mitochondria A. Plasma membrane is invaginateid to form vesicles
C. Lysosomes D. Microsomes B. Cytoplasmic side of vesicle is coated by clathrin
2-14. The fluid mosaic model of membrane includes: filaments
A. Cholesterol molecules increase stability of mem- C. LDL uptake is by endocytosis
brane D. Heparin helps in this mechanism
Chapter 2: Subcellular Organelles and Cell Membranes 23

ANSWERS OF MULTIPLE CHOICE QUESTIONS


2-1 . B 2-2. B 2-3. B 2-4. D 2-5. B 2-6. C 2-7. D
2-8. A 2-9. B 2-10. D 2-11 . D 2-12. D 2.13. D 2-14. C
2-15. D 2-16. B 2-17. D 2-18. B 2-19. C 2-20. C 2-21 . D
2-22. B 2-23. D 2-24. A 2-25. A

PART-3: VIVA VOCE QUESTIONS AND ANSWERS

2-1 . What is the function of Golgi complex? 2-15. What are salient features of facilitated diffusion?
Maturation and processing of nascent proteins, glyco- It is carrier mediated. It does not require energy directly.
2-16. Give an example of facilitated transport.
sylation of proteins, secretion of newly synthesized pro-
Glucose transporters.
teins.
2-17. Give some examples of ion channels.
2-2. What is the function of endoplasmic reticulum? Ion channels for calcium, potassium and chloride.
Biosynthesis of proteins, drug metabolism, desaturation 2-18. What are ionophores?
of fatty acids. They are transport antibiotics which increase the
2-3. Where does protein synthesis takes place? permeability of membrane to ions, e.g., valinomycin,
On the walls of endoplasmic reticulum and also in cytosol. gramicidin.
2-4. What are cathepsins? 2-19. What are the salient features of active transport?
They are intracellular proteolytic enzymes. It requires transporters. It requires energy. Transport is
generally unidirectional.
2-5. What is the function of lysosomes?
2-20. Give examples of active transport systems.
They are bags of hydrolytic enzymes that bring about
Sodium pump, Calcium pump.
degradation of macromolecules. 2-21 . What is the importance of sodium pump?
2-6. What is lysozyme? Cell has low intracellular sodium; but concentration of
It is an enzyme present in external secretions. potassium inside the cell is high; this is maintained by
2-7. What is the most important function of mitochondria? sodium pump.
Generation of ATP. 2-22. What is the clinical significance of sodium pump?
2-8. What are the important metabolic events taking Digoxin increases the contractility of the cardiac mus-
place in mitochondria? cle, by inhibiting the sodium pump.
TCA cycle, Electron transport chain, Beta oxidation of 2-23. What is a uniport?
It carries single solute across the membrane.
fatty acids and urea cycle.
2-24. Give examples of uniport system.
2-9. What are ecto-enzymes? Glucose transporter (GluT2) operating in most of the
Enzymes seen on the outer part of cell membrane. cells is an example. Calcium pump is another example.
2-10. Give examples of ecto-enzymes. 2-25. What is co-transport?
Alkaline phosphatase, 5'nucleotidase. If transfer of one molecule depends on simultaneous
2-11. What is the structure of cell membrane? transfer of another molecule, it is called co-transport
system.
Fluid mosaic model.
2-26. What is symport?
2-12. What are characteristics of fluid mosaic model? In symport, the transporter carries two solutes in the
Membrane is composed of lipid bilayer. Phospholipids same direction across the membrane.
are arranged in bilayers with a hydrophobic core . 2-27. Give examples of symport.
2-13. What are the components of membrane that alter Sodium dependent glucose transporter (SGluT). Amino
the fluidity? acid transport is another example for symport.
Cholesterol and unsaturated fatty acids. 2-28. What is antiport system?
The antiport system carries two solutes or ions in
2-14. What are the different types of transport mecha-
opposite direction.
nisms? 2-29. Give examples of antiport.
Passive and active. Passive type is subclassified as Sodium potassium ATPase and ch loride bicarbonate
simple diffusion and facilitated diffusion. exchanger.
_ _ _ _ _Chapter 3
Amino Acids:
Structure and Properties
/, I I

Chapter at a Glance
The learner will be able to answer questions on the following topics:
Classification of amino acids based on structyre A-b Reactions due to carboxyl group (C...O)
B Based on side chain character -- Reactions due to amino group tN\:-4:i)
c Based on metaboljc fate CD Reactions of SH group {_S.1--\)
l> Based on nutritional regy jrements ,a~ Pepti de bond formation
.tifO lsoelectric prnnt

Proteins are of paramount importance in biological '' • A mIno


"" . cp
deN~
acI"ds wI'th c;1 m1 group: AsparaQ!_ne,
·
systems. All the major structural and functiona l aspects Glutamine (Fig. 3.6). [ A&.j
of the body are carried out by protein molecules. All b. Monoamino dicarboxylic acid · Aspartic acid ,
proteins are polymers of amino acids. Proteins are Glutamic acid (Fig. 3.7).
c. Dibasic monocarboxylic acids: Lysine, Argi-

-
composed of a number of amino acids linked by peptide
bonds. nine (Fig. 3.8). (i--lPtt.J
Although about 500 amjno acjds occur in nature, B. Aromatic amino acids: Phenylalanine, Tyrosine
only are se~ uman bod . Most of the (Fig. 3.9). [f>T"]
amino acids (~ oline) a re O acid C. Heterocyclic amino acids: Trypto~han (Fig. 3. 10),
w hich means tnanhe amino group is attached to the Histidine (Fig. 3.11 ). L 11\J
same carbon atom to which the carboxyl group is D. lmino acid: Praline (Fig. 3.1 1). Cr]
attached (Fig. 3.1 ). E. Derived amino acids:
a. Derived amino acids found in proteins: After
the synthesis of proteins, some of the amino acids
are·~ d. e.g. ~ e (Fig. 3.12) and
are important components of
A. Aliphatic amino acids
collagen. G2 mma carboxylatjon of glutamjc acid
a. Monoamino monocarboxylic acids: residues of proteins is important for clotting
J. ~ e amino acids: Glycine, Alanine (Fig. 3.2)
t-' • Branched chain amino acids: ValiJ:le, Leu~ ~
WP-L-I] ~ - I 00-
l'!,(1'1).,
lsoleucine (Fig. 3.3)
t! · J:jyd?J'ISY- amino acids: § erine, Threonine 0.j
1
yH-NHt
(Fig. 3.4.)
r• Sulfdr-cootajning amino acids: Cysteine, Methio- R
nine (Fig . 3.5) (.Ctv') Fig. 3.1: General structure
Chapter 3: Amino Acids: Structure and Properties 25

1 coo-
l iH-NH;
-] n ',<? 1 0

r-NH; I
ffi jcoo- 1
I NH•3
CH-
~1-
j
CH--CH3 CH2
TH~
I I ~
H CH3 HUH CH2
I 3' I
CH3 CH3
Glycine Alanine
Gly; G Ala; A Valine Leucine lsoleucine
Val;V Leu; L lie; I

Fig. 3.2: - le amino acids Fig. 3.3: Blllflllil!l chain amino acids

.
1
Fl lf coo- 1 coo-
l n coo
I
I

,. +~
+
+ I • C H- NH3
+
.__C H-NH3
l1H-NH3 CH-NH3
I CH2 .. ,tik~ CH2
I
CH24 0H I CH,-@] CH2
I
CH3 § 0 CH 3
Serine; Ser; S Threonine; Thr; T Cysteine; Cys; c ~ Methionine; Met; M

Fig. 3.4 : . . , , amino acids Fig. 3.5 P g ..imino acids

coo- n coo- 1 1t - coo


I • I + I •
CH- NH 3 CH- NH3 a CH-NH3
I I
CH 2 CH2 f3 CH2
I I
160-NH2 I m,d CH2 y CH2

160-NH2 I 1~00- 1
Asparagine; Asn; N Glutamine; Gin; Q Aspartic acid; As1p; D Glutamie acid; Glu; E

Fig. 3.6: Fig. 3.7 · amino acids

'.t
coo-
I
CH- NH3
+
a
0

6H- NH; I
IF\lb' CH- NHI
HO
' - - - - -- - '
H2
I
C
r-:1 +
I CH- NH 3
I 600- 6 00-
CH2
f3 TH2 L -
I Phenylalanine·; Phe; F Tyrosine; Tyr; Y ( ....,OL>-•v•...
CH2 y CH2
I I (Benzene group) (Phenol group)
H2 6 TH2 Fig. 3.9: Aromatic: amino acids
, NH c CH2
I •
C=NH2
I (}
I I ~HJ+

NH2

group) Lysine; Lys; K

Fig. 3.8: mine a cids Fig: 3.10: . . . . (W) with indole group
26 Section A: Chemical Basis of Life

coo-
~CH2

~ 1,H-NH;
coo-
Histidine; His; H (imidazole group)

Fig. 3.11 : Histidine and proline


roline; Pro; P (Pyrrolidine group)
f" '
I

I
H2

H2-NH3
N H2
+
coo-
..

Ornithine 4-hydroxy proli ne


TABLE 3.1: Common amino acids
3 letter I letter Special group Fig. 3.1 2: Some derived amino acids
abbre- abbre- present in side
Name viation viation Chemical nature chain position, is a constituent of 02;ntothenic c;c id
Glycine Gly G Aliphatic, M AMC Nil (vitamin) and coenzyme A.
Alanine Ala A Aliphatic, MAMC Methyl Each amino acid will have three-letter and one-letter
Valine Val V Aliphatic, MAMC Branched chain abbreviations whiich are shown in Table 3.1 as well as
Leucine Leu L Aliphatic, MAMC Branched chain in Figures 3.2 to 3. 11. Som,~:e:::;t:;,;im
_.e~ ~ ~ ~..:.::::.~ ~~
lsoleucine lie I Aliphatic, MAMC Branched chain acig,_uiay not be :separ entifie for hich 3-letter
Serine Ser s Aliphatic, MAMC Hydroxyl abbreviation i~ii:lin -lett r abbreviat n i{ID Simi-
Threonine Thr T Aliphatic, MAMC Hydroxyl larly lei.Ix or Z ~ 1ds f~ tamine/glutamic acid.
Cysteine Cys C Aliphatic, MAMC SH g roup
Methionine Met M Aliphatic, MAMC
f. Special Groups in Amino Acids
Asparagine Asn N Aliphatic, MAMC Amide
Glu tamine Gin Q Aliphatic, MAMC

@ Aspartic acid
Glu tamic
Asp
Glu
D
E
Aliphatic, MADC
Aliphat ic, MADC
acid
Lysine Lys K Aliphatic, DBMC

@ Arginine
Phenyl
Arg
Phe
R
F
Alip hat ic, DBMC
Aromatic
alanine
Tyrosine Tyr y Aromatic Amino acids having non-polar side chains: These
Tryptophan Trp w Aro matic groups a~tli,1ydrophobic (water repel!_antLind lipo-
Hist idine His H Heterocyclic philic. Therefore, the parts of proteins made up of
Proline Pro p lminoacid these amino acids will be hydrophobic in nature.
(MAMC: Monoamino mono See Table 3.2 for classification based on side chain.
carboxylic acid). B. Amino acids having uncharged or non-ionic polar
side chains : These amino acids are i,_ydrophiliS, in
process (Fig. 3.12). In ribosomal proteins and in nature.
histones, amino acids are extensively methy- C. Amino ~cids · harged or~onic olar si e
l~ted and acetylated. chains hydlrophilic
b . Derived amino acids not seen in proteins Acidicarnino acids: They have a negative charge
(Non-protein amino acids): Some derived amino on the R group: Aspartic acid and Glutamic acid.
acids are ~en free in cells, e.g. 9!]ithine (Fig. (Tyrosine, is mildly acidic since it has a weakly
3.12), CJ.!rulline, ~ cys.J.e,jpe. These are ionisable phenolic hydroxyl group)
produced during the metabolism of amino acids. b. Basic amino acids: They have a positive charge
~ e may be considered as derived from on the R group: Lysine, Arginine and Histidine.
tyrosine.
c. Non-alpha amino acids: Gamma amino bu~
acid (GABA) ~ erived-ff8m-'glutamic acid. A. Purely ·& tc>genic: @u..9ne is purely ketogenic
~ - where amino group is in beta because it is converted to ketone bodies.
Chapter 3: Amino Acids: Structure and Properties 27

TABLE 3.3: Classification of a.~,11,0 acids based on their meta-


bolic fate

Neutral amino 2, Both glucogenic and


ketogenic
(5 amino acids)
Glucogenic
acids (having (14 amino acids)
: polar side chains) Leucine Lysine Glycine, Alanine, Valine
(hydrophilic) lsoleucine Serine, Threonine, Cysteine
Phenylalanine Methionine, Asparagine,
Glutamine
Tyrosine Aspartic acid, Glutamic acid
Tryptophan Arginine, Histidine, Proline

Growing children require them in food. But they are


~ j ~ ~l.
t--- - - - - , t - - -- -'ll~- -- - - + - - -- - - - l 'J) i . essential or Dispensable: The remajnjng
10 amino acids are n tial, because their
bon skeletor@.ai4-bo syntbe.slzelt by the body.
So we need not ingest these amino acids as such.
However, they are also required for normal prot~n
• Help In LearnlQg These Little Molecules P~ s Truel synthesis. The nonessential amino acids are listed
'l le"
This stands for
CKm : in Table 3.4. All body proteins do contain all the
nonessential amir)o acids.
Arginine, Hist idine, lsoleucine, Leucine, Threonine, Lysine, c. /J. Conditionally essential amino acids: When a
Methionine, Phenylalanine, Tryptophan and Valine in that order. person is suffering from a moderate to severe chro~
Arginine and Histidine are semi-essential amino acids; while illness, person may lose the ability to manufacture
others are essential.
e~h-
nones5eptjal
w JP
amjnn'----<-
~ s and thus require
supplementation. Problems with digestion will also
B.~ g~ ~ en~ ~ s;g~: .11cllle, necessitate supplementation of "nonessential" amino
1--i:'uenylalaoine, T$asi!J.e and ~JQQhan are partially acids. These amino acids are normally nones~ntial,
'"" -lletogenic and partially glucogenic. During meta- but become essential during times of physiological
bolism, part of the carbon skeleton of these amino stress. Then these amino acids have to be taken in
acids will enter the ketogenic pathway and the other foc)dor through supplements. These conditionally
part to glucogenic pathway. essential amino acids are shown in Table 3.4.
C. Purely glucogenic: AlLtbe..remaiolDg 14 awiaa.aQids Naming (numbering) of Carbon Atoms
are purely glucogenic as they enter only into the
Carbon atoms in amino acids in sequence are named
glucogenic pathway (Table 3.3).
with letters of Greek alphabets, starting from the carbon
atom to which carboxyl group is attached. As examples,
naming of glutamic acid is shown in Figure in 3.7 and
-; A. Essential or in 1s sable: The am· o acids may that of lysine is shown in Figure 3.8.
further be clasmfieaaccording to their essential nature
for growth. Thus, lsoleucine, Leucine, Threonine, ID
: Lysine, Methionine,(! henylalanine, ryptophan,
and Valine are essential amino acids. Their carbon Glycine, alanine, valine, serine, tryptophan, histidine and
skeleton cannot be synthesized by human beings
J and so preforrT)_ed amino acids are to be taken in isoleucine and arginine are bitter
a flavoring agent
s
a 11ciarsweetener con-
food for normal grqwth. N rmal rowth and optimal
~ ~ r, if one such amino acI 1 tains aspartic acid an phenylalanine. All amino acids
· ioJha.diet. See me · · have high melting poir:its (more than 200°C). All amino
B. Partially essential or semi-essential: Histidine acids are soluble in water and alcohol (polar solwnts);
~ nd rginine are semi-indispensable amino acids. but insoluble in nonpolar solvents (benzene).

\
28 Section A: Chemical Basis of Life

TABLE 3.4: Classification of amino acids based on nutritional


14··
importance
13
Conditionally 12..
amino acids amino acids essential amino 11 pK2 = 9.8
(10aminoacids) adds 10··
Methionine Arginine Glycine Glyc· 9
Threonine
Tryptophan
Histidine Alanine
Serine •
® ~··
6··
Valine Cysteine
5 pK1 = 2.4
Leucine Asparagine 4 ..
lsoleucine Glutamine 3
2··
Phenylalanine Aspartic acid
Lysine Glutamic acid
MeTTVILPhly Tyrosine 0.5,....._ _..;.__~ 2
Proline Equivalents of OH-

Fig. 3.14: Titratio n curve of glycine; pH in Y-axis



COOH
pl= pK1+ pK2
I H-C- NH 2 2
H-C- @
I ..,__ - ---i.... I As for example,
R R
+OH . 2.4 + 9.8
p I of gIycme = - - - = 6.1
2
Cation (low pH) Zwitterion (isoelectric pH) Anion (high pH)
• From the graph it is evident that the bjl{fering.,aclli)n
Fig. 3.13: Ionic forms of amino acids • "· ' , . rnaxmwro
I~ · · aod arouoclpKl or at pK2 and minimum
m
/;'\ ____ drr, ~\fl . ~Fig. 3.14). In the case of amino acids having more
Ampholyte and lsoelectri&-- oint ..than two ionizable groups, correspond'.ngly there will be
Amino acids can exist as ampholytes or zwitterions more pK values, e.g. sp it1c ac1 Fig . 3.15). The pK
(German word "zwitter" = hybrid) in solution, depending on
the pH of the medium. The pH at which the molecule
both carboxyl and amino groups of amino acids are
carries no net char is known as isoelectric point or
completely ionized. Thus to be very correct, Zwitterion
isoelectric pH (pl). In ac· ic ol · ic
forms are to be shown as the structures of amino acid!';.
in form and in cll..kaline solution they behave as anions
The pK value of imidazolium group of His · · e is 6 .1,
(Fig. 3.13). At isoelectric point, the amino acid will carry
and the fore eff s-a--bttfferar e physiological
no net charge; all the groups are ionized but the charges
will cancel each other. Therefore, at isoelectric point, pH o 7. . he buffering capacity of plasma proteins and
there is no mobility jn ao electm:aHiekt,-~olubility and h~ oein ~ istidine residues.

® buffering capacity will be minimum at isoelectcic pH .. To


wch a s~lution if we add hxdrochloric acid drop by tjrop
"-'"=::--=..-

l at a particular pH, ~ ole.cules_ac.e-in-Gaiion


f9rm and 50% in zwitterion.1Qrm. Thi pH is pK with . Amino acids having an asyf!:!ooetric ~arbon atom exhibit
regard to COOH). If more HCI is adde , more molecules optical activity. Asymmetry arises when ~
become cationic in nature and solubility increases. are a~
ps
arrJ.e.GarJlQ.o.atom (Fig. 3.16). Glycine
·-
On the other hand, if we .lltte!e Jh e solution from is the simplest amino acid and has no asymmetric carbon

I iso_electric point ,tJith NaoH; molecules ac.quire the atom and therefore, shows no optic;i° actiJity. All others
aQJ.Onic.Jom:i. When 50% of molecules.are anions, that are optically active.The mirror image forms produced
pH is calledfuK2:.twith--respect to NHJ The isoelectric with reference to the alpha carbon atom, are called..Q.a.lli:l
pH {pl) for m~
be calculated :
ino monocarboxylic amino acids can L isomers. The L-amino acids occur in nature and are
therefore called ~atural amino acids. ~ ~ ino acid~

Chapter 3: Amino Acids: Structure and Properties 29

- -
COOH
+OH-
coo- l
I +OH-
coo-
I
cHf NH 3• I
CH-E i] l
I +-- +-- CH-NH 2
CH2 + H• I + H• I
CH2
CH2
booH pK1 = 2.1 I pK3 = 3.9 pK2 = 9 .8
COOH coo-
J Ntt_charge n I ') Net c harge: -1 Net charg0
Fig. 3.15: Ionization of aspartic acid

TABLE 3 .5: pK values of amino acids


COOH
pK3of
p K1 of pK2of extra Extra CH-NH 2
alpho alpha ioniz- ionizable I
Name of carboxy
R
amino able group Decarboxylation
amino acid group group group present Amino acid
pl Corresponding amine
Glycine 2.4 9.8 6.1 Fig. 3. 17: Decarboxylation of amino acid
Valine 2.3 9.6 6.0
Serine 2.2 9.2 5.7 functi
Cysteine 1.9 10.3 8.2 Sulfhydryl waU I
5.1
Glutamine 2.2 cen
9.1 5.6
Aspartic 2.1 9.8 3.9 Beta 3.0
acid carboxyl ... GEN ERA L REACTION
Glutamic 2.2 9.6 4.3 y-carboxyl 3.2 ~0 F AMINO ACIDS
acid
Lysine 2.2 8.9 l to Carboxyl Group
-
10.5 e-amino 9.7
Arginine 2.0 9.0 12.5 Guanidine 10.8 ry_Jecarboxylation
Phenyl- 2.6 9.2 5.9
alanine The a mino acids will underg o alpha decarb oxylatio n to
form the corresp onding amine (Fig. 3.17). Thus, some
Tyrosine 2.2 9.1 10.1 Phenol 5.7 impo rtant amines are produc ed from amino acids. For
Tryptophan 2.4 9.4 5.9 exam pie,
Histidine 1.8 9.2 6.1 lmidazole 7.6 'JH istidine -+ Histam ine + CO
2
(0Tyr osine -+ Tyrami ne + CO
2
Q' Tryptop han -+ Tryptam ine + CO2
L sine -+ Cadave rine + C
CHO COOH J COOH ©,_Glu tamic acid -+ Gamm a amino butyric acid (GABA

HO - y -
I
CH2 0H
H ~
I
y-H IH
I
-i-61 - L -- +CO2 - - - -- --
R R Amid e Formation b'5 F-\C JDlc @
L-glyceraldehyde L-amino acid 0-amino acid The-CO OH group of dicarbo xylic amino acids (other
than alpha carbox yl ) can combin e with ammon ia to form

=~~,btiG~
Fig. 3.16: L- and D-amino acids
the corresponding amide. For examp le,
Asparti c acid + NH -+ Aspara gine
seen in small amoun ts in micr ~aA~ consti-
3
Glutam ic acid + NH -+ Gt utamin e (Fig.3. 18)
3
tuents Gµunicidin~ . P_g!y- These amides are also compo nents of protein
rm!Xin:Acti = \ n~ p linomyc,in, as well as bacterial structu re. The amide group of glutam ine seNes as the
cell wall ~ tidoglyc ans. D-amin o acids have multiple source of nitroge n for nucleic acid synthe sis.
...,,.--- - -

30 Section A: Chemical Basis of Life

coo- coo-
coo-
l • l -E[l \.'"':'!I §
_Jpha k~o-
l
c:(g ,
CH-NH3 CH~
Glutamine
I "' . glutarate I
synthetase I CH Glutamate CH 2

( \. CH
I
2 I 2
CH2
I
CH2
CH I - I -
ATP ADP+ Pi 2
coo
I coo
c~
coo
Glutamine ) I
Glutamic acid
c Hf NH3•
I
I
Fig. 3.1 8: Formation of glutamine
CH 3

Fig. 3.19: Transarnination reaction


HOOC -CH- NH2 HOOC- CH- NH2
I I Due to Sider Chains
CH 2 CH 2
I rt-, Pr. Transmethylation
SH s
+ I The methyl group of Methionine,.atter activation, may be
SH
~ ~tor, which becomes methylated
CH 2 (see Chapter 113).
CH 2
I I Methionine + Accep tor -+ Methylated Acceptor +
HOOC -CH-N H2 HOOC -CH-N H2 ft ~ oc~s~
Cysteine Dicystei ne (Cysti~ )
Este r Formation by the OH Group
The hydroxy amino acids can form esters ~h...Qb.Q§-
Fig. 3.20: Formation of disulfide bridges
r~ric ~ - In this manner, the Serine and Threonine
residues of proteins are involved in the formation of phos-
Due to Amino Group phoproteins. Similarly, ~se ,,bydroxylge9ups can form
Transamination t 9., 9) P -glycosjdjc bnndslwitb- carb
d glycoproteins.
di: id es to form

"d b t f
<>lnh<> <>minn
Th e ~ , .~...........,,...-,min .a6l can e rans err:$
nrn, , ... nf au.uµQ

tc alpha keto acid to f!>rrnlh e .corresQ9.0di WJ new arrJin@


Reaction of the Amid e Group
acw.a og.lllp baketo ac[Q (Fig. 3.19). This is an import ant
of amino The amide groups of G.liJtamjne and Asparagine .ffill..fgrm
reactio n in the body for the interco nversi on
-glycosidic bonds with carbohydrate re§idues to form
acids and for synthesis of nonessential amino acids.
glycoproteins. Amide nitrogen can be donated for
synthesis of niltrogenous compounds.
Oxidative Deamination
The alpha amino group is removed from the amino acid eactions of SH Group
to form the corresponding keto acid and ammonia (see gro!_!Q)and it can form a
Chapter 17). In the body, Glutamic acid is the most
{Qy2teine has a sulfhydryl (S
~ulfid e !§-S) l ~ with gQQ 1h~ ~residue. The two
common amino acid to undergo oxidative deamination.
cysteine residu es can connect two polypeptide chains
© Formation of Carbamino Compound
Carbon dioxide adds to the alpha amjno group of amino
ijCids to form ~ s. The reaction occurs
~-=--.. . .
by the formation of inter.chain disulfide bonds or links
(Fig. 3.20). The
-=~ =-= ~-,

at aJ)5aline eH
and serves as a mechanism for th ce
transport of carbon dioxide from tissues to the lungs by i. ~amf!!!_amino ric acid (GABA , a decivative
hemoglobin (see Chapter 23). of glutannic acid) and dopamine (derived from
Hb-N H +CO -+ Hb-NH --COO H (Carbamino-Hb)
2 2 tyrosine) are neurotransmitters.
Chapter 3: Amino Acids: Structl!lre and Properties 31

1896). This ~ n i m u m of two peptide bonds)


and so i,pdi ·oo acids and ~ l ' ~s wilb
not answer this test. This reaction ca~ sed for

Fig. 3.21 : Peptide bond formation ©


[car l-f~ J which is intensified in strong alkaline medium. This
ii. Histamine (synthesized from histidine) is the reaction causes the yeliow stain in skio by nitric.acid.
mediator of allergic reactions. 4. Millon's test: The phenol group of phenylalanine
iii. Thyroxine (from tyrosine) is an important thyroid and tyrosine containing proteins, when heated with
hormone. C 1"1f.~ CJC:>'f'C\~ f rtJercunc suUate in sui ac1•And soa111ro aifille}
iv. ,.,.,...,~,.,.,., · · ~ntiQper- (or, mercurous and m ric nitrates
culous dryg. aserin · s reactions where amide in nitric acid)form red colored mercyry
groups are added, and so acts a cer dru . phenolate (Millon, 1812- 1867)1. Both
v. Histidine residues are important m the buffering (8anthoprote1c and M1iion's tests are)
activity of proteins{\4'b 'c_,, PlG..~~\-:)
e ative for ta ioca cassava), which
vi. O']!lhl!i:.::e=a!!:!n~d cj~tr;!
7 u!!:Hj~n:::e...:a::.r:.e_:d::;e~ri~v::,at~ives of arginine,
and are essential fo urea s thesis.
s deficient in phenylalanine and Erns1 a lkOWSk'I
5
tyrosine. 1844-1923

PEP 5. Sakaguchi's test for arginine: Free arginine or
arginyl residues in proteins react with alpha-naphthol
Alpha carboxyl group of one amino acid reacts with alpha
and alkaline hypobromite ::!li)ret,bfigtit-red color]
amino group of another amino acid to form a Qeptide
This is ~ he,..g_u~dp ~lP-
bond or CO-NH bridge (Fig. 3.21 ). Proteins are made by
6. Sulfur test for cysteine: When CY.Steine or cystejne-
polymerization of amino acids through peptide bonds.
c,_ontaining protein~ d w~{§trong aliali,
Details of protein structures are given in Chapter 4.
organic sulfur splits and f~ ~,odium 51,lfide, which
on addition of lead aceti e produces lead sulphide
lor Reactions of as a ac re a e Methionine does not answer
Amino Acids and Proteins this test because s methionine is in the
1. Ninhydrin reaction (Ruhemann, 1910) thioether lin age which is difficiult to break. Albumin
Amino acid + 2 molecules of ninhydrin heat and keratin will answer sulfur test positively; but
I
Aldehyde with one carbon atom less + CO2 + purple ~11'.l will give a negative test..
complex. 7. Nitroprusside reaction for SH groups: P
All amino acids when heated with ninhydrin can with free sulfhydryl - ~ ~ ~ ~ i l U - -...,...·~
form complexes; pink, purple or blue in colQ.r. The with sodium nitropru&e-, Jn..i!! m.!!.m
!.!;o
i:t,1.&.1·......,..._.......,........,
color complex is calledf!!unemann's purp@J The Many pmtejns gjye a.L]egativ,e... 1reaction in the native
ninhydrin reaction may be adopted for qualitative sE3te, but when denat11ced, reaction~ ositive,
as well 'as quantitative estim~tion of amino acids. showing the emergence ;;rfree!"SHgroups.
It is often used for detection of amino acids in Color reactions of amino acids are shown as
~romatographu :<f'.::: N~ o.ffk-1') summary in Table 3.6. The amiino acid Selenocys-
2. Biuret reactio~ ~eric ions in an alkaline medium teine is described in Box 3.2. Quantitative estimation
fo!.!JLa vjolet coJo ~eptide bond.nitro~en (Schiff, procedures of proteins are given in Chapter 4 .
BOX 3.2: Selenocyste1ne as the 2 1st amino acid
21st century witnesses the addition of Selenocystelne as the
21st amino acid present in human proteins. An amino acid is
given the individual status, when it is incorporated as such into
proteins during_prote[D biosynthesis, and having a separate
codon. Selenocysteine is present in some enzymes. Instead of SH
3. Xanthoproteic d!st
(sulfhydryl) group in cysteine, SeH (selenium) is present in seleno-
4. Milton's test Phenol (Tyrosine) cyst eine. It is abbreviated as SeCys or SeC. Details are given in
5. Aldehyde test lndole (Tryptophan) Chapter 18, under seri ne. There are about 25 proteins that
incorporate~ ~( ; ) _ _ _ . o _ -
6. Sakaguchi's t est Guanido (Arginine)
Similarly Pyrrolyslne1 Pyl) Is known as the 22nd amino acid.
7. Sulfur test Sulfhydryl (Cysteine) Pyrrolysine is a lysine in an amide linkage to substituted-~ -
8. Nitroprusside test Sulfhydryl (Cysteine) line-5-carboxylate. It is present in methyl transferase enzymes of
fertain bacteria. Both SeC and Pyl are encoded by codons that
9. Pauly's test lmidazole (Histidine)
normally function as stop signals.

LEARNING POINTS CHAPTER 10. Depending on nutritional requirements they may be


es~ tial or nm:,,sseAtial.
1. Most amino acids in the body are alpha amino acids. 11. In solution, amino acids exist as ~ erion ' or
2. Amino acids can be classified based on their (i) 'Ampholytes' at their ch~ uill-
Structure (ii) Side chain characters (iii) M~ ic 12. lsoelectric pH is the pH at which amino acids have
fate (iv) Nutritional requirements. no net charge, no mobility in electric field and are
3. All proteins are mad~ O_;omrnona~ acids less soluble.
linked by peptide bonds. - 13. Each amino acid has a specifiE ctric pH or I
4. Amino acids may ~ classifie~ ased on thw ature of depending on the pK val f ttie 1onzable groups.
side chains into al~ atic, ar~ atic and im1no acids. 14. Amino acids pos ssing an <asymmetric carbon
5. Aliphatic amino acids may be siUf'ple, brani fed chain, atom havE" QPtical activity ~ ~ rail occurring
h~ toxyl, s~ r containing and those with aro(de amino acids are L amino acids.
~

-
s.
6. Acidic amino acids have more than one CDOHgroup
and asic amino acids have more than one amino
group.
7.( Deriveq)amino acids may be found in proteins, or
15. Glycine ~

16. Decarboxylar
involving alpha
_ymmett.ic..carbon atom and there-
fore has no op ical activity.
nd amide formation are reactions
OH group.
they may - ds formed as 17. Transamination, oxidative deamination and forma-
~ ates. tion of carbamino compounds are reactions where
8. A functional classification based on the p.Q!fil_QJ gc04p takes part.
nonpol?r and ionized and non-ionized nature of
side chains is more useful.
9. Based on the metabolic fate, amino acids may be
-
18. Peptide bon¢?zare formed between the
group of ooe amino acid and the
ha amin
ha car
group of the next with elimination of a water mole-
glu~enic, ket~ nic or b~ . cule.
Chapter 3: Amino Acids: Struct,ure and Properties 33

lllll!lftl1'J1111Ce of side chains of amino acids:


a carboxy groups and a amino groups in proteins are unavailable as they form pepti(je
bonds; nature of side chains determine physical properties, including protein folding.

onpolar Uncharged polar Acidic side Basic sidJ


side chain side chains chains chains
Alanine Asparagine Aspartic acid Arginine
Glycine Cysteine Glutamic acid Histidine
lsoleucine GlutamineS'f Lysine
Leucine Serine C
Methionine Threonine f'I~
Phenylalanine Tyrosine --r
Proline
Tryptophan
Found on the outside of proteins that function in an aqueous
Valine
environment and in the interior of membrane-associated
proteins

Found in the interior of proteins that function in an aqueous environment


and on the surface of proteins (such as membrane proteins) that interact with lipids

,.. PART-1: ESSAY AND SHORT NOTE QUESTIONS

3-1 . Classify amino acids, giving suitable examples.


3-2. How will you classify amino acids based on their nutritional importance?
3-3. Define isoelectric pH and give the importance.
3-4. Enumerate different reactions of amino acids, giving one example for each.
3-5. Give an account of the transamination reactions. Give two suitable examples. What is the nnetabolic importance
of transamination? What is the clinical application of transaminase estimation?

SHORT NOTE QUESTIONS


3-6. Decarboxylation of amino acids. 3-8. What are essential amino acids?
3-7. Name the branched chain amino acids.

PART-2: MULTIPLE CHOICE QUESTIONS

3-1 . One of the amino acids listed below is not basic: 3-4. All the following are branched chain amino acids,
A. Arginine B. Histidine except:
C. Glutamine D. Lysine A. Valine B. Leucine
3-2. Which of the following amino acids has a hydroxyl C. lsoleucine D. Thre,onine
3-5. All the following are sulphur containing amino
group?
acids, except
A. Valine B. Threonine
A. Cysteine B. Methionine
C. Leucine D. Histidine C. Homocysteine D. Threonine
3-3. All the following amino acids are acidic, except: 3-6. Guanidinium group is present in:
A. Aspartic acid B. Tyrosine A. Arginine B. Tryp,tophan
C. Glutamic acid D. Threonine C. Histidine D. Prol1ine
34 Section A: Chemical Basis of Life
3-7. lndole ring is present in: 3-16. Nonessential amino acids:
A. Arginine B. Tryptophan A. Are not seen in tissue proteins
C. Histidine D. Proline B. Could be synthesized in the body
3-8. The following amino acids have hydrophobic side C. Have no role in body metabolism
chains, except: D. Have aromatic ring structure
A. Tyrosine B. Lysine
3-17. At isoelectric pH, the amino acids and proteins
C. Leucine Valine
show:
3-9. lmidazole ring is present in:
A. Maximum net charge
A. Arginine B. Tryptophan
C. Histidine D. Proline B. Maximum mobility in electric field
3-10. Pyrrolidine group is present in which of the fol- C. Maximum precipitability
lowing amino acids? D. Minimum buffering action
A. Arginine B. Tryptophan 3-18. This amino acid has 2 optically active centers (asy-
C. Histidine D. Praline mmetric carbon atoms):
3-11. Which of the following amino acids has a non- A. Glycine B. lsoleucine
polar side chain? C. Valine D. Tryptophan
A. Serine B. Valine 3-19. Enzymes are activated by phosphorylation of
C. Asparagine D. Threonine
which amino acid residue?
3-12. All the following are glucogenic amino acids, except.
A. Cysteine B. Serine
A. Glycine B. Serine
C. Glutamic acid D. Lysine
C. Leucine D. Aspartic acid
3-20. The amino acid which has maximum buffering
3-13. Which of the following amino acids has a net
positive charge at physiological pH? capacity at physiologic pH is:
A. Glutamic acid B. Lysine A. Arginine B. Alanine • I

C. Valine D. Leucine C. Histidine D. Glutamic acid


3-14. Which is the ketogenic amino acid? 3-21 . Which does not undergo decarboxylation?
A. Alanine B. Glutamic acid A. Tyrosine B. S-adenosyl methionine
C. Leucine D. Aspartic acid C. Dopamine D. Histidine
3-15. All the following are essential amino acids, except. 3-22. The protein which does not answer aldehyde test:
A. Tyrosine B. Lysine A. Hemoglobin B. Albumin
C. Valine D. Phenylalanine C. Casein Gelatin

ANSWERS OF MULTIPLE CHOICE QUESTIONS

3-1 . C 3-2. B 3-3. D 3-4. D 3-5. D 3-6. A 3-7. B


3-8. B 3-9. C 3-10. D 3-11 . B 3-12. C 3-13. B 3-14. C
3-15. A 3-16. B 3-17. C 3-18. B 3-19. B 3-20. C 3-21 . C
3-22. D

PART-3: VIVA VOCE QUESTIONS AND ANSWERS

;
3-1 . What are branched chain amino acids? 3-7. Which amino acids have hydrophobic side
Valine, Leucine and lsoleucine. chains?
3-2. What are hydroxy amino acids? Valine, Leucine, lsoleucine.
Serine and Threonine. 3-8. Give the names of aromatic amino acids. •
3-3. Name the sulphur containing amino acids. Phenylalanine and Tyrosine.
Cysteine and Methionine. 3-9. What are heterocyclic amino acids?
3-4. Name the acidic amino acids. Tryptophan and Histidine.
Aspartic acid and Glutamic acid. 3-10. Give an example of an imino acid.
3-5. What are the basic amino acids? Praline
Lysine and Arginine. 3-11 . Give examples of derived amino acids.
3-6. Which amino acid contains thioether bond ? Hydroxyproline, hydroxylysine. omithine, citrulline, homo-
Methionine. cysteine.
Chapter 3: Amino Acids: Structure and Properties 35

3-12. Arginine contains which special group? 3-30. Which amino acid has maximum buffering capacity
Guanidinium group at physiological pH?
3-13. Benzene group is present in which amino acid? Histidine.
Phenylalanine 3-31. Which amino acid is opticall}r inactive?
3-14. Phenol group is present in which amino acid? Glycine.
Tyrosine 3-32. What are the isomeric forms of amino acids?
3-15. Tryptophan contains what special group? D and L varieties.
3-33. What are natural amino acids,?
lndole group
Only L amino acids are seen in nature.
3-16. Which special group is present in Histidine?
3-34. What is decarboxylation of a1n amino acid?
lmidazole group.
The carboxyl group is removed from the amino acids
3-17. Name some hydrophobic amino acids.
to form the corresponding aminie.
Valine, Leucine and lsoleucine.
3-35. Give examples of decarboxylation reactions.
3-18. Pyrrolidine is present in which amino acid?
Histidine to Histamine; Tyrosine to Tyramine; Trypto-
Proline
phan to Tryptamine.
3-19. Hydrophobic bonds are formed in protein between
3-36. What is produced when glutamic acid is decar-
which amino acids?
boxylated?
Valine, leucine and isoleucine residues.
Gamma amino butyric acid or GABA.
3-20. What is the basis of classification of amino acids
3-37. What is glutamine?
into ketogenic and glucogenic?
Thal is the amide of glutamic aieid.
Ketogenic amino acids will enter into the metabolic
3-38. What is an amide?
pathway of fats, while glucogenic amino acids enter the
The extra carboxyl group (othor than alpha carboxyl)
pathway of glucose metabolism. can combine with ammonia to lform the corresponding
3-21 . Name a purely ketogenic amino acid. amide.
Leucine 3-39. What is transamination?
3-22. Name some glucogenic amino acids. The alpha amino group of amino acid can be transferred
Glycine; Serine; Aspartic acid. to alpha keto acid to form tile corresponding new
3-23. What are essential amino acids? amino acid and alpha keto acid .
They can not be synthesized in the body; and so, they 3-40. Give an example of transami1nation reaction.
are to be provided in the diet. Glutamic acid + pyruvic acid to alpha keto glutarate +
3-24. How many amino acids are essential? alanine
Eight amino acids are essential; two are semi-essential 3-41 . What is the biological signi1ficance of transamin-
and the rest 10 are nonessential. ation reaction?
3-25. Name any three essential amino acids. These are important for the int,er-conversion of amino
lsoleucine, Leucine, Threonine. acids. Nonessential amino acids are synthesized by
3-26. Is phenylalanine an essential amino acid? this process.
Yes. 3-42. What is the clinical significance of transami-
3-27. What about Tyrosine? nases?
Tyrosine is nonessential. Transaminases in blood are ele1vated in liver and heart
3-28. What is isoelectric point? diseases.
The pH at which the molecule carries no net charge is 3-43. What is the significance of SI-I groups in proteins?
called isoelectric point. The SH group of cysteine can lform a disulphide (S-S)
:; 3-29. What are the characteristic features of iso-electric bond with another cysteine residue. The two cysteine
pH? residues can connect two polypeptide chains by the
At isoelectric point the amino acid will carry no net formation of interchain disulphide bonds.
charge; there is no mobility in electrical field. Solubility 3-44. Phosphorylation is taking pllace on which amino
will be minimum. The tendency for precipitation will be acid?
maximum. Serine and Threonine.
_ _ _ _ _ Chapter 4
Proteins:
Structure and Function

Chapter at a Glance
The learner will be able to answer questions on the following topics:
A D Peptide bonds B D Sequence analysis (study of primary structure)
c Primary structure of proteins B D lsoelectric pH of proteins
C D Secondary structure e. D Precipitation reactions of proteins
C, D Tertiary st ructure Classification of proteins
C.., D Quaternary structure .,gl<'b Quantitative estimation of proteins

The word protein is derived from Greek word, "proteje_~" 0-50 amin acids is a peptide. By convention,
which means~ . As the name shows, the proteins peptide chains containing uiore than 50 aQJ,ino
are of paramount importance for biological systems. acids are called proteins~ Y
Out of the total dry body weight, ~ f In a tripeptide , there are 3 amino acids, but these 3
proteins. Proteins are used for body building; all the can be any of the total 20 amino acids. Thu6203 = 8000
maj~tructural and functional aspects of the body are different perro11ta1tjons ang carobinations..~ possible
carried out by protein molecules. Abnormality jn protein in a tripeptide. An ordinary protein having abour@_o
structure will lead to molecular diseases with profound amino acid~, will have f20100 different poss1b1~ . This
alterations in metabolic functions. number is more tt1an the total number of atoms present
Proteins contain Carbon, Hydrogen, Oxygen and in the whole univeirse. Thus, even though there are only
<_cjf>~ Nitrogen as the major components while Sulfur and Phos- 20 amino acids, by changing the sequence of com-
phorus are minor constituents. Nitrogen is characteristic
of proteins. On an average, the nitrogen content of
ordinary proteins is 16% by weight. All proteins are
polymers of a~ino acids. 1 rn~.,,.t,:;iil,
® ~lneAolds.,. Linked by Peptide Bond
Alpha carboxyl group of one amino acid reacts with Proteins have different levels of structural organization;
alpha amino group of another amino acid to form a primary, secondary, tertiary and quaternary. oJriv--J-
peptide bond o r CO-NH bridge (Fig. 4.1 ). (_ S"(ff) qrq~•.•ffl
Proteins are synthesized by polymerization of amin'I:? Primary Str FtU e ®
acids through peptide bonds. Two amino acids combined Sequence of mino Acids in.Proteins
to form a dipeptide; three amino acids form a tripep-
tide; four will make a tetrapeptide; a few amino acids Protein structure is studied as the primary, secondary,
together will make an oligopeptide; and combination of tertiary and quatemary levels (B9x 4.1). Primary structure
Chapter 4: Proteins: Structure and Function 37

BOX 4 .1: Oef1nit1ons of levels of organization


NH2-CH~ )OH
1. Primary structure of protein means the order of amino acids
in the polypeptide chain and the location of disulfide bonds...,
I -
if any. -
R1
2. Secondary structure is the steric relationshw of amino acids,
close to each other.
3 . Tertiary structure denotes the a,verall arrangement and
inter-relationshi of the v,~,ti,! W+~05 QJ;,.ciQaieins of a
n po ypept1aec
4. Quaternary structure results when the proteins consist Fig. 4.1: Peptide bond formation
of two or more pol)(pep•ide chains held toget her by non-
cov lent for .
H 2N -- CH--CO-NH--CH--CO-NH-CH- COOH
I I I
Amine, terminal Carboxy terminal

Fig. 4 .3: End groups of polypeptide chain

Fig. 4.2: Peptide bond is a partial double bond (T°'oM) terminal (N-1terminal) end and the amino acid contri-
,~1',e.QQ -=- buting the alpha-amino group is named as the first
,.
denotes the number and sequence of amino acids amino acid. (Fig. 4.3). Usually the N-terminal amino
in the protein. The higher levels of organization are acid is written on the left hand side when the sequence
decided by . the primary structure. Each polypeptide of the protein is denoted. Incidentally, the biosynthesis
chain has a unique amino acid sequence decided by of the protein also starts from the amino terminal end.
the genes. The primary structure is maintained by the The other end of the polypeptide chain is the carboxy
covalent bonds of the peptide linkages (Fig. 4.1). terminal encl (C-terminal), where there is a free alpha
Students should have a clear concept of the term carboxyl group which is contributed by the last amino
"sequen ". S foll~ example: acid (Fig. 4.3,). All other alpha amino and alpha carboxyl
Gly - Ala - Val (1) groups are involved in peptide bond formation. Amino
GI - Val - Ala (2) acid resi eptides are named by changing
Both the tripepti es shown above contain the same the suffi '-in,e" to "- I' .g. Glycine to Glycyl.
amino acids; but their sequence is altered. When the H2-Gly-Ala-Val-COOH
sequence is changed 1 the peptide is also
different. In the above example, the amino group of glycine is
free; but carboxyl group of glycine is bonded with amino
Characteristics of a Peptide Bond group of alanine; the carboxyl group of alanine is, in
The peptide bond is a partial double turn, bonded with the amino group of valine; while the
bond. The ~N §ond ~s 'trans) in carboxyl grou of valine is fr e. Therefore this peptide is
nature andh ere i? ' no fw dom._gf named as €!;{c I-alan I-valin It is abbreviated as Gly-
rotati9n because of the partjai ctouble Ala-Val, or. simply as GAV.
bond character (Fig. 4.2). The distance
is 1.32A which is midway between single Branched and Circular Proteins
bond (1.49A) and double bond (1.27A). GN Generally th1:! polypeptide chains are linear. However
The S9! chains are free to rotate on Ramachandran branching points in the chains may be produced by
either side of the peptjde bond. The 1922-2001 ihterchain disulphide bridges. The covalent disulphide
angles of rotation, known as amachandran angles, bonds betwe,en different polypeptide chains in the same
therefore determine the spatial orientation of the peptide protein (i~:£bfil_~ 'f'rl'"'"""'"Lftions of the same ol e -
chain. (Dr GN Ramachandran diq pioneering work on the tide chain (infrachain re art of the rima structur
structural aspects of proteins during 1~50s and 1960s). · Rarely~ rrsteatr of the alpha COOH group, the
gamma carlt>oxyl group of glutamic acid may enter
Numbering of Amino Acids in Proteins into peptide bond formation, e.g. Glutathione (gamma-
In a polypeptide chain, at one end there will be one glutamyl-cysteinyl-glycine) (see Chapter 18). The term
free alpha amino group. This end is called the amino pseudopeptide is used to denote such a peptide bond
40 Section A: Chemical Basis of Life

H H O H H 0 Primary structure
I I R 11 I I R II
-..... N ....
· •c,..··c ..-.,..N , !
C'
... C ... _,....N
'N ·· i . C
, i _, C ...
C ...
is determined by the
sequence of amino acids

?
I R II I I R II I
~ ~ A1p~
helix Secondary structure
H ; 0 H .,_. occurs when the amino
0 H H 0 H H acids are linked by
II R I
.. c .. , _. N.. , c ,
I II R I
_.,....C. f .. N......_
I
C / y hydrogen bonds

/ · '·c·· ··c ! N ···c··· C' ! ' W


I II R I I II R I Tertiary structure


H 0 H H 0 H is formed when alpha helices
and beta sheets are held
Fig . 4 .7: Structure of beta-pleated sheet together by weak interactions

Quaternary structure
chain. Heterodimer contains two different types of poly- consists of more than one
peptides as a functional unit. For example, 2 alpha-chains polypeptide chain

and 2 beta-chains form the fiiimoglobm molecule .


. Similarly, 2 heavy chains and':rii ~t chains form one Fig. 4.8: Levels of organizations of proteins
-I molecule ofdmmu o uli . rea ine inase
is a dime ~te debydrogeoa is a tetramer. TABLE 4.1: Specific structural motifs in common proteins
Protein Structural motif present
Structure-function RelationshitJ Myoglobin Alpha helix and beta pleated sheet
The functions of proteins are maintained because of Collagen Triple helix
their ability to recognize and interact with a variety of Keratin Coiled coil I
molecules. The three-dimensional str tural conformation Elastin No specific motif I
provides and maintains the functi al charact · ics. · Superoxide dismutase Antiparallel beta pleated sheet
The three-dimensional structure, in tu , i~ dependent on
the primary structure. So, any difference in the primary one heme facilitates oxygen binding by other subunits.
suuc\ure may produce a protein which cannot serve its of H• a ep promotes rel,e,ase o~ rqm
Q .P-"f uncytm. To illustrate the structure-function relationship, hemoglobin. Thi os · ·nter · is physiologically
-:> ,>,tflefollowing three proteins are considered; each belongs im ortant and is termed as O . Even a single
'5,;. 7 to a different class in the functional classification. amino acid substitution alters the structure and thereby
f;. \ Enzvmes the function, e.g. in sickle cell anemia (HbS), the 6th
\JJ T' amino acid in the beta chain is altered, leading to
The first step in enzymatic catalysis is the binding of the profound clinical manifestations.
enzyme to the substrate. This, in turn, depends on the ( )
str,\l.c tural cooforma!ion of the acti~ e of the enzyme,nu.u-... ctura/ Protein t;r""®
which is precisely oriented for substrate binding Collagen is the ~ in in mammals and
(see Chapter 5). Carbonic anhydrase catalyses the is the main fibrous component of skin, bone, tendon,
reversible hydration of carbon dioxide. This enzyme cartilage and teeth. Collagen forms a superhelical cable
makes it possible for the precise positioning of the CO2 where the 3 polypeptide chains are wound around
molecule and the hydroxyl (OW) ion for the formation of itself (see Chapter 49). In collagen , eveQ'. 3rd,r_~ue is
bicarbonate ion. The enzyme has a zinc ion located at a glycine. The only amino acid that can fit into the tripTe
a deep cleft co-ordinated to bjstjdjne residues. The CO2 sti:andecLheJix js..glyc il;ie. Replacement of the central
binding residues are very near to the zinc ion. Water glycine by mutations can lead to~brittle bone disease)
bi~ ta.zine-ion.,..gets ionized to liY9roxyl ion and it binds The triple helix of collagen is stabilized by the steric
to the CO2 w hich is proximally located . The substrates repulsion of the rings of hydroxyproline and also by the
are bco.ugbtln_,clos.roximity..foube reaction to pro~ ed. hydrogen bonds between them. In vitamin C deficiency,
/-ct\.
0 ~ QD of Rrolinell,-sine leads tQ.I.e.QJJced

-
Transport Proteing hydro9en bonding and consequent weakness of collagen
Hemoglobin, the transporter of oxygen is a tetrameric (see Chapter 49). The quarter-staggered triple helical
protein (alpha 2, beta 2), w ith each monomer having structure of collagen · oonc.ibJo.Jc · ,;,.~;•1" , __ _ __
Chapter 4: Proteins: Structure and Function 43

BOX 4.4: S1gnif1cance of 1soelectnc pH (pl) CWrecipltation by Heavy Metal Ions


1. The amino acid composition will determine the isoelectric In alkaline medi1101, proteins have net negative charge,
pH (pl) of protein. The alpha amino group and carboxyl or are anions. To such a solution, if salts of heavy metals
group are utilized for peptide bond formation, and hence
are not ionizable. Ail other ionizable groups present in the
protein will infiuence the pl of the protein.
are added, positively charged metal ions can complex
with protein molecules and metal proteinates are
W
2. At the isoelectric point, the number of anions and cations precipitated . Salts of Copper, Zinc, Lead, Cadmium Zn
present on the protein molecule will be equal and the net and Mercury are toxic, because they tend to precipi- Pb
charge is zero. tate normal proteins t gastrointestinal wall. Based
3. At the pl value, the proteins will not migrate in an electrical
field. At the pl, solubility, buffering capacity and viscosity will
on this rinciple, w egg is sometimes used as an cd
be minimum; and· · · · · · · · - tido Ma,
4. On the a0!!L5ide of et. .the. proteins are cations and on . . lJ
_:l~~e : : e,Jhe;'..3, tions.in nature. recipitation by Alkaloidal Reagent
5:-ilie p o sein 4.6; human albumin 4.7; human ' ac1'd , ph osp h otungst'1c ac1'd , t nc
Tungst 1c · hi oro ac!.!.
t·c
insulin s._4; human l lobulins 6.4; human hemoglobin 6.7;
myogIo bin 7; , , "'""'· acid, picric acid, sulfosalicylic acid and tannic acid are
6. Acidic dyes such as eosin will dissociate into H. + dye· powerful protein precipitating agents. These acids lower
which will then attach with protein- NHJ (protein.cations). the pH of medium, when proteins carry net positive
Basic dyes such as E'einatoxylin and methylene are charges. These p ~ "2!ions....are ~e.d...Jttith
dissociated to OH. + dye-, which will then stain Protein- n ati O rm otein-tun state.~ -
coo (anions). Thus, the staining characteristic of a protein is ' . . . . .
determined by the pl of that protein. ~. etc., and thrclu)occu!ent pre~te 1s formed. In
A o .. ._ . . clinical lab.oratory phosphotungstic or trichloroacetic acid
S If O t T'C)I' ,o :"""-9 u.r-A \,eln are usually used for precipitating proteins. Tanning in
a mg U ~'1)0.n<Y\. o!>tP leather processing is based on the protein precipitating
When a neutral salt, such as ammonium sulfate or effect of tannic acid. Under certain conditions, proteins
sodium sulfate is added to protein solution, the shell undergo denaturation , which is a mild form of precipitation
of hydration is removed and the protein is precipitated. reaction (Box 4.5). Heat coagulation is an irreversible
Thi salting out. As a general rule, hjghru: itation process (Box 4.6 and Fig. 4.13).
th r wei ht of a rotein, the salt re uired for
INS
It is almost impossible to correctly classify all proteins.
The following classifications are given only to introduce
a broader idea to the students. ) lM
. . . ( S'°m :.<9fl_.J
Proteins are le~ so.J.u e at their isoelectric pH. lass1ficat1on Based on Funct1om;
p"
~: =~~~in
So_
me proteins_ ~re pre~ipitated immediately wh~n 1. Catalytic proteins, e.g. enzymes

redissolves in highly acidic or alkaline solutions. When 4. Transport proteins, e.g. hemoglobin, myoglobin, albu-
milk is curdled, the casein forms the white curd, because min, transferrin

the 8.t:J tg t1Cf


lactic acid produced by the fermentatjon pcacess lowers
ectrrc po[DU>f casein.
5. Regulatory proteins or hormones, e.g. ACTH, insu-
lin, growth hormone
6. Genetic proteins, e.g. histones
recipitation by Organic Solvents 7. Protective proteins, e.g. immunoglobulins, inter-
When an organic solvent is added to the protein solution, ferons, clotting factors.
water molecules available for proteins are reduced, B
and precipitation occurs.~ _ _ _ ! } reduce the lassification Based on Composition
d ~ n t of the medium which also favors and Solubility
protein precipitation . Hence@coh oJ)s a eowerful protein Simple Proteins
pcacipitatjng agent. This may explain tlie d1sinfectagt
effect of alco6Q.L > According to definition, they contain only amino acids.
44 Section A: Chemical Basis of Life

BOX 4.6 : S1gnif1cance of heat coagulation


Mild heating, treating with urea, salicylate, X-ray, traviolet When heated at isoelectric point, .,some proteins will dena-
rays, high pressure, vigorous shaking and similar physico- ture.irreyersi~ to produce ~i.~k f u @ i ~ e s called
chemical agents produce denaturation. coagulum. This process is calledheatcoagulation. Albumin is
There will be nonspecific alterations in secondar tertiary easily coagula sser extent. (Fig. 4 .13). This
and quaternary structures of protein molecules. Primary is the basis o
structure is not altered during denaturation (Fig. .12). hapter 25).
3. In general, during the process the solubility is creased
while precipitabi lity of the protein is increased. It often
causes loss of biological activity.
4. Nat ive proteins are often resistant to proteolytic nzymes,
but denatured proteins will have more exposed sites for
enzyme action. Since cooking leads to denatu ation of
proteins, cooked foods are more easily digested.
5. Denatured proteins are sometimes re-natu red hen the
physical agent is removed-Fonucleafe. is a good example
for such reversible denaturation. lmmunoglobul n chains
are dissociated when treated with urea. When he urea Fig. 4.13: Heat coagulation. On heating, liquid white portion of
is removed by dialysis, the subunits are reassoci ted and egg becomes solid white coagulum
biological activity of immunoglobulin is regained.
6. But many proteins undergo irrev~rsible denatur ion. For but lack in lysine, e.g. zeio trgm co.I.n , gljadio of
example, albumin cannot be renatured by rem ing the
physical agent.
wheat, hordein of badey.
v. Lectins: Lectins are precipitated by 30-60%
ammonium sulfate. They are proteins having high
affinity to sugar groups. Phytohemagglutinin (PHA),
a lectin from P1:J.rJ3eo1t1s vulgacis (red ki~ an)

..
agglutinates all RBCs and WBCs.
vi. Scleroproteins: They are insoluble in water, salt
solutions and organic solvents and soluble only in
Renal ration:
hot strong acids. They form supporting tissues,
native form is e.g. collagen of bone, cartilage and tendon; keratin
regen rated of hair, horn, nail and hoof.
Denatured protein;
Random coil structure; A ~ e ar apwt:
rl)y:proteiri
function is lost. Primary slrticture i intact.
Conjuga ed Proteins ( '3~~ h
is funct,onai.,:;-
They are combinations of protein with a non-protein part,
Fig. ~~\;,\\ -· called prosthetic group (Table 4.2). Conjugated proteins
may be classified as follows:
i. Albumins: They are soluble in water a d coagu- i. Glycoproteins: These are proteins combined
lated by heat. Human serum albumin ha a mole- with carbohydrates. (8ydroxyl groups of serine or
cular weight of {>9 000 e.g. lactalbumin o milk and threonin~ and Gmide groups of asparagine and
egg albumin. glutamine)orm linkages with carbohydrate residues.
ii. Globulins: These are insoluble in ater, but .§1,Qsd gr u12 and many serum proteins
soluble in diluw salt sofiiffiins. They are also a re glycoproteins. When the carbohydrate content
coagulated by heat, e.g. ei;m globuli , .§filld..m is more than 10% of the molecule, the viscosity is
Q.!Q.b.!Jjins, le.9umin of peas. correspondingly increased ; they are sometimes
Protamines: These are soluble in wat known as mucoproteins or proteoglycans.
acids. They ar not coagulated by hea ii. Lipoproteins: These are proteins loosely combi-
contain large n . ned with lipid components. They occur in _Qjood an.9
dues, and so are strongly b c. Hence they can cell membranes. Serum lipoproteins are des-
combine with other acidic proteins . .._
P_, ro....t,;;+...,.,.-.....-.~ cribed in Chapter 14.
is a common commercial pie iii. Nucleoproteins: These are proteins attached
insu lin. to nucleic acids, e.g. Histones. The DNA carries
iv. Prolamins : They are soluble in 70-80°/4 alcohol, negative charges, which combines with positively
but insoluble in pure water. They are rich in praline charged proteins.
Chapter 4: Proteins: Structure and Function 45

Heme
Histones DNA
Opsin 11 -cls-retinal
uccinate dehydrogenase Protein Riboflavin as FAD
Apoferritin Iron
Poor Proteins
Apoceruloplasmin Copper They l~ k in ma.2-Y essential amino acids and a diet
based on these proteins will not even sustain the original
iv. Chromoproteins: These are proteins with colored body weight. Zein from corn lacks tryptophan and lysine.
prosthetic groups. Hemoglobin (Heme, red); Flavo-
proteins (Riboflavin, yellow), Visual purple (Vitamin l ologically lmpo!1ant Pe~~tides (_Sm) c~
A, purple) are some examples of chromoproteins. When 10 or less number of amino acids are joined
v. Phosphoproteins: These contain phosphorus. together, it is called an oligopeptide. Some of them are
Casein of milk and vitellin of egg yolk are exam- biologically active. A few examples are given below:
pies. The phosphoric aci!i is e~!Q.!tle~ / 4._i. Thyrotropin releasing hormone (TRH) is a
gro~ Q serlQ_e.pnd threQOi._n.e residues of proteins . .::J... ~ ith th ce of Glu-His-Pro; but the
vi. Metalloproteins: They contain metal ions. Exam- I and Pro ar · #
pies are Hemoglobin C!f_an), C~ ochrome Ocon), Txro- ii. Glutathione is a'W~JW-111!:- It is gamma glutamyl
® s~ ~pey,;and Carbonic anhydrase (Zinc). cysteinyl glycine (see Chapter 118). It is involved in
~ ~emb(QneJnteg£ih' and is import
Derlvea Proteins keeping enz mes in active state:.

.. They are degradation products of native proteins. Pro-


gressive hydrolysis of protein results in smaller and ~
smaller chains: Protein peptones peptides
iii. xytocin and vasopressin (ADH) are n o-
peptides; with 9 amino acids. They are secreted by
posterior p i t u i ~
ino acids. iv. Angiotensin I ha 10 mino acids and Angiotensin
A _.. II h~ mino acids. They are pressor agents; they
assification Based on Shape ' ~ate blood pressure. (see Chapter 28).
Globular Proteins Polypeptide hormones (more than 1O amino acids)
escribed in Chapters 11 and 45 .
They are spherical or oval in shape. They are easily
soluble, e.g. albumins, globulins and protamines.
ANTITATIVE ESTIM JION - - -
Fibrous Proteins ® Kjeldahl's Procedure
The molecules are elongated or needle shaped. Their
solubility is minimum. They resist digestion. Collagen, The protein sample is digested by boiling (360°C) with
lastin and keratins are examples. concentrated sulfuric acid in presence of copper sulfate
D and sodium sulfate as catalysts. The nitrogen present
lassification Base.: ~~.:---...:-·utritional Value in the protein is reduced to ammoinia. The liberated
Nutriti9'1r{IY Rich Pr,..,_,..."' (r.avi~ ammonia can be calculated. Thein the quantity of
nitrogen originally present in the protein is assessed.
They a~ o called as complete proteins or first Since proteins, on an average contain 16% nitrogen,
• class proteins. They contain all the essential amino the weight of nitrogen x 100/16, or nitrogen x 6.25 will
acids in the required proportion. On supplying these give the value of proteins present in the original sample.
proteins in the di t, children will grow satisfactorily. A
good example is asein of m"w Advantage
Incomplete Proteins This is the most accurate and precise method. It is
generally used for standardizing a particular protein;
They lac~ ssential amino acid. They cannot that protein is then used for calibrating other proteins
promote ~ wth in children; but may be able to employing other easier methods.
46 Section A: Chemical Basis of Life

Disadvantage
It takes many days to get the result, and is unsuitable for
routine clinical work.

® Bfuref Method Johan Kjeldahl Oliver Howe Richard A Zsigmondy


Cupric ions chelate with peptide bonds of proteins in alka- 1849-1900 Lowry NP 1925
line medium produce a pink or violet color. The intensity 1910-1996 186fr1929
of the color is proportional to the number of peptide
bonds. The color is then compared with a standard pro- albumin is to be estimated, specific antibody against
tein solution treated with the biuret reagent, and esti- albumin is added . The resultant antig1:in-antibody-com-
mated colorimetrically. The principle of colorimetry is plex will form turbidity of the solution. A beam of light
discussed in Chapter 31. (preferably laser beam) is passed through the solution .
The particles in the solution will scatter light. The emer-
Advantage gent scattered light will be proportional to the turbidity
The biuret method is simple one step process, and is the of t_
he solution, which in turn will be proportional to the
most widely used method for plasma protein estimations. antigen.

Disadvantage ©turb lrn.a.,•ru


Nephelometry and turbidimetry are baised on the same
The sensitivity of the method is less and is unsuitable for
principle of scattering of light by coll,oidal particles. In
estimation of proteins in mil ligram or microgram quan-
nephelometry, emergent light scattere:d at 60° angle is
tities.
observed; while in turbidimetry, light ,emerging at 180°
@ Lawry's Metftod angle is detected. Turbidimetry is comparatively cheap-
er. In lmmunoturbidimetry, the particular protein is pre- ..
This is based on the reduction of Folin-Ciocalteau phenol cipitated by specific antibody, and the resultant turbidity
reagent (phosphomolybdic acid and phosphotungstic is measured. The method is very specific, simple, easy
acid) by the tyrosine and tryptophan residues of protein. to perform and cheap.
A blue color is developed, which is compared with that
produced by a known standard. @ ELISA Ta&t
Advantage If proteins of nanogram and picogram quantities are to be
estimated, enzyme-linked immunosorb13nt assay (ELISA)
This method is very sensitive and protein content in
technique is to be employed. This is described in detail
microgram range can be measured.
lilt" in Chapter 31 .

Spectrophotometric Estlmatio <SJ Proteomics


Proteins will absorb ultraviolet light at 280 nm. This
Proteomics is the study of the entire 9alaxy of proteins
is due to the tyrosine and tryptophan residues in the
produced by a cell under different conditions. At a
protein. Quantitation is done by comparing the absor-
particular time, a gene is "on" in a particular cell; but
bance of the test solution with a known standard.
it will be "off" in another cell. Expre:ssion of proteins
during growth and development will be different from
Advantage
the resting cell. Proteins produced by a gastrointestinal
The method is accurate, simple and highly sensitive up cell and a neuronal cell will be entire·ly different. Many
to microgram quantities. proteins undergo post-translational modification, that
• Nephelometry too, at different levels at various organs. But genes are
the same in all cells at all times. Therefiore study of genes
It is based on the measurement of scattering of light by (genomics) will give only a partial picture of what is going
colloids, originally studied by Richard Zsigmondy (Nobel on in nature. Even though DNA determines the basic
Prize, 1925). Nephelometry is defined as the detection genetic structure of an organism, it is the protein which
of light scattered by turbid particles in solution. If actually carries out the body functions. Proteomics aims
·.· ~c...0"\0lN'i @
Chapter 4: Proteins: S fucture and Function 47
at studying the protein structure and function. Human the chains togethe ~..,;.;--ie::-·oWJs dis, 11fide liA-
body contains hundreds of different cells, which express kages between two pgjymmt;de cbaios i11 oligomeric
thousands of proteins, at different times and under the proteins.
influence of different stimuli. Proteomics attempt to study 11 . Secondary and tertiary levels of structure are
this multifaceted picture in toto. maintained by noncovalent bonds.
t
12. The noncovalent bonds maintaining the higher
LEARNING POINTS, CHAPTER 4 levels of structure are hydrogen bonds, ionic bonds
(electrostatic bonds), hydrophobic interactions and
1. Polymers of amino acids linked by peptide bonds vander Waal's forces between the side chains of
are callec@lypeptid~ . amino acids.
2. Peptide bonds are covalent bonds between C=O 13. The two major types of secondary structure are
and NH 2 groups having a t_cag_s configuratipn. alpha helix and beta pleated sheet structure found
3. Depending on the number of amino acids, oligopep- in fibrous proteins. A beta-pleated sheet may further
tides (10), polypeptides (10-50) and proteins (>50) be parallel or anti-parallel.

==·
are fo rmed. 14. Tertiary structure of a protein is the most thermo-
4. Nitrogen ,a~ tent of ordinary proteins is on the s;iv,pamical1Y,,,5.tab~ configuration.
1
average ~ /o by weight. 15. Quaternary structure is present only in certain pro-
5. Protein structure can be defined and studied at more than one polypeptide chain eg.
four levels viz. Primary, Secondary, Tertiary and
Quaternary. 16. Th~ tion of a protein is dependent on subunit
6. Proteins have the primary level of structure which interaction.
denotes the linear sequence of amjno acjds linked 17. Chemical properties of the proteins depend on the
by peptide bonds. colloidal nature of the particles in solution and the
7. The primary sequence is ge,petjcaUy determined @ lure of the side cha1Q);. This explains the precipi-
tation reactions and color reactions of proteins.
• and is unjgue and fixed f~ n produced
18. Solubility of a protein is dependent on the ionic
by a particular species of organism.
8. Primary structure determines the biological activity concentration of the medium. Hence, proteins may
be 'saJ!si.dJn.'_or_'sal!e.d...o.ut'.
of the protein. Alterations lead to loss of functional
19. Denaturation of protein results in l,9.s s of bjolog~ I
capacity. E.g . Si9Js!.e cell hemoglobin (HbS).
~ ty primary structure. Denaturation
9. The N terminal a~ o acid is the 1st amino acid
may be r · .
having a free ajgJJ§. NH2 gmup and t,ie C terminal
20. Proteins can be classified based on (i) Function)
amino acid, (the last amino acid) ha~ free alph~
(ii) Composition (iii) Shape and (iv) Nutritional value.
COOH group .
21 . Methods of protein estimation include~ orime
10. A protein having more t ne polypeptide chain, (Biuret and Lowry's metbo~ nd enzyme linked
like insulin, has interch
immunosorbent assa (ELIS~ l _ ... _.,

~-
Q~'c.c":~~o.,

PART-·J: ESSAY AND SHORT NOTE QUESTIONS


)
4-1 . How does the c harge on a protein molecule vary with t he change in pH of the medium? ~QJJ.·r. ~CU,
V "o.. ~~it.,
:c,.,
4-2. What is end group analysis? What are the reagents used for this purpose?
4-3. Describe the primary, secondary and tertiary structures of proteins. What are the forces , which stab! ze hem? 1 ' ,
4-4. What is the primary structure of a protein? Explain with the help of the structure of Ins ulin.
' 4-5. Name two common types of secondary structures. Mention how they are preserved.
4-6. Name two proteins with quaternary structure. Indicate their subunit make up. How are they maintained?
4-7. Explain the structural organization of hemoglobin molecule. How does the alteration in amino acid sequence
affect the properties of hemoglobin?
4-8. What are the different techn iques used for precipitation of proteins?
4-9. Classify proteins with suitable examples.
4-10. Enumerate the d ifferent techniques available for the estimation of proteins. Explain the principle of any one of
them.
48 Section A: Chemical Basis of Life

SHORT NOTE QUESTIONS

4-11 . Primary structure of proteins. 4-17. lsoelectric pH.


4-12. Alpha helix of proteins. 4-18. lsoelectric precipitation.
4-13. Tertiary structure of proteins. 4-19. Precipitation reactions of proteins.
4-14. Quaternary structure of proteins. 4-20. Heat coagulation. •
4-15. Denaturation of proteins. 4-21 . Decarboxylation of amino acids.
4-16. Conjugated proteins.

PART-2: MULTIPLE CHOICE QUESTIONS

4-1 . During denaturation, proteins will not lose their 4-9. Proteins can be precipitated by the following
structure, with regard to: methods, except:
A. Primary structure B. Secondary structure A. Adding alcohol and acetone
C. Tertiary structure D. Quaternary structure B. Saturating with ammonium sulphate
4-2. The characteristic features of the peptide bond C. Using salts of heavy metals
include all the following, except: D. Shifting the pH away from the iso electric point
A. Does not allow freedom of rotation 4-10. Denatured proteins:
B. It is a partial double bond A. Are soluble
C. Always has cis configuration B. Are difficult to digest
D. Absorbs UV light at 280 nm C. Are biologically inactive
4-3. The force maintaining the primary structure of a
D. Peptide bonds are broken
protein:
4-11. Which of the following is a simple protein?
A. Peptide bonds B. Hydrophobic forces
A. Casein B. Insulin
C. Hydrogen bonds D. Electrostatic (ionic) bonds
4-4. The forces maintaining the secondary, tertiary and
C. Hemoglobin D. Tyrosinase

4-12. In glycoproteins, the carbohydrate chains are com-
quaternary structures of a protein are the follow-
bined through glycosidic linkages with :
ing, except:
A. Electrostatic (ionic) bonds A. Hydroxyl groups of serine or threonine residues of
B. Hydrophobic forces proteins
C. Van der Waals forces B. Epsilon amino nitrogen of lysine residues of pro-
D. Peptide bonds teins
4-5. The amino acid which did not allow formation of C. Guanidium group of arginine residues of proteins
alpha-helix is: D. Phenol group of tyrosine residues of proteins
A. Glutamate B. Praline 4-13. The protein which does not answer the aldehyde
C. Tyrosine D. Histidine test is:
4-6. Tertiary structure of a protein describes: A. Hemoglobin
A. The sequence of amino acids B. Albumin
B. Location of disulphide bonds C. Casein
C. Amino terminal end amino acid D. Gelatin
D. The nature of protein folding 4-14. Proteins may be estimated by the following methods,
4-7. One of the following proteins does not have a qua-
except
ternary structure:
A. Biuret method
A. Albumin
B. Heat coagulation
B. Hemoglobin
C. Kjeldahl's digestion
C. Lactate dehydrogenase
D. lmmunoglobulin G D. Nephelometry
4-8. All the following reagents are used for Identifying 4-15. All the following are examples of tertiary structure
the first amino acid in a protein, except: of proteins, except:
A. Cyanogen bromide A. Alpha helix
B. Fluorodinitrobenzene B. Beta pleated sheet
C. Dansyl chloride C. Triple stranded helix
D. Phenyl isothiocyanate D. Peptide bonds
Chapter 4: Proteins: Structure and Function 49
4-16. Tertiary structure of a protein describes: 4-25. Study of linear sequence of amino acids is done by
A. Sequence of amino acids all techniques listed except
B. Location of disulphide bonds A. End group analysis
C. Amino terminal end amino acid B. Hydrolysis by proteolytic enzymes
D. The nature of protein folding C. Analyzing the content of each amino acid
4-17. Proteins may be denatured irreversibly by: D. Denaturing the protein
A. Adding urea 4-26. Different polypeptide chains are held together by:
B. Bringing to iso electric pH A. Peptide bonds
C. Heat coagulation B. Disulphide bonds
D. Reduction with mercaptoethanol C. Glycosidic bonds
4-18. Lectins are: D. Ester bonds
A. Animal proteins having specific amino acid binding 4-27. Primary structure decides:
site A. Rate of synthesis of protein
B. Antibody molecules acting against cells B. Biological activity of the protein
C. Plant proteins having specific carbohydrate bind- C. Rate of degradation of the protein
D. Effect of proteolytic enzymes on protein
ing site
4-28. Secondary and tertiary levels of protein structure
D. Blood proteins having a lecithin group
are dependent on:
4-19. Ultraviolet light at 280 nm is absorbed by which
A. Presence of disulfide bonds
component of proteins?
B. Primary structure
A. Peptide bonds
C. PH of the medium
B. Sulfhydryl group of cysteine
D. PK value of component amino acids
C. lndole ring of tryptophan
4-29. The protein having predominantly alpha helical
D. lmidazole ring of proline
structure is
4-20. The nature of the bond linking amino acids to each
A. Collagen B. Keratin
other is:
• A. Covalent
C. Fibroin D. Myoglobin
4-30. Which of the following is NOT true regarding the
B. Co-ordinate
tert.iary structure of proteins
C. Ionic A. It is a random coil structure
D. Hydrophobic B. Disulfide bonds are formed between any two cys-
4-21 . How many peptide bonds are present in gluta-
teine residues
thione? C. Position of disulfide bonds are predetermined and
A. 1 B. 2 fixed
C. 3 D. 4 D. Denaturation using reducing agents does not aff-
4-22. Basic difference between two polypeptides is in ect the disulfide bonds
the- 4-31 . Protein having a large number of disulfide bonds is:
A. Structural conformation A. Collagen
B. Primary sequence of amino acids B. Keratin
C. Number of side chains C. Hemoglobin
D. Number of hydrophobic bonds D. Albumin
4-23. Human insulin differs from bovine insulin in: 4-32. Which of the following proteins does not possess
A. Biological activity a quaternary structure?
B. Number of amino acids A. Myoglobin
C. Position of disulfide bonds B. Lactate dehydrogenase
D. Sequence of amino acids C. Hemoglobin
4-24. A covalent bond between the alpha carboxyl group D. lmmunoglobulin M
of one amino acid and alpha amino group of the 4-33. Which of the following is NOT true regarding hem-
neighboring amino acid is called oglobin?
A. Cis double bond A. Has 4 independent subunits
B. lsopeptide bond B. Each subunit has one heme residue
C. Pseudopeptide bond C. Each subunit can bind one molecule of oxygen
D. Peptide bond D. All four subunits are similar
50 Section A : Chemical Basis of Life

ANSWERS OF MULTIPLE CHOICE QUESTIONS


4-1 . A 4-2. C 4-3. A 4-4. D 4-5. 8 4-6. D 4-7. A
4-8. A 4-9. D 4-10. C 4-11 . 8 4-12. A 4-13. D 4-14. 8
4-15. D 4-16. D 4-17. C 4-18. D 4-19. C 4-20. A 4-21 . C
4-22. 8 4-23. D 4-24. D 4-25. D 4-26. 8 4-27. 8 4-28. 8
4 -29. A 4-30. D 4-31 . 8 4-32. A 4-33. D

PART-3: VIVA VOCE QUESTIONS AND ANSWERS


4-1 . How proteins are made up of? 4-13. What is proinsulin?
Proteins are made by polymerization of amino acids Insulin is synthesized by the beta cells of pancreas as
through peptide bonds. (Fig. 4.1 ). a single polypeptide chain with 816 amino acids. The
4-2. What is a peptide bond? middle part is removed as C peptide; the remaining
Alpha carboxyl group of one amino acid reacts with part becomes A and 8 chains.
alpha amino group of another amino acid to form a 4-14. What is a mutation?
peptide bond. Amino acid change in the linear sequence is called a
4-3. What is a dipeptide? mutation.
4-15. Can you give an example?
Two amino acids are combined to form a dipeptide.
Sickle cell anemia due to hemoglobin S (HbS).
4-4. How many peptide bonds are present in a tripeptide?
4-16. What are the forces that maintain the secondary,
A tripeptide is a combination of three amino acids; so
tertiary and quaternary structums of a protein?
there are two peptide bonds.
Hydrogen bonds; Electrostatic bonds; van der Waal's
4-5. What is a polypeptide?
forces and Hydrophobic bonds.
A combination of 10 to 50 amino acids is called as a 4-17. What are the salient features of alpha structure of
polypeptide.
4-6. What are the levels of organizations of proteins?
proteins? •
It is a right handed spiral structure. Each turn is formed
Proteins have primary, secondary, tertiary and quater- by 3.6 amino acid residues.
nary levels of organization. 4-18. What is secondary structure of a protein?
4-7. What is meant by primary structure of a protein? Secondary structure denotes the configurational rela-
It denotes the number and sequence of amino acids in tionship between residues which are about 3-4 amino
the protein. acids apart. In other words, secondary level defines
4-8. What force maintains the primary structure? the organization at immediate vicinity of amino acids.
The primary structure is maintained by the covalent 4-19. What is meant by tertiary structure of a protein?
bonds of the peptide linkages. The tertiary structure denotes three dimensional struc-
4-9. How are the end amino acids of proteins called? ture of the whole protein. It defines tlhe steric relationship
The end where there is a free alpha amino group, is of amino acids which are far apart from each other in
called the amino terminal (N-terminal) end. The other the linear sequence.
4-20. What is quaternary structure of :a protein?
end of the polypeptide chain is called the carboxy ter-
Certain polypeptides will aggregate to form one func-
minal end (C-terminal), where there is a free alpha car-
tional protein. This is referred to as the quaternary
boxyl group.
structure.
4-10. What is a pseudopeptide?
4-21 . Name proteins having quaternary structure.
The pseudopeptide is a peptide bond formed by car-
Hemoglobin; lmmunoglobulins.
boxyl g roup, other than that of alpha position.
4-22. What are the reagents that are used for identifying
4-11 . Can you give an example of a pseudopeptide?
the first amino acid in a protein?
GI utathione (gamma-glutamyl-cysteinyl-glycine). Fluorodinitrobenzene; Dansyl chloride; Phenytisothio-
4-12. What are the salient structural features of insulin? cyanate.
It has two polypeptide chains with 51 amino acids. 4-23. What is isoelectric point of a prcttein?
Chain A has 21 amino acids and Chain B has 30 amino At the isoelectric point, the number of anions and cati-
acids. The two chains are held together by disulfide ons present on the protein molecule will be equal and
bridges. the netcharge is zero.
Chapter 4: Proteins: Structure and Function 51

4-24. What are the features of isoelectric point? 4-35. How are proteins classified on physical basis?
At the pl value, the proteins will not migrate in an elec- Simple proteins, conjugated proteins and derived pro-
trical field ; solubility, buffering capacity and viscosity teins.
will be minimum and precipitation will be maximum. 4~6. Give examples of simple prote•ins.
4-25. What is the isoelectric pH of human albumin? Albumins, Globulins, Protamines, Prolamins, Lectins,
It is 4.7. Scleroproteins.
4-26. How proteins are precipitated from solution? 4-37. Give examples of scleroprotei1ns.
Any factor which neutralizes the charge or removes Collagen of bone, cartilage and tendon; keratin of hair.
water of hydration will cause precipitation of proteins.
4 .38. W hat are conjugated proteins',?
4-27. How is albumin precipitated?
Combinations of protein with a non-protein part, called
By Full saturation of ammonium sulphate.
prosthetic group.
4-28. What about gobulins?
4-39. How are conjugated group subclassified?
Globulins are precipitated by half saturation of ammo-
Glycoproteins, lipoproteins, nucleoproteins, chromo-
nium sulphate.
proteins. phosphoproteins and rnetalloproteins.
4-29. Give example of isoelectric precipitation.
Casein is precipitated when the solution is brought to 4-40. Give some examples of chromoproteins
isoelectric pH. Hemoglobin; Flavoproteins, Visuial purple.
4-30. What is the isoelectric pH of casein? 4-41. Give an example of a nutritionally rich protein (first
It is 4.6. class protein).
4-31 . What are the features of denaturation? Casein.
The secondary, tertiary and quaternary structures are 4-42. Why some proteins are nutriti,onally poor?
lost; but primary structure is preserved. The functional They lack in many essential annino acids and a diet
.. activity is lost. Denatured proteins are insoluble and based on these proteins will not even sustain the body
easily precipitated. weight.
4-32. What is heat coagulation? 4-43. Give an example of nutritionally poor protein.
,. When heated at isoelectric point, some proteins will Zein from corn lacks tryptophan and lysine.
denature irreversibly to produce thick floating conglo-
4-44. What is the advantage of biurnt method?
merates called coagulum. This is called heat coagulation.
The biuret method is simple onH step process, and is
4-33. Give examples of proteins that coagulate.
the most widely used method for plasma protein esti-
Albumin is easily coagulated , and globulins to a lesser
mations.
extent.
4-45. What is the disadvantage of b:iuret method ?
4-34. How are proteins classified?
The sensitivity of the method is less and is unsuitable
They may be classified (a) depending on the function
(b) based on the physicochemical characteristics or (c) for estimation of proteins in milligram or microgram
based on their nutritional value. quantities.


_ _ _ _ _Chapter 5
Enzymology:
General Concepts
and Enzyme Kinetics
Chapter at a Glance

The learner will be able to answer ~ ~ th~r,tJng topics:


i. 0 Classification of enzymes t ./
":::J -~.
0 Factors influencing enzyme activity
l.. 0 Coenzymes +f + Co Enzyme activation
'l., 0 Mode of action of enzymes T fII J ' 0 Inhibition, competitive, noncompetitive [c r_tto(_y
M ichaelis-Menten theory .!, • Allosteric inhibition, suicide inhibition
3.

l\,
l 0
Fischer's template theory
Koshland's induced fit theory
Michaelis constant, Km valu e, Vmax
0 Covalent modification
=JO lsoenzymes

Historical Perspectives
Berzelius in 1835 showed hydrolysis of starch by malt extract and put forward the theory of enzyme catalysis (see Table 1.1).
In 1878, Wilhelm Kuhne coined the word enzyme, which in Greek means "in yeast". Eduard Buchner (Nobel Prize 1907)
showed that cell-free extract of yeast could catalyze the fermentation of sucrose to ethanol. He named this active principle as
Zymase. Sir Arthur Harden in 1897 (Nobel Prize 1929) showed that m , a com lex mixtureo~, each catalyzing
a separate step in the degradation of sucrose. The rate of chemical reactions, chemical equi I num an alysis were studied
by Ostwald (Nobel Prize 1909). In 1926, James Sumner (Nobel Prize 1946) was the first to crystallize the en~ e yrea~e. In
1930, John Northrop (Nobel Prize, 1946) crystallized a number of proteolytic enzymes from gastrointestinal fr'act and proved
that they are all proteins.

Once upon a time there was a rich merchant. In his last catalysts. They enter into the reaction, but come out of
will and testament, he put aside his 17 white horses to the reaction without any chan~ Catalysts are substa-
his 3 sons to be shared thus; 1/2 for the 1st son, 1/3 for nces which ae;!!_erate the ra!9 tt
chemical reac-
the 2nd son and 1/9 for the 3rd son. After his death, the
sons started to quarrel , as the division could not produce.
tions, but do not change the equrfibriuJn. ~Ml-6t ..
whole number. Then their brother-in-law told them that AiiilinMHdioiiiiiljifi"
they should include his black horse also for the sharing
purpose. Thus now they had 17 + 1 = 18 horses, and so Life is possible due to the coordination of numerous
division was possible; 1st son got one-half or 9 horses; metabolic reactions inside the cells. Proteins can be
2nd son got 6 and 3rd son 2 horses. Now all the 17 white hydrolyzed with hydrochloric acid by boiling for a very
horses were correctly divided among the sons. The long time; bu_t
remaining black horse was taken back by the brother-in-
law. Catalysts are similar to this black horse. U D nzyme catalysis is very rapid; usually 1 mo e-
_, A chemical reaction, although theoretically prob- cule of an enzyme can act upon about 1000 molecules
able, becomes practically possible only with the help of of the substrate per minute. Lack of enzymes will lead to
Chapter 5: Enzymology: General Concepts and Enzyme Kinetics 53

a
Eduard Arthur James John Wilhelm
Buchner Harden Sumner Northrop Ostwald
NP 1907 NP 1929 NP 1946 NP 1946 NP 1909
1860-1 91 7 1865-1940 1887- 1955 1891- 1987 1853-1932

block in metabolic pathways causing inborn errors of First digit represents the class
metabolism. Second digit stands for the subclass
The substance upon which an enzyme acts, is called Third digit is the sub-sub class or subgroup
the substrate. The enzyme will convert the substrate Fourth digit gives the number of the particular
Q the product or products. enzyme in the list.
The enzymes are grouped into following six major
classes (Box 5.1).

i. Almost all enzymes are proteins. Enzymes follo lass 1: Ox1doreduc ses
the physical and chemical reactions of proteins.
This group of enzymes will c.aialyze o ~ of ~
ii. They are heat-labile~
~ cate with simultaneous re,duction of geethAc sub-
iii. They are water-soluble.
~ rate4 oenz9rne. This may be represented as
iv. They can ~ . I ecipitated by protein r recipitating
• reagents (a~ oft~§6Hlate or trich~f6ae ~<2~).
fl5 AH 2 +B - A+ BH2
For exam " ,.

e enzyme is Alcohol dehydrogenase; IUB name


is Alcohol-NAD-oxidoreductase; Code number is EC.1.
1.1.1. Oxidoreductases may y}
fili21Jif lY'.genases a
Originally enzymes were give~ himsi~ ames. Some
1 0 nd
of these names, such a epsi~ rypsi~ hymotrypsin, •• _ , .: '
e.g. dehydro-
g ~ s (see Chapter . -- •
etc. are still retained . Later, ii was agreed to call the __.,. •
enzymes by adding th to the substrat · J ass 2: Transferases
Thus, enzyme Lactase acts on su trate lactose, &
This class of enzymes transfers one group (Qlt)er tha019
an d th e products gIucose an d gaIactos are formed.
Enzymes th at hyd ro Iyse starch (amy Iose) a
_ c.r• h)Ldroge,n) from the substrate to another substrate. This
may be represented as
a!llylases; those that dehydrogenate the subs
called dehydrogenases. These are known as the
f the enzymes.
For example,
Hexose + ATP -
~-
A-R + B - A+ B-R

Hexose-6-phosphate + ADP
The name of enzyme is Hexokinase.
j
IQBIIB SysJem of Classification C~2.~\\)
lnternation I Union of Biochemistry and Molecular Bi.,.,.. .........a s 3: Hidrolases
(D
logy (IU suggested the IUBMB system of nomen- This class~ / enzymes can hydrolyze er, er, pep--
w as first introduced in 1964 , tide or glyM sidic bonds by adding water and then break-
ing the bond.
Acetylcholine + Hp-----+ Choline + ace;.-ta
. t_e_ _ _
.....
enz me is Ace !choline esterase(AII diges;,
Uve enzymes are hy~
54 Section A: Chemical Basis of Life

BOX 5.1: Classification of enzymes BOX 5.2: Synthetase and synthase are ' . .
Cla. : Olridon!ductases-Transfer of hydrogen or addition of Synthetases are ATP-dependent enzymes cat alyzing biosyn-
oxygen; e.g. Lactate dehydrogenase (Coenzyme, NAO•); Glucose- thetic reactions; t hey belong to Ligases (class 6). Examples are
6-phosphate dehydrogenase (Coenzyme, NADP•); Succinate t;acbamoyl phosphate synthetase; ~ •succinate synthe-
dehydrogenase (Coenzyme, FAD); dioxygenases. tase; PRPP synthetase anrl 'ililTilWiPG§YOSbGTiil-Se,
a., ~ !es- Transfer of groups other than hydrogen. Synthases are enzymes catalyzing biosynthet ic reactions; but
Example, Aminotransferase. (Subclass: Kinase, transfer of phos- they do not require ATP directly; they belon g to classes o ther
phoryl group from ATP; e.g. Hexokinase). than Li gases. Examples are Glycogen synthase and ALA synthase.
c,-
Glyceraldehyde-3-phosphate Dilhydroxy acetone
phosphate
Enzyme is Triose phosphate isomorase.
~ - lntramolecular t ransfers. They include race-
mases and epimerases. Example, Triose phosphate isomerase.
~ ses-ATP dependent condensation of two molecules, lass&: Ligases
e.g. Acetyl-CoA carboxylase; Glutamine synthetase; PRPP syn-
thetase.
These enzymes link two substrates together, usually
with the siw':!Jtaneous bydcolysis at AIi~ (Latin, Ligare =
to bind). For example,
BOX 5.3: Salient features of coenzymes Acetyl-CoA + CO Malon I-CoA + ADP +Pi
1. The protejn part of the enzyme gives the necessary three- Enzyme i Acetyl-CoA carboxylas
dimensional infrastructure for chemical reaction; but the
A summary of classification is giiven in Box 5.1.
group is transferred from or accepted by t he coenzyme.
2. The coenzyme is essential for the biological activity of the The differences between synthetase and synthase are
enzym e. shown in Box 5.2.
3. Coenzyme is a low molecular weig ht organic substance. It is
heat-stable.
4. Generally, t he coenzymes combine loosely with the enzyme COENZYME § + J.t Ui,.+-~.~
+N u.f!\e.o'1o.-
<Pcfi-
molecules. The enzyme and coenzyme can be separated
easily by d ialysis.
Enzymes may be simple proteins, or complex enzymes,
5. Inside the body, when the reaction is completed, t he co- containing a non-protein part, called the prosthetic
enzyme is released from the apo-enzyme, and can bind to
group. The prosthetic group is called the coenzyme. It
another enzyme molecule. In the example shown in Figure
5.1, the reduced coenzyme, generated in the first reaction is heat stable. Salient featu res of coenzymes are shown
can take part in the second reaction. The coupling of these in Box 5.3. The protein part of the enzyme is then named
t wo reactions becomes essential in anaerobic glycolysis
the apoenzyme. It is heat labile. ThHse two portions
(Chapt er 1OJ for regeneration of NAO•.
6. One molecule of t he coenzyme is able to convert a large combined together are called the holocmzyme.
number of substrate molecules with the help of enzyme. Coenzymes may be divided into two groups:
7. Most of the coenzym es are derivatives of vitamin B complex
sub stances. a. Those taking part in reactions by oxido-
reductases by donating or acc ,epting hydrogen
atoms or electrons.
b. Those coenzymes taking part in reactions transfer-
These enzymes can remove groups from substrates @ ring groups other than hydrogen .
. or break bonds by mechanisms other than hydrolysis. First Group of Coenzymes
~ r example, J'l'\ll:70n ,•
ructose-1 , - In the fi rst group, the change occurring-¾
phate +dihydroxyacetone phosphate the substrate is counter-balanced by the
_ The enzyme is Aldolase (see Chapter 1O for details). coenzymes. Therefore, such coenzymes
may be considered as co-substrates or
Class 5: lsomerliil ,- s~ ry_ sub~ s. In the example
These enzymes can produ optical, geo tric or posi- shown in Figure 5.1, the substrate lactate Otto Heinrich
is oxidized, and simultaneously the COE!n- Warburg
tional isomers of substrate Racemase p imerases,
NP 1931

----------
C, is-trans isomerases are examples. zyme (co-substrate ) is reduced . When the 1883-1 970
Chapter 5: Enzymology: General Concepts and Enzyme Kinetics 55
Glyceraldehyde-3-phosphate
coo- Lactate

L
dehydrogenase dehydrogenase
Glyceraldehyde-3-P ( '\ 1-3 bisphosphoglycerate
CHOH
(') •
NAt NADH+ H+
CH3 NAO+ NADH+

Lactate · - )
Lactate dehydrogenase
Pyruvate
Lactate Pyruvate

F ig. 5.1: One coenzyme molecule can work with diff rent enzymes F ig. 5.2A: Reaction of lactate dehydrogenase
r-1 At>.,. \?Ut..':;) ~- C).CC.e.J)~
H

A _ _CO-NH2 Htf-CO-NH, Thiamine pyrophosphate (TPP) .,/


1-----'-'---'--'-- ----+--=---'---'------=---...-
-LJlr'-IH

l ) ~ r \. N
Pyridoxal phosphate (PLP) Amino ~Jroup -N t'i"
Carbon dioxid e -CD
I 2H H+ I
R
Acyl groups -
R NAO• NAOH One carlbon groups
Fig . 5.28 : NAO• accepts hydride ion Adenosine triphosphate (ATP) Pho ·:11e

place on the coenzyme, NAO• (Fig. 5.2A). In this case,


two hydrogen atoms are removed from lactate, out of
which one hydrogen and two electrons (hydride ion) are
accepted by the NAO• to form NADH, and the remaining
" ~.i.J..U-~ l.\,j medium. The hydro-
, group as shown in
Figure 5.28.

_ ,.,.. _,ond Group of Coenzy e 'Tl0..f"\€X~


Adenosine monophosphate
1hese coenzymes take part in reac~ions transferring
Adenosine diphosphate (ADP)
groups other than hydrogen. A particular group or
Adenosine triphosphate (ATP) radical is transferred from the substr;ate to another sub-
Fig. 5.3: Structure of ATP strate. Most of them belong to vitamin B complex group.
A few such examples are
reactio is reversed, the effect will be exactly the oppo- on
site, su strate getting reduced and coenzyme being . In order to emphasize the
oxidize . (tJ. ® presence of -SH group, the
Here th I for the
0th ~ uch examples are NAOP•-NADPH; FAD-
FADH n <- MN-FMNH •
2 2
===== -
__.
that attaches to it.

denosine Triphosphate (,"TP)


ide Adenine Dinucleotide (NAo+J
ATP is considered to be the energy currency in the
oenzyme synthesized from Nicotinamide, a
body. Fiske and Subbarao first isolated ATP in 1926 and
member o vitamin B co~~- The structure of NAO·
Lohmann in 1929 showed the importance of ATP in mus-
is Nicotinamide-Ribose-P-P-Ribose-Adenine (see Fig. cle contraction . In the ATP molecule, the second and
33.6). Warburg (Nobel Prize, 1931) elucidated the third phosphate bonds are 'high ene,rgy' bonds (shown
structure of NAO•. The reversible reaction of lactate as squiggle bonds in Fig. 5.3). During the oxidation
to pyruvate is catalyzed by the enzyme lactate dehy- of food stuffs, energy is released, a part of which is
drogenase, but the actual transfer of hydrogen is taking stored as chemical energy in the form of ATP. The
56 Section A: Chemical Basis of Life

TABLE 5.2: Metalloenzyrnes


~ CH2
Metal Enzymes containing metals .
- ::.-... NH I @vNH
I
Zinc Carbonic anhydrase, carboxypeptidase, alcohol CHNH2 CHNH2
dehydrogenase I I
COOH COOH
Magnesium Hexokinase, phosphofructokinase, enolase,
glucose-6-phosphatase Protonated form (Acid) Non-protonated (Base)
Manganese Phosphoglucomutase, hexokinase, enolase, Fig. 5.5: Acid base catalysis with the help of histidine
glycosyltransferases
Copper Tyrosinase, cytochrome oxidase, lysyl oxidase, ( T' - ------
~I MODE OF ACTION OF ENZYMSS
Iron
superoxide dismutase
Cytochrome oxidase, catalase, peroxidase, xanthine
oxidase
,-
There are a few theories explaining the mechanism of
Calcium Lecithinase, lipase action of enzymes. Perhaps each of them tries to view
Molybdenum Xanthine oxidase the fact from differe nt perspectives to explain a parti-
cular aspect of the action.
A
- - Reaction kinetics
without enzyme Lowering of Activation Energy
Reaction with enzyme Enzymes lower the energy of activation. Activation
energy is defined as the energy required to convert all mole-
>,
C) cules of a reacting substance from the ground state to the
a;
C:
w tra nsition state. Substrates are remaining in an energy
trough, and are to be placed at a higher energ y level,
n me whereupon spontaneous degradation can occur. Suppose,
Red circle= substrate; black circle= product. C= energy level of
substrate; D= energy level of product. C to A= activation energy in the
we want to make a fire; even if we keep a flame, the wood ..
absence of enzyme; C to B is activation energy in presence of will not burn initially; we have to add kerosene or paper
enzyme; B to A= lowering of activation energy by enzyme
for initial burning. Similarly, the activation energy is to be
Fig. 5.4: Lowering of activation energy by enzymes initially supplied. During enzyme substrate binding, wea k
interactions between enzyme and substrate are opti-
endergonic reactions are carried out with the help of
mized. This weak binding interaction between enzyme and
energy released from hydrolysis of ATP. Fo r example,
substrate provides the major driving force for the enzy-
Hexokinase
Glucose ( ~ "' Glucose-6-phosphate matic catalysis. Enzymes reduce the magnitude of this
ATP AbP activation energy. T his can be compared to making a

@Metalloenzymes tunnel in a mountain, so that the barrier could be lowe-


red (Fig. 5.4). For example, activ ation energy for acid
These are enzymes, which regui(e certain metal:ions for
hydrolysis of sucrose is 26,000 cal/mol, w hile the acti-
their activity. Some examples are given in Table 5.2. In
vation energy is only 9 ,000 cal/mol when hydrolyzed by
certain cases, e .g . copper in Tyros inase, the metal is
sucrase.
tightly bound w ith the enzyme. In other cases, even •
without the metal ion, enzyme may be active; but whe
the metal ion is added, the activity is enhanced. T he cid Base Catalysis
are calle9 ion-activated enzymes, e.g . calc ium ions w ill * ·
is an example of a general
activate pancreatic lipase.
acid and its conjugate base, the ge neral base (Fig. 5.5).
The action of ribonuclease is an example of acid-base
@ Cofactors catalysis. at the active site
T he term cofactor is used as a collective term to include of ribonuclease function as acid and base in catalysis.

-
coenzymes and metal ions. Caeozyrn9 is an or:gaoic co- Histidine 12 acts as an acid and donates a proton. Histi-
factor. dine 119 accepts a pro ton and product is released.
Chapter 5: Enzymology: General Concepts and Enzyme Kinetics 57

A
I
B

I
C
______ 1_________ (A) Unfolded protein
with no activity ____ _

I Folding
process
(B) Folded protein
with enzyme activity
D
35Glu --+ I Asp 52
_,,µ.__ _ _ _ Amino acid side
E
chains from the
I active center of enzyme
F
Enzyme

Fig. 5.6: Active center of lysozyme. A, B, C, D, E, F are carbo- Fig. 5.7: Correct alignment of amino acids in the active center of
hydrate units (substrate). D = N-acetyl muramic acid; E= N-acetyl the enzyme
glucosamine. Bond is broken between D and E, with the help of Glu
and Asp residues in the enzyme, which are opposite to each other
attacks the substrate. This results in covalent bjndjna of
the substrate to the enztme.

t------'-------+--.<.!....C--'-'-==;.;.;.;:...:.== ~ '9 Entropy Effect


Cysteine proteases Pa pain
Aspartyl proteases Renin, retroviral proteases Enzymes enhance reaction rates by decreasing entropy.
Carboxyl proteases Pepsin When orrec osI 10 and i ~ o n the enzyme
Metalloproteases Carboxypeptidases surface, the substrates a sn4r"ned to the transition
Protease inhibitors used ACE inhibitor (Captopril) This is referred to as the Proximity effect. Chemi-
as drugs HIV protease inhibitor (Retonavir) cal reactions need physical apposition of two reactants.
Since substrates usually are present in low concentra-
Strain tions, the collision probability is less and hence the reac-
Binding of substrate to a preformed site on the enzyme tion velocity is low. But a complex formation between the
can induce strain in the substrate. The energy level of enzyme and the two substrate molecules can improve
the substrate is raised. A combination of substrate strain the collision probabilities many fold, causing the rapid
and acid base catalysis is seen in the action of lysozyme. rate of reaction.
The lysozyme substrate has a repeating hexa-
saccharide unit. Binding of the substrate to the enzym roduct Su s rate Orientation Theo~
generates a strained conformation in the enzyme sub- Enzyme has appropriate three-dimensional structure to
strate complex (D in Fig. 5.6). The(i&j:osid1c bo~ keep the substrates in a specific orientation, such that
between N-acetyl glucosa01ine and N-acetyl muramic the reactive groups come to physical apposition, leading
acid on the bacterial cell wall is thus !!l;'.drolyzed. This to speedy reactions (Fig. 5.7). It has been shown that
accounts for bactericidal action of lysoz'yme. Lysozyme the hydroxyl group of the 6th carbon atom of glucose
was purified and studied by Howard Florey (Nobel Prize,
and the terminal phosphate group of ATP are juxtaposed
1945).
with the help of hexokinase. ,, 1.1.-.~ __
'- to ~y.,., !VU)
4
@ Sarine Proteases RAELIS-MENTEN
~ THE
-- ....,;;y;.:._: e\ ~h-
OR,
They are enzymes with a serine residue at the active site
and most of the proteolytic enzymes belong to this group, 1n 1913, Michaelis and Menten put forward the Enzyme~
e.g. trypsin , chymotrypsin, clotting factors (Table 5.3). Substrate complex theory. Accordingly, the enzyme (E)
combines with the substrate (S), to form an enzyme-sub-
® Covalent Catalysii; strate (ES) complex, which immediately breaks down to
In covalent catalysis, a nucleophilic (negatively charged) the enzyme and the product (P) (Fig. 5.8).
or electrophilic (positively charged) group of the enzyme E+S E- S Complex E+P
58 Section A: Chemical Basis of Life

Specific substrate Substrate analog

Enzyme + substrate Enzyme- Enzyme acts


substrate Substrate correctly fixes Enzyme cannot

••
complex on the enzyme; Enzyme- accept analog
substrate complex (ES) formed


-+ Fig. 5.9: Fischer's template theory

Enzyme-product complex Enzyme and products

Fig. 5.8: Enzyme substrate complex

Active
site

•Substrate

Fig. 5.10: Enzyme and substrate are specific to each other. This
is similar to key and lock (Fischer's theory)
2 3 4

1. Enzyme has shallow grooves; substrate alignment is not correct.


2. Fixing of substrate induces structural changes in enzyme. 3. Now
substrate correctly fits into the active site of enzyme. 4 . Substrate is
cleaved into two products.
Fig. 5.11: Koshland's induced fit theory

Alkaline phosphatase hydrolyzes a number of phos- with the substrate. At first, substrate binds to a specific
phate esters including glucose-6-phosphate. The active part of the enzyme. This leads to more secondary binding
cente·r of this enzyme contains a Serine residue, and the and conformational changes. The substrate induces
reaction is taking place in the following two steps: conformational changes in the enzyme, such that
a. E-Serine-OH+Glucose-6-P~E-Serine-O-P+Glucose precise orientation of catalytic groups is eff~ted (Fig.
b. E-Serine-O-P E-Serine-OH+Pi 5.7). A simplified explanation is that a glove is put on
Thus, the overall reaction is a hand. At first, the glove is in a partially folded position,
Glucose-6-P Glucose + Pi but hand can enter into it. When the hand is introduced,
In this reaction mixture, the enzyme substrate.c~m- the glove is further opened. Similarly, conformational
plex, E-Serine-0-f, has been isolated . changes occur in the enzyme when the substrate is
fixed (Fig. 5. 11 ).
When substrate analog is fixed to the enzyme, some
structural alteration may occur; but reaction does not
It states that the three-dimensional structur e active
take place due to lack of proper alignment (Figs. 5.7 and
site of the enzyme is complementary to the substrate.
5.9). Allosteric inhibition can also be explained by the
Thus enzyme and substrate fit each other. Substrate
hypothesis of Koshland.
fits on the enzyme, similar to lock and key. The lock
can be opened by its own key only (Figs. 5.9 and 5.10).
However, Fischer envisaged a rig id structure for enz-
ymes, which _could not explain the ·flexibility shown by
enzymes.

Emil Fischer Leonor Maud Menten Daniel


Conformational changes are occurring at the active site NP 1902 Michaelis 1879-1960 Kosh/and
of enzymes concomitant with the combination of enzyme 1852-1919 1875-1949 1920-2007
Chapter 5: Enzymology: General Concepts and Enzyme Kinetics 59

BOX 5.4: Active center of enzyme TABLE 5.4: Active center of enzymes
1. The region of the enzyme where substrate binding and Name of enzyme Imp ortant amino acid at the catalytic site
catalysis occurs is referred to as active site or active center Chymotrypsin His (57), Asp (102), Ser (195)
(Tables 5.3 and 5.4).
Trypsin Serine, Histidine
2. Although all parts are required for maintaining the exact
three-dimensional structure of the enzyme, the reaction is Thrombin Serine, Histidine

' taking place at the active site. The active site occupies only a
small portion of the whole enzyme.
Phosphoglucomutase Serine
Alkaline phosphatase Serine
3. Generali active site is situated in a crevice or cleft of the
Acetylcholinesterase Serine
enzyme molecule (Fig. 5.7). Tot e active site, the specific sub-
strate is bound. The binding of substrate to active site depends Carbonic anhydrase Cysteine
on the alignment of specific groups or atoms at active site. Hexokinase Histidine
4. During the binding, these groups may realign themselves to Carboxypeptidase Histidine, Arginine, Tyrosine
provide the u.!!i9ue conformatjonal orien!;ion so as to ~ -
Aldolase Lysine
mote exact fitfipg of substrate to the active site (Fig. 5.7).
5. The substrate binds to the enzyme at thea ctive site by non-
covalent bonds. These forces are hydrophobic in nature.
Urea > ammonia + CO2 + energy
The amino acids or groups that directly participate in making
or breaking..the. boods (present at the active site) are called At equilibrium of this reaction, the substrate will be
catalyticres1dues or catalytic groups. only 0.5% and product will be 99.5%. Such reactions are
J,,s;., The active site contai r i ~strate binding site and catalytic
generally irreversible.
site; sometimes these two may be separate.

lsothermic Reaction
When energy exchange is negligible, the reaction is
easily reversible, e.g.
Glycogen+ Glucose-1-phosphate
Catalysis occurs at the active center or active site. Sali-
At equilibrium of this reaction, 77% glycogen will be
ent features are shown in Box 5.4. See Tables 5.3, 5.4
unutilized and 23% glucose-1-phosphate will be formed.
and Figure 5.7 also.
Proteolytic enzymes having a serine residue at the
Endergonic or Endothermic Reaction
active center are called serine proteases, e.g. pan-
creatic proteases (Table 5.3), and coagulation factors. Energy is consumed and external energy is to be sup-
her examples are given in Table 5.4. plied for these reactions. In the body, this is usually
accomplished by coupling the endergonic reaction with
ERMODYNAMIC an exergonic reaction, e.g. Hexokinase catalyzes the

CONSIDERATIONS
-------
From the standpoint of energy, the enzymatic reactions
following reaction:
Glucose Glucose-6-Phosphate + ADP

are divided into 3 types: KINETICS - ----


Exergonic or Exothermic Reaction lVelocity or rate of enzyme reaction is assessed by the
rate of change of substrate to product per unit tim
Here energy is released from the reaction, and there-
In practice, nitial velocity is determine~ If much time }
fore reaction essentially goes to completion , e.g. urease
is allowed to lapse, the velocity may tend to fall due to
enzyme:
decrease in substrate concentration below a critical level.
The velocity is proportional to the concentration
oU..
of reacting molecules.
A+ B C+D
If concentration of A or B is doubled, th rate
i-" oW ~.'cJ_-~
tion is also doubled. If concentrations of A a d Bare do {'CJ''t.A
Howard Christian Stanford William bled together, the velocity becomes 4-fold. (Box 5.5). The
Florey Anfinsen Moore Stein
NP 1972 NP 1972
following facts can be derived from the equations given in
NP 1945 NP 1972
1898-1968 1916-1995 1913-1982 1911-1980 Box 5.6.
60 Section A: Chemical Basis of Life

BOX 5.5: Derivation of equ1l1bnum constant BOX 5.6: Derivation of Michaelis constant /Km)
Va. [Al [Bl Kl
At equilibrium, forward reaction and backward reaction are E+S Ki E-S E+ P
equal, so tha .____.,
"
If concentration of substrate is increased, the forward reaction Kl
is increased, and so K3 as well as total velocity is correspondingly

:J
enhanced. The three different constants may be made into one .t

1
• • • • I
Forward reacti
and backward reaction R2 = - - - --- -
At equilibrium, Rl -·
-----
_
--
Or, Kl [Al [Bl
Or, Kl = [Cl [DJ
K2 = [Al [Bl
K2 [C][D]
Keq or
Equilibrium constant.
- It . f rth h
aelis Constant
th

_,_
~_cm,.;;_.....
...... ---

BOX 5.7 : Factors affecting enzyme act1v1ty ~


--- - ation
__....:.i-._., •,,:...
.
of substrate is made equal to Km, i.e.
1. Enzyme concentration
)\ 2. Substrate concentration Vmax [SJ Vmax [SJ Vmax
3. Product Velocity (v) = - - - = - - - =
[SJ+ [SJ 2[SJ 2
% 4. Irro0r@t11r~
c., 5. Hydroqen ion concentration (P,H) orv=½Vmax
6. Presence of activators
7. Presence of inhibitors
8. Presence of reoressor or dereoressor other words, enzyme makes it quicker to reach the
c_ 9. Covalent modification. equilibrium. G!!!.!¥!):o i.D.£[!ase~ e..,ot reac-
~ L . . - - - - - - - - - -- - - - - - -~
tion, but do not alter the e uili ·

l
l FACTORS INFLUENCING
'ENZYME ACTfV
;;..;..;;.;l:;.T..;..
.;;. Y_ __.---w~"'
The various factors which affect enzyme activity are
enumerated in Box 5.7. These are explained below.

.___ _ _ __ --'-
[E-'--1_ __ _ _ __ _ 4)Enzyme Concentration
Fig. 5.12: Effect of enzyme concentration
Rate of a reaction or velocity (V) is directly propor-
tional to the enzyme concentration, when sufficient
1. The equilibrium constant of the reaction is the
substrate is present. Velocity of reaction is increased
ratio of reaction rate constants of forward and back-
proportionately with the concentration of enzyme, pro-
ward reactions.
vided substrate concentration is unlimited (Fig. 5.12).
2. At equilibrium, forward and backward reactions are
Hence,Qhis property is made use of determining the
equal. Equilibrium is a dynamic state. Even though level of particular enzyme in plasma, serum or tissue9
n concentrations of substrate and ( Known volume of serum is incubated with substrate for a
fixed time, then reaction is stopped and product is quan- 1'
titated (endpoint method). Since the product formed
3. Numerical value of the constant can be calculated will be proportional to the enzyme concentration, the lat-
by finding the concentrations of substrates and ter could be assaye'
products.
4. If Keq is more than 1, the forward reaction is favo...&)Effect of Subs~ate c_oncentratio
red. In such instances, the reaction is spontaneous In a series of test tubes, equal volume of enzyme solu-
and exothermic. tion is taken, but increasing quantity of substrate is added
5. Concentration of enzyme does not affect the Keq. and the rate of reaction is assayed in each tube. The
Concentration of enzyme certainly increases the rate velocity (v) is expressed in micromoles of substrate con-
of reaction; but not the Keq or the ultimate state. In verted per minute.
Vmax Maximum velocity (Vmax)

½ Vmax

[SJ
Km

Fig . 5.13A : Effect of substrate concentration (substrate satura-


Substrate concentration [SJ
tion curve)
Enzyme molecules are shown as half-circles. Substrate molecules
are red dots. (A) Substrate molecules are low; so only a few
enzyme molecules are working and velocity is less. (B) At half-
maximal velocity (Km), 50% enzyme molecules are bound with
substrate. (C) As a lot of substrate molecules are available, all
enzyme molecules are bound. (D) Further increase in the substrate
Enzyme 1 unit will not increase the veloci further.
Ffg. 5.13B: Effect of substrate concentration on enzyme activity

BOX 5.8: Salient lt::c... : ,111

1, Km value i s substrate ,on,entration (expressed in moles/ L)


Km
at half-maximal velocity.
Fig. 5.1 4: Effect of enzyme concentrate on Km 2. It denotes that 50,& of enzyme molecules are bound with
substrate molecules at that particular substrate concen-
tration {Fig. 5.13.B).
If the velocity is plotted against the substrate con- 3. Km Is independent of enzyme concentration. If enzyme
centration, a typical curve (Fig. 5.13A) will be obtained. concentration is doubled, the Vmax will be double (Fig. 5.14).
As substrate concentration is increased, the velo- But the½ Vmax (Km) will remain exactly same. In other words,
irrespective of enzyme concentration, 50% molecules are
city is also correspondingly increased in the initial bound to substrate at that particular substrate concentration.
phases; but the curve flattens afterwards. 4. Km Is the signature of the enzyme. Km value is thus a con-
This is explained in fhe Figure 5.1 3B. At lower con- stant for an enzyme. It is the characteristic feature of a par-
ticular enzyme for a specific substrate.
centrations of substrate (point A in the curve), some
5. The affinity of an enzyme towards its substrate is inversely
enzyme molecules are remaining idle. As substrate is related to the dissociation constant.
increased, more and more enzyme molecules are work- Therefore, the smaller the tendency for the dissociation of
ing. At half-maximal velocity, 50% enzymes are attached the complex, the greater is the affinity of the enzyme for the
substrate.
with substrate (point B in the curve). As more substrate
6. Km denotes the affinity of enzyme for substrate. The less-
is added, all enzyme molecules are saturated (point C). er the numerical value of Km, the affinity of the enzyme for the
Further increase in substrate cannot make any effect in substrate is more.

the reaction velocity (point D). The maximum velocity


obtained is called Vmax (Fig. 5.13B). It represents the In the Figure 5.13A, 50% velocity in Y-axis is extra-
maximum reaction rate attainable in presence of excess polated to the corresponding point on X-axis, which
gives the numerical value of Km. Salient features of Km
... substrate (at substrate saturation level) .

~~-\
value are shown in Box 5.8.
3
,o &,13~\ \'c.
-'r [ he lesser the numerical value of Km, the affinity
aeHs Constant 'm
of the enzyme for the substrate is more. To cite an ?
According to Michaelis theory, the ormation of enzyme- example, Km of glucokina&,e is 10 mrr,s:>1/L and that of
substrate complex is a reversible reaction, while the hexokinase is 0.05 mmol/LJ Thereforel_50% molecules
breakdown of the complex to enzyme + product is irre- of hexokinase are saturated even at a lower concentra-
versible. (See the derivation of the Michaelis constant in tion of glucos, {ln other words, hexokinase has more
Box 5.6). affinity for glucose than glucokinasy ]
62 Section A: Chemical Basis of Life

Km
Vmax
i
•• V

-
-1/Km 1/[SJ

Fig. 5.15: Lineweaver-Burk plot

Fig. 5.16: Sigmoid substrate-saturation kinetics of cooperar e


binding

i
V
down. So when product concentration is increased, the
reaction is slowed, stopped or even reversed. In inborn
venoJu.y-ol~ /. errors of metabolism, one enzyme of a~ etab lie ~athway
.___~..........,___,- ---~ ©
0 10 20 30 40 50 60
is blocked. For example, 1-l.,J Ci
Temperature in •c 2 E2
A~ B C - II ~ D
Fig. 5.17: Effect of temperature on veloci ty

mDouble Reciprocal Plot~ Llf"e


S
t
"\o
If E3 enzyme is absent, C will accumulate, which in
turn, will inhibit E2. Consequently, in course of time, the
whole pathway is blocked. q _
Sometimes it is impractical to achieve high substrate
concentrations to reach the ~aximal velocity ~onditions.~ Effect of emperaturff
So, ½Vmax or Km may be difficult to determine. Then.~
the experimental data at lower concentrations is plot- The velocity of enzyme reaction increases when temper-
ted as reciprocals. The straight line thus obtained is ature of the medium is increased; reaches a maximum
- extrapolated to get the reciprocal of Km. This is called and then falls (Bell shaped curve). The temperature at
Lineweaver-Burk Plot or Double Reciprocal Plot which maximum amount of the substrate is converted
which can be derived from the Michaelis-Menten equa- to the product per unit time is called the optimum tem-
tion. If we plot 1/v against 1/[S], it will give a straight perature (Fig. 5.17). As temperature is increased, more
line graph as shown in Figure 5.15. Intercept in X-axis is molecules get actjvatlan energy, or molecules are at
minus 1/Km, from which the Km can be calculated. increased rate of motion. So their collision probabilities
are increased and so the reaction velocity is enhanced.
1iC) Cooperative Binding, ( But when temperature is more than so0 c, heat
Some enzymes may not strictly follow den'aturation and consequent loss of tertiary structure of •
the Michaelis-Menten kinetics. When the protein occurs. So activity of.the enzyme is decreased.
enzyme has many subunits, and binding Most huma~'-7.'.:'zymes have the optimum temperature
of substrate to one unit enhances the around 37°C~ ~rtain ~acteria living in hot spring~ will
affinity for binding to other subunits have enzymes with optimum temperature near 100 C.
(cooperative binding), a sigmoid shaped Arc~ibald Viviab ;:;,c~ y-
saturation curve is obtained (Fig 5 16) Ht/I NP 1922 ~),Effect of pH
. . . 1886-1977
In such cases, determination of Km Each enzyme has an optimum pH , on both sides of
value, as shown in the previous paragraph, will be which the velocity will be drastically reduced. The graph
invalid. Instead the HIii equation, originally described for will show a bell shaped curve (Fig.5.18). The _e..t:!.
explaining the oxygen binding to Hemoglobin, is decide the amino acid residues at the
employed (Hill was awarded Nobel Prize in 1922). a~ siJ$,. The net charge on the enzyme protein
would influence substrate_binding and catalytic activity)
c) Effect of Concentration of Products Optimum pH may vary d~~ending on th© mpecat1 ire,
In a reversible reaction, S .... P, when equilibrium is reachect,@ oncentration of s11bstcatP~ resence of jogs, etc. Usually
as per the law of mass action , the reaction rate is slowed enzymes have the optimum pH between 6 and 8. Some
Chapter 5: Enzymology: General Concepts and Enzyme Kinetics 63

COOH COOH COOH


I I I
V
CH2
I T""'\ • H- C H- C-
I
H

CH2
II COOH
FADH2 C- H
I I
COOH COOH

Succinate s , FumarateJ' Malonate(

Fig. 5.19: Malonate inhibits succinate dehydrogenase enzyme

P 2 4 6 't 8 10 12
iv. Coagulation factors are seen in blood as zymo-
Fig. 5.18: Effect of pH on enzyme velocity
gen form, their activation pathways are described
{ in Chapter 26.
1N These@_tivities are needed only occasional~ but
when needed, a large number of molecules are to be
produced instantaneously. Hence, the sascade systerp
of chemical amplification of such factors.

"
- 1/Km

Vmax

Km New t<.m [SJ

Fig. 5.20 : Substrate saturation curve in presence and absence of


C01TIPetitive inhibitor

important exceptions are pepsin (with optimum pH_


-2=_2); alkaline phosphatase (optimum pH 9-1 0) ana
acid hosphatase 4-5). • · So, y
le
Eiiiyme ~ctivation
\\ I n presence of certam · .inorganic. .ions, some enzymes
show higher activity. Thus, ~hloride ions activate salivary decreased~
amylase) and €alcium ions activate lipas . Another type In competitive inhibition, the inhibitor will be a struc-
of activation is the conversion of an inactive proenzyme tural analog of the substrate. There will be similarity in
or zymogen to the active enzyme. the three-dimensional structure between substrate (S)
i. By splitting a single peptide bond, and removal of and inhibitor (I). For example, the succinate dehydroge- ~1>'1)
a small polypeptide from trypsinogen, the active nase reaction is inhibited by malonate (Fig. 5.19). Com-
trypsin is formed. This results in unmasking of the petitive inhibition is usually reversible. Or, excess
active center. substrate abolishes the inhibition. In the previous
ii. Similarly trypsin activates chymotrypsinogen. example of 100 moles of E and 100 moles of I, if 900
iii. Alt the gastrointestinal enzymes are synthesized in moles of Sare added, only 1110th of enzyme molecules
the form of proenzymes, and only after secretion are attached to inhibitor and 90% are working with sub-
into the alimentary canal, they are activated. This strate. Thus 50% inhibition in the first example is now
prevents autolysis of cellular structural proteins. decreased to 10% inhibition (Fig. 5.20).
64 Section A: Chemical Basis of Life

f many d rugs may be explained by the


principle of competitive inhibition. A few important examples are Clinical use cha ters
given below: Gout
1. Sulfonamlde 1: They are commonly employed antibacterial
agent s (Fig. 5.21). Bacteria synthesize folic acid by combining r
PABA with pteroylglutamic acid. Bacterial wall is impermeable
3. Penicillin 'rranspeptldase 1Antibiotic
to folic abd. Sulfa drugs, being structural analogs of PABA, will
inhibit the folic acid synthesis in bacteria, and they die. The 4. Sulfonamide Pteroid synthetase Antibiotic
drug is nontoxic to human cells, because human beings cannot 5. Trimethoprim FH2-reducta~e Ant ibiotic
synthesize folic acid. Preformed folic acid is essential for man. 6. rimethamine Do Malaria
Antibacterial effect of sulfa drug was studied by Gerhard 7. Methotrexate Do Cancer 48
Domagk, (Nobel Prize 1939).
8. 6-mercapto- Adenylosuccinate Cancer 48
2. Methotrexat4,: It is 4-amino-N 10 -methyl folic acid. It is a struc-
purine synthetase
tural analog of folic acid, and so can competitiv~ly inhibit
folate reductase enzyme (see Chapter 38). This is essential for 9. S-fluorouracll Thymidylate Cancer 48
DNA synthesis and cell division. Therefore, methotrexate is synthase
used as an anticancer drug. 10. Azaserine Phosphoribosyl- Cancer 48
.J. Dkoumarol: It is structurally similar to v itamin Kand can act amidotransferase
as an anticoagulant by competitively inhibiting the vitamin K 11. Cytosine DNA polymerase Cancer 48
activity (see Chapter 32). arabinoslde
4. l1onlcotlnlc acid hydrazlde (INHJ: It is a commonly used 12. Acyclovir Do Virus
antituberculous drug. It is structu rally similar to pyridoxal,
and prolonged use o~ may cause(flyflObxal de~:;"St, 13. Neostigmine ACh-esterase M asthenia
anc:(Wr]iliera( neurooatb}')see Chapter 33). A selecte 1st of 14. Alpha-methyl Dopa-decarboxylase ~ o n
clinically important drugs working on the principle of com-
petitive inhibition is g iven in Table S.S. 15. Lovastatin HMG-CoA-reductase Cholesterol
low
16. Oseltamivir
(Tamiflu)

9Le
e Substrate

< >.
E-S complex

l7
'----+Prodoct
H 2N COOH

a Noncompetitive
inhibitor
Fig. 5.21 : Competitive inhibition
• Substrate

From the graphs, it is obvious that in the case of


competitive inhibition , the Km Is increased In pres- Inactive
ence of competitive inhibitor. Thus competitive inhi- enzyme
bitor apparently increases the Km. In other words, the Fig . 5.22: Non-competitive inhibition
affinity of the enzyme towards substrate is apparently
decreased in presence of the inhibitor. But Vmax is not competition between substrate and Inhibitor. See
changed. Same Vmax can be achieved at higher sub- Figure 5.22.
strate concentrations. Example_s of competitive inhibi- The inhibitor usually binds to a different domain on
tion and their clinical significance of such inhibition is the enzyme, other than the substrate binding site. Since
shown in Box 5.9, Figure 5.21 and Table 5.5. these inhibitors have no structural resemblance to the
substrate, an increase in the substrate concentra-
B.Non-competitive Inhibit, tion generally does not relieve this Inhibition. The
A variety of poisons, such as iodoacetate, heavy saturation curve of this type of inhibition is shown in
metal ions {lead, mercury) and oxidizing agents act Figure 5.23. A comparison of the two types of inhibitions
as irreversible non-competitive inhibitors. There is no is shown in Table 5.6. Examples are:
Chapter 5: Enzymology: General Concepts and Enzyme Kinetics 65

TABLE S.6: Companson of two types of 1nh1b1t1on


Competitive Non-competitive
• inhibition inhibition
Acting on Active site May or may not J
Structure of inhibitor Substrate analog Unrelated molecule
Inhibition Is Reversible Generally irreversiblt
Excess substrate Inhibition relieved No effect
Km Increased No change
Vmax No change Decreased

1
- 1/Km [S] Drug action Toxicological i

Vmax Without
inhibitor

With
fvmax 1nh1b1tor

- 1/Km - 1/Km 1/(S]


t [SJ
Fig. 5.24: Uncompetitive inhibition
Km

ig . 5.23: Noncompetitive inhibition


The inhibitor combines with the enzymes by form-
ing a covalent bond and then the reaction becomes irre-
a. Cyanide inhibits cytochrome oxidase.
versible. The velocity (Vmax) is reduced. But Km value
b. Fluoride will remove magnesium and manganese
is not changed, because the remaining enzyme mole-
ions. So fluoride will inhibit the enzyme, enolase,
and consequently the glycolysis. cules have the same affinity for the substrate. Increas-
c. lodoacetate would inhibit enzymes having -SH ing the substrate concentration will abolish the com-
group in their active centers. petitive inhibition, but will not abolish non-competitive
d. BAL (British Anti-Lewisite; dimercaprol) is used as inhibition.
C Jan antidote for heavy metal poisoning. The heavy • • • ••
A metals act as enzyme poisons by reacting with theC. .Uncompet1t1ve lnh1b1t1on
i
FBl SH group. BAL has several SH groups with which Here the inhibitor does not have any affinity for free

=~-1
the heavy metal ions can react and thereby their enzyme. Inhibitor binds to enzyme-substrate complex;
poisonous effects are reduced. but not to the free enzyme. In such cases both Vmax
e. Acetylcholin esterase enzyme cleaves acetylcho- and ~m re decreased (Fig. 5~ 4 ). Inhibition of
1i line to form acetate and choline and therefore ter- alkaline tase Re.9art-:is'aloz¥me) by p a~-
minates the action of acetylcholine. Certain chemi- nine is an example of uncompetitive inhibition.
cals e.g. .diworogyl fluorophosphate (DFP) binds • • • d
to the active site, serine of acetylcholine esterase. t>.Suicide lnh1b1t1on ( M..e.c.Y'I OX")~'lo')in~,'tlE:'>'n)
As a result acetylcholine accumulates and over- It is a special type of irreversible inhibition of enzyme
stimulates autonomous nervous system including activity. It is also known as mechanism based inac-
heart, blood vessels and glands. This leads to vomi- tivation. The inhibitor makes use of the enzyme's
ting, salivation, sweating, and in worst cases even own reaction mechanism to inactivate it (mechanism
death. DFP forms an irreversible covalent bond based inactivation). In suicide inhibition, the structural
with acetylcholine esterase, and activity can be analog is converted to a more effective inhibitor with the
regained only if new enzyme is synthesized. help of the enzyme to be inhibited. The substrate-like
66 Section k Chemical Basis of Life

A
s
r -"7"'...::;,.- -.,,c--
With positive
modifier
AA ,___ ____ _ _ _ Sigmoid curve
\ _ _ y ! - Ain
½ Vmax

With negative
modifier
A= Active site; Allo = Allosteric s1le; S = Substrate;
AA= Allosteric activator; In = Inhibitor site;
Ain =Allosteric inhibitor. The enzyme has separate active site
and allosteric site. Figure 1 depicts that the activator is fixed at
the allosteric site, when active site has correct conformation,
and the substrate is correctly fixed. Figure 2 shows that the Fig. 5.26: Allosteric inhibition
inhibitor is fixed at the allosteric site when active site is deformed
and the substrate could not fix.
alloxanthine which is a more potent inhibitor of xanthine
Fig . 5.25: Action of allosteric enzymes
oxidase (see Chapter 38).
The anti-inflammatory action of Aspirin is also
BOX 5.10: Sahenl '" " -~. dllo~l"'"" regulation based on suicide inhibition. Arachidonic acid is converted
1. The inh ibitor is not a substrate analog. to prostaglandin by the enzyme Cyclo-oxygenase (see
2. It is partially reversible, when excess substrate is added Chapter 16). Aspirin acetylates a serine residue in the
3. Km is usually increased active center of cyclo-oxygenase, thus prostaglandin
4. Vmax is reduced synthesis is inhibited, and so infla
2...
5. The effect of allosteric modifier is maximum at or near sub-
strate concentration equivalent to Km (Fig. 5.26). When an f .Allosteric Regulatio
M
c;:~+me.chl
-+2,_eq ~\,I
~()XP>
Inhibitor binds to the allosteric site, the configuration of cata-
lytic site is modified such that substrate cannot bind properly Allosteric enzyme has one c ytic site where the sub-
6. Most allosteric enzymes possess quaternary structure. They strate binds and another separate allosterlc site where
are made up of subunits, e.g. Aspartate transcarbamoylase the modifier binds (a/lo = other) (Fig. 5.25). Allosteric
has 6 subunits and pyruvate kinase has 4 subunits. Examples
and substrate binding sites may or may not be physi-
of allosteric enzymes are shown in Table 5.7.
cally adjacent. The binding of the regulatory molecule
can either enhance the activity of the enzyme (allosteric
compound initially binds with the enzyme and the fi rst Cactivation), or inhibit the activity of the enzyme (allosteric
few steps of the pathway are catalyzed. This new product o)hibition). In the former case, the regulatory molecule is
irreversibly binds to the enzyme and inhibits further
known as the positive @>difier and in the latter case as
reactions.
the negativ difler. The binding of substrate to one
For example, ornithine decarboxylase (ODC) cata-
of the subuni s of the enzyme may enhance substrate
lyzes the conversion of ornithine to putrescine which is
binding by other sub-units. This effect is said to be posi-
necessary for p,Q!yamine synthesis (see Chapter 18).
tive@ operativity. If the binding of substrate to one of
When the ODC in trypanosoma is inhibited , multiplica-
the subunits decreases the avidi!)' of substrate binding by
tion of the parasite is arreste~. Ther~fore inhibitors of
other sites, the effect is called negative operatlvlty.
ODC enzyme such as difluordmethylornithine (DFMO)
' .
has been found to be effective against trypanosomla- In most cases, a combination is observed, resulting in
sis (sleeping sickness). DFMO is initially inert, but on a sigmoid shaped curve (Fig . 5.26). Salient features of
binding with the enzyme, forms irreversible covalent allosteric regulation are enumerated in Box 5.10.
complex with the coenzyme (pyridoxal phosphate) and f' K E
the amino acid residues of the enzyme. In mammalian ey nzyme
cells, the turnover rate of ODC is very high, and so-the· Body uses allosteric enzymes for regulating metabolic
inhibition by DFMO is only transient. So D'FMO kill s the pathways. Such a regulatory enzyme in a particu-
parasites with no side effects to the patient. lar pathway is called the key enzyme or rate limiting
A similar mechanism is observed in the case of enzyme. The flow of the whole pathway is constrained
Allopurinol that is oxidized by xanthine oxidase to as if there is a bottle neck at the level of the key enzyme.
Chapter 5: Enzymology: General Concepts and Enzyme Kinetics 67

A E1 B E2 C ~
---
Allosteric Allosteric
inhibitor activator Chapters In this pathway, if D inhibits E1 , it is called feedback
ATP, citrate AMP, F-2,6-P 10 inhibition.
Heme 22 One example is the inhibition of ALA synthase by
3. Aspartate trans- CTP ATP 38 heme. Other examples are AMP inhibiting the first step
carbamoylase
in purine synthesis (see Chapter 38). Usually such end
4. HMG CoA reductase Cholesterol 14
product inhibition is effected allosterically. Inhibition of
. Pyruvate carboxylase ADP Acetyl-CoA 10
aspartate transcarbamoylase by CTP is another example
. Acetyl-CoA carboxylase Acyl-CoA Citrate 13
for end product inhibition.
7. Citrate synthase ATP 20
8. Carbamoyl phosphate NAG 17 . Induction ~--~--e..-l-~--,J
synthetase I
9. Carbamoyl phosphate UTP 38 Induction is effected through the process of derepres-
synthetase II
sion. The inducer will relieve the repression on the oper-
ator site and will remove the block on the biosynthesis of
The allosteric inhibitor is most effective when substrate the enzyme molecules. Classical example is the induc-
concentration is low. This is metabolically very significant. tion of lactose-utilizing enzymes in the bacteria when
When more substrate molecules are available, there is the media contains lactose in the absence of glucose
less necessity for stringent regulation. (details in Chapter 42). There will be a minimal level of
A few examples are given in detail below: the enzyme inside the cell, but in presence of the indu-
a. Phosphofructokinase-1
cer, the level will go up to thousand or million times within

, . Fructose-6-phosphate +ATP ~ Fructose,1,6


bisphosphate +
hours. By this mechanism nutrients are utilized most
efficiently; while the enzyme synthesis is kept to the
ADP optimum ryptophan pyrrolase and transaminases}EQ ••
This is th4J R41MM i§fep 10 glviiiliisWATP acts as are induced by glucocorticoidG lucokl nase is induced V

J
by insulin LA synthase is induced by barbiturates.
'1r __
an allosteric inhibitor (negative modifier) of PFK1.
When the cellular level of ATP rises, glycolysis can •
slow down and ATP binds to the allosteric site, thus!.Repression ~1-"'-.. _
inhibiting the reaction. A Wgb !eye! of AMP within Even though both inhibition and repression reduce the
the cell jndjcates a low level of ATP. Hence AMP enzyme velocity, the mechanisms are different. In the
acts as an allosteric activator (positive modifier) of case of inhibition, the inhibitor acts on the enzyme
the enzyme. directly; the inhibitory activity is noticed as soon as the
b. ALA synthase inhibitor is added; and the number of enzyme molecules
Succinyl-CoA + - - - delta- amino-levulinic is not changed by the inhibitor.
Glycine acid (ALA) On the contrary, repressor acts at the gene level;
the effect is noticeable only after a lag period of hours or
This is the @ p in Siiiii~im¥Dltbu~ The
days; and the number of enzyme molecules is reduced
end product, heme will allosterically inhibit the ALA
in the presence of repressor molecule. Details of repres-
synthase. This enzyme is the - .,-.u.'fme of heme
sion are given in Chapter 42. A summary of the mecha-
esi~ (see Chapter 22). Other such allosteric
nism of repression is given in Figure 5.27.
enzymes are listed in Table 5.7.
The key enzyme of heme synthesis, ALA synthase
is autoregulated by heme by means of repression. The
GJ. Feedback Inhibition structural gene is transcribed and later translated to pro-
The term feedback inhibition or end-producl inhibition duce the enzyme molecules. The transcription process
means that the activity of the enzyme is inhibited by the starts at the operator site when it is free. When heme
final product of the biosynthetic pathway. is not available, this operator site is open, and therefore
68 Section A: Chemical Basis of Life

-
ooc - Hc coo-
Regulator Operator Structural genes
II
c- coo-
l
CH-OH

! i.
H
I
A~orepressor } Holorepressor
t
ALA synthase
Fumaric acid (trans)

HC- COO- Fumarase


CH2
I -
coo
Corepressor
.,_ieme. TI \,tf\°\
!
Heme
11
H- c- coo-
Maleic acid (cis)
Malicacid

Fig. 5.28: Example of stereospecificity

TABLE 5.8: Examples of covalent mod1ficat1on For example, heme synthesis (see Chapter 22), urea
Enzyme Phosphorylated enzyme cycle (see Chapter 17), gluconeogenesis (see Chapter 10).
Acetyl-CoA carboxylase Inact ive The intermediates have to be shuttled across the mito-
Glycogen synthase Inactive chondrial membrane for this purpose which provides a
Pyruvate dehydrogenase Inactive point where controls can be exerted.
HMG-CoA reductase Inactive
Pyruvate kinase Inactive
PFK2 Inactive

will oxidize only UC0S9 tind no other isomeric


the operator site. Now fupJ,er pcaductjon of ALA syn-
th...s.§e is stopped form. Thus, these enzymes show absolute specificity.

j . Covalent Modification' ~ Bond Specificity


The activity of enzymes may be increased or decreased Most of the proteolytic enzymes are showing group
by covalent modification. It means, either addition of - ="- p ficity. For example, frypsin can hydrolyze .._.
a group to the enzyme protein by a covalent bond; or peptide bonds-f6'fmed by carboxyl groups of arginine or
r ova fa g§ ~eaving a covalent bond. lysine residues in any protein. ,
ogen acti;lii'on by partial proteolysis is an ~ \"'lv,c:J.
example of covalent activation. Addition or removal C roup Specificityi · ~\\~-x:,t.
a particular group brings about covalent modification o
One enzyme can catalyze the same reaction on a group
enzyme protein. This is a reversible reaction.
of structurally similar compounds, e.g. hexokinase can
The most common type of covalent modification is •
the reversible ptotein phosphorylation (see Table 5.8). catalyze phosphorylation of glucose, galactose and
mannose.
The phosphate group may be attached to serine, threo-
nine or tyrosine residues.
@Stereospeclficity
((.Compartmentalizatiori Human enzymes are specific for L-amino acids and
The activity of enzymes catalyzing the different steps D-sugars. Fumarase will hydrate fumaric acid (trans
in a metabolic pathway may be regulated by compart- form) to malic acid; but the corresponding cis form, maleic
mentalization of enzymes. Certain enzymes of the path- acid will not be acted upon (Fig. 5.28). Lactate dehydro-
way may be located in mitochondria whereas certain genase, acting on pyruvate will form only L-lactate, but
o_ther enzymes of the same pathway are cytoplasmic. not the D variety.
Chapter 5: Enzymology: General Concepts and Enzyme Kinetics 69

@ Ribozymes A) lsoenzymes may be For


In the first part of this chapter, it is stated that all in Different Ways ~ LJ.r. v,.... 15

enzymes are proteins. Invariably, all rules will have 1. They may be products of different genes (more
exceptions. Ribozymes are RNA molecules with enzy_
matic activity, which catalyze cutting of in mRNA than one locus) in which case they are known as
:r::~ary trans~ t (see Chapter 40 ). true i"soenzymes. The genes may be located on
different chromosomes, e.g. salivary and pancrea-
tic amylase.
14 ody Metabolism is certain cases, all the different forms are presen
Controlled by Enzyme . . . _ ~ .,,· the same individual, e.g. lactate dehydrog
1
(1!1. a pipe, theoretically water can ltotlV'1be'l~ ~ lle":'"y)j::tt"""' nase (LOH) has 5 isoenzymes and all are seen i
¢ira>rficajjy me rjow 1s Ohldlfettlo(™. In the same way, all persons in the population (see Chapter 6).
although most of the individual enzyme activities are 3. The same locus of the gene may have diffetent
readily reversible, the pathway as a whole, tends to lead alleles alternate forms). Such alleleic isoenzymes
towards the final produ~ are called allozymes. In this case, only one form
/ n the reaction series, B C, as soon as C is will be present in one individual; but all the different
pro~ d, it is utilized for something else, so that the forms will be seen in total population. For example,
reactions as a whole, tend to go in the forward direction more than 400 distinct form of glucose-6-phos-
only. True equilibrium of chemical reactions in the cell phate dehydrogenase (GPD)~have been identi-
is achieved only when the cell is dead. In other words, fied; all of them are produced by the same locus
living cells avoid the state of chemical equilibrium. At oo the X chromosome. When isoenzymes due to
the same time, if we analyze a particular metabolite, the variation at a single locus occur with appr_eciable
concentration is more or less kept constant over_very frequency (more- than 1 % in population), it is said to
l_ong periods. This is called steady-state systerv For be
( example, QJ,Qnd .glucose lev.el is kept ithin a narrow 4. Molecular heterogeneity of enzymes may also be
\ n~. because the factors, which tend to increase the produced after the protein is synthesized -
level are at dynamic equilibrium with the factors that try These are called iso-
to decrease the leve} forms, e.g. sialic acid content in alkaline phospha-
Deficiency of the enzyme or its activit~ will lead to tase (ALP) isoenzymes. Different types of iso-forms
genetic defects, inborn errors of metabolism. may be seen in the same individual.

ISOENZVM
They are physica istinct forms same 1. In Agar gel or polyacrylamide gel electrophoresis
enzyme activity; Multiple molecular forms of an enzyme the isoenzymes have different mobility.
are described as iso-enzymes or isozymes. If One-rupee' n be separated by electro-
coins are examined carefully, there will be minor varia- phoresis.
tion~ of '}dges on the rims and number of dots below 2. Heat stability: One of the isoenzymes may be
i the year.\.ln ,tt,e market all these coins have the same easily denatured by heat, e.g.
face valuek ut to an experienced numismatist, these
variations will explain from which mint it was produced) 3. Inhibitors: One of the isoenzymes may be sensi-
In the same way, different molecular forms of the same tive to one inhibitor,
enzyme synthesized from various tissues are called iso- 4. Km value or substrate specificity may be different
enzymes. Hence, study of isoenzymes is very usefu,I to for isoenzymes,
understand diseases of different organsfuthe subunits
are all the same, the protein is a homomultime., repre- 5. Cofactor requirements may be different for iso-
sented by a single gene. If the subunits are different, enzymes.
protein is said to be a heteromultimer produced by t and the . ·:tl..'-1.:L.:

different gene1
70 Section A: Chemical Basis of Life

6. Tissue localization may be different for isoen- 12. proposes a confor-


zymes. , while mational change in active site to allow binding of
regulatory molecules.
7 . Specific antibodies may identify different types of 13. Enzyme activity is influenced by enzyme concen-
isoenzymes. For example, tration, substrate concentration, pH, temperature
s. and presence of inhibitors. Velocity is directly pro-
portional to the concentration of enzymes.
Related Topics 14. Velocity at saturating concentration of substrate is
lsoenzymes of LOH, CK, ALP; enzymes used for called• •• • • • • • •• ·
therapeutic and diagnostic purposes and immobilized 15. Km value (Michaelis-Menten Constant), the sub-
enzymes are described in Chapter 6. Enzyme linked strate concentration at half maximum velocity is a
immunosorbent assay as well as protein purification constant for each enzyme-substrate pair.
techniques are summarized in Chapter 31. 16. Km value is characteristic of a given enzyme. No

ILEARNING POINTS, CHAPTER 5


two enzymes can have the same Km value. It
denotes the affinity of the enzyme to its substrate.
1. Enzymes are biocatalysts that are essential for bio- Lesser the Km, greater the affi nity and vice versa.
chemical reactions to proceed in the human body. 17. Km value indicates the affinity of enzyme for sub-
2. Biological activity of enzymes is dependent on th e strate; higher the affinity, lower the Km.
structural conformation of the enzyme protein. 18. Velocity is maximum for each enzyme at an opti-
3. Enzymes can be classified into (i) Oxidoreductases mum pH and temperature.
(e.g. alcohol dehydrogenase) (ii) Transferases (e.g. 19. Inhibition may be reversible or irreversible.
hexokinase) (iii) Hydrolases (e.g. acetylcholine 20. Enzyme inhibition can be competitive or noncom-
esterase) (iv) Lyases (e.g. Aldolase) (v) lsomerases
petitive or uncompetitive. Competitive inhibition is
(e.g. triose phosphate isomerase) (vi) Ligases (e.g.
usually reversible.
acetyl-CoA carboxylase).
21. Competitive inhibitor is a structural analogue of
4. Enzymes requiring the presence of a certain metal
substrate that•binds to the catalytic site.
ion for their activity are called Metalloenzymes.
Examples are- in carbonic anhydrase.- in 22. The Competitive inhibitor increases the Km and its
catalase and peroxidase £ If in lipase etc. effect can be reversed by increasing the substrate
5. Apoenzyme (protein part) combines with coen- concentration.
zyme (prosthetic group) to form the fu nctional holo- 23. Many drugs are competitive inhibitors of specific
enzyme. enzymes e.g. Folic acid synthesis is inhibited by
6. Coenzymes may take part in reactions as cosub- sulphonamides since they are structurally similar to
strates, but are regenerated. PABA. Dt.j .
7. Some vitamin coenzymes are derivatives of nuc- 24. Actions of drugs such as Sulfonamides, Methotre-
leotide phosphates eg. NAO+, FAD. Deficiency of
xate, Dicoumarol and lsoniazid are based on the
coenzymes can affect the rate of enzymatic reactions.
8. Area of an enzyme where the catalysis occurs is
called th4111aP'. 25. r binds to a site other than '
the catalytic site and reduces the Vmax.
9. Enzymes catalyze reactions by lowering the acti-
vation energy, but does not change the equilibri um 26. · • •• • • • • • • is irreversible and can be
constant.
10. • •• • • • - • . : ,tates that an enzyme (E)
combines with a substrate (S) to form an enzyme-
27.· ···••acan
caused by toxins or poisons.
be regulated by the binding
of positive or negative modifiers to the allosteric site,
substrate (E-S) complex, which breaks down to thus affecting substrate binding to active catalytic site.
give product (P). 28. is an irreversible inhibition. The
11 . puts forward a rigid inhibitor makes use of the natural reaction of the
structure for the active site where substrate binds. enzyme for inhibition. eg. ornithine decarboxylase.
Chapter 5: Enzymology: General Concepts and Enzyme Kinetics 71

29. 11
by reversible protein phospho-
,-
30. are physically distinct forms of the
rylation or zymogen activation is a common mecha- same enzyme activity. They may be products of the
nism of short term regulation. Rate of synthesis of same gene or different genes.
enzyme proteins may be induced or repressed for
long term regulation.

PART-1: ESSAY AND SHORT NOTE QUESTIONS -


5-1 . Enumerate the major classes of enzymes, giving one example for each.
5-2. What are the salient features of t he active site of the enzyme?
5-3. Explain in detail the factors affecting the velocity of an enzyme reaction.
5-4. What are the different types of enzyme inhibition? Explain with suitable examples.
5-5. What is competitive inhibition? Explain with two examples of its therapeutic significance.
5-6. What are the differences between competitive and noncompetitive inhibition? Give two examples for competi-
tive inhibition.
5-7. What is meant by Km value, and what is its significance?
5-8. Explain the mechanism of allosteric regulation using an example.
5-9. With suitable examples, explain the specificity of enzymes.
5-10. What is the importance of covalent modification of enzymes?
5-11 . What are isoenzymes? Give examples. What are their clinical significance?

SHORT NOTE QUESTIONS

-" 5-12.
5-13.
Michaelis Constant (Km).
Coenzymes.
5-17.
5-18.
Proenzymes.
Zymogens.
5-14. Metalloenzymes. 5-19. Effect of pH on enzyme activity.
5-15. Active site of enzyme. 5-20. Optimum temperature.
5-16. Kosh land's induced fit theory.

PART-2: MULTIPLE CHOICE QUESTIONS

5-1 . An example of lyase is: 5-5. All the following statements are true with regard
A. Glutamine synthetase to the enzymes called synthases and synthetases,
B. Fumarase except:
C. Cholinesterase A. Synthetases and synthases are synonyms
D. Amylase B. Synthetases are enzymes belonging to ligases
5-2. The enzyme belonging to the class ligase is: C. Glutamine synthetase is belonging to class of
A. Glycogen synthase ligases
• B. Glutamine synthetase D. Glycogen synthase belongs to the class transfe-
C. Porphobilinogen deaminase rases
D. Histidine decarboxylase 5-6. The coenzyme:
5-3. Coenzymes are: A. Increases the affinity of apoenzyme to substrate
A. Dialyzable, nonprotein molecules B. Lowers the activation energy
B. Colloidal protein molecules C. Increases the number of active sites on the enzyme
C. Structural analogues of enzymes D. Accepts one of the products of the reaction
D. Different forms of the same enzyme 5-7. All the following metalloenzymes contain copper,
5-4. Digestive enzymes belong to the class of: except:
A. Hydrolases B. Ligases A. Superoxide dismutase
C. Lysases D. Oxidoreductases B. Tyrosinase
72 Section A: Chemical Basis of Life
C. Glutathione peroxidase C. Treatmen t of methanol poisoning by ethanol
D. Cytochrome oxidase D. Use of dicoumarol as an anticoagulant
5-8. Which one of the following enzymes is containing 5-17. In competitive inhibition:
molybdenum? A. Inhibitor has structural similarity to substrate
A. Cytochrome oxidase B. Km is decreased
B. Xanthine oxidase C. Vmax is decreased
C. Carbonic anhydrase D. Reaction rate is independent of substrate concen-
D. Phosphoglucomutase tration
5.9. The enzyme: 5-18. In a competitive inhibition:
A. Reduces the energy of activation A. Km is increased and Vmax is increased
B. Increases total energy of substrate
B. Km is decreased and Vmax is normal
C. Increases the equilibrium constant
C. Km is increased and Vmax is normal
D. Increases total energy of the product
D. Km is decreased and Vmax is increased
5.10. In enzyme kinetics, Vmax denotes:
5-19. The antimetabolite, methotrexate acts by:
A. The amount of an active enzyme
A. Noncompetitive inhibition
B. Substrate concentration
B. Formation of a covalent bond between enzyme
C. Half the substrate concentration
and inhibitor
D. Quantity of enzyme substrate complex
5-11. All the following statements are true with regard C. Increasing the Vmax of the reaction
to enzymes, except: D. Competitive inhibition
A. Enzymes lower activation energy 5-20. Ascaris worm is not digested by intestinal enzymes,
B. They alter equilibrium of the reaction because:
C. They accelerate the chemical reaction A. The worm produces a trypsin inhibitor
D. Most of the enzymes are proteins in nature B. Ascaris protein stimulates digestive enzymes
5-12. The Km value of an enzyme is: C. Ascaris has a tough cuticle
A. The substrate concentration at half maximal velocity D. Proteins of the worm is resistant to trypsin
B. Half the substrate concentration at maximal velocity 5-21 . The following are salient features of allosteric
C. Dissociation constant of enzyme-substrate complex enzymes, except:
D. The total enzyme concentration A. They are usually multiple subunit enzymes
5-13. Lineweaver-Burk (double reciprocal) graph is plot- B. They are involved in metabolic regulation
ted by: C. They do not show any change in Km and Vmax in
A. Substrate concentration against Vmax the presence of modifiers
B. Reciprocal of substrate concentration against D. On binding the inhibitor, usually Km is increased
reciprocal of velocity and Vmax is decreased
C . Reciprocal of initial velocity against substrate con- 5-22. Allosteric enzymes show all the following charac-
centration
teristics, except:
D. Km against initial velocity
A. Co-operative binding of the substrate
5-14. When substrate concentration is equal to Km value:
B. Sigmoid kinetics
A. Half of the enzyme molecules are bound to the
C. Binding between substrate and regulatory sites
substrate molecules and the other half are free
D. Substrate binding site and regulatory site are dif-
B. Maximum velocity is achieved
ferent
C. Maximum enzyme molecules are taking part in the
reaction
5-23. All the following enzymes are examples of cova-
lent modifications, except

D. The reaction is now at equilibrium
5-15. Enzymes which are synthesized in inactive form A. Chymotrypsinogen
are called: B. Pyruvate kinase
A. Coenzymes B. Apoenzymes C. Glycogen phosphorylase
C. Lysozymes D. Proenzymes D. Aspartate transcarbamoylase
5-16. All the following are examples of competitive inhi- 5-24. All are multienzyme complexes, except:
bition, except: A. Fatty acid synthase
A. Inhibition of enolase by fluoride ions B. Pyruvate dehydrogenase
B. Inhibition of dihydrofolate reductase by ametho- C . Alpha-ketoglutarate dehydrogenase
ptrine D. Glycogen synthase
Chapter 5: Enzymology: General Concepts and Enzyme Kinetics 73

5-25. One of the following is an example of repression: 5-36. A catalyst acts by


A. ALA synthase by heme A. Speeding up biochemical reactions
B. Succinate dehydrogenase by malonate B. Getting used up during the reaction
C. Cytochromes by cyanide C. Changing the nature of the reactions
D. Glycogen phosphorylase by glucagon D. Providing the driving force
5-26. All the following are examples of enzyme induc- 5-37. Which of the following is NOT true regarding enzy-
tion, except: mes?
A. Beta galactosidase by lactose A. Highly specific catalysts
B. Tryptophan pyrrolase by glucocorticoid B. Regenerated at the end of reactions
C. Transaminases by insulin C. Remains bound to the product
D. ALA synthase by barbiturates D. Proteins in nature
5-27. Which enzyme requires chloride ion as activator? 5-38. Three-dimensional structure is altered when
A. Amylase B. Lipase A. Enzyme binds to the compound on which acts
C. Trypsin D. Elastin B. Enzyme substrate complex dissociates
5-28. All compounds listed below act as competitive C. Enzymes protein is remaining idle
inhibitors of enzymes, except D. Enzyme dissociates from product
A. Methotrexate B. Sulfonamide 5.39. Which of the following is NOT a major class of
C. Allopurinol D. Fluoride enzymes?
5-29. A competitive inhibitor will affect the enzyme activity: A. Transferases B. Dehydrogenases
A. Precipitating the enzyme protein C. Hydrolases D. lsomerases
B. Decreasing Km value 5.40. The coenzyme which is NOT a derivative of B com-
C. Increasing Vmax
plex vitamin?
D. Irreversibly denaturing the enzyme
A. FAD B. NAO•
5-30. In an enzyme assay, the substrate concentration is
C. TPP D. Carnitine
lower than the Km, then the rate is:
5-41. Which of the following dehydrogenases is NOT
A. Independent of enzyme concentration
coenzyme specific?
B. Independent of temperature
A. Lactate dehydrogenase
C. Proportionate to the substrate concentration
B. Glutamate dehydrogenase
D. Approximately equal to Vmax.
C. Succinate dehydrogenase
5-31. Enzyme activity in biological systems may be affec-
D. Glucose -6-phosphate dehydrogenase
ted by all the following, except
5-42. Which of the following is NOT TRUE regarding
A. Substrate analogues
active site of an enzyme?
B. Inducers
A. Active site occupies the major portion of enzyme
C. Immobilization of enzymes
protein
D. Diisopropyl fluorophosphate
B. It has a three dimensional shape to accommodate
5-32. lodoacetate inhibits enzyme by reacting with which
substrate
particular group?
C. Amino acid residues at active site may take part in
A. Amide B. Sulfhydryl
C. Carboxyl D. lmidazole catalysis
5-33. Enzymes synthesized in inactive form are called: D. Alignment of active site can be altered by regulatory
A. Coenzymes B. Apoenzymes molecules.
5-43. Enzymes bind substrate:
• C. Lysozymes D. Proenzymes
5-34. A constitutive enzyme shows all the features, except A. By covalent bonds
A. Activated by an inducer B. Irreversibly
B. Inhibited by a repressor C. At the catalytic site only
C. Always present in the cell in the same concentration D. Peptide bonds
D. A constituent of the cell membrane 5-44. Enzyme substrate complex breaks down to pro-
5-35. Synthesis of an enzyme is increased by its subst- ducts and enzyme is regenerated:
rate ; this phenomenon is called as: Which of the following equations is correct?
A. Allosteric activation A. [E] + [SJ [ES] Products
B. Induction B. [E] + [SJ .... [ES] E + Products
C. Co-operative activation C. [E] + [SJ [ES] [SJ + Products
D. Feed forward activation D. [E] S Products
74 Section A: Chemical Basis of Life

5-45. The lock and key hypothesis explains all features C. Cannot be extrapolated from the double reciprocal
of enzyme action except: plot
A. Specificity of enzyme action D. A constant for all enzyme substrate pairs .,
B. Binding of substrate to active site 5-54. Enzyme inhibition is applicable in all except:
C. Three dimensional shape of active site A. Regulation of metabolic pathways
D. Effect of regulatory molecules B. Use of drugs as enzyme inhibitors l
5-46. Koshland's induced fit theory explains C. Enhance the action of toxins
A. Rigidity of active site D. To check excess effect of enzymes
B. Effect of modifiers on substrate binding 5-55. Reversible enzyme inhibition is:
C. Complementary shape of active site and substrate A . By a substrate analog
D. Presence of specific catalytic residues at active site B. Exhibited by toxins
5-47. Which of the following factors have NO effect on C. Characterized by a change in Vmax only
velocity of enzyme catalyzed reactions? D. Does not occur in biological systems
A. Concentration of substrate 5-56. The statement which is NOT correct regarding
B. pH of the medium competitive inhibition is:
C. Temperature of the medium A. Reversible
D. Equilibrium constant of the reaction B. Decreases Km
5-48. Velocity of enzyme action is dependent on all C. Decreases Vmax
except: D. Inhibitor binds to active site
A. Concentration of e nzyme molecules 5-57. The inhibitor increases the Km and its effect can
B. Turn over rate of enzyme molecules be reversed by increasing the substrate conce n-
C. Rate of synthesis of enzyme protein tration. Which of the following is an example of
D. Molecular weight of the enzyme protein competitive enzyme inhibition?
5-49. Maximum velocity is seen when: A. Effect of cyanide
A. All enzyme molecules are bound to substrate B. Inhibition of glycolysis by fl uoride
B. Substrate molecules are far in excess of enzyme
molecules
C. Arresting cell division by methotrexate
D. Toxic effect of arsenate •
C. Specificity of substrate binding is high 5-58. Which of the following is a folate antagonist in
D. Activators are present in the medium eukaryotes?
5-50. Km value indicates: A. Sulphanilamide
A. Affinity of enzyme for substrate B. Trimethoprim
B. Maximum velocity C. Amethopterine
C. Specificity of substrate D. Azaserine
D. Saturating concentration of substrate 5-59. All are mechanisms of enzyme regulation In bio-
5-51 . Michaelis-Menten substrate saturation curve is logical systems except:
obtained by plotting [S] against [V] Since Michae- A. Covalent modification
lis-Menten graph is a hyperbolic curve. B. End product inhibition
A. Binding properties of the enzyme are not clear C. Enzyme induction
B. Km cannot be accurately measured D. Enzyme denaturation
C. Velocity is not proportional at very low concen- 5-60. Allosteric enzymes can be regulated by the bind-
!ration of substrate ing of positive or negative modifiers to the allos-
D. Is not helpful to study effect of inhibitors teric site, thus affecting substrate binding to active i-

5-52. A double reciprocal plot, 1/[SJ Vs 1N is a straight catalytic site. Which of the following metabolic
line graph. All are true regarding the Lineweaver intermediates is an effective allosteric modifier?
Burke plot except: A. Pyruvate
A. It is a double reciprocal plot B. Lactate
B. Km can be accurately measured C. Citrate
C. X and Y intercepts directly indicate Km and Vmax D. Glutamate
D. Does not change in presence of inhibitors. 5-61 . Long term regulation of enzyme action is effected
5-53. Which of the following is true regarding Vmax? by:
A. Indicates the initial velocity at Km A. Altering the activity of enzyme proteins
B. Is measured as the number of moles of substrate B. Induction and repression of enzyme protein syn-
converted to product thesis
Chapter 5: Enzymology: General Concepts and Enzyme Kinetics 75

C. Premature breakdown of enzyme protein B. Phosphorylation of hormone sensitive lipase by


D. Partial denaturation of enzyme proteins glucagon
5-62. All the following are examples of enzyme regu- C. Inhibition of heme synthesis by lead
lation in living cells except: D. Increased rate of synthesis of transaminases by
A. Induction of glucokinase by insulin cortisol
,,.
ANSWERS OF MULTIPLE CHOICE QUESTIONS

5-1 . B 5-2. B 5-3. A 5-4. A 5-5. A 5-6. D 5-7. C


5-8. B 5-9. A 5-10. A 5-11 . B 5-12. A 5-13. B 5-14. A
5-15. D 5-16. A 5-17. A 5-18. C 5-19. D 5-20. A 5-21 . C
5-22. C 5-23. D 5-24. D 5-25. A 5-26. C 5-27. A 5-28. D
5-29. D 5-30. C 5-31 . C 5-32. B 5-33. D 5-34. C 5-35. B
5-36. A 5-37. C 5-38. A 5-39. B 5-40. D 5-41 . B 5-42. A
5-43. A 5-44. B 5-45. D 5-46. B 5-47. D 5-48. D 5-49. A
5-50. A 5-51 . C 5-52. D 5-53. B 5-54. D 5-55. A 5-56. C
5-57. C 5-58. C 5-59. D 5-60. C 5-61 . B 5-62. C

PART-3: VIVA VOCE QUESTIONS AND ANSWERS

5-1 . How are enzymes classified? 5-14. Give examples of synthases.


They are classified into 5 major classes. Glycogen synthase, ALA synthase, IMP synthase.
5-2. What are those classes? 5-15. What are coenzymes?
Oxidoreductases, transferases, hydrolases, lyases, They are non-protein part of enzyme. They are low
isomerases and ligases. molecular weig ht organic substances, without which
5-3. Give an example of oxidoreducatase. the enzyme cannot exhibit any reaction. Coenzyme
Alcohol dehydrogenase. accepts one of the products of the reaction; and so
5-4. What is the function of transferases? acts as a co-substrate.
Transfer of groups other than hydrogen. 5-16. How are coenzymes classified?
5-5. Give an example of transferase. (a) Those taking part in reactions catalysed by oxidore-
Hexokinase ductases by donating or accepting hydrogen atoms or
5-6. What is the function of hydrolases? electrons. (b) Those coenzymes taking part in reac-
Cleave the bond after adding water. tions transferring groups other than hydrogen.
5-7. Give an example of a hydrolase. 5-17. Give some examples of coenzymes involved in
Acetylcholine esterase. oxidoreductases.
5-8. Peptidases are classified under which class of NAO•, NADP-, FAD .
enzyme? 5-18. What is the full form of NAD•?
Hydro lases. Nicotinamide adenine dinucleotide.
5-9. What is the function of lyases? 5-19. What is the full form of ATP?
Cleave bond without adding water. Adenosine triphosphate.
5-1 0. Which enzyme will add water to a double bond, 5-20. What is the function of ATP?
without breaking the bond? It is the energy currency in the body. During the oxida-
Hydratase. tion of food stuffs, energy is released, a part of which
5-11 . What is the function of ligases? is stored as chemical energy in the form of ATP.

- ATP dependent condensation of two molecules.


5-12. What is the difference between synthase and syn-
5-21 . Name some enzymes containing copper.
Superoxide dismutase; Tyrosinase; Cytochrome oxi-
thetase? dase
Synthetases are ATP-dependent enzymes Synthases 5-22. Which metal is required for the kinases?
do not require ATP directly; they belong to classes oth- Magnesium.
er than Ligases. 5-23. Which enzyme contains molybdenum?
5-13. Give examples of synthetases. Xanthine oxidase.
Carbamoyl phosphate synthetase; Argininosuccinate 5-24. Name some iron containing enzymes.
synthetase. Cytochrome oxidase, catalase, peroxidase.
76 Section A: Chemical Basis of Life
5-25. What is Michaelis-Menten theory ? 5-40. What is effect of pH on activity of an enzyme?
It is otherwise called Enzyme-Substrate complex theory. Each enzyme has an optimum pH, on both sides of
The enzyme combines with the substrate, to form an which the velocity will be drastically reduced.
enzyme-substrate complex, which immediately breaks 5-41 . What is the explanation for the effect of pH?
down to the enzyme and the product. The pH decides the charge on the amino acid resi-
5-26 What is Fischer's theory? dues at the active site. The net charge on the enzyme
It states that the three dimensional structure of the ac- protein would influence substrate binding and catalytic
tive site of the enzyme is complementary to the sub- activity.
strate. Enzyme and substrate fit like a key and its lock. 5-42. What is the optimum pH of usual enzymes?
5-27. What is Koshland's induced fit theory? Usually enzymes have the optimum pH between 6
Substrate induces conformational changes in the en- and 8.
zyme, such that precise orientation of catalytic groups 5-43. Give some exceptions for this general rule.
is effected. Pepsin (optimum pH 1-2); Alkaline phosphatase (opti-
5-28. What is the active site of an enzyme? mum pH 9-1 0) and Acid phosphatase (4-5).
That area of the enzyme where catalysis occurs is re- 5-44. What is a zymogen?
ferred to as active site or active center. It is otherwise called pro-enzyme. Inactive zymogen is
5-29. What is meant by serine proteases? activated by removal of a piece of the pro-enzyme.
Proteases (proteolytic enzymes) having a serine resi- 5-45. Give an example of zymogen activation.
due at its active center. By removing a small polypeptide from trypsinogen, the
5-30. Give an example of a serine protease. active trypsin is formed.
Trypsin, chymotrypsin, thrombin. 5-46. What is the significance of zymogen activation?
5-31 . What do enzymes on activation energy? Gastrointestinal enzymes are synthesised in the
Enzymes lower activation energy. form of proenzymes, and only after secretion into the
5-32. What is the action of enzymes on equilibrium? alimentary canal, they are activated. This prevents
They increase the chemical reaction, but do not alter autolysis of cellular structural proteins.
equilibrium of the reaction. 5-47. What are the different types of inhibitions of
5-33. What are factors influencing enzyme reaction? enzyme?
Enzyme concentration, substrate concentration, Competitive inhibition: Non-competitive inhibition; Sui-
product concentration, temperature, pH and presence cide inhibition: and Allosteric regulation.
of activators or inhibitors. 5-48. What are the features of competitive inhibition?
5-34. What is Km value? 1. Competitive inhibitor is a structural analogue. 2. It is
Substrate concentration (expressed in moles/L) at reversible. 3. Km is increased. 4. Vmax is not changed.
half-maximal velocity is the Km value. 5-49. Give examples of competitive inhibition.
5-35. What does it indicate? Malonate inhibits succinate dehydrogenase.
It denotes that 50% of enzyme molecules are bound 5-50. Give examples of clinical application of competi-
with substrate molecules at that particular substrate tive inhibition.
concentration. (a) Sulphonamide inhibits PABA incorporation in bac-
5-36. What is the significance of Km value? teria; and so acts as an antibacterial agent. (b) Metho-
Km is independent of enzyme concentration. Km value trexate inhibits folate reductase . (c) Dicoumarol inhibits
is thus a constant for an enzyme. It is the characteristic vitamin K.
feature of a particular enzyme for a specific substrate. 5-51 . What are the features of noncompetitive inhibition?
5-37. What is the relation of Km value with the affinity? 1. Noncompetitive inhibitor has no structural similarity
Km denotes the affinity of enzyme to substrate. Thus, with the substrate. 2. It is generally not reversible. 3. •
the lesser the numerical value of Km, the affinity of the Km is not changed. 4. Vmax is reduced.
enzyme for the substrate is more. 5-52. Give examples of noncompetitive inhibition.
5-38. What is the effect of temperature on enzyme velo- (a) Diisopropyl fluorophosphate inhibits trypsin. (b) /I

city? Fluoride inhibits enolase.


The velocity of reaction increases when temperature 5-53. What is suicide inhibition?
is increased; reaches a maximum and then falls (Bell In suicide inhibition, the structural analogue is con-
shaped curve). verted to a more effective inhibitor with the help of the
5-39. Why it falls? enzyme to be inhibited. The inhibitor makes use of the
When temperature is more than 50°C, heat denatura- enzyme's own reaction mechanism to inactivate it.
lion occurs with consequent loss of tertiary structure of 5-54. What is another term for suicide inhibition?
protein. Mechanism-based inactivation.
Chapter 5: Enzymology: General Concepts and Enzyme Kinetics 77
5-55. Give examples for suicide inhibition. 5-62. Give an example of induction.
(a) Ornithine decarboxylase (ODC) is inhibited by diHu- Induction of lactose-utilising enzymes in the bacteria
ro methyl ornithine (DFMO). (b) Allopurinol is oxidised
• by xanthine oxidase to alloxanthine that is a strong
when the media contains lactose in the absence of
glucose. In humans, ALA synthase is induced by bar-
inhibitor of xanthine oxidase.
bi tu rates.
5-56. What is allosteric inhibition?
• Allosteric enzyme has one catalytic site where the sub-
5-63 . What are constitutive enzymes?

strate binds and a separate allosteric site where the Enzymes whose concentration in a cell is always the
modifier binds. same.
5-57. What are the features of allosteric inhibition? 5-64. What is repression?
(1) The inhibitor is not a substrate analogue. (2) It is Repression acts at the gene level; the number of enzyme
partially reversible, when excess substrate is added. molecules is reduced in the presence of repressor.
(3) Most allosteric enzymes are made up of subunits. 5-65. Give an example of repression .
5-58. Give examples for allosteric inhibition. ALA synthase is repressed by the heme.
ALA synthase, Aspartyl transcarbamoylase, HMG-CoA 5-66. What are the types of specificity?
reductase.
Absolute specificity, group specificity and streospeci-
5-59. What is covalent modification?
ficity.
Addition or removal of a group to the enzyme by a co-
5-67. Give an example for absolute specificity.
valent bond.
5-60. Give some examples of covalent modification. Urea is the only substrate for urease.
Glycogen phosphorylase is active when phosphory- 5-68. Give an example for group specificity.
lated and inactive when dephosphorylated. Trypsin can hydrolyse peptide bonds formed by ca r-
5-61 . What is meant by induction? boxyl groups of arginine or lysine residues.
Induction is effected at the level of DNA. The inducer 5-69. What are isoenzymes?
will relieve the repression on the operator site and will They are physically distinct forms of the same enzyme

-'
remove the block on the biosynthesis of the enzyme activity. They have identical catalytic properties, but
molecules. differ in structure.
_ _ _ _ _ Chapter

Clinical Enzymology

/
Chapter at a Glance
The learner will be able to answer questions on the following topics:

Creatine kinase ( ( 1..:..) iD Cholinesterase (CE)


Alkaline phosphatase Cf\L P) Glucose-6-phosphate dehydrogenase C(-{,P H)
{ Acid phosphatase Cf) c F Amylase and lipase (f. £.
1.-)
JD Prostate specific antigen C Ps A) Enzymes used as therapeutic agents

®
Plasma contains many functional enz;ymes, which are A "biomarker" is a t which is useful
actively secreted into plasma. For example, enzymes of in nctidn of an organ. Cardiac biomarkers
blood coagulation. are used to detect cardiac diseases, which may be:
On the other hand, there are a few non-functional a. Acute coronary syndrome resulting from myocardial
enzymes in plasma, which are coming out from cells ischemia
of various tissues due to normal wear and tear. Their
b. Congestive cardiac failure due to ventricular dys-
normal levels in blood are very low; but are drastically
function (Box 6.1). The different markers are used to:
increased during cell death (necrosis) or disease (Fig.
i. Detect myocardial ischemia at the eacliest
6.1 ). Therefore, assays of these enzymes are very
useful in diagnosis of diseases The reference values ii. Monitor the progression of the condition
of serum enzyme levels in normal people may vary iii. Predict the risk in cardiac dysfunction.
slightly depending on the method used. The normal Commonly used biomarkers for early detection of acut@
myocardial infarction are:
1. Cardiac troponins, Tnl and TnT

Unstable angina
Suspicious ECG i hanges
History suggestive of myocardial infarction
v. Following surgical coronary revascularization
vi. Patients with hypotension and s ne'
Chapter 6: Clinical Enzymology 79

Serial testing of the following cardiac enzymes is usually done to


• g uide the prognosis. No single marker can successfully identify or

. exclude acute Ml w ithin the first 6 ho ur.


Creatine kinase (CK-MB)

t ii. Cardiac troponin L(CTI) and Cardiac troponin T (CTT), These


are z a. ·
iii. Brain natriuretic peptide (BNPY, It is a of
ventricular function.
- CK iv. Lactate dehydrogenase (LOH) and aspartate aminotransferase
- Troponin (AST) were pre:tiously 11seclas.markers.cfmyocard.i,alJunaion,
b ut no..more used in clinical practice. Markers for Risk for fu-
ture G rdiac event are described in Chapter 15.

0 2 3 4 5 6 7 8 9 10 11 12
Days after infarction
Mean percentage in
enzyme blood
MM(CK3)-,.._ s t_ ____;=-----==~+-
Le_a_ !'.'il
_O _ _ _ _--=-i
_% l
,,

Values_
SO .:t 4-0 K and Muscle Diseases
Normal serum value for CK is 15-100 U/L for maws an
10-80 U/L for females. ''0.i. 2 o U he level of CK in serum is very m~ h elevated in mus-
cular dystrophies (500-1500 IUILLJfu female carriers
Kand.Heart Attack of X-linked muscular dystrophy (heterozygous), CK is
CK value in serum is increased in myocardial infarction. moderately raised. £ K level is highly elevated in crush
The pattern of alteration in enzyme level with time is · · cerebrovascular accident~
shown in Figure 6.3 and Table 6.1. The CK level starts
to rise within 3-6 hours of infarction. Therefore, CK
estimation is very useful to detect early cases, where J
ECG changes may be ambiguous.
. The area under the ~ ~!!!l~A~C..!.TROPONINS (CTI/CTT)
peak and slope of initial rise are proportional to the size They are not enzymes. However, Troponins are now
of infarct. accepted as specific markers for myocardial infarction,
80 Section A: Chemical Basis of Life

TABLE 6.3: Charactensllc features of LOH Isoenzymes


Subunit make up of Elecrrophoretic mobility Tissue from which iso- Percentage in human serum
(Mean)
No. ofisoenzyme isoenzyme atpHB.6 enzyme has originated

·.
LDH-1 H4 Fastest Heart muscle 30% I

LDH-2 H3Ml Faster RBC 35%


LDH-3 H2M2 Fast Brain 20% I I a
H 1M3 Slow 10% I
LDH-5 M4 Slowest Skeletal muscle 5%

and hence discussed here. Some more details on troponin


are given in Chapter 49. Measurement of cardiac tro-
ponin~ has ~com_e one of th~ main _tes~s in early ~etectil
••ID
and therefore minor amount of hemolysis will
esult in_a false positive te~t. An inc~ease in LOH values
of an IschemIc episode and In morntonng the patient. is seen in presence of malignancy since the cancer cells
The troponin complex consists of 3 component · redominantly have anaerobic glycolysis. ,
troponin C (calcium binding subunit), troponin I f g..ctp-
myosin ATPase Ufil,iblioiy...,subunit), and troponin (tr 1l- DH and Heart Attack
pomyosin binding subunit). Cardiac isoform is specific In myocardial infarction, total LOH activity is increased,
for cardiac muscle; the amino acid sequence is different while H4 isoenzyme is increased 5-1 O times more.
in skeletal muscle isoform. Troponins are seen in ske- \> He~'t:.
letal and cardiac muscles, but not in smooth muscles. \ ifferentia1 Diagnosis
Troponin I is released into the blood within 4 hou
after the onset of symptoms of myocardial ischemia; Increase in total LOH level is seen in hemolytic ane-
peaks at 14-24 hours and remains elevated for 3-5 mias, hepatocellular damag,e, muscular dystrophy, car-
days after infarction. Therefore, cinomas, leukemias, and any condition which causes
k. It is not necrosis of body cells. Since total LOH is increased in
increased in muscle injury; whereas CK2 may be slightly many conditions, the study of LOH is rarely done now-
elevated in some muscle injury. The initial increase is due adays to diagnosis myocardial infarctio~ -
to liberation of the cytoplasmic fraction and sustaine
elevation is due to the release from myofibrils.
oenzymes of LOH Q~ t:
Serum level of Troponin T (TnT) increases within LOH enzyme is a tetramer with 4 subunits. iJ,t~ nit
6 hours of myocardial infarction, peaks at 72 hours may be either H (heart) or M (muscle) pol:yptide chains.
and then remains elevated up to 10-1 4 days. So, 5 combinations of H and M chain are possibler
H4, H3M, H2M2, M3H and M4 vari ties, forming 5
_. _':__i _i, • - •t . I -• I NII but still have a isoenzymes. All these 5 forms are seen in all persons.
risk of having an adverse cardiac event. High sensi- The isoenzymes are usually separated by cellulose
tive TnT (hsTnT) enables determination of very lo~ a t electrophoresi at pH 8.6 (Table 6.3). M4 form
concentrations. The higher sensitivity of this assay~ in skeletal muscles while H4 form is seen in
allowed for improved identification of patients with AM eart. Normally LOH-2 (H3M 1) concentration in blood is
presenting in the first 3 hours following onset of symp- greater than LOH-1 (H4); but this~ sed in
toms. r.•-llliilll•llllil•ililllllll·•••~!1D myocardial infarction; this is calle~
he current guideline for early diag- was previously used
nosis of AMI includes elevated troponin levels; either as a marker of myocardial ischemia, but new specifi c
high sensitive Troponin Tor ultra sensitive Troponin I. markers have replaced it.

CTATE DEHYDROGENASE @ Brain Natriuretic Peptide (BNP)


LOH LO The natriuretic peptide family consists of three peptides:
Atrial natriuretic peptide (ANP), brain natriuretic peptide
@ Reference Values (BNP), and C-type natriuretic peptide (CNP).
The LOH will convert pyruvate to lactate (see Chapter ANP is produced primarily in the cardiac atria.
10). Normal value of LOH in serum is 100-200 U/L. BNP is present in human brain, but more in the cardiac
~i-? Chapter 6: Clinical Enzymology 81

. Ies. Human pro-BN P Is cleaved by enzymes into


ven tnc BOX 6.3: Enzyme profile in liver diseases
the active BNP and an inactive peptide. These natriuretic Enzymes commonly studied for diagnosis of liver diseases ar ·
peptides defend against excess salt and water retention. 1. Alanine aminotransferase (ALT)

.. Patients with congestive heart failure have high


plasma concentrations of ANP and BNP. The concen-
2. Alkaline phosphatase (ALP)
3. Gamma-glutamyltransferase (GGT).
f trations are correlated with the extent of ventricular
dysfunction. High concentrations of BNP predict poor
long-term survival. In breathlessness, BNP test help ,,,__ AS
..,,,.....P._A
_ R_T.
_'A_J_E
_____ ____......,
in the differentiation of the cause as heart failure or
obstructive lung disease. Patients with COPD (chronic INOTRANSFERASE (AST}
obstructive pulmonary disease) may have increased old literature, it was called as serum glutamate oxalo-
BNP levels. The acetate transaminase (SGOT). AST needs pyridoxal
phosphate (vitamin B6 ) as coenzyme. Normal serum level
of AST is 8-20 U/L. The level is elevated in myocardial
yo ardial Inf infarction. It is significantly elevated in liver diseases.
~ -... __!. _J.-"(Z1.....,1,.1 ~ , n r~UAKI r A marked increase in AST may be seen in primary
yo lo · i rai after myocardi I infarction; put hepatomas. In alcoholic hepatitis AST may be higher than
1 Is not specific and is raised during various otper ALT since mitochondrial enzyme is also liberated.
conditions including muscle injuries. It has the 'l-
_..,,,.,,"'advantage of responding very rapidly, rising '~I n-'l--::=lla.illl-...:KALINE PHOSPHATASE (ALPl
falling earlier than CK-MB or troponin. A nega v . .
value will exclude infarction, and is useful in t e ALP Is a nonspecific enzyme which hydrolyzes aliphatic,
. . 8
(am ;fi4i
.
J
• _kt( e~IY hrn Ics of cbesLpaio 4-5ee Table 6.1). aromatic or heterocyclic compounds. The pH optimum
o- . lschemia modified albumin (IMA) is another can- for the enzyme reaction is between 9 and 10. It is acti-
N~~o didate cardiac marker. Myocardial ischemia alteT' vated by magnesjum and.mangaoe~. ~~o_nsti-
\et?.\ the N-terminus of albumin reducing the ability of co- tuenL,ior:1..oUl.P. It is produced by osteoblasts of bone,
bait to bind to albumin. It has low specificity; therJ- and is associated with the calcification process (see

9) fore generating high number of false positives.


negative value is highly useful, as it rules out t e
Chapter 34 ).

-...:._ ·-~ ... • ... • ' - • !.-=.:_ ' .....


possibility of Ml.
Normal serum value of ALP is U/l. In
children, the upper level of normal value may be more,
nzyme Profiles in ·
because of the increased osteoblastic activity in children.

-
Moderate (2-3 times) increa!jAin ALP level is saen in }
hepatic diseas~ such as i'l,.. n;aiye hepatitis, a1£'holic
ALANINE AMINOTRANSFERASE ~ patitis or hepatocellular carcinoma (see Chapter 24 ).
Very high levels of ALP (10-12 times of upper limit)
J may be noticed in extrahepatic obstruction (obstructive
In old literature, it was called as serum glutamate pyru- jaundice) caused by gallstones or by pressure on bile
vate transaminase SGPJ). The enzyme needs duct by carcinoma of head of pancreas. lntrahepatic
• ~l)_g§pha~e as coenzyme. Details of the reaction cholestasis may be caused by viral hepatitis or by drugs
are shown in [}Qu~i 3.19. Normal seru~ le~I of ALT (chlorpromazine). ALP is produced by w ithe!ialcells 'of
for male is 13-~?"u tL and for female is 1o~b U/L. Very biliary canaliculi and obstruction of biliary passage with
high values (300 to 1000 U/L) are seen in acute hepati- consequent irritation of epithelial cells leads to secretion
tis, either toxic or viral in origin (infective hepatitis). Both of ALP into serum.
ALT and AST levels are increased in liver disease, but Drastically high levels of ALP (10-25 times of
ALT > AST. Rise in ALT levels may be noticed several upper limit) are also seen in bone diseases where
days before clinical signs such as jaundice are mani- osteoblastic activity is enhanced such as I;egeW~
fested. Moderate increase (50 to 100 U/L) of ALT may be .........
(osteitis _JjeiQJ;mans), rickets, osteomalacia, osteo-
seen in chronic liver diseases such as..eifrhosis, .hepati- blastoma, metastatic carcinoma of bone and hyper-
tis C and oen-alcoholic steatohepatitis (NASH). parathyroidism.
82 Section A: Chemical Basis of Life

There a 6 oenzymes of alkaline phosphatase. is necessary for the liquefaction of seminal coagulum. In
Major fractio ound in serum are liver, bone and blood , it is bound to alpha-2-macroglobulin and alpha-1-
intestinal isoenzymes. Out of these, the Alpha-2 antitrypsin; a very small fraction is in the free form also.
variety is heat stable but is inhibited by phenylalao.ine.
It is of inte; tinal/placental origi~ . Heat stable placental
Normal value i.,.,..aa::w.~/ It is very specific for prostate
- - - ~ ~.g / ·.
isoenzyme is found in blood in normal pregnancy. A
--- - -
·- , .., . Values I

. It is named as
Regan isoenzyme (after the first patient in whom it was
detected) or carcinoplacental isoenzyme. Bone alkaline
C ollnesterase (ChE)
phosphatase (BAP), a h~ !~ fraction is used as a Acetylcholine esterase or true ChE can act mainly
marker of metabolic bone disease . ..Qlcio\L +o ~ t . . on acetyl choline. It is present in nerve endings and

~-
'Ii' .
Gamma-Glutamyltransferase (GGT)
in RBCs. Newly formed RBC will have high levels of
ChE which is slowly reduced according to the age of the
It can transfer gamma-glutamyl residues to substrate. cell. Therefore, ChE level in RBCs will be proportional
In the body it is used in ~ h nthesis of glutathione to the reticulocyte count.
(see Chapter 18). GGT h 11 oenzymes. It is seen irreversibly inhibit ChE
in liver, kidney, pancreas, i mal cells and prostate
gland. Normal serum value of GGT is 10-30 U/L. It is
moderately increased in i,tective hepatilis and ~msta~
.z.9pcec17.

*
GGT is clinically important because of its Pseudocholinesterase is nonspecific and can
sensitivity to detect alcohol abuse. GGT is increased hydrolyze acyl esters. It is produced mainly by liver i

in alcoholics even when other liver function tests are cells. Succinylcholine is a widely used muscle relaxant.
within normal limits. GGT level is rapidly decreased Succinylcholine is hydrolyzed by the liver ChE within r
within a few days when the person stops to take alcoho~ 2-4 minutes. But in certain persons the ChE activity may
be absent; this is a genetically transmitted condition. In

such individuals, when succinylcholine is given during
6, ~ Id Phosphatase (ACP) surgery, it may take hours to get the drug metabolized.
s . tma
It hydrolyzes phosphoric acid ester at pH between 4 and
6. Normal serum value for ACP is 2.5--12 U/L. ACP is
secreted by prostate cells, RBC, platelets and WBC.
The prostate isoenzyme is inactivated by tartaric acid.
Normal level of tartrate labile fraction of ACP is 1 U/L. ACP
total value is increased in prostate cancer and highly
It is a . It is an important
elevated in bone metastasis of prostate cancer. In these
enzyme in the hexose monophosphate shunt pathway
conditions, the tartrate labile isoenzyme is elevated. This
of glucose (see Chapter 10). It is mainly used for pro-
assay is very hel ful in follow-up of treatment of rostate
duction of NADPH. Hydrogen peroxide is continuously
cancers. So, (see
Chapter 48). Since blood cells contain excess quantity formed inside the RBC. Peroxide will destroy RBC cell
of ACP, care must be taken to prevent hemolysis while membrane. Glutathione and NADPH will prevent this
taking blood from the patient. Prostate massage may also process. Therefore, NADPH is very essential for pre-
increase the value. So blood may be collected for ACP serving the RBC integrity.
estimation before rectal examination of patient. Drug-induced hemolytic ~ i~P.}GPD is reduced in
drug induced hemolytic anemias. In the GPO deficient
PROSTATE SPECIFIC individuals, B<;c, lifespan ay ~ red d e
ANTIGEN (PSA) will b ~~I.SQ · I~ a~1AJ1-1e.,lil certain
drugs aspirin, mepacrine, primaquine, su a) re taken
It is produced from the secretory epithelium of prostate
by sueh · iduals, there will be sudden damage to
gland. It is normally secreted into seminal fluid, where it
RBCs. Primaquit ~t\~ 1 •roxide formation.
Chapter 6: Clinical Enzymology 83

In GPO deficient cells, the level of NAOPH is low,


leading to unchecked build up of peroxides resulting
in premature cell lysis. This drug-induced hemolytic -- -- -- . ; t ,-:

anemia is characteristic of GPO deficiency.


' - - · . . •. ' - . - I
(
Increase in muscle d isease; not spec

Methemoglobinemia: NAOPH is also necessary for Marked increase in obstructive liver disease
- ~-;--
~-------·~~ --
- - - -
-

• •
- •

;-#
• ,;,~ - • A O 0
. -
Increased in obstructive and alcoholic liver
see Chapter 23). Hence in GPO deficient indi-
II. Myocardial Infarction
l als, methemoglobinemia may also be manifested.

AMYLASE i£:rn -®.. ~°'S"~~- Starts to rise within 3- 6 hours

Starts to rise within 4- 6 hour


This enzyme splits starch to maltose. It is activated by
calcium and chloride .ions. It is produced by pancreas Starts to rise within 3 hours

and salivary glands. Normal serum value is 50- 120 IU/L.


Indicate heart failure
The value is i•llliirilfia!J,f 000 time§ in acute pan-
creatitis which is a life-threatening condition. The peak Rarely used nowadays
values are seen between )jo rs after the onset of Ill. Muscle diseases
disease and returns to normal levels within 2-4 days
Marked increase in muscle d iseases
after the acute phase has subsided. Moderate increase
:::n
in serum levels are seen in Earliest enzyme to rise, but not specific
. ) and _.,.__. • .... Iml. IV. Bone diseases

Marked elevation in rickets and Paget's disease

-
Hydrolyzes triglyceride to beta-monoglyceride and fatty
Heat labile bone_isoenzyme (BAP) is elevated
V. Prostate cancer
acids. The enzyme is present in pancreatic secretion. The
level in blood is highly elevated in acute pancreatitis and Marker for prostate cancer.
Mild increase in benign prostate enlargeme
this •[lj§]fQt:I,- 14 days. Thus lipase remains elevated

..
M• • •• I f ,//1.. f !~•

longer than amylase. Moreover, lipase is not increased Marker for prostate cancer. Metastatic bone di se espe-
in mumps. Therefore, lipase estimation has advantage ciall from a rimar from rostate. Inhibited tartrate.
over amylase. It is moderately increased in carcinoma of
pa reas, biliary diseases and perforating peptic ulcers.
.
tand A value less than 20 mg/di is
pathognomonic of Wilson's hepatolenticular degenera-

clJer,
It is a glycolytic enzyme. Neuron-specific enolase (NSE) tion, in which per toxicity is manifested (see under
is an isoenzyme seen in neural tissues and Apudomas. Chapter 34 ).
NSE is a tumor marker (see Table 48.5) for cancers
associated with Adenosine Deaminase
Estimation of adenosine deaminase in serum and pleural
, etc. fluid is used to differentiate . . illlll;uu;:HJS pleural effusior\
Enzyme patterns in some important diseases are from other causes.
given in Box 6.4.
zymes irrother Body Fluids and Cells
1 eruloplasmin Enzyme levels in other body fluids are measured at time
It is otherwise called Ferroxidase and is described in for specific diagnostic purposes like amylase in urine
detail in Chapter 26. Normal serum level is - ~ u . utfl....l. LOH in CSF, pleural and ascitic fluids and leukocyte
Since it is an acute phase protein, it is increased in all alkt inL A os hatase in neutrophils.
84 Section A: Chemical Basis of Life

TABLE 6 .4 : Therapeutic use of enzymes


Enzyme Therapeutic application Enzyme Used for testing
l. Asparagi nase Acute lymphoblastic leukemia Urease Urea
2. Streptokinase To lyse intravascular clot Uricase Uric acid
3. Urokinase Do Glucose oxidase Glucose
4. Streptodornase DNAse; applied locally Peroxidase Glucose; Cholesterol
5. Pancreatin (tryp sin and lipase) Pancreatic insufficiency; Hexokinase Glucose
o ral administration Cholesterol oxidase Cholesterol
Anti-inflammatory Lipase Triglycerides
AAT deficiency; emphysema Horse radish peroxidase ELISA
Alkali ne phosphatase ELISA

@ nzymes as Ther eutic nts ..t- Restriction endon uclease


Reverse transcriptase
Southern blot; RFLP
Po lymerase chain reaction (RT PCR)
u;treptokinase (from Streptocof . us) or Urokinase (from
urin c s · I r clot nd are therefore used
i myocardial infarcti Pepsi and trypsin are given
to p · · e ective di~estiorlf Asparaginase ¥ Clinical Case Study 6.2
is used as an anticancer drugJA lisr'oh herapeutically A 40-year-old obese female presents to the emergency
useful enzymes is given in Table 6.4 . center with complaints of worsening nausea, vomiting
~ ecombinant human tissue plasminogen activator and abdominal pain. Her pain is located in the midepi-
is used in the ~eatment of cerebral thrombosis, it gastric area and right upper quadrant. Her pain is pres-
produces clot lysi~ A variety of other enzymes are used ently constant and sharp in nature but previously was
in treatment of genetic dis t-APP\~ intermittent and cramping only after eating "greasy"
rn .,. wl.il' foods. On examination, she has a temperature of 37.8°C
nzymes Used in ----·_....ostic Techniques with otherwise normal vital signs. She has significant
A list of enzymes used in clinical laboratory is given in mid-epigastric and right upper quadrant tenderness.
Table 6.5. Methods for blood analyte estimations are Some guarding is present. Her abdomen is otherwise
made specific by using enzymes. For example{_glucose soft with no distention and active bowel sounds. Liver
oxidase is used to estimate gluco'3{ Urease will act only function tests were abnormal , total leukocyte count and
on urea, and therefore, it is conveniently used to quantitate serum amylase level were raised. Ultrasound of the gall-
urea in biological fluid~ f he presence of antibody in bladder revealed numerous gallstones and a thickening
circulation is identified by '°fixing them on antigen and of the gallbladder wall. What is the most likely diagno-
identified by a second antibody tagged with peroxidase. sis? What is the role of amylase in digestion?
Jhese enzymes are effective to produce a color reactiorO
L ELISA test is described in detail in Chapter 31. Restriction .' 9
Clini.eal Case Study 6.1 Answer
endonucleases are used to cut DNA at specific sites; and • n1.
applied in recombinant DNA technology, Southern blotting, [ ~yocardial in~arctio'.J CK isoenzymes, Cardiac tropo-
and other advanced techniques (see Chapter 44 0
nins, myoglobin.

Related Topics
•• Clinical Case Study 6.2 Answer
Tumor markers are described in Chapter 48. Diagnosis: Gallstone pancreatitis.
Role of amylase: Enzyme for carbohydrate metabolism,
·• Clinical Case Study 6.1 used to digest glycogen and starch.
A patient presented with acute chest pain of half hour Acute pancreatitis is an inflammatory process
duration. The biochemical analysis reports are as follows: in which pancreatic enzymes are activated and cause
Blood Glucose - 350 mg%, Serum Cholesterol - 288 mg%, Alcohol use is the most
AST - 55 U/L, ALT - 15 U/L CPK and LOH were eleva- common cause, and episodes are often precipitated by
ted. Give your provisional diagnosis. What are the other C 4l . The next most common cause is ...,,_,
markers which can be estimated in this case? , usually passage of a gallstone into the
Chapter 6: Clinical Enzymology 85

common bile duct. is also a com- 2. fil, f..SJ, fil:.P,_QQT and


til.e.are clinically useful

-
mon cause, and that occurs when serum triglyceride lev- markers in hepatic diseases.
els are greater than 1000 mg/dl, as is seen in patients 3. &..e is also a useful marker in bone diseases.
with Abdominal pain 4. ~ r - . , t alcohol abuse and used as a
is the cardinal symptom of pancreatitis, and it is often marker o~lcoholic liver disease (ALD).
t 5. ACP and PSA,are used in t~ iagnosis of prostate
severe, typically in the upper abdomen with radiation
cancer.
to the back. The pain is often relieved by sitting up and
bending forward and is exacerbated by food. Pat:~:-:!~ 6. p~==.l. !.U:i!.!.sa..~ ~~~=:::
tate cancer.
also commonly have nausea and vomiting, also pre- 7. C olinesterase lChE)._is used as a marker in organo-
cipitated by oral intake. The treatment includes nothing phosphorus insecticide poisoning.
by mouth, intravenous hydration, pain control, and moni- 8. Pancreatic am lasels a marker of acute P,¥freatitis . . 4. ,.nu&
toring for complications. 9. Therap~,utically u~ fyl enzymes str~ioki~e . r<qro_J
or urdiarlase,~p~inaie1 hyall.lMnidase, all5fm1 - 1,: .. ,.~hc.
LEARNING POINTS, CHAPTER 6 antitrypsin, plf'l')Sin and t~in. e'j5
10. Some examples of enzymes used in diagnostic

-
1. Clinically useful biomarkers in diagnosis and pro-
gnosis of myocardial infarction are g<, CTI/CTT methods are peroxidase, glucose oxidase and alka-
and~T. line phosphatase.

PART-1 : ESSAY AND SHORT NOTE QUESTIONS

6-1 . Write briefly about the enzymes that show variations in serum level in Myocardial infarction, and their clinical
i significance.
6-2. Name the enzymes that are used in the diagnosis of myocardial infarction. Explain the pattern of rise following
! an acute infarction.
6-3. Indicate the importance of enzyme studies in (a) Myocardial infarction; (b) Liver diseases; (c) Bone diseases;
(d) Malignancy.
6-4. Describe the importance of serum enzyme estimations in clinical diagnosis.
6-5. Indicate the use of enzyme studies in (a) Hepatobiliary diseases; (b) Bone diseases; (c) Muscle diseases; and
(d) Pancreatic diseases

SHORT NOTE QUESTIONS

6-6. Clinical s ignificance of ALP. 6-11 . Clinical significance of Troponins.


Clinical significance of ACP. 6-12. Clinical significance of CK.
6-7.
Glucose-6-phosphate dehydrogenase deficiency. 6-13. Enzyme profile in myocardial infarction.
6-8.
Clinical significance of serum amylase. 6-14. Enzyme profile in liver diseases.
6-9.
6-15. Clinical significance of transaminases.
6-10. Therapeutic uses of enzymes.

• PART 2: MULTIPLE CHOICE QUESTIONS


6-1 . In myocardial infarction: 6-3. Serum alkaline phosphatase is increased in:
A. Serum LDH-1 level is lower than LDH-2 A. Hypothyroidism
B. Increase in serum ALT level is diagnostic B. Carcinoma of prostate
C. Serum CK-MB levels show a marked increase C. Bone diseases
D. Serum alkaline phosphatase (ALP) is increased D. Myocardial infarction
6-2. How many isoenzyme forms are there for lactate 6-4. ALP (alkaline phosphatase) level in serum is
dehydrogenase (LOH)? elevated in all the following conditions, except:
A. Two B. Three A. Obstructive jaundice
C. Four D. Five 8 . Rickets
86 Section A: Chemical Basis of Life

C. Infective hepatitis C. Alkaline phosphatase


D. Myocardial infarction D. Plasminogen
6-5. Estimation of gamma-glutamyltransferase is useful 6-12. lsoenzymes may be identified or separated by all
to detect one of the following clinical conditions: the following characteristics, except
A. Alcohol abuse A. Electrophoretic mobility
B. Infective hepatitis B. Heat stability
C. Kidney failure C. Difference in Km value
D. Myocardial infarction D. Chemical reaction catalyzed
6-6. Serum acid phosphatase (ACP) level is increased in: 6-13. Enzymes may be used for following, except
A. Rickets A. As therapeutic agents
B. Hepatoma B. For diagnosis
C. Prostatic carcinoma C. As nutrients
D. Excess phosphate intake D. As tumor markers
6-7. Acute pancreatitis may be diagnosed by estimating 6-14. Serum enzymes helpful in the diagnosis of myo-
serum level of the enzyme: cardial infarction are the following, except:
A. Alkaline phosphatase A. CK-MB
B. Acid phosphatase B. Troponin
C. Alanine transaminase C. Glucose-6-phosphate dehydrogenase
D. Amylase D. LDH
6-8. lsoenzymes are: 6-15. All the following statements are true with regard to
A. Examples of polymorphism exhibited by the enzymes liver diseases, except:
B. Having the same amino acid sequence A. In obstructive jaundice, alkaline phosphatase level
C. Physically different but have identical catalytic is markedly increased
properties B. In infective hepatitis, alanine aminotransferase
D. Polypeptide chains that differ by a single amino
acid replacement
level is increased
C. In alcohol abuse, gamma-glutamyltranspeptidase •
6-9. Creatine kinase level in serum is increased in: level is markedly increased
A. Myocardial infarction D. In cirrhosis, creatine phospho kinase level is inc-
B. Infective hepatitis reased
C. Prostate cancer 6-16. Pseudocholinesterase deficiency is manifested:
D. lntravascular hemolysis A. Prolonged apnea after succinylcholine adminis-
6-10. All are true with regard to Regan isoenzyme of tration
alkaline phosphatase, except B. RBC life span is reduced
A. It moves along with alpha-2 band in electrophoresis C. Myasthenia gravis
B. Is heat stable (not destroyed at 65 C for 30 min) D. Peripheral neuropathy
C. Not inhibited by fluoride 6-17. Serum acid phosphatase (ACP) level is increased
D. Seen in carcinoma of lung in all the following conditions, except
6-11 . Which one of the following is functional enzyme in A. Metastasis in bones from carcinoma of prostate
plasma? B. Bone metastasis from carcinoma of breast
A. Amylase
B. Lipase
C. Per rectal examination of patient
D. Hemolyzed blood samples •
ANSWERS OF MULTIPLE CHOICE QUESTIONS
6-1. C 6-2. D 6-3. C 6-4. D 6-5. A 6-6. C 6-7. D
6-8. C 6-9. A 6-10. C 6-11 . D 6-12. D 6-13. C 6-14. C
6-15. D 6-16. A 6-17. B
Chapter 6: Clinical Enzymology 87

PART-3: VIVA VOCE QUESTIONS AND ANSWERS

• 6-1. When does creatine kinase (CK) level increased? carcinoma of lung, liver and gut. Regan isoenzyme is
Myocardial infarction; Muscular dystrophies. otherwise known as carcinoplacental isoenzyme.
6-2. What is the advantage of CK estimation? 6-8. Estimation of gamma-glutamyltransferase is use-
• CK level starts to rise within 3 hours of infarction. ful to detect which condition?
Therefore CK estimation is very useful to detect early Alcohol abuse.
cases. 6-9. Serum acid phosphatase level is increased in
6-3. What serum markers are helpful in the diagnosis which condition?
of myocardial infarction? Prostate carcinoma.
Creatine kinase (CK) CK-MB isoenzyme and Troponins. 6-10. What is the use of isoenzyme study of acid phos-
6-4. What are the isoenzymes of CK? phatase?
CK is a dimer; the subunits are called B for brain and M Tartrate labile isoenzyme is specific for prostate carci-
for muscle. Therefore three isoenzymes are possible. noma.
6-5. What is the advantage of cardiac troponin I? 6-11. What is advantage of prostate specific antigen?
Cardiac Troponin I is released into the blood within 4 PSA is very specific for prostate tissue.
hours after the onset of cardiac symptoms, peaks at 6-12. What are enzymes useful as tumor markers?
12-16 hours. Therefore, CTI is very useful as a marker (a) Regan isoenzyme of ALP for lung tumor; (b) Tartrate
at any time interval after the heart attack. labile isoenzyme of ACP; (c) Prostate specific antigen
6-6. Alkaline phosphatase level in serum is elevated in (PSA) for prostate carcinoma; and (d) Neuron specific
which conditions? enolase (NSE) for cancers of neuroendocrine origin.
Moderate increase is seen in hepatic diseases (infec- 6-13. Which enzyme deficiency is X-linked?
tive hepatitis); high levels in obstructive jaundice; and Glucose-6-phosphate-dehydrogenase.
very high levels in bone diseases such as Paget's 6-14. How the deficiency of GPO is manifested?
disease, rickets. metastatic carcinoma of bone. Drug induced hemolytic anemia.
6-7. What is Regan isoenzyme? 6-15. Acute pancreatitls can be diagnosed by estimating
It is the iso-enzyme of alkaline phosphatase. It is of which enzymes?
placental origin. It is elevated in about 15% cases of Amylase and lipase.

t
_ __ _ _Chapter 7
Chemistry of
Carbohydrates

Chapter at a Glance
The learner will be able to answer questions on the following topics:
Nomenclature and classification of sugars Amino sugars and deoxy sugars
Stereoisomers Pentoses
Glucose, mannose and galactose Sucrose, lactose and maltose
Fructose Starch, glycogen and cellu lose
Reactions of monosaccharides Heteroglycans, mucopolysaccharides
Glycosides

Gtf'h ~)_.I.,
The general molecular formu la ' of carbohydrate is 4. Glycoproteins and glycolipids are components of
C"(HPt For example, glucose has the molecular for- cell membranes and receptors.
p
mula C6 H 6 . Carbohydrates are polyhydroxy aide- · 5. Structural basis of many organisms: Celluloseaof
hydes or ketones or compounds which yield these on ~ s; exoskeleton of insects, c911 wall of micm-

®O
hydrolysis (Fig . 7.1 ). organisms, mucopolysaccharides as gro1,1nd sub-
-.~,..~
..--- 11-" .
stance in higher orga~isms. ; . · •
~·. :!'~=---·~..
-

. , - . - -

{ _1,___ .C.,_
ar~bohydrates are the main sources of energy in
Molecutes raving only, one actual or potential sugar
~' the body. Brain cells and RBCs are almost wholly
group are called monosaccharides (Greek, mono= one;
dependent on carbohydrates as the energy source.
l--~ production from carbohydrates will be
saccharide = sugar). They cannot be .further hydrolyzed
into smaller units. When two monosaccharides are com-
bined together with elimination of a water molecule, it is
2. Storage form of energy (starch and ~ gen). called a disaccharide (e.g. C12H2p 11 ). Trisaccharides
3. Excess carbohydrate is conv d t~ contain three sugar groups. Further addition of sugar
groups will correspondingly produce tetrasaccharides,
pentasaccharides and so on , commonly known as oligo-
saccharldes (Greek, oligo = a few). When more than
10 sugar units are combined, they are generally named
• as polysaccharides (Greek, poly= many). Polysaccha-
R R1 R H
rides having only one type of monosaccharide units are
Ketone Aldehyde
called homopolysaccharides and those having differ-
Fig. 7.1: Keto group and aldehyde group ent monosaccharide units are heteropolysaccharides.
Chapter 7: Chemistry of Carbohydrates 89

TABLE 7.1: Common monosaccho -,..,,; H- C= O H- C= O


I ,
No. ofcarbon Generic A/doses Ketoses H- C- OH
I ~
2
3
bI -H
atoms name (with aldehyde group) (with keto group) CH2- 0H
CH2- 0 H
3 Triose Glyceraldehyde Di hydroxy-acetone
4 Tetrose Erythrose Erythrulose
f
5 Pentose Arabino se, Xylose, Xylulose, Ribulose Fig. 7.2: Stereoisomers
Ribose
6 Hexose Glucose, Galactose, Fructose All monosaccharides can be considered as molecules
Mannose
derived from glyceraldehyde by successive addition of
7 Heptose -~ lose
carbon atoms. Therefore, penultimate carbon atom
is the reference carbon atom for naming the mirror
H =O 'cnages (Fig. 7.3).
2
D- and L-lsomerism of Glucose
3
With reference to the p~ultimate carbon atom (i.e. C5
4
in the case of glucose), the configuration of H and OH1,A,p:~
5 groups is changed and two mirror images are produ
6 (Fig. 7.3). It may be noted that
D-glucose L-glucose

Fig. 7 .3: Penultimate (refe re nce) carbon a to m

Sugars having aldehyde group are called aldoses


and sugars with keto group are ketoses. Depending on
the number of carbon atoms, the · es art!
nose {C3)., tetrose {C4), pentose (C5), hexose
~--~
The presence of ~ atom causes
beptose (Ci') and so on. Commonly occurring
optical activity. When a beam of plane-polarized light is
monosaccharides are given in Table 7. 1.
passed through a solution of carbohydrates, it will rotate
the light either to right or to left. Please note that the
0- and L-notation has no bearing with the optical activi~
Depending on the rotation, molecules are called dextr0;-
are known as stereoisomers. rotatory(+) (d) or levorotatory (- ) (I). Thus D-glucose is
While writing the molecular formula of monosaccharides, dextrorotatory but D-fructose is levorotatory. Equi-
the spatial arrangements of H and OH groups are molecular mixture of optical isomers has no net rota-
important, since they contain asymmetric carbon atoms. tion (racemic mixture). Heyrovsky was awarded Nobel
Asymmetric carbon means that four different groups are rize in 1959 for polarographic analysis of sugars.
attached to the same carbon. The reference molecule \.!)~ ~-=-- - ;
• is @yceraldehyde (glycerose~ hich has a single astereoisomers of G ucose
t asymmetric carbon atom (Fig. 7.2). Configurational changes with regard to C2, C3 and C4
The number of possible stereoisomers depends on will produce eight different monosaccharides. Out of
the number of asymmetric carbon atoms by the formula these, only 3 are seen in human body. They are Glucose,
here n is the number of asymmetric carbon atoms. Galactose and Mannose (Fig. 7.4 and Table 7.2).
There are 8 diaster,eoisomers for aldohexoses. With
rence Carbon Atom of Sugars reference to C5, all of them will have D and L forms.
The configuration of H and OH groups at the second Hence the molecular formula of hexose (C6 H1p 6 )
carbon atom of glyceraldehyde (Fig. 7 .2) may be noticed. represents 16 different monosaccharides, due to spatial
The two mirror forms are denoted as 0- and L-varieties. arrangement of constituent groups.
90 Section A: Chemical Basis of Life

TABLE 7.2: Hexoses of phys1olog1cal importance


Sugar Importance
0 -Glucose Blood sugar, main source of energy in body •
0-Fructose Constituent of sucrose, the common sugar

D-Galactose Constituent of lactose, glycolipids and

D-Mannose
gtw 12coteins - --
Constituent of ~ - rryucoproteins and
'
eins
CH2 0H
'
0-glucose 0-mannose D-galactose

Fig. 7.4 : Epimiu,s of D-gl~se l~ HV-2:; 0 H


H-C= O
V
BOX 7.1: Practical importance of mutarotat1on
I
In a clinial laboratory, once a technician freshly prepared a I I
I.
: H-~-OH H-C-OH H- ~-OH
standard solution (100 mg/ lqG ml) of glucose. She compareq I
with another standard solution similarly prepared one week ag'o. HO-C-H Ho- 9-H 0
The comparison was done b: Q - .. J her 3 Ho -y~~H I
H-C-OH
surprise, the freshly prepared solution showed only one-tenth 4 H-y~ H- 9 ~
strength of the old solution. Testing a second time, she got the I
5 H-C H-C -OH H- C
same results. So she asked for the help of her boss, who suggested I I I
to keep t he new solution, overnight in fridge. The next day, both 6 CH2- 0H CH2 0H CH2 0 H
one-week-old solution and the new (kept overnight) solution
a-0-glucose Traces of intermediary 13-D-glucose
gave identical results. The technician was not able to explain the
reason for this change. Can you explain? ig. 7.5: Anomers of D-glucose

e glucos - · me preferentially acts


F _
nomerlsm of Sugars (~>cLl)r
on the J}--form. Hence, freshly prepared solutions will give lower
When D glucose is crystallized at room temperature,
values. and a fresh solution is prepared, its specific rotation of
polarized light is + 112°; but after 12- 18 hours it changes
Glucose is the in human to +52.5°. If initial crystallization is taking place at 98°C
body. It is the major source of energy. It is present in and then solubilized, the specific rotation is found to be
+19°, which also changes to +52.5° within a iQ rs.
blood (Table 7.2). D-glucose is dextrorotatory. In clinical
This change in rotation with time is called utarota~ nj
practice, it is often called as dextrose (Box 7.1).
This is explained by the fact that D-glucose has two
Galactose is a constituent of lactose (milk sugar)
anomers, alpha and beta varieties. These anomers
and glycoproteins. Galactose is epimerized to glucose
are produced by the spatial configuration w ith reference
in liver and then utilized as a fuel. The term galactose is
to the first carbon atom in aldoses and second carbon
derived from Greek word "gala", meaning milk.
atom in ketoses (Fig. 7.5). Hence, these carbon atoms
Mannose is a constituent of many glycoproteins. Man-
are known as anomerlc carbon atoms. Thus alpha-
nose was isolated from plant mannans; hence the name.
D-glucose has specific rotation of +1 12° and beta-D-
WE -.-- t Sfl'J C"")~,\f-t glucose has +19°. Both undergo mutarotation and at
equilibrium o ~ l ~ u les..are aJ.eha type and ~
,
When sugars are different from one another, ~tiy in a~ e~ to get the specific rotation of +52.5°. The
configuration with regard to a slngle carbon atom, hemiacetal structure of anomeric forms of glucose are
other than the reference carbon atom, they are called shown in Figure 7.5.
epimers. For example, glucose and mannose are an The • • • • • • ·c
epimeric pair which differ only with respect to C2. Simi- forms are dependent on the In
larly, galactose is the 4th epimer of glucose (Fig. 7.4). the previous section 16 stereoisomers of glucose are
Galactose and mannose are not epimers but diastereo- described. Each of them will have 2 anomers; and hence
isomers. there are total 32 isomers for glucose.
(Wu; Chapter 7: Chemistry of Carbohydrates 91

uctose is a Ketohexose
In fructose, the keto group is on the 2nd carbon atom.

..
Thus secon~ om is the anomeric carbon atom.
Fructose has~~ - Each of them has D and L forms
with regard to 5th carbon atom. Fructose has the same
f Emil Fischer Walter Haworth Heyrovsky
NP 1902 NP 1937 NP 1959 molecular formula as glucose, but differs in structural
52-1919 1883-1950 1890-1967 formula. So glucose and fructose are function
(aldose-ketose) isomers.
three Representations of
o niy D variety is seen in biological systems. Fructose
ucose Structure remains predominantly as furanose ring structure
The 1st carbon, aldehyde group is condensed with the · . 7.8). Fructose is a major constituent of honey.
hydroxyl group of the 5th carbon to form a ring. Ring
structure represents• • • • •ill. which is the
an.aldehyde (or keto with a hydroxyl group.
, .
The • ~.::..._ ...... · ; - : - _ .~ _ ; • ' - ·., l ; I

c~oMl
ring structure of glucose were proposed by r erties will be seen together:
in 1883, and hence called Fischer's formula. Fischer • ...111111 I t: I • t ' ;'\_

was awarded Nobel prize in 1902.


Later it was shown that glucose exists in biological
systems not as a rectangle, but as a (Fig .
7.6). This was established by Sir \ll•• •liiiin 1925
who got Nobel prize in 1937. Therefore, the structure
1 carbohydrates containing a
of glucose may be given as the following 3 forms, each
successive form adding more details (Fig. 7.7). free sugar group (aldehyde or keto) will tautomerize to
In solution, beta-D-glucopyranose is the predominant form enediols, where two hydroxyl groups are attached ~ ~
form (63% ), alpha-D-glucopyranose 36% while 1% to the double-bonded carbon. In mild alkaline conditions,
m_?lecules are in glucofuranose ...!2,!:!!)s. The ~ tfea'I glucose is converted into fructose and mannose.
importance is hown in Box 7. 1. The interconversic;m of sugars through a common C,...
enediol form is called (je_B.ruyn-Van Ekenstein'
atiorf (Fig. 7.10). Since enediols are highly
reactive, sugars are powerful reducing agents in alkaline

H
.. H- C= O
6 CH2 0H 6CH 20H

t' 2
I
H- C-OH
I
H-t-OH I 5
0

HO - C-H
t, 3 HO-C-H 0

4
I
H-C-OH
I
I
H- i -OH
4

5 H-C-OH H- C
I I 3 2
6 CH2 -0H CH2 0 H
H
D-glucose. open chain a-D-glucose, closed a-D-glucopyranose, P-D-glucopyranose,
projection formula ·ng structure, Fischer formul Haworth formula Haworth formula

Fig. 7.7: Comparison of different representations of 0-glucose


92 Section A: Chemical Basis of Life

2
1
C=O
H20H 6
OH-H2C
1
CH2-0H
Sugar --+Enediol ~ u • •+--- CuS04

Oxidized Reduced
3 H0-6-H •
Sugar acid cu• - - 2Y
Cu(OH) - -cu20
ellow Red

ft,
4 H-6- oH '
5
I
H-C-OH
I
OH Fig. 7.9: Benedict's test, principle
'
6 CH,- OH I
D-fructose, open chair-it H
Needle-shaped crystals arranged
projection formula o.•D.fructo furanose, Haworth formula like a broom
Glucososazone
Fig. 7.8: Different representations of 0-fructose

H f y=o I HfITTOH
II
H-C-OH ,_----- C-OH ,_-----
I '-t Hedgehog or ' pincushion with
HO-C-H HO-C-H HO-C-H
,. pins" or flower of "touch-me-not" plant
I La<;tososazone
H-C-OH
I I
H-C-OH H-C-OH
I I I
H-C-OH H-C-OH H-C-OH
I I I
CH2-0H CH2-0H CH2-0H
Glucose 1,2-enediol form (intermediary) Fructose Sunflower-shaped or petal-shaped
-
,.
crystals of
Fig. 7.10: . - - - .-
_..,. - , ,
. .
~~-•Ill . _.
. - Maltosazone

l 100~ -C = O
I r
[ COOH Fig. 7 .11 : Shape of osazones under microscope
H-C-OH H-C - OH H-C-OH C
I I I
HO-C-H HO-C - H HO- C-H
I I I
H- C-OH H-C-OH H-C-OH
I I I
H-C- OH H-C- OH H- C-OH
I --i--.
CH2 -0H COOH 1 ! 600H
Gluconic acid Glucuronic acid lucosaccharic acid'

@ Be
Fig. 7 .12: Oxidation products of glucose
c'lCtn- ~~,ot. .
e let's Reaction t2,(1"
o/<'~ ~f">- ~e

sugars will produce the sam

Osazones may be used to differentiate sugars in


logical fluids like urine.

diabetes mellitus. Benedict's reagent contains sodium ldatlon of Sugars B


fl'Oa,CD.3carbonate, copper sulfate and sodium citrate. In a~ a. Under mild oxidation conditions (hyptbro~ o~ s~acid),
UAOO medium, sugars form enediol, cupric ions are reduced, the
W. correspondingly sugar is oxidized (Fig. 7.9). Glucose to produce aldonic acid (Fig. 7. 12). Thus, glucose
e,tw,.Ac, is a reducing sugar (Fig. 7.19B). is oxidized to gluconic acid, mannose to mannonic
,oop acid and galactose to galactonic acid.
Therefore, this is not specific for glucose. Reducing b. When aldehyde group is protected, and the molecule
substances in urine are described in Chapter 11 . is oxidised, the last carbon becomes COOH group
Chapter 7: Chemistry of Carbohydrates 93

H-C=O CH2- 0H CH2 0H CH2-0H


I I I I
2 H-C-OH H- C-OH C=O HO - C- H
' 3 HO- C-H
I
HO - C-H
I
HO- C-H
I
HO-C- H
I
I I I I
4 H- C-OH H- C- OH H-C- OH H-C- OH
f I I I I
5 H- C- OH H- C- OH H- C- OH H- C- OH
I I I I
6 CH2- 0 H CH2-0H CH2- 0H CH2- 0 H
D-glucose 0 -fructose 0 -mannitol

Fig. 7 .13: Reductio n of sugar to a lcohol 1"~\ol?'l·


6 CH2 OH
corresponding alcohol. But
because of appearance of a new asymmetric carbon
atom in this process (Fig. 7.13). Glucose is reduced to
4 4
sorbitol; mannose to mannitol; while fructose becomes
sorbitol and mannitol (Fig. 7.13). Galactose is reduced
3 3
H H to dulcitol and ribose to ribitol.
- - -.- ,~- .. :: ~- -.....__.~
Methyl-a.-D-glucopyranoside Methyl-Jl-0-glucopyranoside l - _ ., . _____._ ....:_,- .:. • I. • - I I .. • - t..., I - ' - I •• '. I ....

Fig . 7.14: Glycosides

to produce uronic acid. Thus glucose is oxidized to p[Oduces changes in tissues when they accumulate In
glucuronic acid, mannose to mannuronic acid and rma l amounts, e .g. cataract of lens.
galactose to galacturonic acid. The glucuronic 1-'.sV'~
D-,.."\l-
acid (Fig . 7.12) · ~''.Y~:tmpo~r-l.-
-.I iaL.u::_; •_• - • - -;i-- ;. - II :. When the hemiacetal group (hydroxyl group of the
(see Chapter 36) anomeric carbon) of a monosaccharide is condensed
and also for with an alcohol or phenol group, it is called a glycoside
c. Under strong oxidation conditions (nitric acid + heat), (Fig. 7 .14). The non-carbohydrate group is called agly-
the first and last carbon atoms are simultaneously cone. Glycosides do not reduce Benedict's reagent,
oxidized to form dicarboxylic acids, known as sac- because the sugar group is masked . They may be
charic acids (Fig. 7 .12). Glucose is thus oxidized to hydrolyzed by boiling with dilute acid, so that sugar is
lucosaccharic acid, mannose to mannaric acid and free and can then reduce copper,
The I . I f tif f 7
, while
Enzyme
hydro ysis thus affords a distinguishing
between the two forms. Some glycosides of medical
importance are given in Table 7.3. 1a~
r Monosaccharides when treated with concentrated sul- ant. forhtztrr ts usectto produce renal damage ip
furic acid undergo dehydration with the removal of 3
• molecules of water. Therefore hexoses give Jiydroxy-
ta nimals .

methyl furfural and pentoses give furfural. The furfural


derivative can condense with phenolic compounds to Hydroxyl groups of sugars can be esterified to form
give colored products. This forms the basis of acetates, propionates, benzoates, phosphates, etc.
Sugar phosphates are of great biological impor-
tance. Metabolism of sugars inside the body starts with
phosphorylation. lucos~
, such as t are irnpoJlaDt lucos&
, hydrogen can reduce sugars. Aldose yields lism (iig. 7.15).
-0~6 0
94 Section A: Chemical Basis of Life

0
II
CH,OH

O
H CH, 0 H J
Glucose H H

Galact ose, digitogenln OH o!-~_-o-l OH OH H OH

xylose
H OH H 0~
Glucose indoxyl

Fig. 7.15: Phosphorylated sugars - )

CHO
I
HO- C- H
I
4 H- C- OH
I
H- C- OH
3 2 I
H NH-CO-CH3 HO - C- H

2-amino-D-glucose N-acetyl-glucosamine
or glucosamine or GluNac

Fig. 7.16: Amino sugars

5 t
OH-H2 CQ OH

H
4
H H OH
1 •
3 2
H0 H

-.,t.J.lJ~,-.•
~deoxy su!Jars with Schjffs reagent (Dye Fuch sine is
(%) Amjno Sugar!g decolorized by sulfurous acid).

Amino groups may be substituted for hydroxyl groups o


sugars to give rise to amino sugars. Generally, the am· . -'"fll'!ll... are sugars containing 5 carbon atoms. Ribose
group is added to the second carbon atom of hexos 18) is a constituent of RNA. N MLUll 1
(Fig. 7 .16). mo sugars will not show reducing prope . Deoxyribose is
een in DNA (Fig. 7.18). Ribulose is an intermediate of
Tfiey will not proouce osazones. Glucosamine is seen in
W4R Sb! '01 ga th>M.y. Arabinose is present in olllilllllllll's
.QLS:S&E&JIIJ : ; Mt and blood group substances.
I 111!1 · • sh optWWW"-5 of the body. The name
Galactosamine is present in 1 I T f rrtil?'.ii' arabinose is derived as it was originally isolated from
)I: !!'!!~~~~~ ! • -@ annosamjne js a constituent of gum arabic. Xylose is seen in.l!lllll!ll!!!!!!lf Xylulose
_-:sn rstsi~ The amino group in the sugar may be
Turther acetylated to produce N-aceiylated sugars sue
as N-acetylglucosamine (GluNac) (Fig. 7.16), N-acetyl- -.____
galactosamine (GalNac), etc. which are important cons- When two monosaccharides are combined together
tituents of 11111:oP«lle.ins, mucopolysaccharides and cell by glycosidic linkage, a disaccharide is formed. The
important disaccharides are:
Chapter 7: Chemistry of Carbohydrates 95

O
H
-H,dG"
6 CH2OH

0
l
.I, 4 o/ H HO /H 20 H

f 3 4
HO H
2
OH Fructose component
Glucose component
Alpha-D-glucosyl-beta-0 -fructoside

Fig. 7.19A: Structure "of suc rose (1- 2 linkage)


Reagent alone With glucose With sucrose Sucrose after
acid hydrolysis

Fig . 7 .198: Benedict's test is positive for glucose. Test is


negative for suc rose; but when sucrose is hydrolyzed, the test
becomes positive (specific sucrose test)
b
0
6CH2 OH 6 CH 2OH
H
H OH H OH
Galactose component Glucose component
4 1a
f3alacto,ff!; ~1eta-1,4-glucose
a.
Fig. 7.20: Lactose '---- 0
H OH H OH
BOX 7.2: Lactose and lactate are different Glucose component Glucose component

LKtose is the milk sugar; a disaccharide made of galactose and Glucosyl-alpha-1,4-glucose


glucose.
Fig. 7.21 : Maltose
Laci.ta or Lactic acid is a product of anaerobic metabolism of
glucose.
sucrase or invertase. Honey contains invert sugar.
Invert sugar is sweeter than.sucrose.
1. Sucrose
2. Maltose and isomaltose
@ ose (_9m)
3. Lactose.
It is the sugar present in milk. It is a reducing disaccha-
crose ('5rt)) (:?,m) ride. On hydrolysis lactose yields glucose and galactose.
It is the sweetening agent known as cane sugar. It is Beta glycosidic linkage is present in lactose. The
structure is given in Figure 7.20. The anomeric carbon
present in sugarcane and various fruits. Sucrose contains
atom of beta-galactose is attached to the 4th hydroxyl
glucose and fructose. Sucrose is not a reducing sugar;
group of glucose through beta-1,4 glycosidic linkage.
and it will not form osazone. This is because the linkage
The lactose may be alpha or beta variety, depending
involves first carbon of glucose and second carbon of
on the configuration of 1st carbon of glucose moiety.
fructose, and free reducing groups are not available (Fig.
Lactose forms osazone which resembles "pincushion
7.19A). When sucrose is hydrolyzed, the products have
with pins" or "hedgehog" or flower of "touch-me-not"
reducing action. A sugar solution which is originally non-
plant (see Fig. 7.11 ). Lactose and lactate should not be
reducing, but becomes reducing after hydrolysis, is inferred
as sucrose (specific sucrose test) (Fig. 7 .19B). confused (Box 7.2).
Hydrolysis of sucrose (optical rotation +66.5°) will
produce one molecule of glucose (+52.5°) and one
molecule of fructose (-92°). Therefore. the prod~ s Maltose contains two glucose residues. There is alpha-1,4
will change the dextrorotation to levorotation, or the linkage, i.e. the anomeric 1st carbon atom of one glucose
plane of rotation is inverted. Equimolecular mixture of is combined with 4th hydroxyl group of another glucose
glucose and fructose thus formed is called Invert sugar. through alpha-glycosidic linkage. Structure is shown in
The enzyme producing hydrolysis of sucrose is called Figure 7.21. Maltose may be alpha or beta depending
96 Section A: Chemical Basis of Life

t
6
O - -cH 2
Fructose Ketohexose
Disaccharides
Glucose + Galactose = Lactose (reducing)
4
Glucose + Glucose = Maltose (reducing)
Glucose + Fructose= Sucrose (non-reducir1g)
3 2
Two glucose residues in@iiiia,;1..6Jiok~g_e H
OH
units, but is highly branched with molecular weight
Fig. 7.22: lsomaltose more than 1 million. The branching points are made by
alpha-1,6 linkage (similar to isomaltos;e, Fig. 7.22).
on the configuration at the free anomeric carbon atom.
f\S\ It is a reducing di ccharide. It forms tal-sha ed drolysis of Starc/-1
~ _,s,Dls of malto eo zone (see Fig. 7.11 ). Starch will form a blue colored complex with iodine;
this color disappears on heating andl reappears when
laomaltose cooled. This is a sensitive test for starch. Starch is non-
It is also a reducing sugar. It contains 2 glucose units reducing because the free sugar groups are negligible in
combined in. alpha-1 ,6 linkage. Thus first carbon of one number. , smaller
glucose residue is attached to the sixth ca rbon of another and smaller fragments are produced. Thus hydrolysis for
glucose through a glycosidic linkage (Fig. 7.22). Partial a short time produces amylodextrin which gives violet ,.
hydrolysis of glycogen and starch produces isomaltose. color with iodine and is non-reducing. Further hydrolysis
The salient features of important sugars are shown produces erythrodextrin which gives red color with •
inBox.L iodine and mildly reduce the Benedict's solution. Later
ach·rodextrins (no color with iodine, but reducing) and
IDES {. Sf1"\ furthe'. on, maltose (no color _with iodine, b~t powerfully
These are polymerized products of many monosaccha- reducmw are fmmed op 9Pntmueg Jifrol SIS.
ride units. They may be: 4
1. Homoglycans are composed of single kind of mono- Action ofAmylases on Starct,r
saccharides, e.g. starch, glycogen and cellulose. a crea c a iefe are alpha-
2. Heteroglycans are composed of two or more diffe- amylases, which act at random on alpha-1,4 glycosidic
rent monosaccharides, e.g. hyaluronic ~ d, chon- bonds to split starch into smaller units (dextrins), and
L1. droi!i.(i_sulfa!e. • ,?t ,..,_,_ ~ 1...C mally to alpha-maltose. Beta-amyla1ses are of plant
n 0 M~Cf\NS. ,~~ _.,~~.,.....
rch \...-=>';· ), "'A..,..,..i;J-W origin (almond, germinating seeds, etc.) which split
l'J;>if"J i,'ff"lv starch to form beta-maltose. They act on amylase to split
Structure of Stare \.-: '6\0 maltose units consecutively. Thus the enzyme starts
. / It is the rate Sou~- its action from one end. When beta-amylase acts on
~ I ces are potatoes, tapioca, cereals (rice, wheat) and other
; food grains. Starch is composed of amylase and amylo-
amylopectin, maltose units are liberated from the ends of
the branches of amylopectin, until the action of enzyme '
:')"'· ~ ctin. When starch is treated with boiling water, 10-20% is blocked at the 1,6-glycosidic linkafJe. The ~ f
J ·•~ is solubilized; this part is called amylose. Amylase is beta-am lase st s at branchin oi ts, leaving a large
~v., made up of glucose units with alpha-1 ,4 glycosidic lin!<- molecule, calle llmlt-dextrin or residu " dextrin.
ages (Fig. 7.21) to form an unbranched long chain wi
a molecular weight 400,000 D or more. The insolubl
Jycogen t 3m) _, fm~Me,\ .a.l-n-f~
part absorbs water and forms paste like gel; this is called It is the reserve carbohydrate in animals. It is stored
amylopectin. Amylopectin is also made up of glucose in liver and muscle. About 5% of weight of liver is made
Chapter 7: Chemistry of Carbohydrates 97

End glucose unit ····•. ·····... \ . . Branch point


BOX. 7.4: lnuhn and Insulin are different

-;~:,;tz;r
lnulin is a polysaccharide (carbohydrate) made up of fructose
units. It is used for renal function studies.
Insulin is a polypeptide (protein) hormone, with wide ranging
actions on carbohydrate and lipid metabolism.

Glucogenin :S_...-~Lg~ +Tu variety of commercial applications, as it is the starting


material to produce~ rs, c~oids, ni~ ellulose and
~ ics.

It is a long chain homoglycan composed of D-fructose


units with repeating beta-1,2 linkage..s. It is the reserve
carbohydrate present various~ in an~ rs, such
as chicory, dahlia, dandelion, onion, garlic. It is clinically
used to find renal clearance. value and glomerular
up by glycogen. Excess carbohydrates are deposited filtration rate. lnulin and lnsulin\Fr"rg'f ferent (Box 7.4).
as glycogen . Glycogen is composed of glucose units~ ...-- -
joined by alpha-1,4 links in straight chains. It also ha _axtrans
These are high! bran d homo ol mers of glucose
1
units with 1-6, 1-4 an 1-3 linka es. They are produced
by micro-organisms. ey have molecular weight
1 million to 4 millions. Since they will not easily go out
!!
of vascular compartment, they are used for intravenous
infusion as plasma volume expander for treatment of
Qlp.OVolemic shock. It may be noted that dextrans are
different from previously described dextrins (Box 7.5}:
It is the supporting tissues of plants. Cellulose constitutes
99% of cotton, 50% of wood and is the most abunda f-
tin'
organic material in nature. It is made up of glucose units
combined with beta-1 ,4 linkages. It has a straight line
structure, with no branching points. Molecular weight is in
the orderof2 to 5 m· · n. Beta-1,4 bridges are hydrolyzed
ut this enzyme is absent i
animal and human dig rve system, and hence cellulos ( 3m)~~ -
cannot be digested. Herbivorous animals have large These are polysaccharides containing more than one
cecum, which harbor bacteria. These bacteria can type of sugar residues. Examples are:
hydrolyze cellulose, and the glucose produced is utilized
by the animal. ~ n ts (.te,rmi~ so..dioefil.9BlJuJo.se gar
(
~th tbe help .QL iot.e.stillaL b~ter.ia- Cellulose has a It is prepared from sea weeds. It contains galactose,

100°C, which upon cooling sets into e


glucose and other sugars. It is dissolved in water at
Agar cannot
be digested by bacteria and hence used widely as a
S!J.12129ctiog agent to culture bacterial colOJJies. Agar is
used as a supporting medium for imn'lunodiffusion and
immunqelectrophm~~s. Agarose is made up of galac-
Alexander Fleming Ernst Chain Howard Florey
NP 1945 NP 1945 NP 1945 tose combined with 3,6-anhydrogalactose units; it is
1881- 1955 1906-1979 1898-1968 used as matrix for electrophoresis (see Chapter 31 ).
98 Section A: Chemical Basis of Life

H
0 H 0

1
0
0

l,,
0-
'1'
H OH H OH H NH-CO-CH3

Fig. 7 .24: Sulfated glucosamine-alpha-1 , 4-iduronic acid. Repeat- Fig. 7 .25: D-gluc uronic acid-be ta- 1, 3-N-acetylgalactosamine-4
ing units in heparin sulfate (units of chondroitin sulfate)

eparin ( t-\efb-M-0) gu\phole,)


~;(
·.(Y .. f.2111tiges-
It is an t1111X2B1Ulant widely used when taking blood in
vitro for clinical studies. It is also used in vivo in suspected
Dextran Glucose, 1-6, 1-4, 1-3 linkages conaffion ;:,revent intr-avascular
~ ~ t~ ~~- ~t~n~r~ l ~c!i
an~sC=J:~
N=-a=ce=ty
: :1::
gl=uc~o=sa=m
=i::
ne:::;=be=t=a =l-4
= 1in=k=s===i ulatloo. It activates antithrombin Il l, which in turn
/' Agar Galactose, glucose inactivates thrombin, factor X and factor IX.
Agarose Galactose, anhydrogalactose Heparin is present in liver, lungs, spleen and mono-
Hyaluronic acid N-acetylglucosamine, glucuronic acid
cytes. Commercial preparation of heparin is from animal
Heparin Sulfated lu samine, L-iduronic acid
lung tissues.
Keratan S Galactose, N-acetylglucosamine It contains repeating units of
Dermatan S L-iduronic acid, N-acetylgalactosamine (Fig. 7.24).

L :.. II
.

lir.:"'

., •
. e
.. · •" . - • -·4·. - - ,~ If-

,
(_~11')
It is present in ground substance of connective tissues
widely distributed in cartilage, bone, tendons, cornea
and skin. It is composed of repeating units of . . .

nd so on (Fig. 7.25).

It is the only GAG, which does not contain any uronic


Mucopolysaccharides are excreted in urine in acid. The repeating units are galactose and N-acetyl-
abnormal amounts in the group of lysosomal storage glucosamine in beta linkage. It is found in cornea and
disorder s known as mucopolysaccharidoses. They tendons.
can be deJected by 20 gel electrophoresis techniques;~
some mucopolysaccharides can also be detected by ~ - rmatan Sulfate
simple urine screening tests like Ce?1est, c ~ {°lon test It contains - ~ - .--_....._ • , _:_ ,. •• r-. ..,-,.,,r', 1' ,it :: ••ll•'lli,,':,'

and Al"i~ blue staining. It is found in skin, blood vessels and


V " 'h\7("\
heart valves. Repeating units in various polysaccharides
J:\ Hyaluronic Acid
/f7it
is present in connective tissues, tendons, synovi
~ ~ .fluid.and vitreous humor. It serves as a lubricant in joint
~ cavities. It is composed of repeating units of
When the carbohydrate chains are attached to a
<h() d'.¢J aw oil. polypeptide chain it is called a proteoglycan. If the
Chapter 7: Chemistry of Carbohydrates 99

carbohydrate content is less than 10%, it is generally Saccharin (Benzoic sulfimide) is an artificial sweet-
named as a glycoproteln. If the carbohydrate content ener. It is used to sweeten products such as drinks,
i is more than 10% it is a mucoprotein. (But some candies, medicines, and toothpaste. Although saccharin
authors use these words as synonyms). They are seen has no food energy, it can trigger the release of insulin in
in almost all tissues and cell membranes. About 5% of humans and rats, apparently as a result of its taste. It is
the weight of the cell membrane is carbohydrates. If the not banned anywhere in the world.
carbohydrate groups cover the entire surface of the cell
membrane, they are called glycocalyx. Glycoproteins Qletary Fiber
act as enzymes, hormones, transport proteins, struc- Dietary fiber is contributed by the unavailable carbo-
tural proteins and receptors. hydrates in the diet. Several different types of dietary
The oligosaccharide chains of glycoproteins are fiber have been found in different types of food items.
composed of varying numbers of the following carbo- They contri~ute the ~ k and assis~ normal bowel
hydrate residue: Glucose (Glu); mannose (Man); galac- movemenfsl)Cellulosef.Viemicellulos~ cti~ inates
tose (Gal); N-acetyl glucosamine (GluNAc); N-acetyl a urns are the usual glycans which form dietary fiber.
galactosamine (GalNAc); arabinose (Ara); Xylose (Xyl); Of these cellulose, a homoglycan made of glucose is
L-fucose (Fuc) (see Fig. 7.17) and N-acetyl neuraminic found in bran, flour and tubers. Pectins are mixtures of
acid (NANA). Glycoprotein metabolism is further elabo- homoglycans found in fruits like apples and berries.
rated in Chapter 10. Gums and alginates are found in legumes and oatmeal.
Carbohydrate group is attached to proteins either as
O-glycosidic linkages or as N-glycosidic linkages. The Bacterial Cell Wall
O-glycosidic linkage is GalNAc to serine or threonine Major constituents o roRaryotic (bacterial) cells are
residues of usual protein . However, galactose is added heteropolysaccharides, consisting of repeating units
to hydroxylysine residues of collagen. The N-glycosidic of N-acetylmuramic acid (NAM) and N-acetylglucosa-
linkages are made by addition of carbohydrate group to mine (NAG). This polysaccharide provides mechanical
nitrogen atom of asparagine or glutamine residues of strength. ~Synthesis of this complex polysaccharide is
proteins. blocked by penicillin. This inhibition is responsible for
F. l:'}i. ~ \ ) ~ ! the bactericidal action of penicillin. Penicillin was dis-
ommon Sugar Substitutes
covered by Sir Alexander Fleming in 1928. Later, Ernst
cesulfame-Potassium (Ace K) is made from aceto Chain isolated and purified penicillin. The first clinical
acetic acid. It is 2.90 times more sweet than sugar; but trial with penicillin was conducted by Howard Florey in
~rje content is negligible. It is present in artificial sweete- 1940. All the three were awarded Nobel prize in 1945.
ners, carbonated drinks and pharmaceutical products.
~ Aspartame is made from aspartic acid and phenyl
LEARNING POINTS, CHAPTER 7
~ . both are amino acids. It is 200 times sweeter
than sugar. Aspartame is not suitable for people with 1. Carbohydrates are polyhydroxy aldehydes or keto-
phenyl ketonuria. nes or compounds, which yield them on hydroly-
{f)saccharln is made from a.otbcaoilic acid. It is ~ sis. Simplest carbohydrates are monosaccharides
times sweeter than sugar. Calorie content is nil. which may be trioses (3C), tetroses (4C), pentoses
® Sucralose is made from sucrose or table sugar. (SC) and hexoses (6C).
2. Carbohydrates are classified into monosaccha-
Sucralose is approximately 600 times as sweet as
rides, disaccharides and polysaccharides, based
sucrose (table sugar), twice as sweet as sacc~arin, and
on the number of sugar/saccharide units they pos-
~ times as sweet as aspartame. Calorie content is nil.
sess. Disaccharides have 2 monosaccharide units,
'. ~ Cyclamate (Sodium cyclohexyl sulfamate) is ~ oligosaccharides around 10, and polysaccharides
ti~ sweeter thg_n sugar. Cyclamate: saccharin mixture more than 10. They could also be classified as
is found to increase the incidence of bladder cancer in aldoses and ketoses based on the functional group
rats. Its sale is banned in the United States. However, they possess.
Cyclamate is approved as a sweetener in over 55 3. Common examples of monosaccharides include
countries. Glucose, Fructose, Galactose, and Mannose.
100 Section A: Chemical Basis of Life

4. Common examples of disaccharides are Sucrose, 11. Optical isomerism is based on the rotation of plane
Lactose and Maltose. polarized light by a pure solution of the sugar. The

t,sofll~)
5. Monosaccharides exhibit stereoisomerism, optical
isomerism, anomerism and pyranose-furanose iso-
prefix 'd' or(+) and 'I' or(-) is used to indicate dextro
and levorotatory compounds respectively. •
'l,.
merism. 12. All reducing sugars form characteristic osazone
6. All carbohydrates are considered to be derived from crystals. Glucose and fructose form needle-shaped
glyceraldehyde by successive addition of carbons. crystals, maltose forms sunflower-shaped crystals
The penultimate carbon atom is thus the reference and lactose forms hedgehog-shaped crystals.
carbon atom for naming mirror images. 13. Amino sugars form important components of muco-
7. Stereoisomerism is the property of monosaccha- polysaccharides. For example, Galactosamine, Glu-
cosamine.
rides, due to the difference in orientation of H and
14. Sucrose is formed from glucose and fructose linked
OH around the reference carbon atom. The stere-
by 1, 2 glycosidic linkage.
oisomers are prefixed as 'D' or 'L'. D sugars are
15. Lactose is formed from galactose and glucose
naturally occurring and human body can metabo-
linked by beta 1,4 glycosidic linkage.
lize only D sugars. 16. Maltose is formed from two glucose molecules
8. A carbon atom bound by four different groups on all linked by alpha 1,4 glycosidic linkage.
its valencies is referred to as an asymmetric carbon. 17. Starch is made of two components; straight chain
When two sugars differ from each other in the con- amylose and branched amylopectin. The linkages
fig uration around one carbon atom (other than the are alpha-1,6 type at branch points while alpha-1,4
reference carbon), they are diastereoisomers. linkages form the straight chain.
9. A pair of monosaccharides which differ from each 18. Action of amylase on starch yields limit dextrins.
other in the configuration around a single carbon 19. Mucopolysaccharides or Glycosaminoglycans
atom are called epimers. Anomers of monosaccha- (GAGs) such as Hyaluronic acid, Chondroitin sul-
rides are produced by the spatial configuration with
reference to the first carbon atom in aldoses and
fate, Keratan sulfate, Dermatan sulfate are asso-
ciated with connective tissue.
,
the second carbon atom in ketoses. 20. Keratan sulfate is the only GAG that does not
10. Two anomers of glucose are alpha-O glucose and contain uronic acid .
beta-O glucose. Mutarotation is the result of anom- 21 . When the carbohydrate chains are attached to a
erism. polypeptide chain it is called a proteoglycan.

PART-1: SHORT NOTE QUESTIONS

7-1. Classification of monosaccharides.


7-2. Reduc ing disaccharides.
7-3. Non-reducing disaccharide.
7-4. Why sucrose is a non-reducing sugar?
7-5. Transport mechanisms of glucose.
7-6. Mutarotation.
7-7. Anomers.
7-8. Epimerism.

PART-2: MULTIPLE CHOICE QUESTIONS


7-1. Glucose and Galactose are: 7-3. A pair of sugars differi ng from each other in the
A. Anomers functional group is called: +
'
8 . Constituents of sucrose A. Anomers 8 . Epimers
C. Diastereoisomers C. Racemers D. Stereo isomers
D. Epimers 7-4. Reducing sugars are differentiated from non-
7-2. 0 -mannose is: reducing sugars by:
A. The 4th epimer of glucose A. Iodine test 8 . Molisch's test
8 . A keto sugar C. Seliwanoff's test D. Benedict's test
C. Is a component of maltose 7-5. In the case of monosaccharides, all properties go
D. Is the 2nd epimer of glucose hand in hand, except:
Chapter 7: Chemistry of Carbohydrates 101

A. Reducing property C. Alpha 1-6 linkage


B. Formation of furfural D. Alpha 1-2 linkage
C. Formation of osazone 7-14. Hydrolysis of maltose will give rise to:
D. Mutarotation A . Glucose only
7-6. All the following are sugar alcohols, except: B. Glucose and fructose
A. Dulcitol B. Mannitol C. Glucose and galactose
C. Xylulose D. Sorbitol D. Glucose and mannose
7-7. The glycosidic linkage seen in sucrose is: 7-15. Which contains alpha glycoside linkages?
A. Alpha 1-4 linkage B. Beta 1-4 linkage A. Chondroitin sulfate
C. Alpha 1-6 linkage D. 1-2 linkage B. Keratan sulfate
7-8. All the following have glycosidic bond, except: C. Hyaluronic acid
A. Maltose D. Amylopectin
B. Sucrose 7-16. Which contains a beta glycosidic linkage?
C. N-acetylglucosamine A. Heparin B. Glycogen
C. Cellulose D. Starch
D. Alpha methyl glucose
7-17. All the following are hompolysaccharides, except:
7-9. The sugar found in milk is:
A. Chitin B. Glycogen
A. Galactose
C. lnulin D. Heparin
B. Glucose
7-18. The heteropolysaccharide which does not contain
C. Fructose
uronic acid is:
D. Lactose
A. Keratan sulfate
7-10. Sucrose consists of:
B. Dermatan sulfate
A. Glucose + glucose
C. Chondroitin sulfate
• B. Glucose + fructose
D. Hyaluronic acid
C. Glucose+ galactose 7-19. On oxidation, glucose will give rise to all the fol-
D. Glucose + mannose lowing, except:
7-11 . Which is a non-reducing sugar? A. Glucuronic acid B. Gluconic acid
A. Maltose B. Sucrose C. Glucosaccharic acid
C. Lactose D. lsomaltose D. Glycoside
7-12. The glycosidic linkage seen in lactose Is: 7-20. Hyaluronic acid is seen in:
A. Alpha 1-4 linkage A. Synovial fluid
B. Beta 1-4 linkage B. Cornea of eye
C. Alpha 1-6 linkage C. Outer shells of insects
D. Alpha 1-2 linkage
\
D. Cartilage
7-13. The glycosidic linkage seen in maltose is: 7-21. Beta glycosidic bond is present in
A. Alpha 1-4 linkage A. Starch B. Heparin
B. Beta 1-4 linkage C. lactose D. Glycogen

ANSWERS OF MULTIPLE CHOICE QUESTIONS


7-1 . D 7-2. D 7-3. A 7-4. D 7-5. B 7-6. C 7-7. D
7-8. C 7-9. D 7-10. B 7-11. B 7-12. B 7-13. A 7-14. A
7-15. D 7-16. C 7-17. D 7-18. A 7-19. D 7-20. A 7-21 . C

' PART-3: VIVA VOCE QUESTIONS AND ANSWERS

7-1 . How carbohydrates are classified? 7-3. What is a polysaccharide?


Based on the number of the sugar units available, they • They contain more than 10 sugar units.
are classified as monosaccharides, disaccharides, 7-4. How are they combined together?
oligosaccharides, and polysaccharides. Through glycosidic linkages.
7-2. What is a monosaccharide? 7-5. How are monosaccharides further classified?
Molecules having only one actual or potential sugar Sugars having aldehyde group are called aldoses and
group are called monosaccharides. sugars with keto group are ketoses.
102 Section A: Chemical Basis of Life

7-6. Name some important monosaccharides. The difference in glucose and fructose is dependent on
Glucose, fructose, mannose, galactose. the first and second carbon atoms; and this is masked
7-7. What are pentoses? by the osazone formation.
Monosaccharides with 5 carbon atoms. 7-25. On oxidation of glucose, what are produced?
7-8. Which is the reference carbon atom in sugars? Glucuronic acid; Gluconic acid and Glucosaccharic
Penultimate carbon atom. acid.
7-9. What is the difference between D and L sugars? 7-26. Name some important disaccharides.
They are mirror images with reference to penultimate Sucrose, lactose, maltose.
carbon atom. 7-27. What is the glycosidic linkage in lactose?
7-10. Which isomer is common in nature? Beta 1-4 linkage.
D variety of sugars are common in nature. 7-28. What is the glycosidic linkage in sucrose?
7-11. What is the difference between glucose and galac- 1-2 linkage.
tose? 7-29. Which disaccharide has no free aldehyde or ketone
Galactose is the 4th epimer of glucose. group?
7-12. Galactose is present in which food? Sucrose.
Lactose is present in milk. Lactose contains galactose 7-30. Sucrose (containing glucose and fructose) is a
and glucose. non-reducing sugar, why?
7-13. What is epimerism? Because the glycosidic linkage in sucrose involves 1st
When sugars are different from one another, only carbon of glucose and 2nd carbon of fructose; so both
in configuration with regard to a single carbon atom reducing groups are masked.
(other than the reference carbon atom), they are called 7-31 . Hydrolysis of maltose will give rise to what?
epimers. Two glucose units.
7-14. Give an example. 7-32. Which is the sugar found in milk?
Glucose and mannose are an epimeric pair which differ
only with respect to carbon atom 2. Similarly, galactose 7-33.
Lactose.
What are the component monosaccharides of lac- •
is the 4th epimer of glucose. tose?
7-15. Anomerism is produced with reference with which Galactose and glucose.
carbon atom? 7-34. Sucrose consists of what monosaccharides?
With reference to the first ca rbon atom in aldoses and Glucose + fructose.
second carbon atom in ketoses. 7-35. Name reducing disaccharides.
7-16. How alpha and beta sugars are produced? Lactose and Maltose.
These are anomers. 7-36. How polysaccharides are classified?
7-17. What is the basis of mutarotation? Homopolysaccharides (homoglycans) and hetero-
It is due to the anomeric carbon atom. polysaccharides (heteroglycans).
7-18. What is the difference between glucose and fruc- 7-37. What is a homoglycan?
tose? They are composed of single kind of monosaccharides.
Glucose is aldohexose; fructose is a ketohexose. 7-38. Give examples of homopolysaccharides.
7-19. What is the principle of Benedict's test? Starch, glycogen.
In alkaline medium, sugar reduces cupric ions, red 7-39. What are heteropolysaccharides?
precipitate. They are composed of two or more different monosac-
7-20. Benedict's test is commonly done for what? charides.
To detect the presence of glucose in urine. 7-40. What are the characteristics of glycogen?
7-21 . Name a few reducing sugars. It is composed of glucose units. It is the stored form of
l
-,
Glucose, fructose, mannose. carbohydrate in animal kingdom. It has a highly bran-
7-22. -Keto group is non-reducing; but fructose reduces ched structure.
Benedict's solution; what is the cause for this ano-
maly?
7-41 . Give examples of heteropolysaccharides.
Agar, hyaluronic acid, heparin, chondroitin sulfate.

(

In alkaline medium, ketone group is converted to alde- 7-42. What are mucopolysaccharides?
hyde, through enediol formation. They contain uronic acid and amino sugars.
7-23. In the case of sugars, which of the properties go 7-43. Wh ich heteropolysaccharide does not contain
hand in hand? uronic acid?
Reducing property; osazone formation and Mutaro- Keratan sulfate.
tation. 7-44. Hyaluronic acid is seen in which tissues?
7-24. Glucose and fn,1ctose will form identical osazones, Connective tissue, Synovial fluid , tendons, vitreous
why? humor.

'
Chemistry of Lipids
-
Chapter at a Glance
The learner will be able to answer questions on t he following topics:

I
Classification of lipids ,______ { Phospholipids
Classification of fatty acids >,- D Phosphatidylcholine or lecithin
B Saturated and unsaturated fatty acids \ D Sphingomyelin
Neot,alf,ts o,t,;acylglymols l Non-phospho,ylated hp;d, ( Gt 1'.f u, \<pi d.()

(~L)
Lipids constitute a heterogeneous grou 1. Simple lipids: They are ~ s Qf~ acjds with
of biochemical importance. Li ids ma glycp,r.ol or other higher alcohols {Table 8.1 ).
compounds which are relativ :===:;:;=====:=~--=====-·
y insol le in wa er, but 2. nd lipids: They are fatty acids .esterified
freely soluble in non-polar organic olvents, such as with alcohol; but in addition they contain oj her
be~ne, ~ roform, ~ . hot alcohol, a~ e. etc. groups. Depending on these extra groups, they are
The functions of lipids are summari e in Box 8 .1. The subclassified in Table 8.1.
clinical applications are shown in B 8.2. ..JI a. Phospholipids, containing phosphoric acid.
"t' b. Nof'}.-phosphorylated lipids {Table 8.1)
ATIONOF 3. Derivetf ltpids, They are compounds, which are
d~ fr~ ~s of lipids, e.g.
fatty acids~ teroids. For details o{ cholesterol and
sterojds. see Chapter 14.
4 . Lipids complexed to other compounds.
1. Storage form of euer9ll (triacylglycerol) '1n"
2>,_ Structural components of biomembranes) W os,etwlipids an
(!)I cholesterol) lit')
Fatty acids, are included in the group of derived lipids.
3. Meta~ ic regulato~ st~ d hormones and p~staglandins)
4. Acta~m .Jllctal}_ts, Q&rgents and Q lsifying agents (amphi·
It is the most common component of lipids in the body.
pathic lipids) They are generally found in ester linkage in different
5. Act as el~ctric insulators io oemORs
6. Provide insulation against changes in external temperature BOX 8.2: Clinical appllcat1ons
(subcutaneous fat)
1. Excessive fat deposits cause obesity. Truncal obesity is a risk
7. Give sbjlpe and contour to t he body factor for heart attack.
8. Protect internal organs by providing a cushioning effect 2. Abnormality in cholesterol and lipoprotein metabolism leads to
(pads of fat) atherosclerosis and cardiovascular diseases (see Chapter 14).
9. Help in absorption of fat soluble vitamins {A, D, E and Kl 3. In diabetes rr.ellitus, the metabolisms of fatty acids and
10. Improve taste and palatability of food. lipoprotelns are deranged, leading to ketosis (see Chapter 13).

l_
TABLE 8.2 : Charactensllcs of common fatty acids
No. of
a. Triacylglycerol or Triglycerides or neutral fat
b. Waxes
Common name
carbon
atoms Chemical nature I
Occurrence
A. Even chain, saturated fatty acids
II. Compound lipids
Acetic 2 Saturated; small chain Vinegar
A. Phospholipids, containing phosphoric acid _
Butyric 4 do Butter
1. Nitrogen containing glycerophosphatides:
Caproic 6 do Butter
i. Lecithin (phosphatidylcholine)
Capric 10 Saturated medium Coconut oil
ii. Cephalin (phosphatidylethanolamine) chain
iii. Phosphat idylserine Laurie 12 do Coconut oil
2. Non-nitrogen glycerophosphatides Myristic 14 do
·-
Coconut oil
i. Phosphatidylinositol Palmitic 16 Sat urated; long cha in Body fat
ii. Phosphatidylglycerol Stearic 18 do do
iii. Diphosphat idylglycerol (cardiolipin) Arachidic 20 do Peanut oil
3. Plasmalogens, having long chain alcohol (Arachis oil)

i. Choline p lasmalogen B. Odd-chain fatty acids


Propionic 3 Saturated; odd chain Metabolism
ii. Et hanolamine plasmalogen
4. Phosphos hingosides, with sphingosine C. Even chain, unsaturated fatty acids
Palmitoleic 16 Monounsaturated (w7) Body fat
Sphingomyelin
Oleic 18 do (w9) do
8. Non-phosphorylated lipids
Erucic 22 do (w9) Mustard oil
1. Glycosphingolipids (carbohydrate)
Nervonic 24 do (ro9) Brain lipids
i. Cerebrosides (ceram ide monohexosides)
Linoleic 18 2 double bonds (006) Vegetable oils
ii. Globosides (ceramide oligosaccharides)
Linolenic 18 3 double bonds (w3) do
iii. Gangliosides (ceram ide + oligosaccharides + N-acetyl-
Arachidonic 20 4 doub le bonds (ro6) Vegetable oils
neuraminic acid)
2. Sulfolipids or sulfatides
i. Sulfated cerebrosides fie Palmitic acid CH3-(CH2) 14-COOH
lo_b_os_id_e_s_ _~~...:::'--""Tl~,c;_- -w.t-~
~.._--- i_i. _S_u l_fa_te_d--"g'- Stearic acid CH3-(CH2 ) 16-COOH
iii. Sulfated gangliosides Some of the common saturated fatty acids are
Ill. Derived lip ids noted in Table 8.2. They are named by adding the suffix
Fatty acids, steroids (see chapter 14), prostaglandins (see 'anoic' after the hydrocarbon . The two carbon acetic
Chapter 16), leukotrienes, terpenes, dollchols, etc. acid and 4 carbon butyric acid are important metabolic
IV. Lipids complexed to other com pounds
Proteolipids and lipoproteins. ...
intermediates. The C16 (palmitic acid) and C18 (stearic
~cid) are most abundant in body fat.
Each animal species will have characteristic pattern
classes of lipids. In the human body, free fatty acids are of fatty acid composition . Thus, human body fat contains
formed only during metabolism. 50% oleic acid, 25% palmitic acid, 10% linoleic and
Fatty acids are aliphatic carboxylic acids and 5% stearic acid. The carbon atoms of fatty acids are
have the general formula, R-CO-OH, where COOH numbered as C1 , C2, etc. starting from the COOH group.
(carboxylic group) represents the functional group. Or, starting from the methyl end, the carbon atoms may
Depending on the R group (the hydrocarbon chain), the be numbered as omega (co)-1, 2, 3, etc.
physical properties of fatty acids may vary. Characteris- ..,,.. 5 19-4 3 2 1
tics of common fatty acids are shown in Table 8.2. Classi- \ d<-~ <"'G!:½-
CH 2 - CH2 - CH2- CH 2 - COOH
fication of fatty acid is given in Table 8.3. co I w2 w3 w4 w5
VJ!!.'\
®---=~- FATTY ACIDS -@) UNSATURATED FATTY ACIDS
They have the general formula CH 3-(CH 2 )"-COOH. They are named by adding the suffix 'enoic' after the
For example, systematic name. They are similar to saturated fatty
Acetic acid CH 3- COOH acids in the reaction of the carboxylic group but also
Butyric acid CH 3(CH2 ) 2- COOH show properties due to presence of the double bond.
l
TABLE 8.3: Class1f1cat1on of fatty acids
1. Depending on total number of carbon atoms:
a. Even chain:
They have carbon atoms 2, 4, 6 and similar series. Most of the 2. aci because they cannot be
naturally occurring lipids contain even chain fatty acids. synthesized by ave to be supplied in the diet.
b. Odd chain: 3. ~ urated fatty acids ar~o designated as
They have carbon atoms 3, S, 7, etc. Odd numbered fatty acids lw3fam.lJY. - Linolenic aci~ ig_,_8.2J
are seen in microbial cell walls. They are also present in milk.

G9
w6 famUz:. - Linoleic and Aracbidoni~ (Fig. 8.2)
2. Depending on length of hydrocarbon chain:
family - Oleic acid
a. Short chain with 2 to 6 carbon atoms
4. Arachidonic acid is the precursor of prostaglandins. Arachi-
b. Medium chain with 8 to 14 carbon atoms donic acid can be synthesized in the body, if the essential
c. Long chain with 16 and above, usually up to 24 carbon atom s fatty acids are supplied in the diet.
d. Very long chain fatty acids (more than 24 carbon). 5. Th® 'taenoiu c;u;14>resent in fish oils is of great nutritional
3. Depending on nature of hydrocarbon chain: importance ( w3 unsaturated fatty acid).
a. Saturated fatty acids (Table 8.2) 6. Eicosanoids (eicosa = twenty) are derived from 20 C ara-
b. Unsaturated fatty acids which may be subclassified into Mono- chidonic acid. They are polyenoic fatty acids. They are pros-
unsaturated (monoenoic) having single double bond or Poly- tanoids (prostaglandins, prostacyclins, thromboxanes) and
unsaturated (polyenoic) with 2 or more double bonds (Table 8.2) leukotrienes. See Chapter 16.

Unsaturated fatty acids exhibit geometrical isome-


rism at the double bonds (Fig. 8.1). All the naturally
ITRANS FATTY ACIDS (lfA)
TFA are present in dairy products and in hydrogenated
occurring fatty acids have the cis configuration. How-
edible oils. They are generally considered to be
ever, in the body during metabolism trans fatty acids are
injurious to health. However, TFA are used in food indus-
formed (see Chapter 16). (t;:r,,~ try as they increase the shelf life of the fried food . Oils
The polyunsaturated fatty acids (PUFA) exist in containing PUFA also have high content of TFA. Fast
cis configuration in naturally occurring lipids. Clinical foQQ preparations have a ,blgh T.fA content. Trans fatty
significance of PUFA is shown in Box 8.3. acids adversely affect composinon of blood lipids and
Many clinical and epidemiologic studies have shown lipoproteins, systemic inflammation, endothelial dys-
positive roles for omega-3 fatty acids in infant develop- function , insulin resistance, diabetes and adiposity.
ment; various mental illnesses, including depression,
Properties of Fatty Acids
attention-deficit hyperactivity disorder, and dementia.
The composition of some of the common oils and fats
These fatty acids are known to have pleiotropic effects,
are given in Table 8.4.
including effects against inflammation, platelet aggrega-
tion, hypertension, and hyperlipidemia . :,drogenation
Unsaturated fatty acids may be converted to the corre-
CH34CH2 ).,--CH
II I sponding saturated fatty acids by hydrogenation of the
HOOC-(CH2).,--CH HC-(CH2h-CH3 double bond.
II
HOOC-(CH2).,--CH . I . (+)2H L.InoeIc
I . ~(+)2H 01e,c~
. (+)2H Steanc
.
LInoenic~
G:is-form /OJpir __,_,, ) ~ ans-form (El~~
Hydrogenation of oils can lead to solidification and
Fig. 8.1 : Cis- and trans-forms of fatty acid
saturation, e.g. Vanaspati.

Linoleic (C 18) 9, 12 (two double bonds) (w6 family)


CH3 (CH2 ), - CH= CH-CH2 - CH = CH-(CH2h -COOH

-----------------------------------------------------------------------------------
18 ro6 12 9 1
Linolenic (C A9. 12. 15 (three double bonds) (w3 family)
18)
CH3-CHz-CH = CH-CH2-CH = CH - CHcCH" = CH- (CH2h -COOH
18 Cil3 15 12 9 1

Arachidonic (C 20) AS, 8, 11, 14 (four double bonds) (w6 family)


CH3- (CH2 )4 -CH = Cf+-CH2- CH = CH-CH2 - CH = CH- CH2 - CH = CH-(CH2 lJ -COOH
20 ro6 14 11 8 5 1

Fig. 8.2: Polyunsaturated fatty acids (PUFA)


106 Section A: Chemical Basis of Life

TABLE 8 .4 : Compos1t1on of 01Is and fats 0. (1)


CH-0-CO - R
Saturated Mono-unsaturated I '
Name fatty acids (%) fatty acids(%) PUFA (%) (2) R-C0-0 - C-H
Coconut o il (*) 86 12 2 I
CH2-0-CO-R
Groundnut oil 18 46 36 a' (3)
Gingelly oil (Til oi l) 13 so 37
Fig. 8.3: Triacylglycerol (TAG) (Trig lyce ride )
Pa lm oil 42 52 6
Corn o il 13 25 62
Sodium and potassium salts of long chain fatty acids
Cotton seed oil 26 19 55
12 48 40 are called soaps. Calcium and magnesium soaps are
Seasame o il
M ustard oil 34("") 48 18 insoluble. Calcium soaps are used in grease.
(rapeseed) Alkyl sulfate (R-CH2- O-SO2- ONa) and alkyl
9 12 79 sulfonate (R-CH2-SO2-O-Na) are not precipitated

--
Safflower oil
(Kardi) by hard water and are used as detergents.
Sunflower oil 12 24 64
Butter 75 20 5 \ E er Formation
Ox(Tallow) 53 42 5
Both saturated and unsaturated fatty acids form esters
Pig (Lard) 42 46 12
with alcohols, especially with glycerol. Fatty acids can
Fish oi l 30 13 57
form mono-, di- or tri-esters with alcohol groups of
(*) t hese saturated fatty acids are medium chain fatty acids.
("") contains erucic acid, 22 C, 1 double bond.
glycerol. Triglycerides or triacylglycerols are also known
as neutral fat (Fig. 8.3).
Glycerol+ fatty acid -+ Monoacylglycerol (MAG)
@ ~ ation . .. Monoglyceride + fatty acid -+ Diacylglycerol (DAG)
When treated with halogens under mild condItIons, the Diglyceride + fatty acid -+ Triglyceride or
unsaturated fatty acids can take up two halogen atoms, triacylglyccerol (TAG)
at each double bond to form the halogenated derivativ~ . . .
of the fatty acid. For example, u, Oxidation at Fatty Acids
Oleic acid + 12 -+ Di-iodo oleic acid All fatty acids undergo oxidation in the body to give
The number of halogen atoms taken up will depend energy. Beta-oxidation is the major process by which
on the number· of double bonds and is an index of th~ fatty acids are oxidized (see Chapter 13). However, the
degree of unsaturati~m (see iodine number, under tn- unsaturated fatty acids can undergo auto-oxidation, due
acylglycerol). to the presence of the highly reactive double bonds and
;::;\ ariety 9f. r ducts r~ f3d.
~ ng Point ..~ :.>4+J '3.
The short and medium chain fatty acids are liquids, l!lttRAL F JS . ._,,-; ~..o.\tt>
whereas long chain fatty acids are solids at 25°C. The
Neutral fats are also called as triacylglycerols (TAG)
solubility in water decreases, while melting and boiling
or triglycerides (TG). These are e sters o7'the' trihydric
points increase with increase in chain length.
alcohol, glycerol with fatty acids (Fig. 8.3).
The unsaturated fatty acids have lower melting
point compared to saturated fatty acids with the sam omenclature of Carbon Atoms /
chain length. For example, stearic acid (C18 fatty acid,
As per International Union of Biochemistry (IUB) the
no double bond) has the melting point 69°C, oleic acid
correct designations are monoacylglycerol (MAG),
(C18, 1 double bond) has 13°C; linoleic acid (C1 8, 2
diacylglycerol (DAG) and triacylglycerol (TAG). But the
double bonds) has - 5°C and linolenic (C18, 3 double
old terminology of mono~ceride, digl~ ride and tri-
bonds) has -10°C.
gly~ ride are still popular, especially among clinical

@sa1t ~ or mation laboratory workers.


The carbon atoms of glycerol are designated as a.
Saturated and unsaturated fatty acids form salts with and a' or as 1, 2, 3 as shown in Figure 8.3, where R
represents the side chain of fatty acids. Enzyme_§_ can

- -
alkali.

CH3- COOH + NaOH -+ CH 3- COONa + Hp distinguish between 1,st3 od 3rd r.arbon atoms.
Chapter 8: Chemistry of Lipids 107

Lipase Lipase
(+) 3 NaOH
( \ • ( ' \ • Monoacyl- Triacytglycerol - - - - - - - Glycerol+3 R-COONa

H20 ::i~Y H20 ::i: glyce


l
+ H20
Fatty acid
+
Glycerol

Fig. 8.5 : Saponification

1xed Triglycerides Triacylglycerols in the body are hydrolyzed by enzymes,


Naturally occurring fats and oils are mixtures of trigly-__ lipases w hich are hydrolases (class 3 enzymes,
cerides. If a~ s of the glycerol Cffa~ter 5). T?1cylglycerol is sequentially hydrolyzed
are esterified to the s me fa acid, a sJ2 triacyl- to diacylglycerol plus monoacylglycerol and finally gly-
glycerol is formed, e.g n a mItIn nol · etc. A mixed ol plus 3 fatty ~ (Fig.8.4).
tr@_cylglycerol is forme , when different fatty acids are • •
esterified to the hydroxyl groups of glycerol. Generally, apomficaf ion
two hydroxyl groups are esterified to similar fatty acid When tria~1cerols are hydrolyzed by l!§J), the
and the third wjth a different one, e.g. palmitic acid in 1 process is known as(ijoonjficatjoo> The products are
and 3 positions, while oleic acid is in.. the 2nd position. glycerol and soaps (Fig. 8.5). Saponification number
When a,!tJFA Is ;reseril it is commonly esterified to the is defined as tlhe nY.DJbec aff milligcaros) of potassium
or /§arbon t,Qm. hydroxide requiired to s ~ ~at. It is
\.Y an indication of the molecular weight of the fat, and is
Physical Properties of Triacylglycerols inversely propo1iional to it. Human fat has a saponifica-
t hey are hydrophobic and insoluble in water. Oils are _ tion number ofl:[94- 19!!1 butter has g 10-23g and coco-
liquids at 20°c ; they are triacylglycerols, which contain oil has ~ ~
,t-
a higher proportion of L.msaturated fatty acids or short Iodine Number
triglycerides. Oils are generally of plant origi!i]
~ ats are solids at room temperature and contain mainly Iodine number of a fat is defined as the number of grams
sa!;'rated long chain fatty acids. Fats are mainly of ~ o.i..iQ.Qio_e taken u~ a,!!l_S,..Qf fat. It is an index of
origin (Table 8.4 ). When the constituent fatty acids have the degree of unsaturation and is directly proportional to
a hi!lbec chajn length and are.,.f,redominantly saturate!l, the content of unsaturated fatty acids. Higher the iodine
'hard fat' is formed, e.g. pig fa.!Jf ats containing medium number, higher is the degree of unsaturation, e.g. iodine
chain triacylglycerols or unsaturated fatty acids are soft umber of ~ ~ @nd that of sµofi~ jl i@
fats. e.g. butter, coconut oil. Coconut oil contains mainly ancidity qf Fat lipid pe,d:>X)(Ud·
dium chain TAG, e.g. !auric and myristic acid~
Fats and oils h.ave~ cy to rancid. The
torage of Energy as Fat term rancidity re!fers to the appearance of an unpleasant
The triacylglycerols are the storage form of lipids in the smell and taste for fats and oils.
adipose tiuue. In a 70 kg normal person, body stores (>r\, Hydrolytic r ncidity is due to partial hydrolysis of
the triacylglycerol molecules due to traces of hydrolytic
contain about 11 kg of triacylglycerol, which is roughly
enzymes present in naturally occurring fats and oils.
equivalent to 100,000 kcal. If the same calories were
( Oxidative rancidity is the result of partial oxidation of
stored as hydrated glycogen, the total weight of this
unsaturated fatty acids with resultant formation of epoxi-
alone would ...have been 65 kg! When stored as TAG,
des and peroxioles of small molecular weight fatty acids
water molecules are repelled and space requirement is
by peroxides and free radicals. The same process, if it
· ·mal. Excess fat in the body leads to obesity.
occurs in vivo will affect the integrity of biomembranes,
ydrolysis of Triacylglycerols (TAG) leading to cell dieath.
This occurs in the body during digestion of dietary Many natural fats and oils may contain antioxi-
fat and mobilization of TAG from adipose tissue. dants (e.g. vitamin E), which prevent the occurrence of
108 Section A: Chemical Basis of Life

oxidative rancidity. PUFA are more easily oxidized; so portion in their molecule (Figs. 8.7A and 8.8). The gly-
vegetable oils with a high content of PUFA are usually cerol along with the phosphoric acid and choline consti-
preserved with addition of antioxidants. tute the polar 'head' of a phospholipid molecule, whereas
Repeated heating of oils would lead to the formation the hydrocarbon chains of the fatty acids represent the

?,m);;'r
and polymerization of cyclic hydrocarbons. These will non-polar 'tail'.
impart an unpleasant taste and color to the oil. Coconu
oil having medium chain saturated fatty acids will with icellar Fon11ation (_
stand such polymerization. When phospholipids are distributed in water. their hydro-

@ Waxes
phobic parts keep away from water, forming molecular
aggregates called micelle (Fig. 8.78). These are involved
They fo rm the secretions of insects, leaves and fruits of in solubilization of lipids in aqueous media and help in
plants, e.g. Lanolin or wool fat, beeswax, whalesperm digestion and absorption of lipids.
oil, etc. They are esters of higher fatty acids with higher, ,
monohydroxy aliphatic alcohols and so have very Ion iposomes Ju.&~ 2,-)i .t;.oe.. no~ l.&
straight chains of 60-100 carbon atoms. They are used
as the base for the preparation of cosmetics, ointm~ . A lipid bilayer will close on itself under appropriQfcon-
polishes, lubricants and candles. (? ditions to form liposomes. Unilamellar or multilamellcy-

(3)
liposomes may be formed. They may be prepared by
PHOSPHOLIPIDS ( <; ~. . -tt sonication of mixtures of phospholipids and cholesterol
(Fig. 8.78). Liposomes are microscopic spherical vesicles.
© Phosphatidates When mixed in water under special conditions, the phos-
i. These are derivatives of pho~ pholipids arrange themselves to form a bilayer mem-
the simplest phospholipid brane which encloses some of the water in a phospholipid
ii. Phosphatidic acid is made up of one glycerol sp,.!:u;Le.. Drugs, proteins, enzymes, genes, etc. may be
to which two fatty acid residues are esterified to encapsulated by· the liposomes which could act as car-
carbon atoms 1 and 2. The 3rd hydroxyl group is r iers for these substances to target organs. Liposome-
esterified to a phosphoric acid (Fig. 8.6). entrapped drugs exhibit superior pharmacological
properties than those observed with conventional formu-
AmphiRathi~ Na ure la~ s. Liposom1:!~ ave important appl~ ions in can-
Phospholipids in general are amphipathic, particularly c~ hemoth~ p~ timicrobial thera~ ene therapy,
Lecithin. They have both hydrophobic and hydrophilic accines an~gnostic imaging.

They are one of !Irie recently developed delivery systems


that act as carrier systems for delivery of peptide based
pharmaceuticals. These are nanoparticulate carrier

Fig. 8.6: L-Phosphatidic acid Lipid bilayer j Liposome I •


Phospholipid

.,~:~.~
·l
Heads orient
Hydrophilic toward water
polar head
water

QQQQOOOQ Phospho•
Hydrophobic
non-polar
ill::~,..
tail
Tails orient away from water

Fig. 8.7 A : Phospholipids form the bilayer Fig. 8.78 : Phospholipids form micelles and liposomes
Chapter 8: Chemistry of Lipids 109

.------- -~- - 1- -- ------,~ ction of Phospholipases


Phospholipases are enzymes that hydrolyze phos-

R co1
H2C-o CO-R1 pholipids. Different phospholipases are involved in
o-?H o- L /CH3
2- the hyd rolysis of specific bonds in lecithin (Fig . 8.8).
c-01P-oic-c-N-CH
~2 [hA + "'CH:
Phospholipase A2 acts on an intact lecithin molecule
hydrolyzing the fatty acid esterified to the beta
(second) carbon atom. The products are Lysolecithin
3 4 and fatty acid . The fatty acid (often a PUFA) liberated
1 = Site of action of ob9sohgljpasc Al from ~ r boA is the precursor for the synthesis
2 = S11e of action of ,en§§g§gfiiiai\¢$2 of prostag andins Lysolecithin is a detergent and
3 = Site of action of J>hQ§PbPHPas., c hemo y~ The enzyme is present in the venom
4 = Site of action ofphospholipase D
of viper snakes. The hemolysis and consequent renal
Fig. 8.8: Lecithin R, and R2 are fatty acids. Red rectangle failure seen in viper poisoning cou ld be thus explained.
depicts glycerol group. The blue rectangle is choline which shows Actions of other phospholipases are shown in Figure
polar or hydrophilic property
8.8. The products formed in each case may be summa-
systems. They comprise the central solid nanocrystal- rized as follows:
line core coated with polyhydroxy oligomers onto which Phospholipase A2 L . h" f 'd
biochemically active molecules are adsorbed. The solid Lecithin yso1ecIt in+ atty acI
core provides the structural stability. The carbohydrate LecIt. h.In Phospholiphase A1 > A cyI gIycerop hosp horyI-

coating stabilizes the biochemically active molecules. choline + fatty acid


The delivery sy~ m has ~ n successfu~)\utilized for Lecithin Phospholipase C 1,2 diacylglycerol
the delivery of ~ . hem~lobin and varioirs antigens. + Phosphorylcholine
Oral delivery of enzymes like serratiopeptidase has also
been achieved. $- Lecithin Phospholipase D > Phosphatidic acid+ choline

om.embranes ------,, Gitung Surfactants


Th e moIecu Ies aI.1gn themse Ives t o f orm--:;'UJOno1ay _ ~ f
ed;
t'Normal lung function depends on a constant supply
rf t ~ -th - II It
crl!!C o IunQ su ac an: . Is p I e 1Ia 1 ce s.
with the polar heads pointing in one dIrect1on and me d <.J
. . . . .
rf --...._,....- . d t l\.....£ . f
ecreases ~ I s1c.e::rea::sIao an preven s CO!Jai,l
1 Se o
nonpolar tails in the opposite direction (Figs. 8.7A and B). lung alveoli. Constituents of surfactants are dipalmi-
Only fatty acid~ with ~ore th~n 6. carbon atoms form toyl lecithin, phosphatidyl~ cerol, cholesterol and
~onolayers. This explains their role as comp~n~nt~ of surfactant proteins A, 8 and c. During..(.~! life," fue lung
b~omem~ran~s. The self-assem~I~ of pho~phohp1ds into synthesizes sphingomyelin b'e'fore 28tlr~m oFgesta-
b1layer~ driven by hydrophob_1c interaction. T~ey also tion. But as fetus matures, mQle lecjthjn is synthesized.
act as !@/,rgents and @ tstfy1ng a~ nts. l~ y The lecithin-sphingomyelin (LS)~ f~ iotic fluid is
act a~ lmonary surfactants. an index of fetal maturity. A @ o~ s full lung
• • • maturity. Low surfactant level can lead o respiratory
osphat1dylchohne or Lec1th1,Tlf'lnb(\I_ ~ distress syndrome (RDS), which is a common cause of
i. This is a n ~ containing phospholipid. T e wora neonatal morbidity.

m.~ le~ s derived from the Greek word, lekithos


It contains glycerol. ttosphatidyl~thanoarmine or Cephalin
ii. The alpha and beta positions are esterified with Cephalin differs from lecithin in that the nitrogen base
fatty acids. UsuaUy the fatty aCld attached to the ~ oJawwe is present instead of choline (Fig. 8.9).
beta-car~ c(PUFA molecu~ (Fig. 8.8). Cephalin is also found in biomembranes and possesses
iii. ThephQsphorjc fcid is added to the third position, amphipathic properties.
to form phosphatidic acid. The phosphate group is >
esterified to the quaterna nitro en base, osphatidy~nositol P!;
(Fig. 8.8). The · molecules Here, phosphatidic acid is esterified to inositol (Fig. 8.10).
zwitterions (pl = 6.7). Phosphatidylinositol bisphosphate or PIP2 is present in
110 Section A: Chemical Basis of Life

H2C-O-CO- R1 H C-0-CO-R
21 1
I
R2 - COO-CH OH R2-COO-CH OH
I I + I I
C-0-P- 0 - C-C-NH 3 C-0-P-O~H
OH
I II I I II
H2 O H. H. 2 0
HO
Fig. 8.9: Cephalin (Phosphatidylethanolamine) OH
OH

Fig. 8.10: Phosphatidylinositol

CHdCH2l12-CH=CH-CHOHiH-CH20H I
Fig. 8.11 : Ethanolamine plasmalogen
L _ _ _ __

Fig. 8.12: Sphingosine


_
NH2
______, I
Sphingosine group 0 +/CH 3
II
CHdCH2)12-CH=CH-CHOH-CH-CH2 O-P-O-CH2-CH2-N-CH3
I
NH d \CH 3
I
I Fatty acid group i=O Choline group

Fig. 8.13: Ceramide Fig. 8.14: Sphingomyelin

biomembranes. This compound plays a vital role in the alcohol s hin osine (Fig. 8.12) which is attached
mejiaiioo at hacmopP...action on biomembranes and acts to a fatty acid in amide linkage to form a ceramide
as a second messenger (see Chapter 45). (Fig. 8.13). The fatty acid has a chain length varying
from C18 to C24.
@ Plasmilogens
I
t
These are phospholipids which have an aliphatic Ion hosph~phingqsides
chain a-~ unsaturated alcohol in ether linkage with the They contain phosphoric acid group. A common
first hydroxyl group of glycerol (Fig. 8.11 ). The second OH phosphosphingoside present abundantly in bi.ome.m.:..
group is esterified to a f~ . The phosphoric acid is branes, especially of the nervous system, is sphingo-
attached to ~ (Fig. 8.11 ). The alco- myelin. It contains choline (Ffg. 8.14).
hols have about C12 to C18 chain length. Plasmalogen
are found in biomembranes in brain and muscle. phlngomyelins (3m)-,(>)11'~
® Phosphatidy~lycerol l ~)
It is formed by esterification of phosphatidic acid to
Sphingomyelins are the only sphirw,2~ that
contain phosphate and have no sugar ffi81e . They
are found in large quantities in nervous system. Different
glycerol. When two molecules of phosphatidic acid sphingomyelins may be formed depending ~ the fatty
are linked with a molecule of glycerol, di9hosphaff~- acid attached. Common fatty acids found are~ ignoceric
g¥ cerol or cardlpfle!o is formed. It is the major lipid of (24 C),li'lervonic (24 C, one double bond) an~ c ·
mitochondrial membrane. Commercially, it is extracted (22 C, 6 double bonds) acids. Because of its amphipathic
from myocardium. Decreased cardiolipin level if£ds to nature sphingomyelin can act as an emulsifying agent

-
~dcial d ~nctioo, . 1s imp I d Ml-heart and detergent. The relative proportion of lecithin ·and
failuri,;iypothyr~ sm and som es yopath1'es.° sphingomyelin is important in biological fluids like yie,
a~ otic fluid, etc. Sphingomyelin combined wit~
Sphingolipids ( ... L) acid is called ceramide, which is a component of ~
The sphy,gosine pontaining lip.ids may~be of 3 types; spfungohp1dµ
Phosphdsphingosides,
' I
glycosphing ipids and sul-
J
- Clinical relevance of antiphospholipid antibody is
fatides. All sphingolipids have the long aliphatic amino
- described in Box. 8.4.
Chapter 8: Chemistry of Lipids 111

BOX 8.4: APL antibody syndrome 2. Compound lipids are Phospholipids, Sphingolipids,
Antiphospholipid antibodies have been found to be present S ulfolipids, etc. Compound lipids containing alcohol
in blood in patients with autoimmune diseases and in patients sphingosine and one or more carbohydrate resi-
with thr_ombotic episodes. In pregnancy, t.b£. presence ~ ese dues are called glycolipids.
ant 1b~ s can cause complications like ~ ~~rriage, edll ®Psia
and ~ erm labor. Here the antibodi!!s are developed against 3. Fatty acids are classified based on (i) Number of
me e phospfiolipid, cardlolipin. carbon atoms, (ii) Length of hydrocarbon chain, and
(iii) Nature of hydrocarbon chain. Depending on the

hosphorylated Lipids ( &,o'


'I
number of carbon atoms, fatty acids may be even
chain or odd ch. , ~which are further ~ u,gdlvided
ycospn;ngolipfds (Glycolipids) into short chain ~ ). medium chain (8-14C) and
They are seen widely in nlll)LO_~ es. This group long chain (1.?-ffio~ .
of lipids do"~ t contain phosphoric atid; instead they 4. Palmitic and s aric acid are the most abundant
contain carbohydrates and ceramide. saturated fatty acids in the body.
Ceramide + Glucose -+ Glucocerebroside 5. Fatty acids may be saturated (no double bonds),
V Ceramide + Galactose -+ Galactocerebroside
mono-unsaturated (one double bond) or polyunsa-
l~ -- ,oi+~ ·. turated (more than 2 double bonds). Poly unsatura-
Btobosides ( eramid Oligo accharides ted fatty acids (PUFA) may be essential or non-
essential. Essential fatty acids are those, which
They contain two or more hexoses or hexosamines ,
cannot be synthesized in the human body and have
attached to a ceramide molecule.
to be supplemented in the diet. As for example,
Ceramide + Galactose + Glucose -+ Lactosylceramide
Linoleic acid, Linolenic acid and Arachidonic acid.
Lactosylceramide is a component of erythrocyte
6. Arachidonic acid is the precursor of prostaglandins.
membrane.
0 ._ _ _ _ _ _ _ - - -- 7. Saponification number is defined as the number of f)Dt<JOo'
nglios,des C ~~,. · milligrams of KOH required to saponify
8. Iodine number of a fat is defined as the number of
r-

They are formed when ceramidtV ~ligosaccharides have


grams of iodine taken up by
at least one molecule of NANA (N-acetylneuraminic
acid) (sialic acid) attached to them.
directly proportional to the degree of unsaturation. f1nre
9. Rancidity refers to the appearance of unpleasant ~ \
Ceramide-Glucose-galactose-NANA; odor and taste to oils and fats. Rancidity can be of b,7te..
This is designated as GM3 (ganglioside M3). two types, Hydrolytic and Oxidative.
Gangliosides contribute to stqbility of royelinated nerve 10. Depending on the position of double bonds from the
f~s. omega end, fatty acids may be omega3, omega6,
omega9.
~ folipids or Sulfatides 11 . Sodium and potassium salts of fatty acids are called
soaps.
These are formed when sulfate groups are attached to
ceram:de oligosaccharides. All these complex lipids are 12. Fatty acids can form esters with hydroxyl groups of
glycerol to form mono-, di- and triacylglycerol.
important components of meJ!].Q@.nes of nervo1 is.tissue.
Failure of degradation of these compounds results in 13. Triacylglycerol or neutral fat are the storage form of
energy in adipose tissue.
ac€:mul ~ n of these ~omp!ex lipids io CNS. This group
of inborn errors is known as ,i'p1d storage d1seas~ 14. Major fatty acids found in adipose tissue fat are
These are described in Chapter 16. oleic acid, palmitic acid and stearic acid.
15. MUFA and PUFA are commonly esterified to the
Relate~ To ics .111 ) se.cond carbon (beta carbon) of glycerol.
· CD<t."tl l\p; o-..:4 16. O ils and fats are mixtures of triacylglycerol. Oils are
ChoTesterol, steroids, bile acids and lipoproteins are des-
liquids at 25°C and fats are solids.
cribed in Chapter 14. Chapter 16 contains the metabo-
17. Butter contains short and medium chain fatty acids.
lism of compound lipids.
18. Phospholipids may be glycerophosphatides or phos-
phosphingosides depending on the alcohol present.
LEARNING POINTS, CHAPTER 8 19. Simplest glycerophosphatides is phosphatidic acid
1. Lipids may be broadly classified into Simple, Com- containing gl~ 2 molecules of fatty-aGid and
pound and Derived lipids. one molecule of phosphoric acid.

)
112 Section A: Chemical Basis of Life

20. Phospholipids are amphipathic in nature since they 24. Phosphosphingosides contain sphingosine as alco-
have a polar head and non-polar tail. Amphipathic hol. Sphingosine esterified to a fatty acid is called
nature is ideal for the role of phospholipids as com- ceramide.
ponents of biomembranes and for micelle formation. 25. Sphingomyelin is the only phosphosphingoside
21. Phosphatidic acid may combine with nitrogenous which contains choline.
base to form amino phospholipids like phosphatidyl 26. Sphingolipids have ceramide attached to carbohy-
choline, phosphatidyl ethanolamine and phosphati- drate residues to form glycolipids like cerebrosides
dyl serine. _ and lactosyl ceramid,z.
22. Phosphatidyl glycerol or cardiolipin is formed by the 27. When one molecule orNANA is attached to the
esterification of one molecule of glycerol simultane- ceramide oligosaccharide, it is called ganglioside.
ous! to 2 mole les of id. 28. Sulfatides are formed when sulfate is esterified to
23. Phospholipas A2 ydrolyses the ester bond bet- ceramide oligosaccharide.
ween the seco h droxyl group of glycerol and a 29. Cholesterol is an animal sterol which is a derived
PUFA . lipid. It is the precursor of all steroids in the body.

PART-1 : ESSAY AND SHORT NOTE QUESTIONS

8-1 . Classify lipids, giving examples.


8-2. What are phospholipids? Name their functions.

SHORT NOTE QUESTIONS


8-3. Rancidity. 8-5. Gangliosides .
8-4. Structure and function of lecithin. 8-6. Cerebrosides.

PART-2: MULTIPLE CHOICE QUESTIONS


8-1 . Which of the following is not a phospholipid? , 8-· . Which of the following lipids is not present in
A. Sphingomyelin -.)3. Cerebroside vegetable oils?
C. Cephalin D. Lecithin A. Cholesterol B. Linoleic acid
8-2. The structure of stearic acid is: C. Oleic acid D. Stearic acid
16 carbon, no double bond 8-8. Neutral fats are:
if!. 18 carbon, no double bond A. Act as surfactant and detergents
C. 18 carbon, two double bonds B. Soluble in water
D. 18 carbon, three double bonds C. Components of biomembranes
The structure of oleic acid can be described as: D. Oxidized to give energy
~ - 8-9. Saponification number of a fat molecule:
A. 16 carbon, one double bond
B. 18 carbon, one double bond A. Decreases with increase in molecular weight of fat
C. 18 carbon, two double bonds B. Increases with increase in molecular weight of fat
D. 18 carbon, three double bonds C. Decreases with increase in number of double bonds
8-4. The structure of linolenic acid is: D. Increases with increase in number of double bonds
A. 16 carbon, one double bond 8-10. The nitrogenous base present in lecithin is:
B. 18 carbon, one double bond A. Choline B. Ethanolamine
C. 18 carbon, two double bonds C. Serine D. Sphingosine
D. 18 carbon, three double bonds 8-11. All the following statements are true with regard to
The following are true with regard to linolenic acid, phospholipids, except:
except: A. They can exist as zwitterions
A. It is a component of lecithin B. They have surfactant properties
B. It is present in sunflower oil C. They are components of biomembranes
C. It can be synthesized by liver D. They are resistant to the action of enzymes
D. It has three double bonds 8-12. All the following alcohols are found in phospho-
8-6. The major fat in adipose tissue is: lipids, except
A. Phospholipid B. Cholesterol A. Sphingosine B. Inositol
C. Sphingolipids ,.o. Triacylglycerol C. Mannitol D. Glycerol
Chapter 8: Chemistry of Lipids 113

8-13. Sphingomyelin on hydrolysis yields all the follow- 8-17. Which one of the following is a dietary essential?
ing, except, A. Oleic acid B. Palmitic acid
A. Sphingosine B. Glucose C. Stearic acid D. Linolenic acid
C. Phosphate D. Choline 8-18. Which of the following lipids gives 2 fatty acids,
8-14. Cardiolipin: one molecule of glycerol and one molecule of
A. Contains Choline phosphoric acid on complete hydrolysis?
B. Is diphosphatidylglycerol A. Diacyl glycerol B. Phosphatidic acid
C. Is part of sphingolipid C. Lecithin D. Cephalin
D. Is found in endoplasmic reticulum 8-19. Which of the following fatty acids is present in
8-15. A ganglioside on hydrolysis gives all the following, surfactant?
except: A. Palmitic acid
A. Fatty acid B. Glycerol B. Stearic acid
C. Sphingosine D. N-acetylneuraminic acid C. Homogamma linolenic acid
8-16. Hydrolysis of fat by alkali is called: D. Arachidonic acid
A. Esterification 8-20. Which of the following is not an amphipathic mol-
B. Saponification ecule?
C. Mutarotation A. Sodium glycocholate B. Cholesterol ester
D. Alkylation C. Phosphatidylserine D. Potassium taurocholate

ANSWERS OF MULTIPLE CHOICE QUESTIONS


8-1. B 8-2. B 8-3. B 8-4. D 8-5. C 8-6. D 8-7. A
8-8. D 8-9. A 8-10. A 8-11. D 8-12. C 8-13. B 8-14. B
8-15. B 8-16. B 8-17. D 8-18. B 8-19. A 8-20. B

PART-3: VIVA VOCE QUESTIONS AND ANSWERS


8-1 . How are lipids classified? 8-13. What is saponification?
Simple, compound and derived. Hydrolysis of fat by alkali is called as saponification.
8-2. Classify fatty acids. 8-14. What is the significance of iodine value?
Depending on the total number of carbon atoms, they It gives the idea of how much double bonds are there
are classified as even chain and odd chain. in oils.
8-3. Which type is prevalent in human body? 8-15. What is Lecithin?
Even chain fatty acids. Phosphatidyl choline.
8-4. Name some saturated fatty acids. 8-16. What is phosphatidic acid?
Palmitic acid, Stearic acid. It is made up of one glycerol, two fatty acid residues
8-5. Name some unsaturated fatty acids. and a phosphoric acid.
Oleic, Linoleic, Linolenic and Arachidonic acids. 8-17. What is Cephalin?
8-6. Name some polyunsaturated fatty acids. Phosphatidyl ethanol amine.
Linoleic, Linolenic and Arachidonic acids. 8-18. What is Cardiolipin?
8-7. How many double bonds are there in arachidonic Diphosphatidylglycerol.
acid? 8-19. What nitrogenous base is present in lecithin?
4 double bonds. Choline.
8-8. Which contains good quantity of PUFA?
8-20. What is sphingomyelin?
Vegetable oils such as sunflower, ground nut oil.
Sphingosirie is attached to a fatty acid to form a cera-
8-9. Which contains very low level of PUFA?
mide. Ceramide with choline is sphingomyelin.
Animal fats.
8-21. Spingomyelin, on hydrolysis produces what?
8-10. Which fatty acids are common in human fat?
Sphingosine, fatty acid, phosphoric acid, choline.
Mainly Oleic acid; then comes Palmitic acid and
8-22. What is the special feature of sphingomyelin?
Linoleic acid.
Sphingomyelins are the only sphingolipid that contain
8-11. What is the difference between fats and oils?
phosphate and have no sugar moiety.
Fats are solids, oils are liquids at room temperature.
8-23. Cerebroside contains what?
8-12. What is the advantage of storing energy as triglyc-
Sphingosine, fatty acid. hexose.
erides in the body?
Space requirement is less as storage does not require 8-24. A ganglioside on hydrolysis, will give rise to?
water Sphingosine, fatty acid, N-acetylneuraminic acid.

SECTION B

General Metabolism

Chapter 9 Overview of Metabolism


Chapter 10 Metabolic Pathways of Glucose
Chapter 11 Regulation of Blood Glucose, Insulin and Diabetes Mellitus
Chapter 12 Metabolic Pathways of Other Carbohydrates
Chapter 13 Metabolism of Fatty Acids
Chapter 14 Cholesterol and Lipoproteins
Chapter 15 Hyperlipidemias and Cardiovascular Diseases
Chapter 16 MCFA, PUFA, Prostaglandins and Compound Lipids
Chapter 17 General Amino Acid Metabolism (Urea Cycle and One-Carbon Metabolism)
Chapter 18 Aliphatic Amino Acids
Chapter 19 Aromatic Amino Acids
Chapter 20 Citric Acid Cycle
Chapter 21 Biological Oxidation and Electron Transport Chain
Chapter 22 Heme Synthesis and Breakdown
Chapter 23 Hemoglobin
..
Overview of Metabolism

Chapter at a Glance
The learner will be able to answer questions on the following topics:
i O Study on six levels of organizations '?> 0 Metabolic profile of organs
2 0 Metabolic pathways and control mechanisms Jt O Metabolic adaptations during starvation

PERIMENTAL STUDY OF To the perfusion fluid any cowpound may be added and
the flu.!£i emerging from the organ is a~fyzed for the
METABOLISM
metabolftes of the compound.
The study of metabolic sequences may be conducted
at six levels of organizations, each at deeper levels Level 3: Organ Slices
of cellular architecture, and ~ - The next lower level of study is using the slices of
s ectives to the s organs, abou( so micrometer tha :\_9tto Warburg (Nobel
Prize 1931 ) was the first scientisrlo study metabolic path-
Level 1: Intact Organism ways using organ slices. (Thµa.strument f ~y of
The essential nature of amino acids and vitamins, etc. tissue respiration is known a~ ~ urg apparatus)} The
was studied by feeding animals with dietc; lackin i advantage of this procedure is thaTihe cellular organelles
one of the ingredients of food.~ 1842 Friedrich Wohl were preserved inta Metabolic transformations of nutri-
showed that benzoic acid when injected Is excreted as ents could be studied in detail. If rat liver slices are incu-
hippuric acid (b~ ~); this was the starting bated with medium containing glucose, carbon dioxide is
point of metabolic study in animal~ adiolabeled iron evolvedj
( 59Fe) is given, and incorporation of the radioactivity in

bone marrow and erythrocyte precursors are studied,


Level 4: Intact Cells and Tissue
which ~ovides information regarding the life span of Culture Set-up
RBCs_J he studies on inborn errors have been of great Tissues or cells can be kept in defined culture medium for
help In understanding normal processes inside the a few days for metabolic studies. The medium contains
body. It is easy to study individual enzyme systems in nucleotides, carbohydrates, amino acids, vitamins and
microorganisms.\}y utilizing mutant strains of bacteria, growth f tors. The pH of the medium should be kept
metabolic defects may be elucidatec£J arounc(r?.2. If labeled glucose is added in the culture,
the utttiza I0n of glucose and its incorporation into
Level 2: Organ Perfusion glycogen, etc. could be identifled. If labeled nucleo-
The organ can be isolated preserving its blood vessels. tides are added in the culture, cells take them up for
The organ is canulated and perfused with~ ~) DNA synthesis and the uptake of radioactivity will be
118 Section B: General Metabolism

pr~ ortjonal to 11!.,cell division. Activities 15


N-labeled glycine was followed by appearance of the
of drugs can be studied in cell culture label in different compounds like hemoproteins, nucleic
system. Biologically useful substances acids, and creatinine.
can be harvested from tissue culture
set-up. For example, specific monoclonal Studies on Metabolism
antibodies could be obtained from the
Alexis Carrel
Four aspects of metabolic pathways are studied:
supernatant of cultured hybridoma
NP 1912 a. Sequence of reactions
cells (see Chapter 44).
1873-1944 b. Precursor- product relationship
The cells from cancer tissues have r---:----,
c. Mechanism of reaction
indefinite capacity to grow into any num-
d. Control mechanisms.
ber of passages. This immortalization
Metabolic pathways may also be studied by creating
is characteristic of cancer tissues. A good
..-- perturbances to the system, such as:
example is the Hela cell line from cer-
vical cancer tissue, now growing in a. by causing metabolic- @
laboratories all over the world. This cell . d .h
F rte nc
b. by studying or anisms with metabolic
line was originally started in 1938 from Wohler ~ c . genetic ani ula , e.g. gene knock-out.
180 1882
a patient, Henrietta Lacks whose first (}- ;,:..;/
and last names were abbreviated to name the culture. A METABOLIS
obel Thousands of chemical reactions are taking place inside
Prize, 1912). a cell in an organized, well coordinated, and purposeful
manner; all these reactions are collectively called as
Level 5: Homogenates metabolism. Metabolism serves the following purposes:
The tissue is homogenized in an isotonic medium and 1. Chemical energy is obtained from the degradation
cell wall is broken by ultrasonic vibration and cellular of energy-rich nutrients.
organelle are separated. For example, isolated mitochon- 2. Food materials are converted into the building
drial preparation will show enzymes of electron transport block precursors of cellular macromolecules. These
chain . building blocks are later made into macromolecules,
such as proteins, nucleic acids, polysaccharides, etc.
Level 6A: Purified Enzymes Biomolecules required for specialized functions
Enzyme preparations may be used to study individual of the cell are synthesized .
metabolic reactions, their regulation, co-factors, etc. 3. Metabolic pathways are taking place with the help
of sequential enzyme systems. These pathways
Level 6B: DNA or Genomics are regulated at three levels:
a. Regulation through the action of allosteric enzy-
Present day research work mainly involves the studies
mes, which increase or decrease the activity
at genetic level (molecular biology). For example,
under the influence of effector molecules.
phenylketonuria is due to a mutation in the gene coding
b. Hormonal regulation. Hormones are chemical
for the enzyme phenylalanine hydroxylase. The full
messengers se,£reted b~ q___iff.ernnt endocrine
complement of genes within the cells (genomics), their
glands.
expression and regulation (transcriptomics) and the
c. Regulation at the DNA level; the concentration
@ ne products (proteomlcs) can be studied.
~ -L
axa: nU U r r L- of the enzyme is changed regulation at _the
level of synthesis of the enzyme.
Use of Radioisotope Tracers'
The isotope studies provide valuable information regard- ! Types of Metabolic Pathways .
ing precursor- product relationship, rate of metabolism A. Catabollc (degradation) pathways, where energy
and anatomical distribution (see Chapter 50). When rich complex macromolecules are degraded into
14c-labeled glucose is administered, the metabolites smaller molecules. Energy released during this pro-
can be traced to-- different organs. Administration of cess is trapped as chemical energy, usually as ATP.
I',

)q CC'--9-bJ"_' IJ"6'v ,.
J..l kc• 0
Chapter 9: Overview of Metabolism 119
j_q "'\ g 'f::_C
TABLE 9. 1: Energy reserves of man BOX 9.1: Energy ut1ilzat1on of average person
Weight Energy equ,valent The energy consumption varies based on lifestyle in adults.
Stored fuel (in gram) (in kilo calories) Approximately 300 g of carbohydrates (1,200 kcal), 7 ~ f
Glycogen in liver 70 280 proteins (294 kcal) and 80 g of fats (720 kcal) are consumed by a
person with a sedentary lifestyle. Therefore about 60% calories
Glycogen in muscle 120 480
are derived from carbohydrates, 15% from proteins and rest from
Glucose in body fluids 20 80 fats. The energy reserves provide energy in between meals aod
Fat In adipose tissue 15,000 135,000 after over;;i ht-fa.stin I co enol sis and luconeo ene ·s).

Protein in muscle 6.000 24,000


metabolism or Internal respiration or cellular
B. Anabolic (biosynthesis) pathways. The cells syn- respiration (see Fig. 21 .1).
thesize complex molecules from simple precursors. Metabolism of carbohydrate, lipids and amino acids
This needs energy. are interrelated and details are given in the next 9
C. Amphibolic pathways are seen at cross-roads of chapters of this book.
metabolism, where both anabolic and catabolic
pathways are linked. :::'.) .__E_TABOLIC PROFILE OF ORGANS
Metabolism is a sum of the following ~ jo ~ s The metabolic pattern or metabolic profile of different
of pathways:
organs is different depending on its function. Moreover,
1. Fuel oxidative pathwayl ood~ ~rials consumed
the organs are able to adapt to metabolic alterations in
are converted to energy ,C'
fej state-am:! starvation. The storage forms of fuels are
2. Fuel storage and mobilization path~ y: Stored fuel
shown in Table 9.1.
mobilized when there is no iriJ<lk§' J ood or during
Calories are stored in the body as faLand..gl¥cogen.
strenu ous exercise "?".''
The approximate percentage of storage form of energy
3. Biosynthetic pathway: For the sy~ @ ~is of proteins
(total fuel reserve) present in a normal human body is,
and other such macromolecule<..Q°ti~sic materials
~',,-
needed for biosynthetic pa hways include certain
fat 85%, glycogen 1%, and proteins 14%. Box 9.1 shows
the energy utilization of an average person.
amino acids, essential fattyacids and vitamins
4. Detoxification and waste 9i~gsaj pathway: For
-
Fat stores are rei•••••P:inly on
• even though adipose tissue fat is undergoing
removal of toxic waste p~lfCts-frofn the body.
turnover on a daily basis. Caloric homeostasis is _ ,
Additional pathways, wniCh..help-tnese essential path-

.
.. - - ' : - • ..-'l", •

ways are transport and intercellular signaling pathways. -w,


- - - - -- - . Similarly metabolic profile of
tt Stages or Phases of Metabolism various organs and tissues change to adapt to physiolog-
ical and pathological states, so that caloric homeostasis
The degradation of foodstuffs occurs in three stages.
is maintained unless extreme conditions set in.
i. In the first stage, digestion in the gastrointestinal
tract converts the macmmo!ecules iota swall 11oits 1. Brain
For example, proteins are digested to amino acids.
This is called primary metabolism. Although brain represents only 2% of adult body weight,
ii. Then these products are absorbed, catabolized it needs • • • • • • • •• About 750..ml.., of bloog_
to smaller components, and ultimately oxidized to circulates through the brain per minute. Neuronscan

l
CO2 • The reducing eauiv,ajents are mainly gener- survive only a few minutes without blood supply. Occlu-
ated in the mitochondria by the final common oxi- sion of blood supply to brain causes unconsciousness
dative pathway, citric acid cycle. In this process, within 10 seconds. There is no stored fuel in the brain
NADH or FADHQare qruierated. This is called sec-
• Glucose can freely enter the brain cells.
ondary or intermediary metabolism.
iii. Then these reduced equivalents enter into the elec- The total consumption of glucose by brain is about
tron transport chain (ETC, or Respiratory chain), 120 g/day (480 kcal). Thus, about 60% of the total car-
where e'.!.e;Y is released . This is the tertiary bohydrate intake by the body is metabolized by the
120 Section 8 : General Metabolism

Liver Brain
Fed state -+ Glucose-+ Brain

Starvation -+ Ketone bodies -+

Fig . 9.1: Metabolic alterations in brain


Glucose - -'---+

Food
Glycogen
l
Nil F a t - -- - -'-- .. Fatty acid

i
VLDL- ---

Adipose tissue

Fig. 9.2: Metabolism in well-fed state , \ ,,


Q,_,___ ~ C ( ) ~ C>U-'"c.

= Sb\'. (:j)f>~
.Skeletal Musel P-f'I = ~oJtt-.\-hry
The skeletal muscle forms about 4~% of the total weight
of the body. About OJ,% muscle weight is due to glycogen
Adipose
tissue
t
Gluconeogenesis ..---Alanine
content. Following a meal, the muscle glycogen content
increases by about 1% of the total weight.
Fig. 9.3: Metabolism in fasting state
A . Muscle metabolism after a meal: The uptake and
brain. Moreover, about 25% of the oxygen consumed storage of glucose by the skeletal muscle is under the
influence of insulin. Following__a_meaJ? the level of the
by the adult body is due to glucose oxidation in brain. In ·r
children, this may be as high as 50%. gl~a..and insulin ar~ . So glycogen synthesis is
enhanced (Fig. 9.2). The resting muscle uses fatty acids
A Brain under conditions of anoxia: In anoxia the rate as a ~,;:iinr fif IPII (Aai;50011n ') .
of lactate production by glycoly~is rises to 5 or 8 times . . .
withih one minute. The(!Jisfeur eff½s) (see Chapter 1o)1?>. Muscle metabolism . dl.lfmg exerc1~e_ : Muscle us~s
is the brain's protection against conditions of anoxia . glycogen for short active spurts of. a~ §lyco_,.9en is
e le ~ ~ ~ a l . rapidly broken down to form 1~ 1':fhe lactate has to be
transported to liver to undergo gluconeogenesiS-(f,?ri's
B, Brain ~nd acetoacetate: The _brain is unable ~o utilize cycle in Chapter 1O). Mracle tlowever u es,fatty ado as
fatty acids as a source of fuel since the fatty acids com- f f b. • d d" t
. . ue I or ae o Ic exercIs an IOQ Is ance running.
plex:d to album,~ are un; le.~o t rave~j~e e
th
bz:°d
~r; nc .Muscle metabolis; ; during starvation: During star-
b~ . Bu~, brain can 1 . vation, maximum glucose is spared for the brain. The
This Is again a
free fatty acjd (l;.fA) mobilized from adipose tissue
s.
is the>-p referred fuel for muscle during starvation, FFA
C Brain and starvation: During starvation, a significant does ~qttif-e-t~in, and during fasti ·~sp lin level •
part (
is low {Tobie 9.3). uring prolonged starvation.1., muscle
met by 0 r
!(Fig. 9.1). protein breakdown occurs and alanine is relea~ d to
Under conditions of partial anoxia , the production of the bloodstream. It is transported to liver to provide
ammonia is increased. -rfils is immediately trapped as substrate for gluconeogenesis (glucose-alanine cycle in
glutamine. The NH2 group of glutamine and glutamate Fig. 10.27). The metabolic fuel during prolonged fasting
can be used for synthesis of oth er amino acids (s~e is ketone bodies. Branched chain amino a1\in.r......i;,re
Chapter 18). utilized by the skeletal muscle (Fig. 9.3 and Table 9.2).
Chapter 9: Overview of Metabolism 121

TABLE 9.2: Adaptations during starvation


Skeletal Cardiac Adipose
Fed stote Skeletal muscle Cardiac muscle
Brain muscle muscle tissue
Preferred fuel Fatty acids FFA, ketone bodies,
After a meal Glucose Glucose, Glucose, Fatty acids;
at rest lactate
a Fatty acids pyruvate glucose
Exercise Glycogen to lactate Fatty acids
Fasting Glucose Fatty acids Fatty acids Fatty acids
Starvation Protein breakdown; release Fatty acids, branched (short-term)
Adaptations of amino acids; FFA, ketone chain amino acids Ketone Fatty acids;
Fasting Glucose; Ketone
bodies and branched chain and ketone bodies (long-term) ketone bodies; bodies ketone
amino acids utilized utilized bodies Branched bodies
chain aa
Exercise Glycogen Fatty acids
!.Adipose Tissue
It is the storehouse of energy in the body (about 135,000r C d" M I
. . J ar 1ac usc e
kcal) (see Table 9.1). The energy 1s stored in the
concentrated form, triacylglycerol. The chylomicrons Heart consumes more energy than any other organ. It
and VLDL are hydrolyzed by liJ2oprotein lipas~ present utilizes about 6 kg of ATP per day, 20-30 times of its own
on capillary walls. It is activ..ated_gy insulin. The fatty weight. Cardiac muscle derives its energy by oxidative
acids are re-esterified to form triacylglycerol (see Chap- metabolism of fatty acids (60-90%) and glucose 10-40%.
ter 13). The glycerol is derived from dihydroxyacetone Ketone bodies are also normally metabolized.
phosphate (DHAP), an intermediate of glycolysis. There- In addition, creatine phosphate is an important
fore, for storage of triacylglycerol, both fatty acid synthe- energy store in the muscles. The creatine kinase system
sis and glycolysis should operate. The uptake of glucose, acts as an energy buffer, by keeping ATP level constant.
glycolysis and lipogenesis are all favored by insulin. In a failing heart, the uptake and utilization of fatty
Table 9.3 shows the major metabolic fuels of different acids and glucose occurs. In advanced heart failure,
organs during various physiological conditions. insulin resistance also develops, further decreasing the
During fasting, triacylglycerols in the adipose tissue glucose utilization. At the same time, the metabolism of
are hydrolyzed. Cyclic AMP mediated activation of hor- a hypertrophied heart switches from fatty acid utilization
mone sensitive lipase occurs in response to the high to glucose.
glucagon-insulin ratio. Glucocorticoids also have a stim-
·ulant lipolytic effect during fasting. 1* Effect of Exercise on Metabolic Profile
Long-distance running is the typical example of aerobic
If. Liver exercise, where as sprinting or weight lifting exempli-
The liver plays a central role in metabolism by providing fies anaerobic exercise. During anaerobic exercise, the
adequate quantities of metabolic fuel for other organs. major organ involved is the skeletal muscle with very lilt.le
Almost all the metabolic pathways operate in the liver; a involvement of other organs. The relative ischernia__cre
notable exception being ketolysis. ated by the compression of blood v~ sels in the mu~ I~
A.Liver metabolism in fed state: Under well-fed condi- will necessitate the use of gJycogen and phasphocreatine
tions, the liver takes up glucose from circulation and available in the muscle to supply the required energy.
stores it as glycogen. Similarly the fatty acids syn- During moderate aerobic exercise, the muscular
thesized by the liver are incorporated into VLDL and stores of glycogen are used, but in a normal individual
secreted into bloodstream (Fig. 9,2). Liver is the major this is not sufficient to provide a continuous supply of
site of degradation of amino acids ~nd detoxification of ATP for exercise like long distance running . Muscles
ammonia into urea (see Chapter 17). During starvation, start ~ ing fatty acjd{ R;st after a vigorous muscular
liver provides glucose by glycogenolysis and later by activity often results in repletion of the exhausted glyco-
gluconeogenesis so that the obligatory requirements gen stores.
of the brain arE: met (Fig. 9.3). Moreover, liver also pro- In muscle developed by exercise and training, the
duces the ketone bodies, an alternate source of fuel. size and number of mitochondria are more as well as the
But the liver cannot use ketone bodies as its own fuel. level of enzymes for fatty acid oxidation and ketone body
122 Section B: General Metabolism

BOX 9.2: Long distance runners do not compete with sprinters''


Long-distance running is an example of aerobic exercise.
Metabolic profile of organs changes during aerobic exercise
with fa acids and ketone bodies bein the refer I for
the skeletal muse e. Because g ycogeno!ysjs js not sufficient to
meet the energy demands of prolonged aerobic exercise. a,
C)
C
Anaerobic exerci1e, on the other hand, has no effect on nl
.c
the metabolic profile of organs other than skeletal muscle. 0
a,
The skeletal muscle depends on its own glycogen stores and .;?
phosphocreatine to meet the demand for ATP. 1ii Blood glucose
ni
0::

utilization. Hence the trained muscle can better utjljze


non-carhabydcate sq, 1rces of ener9v. So exhaustion is
delayed (Box 9.2).
Liver glycogen

etabolic Adaptations during Starvatio Days 2


When blood glucose levels decline as in starvation, it
Fig. 9.4: Relative changes of important parameters during star-
is normalized by glycogenolysis and gluconeogenesis. vation
Once the glycogen is utilized, fatty acids, ketone bodies
and finally body proteins are utilized for ener~y production.lit Third Stage: Lipolysis
•1 First Stage: Glycogenolysis The prevailing state of high-glucagon-low-insulin ratio
,iitimulates cAMP-mediated lipolysis by increasing the
Table 9.2 and Figure 9.3 show the changes in activities
activity of hormone-sensitive lipase. Then skeletal mus-
during starvation. During early fasting, at first, blood
cle, heart and kidney will shut down their glucose utili-
\ glucose level is maintained by hepatic glycogenolysis.
zation; and will depend mainly on fatty acids for energy
The glycogen stores are ~ ~ r , e.bmJt 18 hours.
needs. The increased rate of lipolysis and beta oxidation
The primary requirement for glucose is to meet the
provides an alter~ate source of fuel as acetyl-CoA and
demands of the brain.
subsequently ketone bodies. Ketone bodies provide
ii Second Stage: Gluconeogenesis fuel for tissues like heart muscle, skeletal muscle and
to some extent the brain.
Even before the glycogen stores are depleted, gluco- W"
neogenesis is accelerated (Figs. 9.3 and 9.4). The amino i\l'Fourth Stage: Acidosis
acids released from muscle form the major substrate
for gluconeogenesis. The amino nitrogen is transferred Since ketone bodies are acidic in nature, their over-
from other amino acids to pyruvate to form alanine. Thus production and accumulation leads to acidosis. When
the amino group reaches the liver as alanine where it . the bicarbonate buffering capacity is exceeded, the
is transaminated to give pyruvate for gluconeogenesis. pH falls and hyperventilation occurs as a compensa-
This glucose-alanine cycle (see Fig. 10.27) serves to tory mechanism. Since glucose levels are comparatively
transport the amino nitrogen of other amino acids to low during prolonged starvation, secretion of insulin is
liver in a harmless form. Glutamic acid also serves as an reduced, but more glucagon is secreted. A fall in the
important mode of transport of amino acids to liver (see T3 (thyroid hormone) level leads to decrease in basal
Chapter 17). ~ergy requirements by 25%.
The branched chain amino acids liberated by "
• t b - . .
F"fth
1
Stage: Death f rom Starvat·10n
museIe prot em ca a o1ism especIa 11y 1eucIne and
isoleucine are utilized by the muscle to give energy. s·rain Metabolic acidosis and dehydration, unless corrected
can preferentially take up the glucogenic valine from the efficiently, will lead to death. A normal person has fuel
bloodstream. The plasma level of branched chain amino reserves to live up to 45-60 days. Examples are avail-
acids reaches a peak by 5th day of starvation. able in history. As part of the freedom struggle, Sri Jatin
Chapter 9: Overview of Metabolism 123
11\~
Das took a fast unto death, who died on the 61 st day of 16. Storage form of lipids is triacylgl}~OI which can be
his hunger strike on 13th September 1929. stored in unlimited quantities in adipose tissue.
17. In the fasting state energy is genmated by degrading
stored energy sources. Catabolism is enhanced .
LEARNING POINTS, CHAPTER 9
1a.( Bra\n 1s the f!lIDOLconsumer of ~~lucose since there
1. Studies using intact animals is now restricted to is no storage- foan. Brain is continuously at work.
procedures which do not result in any suffering or 19. A fall in glucose level can be detriimental to the brain.
sacrifice of animals. The animals used for experi- 20. In prolonged starvation, brain also uses keton~
mental purposes should be rehabilitated after the
bodies as fuel.
study. Study of intact cells has been made possible
21. @ r~ n cannot take up fatty acids flrom pia's~ to me~ta )
with tissue culture set up.
its energy demands, beca~ fatty ac,d-albumrn~
2. Studies using isotope tracers can be done in cells
complexEannot crassJfie_bioJ1caio..bar]ier.
obtained by tissue culture.
22. For short periods of muscular exercise the glycogen
3. Organ slices can be studied using organs which
is broken down to supply energy.
have been surgically removed. Study of intact cells
has been made possible with tissue culture set up. 23. Prolonged aerobic exercise uses fatty acids and
Tissue homogenates have been used to isolate oxidative metabolism of glucose.
cellular organelle and study their functions. 24. During starvation als muscle ses fatty acids and
-..;::,,-
ketone bodiec:<.~---

-
4. Purified enzymes and proteins may be used to
study i~ividual reactions and regulation _
5. Genomic studies are the main focus of current
- ~ ··
25/ Cardiac' my_scle..-al~ r . . . . a. c i ~
bodies as its preferred fuel under fed and fasting
research in medicine using a,en~knock out techni- state. N~ ~ ,OfheNJ
~es. 26. Cardiac muscle however has an <:,bligatory require-
6. Use of radioactive tracer is widely employed to ment of glucose which is continuously provided by
study the utilization and fate of compounds. the blood perfusing the heart.
7. Metabolic reactions generate chemical energy by 27. ~ iver is metabolically most active orqa~ Almost all
the degradation of compounds~..D metabolic pathways are active iin liver.
8. Precursors of cellular macromolecules are used for 28. During the fed state, liver stores glucose as glyco-
synthesis reactions gen. In the fed state, liver secrets TAG as VLDL
9. Amphibolic reactions 'arelti6'~e. where anabolic which can be taken up and utilized by peripheral
and catabolic reactions ar~i~ke'9 tissues (skeletal muscle and adiipose tissue).
10. All metabolic pathways are catalyzed by enzyme 29. After overnight fasting (8-10 hrs) the plasma
systems which are regulated by ~fferent Jffiha- glucose is maintained by glycogenolysis and gluco-
nisms. M~abolism has 3 phases;'primary, set'6nd- neogenesis.
ary an d
c.. ,ary.
.
30. 1:Jl' 24 houri of faiting. the glycogen stores are
11.(Erimary pb~ includes the digestion of macronutri- exhausted and fatty acids are mobilized from adi-
ents into small molecules which can be absorbed. pose tissue.
12.( § econdary pba~ includes all metabolic pathways 31. Muscle also produces alanine foir gluconeogenesis.
converging at citric acid cycle. 32. By third day of starvation, ketogenesis is active.
13C T.hjrd phasw ccurs at the mitochondrial electron Cardiac, skeletal muscle and aclipose tissue derive
transport chain where the reduced coenzymes are major fraction of energy from ke·tone bodies.
oxidized to generating energy as ATP. This is called 33. Substrate for gluconeogenesis is provided by gly-
oxidative hosphorylation. cerol part of fat and amino acid ireleased by muscle
14. Fed state represents a condition where sufficient protein breakdown.
energy is available. In this state, synthetic pathways 34.\ Ketone bodies start replacrn glucose as e ma r
predominate. source of energy for most tissues including brain.
15. ~ ~-oicarbohydrate is glycoS@J in liver 35. _However ac1;11m1rlation bodi9s in pro-
and~ longed starvation can lead ~s.,t~at[on ketosis.
_ _ _ _ _ Chapter 10
Metabolic
Pathways of Glucose

Chapter at a Glance
The learner wi ll be able to answer questions on the following topics:
)0 Digestion of carbohydrates O Glycogenesis; glycogen synthesis
lo Absorption of glucose and glucose transporters
0 Regulation of glycogen; Cyclic AMP
9, { D Glycolysis pathway and its regulation O Glycogen storage diseases
Energy yield from glycolysis D Hexose monophosphate shunt pathway
Cori's cycle
Glucose-6-phosphate dehydrogenase deficiency
D Pyruvate as a junction point
0 Glucuronic acid pathway
D Gluconeogenesis and malate shuttle
Essential pentosuria
0
D Glucose alanine cycle Polyol pathway
D Glycogenolysis; degradation of glycogen

,l,..a..9..~\t,°t".A
Historical Perspectives ~=~----
__~.,.n.., "::V , v_y "' F ' ''
Abbate Spallanzani (1768) showed that living tissues take up oxygen and give off carbon dioxide. In 18130, Louis Pastu ur
demonstrated the fermentation process o.f..glf.f,9_Se to alcohol by yeast. ,sck..(.1882) indicated that chemicail energy 1n mus le
is converted to contraction of muscle. lnlf~~ ustav G.eQ[ge..,Embden (1 874;1933) studied the lactic a1cid formation from
pyruvate. In 1919, Otto Fritz Meyerhof (1884-1951) enunciated most of the steps of the glycolytic pathway (Nobel Prize, 1922).
Hexokioase eozyro<> was first identified by v.2,n El.l.leL.Cllelpin in 1915 (Nobel Prize, 1929). Other enzym1es in the glycolyt~c
pathway were then identified rapidly; pyruvate decacbaxyla!'le by Neuberg in 1911 , pb.Qsphofructokinase by ~uu:iru:.der
in 1920 (Nobel Prize, 1929); phosphohexo:;e isomerase by L~hmann in 1933; p_yruvate kinase by P,amas..lJ:1-1934.;.enolase by
_l'{eyerhof jn 1!U,5; phosphoglucomutase by l.,tlloir in 1938 (Nobel Prize, 1970); ceraldeh de hosphate deh drogenas~ by
ti,!_~ arburg in 193.9~hosphog!yceromutase,.by 9.uthedand in 194~_(Nobel Prize, 1971 ). Between 1935 an 1H43, all enzymes of
~~'cl gTycolytic pathway were crystallized and characterized by Warburg. He was awarded Nobel Prize in 1931 for his earl ier work
i"l"~~ on cellu lar respiration. He was awarded Nobel Prize for a second time in 1944 for his contributiQ.{ls in glyccilysjs; but Hitler did
"'I( "\:, not permit him to receive it! P~ as was murdered by Stalinistic regime. '

tq\ --- --------·- -- --- acid will inhibit t~ action o f salivary amylase. In the
\.:1/ pancreatic juice another c1lpba-amx~ is available,
~liydr tes re presen S\-C~
In the d i e trb which will hydrolyze the alpha-1,4 gl1ycosidic linkages
~ari.des (s ch, ~ c gen)t.and to a minor extent, randoml~ so as to pr9duce smaller subunits like malt-
as"~ c and flk tose). They arewruo- ose, isomaltose, dexfrins and ~hed or..lL~<i~r-3-
~lO moOQ~ t'nm;units in the gastrointestinal olig&13'ccharides. The eel~ of rus ~ ~ s \ m e
_!Jr' ·tract. Cooking makes the digestion process easier. The contai~the enzymes, sJease, tase, iso~ltase
process of digestion starts in mouth by the salivary and laefase, They hydrolyze the corresponding disac-
/? ~ mylase. However, the time available for digestion charides into component monosacch,arides, which are
mouth is limited, because the gastric hydrochloric then absorbed. b
_,,,. rOll~
l.On$u.:n-,f1'l!W\ tr~~ f"Y\~ - S~ 'J'n~ nh1~ .,. .fibS?.o~f
Chapter 10: Metabolic Pathways of Glucose 127

RBC, brain, kidney,


Properties
Glucose uptake in most of
"
colon, retina, placenta cells
Serosal surface of Low affinity; glucose uptake
intestinal cells, liver, in liver; glucose sensor in
beta cells of pancreas beta cells
Neurons, brain High affinity; glucose into Cytosol
brain cells

Fig. 10.1: SGluT. Sodium and glucose'c\i- r


luminal side; sodium is then pumped out
uctose transporter; poor
ability to transport glucose
Glucose from ER to Absorption of Glucose
cytoplasm
Intestine, kidney Cotransport; from lumen
Glucose has s~cific transpactecs. which are tral]S·
into cell membr@e pr~teins. Table 10.1 shows a summary of the
glucose transporters.

Clinical Application; Lactose Intolerance


Co-transport from Lumen
Lactase hydrolyzes lactose to glucose andmfflac ose .
. "' to Intestinal Cell '
Lactase is present in the brush bomer of .(en er c es. C)l.,.)f'lt,..
Deficiency of lactase leads to lacTose intolerance. In This~f~~\_Jsle [l~iated by S.Q.diwn Dependent
this condition, lactose accumulates in the gut. !.!ril,filJt Glucose Yransporter-1'sGluT-1) (Fig.10.1). Absorp-
dj;mbea apd f)at11leoce are seen. The causes may tion from i~stinal lume~ntestinal cell is by
be congenital or acquired. As 8 ge advances, lactase co-transport mechanism (secondary active transport) -Ki.& tp.k,
.:. activity decreases and seco~ry lactose jntolera~e (see Chapter 2). A membrane bound carrier protein} ~
occurs. Acquired lactose intolerance can also occur carries glucose, along with sodium. This sodium is later eJ'I~
'4hen there is a sudden change to a milk based diet. expelled by the sodium pump with utilization of energy.
Lactase is an inducible en~ milk. is withdrawn So energy is nee~~ ectly (details in Cha ter 2). ~"""'
temporarily, di~ cbea will He: limited.(Curg'.fS also an The transporter in in stin~1, ,ed GluT and a.A\-€\
e ective treatment, because the lactobacilli present in the transporter in the S uT-2. The first
curd contains the enzyme lactase. Lactase is abundantly 1
one is involved in ghJcose:9ajac m !absorption. TheW~ ~
seen in ~east, which could also be used in treatment.
~ est~ congenital renal olvcosyria.
@clinical application: Common treatment for diarrhea
ABSORPTION OE
is oral rehydration fluid. It contains gl.!:!£Q_§e and °'3 "'·
CARBOHYDRATES sodium.,, sence of sodium and glucose gether in
oral rehydrati fluid allows absorption sodium to
·l:iHJ__cl}!Q!:ige_Jeyel nd glucose to

b\a}O-C ·

Lazzaro Louis Arthur von Euler- Otto Warburg Karl Luis Le/oir
Spallanzani Pasteur Harden Chelpin NP 1931 Lohmann NP 1970
1729-1799 1822-1895 NP 1929 NP 1929 1883-1970 1898-1978 1906-1987
1865-1940 1873-1964
, ;

...
128 Section B:

Glucose in intestinal cells Glucose outside the cell

1 Transporter changes direction .\


• L\\1€..,'1,
· KlO-'i\
•yo.no
1
.\tj'I'
\'l\C~' Q..'- Glucose ig bloodstrel(Tl •
a\l>-· ~t:---- -- - - - - - - .......;;..__ _
1,c1:
___J

Fig. 10.2: Glucose absorption (GluT2) Fig. 10.3: GluT4. Glucose transport in cells
r.~"
Another Uniport System C~ e , GLUCOSE METABOLISI\I
Releases Glucose into Blood tD £'GLUi)
Clinical Importance of Glucose
The same intestinal epithelial cells have a different
transport mechanism on the membrane facing capil- Glucose is the ~ e~ source of energy for most of the
laries (Fig. 10.2). lntestjnal cells__releas_e_gl.ucose.Jnto body tissues. Brain eels derive the energy mainly from
bloodstream by the carrier mechanism called Glucose glucose. When the glucose metabolism is deranged, life-
Transporter Type 2 (GluT2). This transporter is not threatening conditions may occur. A minimum amount
dependent on sodium. It is a uniport, facilitated of glucose is always required for normal functioning.
diffusion system (Fig. 10.2). GluT2 is involved in abs- Normal fasting plasma glucose level is 70 to 110 di.
orption of glucose from bloodstream to cells. GluT2 is
After a heavy carbohydrate meal, in a normal person,
present in intestinal epithelial cells, liver cells, beta
.____, . . -cells
-- this level i{ggow 150 mi@)
of pancreas and ~ Y- GluT2 e11ables the pancreas to
monitor the glucose level and adjust the rate of insuli._ ._ _
secretion (see Chapter 11). LYCOLYSIS (EMBDEN•
MEYERHOF.J>ARNAS PATHWAY)
Glucose Transporter 4
GluT4 is the major glucose transporter in skeleta efinition : In glycolytic pathway glucose is converted
muscle and adipose tissue (Fig. 10.3). GluT4 is under to pyruvate (aerobic condition) or lactate (anaerobic
the c~*cal ot jow)in. But otbec glllcase transporters are condition), along _wi~h pro~uction of a small quantity of
n un the trol of instffin. energy~ Glycol~s1s Is ~e:ved. f~om the Greek words,
Clinical applic~ :J.ns_u i promotes the translocation glykys - sweet, and lys1s - splitting.
of intracellular molecules to the cell surface an@ Site of reactions: All the reaction steps take place in:
thus increases gluc!,)se uptake. I~ Type 2 diabetes • the cytoplasm.
mell_itus (s~~ ~ a'8\rr 11 ~.EP~~ GluI4 is.reduced,
leading to insulin res ancelnn:iys.cl~s. In ignificance of Glycolysis Pathway
diabetes, entry of glucose into muscle is only half of
normal cells. GlyI5Js..thafructose.transporter. It is the only pathway that is t¾ing place in all the cells
of the body. Glycolysis is _the ~ ly source of energy in
Absorption t f other~ nosaccharides erythrocytes. In strenuous e~ercise, when muscle tissue
lacks enough oxygen, a~aero~ glycolysis forms the
9'. I \' Gluc are absorbed by the same
tl transporter, SGI It Is an energy dependent process, major source of energy for rn'Qscles.,.Glycolysis is the •
it,~1._, against a concentration gradient, and therefore absorp- preliminary step before complete oxida~'!n. The glycolytic
-y;:/ tion is almost complete from the intestine ...Q~ mono- . path"Y£ Y p~des carbon skeleti~ for:.sy~ thesis of~
.1- , .. [saccharides are absorbed by t arrier mediated facilitated esserftral ~ o a~ids as well as ~ 19cetii par1 of fat. Most
~ - transport. Therefore, absoro1LOD is not complete, and the of the reactions of the-gty~olytic pathway are reyersible,
~ cJremaining molecules in t,he intestine will be fermented which are also usec:NoFO-fcrconeogenesis. A summary of
,.,, \ by bacteria. ( Ot.L\."<~~ , the pathway is shown in Figure 10.4.
I.> \ \C, 0{\\1),.1
~ · w»tt- fo ~- ~om<-- kou.Ul
er
:p-.a.~ '"~ g chi£~.,.,_~ 0-\v "' u 'YE°9t . ~)\,_-tr
·• b.'~-2 lr(J~O'\¼ jn~uh'°f'. T ~ ttot''.~
tc'~ I..!. " L ve_q_
Chapter 10: Metabolic Pathways of Glucose 129

_-o Q
Glucose
e
(1) Hexok nase
•1-
c.,&A"r;;I
V- ATP
,o-r-o-c
757
II H2

\ ..~°:>~ Wit-ADP 4
Glucose-6-phosphate HH : HH
(2) Phosphohexose
H
t=.
isomerase 2 OH
Fructose-6-phosphate H OH H H

krn~e
e, I
J V r-·~
(3) Phosphofructo

.l, ""'
P,
.i(.C>''\
ROS
ADP
ATP

A'l l 0 Glucose
~ATP
\,____Mg+/
ADP

- - - ---'=--""'-'--- -+• Glucose-6-phosphate


' ·.;.. 1~ F'ructose-1 , 6-bisphosphate
'f','("-"'.
7'<..P Hexokinase

(4) ~ e
1
~~,. j1 PIP~pAl Step 1 of glycolysis; irreversible step

Glyceraldehxde-3-phosphate + DHAP\ Ol<\~ . •


(5) Glyceraldehydef ~
3-phosphate
V l t = . NAD•
Pi
---,J-:~ 9p\1
dehydro~ NADH+H+ 1>
1,3-Bisphosphoglycerate (_rto~
(6) 1,3-bispho~ ( 1,~ ADP
glycerate ~ - - -+l~~JP Otto Meyerhof Jakub
Gustav Georg
NP 1922 Embden Parnas
(7) Phospho-
glyceromutase
..r
i1 CS-?G
3-Phosphoglycerate
1884-1951 1874-1933
'
1884-1949
....

(8) Enola~ , ~ .J. i1~;,


2-phosphoglycerate

Occurrence
Km value
Phosphoenol pyruvate
Affinity to
substrate
Specificity Acts on qlucose, Acts only on glucose
tr.uc.lgse and
l~
Induction Not induced Induced by insulin and glucose
Function Even when blood Acts only when blood glucose
Fig . 10.4: Summary of glycolysis (Embden-Meyerhof - Parnas sugar level is low, le·1el is more than 100 mg/dl ;
pathway). Steps 1, 3 and 9 are key enzyems; these reactions are glucose is utilized then glucose is taken up by liver
irreversible. Steps 6 and 9 produce energy. Steps 5 and 10 are by body cells cells for glycogen synthesis
coupled for regeneration of NAO·

lucose Entry into Cells_ .. ~f'\ enz me. ~e kinase rep tio,il is · le. But this
.,,~ irreversibil" is c~ vented by another enzyme
Glucose transporter-4 (GluT · transports glucose from ,"•glucose-6 phosphatase (see gluconeogenesis).
the extracellular fluid to muscle cells and adipocytes (see Hexokinase and glucokinase may be considered
Table 10.1 ). This translocase is under the influence 9f as isoenzymes; their properties are compared in Table
i~ulir;i. In diabetes mellitus, insulin deficiency hinders the 10.2. Glue.okioase js under the influence of io.su~
entry of glucose into the peripheral cells. But Glu ·s the but hexokinase is not. Hexokinase is present in most
transporter in liver cells; it is not under the control of· sulin. tissues. Glucokinase is present in and ~ ells.
No1l .-~ ~Glucokinase is induced by insulin. The metabolic fates
tep 1 of Glycolysis of glucose-6-phosphate are shown in Figure 10.6. The
phosphorylation of glucose tra s it within the .c_eUs.
Glucose is phosphorylated to glucose-6-phosphate (Fig.
10.5). The enzyme is Hexoklnase (HK), which splits th i tep 2 of Glycolysis
ATP into ADP, and the Pi is added on to the glucose. Glucose-6-phosphate is isomerized to fructose-6-phos-
The energy released by the hydrolysis of ATP is utilized phate by phosphohexose isomera_Je. This is readily
for the forward reaction. He..xokioase is a ke lycol ic reversible. "
.::---
130 Section B: General Metabolism

Glycolysis to pyr~vate o
-/:Y-f-
E
Glucose ---+ Glucose-6-phosphate Glucose ( • ' ' - C
~ H2 0 CH2OH
Glycogen ".\,v l J'.l"l -o
'---- - -- - -- -- - - -- S
....hc...iup.
npt p°"a'-"
th+w+art-
y -.r-i.----J HH HO OH
Fig. 10.6 : Fate of glucose-6-phosphate

gr•
: ATP I ADP
BOX 10.1: D,phosphate and b,sphosphatc r1re different
When t~ phosph~te are I"
attached to a parent compound, it is called i
r and then
¥-- phosphate \ Mg++ __ t ....,~-Fructose-1 ,6-
Fructose-6- ---~......:::::- L.:::_
Phosphofructoklnase bisphosphate
adenosine-di-phosphate (see Fig. 5.3).
But wher("fil5ospbodc acid~ are present at ~~felf:nL
sites of the compound, it is named as bisphosphate,
- tructose-1,6-bisphos hate (Fig. 10.7).
CH 2OPO3
= H-C=O
I Triose ~ osphate
isomerase
I
C=O H-C-OH
Dihydroxyacetone phosphate
CH 2OPO 3 I I -
CH OPO -

)-r.~
I
: CH2-OH 2 3
CH20Po
®=
/ Site or cleavage I
C=O
~e.,•
t<"I'
Oil\ydroxyacetone phosphate Glyceraldehyde-3-phosphate

I
CH -0H
Fig. 10.9: Step 4-A: isomerization; reversible
/ 2
H- Cl·OH tep 4-A of G/yco/ysi
c~ H- C=O
H-C-OH Ptt~ \ Dihydroxyacetone phosphate is isomerized to glyceral-
I =
H- 6.0 H~
dehyde-3-phosphate by the enzyme ph~s~hcµriose
CH OPO
2 3 I -
CH2 OPO; isomerase. Thus net result is that glucose Is now
Fructose-1,6-bisphosphate Glyceraldehyde-3-P
cleaved int°'-2Tl]Olecules of glyceraldehyd~phate
Fig. 10.8: Step 4 of glycolysis; reversible (Fig. 10.9). The steps 4 and 4-A are together called the
SpJ itting Pbu i>. '("Cy°'-\
tep 3 of Glycolysis ( ~"mO~'(\j f~ Glycerol portion o th~ ral t can enter into
· l 1 glycolytic or gluconeogenic pathways at this point.
Fructose-6-phosphate is further phosphorylated to fruc-
tose1 ,6-bisphosphate (Fig. 10.7 and Box 10.1). Th tep 5 of Glycolysi$
.., enzyme is phosphofructoklnase (PFK). It is the e.Jim1- •1
Glyceraldehyde-3-phosphate is dehydrogenated and
~\q... mig enzyme of glycolys1s) t->FK is an all~feric, in~ ible,
simultaneously ribosphorylated to 1,3-bisphosphogly-
re¢atory enzyme. It is an important key enzyme of this
cerate (1,3-BPG) with the help of NAO• (Fig. 10.10).
pathway. This is again an activation process, and t!}e
The enzyme is glyceraldehyde-3-phosphate dehydro-
energy is derived from ATP. This reaction is an irreversi
genase. The product contains a high energy bond. This
ble step in glycolysis. However duri~ gluconeogenesis,
is a ~eversible ceac!ion. During this reaction, NAO• is
this difficulty is circumvented by fructose-1 ,6-bisphos- reduced to NADH.
phatase. The steps 1, 2 and 3 together are called as the
preparatory phase. 1° - tep 6 of Glycolysis'
Step 4 of G/ycolysis The energy of 1,3-BPG is trapped to synthesize one
V ,.. ATP molecule with the help of bisphosphogl cer te
One molecule of fructose-1 ,6-bisphosphate (6 carbons) Ki' e (Fig. "'I0'. 11 ). This is an example o level
is cleaved into two molecules of 3 carbon units; glyceral- iJ;;.iiihorylatloa(, where energy is trap()ed direct))l.fiom
dehyde-3-phosphate and dihydroxyacetone phosphate the substrate ~ he help of the c;'mplicated electron
(DHAP) (Fig. 10.8). Since the backward reaction is an transport chain reactions. When energy is trapped by
aldol condensation, the enzyme is called aldolase. This oxidation of reducing equivalents such as NADH, it is
reaction is reversible. called oxidative phosphorylation. Step 6 is reversible.
Chapter 10: Metabolic Pathways of Glucose 131
r- 1.

CHO Glyceraldehyde-3• COO--Po; 6'~~ coo-


I phosphate dehydrogenase I Bisphosphoglycerate kinase I
CH- OH
I 4 7+ Pi\ • ,H-OH -
4
fig+~
• CH- OH
I _
CH2OPO3- NAO+ NADH+H+ CH2OPO3- J + CH2OPO3-

Glyceraldehyde-3-phosphate {1;3-bisJ)hosphogl~ 1,3-bisphosphoglycerate ADP IATP I 3-phosphoglycerate

Fig. 10.10: Step 5 of glycolysis. Reversible step1 NADH gene- Fig. 10.11: Step 6 of glycolysis. Reversible reaction. ATP genera-
rating step I , -
1 tion step
(.,.C,, .

Phosphoglycero-
coo- mutase COO- Enolase coo-

l l
C=O
CH-OH l B-o-Po~ ~ •
I I
I -
CH OPO -
CH2.loHl
HO
2 CH 3
2 3 Phosphoenol pyruvate (PEP) Pyruvate
3-phosphoglycerate 2-phosphoglycerate Phosphoenol pyruvate

Fig. 10.12: Steps 7 and 8 of glycolysis Fig. 10.13: Step 9. ATP production (irreversible}

spontaneously isomerized int ~~::;=:=:::'J1.the stable


Lactate dehydrogenase
coo form of pyruvate. One mole of
1
this reaction. This is again an example of substrate
level phosphorylation (Fig. 10.13). The pyruvate kinase
CH3 CH3
Pyruvate Lactate

Fig. 10.14: Step 10; LOH reaction; reversbile two enzymes (pyruvate carboxylase and phosphoenol-
;. uvate carboxykinase) (see gluconeogenesis).

p 10 of Glycolysis H:! d
3-phosphoglycerate is isomerized to 2-phosphoglycerate
In anaerobic condition, ·
by shifting the phosphate group from 3rd to 2nd carbon
by tg te dehydro~ (LOH) (Fig. 10.14). (Greek; ·.
atom (Fig. 10.12). The enzyme is phosphoglyceromu-
an=not; aer=air; bios=life). LOH has 5 isoenzymes. A
tase This is a readily reversible reaction.
p summary of glycolysis is shown in Figure 10.4.
8 of Glycolysis In aerobic conditio , e pyruvate enters~ e~
..acid.cyc~ (or complete oxidation. The end product of ana-
2-phosphoglycerate is converted to phosphoenol pyru- erobic glycolysis i~ ~ Cori's cycl~
vate by the enzyme enolase. One water molecule is
removed (Fig . 10.12). A high energy phosphate bond ignificarice of Lactate Production "\
· duced. The reaction is reversible. En ·res
Steps 5 and 10 are Coupled i q Q_~Ll J
and by removing magnesium ion f uond ill
ersibly inhibit this enzyme. Thus, fluor stop )J.Jcose two mole-
the whole glycolysis. So whe~ collecting blood for sugar cules o '::: t{;A::;O!:.•:...a
:.::r:.:..e~re:.:::d:.:.u..:.
c:.:;
e~ = ~~ .1} The availability
estimation, fl~oride is added to blood. If no ,""glucose of co~ n mes inside a ce)J is limited. Therefore, this step . ,p-;
is metabolized by the blood cells, "so that lower blood becomes a bottleneck in the whole ~ e.
sugar values are obtained . This NADH is to be reconverted to NAO•. This can
\ be done by oxidative phosphorylation. However, during
p 9 of Glycolysis exercise, there is lack of oxygen. So, this reconversion
Phosphoenol pyruvate (PEP) is dephDsphorylated is not possible. Therefore, the cell has to couple some
to pyruvate, by pyruvate kinase. First PEP is made other reaction in which NAO• is regenerated in the
into a transient intermediary of enol pyruvate; which is cytoplasm itself. Hence pyruvate is reduced to lactate;
t' Cle,,..""vbZc 0J lJC!lhf;~ 1 "'7\vt2 -00 "'6" eo.ctt.un '"'hr t ~,z;tl.f
tlf''°"\'+-1-,• do.,\ve..0- 1)'°~ bo
cy c.\.e
fe,.. ,iurl:ow,\1-.oul
_-ti> NFi\J~, 1t'J-J.-1.
bv,r-.q
132 Section B: General Metabolism 1M~~ ~vi:~ N ~D

Glyceraldr:=ehyde-3-P NAO+ TABLE 10.4 : Energy yield (number of ATP generated)

7
per molecule of glucose in the glycolytic pathway, under
Gly3PDH anaerobic conditions (Oxygen deficiency)
(Ste p 5)
NADH
!
No. ofATPs gained
1,3-bisphospho- Step Enzyme Source per glucose mo/

*
glycerate +
1 Hexokinase Minus 1

L
NADH NAO @

!
3 Phosphofructokinase Minus 1
6 1,3-bisphosphoglycerate kinase ATP 1x 2 = 2
\ ,LOH Lactate
Pyruvate
t 1
(S ep O) n \ : "" 9 Pyruvate ki nase ATP 1X2 = 2
" e~,en~""""tl'\ Total = 4 minus 2 = 2
Blocked in lack of oxygen 9\_ N I)
Acetyl CoA t) cle, BOX 10.2: Chicken 1s white. but duck 1s red 11
Fig. 10.15: Lactate formation is necessary for .~ f Actively contracting muscles that rapidly consume ATP can also
Nf .OH lo [)18.D:.d,_uring anaer®,iasis. · regenerate ATP entirely by anaerobic glycolysis. Fast twitch white
muscle fibers have very few mitochondria and predominate in
muscles capable of short bursts of activity (sprints). Slow twitch
red muscle fibers rich in mitochondria are found in muscles that
(1) contract slow ly and steadily (distance runners). Chicken mes
~---1--------+----=-'---_;;;:=.::..<---~---l only short bursts and has white fibers. Ducks on t he other hand
GK Insulin
are migratory and have red fibers suitable for sustained activity.
G) PK Insulin, Fl ,6-BP ) '------''----'---- - - - - - - - - - - - - - - = - -___,
(;;'l f---t-....:..._c_---+-'----'--'-'---"--.:........:........:....::L.::.:..;:____~~lR
PFK Insulin, AMP. F-6-P, Glucagon, ATP, citrate, low pH, cAMP
F-2,6-BP TABLE 10.S: Energy yield (number of ATP generated) per
Ji : U~e J e r\r-.r,1:-u<?.9-'~ rrr,1..,,~t'"""· • molecule of glucose in the glycolyttc pathway. under aerobic
G-) n ~rr,-6'"' ' " ' ~ . \ . (' I'\ :i:. -~ ,~, conditions (oxygen is available)
the NAO• thus generated is reutilized for uninterrupted!' J!!IIII No. ofATPs gained
operat~ the 5th step (Fig. 10.15). Step Enzyme Source per glucose mot
In~ . there are r:io mitochondria . Hence RBCs 1 Hexokinase Minus 1

derive energy ® lY-tl'l~fugb - g ~ . where the end- 3 Phospho fructokinase Minus 1

P~ - =- 5 Glyceraldehyde-3- phosphate
dehydrogenase
NADH 2.5 x 2 = 5

6 1,3-bisphosphoglycerate kinase ATP 1x 2= 2


C Energy Yield from Glycolysis in
9 Pyruvate kinase ATP 1 X 2 =. 2
Anaerobic Conditions Total = 9 minus 2 = 7

During anaerobic (oxygen deficient) condition, when


one molecule of glucose is converted to 2 molecules provides~ this reaction generates 2.5 x 2 = 5
of lactate, there is a net yield of 2 molecules of AIE. ATPs. Th~ oxygen is avai~ le, the net gain of
During steps 6 and 9, four ATPs are synthesized . But 2 energy from the glycolysis pathway is 7 ATPs (Table
molecules of ATP are used in the steps 1 and 3, hence 10.5). Hence the ATP yield from glycolysis is different in
the net yield Is only 2 ATP (Table 10.1 The whole anaerobic and aerobic conditions (compare Tables 10.4
reaction is summarized as - .2.. Q.+ J. and 10.5). In aerobic conditions, pyruvate is converted
@ ucose + 2 Pi + 2 2 Lactate + 2 ATP to acetyl-CoA which enters the TCA cycle for complete
See Box 10.2 for biological significance of the oxidation. Complete oxidation of glucose through
anaerobic glycolysis. glycolysis plus citric acid cycle will yield a net 32 ATPs
(Table 10.6).
P Energy Yield from Glycolysis Note: Till a few years back, c~ulations were made
in Aerobic Conditions assuming that NADH and .l;&Q_H
ge.!l..eJates.2A;i:es. i I amount to a net generation of
When oxygen Is In ~ . the two NADH mole_ci.tles, 38 ATP per glucose u e. Recent experiments show
generated in step 5, canenter the mitocbQndrial electron that these.~ ~_t:;;~~!:!~~!~~~~~ nd net generation is
transport chain for complete oxidation. As each NADH only 32 ATPs.
Chapter 10: Metabolic Pathways of Glucose 133

TABLE 10.6: Energy yield (number of ATP generated) per molecule of glucose when it is completely oxidized through glycolysis plus
citric acid cycle, under aerobic conditions
No ofATPs gained
Method of ATP No.of ATPs as per old
Pathway Step Enzyme formation calculation
Glycolysis 1 Hexokinase Minus 1 Minus 1
Do 3 Phosphofructokinase Minus 1 Minus 1
Do 5 Glyceraldehyde•3·P DH NADH Respiratory chain 2.5 X 2 = 5 3 x 2=6
Do 6 1,3·BPG kinase ATP Substrate level 1 x2=2 l x2= 2
Do 9 Pyruvate kinase ATP Substrate level 1 x 2 =2 1X2=2
Pyruvate to acetyl•CoA . Pyruvate dehydrogenase NADH Respiratory chain 2.5 X 2 = 5 3x2=6
TCA cycle 3 lsocitrate DH NADH Respiratory chain 2.5 X 2 = 5 3 x 2=6
Do 4 alpha ketoglutarate DH NADH Respiratory chain 2.5 X 2 = 5 3X2=6
Do 5 Succinate thiokinase GTP Substrate level I x 2 =2 I X2=2
Do 6 Succinate DH FADH2 Respiratory chain 1.5 X 2 = 3 2x2 =4
Do 8 Malate DH NADH Respiratory chain 2.5 X 2 = 5 3 x 2=6
Net generation in glycolytic pathway 9 minus 2 = 7 10 minus 2 = 8
Generation in pyruvate dehydrogenation =S =6
Generation in citric acid cycle =20 =24
Net generation of ATP from one glucose mol =32 = 38

Nore: Previously, calculations were made assuming that in the electron transport chain, NADH produces 3 ATPs and FADH2 generates 2 ATPs.This will amount
net generation of 38 ATP per glucose molecule. Recent experiments show that these old values are wrong. Please also see Chapter 21 for details.
e
egulation of Glycolysis Glucose
-::1
The regulation is summarized in Figure 10.16. The '········· G6P
regulatory enzymes or key enzymes of glycolysis are:
1. ~ xokinase, step 1 (glucokinase, in liver)
2. Phosphofructokinase, step 3 (Table 10.3)
Glucocorti·
coides -
-
-
l
H
F6P _ . F2,6BP
+ ........ .J...... Insulin
........ .:
..............ADP
Pyruvate kinase, step 9. : FBP · ····················,
i t! ·············~ :
ctors Regulating Glycolysis I
:
G3P
u
+- DHAP i
:
: BPG :
Hexokinase having a hi_g_h affinity for glucose will act
even at low glucose concentrations. $0, brain and RBCs
~ . )lf
3PG
i:
can get necessary energy. Glucose-6-phosphate has a i: H
2PG
i
i
feedback inhibitory effect on the enzyme. G,.!Ygokinase
i H !
with a loi affinity a~ higll..Km-tor..g!1.1GGS0-is-;::>ffiSel:lt
o; ly irfTf, r and bet~ ells of pancreas. These are the $ , ···········•·····---·• _
PlEP
r . .. . ........ ADP
tissues where glucose is available in plenty. In the liver ''' ''
'
glucokinase phosphorylates glucose which can be used
for glycogen synthesis. l~
the glue · ·
eta cells of pancre~
·n to
::
'
''
:
\..
i
\
i
Pyruvate

release of il]§Yli e Chapter 11 ). Insulin also induces


glucokmase. Phosphofructokinase (PFK) (step 3) is the
most important rate-limiting enzyme for glycolysis path- ~""
way. re the most important allosteric
inhibitors. (Fig. 10.16). Fig . 10.16: Summary of regulation of glycolysis
Fructose-2,6-bisphosphate (F-2,6-BP) increases
the activity of phosphofructok.i°nase. F-2,6-BP is for- enzyme called (I ). The
med from fructose-6-phosphate by the action of an activities of both the enzymes (PFK2 and Fructose-2,
134 Section B: General Metabolism
coo- coo-
l
CH-O-PO3
= I
CH-OH
Irreversible sreps in glycolysis
I _ I _
Pyruvate kinase (step 9) @
yruvate carboxylase; CH2-O-PO3 CH2-O-PO 3
hosphoenol- pyruvate @- - - f?,f<,,A €,9~A

carboxykinase tP€ c ~) Mutase
1,3-blsphos- - - - - + 2,3-bisphos-
Phosphatase
• 3-phospho-
Phosphofructoklnase (step 3) Fructose-1 ,6-bisphosphatase phoglycerate phoglycerate /~ glycerate
Hexokinase (step 1) Glucose-6-phosphatase H20 Pi

Fig. 10.18: BPG shunt; step 6 of glycolysis is bypassed in eryth-


rocytes. Compare with Figure 10.11

.Carl Cori reaches liver, where it is oxidized to pyruvate. Thus it


NP 1947
is channeled to gluconeogenesis. Regenerated glucose
1896-1984
can enter into blood and then to muscle. This cycle is
called Cori cycle (Fig. 10.17).
@ significance of the Cori cycle: The lactate
produced in the muscle is etfi'6jeptly reutilized by the
body. During exercise, lactate production is high, which

-
is utilized by liver to produce glucose. Th~ cess.nee<:ls
ATP. The resultant increased oxygen consumption is the
explanation for the oxygen debt after vigorous exercise.

asteur Effect
Fig. 10.17: Cori cycle. Contracting muscle has lack of oxygen. Under aerobic conditions, glycolysis is inhibited. This
So pyruvate is reduced to lactate. This can be reconverted to
~ t o ~ effect~tgx ~ on qlycolysis is known as

r
glucose in liver by gluconeogenesis.(!.actate to pyruvate ratio Is
m:rrmaily approximately 25)> Pasteur effect.
\-\'f ft'<-O'>t\.PI \'Jll&
arburg Hypothesis CP.rNC£.R
6-bisphosphatase) are reciprocally regulated. The two
enzyme activities are present on the same polypeptide Otto Warburg showed that cancer cells utilize energy
chain , a tandem enzyme.Addition of a phosphate group to from qlycolysis and they require less ax~en than their
the tandem enzyme activates F-2,6-bisphosphatase and normal counterparts; this is called Warburg hypothesis
inactivates PFK2 leading to a fall in F-2,6-bisphosphate (1923). Inhibition of glycolysis severely depletes ATP in
and glycolysis slows down. Dephosphorylation has the can ells and leads to massive cell death. Because
opposite effect. the Warbur effect and hypoxia re frequently seen in
Pyruvate kinase is a regulatory enzyme of glyco- human cancers, these findings may have broad clinical
lysis. When energy is in plenty in the cell , glycolysis is
inhibited. Insulin increases its activity whereas glucago·,ft;".......-..= · .
inhibits. Insulin favors glycolysis by activating the ke apoport Luebermg Cyc:;.Je
~; glycolytic enzymes. Glucocorticoids inhibit glycolysis (BPG Shunti
and favors gluconeogenesis (Tables 10.3 and 10.7). Q in the erythrocytes, step 6 of glycolysis is bypassed.
Bisphosphoq!ycerate mutase.converts 1,3-bisphospho-
Cori Cycle or Lactic Acid Cycle glycerate (BPG) to 2,3-BPG. Then BPG-phosphatase
remo~e..pllos~ roup to form 3-phosphogly-
Q) oefinition: It is a process in which Qlµcose is converted..
to lactate inJba.muscle;.. and in the liver this lactate is
cerate (Fig. 10.18). i ,u;i-r-
.J
re:£9L1Yerted ·nt~ucese-_(Fig. 10.17). The muscl@ Significance of BPG
cramps, often associated with strenuous muscular exer-
cise, are thought to be due to lactate accumulation. Lac- 2,3-BPG binds to hemoglobin, and reduces the affi-
tic acid from muscle diffuses into the blood . Lactate then nity toward oxygen. So, in pr~ ce oL.2.3-BPG,
-\t ' Ra_p-Oll>O'i~ ~€$¼"3 d~ NO ~i? ~ ~ ~ & ' { ' ) . \ \
I -\'r-,0\J..<Jr\ ?, -¼ q~
Chapter 10: Metabolic Pathways of Glucose 135
II

CH3-CO-COO~
H Thiamine-PP ~ CH3- co-S-L- SH X HSCoA
"""'1!1111~
Pyruvate
11,Wulil · \!31
co
2 CH3-CHOH-TPP / L\ / L \ CH3-CO-SCoA

s------s Hs sH (PR.ot>uc:i-:_ .._


Enzyme 1 = pyruvate dehydrogenase J>~1 (Oxidized (reduced C)..JI,
Enzyme 2 = dihydrolipoyl transacetylase (1>»\.1'IV lipoic acid) lipoic acid) u-_9.\«\!l)
Enzyme 3 = d1hydrolipoyt dehydrogenase (. ' t'""") •••

Summary: PDH OX\DAT\ \1£,


Pyruvate T"\ ( \ Acetyl-CoA

CoASH CO2 NAO.. NADH+H.. ~CAR80)l'IL.Pr110N


Fig. 10.19: Details of pyruvate dehydrogenase reaction . See summary in the inset.

is essential for utilization of pyruvate. An enzyme


ti~ues U bound(fiydroxyl ethyl T fu
formed.
centration in the RBC increases, thus favoring the B. Dihydrolipoyl transacetylase (Enzyme 2): Then,
release of oxygen to the tissues even when pO2 is low. IJ.¥droxyethyl group is oxidized.

tat©
The compe_nsatory increase in 2,3-BPG in high altitudes C. Dihydrolipoyl dehydrogenase (Enzyme 3): The
favors oxygen dissociation. last step is the oxidation of lipoamide.
§r.culation;1n this shunt pathway~ TP is genera . A ,imilar enzyme complex brings about the oxidative
Please compare Figures 10.18 and 1o. . decarboxylation of alpha ketoglutarate to succinyl-CoA
in the TCA cycle (see Chapter 20).
METABOLIC FATE OF PYRUVATE t-l'C. l ~on: Alet: lost~'; i'1_\bi~ ;ruhe i r~ ucts
ace~l~~nd NA . ) "
Under aerobic conditions, pyruvate is comLedecL-to n, r) t
~J:Q.ol\ which enters the TCA cycle to be oxidized to Importance of Pyruva Dehydrogenase
CO 2• ATP is generated. .<~ Completely e}'l rs 'There is no.eafbwjY,
Glycolysis is taking place in cytoplasm. So ~_ume available in the body to cifE_umvent this,.step,,. Glucose
is gea,.e rate.d..ia..cyto_p!a~1:nsporte,d into mito- through this step is converted to acetyl-CoA from which
chondria by a pyruvate transporter. fa~cids.can.b.e~yntbesized. But the backward reaction
is not possible, and so there is no net synthesis of
Pyruvate Dehydrogenase Complex 9!!!.CAu.from,fat. HencQDH reaction ajhg romm;uaii)
.140~ . step toward com lete ox· o gly£Q_se .
Inside the ~ochon_d.cia, pyruvate is oxidatively decar- Diseases associated with glycolysis pathway are
boxylated to acetyl-CoA by pyruvate dehydrogenase described in Box 10.3.
(PDH). It is a multi-enzyme complex with ~ s
and~ enz~s. The coenzymes needed are: Pyruvate as a Junction Point
1. Thiamine gy_roph, hate (TPP)
2. (Cp.™vm.e.A.(.C~ \..c9'{(" , . ~>eJ.
3. FAD ,,- t
Z.Si+ 8C

WU
4 . NAO•
5. Lipoamide. The lipoic ~id, otherwise called .!!}i~ c =...;;...;.;;...;;._ ...:..;....:..;.~ ,ch is use eo enesis.
acid has two sulfur atoms and 8 carbon atoms. It These pathways are summarized in Figure 10.20.
can ac_seot or donate hydrogen ato~ . ~ON -..1-tO
6. The enzyme part of the PDH complex is made up of GLUCONEOGENESIS
three component enzymes (Fig. 10.19). - - --,- m
0
1-~-+-+-
A. Pyruvate dehydrogenase (Enzyme 1): Catalyze~ Deflnition \,,..\,n&.,ovo
C~
oxidative decarboxylation. TPP is required for this It is the process by which glucose molecules are produ-
step. So, Thiamine, a B complex group vitamin ced from non-carbohydrate precursors. These include
(_~)
136 Section B: General Metabolism

COO- Pyruvate carboxylase COO-


Ladle acid'B,is. It is seen in hypoxia, s11tick, pulm'i5nary I Biotin I
C=O+ CO2 C=O
failure, alcohol abuse (see Chapter 12), and diabetef mellitus
(see Chapter 11 ). bH 3 ~ \ fH2
2. Deficiency of glycolytlc enzymes. These conditions ATP ADP+Pi COO-
are rare, out of which pyruvate kinase deficiency and Pyruvate Oxaloacetate
(J{ hexokinase deficiency are comparatively common. Though
~ rare, these deficiency states can lead to hemolytic anemia, Fig. 10.21 : First step in gluconeogenesis
efto ecause nergy epleted RBCs are destroyed. 1-j~ ase
deficient R s ave a low level of 2,3 BPG and a h1g:;~~7
Ar

:i
for oxygen. On the other hand, a deficiency of e Cytoplasm Mitochondria
kinase leads t BP
Malate . = = ~- - ==== = ~ Malate
NAD+ +
MDH MDH
t NAD+ +H

NADH NADH
Oxaloacetate • Oxalo~
deficiency in alcoholism causes lactate accumulation in tissues. <" Glu , rrt,-\-
Inherited aldolase deficiency may lead to ·c.a sis. - ~''tt.-
Glu ~
AST
AST .,.,.-,i.
AKG AKG

Aspartate -----,-- - . . i -- - - Aspartate


(\.\l\-uW
Pho!~!_OOI pyruvate Lactate

Gfua.r !PYRUVATE
PK ½.
,:;:;;=,===- Alanine
Fig. 10.22:1Malate-aspartate shutt1J . MOH = Malate dehydro-
~ enase. AST = Aspartate aminotransferase. Glu = Glutamic acid.
AKG = Alpha ketoglutaric acid
PCl l PDH
Oxaloacetate Acetyl CoA - + Fatty acid Gluconeogenesis involves s es of gly-

ASTl i MDHl l colysis, bu it is not a reversal of glycoly · . The i,.[Le-


versible stees 1if Ql,tcolysis are circumvented b~ four
Aspartate Malate TCAcycle
enzymes which are designated as the k~ _enzym.es. of
PK = pyruvate kinase; LDH = lactate dehydrogenase; ALT= alanine gluconeogenesis (Table 10.7).
aminotransferase; PDH = pyruvate dehydrogenase: PC = pyruvate
carboxylase; MDH = malate dehydrogenase: AST = aspartate
aminotransferase; PEPCK = phosphoenol pyruvate carboxy kinase.t. Pyruvate Carboxy/ase Reaction
Fig. 1 o.2o: Pyruvate; metabolic junction point Pyruvate in the cytoplasm enters the mitochondria. Then,
@ctate, glu~enic
(a\ €)
amino acids, glycerol part of fat and
carboxylation of pyruvate to oxaloacetate is catalyzed
by a mitochondrial enzvme, @ ruvate carboxylas~ Fig.
~ ropionyl-CoA derived from odd chaic...fatty acids. (Fig. - -
10.21), which needs the coenzyme biotin ~md energy
10.24 ). • .
@ Site r~ft-TL"/ M,~ 6crlrot . by~ hydrolysis. ~t
@ Ma/ate Aspartate
Gluconeogenesis occurs mainly in th~ and to a
lesser extent in the ~ al co~ The pathway is P.artly Oxaloacetate is generated inside the mitochondria.
mitochondrial and partly cytoplasmic. This oxaloacetate has to be transported from mito-
chondria to cytosol. T.h~is is achieved by the malate
©
C Key Gluconeogenic Enzymes ~rtate shuttle. Oxal Jl-
te is first converted to
1. Pyruvate carboxylase 4 f'C. !" p~ Re, which tra~ ~ ~ h~ J \ ~ ~and reaches cyto-
2. Phosphoenolpyruvate carbo~[n9 _se...t-t6f-C"-' plasm. Malate is then re-converted to oxaloacetate.
3. Fructose-1-6-bisphosphatase-.,, AL,~ Malate d e h W r n o e o ~ ~ itochondria
4. Glucose-6-phosphatase and cytopla'tm (Fig. 10.22).
.7 G. \v \1-:o. d.d ..
Chapter 10: Metabolic Pathways of Glucose 137

coo- A
'co coo-
l '--- 2
l
C=O
I
CH2
PEPCK
4 r CO- P
II
CH
1----..
2
GTP GDP
( c~ (ITP) (IDP)
Glycerol

j
Oxaloacetate Phosphoenolpyruvate

Fig . 10.23: Phosphoenolpyruvate carboxykinase

Glyceraldehyde-3-phosphate - - - - DHAP
ate Carboxykinase
In the cytoplasm, PEPCK enzyme then converts oxalo- !i
1,3-Bisphosphoglycerate (1,3-BPG)
acetate to phos hoenol pyruvate bY. removing a molecule
~'\,e- l t , - .ADP
o~ 2
• GTP or ITP onates the phosphate (Fig. 10.23).
t---- ATP
The net effect of these two reactions is the
3-Phospr t ycerate
conversion of pyruvate to phosphoenol pyruvate. -

2-Phosphoglycerate

., t
Phosphoenol pyruvate
Partial Reversal of Glycolysis
The phosphoenol pyruvate undergoes further reactions ·f
Lactate
7
Glycine, Alanine,
catalyzed by the glyzilytic.....,filgy~ to form fW-~ Serine, Cysteine
1e-bise_hosphate (see glycolysis steps 8,7,6,5 and 4). fu<
All these reactions are fr~ JffilefS.ible. 1 Acetyl CoA

Fructose-1, 6-bisphosphatase \
Fructose-1 ,6-bisphosphate is then acted upon by fruc-
tose-1 ,6-bisphosphatase to form fructose-6-phosphate.
Asn
Asp
-----? Oxalo ~
acetate Citrate

This will (see step 3 of

j/E:~::
Malate \
glycolysis).
Fructose-1 ,6-bisphosphatase (enzyme) Phe,
r
Fructose-1 ,6-bisphosphate-,Fructose-6 phosphate+ Pi Tyr
\ Alpha keto
Asp,
Then fructose-6-phosphate is isomerized to glu- Asn - Fu~ rate /ta rate
cose-6-phosphate by the freely reversible reaction cata-
lyzed by hexose;_Qhospqate · mera (second step in "' / lie,
Propionate Succinyl CoA - - - - Met, Val
glycolysis)_. r r 'l Key gluconeogenlc enzymes:
1=Pyruvate carboxylase; 2=Phosphoenol pyruvate
carboxykinase; 3=Fructose-1 , 6-bisphosphatase;
Glucose-6~phosphatase Reactiorl 4:Glucose-6-phosphatase
Key glycolytic enzymes .
The glucose-6-phosphate is hydrolyzed to free glucose S=Hexokinase; 6=Phosphofructok1nase: 7:;Pyruvate kinase
by glucose-6-phosphatase. Substrates for gluconeogenesis are shown inside brown squares.

Glucose-6-phosphate + Hp -, Glucose + Pi Fig. 10.24: Gluconeogenic pathway


Glu1i,:;-phospha.,ti e is active in liver. It is pre-

i
sent in and in,tgs't!nal muco~a to a lesser extent, Significance of Gluconeogenesis
but is absent in muscle. through gluco-
The detailed steps of gluconeogenesis are shown in neogenesis, because glucose-6-phosphatase is pre-
Figure 10.24. sent mainly in liver. So liver pJays the major role in
"- '>
lli...v-~\.--
Ne\
138 Section B: General Metabolism

coo- Lactate dehydrogenase COO- Alanine aminotransferase COO-


(LOH) I (ALT)+ PLP I
C=O fHt N~ --(~-\~--• , ~O

CH3 NAO
( '\ NADH + H CH3 CH3 a-keto-
glutarate
Glutamate CH3

Lactate Pyruvate Alanine Pyruvate

Fig. 10.25: Reversal of step 10 of glycolysis Fig. 10.26: Transamination of alanine

B maintaining the blood glucose level. During starvation


glucone~
The sto~
esis maintains the blood glucose level.
ly~ ~n is deeleted within the first j 2- 18 i
Pyruvate

Z
hours of fasting. On prolo~ ) tarvation, the gluco-
neogenesis is speeded up ar\<U!'~ rr ca tabolism provi-
~des the substrates, namely glucogenic amino acids.
i LORt'"- C4t.le.
o/"1 (Qluconeogenesis by renal cortex increases:) ® Lactate - -- - - Lecta't'e - - - Lactate

pCJ c ci'nergy requiremen · e reactions catalyzed by pyru- ~ILl:A


Alanine .,___.:=-.._ ~ Alanine - - - Alanine
f'O'C,~ ate car~ ylase, p oenolpyruvate carboxy kinase
'f' b.~ and phosptioglycerate kinase so
( ~ ') o produce one-
~'\:<fl half molecule of glucose; or 6 ATPs are required to gen-
:Jv:~.

1!:63@
erate one glucose molecule. Liver _
Blood Muscle
n,e>,I";..,
I",, ...
I' Fig . 10.27: Glucose-alanine cycle
Glycolysis
lucose , 'Lactate
Gluconeogenesi ·· the glycolytic pathway to form pyruvate, which in turn,
can be transaminated to alanine. Glucose-alanine cycle
Substrates for Gluconeogenesis is important in conditions of starvation (Fig. 10.27). Thus
Lactate and glucogenic amino acids are the most net transfer of amino acid (nitrogen) from muscle to liver
important substrates for gluconeogenesis. and corresponding transfer of glucose (energy) from liver
to muscle is affected. Alanine cycle is intimately related
Lactate "' } with Cori's cycle (compare Figs. 10.17 and 10.27).

The l~ ate.Jorroed-ifldea-ml:t§S!'l is transported to th,€r}G/ cero/


liver. In the liver cell lactate dehydrogenase convert~ y; _
lactate to pyruvate (Fig. 10.25). The pyruvate enters the The glycerol part of fat is phosphorylated j n the liver·
gluconeogenic pathway to form glucose. cytosoffiyALElo jfilycerol jphosp ~ It is then oxidized
to dihydroxyace~ hate by an NAO• dependent
(g) Glucogenic Amino Acid~ dehydrogeriasel Fig. 10.28).
Glucogenic amino acids are shown in Figure 10.24. {Ala
ropionyl-CoA
nine, glutamic acid, aspartic acid, etc.). When glucose is
not readily available (starvation or diabetes mellitus), the Propionyl-CoA is formed from ~ n.fa.t1¥-~ nd
~ ic amino acids are transaminated to correspond- carbon skeleton of some amino acids. It is converted to
ing carbon skeletons (Fig. 10.24). These then enter the succinyl-C9A (see Fig. 13.10) and is a rpinQ[ s~e for
TCA..cycle and form oxaloacetate or pyruvate. Alanine glucose.
released from the muscle is the ~llestfate for gluco- Important: Even chain fatty acids cannot be converted
rleogenesis (Fig. 10.26). Muscle wastage seen in uncon- to glucose; the~ are not substrates for gluconeogenesis.
trolled diabetes mellitus could b plained by this factor. Ace'°Y1 <.of\ Bul 'v. 1~\tt
I/& - ~- r .; egulation of Gluconeogenesis J.., r?J,-
® Glucose-Alanine Cycle Gluconeogenesis and glycolysis are reciprocally regu-
Alanine is transported to liver, transaminated to pyruvate lated so that one pathway is relatjyelv inactive when the
and converted to glucose. This glucose may again enter other is active. The regulatory steps are:
Chapter 10: Metabolic Pathways of Glucose 139

Glycerol Glycerol-3-phosphate
kinase dehydrogenase Fructose-6-phosphate

(\ • Glycerol-3-
phosphate
( '\ Dihydroxy-
acetone
phosphate
(+) Citrate
ATP

ATP ADP NAO+ NADH + H+ (-) AMP


(-) F-2, 6-P
ADP

Fructose-1,6-bisphosphate

Fig. 10.29: Reciprocal regulation of PFK (glycolytic enzyme) and


Fructose-1,6-bisphosphatase (gluconeogenic enzyme)
Fig. 10.28: Gluconeogenesis from glycerol

Glucose TABLE 10.8: Regulatory enzymes of gluconeogenesis (Com-


pare with Table 10.3)

Gluconeogenesis
Glucagon (+)
1l Glycolysis Enzyme
PC
Activation
Cortisol, Glucagon
Inhibition
Insulin, ADP
Glucocorticolds (+) f\f\\ p
Adrenalin, Acetyl-CoA
Insulin H Prt>t>
A'iP> G \:rols, (+) Insulin PEPCK (i,nO..~ Insulin
do
c..Am-P Pyruvate F-1,6-bisp hosphatase do F-1,6-BP,AMP,F-
2,6-BP
Hormonal regulation of gluco
G-6-phosphatase do Insuli n

• Pyruvate Carboxylase
It is an allosteric enzyme. Acetyl-CoA is an activator of BOX 10.4: Clinical significance of pyruvate
pyruvate carboxylase so that generation of oxaloacetate 1. Pyruvate carboxylase deficiency. It Is seen as an inb orn
is favored when acetyl-CoA level is sufficiently high error of metabolism, where mental retardation is manifested.
(Fig. 10.21). Its incidence Is 1 in 25,000 births. Lactic acidos1s is noticed.
2. Malignant hyperthermia. This may occur Yfil.en halothane
i given as an anesthetic to certain persons. The ryanodine
receptor, a calcium-release channel is defective, leading
to inappropriate release of calcium from sarcoplasmic
reticulum. This results in uncpntrolled heft ~ neration,
dama~ of muscle cells~TP depletion, lactic acidosis and
rhabddrnyolysis. CPK is markedly elevated. This defect is seen
in 1 per 50,000 population.
3. Ethanol (Ethyl alcohol). Inhibits gluconeogenesis. During
the metabolism of ethanol the level of cytoplasmic NADH
is raised. Thus, the Pyruvate Malate - > Oxaloacetate
Gluconeogenesis is enhanced by ATP. reactions are reversed. So, ingestion of alcohol
r~s in hye,oglycemia. Lactate also accumulates as NADH
Hormonal Regulation of Gluconeogenesis level is high (see Chapter 12).

The hormones uca and ocorticoi ncrease


gluconeogenesis (Fig. 10.30). Glucocorticoids induce the GN Ratio or ON Ratio
synthesis of hepatic aminotransferases thereby provid-
During gluconeogenesis proteins are degraded and
ing substrate for gluconeogenesis. The ~ lucaggn
end-product urea is e1creted through urine. The ratio of
favors induction of synthesis of gluconeogenic enzymes
glucose (dextrose) to· urea nitrogen in urine is term~ ~
(PEPCK, Fructose; 1,6-bisphosphatase and glucose-6-
GN ratio or ON ratio. In such animals, the ratio 1s ~ 1.36
phosphatase). At the same time, glycolytic enzymes HK,
That means 1 g of nitrogen (from protein) will form
PFK and PK are depressed. Insulin inhibits the process
(Fig . 10.30). A summary of regulatory enzymes of. ;!.6 Lucose. · , l_lll ..-
gluconeogenesis is shown in Table 10.8 (compare with GN ratio is enhanced
Table 10.3). Clinical significance of pyruvate metabolism when catabolism is increased, e.g. insulin ciP.ficiency,
is shown in Box 10.4. starvation, pyrexia, hyperthyroidism and ~
A\) t:.~z~e~ DD ~\~Cft'oen meto~,~in
140 Section B: General Metabolism O.le CY.TO PLASM 1C

Glycogen
phosphorylase
Glycogen Glycogen + Glucose-1-
G
with (n) + Pi with (n-1 ) phosphate
glucose (PLP) glucose
residues residues

Blood Fig. 10.32: Reaction of glycogen phosphorylase -: •


glucose f':
Gi\ij~.s CJ>_ol ; ~UL~H~
Fig . 10.31: Functions of muscle and liver glycogen Functions of Glycogen 1.."6<:>1'1~

Glycogen is the s!arage form of carbohydrat s in the


~ n bod~ lhe major ~ites of ~t_?rage are Ive . and
muscle~e major function of hver glycogen Is to
provide glucose during fasting. The glycogen content
~f ~ (10 q/100 g tissue) is more than in the skeletal
Glucose-1-phosphate ~ (1;-2,.g/10Q g). B~ the total gu~ tity of muscle
Glycogen phospho units released
glycogen is more than liver glycogen because of the

l
sequentially
larger muscle mass. When blood glucose level lowers,
00000
0 0 0 0-0-0-0-0-00-- liver glycogen is broken down and helps to maintain
blOfil~Cose ~v~I (Figo.11 Q._3_\l-
Act1on of glycogen ( Transferase enzyme )
phosphorylase stops near ~ sfers a tn§pccharide 0..).'fter ":foobc..o~~o~r whi~h
the branching point unit to another.branch causes ~~63®r:amtasWiffn li~tA]_o'lR 5 h~ u e_r
taking food, the blood sugar tends tu.fall. But, glycogen
~ -- is lyzed to glucose so that the energy needs are met.

(,Oebranching enzyme)
µ
hydrolyses alpha-1 , 6 linkage

0-0-0-0-0-0-0-0--
Glucose unit 1s released:
branch point is removed

After about 18 hours fasting, most of the liver glycogen
is depleted, when depot fats are hydrolyzed and
energy reguireme~i~t ~ - The
function of muscle glycogen Is to act as reserve fuel for
G en phospho; las y
further acts
l
00
Glucose-1-phosphate units
are released sequentially
muscle contraction. All the enzymes related to glycogen
metabolism a r e ~.
......,....__

Fig. 10 .33: Glycogenolysis


DEGRADATION OF GLYCO~ 1
(GLYCOGENOLYSIS)_,.,.='9' ."
GLYCOGEN METABOLISM
r:r_J{ll:n,...HQ
hosph ilase ,~fr
I i -y.
c.-
1
-

Carl Cori and Gerty Cori were awarded Nobel Prize ir:t, Glycogen phosphorylase removes gluco~e as glucose-
1947, for their work on glycogen degradatii:m. Based on 1-phosphate from glycogen (phosphorolysis) (Fig. 10.32).
his work on glycogen synthesis, Luis Leloir (Argentina) It contains pyridoxal phosphate (PLP) as a prosthetic
was awarded Nobel Prize in 1970. Earl Sutherland group. The alpha-1,4 linkages in the glycogen are
(Nobel Prize, 1971 ) studied the roje of cyclic 4,MP as ~ ea-f°e d. r-emoves glucose units one at a time. Enzyme
the second messenger in glycogenolysis. Glycogen ~ equentially hydrolyzes alpha-1,4 glycosidic linkages, till
is a @opolysaccha~ with glucose units linked in it reaches a glucose residue, 3-4 glucose units away
alpha-1 , 4 linkages (straight_!l,ne) and ajpha-1 , 6 linkages from a branch point (Fig. 10.33)(li cannot attack the 1,6
(branching oojnt). Branching makes the molecule more linkage at branch paint> If glycogen phosphorylase alone
globular and less space-consuming ..• For details of acts on a glycogen molecule, the final product is a highly
structure, see Figure 7.23. branched molecule; it is called limit dextrin.
L. , rp o CJ '1d .~)(r!f '-< cm tr1p1e 1 1)m 2\crti vo¥.'->r-- f')~ •
-: \_~ r"IQ.\ rn'-'.SrJ.e q r- C ?1'I'
.., Chapter 1O: Metabolic Pathways of Glucose 141

CH 2OH .
H2 GLYCOGEN SYNTHESIS

O
H H -o-:x- 0C- ~
OH (GLYCOGENESIS)
0
0 o-r-o-
II
OH H
The glycogen synthesis occurs by a pathway distinctly
different from the reversal of glycogen breakdown,
0 OH
H OH Cf which would prevent the operation of futile cycles. The
H OH
Glucose-6-phosphate steps are:
e reaction
-\ '\J> \)1 ()..'(-!.~
• •
Act1vat1on of Glucose
NO O
il
V-.
a.:
Deuranc h"mg by 1"() 0:-DI!.)
0 .1 er. •~ 1'1.A" UDP glucose is formed from gllJ...cose-1-phos.e,hate and
1 F V""'cu)s=- ~..., UTP (uridine triphosphate) by the enzyme UDP-glucose
QV.,

Bifunctional (Two) Enzymes


:· ,r.~J yrophosphorylase. · ·
A block
-.:-
of 3 glucose resjdues (trisaccharide runif re; ~ _t<..._
..,a\'. UDP-glucose-
trcmsfe,o:ed from the branching point to another brnllCb. t
pyrophosphorylase
This enzyme is alpha-1 , 4 -, alpha-1, 4 glucan trans-
Glucose-1--- - -~ UDP-glucose + PPi
ferase. Now the branch point is free. Then alpha-1,
6-glucosidase (debranching enzyme) can hydrolyze phosphate -+'~ TP foo:r c •1.-t'"i ..-a.~ v~ Clf1run
~\,.'6b bo.c.J-
the remaining glucose unit held in alpha-1 ,6 linkage at \rn 00)'.. \ 1
the branch point Fi . 10.33). This glucose residue is .ta; lycogen Synthase
I a a r e •
. The transferase and alpha-1, The glucose moiety from UDP-glucose is transferred to
6-glucosidase will together convert the branch point to a a glycogen primer (glycogenin) molecule. The primer
linear one. With the removal of the branch point, phos- is essential to accept the glycosyl unit. The p.ritner
rylase enzyme can proceed with its action. . is made_up_oLa protei0=.earbobydrate complex. An
- , _,~l).i.\,q aJigasaccbacid.e c ~ l ~ e units is added to the
hosphoglucomutase r P("' 'f-.f' Qf"ITI'! primer (glycogeni[l). , _ . ~cl
•()11'6~ 1 (JD()JI I Tl .» e\r
Phosphorylase reaction produces glucose-1-phosphate ~ : .,. . o) Glycogen synthase "- ~ ,._
while debranching enzyme releases glucose. The glu- Gl~ ~r (n) - - - - -~Glycogen (n+1 )
cose-1-phosphate is converted to glucose-6-phosphate + UDP-glucose . + UDP (. 1'2unoo~
· ~ e l \ ~ ~ ~bo,-_ ~--
b~ hosphoglucomutase (Fig . 10.3 ).
E
cose
-6- h h t .
4

p osp a ase m 1ver


r
'Tn,C-in4..J't-
e\J" \l:
In the next step, activated glucose units are
sequentially added by the enzyme glycogen synthase
(Fig. 10_35 ). The gl~ ~ addedtothe p.ctmer to
Next, hepatic glucose-6-phosphatase hydrolyzes glu- form an alpha-1 , 4 glycosidic linkage.
cose-6-phosphate to glucose. The free glucose is relea e .
sed to the bloodstream. ranchmg Enzyme
The glycogen synthase can add glucose units only
Muscle Lacks Glucose-6-phosphatase in alpha-1,4 linkage. A branchjng~.z,y.me....i5-n.ead,ed .
1
to create the aleba;,1..6 lioL~ When the chain is tt f\
l~ngth.ened.Jo 11-12 gl!,/_ccts.e.J.esid.!-les, the branching <i?.-
enzyme will transfer a block of ~ gtucose residues ~ ' ( "
activity, I ste df in muscle, glucose-6-phosphate under-
from this chain to another site on the growing molecule 3S,i..u
goes'glycolysi o produce ATPlor muscle contraction.
with 1,6 linkage (Fig. 10.35). To this newly created ~ rr-.o.,r
' ~h J\'riAeYO~'e,
branch, further glucose units can be added in alpha-1,4 c' '°'"
Energetics linkage by glycogen synthase.
The energy yield from one glucose residue derived
Regulation of Glycogen Metabolism
from glycogen ~G_ ATP molecule§} because no ATP is
requi(ed for initial phosphorylation of glucose (step 1 The synthesis and degradation pathways are recipro-
of glycolysis). If glycolysis starts from free glucose only pally.regulated to r :tent futile cycles. The phospho-
2 ATPs are produced. rylated form of glycogen phosp orylase is active;
142 Section B: General Metabolism

In 1992, Nobel Prizes were awarded to Edwin Krebs


and Edmond Fisher for their work on protein kinase,

c,,c:'~,"o,,! ~s)e
(many cycles)
UDP
1 Glycogenin

alpha-1,4 linkages
phosphorylase kinase and protein phosphatases.
The intracellular concentration of cAMP therefore
decides the level of active PKA. The cAMP level depends

Branching enzyme
transfers
j on the activity of adenylate cyclase and phosphodi-
esterase. Cyclic AMP level is increased by glucagon and
decreased by insulin. P h~hO&.~!:
6 glucose residues
to form a new branch Adenylate cyclase ~ f!JI 0

ATP 1r ::;... 5'AMP


Alpha-1, 6 linkage
+ Glucagon +Insulin ?.~

Repeat action of
glycogen synthase and
branching enzyme
l Glycogen in
hosphory/ase Kinase Activation
The active protein kinase can now co Q_'ie.ct the phos-
•VT

pho.,2'.lase kinase to an active phospoo'3/!atei!,,form, which


converts phosphorylase-b (inactive) to phosphorylase-a
(active).
C]
"'..:
a;...
lc-1u-m
- t:-:ri'-=g-=
ru-=
e-::
rs:-:m
::-u
~s:-:c:-;-1e= -c:-:o:-:n::-7tr:a::::c::;
ti~
OJ) as well as
~li coqen breakdown through the action.of ghosphol)'lase
~ e. Th~ ~si.s..is linked to rate of

~ 1 phosphorylation
1 of the active phosphorylase
'---- - - - - -- - -- - - - - - - - ~ pr~; e~~ phosphatase 1 (PP1) involves removal of
Fig. 10.35: Formation of branches in glycogen phosphate group from phosphorylase-a. When cAMP
level is high, PP1 is inhibited. The effeclol cycUe-AMFlis
but glycogen synthase becomes inactive on phos- OQL,o~~asiA9-loo..pbospl:lol)4atiGA-ef-em~ymes,
phorylation. The hormonal control by covalent modifi- but also by decreasing depbos.phol)'.iation.
cation and allosteric regulation are interrelated. These
hormones act through a second messenger, cyclic AM lycogen Phosphorylase in
(cAMP) (For structure see Fig 38.7). The mechanism is Liver and Muscle
shown in Figure 10.36.
Liver: The liver phosphorylase-b is the inactive form.
It becomes active on phosphorylation. The active
Generation of Cyclic AMP (cAMP)
enzyme is denoted as phosphorylase-a. The enzyme
Both liver and muscle phosphorylases are activated by a is inhibited by ATP and glucose-6-phosphate ~ee Fig.
c clic A mediated activation cascade triggered by 10.36).@lucose is an a"ikisteric inhibitor of phosphorylase
ttie ormonal signal. The hormones epinephrine and a. Insulin favors this effect by promoting the uptake and
glucagon can activate liver glycogen phosphorylase phosphorylation of glucose.
but glucagon has no effect on the muscle. When the Muscle: Skeletal muscle glycogen is degraded only
hormone binds to a specific receptor on the plasma when the demand for ATP is high. The regulation of
membrane, the enzyme adenyl cyclase is activated glycogenolysis in skeletal muscle is by epinephrine. ,
which converts ATP to cyclic AMP (cAMP). When level Glucagon has no effect on muscle glycogenolysis. AMP
of cyclic AMP rises, it will activate a protein kinase (PKA) formed by degradation of ATP during muscle contraction
by phosphorylation of a serine residue (Fig. 10.36). is an allosteric activator of phosphorylase-b.

® Protein Kinase Activation The active form of phosphorylase is referred to


as 'a' (active, phosphorylated) and the relatively inac~
Earl Sutherland (Nobel Prize, 1971) studied the role of tive dephosphorylated form as 'b'. On the other hand,
cyclic AMP as the second messenger in glycogenolysis. the active glycogen synthase (a) is dephosphorylated
Chapter 10: Metabolic Pathways of Glucose 143

6
Epinephrine/Glucagon attaches to receptor
', Glycogen synthase
'- - - (Active) (Dephosphorylated)
Cell membrane

I \----- Adenyl cyclase I/ v---- ATP


r ' ..-G,~:1-.,..r,
Sutherland
~ ADP
ATP Cyclic AMP + Pi NP 1971

1 Glycogen synthase
1915-1974

Inactive Active protein kinase (Inactive) (Phosphorylated)

!
protein kinase (Glycogen synthesis inhibited)

Phosphorylase kinase Phosphorylase kinase

(\
(inactive) (active)
(dephosphorylated) (phosphorylated)

ATP ADP

Glycogen phosphorylase-b Glycogen phosphorylase-a


(inactive) (dephosphorylated)

ATP
(\ (active) (phosphorylated)

+
ADP Glycogen ---'--+ Glycogen (n-1 units)+
(n residues) Glucose-1-phosphate
(Glycogen breakdown is favored)

Fig. 10.36: Cyclic AMP-mediated activation cascade

and phosphorylated (b) is inactive. Phosphorylase-b is Glycogenolysis


sensitive to allosteric effectors like AMP but phospho- Glycogen (n)
rylase-a is not sensitive. High concentration of ATP and
Epinephrine (+) ~ Pi
glucose-6-phosphate in the cell will inhibit phosphory- Glucagon (+) Phosphorylase
lase-b. glucose-1-phosphate

Glycogen (n-1)
Glycogen Synthase
Glycogen synthesis
Glycogen synthase and phosphorylase activities are
Glycogen (n)
reciprocally regulated. The same protein kinase, which UDP-glucose
phosphorylates the phosphorylase kinase would also
[ Insulin (+) ycogen synthase
phosphorylate glycogen synthase. The activity of the
glycogen synthase is markedly decreased on phospho- UDP
Glycogen (n+1)
rylation. Insulin promotes glycogen synthesis by favor-
ing dephosphorylation . Glycogen synthase is active in Fig. 10.37: Effects of hormones on glycogen
the dephosphorylated state. Relative rates of glycogen
synthesis and breakdown are controlled by the action of stimulus of hypoglycemia. The key enzyme for glycogen
synthesis is glycogen synthase. Its activity is decreased
PKA, phosphorylase kinase and protein phosphatase.
by glucagon and epinephrine. Activity is enhanced by ins-
The regulation of glycogen phosphorylase and synthase
ulin, under the stimulus of hyperglycemia (Fig. 10.37).
is a typical example of multisite phosphorylation (pri-
Glycogen metabolism is regulated by coordinated
mary and secondary sites) for metabolic regulation .
regulation of glycogen synthase and glycogen phos-
phorylase. The regulatory mechanisms include allos-
Summary of Regulation teric control as well as hormonal control b covalent
The key enzyme for glycogenolysis is phosphorylase. rm-Pificati96)of enzymes. Th allosteric effect are
It is activated by glucagon and epinephrine, under the ATP, gluco'~-6-phosphate an AMP.
l-\~m on oJ 1 b\\v.t~oo.\ £.pi-neph~'l'\t...,
X.<-6u)'"' .
144 Section B: General Metabolism

Type I von Gierke's disease

Type II Pompe's disease Accumulation of n in lysosomes of Liver, heart and muscle;


' " ,._J r"l'f"\)..l infantile form deat before 2 years€ ~e.nile form myopat@;wf
t' a:,1:·•Q • N.- orm muscu ar ystrop like r Mu.I.<' " \ ~

limit dextrinosis Debranching enzyme ~ ( 9 -t Highly branched dextrin accumulates; Fasting hypoglycemia;
Cori's dise se (Liver and muscle) c;., hepatomegaly, myopathy

Amylopect inosis Branching enzyme f"lt.~_,<fJJ. Glycogen with few branches; h!_Patospleno~aly, irr~ t death
Anderson's disease (Liver and muscle) • · before age of 5

Excerdse.io.tolerance; accumulation of glycogen in muscles,


myoglobinuna Cl'Y\b u:ts,o.-')

Type VI Her's disease Hypoglycemia, hepatomegaly

GLYCOGEN STORAGE DISEASES BOX 10.5: Pompe s disease


Enzyme replacement therapy is now available for treatment
These are inborn errors of metabolism. That phrase was
of · ntile-onset Pompe's disease. The enzyme designated as
coined by Sir Archibald Garrod.
recombinant human acjg, sleha qlycosjdase. r~ )
is administered intravenously and is approved in US by FDA. The
Glycogen Storage Disease Type-I drug is very expensive currently and has to be taken life-long.

It is also called von Gierke's Oisea ::;s


~e~ F-;2::!:::
c~m1::l:::
m~on
type of glycogen storage disease i ty · . HEXOSE MONOPHOSPHATE
Incidence is 1 in 100,000 live birt . alient features
of the disease are:
311 ~Qnru.::ISl~
1. Glucose-6-phosphatase is de1lcie~. -®
2. Fasting hypoglycemia that does not respond to "Dickei'5pHorfpcker pathway"; "Shunt pathway" or "Phos-
stimulation by adrenaline. The glucose cannot be phoglucon\?e oxidati~~thway". Instead of glucose
released from l~r d~ri~ over night fasting (see going through the ~1~6 olyflcp~ay, it is'-hunt through g
Box 12.1 for neona~h O ycemia). ~
t 1s athway; so It Is known as the shun pathway. In
· a\ L , td- • \:P~
3. Hyperlipidemia, lac· 1c acidosis and7<e sis . co I s· , ere a~ f b. nhr.crrh · ·
~.3e...[1termed1ates;
4. Glucose-6-phosphat~ Is accumulated, so it is chan- but in tj)js pathw ~Ti a os hate~ ly;
neled to HMP shunt pathway (described below) pro~ ence this is called hexose monophosphate (HMP)
ducing more ribose and more purine nucleotides. pathway. The rea ctions involve the intermediate fofma-
5. Purines are then catabolized to uric acid, leading to tion.of pentose. phosphate~; hence thi_s is._also called
hyperuricemia (see Chapter 38). pentose P{)Bll'~te P~~m~Y- MS4.t;.\o_
6. Glycogen gets deposited in liver. Massive liver . About 1(Wc; ofgmco~molecules pe\ day arf\_;Jlte 'ng
enlargement may lead to cirrhosis. this pathway. The liver and RBC metah~bout30%
7. Children usually die in early childhood. o ~ a y. The major purpose of 1his
8. Treatment is to give small quantity of food at fre-
quent intervals.
There are many types of glycogen storage diseases,
depending on the deficient enzymes. Types I to VI are
described in Table 10.9 and Type II in Box 10.5. They
are very rare, incidence being 1 in 1 million births.
Blood glucose level is maintained within a very narrow Edmond Edwin G Otto von Archibald
limit in normal persons. The homeostasis of blood sugar Fischer Krebs Gierke Garrod
NP 1992 NP 1992 1877-1945 1857-1936
and abnormal conditions are explained in Chapter 11. b. 1920 191FJ....2009
Chapter 10: Metabolic Pathways of Glucose 145

Step 3, NADPH is again Generated


This is an @ idative step coupled witb decarb ~~ @
The enzyme is 6-phosphoglucona~ dehydrogenase. The
Overview of the Shunt Pathway 6-phosphogluconic acid is dehycft~ ~ -
The HMP shunt pathway has oxidative and non- o-6,nospMoglQconaf.e It is a ~~:;;:A.:...;,~~:,,• d, and
oxidative phases. During the oxidative phase, glucose-6- spontaneously under oxylation to form ribu-
phos~ eis oxidized with the genera · of 2 molecules lose-5-phospbat . The carbon of CO2 is derived from
~ A P and one molecule of P.~ ~.._..I.U.U.~=te, COOH group of gluconic acid (Fig. 10.38). In this step, a
witnn,e- iberation of one molecule o 0 2 • During the second molecule of NADPH is generated.
non-oxidative phase, the pEiQtose phosphates are con-
verted to intermediat~of glycolysis.

IOXIDATIVE PHASE Step 4, lsomerization


The ribulose-5-phosphate is then isomerized to ribose-
Step 1 of HMP Pathway 5-phosphate or epimerized to xylulose-5-phosphate
Glucose-6-phosphate is oxidized by NADP• depend- (Fig. 10.39).
ent Glucose-6-phosphate dehydrogenase (GPO).
6-phosphoglucono lactone is formed. One molecule of Step 5, Transketolase Reaction
NADPH is formed in the reaction (Fig. 10.38). This is
Transketolase is a thiamine pyrophosphate (TPP) depen-
the rate-limiting step. Regulation is effected by this
dent enzyme. It transfers two-carbon unit (with keto
enzyme. :\,,;,o~T"I Nf'\1:>P~ group) from xylulose-5-phosphate to ribose-5-phos- {"'+

Step 2 of HMP Pathway


phate to form a 7 carbon sugar, sedoheptulose-7-phos- l
phate and glyceraldehyde-3-phosphate (Fig. 10.40). '1- '3
The lactone is hydrolyzed by gluconolactone hydrol~ Transketolase enzyme will transfer the group from a
to form 6-phosphogluconic acid (Fig. 10.38). @_onor ketose to an aldose acceptoj:) In thia~ . defi-
ciency, transketolase activity is decreased.

BOX 10.6. N/\D ,md NADP ,Ht' d1ffprenl Step 6, Transaldolase Reactio11
NADH ts used for reducing reactions in catabolic pathways, e.g.
The next group transfer reaction involves the transfer
and is g; e t . <
pyruvate to lactat . ADH enters the electron transport chain,
-:J of a 3 carbon unit, from sedoheptulose-7-phosphate to
glyceraldehyde-3-phosphate to form fr.JJctose-6-phos-
phate. Here also the donor is a ketose and acceptor is
an aldose (Step 6, Fig. 10.41 ).

Step 7, Second Transketolase Reaction,


]YADP differs from in having an additional phosph~ e
group (see Fig. 33.6). These ~o coen~ ~ ,.are sp.ecific.Jor
enz m s; they are not interchangeable.

CHO

i
OH
HO

OH
OH
CH2OPO3=
NADPH+H•
+OH
CH2 OPO 3=
Glucose-b-phosphate Ribulose-5-phosphatc

~e)~t~~)
Fig. 10.38: Oxidative phase of HMP shunt pathway; Steps 1, 2 and 3
i . 1)eh'-3~~~ohon 2.. - \.'~d•·nlhcm i.
cox~) s.1.,~'l",,I
I..:. dec:.a...-Ybo~ \.J \.O..,~
146 Section B: General Metabolism
r,ve.Y..e
.,.. kr6n ,. ,.

t::
Ribose-5-phosphate ceraldehyde-3-phosphate
CH2 0H

i lf
CHO
Transketolase
0 + OH

+oH
R
HO =c5H) CH20P03=
H
CH20P03=
OH H H
CH2 0 P0 3=

c~
CH2 0P03 = CH 20P03 =
HO

Xylulose-
5-phosphate
Phosphopentose
Eplme.uise
Ltf"t:,)
~==== ulose-
Phosphopentose

5-phosphate
l pf>\.J
=== :=, Ribose-
5-phosphate
H
Of CH2 0P03=
OH
OH
OH
CH20P03=
Xylulose-5-phosphate Sedoheptulose-7-phosphate

Fig. 10.39: Step 4 of HMP shunt pathway Fig. 10.40. Step 5; first transketolase reaction

Suppose, 6 molecules of glucose enter the HMP shunt pathway:


GPD 1 2
6 x glucose-6-P-7--,-""'"'\--+• 6 x 6-phospho- 6 x 6-phospho-gluconate
/ t
gluconolactone r - 6 x NADP+
6 NADP+ 6 NADPH 3
~ 6xNADPH

5 x glucose-6-P
__:N'd:
. . :. =-.;..._-- -- -~ 6 x 3-keto-6-phosphogluconate

5 glucose are regenerated;


6 CO2 eliminated; 3
12 NADPH generated; 6 • CO, eHm;rn,t,d
1 glucose is completely oxidized
6 x ribulose-5-P
I
~-
8
~ 4 x xylulose-5-P
I
2 x xylulose-5-P 2 x
i
xyiulose-5-P
5
5 x Fructose-6-P

., t?>PG,if'\
2 x sedoheptulose-7-P

2,-t?- ~4+4-) 6
3-t~ 2 x erythrose-4-P 2 x fructose-6-P

2 x gr c: raldehyde-3-P 2 x fructre-6-P

1------ 1 x fructose-6-P J
1----- - 2 x fructose-6-P - - - -- -- -- --'-
- -- - - 2 x fructose-6-P - -- - - -- - -- - - -- - - -- - - - -

Enzymes·
1 = Glucose-6-phosphate dehydrogenase (GPO) 5 = Transketolase
2 = Gluconolactone hydrolase 6 = Transaldolase
3 = 6-phosphogluconate dehydrogenase 7 = Second transketolase
4A = lsomerase 8 = Reversal of glycolysis
48 = Eplmerase

Fig: 10.41 : Summary of shunt pathway; numbers show the steps referred to in the text
Chapter 10: Metabolic Pathways of Glucose 147

form fructose-6-phosphate and glyceraldehyde-3-phos- BOX 10.7: S1gnif1c;.mce of HMP shunt p;ithw;iy
phate (Step 7, Fig. 10.41). Metabolic importance:
1. To produce NADPH and pentose phosphates
.step 8, Regeneration of NADPH is required for
i. Reductive biosynthesis-fatty acids, cholesterol and ste-
Glucose-6-Phosphate roid hormones
Two molecules of glyceraldehyde-3-phosphate formed in ii. Free radical scavenging

step 7 are condensed to form one fructose-6-phosphate iii. Maintain RBC membrane integrity by keeping GSH in
reduced state
(reversal of step 4 of glycolysis). Fructose-6-phosphate
iv. Prevention of methemoglobin formation
is then converted to glucose-6-phosphate (reversal of v. Detoxification by hydroxylation
step 2 of glycolysis). A summary of the whole pathway is vi. Maintain the transparency of lens
depicted in Figure 10.41 . vii. Bactericidal activity of macrophages
2. Ribose-5-phosphate is required for nucleic acid synthesis
Regulation of HMP Shunt Pathway 3. Clinical importance
I. Glucose-6-phosphate dehydrogenase deficiency
ii. Drug-induced hemolytic anemia
iii. Methemoglobinemia
iv. Thiamine deficiency leads to reduced transketolase activity

athway is Significantly
Active in the Following Organs
i. Liver
ii . Adipose tissue
summarized as:
iii. Adrenal cortex
G-6-P + 12 NAOP• + 7Hp - - -- - iv. Mammary glands
6 CO2 + 12 NADPH + 12 W + Pi v. Testes and ovaries
vi. RBCs
Summary of Shunt Pathway vii. Lens of eye
A. The oxidative phase of the pathway is seen in
Suppose, 6 molecules of glucose (6 >< 6 = 36 carbons) the above organs, where NAOPH generation is
are entering in this pathway. The first carbon atoms of all required for lipid synthesis or steroid synthesis.
6 glucose molecules are removed as 6 molecules of CO2 • B. The non-oxidative phase is present in all tis-
(This is equivalent to complete oxidation of 1 mole- sues, and so synthesis of ribose is possible in
cule of glucose). In this process, 12 NAOPH are gene- all tissues of the body.
rated. The remaining 6 molecules of 5-carbon pentoses
(6 x 5 = 30C) are interchanged in such a way that Generation of Reducing Equivalents
molecules of glucose (5 >< 6 = 30C) are regenerated. The major metabolic role of the pathway is to provide
This is summarized in Figure 10.41 . cytoplasmic NADPH for reductive biosynthesis o ~
acids..cbalesteral and steroids (see Chapters 13 and 14).
Physiological Significance
of the Pathway ree Radical Scavenging
Free radicals (super oxide, hydrogen pemxide.},,ara-0en-
A. The oxidative phase of the pathway is seen in
ti~ ous1¥-Pmdu.ced in all caUs. These will ~ y__QW.
the above organs, where NADPH generation is
QLoteia.s~ds and. alL hiawolecuJ.es, and in turn
required for lipid synthesis or steroid synthesis. cjills..are-d cad. The fre · e inactivat
B. The non-oxidative phase is present in all tissues , by enzyme syste e superoxide · mutase (SOD),
and so synthesis of ribose is possible in all tissues peroxidase (POD) and glutathione reductase (.&?R). -(/;)
of the body. ~duced GR is regenerated with the help of NADE>H @
Please see summary in Box 10.7. (Fig. 10.42).
148 Section B: General Metabolism

SOD Glucose Macrophage and PMN (Respiratory burst)

1 l
o; +o; + 2H+ H202 + 0 2
0
POD ~·

( \
H202 2H 20
NADPH* 02
(,ed,o,d) 2 G ~ : ; - S G (o,;dOed) NADPH Oxidase
SOD MPO
--+ H20 2
I i
NADP+ o;- HCIO
+c1-
GPD • Bacteria
NADP+ NADPH Glucose
killed
Fig. 10.42: Free radical scavenging enzymes. SOD = superoxide Fig. 10.43: Generation of ROS in macrophages. MPO = myelope-
= =
dismutase; POD glutathione peroxidase; GSH glutathione; GR = roxidase
glutathione reductase; GPO = glucose-6-phosphate dehydrogenase

Erythrocyte Membrane Integrity coenzymes. Reversal of non-oxidative phase is present


in all tjssues~ rib~e could be made available.
NADPH is required by the RSC to keep the glutathione
in the reduced state (Fig. 10.42). In turn, reduced hat about ATP?
glutathione will detoxify the peroxides and free radicals
formed within the RSC (see Chapter 30). So, NADPH, ATP is neiJher utilized nqr produced by the HMP shunt
glutathione and glutathione reductase together will pathway. Cells do not use the shunt pathway for energy
preserve the integrity of RBC membrane. production .

O Prevention of Methemoglobinemia ~ Clinical Significance of Shunt Pathway

NADPH is also required to keep the iron of hemoglobin


in the reduced (ferrous) state and to prevent the accu-
mulation of methemoglobin (see Chapter 23). ~ -
moglobin cannot carry oxygen.

Detoxification of Drugs
Most of the drugs and other foreign substances are
,......a..;.;_..;;._,,.~~ .;.;.;::.::
olytic
detoxified by the liver microsomal P450 enzymes, with .:;.;,.,__i.,,. The eficiency is manifested only when exposed
the help of NADPH. to certain drugs or toxins, e.g. intake of ntimalarial

© Lens of Eye drugs. like primaqujne. Primaquin stimulates peroxide


formation Inside Rst. In GPO deficient cells, the level
MaxiCilJJlD concentration of is seen in lens of of NADPH is low; hence further production of peroxides
eye. NADPH is required for ~ c y will lead to cell lysis.

-
of lens. Similarly, in estion of toxic glycosjdes present in
f<lY,a-..beans ,may have simjlac,_ e~c avlsm). Sulfa
Macrophage Bactericidal Activity drug ~an . ad alg> QI:ecipitate thehemoly-
is will lead o jaundic fl"-5evere anemia.· The
NADPH is required for production of reactive oxygen
geographical distribution of~ deficienc corre-
species (R~) (superoxide anion radical) by macro-
lates well with the malarial f n~ lcity. The enzyme
phages to kill bacteria (Fig. 10.43).
deficiency offer re ·s o Plas odium infection
UP Availability of Ribose (see Fig. 23.10). The parasite requires reduced glu-
tathione for its survival, which will not be available in
Ribose ar.d deoxyribose are required for DNA and adequate amounts in deficiency of GPO. GPO defici-
RNA synthesis. Ribose is also necessary for nucleotide ency is reported from different parts of India.
Methemoglobinemia
GPO deficient persons will show i n c r e a s ~ -
globin in circulation, even though cyanosis may not 6e
!Jlenif~ -
C.
1amine Deficiency { 0 ,)
The transketolase reaction is measured in RBCs as
an index of the thiamine status of an individual. The
occurrence and manifestation of ermc e s orsa-
ko s s n rome encephalopath which is seen in
atcoboliC~JS1r:lG-thes& WI "1 ~wiile,_g_eficiencnis due to
a genetic defect in ttm enzymeftaAHift{'H( . .
GLUCURONIC ACID PATHWAY L
The pathway is shown in Figure 10.44.

Importance of the Glucuronic


Acid Pathway
It provides UDP lucuronic
=
id, which is the active

I ~'"1
form of glucuronjc acid . It is used for the following

purposes:
1. Conjugation of bili~in
2. Conjugation of ster~s .... - r
3. Conjugation of various drugs which will make them
more w~ soluble and more e ~ excretable.
4. Synthesis of glycosamino glycans ~ )-

Effect of Drygs
A~ it£.p(U/II ~f"c,_ pa:...
Ba r b ~. anti'pyrine atld aminopyrine wilf increase
the .u~ id pathway, leading to availability of more
glucuroni}te for conjugation purpose.

Vitamin C in Lower Animals


The enzyme - nola oxidase Is abse t in
human beings, p~ s . .9.!:!_inea pigs and ~ ts. Hence
ascorbic acid cannot be synthesized by these orga-
POLYOL PATHWAY
nisms. Hence ascorbic acid became essential in diet for Of GLUCOSE
human beings.

E~ al Pentosuria
It is one of the_....rrwmbers of the (Qarrod's tetrj'a) The
incidence is 1 in 2,500 births. It is an inborn error of
metabolism. In the pathway (Fig . 10.44), b,-xx,lulose is
converted to ~ yllJ.Lose by ~ ~mes, xylitol dehy -
Glucose when converted to sorbitol, cannot diffuse
drogenase and xylulose reductase. Absence of any
of these enzymes leads to pentosuria. L-xylulose is out of the cell easily and gets trapped there. Sorbitol is
150 Section B: General Metabol ism

mg%, Unconjugated bilirubin -8.5 mg%. AST-30.0


CHO CH 20H CH20H
U/L, ALT-35 .0 U/L, ALP-70 IU/L. LDH-10 00 U/L.
OH OH 0 Jaundice was triggered on many occasions by bacterial
HO HO HO and viral infections as well as some antibiotics. What is
OH OH OH the likely diagnosis?
OH OH OH
0
CH:z--OH CH 2-0H CHrOH • · Clinical Case Study 10.4
A 3-year-0ld child was brought to the pediatrician for com-
plaints of fever and cough. On examination, all systems
were normal and no evidence for any genetic disorder.
On the safer side, the doctor performed urine screening to
Fig . 10.45: Polyol pathway of glucose metabolism rule out an inborn error of metabolism. As a surprise, Ben-
edict's test result came as positive! Blood glucose -100
normally present in lens of eyes. But in diabetes mel- mg%, urine was negative for glucose as well as galac-
tose; Bial's test was positive. What is the likely diagnosis?
litus, when gl~cose JeveUS-.hi_gh, the soJ:bitQl-!,pn~
t@tioILalsoJoc~ s~o.!h e lens. This,ia~s to a.smo!ic What is the biochemical basis for this disease?
a mag.e_or tl1a.:
a
e nd development of cataract.
Galactitol also causes cataract (see under galactose Clinical Case Study 10.1 Answe r
metabolism).
Fructose is present in semen in large quantities. It isThe likely cause is lactose intolerance. Abdominal dis-
pr~ticed by the polyol pat~way. T~e polyol ~athway is tension and bowel cramps after intake of milk is due
active in brain and fructose Is seen In CSF. This pathway to undigested lactose. This lactose can be ultimately
"r s inactive in liver. excreted in stool giving a positive Benedict's test.
There is absence of lactase enzyme, which digests
0 lactose. This is normally produced by cells lining small
• · Clinical Case Study 10.1 intestine. Highest levels of lactase are present shortly
after birth, and this declines with age. Lactose fermenta -
A newborn baby had severe abdominal distension, sev-
ere abdominal pain, and diarrhea after being fed breast tion in the intestine produces large amounts of gas (a
mixture of hydrogen, CO2 and methane) which produces
milk. Stool analysis revealed the presence of reducing
abdominal cramps, bloating and flatulence. Resultant
sugar. What is the possible defect? What is the confirm-
elevated osmotic pressure can produce diarrhea. Symp-
atory diagnostic test? How can it be treated?
toms usually appear 30 minutes -1 hour after consum-

£ Clinical Case Study 10.2


ing milk or milk products.
Confirmatory diagnosis is made by stool acidity test.
The stool acidity test is used for infants and young chil-
A 2-year-o ld child presented with liver enlargement.
dren to measure amount of acid in stool. Undigested
On investigation, Blood sugar-SO mg%, Uric acid-10
lactose creates lactic acid and other fatty acids that can
mg%, Lactic acid-15 mg% and ketone bodies were
be detected in stool sample. Glucose also may be pre-
present. Plasma choleste rol-300 mg%. What is the
sent in stool.
likely diagnosis? What is the biochemical basis of the
Gradual introduction of small amounts of milk/milk
disorder and its treatment?
products may help adaptation in some patients. Lactose-
0 free, lactose-reduced milk, Soy milk and other products
• · Clinical Case Study 10.3 may be beneficial. Lactase enzyme or tablets can be
administered.
A 6-year-old boy presented with general weakness.
There was pallor, fatigue, shortness of breath and a
rapid heart rate. History revealed that he had prolonged
0
a Clinical Case Study 10.2 Answer
neonatal jaundice. Results at that time were as follows- The diagnosis is von Gierke's
disease (GSD Type I).
Total Bilirubin- 10.0 mg%, Conjugated bilirubin -1 .5 The important biochemical
features are hypoglycemia,
Chapter 10: Metabolic Pathways of Glucose 151

hyperlactatemia, metabolic acidosis, hyperlipidemia and 6. Energy generating steps of glycolysis are cata-
hyperuricemia. Clinical features include convulsions, lyzed by the enzymes glyceraldehyde-3-phosphate
enlargement of liver and kidneys, doll's face (rounded dehydrogenase: (NADH); 1,3-bisphosphoglycerate
cheeks due to fat deposition), growth retardation, normal kinase (ATP) and pyruvate kinase (ATP).
mental development, etc. Late complications include 7. Cori's cycle ensures efficient reutilization of lactate
renal stones, tubular defects, hypertension, changes in produced in the muscle.
skin and mucous membrane and altered platelet function 8. Energy yield per molecule of glucose in the glyco-
leading to bleeding . Renal conditions may necessitate lytic pathway under anaerobic conditions is 2 ATPs.
dialysis and transplantation. 9. Under anaerobic condition pyruvate is reduced to
No specific treatment is available. Primary goal of lactate by lactate dehydrogenase. Under aerobic
dietary therapy is to correct hypoglycemia and maintain conditions, it is oxidatively decarboxylated to acetyl-
normoglycemia by frequent small quantity of food. Intake CoA by the enzyme complex Pyruvate dehydro-
of fructose and galactose is to be limited, because they genase (PDH).
are converted to lactate, and not to glucose. Nasogastric 10. The PDH enzyme complex requires 5 cofactors for
glucose infusion, parenteral nutrition and oral admi- its activity viz. NAO•, FAD, TPP, Lipoamide and Co A.
nistration of raw cornstarch are useful options. 11 . The PDH reaction is a totally irreversible reaction.
Hence, there is no net synthesis of glucose from fat.
0 12. Key enzymes of gluconeogenesis are; pyruvate
-•• Clinical Case Study 10.3 Answer carboxylase, phosphoenolpyruvate carboxykinase,
GPO deficiency. Jaundice is hemolytic in type, kemicterus fructose-1,6-bisphosphatase and glucose-6-phos-
and anemia are present. Liver enzymes are normal and phatase.
lactate dehydrogenase levels are very high. Hemolytic 13. Major substrates for gluconeogenesis are lactate
anemic in GPO deficiency is triggered on many occasions and glucogenic amino acids.
by bacterial and viral infections as well as some antibiotics. 14. Glycogen is the storage polysaccharide of the body.
It is stored mainly in the liver and muscle.
0
8 Clinical Case Study 10.4 Answer; 15. Glycogen phosphorylase is activated by glucagon
and adrenaline, while glycogen synthase is acti-
Essential pentosuria. See chapter for biochemical basis. vated by insulin.
16. Glycogen storage diseases (GSD) are Inborn errors
LEARNING POINTS, CHAPTER 10 of metabolism. Type 1 is called van Gierke's disease.
17. The HMP shunt pathway, also known as pentose phos-
1. Deficiency of Lactase results in lactose intolerance.
phate pathway (PPP) generates NADPH required for
2. Insulin dependent GluT4 has been implicated in
reductive cytoplasmic biosynthesis of biomolecules
Type 2 diabetes mellitus.
3. Glycolysis occurs both in aerobic and anaerobic such as steroids, fatty acids and cholesterol.
conditions. Anaerobic glycolysis is the major source 18. NADPH generated as a result of HMP pathway is
of energy for muscles, when the muscle tissue lacks essential to maintain transparency of the eye lens,
oxygen. to prevent methemoglobinemia and to maintain
4. Phosphofructokinase (PFK) is the regulatory or rate erythrocyte membrane integrity.
limiting enzyme of glycolysis. It is an allosteric and 19. The pathway also provides pentose sugars (ribose
inducible enzyme. AMP is allosteric activator, while and deoxyribose) for nucleic acid synthesis.
citrate and ATP are allosteric inhibitors. 20. GPO deficiency is a common clinical condition,
5. The reaction catalysed by 1,3-bisphosphoglycerate transmitted as X linked recessive trait. Ingestion of
kinase and pyruvate kinase are examples of sub- fava beans (favism) and anti-malarials such as pri-
strate level phosphorylation. maquine precipitates the manifestations.

PART-1: ESSAY AND SHORT NOTE QUESTIONS


10-1. Describe digestion and absorption of carbohydrates.
10-2. What is the major catabolic pathway of glucose under anaerobic conditions? Mention the steps in the pathway
and indicate the key enzymes.
152 Section B: General Metabolism

10-3. Describe the process of glycolysis. Explain how many molecules of ATP are formed in anaerobic and aerobic
conditions.
10-4. Assuming that oxygen is available in plenty, what are the steps in which emergy is trapped in glycolytic pathway?
Give the substrate, product, enzyme and coenzyme of each of these em!rgy-trapping steps.
10-5. How is glucose metabolized by the RBCs for producing energy? Indicate the energy yield from 1 molecule of
gl ucose in the RBCs.
10-6. In anaerobic glycolysis, lactic acid is generated. What is the reason for reduction of pyruvate to lactate?
10-7. What are the irreversible steps in glycolysls? How are these blocks circumvented?
10-8. Trace the pathway of gluconeogenesis starting from alanine. Mention thie key enzymes.
10-9. " Gluconeogenesis is not a simple reversal of glycolysis"; justify your ainswer by giving the reactions.
10-10. How is glycogen broken down in the body? Explain the hormonal regulation of the pathway.
10-11 . Outline the sequence of reactions involved in the breakdown of glyco1~en in the skeletal muscle. Explain the
purpose served by this process.
10-12. Write the reactions of the oxidative phase of the hexose monophosphate shunt pathway. Which tissues have
this pathway?
10-13. What is the significance of HMP shunt pathway?

SHORT NOTE QUESTIONS


10-14. Regulation of glycolysis. 10-24. Action of g llucagon on glycogenolysis.
10-15. Key enzymes of glycolysis. 10-25. Cyclic AMP.
10-16. Phosphofructokinase. 10-26. von Gierke's disease.
10-17. Substrate level phosphorylation. 10-27. Glycogen storage diseases.
10-18. 2,3-bisphosphoglycerate (2,3-BPG). 10-28. Signiftcanc,e of HMP shunt pathway.
10-19. Cori cycle. 10-29. Transketola1se.
10-20. Substrates for gluconeogenesis. 10-30. Key enzym1! of HMP shunt pathway.
10-21. Regulation of gluconeogenesis. 10-31 . Drug induc,ed hemolytic anemia.
10-22. Key enzymes of gluconeogenesis. 10-32. UDP-glucuronic acid.
10-23. Malate shuttle.

PART-2: MULTIPLE CHOICE QUE TIONS


10-1. Glucose is absorbed by gastrointestinal cells by: 10-5. An example! of substrate level phosphorylation is:
A. Passive diffusion A. Phosphoglycerate kinase
B. Carrier mediated co-transport system, along with B. Enolase
sodium C. Pyruvatei kinase
C. Carrier mediated antiport system, along with pota- D. Glyceraldehyde-3-phosphate dehydrogenase
ssium 10-6. All the dehydrogenases listed below are NAO•
D. Needs no ATP dependent, except
10-2. The product of glycolysis in erythrocytes is: A. Lactate dehydrogenase
A. NADPH B. Glucose--6-phosphate dehydrogenase
B. Lactate C. Pyruvate! dehydrogenase
C. Pyruvate D. Glyceral,dehyde-3-phosphate dehydrogenase
D. Carbon dioxide 10-7. Which enzymes glycolytic pathway is inhibited by
10-3. Glucokinase is more active after a meal, because: fluoride iorns?
A. It is an inducible enzyme A. Hexokinase B. Phosphofructokinase
B. It has more affinity to glucose than hexokinase C. Aldolase D. Enolase
C. It is present in all tissues 10-8. As the end product of glycolysis, pyruvate and
D. Can act on all monosaccharides NADH are formed. During anaerobiosis, the NAO•
10-4. Which enzyme catalyzes an irreversible reaction? Is regenerated by:
A. Transketolase A. Glyceraldehyde-3-phosphate dehydrogenase
B. Phosphofructokinase B. Oxygen
C. Aldolase C. Glutamaite dehydrogenase
D. Glyceraldehyde-3-phosphate dehydrogenase D. Lactate dehydrogenase
Chapter 10: Metabolic Pathways of Glucose 153

10.9. As the end product of glycolysis, pyruvate and C. Pyruvate kinase


NADH are formed. During aerobic conditions, the D. Lactate dehydrogenase
NAD• is regenerated by: 10-19. Malate shuttle is necessary for:
A. Glyceraldehyde-3-phosphate dehydrogenase A. Gluconeogenesis B. Providing NADPH
B. Oxygen C. ATP generation D. Glycogenolysis
C. Glutamate dehydrogenase 10-20. Gluconeogenesis is inhibited by:
D. Lactate dehydrogenase A. Glucagon B. Growth hormone
10-10. All the enzymes listed below are regulatory enzy- C. Insulin D. Glucocorticoids
mes, except 10-21 . All the following are key gluconeogenic enzymes,
A. Glycogen phosphorylase except:
B. Glucose-6-phosphate dehydrogenase A. Pyruvate carboxylase
C. Pyruvate kinase B. Phosphoenolpyruvate carboxy kinase
D. Lactate dehydrogenase C. Phosphofructokinase
10-11. ATP is generated in all the reactions, except D. Glucose--6-phosphatase
A. Glyceraldehyde-3-phosphate dehydrogenase 10-22. Blood glucose level can be raised by gluconeo-
B. 1,3-bisphosphoglycerate kinase genesis only by liver because:
C. Pyruvate kinase A. Glucokinase is present exclusively in liver
D. Hexokinase B. Glucose-6-phosphatase is active only in liver
10-12. The key enzyme of glycolysis is: C. Transaminases are present in liver
A. Glucose-6-phosphatase D. Liver is the site of storage of glucose as glycogen
B. Glyceraldehyde-3-phosphate dehydrogenase 10-23. How many ATP molecules are required to convert
C. Phosphohexose isomerase 2 molecules of pyruvate into glucose?
D. Phosphofructokinase A. Two B. Three
10-13. Catalytic activity of phosphofructokinase is increa- C. Six D. Eight
sed by all the following, except: 10-24. All the following are substrates for gluconeoge-
A. AMP nesis, except
B. Fruclose-2,6-bisphosphate A. Palmitic acid B. Lactic acid
C. ATP C. Alanine D. Glycerol
D. Fructose--6-phosphate 10-25. All the following statements are true with regard to
10-14. Complete oxidation of one molecule of glucose glucokinase, except
yields how many ATPs? A. It has lower affinity for glucose than hexokinase
A. 12 B. 24 B. It is found only in liver
C. 32 D. 129 C. It is induced by insulin
10-15. Rapaport Leubering Cycle in RBC produces: D. It phosphorylates glucose under conditions of fasting
A. More ATP than produced in Embden-Meyerhof 10-26. Which glycolytic enzyme uses inorganic phosphate
pathway for the reaction?
B. More NADPH than HMP shunt pathway A. Pyruvate kinase
C. 2,3-BPG which enhances oxygen dissociation B. 1,3-bisphosphoglycerate kinase
D. 1,3-BPG which contains a high energy bond C. Glyceraldehyde-3-phosphate dehydrogenase
10-16. All the following coenzymes are Involved in the D. Hexokinase
pyruvate dehydrogenase reaction, except 10-27. All will inhibit phosphofructokinase, except.
A. Thiamine pyrophosphate (TPP) A. Fructose-2,6-bisphosphate
B. Biotin B. ATP
C. NAD• C. Cyclic AMP
D. FAD D. Citrate
10-17. There is no net synthesis of glucose from fatty 10-28. 2,3-bisphosphoglycerate is:
acids, because of the irreversible nature of the A. Binding to foetal hemoglobin
reaction:
B. A normal intermediate of glycolytic pathway
A. Pyruvate to oxaloacetate C. A high energy compound
B. Pyruvate to acetyl-CoA D. Needed for triglyceride formation
C. Phosphoenolpyruvate to pyruvate 10-29. All the following statements are true with regard to
D. Oxaloacetate to phosphoenolpyruvate gluconeogenesis, except:
10-18. Pyruvate is converted to acetyl-CoA by: A. It maintains blood glucose level during fasting
A. Pyruvate dehydrogenase B. Allows use of proteins as an alternate source of
B. Pyruvate carboxylase energy
154 Section B: General Metabolism

C. There is net conversion of acetyl-CoA to glucose 10-40. The HMP shunt pathway is important for all the
during gluconeogenesis following, except
D. Glycerol part of fat forms a minor source for gluco- A. Generation of ATP
neogenesis B. Fatty acid biosynthesis
10-30. Pyruvate can be directly converted to all the follow- C. Synthesis of reduced glutathione
ing, except D. Synthesis of ribose
A. Alanine B. Lactate 10-41. All are true with regard to NADPH, except:
C. Oxaloacetate D. Phosphoenolpyruvate A. Generated by glucose-6-phosphate dehydroge-
10-31. Gluconeogenesis is taking place in : nase
A. Brain B. Liver B. Used for reductive biosynthesis of fatty acids
C. RBC D. Muscle C. Produces peroxides and free radicals in RBC
10-32. Both glycolysis and gluconeogenesis pathways D. Preserves integrity of RBC membrane
utilize all the following enzymes, except: 10-42. The hormone activating the enzyme phosphory-
A. Phosphofructokinase lase is:
B. Glyceraldehyde-3-phosphate dehydrogenase A. Epinephrine B. Insulin
C. 1,3-bisphosphoglycerate kinase C. Growth hormone D. Glucocorticoids
D. Phosphoglucomutase 10-43. Von Gierke's disease is characterized by the defi-
10-33. Lactic acidosis is seen in the following cases, except ciency of:
A. OXPHOS Diseases A. Glucose-6-phosphatase
B. Defective metabolism of pyruvate B. Glyceraldehyde-3-phosphate dehydrogenase
C. Excessive ingestion of alcohol C. Phosphofructokinase
D. Deficiency of glucose-6-phosphate dehydrogenase D. Phosphorylase
10-34. Which of the following metabolic intermediates is 10-44. McArdle's syndrome is due to the deficiency of:
not generated from glucose? A. Glucose-6-phosphatase
A. Dihydroxy acetone phosphate B. Glucose-6-phosphate dehydrogenase
B. 2,3-bisphosphoglycerate C. Muscle glycogen phosphorylase
C. Beta hydroxybutyrate D. Debranching enzyme
D. Glycerol-3-phosphate 10-45. Glycogen is:
10-35. Which reactions does not occur in the liver? A. Having unbranched or straight line structure
A. Acetoacetate to acetoacetyl-CoA B. Hydrolyzed to equal quantity of glucose and galac-
B. Hydroxylation of Cholecalciferol tose
C. Glucose to UDP glucose C. Hydrolysed by phosphorylase
D. Oxaloacetate to phosphoenol pyruvate D. Seen in brain
10-36. Transport of glucose from Intestinal lumen to 10-46. One glycosyl residue in glycogen is converted to 2
mucosal cell: lactic acid molecules, the net yield of ATP is:
A. Requires hydrolysis of ATP A. Two B. Three
B. Occurs by facilitated diffusion C. Four D. Thirty eight
10-47. Which enzyme is active in the phosphorylated state:
C. Is sodium dependent
D. Occurs by an antiport system A. Glycogen phosphorylase
10-37. Which enzyme generates NADPH? B. Pyruvate kinase (PK)
A. Pyruvate carboxylase C. PFK-2 (phosphofructokinase-2)
D. Glycogen synthase
B. Pyruvate dehydrogenase
10-48. Glycogen synthase is activated by:
C. Glucose-6-phosphate dehydrogenase
D. Lactate dehydrogenase A. Insulin B. Protein kinase
10-38. Transketolase activity is decreased in the defi- C. Adrenaline D. ATP
10-49. Wh ich enzyme uses NADP as coenzyme?
ciency of:
A. Pyruvate carboxylase
A. Thiamine pyrophosphate (TPP)
B. Pyruvate dehydrogenase
B. Nicotinamide adenine dinucleotide
C. Glucose-6-phosphate dehydrogenase
C. Flavin adenine dinucleotide
D. Lactate dehydrogenase
D. Pyridoxal phosphate
10-50. Glucose-6-phosphate dehydrogenase enzyme:
10-39. Deficiency of glucose-6-phosphate dehydrogenase
A. Is a member of glycolytic pathway
causes :
B. Is necessary for production of NADPH
A. Cataract B. Hemolytic anemia C. Requires the coenzyme ATP
C. Hypoglycemia D. Mental retardardation D. Is inhibited by insulin
Chapter 10: Metabolic Pathways of Glucose 155

10-51. Transketolase reaction ; 10-61. All are true regarding sorbitol, except:
A. Takes place between xylulose phosphate and A. Deficient production in diabetes mellitus
ribose phosphate B. Osmotically active
B. Occurs between glyceraldehyde phosphate and C. Used as a sweetner
fructose phosphate D. May be oxidsed to fructose
C. Needs pyridoxal phosphate as coenzyme 10-62. Alpha 1,4 linkages are hydrolysed by:
D. Occurs between glyceraldehyde phosphate and A. Lactase 8 . Sucrose
sedoheptulose phosphate C. Maltase D. lsomaltase
10-52. Hemolytic episode after administration of anti 10-63. Absorption of glucose from intestinal lumen to
malarial drug is due to the deficiency of enzyme: bloodstream involves all the following transport
A. 6-phosphogluconate dehydrogenase mechanisms, except
B. Glucose-6-phosphate dehydrogenase A. Cotransport
C. Glucose-6-phosphatase B. Active transport
D. Hexokinase C. Facilitated diffusion
10-53. Oxidative phase of HMP pathway produces all, D. Passive diffusion
except 10-64. Lactose intolerance is:
A. CO2 B. Pentose phosphates A. Seen only in newborns
C. ATP D. NADPH B. Due to relative deficiency of lactase
10-54. NAOPH is requ ired for all, except: C. Does not respond the lactose free diet
A. Phagocytic cells for respiratory burst D. Requires replacement of lactase
B. Drug hydroxylation 10-65. Which of the following tissues can derive energy
C. Substrate for electron transport chain only from glucose?
D. Detoxification of Reactive oxygen species A. Brain
10-55. Which of the following is not an intermediate of B. Cardiac muscle
HMP pathway? C. Erythrocytes
A. Fructose-6-phosphate D. Adipose tissue
B. Erythrose-4-phosphate 10-66. Insulin dependent uptake of glucose is seen in:
C. 2,3 bisphosphoglycerate A. Liver b. Brain
D. Ribose-5-phosphate C. Skeletal muscle D. Erythrocytes
10-56. Which of the following coenzymes is required for 10-67. Which of the intermediates is formed only in RBCs?
the HMP pathway? A. 1,3-BPG B. 2,3-BPG
A. PLP B. Biotin C. PEP D. 3-PG
C. FAD D. TPP 10-68. Which of the enzymes listed catalyzes a substrate
10-57. Which of the following is NOT true regarding the level phosphorylation?
HMP pathway? A. Phosphofructokinase
A. Glucose-6-phosphate is shunted through the HMP B. Hexokinase
pathway to glycolysis C. Pyruvate kinase
B. Used for energy production D. Glucokinase
C. Ribose-5-phosphate is produced for nucleotide 10-69. Lactate is the product of glycolysis in RBCs
synthesis because:
D. Pathway is essential for the erythrocytes. A. RBCs can convert lactate to glucose
10-58. HMP pathway is essential for all, except B. Formation of lactate produces more energy
A. Transparency of lens C. NAO' is regenerated in the cytoplasm
B. Integrity of RSC membrane D. Accumulation of lactate activates glycolysis.
C. Reduce methemoglobin level 10-70. Lactic acidosis can result from deficiency of:
D. Provide 2,3-BPG for RBCs A. Lactate dehydrogenase
10-59. Deficiency of HMP pathway can cause all, except 8. Pyruvate dehydrogenase
A. Hemolytic anemia C. Pyruvate kinase
B. Methemoglobinemia D. Phosphoenolpyruvate carboxykinase
C. Chronic granulomatous disease 10-71 . Which of the following cannot be converted back
D. Thromboembolic episodes to glucose?
10-60. Which of the following drugs can precipitate GPO A. Acetyl-CoA
deficiency? B. Lactate
A. Primaquin B. Aspirin C. Alanine
C. Statin D. Penicillin D. Propionyl-CoA
156 Section B: General Metabolism

10-72. Which of the following is not true regarding g luco - 10-73. Under conditions of starvation, the enzyme which
neogenesis? is inhibited is:
A. Requires ATP A. Glucose-6-phosphatase
B. Inhibited by a high NADH/NAD• ratio B. Alanine aminotransferase
C. Takes place in liver C. Glucokinase
D. Activated by insulin D. Pyruvate carboxylase

ANSWERS OF MULTIPLE CHOICE QUESTIONS


10-1. B 10-2. B 10-3. A 10-4. B 10-5. D 10-6. B 10-7. D
10-8. D 10-9. B 10-10. D 10-11. D 10-12. D 10-13. C 10-14. C
10-15. C 10-16. B 10-17. B 10-18. A 10-19. A 10-20. C 10-21. C
10-22. B 10-23. C 10-24. A 10-25. D 10-26. C 10-27. A 10-28. C
10-29. C 10-30. D 10-31. B 10-32. A 10-33. D 10-34. C 10-35. D
10-36. C 10-37. C 10-38. A 10-39. B 10-40. A 10-41. C 10-42. A
10-43. A 10-44. C 10-45. D 10-46. B 10-47. A 10-48. A 10-49. C
10-50. B 10-51 . A 10-52. B 10-53. C 10-54. C 10-55. C 10-56. D
10-57. B 10-58. D 10-59. D 10-60. A 10-61. A 10-62. C 10-63. D
10-64. B 10-65. C 10-66. C 10-67. B 10-68. C 10-69. C 10-70. B
10-71. A 10-72. D 10-73. C

PART-3: VIVA VOCE QUESTIONS AND ANSWERS

10-1 . Glucose is absorbed at the luminal side of gastro- 10-10. What is the difference between glucokinase and
intestinal cells by which mechanism? hexokinase?
Carrier mediated co-transport with sodium, named as Both add phosphate group to glucose. But glucokinase
Sodium dependent glucose transporter (SGluT). is present only in liver: acts specifically on glucose,
10-2. How glucose is released from intestinal cells into and is active when glucose level in blood is increased
the bloodstream? after food intake.
Glucose transporter type 2 {GluT2). 10-11. Which tissues prefer anaerobic glycolysis?
10-3. How glucose is taken up by cells? RBCs, exercising muscle, and cancer cells.
In tissues GluT2 helps in absorption of glucose from 10-1 2. During glycolysis, energy is produced during
blood. which steps?
10-4. What is the importance of GluT4? Step 5, Glyceraldehyde-3-phosphateto 1,3-bisphospho-
It is the glucose transporter present in muscle and glycerate; Step 6. 1,3-bisphosphoglycerate to 3-phos-
adipose tissue. phoglycerate; and Step 9, Phosphoenolpyruvale lo
10-5. What is its clinical significance? pyruvate.
Insulin induces these transporters. In diabetes mellitus, 10-13. Fluoride ions inhibit which enzyme?
entry of glucose into muscle is decreased, because Enolase (step 8).
GluT4 is reduced. 10-14. What is the clinical significance of the above inhi-
10-6. What is glycolysis? bition?
In this pathway, glucose is converted to pyruvate or Fluoride is used to prevent glycolysis , as preservative
lactate, along with production of a small quantity of for blood before glucose estimation.
energy. 10-15. In glycolysis, NAO• is reduced to NADH in wh ich
10-7. In which condition pyruvate is produced? reaction?
In aerobic condition pyruvate is produced. Glyceraldehyde-3-phosphate dehydrogenase reaction.
10-8. In which condition lactate is produced? 10-16. What are substrate level pt,osphorylation steps in
When oxygen is lacking, lactate is produced. glycolysis?
10-9. What is the significance of glycolysis? 1,3-bisphosphoglycerate kinase (step 6) and pyruvate
Anaerobic glycolysis forms the major source of energy kinase (step 9).
in actively contracting muscles. Moreover, glycolysis is 10-17. What is the purpose of lactic acid production under
the only source of energy in RBCs. anaerobic conditions?
Chapter 1O: Metabolic Pathways of Glucose 157

NADH generated in the 5th step has to be oxidized to Pyruvate carboxylase; Phosphoenolpyruvate carboxy
NAO'. This can be done by oxygen. But when oxygen kinase; Fructose-1 ,6-bisphosphatase and Glucose-6-
is lacking, the 5th step has to be coupled with the 10th phosphatase.
step for regeneration of NAO'. 10-34. Malate shuttle is used for what purpose?
10-18. What is Cori cycle (or lactic acid cycle)? Reactions of gluconeogenesis are taking place in
During exercise, lactate is produced in muscle. This cytosol. Hence the oxaloacetate has to be transported
lactate diffuses into the blood. Lactate then reaches from mitochondria to cytosol. This is done by malate
liver, where it is oxidized to pyruvate. It is converted shuttle.
to glucose. This glucose can enter into blood and then 10-35. Gluconeogenesis is seen in which tissue?
transported to muscle. Liver.
10-19. What is the purpose of Cori cycle? 10-36. How many ATP molecules are required to convert
By this means, the lactate is efficiently reutilized. 2 molecules of pyruvate into glucose?
10-20. What are the key glycolytic enzymes? Six.
Glucokinase; phosphofructokinase; pyruvate kinase 10-37. Blood glucose level can be raised by gluconeo-
10-21. What is the action of insulin on glycolysis? genesis only by liver, why?
Insulin stimulates glycolysis. Glucose-6-phosphatase is present only in liver.
10-22. What is the energy yield from glycolysis during 10-38. Muscle glycogen will not serve as a precursor of
anaerobic conditions? blood sugar, why?
2ATP. Glucose-6-phosphatase is absent in muscle.
10-23. In aerobic glycolysis, the net yield from one glu- 10-39. Which amino acids are both ketogenic and gluco-
genic?
cose molecule is how much?
Tyrosine and Tryptophan.
?ATP.
10-40. What hormone will inhibit gluconeogenesis?
10-24. During complete oxidation, what is the net yield of
Insulin.
ATP from one glucose molecule?
10-41. What hormones will stimulate gluconeogenesis?
32ATP
Glucagon and glucocorticoids.
10-25. How many ATPs are generated per one rotation of
10-42. What is the significance of gluconeogenesis?
the citric acid cycle?
Gluconeogenesis is necessary to maintain blood glu-
10ATP.
cose level, especially under conditions of starvation.
10-26. What is function of 2,3-bisphosphoglycerate?
10-43. What is the key enzyme of glycogenolysis?
It reduces the affinity of hemoglobin toward oxygen.
Glycogen phosphorylase.
10-27. What are the steps in which carbon dioxide is
10-44. Which hormones enhance glycogenolysis?
released from a glucose molecule?
Adrenaline and glucagon causes glycogenolysis
Pyruvate dehydrogenase; lsocitrate dehydrogenase;
10-45. What is the mechanism of action of adrenaline?
Alpha ketoglutarate dehydrogenase. Adrenaline increases cyclic AMP level which activates
10-28. What are the coenzymes necessary for oxidative glycogen phosphorylase
decarboxylation of pyruvate? 10-46. What will activate glycogen phosphorylase?
Thiamine pyrophosphate; NAD•; FAD; Lipoic acid; Epinephrine, Glucagon, Cyclic AMP.
CoenzymeA. 10-47. What is cyclic AMP (cAMP)?
10-29. There is no net synthesis of glucose from fatty It is a second messenger.
acids, why? 10-48. How cyclic AMP is produced?
Pyruvate to acetyl-CoA is a totally irreversible reaction. Adenyl cyclase enzyme acts on ATP to produce cAMP.
10-30. Which enzyme irreversibly channels glucose to 10-49. Which is the defective enzyme in von Gierke's
energy production? disease (glycogen storage disease type I)?
Pyruvate dehydrogenase is the irreversible reaction. Glucose-6-phosphatase.
10-31. What is gluconeogenesis? 10-50. What is the characteri stic clinica l features of von
Production of glucose from non-carbohydrate sources. Gierke's disease?
10-32. What are those non-carbohydrate sources? (What Fasting hypoglycemia, which does not respond to
are the substrates for gluconeogenesis?) adrenaline is very characteristic.
Glucogenic amino acids and lactate. 10-51. Which enzyme generates NADPH?
10-33. What are the key gluconeogenic enzymes? Glucose-6-phosphate dehydrogenase
158 Section 8: General Metabolism

10-52. Which is the key enzyme ofhexose monophosphate 10-60. What is the manifestation of glucose-6-phosphate
shunt pathway? dehydrogenase deficiency?
Glucose-6-phosphate dehydrogenase. Drug-induced hemolysis.
10-53. What are the tissues in which HMP shunt pathway 10-61. Acute hemolytic episode after administration of
is significant? antimalarial drug is due to what?
Liver, adipose tissue, RBC, adrenal cortex, ovary, Deficiency of glucose-6-phosphate dehydrogenase.
testis, mammary gland, lens. 10-62. What is the most common enzyme deficiency?
10-54. What hormone controls HMP shunt pathway? Glucose-6-phosphate dehydrogenase deficiency.
Insulin stimulates the pathway by activating the key 10-63. What is clinical significance of transketolase?
enzyme. The transketolase available in RBCs Is an index of the
10-55. What is the purpose of HMP shunt pathway? thiamine status of an individual.
It generates NADPH. 10-64. When is transketotase activity decreased?
10-56. What is the use of NADPH in biological systems? Deficiency of Thiamine pyrophosphate (TPP).
For reductive biosynthesis. 10-65. What is the purpose of uronic acid pathway?
10-57. Which synthetic pathways need NADPH? It is used for conjugation of bilirubin; steroids, synthesis
Fatty acid biosynthesis; synthesis of cholesterol, ste- of glucosaminoglycans.
raid hormones. 10-66. In lower animals, uronic acid pathway is used for
10-58. Apart from reductive synthesis, NADPH is used for what?
what purpose? For synthesis of ascorbic acid (vitamin C).
It is necessary to keep the integrity of RBC membrane; 10-67. What is essential pentosuria?
for keeping transparency of lens; and for superoxide Excretion of pentose (L-xylulose) in urine.
production inside macrophages. 10-68. What is it due to?
10-59. Is NADPH used for ATP generation? Due to the deficiency of xylulose reductase or xylulose
No. NADPH is not used for ATP generation. reductase.
_ _ _ _ _ Chapter 11
Regulation of Blood Glucose,
Insulin and Diabetes Mellitus

Chapter at a Glance

The learner will be able t o answer q uestions on t he following topics:

D Factors mainta ining blood glucose D Insulin, synthesis and secretion


D Normal plasma glucose level D Phys iologica l action of insulin
D Effect s of horm ones on glucose level D Glucagon
D Oral glucose to lerance test {OGTT) D Diabetes mellitus types
D Diagnostic criteria for diabetes mellitus D M etabolic derangements in diabet es
D Impaired glucose tolerance D Clinical aspects of diabetes mellitus
D Reducing substances in urine D Laboratory investigations in diabet es
D Benedict 's t est D Glycat ed hemoglobin

Historical Perspectives
The word "insulin" is derived from Latin, insula, meaning island (islet). In 1869, Langerhans identified the alpha and beta cells
in islets of pancreas. In 1889, von Mering and Minkowski produced experimental diabetes by pancreatectomy. In 1922, Banting
and Best extracted insulin from pancreas. Insulin was the first hormone to be isolated in a pure form. They injected the extract
to a diabetic dog, Marjorie, who was kept alive by regular insulin injections. For this work Banting was awarded Nobel Prize in
1923. But Best was deleted in the list. As a compensation, Banting declared that half his share of the prize will go to Best. In
1954 , Sanger studied the amino acid sequence of insulin. For this work Sanger got Nobel Prize in 1958.
The term "diabetes" is derived from the Greek words dia {= through), bainein (= to go) and diabetes literally means pass
through. The disease causes loss of weight as if the body mass is passed through the urine. The Greek word, mellitus, means
sweet, as it is known to early workers, that the urine of the patient contains sugar. Diabetes mellitus is a disease known from
very ancient times. Charaka in his treatise (circa 400 BC) gives a very elaborate clinical description of madhumeha (= sweet
urine). He had the vision that carbohydrate and fat metabolisms are altered in this disease. In Western literature, Thomas Willis
in 1670 noticed the sweet taste of diabetic urine. In 1838, Bouchardt and Peligot proved that the sugar of diabetic urine is the
same as that present in grape sugar. A crude test for urine sugar was first developed by Trommer in 1841. Qualitative test for
urine sugar was perfected by Hermann Fehling (1848) and semi-quantitative test by Stanley Benedict (1908). Folin in 1919
identified a method for quantitative determination of sugar in blood .

REGULATION OF brain has an obligatory requirement for glucose. Factors


maintaining the blood glucose are shown in Box 11 .1
BLOOD GLUCOSE
and Figures 11 .1 and 11.3.
Glucose level in blood is maintained within narrow limits.
This is a very finely and efficiently regulated system. It Postprandial Regulation
is essential to have continuous supply of glucose to the Following a meal, glucose is absorbed from the intestine
brain. It can utilize ketone bodies to some extent, but and enters the blood. The rise in the blood glucose level
160 Section B: General Metabolism

i
BOX 11 .1 : F.ic-" r" rn2,~t,i1r·111,1 l,lo,)cJ sugar High blood
1. The plasma glucose level at an instant depends on the balance
between glucose entering and leaving the extracellular fluid
Lowblood
gl,rose I!!! glucose

2. Hormones maintain this balance (Fig. 11 .1 )


Pancreas
3. The major factors which cause entry of glucose into blood are:

~'""''":J ~ @i;"
Alpha cells Beta cells
a. Absorption from intestines
b. Glycogenolysis (breakdown of glycogen)
c. Gluconeogenesis
d. Hyperglycemic hormones (glucagon, steroids)
4. Factors leading to depletion of glucose in blood are:
Glycogenolys~·s Peripheral tissue
a. Utilization by tissues for energy in liver; cells take
b. Glycogen synthesis glucose glucose from
released to blood blood
c. Conversion of glucose into fat (lipogenesis)
d. Hypoglycemic hormone (insulin) Nonna! blood glucose level attained

Fig. 11 .1: Homeostasis of blood glucose

i
Liver Liver Glycogen synthesis
Muscle
Gluconeogenesis
;;~~~:':'°'
Glycogenoly~ r
Glucose Glucot i
FFA

Intestine Adipose tissue Intestine Adipose tissue

Fig. 11.2A: Blood glucose regulation during fasting state (high Fig. 11.28: Blood glucose regulation during postprandial state
Glucagon). In fasting state, blood glucose level is maintained (high insulin). In postprandial state, glucose level is high; then
by glycogenolysis and gluconeogenesis; further, adipose tissue blood glucose level is lowered by tissue utilization, glycogen syn-
releases free fatty acids as alternate source of energy. Red arrows thesis and lipogenesis. Red arrows indicate activation; blue arrow
indicate activation; blue arrow indicates inhibition indicates inhibition

stimulates the secretion of insulin by the beta ce lls of that supplies the glucose for maintaining blood gluco se
islets of Langerhans of pancreas. The uptake of g lucose level (Fig. 11 .1 ). Hormones like glucagon, epinephrine,
by extrahepatic tissues, except brain is dependent on glucocorticoids, growth hormone, ACTH and thy roxine
insulin . Moreover, insulin helps in the storage of g lucose will tend to inc rease the blood glucose level. They a re
as glycogen or its conversion to fat (Fig . 11 .2A) referred to as anti-insulin hormones or hyperglycemic
hormones. An overview of the regu latory mechanism is
Regulation in Fasting State s hown in F igure 11.3. Effects of hormones are shown in
Normally, 2 to 2½ hours after a mea l, the blood glu- Box 11 .2.
cose level falls to near fasting levels. It may go down
further; but this is prevented by processes that contri-
Determination of Glucose in Body Fluids
bute glucose to the blood. Fo r another 3 hours , hepatic Estimation of glucose is the most common analysis
glycogenolysis will take care of the blood sugar level. done b clinical laboratories. The blood is collected using
Thereafte r, gluconeogenesis will take c harge of the an anticoagulant (potassium oxalate) and an inhib ito r of
situation (Figs. 11.2A and B). Liver is the major o rgan glycolysis (sodium fluoride).
Chapter 11 : Regulation of Blood Glucose, Insulin and Diabetes Mellitus 161

Hyperglycemlc Factors Hyperglycemic Hormones I Hypoglycemic Factors


(Sources of blood glucose) Glucagon/Adrenahn {F~~ removing glucese
Corticosteroids from blood)
Growth hormone
ACTH, Thyroxine
Glycolysis in all cells; and
Absorption from GIT
TCA cyde in most cells; Glucose to CO2
(Starch to glucose)
and water
Pl.ASMA GLUCOSE
-----~
Fasting: ~110 mgldL l • Glycogen synthesis in liver and skeletal
Glycogenolysis in liver - 2 hours poatprandlal: < 1.0 mgldL
(Glycogen to glucose) •
~ m,~I•

Gluconeogenesis in liver /
Hypoglycemic Hormone Lipogenesis (synthesis of
(Amino acids to glucose)
Insulin fatty acid and fat deposit)

F ig. 11 .3: Overview of regulation of blood sugar

BOX 11 .2: Effects of hormones on glucose level in blood (glucose oxidase peroxidase) method is most commonly
A. Effect of insulin (hypoglycemic hormone) used to assess the blood glucose level. The reaction
1. Lowers blood glucose generates a color, which is read in a photometer. The
2. Favors glycogen synthesis newer automated systems use hexokinase method.
3. Promotes glycolysis The above GOD reaction mixture is immobilized on
4. Inhibits gluconeogenesis a plastic film (dry analysis). The intensity of the color is
B. Glucagon (hyperglycemic hormone)
measured by reflectance photometry. The instrument is
1. Increases blood glucose
named as glucometer. It is useful for patients to have
2. Promotes glycogenolysis
3. Enhances gluconeogenesis self-analysis at home. But the instrument is less accurate.
4. Depresses glycogen synthesis
5. Inhibits glycolysis (Details given below) Commonly Employed Terms
C. Cortisol (hyperglycemic hormone) Regarding Glucose
1. Increases blood sugar level
1. Blood sugar analyzed at any time of the day, without
2. Increases gluconeogenesis
3. Releases amino acids from the muscle
any prior preparations, is called random blood sugar.
D. Epinephri1 ..: or Adrenaline (hyperglycc:-:iic) 2. Glucose estimated in the early morning, before tak-
1. Increases blood sugar level ing any breakfast is called fasting blood glucose.
2. Promotes glycogenolysis Fasting state means, glucose is estimated after an
3. Increases gluconeogenesis overnight fast (12 hours after the food) (postab-
4. Favors uptake of amino acids
sorptive state).
E. Growth hormone (hyperglycemic)
3. The test done about 2 hours after a good meal is
1. Increases blood sugar level
2. Decreases glycolysis
called postprandial blood glucose (Latin = after
3. Mobilizes fatty acids from adipose tissue food).
4. When blood glucose level is within normal limits, it
Fluoride inhibits the enzyme, enolase, and so glyco- is referred to as normoglycemia. When values are
lysis on the whole is inhibited . If fluoride is not added, above the normal range, it is known as hypergly-
cells will utilize glucose and fa lse low value may be cemia. When values are below the normal range,
obtained. Capillary blood from finger tips may also be it is called hypoglycemia. (Greek, hyper= above;
used for glucose estimation by strip method. hypo = below).
5. When the blood glucose is below 50 mg/di, it is a
Enzymatic Method very serious condition. Hyperglycemia is harmful in
This is highly specific, giving 'true glucose' values (fasting the long run; while hypoglycemia even for a short
70-110 mg/dl). In the medical laboratory, the GOD-POD while is dangerous, and may even be fatal.
162 Section B: General Metabolism

TABLE 11.1: The plasma sugar levels in OGTT In normal BOX 11 .3: D1agnost1c cntena for diabetes melhtus
persons and In d1abet1c patients
1. If the fasting plasma sugar is more than 126 mg/dl, on more
Criteria for Criteria for than one occasion (Table 11 .1)
Normal persons diabetes diagnosing /GT 2. Or, if 2-hour post-glucose load value of OGTT is more than
Fasting < 110 mg/dl > 126mg/dL 110 to 200 mg / dl (even at one occasion)
126 mg/ dl 3. Or, if both fasting and 2-hour values are above these levels,
1 hr (peak) < 160 mg/dL Not prescribed Not prescribed on the same occasion
after glucose 4. If the random plasma sugar level is more than 200 mg/dl, on
2 hr after < 140 mg/dL > 200mg/dL 140to more than one occasion. Diagnosis should not be based on a
glucose 199 mg/ dl sing le random test alone; it should be repeated
5. Glycated hemoglobin (Glyco-Hb) or HbA 1c level more
than 6.5% at any occasion. As per the recommendations of
6. The ability of a person to metabolize a given load of American Association of Clinical Chemistry and American
glucose is referred to as glucose tolerance. Diabetes Association, HbA 1c level is the preferred method
for initial diagnosis of diabetes mellitus.
Conducting the Glucos e Tolerance Test
At about 8 am, a sample of blood is collected in the fast- Classical Oral Glucose
ing state. Urine sample is also obtained. This is denoted Tolerance Test (OGTT)
as the "O" hour sample.
Glucose tolerance test is artificial, because in day-to-day
Glucose load dose: The dose is 75 g anhydrous
life, such a large quantity of glucose does not enter into
glucose (82.5 g of glucose monohydrate) in 250-300ml
blood. However, the GTT is a well-standardized test,
of water. This dose is fixed for an adult, irrespective of
and is highly useful to diagnose diabetes mellitus in
body weight. (When the test is done in children , the
doubtful cases.
glucose dose is adjusted as 1.75 g/kg body weight).
In order to prevent vomiting, patient is asked to drink it
Indications for OGTT
slowly (within about 5 minutes). Flavoring of the solution
will also reduce the tendency to vomit. 1. Patient has symptoms suggestive of diabetes
Sample collection: As per current WHO recommenda- mellitus; but fasting blood sugar value is inconclusive
tions, 2 samples are collected, one at fasting ("O" hour (between 100 and 126 mg/dl).
sample) and 2-hour post-glucose load. Urine samples 2. During pregnancy, excessive weight gaining is
may also be collected along with these blood samples. noticed, with a past history of big baby (more than
This is sufficient to get a correct assessment of the 4 kg) or a past history of miscarriage.
patient. 3. To rule out benign renal glucosuria.
4. GTT has no role in follow-up of diabetes. It is indi-
Normal Values and Interpretations cated only for the initial diagnosis.
As per WHO recommendation, In a normal person,
fasting plasma glucose is 70-110 mg/dl. The present Preparation of the Patient
day tendency is to view values above 100 mg/ml as
The patient is instructed to have good carbohydrate
suspicious. Value more than 100 mg/dl is one of the
criteria for the metabolic syndrome . diet for 3 days prior to the test. Patient should not take
Following the glucose load, in normal persons, the food after 8 PM the previous night. Should not take
level rises and reaches a peak within 1 hour and then any breakfast. This is to ensure 12 hours fasting.Th e
comes down to normal fasting levels by 2 to 2½ hours. patients are advised to remain in the hospital during the
This is due to the secretion of insulin in response to the waiting period of two hours without any active exercise.
elevation in blood glucose. None of the urine sample Figure 11.4 represents the graph, when plasma glucose
shows any evidence of glucose. Diagnostic criteria for values are plotted on the vertical axis against the time
diabetes mellitus are given in Table 11.1 and Box 11 .3. of collection on the horizontal axis.
Chapter 11: Regulation of Blood Glucose, Insulin and Diabetes Mellitus 163

increased birth weight. After the child birth, the women


should be reassessed.

Alimentary Glucosuria
Here the fasting and 2-hour values are normal; but an
exaggerated rise in blood glucose following the inges-
tion of glucose is seen. This is due to an increased rate
of absorption of glucose from the intestine.This is seen
50
in patients after a gastrectomy or in hyperthyroidism.

Renal Glucosuria
0 2 Hours Normal renal threshold for glucose is 175-180 mg/dl.
Fig. 11.4: Oral glucose tolerance test (OGTT) If blood sugar rises above this, glucose starts to appear
in urine.
Causes for Abnormal GTT Curve Generally, the increased blood sugar level is reflec-
ted in urine. But when renal threshold is lowered,
Impaired Glucose Tolerance (JG T) glucose is excreted in urine. In these cases, the blood
sugar levels are within normal limits. This is called renal
It is otherwise called as Impaired Glucose Regulation
glycosuria.
(IGR). Here blood sugar values are above the normal Renal threshold is lowered physiologically in preg-
level, but below the diabetic levels (see Table 11.1 ). nancy. Renal glucosuria is associated with renal dise-
In IGT, the fasting plasma glucose level is between ases with renal tubular transport defects; e.g. Fanconi's
110 and 126 mg/dl and 2-hour post-glucose value is syndrome. In these cases glucosuria is seen along with
between 140 and 200 mg/dl (Fig. 11.4). amino aciduria and phosphaturia.
Such persons need careful follow-up because IGT In some cases, renal threshold may be increased
progresses to frank diabetes at the rate of 2% patients when glucose will not appear in urine, even though
blood sugar is elevated. Here GFR is decreased with
per year.
minimal or no impairment of tubular reabsorption. This
is seen in old age (arteriosclerosis) and in Kimmelsteil-
Impaired Fasting Glycemia (/FG)
Wilson Syndrome (diabetic nephrosclerosis).
In this condition, fasting plasma sugar is above normal
(between 110 and 126 mg/dl); but the 2-hour post- Factors Affecting GTT
glucose value is within normal limits (less than 140mg/dl). In acute infections, cortisol is secreted, and so curve is
These persons need no immediate treatment; but are to elevated and prolonged. In hyperthyroidism there will
be kept under constant check up. be steep rise in curve. A flat curve is seen in hypo-
thyroidism.
Gestational Diabetes Mellitus (GDM)
REDUCING SUBSTANCES IN URINE
This term is used when carbohydrate intolerance is
Normally glucose is not excreted in urine. But if blood
noticed, for the first time, during a pregnancy. A known
sugar is more than 180 mg/dl, urine contains glucose.
diabetic patient, who becomes pregnant, is not The blood level of glucose above which glucose is
included in this category. excreted is called renal threshold.
Women with GDM are at increased risk for subse- The excretion of reducing substances in urine is
quent development of frank diabetes. GDM is associ- detected by a positive Benedict's test. (see Chapter
ated with an increased incidence of neonatal mortality. 7). About 0.5 ml of urine is boiled with 5 ml Benedict's
Maternal hyperglycemia causes the fetus to secrete reagent for 2 minutes (or kept for 2 minutes in water bath
more insulin, causing stimulation of fetal growth and which is already boiling). The formation of a precipitate
164 Section B: General Metabolism

TABLE 11.2: Reducing substances In unne


Sugars Noncarbohydrates
Glucose Homogentisic acid s.(
s-.. - 1
..s-
Fructose Salicylates
Lactose Ascorbic acid -~
~-
Galactose, pentoses Glucuronidesof drugs
21
30
is observed on cooling. The test is semi-quantitative Pro-insulin Active insulin
and the color of the precipitate roughly parallels the Fig. 11 .5: Insulin biosynthesis
concentration of reducing sugar. Blue color indicates the
absence of sugar in urine. The green precipitate means Insulin
0.5%; yellow (1%); orange (1.5%) and red indicates 2%
or more of sugar (1 % means 1 g per 100 ml). Nowadays, Structure of Insulin
strips are available, which when dipped in urine will give Insulin is a protein hormone with 2 polypeptide chains.
the color, if it contains sugar. The A chain has 21 amino acids and B chain has 30
Any reducing sugar will give a positive Bene- amino acids. These two chains are joined together by
dict's test. So differentiation of various sugars, which
two interchain disulfide bonds, between A7 to 87 and
may be present in urine has practical importance. Such
A20 to 819. There is also an intrachain disulfide link
conditions together are sometimes called as "mellituria".
in A chain between 6th and 11th amino acids (see
The substances in urine answering Benedict's test are
enumerated in Table 11 .2. Chapter 4, Fig. 4.4).
When reducing sugars are excreted in urine, the
condition is referred to as glycosuria. To denote the
Biosynthesis of Insulin
excretion of specific sugars the suffix 'uria' is added Insulin is a protein synthesized and secreted by the beta-
to the name of the sugar, e.g. glucosuria, fructosuria, cells of the islets of Langerhans of the pancreas. The
lactosuria. Glucosuria means glucose in urine; glycosuria insulin is synthesized as a larger precursor polypeptide
means any sugar in urine. Since glucose is the most chain, the pre-pro-insulin. It has 109 amino acids. It
common reducing sugar excretea in urine, the term is rapidly converted to pro-insulin in the endoplasmic
glycosuria is often (though incorrectly) used to denote
reticulum by removal of leader sequence of 23 amino
the excretion of glucose.
acid residues. The pro-insulin with 86 amino acids is
When blood glucose level exceeds the renal thresh-
transported to Golgi apparatus where it is cleaved by
old (175-180 mg/dl), glucose is excreted in urine. Dia-
a protease (Fig. 11.5). Thus C-peptide or connecting
betes mellitus is the most common cause. Transient
peptide with 33 amino acids is removed (The number of
glucosuria may occur in some people due to emotional
stress. Excessive secretion of anti-insulin hormones like amino acids in C peptide may vary according to species).
cortisol (anxiety) and thyroid hormone may cause gluco- Insulin with 51 amino acids is thus formed.
suria. Once the stress is removed, the glucosuria disap-
pears. Secretion of Insulin
Insulin secretion is in response to an elevation of glucose
Lactosuria
level. The GLUT2 allows the entry of glucose into the
It is the second most common reducing sugar found in beta cell. Glucose is fu rther metabolized producing ATP.
urine. It is observed in the urine of normal women during This closes potassium channels and calcium channels.
3rd trimester of pregnancy and lactation. The condition The influx of calcium causes release of insulin into the
is harmless. In pregnancy, it is important to distinguish blood . The insulin in turn lowers glucose level. Insulin and
lactosuria from glucosuria when gestational diabetes C-peptide are synthesized and secreted in equimolar
mellitus is suspected . Fructosuria, Galactosuria and
quantities. Therefore, measurement of C-peptide is an
pentosuria are described in Chapter 12.
index of rate of secretion of insulin.
Chapter 11: Regulation of Blood Glucose, Insulin and Diabetes Mellitus 165

Glucose ent~ Ca++entry When insulin binds with alpha unit, the beta unit transmits
K -ATP channel signal to cytoplasm
GLUT2

Glucose/
more;
ATP more

@
/ "'"~"@

@
Insulin in granules @

Secreted outside cell • • • • •


Fig. 11.6 : Mechanism of insulin secretion

Factors Increasing Insulin Secretion


Activated beta unit then phosphorylates IRS;
1. Glucose: As blood glucose level increases, the which then leads lo cellular responses
insulin secretion also correspondingly increases.
Fig. 11.7: Insulin receptor
The mechanism has been described in previous
paragraph (Fig.11.6). with type 2 diabetes mellitus (OM). In such cases,
2. Gastrointestinal hormones: Insulin secretion is administration of GLP-1 improves glycemic control.
enhanced by secretin, pancreozymin and gastrin.
After taking food, these hormones are increased. Degradation of Insulin
3. Proteins and amino acids: Leucine and arginine Insulin is rapidly degraded by the liver. Plasma half-life
are stimulants. is less than 5 minutes. An insulin-specific protease
4. Drug: Tolbutamide. (insulinase) is involved in the degradation of insulin.
5. lncretin hormones: The incretins are hormones
that work to increase insulin secretion. Nutrient Mechanisms of Action of Insulin
intake stimulates the secretion of the gastroin- Insulin Receptors
testinal incretin hormones, glucagon-like peptide-1
Insulin acts by binding to a plasma membrane receptor
(GLP-1 ) and glucose-dependent insulinotropic
polypeptide (GIP). Glucose in the small intestine on the target cells. Insulin receptor is a glycoprotein with
stimulates incretin release. lncretin stimulation of 4 subunits; 2 alpha and 2 beta subunits. The alpha units
beta cells causes them to secrete more insulin in (135 kD) are located on the extracellular side, to which
response to the same amount of blood glucose. insulin binds. The beta subunits (95 kD) are exposed
Decreased responsiveness to GIP and markedly on the cytoplasmic side (Fig. 11.7). Beta subunit has
reduced GLP-1 concentration occur in individuals tyrosine kinase activity (receptor tyrosine kinase). In

Frederick John James Frederick Paul Josef Oskar John Abel


Banting James Bertram Sanger Langerhans von Mering Minkowski 1857-1938
(Right) NP 1923 Richard Co/lip 1918-2013 1847- 1888 1849-1908 1858-1931
(1891-1941) Macleod 1892-1965 NP 1958
Charles Best NP 1923 and 1980
(Left) 1899-1978 1876-1935
Marjorie (middle)
166 Section B: General Metabolism

obesity, the number of receptors are decreased and iii. Phosphofructokinase


target tissue becomes less sensitive to insulin (diabetes iv. Acetyl-CoA carboxylase
mellitus type 2). B. Insulin represses the following enzymes:
i. Glucose-6-phosphatase
Signal Transduction
ii. Phosphoenolpyruvate carboxykinase
Insulin binds to the alpha subunit of the receptor. iii. Fructose-1,6-bisphosphatase
Oligomerization of alpha units would trigger the tyrosine
kinase activity of the beta subunit. The phosphorylated
Activation of Enzymes
sites act as docking sites for insulin receptor substrates
- IRS1 and IRS2. This recruits GLUT4 to cell surface Insulin activates the existing molecules of enzymes by
and stimulate glycogen synthesis. covalent modification (phosphorylation or dephospho-
rylation ). There are more than 50 enzymes activated
Gene Transcription
by this mechanism. A small selected list is shown in
(New Enzyme Synthesis)
Table 11.3.
Insulin acts at the transcriptional level to regulate syn-
thesis of more than 100 proteins.
Other Effects
A. Insulin induces the following enzymes:
i. Glucokinase Insulin increases DNA synthesis, cell growth and ana-
ii. Pyruvate kinase bolism. Insulin inhibits glycogen phosphorylase (see
Chapter 10). Insulin increases the recruitment of GluT4
TABLE 11.3: Insulin acting through covalent modification in cells.
Enzyme Activity Mechanism
Glycogen synthase Increase Dephosphorylation
Physiological Actions of Insulin
Pyruvate dehydrogenase Increase Dephosphorylation
Pyruvate k inase Increase Dephosphorylation
(Metabolic Effects of Insulin)
Acetyl-CoA carboxylase Increase Dephosphorylation Insulin plays a central role in regulation of the metabolism
HMG-CoA reductase Increase Dephosp horylation of carbohydrates, lipids and proteins (Table 11.4).

TABLE 11.4: B1olog1cal effects of insulin


Metabolism Key enzyme Action of insulin on the enzyme Direct effect Overall effect

l
Carbohydrate Translocase Stimulation Glycolysis
Glucokinase Stimulation favored
Phosphofructokinase Stimulation
Pyruvate kinase Stimulation
Pyruvate carboxylase Inhibit ion Gluconeogenesis Hypoglycemia
PEPCK
Fructose-1,6-bisphosphatase
Glucose-6-phosphatase
Glycogen synthase
Glycogen phosphorylase
Inhibit ion
Inhibition
Inhibition

Activation ]
Inactivation
l depressed

Glycogen deposition -

GPD Stimulation
1 Generation of NADPH l
Lipid Acetyl-CoA carboxylase
Glycerol kinase
Stimulation
Stimulation
j Lipogenesis favored J Glucose is used for lipogenesis;
glucose lowered; decreased
Hormone sensitive lipase Inhibition Lipolysis inhibited ketogenesis
HMG-CoA reductase Stimulation Cholesterol synthesis

lj
Protein Transam inases Inhibition Catabolism inhibited
Ornithine transcarbamoylase Inhibition
RNA polymerase Protein synthesis General anabolism
and ribosome assembly Favored favored
Chapter 11 : Regulation of Blood Glucose, Insulin and Diabetes Mellitus 167

Uptake of Glucose by Tissues ii. Protein synthesis is promoted and degradation is


retarded. Insulin is an anabolic hormone.
Insulin facilitates the membrane transport of glucose.
iii. Insulin is an essential growth factor for all mammalian
Facilitated diffusion of glucose in muscle is enhanced
cells. These effects are summarized in Table 11.4.
by insulin. In diabetes mellitus, the transporter, GluT4 is
reduced (see Chapter 10). However, glucose uptake in HYPERGLYCEMIC HORMO NES
liver (by GluT2) is independent of insulin.
1. Glucagon
Utilization of Glucose 2. Epinephrine or Adrenaline
3. Glucocorticoids
Glycolysis is stimulated by insulin. The activity and 4. Adrenocorticotropic hormone (ACTH)
amount of key glycolytic enzymes (glucokinase, phos- 5. Growth hormone
phofructokinase and pyruvate kinase) are increased. 6. Thyroxine
Glycogen synthase enzyme is activated, and so insulin All these are anti-insulin hormones.
favors glucose storage as glycogen (see Chapter 10).
GLUCAG ON
Hypoglycemic Effect
It is a polypeptide hormone with 29 amino acids. It is
Insulin lowers the blood glucose level by promoting uti- secreted by the alpha cells of pancreas. Enteroglucagon
lization and storage. Gluconeogenesis is inhibited is a peptide hormone secreted by duodenal mucosa,
by insulin by repressing the key enzymes, pyruvate having same immunological and physiological proper-
carboxylase (PC) phosphoenolpyruvate carboxykinase ties of glucagon. Glucagon is synthesized as a longer
(PEPCK) and glucose-6-phosphatase (see Chapter 10). proglucagon precursor. The major regulator of secretion
Insulin inhibits glycogenolysis by favoring the inac- of glucagon is glucose. An increase in blood glucose
tivation of glycogen phosphorylase and inhibiting level inhibits secretion of glucagon.
glucose-6-phosphatase. The net effect of all these three
mechanisms, blood glucose level is lowered.
Physiological Actions of Glucagon
Glucagon is the most potent hyperglycemlc hormone.
Lipogenesis It is anti-Insulin in nature. Therefore, the net effect is
decided by the insulin-glucagon ratio (Fig . 11 .8). Glu-
Lipogenesis is favored by providing more acetyl-CoA
cagon is mainly glycogenolytic. The active form of
by pyruvate dehydrogenase reaction. Insulin increases
glycogen phosphorylase is formed under the influence
the activity of acetyl-CoA carboxylase and provides
of glucagon. Liver is the primary target for the gly-
glycerol for esterification of fatty acids to TAG (see
cogenolytic effect of glucagon. It depresses glycogen
Chapter 13). Insulin also provides NADPH by increasing
the GPO activity of the HMP shunt pathway.

Anti-/ipolytic Effect High blood


glucose
Insulin inhibits lipolysis in adipose tissue due to inhibi-
tion of hormone-sensitive lipase. The increased level
of FFA in plasma in diabetes is due to the loss of this
inhibitory effect on lipolysis.

low blood
Other General Effects glucose
i. Insulin depresses HMG-CoA synthase and so keto-
Glucose Glucagon
genesis is decreased. Insulin also favors fatty acid uptake/storage glucocorticoids
synthesis from acetyl-CoA. All these factors reduce
Fig. 11 .8: Combined action of insulin and glucagon will keep the
the availability of acetyl-CoA, so that production of blood sugar level within normal limits. High blood sugar stimulates
ketone bodies reduced . insulin secretion. Low blood sugar causes glucagon secretion
168 Section B: General Metabolism

TABLE 11.S: Comparison of action of insulin and ant1-1nsulln hormones


Metabolism Key enzymes Insulin Glucagon G/ucocorticoids Growth hormone

Glycolysis GK, PFK and PK Stimulation Inhibition

Gluconeogenesis PEPCK, G6 Pase, F-bisphosphatase Inhibition Stimulation Stimulation St imulation

Glycogen synthesis Glycogen synthase Activation Inhibition

Glycogenolysis Phosphorylase Inactivation Activation


Lipolysis Hormone-sensitive lipase Inhibition Stimulation Stimulation Stimulation

Ketogenesis Carnitine acyltransferase Inhibition Stimulation Stimulation

Protein breakdown Transaminases Inhibition Stimulation

Protein synthesis Anabolism Catabolism Anabolism

Blood sugar level Decreases Increases Increases Increases

synthesis. Gluconeogenesis is favored by glucagon by suffer from this disease. It is a major cause for morbidity
inducing enzymes like PEPCK, glucose-6-phosphatase and mortality. Insulin deficiency leads to increased blood
and fructose-1 ,6-bisphosphatase. Glucagon increases glucose level. In spite of this high blood glucose, the entry
plasma free fatty acid level. In adipose tissue gluca- of glucose into the cell is inefficient. Hence all cells are
gon favors beta-oxidation, as it activates carnitine starved for glucose.
acyl transferase. The mitochondrial acetyl-CoA level Criteria for diagnosis of diabetes mellitus are shown
increases. Ketogenesis is favored. in Table 11 .1, under glucose tolerance test. The disease
may be classified as follows (WHO recommendation,
Mechanism of Action 1999):
Glucagon combines with a membrane bound receptor.
This activates G protein and adenylate cyclase (see Type 1 Diabetes Mellitus
Chapter 45). Thus ATP is converted to cAMP. Cyclic
(Formerly known as Insulin-dependent diabetes mel-
AMP activates glycogen phosphorylase, and inactivates
litus; IDDM). About 5% of total diabetic patients are of
glycogen synthase.
type 1. Here circulating insulin level is deficient. It is sub-
Anti-insulin Hormones classified as:
a. Immune mediated and
Regulation of carbohydrate metabolism in general de-
b. Idiopathic.
pends on the balance between insulin and anti-insulin
hormones. A summary is given in Box 11.2. See also
Table 11 .5. Glucocorticoids act mainly by stimulating Type 2 Diabetes Mellitus
gluconeogenesis. But growth hormone antagonizes (Formerly known as noninsulin dependent diabetes
insulin in ma ny key metabolic reactions (Table 11.5). mellitus; NI DDM). Most of the patients belong to this
Bernado Houssey demonstrated that in pancreatecto-
type. Here circulating insulin level is normal or mildly
mized animals, the requirement of insulin was about 100
elevated or slightly decreased, depending on the stage
units per day. When anterior pituitary was also ablated
of the disease. This type is further classified as:
in such animals, the requirement of insulin came down to
10 units or so. This shows that growth hormone antago- a. Obese
nizes insulin. Houssay was awarded Nobel Prize in 1947. b. Nonobese

DIABETES MELLITUS Diabetic Prone States


Diabetes mellitus is a metabolic disease due to abso- a. Gestational diabetes mellitus (GDM);
lute or relative insulin deficiency. Diabetes mellitus is a b. Impaired glucose tolerance (IGT);
common clinical condition. About 10% of the total popu- c. Impaired fasting glycemia (IFG);
lation, and about 115th of persons above the age of 50, d. Metabolic syndrome (described below)
Chapter 11 : Regulation of Blood Glucose, Insulin and Diabetes Mellitus 169

Secondary to Other Known Causes BOX 11. 4 : Cntem1 for d1agnos1s of metabolic syndrome
i. Elevated waist circumference: (For men >90 cm and for
a. Endocrinopathies (Cushing's disease, thyrotoxico- women, >80 cm).
sis, acromegaly); ii. Elevated triglycerides: > 150 mg/ dl
b. Drug induced (steroids, beta blockers, etc.); iii. Reduced HDL ("good") cholesterol: For men, <40 mg/ dl; for
c. Pancreatic diseases (chronic pancreatitis, fibrocal- women, < 50 mg/dl
culus pancreatitis, hemochromatosis, cystic fibrosis) iv. Elevated blood pressure: > 130/ 85 mm Hg
d. Anti-insulin receptor autoantibodies (Type B insulin v. Elevated fasting glucose: > 100 mg/dl
vi. Insulin resistance (hyperinsulinemia)
resistance)
vii. Additional parameteri include coagulation abnormalities,
e. Mutations in the insulin gene or insulin receptor
hyperuricemia, microalbuminuria, non-alcoholic steatohepa-
gene (acanthosis nigricans) titis (NASH) and increased CRP
f. MODY (Maturity Onset Diabetes of Young). viii. Diagnosis is made, if the first criterion and any two of other
MODY was previously considered to be a third criteria are present.
form of type 2 diabetes. However, with the discovery of
specific mutations leading to MODY, it is now classified where it is re-esterified. The consequent increase in
under secondary diabetes. MODY is characterized by diacylglycerol (DAG), a second messenger, leads to
onset prior to age 25, impaired beta cell function and reduced signal transduction by insulin leading to insulin
insulin resistance. Mutations of about 10 different genes resistance.
have been correlated with the development of MODY. A high-caloric diet coupled with a sedentary lifestyle
are the major contributing factors in the development
Type 1 Diabetes Mellitus (T1 DM) of the insulin resistance. A major susceptibility locus
It is due to decreased insulin production. Circulating for type 2 diabetes, named as NIDDM 1, is located on
insulin level is very low. These patients are dependent chromosome 2. Lipoprotein (a) or Lp(a) (see Chapter
on insulin injections. Onset is usually below 30 years 15) is associated inversely with risk of type 2 diabetes.
of age, most commonly during adolescence. They are
more prone to develop ketosis. Metabolic Syndrome (MetS)
An autoimmune basis is attributed to most of these It is a combination of abdominal obesity, atherogenic
cases. Circulating antibodies against insulin is seen in dyslipidemia (hypertriglyceridemia and low HDL cho-
50% cases. Type 1 diabetes mellitus is an autoimmune
lesterol), elevated blood pressure and elevated plasma
disease in which pathologic, autoreactive T cells of the
glucose. The characteristic features are abdominal
immune system attack the insulin-secreting pancreatic
obesity, insulin resistance and decreased glucose tole-
islets of Langerhans. There is excessive secretion of
rance (Box 11.4). The body cannot properly use glucose
glucagon in IDDM patients.
even in presence of normal insulin level. In other words,
body cannot use insulin efficiently. Therefore, the meta-
Type 2 Diabetes Mellitus (T2DM) bolic syndrome is also called the insulin resistance
95% of the patients belong to this type. The disease syndrome. People with Mets are at increased risk of
is due to the decreased biological response to insulin, coronary heart disease and type 2 diabetes. The Mets
otherwise called insulin resistance. So, there is a has become increasingly common in the developing
relative insulin deficiency. Type 2 disease is commonly countries. Diagnostic criteria are shown in Box 11 .4.
seen in individuals above 40 years. These patients are Abdominal obesity is the most prevalent manifesta-
less prone to develop ketosis. About 60% of patients tion of metabolic syndrome. Obesity and adipocytokines
are obese. These patients have insulin resistance and are discussed in Chapter 35.
high/normal plasma insulin levels.
Insulin resistance develops as a consequence of
Metabolic Syndrome (MetS) and
excess accumulation of fat in liver and skeletal muscle. Polycystic Ovary Syndrome (PCOS)
The free fatty acid level increases, exceeds the capacity They have overlapping features. The common factors
of mitochondrial oxidation and spills over to cytoplasm are insulin resistance and obesity. In PCOS there is
170 Section B: General Metabolism

Skeletal muscle

l
Glucose - - - - - - - - - - - - - - Hyperglycemia
1
l\
Fal1)' acid

Amino acids 1 Glucosuria

l
Gluconeogenesls - - - - - -- - -- Oxaloacetate + Cit,{
Acetyl-CoA - - - - - - Ketone bodies

~ sterol l
Increase

Decrease
TCAcycle
l
Atheroscleros,s and
cardiovascular diseases
Ketonuna

Alpha ketoglutarate
Glycolysis is inhibited; gluconeogenesis is favored. Fat is broken down; FFA is increased; Acetyl-CoA is in plenty. This could not be fully
utilized in TCA cycle, because availability of oxaloacetate is reduced. So acetyl-CoA is shunted to ketone body formation.

Fig. 11.9: Metabolic derangements in diabetes mellitus

hyperandrogenism. Those with PCOS have an increa-


sed risk for coronary vascular disease as in patients
with Mets. Overweight adolescents with PCOS are at
increased risk of developing impaired glucose tolerance
and type 2 diabetes mellitus.

Pathological Alterations Bernardo Hermann Otto O/of


Houssay von Fehling Folin
in Diabetes Mellitus NP 1947 1812-1885 1867-1934
1887-1971
Derangements in
Stanley Rossiter Benedict ( 1884-1936) dis-
Carbohydrate Metabolism covered the Benedict's Reagent in 1908,
while working as a PhD student at Yale Uni-
Insulin deficiency decreases the uptake of glucose
versity. He was Professor of Physiological
by cells. The insulin dependent enzymes are also less chemistry in Come/I University Medical Col-
active. Net effect is an inhibition of glycolysis and stimu- lege from 1912 till death.
lation of gluconeogenesis leading to hyperglycemia.
Clinical Presentations in
Derangements in Lipid Metabolism
Diabetes Mellitus
Enhanced lipolysis leads to high FFA levels in plasma
The cardinal symptoms of diabetes mellitus are gluco-
and consequent accumulation of fat in liver leading to
NAFLD (Nonalcoholic fatty liver disease). More acetyl- suria, polyuria, polydypsia and polyphagia. When the
CoA is now available, which cannot be efficiently oxidized blood glucose level exceeds the renal threshold glucose
by TCA cycle, because the availability of oxaloacetate is is excreted in urine (glucosuria). Due to osmotic effect,
limited. The stimulation of gluconeogenesis is responsible more water accompanies glucose (polyuria). To com-
for the depletion of oxaloacetate. The excess of acetyl- pensate for this loss of water, thirst center is activated,
CoA therefore, is diverted to ketone bodies, leading to and more water is taken (polydypsia). To compensate
ketogenesis (see Chapter 13). This tendency is more the loss of glucose and protein , patient will take more
in type 1 disease. There is hyperlipidemia, especially food (polyphagia) (See Fig. 11.9).
an increase in NEFA, TAG and cholesterol in plasma. The loss and ineffective utilization of glucose leads
to breakdown of fat and protein. Thig would lead to loss
Derangement in Protein Metabolism of weight. Important differential diagnosis for weight
Increased breakdown of proteins and amino acids for loss are diabetes mellitus, tuberculosis, hyperthyroidism,
providing substrates for gluconeogenesis is responsible cancer and AIDS . Often the presenting complaint of the
for muscle wasting. patient may be chronic recurrent infections, such as
Chapter 11: Regulation of Blood Glucose, Insulin and Diabetes Mellitus 171

boils, abscesses, etc. Any person with recurrent infec-


tions should be investigated for diabetes. When glucose
: . t 1s lactic ac1dos1s?

Lactate is the ncirmal end product of anaerobic glycolysis. All


level in extracellular fluid is increased, bacteria get tissues can produce lactate and liver can metabolise it. The
good nutrition for multiplication. At the same time, mac- blood level seldom exceeds 1.5 mmol/ L. Under conditions
of decreased oKygen availability, as in vigourous excercise,
rophage function of the host is inefficient due to lack of
the rate of lacttate production increases. The term lactic
efficient utilization of glucose. In India, tuberculosis is acidosis denotes a pathological state, when the lactate
commonly associated with diabetes. level in blood i!; more than 5 mrnol/L. So blood pH is low,
with decreased levels of bicarbonate. Collection of blood for
lactate estimation has to be done avoiding tissue hypoxia, so
ACUTE COMPLICATIONS OF that falsely elevated values are not obtained.
DIABETES MELLITUS
products (AGE) deposition in tissues and endothelium
Diabetic Ketoacidosis
lead to all the chronic complications of diabetes mellitus.
Ketosis is more common in type 1 diabetes mellitus. Vascular disea1ses: Atherosclerosis in medium-sized
Ketone body formation and explanations for ketosis are vessels, plaque formation and consequent intravascu-
described in Chapter 13. Normally the blood level of lar thrombosis may take place. If it occurs in cerebral
ketone bodies is less than 1 mg/dl and only traces are vessels, the resiult is paralysis. If it is in coronary artery,
excreted in urine (not detectable by usual tests). But myocardial infarction results. In the case of small ves-
when the rate of synthesis exceeds the ability of extra- sels, the process is called microangiopathy, where
hepatic tissues to utilize them, there will be accumula- endothelial cells and mural (cement) cells are damaged.
tion of ketone bodies in blood. This leads to ketonemia, Microangiopathy may lead to diabetic retinopathy and
excretion in urine (ketonuria) and smell of acetone in nephropathy.
breath. All these three together constitute the condition
Complications in eyes: Early development of cataract
known as ketosis.
of lens is due to the increased rate of sorbitol formation,
caused by the hyperglycemia. Retinal microvascular
Diagnosis of Ketosis
abnormalities lead to retinopathy and blindness.
Ketone body fonnation, causes of ketosis, clinical fea-
Neuropathy: P,eripheral neuropathy with paresthesia
tures and management of ketosis are elaborted in
is very commoni. Decreased glucose utilization and its
Chapter 13.
diversion to sorbitol in Schwann cells may be one cause
Lactic AciJosis for neuropathy. Another reason proposed is the produc-
tion of advanced glycation end products. Neuropathy
It is another acute complication. It occurs due to over- may lead to risl< of foot ulcers and gangrene. Hence,
production and/or under-utilization of lactic acid. Over- care of the feet in diabetic patients is important.
production can result from an increased rate of anaerobic Kimmelstiel-Wilson syndrome is another compli-
glycolysis due to hypoxia. Under-utilization may be due cation of diabetes, resulting from nephrosclerosis, char-
to impairment of TCA cycle. Lactic acidosis is seen when acterized by proteinuria and renal failure. Persistent
diabetic patients are treated with biguanides. This drug hyperglycemia l,eading to glycation of basement mem-
inhibits TCA cycle and gluconeogenesis (Box 11 .5). brane proteins may be the cause of nephropathy.
Pregnancy: Diabetic mothers tend to have big babies,
Chronic Complications of
because insulin i:s an anabolic hormone. Chances of abor-
Diabetes Mellitus tion, premature !birth and intrauterine death of the fetus
When there is hyperglycemia, proteins in the body are also more, if the diabetes is not properly controlled.
may undergo glycation. It is a nonenzymatic process.
Glucose forms a Schiff base with the N-terminal amino Hyperosmolar Nonketotic Coma
group of proteins. The glycation first occurs in circulating It can result due to elevation of glucose to very high levels
proteins like hemoglobin, albumin and LDL and then (900 mg/dl or nnore). This would increase the osmola-
to extracellular proteins. The advanced glycation end lity of extracellular fluid (ECF). Osmotic diuresis leads to
172 Section B: General Metabolism

is better to do once in every month, so as to analyze the


RBC -Sugar effectiveness of the treatment.
group
added
Normally the level of Hb A 1c is less than 5.5%. The
value 5.5% denotes very good control of diabetes by
treatment measures; 7% means adequate control; 8%
Normal glucose level High glucose level inadequate control and 9% means very poor control. Any
Fig. 11 .10: Glycation is parallel to the blood glucose value above 5.5% is to be closely watched and values
between 5.6 and 6.4 are to be considered as impaired
water and electrolyte depletion. The coma results from glucose tolerance. 6.5% and above means the person
dehydration of cerebral cells due to hypertonicity of ECF. is diabetic. An elevated glycohemoglobin indicates poor
control of diabetes mellitus. The risk of retinopathy and
Laboratory Investigations in Diabetes renal complications are proportionately increased with
elevated glycated hemoglobin value. Reduction in 1%
Fasting blood glucose estimation, random blood glucose of glycoHb will decrease long-term complications to an
assessment and oral glucose tolerance tests are used extent of 30%.
for the diagnosis (Table 11.1 ). HbA 1c estimation is yet Advantages of HbA1c over fasting blood sugar
another important diagnostic test. estimation are (1) For HbA 1c, fasting sample is not
required; the test may be done at any time. This adds
Blood Glucose Level convenience to the patient. (2) HbA 1c sample is stable;
For monitoring a diabetic patient, periodic check of while blood sugar level is lowered during transportation
fasting and postprandial blood glucose are to be done. to laboratory, unless precautions are taken. (3) HbA1c
Blood glucose level has to be maintained within the reflects long-term glucose control, while blood sugar
normal limits. Persistent hyperglycemia is the most estimation will show the result of a particular time only.
important factor, which leads to chronic complications. (4) HbA1c is a better index for predicting complications.
Because of all these reasons, HbA 1c has become the
Glycated Hemoglobin preferred test nowadays.

The best index of long-term control of blood glucose Disadvantage of HbA 1c estimation
level is measurement of glycated hemoglobin or glyco- Any type of anemia, (e.g., abnormal hemoglobins,
hemoglobin. Enzymatic addition of any sugar to a protein hemoglobinopathies) where RBC life span is reduced,
is called "glycosylatlon", while nonenzymatic process is will reflect in lowered HbA 1c value, because the time
termed "glycation". When once attached, glucose is averaged value is less.
not removed from hemoglobin. Therefore, it remains
inside the erythrocyte, throughout the lifespan of RBCs How Frequently the Estimation to be Done
(120 days) (Fig . 11.1 0). The glycated hemoglobins are HbA1c estimation indicates the average blood glucose
together called HbA 1 fraction. Out of this 80% molecules concentration for the past 120 days (life span of RBCs).
are HbA1c, where glucose is attached to the N-terminal However, the value is weighted towards the younger
valine of beta chain of hemoglobin. RBCs. About half of the value is contributed by the
RBCs of the age of 1 month or less. Therefore, it is ideal
Interpretation of to repeat the test every month. This will give an idea to
Glycohemoglobin Values the physician that the treatment is effective or not.
The determination of glycated hemoglobin is not for dia-
gnosis of diabetes mellitus; but only for monitoring the Fructosamines
response to treatment. The Hb A 1c level reveals the Along with other proteins, albumin is also glycated in
mean glucose level over the previous 10-12 weeks. diabetes mellitus. Glycated albumin is more correctly
It is unaffected by recent food intake or recent changes called as fructosamine albumin. As half-life of albumin is
in blood sugar levels. The estimation should be done at about 20 days, glucoalbumin concentration reflects the
least every 3 months in all diabetic patients; however, it glucose control over a recent past, for a period of last
Chapter 11 : Regulation of Blood Glucose, Insulin and Diabetes Mellitus 173

glucosidase inhibitor, acarbose inhibits the alpha-glu-


100 cosidases present in the small intestinal brush border.
So absorption of glucose is reduced. This allows the
Insulin production decreases pancreas to moire effectively regulate insulin secretion.
Oral hypoglyce1mic agents: There are several types of
80
::i' Epinephrine and glucagon oral hypoglycemic agents (OHA) now in use. The con-
"C
m production increases
.s
QI
ventional types are sulfonylurea and biguanides (Met-
formin) used in type 2 DM.
CII Growth hormone production increases
0
u Insulin injections: Insulin is the drug of choice in type
:::, 60 Cortisol production increases
ci
"C • anxiety 1 disease. It is also used in type 2 disease, where oral
0

7
0 •headache • palpitation drugs are not sufficient. The availability of human insulin
iii • confusion • tremor
• slurred speech • sweating prepared by recombinant DNA technology has markedly

7
40 • seizures improved the response of patients.
• coma
• death Prevention of c:omplications

Fig. 11 .11 : Hypoglycemia is fatal Hypoglycer,n ia


2-3 weeks. Estimation of serum fructosamine is preferred Hyperglycemia causes harm; but hypoglycemia is fatal.
in gestational diabetes mellitus. It is also useful in cases A fall in plasma glucose less than 50 mg/dl is life-
of decreased life span of RBCs (e.g. anemias), where threatening (Fig " 11.11 ). Causes of hypoglycemia are:
HbA 1c estimation will give erroneous answer. 1. Overdose of insulin: This is the most common
cause. The: · differentiation of hypoglycemic coma
Complete Lipid Profile from hyperglycemic coma (ketosis) is important,
Total cholesterol, triglycerides, HDL and LDL cholesterol since treatment is exactly opposite. The diagnosis
levels may be done once in 6 months (see Chapters is mainly,based on blood glucose estimation.
14and15). 2. Postprandiial hypoglycemia: 2- 3 hours after a
meal, transient hypoglycemia is seen in some per-
Kidney Function Tests sons. This is due to over-secretion of insulin.
Blood urea and serum creatinine may be done at least 3. lnsulinomc1: Insulin secreting tumors are rare.
twice an year (see Chapter 25). 4. Von Gierke1's disease (see Chapter 10).

Microalbuminuria and Frank Albuminuria 0


• Clinical! Case Study 11.1
Presence of albumin (50 to 300 mg/day) in urine is
"A 45-year-old obese male had a tooth infection. Prior to
known as microalbuminuria (see Chapter 25). It is a pre-
extraction he was advised to have a routine blood and
dictor of progressive· renal damage. Albumin more than
300 mg/day indicates overt diabetic nephropathy. Micro- urine examination. The results were:
Total WBC count: 35,000/cmm
albuminuria is to be checked at least once in an year.
Differential count: P70, L20, E7, M2, B1
Management of Diabetes Mellitus ESR: 45 mm/hr
Urine albumin: Trace
Diet and exercise: This is the first line of treatment. A Urine sugar: Orange precipitate
diabetic patient is advised to take a balanced diet with Urine ketone bodies: Nil
high protein content, low calories, devoid of refined sugars Urine bile salts: Nil
and low saturated fat, adequate PUFA, low cholester- Urine bile pigments: Nil.
ol and sufficient quantities of fiber. Vegetables are the A. What are the further investigations to be done in
major sources of minerals, vitamins and fibe r. Alpha this patient? Explain the rationale behind each test.
174 Section B: General Metabolism

B. When the diagnosis is confirmed and treatment interval, 4.0-5.5.0%). What are the various types of
started, how will you monitor the patient? methods used for measuring HbA1c? How do Hb
C. What are the possible complications that can be variants interfere with each of these HbA 1c methods?
avoided by proper monitoring of the patient? What actions should be taken when a spurious HbA1c
e result is present?
• Clinical Case Study 11.2 e
A person is brought to the Emergency Department in a • Clinical Case Study 11 .7
comatose state. The following test results were obtained A 40-year-old male was brought to the emergency room
Blood sugar - 400 mg%, Benedict's test (Urine) - Red complaining of dizziness and weakness. History revealed
precipitate, Rothera's test (Urine) - Positive, Serum that he had skipped breakfast. Random blood sugar
Bicarbonate - 12 mEq/L, Plasma pH - 7.14. What is value was 40 mg%. What is the probable diagnosis?
your probable diagnosis?
e
e
a Clinical Case Study 11.3
• Clinical Case Study 11 8
A40-year-old man presented with complaints of frequent
A person is brought to the Emergency Department episodes of dizziness and numbness in legs. On exami-
in a comatose state. The following test results were nation, he is obese, leads a sedentary lifestyle, has a BP
obtained - Blood sugar-40 mg%, Benedict's test (Urine) of 200/120 mm Hg, has fasting hyperglycemia, hyperin-
- Negative, Rothera's test (Urine) - Positive, Serum sulinemia, dyslipidemia and glucose intolerance. What
Bicarbonate - 12 mEq/L, Plasma pH - 7.14. What is is the diagnosis? What is the pathogenesis involved?

ae
your probable diagnosis?


1?. Clinical Case Study 11 .4
Cllnlcal Case Study 11 .9.
A 30-year old woman during her second pregnancy had
A 19-year-old with 4 year history of juvenile diabetes a glucose tolerance test and the results are:
mellitus was brought to the Emergency Department in Fasting glucose level: 125 mg/dl
state of coma. The following laboratory results were 1 hour glucose level: 210 mg/dl
obtained - Blood sugar - 1300 mg%, Plasma pH - 7.1 , 2 hour value: 170 mg/dl.
pCO2 - 13 mm Hg, Pulse rate - 120/min, Respiratory A. Plot a GTT graph with these results.
rate - 28/min. What is your probable diagnosis? What is B. Comment on the GTT results.
the pathophysiology of the above condition? C. What will be the result of Benedict's test with the urine
e sample collected along with each blood sample?
• Cllnlcal Case Study 11 .5 D. How will you follow-up the patient?
E. What is the importance of assessing the glucose
A 55-year-old man with long standing diabetes melli-
tolerance in a pregnant lady?
tus presented with fever, pruritis, delirium and low urine
F. How do you rule out lactosuria in this case?
output. His blood urea level was 135 mg% and urea clea-
rance was 35 ml/min . What is the most likely diagnosis? e
• Clinical Case Study 11 .10
e
a Clinical Case Study 11 .6
An apparently healthy man, on a routine checkup, was
found to have fasting blood sugar of 80 mg/dl, and
A 52-year-old woman with a medical history of hepatitis B, urine showed no abnormal constituents. After a heavy
hyperlipidemia, hypertension and anemia, presented to breakfast of one-and-half hours, his blood sugar was.
the medicine department for a routine visit. Laboratory 140 mg/dl and urine sample at that time was positive
tests 3 months previously had revealed an impaired for Benedict's test.
fasting glucose concentration of 118 mg/dl) [reference A. What is the diagnosis?
interval, 70-110 mg/dl]. Therefore, a hemoglobin HbA1c B. How do you further investigate?
analysis was performed. The initial HbA1c evaluation C. What is the line of treatment?
by HPLC showed an HbA1c value of 12.8% (reference D. What is the course of this disease?
Chapter 11: Regulation of Blood Glucose, Insulin and Diabetes Mellitus 175

0 falls below 50 m~1%. CNS symptoms include behavioral


• Clinical Case Study 11.1 Answer changes, confusion, fatigue, seizures, loss of conscio-
usness, and if severe and prolonged, death.
Likely diagnosis is uncontrolled diabetes mellitus. Often,
Spontaneous hypoglycemia may be (1) fasting, and
diabetes is noticed by a routine urine examination. Fast-
(2) postprandial. Fasting hypoglycemia may be subacute
ing and postprandial blood sugar should be estimated
or chronic and usually presents with neuroglycopenia.
on this patient (see Chapter 24 for further details).
Postprandial hypoglycemia is usually acute and symp-
0 toms of neurogenic autonomic discharge like sweating,
8 Clinical Case Study 11 .2 Answer palpitations, anxiBty and tremulousness are seen.
Diabetic ketoacidosis (OKA). There are many causes for hypoglycemia. Fasting
hypoglycemia maIy be due to drugs (e.g. insulin), critical
.i, Clinical Case Study 11.3 Answer illness (hepatic, irenal and cardiac, sepsis), endocrine
problems, insulinoma, endogenous hyperinsulinism,
Starvation ketosis.
other 13-cell disorders, autoimmune disorders and certain

a0 Clinical Case Study 11 .4 Answer


inborn errors of metabolism. Postprandial hypoglycemia
may be due to ailimentary (postgastrectomy), galacto-
Diabetic ketoacidosis (OKA). semia, hereditary fructose intolerance and idiopathic.
t, Treatment of hypoglycemia is oral glucose or IV
• • Clinical Case Study 11.5 Answer glucose.
Diabetic nephropathy.
0 a0 Cllnlcal Case Stucty 11.8 Answer
• • Clinical Case Study 11 .6 Answer The patient has "insulin resistance syndrome" or meta-
In an effort to determine if the unusual HbA 1c result bolic syndrome. Metabolic syndrome is multifactorial
was due to potential hemoglobinopathies, Hb variant in origin; there are 6 major factors involved, abdominal
analysis was done and presence of HbS and HbF identi- obesity, atherogeinic dyslipidemia, hypertension, insulin
fied. In this particular case, increased HbF caused the resistance (with or without the presence of glucose into-
abnormal HbA 1c value. HbA 1c assays can be divided lerance), proinflannmatory state and prothrombotic state.
into methods that use molecular charge (HPLC and The defect may be due to; (1) Prereceptor pathology
electrophoresis) and methods that use molecular struc- - mutations in insulin molecule, anti-insulin antibodies,
ture (immunoassays). Hb variants (or their glycated forms) (2) receptor defects-decreased number of receptors,
may interfere with HbA1c assays based on HPLC and reduced insulin binding , insulin receptor mutations, insulin
electrophoresis by coeluting/comigrating with Hb A and/ receptor blocking antibodies, (3) postreceptor defects-
or HbA 1c. When a spurious HbA1c result is obtained, defective signal transduction, mutations in GluT4, (4)
the possibility of interference by Hb variants should be combination of defects, (5) other pathologies-Werner
considered, Fructosamine and daily testing of glucose syndrome, ataxia telangiectasia, lipodystrophic states,
may be used to monitor glycemic control. These alter- etc. (6) increased production of insulin antagonists, and
native tests may also be used for patients who have an (7) glucose intolerance.
altered erythrocyte lifespan and changes in the degree Other laboratory tests helpful are apo-8, hs-CRP,
of glycation. HbA1c testing cannot be used for these fibrinogen, uric acid, urine microalbumin and liver func-
individuals. tion tests. Treatment includes treating insulin resistance,
0 lipid abnormalities, prothrombotic state, hypertension,
• Clinical Case Study 11.7 Answer impaired fasting glucose and lifestyle modifications. Diet
Patient has hypoglycemia, probably due to fasting. Under and exercise are the keystones to the clinical manage-
physiological conditions, brain derives fuel from glucose. ment.
Hypoglycemia is considered when blood glucose falls to 0
below 60 mg%. Symptoms begin at this concentration • Cllnlcal lCase Study 11 .9 Answer
of glucose; brain symptoms appear when glucose level Gestational diabetes mellitus (see Chapter 24).
176 Section B: General Metabolism

0 signal transduction pathway, which leads to regula-


• Clinical Case Study 11.10 Answer tion of gen•e transcription, DNA synthesis and acti-
vation of enzymes.
Renal glycosuria. Normal renal threshold for glucose is
10. Diabetes mellitus is of two types, Type 1 and Type 2.
180 mg/dl (see Chapter 24 ).
Type 1 is also known as insulin dependent (I DDM),
while the type 2 was previously known as noninsu-
LEARNING POINTS, CHAPTER 1~ lin dependemt diabetes mellitus (NIDDM).
1. Major factors that cause increased level of glucose 11 . Secondary diabetes mellitus can be; manifested in
in blood are; absorption from intestines, glyco- endocrine disorders (Cushing's syndrome, Thy-
genolysis and gluconeogenesis. A continuous and rotoxicosis ), drug induced (beta blockers, ster-
adequate supply of glucose is essential for the oids), seen in pancreatic diseases (chronic pan-
brain, erythrocytes and renal medulla. creatitis).
2. Major factors that cause depletion of glucose in 12. Diabetic kHtoacidosis (DKA), lactic acidosis and
blood are; utilization by tissues, glycogenesis and hypoglycemia are acute metabolic complications of
conversion to fat. Blood glucose level varies signifi- diabetes mellitus.
cantly during the fasting state and in postprandial 13. Retinopathy, neuropathy and vascular diseases are
state (after food). chronic complications of diabetes mellitus.
3. An elevation in blood glucose level stimulates secre- 14. Glycated h,emoglobin (HbA1c) is used as an index
tion of insulin. Insulin in turn favors uptake of glucose for long-term control of blood glucose level.
by body cells and glycogen synthesis. Hyperglyce- 15. The fastin~J plasma glucose (FPG) denotes glu-
mic hormones are Glucagon, Cortisol, Adrenaline cose level after overnight fasting of 8-10 hours.
and Growth hormone. Insulin is a hypoglycemic Postprandial glucose (2 hours PPG) is measured
hormone. 2 hours after taking food.
4. The glucose levels are measured in plasma after 16. The term random plasma glucose (RPG) is used
collecting blood in a tube with oxalate and fluoride. when blood is collected regardless of the time of
True glucose values are given by enzymatic the previous meal.
method (GOP-POD method). Indications for an 17. Normal fasting blood glucose value is 110 mg/dl
oral glucose tolerance test (OGTT) are; patient and 2 hours postprandial value is 140 mg/dl. Hyper-
with symptoms suggestive of diabetes mellitus, glycemia refers to elevated glucose levels.
excess weight gain during pregnancy and to rule 18. A plasma leivel of glucose below 50 mg/dl is called
out benign glycosuria. hyperglycemia.
5. Contraindications for an OGTT are; known case of 19. Fasting value between 111 and 126 mg/di and
diabetes mellitus, to follow prognosis of diabetes 2 hours PPG between 140 and 199 mg/dl indicate
mellitus and performing on acutely ill patients. impaired glucose tolerance.
6. Conditions that can be assessed by OGTT are 20. A diagnosis of diabetes mellitus is made when
impaired glucose tolerance, impaired fasting gly- fasting plasma glucose is above 126 mg/di and
cemia, gestational diabetes, alimentary glucosuria 2 hours PPG above 200 mg/dl.
and renal glucosuria. 21 . Renal threshold for glucose is 180 mg/dl, above
7. Reducing substances in urine other than glucose which glucose is excreted even at a lower level. In
are fructose, lactose, galactose, pentoses, homo- renal glyccisuria the glucose tolerance is normal,
gentisic acid, salicylates, glucuronides and ascor- but glucose is excreted in urine.
bic acid. 22. Insulin is secreted in response to an increase in
8. Insulin has the following biochemical effects: plasma glucose level.
increases uptake of glucose by cells, enhances utiliza- 23. Insulin receptors are located in the plasma mem-
tion of glucose, hypoglycemic, antilipolytic, antiketo- brane with 2-alpha and 2-beta subunits.
genic and favors lipogenesis. 24. Binding of insulin to alpha subunits activates the
9. Insulin acts via a specific insulin receptor present tyrosine kinase activity of beta subunit which will
on cells of insulin responsive tissues. This affects a phosphorylate insulin response substrate (IRS).
Chapter 11 : Regulation of Blood Glucose, Insulin and Diabetes Mellitus 177

The activated IRS in tum will activate other enzyme 27. Glycolysis and glycogen synthesis are stimulated,
system and cascades causing metabol c effects. thus decreasing glucose level in plasma.
25. Insulin has its effects at the level of gen es by induc- 28. Insulin also favours lipogenesis where by excess
tion and repression of enzymes as well as covalent glucose is converted to fat for storage.
modification of enzymes. 29. Gluconeogenesis and glycogenolysis are inhibited
26. Insulin recruits GluT4 in cells to the membrane by insulin, favouring fall in plasma glucose level.
enhancing glucose uptake.

PART-1 : ESS Y AND SHORT NOTE QUESTIONS


11-1. What is the normal fasting blood glue ,se level? How is it regulated?
11-2. What are the hormones influencing blood sugar level and how are these hormones acting?
11-3. Give the normal blood glucose levels in fasting conditions and after a carbohydrate meal. What are the
mechanisms by which normal range i; maintained?
11-4. Discuss the changes in metabolism during diabetic mellitus.
11-5. What are the enzymes influenced by I ,sulin? What are the derangements seen in diabetes mellitus?
11-6. What are the indications of glucose tolerance test? What precautions are to be taken before doing a GTT? What
are the abnormal curves obtained? W at is impaired glucose tolerance?
11-7. Name the reducing sugars that may c ppear in urine and give the differential diagnosis of these clinical condi-
tions.

ORT NOTE QUESTIONS

11-8. Renal threshold for glucose. 11-13. HbA1c.


11-9. Benedict's test. 11 -14. Renal glycosuria.
11-10. Glucose tolerance test (GTT). 11-15. Key enzymes influenced by insulin.
11-11 . Reducing sugars in urine. 11-16. Give the normal blood level of glucose.
11-12. Glycated hemoglobin.

PART-2: ULTIPLE CHOICE QUESTIONS

11-1 . The normal fasting plasma glucose level is: A. Untreated diabetes mellitus with ketosis
A. 40-60 mg /100 ml B. Diabetes mellitus treated with overdose of insulin
B. 70-110 mg/100 ml C. Renal glycosuria
C. 120-150 mg/100 ml D. Thyrotoxicosis
D. 60-180 mg/100 ml 11-5. Activities of all the following enzymes are enhan-
11-2. Insulin activates:
ced in starvation, except:
A. Lipolysis B. Ketogenesis
A. Phosphoenolpyruvate carboxykinase
C. Gluconeogenesis D. Glycolysis
11..J. When a standard oral glucose tole1ance test is
B. Carnitine acyl transferase
done, the blood glucose levels of the patient were C. Pyruvate ca rboxyiase
found to be as follows: 0 min (fasting)= 120 mg/ D. Acetyl-CoA carboxylase
dl; 60 min= 170 mg/dl ; 120 min= 150 mg/dl. The 11-6. Insulin increases activity of all the following enzy-
patient has: mes, except
A. Normal glucose tolerance A. Acetyl-CoA carboxylase
B. Impaired glucose tolerance 8. Hormone sensitive lipase
C. Mild diabetes mellitus C. Glycogen synthase
D. Severe diabetes mellitus
D. Glucose-6-phosphate dehydrogenase
11-4. A comatose patient with tremors ad nitted to the
11-7. The following are llpolytic hormones, except
hospital has a blood glucose value >f 300 mg/dl
A. Glucagon B. Cortisol
and his urine was positive for both F.othera's test
and Benedict's test. He is suffering from: C. Epinephrine D. Insulin
178 Section B: General Metabolism

11-8. Growth hormone causes: C. Diabetes mellitus treated with overdose of insulin
A. Decreased peripheral utilization of glucose D. Renal glucosuria
B. Decreased hepatic production of glucose by glu- 11-17. Benedict's reagent will be reduced by all the fol-
coneogenesis lowing substances seen in urine, except:
C. Increased glycolysis in muscle A. Fructose B. Galactose
D. Decreased lipolysis C. Pentoses D. Sucrose
11-9. All are true with regard to Glucagon, except: 11-18. All affect glucose tolerance test, except:
A. Increases lipolysis A. Liver diseases
B. Stimulates glycogenolysis B. Acute infections
C. Favours gluconeogenesis C. Vitamin B12 deficiency
D. Produces glycogen synthesis D. Hyperthyroidism
11-10. All are true with regard to Glycohemoglobin, except: 11-19. Which statement is true with regard to insulin?
A. Seen as high levels in sickle cell anemia A. Secretion is from the beta cells of pancreas
B. Otherwise known as HbA1c B. Insulin favours gluconeogenesis
C. Useful index of long-term control of blood glucose C. Insulin molecule has two chains connected by
level disulphide bridges
D. About 3-5% of hemoglobin in normal cases D. Insulin level in blood is regulated by the growth
11-11. A lactating mother's urine was found to be positive hormone level
for Benedict's test. Her fasting blood sugar was 80 11-20. All the following cellular activities are decreased
mg/dl and postprandial blood sugar after one and under the influence of insulin, except:
half hours after a breakfast was 140 mg/dl. She A. Protein breakdown
may be a case of: B. Ketogenesis
A. Untreated diabetes mellitus C. Glycogen synthesis
B. Diabetes mellitus treated with inadequate dose of D. Lipolysis
insulin 11-21. Insulin increases activity of all the following enzy-
C. Lactosuria mes, except:
D. Renal glucosuria A. Acetyl-CoA carboxylase
11-12. Diabetes mellitus is caused by: B. Hormone-sensitive lipase
A. Anti insulin antibodies C. Glycogen synthase
B. Low levels of glucocorticoids D. Glucose-6-phosphate dehydrogenase
C. Receptor abnormalities on target tissues 11-22. Insulin secretion is mainly controlled by:
D. Reduced secretion of growth hormone A. Blood lipid level
11-13. Which tissue is most insulin sensitive? B. Hypothalamic mechanism
A. Brain B. Liver C. HCI in stomach
C. Adipose tissue D. Cardiac muscle D. Glucose level in blood
11-14. Which laboratory finding is abnormal in a pregnant 11-23. All are true with Insulin receptor, except:
woman at 28 weeks gestation: A. It is a glycoprotein
A. Serum alkaline phosphatase level of 25 KAU/dl B. Alpha subunit has tyrosine kinase activity
B. Plasma 1hour postprandial glucose of 200 mg/d l C. It has 4 subunits
C. Alpha fetoprotein level of 120 ng/ml D. Alpha subunits are located on the exterior of cell
D. Plasma triglyceride level of 150 mg/dl membrane
11-15. In the enzymatic method of glucose estimation, 11-24. The following are lipolytic hormones, except:
the following reactants are used, except: A. Glucagon B. Cortisol
A. Glucose oxidase B. NAD+ C. Epinephrine D. Insulin
C. Peroxidase D. Hydrogen peroxide 11-25. When glucagon binds to its receptors on the liver
11-16. An apparently healthy man, on a routine check up, cells, the following changes take place, except:
was found to have fasting blood sugar of 80 mg/dl, A. Activation of adenylate cyclase
and urine showed no abnormal constituents. One B. Increase in concentration of cAMP
and half hours after a heavy breakfast, his blood C. Inactivation of glycogen phosphorylase
sugar was 120 mg/dL and urine sample at that time D. Activation of protein kinase
was positive for Benedict's test. He is suffering from : 11-26. Growth hormone causes:
A . Untreated diabetes mellitus A. Decreased peripheral utilization of glucose
B. Diabetes mellitus treated with inadequate dose of B. Decreased hepatic production of glucose by gluco-
insulin neogenesis
Chapter 11: Regulation of Blood Glucose, Insulin and Diabetes Mellitus 179

C. Increased glycolysis in muscle C. Preserve glucose and prevent clotting


D. Decreased lipolysis D. Get quick results.
11-27. All the following enzymes are activated by glu- 11-33. A random sample of blood is collected:
cagon, except: A. Any time regardless of the previous meal
A. Phosphorylase kinase B. 1 hour after a meal
B. Phosphoenolpyruvate carboxykinase C. 2 hours after a meal
D. 3 hours after a meal
C. Liver phosphorylase
11-34. Insulin receptor:
D. Phosphofructokinase
A. Is composed of extracellular subunits only
11 -28. All are true with regard to the Juvenile diabetes
B. Is made of 4 similar subunits
mellitus, except:
C. Has binding site for insulin in all four subunits
A. Patients need insulin injections D. Possesses tyrosine kinase activity
B. Frequent ketoacidosis is common 11-35. Insulin favours:
C. Insulin resistance is the cause A. Ketogenesis
D. Genetic susceptibility is inherited B. Gluconeogenesis
11-29. Which of the following lab results indicate poor C. Lipogenesis
glycemic control? D. Cholesterol synthesis
A. PPBS of 180 mg/dl 11-36. Which of the following processes are Inhibited by
B. Serum cholesterol of 240 mg/dl glucagon?
C. Glycohemoglobin of 10% A. Release of fatty acids from adipose tissue
D. Blood urea level of 35 mg/dl B. Elevating plasma glucose level
11-30. Which of the following enzymes is active in dia- C. Uptake of alanine and lactate by liver
betes but not in starvation? D. Storage of glucose as glycogen
11-37. Which of the following is NOT true regarding type
A. Hormone-sensitive lipase
2 diabetes mellitus?
B. Pyruvate carboxylase
A. Often affects people above 40 years of age
C. Carnitine acyltransferase
B. More prevalent in obese people
D. Aldose reductase
C. Characteristic feature is insulin resistance
11-31. Glucose level in plasma is increased by:
D. Results from beta cell destruction due to auto-
A. Gluconeogenesis immune disease
B. Glycolysis 11-38. Complications of diabetes mellitus mainly result
C. Glycogen synthesis from :
D. Glycosuria A. Non enzymatic glycation of proteins
11-32. Blood is collected in fluoride oxalate bottle to: B. Excess consumption of sugars
A. Prevent clotting C. Excretion of glucose in urine
B. Preserve glucose D. Delayed secretion of glucagon

ANSWERS OF MULTIPLE CHOICE QUESTIONS

11-1 . B 11-2. D 11-3. B 11-4. A 11-5. D 11-6. B 11-7. D


11-8. A 11-9. D 11-1 0. A 11-11. C 11-12. C 11-13. C 11-14. B
11-15. B 11-16. D 11-17. D 11-18. C 11-19. C 11-20. C 11-21. B
11-22. D 11-23. B 11.24. D 11-25. C 11-26. A 11-27. D 11-28. C
11-29. C 11-30. D 11-31. A 11-32. C 11-33. A 11-34. D 11-35. C
11-36. C 11-37. D 11-38. A

PART-3: VIVA VOCE QUESTIONS AND ANSWERS

11-1 . What is the normal level of fasting blood sugar? 11-3. Which hormone is hypoglycemic?
70-110 mg/dl. Insulin.
11-2. What are the sources of blood glucose? 11-4. What are the major actions of Insulin?
(a) Absorption from intestine; (b) hepatic gluconeo- Insulin decreases blood sugar; it stimulates glycolysis;
inhibits gluconeogenesis; enhances glycogen synthe-
genesis; and (c) hepatic glycogenolysis.
sis and inhibits lipolysis.
180 Section 8 : General Metabolism
11-5. What are the hyperglycemic hormones? 11 -21 . What are the pathways stimulated by insulin?
Glucagon, adrenaline, corticosteroids, growth hormone. Glycolysis, Glycogen synthesis, HMP shunt pathway,
11 -6. W hat are the major actions of glucagon? lipogenesis.
Promotes glycogenolysis; enhances gluconeogenesis, 11-22. What are the enzymes stimulated by Insulin?
depresses glycogen synthesis, inhibits glycolysis. Phosphofructokinase; Glycogen synthase, Glucose-6-
11-7. What is the major indication for doing an oral phosphate dehydrogenase; Acetyl-CoA carboxylase.
glucose tolerance test (OGTT)? 11 -23. What are the pathways inhibited by insulin?
Patient has symptoms suggestive of diabetes mellitus; Gluconeogenesis. Glycogenolysis, lipolysis, Ketoge-
but fasting blood sugar value is inconclusive (between nesis.
110 and 126 mg/dl). 11-24. What are the enzymes inhibited by insulin?
11-8. What is the criteria to diagnose diabetes mellitus,
Glucose-6-phosphatase, Glycogen phosphorylase,
with regard to fasting blood glucose level? Hormone-sensitive lipase.
Fasting blood glucose level is more than 126 mg/dl, 11-25. How is insulin secretion controlled?
on more than one occasion.
Glucose is the major stimulant of insulin secretion.
11-9. What is impai~ed glucose tolerance (IGT)?
11-26. How glucose stimulates insulin secretion?
When fasting plasma glucose level is between 110 and
GluT2 receptors act as sensor mechanism for glucose
126 mg/dl or 2-hour post-glucose value is between
level. Cyclic AMP along with calcium causes the insulin
140 and 200.
secretion.
11-10. What is Gestational Diabetes Mellitus (GDM)?
11-27. Which tissue utilises glucose maximally?
This term is used when carbohydrate intolerance is
At basal rates, brain utilises 60% of sugar oxidized.
noticed, for the first time, during a pregnancy.
11-28. What are the cardinal symptoms of diabetes mel-
11-11 . What is the clinical significance of GDM?
litus?
Women with GDM are at increased risk for subsequent
Polyuria, polydipsia, polyphagia, weight loss.
development of frank diabetes. GDM is associated with
11-29. What is the reason for polyuria in diabetes?
increased birth weight of child and increased incidence
of neonatal mortality. When the blood glucose level exceeds the renal
11-12. What is renal glucosuria? threshold glucose is excreted in urine. Due to osmotic
effect, more water accompanies the glucose.
Glucose is excreted in urine due to a lowering of renal
11-30. What is the reason for polydipsia in diabetes?
threshold. The blood sugar levels are within normal limits.
11-13. What is normal renal threshold for glucose? To compensate for this loss of water, thirst centre is
180 mg/100 ml. activated, and more water is taken (polydipsia).
11-14. What are reducing substances seen in urine? 11-31. What is the reason for weight loss in diabetes?
Glucose, fructose, lactose, galactose, pentoses, ascor- The loss and ineffective utilisation of glucose leads to
bic acid, glucuronides. breakdown of fat and protein. This would lead to loss
11-15. Fructosuria is due to what? of weight.
Due to the deficiency of fructokinase or aldolase B. 11-32. What is the reason for polyphagia in diabetes?
11-16. What is lactosuria? To compensate the loss of glucose and protein, patient
It is observed in the urine of normal women during 3rd takes more food.
trimester of pregnancy and during lactation. 11 -33. What is microalbuminuria?
11 -17. What is the clinical importance of lactosuria? Albumin 30 to 300 mg/day in urine. It is a predictor
The condition is harmless. But it is important to dis- of progressive renal damage, atherosclerotic diseases
tinguish lactosuria from glucosuria. and cardiovascular mortality.
11-18. What is the test for reducing sugars in urine? 11-34. What is the difference between glycosylation and
Benedict's test. glycation?
11-19. Where ls insulin synthesized? ·-:, Enzymatic addition of any sugar to a protein is called
Beta cells of islets of Langerhans of Pancreas. "glycosylation", while nonenzymatic process is termed
11-20. What is pro-insulin? "glycation".
Insulin is synthesised as a large single polypeptide. 11-35. What is significance of glycated hemoglobin?
Middle part of it is then removed, to form the A and B An elevated glycohemoglobin indicates poor control of
chains of insulin. diabetes mellitus.
Metabolic Pathways of
Other Carbohydrates

Chapter at a Glance
The learner will be able to answer questions on the following topics:
D Fructose metabolism
D Glycoproteins
D Galactose metabolism
D Blood group antigens
D Metabolism of alcohol
D Mucopolysaccharidosis
D Amino sugars

IFRUCTOSE METABOLISM- - - - - Therefore, in theory, fructose will be better utilized


in patients with diabetes mellitus, because the first few
Fructose is a ketohexose present in fruits, honey and
enzymes of fructose utilization do not require insulin. But
sucrose. Soft drinks have fructose as the sweetener.
in clinical conditions, fructose was found to be deleterious
Corn syrup, which has a high fructose content and is
in diabetic patients. Fructose rapidly enters the tissues,
sweeter than sucrose. Fructose is promptly metabolized
leading to enhanced fatty acid synthesis, raised serum
by the liver.
Fructose is phosphorylated by fructokinase, an triglycerides and increased LDL cholesterol level in
enzyme.,wesent in liver with a high affinity for fructose blood; all these are atherogenic and harmful.
(Fig. 12.1 ). Fructokinase phosphorylates the substrate Fructose metabolism in liver bypasses the PFK con-
at 1st position, whereas hexokinase action is on the trol point; hence fructose increases the flux of glycolytic
6th position. Fructokinase is not dependent on insulin. pathway, leading to lipogenesis. Moreover, glycerol
So fructose is more rapidly utilized in normal persons, phosphate required for TAG synthesis is provided by the
because HK and PFK metabolic bottlenecks are not metabolism of fructose, leading to the increase in TAG
encountered in fructose metabolism. pool in the body.

Dihydroxyacetone phosphate

CH 2OH Fructoklnase yH29P03= Aldolase B yHzOP03=j

T ©
I
C=O C=O C-0
I I CHO

r
I

'\~
HO-C-H HO-C-H CH2OH Trlose kinase I
I ATP ADP I Absent in H-C-OH
+
H-C-OH H-y-OH Hereditary CHO
I I tHztjP03=1
H-y-OH H-y--OH Fructose ATP ADP
Absent in Intolerance H-y-OH
CH2OH CH2OH Glyceraldehyde-
Fructosuria CH2OH 3-phosphate
Fructose-1-
Fructose
phosphate Glyceraldehyde 3P~f\
Fig. 12.1: Fructose entering glycolysis
182 Section B: General Metabolism

BOX 12.1: Neonatal hypoglycemia 1


Galactose - --------+~ Galactose-1 -phosphate
Neonatal hypoglycemia is seen in:
fM9+\
1. Glycogen storage disease type I (see Chapter 1O) ATP ADP UDP glucose
2. Fructose intolerance (see Chapter 12)
Glucose-1-phosphate
3. Galactosemia (see Chapter 12)
GALACTOSEMIA
4. Medium chain fatty acyl-CoA dehydrogenase deficiency (see
Chapter 16)
5. Long chain fatty acyl-CoA dehydrogenase deficiency (see UDP glucose
Chapter 13)
Premature infants are more prone to hypoglycemia because
a. Decreased level of PEPCK leads to decreased gluconeogenesis
b. Low hepatic glycogen stores
Galactose-1-phosphate
c. Inability to produce and utilize ketone bodies 1= galactokinase.
d. Larger brain : body ratio. 2= galactose-1-phosphate uridyl transferase (GALT).
3= UDP-gal-epimerase (uridine diphosphate galactose epimerase).
4= galactose-1-phosphate pyrophosphorylase.

Phosphorylation of fructose by fructokinase depletes Fig. 12.2: Summary of galactose metabolism


the cell of ATP. Low ATP levels enhance oxidative
phosphorylation leading to lowering of Pi levels in cell. and associated hypoglycemia (Box 12.1 ). Vomiting and
This will remove the inhibitory effect on adenosine loss of appetite are seen. The infants often fail to thrive.
deaminase. There is increased rate of conversion of Hepatomegaly and jaundice may occur. If liver damage
AMP to IMP and then to uric acid. Accumulation of fruc- progresses, death will occur. Fructose is also excreted
tose-1-phosphate also inhibits gluconeogenesis. In in urine when urine gives positive Benedict's test. With-
muscle, fructose is phosphorylated by hexokinase to drawal of fructose from the diet will immediately relieve
fructose-6-phosphate. the symptoms.
The fructose-1-phosphate is then cleaved by the
enzyme, fructose-1-phosphate-aldolase or aldolase B. Fructosuria
The products are glyceraldehyde and dihydroxyacetone
phosphate (Fig. 12.1 ). Fructose is mainly metabolized This is a benign metabolic defect due to deficiency of
by liver, but free fructose is seen in large quantities in fructokinase (Fig. 12.1). There is no abnormality other
seminal plasma. The energy for mobility of spermato- than excretion of fructose in urine. Fructose is not die-
zoa is mainly derived from fructose. Fructose is secreted tary essential. Urine gives positive Benedict's and Seli-
by seminal vesicles. wanoff's tests. Incidence is 1 in 130,000 birth•
In some persons, azoospermia is seen due to a
block in the duct. In such persons, fructose is estimated IGALACTOSE METABOLISM
in semen. If fructose is present, the block is above
The term galactose is derived from Greek word, "gala"
the seminal vesicular duct; if absent, block is after the
= milk. Galactose is an aldohexose and is the 4th
seminal vesicles.
epimer of glucose. Galactose is a constituent of lactose
Hereditary Fructose Intolerance (HFI) of milk sugar, and is taken in the diet. Galactose is not
an essential nutrient, because UDP glucose can form
It is an autosomal recessive inborn error of metabolism.
UDP galactose (Fig. 12.2). Galactose is metabolized
Incidence of the disease is 1 in 20,000 births, while 1
almost exclusively by the liver and therefore galactose
in 70 persons r~ ~ - rs of the abnormal gene. The
tolerance test is done to assess the functional capacity of
defect is in ~ dolas ; hence fructose-1-phosphate
cannot be metabo Ize (Fig.1 2.1). This is seen when the liver. UDP galactose is the active donor of galactose
sucrose (containing fructose) is introduced in the diet of during synthetic reactions. Galactose is necessary for
infants, usually around 6 months of age. Accumulation synthesis of the following:
of fructose-1 -phosphate will inhibit glycogen phosphory- i. Lactose Synthesis:
lase. This leads to accumulation of glycogen in liver UDP glucose Epimerase UDP galactose
Chapter 12: Metabolic Pathways of Other Carbohydrates 183

Galactosemia
;;;,,__ _ _ _ _ Mental retardation
There is deficiency of enzyme galactose-1-phosphate
:---.,,- - - - - Congenital cataract uridyl transferase. It is an inborn error of metabolism.
- - - - Jaundice The incidence is 1 in 35,000 births. Herman Kalckar
Hepatomegaly described it in 1958. Due to the block in this enzyme,
Kidney damage galactose-1-phosphate will accumulate in liver. This will
inhibit galactokinase as well as glycogen phosphorylase.
Hypoglycemia is the result (see Box 12.1 ).
Bilirubin uptake is less and bilirubin conjugation is
reduced; so unconjugated bilirubin level is increased in
blood (for bilirubin, see Chapter 22). There is enlargement
of liver, jaundice and severe mental retardation. Free
Galactose and galactose accumulates, leading to galactosemia. It is
Amino acids in Herman Kalckar
urine partly excreted in urine (galactosuria). Galactose is
reduced to dulcitol (see Chapter 7). The accumulation of
Fig. 12.3: Clinical features of galactosemia
dulcitol in the lens results in cataract due to its osmotic
effect. This is called congenital cataract and is a very
UDP galactose + glucose Lactose synthase
characteristic feature of galactosemia. Galactose-1-
Lactose
phosphate may get deposited in renal tubules, producing
Lactose synthesis is seen in lactating mammary glands tubular damage leading to generalized aminoaciduria.
(see Chapter 29). These salient clinical manifestations are summarized in
ii. Synthesis of glycosaminoglycans Figure 12.3.
iii. Synthesis of cerebrosides
Diagnosis: Clinical manifestation including congenital
iv. Synthesis of glycolipids
cataract and presence of galactose in urine as well as
v. Synthesis of glycoproteins.
elevated blood galactose levels will help in the diagnosis.
Galactose Utilizing Steps Collection of fetal cells by amniocentesis may be useful
in prenatal diagnosis. Heterozygous parents could be
1. Galactokinase reaction: Galactose is first phos- detected by elevated galactose level in blood after a
phorylated by galactokinase to galactose-1-phos-
galactose load.
phate (Step 1, Fig .12.2).
Treatment: If lactose is withdrawn from the diet, most
2. Galactose-1-phosphate uridyl transferase (GALT):
of the symptoms recede. But mental retardation, when
This is the rate-limiting enzyme in galactose meta-
established, will not improve. Hence early detection is
bolism (Step 2, Fig.12.2). UDP-galactose may be
most important. For affected infant lactose-free diet
used as such for synthesis of compounds contain-
is given. Such special diets may be withdrawn after
ing galactose (e.g. lactose).
4 years, when galactose-1-phosphate pyrophosphorylase
3. Epimerase reaction : By this reaction, galactose is
(Step 4, see Fig. 12.2) becomes active.
channeled to the metabolism of glucose (Step 3,
Fig. 12.2). Since the reaction is freely reversible,
even if the dietary supply of galactose is deficient,
Galactokinase Deficiency
UDP-glucose can be epimerized to UDP-galactose. A variant of the disease occurs due to the deficiency of
4. Alternate pathway: The galactose-1-phosphate galactokinase. But here the symptoms are milder. This
pyrophosphorylase in liver becomes active only is because galactose-1-phosphate is not formed and
after 4 or 5 years of life. The enzyme will produce hence no toxic effects of this compound are manifested.
UDP-galactose directly, which can be epimerized to However, cataract is seen. Galactokinase deficiency is
UDP-glucose (Step 4 , Fig. 12.2). reported to be 1 in 40,000 births.
184 Section B: General Metabolism

Alcoholdehydrogenase Aldehyde dehydrogenase


----::_""''-:-----•
Ethanol------
CH 3 - CH2 -OH
NAO+
7
/
NAOH + H+
Acetaldehyde
CH 3 - CHO T\
- - - - - , . . . . _ , - - - - - - • Acetic acid

NAO+ NAOH + H
+
CH3 - COOH

Fig. 12.4: Alcohol metabolism

IMETABOLISM OF ALCOHOL ii. Deficiency of pyruvate leads to inadequate for-


mation of oxaloacetate. This results in depression
Alcohol absorption starts from the stomach itself, but most of gluconeogenesis, leading to hypoglycemia.
of it is absorbed by intestine. Only 1% of the ingested iii. Reduced oxaloacetate, decreased pyruvate and
alcohol is excreted through the lungs or urine. Major high NADH level causes suppression of TCA cycle.
fraction of the alcohol is oxidized in the liver. So Acetyl-CoA is accumulated, which favors keto-
genesis.
Alcohol Dehydrogenase (ADH) iv. Increased level of acetyl-CoA causes increased
fatty acid synthesis; but fatty acid is not oxidized.
It is an NAO• dependent cytoplasmic enzyme. It oxidizes
So fat is accumulated in liver, resulting in fatty liver
ethanol to acetaldehyde (Fig. 12.4). In some individuals
and steatosis.
the enzyme is mutated. This mutation rate is more in v. Alcohol causes CNS depression.
Orientals. In such individuals, alcohol metabolism is vi. In pregnancy, alcohol consumption leads to fetal
slower and even small quantity of alcohol may produce alcohol syndrome (FAS).
symptoms of intoxication.
Chronic Alcoholism
Aldehyde Dehydrogenase Chronic alcoholism leads to liver disease or cancers of
The acetaldehyde is further oxidized to acetate by a the liver, lungs or breasts. Genotypes, drinking history,
mitochondrial NAO• dependent enzyme (Fig. 12.4). gender, quantity, etc. are the factors which influence
The acetate is then converted to acetyl-CoA. The acti- alcohol metabolism.
vity of alcohol dehydrogenase is more than aldehyde
Alcoholism and Liver
dehydrogenase. So acetaldehyde accumulates in liver.
Aldehyde is toxic, which in excess may lead to cell Accumulation of fat in liver cells leading to fatty liver.
death. The activity of aldehyde dehydrogenase is less in Accumulated toxic effect of acetaldehyde leads to
Indians, when compared to Europeans. cellular death. This is followed by replacement by fibrous
tissue. Fibrosis of liver is called Cirrhosis. When liver
functions are reduced (see Chapter 24) hepatic coma
Microsomal Ethanol Oxidizing
results.
System (MEOS)
It is another mechanism of detoxification of alcohol. It Alcoholism and Nervous System
is cytochrome P450 dependent and is inducible. This In chronic alcoholics, the brain ventricles are enlarged,
accounts for metabolic tolerance of alcohol observed in neurons are lost, neurodegenerative changes set in and
chronic alcoholics. memory is affected. In alcoholics, combined thiamine
deficiency leads to Wernicke's disease . Aldehyde inhi-
Biochemical Alterations in Alcoholism bits pyridoxal phosphate; hence neuritis is very common
Both the oxidation steps of alcohol produces NADH, in alcoholics (see Chapter 33). Alcohol abuse is asso-
resulting in a high NADH/NAD• ratio. As a result, several ciated with depression, severe anxiety, insomnia, suicide,
metabolic adaptations occur: heart disease, stroke, cancers, liver cirrhosis, amnesia,
i. In the cytoplasm, the high NADH level favors con- temporary cognitive deficits, sleep problems, peripheral
version of pyruvate to lactate, leading to lactic neuropathy; and fetal alcohol syndrome. Hyperuricemia
acidosis. and gout are exacerbated by alcohol.
Chapter 12: Metabolic Pathways of Other Carbohydrates 185

Epimerase Glu-6-P
N-acetyl-glucosamine N-acetyl-
mannosamine i
F-6-P - - - - - . GluN-1 -P - -- • UDP-GluN

i i
l
~ ATP
GluN-6-P - - - GluNAc- 1-P

, N-acetyl- +-- -- -- --
Mutase
~ ADP
i i
N-acetyl- GluNAc-6-P ---+ GluNAc Glycoproteins
i
!
mannosamine- mannosamine- and GAG
-6-phosphate -1-phosphate
ManNAc-6-P
+ Phosphoenol pyruvate i GalNAc /
J
Aldolase enzyme NeuraNAc-9-P

l
N-acetyl-neuramlnic acid -9-phosphate
Fig. 12.6: Interrelations of amino sugars
P = phosphate; Glu = glucose; F = fructose; GluN = g lucos-
amine; Neura = neuraminic; NAc = N-acetyl; UDP = uridine
Fig. 12.5: Synthesis of N-acetyl neuraminic acid (NANA) diphosphate; GAG = glycosamino glycans

Amino sugars are used for synthesis of glycosami-


Type Eponym Deficient enzyme Clinical findings noglycans and gangliosides. Mannose (6C) and pyruvate
Hurler's L-lduronldase MR+++; Skeletal defor- (3C) combines to form 9C compound, N-acety1 neura-
mity++; Corneal opacity++;
minic acid (NANA). The steps are outlined in Figure 12.5.
OS and HS in urine
Lysosomal storage disorders due to defective degra-
II Hunter's lduronate MR+; Skeletal deformity++;
sulfatase deafness; no corneal dation of GAG are called Mucopolysaccharidoses. The
clouding; OS and HS in urine major features of different types of such disorders are
Ill Sanfillppo's N-Acetyl MR++; Skeletal deformity+; given in Table 12.1. All amino sugars are derived from
glucosaminidase, corneal clouding+; HS in
Heparan sulfatase urine; 3 different types are
glucose (Fig. 12.6).
reported
IV Morquio's Galactosamine
sulfatase,
MR+; Skeletal deformity+;
epiphyseal dysplasia+;
IGLYCOPROTEINS- - - -
13-0-galactosidase Corneal opacity+; KS and CS Glycobiology is the study of the sugars in health and
in urine; 2 types reported disease. The glycome is the entire component of sugars
MR= mental retardat ion; CS = chondroitin sulfate; KS = keratan sulfate; in the cell. Glycomics is the comprehensive study of
HS= heparan sulfate; OS = dermatan sulfate
the carbohydrates in the cells as a whole, functions and
metabolism.
Laboratory Findings in Chronic Alcoholism Glycoproteins have a protein backbone to which
oligosaccharide groups are attached. Only 8 monosac-
Increase in serum levels of gamma-glutamyl transferase
charides are commonly found in such side chains; some
(GGT) and alanine transaminase {ALT) (see Chapter 6)
of them will be further acetylated or sulfated. These are
are seen. Decrease in aldehyde dehydrogenase activity
glucose, galactose, mannose, galNAc, gluNAc, xylose,
(in liver cells and RBCs) is the best marker for alcohol fucose and N-acetyl neuraminic acid (sialic acid).
abuse. Proteoglycans are complexes of glycosamino-
glycans and proteins. As the carbohydrate content is
~ ETABOUSM OF AMINO SUGARS increased, viscosity is increased and solubility is decrea-
sed. The carbohydrate content of mucins is generally
The amino sugars, N-acetyl glucosamine, N-acetyl gal-
more than 50%. Mucus consists of 5-10% of mucins.
acto-samine and N-acetyl neuraminic acid are synthe- These monomers are further linked together by disulfide
sized from fructose-6-phosphate. The amino group is linkages, to form oligomers. Mucins will form a protec-
derived from amide group of glutamine. The reaction is tive barrier on epithelial surfaces. They are also found
catalyzed by an amido transferase. This is irreversible; it in secretions of the gastrointestinal, respiratory and
is the rate limiting step in amino sugar synthesis. urogenital tract.
186 Section B: General Metabolism

TABLE 12.2: Functions of glycoprote1ns TABLE 12.3: Glycosam1noglycans


Function Example ofglycoprotein GAG Repeating sugar units Linkage Tissues
Structural substance Collagen, bacterial cell walls Hyaluronic N-Acetyl glucosamine fj-1,3 Synovial fluid,
Enzymes Ribonuclease-8, Prothrombin acid and Glucuronic acid vitreous humo r
Transport proteins Cerulop lasmin, Transferrin Chondroitin N-Acetyl galactosamine f3-1,3 Cartilage,
Ho rmo nes Thyroglobulin, Erythropoietin, TSH sulfate Gl ucuronic acid bone, cornea

Immunity lmmunoglobulins, Blood group antigens Keratin sulfate N-Acetyl g lucosamine I} 1,4 Cornea,
Ty pes I and II and Galactose car tilage
Lubricant Mucin
Heparan N-Acetyl glucosamine a -1 ,4 Skin
M essage transfer Receptor proteins on cell surfaces
Cell adhesion Selectins and integrins
The H locus codes for fucosyl transferase. In a per-
Glycoproteins are widely distributed in tissues. All son belonging to blood group A, N-acetyl-galactosami-
plasma proteins are glycoproteins. nyl transferase is present. In group B person, galactosyl
In glycoproteins, the carbohydrate groups are atta- transferase is seen. Lack of both leads to blood group
ched to the polypeptide chain by the following types of 0 , while AB persons have both enzymes.
linkages:
a. Through the amide group of asparagine to N-acetyl Proteog lycans
glucosamine (N-glycosidic linkages). These are widely distributed complex molecules having
b. Through hydroxyl group of serine, threonine, hydroxy- glycosaminoglycans (GAG) and proteins (Table 12.2).
lysine and hydroxyproline to N-acetyl glucosamine The GAGs containing repeating disaccharide units are
or galactose or xylose (beta-O-glycosidic linka- covalently bound to the peptide chain to form proteogly-
ges). cans. Structures of the repeating sugar units are shown
There can be many (usually 20-30) oligosaccharide in the end of Chapter 7. The different GAGs found in
chains attached to a single protein. Each sugar different tissues are shown in Table 12.3.
chain may range from 1 to 20 in length.
Enzymatic addition of specific carbohydrate unit on Biosynthesis of Glycoproteins
the protein is called glycosylation, while nonenzymatic
The biosynthesis of GAG is taking place in endoplas-
spontaneous addition is called glycation.
mic reticulum and Golgi bodies, with the help of specific
Blood Group Substances (Antigens) glycosyl transferases.
The glycoproteins having mannose-6-P as the end
The RBC membrane contains several antigenic sub-
residue will be directed to lysosomes. Mannose phos-
stances, based on which persons are classified into dif-
phate acts as the tag to deliver such glycoproteins into
ferent blood groups. More than 160 different antigens
lysosomes. Defect in this correct tagging will lead to
are known. Of these, ABO system and Lewis system
defective targeting. See I-cell disease, described in
are known to involve glycoproteins. ABO system is asso-
Chapter 41 .
ciated with 3 blood group substances on RBCs desig-
nated A, B and H antigens. The H antigen is the basic
Congenital Disorders of
structure. It has the following structure:
Glycosylation (COG)
Fucose-Gal- GalNAc- Protein
These are autosomal recessive disorders. They are
The RBCs carrying such H antigen are denoted as
multisystem disorders, mostly affecting CNS, with men-
blood group 0 . A and B antigens differ from this in hav-
tal retardation .
ing additional sugar residues.
A antigen: - Fucose-Gal-GalNAc-Protein
Paroxysmal Nocturnal
I
GalNAc Hemoglobinuria (PNH)
B antigen: - Fucose-Gal-GalNAc-Protein There is somatic mutations in PIG (phosphatidyl inositol
I glycan) gene in hematopoietic cells. During sleep, there
Gal is a slight drop in pH of blood, causing mild RBC lysis by
Chapter 12: Metabolic Pathways of Other Carbohydrates 187

TABLE 12.4: In born errors associated with carbohydrate metabolism


Name Incidence 1out of Defective enzyme Salient features [ Chapter no.
Glycogen st orage disease, Type I 100,000 Glucose-6-phosphatase Hepatomegaly, cirrhosis, 10
(von Gierke's disease) hypoglycem ia, ket osis, hyperuricemia
Do, type II (Pompe's disease) 175,000 Lysosomal maltase Generalized glycogen deposit; lysosomal 10
storage d isease
Do.type Ill (Cori's disease) 125,000 Debranchlng enzyme Hepatomegaly, cirrhosis 10
Do,type IV (Andersen's disease) 1 million Branching enzyme Do 10
Do, t ype V (McArdle's disease) 1 millio n Muscle phosphorylase Exercise intolerance 10
Lactose intolerance Lactase Milk induced diarrhea 10
Fructose intolerance 20,000 Aldolase B Hypoglycemia, vomiting, hepatomegaly 12
Fructosuria 130,000 Fructokinase Benign reducing sugar in urine 12
Galactosemia 35,000 Gal-1-P-uridyl transferase Hypoglycemia ; hepatomegaly; mental 12
retard ation; jaundice; congenital cataract
Essential pentosuria 2,500 Xylitol dehyd rogenase Benign reducing sugar in urine 10
PC deficiency 25,000 Pyruvate carboxylase Mental retardation 10
GPO deficiency 5,000 Glucose-6-phosphate Drug-induced hemolytic anemia 10
dehydrogenase
HK deficiency Hexokinase Hemo lytic anem ia 10
PK d eficiency Pyruvate kinase Hemolytic anemia 10
PDH deficiency 250,000 Pyruvate dehydrogenase Neuro nal loss in brain; muscular hypotonia; 10
lactic acidosis

complement factors, which leads to excretion of hemo- ground substances. In general, defective degrada-
globin in urine. Normal persons can withstand this lysis. tion of heparan sulfate leads to mental retardation
predominantly whereas accumulation of other GAGs
Rheumatoid Arthritis leads to mesenchymal abnormalities. A characteristic
There is reduced glycosylation of circulating immuno- finding is the excessive excretion of any one of the
globulin G, these are aglycosyl lgG molecules. These will GAGs in urine. All these diseases are inherited as auto-
activate complement system, causing chronic inflamma- somal recessive traits, except Hunter's disease, which is
tion of synovial membranes of joint cavities. X-linked (see Table 12.1).
The inborn errors associated with carbohydrate
Viruses
metabolism, are shown in Table 12.4.

"
Many viruses will attach on human cells through spe-
cific glycoproteins. HIV virus (causing AI DS), attaches
on lymphocytes by r:ieans of a surface glycoprotein
•i i• Clinical Case Study 12.1
(gp120) of the virus (see Chapter 47). Influenza virus A 2-year-old child presented with liver enlargement.
attaches by hemagglutinin (H). So neuraminidase inhibi- Investigations showed the following results: Blood sugar
tors are used as antiviral drugs against H1 N1 . - 50 mg%, Uric acid - 10 mg%, Lactic acid - 15 mmol/L,
Plasma cholesterol - 300 mg% and ketone bodies were
M ucopolysaccharidoses present.
These are a group of inborn errors of metabolism char- What is the likel y diagnosis?
acterized by excessive intralysosomal accumulation What is the biochemical basis of the disorder and its
of GAG in various tissues. They are progressive disor- treatment?
ders. The clinical manifestations include coarse facial
features, thick skin and corneal opacity due to accumu-
lation of GAG.
a0 Clinical Case Study 12.2
Mental retardation, growth deficiency and skeletal A 3-year-old boy is brought to the emergency depart-
dysplasia are also seen due to defective formation of ment after several episodes of vomiting and lethargy.
188 Section 8 : General Metabolism

His pediatrician has been concerned about his failure Biochemical basis of disorder: Because of a genetic
to thrive and possible hepatic failure along with recur- disorder, the hepatic aldolase B enzyme is defective,
rent episodes of the vomiting and lethargy. After a care- and functions normally in glycolysis but not in fructose
ful history is taken, it is found that these episodes occur metabolism. Glucose production is inhibited by eleva-
after ingestion of certain types of food, especially high ted fructose-1-phosphate. When fructose is ingested,
in fructose. His blood sugar was checked in the emer- severe hypoglycemia results.
gency department and was extremely low. Treatment Avoid dietary fructose.
1. What is the most likely diagnosis? Clinical correlation: Deficiency of aldolase B is an auto-
2. What is the biochemical basis for the clinical symp- somal recessive disease, leading to fructose intolerance.
toms? It does not cause difficulty as long as the patient does
3. What is the treatment of the disorder? not consume any foods with fructose or sucrose. Fre-

a0
quently, children with fructose intolerance avoid candy
Clinical Case Study 12.3 and fruit. However, if chronically exposed to fructose-
containing foods. infants and small children may have
A 10-year-old child was brought to the OP Department
poor weight gain and abdominal cramping or vomiting.
with complaints of constant dribbling of thick mucus from
mouth and not responding to surroundings. Clinical his- 0
tory revealed delayed developmental milestones since •ti• Clinical Case Study 12.3 Answer
2 years of age. The child has coarse facial features and
Diagnosis is Hurler's syndrome, a type of mucopoly-
thick mucus and skeletal deformities. Urinalysis revealed
presence of heparan sulfate and dermatan sulfate. saccharidoses (MPS-I). There is deficiency of a-L-
ldurionidase. This leads to defective degradation of der-
What is the probable diagnosis?
matan sulfate and heparan sulfate. They accumulate in
What is the biochemical basis of this disorder?
tissues and are also excreted in urine. Clinical features

a0 Clinical Case Study 12.4


include progressive deterioration, hepatosplenomegaly,
progressive mental retardation , coarse facial features,
A middle aged man complained of fatigue. There was loss of physical skills, hearing loss (conductive hear-
vague abdominal pain. On examination, there was enlarge- ing loss) and language problems, skeletal deformities
ment of the liver. Laboratory results obtained were as (including disproportionate growth of trunk and extremi-
follows - Serum AST - 120 U/L, ALT - 80 U/L, ALP - ties). etc. These children often die by 10 years of age.
68 U/L, GGT - 170 U/L, Bilirubin - 1.0 mg/dL, glucose Urine analysis for Mucopolysaccharides is diag-
- 60 mg/dL, Uric acid - 8.0 mg/dl. CBC and urinalysis nostic in these patients. Urine cetyl trimethyl ammo-
were normal. nium bromide test for mucopolysaccharides is positive.
What is the diagnosis? Absence of a- L-ldurionidase in WBC is diagnostic.
Interpret the biochemical data on the basis of your diag- Treatment options are bone marrow transplantation and
nosis. umbilical cord blood transplantation, before 18 months
of age. There is no cure for the disease. Gene therapy

a0 Clinical Case Study 12.1 Answer


trials and enzyme replacement therapies are ongoing.
These options are partially beneficial (see Table 12.1 for
Galactosemia . See Figure 12.3 for details. Hypogly- further details).
cemia causes increased fatty acid catabolism leading to 0
increased acetyl-CoA production. This excess acetyl • • Clinical Case Study 12.4 Answer
-CoA leads to increased synthesis of cholesterol and
The patient has alcohol related liver disease. Hypo-
ketone bodies.
glycemia and hyperuricemia are due to alcoholism.
Alcoholism is associated with fatty liver, hepatitis and
0
cirrhosis. High GGT also points toward a diagnosis of
• · Clinical Case Study 12.2 Answer
alcoholism. There can be acute or chronic inflammation
Diagnosis: Fructose intolerance. and parenchymal necrosis, which is often reversible.
Chapter 12: Metabolic Pathways of Other Carbohydrates 189

Fatty liver is caused by impaired fatty acid oxidation


and increased lipogenesis, due to changes in [NADH)/
ILEARNING POINTS, CHAPTER 12
[NAD] ratio in liver. Oxidation of alcohol by alcohol 1. Deficiency of aldolase B enzyme leads to an inborn
dehydrogenase leads to increased NADH. This NADH metabolic disorder termed hereditary fructose
can compete with other substrates like fatty acids for the intolerance (HFI).
respiratory chain , inhibiting their oxidation and causing 2. Deficiency of galactose-1-phosphate uridyl trans-
increased esterification of fatty acids to form triglycer- ferase is the cause of galactosemia.
ides and causing fatty liver. Oxidation of ethanol pro- 3. Characteristic features of galactosemia are con-
duces acetaldehyde and acetate. Increased (NADH]/ genital cataract, generalized aminoaciduria, hepa-
[NAD] ratio also causes increased [lactate]/[pyruvate] tomegaly, jaundice and mental retardation. Lactose-
ratio leading to elevated lactate, which decreases uric free diet forms the main stay of treatment.
acid excretion aggravating gout. 4. Metabolism of alcohol involves two enzymes; al-
In alcoholism, usually ALT is higher than AST cohol dehydrogenase and aldehyde dehydroge-
(Please note that in this patient it is not the case), GGT nase.
is high and ALP may be normal. Serum bilirubin may be 5. Gamma glutamyl transferase (GGT) is used as a
high , serum proteins may be reduced, and hypoglyce- clinical marker of alcoholism.
mia, hyperuricemia, ketosis and metabolic acidosis may 6. Blood group antigens are glycoproteins.
be seen. Fasting hypoglycemia is due to high lactate 7. Mucopolysaccharidoses (MPS) are a group of inborn
and low pyruvate . Level of b hydroxy butyrate is ele- metabolic disorders caused due to accumulation of
vated due to high NADH leading to ketosis. Ketoacidosis mucopolysaccahrides in tissues. Features include
and lactic acidosis are accompanying features. mental retardation.

PART-1: ESSAY AND SHORT NOTE QUESTIONS

12-1 . Describe the process by which galactose is converted into glucose. Indicate the metabolic errors associated
with this pathway.

SHORT NOTE QUESTIONS

12-2. Enzyme defect in congenital cataract. 12-5. Essential pentosuria.


12-3. Lactose intolerance. 12-6. Metabolism of alcohol.
12-4. Hereditary fructose intolerance.

PART-2: MULTIPLE CHOICE QUESTIONS

12-1. All are true with regard to fructose intolerance, 12-3. Features of galactosemia include the following,
except except
A. Defective enzyme is aldolase B A. Cataract B. Hepatosplenomegaly
B. Fructose-1-phosphate accumulates C. Mental retardation D. Hemolytic anaemia
C. Glycogen phosphorylase is inhibited 12-4. Which inborn error will result in mental retardation?
D. Urine is positive for Rothera's test A. Fructosuria
12-2. All are correct with regard to fructose metabolism, B. Essential pentosuria
except C. Galactosemia
A. Fructose is metabolized in liver D. Glucose-6-phosphate dehydrogenase deficiency
B. Free fructose is seen in seminal plasma 12-5. Excess intake of alcohol may produce lactic aci-
C. Block in seminal vessels is indicated by the abs- dosis, because:
ence of fructose in semen A. Alcohol is oxidized to lactic acid
D. Compared to glucose, fructose is sluggishly meta- B. Alcohol is oxidized to acetaldehyde and then to
bolized lactic acid
190 Section B: General Metabolism

C . NADH is g enerated, which converts pyruvate to 12-9. Fructokinase catalyses which reaction?
lactate A . Fructose ---> fructose-6-phosphate
D . Ethanol induces Cori cycle, by wh ich lactic acid B. Fructose---> fructose-1-phosphate
produced from muscle is taken to liver C. Fructose ---> fructose-1 ,6-bisphosphate
12-6. N-acetyl neuraminic acid (NANA) (sialic acid) is D. Fructose-> fructose-2,6-bisphosphate
synthesized by: 12-10. Which is true with regard to fructosuria?
A . N-acetyl glucosamine + glyceraldehyde-3-phos- A . It is due to the absence of Aldolase-B

phate B. Severe mental retardation is seen


C. Urine is positive for Benedict's test
B. Galactose-6-phosphate + pyruvate
D. Glycolysis is inhibited
C. Glucose-6-phosphate + lactate
12-11. In glycoproteins the outermost carbohydrate resi-
D. N-acetyl mannosamine-6-phosphate + phospho-
due is donated by:
enolpyruvate
A. ADP-xylose B. UDP-glucosamine
12-7. Mannose derivatives are produced from:
C. CMP-NANA D. GDP- fucose
A. Glucosamine
12-12. Cataracts are formed due to accumulation of
B. N-acetyl glucosamine
C. Glucose-1-phosphate A . Xylitol in essential pentosuria
D. UDP-glucose B. Galactitol (dulcitol) in galactosemia
12-8. Hereditary fructose intolerance is usually mani- C . Mannitol in galactosemia
fested when the child is about 6 months old, because D. Ribitol in renal glycosuria
12-13. Excess ingestion of alcohol may produce hypo-
at that age
glycemia, because ethanol:
A . Defective e nzyme is produced in liver
A. Inhibits gluconeogenesis
B. Fructose is introduced in diet B. Favours glycogen synthesis
C. Mother's milk is reduced C. Increases secretion of insulin
D. Starch diet is being introduced D. Inhibits absorption of g lucose

ANSWERS OF MULTIPLE CHOICE QUESTIONS


12-1. D 12-2. D 12-3. D 12-4. C 12-5. C 12-6. D 12-7. B
12-8. B 12-9. B 12-10. C 12-11. C 12-12. B 12-13. A

PART-3: VIVA VOCE QUESTIONS AND ANSWERS

12-1. What is fructose intolerance? 12-7. Why five years? Why not life-long?
Due defective aldolase B. So, fructose-1 -phosphate By five years, the a lternate pathway (galatose-1-phos-
accumulates. phate pyrophosphorylase) becomes active.
12-2. What is the clinical application of fructose esti- 12-8. Why intake of alcohol produce lactic acidosis?
mation in semen? During alcohol oxidation, NADH is generated, which
Fructose is secreted by seminal vesicles. A block in converts pyruvate to lactate.
seminal vessels in indicated by the absence of fructose 12-9. Why excess intake of alcohol produce hypoglyce-
in semen. mia?
12-3. Neonatal hypoglycemia is seen in which conditions? Because ethanol inhibits gluconeogenesis.
Gly_!'.:ogen storage disease, type I; Galactosemia; Fruc- 12-10. What are the features of chronic alcoholism?
tose intolerance. Polyneuropathy; Fatty liver; Cirrhosis, encephalopathy.
12-4. Congenital cataractis seen in which condition? 12-11 . In glycoproteins, carbohydrate residues are atta-
Galactosemia. ched to which group of the polypeptide chain?
12-5. Galactosemia is due the absence of which enzyme? Hydroxyl group of serine or threonine.
Galactose-1-phosphate uridyl transferase. 12-12. Increased glucosaminoglycans in urine is seen in
12-6. What is the treatment for galactosemia? which condition?
Lactose free diet is given for first five years of life. Mucopolysaccharidosis.
.,______ _ __ Chapter 13
Metabolism of
Fatty Acids

Chapter at a Glance
The learner will be able to answer questions on the following topics:
D Digestion of Lipids D Omega Oxidation
D Absorption of Lipids D De Novo Synthesis of Fatty Acids
D Chylomicrons D Synthesis of Triacylglycerols
D Beta-Oxidation of Fatty Acids D Metabolism of Adipose Tissue
D Oxidation of Odd Chain Fatty Acids D Fatty Liver and Lipotropic Factors
D Alpha Oxidation D Ketogenesis and Ketolysis

I DIGE~PT,IP N OF LIP!,_rDS ,...i


9-,' ~ej, ,..9,
,... -oo-%-°J
reased. This process is -
0
t~nsion is reduced; and surface ace.a_of droplets is inc-
favored.by: ·
-- • ~o
The major di~ary lipids are tri'acylglycerol (TAG) (trigly- Bile salts (detergent action) _,. " ,~ ~61Jo\~~\eY- e;S
1
cerides), cholesterol and phospholiQid The average y Peristalsis (mechanical mixing) ,,u
normal Indian diet contains about 20-30 g of lipids per _ Phospholipids. , , , P~ I.l:..Ot9.. \-ec.r-- ~0
day. Western j iet generally contains-tw_o or_..!bree.limes "
3
' , 7:. c;.oJn -..,.\,dJcu.
v.~v-.e, f"e..C~\C~
oi (} Bile Salts are Important
. . .
more than this quantity. '1

e D1gest1on in
/ii,
Stomach -,
'
for Digestion of Lipids
The bile salts present in the bile (sodium glycocholate
The lingua(i)pase from the mouth ent~ ~ ,§.\qlJlfch and sodium taurocholate) lo~r surface _tension~They
i:r-.

along with the food. It h-;;;s an optimum p H ofIT~ 5~The emulsify the fat droplets. The emulsification increases
enzyme therefore continues to be active in the stomach. the sliliace area of the particles for enhanced activity of
It acts on sh_ort chain triglycerides (SCT). SCTs are pre- enzymes (Fig. 13.1 ).
sent in milk, butter and hee. The action of lingual lipase - O -=========~
0
is observed to be more significant in the newborn infants.t)Lipolyti c Enzymes in Intestines
~'}Afric lipase is acid stable, with an optimum pH about 1 _ Pancreatic lipase with co-lipase ( R\.-)
_ 4. It is secreted by Chief cells, the secretion is stimu- 2 _ Cholesterol esterase C. (.E.)
lated by Gastcin. Up to'@_0% digestion__,,of triglycerides 3 _ Phospholipase ~- c~ C.f:l~ c. 1
occurs in stomach. The bile (pH 7.7) , entering the duodenum serves to
neutralize the acid chyme from the stomach and provides
@ Digestion in Intestines _, a pH favorable for the action of pfil!_creatic _enz~mes.
Emulsification is a pre-re_guisiter for digestion of lipids. A list of physiologically important lipases is shown in
The lipids are dispersed into smaller droplets; $.!Ifface Table 13.1.
~rnw..(, "\j co}\on e ~u< d.-~~~'f/2)~

~ , t'-U' ~ \ . ~ _,, CD Ss.~ CT) £,-~o-1'


192 Section B: General Metabolism

rt t_.
,,.. , .. TABLE 13.1 : Phys1olog1cally important llpases
'
.._ /\
i
I
,,,'
, Lipase
Lingual/acid-
Site ofaction Preferred substrate Product(s)

..,. ' -I
Fat Mouth, TAGs with short and FFA+ DAG
-,,,,,,, stable lipase stomach medium chain FAs
'
, I'
- "if .. ___,. I
,,I Pancreat ic
lipase+
Small
intest ine
TAGs with long-
chain FAs
FFA +
2MAG
,. ,, ,,.
I l
.. ,. ,
, co-lipase
I Intestinal lipase Small TAGs with medium 3 FFA +

,,' I
with bile acids
Phospholipase
intestine
Small
chain FAs
Pls with unsat. FA
glycerol
Unsat FFA
A + bile acids intestine on position 2 Lysolecithin
Fig. 13. Acti Ie The hydrophobic portions of bile
Capillary TAGs in chylomicron FFA+
salts intercalate into the large aggregated lipid, with the hydr.
walls orVLDL glycerol
philic domains remaining at the surface. This leads to breakd
of large aggregates into smaller and smaller droplets. Thus t Adipocytes TAG stored in FFA + DAG
surface area for action of lipase is increased adipose tissue

~ .
CH2- HO + RrCOOH
Lipase I Lipase
CH-0- CO- R2
I
CH - 0-CO-R
+ H20
2 3

.
Triacylglycerol 2,3-diacylglycerol

Fig. Complete hydrolysis of triglyceride. In the intestines, general y fats are only partially hydrolyzed
f ,_cv C\ t,;.
'i)()t"'"U.'-" \-o "f("'\-e. CV bi
cf) Digestion of Triacylglycerols
Pancreatic lipase can easily hydrolyze the fatty acids
esterifie™ a~ d~ s of glycerol
I 1-.· forming - on l'iefd and two molecules of fatty
'. P ,,rc---..I·acid (Fig. 13.2). Then ari isomerase shifts the ester bond
\~ _ from position 2 to 1. The bond in the 1st position is then
-V' ,w .drolyzed by the lipase to form free glycerol and fatty Bergstrom
~-€ acid (Fig. 13.2). The major end products of the diges- NP 1982
\ e.~~tion ofTAG are 2-MAG (78%), 1-MAG (6%), glycerol and 1916-2004
J fatty acids (14%). Thus digestion ofTAG is partial (incom-
A~ 4pl~). Cholesterol ester may be hydrolyzed to free cho- Long chain fatty ac,d
OH OH

~· ~ lesterol and f ~ i d . The action of phospholipase


tf (. produces lysophospholipid and a fatty acid (see Fig. 8.8).
r,. .,
Fig. 13.3: Micellar formation
' ~L\..H er---"- 9,ve f'fP (l.Jj eN DyM·
\.3,/Colipase blood. The theory proposed by Bergstrom (Nobel Prize,
·,Prf:i
The binding of colipase to the ~ lglx:cerol malei;;..ules 1@f) has the following ~teps.
at the oil:water interface is obligatory for the action of (~ } . ~t c;re._-',-JOf l-
lipase. The colipase is secreted bv the eancreas as an Mixed Mice/le Format,o:3 <..., 1 r'\-''- C 11
~ Jive zymogen (molec~ ar w~ht 11 ,000). It is acti- The bile sa micelle incorp~ ~ prog~cts of digestion,
v ~sin .

I
R~ ~y~,o~>:f \~ ,1 ~ f
c'tfolesterol, f)h cisbholipids and ljio~hospholipids into
ABSORPTION OF LIPIDS -:J f(' 0 ~ f 'molecular aggregates to form__mixed micelle (Fig. 13.3).
~ mely 2 '- 0io~°Jglycerols:~'9P~~~rin fatty acids,

t,t'v-1<" .A~ h . II hD-Wnb)u o<\-6t10..n ~th h d h·I·


Ab f fL Ch • F tty A "d
5
¥!'- ; e mice es are sp errcaI pamcIes wI a y rop IIc
sorp ion° ong am a CJ exterior and hydrophobic interior core (Fig. 13.3). Due
Long chain fatty acids (chain length more th~ ~ ~ ar- to their amphipathic nature, the bi,!.e salts help to_form
bans) are absorbed to the lymph and not directly o the micellar aggregates. Micellar formation is essential for
Chapter 13: Metabolism of Fatty Acids 193

Gallbladder- -tl--- Stomach


Bile salts- -~
(emulsifying - ;;;;.
-= ...,..
--=.::::.,;...
;;;;:;.~
- ,- - - Pancreas
agent) -==-.::;;=r-- -"'!::itl?- Hydrolysis
of TAG

Chylomicrons
carrying
Adipose - ----1+-- -<,, TAG
tissue with
stored fat -,,..=-------l--f-- Albumin
carrying
Muscle and- - -~,a.i.+- free fatty
peripheral acids
tissues

Lacteal Oxidation and


I mph vessel generation or ATP
Fig. 13.5: Summary of utilization of fat
Fig. 13.4 : Digestion and absorption of triglycerides
LC = long chain; SC FFA = short chain free fatty acid. Triglycerides r 5 0 N')'('{)
with long chain fatty acids are absorbed into lymph vessels as
chylomicrons. Short chain fatty acids directly enter portal vein 1. M inor digestion of triacylglycerols in mouth and stomach
by
(blood vessel). lingual (acid-stable) lipase
,. "" ,.._ i , • 0 )'o,o~ 2. Major digestion of all lipids in the lumen of the duodenum/
, • ''-"- 1 jejunum by pancreatic lipolytic enzymes
the absorption of fat-soluble vitamins such as vitamin 3. Bile acid facilitates formation of mixed micellesGI \
A , D and K. The l)licelle!i are aligne~d a.t the miivillous 4. Passive absorption of the prd~\r~ iffito1ys~~ ~he
d A""
su rface of the jej llnal l
muco sa. Fat - cids, 2- e, G and mixed mice11e into the intestinal epithelial cell
other digested product pq§_sively diffus 5. Q!e-esterificat§) of 2-monoacylglycerol with free fatty acids ( c.,F~)
osal cell (Fig. 13 .4)_ '60-tN O'T L c...FPi "'~~~>'0- insidetheintestinalenterocyte
6. Assembly of chylomicrons containing Apo B48, triacylgly-
., fl fl c- ~· cerols, cholesterol esters and phospholipids and export from
Enteroh(j]patic Circulation of Bile Salt~, intestinal cells to the lymphatics.
~O-Q..\\ ci.obPo-. (.Nc £.o~) j .__ '1 1 C"i
The 1::i1le salts are left behind which are moldy reab- t"2J i\ / .J c:1. re ....,c-..f>
sorbed fr9m the ileum a nd returned to
' he J1c.ver to be Chy/omicrons. L_,. • "'' K'\O<-t- ';-:)e), ~--r0,:- 1~ w < '
r~-excret~d ~enterohepatic circulation). About (9,8% ....Qf The 1...k~.
choleiter-0~ ter and phos~AOligid '.11alecules
dietalY]ip ~ are normally_a~ ed . along with apopr&eins B48, and a are incorpora-
-'f\ .7.1~ ,,, ~ ~ ~~ coi\i-N.~ -µ\:,J..~ted into chyl~ i~ 1 3 .5). The chyle (milky fluid)
~ ~nficat,on lns,c l e ' tucosa/
'i't~ Cell '\ from the intestinal mucosal cells loAded with chylomi- ,,,..,,,·,•f'
(_ ~~\°t-1€51. 'p~ aJ::.. ~) ,,.,,--,---.~ '.,,p..:_t.! oop_i j,I " ,,.. 'o-r~'t--
Once inside the intestinal mucosa! cell, the ng chain crons are '~ t h r o u g h the ~al!l 1Ato t'- t~o- •,.p .)v..
fatty acids are re-esterified to fo rm triacylglycerols. The ra..9ic duct a Rd the11 e111ptied i11tl>s__ystemic circulation--- • atH b.~
fatty acids are-firsC activated~ atty__ac')'heoA by the (Fig. 13.4). The serum may appear milky after a high '"' ',rv.r.
e~ zy~ :tcy . oA syntfle tasy pr
t,Q~in'tr~,(see Fig. f ~ I (post~pr~ndial ~ipemia) due to th~ pre~ence of
13.8). This needs J~sis of two high energy bonds. Two c~yl?m1crons m circulation. Normally the h~em1a cle~rs
such activated fatty acids react with monoacyl-glycerol w1th1n a few hours by the uptake of chylom1crons by t1s-
(MAG) to form the triacylglycerol. Maj ority of molecules sues. The major steps of lipid absorption are shown in
8 13 1
follow this MAG pathway (Figs 13.4). Free glycerol ~ · · ,.:; µ v-t''ftt\,r.,~~
absorbed f rom intestinal lumen direct! enters in SCFA Absorption is Different ,,, RPrr..J ~\,..i.lr
bloodstream. So ee glycerol is not available for re-est , q_'y,il~ ~~o=
rification. But the cells can derive glycerol phosphate Sh ort c hain fatty acids (SCFA) (seen in milk, butter, c.,.-~
from glucose by glycolysis, and add 3 molecules of acyl ghee) and medium chain fatty acids (MCFA) (in coconut
groups to synthesize TAG. "- ' ' oil and mother's milk) do not need re-esterification. They
( \ " r,,A
1 •1'-..J'J
•\1°ff'U'\
<;.v
'

G_ uj-u-o\ ®
~.11e.:l
10,~ w<&U--- 1-;,,,.J. c.!o-e'.:.
194 Section B: General Metabolism ,,.,. K_o. mo\:,
'
BOX 13.2: Absence of d1gest1ve Juices L BETA-OXIDATION OF FATTY ACIDS
1. In pancreatic deficiency: Steatorrhea; unsplit fat is present
This) process ii./<nown as beta-oxidation, because the
in stools
oxm:ation and splitting of tw.9-carboOJW.l!s occur at the
2. When bile is not available: Absorption is defective; split fat
l'r 6eta.carb..on_.atom. The oxidation of the hydrocarbon
is present in stools; defective absorption of vitamin K leads to
Rrolon ed rothrombin time. chain occurs by a sequential cleavage of two-carbon
atoms (Fray Knoop, 1904).

can directly eriterjDto blood vessefs. then top-9rtal v.e.in , Preparative Steps for Beta-Oxidation
finally !o___!tter where they are immeeiately utilized for
energf fig. 13.4). Their absorption is rapid. They are The c9,~nzyme A is a~mplex molecule containing B
better absorbed than long chain fatty acids. Peculiarities com1t1lf~ min Q.9n~ ffienic_gg_d and a molecule of
in digestion and absorption of medium chain fatty acids bi ~ tmer.capto ethaooJarnine; this SH group forms thio-
8
are given Table 16.1.
0
ester bond in acyl-CoA (Fig. 13.6). To emphasize the
function of the SH group, the CoA is sometimes written

® Abnormalities in Ab~~on of Lipids ®c~i;lVAT(ON :


@ ~~fective digestion ~~ j ~ a ~ r ~aily excretion of Preparative Step 1: Activation of Fatty cids
fat ~ feces is more than o g per day. (Greek word, "steat", """' , e' '· '
means fat). It is due to chJQQiC diseases of paocre..s. In Fatty acids are activated to their coenzyrne A (CoA)
such cases, unsplit fat is seen in feces (Box 13.2). derivative. This activation is taking place in cytoplasm.
@ Defective absorption : On the other hand, if the ATP is hydrolyzed to AMP and PPi and the energy from
absorption alone is defective, most of the fat in feces hydr~sis o~Pi drives the reaction forward. Thus two
. . may be split fat, i.e. fatty acids and monoglycerides. (Box hig l!_en~ y ~ ond~ : utilized in thiOOction.
13.2). Defective absorption may be due to diseases: l e enzyme is~jokjn;fs~ brfa'ttyacyl-CoA syn-
~ \[.t..~ A. Celiac disease, sprue, Crohn's disease. ~ h as (Step 0, Fig. 13.8). Acetyl group and acyl groups
~....owr\.- B. Surgical removal of intestine. are I erent; see Box 13.3.
bI , t\<U2- N.J'C. Ob~ uction ofr,Rile duct: This may be due to Three different enzymes, one each for short chain,
k\w.~ gall~,&_es, tumo,W of head of pancr~s. enlarged medium chain and long chain fatty acids have been
C ~' lymp~ands, etc. The result is d_§ficjency of bile salts i~'.fied. Small ch~ ~ci_gs also be activated
t)o.rut \c-,e-- In such cases, triacylglycerols with short chain and b~uo ihorase enz e, using succiny!-CgA (see under
bo.o.x,.,_ medium chain fatty acids {SCT and MCT) are diges- ketone bodies .
&; • ted and absorbed properly, because they do not Cf?;) tRANSP0Rf :
require micellerization for absorptiQfl. Since milk "Preparative Step 2: Role of C~ rnitine
fat and coconut oil are made up of MCT, they are Fatty acids are activated in the cytoplasm: but beta-
therapeutically useful in malabsorption syndromes. oxidation is in mitochondria. So transport of fatty acids
Chyluria. There is an abnormal connection between through t_tti~eArpit?chondrial membrane is essential._The
l ~I:.?" lhe urinary tract and lymphatic drainage system of the lq_ng ch~intatW acyl-CoA cannot ass through the inner
~-:~· ~ ntestine. Urine appears milky due to lipid droplets.
.-_orllr',. . mitochondr~me_ mbrane. Therefore a transporter, car-
Y""'"""- Chylothorax can result from an abnormal connec-
. b nitine is involved in transfer of fatty acids. Carnitine is
t1on etween the pleural cavity and thoracic duct.
• ~ I f,
b t h d
e a~rox y
t · th · b
- <!!Jl_ma- n e 1 ammonium utyrate,
r,v-~ s~ 01I <' -
~ Fate of Chy/omicrons ()f>,O~id. ' ~,-e.,~ ~

The absorbed (exogenous) triacylglycerols are trans-


· bl d h .
~~,.-
_,_r.,"-
-- (CH ) - N+-CH CPIOH CH - COOH.

repara
- --3·
~t . St
1ve
ll

ep .
'
2 2

ported m oo as,c ~10 1 1crons. They are taken up by ..


adip9se tis~'J.os ~ lefa o~ le and liver. Liver synthe- Carmtme Acy/-Transferase
sizes endogenous tnacylglycerols~ These are trans- The enzyme carnitine acyl transferase-1(CAT-I) will trans- ·
ported as VLDL (very low density lipoproteins) and are fer the fatty acyl group to the hydroxyl group of carnitine
transported to adipose tissue. A summary of lipid trans- to form acyl carnitine (Fig. 13.7) The reaction occurs on
port is shown in Figure 13.5. the cytosolic side of inner mitochondrial membrane.
fr L,

Beta-mercapto
I a. ,:J
Chapter 13: Metabolism of Fatty Acids 195
r, J JJ)

ethanolamlne Beta-alanine • ,nu~ ~ ADP


- CH2-CH2-NH ~ CH2-CHrNH - CO-CHOH-qtH3h·CH2 - - - - - 0-P-0-P-0-R1boaa Adenine

Fig. 13.6 : Structure of coenzyme A (CoA) (CoA-SH) /


f":.- _

X
BOX 13.3: Acetyl and Acyl groups are different Mitochondrial
membrane--tt-·
1. Acetyl-CoA is the combination of acetate or acetic acid
(2 carbon unit) with Coenzyme A.
Ac11l-
2. Acyl-CoA means acyl group (any fatty acid, C4 to C26 in
Carnitine co:a.
length) combined with Coenzyme A.

®
Preparative Step 4: Translocase
Acyl
carnitine
CoA Co.~
A P_!:.~tejn tran_slo~ will gm:yJh,.e_ac_ylcamitine..across
the me brane to the matrix of mitochondria. On the Cytosol Mitochondri;1
matrix side of the ~ e another enzyme, carnitine
acyl transferase-lll{S1T-!!),-'will transfer the acyl group Fig. 13.7: Role of camitine in transport of acyl groups
( = Carnitine acyl transferase
back to co-enzyme A ~cule (Fig. 13.7). Carnitine is
.....,
returned to the cytosolic side by the translocase.
Beta-Oxidation Steps
Clinical Applications The next 4 reactions are sequentially repeated for com-
MC.f'F'I, .SC ff!\ plete oxidation of fatty acids. After one round of four
• 1. Medium chain and short chain fatty acids do @ metabolic steps, one acetyl-CoA unit is split off and acyl-
req uire carnitine for transport across the inner mito- CoA with 2 carbon atoms less is generated. This would
chondrial membrane. So, medium chain and short undergo the same series of reactions again until the
chain f~ acids re easily oxidized. fatty acid is completely oxidized.
2. Carnitme dl _
1 1e~5;~repo~ed in. P.rn.nts,
in wh . dtattv'aciB°ox~~s no ice . So Step 1: FAD Linked Dehydrogenase ,. IJ~ ~o.tt,-t\)
~ r ~ ~ se,Js0 utilized , resulting in episodes of The fatty acyl-CoA is dehydrogenated to a transenoyl-
hypoglycemia. CoA with FAD accepting the hydrogen atoms (Step 1,
.. , _ ,. __,., :. Fig. 13.8). FADH 2 when oxidised in electron transp'.lrt
Carmtme -=> NI n n G.rui €. } ~ain will produce 1.5 ATP molecules.

Carnitine is s nthesized in the @ c and ~ s "om Step 2: Hydration \--rtJ :::h·'l ri..ro~'\
I sirre and met ·onin . During ro or pregnanc the I

requirement o migbt exceed.its natural produc- This is catalyzed by an enoyl-CoA hydratase (step 2,
tion. Human genetic disorders, affecting different steps Fig. 13.8). This step forms a beta-hydroxy fatty acyl-
of carnitine metabolism will cause deficiency of fatty acid CoA. The L isomer alone is formed during the hydration
of the trans double bond.
oxidation.
During the aging process, camitine concentration in
I' t I~ b
cells diminishes. B e are particularly affected adver-
Step 3: NAO+ Dependent Dehydrogenase~
sely, leading t~ opo in elderly subjects. Admin- The beta-hydro~y fatty acyl-CoA is again oxidized to fo1m
istration of carnitine 1s capable of improving the clinical beta-keto fatty acyl-CoA. (Step 3, Fig. 13.8). This dehydro-
condition. .iLl ~J ~!oh~ genase acts only on L isomer. The NADH when oxidiz13d
v The class~ pifi~entati~ frprffuary carnitin~~fi- in electron transport chain will generate 2.5 ATPs.
ciencBs hep~ megaly, ele~ ted transaminases, and
hyper'a~monemia. T~ causes for seJttl[1dary carnitine Step 4: Cleavage r \"\Ar\ q fl c. t" l. ~ '
deficiency are organ~ cidurias, rd dfi!'g -induced (val- The beta-keto fatty acyl-CoA now undergoe~ thiolytic
proic acid,~gyudine). \URlNm~eet\ cleavage, splitting off a molecule of acetyl-CoA and
;lf <!- WXj wid:lii ~t
196 Section B: General Metabolism

Fatty acid \ Fatty acyl-CoA


R - CH2-CH2 CH 2- CO- OH ( C1s ---+ Acetyl -CoA
@ AMP+PPi
Cu ---+ Acetyl -CoA
(Step 0) Fatty acyl-CoA synthetase or Thiokinase
(Cytoplasm)
C12 ---+ Acetyl -CoA
Mitochondrial Membrane Carnltlne Transporter
(Mitochondrial side) ---+ Acetyl -CoA
Fatty acyl-CoA C10
R- CHrC~ r CO-SCoA
Ca ---+ Acetyl -CoA
Acyl-CoA-r : FAD
dehydrogenase
ji1f (Step 1) FADH - 1.5ATP Cs ---+ Acetyl -CoA
2
~Oc).. by R- CH 2 CH=CH- CO-SCoA
J OJ"MJ a-fl-unsaturated fatty acyl-CoA C4 ---+ Acetyl -CoA

yo~<Ihtj Enoyl/CoA- 1 Acetyl -CoA


IL.:. r,,1,- hydratase +H20
~oo.. (Step 2)
Fig. 13.9 : Summary of beta-oxidation of palmitic acid (16 C )
R- CH2- CHOH- CHr CO-SCoA It undergoes 7 cycles, which give rise to 8 molecules of acetyl-CoA.
II-hydroxy fatty acyl-CoA

ll
ll-hydroxy-t NAO+ BOX 13.4: Summary of beta-ox1dat1on
f.itty acyl-CoA-
dehydrogenase + When one molecule of palmitate undergoes beta-oxidation, the
(Step 3 NADH+H- 2.5ATP net reaction is:
R- CH2- CO - CHr CO-SCoA

®
l
f.l -keto fatty acyl-CoA Palmitoyl-CoA 8 Acetyl-CoA
+ 7 FAD + 7 FADH2
!3-k~· Thiolase + CoA-SH + 7 NAO' - -- + 7 NADH
(Step 4)
+ 7 H2O + 7 H"
R- CH 2 CO-SCoA + CH 3 CO-SCoA
+ 7 HSCoA
Fatty acyl-CoA + Acetyl-CoA
I ( - l,C..) I
(Steps 1,2,3,4)
...._;_;.__ ___;_ _ TCAcycle (10ATP)
(further cycles) gives 10 molecules of ATP. Each molecule of FADH2
Fig. 13.8: Beta-oxidation of fatty acids
produces 1.5 molecules of ATP and each NADH gene-
Important to remember that the first step is FAD dependent rates 2.5 molecules of ATP, when oxidized in the elec-
and the third step is NAO• dependent. tron transport chain . Hence, the energy yield from one
molecule of palmitate may be calculated as:
leaving behind a fatty acid with 2 carbon atoms less 8 acetyl-CoA x 10 = 80 ATP
(step 4, Fig . 13.8). 7 FADH2 x 1.5 = 10.5 ATP
7 NADH x 2.5 = 17.5 ATP
Further Cycles Gross total = 108 ATP
The newly formed fatty acyl-CoA will sequentially under- Net yield = 108 minus 2 =106 ATP
go further cycles of step s 1, 2 , 3 and 4 of beta-oxidation (In the initial activation reaction, the equivalents of 2
until the fatty acid i(s_ofiip_Ieteiy__sor1vertei!_@ acetyl-C0) high energy b s a e utilized). The efficiency of beta-
(Fig. 13.9). A summary is shown in Box 13.4. oxidation is bou 33°0 The differences in oxidation of
unsaturated fa y I s are shown in Chapter 16.
Energetics of Beta-Oxidation A few years back, calculations were made assum-
(ATP Yield) ing that NADH produces 3 ATPs and FADH generates
2 ATPs. This will amount to a net generation of 129 ATP
Palmitic acid (16 C) needs 7 cycles of beta-oxidation . per palmitate molecule. Recent experiments show that
So, it gives rise to 8 molecules of acetyl-CoA. Every these old values are wrong, and net generation is only
molecule of acetyl-CoA when oxidized in the TCA cycle 106ATPs.
Methylmalonyl-CoA '\ Ketoacidos1s, hypotonia, 1- 1')
mutase or B12 yiJ:,/ hypoglyce'mia, hyper- ,::,.
coenzyme ammonemrci, hyperuricemi : '
Ketoacidosis, hypotonia, 7-
vomilng, lethargy
Acidosis, hyperammone,rnia;
hypoglycemia, fatty liver
Long chain acyl-CoA Nonketotic hypoglycemia,
dehydrogenase low carnitine, increased acyl '
carnitine
Glutaryl-CoA Ketoacidosis, convulsionf
dehydrogenase progressive neurological '
defects, cerebral palsy '

Regulation of Beta-Oxidation
i. The availability of free fatty acid (FFA) regulates the Fig. 13.1 O: Metabolism of propionyl-CoA

I net utilization through beta-oxidation.


ii. The level of FFA, in turn, is controlled b luca- removal are the general lines of managemer3 Exam-
gon:insulin ratio G cagon increases FFA lev and ples of these disorders are given in Table 13.2.

I insulin has the oppos, e e e . C:.--- L =D ½ \">'tq~


iii. CAT-I is the regulator of entry of fatty acid into mito-
chondria. Malonyl-CoA inhibits CAT-I activity. Thus
during de novo. synthesis of fatty acid, beta-oxida-
OXIDATION OF ODD 3 v(\:a..r-n'q-~
CHAIN FATTY ACIDS Vi't }3~, BJ, Bl i_
Th e odd c ham
• fatty ac,'ds are ox,·d·Ized exac~ t,..ct'.
ly m \ . e -
tion is inhibited. &tti 1c .,. ame manner as even cham · f atty ac,·d s. However, a1·ter
Defects in Beta-Oxidation ':'S '\'"' OQ).U- successive removal of 2-carbon units, at the end, oni:i 3
carbon unit, propionyl-CoA is produced.
Abnormalities in transport of fatty aci s into mitochon-
dria and defects in oxidation can lead deficient energy Fate of Propionyl-CoA
production by oxidar of long chai fatty aci~ Com- Carboxylase: Propionyl-CoA is first carboxylated to
mon featureH re h keto ic h o I cemia ~ - D-methyl-malonyl-CoA by a biotin dependent carbo.)(y-
roonemja, skYetal muscle weakness and I~ . lase. Biotin is a member of vitamin B complex group.
Acyl carnitine accumulates when the transferases One molecule of ATP is utilized lo supply energy (step 1,
or translocase is deficient. Dietary supplementation of Fig. 13.10).
carnitine has been found to improve the symptoms in Racemase: Then racemase acts upon D-methylmalonyl-
some cases. CoA to give L-methylmalonyl-CoA. (step 2, Fig. 13.10).
.,:,11 .;J_C C.Q.X n,'¼ '(\ e....
. . Organic Acidurias ~ c lU\ ~
Mutase: Then L-methylmalonyl-CoA is rearranged to
form succinyl-CoA by L-methylmalonyl-CoA mutase.
f They are disorders of metabolism of fatty acids, The reaction needs vitamin 8 12 coenzyme (Step 3, Fig.
¼ranched chain and aromatic amino acids and citric acid 13.10).
Succinyl-CoA then enters TCA cycle, finally conver-
cycle. They are all characterized by the accumulation
ted to oxaloacetate, and is used for gluconeogenesis.
of o~ic acids in body tissues and their excretion in
Propionyl-CoA is also derived from the metabolism of
urin~he patients present with acidosis, vomiting, con-
valine and isoleucine (see Chapter 18).
vulsions and coma. The children often die in infancy;
in case they survive, there is severe mental and physi- Propionate is Gluconeogeni
cal retardatio.fil{9iagnosis is confirmed by showing the
,..eresence of organic acids in urine by chromatography} Ordinary fatty acids are cleaved to acetyl-CoA units which
L Dietary restri.ction, cofactor therapy and substrate on entering the Krebs cycle are completely oxidized to
198 Section B: General Metabolism

CO2, and hence as a general rule, fatty acids cannot Infantile Refsum's Qis.aase
be used for gluconeogenesis. However, propionate is
It is a(tSer o Xl.J·soS~ . , similar to Zellweger synd-
entering into the citric acid cycle at a point after the CO2 rome and adr e'!li e ~o_Eh-y (see Chapter 41 ).
elimination step~ so propionate can be channelled to
= -
gluconeogenesis. Thus 3 carbon units from odd car-
Hence phyta ic acid accumulates along with VLCFA.
Children do not survive long. 'f.oll'5
*bon fatty acids are gluconeogenic. Cow's milk con- &o,.'Y\if,
tains significant quantity of odd chain fatty acids. Lc@GA OXIDATION :;,,le] ~Mil J
CGio.-u.u.1ev Rarrde,'J . . th t k' I
It Is a minor pa way a 1ng p ace . .
1
J
1crosomes, wI·th
th,,.,r- '
Inborn Errors of Propionate Metabol ism the help of hydroxylase enzymes involving NADPH and
1. Propionyl-CoA carboxylase deficiency. It is chara- cytochrome P-450. The CH3 group is converted to CHpH
§._..-1- cterized by propionic acidemia, ketoacidosis, and and subsequently oxidized with the help of NAO· to a
developmental abnormalities. COOH group to produce@1carboxylic acJ~. w-oxidation
2. Methyl malonic aciduria. Some patients respond becomes important when 13-oxidation is defective and
to treatment with pharmacological doses of B12 . This dicarboxylic acids (6C and BC acids} are excreted in
group had deficiency in the formation of adenosyl urine causing dicarboxylic aciduria. Peroxisomal oxi-
B12 with deficient mutase activity. The second type dation is described in Chapter 16, under VLCFA.
did not respond to cyanocobalamin and had defi-
ciency of the enzyme racemase or mutase. Methyl- Inherited Disorders
malonate affects tht£'®abo~ of brQin leading to Inherited defects in the 1'.nzymes of b~-oxidatioJ;Hlnd
meQ,tal retardati~ ·n these cases. r _, ketog~esis also lead to'Wonket~~~oglycemia, ~ ma,
=~ v-iti (~ n
1
and fatty liver. Defects are known in 3-hydroxyacyl-CoA
1 ALPHA-ox1DAT10N _,,.;;; ..c "irCl--r' dehydrogenase, 3-ketoacyl-CoA thiolase and HMG-CoA
It is a process by which fatty acids are oxidized by lyase deficiency. Dicarboxylic aciduria is characterized
re.rD,Q',illl carbon-atoms, o,oe~ a.t~ om the car OET by the excretion of dicarboxylic acids and by nonketotic
end. The process is important io brain. The fatty acid hypoglycemia. It is caused by a lack of mitochondrial
does not need activation. Hydroxylation occurs at the medium-chain acyl-CoA dehydrogenase.
alpha-carbon atom. It is then oxidized to alpha-keto More than 25 enzymes havebeenidenti~ for fatty
acid. The keto acid then undergoes decarboxylation acid metabolism in humans, out of which at>;Q.ub 5 are
yielding a molecule of CO2 and a fatty acid with one car- known to be associated with metabolic disorders.
bon atom less. This process occurs in the endoplasmic
reticulum, does not require CoA, but does not gene- DE NOVO SYNTHESIS
rate energy. Some fatty acids undergo alpha oxidation OF FATTY ACIDS f \-~
in peroxisomes also. -----
The process of fatty acid synthesis was studied by
Alpha-oxidation is mainly used for fatty acids that
Feodor Lynen, w h o ~ ize in 1964. The pathway
-
have a methyl group at the beta-carbon, which blocks
is referred to as L nen s s ira It is not a reversal of
beta-oxidation. A major dietary methylated fatty acid is
oxidation . Important di nces in synthesis and break-
phytanic acid. It is derived from phytol present in chlo-
down of fatty acids are given in Table 13.3.
rophyll, milk and animal fats.
This pathway operates in the cytoplasm. So it is
referred to as extramitochondrial las ·c fatty
Refsum's Disease acid synthase system.
It is a metabolic error due to lack of alpha-hydroxylase The major fatty acid synth~sized de novo is palmitic
(phytanic acid oxidase) so that alpha oxidation does not acid, the 16C aturated fatty acid. The process occurs
occur and phytanic acid accumulat~ in the tissues. The in b"·adipose tissue, kidney, ~ n, and m mmary
patie,m presents wit~evere nearologic~ symptoms, glands.
polyne_1Kopathy, retinitis pigmentosa, nerve~afness and
..tet> Q ~ " ' j 1,... ~~N>•r·
ti; I,. ,..,., CoT:f .....
cerebe'llar ataxia. Regression of symptoms is observed ransport of /1+.. etyl-CoA to Cytoplasm
with restricted dietary intake of phytanic acid. Milk is a Acet I-CoA is formedinside the mYt'~ Ro~ l from pyru-
good source of phytanic acid, which may be avoided. vate. The inner membrane is not freely permeable to
....
.. . . .
TABLE 13.3: Difference 1n the twc pathways Fatty acid

Site
Intermediates
Beta-oxidation
Mitochondria
Present as Ce>A
Gllft!llllMS
Fatty acid synthesis
Cytoplasm
Covalent lyilliikicl to
~of/0
l MITOCHONDRIA
beta-oxidation

Acetyl-CoA

Enzymes Presentas iode. Multi-enzyme ~


pendent proteins
Sequential unit s Two carbon units split Two carbon units added,
off as acetyl-CoA as 3 carbon malonyl-CoA
Coenzymes N,\D,l..anc:IFAO are NADPH used as reducing
Oxaloacetate
reduced
Transporter CarnitinJ! \(:NADH
Regulation lnsuliun .J,
Glucagon t ~NAD+

Malaw
acetyl-CoA. Hence the,acetY.1-CoA units are ~ d
to the cytoplasm citra Fi9i 13.11 ). In the crtoplg2~1 NADP+ .j
citrate is cleaved to o~aloa~e ~nd ace~ CoA' in th"e1
cytoplasm. The enzyme is ATP citrafe lyase. The oxalo-
acetate can return to the mitochodtia as malat or
NADPH --1
pyruvate (Fig. 1 1. < e.,N
4~ · ~ ~ ~ .~ - - - ~~ -- - ~
l Y'> . Fig. 13.11: Transfer of acetyl-CoA from mitochondria to
Fatty'Acid Synthase (FAS) Complex cytoplasm by malate-oxaloacetate shuttle
= = =
1 citrate synthetase; 2 ATP-citrate lyase; 3 malate
This system exists as a multi-enzyme complex. The dehydro-genase; 4 = malic enzyme
enzymes form a dimer with identical subunits. Each
subunit of the complex is organized into 3 domains with Third Domain or Releasing Unit l I )
7 enzymes (Fig. 13.12).
It is involved in the release of the pa Imitate synthesized.
Advantages of Multi-enzyme Complex It contains th jo-esterase JbE~ or de-acylase (Fig. 13.12).
lo. Th~ \N>.. ~ ' t ~ -
in_teraGt Step 1: Carboxylation of Acety/-CoA 3oo~
The first step in the~ \.synthesis is the•.wroo_xx- rod!t
b. of ~ yl-CoA to form~ . Acetyl-CoA
carboxylase is not a part of the multi-enzyme complex.
C. But it is the rate-limiting enzyme. a member
t): ' CJIL of B complex _vi tamios, is necess.ary for this reaction
First Domain or Condensing Unit 0
u.1step 1 in Fig. 13. 13). --
It is the initial substrate bin\~~ site. The en= invl The enz~me i~ aJ!,R_~eric~_lly regulated1) ~~- i1?.!o .d-1
ved are b £ ~ ~ ~ a s e or condensin.g_enz.~ effectors being g trate (pos1t1ve) and palm, oyl-Co J
(CE);o.~ etyl transferase'(AI) and malo ny) transacylase (negative). The reaction is similar to carboxylation of
(MT) (Fig. 13. 12). - ~ M . ~ '\:.&.~ pyruvate to form oxaloacetate.
roi c..~1 The a
Second Domain or Reduction Unit the 2-carbon units am
( 4) added as 3-carbon, malonyl units. The whole reaction
It contains the dehydratase (DH); enoyl r.edu.ctase (ER); sequence occurs while the intermediates ace bouru:Uo
beta-keto acyl ~ as.e (KR) and a~ ~e.r.. prote~ ACP (acyl carrier protein). _. A 't·\;..~-' , a,.~
(AC~ _Fig. 13.12). The acyl carrier protein ha~ - =-
~ tottieine group, to which the acyl groups are atta- Step 2: Three C and Two C Units are Added
ched in thioester linkage. So ACP acts like the CoA A. Acetyl transacylase (AT in Fig. 13.12) catalyze!;
carrying fatty acyl groups. the transfer of the acetyl group (2 carbons) to the
0.'
'( <;,),...."f". Condens ing unit , , Reduction unit
@
\
CE - AT - MT----''--.,.----- DH - ER - KR - ACP- - - - -- .

',,,,,
yys ', Translocation
SH
SH '
I ', I
SH ',

',
' ',
TE - - - - - - - - - -ACP- KR - ER - DH - - - - - ' - , - MT- AT - CE
R9lualng 111111 Reduction unit ' Condensing unit

CE = Condensing enzyme; AT= Acetyl transacylase; MT = Malonyl tr,insacytase; DH = Dehydratase


ER = Enoyl reductase KR = Keto acyt reductase; ACP = Acyl carrier protein TE = Thie esterase

~~:J
lil!d .. ,.
Fig. 13.12: Fatty acid synthase complex
Upper and lower units are two monomers of the complex. Dotted line represents functional division.
f'Qrf'> 1~ L'tllte ~oo~__,,c,..\t\
• ~l V t
cyst~inyl SH group of the condensing enzyme Cycling of Reactions
(CE) of the other monomer of the fatty acid sm--
thase complex (step 2A in Fig. 13. 13). The butyryl group (4C) is now transferred to the SH
B. One molecule of acetyl-CoA (2 carbon ) and one group of the condensing enzyme on the other mono-
molecule of malon I-CoA 3 carbon) bind to the mer and a 2nd malonyl-CoA molecule binds to the
.~ ti-enzyme comp~ . Malonyl transacylase (MT fili(~mlQ.:Pantot; ! sH rou ~~ he sequ~J'.\ce of reac-
(.t'I°<'~ :~v-fn~ e r s the malonyl group to the SH tior~ , namely con · ;}tion, reduc ion, de dr ion and
~.,~ group of the ACP (step 28 in Fig. 13.13). reo'at~n (steps 3,4,5,6) are repeated. The cycles are
~ ,- repeated a total of seven times, till the 16-Carbon pal-
/ Step 3: Condensation mitic acid is formed. rr co , .,,..
M.e..-r N\a.llLL "- CP-" \\
~ C ) a~ inafo-,,yr (3g units are condensed to
form aceto acetr AcP (4C). During this process one Step 7: Palmitic acid is Released
carbon is lost ast;02 (step 3 in Fig. 13.13). The enzyme
Th io-esterase or deacylase activity (TE) releases
is called condensing enzyme (CE in Figs.13.12). palmitate from the multi-enzyme complex (step 7, Fig.
13.13).
Step 4: Reduction The end point is Palmitic acid 16 C) in liver and
Acetoacetyl ACP is reduced by NADPJ:L.d.epeodent adipose tissue. Bu in rnar:nma land, the
reductase (KR in Fig. 13.12) to form ~ tty end products are Ca ric 10 C) and Laurie 12_C) acids.
a~ P (step 4 in Fig. 13.13). r
-OH A Ad') Mother's milk contains these me rum-c arn fatty acids.
Cow's milk contains odd numbered fatty acids.
i~~~~ tep 5: Dehydration
-~ It is then dehydrated by a ehydratase (D j) to form Summary of De Novo Synthesis
e- gru:zyl ACP otherwise known as (unsaturated acyl ACP) The net reaction of de nova synthesis of fatty acid may
(step 5 Fig. 13.13). be sum
'°"V(('
-+
Step 6: Second Reduction
The enoyl ACP is again reduced by enoyl reductase Fatty acid synthesis is not an exact reversa of beta-
(ER) utilizing a 2nd molecule of NADPH to forfflbu~ ox dation. A comparison of these pathways is given in
ACP (step 6 in Fig. 13.1 3). '-r- Taole 13.3.
~,4,..'.> Cl.Sl~
Chapter 13: Metabolism of Fatty Acids 201

Coenz}'i es of Fa.tty Acid Synthesis


ROS(£)NOt f'/' P Pr\ -•
cetyl~ boxy!ase~ An important point to remember is that he oenz me:
(Step 1) Biotin ~-r
Acetyl-CoA + CO2 utilized for de novo synthesis is NADe.lj. The main
CH3- CO-SCoA ( \ ' M•loo-
~OOH-CHj'°CO-SCoA source..,.o.LI\JAD.lli Jo · ·s PentGso
Phos hate Patbway. Tissues h · ctive Ii o en ·,)
ATP ADP+ Pi
(liver, adipose tissue, 2'~W!t'i" mammary gland} have an IP
\N oo&.e.l\ 1 . / J ~

l
(Step 2A) active HMP shunt pa ay also.
Acetyl transacylase (AT)
Acetyl-CoA + (CE)-SH-+ Acetyl S- (CE) + CoA

(Step 28)
Regulation of Fatty Acid C thesis '1"
Malonyl transacylase (MT) Availability of Substrates o.e (l) eJ
Malonyl-CoA + ACP-S~ Malonyl-S-ACP + CoA
Fatty acid synthesis oc9J1!.is when carbohydratiLJs abun-
(CE)- S- CO- CH 3 + ACP- S- CO- CH2 COOH
dant and the level off~ s .is low. The availability of
Acetyl- S- (CE) MalonylACP
l~C) ( a c.) q.tcate io the cytoplasm is the fll.O,SWIJlPOrtsot rea.ulaj_Ory
(Step 3) CO2 factor producing a short-term effect.
Condensing enzyme
or keto acyl synthase (CE)-SH

(rO
ACP- S-CO- CH2- CO- CH3 e
At-./P
cetoacetyl ACP ~ )
(Step 4 ) beta keto acyl ACP
Keto acyl reductase NADPH + fatty acid synthesis is enhanced when eoei:mw:bar9e- is
bi9bCEafty..acid syaihe~is de~1eases when glucose level
NADP+
a'f!)rner~ arge is low. The enzyme is inhibited by pal-
"'.a\-.
ACP- s - co- CH2- CHOH-CH3
Beta-hydroxy butyryl ACP e
mitoyl-CoA, the end product.
;cu?; (.ll!!'9)
11
v:---1
,•

Dehydratase~
Insulin Fa vors Lipogenesis ~'d)...o J~
(Step 5)
H2 0 Insulin enhances the u tal,<e oL ll,ico e by/adipocytes
ACP-S- CO - CH==CH-CH3 (Enoyl ACP) and increases the activity of py,a,~ ~aatl¥d~~ase,
1 Pl>l-1
= ~""'"""...._,,.,._t0xytase and 91¥-C,eu..,i-+u.1
Enoyl reductaser : NADPH + H+ transferase (see Table 11.4 ). Insulin also depresses (G 11
(Step 6}
hormone-sensitive lipase (Fig . 13.14 ). J
NADP+

ACP-S-CO- CH 2- CH2- CH3 Glucagon Inhibits Lipogenesis

__
(Steps 3,4,5,6)
,__......;___;_ _;_ __l Butyryl ACP (4 carbons)

Repeat cycles 6 times (total 7 cycles)


Glucagon and epinephrine are lipolytic. They il:Ulihit
aceM-C.oA carbox¥.lase by keeping the enzyme in the
inactive phosphorylated state. These hormones are sec-
,___ l Thio esterase
("'i'°' (Step?) reted when the ener~ charge is low and ~ ~ vels are
C +H2 O high. 1'.
PalmiUc acid (16 carbons)
- Regulation at the Gene Level
A diet rich in_carboJiydrates leads to stimulation of both
the glycolytfc and Ii , enic pathways. Genes e~ocliw lah,D
. . J ..N~l"\j!u. ~ t'I
Feodor Ly nen gJucokrnase (GK) and ILYec pyruy maSf! (L- 'K) f
NP 1964; 1911-1979 ,,._ u:,f"
glycolysis and ATP citrate lyase, '¾c ,anf f ~
Fig. 13.13: De novo synthesis of fatty acid (Lynen cycle). Steps 4 genesis are regulated by modulation of their transcrip-
and 6 utilize NADPH tion rates, rrti'i , . 11~ ..,
~ - ~v10<:i'lc:> ... ,..,~.,,.
-n "- , ..,...o-.\V
~-
.... Jt----~~ (~?-.'t>O\-, ~(" e'I ~-~J~,lJ. C.l""''Cj\/ dc..,~c
DkohpM~'~on ,r.-, •~\ WI'\ C.OtV>-Qx1\o.H I "{\~ r / I 1r
202 Section B: General Metabolism
e,
~ .~ . ~activation through cAMP)

activates Cffiffeihe inactivates


0 phosphodiesterase; p osp odiesterase;
Hormone so cyclic AMP is so cyclic AMP level
receptor hydrolyzed; remains high;
so HSL inactivated so lipase activated

Membrane
ATP •
\ ·•--.:t,
=···
Cyclic AMP
,__ P_h_
os_p_hod_ie_s_te_ra_se
_ _ }AM~ ~ &A

Inactive protein kinase r • Active protein kinase ( AP\-'-)


i

... ..
ch~~

T\
Inactive Active
hormone-sensitive hormone-sensitive
lipase lipase
(no..Qh9sphate group) ATP ADP • (phosphorylated)

Triglyceride - - - - - - - - " - -- - - + FIN fatty acid

Fig. 13.14: Cascade activation of hormone-sensitive lipase

ISYNTHESIS OF Dihydroxy acetone phosphate (·~t')


>
lIB_IACYLGLYCEROc_;.;L;:.S
. :;:._ _
Fatty acyl-CoA
Liver and adipose tissue are the major sites of triacylgly-

-
Acyl transferase

- r=
cerol (TAG) synthesis. T TIX synthesis in adipose HSCoA

tissue is for storage of energy. But TAG is secreted ~yl dihydroxy acetone phosphate
as VLDL and is transported. The TAG is synthesized NADPH+H•
by esterification of fatty acyl-CoA with either glycerol- Reductase
NADP+
3-phosphate or dihydroxy acetone phosphate (DHAP)
(Fig. 13.15). The glycerol part of the fat is derived from Lysophosphatidic acid { 1-.P'P.l)
the metabolism of glucose. DHAP is an intermediate Fatty acyl-CoA
of glycolysis. Glycerol-3-phosphate may be formed by Acyl transferase

r=
phosphorylation of glycerol or by reduction of dihydroxy HSCoA

acetone phosphate (DHAP). Phosphatidic acid C t>·'A)


In adipose tissue,(a "'"'1•:z:•ce
• r•o"!'l'"'!k"'!i•n•a•se
""""'!i•s• d
"'!'e
• fi!!!11c•i•e•r -..
t) Phosphatidate HzO
phosphatase
and the major source is DHAP derived from glycoly-
Pi
sis. However i~ ver, glycerol kinase is activ~)he fatty 1,2-diacylglycerol( 1., 'l.- t>PC"l)
acyl-CoA molecules transfer the fatty acid to the hydroxyl
Fatty acyl-CoA
groups of glycerol by specific acyl transferases. Acyl transferase
In addition to these two pathways, in th~ HSCoA
nld66§§1 eeils)ie TAG synthesis occurs by ~ Triacylglycerol l~)
pathway. The 2-MAG absorbed is re-esterified with fatty
acyl-CoA to form TAG (see Fig. 13.4). Fig. 13.15: Triacylglycerol synthesis

~ ETABOLISM OF ADIPOSE TISSUE tissue are not inert. They undergo a daily turnover with
The adipose tissue serves as a storage site for excess new triacylglycerol molecules being synthesized and a
calories ingested. The triglycerides stored in the adipose definite fraction being broken down.
Chapter 13: Metabolism of Fatty Acids 203

Adipose Tissue in Well-fed Condition TABLE 13.4 : Changes 1n adipose tissue


Well-fed state During fasting
Under well-fed conditions, active lipogenesis occurs in
Lipogenesis increased Lipogenesis inhibited
the adipose tissue. The dietary triglyqerides are trans-
Lipolysis inhibited Lipolysis increased
ported by chylomicrons. Liver endogenously synthesize Insulin inhibits HS-lipase Glucagon activates HS-lipase
triglycerides which~secreted as VLDL. Both chylo- 11
Liooprotein lipase active FFA 1n blood increased
-
microns and VLDL are aken up by adipose tissue and
"'
Cl.,v'°~"'
' v~
L,-
stored as TAG. T poprotein molecules are broken
down by the lipoprotein lipase present on the capillary Adipokines
wall. In well fed condition, glucose and insulin levels are Adipokines are adipose tissue dervived hormones. The
increased . GluT4 in adipose tissue is insulin-depen- important adipokines are leptin, adiponectin, resistin, LATT
dent. Insulin increases the activit~ of key glycolytic TNF-alpha (tumor necrosis factor) and IL-6 (interleukin-6).
enzymes as well as pyru te e yqrogenase, acetyl- Leptin is a small peptide ..{)t;p.Quced by agle,Q,Sytes.
CoA c bOxyfase and glycerol phGl ate acyl trans- LeQl_in re0e15lors are present in specific regions of the
ferase. The stimulant effect of insulin on MP pathway brain. The feed in9 behavior is regulated by leptin. A
also enhances-li.pogenesis. Insulin also causes inhibi- fect'ln"ieptih or its receptor, can lead to o6esify) Dec-
tion of horm e sensitive lipase, and so lipolysis is reased level of leptin increases the chances of obesity.
decreased (Fig. 13.14 and Table 13.4). Adiponectin is another polypeptide, which increa-
ses the insulin sensitivity of muscle and liver. LoWievels
Adipose Tissue in Fasting Condition of adiponectin will a_ccele.ranra~cferosfs. Low levels
The metabolic pattern totally changes under conditions are also observed in patient ith rne,tabolic syrrdrome
of fasting. TAG. from the adipose tissue is mobilized
under the effect of the hormones, ~n,aod epi- W i Adipose Tissue
na_pllfin.e. The cyclic AMP mediated activation cascade It is mainly oncern~ ith energy storage. It is made
enhances the intracellular hormo e-sensitive lipase up of I cells, wif few mitocho~ The
(Fig. 13.14 and Table 13.4). This acts on TA.Q and liber- triglycerides form the major component of whi~q§e
ates fatty acids. Under co ditions of starvation, a high tissue (about 80%) with oleic acid being the most abun-
glucagon, ACTH, qlucocortjcojds anq yroxine have dant fatty acid (50%). 'iOt,
lipolytic effect. The re:eased freB tatt)r<lcids (ff.A) are Br.o adipose >issue is involved in thermogene-
taken up by peripheraf ~ yes as a fuel. sis. The b own colo( is due to the presence of numer-
ous ~orfcffia { is primarily important in new born
Adipose Tissue and Diabetes Mellitus human beings and adult hiber ~ ing animals.
In diabetes, lipolysis is enhanced ~ igh FFA level Th.er_mo_genesis Is a process found in brown aqi-
in plasma is noticed. Insulin aot t rou~ receptors on pose tissue. It liberates heat by unco~ ling oxidation
the ce)I surface of adipocytes. "These receptors are from phospl)_orylatio~. So energy Is released as heat,
decreased, leading to insulin insensitivity in diabetes. instead of trapping it In the high energy bor,ds of ATP by
the action of the uncoupling protein, thermogenin.
Adipose Tissue and Obesity
Liver-Adipose Tissue Axis
The fat content of the adipose tissue can increase to
unlimited amounts, depending on the amount of excess Role of liver in fat metabolism is shown in Box 13.5. Liver
calories taken i~ is leads to i i ity. A high level of produces fatty acid and TAG (triacylglycerol), which is
plasma insulin 1W-n @oti'ilO. ut insulin recep- transported as VLDL (very low density lipoprotein) in
tors are decreased; and there i Ipheral resistance the blood. The fatty acids from VLDL are taken up by a1j i-
against insulin action en fat droplets are overloaded, pose tissue with the help of lipoprotein lipase, and stonad
the nuc eus o adipose issue cell is degradErd, cell is as TAG. This neutral fat is hydrolysed by hormone-sensi-
destroyed, and TAG becomes extracellular. Such TAG tive lipase into NEFA (FFA), which in the blood is carried
cannot be metabolically reutilized and forms the dead by albumin. The NEFA is utilized by the peripheral tiSSUHS,
bulk in obese individuals. See Chapter 35 for details.'" excess of which can be taken up by liver cells. Thus thme
204 Section B: General Metabolism

BOX 13.5: Role of liver 1n fat metabolism Liver


1. Secretion of bile salts
2. Synthesis of fatty acid, triacyl ~ I and phospholipids
3. Oxidation of fatty acids
4. Product ion of lipoproteins
5. Production of ketone bodies
6. Synthesis and excretion of cholesterol.

Blood glucose Oxidation


FFA = free fatty acids; FA = fatty acid; TAG = triacyl-
glycerol; VLDL = very low density lipoproteins

Fig. 13.17: Causes for fatty liver

.·. ov,1 sla._-~" ~o..~H•


liver can occur in diabetes mellitus and starvation due to
I,
increased lipolysis 1n adipose tissue (§iep H?.'13.17).

Excess Calorie Intake


= =
TAG triacylglycerol; VLDL very low density lipoprotein; Excess calories, either in the form of carbohydrates or
= =
LPL lipoprotein lipase; HSL hormone sensitive lipase; as fats, are deposited as fat. Hence obesity may be
FFA = free fatty acids
accompanied by fatty liver (Step no. 2 in Fig. 13.17).
Fig . 13.16: Liver-adipose tissue axis

Toxic Injury to Liver


is a constant flux of fat molecules from liver to adipose
tissue and back (Fig. 13.16). i. In toxic injury to the liver due t o ~ni(l{l;by com-
pounds like carb lC tetr chloride, a nic, lijaa, etc.,

'
FATTY LIVER AND the capacity to synthesize VLDL is affected leading
to fatty infiltration of liver (step no. 3 in Fig. 13.17).
( LIPOTROPIC FACTORS
ii. In protei calorie malnutrition, amino acids req-
Fatty liver~efers ;Jth)dffosltio~of ek!ss tfigtycerides ire to synt esise apoproteins may be lacking.
in the liver cells. The balance between the factors caus- iii. Hepatitis B virus infection reduces the function of
ing fat deposition in liver versus factors causing removal hepatic cells.
of fat from liver, determines the outcome.
Alcoholism
Causes of Fatty Liver It is the most common cause of fatty liver and cirrhosis in
A. Causes of fat deposition in liver India. The metabolism of alcohol is described in Chapter
1. Mobilization of fatty acids (FFA or NEFA) from 12. Alcohol is oxidized to acetaldehyde. This reaction
adipose tissue . produces increased quantities of NADH, which converts
• -.@ More synthesis of fatty acid from glucose. oxaloacetate to malate. As the availa bility of oxaloace-
.' B. Reduced removal of fat from liver tate is reduced, the oxidation of acetyl-CoA through citric
, 3. ·n· to liver. Secretion of VLDL needs acid cycle is reduced (block in step 4 of Fig. 13.17). So
'•, ,,,....-:\ synth · Bf :too and apo.C. fatty acid accumulates leading to TAG deposits in liver.
·- \V Qijci'easeo oxidation of fat by hepatic cells. An
increase in factors (1 ) and (2) or a decrease in Nonalcoholic Fatty Liver Disease
factors (3) and (4) will cause excessive accumu- (NAFLD) and Nonalcoholic
lation, leading to fatty liver. These pathways are Steatohepatitis NASH
summarized in Figure 13.17. Hi h fat diet and unco
most common causes
Excessive Mobilization of Fat
cytes. As it progresse
The capacity of liver to take up the fatty acids from blood which is t
far exceeds its capacity for excretion as VLDL. So fatty (NASH).
Chapter 13: Metabolism of Fatty Acids 205

Fatty Liver Progresses to Cirrhosis


CH 3- CO-SCoA + CH3- CO-SCoA
Fat molecules infiltrate the cytoplasm of the cell (fatty
Aoetyl-CoA + Aoetyl-CoA
infiltration ). These are seen as fat droplets, which are
merged together so that most of the cytoplasm becomes Step 1 l
laden with fat. The nucleus is pushed to a side of the !'--.coA-SH
cell, nucleus further disintegrated (karyorrhexis), and CH3- CO- CH2- CO-SCoA
Acetoacetyl-CoA
ultimately the e atic II is lysed. As a healing pro-

E
oetyl-CoA
cess fibrous tissue ·sJaid..dow!l, caus· fibrosis of liver,
Step 2 H2 0
otherwise known as cirrhosis. ...,,, ction tests (see
Chapter 24) will show abnormal v CoA
COOH-CH2- C(CH3 ) (OH)- CH2 - CO-SCoA
Betahydroxy beta methyl glutaryl-CoA (HMG-CoA)
Lipotropic Factors
They are required for the normal mobilization of fat Step 31
from liver. Therefore deficiency of these factors may
CH3 - CO- CH2- COOH + CH3- CO-SCoA
result in fatty liver. They can afford protection against
the development of fatty liver. Acetoaceta~te + : c:: ~ : :
1. Choline
Step 4
2. Lecithin and methionine: They help in synthesis
NAO•
of apoprotein and choline formation. The deficiency
of methyl groups for carnitine synthesis may also CH3- CO- CH 3 CH 3 - CHOH- CH2 - COOH
Acetone Beta-hydroxy butyrate
hinder fatty acid oxidation.
3. Vitamin E and selenium give protection due to Step 1 = Aci?t : CoA synthase;
their antioxidant effect. Step 2 = HM A synthase;
4. 0 ega-3 fatty acids present in marine oils have a Step 3 = H • oA lyase;
protective effect against fatty liver. Step 4 = Dehydrogenase; (.D t-r)
Step 5 is nonenzymatic and spontaneous .

METABOLISM OF ..\\~ 0£.~\1


· -A or-J· Fig. 13.18: Ketone body fo rmation (Ketogenesis)
Q.~~•r
KETONE BODIES
Carbohydrates are es,3s~e\Qn!!!ti~l~f~o~r
Step 2: Production of HMG-CoA
or fat is burned under the fire of carbohydrates. e One more ·acetyl-CoA is added to acetoacetyl-CoA to
acetyl-CoA formed from fatty acids can enter and et form HMG-CoA (beta-hydroxy beta-methyl glutaryl-CoA).
oxidized in TCA cycle only when carb ydr tes are The enzyme is HMG-CoA synthase. Mitochondrial
available. HMG-CoA is used for ketogenesis , while cytosolic
During starvation and diabetes mellitus, acetyl-
fraction is used for cholesterol synthesis.
CoA takes the alternate route of formation of ketone
bodies. [·: e\ ~~e. Co.9-~1>h. '" ,te Step 3: Lysis
c~]
Ketogenesis Then HMG-CoA is lysed to form acetoacetate. HMG-
Acetoacetate is the primary ketone body while beta- CoA lyase is present only in liver.
hydroxy butyrate and acetone are secondary ketone
bodies. They are synthesized exclusively by the liver mito- Step 4: Reduction
chondria. The steps involved are shown in Figure 13.18. Beta-hydroxy butyrate is formed by reduction of aceto-
acetate.
Step 1: Condensation
Two molecules of acetyl-CoA are condensed to form
Step 5: Spontaneous Decarboxylation
acetoacetyl-CoA. Acetone is formed (Fig. 13.18).
206 Section B: General Metabolism
insulin causes accelerated lipolysis. More fatty acids are
released into circulation. Oxidation of these fatty acids
Blood
increases the acetyl-CoA pool. Oxidation of acetyl-CoA
Extrahepatic tissues
by TCA cycle is reduced, since availability of oxalo-
acetate is less.
Starvation: In starvation, the dietary supply of glucose
Acetyl-CoA - - - - FFA Acetyl-CoA
is decreased. Available oxaloacetate is channelled to

l
Ketone bodies Ketone bodies
J~= l
gluconeogenesis. The increased rate of lipolysis is to
provide alternate source of fuel. The excess acetyl-
CoA is converted to ketone bodies. The high glucagon
favors ketogenesis. The brain derives 75% of energy
Citric acid cycle

Acetone
in lungs
Ketone
bodies in urine
i
CO2
from ketone bodies under conditions of fasting. Hyper-
emesis (vomiting) in early pregnancy may also lead to
starvation-like condition and may lead to ketosis.
Fig. 13.19: Formation, utilization and excretion of ketone bodies
Explanation for Ketogenesis
Ketolysis
i. During starvation and diabetes mellitus, the blood
The ketone bodies are formed in the liver; but they are level of glucagon is increased . Glucagon (see
utilized by extrahepatic tissues. The heart muscle and Chapter 11) inhibits glycolysis, activates gluconeo-
renal cortex prefer the ketone bodies to glucose as fuel. genesis, activates lipolysis, and stimulates keto-
Tissues like skeletal muscle and brain can also utilize genesis. High glucagon-insulin ratio is potentially
the ketone bodies as alternate sources of energy, if glu- ketogenic.
cose is not available. Acetoacetate is activated to ace- ii. Insulin (see Chapter 11) has the opposite effect; it
toacetyl-CoA by thiophorase enzyme. favors glycolysis, inhibits gluconeogenesis, depres-
Almost all tissues and cell types can use ketone ses lipolysis, and decreases ketogenesis. The
bodies as fuel, with the exception of liver and RBC. ketone body formation is regulated at the following
Thiophorase 3 levels:
Aceto acetate Acetoacetyl-CoA
+ Succinyl-CoA Succinate Level 1: Lipolysis
Then acetoacetyl-CoA enters the beta-oxidation
Precursors of ketone bodies are free fatty acids. So
pathway to produce energy. Summary of ketone body
mobilization of fatty acid from adipose tissue will influ-
metabolism is shown in Figure 13.19.
ence ketogenesis (Fig. 13.20). Insulin inhibits lipolysis,

IKETOSIS while glucagon favors lipolysis.

Normally the rate of synthesis of ketone bodies by the Level 2: Entry of Fatty Acid to Mitochondria
liver is minimal. So they can be easily metabolized by the
The mobilized fatty acid then enters mitochondria for
extrahepatic tissues. Hence, the blood level of ketone
beta-oxidation. Carnitine acyl transferase I (CAT-I) regu-
bodies is less than 1 mg/dl. Ketone bodies are not
detected in urine. But when the rate of synthesis exce- lates this entry (see Fig. 13.7). Malonyl-CoA is the major
eds the ability of extrahepatic tissues to utilize them, regulator of CAT-I activity. In diabetes and starvation,
there will be accumulation of ketone bodies in blood. glucagon is increased, which decreases malonyl-CoA
This leads to ketonemia, excretion in urine (ketonuria) (Fig. 13.20).
and smell of acetone in breath. All these three together
constitute the condition known as ketosis. Level 3: Oxidation of Acetyl-CoA
When the above two steps are increased , more acetyl-
Causes for Ketosis
CoA is produced. Normally, acetyl-CoA is completely
Diabetes mellitus: Untreated diabetes mellitus is the oxidized in the citric acid cycle. In both diabetes mel-
most common cause for ketosis. The deficiency of litus and starvation, the oxaloacetate is channelled to
Chapter 13: Metabolism of Fatty Acids 207

Starvation/Diab ties M ' ill.IS Supportive evidence may be derived from estimation of
(Increased Glucagon/Decreased Insulin) serum electrolytes, acid-base parameters, glucose ana

L Level 1: Lipolysis activated


Increased NEFA in plasma
urea estimation.

Differential Diagnosis of Ketosis

---+ Level 2:
l
Increased CAT-I
The urine of a patient with diabetic ketoacidosis will
give positive Benedict's test as well as Rothera's test.
Decreased malonyl-CoA
But in starvation ketosis, Benedict's test is negative, but
i
Increased beta oxidaion
Rothera's test will be positive.
- - - . Level 3: Increased gluconeogenesis
Decreased oxaloacetate Management of Ketoacidosis
i. Treatment is to give insulin and glucose. When
Ketogenes1s -+- Increased generation of acetyl-CoA glucose and insulin are given intravenously, potas-
and decreased utilization of acetyl-CoA
sium is trapped within the cells. Hence, the clinician
Fig. 13.20: Summary of ketosis should always monitor the electrolytes.
ii. Administration of bicarbonate, and maintenanc1:i
gluconeogenesis; so the availability of oxaloacetate is of electrolyte and fluid balance are very important
decreased. Hence acetyl-CoA cannot be fully oxidised aspects.
in the TCA cycle. When oxaloacetate is diverted for
gluconeogenesis; citric acid cycle cannot function opti- • · Clinical Case Study 13.1
mally. Thus, on the one hand, acetyl-CoA is generated in
A teenage girl was brought to the hospital with com-
excess, on the other hand, its utilization is reduced. This
plaints that she gets too tired and has muscle pains. .A.
excess acetyl-CoA is channeled into ketogenic pathway.
consulting neurologist found muscle weakness in arms
(See Fig. 11.9 also).
and legs. Biochemical investigations revealed elevated
amounts oftriacylglycerols esterified with long chain fatly
Salient Features of Ketosis
acids. Muscle biopsy report showed significant number
1. Metabolic acidosis. Acetoacetate and beta-hy- of lipid vacuoles. What is the probable diagnosis? What
droxy butyrate are acids. When they accumulate, is the cause of these symptoms?
metabolic acidosis results. (see Chapter 27).
2. Reduced buffers. The plasma bicarbonate is used 8 Clinical Case Study 13.2
up for buffering of these acids.
A 22-year-old primigravid female at 36 weeks of gesta-
3. Kussmaul's respiration. Patients will have typical
tion presented with nausea, vomiting, and malaise ov1:ir
acidotic breathing due to compensatory hyperventi-
the last several days. On examination she had high
lation.
blood pressure (190/110 mm Hg) and yellowish discol-
4. Smell of acetone in patient's breath .
oration of sclera. Laboratory results revealed protei 1-
5. Osmotic diuresis induced by ketonuria may lead
uria, impaired liver function tests, prolonged clotting
to dehydration.
time, hyperbilirubinemia , hypoglycemia and hypofibrino-
6. Sodium loss. The ketone bodies are excreted in
genemia. The condition was diagnosed as acute fa1ty
urine as their sodium salt, leading to loss of cations
liver of pregnancy. An emergency cesarean section was
from the body.
done, but hypoglycemia worsened and the patient went
7. Dehydration. The sodium loss further aggravates
into coagulopathy, renal failure and hepatic coma. What
the dehydration.
is the cause of acute fatty liver of pregnancy? What is
8. Coma. Dehydration and acidosis are contributing
the cause for hypoglycemia?
for the lethal effect of ketosis.

Diagnosis of Ketosis •• Clinical Case Study 13.3


The presence of ketosis can be established by the A 65-year-old man presented with anemia, weight loss
detection of ketone bodies in urine by Rothera's test. and passage of bulky pale stools. On examination, ne
208 Section B: General Metabolism

had hepatosplenomegaly. His plasma electrolytes were e


normal. Further laboratory tests were: •ii• Clinical Case Study 13.2 Answer
Plasma, total proteins : 5.2 g/ L
Short history of illness, hypoglycemia, liver failure, renal
Albumin : 2.5 g/L
failure and coagulopathy are suggestive of acute fatty
Calcium : 6.8 mg/dL liver of pregnancy. Condition usually sets in during the
Phosphates : 2 mg/dL second half of pregnancy and closer to term, it can also
Alkaline phosphatase : 300 U/L manifest in the postpartum period. Diagnosis is made inci-
His fecal fat excretion was 55 g over three days (nor- dentally with elevated liver enzymes. Patients develop
mal less than 21 g) and his plasma 25-hydroxy cholecal- jaundice, encephalopathy, and profound hypoglycemia .
ciferol was 28 nmol/L (normal 40- 160). Fatty liver due to any condition except alcohol is
A. What is the most probable diagnosis? termed as nonalcoholic steatohepatitis. Causes are
B. What will be the cause for steatorrhea and what obesity, diabetes mellitus, hypertriglyceridemia, drugs
functional analysis could be done? and poisons, endocrine disorders and acute fatty liver of
C. How do you interpret low plasma albumin? pregnancy. Women with fatty liver of pregnancy develop
D. How do you interpret low plasma vitamin D, calcium defect in fatty acid oxidation due to reduced long chain
and increased ALP values? 3 hydroxy acyl-CoA dehydrogenase activity. The defect
E. What is the cause for anemia in this patient? is in mitochondrial processes and reduced b oxidation.

e There is fatty infiltration of liver and glycogen levels are


depleted. Gluconeogenesis is impaired and hypoglyce-
6 Clinical Case Study 13.1 Answer mia develops .
Likely cause is carnitine deficiency. Causes of carnitine e
deficiency may be - (1 ) Inadequate intake, (2) Enzyme •ii• Clinical Case Study 13.3 Answer
deficiencies, (3) Decreased endogenous synthesis due to
Steatorrhea is confirmed by abnormal fecal fat excretion
severe liver diseases, (4) Excess loss of carnitine due to
and the complaint of "passage of bulky pale motions".
diarrhea, renal losses, hemodialysis, (5) Hereditary dis- Plasma levels of bilirubin, ALT and GGT were normal
eases, (6) Increased requirement, as in ketosis, critical ill- in this particular case. Liver function tests to be done to
nesses like sepsis and major burns, major surgery of GIT, rule out chronic liver diseases. Vitamin B 12 absorption
and (7) Mitochondrial impairment as with certain drugs test will be positive in diseases of terminal ileum, e.g .
(zidovudine, valproate). It can cause a heterogeneous regional enteritis.
group of disorders. Muscle metabolism is impaired, caus- Low plasma albumin is due to inadequate absorp-
ing myopathy, hypoglycemia, or cardiomyopathy. Infants tion of amino acids. Low plasma vitamin D, calcium
typically present with hypoglycemia, and hypoketotic en- and increased ALP are due to decreased absorption
cephalopathy. of fat-soluble vitamins. Vitamin D deficiency leads to
The symptoms of carnitine deficiency range from decreased gut calcium absorption; this in turn , leads to
mild muscle cramps to severe weakness and death . low plasma calcium. This activates osteoblasts, which
Muscle, kidney and heart are primarily affected. Muscle accounts for the increased ALP level. Anemia is seen
weakness during prolonged exercise is a predominant because excess fat in gut will interfere with absorption
feature, because muscles rely more of fatty acids as a of iron.
long-term source of energy. Medium chain fatty acids,
which do requi re carnitine, are normally metabolized in l_hEARNING POINTS, CHAPTER 13 _
these patients. 1. Digestion of lipids involves the following enzymes,
Diagnosis is made by very low carnitine level in Lingual lipase, gastric lipase and pancreatic lipase.
plasma and muscle (1- 2% of normal). Fasting ketogene- Pancreatic lipase is the major digestive enzyme
sis may be impaired if dieatary intake is impaired. Fast- and requires bile salts.
ing urinary organic acid pattern may show hypoketotic 2. Lipids are absorbed by emulsification and micelle
dicarboxylic aciduria pattern. Carnitine assay in cultured formation with the help of bile salts.
fibroblast and lymphoblast will demonstrate low level. 3. Short and medium chain fatty acids are absorbed
Treatment consists of dietary L-Carnitine. directly without re-esterification.
Chapter 13: Metabolism of Fatty Acids 209

4. Defective absorption of lipids occurs in celiac dis- 16. Acetoacetate is the primary ketone body. Beta
ease, Crohn's disease. hydroxy butyric acid and acetone are secondary
5. Mammalian tissues oxidize fatty acids primarily ketone bodies.
by the beta-oxidation pathway which occurs in the 17. Ketosis is seen in diabetes mellitus and starvation.
mitochondria. 18. Rothera's test is commonly used to detect pres-
6. Transport of fatty acids {long chain acyl-CoA) ence of ketone bodies in urine.
through the inner mitochondrial membrane is facili- 19. Acetyl-CoA formed from fatty acids is further oxi-
tated by carnitine acyl transferase and translocase. dized in TCA cycle to generate energy, when avail-
7. Net yield of ATP from one molecule of palmitic acid ability of oxaloacetate is sufficient.
is 106ATP. 20. Under conditions of fasting and starvation , the oxa-
8. Oxidation of odd chain fatty acids produces propio- loacetate is channelled to gluconeogenesis. Excess
nyl-CoA, which may be further metabolized by the acetyl-CoA is then used for ketogenesis by liver.
TCAcycle. 21. A similar situation is seen in uncontrolled diabetes
9. Alpha oxidation and omega oxidation are two other mellitus where gluconeogenesis and lipolysis are
modes of fatty acid oxidation. both enhanced.
10. De novo synthesis of fatty acids occurs in the cyto- 22. The excess acetyl-CoA is converted to ketone
plasm with the help of a dimeric multi-enzyme com- bodies in hepatic mitochondria .
plex termed Fatty acid synthase. 23. The HMG-CoA formed is cleaved by liver enzyme
11 . Synthesis of fatty acid requires NADPH, while lyase to the primary ketone body acetoacetate.
degradation requires NAO and FAD. 24. Acetoacetate may be reduced to beta-hydroxy buty-
12. Insulin favors fatty acid synthesis. rate or spontaneously decarboxylated to acetone.
13. The white adipose tissue is concerned with energy 25. Ketone bodies are synthesized by liver and meta-
storage and the brown adipose tissue is concerned bolized by extrahepatic tissues, mainly cardiac
with thermogenesis. muscle and skeletal muscle.
14. Obesity is the result of an increase in the fat content 26. Under conditions of starvation, brain starts metabo-
of the adipose tissue. It is associated with insulin lizing ketone bodies for energy needs.
resistance. 27. Since ketone bodies are acids, metabolic acidosis
15. Fatty liver refers to deposition of excess triglyceri- occurs. Excessive accumulation of ketone bodies
des in the liver cells. It is facilitated by lipotropic can be dangerous since it can result in acidosis,
factors such as methionine, choline and lecithin . dehydration and coma.

PART-1: ESSAY AND SHORT NOTE QUESTIONS

13-1 . Classify lipids, giving examples.


13-2. Name the dietary lipids. Explain digestion and absorption of fats.
13-3. Explain the role of bile salts in the digestion and absorption of dietary lipids. Mention the changes observed in
obstructive jaundice.
13-4. Explain the steps of beta-oxidation of Palmitic acid, giving energetics.
13-5. Describe the beta oxidation of fatty acids. What are the coenzymes required for beta oxidation, and in which
steps?
13-6. Give the sources and fate of Acetyl-CoA.
13-7. Describe the reactions of de novo synthesis of fatty acid and how is it regulated ?
13-8. Describe the de novo synthesis of fatty acids. What is the coenzyme required, and how is it generated?
13-9. How are the fatty acids in adipose tissue mobilised and transported to other tissues? Explain the effect of hor-
mones in this process.
13-10. How is propionyl-CoA formed? How is it further metabolised?
13-11 . What is fatty liver? Explain the causes of fatty liver. Indicate how lipotropic factors can prevent fatty liver.
13-12. What are ketone bodies? Explain the reactions leading to the formation of them. How are they utilized in the
body?
13-13. Name the ketone bodies. Give two conditions characterised by excessive production of ketone bodies. Explain
the metabolic derangements and consequences of ketosis.
210 Section B: General Metabolism

SHORT NOTE QUESTIONS


13-14. Rate limiting enzyme of fatty acid biosynthesis. 13-22. Dicarboxylic aciduria.
13-15. Carnitine. 13-23. Effect of insulin on lipolysis.
13-16. Role of citrate in fatty acid synthesis. 13-24. Hormone-sensitive lipase.
13-17. ATP citrate lyase. 13-25. Fatty liver.
13-18. Oxidation of odd chain fatty acids. 13-26. Lipotropic factors.
13-19. Metabolism of propionyl-CoA. 13-27. Ketosis.
13-20. Refsum 's disease. 13-28. Ketogenesis.
13-21 . Alpha oxidation.

PART-2: MULTIPLE CHOICE QUESTIONS


13-1 . The complete digestion oftriacyl glycerol {triglyce- 13-8. Name the defective enzyme in methylmalonyl aci-
ride) in the gastro intestinal tract needs all the duria:
following, except A. Propionyl-CoA carboxylase
A. Pancreatic lipase 8. Methylmalonyl-CoA mutase
8. Hormone-sensitive lipase C. Tyrosinase
C. lsomerase E. Cystathionine synthase
D. Bile salts 13-9. Refsum's disease is due to lack of enzymes of:
13-2. Bile salts help in the absorption of dietary lipids by: A. Beta oxidation 8 . Omega oxidation
A. Producing the micellar state of lipids C. Alpha oxidation D. Desaturation
8 . Incorporating cholesterol into chyclomicrones 13-10. Refsum's disease is due to accumulation of:
C. Converting triglycerides into monoglycerides A. Phytanic acid 8 . Sphingomyelin
D. Providing the optimum pH for lipase activity C. Glucocerebroside D. Ganglioside
13-3. All are correct with regard to Carnitine, except 13-11. Acetyl-CoA formed in the mitochondria is made
A. It transports fatty acids from cytoplasm to mito- available in the cytoplasm for de novo synthesis
chondria of fatty acids by the activity of the enzyme:
8 . Carnitine is involved in fatty acid synthesis A. Carnitine acyl transferase
C. Medium chain fatty acids do not require carnitine 8 . ATP citrate lyase
D. Carnitine is synthesized from lysine C. Acetyl-CoA carboxylase
13-4. During each cycle of beta-oxidation of fatty acids, D. Pyruvate dehydrogenase
all the following are generated, except: 13-12. The key enzyme in fatty acid synthesis is:
A. Fatty acyl-CoA , with 2 carbon atoms less A. Acetyl-CoA carboxylase
8 . Acetyl-CoA 8 . Beta hydroxy acyl dehydratase
C. FADH2 C. Enoyl reductase
D. NADPH D. Acetyl transacylase
13-5. The net gain of ATP per molecule of palmitic acid 13-13. Acetyl-CoA carboxylase is activated by:
on complete oxidation Is: A. Palmityl-CoA 8 . Carbon dioxide
A. 12 8. 38 C. Citrate D. Malonyl-CoA
C. 106 D. 135 13-14. For the de no fatty acid synthesis, the coenzyme
13-6. The coenzyme required for propionyl-CoA carboxy- required is:
lase Is: A. NADH
A. Thiamine pyrophosphate 8 . NADPH
8. NADPH C. Thiamine pyrophosphate
C. Biotin D. FADH2
D. Pyridoxal phosphate 13-15. Sources of NADPH for fatty acid synthesis include
13-7. Name the defective enzyme in propionic acidemia: all the following reactions, except
A. Propionyl-CoA carboxylase A. Glucose-6-phosphate dehydrogenase
8 . Methylmalonyl-CoA racemase 8. 6-phospho gluconate dehydrogenase
C. Tyrosinase C. Cytoplasmic malate dehydrogenase
D. Propionic acid dehydrogenase D. Cytoplasmic isocitrate dehydrogenase
Chapter 13: Metabolism of Fatty Acids 211

13-16. In the biosynthesis of triacylglycerol from glyce- 13-28. All the em:ymes are required for de novo synthesis
rol phosphate, all the following are intermediates, of fatty acid, except:
except: A. Acetyl ..CoA carboxylase
A. Lysophosphatidic acid B. Glucose-6-phosphate dehydrogenase
B. Phosphatidic acid C. Thiolase
C. 1,2-diacylglycerol D. Malonyl transacylase
D. Monoacylglycerol 13-29. The action of caffeine include all the following, except
13-17. In adipose tissue, activity of hormone-sensitive A. Inhibition of phospho diesterase
lipase is increased by all the following hormones, B. Prolongation of action of cyclic AMP
except C. Increase in activity of hormone-sensitive lipase
A. Adrenalin B. Growth hormone D. Inhibition of synthesis of prostaglandins
C. Corticosteroids D. Insulin 13-30. Ketosis is often seen in:
13-18. Chain elongation of fatty acids takes place in which A. Nephritis
of the following subcellular fraction? B. Coronary artery disease
A. Cytoplasm B. Nucleus C. Muscle degeneration
C. Microsomes D. Golgi complex D. Diabetes mellitus
13-19. HMG-CoA is a precursor of all the following, except: 13-31 . The pathway which takes place in the mitochon-
A. Arachidonic acid B. Cholesterol dria. is:
C. Acetone D. Beta hydroxy butyrate A. KetogEmesis
13-20. HMG-CoA is directly converted to all the following, B. Glycolysis
except: C. HMP shunt pathway
A. Acetoacetyl-CoA B. Mevalonate D. Cholesterol synthesis
C. Acetoacetate D. Acetyl-CoA 13-32. Utilizationi of ketone bodies by peripheral tissues
13-21. Fatty liver may be prevented by the following, except needs:
A. Ethanol B. Choline A. NADPH dependent dehydrogenase
C. Methionine D. Lecithin B. Biotin dependent carbon dioxide fixation reaction
13-22. Ketone bodies are produced mainly in : C. Carbon dioxide elimination reaction
A. Brain B. Liver D. Succinyl-CoA dependent thiophorase reaction
C. Erythrocytes D. Skeletal muscles 13-33. All are trw! with regard to ketone bodies except
13-23. Ketosis is due to increased: A. Acetoacetate, acetone and betahydroxy butyrate
A. Hepatic glucose liberation B. Seen in urine in uncontrolled diabetes mellitus
B. Carbohydrate utilisation C. Mainly produced in adipose tissue
C. Gluconeogenesis D. Seen in urine during prolonged starvation
D. Fatty acid oxidation 13-34. Defects ini oxidation of fatty acids can produce all
13-24. Which of the following tissues are capable of oxi- except
dizing ketone bodies? A. Hypoglycemia B. Ketonemia
A. Liver B. Heart C. Myopa1thy D. Hyperammonemia
C. RBCs D. Plasma 13-35. Defective digestion and absorption of fats can lead
13-25. Ketone bodies in urine are identified by: to the foll,c,wing, except
A. Heat and acetic acid test A. Steatorrhoea
B. Rothera's test B. Hypocalcemia
C. Benzidine test C. Defective coagulation
D. Benedict's test D. Difficullty in dim light
13-26. Beta-oxidation of fatty acids yields all the follow- 13-36. Which of the following tissues cannot derive
ing, except energy trc,m fatty acids?
A. NADH B. Acetyl-CoA A. Brain B. Cardiac Muscle
C. FADH2 D. Malonyl-CoA C. Skeletal Muscle D. Erythrocytes
13-27. Regarding propionic acid metabolism 13-37. Wh ich of the following processes does not occur
A. Propionyl-CoA carboxylase is biotin dependent in adiposi! tissue?
B. Methyl-malonyl-CoA racemase is vitamin 812 A. Formation of triose phosphates
dependent B. Generation of NADPH
C. Succinyl-CoA is converted to propionyl-CoA C. Hydrolysis of Triacyl glycerol
D. Leucine is catabolized to produce propionyl-CoA D. Phosphorylation of glycerol
212 Section B: General Metabolism

13-38. The enzyme which is active only in the liver is 13-39. Fatty acidl synthesis differs from beta oxidation in
A. HMG-CoA reductase all, except
B. Carnitine acyl transferase A. Uses NAD PH as reducing power
C. Lecithin cholesterol acyl transferase B. Requires coenzyme A
C. Catalysed by multienzyme complex
D. HMG-CoA lyase
D. Activated by insulin

' ANSWERS OF MULTIPLE CHOICE UESTIONS

13-1. B 13-2. A 13-3. B 13-4. D 13-!5. C 13-6. C 13-7. A


13-8. B 13-9. C 13-10. A 13-11 . B 13-1 :2. A 13-13. C 13-14. B
13-15. D 13-16. D 13-17. D 13-18. C 13-19. A 13-20. A 13-21 . A
13-22. B 13-23. D 13-24. B 13-25. B 13-21:i. D 13-27. A 13-28. C
13-29. C 13-30. D 13-31. A 13-32. D 13-3:3. C 13-34. B 13-35. B
13-36. D 13-37. D 13-38. D 13-39. B

PART-3: VIVA VOCE QUESTIONS AN ANSWERS


13-1. Complete digestion of triacylglycerol (triglyceride) 13-10. What is steatorrhea?
needs what? When daily excretion of fat in feces is more than 6 g per
Pancreatic lipase: Co-lipase; lsomerase and Bile salts. day, it is called steatorrhea. It is due to defective diges-
13-2. What are the final end products of digestion of tri- lion as in chronic diseases of pancreas. In such cases,
glyceride? unsplit fat is excreted in feces.
2-monoacyl glyceride, 1-monoacyl glyceride, glycerol 13-11. What hapIpens in defective absorption?
and two fatty acids. If the absorption alone is defective, most of the fat in
13-3. What is the difference for absorption of short chain
feces may be split fat, i.e. fatty acids and monoglycer-
fatty acid?
ides.
Short and medium chain fatty acids do not need 13-12. What is cause for defective absorption of fat?
re-esterification. They directly enter into blood vessels
It may be due to diseases in intestinal mucosa, e.g.
(Not to lymph vessels).
coeliac dis,ease, sprue. Crohn's disease. Any condition
13-4. Where will you find short and medium chain fatty
leading to a deficiency of bile salts can also result in
acids?
malabsorption of fat, e.g., gall stones, tumors of head
Butter, ghee, coconut oil and mother's milk.
of pancreas, etc.
13-5. How bile salts help in absorption of lipids?
13-13. How fatty acids are activated in preparation of
By emulsifying the lipids and producing micelles of
oxidationi?
lipids.
13-6. What are micelles? Fatty acids are activated to their coenzyme A (CoA)

The micelles are spherical particles with a hydrophilic derivative.


13-14. How much ATP is required for this reaction?
exterior and hydrophobic interior core. Monoglycer-
ides, long chain fatty acids, cholesterol, and phospho- One molecule of ATP is hydrolyzed to AMP and PPi.
lipids are incorporated into molecular aggregates to Thus two high energy bonds are utilized in this reaction.
form mixed micelles. 13-15. What are t:he coenzymes needed for fatty acid oxi-
13-7. What happens to the fatty acids in the mucosa! cell? dation?
Once inside the intestinal mucosa! cell, the long chain FAD and NAO'.
fatty acids are re-esterified to form triglycerides. 13-16. What is the function of carnitine?
13-8. What is the further fate of this triglyceride? The long chain fatty acyl-CoA cannot pass through the
The triglycerides are incorporated into chylomicrons inner mitochondrial membrane. Therefore a transpor-
which are enters the lymph channels. ter. carnitine is involved in transfer of fatty acids.
13-9. What is enterohepatic circulation of bile salts? 13-17. What about medium and small chain fatty acids?
They are reabsorbed from the ileum and returned to Medium chain fatty acids do not require carnitine for
the liver to be re-excreted again to gut. transport; s,o they are easily oxidized.
Chapter 13: Metabolism of Fatty Acids 213

13-18. What is the net generation of ATP, when one mole- From dihydroxy acetone phosphate, derived from glu-
cule of palmitic acid (16 carbon) is oxidised com- cose.
pletely? 13-34. White adipose tissue is concernt;!d with what?
106. Energy storage.
13-19. What is the product of beta-oxidation of odd chain 13-35. Brown adlipose tissue is involved in what?
fatty acids? Thermoge1nesis.
Propionyl-CoA. 13-36. What enzyme is involved in lipolysis?
13-20. What is the metabolism of propionyl-CoA? By hormone-sensitive lipase.
Propionyl-CoA is first carboxylated to methyl malonyl-
13-37. What is its action?
CoA then to form succinyl-CoA. The succinyl-CoA then
It hydrolyses triglyceride into fatty-acid.
enters TCA cycle.
13-38. In blood, fatty acids are transported as what?
13-21. What are the coenzymes required for the conver-
Albumin is the carrier of free fatty-acid.
sion of propionyl-CoA to succinyl-CoA?
13-39. Which hormones activate hormone-sensitive lip-
Biotin, ATP, Vitamin B1 2.
13-22. Succinyl-CoA is generated from which substances? ase? or, what are the lipolytic hormones?
Odd chain fatty acids, Propionic acid, Valine, lsoleu- Growth hormone; Corticosteroids; ACTH; Adrenalin;
cine, Threonine Glucagon
13-23. Succinyl-CoA is utilised for what purposes? 13-40. In diabet«?s mellitus, non-esterified fatty acid level
Porphyrin biosynthesis; Activation of acetoacetate; in blood is increased; why?
Oxidation in TCA cycle. Insulin inhibits hormone-sensitive lipase; in diabetes,
13-24. Where is alpha oxidation taking place? this inhibition is removed, so, more lipolysis is taking
In endoplasmic reticulum (microsomes). place.
13-25. Refsum's disease is due to what? 13-41. What are the sources of acetyl-CoA?
Accumulation of phytanic acid , due to defective alpha
Pyruvate; fatty acids, acetoacetyl-CoA; Leucine.
oxidation.
13-42. What substances will prevent fatty liver?
13-26. What is the rate limiting enzyme of de novo syn-
Choline; Methionine; Lecithin.
thesis of fatty acid?
13-43. What are ketone bodies?
Acetyl-CoA carboxylase.
13-27. What is the reaction? Acetoacetate; Beta hydroxy butyric acid; and Acetone.
Acetyl-CoA + CO2 to produce Malonyl-CoA 13-44. Ketone bodies are formed in which tissue?
13-28. What are co-enzymes required for the reaction? Liver.
Biotin and ATP 13-45. KetogenEisis is taking place in which subcellular
13-29. How NADPH is made available? organelle1?
In the HMP shunt pathway, glucose-6-phosphate de- Mitochondria.
hydrogenase reaction produces NADPH. 13-46. What is the rate limiting step in ketone body forma-
13-30. Acetyl-CoA is used for what purposes? tion?
Fatty acid synthesis; Oxidation in citric acid cycle for
HMG-CoJ~ synthase.
generation of energy; Cholesterol synthesis and Ke-
13-47. Ketone body utilization is taking place in which
tone body formation
organs?
13-31 . How is fatty acid synthesis regulated?
Ketolysis is taking place in extra hepatic tissues. (All
Key enzyme, acetyl-CoA carboxylase, is stimulated by
Citrate and inhibited by palmitoyl-CoA. other tissues, except liver).
13-32. Which hormone enhances the synthesis of triacyl 13-48. Utilization of ketone bodies by peripheral tissue
glycerol? needs which enzyme?
Insulin. Succinyl-CoA dependent thiophorase.
13-33. In adipose tissue, what is the source of glycerol 13-49. What test is used to identify ketone bodies?
phosphate for triglyceride formation? Rothera's test.
_ _ _ _ _ Chapter 14
Cholesterol and
Lipoproteins

Chapter at a Glance
The learner will be able to answer questions on the following topics:
F' Struct ure of cholesterol Low density lipoproteins
,e. Biosynthesis of cho lesterol High density lipoproteins
Plasma lipids Free fatty acid
Chylomicrons Formation of bile acids and bile salts
Very low d~nsity lipoproteins

.S: ou...')" c..e.. : G: ~q "-So\¥-, Y'O~, c.h~ ·@


')/ The word cholesterol is derived from Greek words, chole BOX 14.1: Functions of cholesterol
\ :_~~ b~ steros =~; = ol aJsQ!lQ!. Almost all nucleated Cell membranes: Cholesterol is a component of memb@nes
1.
~ells (including arterial walls) can synthesize cholesterol. and has a modulating effect on the fluid state of the membrane.
2. Nerveconduction:Cholesterol is used to ~ e nerve fibers.
tis widely distributed in the body. In a 70 kg man, a total
of about 140 g of cholesterol is available; out of which 3. Bile acids and bile salts are derived from cholesterol. Bile salts
M !. N'\S. :fre important for fat absorption. = , ""'\
about 30 gin brain and nerves, 30 gin muscles and 30 g 1 i:i) re, 0
- .lt\-\ci~ 4. Steroid hormones: Glucoc:orticoids, a ~ s and §l[Q·
in adipose tissue. Cholesterol is solublein cnrorororm gens are from cholesterol. '
and other fat solvents. It is the most important animal s. Vitamin D3 is from ~o-cholesterol.
steroid. In plants, cholesterol is absent, but other plant 6. Esterlficatlon: The..Q!::!~~!cholesterolisesterifiedtofat-
sterols are present. In bacteria an~~a~ comp,o~ndsP...i\d,N'.<lth-.y acids to forrr@eJ1e~~-Thisesterification occurs in
similar to steroids exist, known a~liopanoid~ ~o~plete e body by transfer otaPUFA moiety by lecithin cholesterol
ac ~..-.e. y..!)C\a\o~ -
structure of cholesterol was enunciated by Heinrich ............
· · -,
Wieland in 1918, who got Nobel Prize in 1927. a,cec\lc.. ( Fat"\) ~
Structure of Cholesterol ~aj'oh k ~l'-4\6-
I CHOLESTEROL _ _ __ W.g~o.. .
All steroids have cyclopentanoperhydrqph~nanthrene
ring system (Fig. 14.1 ). It is a fused ring system made
Clinical Significance of Cholesterol up of 3 cyclohexane rings designated as A, B and C and
The level of cholesterol in blood is related to the devel- a cyclopentane ring D. The six-membered rings are in
opment of ~ sand l'!:rl_oc_a_rdial infarction. a phenanttyene arrangement. Total 27 carbon atoms.
Abnfflality of cholesterol ~bolism may lead to c..w:-
diov~ular accidents and h ~ attacks. These are exp-
..
One hyd.:ixyl group at third position which is charac-
teristic of all sterols. The OH group is beta-oriented, pro-
lained in detail in Chapter 15. Functions of cholesterol in jecting above the plane of ring. Double bond between
the body are enumerated in Box 14.1. Salient features of carbon atoms 5 and 6. An ight carb6nbeta oriented
steroids are shown in Table 14.1.
--==---
side Ghain attached to 17th carbo~ -
"-:-r:--,..;:
Chapter 14: Cho,escerol and Lipoproteins 215

CoA
70-S-Co.A, yH20H
Tota/no. No. ofcarbon

Name ofsteroid
of carbon atoms inside
atoms chain Importance
fH2 rH3
HO- T-CH3 HO- T-CH3
Cholesterol 27 8 Most important
animal sterol CH2 2NADPH+2H• 2NADP CH2
Bile acids 24 5 Emulsifying agents I I
COOH COOH
Glucocorticoids 21 2 Influences HMG-CoA Mevalonate
and metabolism
Mineralocortioids as well as fluid and Fig.14.2: Step 3 of cholesterol synthesis
electrolyte balance
Testosterone 19 Male sex hormones
Estrogens 18 Female sex hormones
Step 1 : Conidensation
The acetyl-CoA is provided by the ATP-citrate lyase reac-
tion as in the ca:se of fatty acid synthesis. Two molecules
of acetyl-CoA condense to form aoetoacetyl-CoA cata-
lyzed by cytoplasmic acetoacetyl-CoA synthase (see
Chapter 13, fatty acid synthesis).

Step 2: Production of HMG-CoA


A third molecule of aoetyl-CoA condenses with aceto-
acetyl-CoA to form beta-hydroxy beta-methyl glutaryl-
CoA (HMG-CoA). The enzyme is HMG-CoA synthase.
(see Chapter 1:3, ketogenesis). HMG-CoA is present in
both cytosol as well as mitochondria of liver. The mito-
Cholesterol ester present in the diet is hydrolyzed by
chondrial pool is used for ketogenesis whereas the
-'chole,e_ erol-esfer~~ TheCfrei cholestemUs incorj)ofa-'
cytosolic fractiion is utilized for cholesterol synthesis.
ted into bi!e~ e_and absorbed into the mucosal
cell. Absorption needs miceHar formation. Inside the
mucosa! cell, chol~terol is r~sterifieq_and iocorpor-
Step 3: The· Committed Step
ated into chylomicrons. The chylomicrons reach the The reduction of HMG-CoA to mevalonate is catalyzed
stream through.__1¥;11Ph~s (lacteals) (see Chap- by HMG-CoA rnductase. It is a microsomal (endoplas-
ter 13). Plant sterols (sitosterol) decrease absorption of mic reticulum) Bnzyme. It uses 2 molecules of NADPH
. -A .\f-D ~oo.N'-":>
• c~...,.,, ~ ·
cholesterol.
~\()DJ',
(Fig. 14.2). Steps 1 and 2 are shared with ketogenic
. ~r r-- - ~ , ~ .z..--~'\c.'t-ll> pathway; but sltep 3 is the first reaction that is unique
I BIOSYNTHESIS OF CHOLESTEROL to the cholesterol biosynthetic pathway. This is the rate-
limiting step.
All carbon atoms of cholesterol are derived from acetyl-
CoA (Konrad Bloch , 1940, Nobel Prize in 1964). The
Step 4: Production of 5 Carbon Unit
biosynthetic pathway was described by Sir John Corn-
forth and Vladimir Prelog; both of them got Nobel Prize i. Three phosphate groups are successively added
in 1975. The major sites of synthesis of cholesterol are to mevalonate to produce 3-phospho-5-pyrophos-
liver, adrenal cortex, testis, ovaries and intestine. phomevalonate.
AJLnucleate~Jls can synthesize ch_$>fusterol, including ii. This then undergoes decarboxylation to give iso-
arterial walls. The enzymes involved in the synthesis of pentenyl pyrophosphate, a 5 carbon unit (Fig. 14.3).
cholesterol are partly located in the endoplasmic reticu- Steps 1, 2. 3 and 4 together may be considered as
lum and partly in the cytoplasm. the first phase of the cholesterol synthesis.
--~
Cho\.~ \'r\~~!

, .:--_ ...._,
'(-.; ~le~\ \ A.cm- c.'r-.o'lu \~~\ ~ -\: e.Q
216 Section B: General Metabolism ooi.. ~ , ~~\-, \ c\r\o\atero\

0 0
II II Step 1
H2C- O- P- O- P- OH
Acetoacetyl-CoA synthase
1 I I
CH2 OH OH Acetyl CoA + Acetyl CoA "'-: Acetoacetyl-CoA
I \ + Acetyl-CoA
~c ep 2
H2C "cH3 CoA
MG-CoA synthase

t:
Fig. 14.3: lsopentenyl pyrophosphate; 5 carbon unit CoA
Beta hydroxy beta methyl glutaryl-CoA (HMG-CoA)

Step 5: Condensation of 5-Carbon Units Step 3; Rate limiting 2NADPH+2H+


HMG-CoA reductase
See Figure 14.4. step 5A, 5B, 5C and 50. A total of six 2NADP+
5-carbon units are condensed to form a 30 carbon com- Mevalo~6A~~rbons)
pound , Squalene. In summary
Step 4-A; Kinase
5C + 5C -+ 10C; 10C + 5C -+ 15C; 15C + 15C _. 30C t--.. ADP
5-phosphomevalonate (6 C)

Step 6: Cyclization Step 4-B; Kinase t ATP

Squalene is oxidised and cyclised to form 30C lanos-


terol. It is the first steroid compound synthesized. Step 4-C; Kinase :./ 1
5-pyrqphosphomevalonate (6 C)
+ ATP

Step 7: Cutting to Size


Step •-D; : :c:::::::rophospl:o;;;alonate (6 C)
A. Next, the 3 additional methyl groups on carbon atoms
4 and 14 are removed to produce zymosterol. f·-.co2

'i
lsopentenyl pyrophosphate (5 Carbons)
B. Then the double bond migrates from 8-9 position
to 5-6 position, when desmosterol is formed. Des-
mosterol is present in fetal brain. It is absent in adult
Step 5-A; lsomerase
l
Dimethyl allyl pyrophosphate (5 C)
brain and reappears in gliomas (brain tumor). ~'Y( . + Dimethyl allyl
Step 5-B, Transferase l pyrophosphate
C. Finally, the double bond in the side chain (between Geranyl pyrophosphate (10 C)
carbon 24-25) is reduced by NADPH when choles- -+< lsopentenyl
terol is formed. A summary of the whole pathway of Step 5-C ; Transferase pyrophosphate
PPi
cholesterol synthesis is given in Figure 14.4.
Farnesyl pyrophosphate (15 C)
NADPH J+ Farnesyl
Regulation of Cholesterol Synthesis
&

Step 5-0; Squalene synt~~~V ! pyrophosphate

Regulation at transcription : The regulatory enzyme Squalene (30 C)

l
NADPH
is HMG-CoA reductase. Long-term regulation involves Step 6-A: Epoxidase
regulation of transcription of the gene for HMG-CoA 02
Squalene epoxide
reductase. When sufficient cholesterol is present in the
cell, transcription of the gene for HMG-CoA reductase
is suppressed, and cellular synthesis of cholesterol is
1 Step 6-B: Cyclase
Lanosterol (30 C)
decreased. When c_!1olesterol jn djet i ~ , synthesis is
J.ocreased (Fig. 14.5). Cholesterol regulates the expres-
1 Step 7-A

Zymosterol (27 C) d.
sion of tiMG-CoA reductase gene and LDLR (LDL
r~ ce toI) g ~ --- l step7-B /

Covalent modification: Short-term regulation is by Desmosterol


Konrad John Vladimir
covalent modification of the enzyme. HMG-CoA is active Bloch Cornforth Prelog t step 7-C
NP 1964 NP 1975 NP 1975 Cholesterol (27 C)
in the ~s.p.b,PlY.lakd state. 1912-2000 1917-2013 1906-1998
Insulin and thyroxine increase the activity of HMG-
CoA reductase. Fig. 14.4: Cholesterol biosynthesis
:_r-.(J- i :.c: 1:>Gp) o~\)h1.n ak,
¼\: ~ \ a l-- ~ ~)w&frw-ryl, e,h,L- = Govol.em._ f1\®~C-
4
HD:-cM~~, Co...\-C'l.JJ Lr.A~~ rho\e.21:ero\ €9..~
l-e-c ,th,!\ C. o.~ Cz..J f\<ilrno... Chapter 14: Cholesterol and Lipoproteins 217

Inhibitors Activators Dietary Cholesterol from


cholesterol extrahepatic
~ } holeslerol @
Transcription l tissues
!
!
Chylomicron
remnants HDL

Translation

-~ lucago~ f'.) -
!
Enzyme ~ Insulin
ort,sone HMG-CoA Thyroxin
·statins· reductase
' ->.! ic~ r)
HMG-CoA reductase - - - - - . Cholesterol
Fig. 14.5 : Regulation of cholesterol synthesis

TABLE 14.2: Plasma ltptd profile


Analyre Reference value
Major routes by which cholesterol leaves the liver
Total plasma lipids 400-000 mg/dl
Total cholesterol 140-200 mg/dl A01- Fig.14.6: Cholesterol pool in the liver
,,....._r'r--
HDL cholesterol, male 30- 60 mg/ dl so•;.
HDL cholesterol, female 35- 75 mg/dl day. Out of this total 1000 mg, abotlt 500.fng of choles-
LDL cholesterol, 30-39 years 80-130 mg/ dl terol is excreted through bile. This cholesterol is partly
Triglycerides, male 50- 150 mg/ dl :2-o·f reabsorbed from intestin~ ~~ ~tes contain plant
Triglycerides, female 40-150 mg/ dl sterols which inhibit th reabsol])tion of cholesterol. The
Phospholipids 150-200 mg/dl 'JI()'/. unabsorbed portion is acted upon by intestinal bacteria
Free fatty acids (FFA) (NEFA) 10- 20 mg/dl to form ct!,.olestanol and ccmrostanol. These are excreted
(fecal sterols). About 500 mg of cholesterol is c~n'ierted
~ ortisol and_gUJ..Qggon decrease its activity. tQ_bile asjds, ~hich are excreted in the bile as bile salts
~ _))rugs: Jov'ifs
a iCl and othe 'statin" roup of drugs (Fig. 14.6).
are competitive inhibitors of HM -CoA reductase. So,
they are used in clinical pra · e to reduce cholesterol Liver and Cholesterol
level in blood. '(Y\~'\.c,...~ \ O'\ .ho The liver has a major role in controlling the plasma
,I, : : . ~ ~ J ehe_c.'n~
P . 1 .c, :"'e.'n\ n, "-;b., levels of LDL cholesterol.
Cholesterol Pool and l1"'i " 1. Liver synthesizes cholesterol.
Cholesterol Metabolism 2. Liver removes cholesterol from lipoij'6tein rem-
nants and Hgt.
The free cholesterol within the cell has the following
3. Liver is the only organ that can e?5crete cholesterol
fates: (1) Incorporated intifg?fl ~ 111.braoes. (2) Metabo-
through bile.
lized to steroid 'hormones, especially in adrenal cortex
4. Liver converts cholesterol to bile acids.
and gonads. (3)___E'stefified with saturated fatty acids and
stored in the cell. The enzyme'Ac~acyl cholesterol
PLASMA LIPIDS'@ - - - -
acyl transferase) helps in this reaction. (4) Esterified with
poly-unsaturated fatty acids (~UF~ by the action of Total plasma lipid is 400-600 mg/dl. Normal values of
LCAT {le~ ithin cholesterol ac I transferase) and finally lipid fractions are shown in Table 14.2. Out of this, 40% is
excreted through liver. cholesterol; 30% is phospholipids; 20% is triglycerides.
Since lipids are insoluble in water. the need the help of
carriers in plasma. Therefor ey are complexed with
Excretion of Cholesterol
e>r-, cnorr- proteins to form lipoproteins. The protein~ rt of lipo-
Average~ contains about 300 mg of cholesterol per protein is caIled ae,olipaproteJn. The lipoproteins are
day. Body synthesize about 700 mg of cholesterol per usually abbreviated as Lp.
-- ,.@ O~
218 Section B: General Metabolism

Classification of Lipoproteins General Characteristics


Depending on the density (by ultra centrifugation) or on of Lipoproteins
the electrophoretic mobility, the lipoproteins in plasma
are classified into five major types (Figs. 14_7 and 14_8 ). The salient characteristics and compositions of lipopro-
1. Chylomicrons _ contain apoprotein B-48. teins are giv{&in Table 14.3. The lipopro1f$. molecules
2. Very low density lipoproteins (VLDL) or pre-beta have a_R~l~r periphery mft.:<te of proteins, p ~ o f
lipoproteins. Main apoprotein is B-100. p - r o l . The inner £Ore consists
3. Intermediate density lipoproteins (IDL) or broad- of the bydropl:lobic TAGs and tails of phospholipids. The
beta lipoproteins apoproteins also increase the solubility of lipids.
4. Low density lipoproteins (LDL) or beta-lipopro-
teins. Major apoprotein in LDL is B-1 oo. @Separation by Ultracentrifugation ~
5. High density lipoproteins (HDL) or alpha-lipopro- . r-:'
o< ::lfcl.-ns1 \1.. ..-~ =l --,..r.?(.)
teins. Major apoprotein in HDL is apo-A. T_he hpo~rotems are separatedon the basis of their den-
Free fatty acids (FFA) or nonesterified fatty acids s1ty. Fat 1s less dense than water; so fat floats on water.
(NEFA) are complexed with albumin. FFAs are not Lipoproteins with high lipid content will have a low den-
generally included in the classification of lipopro- sity and so float on centrifugation. Those with high pro-
teins, because they are loosely bound to the protein. tein content will sediment easily and have a high density.
Depending on the fl,2tation constan.LJSf), different frac-
tions can be separated (Table 14.3 and Fig. 14.7).
Ultracentri- Electrophoresis
fugation Lipoproteins
(density g/l)
Lipoproteins
(mobility)
eparation by Electrophoresis
The basic principle of electrophoresis is given in Chap-
Chylomicron ter 31. The serum is applied on cellulose acetate, elec-
Chylomicron (origin)
tric current is applied for 2 hours, the strip is dried and
LDL (beta)
stained with lipid dyes, such as Oil Red 0 . As a general
VLDL
rule, those with higher protein content will move faster
IDL IDL (broad P)
LDL
towards the anode and those with less proteins have
VLDL (pre-p)
HDL
minimum mobility (see Fig. 14.7).
~ DL (alphaD

FFA FFA-albumin Apo-lipoprotei ns


The t[fotein part of lipo_protein is called apolipoprotein
(apo-Lp) or apoprotein. For details of apoproteins, see
VLDL = very low density lipoprotelns; IDL = intermediate density Table 14.4. All apoproteins are mainly synthesized in
lipoproteins; LDL = low density lipoproteins; HDL = high density
lipoproteins; FFA = free fatty acid liver. Intestinal cells produce small quantities of apo-A.
Apart from solubilizin the Ii i rt, the protein com-
Fig. 14.7: Comparison of electrophoretic and ultra-centrifuge pat-
terns of lipoproteins

60%
90%

2% -r·a :• · -...::
'11111111111111 , 20%
3%

Chylomlcron (CM) Very low-density lipoprotein (VLDL) Low-density lipoprotein (LDL) High-density
llpoprotein (HDL}
D Triacylglycerol Protein Ill Phospholipids D Cholesterol

Fig. 14.8: Comparison of composition of different lipoproteins


Chapter 14: Cholesterol and Lipoproteins 219

TABLE 14.3: Charactenstics of different classes of hpoproteins


i Chy/omicron VLDL IDL LDL HDL FFA (•)
Densityg/ L <0.95 0.95- 1.006 1.006-1.019 1.019- 1.063 1.063-1.121 1.28- 1.3
Diameter (nm) 500 70 30 25 15 -
Electrophoretic mobility origin pre-beta broad beta beta 1B alpha ex. albumin
% Composition I
G.c,cd.\ ~"'cl
Protein 2 10 20 20 I 30--00 , I 99 I
TAG <so') 50 30 10 10 0
Phospholiplds 10 20 20 20 (f0-30) 0
Cholesterol 10 20 30 63) 10-30 0
FFA 0 0 0 0 0 (1")
Apoproteins A, B-48, C-II, E B-100, C-11, E B-1 00, E B-100 A-I, C, E Albumin
Transport function TAG TAG ~~ bt~1¥, Cholesterol Cholesterol FFA
,, j
..... from gut to
muscle and
from liver o
muscle and
from liver to from peripheral from adipose T to
peripheral tissues tissues to liver muscle and liver
l'tadipose tissue
adipose tissue q - \.9 l <3-"'> t~ 4 l 'l-'J(4 +-9'
(") Free fatty acids (FFA) are not generally included in the lipoproteins. They are seen in circulation, weakly bound to albumin.

LDL; VLDL; chylomicron Arginine rich; ligand for hepatic uptake


Lp(a) Attached to B-100; impairs fibrinolysis; highly atherogenic

, -- ~ ~ ~ ~:;::== =~AAip~o~-si-:44EB~
- =;;;n~lo't'~o~)-_:::
J __ produce the 8-48 protein (Fig. 14.11). 8-48 is so
0-,-e. ro, r-.o r amed because it is only 40% of the size of 8 -100.
74. Apo-C-11. ActivatesJipoprot!'tl!:! lipase')
t 5. Apo-E. It is present in chylom icrons, LDL and
\ VLDL. Apo-E has I, 11, Il l and IV isoforms. A po-E-IV
\."ii. isoform is implicated in the development of senile
~ , ..
Cholesterol
Phospholipid outer cover dementia and Alzheimer's disease.

'--- - Triglycerides in the interior I CHYLOMICRONs@


Fig. 14.9: Structure of chylomicrons
Synthesis of Chylomicrons
1. A po-A-I. Activates lecithin-cholesterol acyl trans Chylomicrons are formed in the intestinal mucosa!
ferase {LCAT). It is the ligand for HDL receptor. It is cells, and secreted into the lacteals of lymphatic system .
anti-atherogenic. It is specific for HDL. They are rich in triglyceride (Fig. 14.9). If lipem ic
2. Apo-B-100. Only apoprotein component of LDL; it serum is kept overnight in the refrigerator, chylomicrons
binds to LDL receptor on tissues (Fig.14.10). rise as a creamy layer to the top, leaving the s ubnatant
3. Apo-8-48. Majo r apoprotein of chylomicrons. It is clear. When the chy lomicrons are synthesized by the
synthesized only in intestinal cells. Apo-8- 100 and intestinal mucosa, they contain only ~ B-48 and
apo-8 -48 are products o f the same gene, but in the apo::A. A.QQ:C, and ~o- are added from HD L .rn bl ocd
, - - r:-- -..
intestine, the mRNA undergoes editing, so as to during transport (Fig. 14.10).
220 Section 8 : General Metabolism

Phospholipld
B-48e
Triglyceride
Cholesterol
c-u•

B-48e
l e A
B-48e e A Fig. 14.1 1: Low density lipoprotein (LDL)
,')..' Adipose c-11• E
Nascent chylo tissue
Chylo remnantl
onlyTG Mainly Ch (YERY LOW DENSITY LIPOPROTEINS
Synthesis of VLDL
Triacylglycerol synthesized in liver is incorporated into
Apo-E-receptor VLDL along with hepatic cholesterol. Apo-B-100 is the
major lipoprotein present in VLDL when it is secreted.

ral tissues, apo-C-11 acti-


Chylo =chylomicron; Ch =cholesterol; TG =triglyceride. vates Lpl which Ii rates fa acids that are taken up
B-48, C-11, A and E are apoproteins.
by adipose tissue d muscle. The€..,~ now des-
ignated as IDL (intermediate density lipoprotein) and
Fig. 14.10: Metabolism of chylomicrons contains less ofTAG and more of cholesterol (Table 14.3
and Fig.14.17). The major fraction of IDL further loses
Metabolism of Chylomicrons triglyceride, so as to be converted to LDL (low density
Main sites of metabolism of chylomicrons ar adipose lipoprotein). This conversion of VLDL to IDL and then
tissue and{skeletal muscl~. The enzyme lipo rotein.lip- to LDL is referred to asJ,poprotein cascadi:i>athway
@ Lpl) i; focated at the endothelial layer of capillaries (Fig.14.17). A fraction of IDL/is taken up by the hepatic
V
of adipose tissue, muscles and heart; bu otfn i~e . receptors. \J L O L.- ...,. L 0 \.- -1-Dl..
Apo-C~I present in the chylomicrons activates fle Lpl
Function of VLDL
(Table 14.4).The Lpl hydrolyze~ present
in chylomicrons into fatty acids~ d g cerol. Muscle or The VLDL carries endogenous TAG from liver to
adipose tissue cells take up the liberated fatty acids (Fig . peripheral tissues for energy needs.
14.10).
I LOW DENSITY LIPOPROTEINS
Liver Takes up Chylomicron Remnants Low density lipoproteins (LDL) transports cholesterol
from liver to peripheral tissues. The on.!y_apoprot~n pre-
As the TAG content is progressively decreased, the
sent in LDL is a o-B-1-9..Q..(Fig.14.11). Most of the LDL
chylomicrons shrink in size. These r~ nants containing
particles are derived from VLDL. The half-life of LDL in
qpa:B-48 and apo-E are taken up by hepatic cells by blood is abou 2 days.
receptor mediated endocyto~ s. Apo-E binds the ./

hepatic receptors (Fig. 14.10). Metabolism of LDL and LDL Receptors


The LDL is taken up by peripheral tissues by receptor-
Function of Chylomicrons mediated endocytosis. LDL receptors are present on all
Chylomicrons are the trans ort form of dietary tria~ c- cells but most abundant in hepatic cells. LDL receptors
erides from inte tine o th dipose tissue for stora are located in specialized regions called clathrin-coated
and to muscle heart for their e ~~ e~ . pits (Fig.14.12). Binding of LDL to the receptor is by
r~('...d_-v-::;o..\d'~ 1?_9[>· :)o~ LDL c.eut '
-----------~--'------"'\
Cjt:_=-_L~D~I,.._. .::::..C~t-_\-_f_:_Dl.~
·C_+_V_~_'D_L_·_C.
- Chapter 14: Cholesterol and Lipoproteins 221

, /LDLwith
Clathrin- ~ ¥" cholesterol
coated pits ~ - B- 100

Cytosol
1
~! o
Recycling
Liver - VLDL- LDL

Coated
vesicle

@
HDL - -----
Lysosome

Fig . 14.13: Forward and reverse transport of cholesterol

~ --+ ~~ ii . Cholesterol may be incorporated into the mem-


LDL digested Cholesterol
deposited
Joseph L Goldstein brane .

Fig. 14.12: Uptake and fate of LDL


NP 1985 b. 1940
-
iii. Cho,esterol may be esterified to a MUFA by acyl
---
cholesterol acyl transferase (ACAT) for storage.
The cellular content of cholesterol regulates further
BOX 14.2: LDL cholesterol is "bad· endogenous synthesis of cholesterol by regulating
LDL, especially oxidized and glycated LDL, creates a pro-coagu- HMG-CoA reductase.
lant surface on the endothelium, causing thrombus format ion.
Oxidized LDL is found in higher levels in cigarette smokers,
patients with diabetes mellitus and in insulin resistance. LDL and Clinical Applications
The LDL concentration in blood has positive correlation
apo-B-100. When the apo-B-100 binds to the apo-B-100 with incidence of cardiovascular diseases. A fraction
receptor, the ~ceptor-LDL complex is_internaliz~by of cholest~ I is taken up by macrophages. 1ncrEti3sed
endocytosis. l~vels of 1.Q,L or oxida ·on of LDL increases uptake of
cholesterol by rr.li;ltropbages. LDL infiltrates through

-
The endosome vesicle thus formed fuses with lyso-
somes. The LDL particle, along with apoproteins and arterial walls, and are taken up by macrophages. This
cholesterol ester are hydrolyzed to form fr~olesterol. is the starting event of a heroscierosi leading to myo-
-"-- ___,,A.
The free receptors can now return to the membrane cardial infarction (see coronary artery diseases in Chap-
surface to bind further LDL molecules (Fig. 14.12). For ter 15). When macrophages are filled with cholesterol,
their work on LDL receptors, Michael Brown and Joseph foam cells are formed. They get deposited in the sub-
Goldstein were awarded Nobel Prize in 1985. endothelial space and leads to the formation of athe.!9-
The free cholesterol is either incorporated into plasma matous plague (see Fig. 15.3). Th~ results in increased
membranes or esterified (by ACAT) and stored within chances of thro<,Wbosis and corb~ary artery disease
the cell. The excess cholesterol tends to be deposited (see Fig. 15.4).
within the arteries, leading to atherosclerosis. Since LDL-cholesterol is thus deposited in tissues, the

Function of LDL
LDL (low density lipoprotein) variety is calle~~
->-
Ji~
terol" in common parlance (Fig.14.13 and Box 14.2).
About 75% of the plasma cholesterol is incorporated into lns.ulio...aru:1Jri-iodothyronine (T3} increase the bind-
-4= --- --
the LDL particles. LDL transports cholesterol from liver ing of LDL to liver cells. This explains the ~ er ~s-
to the peripheral tissues. The cholesterol thus liber- tec.oJecmaseBriTrLaiatfetes an oth ro1d1sl'n.
ated in the cell has three major fates: Defects in LDL receptor synthesis leads to fami-
i. It is used for the synthesis of other steroids like lial hypercholesterolemia, which is further described in
steroid hormones. Chapter 15.
222 Section B: General Metabolism

Lp(a) has homology BOX 14.3: Lp(a) and apo-A are different
with plasminogen
Apo-A is a constituent of HDL. This"A" is always written in capital

l
Plasminogen
Lp(a) apo-B-100 letters. It is seen in all persons. It Is antiatherogenic.

l
Activation to plasmin Lp(a) binds to fibrin
Lp(a) is seen in high levels only in some persons. When present,
it is associated with LDL This •a• is always written in small let-

!
and Inhibits plasmi- ters. It is highly atherogenic and connected with heart attacks in
nogen activation younger age group.

Plasmin binds to fibrin

l ! Lecithin is a component of phospholipid bilayer of


the HDL disk. Structure of lecithin is shown in Chapter
Fibrin lysed Fibrin
Fibrin not 8. The second carbon of lecithin contains one molecule
lysed
o~ ~ _acid (PUFA). It is ~ferr to
the third hydroxyl group of cholesterol to form esterol
Fig. 14.14: Lp(a) competitively inhibits plasminogen activation;
and so inhibits fibrinolysis =
ester. The esterffied cholesterol which is more hydrophobic,
moves into the interior of the HDL disk. This reaction con-
tinues; till HDL becomes spherical and a lot of choles-
Lipoprotein (a) terol esters are formed. ~ture HDL spheres are taken
up by liver cells by ~-Mm'-djat~regeptpr mechanism
Lipoprotein (a) or Lp(a) should not be confused with
(Figs. 14.15 and 14.17). Hepatic lipase hydrolyzes HDL
apo-A (Box 14.3). Lp(a) is very strongly associated with
phospholipid and TAG. Cholesterol esters are released
myocardial infarction and is sometimes called the "little
rascal". Lp(a), when present, is attached to apo-B-100 into liver cells. The cholesterol that reaches the liver is
by a disulfide bond . In 40% population, there is no used for synthesis of bile acids or excreted as such jn bile.
detectable level of Lp(a) in serum. I@% of population, The cholesterol ester from HDL is transferred to
the~ concentrationjn blood is mQJ1tttlan 30 mg/QL; VLDL'. IDL and LDL by a C ~ s f e r
and these persons are susceptible for heart attack at a Protein .ifETP~.
younger age. Lp(a) is associated with heart attacks a(s\F ncf of HDL
IO 5
the age of 30 or 40 years. Indians have a higher /eve~ U n
of Lp(a) than Western populations. Lp(a) has ig.Qificant i. HDL is the · t~port form of cholesterol from
homoJQgy_-WittLplasllll!J.Q.9en. So, it interferes with plas- peript)eral tissue Iver, which is later excreted
rrinogen activation and impairs fibrinolysis (Fig.14.14). through @i)i e 14.3). This is called reverse
This leads to unopposed intravascular thrombosis and cholesterol transport by HDL.
possible myocardial infarction. -~'f'\ ii. The only excretory route of cbclesteml from the
1
/4 lo':\ ~ bodyisth@
I HIGH DENSITY LIPOPROTEfN ~ ~ ~ii. Excretion of cholesterol needs prior esterification
High density lipoproteins (HDL) transport cholesterol with PUFA. Thus PUFA will help in lowering of cho-
from perip~ues to the liver. The major apoprotein lesterol in the body, and so PUFA is aotiatbero-
in HDL is ( t p ~ genie.

@ Metabolism of HDL © clinical Significance of HDL


t-A ~"\(" t,'::}-
The intestinal cells t5Ynthesize components ofHDL and The level of HDL in serum is inversely related to the
._
release int8m~The nascent HDL in plasma are dis-
_. incidence of myocardtal infarction. As it is futiathero-
coid in sha~e (Fi~ ~ -15). The f~~cholesteml deril(ep genic" or "protectiv.2l, nature, HDL is known as "go~d,.
~ om perjphern sue cell~L~e..,,taken up by the~ L. The cholesterol" in common parlance (see Fig. 14.13)7i
apo-A-1 of HDL activates LCAT (lecithin cholesterol acyl .~
·s nvenient " in HDL stands for
transferase) present in the plasma. The LCATthen binds
, , e,+ c..b
~ •- DL level below 3 ~/~ increases the risk,
to the HDLdisk. The reaction is shown in Figure 14.16. while level above 60 mg/dl protects the person from
Chapter 14: Cholesterol and Lipoproteins 223

Liver Intestine
PL, A1, C, E

Pl.
Discoid
l Heart and
blood vessels
c;,i Nascent HDL
! LCAT Ch
PL
Excretion of A1 cE
Ch+ ChA .,S~ .~ Mature HDL
PL= phospholipid, HDL = high densitr,lp~ro\~~~ cholesterol, ChE = cholesterol ester.
ChA = cholic acid, LCAT = lecithin cholesterol acyl transferase
Fig. 14.15: HDL metabolism

l , c i t h ~;th;" Intestine Liver

I•••
Cholesterol Cholesterol ester
Fig. 14.16: Lecithin cholesterol acyl transferase (LCAT)
Chyle

HDL 11

ee •;
remnants
coronary artery diseases. A summary of the lipoprotein
•••
1
Blood
metabolism is shown in Figure 14.17.
l •
Chylomicrons vessel

HDL-specific Receptors
\
Blood
In liver, specific recepto , R-8 bjnds apo-A-1 present vessel
o..Q..!:!Q..L. Then~ les erol is internali .

I FREE FATTY ACID FFA)-' Peripheral tissues Peripheral tissues

Fig. 14.17: Summary of lipoproteln metabolism

Cholesterol is an essential component of biomem-


branes._Since it has a hjgh dielectric constant, its pres-
ence on nerve sheaths has an insulating eff~ct during
t~e transmission of nerve impulses.
are transported to peripheral tissues in""--_,, Note: The steroid ring is not degraded in the huma~
albumin. body. Cholesterol is ~cii
used for providing energ) \l5)

I FORMATION OF BILE ACIDS


Windaus (Nobel Prize, 1928) in 1918 showed the con-
version of cholesterol to bile acid. Complete structural
analysis was done by Heinrich Wieland in 1918, who got
Nobel Prize in 1927.
Functions of Cholesterol
Synthesis of Bile Acids
Precursor of all steroid compounds in the body like b~
acids, steroid hormones (see Chapter 45) and vitamin Bile acids are synthesized in the liver from cholesterol.
0 3 (see Chapter 32). - They contain 24 carbon atoms. All of them have an
~ -

224 Section B: General Metabolism

y. '3, Cholesterol (27C)


Cholic a cid
(3.7.12-trihydroxy (+)3 OH groups~ (+)2 OH groups
cholanic acid)
Cholestane 3,7 diol

1'-Prnpkmate

rl•I
Chenodeoxycholic acid (24C)

HS- CoA

Chenodeoxycholyl-CoA
V Taurine

~ CoA
Glycocholic acid Taurochenodeoxycholic acid
alpha-oriented (projecting below the plane of ring) Fig. 14.19: Formation of bile salts
hydroxyl group at position 7. The r~ctioos foe synth96is
of bile acjos are summarized below:
1. Cholesterol hydroxylated at ¥!.f.12 QOS~ S
2. Removal of 3-carbon unit, to make it 24 C
3. Conjugation with glycine
4. Secretion into intestinal canal
5. In the intestine, deconjugation and removal of Adolph Adolf Otto Heinrich Otto
hydroxyl groups. Strecker Windaus Wieland
1822- 1871
* Hydroxylation Reactions
NP 1928 NP 1927
1876-1959 1877-1957

One hydroxyl group is adde-,,..-..,_,..~~- nzyme 7-alpha-

group is added at 2th car


-
hydroxylase. This is the r te-limiting s
· holic
acid. Chenod~ cholic acid , anot er primary bile
Functions of Bile Salts
acid has only Nf6 hydroxyl groups at positions 3 and 7
(Fig.14 .18). Ring B is reduced in all cases. s. They can form
>--..l'·~ i
hich bring about
Removal of.3 Carbon Unit icelle also plays an
- =-=
important role in keeping the cholesterol io sahrt;on.
Cleavage takes place at 24 C, with removal of propionic
acid (3 carbon) unit. @ Enterohepatic Circulation of Bile Salts
Formation of Bile Salts Of the total bile salts reaching the intestine (15-30 g/
day) only a very small fraction, about 300-500 mg/day
The primary bile acids are now conjugated with either
is excreted through feces (see Fig. 14.6). The rest is
glycine or taurine to form bile acids (Fig. 14.19). The
major conjugated bile acid is glycocholic acid. Conjuga- (re,absorbed from ileu'!)) reaches liver and re-excreted
tion adds more polar groups and increases the efficiency through bile. This is referred to as the enterohepatic cir-
of bile acids as surfactants. T ·u ated ii s culation. When (b1Te acidbmdin resin (ch~ mine)
are excrete b e. In the bile they exist as is given, the reabsorption of bile aci ~ hed.H~nce
bile salts od ium or potassium salts of conjugated bile more choleste~ol gets co~
acids). le~ al is decce.as.ed.

® Secondary Bile Acids/Bile Salts Bile


Intestinal bacterja the primary bile acids. It is the chief secretion of liver, the largest gland in the
Then bile acids are partly converted to secondary bile acids body. Daily volume of secretion is about 500 ml. The
/fureted bile"> is filored jn the gallbladder and released
on.ctemaod. The pH of bile in hepatic duct is 7.8, and in
gallbladder is 7.4. An enzyme present in bile is alkaline 1. Cholesterol has a cyclopentanoperhydrophenan-
phospha e _ _ threne ring with a total of 27 carbon atoms. Acetyl-
Cholereti s are substance?wm the CoA is the precursor of cholesterol. Cholesterol is a
secrei1onof bile by the liver holagogue constituent of cell membranes. It is the precursor of
all steroid hormones, bile acids and vitamin 03.
rele~ e of bile m the gallbla,,_.,,_,rn st important
2. Cholesterol is synthesized mainly in liver, adrenal
choleretics are ii o ~ tin and
cortex and gonads. The rate limiting enzyme of the
vagal stimulation. Cholecyst inin is the most power-
cholesterol biosynthetic pathway is HMG-CoA red-
ful cholagogue. The release of cholecystokinin itself is uctase.
stimulated by fatty acids and amino acids in duodenum. 3. Normal serum cholesterol range is 150-200 mg%.
4. Lipoproteins are of 5 major types; Chylomicrons,
Functions of Bile VLDL, IDL, LDL and HDL.
i. The alkaline pH of the bile serves to rieutralize the 5. Chylomicrons contain the apo-B-48. Chylomicrons
cl,Cidity of the gastric juice. help in the transfer of triglycerides from the intes-
tine to the muscle and adipose tissue.
ii. The bile salts are ~cjent surfactants and detergents.
6. VLDL helps in the transfer of triglycerides from the
iii. Bile is the only route of excretion for bilirubin, the
liver to the peripheral tissues.
enq.product of heme catabolism. 7. LDL carries cholesterol from the liver to the heart,
iv. It s~ ta_excrete cholestei:ol, thus regulating the while HDL carries cholesterol from the heart to the
body cholesterol pool. liver. LDL contains apo-B-100, while HDL contains
v. Bile serves as the rae.dil.JllL()f excretion for s.e!Ler al apo-A-I
drugs, which are detoxified by the liver. 8. LDL is 'bad' cholesterol and HDL is 'good' cholesterol.
9. Higher concentration of Lipoprotein (a) or Lp(a)
Cholesterol and Cardiac Diseases increases risk of a myocardial infarction.
10. Free fatty acids in plasma are transported bound to
The level of cholesterol in blood is related to the develop- albumin, and are taken up by peripheral tissues.
ment of atherosclerosis. Abnormality of cholesterol 11 . Increase in TAG or cholesterol or both in plasma
metabolism may lead to cardiovascular accidents and leads to hyperlipidemias, which can lead to early
heart attacks (see Chapter 15). atherosclerosis and coronary artery disease.

PART-1 : ESSAY AND SHORT NOTE QUESTIONS

14-1. Enumerate the major steps of synthesis of cholesterol. Name the rate-limiting step of cholesterol synth esis.
Describe the role of cholesterol in atherosclerosis.
14-2. Classify lipoproteins. Explain their biological significance.
14-3. What is the normal cholesterol level in plasma? Explain how the cholesterol is transported from liver to periphe-
ral tissues and back.
14-4. How are dietary triglycerides absorbed and transported in plasma? Explain briefly the transport of dietary TAG
from intestine to liver.
14-5. How are endogenously produced triglycerides transported in plasma? Explain the transport of them from liver.
Indicate the abnormalities associated with this process.

SHORT NOTE QUESTIONS

14-6. Biologically important compounds derived from 14-10. Give the normal blood level of cholesterol.
cholesterol. 14-11. Apo-B-100.
14-7. HMG-CoA reductase. 14-12. Chylomicrons.
14-8. Regulation of cholesterol synthesis. 14-13. Lipoprotein lipase.
14-9. Key enzymes of cholesterol biosynthesis.
226 Section B: General Metabolism

PART-2: MULTIPLE CHOICE QUESTIONS

14-1. All are useful substances produced from choleste- C. Cholesterol from peripheral tissues to liver
rol, except D. Free fatty acids (NEFA) from adipose tissue
A. Vitamin D B. Bile salts 14-12. Function of LDL Is the transport of:
C. Bile pigments D. Cortisol A. Triglycerides from intestine to adipose tissue
14-2. The ring system of cholesterol is called: B. Cholesterol from liver to peripheral tissues
A. Cholanthrene ring C. Cholesterol from peripheral tissues to liver
B. Cyclopentano phenanthrene D. Free fatty acids (NEFA) from adipose tissue
C. Naphthoquinone 14-13. The cyclopentano phenanthrene ring structure is
D. Corrin ring seen in all the following compounds, except
14-3. During cholesterol blosynthesis, the first sterol A. Testosterone B. Adrenocorticotropin
ring synthesizes is: C. Glycocholic acid D. Vitamin D
A. Squalene B. Zymosterol 14-14. All are intermediates in the biosynthesis of choles-
C. Lanosterol D. Desmosterol terol, except
14-4. The key enzyme In the pathway of cholesterol blo- A. Acetyl-CoA B. Cholyl-CoA
synthesis is: C. Acetoacetyl-CoA D. HMG-CoA
14-15. All are true with respect to plasma lipoproteins, except
A. HMG-CoA synthase
B. HMG-CoA lyase A. Lipoprotein lipase is activated by apo-C-11
C. HMG-CoA reductase B. LCAT is activated by Apo-A-I
C. Apo-E is involved in receptor mediated uptake of
D. Mevalonate kinase
chylomicron remnants
14-5. Cholesterol contains how many carbon atoms?
D. Apo-A-I is atherogenic
A. 6 B. 12
14-16. Select the correct statement regarding HDL:
C. 27 D. 30
A. Normal HDL cholesterol in serum is 150- 250 mg%
14-6. The serum lipoprotein with the highest cholesterol
B. It contains apoprotein 8-48
content Is:
C. It is associated with enzymatic conversion of free
A. Alpha lipoprotein (HDL)
cholesterol to cholesterol ester
B. Beta lipoprotein (LDL)
D. Mature HDL has discoid shape
C. Prebeta lipoprotein (VLDL)
14-17. Function of albumin is the transport of:
D. Chylomicrons
A . Triglycerides from intestine to adipose tissue
14-7. The main apoprotein present in the low density
B. Cholesterol from liver to peripheral tissues
lipoprotein (LDL) is:
C. Cholesterol from peripheral tissues to liver
A. A and D B. B-1 00
D. Free fatty acids (NEFA) from adipose tissue to
C. E2 and C D. B-48
peripheral tissues
14-8. The main apoprotein present in chylomicron:
14-18. All are true in familial hyperlipoproteinemia Type
A. D B. B100 II-A, except
C. E2 D. B48 A. TAG level in blood is increased
14-9. The main apoproteln present In HDL Is: B. LDL level in blood is increased
A. A1 B. B-100 C. LDL receptor defect is seen
C. E2 and C D. B-48 D. Cholesterol level in blood is increased
14-10. Function of chylomicron is the transport of: 14-19. The plasma sample of a 35 year old man after over-
A. Triglycerides from intestine to adipose tissue night refrigeration showed a creamy layer on top
B. Cholesterol from liver to peripheral tissues and opalescence below. The condition which is
C. Cholesterol from peripheral tissues to liver excluded is
D. Free fatty acids (NEFA) from adipose tissue A. Diabetes mellitus
14-11. Function of HDL is the transport of B. Alcoholism
A. Triglycerides from intestine to adipose tissue C. Hypothyroidism
B. Cholesterol from liver to peripheral tissues D. Familial hypercholesterolemia

ANSWERS OF MULTIPLE CHOICE QUESTIONS


14-1. C 14-2. B 14-3. C 14-4. C 14-5. C 14-6. B 14-7. B
14-8. D 14-9. A 14-10. A 14-11 . C 14-12. B 14-13. B 14-14. B
14-15. D 14-16. C 14-17. D 14-18. A 14-19. D
Chapter 14: Cholesterol and Lipoproteins 227

PART-3: VIVA VOCE QUESTIONS AND ANSWERS

14-1. What is the ring structure present in cholesterol? 14-16. What is lipoprotein(a) or Lp(a)?
Perhydro cyclopentano phenanthrene ring. It has homology with plasminogen. So it interferes with
14-2. What are substances derived from cholesterol? plasminogen activation and impairs fibrinolysis. This
Glucocorticoids, mineralocorticoids, testosterone, estro- leads to unopposed intravascular thrombosis and pos-
gen, bile acids. sible myocardial infarction.
14-3. What is the rate limiting step in the cholesterol bio- 14-17. What is the significance of Lp(a)?
synthesis? Lp(a) is associated with heart attacks at the age of 30
HMG-CoA reductase. or 40 years.
14-4. How cholesterol is excreted? 14-18. What is the function of HDL?
Through bile, partly as cholesterol itself, and partly as Transport of cholesterol from peripheral tissues to liver.
bile salts. 14-19. What is "good cholesterol"?
14-5. What is the normal level of total cholesterol? HDL cholesterol.
140-200 mg/dl. 14-20. Why is it called so?
14-6. Maximum cholesterol content is in which lipo- HDL transports cholesterol from peripheral tissues to
protein? liver, and so helps in excretion of cholesterol from the
LDL (Low density lipoprotein). body. So HDL is antiatherogenic.
14-7. What is the function of chylomicron? 14-21 . What is the main apoprotein present in HDL?
Transport of triglycerides from intestine to adipose Apo-A-I; it is the ligand for HDL receptor.
tissue. 14-22. What is LCAT?
14-8. What is the main apoprotein present in chylo- Lecithin cholesterol acyl transferase.
micron? 14-23. Where is it present?
Apoprotein, B-48. LCAT present in plasma is activated by apo-A-I, when
14-9. Highest content of triglycerides is seen in which LCAT binds to HDL disk.
lipoprotein? 14-24. What is its importance?
Chylomicrons. The free cholesterol is esterified by LCAT; the esteri-
14-10. Which carries endogenous triglycerides? fied cholesterol is then incorporated into HDL. So for
VLDL (Very low densinty lipoprotein). excretion of cholesterol, LCAT is necessary.
14-11. What is the main apoprotein present in LDL? 14-25. What is the importance of PUFA in cholesterol
B-100; it is the ligand for LDL receptor. metabolism?
14-12. How LDL is taken up by the cells? PUFA present in lecithin is transferred to cholesterol
When the apo-B-100 binds to the receptor, the recep- by the enzyme LCAT. The esterified cholesterol is then
tor-LDL complex is internalised by endocytosis. taken by HDL, and finally excreted through liver. So,
14-13. What is the function of LDL? for excretion of cholesterol PUFA is required. Thus,
Transport of cholesterol from liver to peripheral tis- PUFA will lower the blood level of cholesterol.
sues. 14-26. Free fatty acids of plasma are bound to what?
14-14. What is "bad cholesterol"? Bound to serum albumin.
LDL cholesterol. 14-27. Triglycerides present in adipose tissue are hydro-
14-15. Why is it called so? lyzed by what enzyme?
LDL transports cholesterol from liver to peripheral Hormone sensitive lipase.
tissues, where it is deposited, and causes athero- 14-28. Bile acids are derived from what substance?
sclerosis. From cholesterol.
_ _ _ __Chapter 15
Hyperlipidemias and
Cardiovascular Diseases

Chapter at a Glance
The learner wi ll be able t o answer questions on the following topics:
0 Hyperlipidemias O Prevention of Atherosclerosis
0 Atherosclerosis and Coronary Artery Disease O Hypolipoproteinemias
\
0 Risk Factors for Coronary Artery Disease

I CLINICAL SIGNIFICANCE
OF CHOLESTEROL
coronary artery disease (CAD). It is estimated that by
the year 2020, it will account for 33% of all deaths.

Students should be familiar with cholesterol and lipo- Atherosclerosis and LDL \flc.e..
proteins described in detail in Chapter 14. A summary Stage I: Formation of Foam Cells
of lipoproteins is given in Table 15.1 and their metabolic
relati nships are shown in_Eigure---4 5:1 . is said to Increased levels of cholesterol for prolonged periods
be "b~ tmles erol an HD is ,,._"goo cholesterol (Fig. will favor deposits in the ~ ntimal reg.ion of a@ries.
15.2). High cholesterol leve s are associated with athero- Aorta, coronary arteries and cerebral vessels are pre-
sclerosis. Abnormality of cholesterol metabolism may dominantly affected by the atherosclerotic process. The
lead to cardiovascular accidents and heart attacks. LDL cholesterol, especially oxidized LDL particles are
deposited in the walls of arteries. Plasma LDL is mainly
catab · d via apo-B-LDL receptor pathway. But a small
p~a~rt~O!!.JJ~Rfil!~~~~l!a!l~.Clt.-~~~~~ake
Greek word, scleros· b:t.., macrapbaqFjls. Fr idative
obstructjon and m')(aimdia.Liofa[;~ damage of LDL will accelerate this process. Later, the
macrophages become overloaded with cholesterol, and

TABLE 15.1: Characteristics of different classes of llpoprote1ns


Chy/omicron VLDL LDL HDL
Density g/ L <0.95 0.95- 1.006 1.019- 1.063 1.063- 1.121
Diameter (nm) 500 70 25 15
Electrophoretic mobility origin p re- beta beta alpha
Apoproteins A, 8-48, C-11, E 8-100, C-11, E 8-100 A-1, C, E
Transport function TAG from gut to muscle and TAG from liver to muscle Cholesterol from liver to Cholesterol from
adipose tissue peripheral tissues peripheral tissues to liver
Chapter 15: Hyperlipidemias and Cardiovascular Diseases 229

Intestine Liver

l•••Chylomicron
remnants

••• Blood e •
••
t
LDL
HDL lt • Liver -

"Good"
VLDL- LDL

l
cholesterol
C hylomicrons vessel • • • {Excretion)
\ HDL .,__- - - - -

,,
Blood
vessel
Heart

Fig. 15.2: Forward and reverse transport of cholesterol


Peripheral tissues Peripheral tissues

Fig. 15.1: Summary of lipoprotein metabolism

Thrombus formed . ----.i"'-1.


Fig. 15.3: Left, cut section of normal artery; middle, early plaque in the artery
formation; right, advanced plaque formation
Infarction of muscles - -- -
supplied by the artery
these are then called "foam cells". These form the
fu. Release of troponins
{§llmark of atherosclerotic plaques. ) ® ~ tt>~~.__
in_to_g_e_ne_ra_l_c_irc_u_la_tio
_n_ __ _ _ _ __ _ _ _ _
. . Fig. 15.4: Thrombosis in the artery leads to infarction of the area
Stage II: Progression of Atherosclerosts supplied by the artery

Smooth muscle cells containing lipid droplets are seen


in the !esjQD. During early stages of atherosclerosis, the narrow lumenk , r ~ and there is tendency
condition is reversible if plasma lipid levels, especially for clot formation. '---'->--"
LDL-cholesterol levels are lowered. But wben ligid ,....;, \<Te..ol.\-
accumulates, the le · s unchecked and the Myocardial Infarction (Ml)
arterial changes becom reversi e. ~ t \- Finally, a clot is formed which occh,tde§_one of the major
_.;;, ,-.:,v- \
v~s.(rhrombosis)n ggronary arteryJeads to is~he-
Stage Ill: Fibrous Proliferation mia of the tissue supplied, due to hindrance to oxygen
Due to liberation of varie~~JJrowth factors by ma- supply (Fig. 15.3)(Qlt1mately infarction (deat~ of tis-
crophages and platelets c'otra~e<tls
~mulated. Thus sue) occurs (Fig. 15.4). Result is~ nefficient eootractig,,n
there is a definite co on inflammation in athe- of heart muscle, and if allowed to progress further, death
rosclerosis. This ch oni~ eads to in~ re d of the muscle cells in the affected region. Usually the
plasma high sensitive C-reactive protein (hs-CR .1' diagnosis can be made from the clinical hir!1iry, the
..?l~ el~ ocardiogram and cardiac markers (troponin T,
Stage JV: Advancing Fibrous Plaque C~ MB, etc, described in Chapter 6). Size of the infarct
This leads to narrowing of vessel wall when proliferative may be reduced by immediate administration of tissue
changes occur (Fig. 15.3). The blood flow through the plasminogen activator (t-PA).
(Z
230 Section B: General Metabolism

BOX 15.1: Clinical conditions 1n which serum cholesterol


level 1s increased 1.
: . hould check hp1d profile?
Suspected cardiovascular disease, coronary artery disease
1. Diabetes mellitus: Acetyl-CoA pool is Increased and more and peripheral vasculac disectse
'{>~ molecules are channeled t o cholesterol.
N o.DJ'\1

V, 2. Obstructive jaundice: The excretion of cholesterol through


2. All patients w ith diabetes mellitus, atleast once in 6 months
T,.\011"'\ l:M,; AaJl-
3. Thyroid, liver and rel'lar diseas~s. where lipid metabolism may
_ ode 1s blocked. be altered
t" , 3 tjypothyroidism: The receptors for HDL on liver cells are
4. All persons above 40, shou ld be checked once in a year.
decreased, and so excretion is not effective.
4. Nfphrotic syndrome:fibumin is lost th rough urine, globulins
(including lipoproteins) are increased as a compensatory In routine cli1nical practice only the first four para-
mechanism.
meters are measured in a fasting sample.
5. FQI!lilial hyperlipoproteinemias: See Fredrickson's classifica-
tion (Table 15.4) Box 15.1 shiows the conditions in which abnormal
levels of serum cholesterol are seen Box 15.2 gives the
indications for checking the lipid profile.
Class 1: Modifiable risk factors; Interventions have been
proved to lower CAD risk

2.
Cigarette smoking
High total cholesterol I RISK FACTORS FOR
ATHEROSCLEROSIS
@Y
--
3. High LDL cholesterol Risk factors for atherosclerosis and future myocardial
4. Low HDL cholesterol
infarction (Ml) am shown Box 15.3. Out of these, the total
5. High fat/cholesterol diet
cholesterol, HDL. and LDL levels are the most impor-
Jr.' Left ventricular hypertrophy (LVH)
..;r. Thrombogenic factors , tant indices (Box 15.4, item A). Cardiac markers, which
Class 2: Modifiable risk factors; Interventions are likely to indicate the presence of acute myocardial infarction
lower CAD risk are listed in Box 15.4, item B.
....a. Upoprotein (a) or Lp(a)
$. Diabetes mellitus
.,ffi. Hypertension
Serum Choliesterol Level
-.)1 . Physical inactivity
Framingham epidemiological study demonstrated that
_.)-2. Obesity
increase in serum cholesterol level is associated with
13. High triglycerides C) , as c..reasecLcislWlL..deatb-fr om-GH . For every 10%
14. High homocysteine ./1 C. -
15. Increased high-sensitivity-CRP (hs-CRP) 'yn lowering of cholester ( c!HD mortality).,as reduced by
J8'. Stress 13%_.,•~-in healthy persons, cholesterol level varies from
Class 3: Nonmodlflable risk factors 150 to 200 mg/dl (Table 15.2). If other risk factors are
17. Age pre e o st,~rol level should b~. t eferably
18. Male gender b,l_o w 180 ..._ml)/~µ. Values a~i'.? 22ij m will have
19. Family history of CAD m o ~ n d valu 40 m dl will need
active treatmen~;
sterol which affoirds protection against atherosclerosis.
~ LASMA LIPID PROFILE Plasma cholesterol levels would tend to slowly rise after
The sample of serum should be taken after 12 hours of the 4th decade of life in men and postmenopausal women.
fasting . A complete lipid profile is assessed by estimat-

1. Total cholesterol
2. HDL-cholesterol
t
ing the following parameters in plasma/serum.
(- ,,.,\-n CX-- ~
1·r ·· -, -
Ir,, tz1"4-.\
LDL-Choles1:erol Level
National Cholesterol Education Program (NC~ P) identi-
fied elevated LDL--C as a primary risk factorforCH D. Blood
3. LDL-cholesterol levels less than ·130 mg/dl are desirable (Table 15.2).
4. Triglycerides } <. Levels between 130 and 159 are borderline; while above
5. Apo-B level 1 160 mg/dl carry definite risk. Hence LDL is "bad" chole-
6. Apo-A-I level f 6c:;...(;.va..... 'o~ sterol (Fig. 15.2). Ox~ edD)Uinitiates fatty streaks,
7. Lp(a) level ' K.i.H .. dJ.~c:_ \"\f ·· which is the starting p~
-
eroma formation.
---::::::....;.
*' ®\ ~gU<P
1 V.
•,r0j,c.o..~'vP\...., A c..U.'ce.. :e. "6'ful·
Chapter 15: Hyper/ipidemias and Cardio ascular Oise ses 231

Non-HDL cholesterol or Atherogenic cholesterol = (LDL+ VLDL+


risk risk
IDL+ Lpa). As per NCEP guidelines, t he value of non-HDLC is
Total c olesterol (mg/ dl) <200 200- 240 >240 Important for the risk evaluation.
LDL ctTtilesterol (mg/d l ) <130 130-160 >160 <130 (100-130) mg/ dl = Very little risk
HDL cholesterol (mg/dl) >60 35-60 <35 <160 (130- 160) mg/ dl = Borderline high
<190 (160-190) mg/ dl = High risk
Triglyceride (mg/dl ) <200 200-400 >400
>190 mg/di = Very high risk
Another way of expressing the risk is as follows:
LDL-C Non-HDL-C
A. Risk markers of cardiac disease (prediction) (described in Risk grade 1 < l 60mg/dl <1 90mg/ dl
this Chapter) Risk grade 2 <1 30 <160
1. Total cholesterol level in serum Risk grade 3 < 100 < 130
2. LDL cholesterol and Apo-B100 level
3. HDL cholesterol and Apo A 1 level
Non-HDL Cholesterol
4. Plasma hsCRP
5. Lp(a) level A value of more than 160 mg/dL carries pigb rjsk (Box
6. Serum triglycerides 15.5).
7. Blood HbA le
8. Serum homocysteine level High Sensitive C Reactive
B. Cardiac markers of acute coronary syndrome (SH Chapter 6) Protein (hsCRP)
1. Creatinine kinase (CK-MB)
2. Cardiac troponins (cTnT and cTnl) It is also called ultra~ nsitive CRP. It measures low levels
3. High sensitivit y t roponin of CRP {1-10 ng/dL). It is a marker for risk for athero-
4. BNP and NTproBNP sclero~is and is used as a predictor for future myocardial
infarction within the next 12 months. The hsCRP test
HDL-Cholesterol Level · cleaJly adds to the predictive value.
The HOL level(above 60 mgl<ii..)protects against heart Less than 1 mg/L (0. 1 mg/dL) is considered as low
disease. (Table 15.2). Hence, HDL is " good" cholesterol. risk and single measurement is sufficient. Levels between
A level below 40 mg/dL increases the risk of CAD. For 1-3 mg/L ace borderline, indicating some risk. Levels
every 1 mg/dL drop in HOL, the risk of heart disease more than_3 mg /L is having high risk for future Ml , and
rises 3%. If the ratio of total cholesterol/HOL is more will need active medical intervention. If the hsCRP value
than 3.5, it is dangerous. Similarly, LDL : HDL ratio more is more than 10 mg/L, it indicates significant a.9:!..te r;,hase
than 2.5 is also detrimental. reaction, and is not indicative of any cardiac pathology.
...-Thus, hsCRP is tested only when other inflammatory
Apoprotein Levels and Ratios (B~ AJ) conditions are ruled out. Atherosclerosis has an inflam-
matory component, which causes production of CRP in
Apo-A-I is a measure of HOL-cholesterol (good) and
apo-B measures LDL-cholesterol bad). Ratio of Apo-B: small quantities. This CRP binds selectively to LDL, acti-
vates complement, resulting in plaque formation. CRP is
Apo-A-I is the st reliable index. e ratiio rn. is very
good; the ratio 1.4 as th.e highest risk of card '{9scular further described in Chapter 26.

accidents.
Serum Triglyceride
Lp(a) Normal level is 50-150 mg/dl. Blood level more than
Lp{~ h~ rinolysis. Levels more than 30 mg/dL 150 mg/dl is injurious to health. ( AOO- J.t·OO ':'5 /d.L
increase the risk 3 times; and when increased Lp(a)
is a~ with jncre.asecU.DL, the risk is increased
Diabetes Mellitus J"i ~g') bcm:la-U~)
6 tim s. (S~ Lp(a) in Ch.apter 14). Nicotinic acid will Cardiovascular disease is respon~ for 80% of total
reauce serum Lp(a) level. mortality in diabetes. It is associated with an increase
wrct& ~¥;. 1M, ~ -
232 Section B: General Metabolism li~d.,y ,;.;g --'> A~\. t,O~ Cho\-. ~ o}h

in LDL, high TAG and low HDL levels. In the absence BOX 1S.6 : Treatment policy in high risk patients. Targets to be
,.of insulin, hormone-sensitive lipase is activated, more achieved
ee fatt ac· are formed, which are catabolized to 1. Reduce total cholesterol below 180 mg/ dl
produce ce - o These cannot be read.il,! utilized, as 2. Decrease LDL-cholesterol below 130 mg/ dl; In persons with
.£Pi. the a~ailability of oxaloacetate isceducect and citric acid documented CHD, the goal of therapy is to reduce LDL to

\l;;/ cycle is sluggish So ~E·c~ ,


is increased, and
below 100 mg/ dL. In general, lowering of LDL cholesterol by
1 mg/dl ( 1 mmol/ L) reduces the risk of cardiovascular disease
it is channeled to cholesterol sycv<esi? by25%
In diabetes, atherogenic LDL is increased while 3. Keep HDL-cholesterol above 40 mg/ di.
atheroprotective HDL is decreased. The glycation and
oxidation of LDL will promote the uptake by macro-
/ phages. At the same time, the level of HDL in diabetic
IPREVENTION OF @ _:,
patients and those with metabolic syndrome is low. Gly- ~ THEROSCLEROSIS - - - -
cation of Apo-A-I decreases its ability to stimulate LCAT, Almost 90% of CAD is predictable, preventable and
and thereby the esterification and efflux of cholesterol curab~ Lifestyle chaflj9is are requ~i d, which include

1
from the cells. reg_l:!lc!~ xercise, balarfced die es~ ion of smoking,
Smoking __., )',vl\tJ.n. m;int~ ~ roper weight, cdhlr:o1 o.Lh.yQfilte.o_sion,
di ~ ~ dysliQjg,emia. The aim is to reduce total
Cigarette smoking is the most important modifiable risk cholesterol below 180 mg/dl ; to decrease LDL-cho-
factor for CAD Box 15.3). Risk from smoking is dose- lesterol below 130 mg/di and to keep HDL-cholesterol
dependant; ~e on t a~at which the person above 35 mg/dl (Box 15.6).
started smoking an umber4 of cigarettes smoked
Reduce Dietary Cholesterol

--
per day. Smoking enhances oxidation of LDL, reduces
HDL, increases CRP and augments a regation and
Cholesterol in the diet should be kept less than 200 mg
per day. ~ s and ~ at contain high cholesterol. One
egg yolk contains about 500 mg of cholesterol (Fig.
15.5A). One double omelet increases the blood choles-
terol § m~ th.~n the original level.
Hypertension f'M=--
Vegetable Oils a~ PUFA B
Systolic blood pressure more than 160 further increa-
ses the risk of CAD. An-.kl~asaJo 10 mm oL will Vegetable oils (e.g. stJOfiower oil) and fi ails contain
rfil!LJ.C..e l.ife~ oectanc'l by 10. y~ s. n~ ea.s.e..oc1f-5 mm Hg polyunsaturated fatty acids (PUFA). They are required
of diastolic pressure is associated with 34% increase in for the esterification and final excretion of cholesterol.
stroke risk. - ' -;r...<t:. So PUFA is helpful to reduce cholesterol level in blood
(Fig. 15.5B)~ mega-3 fatty acid;;from fish oils reduce
Obesity and Sedentary LDL and decrease the risk of CAD. i""tc_;_:--:,,"Tj-r":·r- ded
The classical description of Pickwick (in Pickwick papers) intake of omega-3 fa acid fish oils) is PA
by Charles Dickens reminds of a person with high risk and DHA combined). Clinical studie ave suggested
for CAD. People with "apple type" of obesity or truncal tha~ct.,ocosahexenoic aci ) P
obesity are more prone to get myocardial infarction. Aper- enoi c iicici'ffower · · , w e ii
son is obese when BMI exceeds 27.8 kg/m 2 in men and r~ u e s ; as well as reduce the risk..Qf.h_eart
-,.. - l '~ attack.
27.3 kg/m2 in.. women. ~ ~ifUauses gl~ e ilJ!2Ie-
rance, insulin-~ sistance, ~ pertJ-rlsion and d'/Sfioidemia.
Adipose tissue releases a large number of bioactive Moderation in Fat Intake
mediators that influence insulin resistance leading to The accepted standard is t h a t ~ oUotal calwies
endothelial dysfunction and atherosclerosis. A summary may be obtained from fat, out of which about one-third
of adipose tissue function is given in Chapter 35.

;i.01- --,
c
from saturate-a, another one-third from monounsaturated
v3, _., ~
Y; --
'b•

\..._., v 3 - Pv P~
Chapter 15: Hyperlipicfemias and Cardiovascular Diseases 233

Figs. 15.5A and B: (A) Reduce dietary cholesterol by avoiding Fig. 15.6: Green leafy vege- Fig. 15.7: Avoid cigarettes
egg omelet; (B) Sunflower oil and other vegetable oils contain tables are very 901Jd for heart.
PUFA

and the rest one-third from polyunsaturated fatt1~ids.


The recommended daily allowance will be about 'if>~~
BOX 15.7 : Plan
action
. -....
"' I '"' .. I I .. I

of oils and about_J-3 g_gf PUFA per day for a normal Plant derived fiber: Reduces serum cholesterol
adult. - '5 1. no..p:rn Legumes: Reduces cholesterol even on high fat diet
'
Onion and garlic: Beduces serum cholesterol and triacylglycerol
Embelia Ribes (Vidanga): Dried berries alone or along with amla
Green Leafy Vegetables has hypolipidemiic effect _,
Due to their high fiber content, leafy vegetables will Cammiphora Muku/ (Guggulu): Hypolipidemic and cardiopro-
tective
increase the motility of bowels and reduce reabsorption
Cyperus Rotundus (Musto): Hypolipidemic; improves metabolic
of bile salts Fi . 1~ tables also contain plant activity
sterols @tostero which decrease the absorption of Spices, flavanoids, red wine: Natural antioxidants prevent oxida-
cholesterol. A out 400 B'day of fruit and vegetables are tive modification of LDL
desirable. R.<;.

( Avoid Sucrose and Cigarette cholesterol will be converted to bile acids, thus
reducing the cholesterol level.
Cigarett~ oking Fig. 15.7) is t s1ngle most im iv. is widely used to ~ ')t!Gs,Jo.uDa-
tant modifiable risk factor for CAD (Box 15. c os tion, because of its anti-platelet activity (see Chap-
will rais£ plasma TAGs. High ca ohydrate diet, e'sr/e- ter 16).
cially sucrose, should be avoided by patients with 1
v. Anti-oxidanits such as • vitamin E · wilLroini.rn.Lz.e
hypercholesterolemia. oxi~ ~ and so, atheroscl;rosis may be
reduced.
Exercise vi. Plant deriv,ad·products having cholesterol-lowering
Regular moderate ex~~ @ min per day) will lower action are enumerated in Box 15. 7. The guggul
LDL (b~ lesterol~ raise Hpl (gqg(cholesterol) (resins) from the Mukul myrrh tree (Commiphora
levels in blood. It will also reduce obesity. Individuals Mukul) has cholesterol lowering action.
spending more than 2000 kcal/week in exercise are at
a lower risk. Avoid Trans Fatty Acids (TFA) ·
H • •d • D Trans fatty acids (with double bonds having trans con-
y po11P1; emic rugs \tt,j7 J figuration) are formed during the partial hydrogenation
i. HMG ~ A re9ucta~hibitors ("statins"): Ator- of vegetable oils. They are widely used in food industry
vastatin and ~ stalin are popular drugs in this because of their long shelf-life. Trans fatty acids (TFA)
group. They are effective in reducing the cholesterol are found to be, more atherogenic than saturated fattv
level and decreasing the incidence ?~AD. acids. It alters se& etion and composition of apo-8100
ii. Bi~ a}cid binding resins (Chol~fyramine and containing lipo~!f:Rteins (LD~ nd VLDL). It increases
Co · ipol) decrease the reabs.9.!B!ion of bile acids. catabolism of ~ -A-1, decr~ es HDL and increases
iii. Probuc I increases LDL catabolism and prevents u ® ev~ Reclucing the intake of TFA to 2- 7 g/day is
accumulatio ;of ~
. in arterial walls. So more now strongly advised .
( cm~o.6 , ome.~ o.. ,1
234 Section B: General Metabolism

TABLE 15.3: Class1ficat1on of hypoilpoproteinem,as


Disease Lipoproreins Cho/es terol Triacylglycerols Clinical findings
Familial hypobetalipoproteinemia LDL decreased Decreased Normal Decreased risk of coronary artery
B-100 decreased
Abetalipoproteinemia VLDLv; LDLH Markedly Decreased Malabsorption; mental and physical retardation;
B-4aJ,; a-1ooU decreased a,canthocytosis

Hypoalphalipoproteinemia HDLJ, Normal Normal In creased risk of coronary artery disease


M l.
Familial alpha HDL V Normal Normal Increased risk of CAD
Lp-deficiency A-IH

PUFA, in Excess, may be Harmful


r ~,
U\-~ ( f'l\\1-c.J.)
HYPOLIPOPROTEINEMIAS
PUFA can definitely reduce cholesterol level. But there AbetalipoprClteinemia
should be moderation. It is known that the diet should
All apo-B containing lipoproteins are reduced since
contain correct type and quantity; the optimum ratio of
microsomal triglyceride transfer protein is defective.
omega-,,6 to O,lJlega-3 fatty_~s is 4:1. Very high intake
Hence TAG is not incorporated into VLDL and chylomi-
of ~ a - o will cause lowering of HDL, elevation
crons. (Table 15,.3). Beta lipoprotein (LDL) is absent.
of plasma rigl rides, and will promote platelet aggre- Fat-soluble vitamins are not absorbed, causing mental
gation. Vegetable oils (sunflower oil) containing PUFA and physical retardation . Serum levels of triglycerides,
are rich in omega-6 variety; while ghee and butter are cholesterol and phospholipids are extremely low. Blind-
low in omega-6. i s - ~ in fish oils. ness may occur as a result of degenerative changes in
Normal Indian diet ~nsi¥g pulses and retina. Erythrocytes have spiny projections (acantho-
vegetables provides "invisible oils" , which contains
cytes).
about 10 g of PUFA/ day (out of which about 2 g is
omega-3 and the rest 8 g is omega-6). Further intake of Hypoalphalipoproteinemia
omega-6 oils is unnecessary and may be harmful. This is inherited as an autosomal dominant trait. Serum
The optimal ratio for omega-6 to omega-3 in diet is
HDL is decrease,d. There is increased risk for coronary
4:1. In an average Indian diet, this is about 30:1. In sun-
artery diseases (Table 15.3).
flower oil, this value is 160:1, and therefore, unnecessary
addition of such vegetable oils will further deteriorate the Tangier DisE~ase
condition. Although contains saturated fatty acids, coco-
It was first described in patients from Tangier Island in
nut oil has the omega ratio 3: 1, and therefore is superior
North-West Afric:a. It is a relatively benign, autosomal
to sunflower oil in this respect.
dominant condition. It is characterized by a defect in the
The general advice against the use of ghee and
efflux (flowing out) of cholesterol from cells, and reduc-
coconut oil needs re-evaluation. This misinformation
tion of HDL levels in the blood . The biochemical defect is
arose, when ~ong chain saturated fa acids LCSEA})
the absence of "ATP-Binding Cassette Transporter-1 " I
were shown to incre se cholesterol level. Since butter
(ABC-1 ), which is involved in transferring cellular cho-
~ lso contain saturated fatty acids. people
lesterol to HDL. So, plasma HDL is low and alpha band
equated them with LCSFA. Now it is known that ghee
is not seen in edectrophoresis. Cholesterol esters are
and coconut oil contain small chain (SCFA) and medium
chain fatty acids. The drastic differences in metabolisms accumulated in tissues. Manifestations are large orange
of LCFA and SCFA are given in Chapter 16. In sum- yellow tonsils, muscle atrophy, recurrent peripheral
neuropathies and atherosclerosis.
mary, ghee and coconut oil, within normal limits, neither
decrease nor increase cholesterol levels. But it is to be
noted that consumption of ghee (any oil in general), inc-
I HYPERLIPIDEMIAS
reases the total fat intake as well as calorie intake. That The most widely accepted Fredrickson's classification
is harmful. Again, moderation is the key. is shown in Table 15.4. In all cases of hyperlipidemias,
Chapter 15: Hyperlipidemias and Cardiovascular Diseases 235

-, • I I t .. It t t • I , creased)
Lipoprotein
fraction Cholesterol Appearance of
Type elevated level TAG/eve/ plasma after 24 hr Metabolic defect Features Management
Type I Chylomicrons N ii Creamy layer over Lipoprotein Eruptive Restriction of fat intake.
clear plasma lipase deficiency xanthoma; Supplementat ion w ith
hepatomegaly; medium chain triglycerides
Pain abdomen.
Type IIA LDL ii N Clear LDL Receptor Atherosclerosis, Low cholest erol diet.
defect; Apo-8 i coronary artery Decreased int ake o f
disease, Tuberous saturated fat. Give PUFA
xanthoma and drugs like statins.
Type 118 LDL andVLDL ii i Slightly cloudy Apo-8 t Corneal arcus Do
Apo-CII
Type Ill Broad beta- it i Cloudy Abnormal apo-E; Palmar xanthoma. Reduction of weight,
VLDL and Apo-CII"' High incidence of restrictio n of fat and chol.
Chylo microns vascular disease Give PUFA and drugs
Type IV VLDL t it Cloudy or m ilky Over-pro duction Associated with Reduction of body weight.
ofVLDL; Apo - diabetes, heart Restrict carbohydrate and
CII t disease, obesity. cholest erol
Type V VLDL N tt Creamy layer over Secondary t o Chronic High PUFA intake,
Chylomicrons milky plasma other causes pancreatitis hypocholipidemlc drug

triglycerides. They disappear when the lipid level falls.


Deposits of lipids in cornea lead to corneal arcus; indi-
cating hyper cholesterolemia.
Hyperlipidemias, in the order of highest to lowest
Anode Cathode
(+) (-)
incidence are Type IIA, IIB, IV, I, Ill and V.

Type IIA (Primary Familial


Type II A Hypercholeisterolemia)
There is elevation of LDL. Patients seldom survive the
Alpha Beta second decade· of life due to ischemic heart disease
Chylomicrons
Pre-Beta Application point (Table 15.4 and Fig. 15.8). The cause is LDL recep-
tor defect. Receptor deficiency in liver and peripheral
Fig. 15.8: Electrophoretic pattern of hyperlipidemias
tissues will result in the elevation of LDL levels in plas-
the elevated lipid fraction is either cholesterol or TAG ma, leading to hypercholesterolemia. The LDL receptor
or both. defect may be clue to the following reasons.
The elevation of lipids in plasma leads to the depo- 1. LDL receptor deficiency.
sition of cholesterol on the arterial walls, leading to 2. Defective binding of B-100 to the LDL receptors.
atherosclerosis. (See under coronary artery diseases). This defeclt is known as B-3500 or".familial defe-
The coronary and cerebral vessels are more commonly ctive apo 13.
affected. Thromboembolic episodes in these vessels 3. Receptor-L.DL complex is not internalized .
lead to ischemic heart disease and cerebrovascular Secondary type II hyperlipoproteinemia is seen in
accidents. hypothyroidism, diabetes mellitus, nephrotic syndrome
The deposition of lipids in subcutaneous tissue and cholestasis (Table 15.5).
leads to xanthomas . The type of xanthoma depends Salient features of other types of hyperlipoproteine-
on the nature of lipid deposited. Eruptive xanthomata mias are showin in Table 15.4. A summary of clinical
are small yellow nodules associated with deposition of classification of hyperlipidemias is shown in Box 15.8.
236 Section B: General Metabolism

TABLE 1s.s: Secondary hyperl1p1dem1as BOX 15.8: Clinical class1f1cat1on of hyperllp1derrnas (for treat-
Serum cholesterol Serum triglyceride ment purpose)
Diabetes Increased Increased Classification Type Salient features
Nephrotlc syndrome Increased Increased Hypercholesterolemia Type Ila Increased LDL
Hypothyroidism Increased Increased Hypertriglyceridemia Type I, Type IV and Increased VLDL
Biliary obstruction Increased Normal TypeV
Pregnancy Normal Increased Increased chylomicrons,
Alcoholism Normal Increased Combined hyperlipidemia Type lib and Type Ill Increased VLDL,
Oral contraceptives Normal Increased Increased IDL

!t
¥ Clinical Case Study 15.1
likely diagnosis? What biochemical shuttle may be active
to produce more adenosine triphosphate (ATP) per
glucose molecule?
A 48-year-old male presents to the clinic because of
concerns about heart disease.The patient reports chest 0
pain occasionally with ambulation around his house
• · Clinical Case Study 15.3
and is not able to climb stairs without significant chest A48-year-old male presented to OP with chest pain. Family
pain and shortness of breath. The physical examina- history shows that his father died of a heart attack at the
tion is normal, and the physician orders an electrocar- age of 46, and his elder brother also had a heart attack at
diogram (ECG), exercise stress test, and blood work. the same age. The patient reports that he gets chest pain
The patient's cholesterol result comes back as 350 mg/ occasionally with ambulation and is not able to climb stairs
dl (normal 200). The physician prescribes medication, without significant chest pain and shortness of breath. His
which he states is directed at the rate limiting step of plasma cholesterol level was 450 mg%. What is the pos-
cholesterol biosynthesis. sible diagnosis?
What is the rate-limiting step of cholesterol metabo-
lism? What is the class of medication prescribed? 0
• • Clinical Case Study 15.1 Answer
0
• · Clinical Case Study 15.2 Diagnosis: Hypercholesterolemia.
Rate-limiting step: The enzyme hydroxymethylglutaryl-
A 51-year-old male presents to the emergency center with
CoA reductase (HMG-CoA reductase) catalyzes an
chest pain. He states that he has had chest discomfort
early rate-limiting step in cholesterol biosynthesis.
or pressure intermittently over the last year especially
Likely medication: HMG-CoA reductase inhibitor,
with increased activity. He describes the chest pain as a
otherwise known as "statinn medications.
pressure behind his breastbone that spreads to the left
Clinical correlation: Hyperlipidemia is one of the most
side of his neck. Unlike previous episodes, he was lying
treatable risk factors of atherosclerotic vascular disease.
down, watching television. The chest pain lasted approxi-
In particular, the level of the low-density lipoprotein
mately 15 minutes then subsided on its own. He also noti- (LDL) correlates with the pathogenesis of atherosclero-
ced that he was nauseated and sweating during the pain sis. Exercise, dietary adjustments, and weight loss are
episode. He has no medical problems that he is aware the initial therapy of hyperlipidemia. If these are not suffi-
of and has not been to a physician for several years. On cient, then pharmacologic therapy is required. The exact
examination, he is in no acute distress with normal vital LDL targets depend on the patient's risk of cardiovascular
signs. His lungs were clear to auscultation bilaterally, and disease. For example, if an individual has had a car-
his heart had a regular rate and rhythm with no murmurs. diovascular event previously (heart attack or stroke),
An electrocardiogram (ECG) revealed ST segment eleva- the LDL target is 100 mg/dl ; 1 to 2 risk factors with-
tion and peaked T waves in leads II, Ill, and aVF. Serum out prior events = 130 mg/dl ; and no risk factors =
troponin I and T levels are elevated. What is the most 160 mg/dl.
Chapter 15: Hyperlipidemias and Cardiovascular Diseases 237

0 The condition may be homozygous (which is a rare


•ii• Clinical Case Study 15.2 Answer condition, with an incidence of 1 in 1 million) or heterozy-
gous (which is much more common, with an incidence of
Likely diagnosis: Acute myocardial infarction.
1 in 500 persons). Men are more prone to develop CAD
Clinical correlation: Patienrs symptoms in this case
than women. Symptoms appear later in heterozygotes.
are very typical of myocardial infarction , that is, chest
pressure or chest pain, often radiating to the neck or
to the left arm. The pain is usually described as deep
and "squeezing chest pain." Cardiac muscle is perfused 1. LDL carries cholesterol from the liver to the heart,
by coronary arteries with very little redundant or shared while HDL ,carries cholesterol from the heart to the
circulation; thus, occlusion of one coronary artery usu- liver.
ally leads to ischemia or necrosis of the corresponding 2. Hyperchole:sterolemia is seen in diabetes mellitus,
cardiac muscle. Laboratory confirmation of myocardial hypothyroidism, nephrotic syndrome, obstructive
infarction (death of cardiac muscle) includes ECG show- jaundice and in familial diseases.
ing elevation of the ST segment and/or increase of the 3. When LDL concentration in blood increases, cho-
cardiac enzymes. When there is insufficient oxygen lesterol gets deposited in the subintimal region of
available for the cardiac muscle, then the glycolytic path- arteries.
way must be used, which leads to a very small amount 4. Oxidized LDL (free radical damage) or glycated
of ATP per glucose molecule. LDL (hyperglycemia in T2DM) have more chances
of getting deposited. These modified LDL particles
0
• • Clinical Case Study 15.3 Answer are taken up by macrophages. Macrophages over-
·- loaded with cholesterol are called foam cells, which
The patient might be suffering from familial hypercho-
form the beiginning of an atherosclerotic plaque in
lesterolemia (FH). An LDL-C higher than 200 mg% in a
the arterial endothelium.
patient less than 20 years is suggestive of heterozygous
5. Macrophages and platelets release lymphokines
FH. It is an autosomal dominant condition where total cho- and growth factors, contributing an inflammatory
lesterol and LDL-C show severe elevation. Sometimes, it component, leading to raised hsCRP level.
is also a moderate elevation. It carries a risk premature 6. Clot formation and coronary artery thrombosis
CAD and hence early detection and treatment are impor- result in acute myocardial infarction.
tant. Exercise, dietary adjustments and weight loss are the 7. Plasma total cholesterol level should be 150-
initial steps, but if they fail drugs may be needed. 200 mg/dl, and preferably below 180 mg/dl.
FH is due to a defect in LDL receptor. LDL receptor 8. The HDL is atheroprotective cholesterol; it should
activity may be completely absent or up to 25% activity be more th.an 40 mg/dl for men and 45 mg/dl for
may be present. There are 3 types; in the first type LDL women. LDL-C values below 130 mg/dl are desir-
receptor is absent, in the second type there is mutation able. Desirable TAG level is less than 150 mg/dl.
in the terminal region so that binding is affected and in 9. Ratio between apo-B and Apo-A-I is a reliable
the third type, there is mutation in the C terminal region index, ideal being 0.4.
so that endocytosis is affected. Cholesterol synthesis 10. Lp(a) is highly atherogenic; values above than
continues even when plasma cholesterol is very high in 30 mg/dl predicting high risk.
these patients. 11 . Avoidable risk factors are smoking, sedentary life
In children with FH , typically cholesterol levels may style and overweight. Controlling of T2DM and
be above 600 mg%, and LDL-C may be 200 - 400 mg%. hypertension will also reduce the risk. Regular
Foam cell formation , plaque cell formation and prema- moderate exercise (walking for 1.5 to 2 km per day)
ture CAD are typical features. Cholesterol may accumu- is the basis of weight reduction.
late in other areas, leading to xanthelasma and variety 12. Not more tlhan 20% calories should be from fats.
of xanthomas. Corneal arcus and valvular abnormalities The fats should be proper mixtures of SFA, MUFA
are seen secondary to cholesterol deposition. and PUFA.
238 Section B: General Metabolism

13. Vegetable oils supply mainly omega-6 fatty acids, 18. Tangier disease due defective efflux of cholesterol
where as fish is rich in atheroprotective omega-3 from cells occurs when ABC-A1 protein is deficient,
fatty acids. The ideal ratio of omega-6 to omega-3 with cholesterol ester accumulating in tissues, e.g.
in the diet is 4:1. Orange tonsil.
14. When diet control and exercise do not bring the 19. Frederickson's classification is based on the type
cholesterol to desired levels, use of hypolipidemic of lipoprotein elevated. Six types have been descri-
drugs may be considered. bed. Most common is Type IIA, which is due to defec-
15. "Statins" that inhibit HMG-CoA reductase is the tive LDL receptor, leading to decreased uptake of LDL
most common drug prescribed to lower cholesterol 20. For purposes of management, a clinical classifica-
levels. In addition, antiplatelet agents like aspirin tion to 3 typ,es is used, (a) hypercholesterolemia,
are also given. In persons with significant hypertri- (b) hypertrigilyceridemia and (c) combined hyper-
glyceridemia, fibrates may be given. lipidemia.
16. Abetalipoproteinemia is due to a deficiency of TAG 21. Hypertriglyceridemia is seen in Type I where lipo-
transfer protein which interferes with the incorpo- protein lipase activity is deficient. In type IV and
ration of apo-B-100 and apo-B-48 into VLDL and Type V diseases, the TAG pool is increased .
chylomicrons. 22. Combined hyperiipidemia is seen in Type IIB, where
17. Hypoalphalipoproteinemia is due to low HDL level apo-B levels are high. In Type Ill disease, the apo-E
with high risk of CAD. deficiency causes defective hepatic uptake of IDL.

PART-1: ESSAY AND SHORT NOTE UESTIONS


15-1. What is the normal cholesterol level in plasma? What Is its clinical sig nificance? What are the dietary precau-
tions to reduce hypercholesterolemia?

SHORT NOTE QUESTION

15.2. HDL-cholesterol. 15-4. Preventio rn of atherosclerosis.


15-3. LDL-cholesterol. 15-5. Polyunsaturated fatty acids.

PART-2: MULTIPLE CHOICE QUE TIONS

15-1. HDL cholesterol is said to be "good" cholesterol, A. TAG level in blood is increased
because B. LDL level in blood is increased
A. HDL contains contains enzymes to break down C. LDL receptor defect is seen
cholesterol D. Cholesterol level in blood is increased
B. HDL carries cholesterol from liver to tissues where 15-4. Normal blc,od cholesterol level is:
it is broken down A. 40-60 rng/ 100 ml
C. HDL carries cholesterol from tissues to liver from B. 70-110 mg/100 ml
where cholesterol is excreted C. 120-150 mg/100 ml
D. HDL inhibits cholesterol synthesis D. 150-200 mg/100 ml
15-2. All the following statements are true with regard to 15-5. Hyperchol,esterolemia is seen in the following con-
type IIA hyperlipoproteinemia, except ditions, except:
A. Premature artherosclerosis A. DiabetEi s mellitus
B. Elevated plasma LDL cholesterol B. Thyrotoxicosis
C. Creamy layer on top of the serum C. Nephroitic syndrome
D. Xanthomata D. Obstructive jaundice
15-3. All are true in familial hyperlipoprotenemia Type IIA, 15-6. The characteristic finding in hypobeta-lipoprote-
except: inemia is:
Chapter 15: Hyperlipid'emias and Cardiovascular Diseases 239

A. Acanthocytosis 15-11 . Hyperchc,lesterolemia in a 15-year-old girl may be


B. Orange tonsils due to th1e following causes, except
C. Eruptive xanthomas A. Nephrotic syndrome
D. Corneal arcus B. Hypothyroidism
15-7. Which of the following dietary modifications have C. Type IV hyperlipoproteinenmia
minimum hypocholesterolemic action? D. ObstnJctive jaundice
A. Inclusion of fresh fruits and vegetables in diet 15-12. An obesE! person may have all the followi ng bio-
B. Dietary cholesterol intake less than 300mg/day chemical abnormalities, except
C. Intake of whole wheat bran A. Increased glucose tolerance
D. Consuming fish as the only nonvegetarian food. B. Hypertriglyceridemia
15-8. Premature atherogenesis will not set in when----- is C. Chro111ic respiratory acidosis
defective:
D. High plasma insulin levels
A. Cholesterol efflux regulatory protein 15-13. Which of the following is the most common type of
B. LDL receptor hyperlipic:lemia in Diabetes mellitus?
C. Cholesterol ester transfer protein A. Eleva1ted triglycerides with normal cholesterol
D. Apolipoprotein B B. Normal triglycerides with elevated cholesterol
15-9. Hypoalphalipoproteinemia is characteristic of C. Elevatted cholesterol and decreased triglycerides
A. Fish eye disease D. Elevatted cholesterol and triglycerides.
B. Bassen-Kornzweig syndrome 15-14. The Lipoprotein deficient In Tangier disease:
C. Floating beta disease A. HDL B. LDL
D. Tangier disease
C. Lp(a) D. IDL
15-10. A family gives a history of several members com-
15-15. Secondary hyperlipidemia occurs in all the follow-
pleting a century of healthy life. The cause maybe
all except: ing conditions, except
A. Sedentary lifestyle A. Hypothyroidism
B. Normal adipose tissue liver axis B. Alcoholism
C. Elevated HDL levels C. Chronic pancreatitis
D. Consumption of a fish based diet D. Nephrotic syndrome

ANSWERS OF MULTIPLE CHOICE UESTIONS


15-1 . C 15-2. C 15-3. A 15-4. D 15-5. B 15-6. A 15-7. C
15-8. C 15-9. D 15-10. A 15-11 . C 15-1.2. A 15-13. B 15-14. A
15-15. C

PART-3: VIVA VOCE QUESTIONS AN ANSWERS

15-1 . What are the salient features of hyperlipoproteine- 15-5. What are other risk factors associated with coro-
mia Type IIA? nary artery diseases?
Premature atherosclerosis; Elevated plasma LDL cho-
Cigarette smoking, hypertension, diabetes mellitus,
lesterol; Prominent beta band on electrophoresis.
serum trig1lyceride level above 200 mg/di; homocyst-
15-2. What is it due to?
Defect in LDL receptor. eine level; sedentary life style, obesity.
15-3. Hypercholesterolemia is seen in what conditions? 15-6. What advise you will give to a person with
Diabetes mellitus; Nephrotic syndrome; Obstructive increased! cholesterol level?
jaundice; Hypothyroidism. Reduce cholesterol content of food; include PUFA and
15-4. What are the important risk factors of coronary
omega-3 fatty acids in diet; reduction of total fat intake;
artery diseases?
increase gIreen leafy vegetables; exercise.
Serum cholesterol level above 200 mg/di; LDL-cho-
lesterol level above 160 mg/di; HDL-cholesterol level
below 35 mg/di; Lp(a) above 30 mg/di.
_ _ _ _ _Chapter 16
MCFA, PUFA, Prostaglandins
and Compound !Lipids

Chapter at a Glance
The learner will be able to answer questions on the following topics:
0 Medium chain fatty acids (MCFA) 0 Leukotrienes
0 Monounsaturated fatty acids (MUFA) 0 Very long chain fatty acids (VLCFA)
0 Polyunsaturated fatty acids (PUFA) 0 Glycerophosphatides
0 Saturation and desaturation of fatty acids 0 Phosphatidylcholine synthesis
0 Essential fatty acids 0 Sphingolipid .and sphingomyelin synthesis
0 Eicosanoids 0 Lipid storage diseases
0 Prostaglandins

Fatty acids having carbon atoms 4 to 6 are called small catalyzes the complete hydrolysis of SCT/MCT into
chain fatty acids (SCFA); those with 8 to 14 carbon atoms glycerol and small/medium chain fatty acids. These free
are known as medium chain fatty acids (MCFA); those MCFAs diffuse directly into portal circulation. SCFA
with 16 to 18 carbon atoms are long chain fatty acids and MCFA are preferentially oxidized by peripheral
(LCFA); and those carrying 20 or more carbon atoms cells, and so they are not deposited in adipose tissues.
are named as very long chain fatty acids (VLCFA) A comparison is uiven in Table 16.1 .
(see Table 8.3).
The SCFAs, butyric acid (4C) and caproic acid (6C) Very Long Chain Fatty Acids (VLCFA)
are seen in butter and ghee; the name butyric is derived
Fatty acids having 20 or more carbon atoms are called
from butter. MCFAs, capric acid (10C), lauric acid (12C)
very long chain fatty acids (VLCFA). Eicosapentaenoic
and myristic acid (14C) are present in coconut oil and
acid (EPA) (C-20, 5 double bonds) and docosahexaenoic
human milk. Normal skin has a very thin layer of sebum
acid (DHA) (C-2,!, 6 double bonds) are good examples
secretion with MCT (medium chain triglyceride) contain-
of VLCFA. DHA is synthesized in liver from linolenic acid
ing lauric acid, which prevents bacterial entry into the
(Omega-3, C-18, 3 double bond}. DHA is available in
body.
large quantities in fish oils. DHA is especially required for
the development of brain and retina. Low level of DHA
STION OF MEDIUM
in blood is seen in patients with retinitis pigmentosa.
N FATTY ACIDS- - - - In human beings, DHA accumulates in brain before
Digestion and metabolism of SCFAs and MCFAs are birth and for up to, 12 weeks afterwards. Outer segments
entirely different from those of LCFAs. Triglycerides con- of retinal rods contain high concentrations of DHA, which
taining small and medium chain fatty acids (SCT and MCT) gives high fluidity to the membranes. This is required
do not require prolonged digestion. Pancreatic lipase for the lateral anid rotational movement of rhodopsin
and bile salts are not required. MCT-specific lipase within the membrane during photoactivation.
Chapter 16: MCFA, PUFA, Prostaglandins and Compound Lipids 241

Examples Butyric acid (C4) in butter and !auric acid (Cl 2) in Palnnitic acid (C16) and stearic acid (C18) in
coconut oil veg,etable oils and animal fats
Di estion in stomach Hydrolyzed Not hydrolyzed
Pancreatic lipase Not necessary Esse:ntial
Bile salts Not necessary Abs,olutely essential
Inside intestinal cells TAG is hydro I ed to form fatty acids Free• fatty acids are re-esterified to form TAG
Absorbed Directly to blood To li1mphatics, then to thoracic duct
Absorbed as Free fatty acid carried by albumin TAG, carried by chylomicrons
Immediate fate Oxidized by peripheral cells Deposited in the adipose tissue
Carnitine Not required for oxidation Required for oxidation
Clinical application No effect on atherosclerosis Hypercholesterolemia and atherosclerosis

VLCFAs (20C and above) are partly oxidized in (1st step in Fig.13.9) is not needed. Thus in the case of
peroxisomes to smaller (18C) fatty acids, which then unsaturated fatty acids, the energy yield is less by 1.5
leave peroxisomes to enter mitochondria. This peroxi- ATP molecules per double bond.
somal oxidation differs from beta-oxidation in that the
electrons from FADH 2 (step 1 of beta-oxidation) are directly I POLYUNSATURATED
donated to oxygen to form hydrogen peroxide. So this
step does not produce ATP.
lEMJy ACIDS (PUFA}_ _
Deficient oxidation of VLCFA by peroxisomal The important poly unsaturated fatty acids are:
enzyme systems leads to adrenoleukodystrophy (see 1. Linoleic acid (18C, 2 double bonds)
Chapter 2), where VLCFAs are accumulated and myelin 2. Linolenic acid (18C, 3 double bonds)
sheaths are destroyed. It is an X-linked condition. The 3. Arachidonic acid (20C, 4 double bonds)
child usually dies during the first decade of life. They are pn:!sent in significant quantities in veget-
able oils such as sunflower oil (see Fig. 8.2 and Table 8.4 ).
I MONOUNSATURATED They are used to esterify cholesterol, whereby the latter

lEAI:rY
ACIDS (MUFA
can be excreted. So, PUFA in general are antiathero-
genic (see Chapters 14 and 15). Functions of PUFA are
Palmitoleic (C16, 1 double bond) and oleic (C18, 1 double shown in Box 16.1. Clinical manifestations of PUFA/ EFA
bond) acids are present in human body fat, as well as deficiency are shown in Box 16.2.
many vegetable oils. Erucic acid (C22, 1 double bond) is Fish oils contain PUFAs with 5 or 6 double bonds.
a constituent of mustard oil and rapeseed oil. Nervonic They are important for development of human brain.
acid (C24, 1 double bond) is present in substantial
quantities in brain. Lipid Peroxiidation
In vitro, peroxidation would lead to rancidity of fats and
Modified Beta-Oxidation of MUFA oils. In vivo the membrane lipids are more liable to attack
The oxidation of unsaturated fatty acids proceeds as in by free radicals ,and produce damage to the integrity of
the case of saturated fatty acids, till the double bond is the membrane (see Chapter 30). In naturally occurring
reached . Thus palmitoleic acid (16 C monounsaturated) lipids antioxidanits prevent lipid peroxidation. Vitamin E
undergoes 3 cycles of beta-oxidation to yield t.3-cis or tocopherol is an important antioxidant in the human
enoyl-CoA with 10 carbon atoms. Here the double body.
bond is cis type; the dehydrogenase cannot act on that
Elongation iof Fatty Acids
bond. Therefore, an isomerase changes the cis 63
double bond to 62-trans double bond. The double bond The Microsomal system (microsomal fatty acid
between 3rd and 4th carbon atoms is shifted to between elongase system) elongates saturated or unsaturated
2nd and 3rd carbon atoms. It will then undergo 2nd, 3rd fatty acyl-CoA by successive addition of two-carbon units.
and 4th step reactions of beta-oxidation (see Fig. 13.9). Malonyl-CoA is the donor of two carbon acetyl groups.
So in this cycle the FAD dependent dehydrogenation NADPH is required for the reaction. This system can
242 Section B: General Metabolism

BOX 16.1: S1gnif1c,mce of PUFA BOX 16.2: Clln1cal s1gnif1cance of PUFA and EFA
1. PUFAs are seen in vegetable oils. 1. Persons with normal diet w ill not have any deficiency; but
2. Linoleic and linolenic acids are nutritionally essential; and are t hose who are on parenteral nutrition for long periods will have
called Essential Fatty Acids (EFA). deficiency.
3. Prostaglandins, thromboxanes and leukotrlenes are produced 2. PUFAs are used for esterification and excretion of cholesterol.
from arachidonic acid. PUFA will reduce serum cholesterol level (see Chapter 15)
4. PUFAs form integral part of mitochondrial membranes. In defi- 3. Deficiency of EFA causes acanthocytosis, hyperkeratosis,
ciency of PUFA, the efficiency of biological oxidation is reduced. acrodermatitis and hypercholesterolemia.
5. They are components of membranes. Arachidonic acid is 4. EFA deficiency is connected with acrodermatitis enteropathica,
10-15% of the fatty acids of membranes. hepatorenal syndrome and CNS manifestations.
6 . As double bonds are in cis configuration; the PUFA molecules 5. Elevated PUFA levels are seen in Zellweger's syndrome.
cannot be closely packed. So PUFAs will increase the fluidity 6. DHA levels in blood are low in patients with retinitis pigmentosa.
of the membrane. 7. Trans fatty acids will compete with EFAs, and may increase the
7. As PUFAs are easily liable to undergo peroxidation, the EFA deficiency and decrease fluidity of membranes.
membranes containing PUFAs are more prone for damage by 8. Trans fatty acids decrease HDL-cholesterol and may cause
free radicals. atherosclerosis.
8. The production of DHA (docosahexaenoic acid) is synthesized
to a limited extent from alpha linolenic acid. DHA is present in
fish oils. DHA is present in high concentrations in retina, cerebral Essential Fatty Acids (EFA)
cortex and sperm s.
Normal dietary allowance of PUFA is 2-3% of total
calories. Linoleic acid and linolenic acid are the only
NADH + H+ NAO+
y-linolenoyl-CoA
fatty acids which cannot be synthesized in the body.
Linoleoyl-CoA
- (18C , A6, 9, 12) Their structures are shown in Figure 8.2. They have to be
(18C, 9 • 12 ) 6 6 desaturase (Linolenic)
(Linole,c) provided in the food; hence they are essential fatty acids.
02 2Hp Arachidonic acid can be formed, if the dietary supply of

t
NADPH NADP+ linoleic acid is sufficient. For deficiency manifestations
Homogamma- of EFA, (see Box 16.2). The relation of PUFAwith chole-
y-linolenoyl-CoA - linolenoyl-CoA
Elongase (20C , 6 8, 11, 14) sterol is described in Chapters 14 and 15.

H
___ __ ___ Malonyl-CoA ___ CO2 ________ _

02 2H 20 Gamma-Linolenic Acid (GLA)


Homogamma Arachidonoyl-CoA
linoleoyl-CoA - (20C, AS, 8, 11, 14) It is an essential fatty acid of the omega-6 family. In
65 desaturase
the body, GLA is produced from linoleic acid. GLA is
NADH + H+ NAO+
desaturated to arachidonic acid (AA). GLA may prevent
Fig. 16.1: Desaturation and elongation of linoleic acid to arachi- cardiovascular diseases by preventing atherosclerosis.
donic acid
Dietary sources of GLA are plantseed oils. GLA is also
elongate fatty acids having 10 carbon units onwards up found in human milk.
to the length of 22 or 24 carbons. The steps in elongation
Clinical Significance of Omega-3,
are similar to de novo synthesis (see Chapter 13).
and Omega-6 PUFAs
Desaturation of Fatty Acids The term omega refers to the terminal carbon atom far-
Monounsaturated fatty acids can be synthesized from thest from the functional carboxylic acid group (-COOH).
saturated fatty acids by a 69 desaturase enzyme system Thus, an omega-3 fatty acid has a site of unsaturation
present in the endoplasmic reticulum. The reaction between the third and fourth carbons from the omega
utilizes NADH and molecular 0 2 and cytochrome b5. end. There are three major types of omega-3 fatty acids
Thus stearic acid is desaturated to form oleic acid. in foods; these are ALA (alpha linolenic acid), eicosa penta-
The introduction of double bonds is possible only enoic acid (EPA), and docosa hexaenoic acid (OHA).
between an existing double bond and carboxyl end of The body converts ALA to EPA and then to DHA.
the fatty acid (but not between the omega end and an Most of the ALA consumed in the diet comes from
existing double bond}. Hence, linoleic acid cannot be fruits and nuts. The highest concentrations of EPA and
synthesized from oleic acid. However, linoleic acid can DHA are found in cold water fishes such as salmon, tuna,
be converted to arachidonic acid by elongation and and herring. The most important PUFAs, biologically, are
desaturation (Fig. 16.1). EPA and DHA.
Chapter 16: MCFA, PUFA , Prostaglandins and Compound Lipids 243

(A)
,,c:,9

11
7

13
5 3

20

OH
John R Vane
NP 1982
1927-2004
Ulf von Euler
NP 1970
1905-1983
Bengt
Samue/sson
NP 1982
b. 1934
(B) h 20
OH OH
Most of the omega-6 PUFAs consumed in the diet
9
are from vegetable oils such as soybean oil, corn oil,
(6
J:::
etc. Linoleic acid is converted to arachidonic acid (see (C)
Fig. 16.1 ). When gamma-linolenic acid (GLA) is available 20
'12
in food, GLA is converted to arachidonic acid.
Omega-3 and omega-6 fatty acids are incorporated
into cell membranes. These membrane lipids serve Figs. 16.2A to C: (A) Prostanoic acid; (B) Prostanglandin-F2;
(C) Thromboxane A2
as precursors for the synthesis of important signaling
molecules involved in cell growth and inflammation.
The most important omega-6 PUFA is arachidonic awarded to Beni~t Samuelsson (biosynthesis of PGs)
acid. On stimulation of the cell, arachidonic acid is and Sir John Vane (effect of aspirin on PGs). PGs were
released from cell membranes through the action of phos- originally isolated from prostate tissue and hence the
pholipase A 2 (PLAJ The released arachidonate then name. But they are present in almost all tissues. They
serves as the precursor for the synthesis of the prosta- are the most potent biologically active substances;
glandins (PGs), thromboxanes (TXs), and leukotrienes. as low as one n!~/ml of PG will cause smooth muscle
(l Ts). These eicosanoids will cause platelet and leuko- contraction. The diverse physiological roles of prosta-
cyte activation , signaling of pain, induction of broncho- glandins confer on them the status of local hormones.
constriction, and regulation of gastric secretions. These
activities are targets of nonsteroidal anti-inflammatory Chemical Stiructure
drugs (NSAIDs), COX-2 inhibitors, and leukotriene All prostaglandins are considered to be derived from
antagonists.
the 20C cyclic saturated fatty acid, prostanoic acid (Fig.
Dietary omega-3 PUFAs compete with the pharma-
16.2A). The five carbon ring is saturated. All naturally
cological activities of omega-6 PUFAs because they
occurring PGs have an alpha-oriented OH group at
displace arachidonic acid from cell membranes. Increas-
C15.
ing dietary consumption of omega-3 PUFAs will reduce
the activity of leukocytes and platelets.
Classification of Prostaglandins
I EICOSANOIDS According to thei attachment of different substituent
groups to the rin!g, PGs are named with capital letters
They are 20C compounds (Greek, eikosi = twenty),
such as A, B, E and F (Table 16.2). PGF is designated as
derived from arachidonic acid . Their names are:
alpha to denote the projection of the OH group in natu-
1. Prostanoids, containing:
rally occurring prostaglandins.
a. Prostaglandins (PGs);
In the same series, depending on number of double
b. Prostacyclins (PGls);
bonds on the side chains they are denoted by a sub-
c. Thromboxanes (TXs)
script after the capital letter, e.g. PGE 1, PGE2 , PGE3 , etc.
2. Leukotrienes (LTs)
(Table 16.2).

IPROSTAGLANDINS The primary prostaglandins PGG and PGH, (the endo-


peroxides) are inttermediates in the synthesis of others.
Prostaglandins (PGs) were first isolated by Ulf van Euler Only 5 PGs are widely distributed in the body. They
in 1935 (Nobel Prize in 1970). In 1982, Nobel Prizes were are PGD2 , PGE2 , PGF2 and PGl 2 and thromboxane Az.
244 Section B: General Metabolism

Table 16.2: Salient features of prostagland1ns Membrane bound phospholipid


Name
PGA
Substituent groups
Keto group at C9; double bond Cl0 and 11 I._ Phospho- ~
lipase ~
Co_rtisol (:-)
Epinephrine (+)
PGB Keto group at C9; double bond CB and 12 Thrombin (+)
Arachidonic acid

l Aspirin H
PGD OH group at C9; keto group at Cl 1 Cyclo- ~
PGE Keto group at C9; OH group at Cl 1 oxygenase lndomethacin H
PGF OH groups at C9 and Cl 1 Prostaglandin G2
IPeroxidase
i
PGG Two oxygen atoms, interconnected to each other, and
bonded at C9 and Cl 1; hydroperoxide group at Cl 5 Prostacyclin Thromboxane
PGH Same ring as PGG; but C15 has OH group
syntha,se PG]; : y; hase T><Ai
PGI Double ring. Oxygen attached to C6 and C9, to form
another 5-membered ring. Hence called prostacyclin. o ,..a.. o'>-'
,-.Y(!I 't>;

7 PGD, -~
".;\ &,<
Reductase & '
Structure of PGF2 is shown in Figure 16.28. Throm-
boxanes have 6 membered oxane ring (Fig.16.2C).
PG\+--Pr --+ looci"'• PGs
Biosynthesis of Prostaglandins Adenyl cyclase
Prostaglandins are derived from the following PUFA. ATP - - - - - • Cyclic AMP - + Biological effects I I
1 series (1 double bond)-from Linoleic acid Fig. 16.3: Synthesis and action of prostaglandins
2 series (2 double bonds)-from Arachidonic acid
3 series (3 double bonds)-Eicosapentaenoic acid
BOX 16.3: Mi>chan1sm of c1ct1on of ilsp1r111
Naturally occurring PGs belong to the 2 series. Aspirin irreversibly acetylates and inhibits cyclo-oxygenase.
i. PGs are not stored as such; the precursor fatty Platelets cannot regenerate cyclo-oxygenase and so thromboxane A2
acids are present in membrane as phospholipids. is not formed in platelets. Hence, there is decreased platelet aggre-
gation. Therefore, aspirin is useful in prevention of heart attacks.
The arachidonic acid is released by the action of
By inhibiting cyclo-oxygenase, aspirin also reduces PGl2; but endo-
phospholipase A2 on phospholipids (Fig.16.3). thelial cells after a few hours will resynthesize cyclo-oxygenase. So
ii. Synthesis is catalyzed by prostaglandin H synthase aspirin completely blocks TXA2, but only partially inhibits PGl2. Other
(PGHS) . It contains two separate enzyme activities, anti-inflammatory drugs (indomethacin and ibuprofen) also cause
irreversible inhibition of enzyme. Paracetamol is a reversible Inhibitor.
cyclo-oxygenase and peroxidase.
iii. PGG 2 and PGH2 are formed as intermediates during
the synthesis of other PGs. Specific enzymes functions. The inducible form mediates the inflammatory
convert PGH2 to other prostaglandins (Fig. 16.3). response. Prostaglandins have only very short half-life,
of about 30 seconds.
Regulation of Synthesis
Mechanism of Action
The phospholipase (PL) is activated by epinephrine,
thrombin, angiotensin II, bradykinin and vasopressin. Prostaglandins are local hormones. They function
Steroids inhibit PL and prevent release of arachidonic through G-protein coupled receptors (see Chapter 45).
acid from membranes (Fig.16.3). Cyclo-oxygenase is In most tissues, PGE increases cAMP (cyclic AMP)
activated by catecholamines and inhibited by nonsteroid level. But in adipose tissue and in renal tubular cells,
anti-inflammatory drugs (NSAIDs). Aspirin acetylates PGE lowers cAMP level. PGI activates adenyl cyclase
serine at the active site and irreversibly inhibits the and TXA inhibits it (Fig.16.3).
cyclo-oxygenase (Box 16.3).
Cyclo-oxygenase is a "suicide" enzyme, self cataly- Biological Actions and
zed destruction rapidly inactivates the enzyme (see Clinical Applications
suicide inhibition in Chapter 5). This would prevent exces-
Effects on CVS
sive production of PGs. Cyclo-oxygenase exists in two
different forms. COX- 1, the constitutive form produces Prostacyclin or PGl 2 is synthesized by the vascular
prostaglandins, that mediate gastric, renal and platelet endothelium. Major effect is vasodilatation. It also
Chapter 16: MCFA, PUFA, Prostaglandins and Compound Lipids 245

Table 16.3: Proslacycl1ns and thromboxane Arachidonic acid

Structure
Site of formation
PG/,
Cyclopentane ring
Endothelium
TXA?
Oxane ring
Platelets
1'.~r,....,......
Cyclic AMP level Increased Decreased 5-hydro-peroxy-eicosa-tetra-enoic acid (5 HPETE)

l
Platelet aggregation
Blood vessel
Inhibited
Vasodilatation
Enhanced
Constriction
l''"' · " ' ' " ' \ ~
- H 2O
LTA synthase
Peroxidase

HETE

l•
Bronchioles Relaxation Constriction + Glutathione

inhibits platelet aggregation and has a protective


H,o LXA, Gl,tamioacid
effect on vessel wall against deposition of platelets. But
any injury to the vessel wall would inhibit PGl2 synthesis
Leukotriene B4 LTD4 "\. • LTE 4
so that platelet aggregation occurs to promote thrombus
formation. Glycine
Thromboxane (TXA2) is the main PG produced by
platelets. The major effects are vasoconstriction and Fig. 16.4 : Synthesis of leukotrienes. LT= leukolriene; LXA = lipoxin;
HETE = hydroxyeicosatetraenoic acid
platelet aggregation. Prostacyclin and thromboxane
are opposing in activity (Table 16.3). Prostaglandins lower
that there are 4 double bonds in the structure. The slow
the blood pressure.
reacting substance of anaphylaxis (SRS-A) contains
LTC4 , LTD4 and LTE4 • They cause smooth muscle
Effects on Ovary and Uterus
contraction, constrict the bronchioles and produce
The PGF2 stimulates the uterine muscles. Hence PGF2 vasoconstriction. SRS is the mediator of hypersensitivity
may be used for medical termination of pregnancy. reactions such as asthma. LTB4 is a potent inflammatory
Yet another use is in inducing labor and arresting molecule through its action on neutrophils.
postpartum hemorrhage. PGs are involved in LH induced
ovulation. In cattle, if PG is given, luteolysis takes place Lipoxins
and animal goes into estrus. Better fertilization rate is
They are a group of compounds produced by leukocytes.
achieved with timely artificial insemination .
LXA4 is the most common variety (Fig. 16.4 ). It is anti-
inflammatory and decreases immune response.
Effects on Respiratory Tract
The PGF is a constrictor of bronchial smooth muscle; 1.§_YNTHESIS OF COMPOUND LIPIDS
but PGE is a potent bronchodilator. PGE series are
used in aerosols for relieving bronchospasm. Structures of lecithin, cephalin, phosphatidyl inositol, plas-
malogen, sphingosine, ceramide and sphingomyelin are
Effects on Immunity and Inflammation shown in Chapter 8. Their metabolism is briefly descri-
The PGE2 and 0 2 produce inflammation by increasing bed below.
capillary permeability. Erythema and wheal are produ-
Synthesis of Glycerophosphatides
ced at the site of injury. The anti-inflammatory effect of
PG synthesis inhibitor (aspirin; cortisol) is explained in Phosphatidic acid is an important intermediate in the
Box 16.3. synthesis of phosphoglycerides as well as triacylglycerol.
The phosphatidic acid itself may be formed from
Leukotrienes (L Ts) glycerol-3-phosphate or dihydroxy acetone phosphate
They are produced from arachidonic acid. The pathway (Fig.13.15). The synthesis of glycerophospholipids can
is shown in Figure.16.4. LT 8 4 is produced in neutrophils; occur either by activation to CDP-choline and CDP-ethano-
it is the most potent chemotactic agent (factor attracting lamine or by formation of active diacylglycerol, CDP-
cells to the inflammatory site). The number 4 denotes diacylglycerol.
246 Section B: General Metabolism

1-palmitoyl-2-oleayl lecithin Choline kinase


Choline + ATP- - - - - - - - + Phosphocholine + ADP
Phosphocholine
v :: :~holipase A2 cytidyl transferase
Lecithin Lysolecithin Phosphocholine + CTP - - - - + CDP-choline + PPi

\,, L •
f:'. oleic acid
1-palmitoyl lysolecithin Phosphatidylcholine Elhanolamine

l
LLAT (1-palmitoyl, phosphotransferase
2-arachidonyl lecithin) 1, 2-diacylglycerol - - - - - - + Phosphatidylcholine
+ CDP-choline + CMP
1-palmitoyl lysolecithin ( \ • f"hosphatidylcholine
Ethanolamine
Arachidonyl- kinase

1
CoA HS-CoA Ethanolam ine + Phospho-ethanolamine + AD

1-palmitoyl • Dipalmitoyl Phospho-ethanolamine


cytidyl transferase
lysolecithin Palmitoyl-n lecithin Phospho-ethanolamine+CTP- - - + CDP-ethanolamine + PPi

1
1-palmitoyl
CoA HS-CoA

-----7""""'?""""""'""-+• Dipalmitoyl
1, 2-diacylglycerol +
CDP-ethanolamin,e
Phosphotransferase
-----+ Phosphati~yl-
ethanolamme + CMP
lysolecithin ( • lecithin
1-palmitoyl Glycerophosphoryl Phosphalidyl - - - - - - - + Phosphatidyl serine
lysolecithin choline ethanolamine + Serine + ethanolamine

Fig. 16.6: Synthesis of phosphatidyl choline; phosphatidyl etha-


Fig. 16.5: Synthesis of phosphatidylcholine
nolamine; phosphatidyl serine

Synthesis of Phosphatidylinositol to cholesterol forming lysolecithin and cholesterol ester


(see Chapters 14 and 15).
In the CDP-diacylglycerol pathway, phosphatidic acid
first reacts with CTP to form CDP-diacylglycerol. The CDP Synthesis o,f Sphingolipids
diacylglycerol can react with the alcoholic group of serine
They are important components of biomembranes as
or inositol to form the corresponding phosphoglyceride
well as the brain. The most important phosphosphingo-
and releasing CMP. The phosphatidylinositol undergoes
lipid is sphingomyelin. Ceramide is the basic structural
further phosphorylation by a specific kinase to form
unit of all sphingolipids.
phosphatidylinositol diphosphate (PIP2) which acts as
a signal transducer.
Synthesis of' Ceramide
Synthesis of Phosphatidyl Choline It is formed from sphingosine and fatty acyl-CoA. Sphin-
gosine is formed in the endoplasmic reticulum from
The major pathway for lecithin and cephalin synthesis,
palmitoyl-CoA and serine in the presence of pyridoxal
especially in liver and brain , involves the activation of
phosphate (Fig:I 6.7).
choline or ethanolamine to phosphorylated derivative
and then to form the CDP derivative (Fig.16.6). Finally Synthesis of Sphingomyelin
phosphocholine or phosphoethanolamine is transferred
Ceramide + CDP choline -+ Sphingomyelin + CMP
to diacylglycerol to form the corresponding phospholipid.
Ceramide reacts with CDP choline to form sphingomyelin.
First one fatty acid residue is removed from glycerol.
Then, arachidonic acid is added by the action of LLAT Niemann Pick's disease
(Lecithin-lysolecithin acyltransferase) (Fig.16.5).
This is an inborn error of metabolism due to failure of
degradation of sphingomyelin. The enzyme sphingo-
Plasmalogens
myelinase is deficient in this condition (Table 16.4 ).
Dihydroxy acetone phosphate is acylated and then
choline is added. Finally the alkyl group in the first car- Synthesis of Glycosphingolipids
bon atom is desaturated. These carbohydlrate containing lipids are synthesized
Yet another important enzyme acting on lecithin is by transfer of an active glycosyl or hexosamine residue
LCAT which transfers a PUFA from 2nd carbon of glycerol from its UDP derivative.
Chapter 16: MCFA, PUFA, F>rostaglandins and Compound Lipids 247

Synthesis of Cerebrosides synthesis, the active form of NANA (N-acetyl neuraminic


acid) is used. The synthesis is shown in Figure 16.9.
The two most common cerebrosides are glucocere-
broside and galactocerebroside (Fig. 16.8) Tay-Sachs Disease
This is an inborn error of metabolism due to failure of
Gaucher's Disease
degradation of gangliosides. The enzyme hexosami-
This is an inborn error of metabolism due to failure of nidase A is deficient in this condition (Table 16.4 ).
degradation of glucocerebrosides. The enzyme beta gluco-
sidase is deficient in this condition (Table 16.4 ). Lipid Storag'e Diseases (Sphingolipidoses)
They form a group of lysosomal storage diseases.
Synthesis of Sulfatides The sphinf JOlipids are normally catabolized by a
Cerebroside sulfatides are sulfuric acid esters of cere- series of bond specific lysosomal hydrolases like alpha
brosides. The major sulfolipid of brain is galactocere- and beta glucosidases, galactosidase, neuraminidase,
broside-3-sulfate (Fig.16.8). PAPS is phosphoadenosine hexosaminidase and arylsulfatase.
phosphosulfate or active sulfate formed from sulfur The diseases result from failure of breakdown of
containing amino acids. a particular sphingolipid due to deficiency of a single
enzyme.
Synthesis of Gangliosides The children afflicted by these diseases are severely
retarded mentally and seldom survive fo r long.
Gangliosides contain one or more sialic acid residues.
All these diseases can be diagnosed prenatally by
They are present in high concentrations in the CNS amnio centesis and culture of amniotic fluid cells. Since
and also on surface of membranes. For ganglioside the children born with these diseases will have serious
mental deficits, the pregnancy may be terminated.
Pyridoxal phosphate Replacement of deficient enzyme has been tried in
Palmitoyl-CoA ---/""'"'"-"""':::--- - - • 3-keto dihydro-
/ + sphingosine Gaucher's disease, with limited success.
Serine CO2 Rate of synthesis of the lipid is normal, only degra-
dation is affected . The extent of the enzyme deficiency
3-k~to di~ydro----/-=-":::--- - -• Dihydrosphinganine
Reductase

sphmgosme / , + Galactosyl transferase


NADPH + H+ NADP+
Ceramide + ------------------·--····--------+ Galactocerebroslde +
UDP galactose +UDP
Acyl transferase
Dihydrosphinganine Dihydroceramide
Glucosyl transferase
+ Fatty acyl-CoA +CoA
Ceramide + ---------------------------------+ Glucocerebroside +
Dehydrogenase UDP glucose +UDP
Dihydroceramide ---(---::::- =:::'\---+• Ceramide
Galactocerebroside Galactocerebroside-
FAD + PAPS 3-sulfate + PAP

Fig. 16.7: Synthesis of ceram ide Fig. 16.8: Synthe,sis of cerebrosides

1. Gaucher's disease Glucocerebro side 3 types- adult, infant ile, juvenile hepatosplenomegaly, erosion
of bone, moderate anemia

2. Niemann-Pick Sphingomyelinase Sphingomyelin Severe CNS dannage, mental retardation, hepatosplenomegaly


disease Cherry red spot in macula. Death occurs by 2 years of age

3. Metach romatic Arylsulfatase A Sulfogalactocerebroside Accumulates in most tissues. Neurological deficit, difficulty in
leukodyst rophy speech and opt ic atrophy. Demyelination is also seen

4. Tay-Sachs disease Hexosaminidase A Ganglioside (GM2) Incidence 1 in 6,000 births. M ental retardation. Cherry red spot in
the macula. Progressive deterioration. Death by 3- 4 years

5. Generalized ~-galactosidase Ganglioside (GM I ) Mental retardation, hepatomegaly, skeletal deformit ies. Foam
gangliosidoses cells in bone m.arrow. Cherry red spot in the retina
248 Section B: General Metabolism

0
Ceramide- Glu- Gal + CMP-NANA •i i• Clinical Case Study 16.1 Answer
Diagnosis: Gastric ulcer due to NSAID
~ CMP
Biochemical etiology: NSAID inhibits gastric
Ceramide- Glu - Gal- NANA ----------------------------• GM3
enzyme (COX-1 ) required for synthesis of prostaglandins
UDP-N-Acetyl-Galactosamine that have a protective effect on the gastric mucosa.

!'--.uoP Decreased gastric side effects with coxibs:


Traditional NSAIDs, such as ibuprofen and aspirin,
Ceramlde - Glu- Gal--NANA ---------·------------------• GM2 inhibit both COX-1 and COX-2. The coxibs are selective
I inhibitors of COX-2, allowing continued production of
Gal NAc
protective prostaglandins by gastric COX-1 .
Fig. 16.9: Synthesis of gangliosides Difference between aspirin and other NSAIDs:
Aspirin covalently modifies platelet COX-1 , thus irre-
is the same in all tissues. A chart showing the salient versibly blocking thromboxane formation and reducing
features of the disease is given in Table 16.4. platelet function for the lifespan of the affected platelet
(platelets cannot synthesize new proteins). The inhibitory
Multiple Sclerosis action of other NSAI Os on platelet COX-1 is not covalent
and is eventually reversed when the agents' blood
It is a demyelinating disease. Phospholipids (ethanol levels decline.
amine plasmalogen and sphingolipids) are lost from Nonsteroidal anti-inflammatory drugs (NSAIDs),
white matter of the central nervous system. Cerebrospi- also known as prostaglandin synthesis inhibitors or
nal fluid contains increased quantity of phospholipids. cyclo-oxygenase (COX) inhibitors, can induce upper GI
irritation or ulcers. The NSAIDs include a wide variety
0
•'ii• Clinical Case Study 16.1 of medications including aspirin, ibuprofen, naproxen,
and indomethacin. These medications are used for
A 63-year-old female presents to the clinic with recurrent pain, inflammation, dysmenorrhea, headache, arthritis,
mid-epigastric pain over the last 3 months. She reports or fever. These compounds act as anti-inflammatory
some relief shortly after eating, but then the discomfort and antipyretic agents by inhibiting COX catalysis by
returns. She has tried various over-the-counter medica- prostaglandin H synthase (PGHS). PGHS has two
tions without relief. She also reports feeling tired and isoenzymes:
has had to increase the amount of ibuprofen needed PGHS-1 (or COX-1) is generally a basal enzyme
for relief of her arthritis. She denies nausea, vomiting, found in various tissues including platelets and gastric
and diarrhea. On examination, she is found to have mild mucosa; PGHS-2 (or COX-2) is an inducible enzyme
midepigastric tenderness. Blood microscopy revealed typically expressed in response to cytokines and mite-
a microcytic anemia and normal white blood cell count, gens at sites of inflammation or cell proliferation .
The prothrombotic and vasoconstrictive actions of
consistent with iron deficiency. The patient was referred
COX-1-derived thromboxane in the vasculature are oppo-
to a gastroenterologist who performed an upper GI
sed by an antithrombotic and vasodilative prostaglandin,
endoscopy that identified gastric ulcers. He stated that
prostacyclin , that originates from COX-2 in vascular
he suspected that the ibuprofen, a nonsteroidal anti-
endothelial cells. The COX-2 selective coxibs thus tend
inflammatory drug (NSAID) was the causative agent and to decrease prostacyclin levels in the vasculature without
suggested switching from ibuprofen to a coxib, such as reducing the thromboxane levels. This tendency is
celecoxib. thought to explain the small but significant increase in
1. What is the likely biochemical etiology of the cardiovascular risk that recently led to withdrawal of
disorder? two coxibs from the market.
2. Why do coxibs generally have a lower incidence of
upper GI problems than other NSAIDs? I LEARNING POINTS, CHAPTER 16
3. What is the major difference between aspirin and 1. Fatty acid with 4 to 6 carbon atoms are called short
other NSAIDs with regard to platelet function? chain fatty acids (SCFA) (e.g. Butyric acid) and
Chapter 16: MCFA, PUFA, Prostaglandins and Compound Lipids 249

those with 8 to 14 carbon atoms are called medium 13. The arachidlonic acid is converted to prostaglandins
chain fatty acids (MCFA) (e.g. Laurie acid). Fatty by cyclo-oxygenase pathway.
acids having more than 20 carbon atoms are called 14. Thromboxanes are the major prostaglandins pro-
Very Long Chain Fatty Acids (VLCFA). duced by platelets causing aggregation and vaso-
2. Monounsaturated fatty acids (MUFA) have a single constriction .
double bond like oleic acid (C18) or palmitoleic acid 15. Prostacyclins synthesized in endothelial cells inhibit
(C16). Poly unsaturated fatty acids (PUFA) have 2 platelet aggIregation and cause vasodilatation.
or more double bonds. 16. PGF2 stimulates uterine contraction. Most impor-
3. MCFA digestion does not require pancreatic lipase tant therapeutic uses are in inducing labor, termi-
and bile salts. They diffuse directly into the portal nation of pregnancy and controlling postpartum
circulation. hemorrhage. Leukotrienes LTC4 , LTD4 and LTE4
4. Linoleic acid (C18, 2 double bonds) and linolenic constitute tlhe slow reacting substance of anaphy-
acid (C18, 3 double bonds) are essential fatty acids laxis (SRS-.A).
to be supplied in the diet. Arachidonic acid (C20, 4 17. Cyclo-oxygenase is an example of a 'suicide en-
double bonds) and Eicosapentaenoic acid (C20, 5 zyme'. Aspirin irreversibly inhibits the enzyme. Inhibi-
double bonds) are also biologically important PUFA. tors of cyclo-oxygenase are used as anti-inflamma-
5. Unsaturated fatty acids require isomerization of tory agents (NSAID) and antiplatelet drugs (Aspirin).
double bonds at specific carbon atoms before beta- 18. Cortisol inhibits phospholipase A2 thus reducing
oxidation can continue. PG synthesis.
6. VLCFA undergo initial shortening in peroxisomes 19. Compound lipids contain molecules other than
before they can undergo beta-oxidation. fatty acid and alcohol. Sphingomyelins are the only
7. Defective peroxisomal oxidation can lead to Zell- sphingolipicls, which contain phosphate and does
weger's syndrome (empty peroxisomes) and not contain a sugar residue.
Adrenoleukodystrophy. 20. Cardiolipin or diphosphatidyl glycerol is formed by
8. MUFA can be formed in the body by desaturation of reaction between CDP-diacylglycerol and glycerol-
saturated fatty acids. Thus oleic acid can be formed 3-phosphate.
by desaturation of stearic acid. The microsomal 21 . Sphingosin,e is the alcohol present in glycolipids
desaturase system requires NADH, molecular oxy- and sphingomyelin.
gen and cytochrome B5. 22. Sulfatides are formed by addition of active sulfate
9. Nutritionally essential linoleic and linolenic acid can from PAPS ..
be further elongated and desaturated to form Homo- 23. Gangliosidei synthesis requires one or more sialic
gamma linolenic acid (GLA) and arachidonic acid . acid (NANP1) residues.
10. Omega-3 fatty acids are found mainly in fish liver 24. Defective catabolism of these compound lipids can
oils and have atheroprotective effect. lead to a group of lysosomal lipid storage disorders.
11. Arachidonic acid is the precursor of prostaglandins, (Gaucher's, Neimann-Pick's, Tay-Sachs)
thromboxanes and leukotrienes. 25. All these conditions are characterized by accumu-
12. Prostaglandins are local hormones synthesized lation of a single compound lipid, severe mental
from arachidonic acid. retardation and neurological deficit.

PART-1: ESSAY AND SHORT NOTE UESTIONS


16-1. How are prostaglandins synthesized? Indicate their importance as loc;ill hormones. Mention the effects of drugs
acting on this pathway.

SHORT NOTE QUESTION


16-2. Cyclo-oxygenase. 16-6. Gaucher's disease.
16-3. Mechanism of action of aspirin. 16-7. Niemann--Pick disease.
16-4. Effect of prostaglandin on smooth muscles. 16-8. Tay Sachs disease.
16-5. Lipid storage diseases.
250 Section B: General Metabolism

PART-2: MULTIPLE CHOICE QUESTIONS


16-1. Triglycerides containing medium chain fatty acids 16-8. Phosphatidylcholine is synthesized by combining
are hydrolyzed in the gastrointestinal tract by: diacetyl glycerol with activated choline. Which deri-
A. Epithelial triglyceride lipase vative of choline is used for this reaction?
8 . Pancreatic lipase A. UDP-choline 8. GDP-choline
C. Hormone sensitive lipase C. ATP-choline D. CDP-choline
D. Lipoprotein lipase 16-9. The lipid storage disease which can manifest in
16-2. Polyunsaturated fatty acids are required for the adults is:
following purposes, except A. Tay-Sachs disease 8. Niemann-Pick's disease
A. Synthesis of leukotrienes C. Sandhoff's disease D. Gaucher's disease
8 . Synthesis of prostaglandins 16-10. Which of the carbon atoms is omega carbon?
C. Esterification of cholesterol A. Carbon atom having the functional group
D. Conjugation of bile acids 8. Carbon atom having the first double bond
16-3. Desaturation of fatty acids is taking place in which C. Carbon atom bearing the methyl group
subcellular fraction? D. Carbon atom having the double bond nearest to the
A. Mitochondria COOH group.
8 . Cytoplasm 16-11. Which of the following is required for the oxidation
C. Endoplasmic reticulum of VLCFA?
D. Lysosome A. Oxidation at the beta carbon
16-4. All the statements regarding prostaglandin are 8 . Transport into mitochondria
correct, except: C. lsomerization of the double bond
A. Leuktrienes are derivatives of prostaglandins D. Shortening in peroxisomes
8. Arachidonic acid is the precursor for PG2 series. 16-12. All the following are biologically important features
C. Naturally occurring prostaglandins belong to PG2 . of PUFA, except:
D. Synthesis is inhibited by aspirin A. Undergo peroxidation when present in membranes
16-5. The mechanism of anti-inflammatory action of aspi- 8 . Used for esterifying cholesterol
rin is that it: C. Mainly used for energy generation
A. Reversibly inhibits cyclo-oxygenase D. Precursor for synthesis of eicosanoids.
8. Irreversibly inhibits cyclo-oxygenase 16-13. Which of the following inhibitors of PG synthesis
C. Inhibits phospholipase does NOT act on Cyclooxygenase?
D. Inhibits lipo-oxygenase A. Cortisol 8 . Aspirin
16-6. Prostaglandins are stored: C. Paracetamol D. NSAID
A. In adipose tissue 16-14. Hydrolysis of one molecule of sphingomyelin
8 . As plasma lipoproteins gives all except:
C. As granules in mast cells A. Phosphoric acid 8 . Choline
D. As membrane phospholipids C. NANA D. Fatty acid
16-7. Polyunsaturated fatty acids are required for the 16-15. The lipid storage disorder characterized by cherry
synthesis of the following compounds, except red spot in the retina is
A. Lecithin 8 . Cholesterol A. Gaucher's disease 8. Zellweger syndrome
C. Prostacyclin D. Leukotrienes C. Fabry disease D. Tay-Sachs disease

ANSWERS OF MULTIPLE CHOICE QUESTIONS


16-1 . A 16-2. D 16-3. C 16-4. A 16-5. B 16-6. D 16-7. B
16-8. D 16-9. D 16-10. C 16-11. D 16-12. C 16-13. A 16-14. C
16-15. D

PART-3: VIVA VOCE QUESTIONS AND ANSWERS


16-1 . What is MCFA (Medium chain fatty acids)? 16-3. What is VLCFA (Very long chain fatty acids)?
They contain 8 to 14 carbon atoms. They contain 20 or more carbon atoms.
16-2. What is LCFA (Long chain fatty acids)? 16-4. What is the difference in digestion of MCT from
They contain 16 to 18 carbon atoms. that of LCFA?


Chapter 16: MCFA, PUFA, Prostaglandins and Compound Lipids 251

MCT containing triglycerides are digested by MCT- Prostaglandins are derived from the PUFA. The Series 2
specific lipase. Pancreatic lipase and bile salts are not (with 2 double bonds) are derived from. Arachidonic
required. acid. All naturally occurring PGs belong to the 2 series.
16-5. What is the difference in absorption of MCFA from 16-14. What are the enzymes necessary for prostaglandin
that of LCFA? synthesis?
MC FA is absorbed directly to blood (not to lacteals, as The arachidonic acid is released by the action of phos-
in the case of LCFA). A fter absorption, MCFA is carried pholipase A2. Prostaglandin synthesis is catalyzed
by albumin in blood, whereas LCFA are absorbed as by Prostaglandin H synthase (PGHS). It contains two
triglycerides and carried by chylomicrons. separate enzyme activities, cyclo-oxygenase and
16-6. When unsaturated fatty acids are oxidised, how peroxidase.
many ATP is formed? 16-15. How prostaglandin synthesis is regulated?
The energy yield is less by 1.5 ATP molecules per The phospholipase is activated by epinephrine. Steroids
double bond, when compared to the corresponding inhibit PL and prevent release of arachidonic acid from
chain length saturated fatty acid. Because, the FAD membranes.
dependent dehydrogenation (step 1 of beta-oxidation) 16-16. What is the importance of cyclo-oxygenase?
does not occur at the double bond. Cyclo-oxygenase is activated by catecholamines and
16-7. Name polyunsaturated fatty acids. inhibited by nonsteroid anti-inflammatory drugs (NSAIDS).
Linoleic, linolenic and arachidonic acids. Cyclo-oxygenase is a •suicide" enzyme.
16-8. What are essential fatty acids? 16-17. What is the mechanism of action of aspirin?
Those cannot be synthesized by the body. So they are Aspirin acetylates serine in the active site and irrever-
to be provided in the diet. sibly inhibits the cyclo-oxygenase.
16-9. Name the essential fatty acids. 16-18. What is the mechanism of action of prostaglandins?
Linoleic and linolenic acids are the only fatty acids Prostaglandins are local hormones. and function
which cannot be synthesized in the body. through G-protein coupled receptors.
16-10. Where is desaturation of fatty acid takes place? 16-19. What is the importance of DHA?
In the endoplasmic reticulum. DHA is synthesized linolenic acid. DHA can be obtained
16-11 . What are the substances derived from PUFA? from fish oils or from milk. DHA is especially required for
Prostaglandins, Prostacycline, Thromboxanes, Leuko- the development of brain and retina. Low levels of DHA
trienes. in blood is seen in patients with retinitis pigmentosa.
16-12. What are the functions of PUFA? 16-20. Niemann-Pick disease is due to deficiency of?
Synthesis of prostaglandins, synthesis of phospholipids Sphingomyelinase.
and esterification of cholesterol. 16-21 . What is accumulated in Niemann-Pick disease?
16-13. Prostaglandins are derived from what? Sphingomyelin.
_ _ _ _ _Chapter 17
General Amino Acid
Metabolism (Urea Cycle antt
One-Carbon Metabolism)
Chapter at a Glance
The learner will be able to answer questions on the following topics:
l Digestion of proteins -:r Formation of ammonia
Absorption of amino acids Urea cycle
3 Meister cycle ) ' .9 Urea cycle disorders
)t Cathepsins, proteasomes ,1' 0
10 Urea level in blood
lnterorgan transport of amino acids O It One carbon metabolism
{. Transamination and transdeamination .4ovJ Hyperammonemia

--
CO>\.,~ P1f'(1te\ ·C. ~ l (_L-1 -- 1/
V u
The main role of amino acids is in the ~ ntbe~ ,if l DIGESTION OF PROTEINS El.
strycturill and functional proteins. A 70 kg man has
an average protein turnavec rate of @oo g per da) (same The dietary proteins are denatured on coo~ and
amount synthesized and same amount broken down). therefore mo~ i asily digested. All these e!)_z~ s
: The(nonessenii)I amino acids are either ~~m, are hydrol~ s tciat s 3 enzt > ~~ natu r e ~ i c
the diet or synthesized in the body. The>'f!~aJ e ~ are secrete&f.~t~;ctive 2_~':Jns which are
amino acids are obtained from the diet. Even if one ~rted to their active "#6r'i! in the i estinal lumen.
amino acid is deficient, protein synthesis cannot take This would prevent autodigestion of the secretory acinr.
' place. The body amino acid pool is always in a dynamic The protEiolytic enzymes include:
steady state. In an adult, the rate of synthesis of proteins 1. @n~ot'eptidase~ They act on peptide bonds ioside
balances the rate of degradation, so that ~ n the protein molecule, so that the protein becomes
-\ b~ is maintained proteins are generaUy not US!;1d successiv~ small1S\and s~~er units. Thi~ roup
\, for ruoviding energy (Fig. 17 .1 ). includes P~ sin, Try~fn, Chy~ trypsin and El~ tase.
~"' T, EJoJt (~he.) c;p,, == Pf T
tf (Y'tL '\ -~ cb:-. + Ammonia g/day)
)' Dietary proteins- - - -- - Body _____.. Keto acid _
(50-75 g/day) amino ,I ~ x ldatlon
l:P 1'

l
acid pool
(J).,N, (100 g) l>eo..tofl'I otl,
luconeogenesis
Breakdo~ n of _ _ _ _ _ _ . / a£i-et J.rivl"j
body proteins onessential amino acids
(300-400 g/day)
Syntl;tuia of body pto~ns f300-400 gtday) isvntbub_of.&o!cla~iud ~ .1:-1cts (heme, purine,, pyrimidines, cr~tjne;- , · -:,
______ . , a ; : - ~onzymei;, _
.t,onnones, etc.)• - ,:_ ;_ ·.:_· -•.:.; -;_.::-
0
• _ - - ::·

Fig. 17.1 : Overview of metabolism of aminp acids


sn - ~ '['f\,-e>:n v-f~- ,ti) x
J:e (.).1'¥-~~y-,t,o y&~~ .i..We9; f~G,g9,"t6)C-J
rQ•. - ' , ,
J~..
Chapter 17: General Amino Acid Metabolism (Urea Cycle and One-Carbon Metabolism) 253

bond located at the ends of the polypeptide chain. Ala, Gly, Ser
This group includes: 1~ ~ -----+-_:__:_:__ _ ___:::..!..,__:J_.:...__ _----1
a. Carboxypeptidase, which acts only on the
( -terminal basic amino acid
peptide bond at the(sarboxy term]bal>end of the
chain.
b. Aminopeptidase, which acts only on the peptide the peetide hormones: Chol~ Jdnin an ancreo-
i
dlfbf1iin
l PX)
nM ~,S~"M
zymin. ·
M,on W ~Jhe chi iQ
pro\ em is e#efiifai y eitr~i1t :ff • Pancreatic ju}'jf) contains ~ importan~ dopepti-
A. Stomach das~ namely Tl)Tpsin, Chyrirotrypsin, E'fas'tase and
B. Carboxypeptidase.
C. These enzymes are also secreted as zymogens
/'Jv, Pn~ • (trypsinogen, chymotrypsinogen and proelastase), so that
Gas fie Digestion of Proteins
-C...~ P i ! c - thei nf.;a~~io~eus are not aotolyzedj\ll the three
In the stomach{hy~ ~nc'acid IS secreted (see Chapter areeC ase: i.e. the active centers of these
24 ). It rnakes e~pH optimum for the ~ction of epsin t . . .d
, - enzymes con am serine res, ues.
and also acJivates pepsin. The acid als ~PP. '-· "·
proteins. 1fu=-.&,•,,( t ~ ....,iua ~ ) 9-'2. -i""G =fJ ~~~noui
(U,'r:~ oJ OJTT ~ lJSltJJ\... ' ) E:~-
Renn'm ® CJ,1 ~moon L Cm) Trypsinogen is a~ a~ terokjnaspl (enteropep-
tidase) present on thd'.iritestioal mjcroyjUuslnembranes.
Rennin otherwise called Chymosin, is active in infants
Once activated, the trypsin activates other enzyme mole-
and is involved in the curdling of milk. (Box 17.1 ). It is . . .
absent in adults. Milk p rotein, casein is converted to cules. Trypsin is activated by the(remaval af a bexaReP
paracasein by the action o(Turinfu)This denatured protein tide from N-terminal end . Trypsin catalyzes hydrolysis
is ; asilyd igested further by pepsin. . of the bonds formed by carboxyl groups of Arg and Lys.
C'o.&"L11' :W.DD'f) c . f>~ Acute pancreatitis: P~ atll(e activa1iool>f trypsi-
Pepsin ( P) r J (;"Q
1
~ i~ c-, nogen inside the pancreas itself, will result in the autg-
- "eferej,i, ~fi~ digestion..oLpaocreatic c.eJls, The result is acute pancrea-
lt is ~ecreted by the ecsc . s as inacti~e titis. It is a life-threatening condition.
pepsmogeri. The conversion of ~eJ>smogen to pepsin
is brought about b rem ova1 or ~4 ~~1cids from@;hymotrypsin l C,)
tha. tl,l-ter0lioat.evd, by the hydrochloric acid. The opti-
mum pH for activity of pepsi~ s arourtd 2. epsin is an I.JlYRSin wHiacvt:,n chymotrypsinogen, in such a manner
endope tidase, (Table-tt'. 1). Pepsin catalyzes~ dr~- that &,_B a,11d C peoti,®~ f9rmed. These 3 segments
- are approximated, so that the"active site is formed. Thus
sie\of th~ onds formed by carboxyl groups of e yr, . - . . {{)

~=
\&.I . . x- ) ~ I' ( selective proteolys_w produces the catalytic sit
Trp and ~ Y the act1~ f pepsin, proteins are bro-
ken into pr~ses and peptt>nes. @Jarboxypeptidases (C p)
Pa~ atic Diges~ Trypsin and chymotrypsin degrade the proteins into
small peptides; these are fucth~ hydral¥3ed into dipep-
The optimum pH for the activit of Pt ncreatic enzy~ep tjges aod trjpeptide.s by carbox tidasesl pre ·
(p~ by the lkahne · a n d ~ c the pancreatic juice. The procarboxypeptidase is
juice. The secretion ~ 1eatic juice is stimulated by

PZ CC.\<
II O'OC
254 Section 8 : General Metabolism

Intestinal Digestion of Proteins


The half-life of proteins is highly variable. ~ ithine de~ooxylase
CQmplete digestion of the small peptides to the level
has only 11 minutes. Half-life of'hemo lobin depends on the life
of amino acids~ rought about by enzymes present in span of RBCs. The lens protein, r sta m remains unchanged
i~stinal Lu!s,_(.~uc.s;~teric,us). The luminal surface throughout the life of the organism: amaged or defective proteins
are prematurely degraded.
~ntestinal epithelial cells contains the following enzymes:
;\ Q3) mino e tidase lease the N-terminal amino
<'s)t/0) ac · · a ~ ri e tidas will complete the
digestion of proteins.

@ Absorption of Amino Acids


The absorption of amino acids occurs mainly in the small
intestine. It is an energy requiring process. These trans- Aaron Ciechanover Avram Hershko Irwin Rose
NP2004 NP 2004 NP2004
port systems are carrier mediated and/or ATP sodium b. 1947 b. 1937 1926 - 2015
dependent symport systems. There are 5 different car-
riers for amino acids:
~t
t 1.
Neutral amino acids (Ala, Val, Leu, Met, Phe,
Tyr, lie)
a. Hartnup disease (~e Chapter
b. lmi,no glycinuria
c. Cystinuria-(see ctmpte'- 18)
'P 1.'-t-.a)
):ff) )r,O \~ vJ,f,t;__ ,
-,. ~,(te~~
\1\
~ utt 2. Basic amino acids (Lys, Arg) and Cysteine
3. lmino acids and Glycine
4. Partial gastrectomy, pancreatitis, carcinoma of pan-

(b 4 . Acidic amino acids (AspJ Glu)


creas and cystic fibrosis may affect the digestion
and absorption of proteins.
P''\; 5. Beta amino acids (bet alanine,). t ,
t" /"7 1 \MI 'I. \.r ! TJ
5. Protein-losing enteropathy: There is an excessive

y
~ Miester Cycle (Gamma-Glutamyl Cycle) loss of proteins t u h th astrointestinal tract.
p. 1?, (.,
intestines, kidney tubules and brain, the absorption Intracellular Protein Degradation
~uT _ of neutral amino acids is effected b the gamma-gluta-
All proteins in the body are constantly being degraded.
1?JO{td,myl cv,sLe;.l llEtripeptide glutathione H) (gamma-
glutarh~ in I;ll~ i~) reacts with [lino acid to
form gamma-glu ~ i n o aci . hat is then cleaved
- -==--
to give the free amino acid. The -----~ - __.
- - - -4
~--
- ... ' -
__ ..........._..,,_~ ~-
• • •

sequence (areas rich in praline, glutamate, serine and

DiRe£Ycies and tripeptjdes can e~ er


ofJDuco.§.iil.iliills ; they are i mediate! h d into
single'aroioo aci~~~are then transported into portal se~ ~ter 2 ) .
. Th
vein. ese are 1mmunoge~. causing an I o y rea \!} c.;atheps ns r ,o
. . . t·b d ~ ~m't?,v O c.i.,.; ..l
°' '9eA"t
tion , leading to food allergy. ·
In th@ agolysosome~ the particles are broken down
Clinical Applications by enzymes known as cathepsins. The term cathepsin
• is a Greek word, meaning '!.2, digest'. Cathepsins are~
1. The deficiency of the enzyme 5-oxtjprolinase leads
to oxoprolinuria (pyroglutamic aciduria). in number, ~ d a& A tg I . ~ ost of them are active
The allergy to certain food proteins (milk, fish) is at pH aroun~ l Acidic)
believed to result from absorption of partially dige
led proteins. CFvoJ.
al\~~) Ubiquitin
Defects in the intestinal amino acid transport systems Intracellular protein b.IJ~;cl.Qwn also occurs ~ ~n-
are seen in inborn errors of metabolism such as: dent of lysosomes This involves u itin. It is so named,
Brain
Kidney

I ~G
Valine? Glutam,ne
Alanine

l
Muscle
lanine - -- - - -+
Muscle
*
~e

Fig. 17.2: lnterorgan transport of amino acids during fasting con- Fig. 17.3: lnterorgan transport of amino acids after taking food
ditions (postprandial condition) ~ -

2 ? I• '.
In the Fed State " Ol '- ,, l rr ) CJ-, ' •
Diet - - - - ·~ ---+ Ammonia -+ Urea (57%)
Amino acids absorbed from the diet are taken up by dif-
---+ Plasma proteins (7%) ferent tissues (Fig. 17.3). Both muscle and brain take up
Amino acid
pool branched chain amino acids, and release glutamine and
---+ Body proteins (14%)
Body- - + alanine. The glutamine is delivered to kidneys to aid in
protein
catabolism ---+ Circulation (22%) regulation of acid-base balance, while alanine is taken
up by liver.
Fig. 17.4: Amino acid pool
Gener~ Metabolism of Amino Acids
because it is io all cells_abundantly. It is a small
1. The ahabolic reactions where proteins are syn-
protein with 7§,c~ idu~ (molecular weight, 8.5 kDa). thesized.
Ubiquitin is attached to proteins. 2. Synthesis of specialized products such as heme,
(D ' cr~e, purines and pyrimidines.
Proteasomes -- Pzro,ient ")'r\ 3. T~ tabolic reactions where dietary proteins
Ubiquitin tagged proteins are immediately broken down ~ d body proteins are broken down to amino acids.
~ ransamination: Amino group is removed to pro-
-=
inside tbe proteasomes of the cells. Ciechanover, Hershko
and Rose were awarded Nobel Prize in 2004 for their
duce the carbon skeleton (keto acid). The amino
group is excret~ds ure1 f- ·- · - - ·
discovery of ubiguitin-mediated protein degradation. li'\The carbon sk ton is us d for synthesis of non-
\J_)ssential amin a ~
lnterorgan Transport of Amino Acids 6. It is also used for 'gruconeogenesis or for com-
plete oxidation.
Breakdown of muscle protein is the source gf _amino
7. Amino acids are used for minor metabolic functions
acids for tissues while liver is the site of disposal (Figs. like conjugation, methylation, amidation, etc. Figure
17.2 and 17.3). 17.1 gives a summary of amino acid metabolism. The
,(~ c:9.>~' amino acid pool in the body is shown in Figure 17.4.
In Fasting State J,.e.e.
The musde releases mainly alanine and glutamine of which FORMATION OF AMMONIA
by liver and glutamine by kidneys The sources and fate of ammonia are shown in Figure
es the amino rou and converts 17.5. The first step in the catabolism of amino acids is to
it to ure ~~msJ!t r on skeleto is used for gluco- remove the amino group;; ammonia. This
neog;.:;ti. S de s should also see glucose-alanine source of ammonia. However, ~ s of ammo-
cycle, under gl cbfili'agth~sft)~}ig.
10.27). The brain nia may also be formed from c.51abolism of purine and
predominantly takes up branched chain amino acids. pyrimidine bases.
'8"" ~\<. A'-~
j 0£4\\-a. ha.{
rvce¥ °-p (';06~,,l)Of) 0J
256 Section B: General Metabolism )SY/>
. fo X 'fC
'-" V

-,-----=-- Glutamine coo-· coo-


I
C ~H +
Ammonia I
pool --=--• Asparagine
CH2
I
CH2
<t 2 ~
CH2 ~
I I
coo-· coo-
Glutamate Alpha keto glutarate
Fig. 17.5: Sources and fate of ammonia

(j.mmonia is highly to,s,i~specially to the nervous


system. Detoxification of ammonia is by conversion to
--- '
urea and excretion through urine. coo-
@ Transamination §)€) CH3
Transamination is the exchange of the alpha arnioo Pyruvate
Summary
group between one alpha amino acid and another alpha
amino acid 1 + keto acid acid 2 + keto acid 1
ket~ forming ~ alpha a~ acid.
Fig. 17.6: Transarnination reaction. In this example, enzyme is
Am~ id 1 + kekuiMl 2 2+~ cid 1 Alanine aminotransferase (ALT) and pyridoxai phosphate is the
As an example, amino groups are interchanged coenzyme. The reac:tion is readily reversible
between alanine and glutamicacid (Fig. 17.6). In almost all
cases, the amino group is accepted by alpha ketoglu- give amino acid no. 2 to equalize the quantity of both.
tarjc ae:ctso that glutamic acid is formed. The enzymes This is called eqlllalization of quantities of non-essential
catalyzing the reaction as a group are known as tjmjno- amino acids.
fransferasis) These enzymes have · al hos /_.., Yr·Y"O-Y
® ehate as prosthetic 9iQ.ldQ.. (Fig. 17.6). The reaction i N xceptions Pr ~~th.ie\--~ ~'J
@~ - Lysine, threonine and proline are not transaminated.
• • . ·r.
J..: 1olog1cal 5 1gm 1cance o
f r:;; They follow direc1t degradative pathways.
.-
1
Transamina_
t ion ~ Clinical Significance of Transamination
1

~,I
(;) First Step of Catabolism r~""2r,l\ g Aspartate amino transferase (AST) and Alanine amino
In this first step, ammonia is removed, and the carbon transferase (ALT) are induced by glucocorticoids, which
sk eIeton of the ammo· ac,·d lPJW:,
t S in t h a r,c path way.
· to cad'.3 favor gluconeogenesis. AST is increased in myocardlc;il
A 'I'f'~~ \..~")
I.JVSynthesis of Non-essenr,a/ Amino Acids
'--> ~4B~'& R,9""-t
infarction and .i.\
r
· live dise~s~_lhe~-
im(?Ortance is given in Chapter 6. V\t.J,Y(.

B ransdeamination 1· ~\o...
t1 11....
Bymeansoftransamination, allnonessentialaminoacid (Of\,(.Lll)'lrxm
l"\~\f,> can be synthesized by the body from keto acids avail- .._ J
1'" _.IJ, able from other sources. For example, pyruvate can be 1. The amino woup of most of tn-e amino acids is
!~, transaminated to synthesize alanine. Similarly oxalo- released by a coupled reaction, transdeamination,
_J•~ acetate produces aspartic acid. Alpha ketoglutarate that is transamination followed by oxidative
µ,"' is transaminated to form glutamic acid. Those amino deamin'1ltlori.
; / acids, which n t be s nthesized in this manner, are 2. Transamination takes place in the cyfc$Pasm of all
therefore s enti ; they shou e made available in the the cells of the body; the amino group is transported
food (See Box 3.1 for essential amino acids). to liver as glutamic acid.,_w.hlG~ finally oxidatively
. .<kak.0.9
n, nterconversion of Amino Acids <aeaminated in the roitnchondr.ia_6f hepatocytes.
3. Thus, the two, components of the reaction are physi-
If amino acid no.1 is high and no. 2 is low; the amino
group from no.1 may be transferred to a $ cally far away, but physiologically they are coupled.
Hence, the term transdeamination (Fig. 17 .7).
ru,,i • Q}a,.2.
0..0.,1 ~o.. 0..0-2..
f'\a> bh- toi l Y'A:l~iljDO'OJ ~ f ' \fl.i( k-0 :l •lf9-9"'
~ . , ~fubi p i ~16.a.k,o,. y,ga.A/JO-{ (G~ pt"~~f\ doli!~ \
enera/ Amino Acid Metabolism (Urea Cycle and One-Carbon Metabolism) 257
r -----:-- - - - --iw-~- ..::,..,;;;i.....:__.s;.,~~~::!...!_T V ab
Any amino acid

Corresponding Glutamate
ketoacid :)

Transport from tissues to liver


-~
~ = " ,.
Oxidative · e,I?_.__ ...riv
( _ Deamination in liver ~~'teJ"
COO Glutamate C00-
6H H + dehydrogenase I
I -N 3 ~ 20 (~Dr-\) ?=O Deaminations
CH2 \ • CH2
I I
1H2 NAO+ NADH+H CH2
coo- I boo- c@
Glutama I~ Alpha keto glutarate
: Transamination + deamination = transdeamination
~~,4io'l'\·,l;."-~
ative Deamination of Glutamate
Only liver mitochondria contain glutamate dehydro-
genase (GOH) which deaminates glutamate to alpha- i ti i also undergoes non-oxidative deamina-
ketoglutarate plus ammonia. So, all amino acids are tion to form ocanic c· ; catalyzed by histidase (see
first transaminated to glutamate, which is then finally Chapter 19)) mmonia may also be produced by degra-
deaminated (transdeamination) (Fig. 17 _7 ). Amino dation of purines and pyrimidines due to ~
111
acids are deaminated at the rate of about 50- 70 gram

per day: - bS :t,O


During the transamination reaction the amino group
-=::;,,=;;;==!::-~

I ti • f\11 .Jt~~• I,,),> ff;"). ti~• \~, •~t f toJiZl-. J._(J'6 j-~f
faction 'in the gastrointestinal tract.

DISPOSAUDETOXIFICATION I)

of all other amino acids is funneled into glutamate. Hence, OF AMMONIA ~1oo,.dlL «9\-~+~
t~e glut~mate dehydrogena~e reaction is the fi~al re~c- First Line of Defense /'> ¼ ~Oril-
111
tion, which removes the amino group of all amino acids (Trapping of Ammonia) ~ , w
(Fi~.17.7). ltneedsNAD•as en~ . . ½cd\e.q,~ ,,tu)
enzyme; it is activate Dg.,and inhibited b GTP. Being highly toxic, ammonia 'Shouldoe eliminated or
The hydrolysis of glutamine also yields NH but this detoxified , as and when it is formed. Even very minute
3
occurs mainly in the kidney where the NH; excretion is quantity of ammonia may produce toxicity in central
d for acid-base regulation. £em nervous system. But, ammonia is always produced by
r 'L"' \);h \4-:.,{J almost all cells, including neurons. The intracellular
r Pathways of De, minati ~IJ· ,':}.f ammonia is immediately trapped by glutamic acid to
1. L-amino acid oxidase can ~i ct on all amino acids form glutamine. especially in brain cells (Fig. 17.9). The

-
except hydroxy.A~J..1'18 acids and dicarboxylic a23i.!JO glutamine is then t.@_nsported to lj~ r. where the reaction
acids. It uses'l=~ cl£benzyme. The peroxfde is reversed by the enzyme glutaminase (Fig. 17.9). The
formed in this reaction is decomposed by catalase ammonia thus generated is immediately detoxified into
in the peroxisomes (Fig. 17.8). urea.' Aspartic acid may also undergo similar reaction to
2.(®amino ci xidase can oxidiz~ and form asparagine (see Chapter 18).
a mino · that ay be form~cterial r-ho-N.p~r
me a olis . ses a t!oenzyme.
Transportation of Ammonia ~ o ,
3. Small quantities of ammonia may be formed in Inside the cells of almost all tissues, the transamination
the body through minor reactions like o~ of amino acids produce glutamic acid. However, gluta-
' 1 2 ~ (mono amine oxidase) (see mate dehydrogenase is available only in the liver. There-
Chapter 19, under Tyrosine metabolism). fore, the final deamination and production of ammonia
"-4"'~ I.~
I

258 Section B: General Metabolism

--CH2-CH2--COO" Ammonia + CO2


Glutamic acid
!1
Carbamoyl phosphate
NH3: t :ATP
Glutamlne aynthetase
H2O ADP+ Pi
2
,,..,. • .....,.,_·,+f},,- · ooc--CHNH3•-cH2- CH2--C~H2
Glutamine O~mithine Citru~llicneAspartate
2 IUrea I 3
H0 Glutamlnase
[ NH 3 ..-1
Glutamic acid
Fumarate

Argininosuccinate
Argi~
Fig. 17.9: Ammonia trapping as glutamine

Fig. 17.10: Urea cycle. summary


BOX 17.3: Mammals excrete ammonia as urea but birds
excrete ammonia as uric acid
M illions of "gooney" birds nest on some islands of Pacific Ocean, off
the coast of Peru. Over the centuries, their droppings formed big CPS-I CPS-I/
hills. These "guano• deposits, containing mainly uric acid, is now 1. Site Mitochondria Cytosol
being exploited commercially as fertilizer containing nitrogen.
2. Pathway of Urea Pyrimidine
3. Positive effector NAG Nil
is taking place in the liver (Fig. 17.7). Thus,~ 1-...:.:
4-~ 5:.::
o.::
ur:..::
ce:..:..::
fo::..rN
:...:..__ _+-
A_m_m~o_n_ia_ _ _+-
G_lu_ta_m_i_ne_ __,,
acRt"'actn'S""the Ii, ,k betweerramino-groops-of amino s. inhibitor Nil CTP
acids and ammonia) The concentration of glutam,c ac,d N H..,(N ~
in blood is 10 times more than other amino acids. Gluta- solution of ammonium cyanate. The urea cycle is the first
mine is the transport form of ammonia from brain and metabolic pathway to be elucidated in 1932, by Hans
intesti ~ liver; while alapioe is tbe t~pod ~rm from Krebs (1900-1981) and Kurt Henseleit (1907~19~2).
muscle toffver. f"f'U~e.. Al~ Hence, the cycle is known as (!sr.ebs-Heoseie1t LJ@_
F.ma1 o·Isposa1 _ "'-<0-Lfl\\l f J1 r\'- ~ de · As ornithine is. the ftr§t rnembe.r.of tbe..raaction, it
. -
'-;» ~V'1'1 !'\q -1tu.. ~ '' . is also called as Orrnthme c~.
The ammonia from all over ttm body thus reaches liver. The two nitrogen atoms of urea are derived from
It is then detoxified to urea by liver cells, and excreted two different sources, one from ammonia and the other
through kidneys. Urea is the end product of protein directly from lhe alpha amino group of aspartic acid.
metabolism.
Since mammals including human beings excrete
Step 1: Formation of
Carbamoyl Phosphate
One molecule of ammonia condenses with CO2 in the
presence of two molecules....o.f.ATP to form carbamoyl
A oug , ammonia is toxic and has to be imme- phosphate. The reaction is catalyzed by the mitochondrial
diately detoxified, in kidney cells, ammonia is generated enzyme carbamoyl phosphate synthetase-1 (CPS-I).
from glutamine by the action of glutaminase. The pur- (Figs. 17 .1 0 and 17 .11, Step 1). An entirely different
pose is to excrete hydrogen ions as ammonium ions, so
cytoplasmic enzyme, carbamoyl phosphate synthetase-11,
as to maintain acid-base balance (see Fig. 27.5).
(CPS-II) is involved in pyrimidine nucleotide synthesis
9JAr (see Chapter 38). The differences of CPS-I and II are
9>~;1,,(JJ'Y
~<1"
~ REA CYCLE ._,,. shown in Table 17.2. CPS-I reaction is the rate-l~ g
In 1773, Rouelle isolated urea from urine. Frederic step in urea formation. It is irreversible and! J!.osteri~ lly
Wohler in 1828 obtained urea by boiling an aqueous regulated . p. 1"-h
:#= ·. uroCli-Q. u -410'1Thd. t~ fV) \'TO~
c_rm'x t,,,~ ir~ c.y'ro~rn - A/c;::--eJ_OQ)f)
Chapter 17: General Amino Acid Metabolism (Urea Cycle and One-Carbon Metab~ ~~ 59

Fig. 17.11 : U rea cycle and its relation w'ith citric acid cycle
\
Step 2: Formation of Citrulline (Figs. 17 .10 and 17 .11, Step 4 ). The e..!)ZWejS inhibited
QY-1.uroarate.. But this is a'iQided by the c toRlasmic
The second reaction is also mitochondrial. The carba-
lo_p aliza!iQn._o.Lthe eo__zy__m.e. The fumarate formed may be
moyl group is transferred to the NH2 group of ornithine funnelled jnto TCA.cycle to be converted to malate and
by ornithine transcarbamoylase (OTC) (Figs. 17.10 then to oxaloacetate to be transaminated to aspartate
and 17.11 , Step 2). The citrulline leaves the mitochon- (Fig. 17.11 ). Thus the urea cycle is linked to TCA cycle
dria and further reactions are taking place in cytoplasm. through fumarate. The 3rd and 4th steps taken together
proteins 1JCL.U1 may be summarize as:
. d , " _,
..
' c.. ·r
"',.
01
1
Citrulline + aspai ate -l Arginine + fu1Ilj3rate
A similar reaction of donation of amino group by
Step 3: Formation of Argininosuccinate aspartate takes place in purine nucleotide synthesis also
One molecule of aspartic acid adds to citrulline form- (see Chapter 38).
ing c(larbon to ni@ gen bong),vhich provides the 2nd
nitrogen atom of urea. Argininosuccinate synthetase Step 5: Formation of Urea
catalyzes the reaction (Figs. 17.10 and 17.11 , Step 3). The final reaction of the cycle is the hydrolysis of arginine
This needs hydrolysis of ATP to AMP level , so two high to urea and ornithine by arginase (Figs. 17.10 and 17.11 ,
energy phosphate bonds are utilized. Step 5). ThelorrJitbine returns to.the_mito_choodcia to react
with another molecule of carbamoyl phosphate so that
Step 4: Formation of Arginine the cycle is repeated. Thus ornithine may be considered
Argininosuccinate is cleaved by argininosucclnate as a catalyst which enters the reaction and is regen_e-
lyase (argininosuccinase) to arginine and fumarate rated.
OTC ~iOd\C-Lj :: -*Hnke.0. ~o~cl£sl
~ : lbl'b'& <9'v o}'nm,u..J-~ hee~

Hyperammonemia type I Very high NH3 levels in blood. Autosomal recessive. Mental etardation. Incidence is 1 in
100,000
Hyperam monemia type II (OTC) Ornithine Ammonia level high in blood. Increased g lutamine in blood, CSF and urine. Orotic aciduria
t ranscarbamoylase due to channeling of carbamoyl phosphate into Pyrimidine synthesis. X-linked 1
Failure to import ornithine fro m cytoplasm t o mitochondria. Defect in ORNT1 g ne.
transporter protein yperornithinemia, hyperammonemia and homocitrullinuria is seen (HJiH syndrome).
~c,. Decreased urea in blood. Autosomal recessive condition t<...O~
I,;\
~.,,. l~
Arg ininosuccinate Autosomal recessive inheritance. High blood levels of synthetase ammo nia and cit rulline.
\.9'
I----------+-=------+----_:....,------------':,__-'--
Citrullinuria (Citrulline excreted in urine 1-2 g / day).
O, Arg ininosuccinic aciduria Argininosuccinate Argininosuccinate in blood and urine. Friable brittle t ufted hair (Trichorrhexis nodosa).
- -- - - -------;
\!,' f - ---'- - - - - - + -:.t..
ly..:..
as:..:
e_ _ _ _-+-.:.:.
ln:..:
c:..:
id.:.:
en___c:..:
e:..:
2___in_ 2
::..:0:..:
0.:.:,00..:..0
:....::..:
bi.:.:. .:.:.._ _ _ _ _JI!.·-="'m.:.:.o.._\.,,L.c-_f'-.-st:~~--1::A~_;_:,mai=,Fr,:..._--,
rt.:..:.
hs
ti' Hyper argininemia Arginase Argini ne increased In blood and CSF. Instead of arg inine, cysteine and lysine are lost in
urine. Incidence 1 in 100,000
L-----------'--------'---------,~---'-------------------~
t)m))"f1"9 ~L,orr-h
Energetics of Urea Cycle isorders of Urea Cycle
The overall reaction may be summarized as: ~ ~Deficiency of an of the urea cycle enzymes would result
NH3 + CO2 + Aspartate -+ Urea + Fumarate 1" in perammonemi When the block is in one of the
In the urea cycle 2 ATPs are used in the first reac- eru:lier..ste~ q g ition is [1.0@..S.evereofiQ~ ~~~,"1::11
tion. Another ATP is converted to AMP an~ ~ nia itself accumulates. Deficiencies of late~ i met~-1.
equivalent to @ .The urea cycle consumes 4 high result in the accumulation of other intermediates, which
energy phosphate bonds. However, fumarate formed are l~oxic and hence symptoms are less. As a general
in the 4th step may be converted to malate. Malate when ffl~iption, disorders of urea cycle are characterized
'J \
oxidized to oxaloacetate produces 1 NADH equivalent to Y yperammon!,_T ip, ~rnc~ppalopathy agd resp:ratory
2.5 ATP. So, net energy expenditure is only 1.5 high alkc1losis. Clinical sy~ toms include '@Olllio , ircila6iti~
energy phosphates. The urea cycle T c cle are lethargy and severe mental retardation. Infan ts a~ear
interlinked, and so, it is called as "urea bicy~le°y uarmal at birth, but within days progra"ssive leth~

®CB) Regulation of the Urea Cycle


Coarse Regulation
sets in.
nJ~ eat~~n~~ !f~~Y~~~ ~ J~~!:.-.}.ow protein diet
~1tn -sJ1/ib,e~ arginine ancl' energy 'IJ'yfrequent feeding
minimize brain damage since ammonia levels do
During starvation, the activity of urea cycle enzymes not increase very high (Table 17.3).
is elevated to meet the increased rate of protein cata- i. Ornithine transporter deficiency is characterized by
bolism. H ~ ' hyperornithinemiaH ~ p ~nemia and homo-
/::\ F R I t· ,,,..,,,., ,a~~ ci ullinuri]fi:fH~ synatome). ~inc ornithine is not
\.!:)I me egu a ,on ~ - ...,._ 1 - bl . h . h d • c~
~_,_ "L ·
var1a e ,n t e m,toc on na, 1y , e rs carbamylated
The major regulatory enzyme is CPS-I. N-acetyl glu- ~ ~ to form homocitn /llioiei 4-4~-~
tamate (NAG) will stimulate this reactio?'. ,It is forrlfed j'
ii. Ornithine transcarbamoylase deficiency is the
from glutamate and acetyl-CoA. Arginine is an onl~ r~a ~yc/e t¾>\,
'frsich is int}erited as an
Qf f:iAG 5yntha§Le. l,;. ~'t-\J 01'\ ~\, ,tt~ -litikecl ra, . l:J'evate leve s ra~rn~ia are asso-
C rt t I" f 8 ciated with high glutamine levels in CSF and blood.
~ ) ompa men a iza ion iii. Argininosuccinate lyase deficiency leads to
The urea cycle enzymes are located fn such a way that argininosuccinic acidemia and therefore metabolic
the first two enzymes are in the mitochondrial matrix. acidosis. Hyperammonemia is less severe and
The i ~ e on its own formation argininosuccinate is elevated in CSF and excreted
is minimized because ar ininosuccinate I se is in the in ~~.t
ti~ical clinical feature is friable tufted hair
cytoplasm, whil fumarase is in · chondr' ig. 17.11 ). (trictili}rhlx,s nodosa). '--> Na.\ern~ k
(0~ b,Q \
'fu.,m~a..,~ 'mi~/.-
~:i ~t) C~)
Chapter 17: General Amino Acid Metabolism (Urea Cycle and One-Carbon Metabolism) 261

iv. Arginase deficiency is the most mild variety with


accumulation and excretion of arginine are seen. Ammonia, pH, Electrolytes (Blood)
Ketones (Urine)
Symptoms appear by 2-4 years of age.
The accumulation of ~mmonia in blood (normally
less than 50 mcg/dl) and body fluids results in toxic
NH3 f NH3 normal NH, normal / NH3 normal
symptoms. Nowadays, defects in enzymes of urea cycle No acidosis No acidosis acidosis No acidosis
are detected in e n _ d b estimatin.9 metabo- Anion gap
• DNPH +ve(U)
lites b tandem ~a~s spe fro~ try. e V '('..-r,. ketonuria DNPH-ve
Brain is very sens, 1ve to ammonia. Different clini-
cal disorders are shown in Table 17.3. y h.i!g_.may be put
on a low Qroteil]Jliet and frequent small feeds are given.
Attempts may be mad~te eliminate the amino nitrog~n Urea cycle MSUD OAD
defecV (PA, IVA,GA
in ot!1er fo~s. e.~ s liippuric acid (Benzoyl conjugate MMA)
1 'HHH s ndrome
of glycine) or phenyl acetyl glutamine.
Since Citrulline is present in significan.J..q1:.1antities in •Hyperammonemia, hyperomithinemia, homocitruu·
·1 ' •Non ketotic hyperglycinemia
m1 k, br~st milk is tg_be avoided in ~itrulti@.Tj a.

Hepatic Coma ired /r;--,. Fig. 17.12: Differential diagnosis of hyperammonemia


Hypera m nemia) m -m OAD-Organic acid disorders, MSUD- Maple syrup urine dl~ase,
IVA-lsovaleric acidurias, MMA-Methylmalonic acidurias, PA-
-- ---- ,.,,.._._,.._r-y-"
.__,, ' . ---:. / Propionic aciduria. GA- Glutaric aciduria, NKH- Non k totic
In dis _?ses ~f the liyer, hepa~c..tail1;re ; ~)inally lead to hyperglycinemia, HHH syndrome-Hyperammonemia hyperpmi-
hepa ic coma and<death. Hyperammonemia is the char- thinemia homocitrullinuria syndrome, DNPH test-Dinitrophenyl-
acteristic feature of ffverfa~Sre.1Th~ ~ i dJ i~n is also hydrazine test, AA-Amino acid, QA-Organic acid ~\
known as portal systemic ence~al6pa'tfiy~ ormally, the
11
p-,, ~ 0-rrfN½ ifJO JL6\lxrl ·
ammonia and other toxic COJIPOunds produced by intes- . An important point to remember is that ~rea le ~I
tinal bacterial metabolisware transported to liver by por- 1s dependent on age. The value of 20 mg/di 1s seen n
tal cirxulation and det {ified by the liver. But when there young adults, while the va~ue of 40 mg/di i~ for seni r
· ,., ,.. Y ,, J "' - 1 • J..J( .1.Y-,ditizen s ~,;1("~:.~ le~o\ ""' S b~
1s o_ a S! '.S em, 1 ..Q__ me toxins bypass ·
the liver and their concentration in systemic circulatiq~ Urinary excretion of urea is 15 to 30 g/day (6-15 grams
rises. F,~
'(WZ- in ~ nitrogen/day). This corresponds to the breakdown of
The signs and ~Yml?, ms are main~ p;-~aining to @'"o to 80 grams of proteins per da0 Urea constitutes
CNS dysfun_ction {altered sensorium, con~ lsions) f r
1 80% of urinary organic solids.
manifestati~ns_of fpilur~ liverJ ig,\ction (a~ ites, ja'iJn-
dice, hep~ omegaly, ederna~J :fem rrhage~s id !f nevi}, Hyperammonemia
'1'-'
Measurement of ammonia levels is a very critical 8P a-
""' ' -f ,,,
The managemenb9f ,the cbhaiffon 1s difficu1t. Th 0

main lines of treatment are t . ~ ~in diet. 2. Bowel meter for the assessment of a variety of ge~ti£ and
disinfection using antibiotics and clearance by _lactulose. acquired conditions. The most important gen€tic causes

"'00")
3. Avoid hepatotoxjg dru~ . 4 . Maintenance of electro- of hyperammonemia are urea cycle disorders and organic
lyte and acid base balan 'NU 'k~~, acidurias (Fig. 17.12). Organic acidurias usually pre-
. ~e.ch-o.l ~ .e..t-e...~ , sent with life-threatening metabolic acidosis and hyper-
Urea Level m Blood 0n\,:,l:>- ~ ~ Q kO , ~ ammonemia; w hereas urea cycle disorders often have
In clinical practice, blood urea level § ?a~n as an indi- metabolic alkalosis along with hyperammonemia. Ammo-
cator of renal function . The normal urea level in plasma nia levels may be elevated in hepatic coma (hepatic
is from 20-40 mg/di. Blood urea level is increased encephalopathy). In this condition liver function tests will
where renal function is inadequate. Details of causes of also often be grossly abnormal. Other conditions where
uremia are given in Chapter 25. When protein intake is ammonia is elevated include cor pulmonale, pulmonary
high, the urea level in blood will be slightly increased, emphysema and renal failure. Treatment policy is shown
but within the upper limit of the normal values.
Prmm1:1'0, <Ji ~ - ,
L.:+.:Y~,:-hepatic coma.
-Fd\on
\ ) ~ Q_ - L l~
262 Section 8 : General Metabolism

Table 17.4: One-carbon compounds One-carbon One-carbon units One-carbon


donors or with carriers utilization
Group Structure Carried by generators

1,).. Formyl - CHO N5- formyl-THFA and N' 0- formyl-THFA

j Formimino -CH=NH N5-formimino-THFA Formate -...,. . _ _ _ _ . C.92 + H2O


C .3 Metheny! =CH- N5,N •0 - methenyl- THFA Tryptophan- Fonnyl THFA -.............. - C 2 p~ rine

ll
17 • Hydroxymethyl -CH 2OH N10 - hydroxymethyl- THFA
~Formyl
G $ Methylene - CH2- N5,N10 -methylene-THFA N Methiony1 RNA

F- 1 Methyl -CH3 N5- methyl- THFA and methyl Histidine~ F6nnimino ,


cobalamin THFA -+ Metheny! THFA -+ C 8 purine

ll
l '.
Choline ~ Hydroxy-+ Methylene- + Glycine
Serine Methyl THFA Se ne
N THFA dU P-+ dTMP
Reductase

Melhy1THFA

E
T nsmethylation
S rlne _,. Choline
---• Creatin
Fig. 17.13: Tetrahydrofolic acid (THFA). Methylene group is atta-
Epinephrine
ched to N 5 and N 10 methionine

Fig. 17.14: One- rbon gene lion and utilization


B,1. ,
l9NE-CARBON METABOLISM N. c_doo..\ o..W'I"°("',
3. Histidine con ributes to N5-formimino-THFA through
One-carbon (1 C) groups play a pivotal role in donating
FIGLU (see Chapter 19).
carbon atom~ for synthesis of different types of com- 4. Tryptophan donates formyl-THFA (see Chapter 19).
pounds. The different one-carbon groups of the 'one- 5. Choline and betaine are donors of hydroxymethyl
carbon pool' of the body are: groups (see Chapter 18).
1. Formyl group As serjne js ei111v1 rte$:chojjne, 3 one-carbon units
2. Formimino group are used up. During the conversion of
3. Metheny! group these methyl groups are recovered (see Chapter 18).
4. Hydroxymethyl group Hence this pathway is called the "salvage pathway" for
5.~ Methylene group one-carbon units.
. ' Methyl group (Table 17.4 .
C .J The one-carbon grou s, ~c_e_p_t_m
- et_h_y_l _g-ro_u_p_,-a-re Interconversion of One-Carbon Groups
(
0
?;if came y etra- hydrofolic a · (THFA). THFA is pro- The different one-carbon groups are interconvertible
. uced-f om folic acid (see Chapter 33) . .t!_: ang_N10 aJQ!!1s as shown in Figure 17.14. & one-carbon....units are ulti-
ofJHl:fr. carry the,2_ne-carbon groups. The attachment of mately siphoned into methyl:rHFA. This is because, the
methylene (-CH2-) group is shown in Figure 17.13. reduc~se repctip~ methylene to methyl group (Fig.17.14)
is an(irrev~rsib e step/
Generation of One-Carbon Groups From methyl-THFA, the B1l coenzyme accepts the
The one-carbon groups are contributed to the one- methyl group to form methyl cobalamin. It then transfers
carbon pool by amino acids. the methyl group to homocysteine to form methionine. In
1. reaction (see Fig. 18.1) is the 8 12 deficiency, deficiency of folic acid is also observed;
primary contributor for methylene THFA.t~ ) this is because, tra sfer th I 9!.QJJP-.1(Qm

-
2. Glycine cleavage system also produces~Tethyl~ methyl-THF ccur. THFA is not regenerated;
groups (see Fig. 18.3). this is called folate trap (See Folic acid in Chapter 33).
Chapter 17: General Amino Acid Metabolism (Urea Cycle and One-Carbon Metabolism) 263

Utilization of One-Carbon Groups hyperargininemia, which is the least severe). NAG syn-
thase (NAGS) dleficiency mimics CPS-I deficiency.
A summary of the generation and utilization of one- Clinical symptoms of urea cycle disorders are leth-
carbon groups is shown in Figure 17.14. The one-carbon argy, anorexia, hyperventilation or hypoventilation, hypo-
units are used for synthesis of the following compounds: thermia, seizures and coma. Hyperammonemia may be
1. C2 of purine { ~m@ j triggered by illness or stress, which along with a normal
2. Formylation of methionyl tRNA anion gap, is tt1e classical hallmark of urea cycle dis-
3. CS of purine £S ,e,} order. Definitive: diagnosis depends on enzyme assay
4. Gly~ine (see Fig. 18.1)Jl p ~\ from liver biopsir specimen and genetic analysis.
5. Senne J · 1 Treatment includes dialysis to reduce ammonia
6. Choline (see Chapter 18) (F-) levels, intravenous arginine chloride and nitrogen scaven-
7. Deoxy TMP... '"''11),&J ger drugs (phenyl acetate, benzoate) to activate altera-
8. Transmethylation reactions including creatine, tive pathways for ammonia excretion, and protein
choline and epinephrine synthesis restricted diet. Chronic therapy includes protein restricted
9. Excreted as carbon dioxide. diet, phenyl bultyrate, arginine, citrulline supplements,
0 and if necessary, liver transplantation. Genetic coun-
•• Clinical Case Study 17.1 seling and prenatal diagnosis are advised.

A 6-month-old infant began to vomit occasionally and


ceased to gain weight. At 9 months of age he was
LEARNING POINTS, CHAPTER 17
readmitted to the hospital. Routine examination and labo- 1. Proteolytic enzymes are secreted as inactive zymo-
ratory analysis were normal. After one week, he became gens. This prevents autodigestion of the secretory
drowsy, had fever, pulse was elevated, and there was acini. As for example, Trypsinogen. The chief cells
hepatomegaly. EEG was done and was grossly abnor- of the stomach secrete pepsin. Optimum pH for its
mal. Blood ammonia was elevated and urine contained activity is 2.0.
high amount of glutamine and uracil. What is the prob-
2. Pepsin catalyzes hydrolysis of the bonds formed by
able diagnosis? What is the pathogenic mechanism
carboxyl gri0ups of Phe, Tyr, Trp and Met.
involved?
3. Activation of trypsin is by 'enterokinase' present in
0 the intestin:al microvilli.
• Clinical Case Study 17.1 Answer 4. Acute pancreatitis is a condition resulting from
Hyperammonemia in this patient may be due to urea premature activation of the zymogen trypsinogen
cycle disorder. Excessive excretion of uracil and/or orotic inside the pancreas itself.
acid results from accumulation of carbamoyl phosphate, 5. Trypsin catalyzes hydrolysis of the bonds formed
and is due to absence of the enzyme ornithine transcar- by carboxyll groups of Arg and Lys.
bamoylase (OTC). Glutamine is normally converted to 6. Important jproteolytic enzymes of the body are
glutamate and ammonia by the kidneys; but when the
Pepsin, Trypsin, Chymotrypsin, Carboxypeptidase
level exceeds, it is excreted in urine.
A and B and Elastase.
Ornithine transcarbamoylase (OTC) deficiency is
7. Amino acids have five different transport systems
inherited as X-linked dominant condition and the disease
for their absorption viz. for neutral amino acids, for
is more severe in males. Drugs like valproate and corti-
basic amino acids, for imino acids and glycine, for
costeroids increase the risk. Other causes of urea cycle
disorders are carbamoyl phosphate-I deficiency (cha- acidic amino acids and for beta amino acids.
racterized by severe hyperammonemia in the newborn 8. Absorption of neutral amino acids in renal tubules
period), argininosuccinic acid synthetase deficiency and brain is by the 'gamma-glutamyl cycle'.
(leading to citrullinemia and severe hyperammonemia), 9. Transamination is an important general metabolic
argininosuccinic acid lyase deficiency (leading to argini- reaction of amino acids, where one amino acid can
nosuccinic aciduria, with rapid onset hyperammonemia transfer its amino group to another alpha-keto acid
in neonatal period) and arginase deficiency (leading to to form a 1new amino acid and a new keto acid.
264 Section B: General Metabolism
Transaminases require pyridoxal phosphate (PLP) 25. The first step of urea cycle occurs in the mitochon-
as cofactor. dria and uses 2 molecules of ATP to form carba-
1O. Reactions that form ammonia in the body are moyl phosphate from NH3 and CO2 •
transamination, transdeamination, oxidative deami- 26. The carbamoyl phosphate combines with omithine
nation and nonoxidative deamination. to form citrulline.
11 . First line of defense against ammonia toxicity in the 27. Citrulline comes out into cytoplasm and further
body is its trapping by glutamine, especially in the reactions occur in the cytoplasm.
brain cells. Second line of defense is constituted by 28. Then aspartate condenses with citrulline with hy-
the urea cycle. drolysis of two high energy bonds to form argini-
12. The two nitrogen atoms of urea are derived from nosuccinate. The carbon skeleton of aspartate is
ammonia and aspartic acid. released as fumarate that enters the TCA cycle.
13. Carbamoyl synthase-I is the rate-limiting enzyme of 29. Arginine is then cleaved to urea and ornithine by
the urea cycle. The reaction is irreversible. arginase, an enzyme present only in liver.
14. Deficiency of all the urea-cycle enzymes is known. 30. Urea cycle enzymes are activated during starvation
They constitute the set of disorders called 'urea- since rate of protein catabolism is more.
cycle defects'. 31 . The regulatory enzyme is CPS-I; it is activated by N
15. Normal plasma urea level is 20 - 40 mg/di. acetyl glutamic acid.
16. The one carbon units are; N5 -formyl-THFA, N10- 32. Presence of more than 60 micromoles/L of urea in
formyl-THFA, N5 N10-Methylene THFA, N5, N10-Methe-
blood and body fluids is toxic to the brain.
nyl-THFA and N5 -methyl-THFA.
33. Liver fai lure can also cause hyperammonemia
17. Proteins undergo regular turnover in the body. Pro-
which will result in hepatic coma.
teins to be degraded are tagged with ubiquitin by
34. Estimation of urea in blood is a very common test
specific enzymes. The ubiquitinylated proteins are
done in the laboratory to assess the kidney function.
degraded in proteasomes.
35. Serine is the most important donor of one-carbon
18. Cathepsins are lysosomal hydrolases which also
pool. Degradation of glycine (N 5 , N10-methylene),
have an important role in intracellular protein deg-
histidine (formimino) and tryptophan (formyl) can
radation.
also contribute to one carbon pool.
19. Amino acids take part in anabolic reactions to syn-
thesise proteins, peptides and other nitrogenous 36. From the N5-methyl-THFA, the methyl group
compounds. can be transferred to cobalamin to form methyl
20. Essential amino acids are those whose carbon cobalamin .
skeleton cannot be synthesized in the body and 37. The only acceptor of this methyl group from cobala-
has to be supplied in the diet. min is homocysteine, which is then remethylated to
21. Transamination is also important in the synthesis methionine.
of nonessential amino acids and interconversion of 38. This reaction is important for the recycling of THFA
amino acids. and hence 812 is an essential cofactor in one-
22. Clinically important transaminases are ALT and AST. carbon metabolism.
23. Glutamic acid and glutamine are the nontoxic trans- 39. Major use of one-carbon group is for the synthesis of
port forms of ammonia in plasma. purines and pyrimidines for nucleic acid synthesis.
24. The urea cycle occurs only in the liver and is a partly 40. Salvaging of one carbon groups can occur by cho-
mitochondrial and partly cytoplasmic pathway. line -betaine interconversion.

PART-1: ESSAY AND SHORT NOTE QUESTIONS


17-1. Explain the term transamination. Give one suitable example. What is the metabolic importance of transamina-
tion? What is the clinical application of transaminase estimation?
17-2. Describe the reactions of the urea cycle. Discuss the interrelation of urea cycle and citric acid cycle.
17-3. Give details of the steps by which ammonia is detoxified in the brain and in liver.
17-4. What is uremia? Describe the reactions of urea cycle. Discuss the diagnostic significance of blood urea.
Chapter 17: General Amino Acid Metabolism (Urea Cycle and One-Carbon Metabolism) 265

SHORT NOTE QUESTIONS

17-5. Proteolytic enzymes of gastrointestinal tract. 17-8. Transdeamination.


17-6. Digestion of proteins. 17-9. Name one-carbon units.
17-7. Transamlnation. 17-10 Decarboxylation of amino acids.

PART-2: MULTIPLE CHOICE QUESTIONS

17-1. Which enzyme is an exopeptidase? C. Both from ammonia


A. Trypsin B. Chymotrypsin D. Ammonia and ornithine
C. Pepsin /4 Carboxypeptidase 17-8. Ammonia is trapped in brain by:
17-2. Trypsinogen is activated to trypsin by removal of a A. Glutamine synthetase reaction
hexapeptide by : B. Glutaminase reaction
A. Pepsin B. Chymotrypsin C. Urea synthesis cycle
e-:- Enterokinase D. Pancreozymin D. Glutamate dehydrogenase reaction
17-3. All the following are transamination rea ctions, 17-9. Normal blood urea level is:
except: A. 10-20 mg/di B. 20-40 mg/di
A. Pyruvic acid to alanine C. 20-40 g /di D. 30-60 mg/di
B. Oxaloacetic acid to aspartic acid 17-10. Blood urea level is markedly increased in:
..Z: Hydroxypyruvic acid to serine A. Liver diseases B. Renal diseases
8 . Glutamic acid to glutamine C. Cardiac diseases D. Protein intake
17-4. All are true with regard to transamination reactions, 17-11 . The major donor of carbon atoms to the one-
except: carbon pool is:
A. Serine B. Tyrosine
A. It is the starting step of metabolic degradation of most
C. Threonine D. Proline
of the amino acids
17-12. All the amino acids contribute carbon atoms to the
B. This process synthesize non-essential amino acids
one-carbon pool, except:
C. It requires pyridoxal phosphate as coenzyme
A. Tryptophan B. Histidine
D. Reaction takes place only in liver
C. Valine D. Serine
17-5. Ammonia is formed with the help of all the follow-
17-13. All the following gastrointestinal enzymes are
ing enzymes, except:
secreted as proenzymes (zymogens), except
A. Transaminases
A. Trypsin B. Chymotrypsin
B. Glutamate dehydrogenase
C. Pepsin D. Ribonuclease
C. Amino acid oxidase
17-14. Succinyl-CoA is formed from following except:
D. Threonine dehydratase
A. Valine B. lsoleucine
17-6. Which amino acid is oxidatively deaminated in
C. Propionyl-CoA D. Aspartate
liver?
A. Aspartic acid B. Alanine 17-15. Pyridoxal phosphate is the coenzyme of which
C. Glutamic acid D. Valine reactions?
17-7. During urea cycle, the two nitrogen atoms are deri- A. Transamination reactions
ved from : B. Glutamate dehydrogenase
A. Ammonia and arginine C. L-amino acid oxidase
B. Ammonia and aspartic acid D. Formimino glutamic acid to glutamate

ANSWERS OF MULTIPLE CHOICE QUESTIONS


17-1. D 17-2. C 17-3. D 17-4. D 17-5. A 17-6. C 17-7. B
17-8. A 17-9. B 17-10. B 17-11. A 17-12. C 17-13. D 17-14. D
17-15 A
266 Section 8 : General Metabolism

PART-3: VIVA VOCE QUESTIONS AN ANSWERS

17-1. What is the action of pepsin? 17-17. What are the physiological significance of trans-
It hydrolyses peptide bonds formed by carboxyl groups aminatiori1?
of Phenylalanine, Tyrosine, Tryptophan and Methionine. It synthesises nonessential amino acids; it is the first
17-2. What are zymogens? step of amino acid break down pathways.
They are proenzymes; inactive at the time of secretion, 17-18. What is the clinical significance of transamination?
but will be activated in the gastro intestinal tract. Transaminases are increased in blood in liver and car-
17-3. What is its biological significance? diac diseases.
Zymogens prevent autodigestion of the cells. 17-19. What is transdeamination?
17-4. What is its clinical significance? Transamination takes place in all the cells of the body;
Acute pancreatitis results when trypsinogen is activated
the amino group is transported to liver as glutamic acid
prematurely.
which is finally oxidatively deaminated in liver. Thus
17-5. What are the enzymes in pancreatic juice?
the two components of the reaction are physically far
Pancreatic juice contains the endopeptidases trypsin,
away, but physiologically they are coupled. Hence the
chymotrypsin and elastase.
term transdeamination
17-6. How trypsinogen is activated ?
17-20. Which amino acid is oxidatively deaminated in liver?
Removal of a hexapeptide from N-terminal end by
enterokinase (enteropeptidase) Glutamic acid.
17-7. What is the action of trypsin ? 17-21 . Nitrogen atoms in the urea is derived from what
It hydrolyses peptide bonds formed by carboxyl groups precurso1rs?
of arginine and lysine. One from ammonia and another from aspartic acid.
17-8. How chymotrypsinogen is activated? 17-22. Ammonia1is trapped in brain by what?
It is activated by trypsin. Glutamine! synthetase.
17-9. What is an endopeptidase? 17-23. What is the key enzyme of urea synthesis?
It acts on peptide bonds inside the protein molecule, Carbamoyl phosphate synthetase.
so that the protein becomes successively smaller and 17-24. What are t he two carbamoyl-hosphate synthetases?
smaller units. CPS-I is involved in urea synthesis; CPS-II is required
17-10. Give some examples of an endopeptidase. for pyrimidine synthesis. CPS-I is seen in mitochon-
Trypsin, pepsin. dria, while, CPS-II is in cytosol.
17-11 . What are exopeptidases? 17-25. What is the normal blood urea level?
They act at one end of the protein molecule, liberating
20-40 mg/di.
amino acids sequentially, one at a time.
17-26. Blood unea level is markedly increased in which
17-12. Give an example.
condition?
Carboxypeptidase.
Renal diseases.
17-13. Carboxypeptidase contains which metal?
17-27. Tetrahydrofolic acid is used for what purpose?
It is a metalloenzyme containing zinc.
It is the ca1rrier of one carbon compounds.
17-14. In fasting state, nitrogen is transported from mus-
cle as what form? 17-28. Name on1~-carbon compounds?
In fasting state, the muscle releases alanine and gluta- Form,:I, formimino, methenyl, hydroxymethyl, methy-
mine of which alanine is taken up by liver and gluta- lene, and methyl.
mine by kidneys. 17-29. Which an~ the donors to one carbon pool?
17-15. Transamination of glutamic acid produces what? Serine, choline, glycine, tryptophan, histidine.
Alpha ketoglutaric acid 17-30. One carbon units are used for what?
17-16. What is the coenzyme for transamination reaction? C2 of purine, CB of purine, serine, choline, creatine,
Pyridoxal phosphate epinephrine.
Aliphatic Amino Acids

Chapter at a Glance

The learner will be able to answer questions on the following topics:


Glycine O Cystinosis, Cystathioninuria
Creatine, creatine phosphate and creatinine O Glutamic a,cid, Glutamine .,
Serine, choline Aspartic acid, Asparagine /
0 Alanine O Lysine •
Methionine Arginine, nitric oxide
Transmethylation reactions Ornithine, IPOlyamines •
Cystei ne, Glutathione D Valine, leuc:ine, isoleucine •
0 Homocysteine and homocystinurias

---Am \~o<:>cs · ,:-::,~...., me\a.h .,, &~edi -1 \f ).),Jc9e


I GLYCINE (GLY) (G) =- .,. ~J. -r :trop one-carbon unit;. This is the reversal of the glycine
l'\.0,-----:g;r~ cleavage system (Fig.18.3).

glucogenic. Glycine is formed: r~


It is the simplest amino acid. It is<@on-essent,~ and is
+ f -C un;'i}
1. From Serine. The beta carbon of serine is removed
4. Glycine amino transferase can catalyze the syn-
thesis o glycine.J!91+1-glyoxylate and glut,a/!1~~ or
alanine. This reac ,on f[QTTgtyjayQLS-sYn~ s of
which enters the one-carbon pool with the help of 1 ·
g ycme. (]'
t • • n..-u>4 -lL... q
c.ltJa,"'-
'!'.......
THFA (tetrahydro-folic acid). The alpha carbon of ~t""'j ...,,.. J
serinebecomesthealphacarbonofglycine(Fig.18.1). Utilization c:>f Glycine
2. From Threonine by the activity of threonine aldo~ / . Cl S t
lase (Fig.18_2). eu,dJ:>\ c.8'<"d..t.~~1
yc,ne eavage ys em
3. Glycine synthase. Glycine can be synthesized by Glycine underg1oes oxidative deamjnat[Qn (reversal of
the glycine synthase reaction from CO2 , NH3 and glycine synthase) to form NH3 , CO2 and the one-carbon

(~~N'I~)
Serine hyaroxymethyl transferase coo-
COO- Glycine
l
~-t~ _
+ I +
Serine Glycine CH- NH Threonine
3 CH2- NH3
____;ca_ld_ol-'-
as'""e_ _
coo- coo- +
l
CH-NH 3
+ l +
CH2-NH 3 I @) CHO
CH20H
CH3 I Acetaldehyde
Threonine CH 3

Fig. 18.1: Formation of glycine from serine Fig. 18.2: Formation of glycine from threonine

f\J ~ E...~ ( ~~IQ_ Pi'\ao\


268 Section B: General Metabolism

Proteins Proteins
Glycine
i l~ r-
Glycine ~eavage system
Serine - G ......,____-.......,.
ammonia -+ Urea
~~]' ...,_ 1 carbon unit
-""(:,.\ul!lic.~"lolt- I Serine -+ Pyruvate -+ Glucose
Amino methyl group "'J Cysteine
, Choline
+ CO2 + Glyoxanc acid -+ Oxalate
THFA ( t·,,o
) amlde * NADH+H NH.c3 +} Creatin~-+ CP -+ Creatinine
@ 0 0
Reduced +
1
c;.'c.;#
5 7 of purine ring C l.~g
·Tie!>
Melhylene lipoamide NAO ALA -+ Heme
v THFA tA~~~ ,. :r •
) ')-"" f Cl> Glutathione
. NH3 H11- .., . ._ L ~(.,J.J}>
...< \,v
'<~ -t::-
t
r....eS ·~ • ,..,., ~t>l.,) N\'f:
) Bile salt
v~• A= Glycine decarb<?~ase B = Aminomethyl transferase· ( liI> ) ..__ _ _ _ __ ___c_o_n_iu_ga_ti_
·o_n,_e_.g_.h
_i_
pp_u_ric_ ac_id_ ___,
C = Meth lenelf'Al..?t, nt as D = Lipoamide dehydrogenase; Fig. 18.4: Overview of glycine metabolism
THFA = Tetrahydrofolic acid; PLP = Pyndoxal phosphate
Fig. 18.3: Glycine cleavage system. Glycine is completely deg- total muscle weight. It is synthesized from 3 amino acidsJ
raded to CO2 , ammonia and one-carbon unit methylene THFA. ~ e. argioia.e and methionine. ('; r r
The reactions a.qi~ · reversible, when the enzymes are A. First step (Guanidoacetic acid); The amidino group
1
together calle~d~G~1~ · ~ ~ ~I"'
of arginine is transferred to glycine to form guanido-
acetic acid, catalyzed by arajdo transfetase·(Step 1,
unit methylene THFA (Fig. 18.3). This pathway is the
Fig.18.5). It is seen in mitochondria of kidney and
major catabolic route cine. The glycine cleavage
pancreas, but not in liver.
system is ulti-enzyme co It needs the,.w -
B. Second step (Creatine): Guanido acetic acid is
enzymes, 'A , lipoamide, tetra ydrofolic acid and p?ri-
m;!9Ylated lby S-adenosyl methionine (SAM) by
doxal hos hate. . ,.,_ u- re-"" w,\J.,t) _ o ~;_
met~mJ§:~e to form creatine. This methyla-
t::':)..
,." ,_ .. --,
, -¥'
b> • J \:-o ·1 tion reaction takes place in liver (Step 2, Fig.18.5).
Vl!l2.Gluc;og'fmi9c ath~ay s lAP- ..Q..."'p C. Third step (Creatine phosphate): Creatine is phos-
Glycine is~ airfty7hatm~ci into the glucogenic pathway phorylated to creatine phosphate (Step 3, Fig.18.5).
by getting first converted to serine. This is the reversal Th~ enzy~j ere~ ine kinase (CK) is present in
of serine hydroxy methyl transferase reaction (Fig.1 8.1 ). muscle, brtfin ana Iver. The reac . n .eds hydro-
The serine is then converted to pyruvate by serine dehy- lysis of ATP . The stored~ atine ho h e in the
dr~tase (Fig.18.9). b ~'rm'""' ,S S. ') u > f?
muscle serv«:?s as an immediate s
4,,~ he muscle. During muscle contraction, the energ•y
·

Special Metabolic Functions of Glycine ,tv.c,uc<.. is first derived from ATP hydrolysis. Thereafter, the
ATP is regenerated by the hydrolysis of creatine
Glycine may be used for the biosynthesis of the follow-
phosphate (Fig.18.6). This is called the Lohman n's
ing compound§ (Fig.18.4 ):
reaction.
i. Creatine, creatine phosphate and creatinine D. Fourth step, (Creatinine): The creatine phosphate
ii. Heme µ~ 9.1.,.o.9- .
may be conv,erted to i s anhy_g_ri9e, creatinine (Step 4,
iii. Purine nucleotides rrein --~~ " ~e. Fig.18.5). It is a~ -enzymatic. spontaneous reac-
iv. Glutathione ha.n..,,-cc...."', tion. Creatinine is excreted in urine. The blood level
v. Conjugating agent
vi . Neurotransmitter. lt--o..e. J.o G.w.. I ½ of creatine and creatinine, and urinary excretion
of creatinine are more or less constant. But creati-

l
nine level is dependent on the total muscle mass.
CREATINE AND
CREATINE PHOSPHATE Clinical Applications
The word creati e is derived from the Greek term, kreas, ~ ; serum creatinine level is 0..Z..toJA..mgl.dl and
which means flesh. Creatine constitutes. about 0.5% of serum creatine level is Q2=0_4 mg/di. Urine contains
-OM--:
0 L.,, m'- .
C,-'ll"tat\'Af. f" .3l' 'lt'11\ C..~0$ln;-r-a,

,, 1s7rg
rc')Qn
,

,..., Creatine kinase


Creatine~--+ Creatine
CK
Creatine
Glycine Arginine ( phosphate
ATP .A,DP ADP ATP
+
Energy for muscle

Fi g. 18.6: Creatine phospha e, Lohmann's rea~ f, -


('J2 t~'<l:.~ ( _-....,...,.~ ""'- _. __,,,
synthesi~ or transport pres nt wi ti severe neurological
symptoms and .t{grofound depletion of brain creatine.
Ornithine
~eS-s-'"I G.~c\""" l.> 1 1 ' ~ ~ ¥
Synthesis"<>f Heme -D
The enzyme ALA synthase condenses glycine with
coo-
l succinyl-CoA to form delta amino levulinic""acid (ALA)
CH 2 (see Chapter 22'.). It is t key enzyme of heme synthesis.
I SAH 1~
N@:;) Synthesis of Purines
I Creatine
C=NH c-c.,·"' c\,i;,•;' The whole molecule of gl>:3iP,P. is incorporated into the
I I

NH 2
ATP g-rs(»-'O' purine ring (C4, C5 and N7 ) (s~~apter 38).
, A-1f l
"\r-t 9 ~~R
Synthesis of Glutathione .._
J.D\luJ\OM
- ~ OC
coo A DP CC~ 9 I Glutathione is ;a ! ( i ~ formed from glutamic acid,
~H -4 '--i C~~H2 cystei_ne a~d @~ The functions of glutathione are
2
I Creatine Creati- ' N-CH3 described In thei section on metabolism of cysteine.
N-CH3 phosphate_ nin\ I
I ('~ C=NH Glycine as a Conjugating Agent
C=NH m:M) -~ Pi+H20 I
, - svJl(v-, , N A. Bile acids . Glycine is used to conjugate bile acids,
NH-Po; \,l}V~ H
to pr~duce: pile salts. ~yeocbalic acid and EtC't>~
_s.be~ ~ acid are the main conjugated
bile a ids (see Chapter 14).
B. Benzoic acid. It is used as Q!eservative in foods.
4 = Spontaneous (non-enzymaticl,
SAM = S-adenosyl methionine
Glycine is iused for detoxification of benzoic acid to
- SAH = S-adenosyl homocysteine form hippuric acid (~ Chapter 36).
Glycine: + Benzoyl-CoA -+
Fig. 18.5: Creatine metabolism •
>v(l..(l.0 + NC\Av-.e-. Benzoyl glycine (hippuric acid) + CoA
ne ~e amounts of creatine in normal males. But in T~ a tion occurs in liver and so it is an index ~a\)
m ~r dystrophies, the ~od c~atine and urinary of~ r functic~. Hippuric acid was first isolated from
~~::!!~jg_.c.reased. Creatinine level in blood 1s a~ ~ e ~ j hence the name.
indicator of renal function (see Chapter 25). The enzyme~
CK is clinically important as it is elevated in myocardial Glycine as a Neurotransmitter
infarction (see Chapter 6).
The spontaneous loss of creatine and of phospho-
creatine to creatinine ~uires that creatine be continu-
ously replaced ; this occurs by a combination of diet
and efl.dogenous synth_!!sis . ~tiaos 9btairi.-aJJ:oost
no d~ reatine synthesis makes major Glycine as a Constituent of Protein
demands on the metabolism of glycine, arginine, and
methionine. Children with inborn errors of creatine In collagen, ev~~ 3rd :_Q}ino acid i€. i : ?

-~~-
\Jn..A.. 6
l ~ _., rr...,.q~
Cn.a}\n,l\ e. Re.J'\O.}~
... __\•,
\~
. , C K.
270 Section B: General Metabolism

Dehydrogenase
3-phosphog.occerato ( \ • 3-phosphohydroxypyruvate

______ _ _ _ NAO+ _ NADH+H+_8 ______ _ _ _


Phosphoserine
COOH
I
COOH
I
transaminase
3-phosphohydroxy- _ _ ___,,_~"---+•
7
Phospho~erlne
CHO COOH pyruvate I •
___ _ _____Glutamate __ _ Alpha keto !lutarate __ _
1 = alanine glyoxalate aminotransferase
2 = glyoxalate reductase Phosphoserine phosphatase
Phosphoserine + H 2O - - - - - - Serine + H 3PO 4
Fig. 18.7: Cause of oxaluria. Enzyme 1 is located in Peroxisomes,
but in patients it is located in cytoplasm and hence inactive F ig . 18.8: Formation of serine
T- ,~\c..~t..,, o-.6 ..... ~1.,,.u
@ Metabolic Errors io Relation to Gl~~ ERINE (SER) (S) ...
@ N onketotic Hyperg/ycinemia (NKH): Serine is an aliplhatic hydroxy amino acid. It is non-
It is due to defect in glycine cleavage system. Gly- essential and glucogenic. Sources of serine are:
ci~ level is inc'.eased in bJaQd, urine and CSF. Severe 1. Ph~_sp~ogly 1C?.!::r::~: This is th_e major source of
m~ tal retardation and se~ es are seen. There is no serine in the body. The steps involve dehydroge-
effective management. Diagnosis is achieved by~ - nation, transamination and removal of phosphate
teci CSEtblaad gtycioe catjQ. Diagnosis is importl ~r group (Fig .18:.8). " \ H , ' Yt"It \
, ..;:: genetic counseling. A variant of this condition is i~ 2. From gJycinH by reversal of serine hydroxymethyl
1
-t'-~t- nonketotic hyperglycinemia. Hyperglycinemia Is also transferase maction (Fig.18.1).
- seen in organic aciduria@. 3. Serine may also be formed by transamination of
f oP-°' ....,, ...,.!J,~ hydroxypyruvate with alanine.
(P~Di)Primary Hyperoxaluria (~ P'" 1 Q,~ Alanine + t:iydrox pyruv~-+ Pyruvate + Serine
Increased excretion of oxalates is_p_p"€erved (up to 600
mg/day, compared to a normal o~ mg/day). The oxa- Catabolism qf Serine
luria is dJ.!.e to increased__proouction of oxalates. It is an 1. Deamination to pyruvate (Fig. 18.9)
autosomal recessive trait. isease is due to a pro- 2. Transaminati,on to hydroxypyruvate
tein tarqetting defect. ormall the enzyme alanlm
3. Serine is glwi::ogenic.
~c;·Pt, )glY.oxalate amino transf e no.1 in Fig.18.7) is loca- ,....• 1'>0 P.,,,J!!.J,r-.
ted in hepatic peroxisomes; but ~ the Metabolic Funclions of Serine
enzyme is pwseot io rnitachoQ.Qria (see also Box 41.1).
1. One-carbon group: Serine donates one-carbon
So, enzyme is inactive. This leads to excess production
group to the one-carbon pool. One carbon is r~ -
of~ ~alate. Renal r~~position of ~ lates would cause
oved from serine, and glycine is formed (Fig. 18.1).
neplirolithiasis, renal colic and hematuria. Extrarenal
Q)(ajpsis.ma}'.' be seen in beart,_ blood vessels, bone,etc.
2. Cysteine: Serine is used for the formation of
cysteine (Fig.18.14 ).
T e 2 primary peroxaluFia is a milder condition
Serine + H.omocysteine -+ Cysteine + Homoserine
c~using only u · hI aA~ results from deficient acii- t'" _
3 Alanine: Serine is converted to alanine by dehy-
v1ty of r.oxalafe oxid . 2 in Fig.18.7).
dration followed by transamination.
The principle of management crease oxa ate
4. Phosphatidyl serine (see Chapter 8).
ex..crmi.Qn by in&reased water intake. Also try to minimize
5. Drugs: Serine analogs are u ed as drugs and they
dietary · take of 'f ates by re~ ~fa--in!~ of
leaf~ veQ'6ta,t>~ ~- sesame seeds, t~ e\:rrlot, - - -
inhibit nucteaitide s nth.esis. Az.a.serine is an anti-
cancer drug and c closerine is an antituberculous
sp~Rach , rhJtarb, etc. In normal persons, oxalate can
drug.
arisefrom
a. glyoxalate m.!:labolism,
b. from ing~stion of I~ vegetables
Choline Synthesis
c. from as'corb1c acid degradatiori7'he third source is A. Serine is ecaFsoxylated to ethan61amine by a
minimal in human beings. pyridoxal phosphate dependent decarboxylase
(PLP) Serine-
dehydratase
coo
I +
' . CH2-NH 2

r\
Serine decarboxylase

\
Serine lmino acid Pyruvate CH-NH3

\
I I
coo- COCJ coo- CH2-OH
(PLP) CH2-OH
I -+ H2O I H2O NH3 I
Serine
CO2
CH-NH3 C = NH C=O Et hanolamlne
I I I
CH 2-OH CH3 CH3 Fig. 18.10: Decarboxylalion of serine

Fig. 18.9 : Deamination of serine to pyruvate


Protein
Phospho- t i
glycerate --+ S enne
+ COOH CO2

---k> Alanine/ /"'


Glycine Serine Ethanolamine Pyruvate -+ Glucose
Glycine /
Ethanolamine -+ Choline
+CH, Choline-+ Acetylcholine
Phosphatidylserine-+
Sarcoslne or / Methyl other phospholipids
methyl glycine ethanolamine

CH2 0H --+ i~~n


pool
-+ + CH3 1® Sphingosine
Cysteine
Selenocystelne

£>imethyl glycine / \ imethyl ethanolamine F ig . 18.12: An overview of serine metabolism


OOC-CH2-N\CH3 CH2OH-CHrN- CH 3

·
tD CH3
CH3 +CH 3
1 "'CH3
®
is added by protein kinases which are se~/threonine
kinases. In gJYJcoproteins, the C1:f ~ y ~ o up
are usually attached to the hJ2!oW rou s of selioe
or threonine residues of the protein. Serine forms the
Betaine Choline or active catalytic residue of many enzymes (serine pro-
or trlmethyl glycine trimethyl ethanolamine
teases), e.g. trypsin and coagulation factors.
An overview of serine metabolism is given in Figure
18.12.
/
SelenocystE!ine (SeCys) OOC- C - C -S,eH
, 'H
Fig . 18.11 : Glycine~( e-cholin (21st amino acid)
~e.,~dono Selenocysteine i s abbreviated as SeCys. It is seen at
(Fig.18.10). Figure 18.11 -s ows the addition of 3 the active site of the following eri.,zymes: a) Thioredoxin ' ~~-~J
methyl groups to ethanol amine to form choline. ''~'..,-- v -
reductase; b) Gl~ thione peroxidase, which scaven-
B. Choline is used for acetylcholine synthesis, which ges peroxides; c) De-iodinase that removes iodine from
is an important neurotransmitter. thyroxine to malke triiodothyronine and d) p ele.nopco.:
C. From choline, 3 one-carbon groups (-CH3) can be tei n~ . a glycoprotein seen in mammalian blood. Their
removed. So, choline is an important one-carbon concentration fallls in selenium deficiency.
donor (Fig. 18.11 ).
-
Its structure is COOH-CHNH 2- CH,-SeH. Biosyn-
thesis of selenocysteine is by two steps: • I

Serine as a Component of Protein


In phosphoproteins, serine serves to esterify phos-
Se +ATP-+ Se-P + AMP + Pi
Serine+ Se--P -+ SeCys + Pi
- ~· l.,L

Me~\,e:::;
.
phate groups , e.g. casein. Glycogen phospj,orylase is SeCys is inserted as ~uc durini p,i:otein _biosyn-
activated by phosphorylation, while pyruv5ttkinase and thesis. Its genetic code is,__Y~A. The tRN~~~c rs first
r i
phosphofructok inase-2 are activated by aep ospho~ ctiarged with smi~ then it is-'c2nverted into SeC_y..s.
lation. This covalent modification serves as a mecha- This is then i~~~dJ_nto the correct ~ ition, when pro-
nism of regulation of enzyme activity. The phosphate group tein is synthesiz,ad.
272 Section 8: General Metabolism

~ LANINE (AL~A) _ Meth ionine S-ade nosyl


Alanine is a non-essential glucogenic amino acid. homoc ys teine
(SAH)
Alanine can be formed by transamination of pyruvate. -,~
The enzyme is alanine amino transferase (ALT) (see COO- '"!9If COO- ' \ U.l COO-
I + I • '-- I +
Fig. 3.19). CH-NH 3 -t-Mu CH-NH3 GC..'r\.~ CH-NH3
Pyruvate + Glutamate -+ Alanine + alpha ketoglutarate I I /?\ •I
This reaction requires pyridoxal phosphate (PLP).
CH2 / 1 CH2 1 -
2 CH2 +
¢H ATP PP i+Pi bH Acceptor Methyl I
ALT level in blood is increased in liver diseases; this I 2 r- 1 !acceptor 1H2
clinical significance is indicated in Ch_a~,ter 6. lr'_Q_der
c~ ns of starvation. tbe glucok~)~ of Ev!feli~
S-CH3 +
1-§
Adenosyl
f
Adenosyl
sqecial metabolic significance (see Fig. 10.27). Alanine
is quantitatively the most important amino acid taken
up by the liver from peripheral tissues, particularly from x:p,(¼ 0 ch •)- Ad~ ;.. 3 A
skeletal muscle. It forms a major participant in inter- <tfk1 ,, 4
Methionin e - - - - - - - Homocyste ine
organ transport of nitrogen (see Fig.17.2).

Beta alanine
+ CH3 (1-C unitj
Vitamin B 12
Cysteine synthesis
!·"~
Here the amino group is attached to the beta carbon ~~~Of")'l'tC- F - , U . ~ ~ + ~'ISIN.
atom. It is fo rmed during the ~at~ of th.s).QWroidioe
=
1 methionine adenosy~ ~ ~e (MAT); 2 methyltransferase; =
3 =adenosine homocysteinase; 4 =homocysteine
bases, cytosine and ~ (see Chapter 38). It is mainly
melhyltransferase, with vitamin B12 as coenzyme
used for the synthesis of Coenzyme A (see Chapter 33).
Fig. 18.13: Formation of active methionine
l!_HREONINE THR) (~T)_
It is an es~tial amiDQ_ acid. It is 9!llcogenic. Threo- 2. Methyl transfer. In methionine, the thio-ether link-
nine has 2 asymmetric carbon atoms, hence it has 4 age (~ C) is very stable. In SAM, due to the
diastereoi~ hreonine. L-threonine, L- presenceoT a high energy bond, the methyl group
">KalJ.otbreooine and D-allothreoni e Tq~ ~do s not is labile, and may be transferred easily to other
r directly undergo transamination, ~ \'tl..l?i'a'erc +- -~ 00 acceptors (Step 2, Fig.18.13; and Table 18.1).
A nation forming alpha ketobutyric acid reaction similar to 3. Homocysteine. From the S-adenosyl homocys-
'N~ LW~ rine, Fig. 18.9). The enzyme is threonine dehydratase. teine (SAH), the adenosyl group is removed to form
The H gro f threonine residue in protein serves homocysteine, which is the hjaher homolog~ of
-a si ·on (as in the case of c~steine (St13p 3, Fig.18.13).
is OH group also serves for combining car- 4. ~ nine synthesis. Homocysteine can be con-

\oJ:~@
C
1..i.tJu.,....u..a.Lw..1.......,u.w=,.!.l,l..l,,W,!,,IS;,!lai!;> , ~-te ~ glycopro- verted to me!thionine by addition of a methyl group.
teins. /.::7 (r\e,rvv.J v
'J ..... rv!. This methyl group is donated from one-carbon
·· J .,gool with the help,.ofvita~ Step4,F lg.18.1 3).
IMETHIONINE (MET) (M)

__ 5. Homocysteine degradation: Homocysteine con-
It is sulfur-containing, e ~ial, glu~ eni~ amino ~enses with1 serine _to form ~st~thionine:)This
acid. Degradation of methionine results in the synthesis . is cata_lyz~dl by pyndoxal phpsp hate _dependent
e; of cysteine. The sparing action of cysteine on methio- (l~ c stath1ornrn:i-beta synthase (No.5, Fig. 18.14).
nine is thus explained . Absence of this enzyme leads to homocystinuria.
~ ~,...... 1. Activation of methionine to SAM : In the major 6. Cysteine sy·nthesis: In the next step cystathionine

j pathway, methionine is activated to 'active methio-


nine' or S-aaenosyl methionine (SAM). The adeno-
COOrl syl group is transferred the sulfur atom (Step
is hydrolyzed by cystathionase to form cysteine
and homoserine (No.6, Fig. 18.14). Net result is that
the SH grouip from methionine is transferred to
1, Fig. 18.13). SAM is the main source o met s~ .e o fo1rm cysteine. This is c a l l e ~ ~
groups in the body. ration reactior:1. _~l ~~
-+'()-
-
r

_/
'""1~ 9,.~i°"IC. \:'ru.Q_. ' N\eln~O()) N2_

·HOfi\O e..n f"I e_ :::


) { ~- - -~ \YI~
Chapter 18: Aliphatic Amino Acids 273
(? 'C'C O \'QC ~..:ctiif''k
,-- - ----J/,.- --=-===---1.. .a.::--:,--- - - - ,
·oo C-CH- CH2 --CH1 H OOC-CH-CH 2-0H
I - I =-=-
NH;
Homocysteine (PLP) 5 ! -H 0
2
® NH;
Serine

*
· coO- CH - CH2- CH2-CH2-CH-COO-
I I I
NH; SH NH;
Ethanolamine C. hhd!! Cystathlonine
Carnosine
Acetyl serotonin
adz~
GA.l!h.~ J::..g.p) 6 (+)H20 ©
l:'t~oo-
°'-:f

-OOC-i:tr-@
Serine ~ omoserine Cysteine
Histidine
Lysine Methyl lysine
tRNA Methylated t RNA
.2)_,
i ~NH3

7. Final oxid~ton: Homoserine is dWminated and Alpha keto hydroxy butyrate -OOC-C-CH2-CH 2-0H

then deca oxylate~ It finally~


._. '-' g
f:+ CO2 H20
enters into the TCA cyc~ I - C oA (see$
Chapter 13), which is converted to glucose. ( Propionyl-CoA _ ___.,.. C'\, " " 'f'l'\.L t II n
LC0A - S-CO - CH2- CH 6 v..,...x..\,, 1~ VJ

5 = Cystathionine beta synthase;


Methionine in Transmethylation 6 = Cystathlonase
Reactions
Fig. 18.14: Cysteine formation ,
The methylation reactions are shown in Table 18.1. 1\ ·c- "rf2t~ e>\ x ~ ~ oT t\O~\'C)
Some important products are: methionine. Methionine -> SAM-+ SAH-+ Homocysteine
1. Creatine (Fig. 18.5) -+ Cystathionine -+ Cysteine (Figs.18. 13 and 18.14).
2. Epinephrine (see Fig. 19.4) See Trans-sulfuration, item No.6, under methionine
3. Choline (Fig. 18.11) degradation.
4. Melatonin (see Fig. 19.11)
These reactions are called
Degradation of Cysteine
:~~:=::;::;:=:;:::=-
tions, and these are carried out with the elp o ~ 1. Transamination: Cysteine is transaminated to
sy!~hi~ ) (Step 2, Fig. 18.13). Methyl groups form beta mercapto pyruvic acid and finally pyru-
vate (Fig .18.16). The ~eta mercapto pyruvate can
~re originally derived fror, t~ ~c r~ ool (Details
tr~fer the ~ to C~ to form thiocyanate (SCN). -
In Chapter 17). The ~ I-THF can transfer the
2. The sulfur may be removed either as H2S or ele-
methyl group to homocysteine (Step 4, Fig.18.13).
mental sulfur or as sulfite.
Vitamin B12 is the co-enzyme for the reaction . This
3. Cysteine on decarboxylation gives beta mercap-
would account for the defic· off 'c a, · soc· ted
to ethanolamine (Fig.18.17). This is sed for s a,-
wi~ : ! i~ p). SAM is the methyl donor !b_es1s o coenz me JQsee Chapter 33).
for alltfle trans~reactions.
A summary of methionine metabolism is shown in Metabolic Functions of Cysteine
Figure _18.15. The roles played by vitamins are alsrlJFormation of Glutathione
shown m that~1 ,

CYSTEI VS) (C)


?
Jl;r!![• Glutathione is gamma glutamyl cysteinyl glycine (Fig.
18.18). Glutathione is generally abbreviated as GSH, to
indicate the reactive SH group. It was isolated in 1921
It is n~ sential and g l ~ . Cysteine is present by Sir Frederick Hopkins (Nobel prize, 1929).
in large quantity in ~ec:aiiO,_Qf haiLaru:Lnails. Glutamate+ Cysteine -+ gamma glutamyl cysteine Cr~ c.)
Formation of Cysteine is by using the carbon ske- Glutamyl cysteine +glycine-> glutathione (r:-:,J (;.'
leton contributed by serine and sulfur originating from Both steps need hydrolysis of each ATP.
f
\100 C-~ - c.1'-l~-c.f1-,tt{
tJH.,_
ATP PPi+Pi Transamination
Cysteine ---+ I
Methionine
PLP pyruvt~f') @
THFA S-Adenosyl
methionine
Acceptor
coo- @ coo- coo-
l --. 1 ,- l
' CH, NH3 '"'- C-'O C=O
I - I -- I
Methyl transferase CH2-SH CH2f ~~ ; CH3 ,
I
Methyl-

......_
THFA . 18.16: Pyruvate formation from cysteine

i Homocysteine S-Adenosyl 00n€\o.Y1'l~

l!
Homocysteine Methylated j

FOLIC acceptor (SH) i,d.tu.,~


ACID Be ~'nN- { + 'i-" mO"ri1"'i .
NH3 CH 2 iv~~>
Cystathionine Aclenosine _ I I _
OOC-CH-CH 2-CH2-CO- NH-CH-CO- NH-CH2-COO
VITAMIN B8
y-glutamyl j cysteinyl j glycine
Cysteine Frederick Hopkins

Fig. 18.15: Summary of methionine to cysteine conversion. Note


NP 1929
1861-1947
fq\u.~
the role played by vitamins
Fig. 18.18: Glutathione
Cysteine Beta-mercapto
ethanolamine C~
RBC Membrane Integrity
,-
~oo-:)
CH-N H;
CH2-NH2
I
CH2-SH Glutathione is present in the RBCs. This is used for inacti-
I vation of free radicals formed inside RBC. The enzyme
CH2-SH
is tglufathfone peroxidase 1 a selenium containing
Fig. 18.17: Decarboxylation of cysteine enzyme (GPx in Fig.18.19). The glutathione is regene-
- e -'f.z.) otv;;>>--'<kl- \1
O'"[ (superoxide) SOD H; 0 2 n •2H20
-
rated by an NADPH dependecu glutathione reductase
-----
(GR in Fig.18.19). The NADPH is _§lrived from the glu-
c.9se.:§.-phos hate (GPO~ pathwa;--The occur-
GSH = glutathione 2GSH GS-SG rence of hemolysis · PD deficienc is attributed to
SOD = superoxide dismutase r<R
GPx = glutathione peroxidase the dt:c.mase.cl!:filj of red tat.bl.gne (see
GR = glutathione reductase
GPO = glucose-6-phosphate 2NADP+ NADPH + H+
Chapter 30). /
dehydrogenase
Met-Hemoglobin.
osphate GPO
The met-Hb is unavailable for oxygen transport. Gluta-
Fig. 18.19: Free radical scavenging thione is necessary for the reduction of met-hemoglobin
(ferric state) to normal Hb (ferrous state).
® Amino Acid Transport ( T\.(1 e.tt~)' J
code.) - - ----r-.- - -----
2Met-Hb-(Fe3 •) + 2GSH _, 2Hb-(Fe2 +) + 2H+ +GS-SG\
The role of glutathione in the absorption of amino acid is '---- 7
described in Chapter 17, under absorption of amino acid®:;onjugation for Detoxification
@coenzyme Role Glutathione helps to detoxify several compounds by
Metabolic role of GSH is mainly in reduction reactions transferring the cysteinyl group, e.g.
~ \ u } " ~ 2GSH --. GS-SG + H2 a. organophosphorus compounds4 CP~ 9'{~
(Reduced) (Oxidized) b. halogenated compounds-+chlPn~ ~)C ~<'>
The hydrogen released is used for reducing other c. nitrogenous substances (chlorodinitrobenzene)
., substrates. A fe~u,mples are shown below: " d. heavy metals ~ "to@) .&'v..e.\~ t.,1
i ileyl~ce1oacetate -, fumarylacetoacetate CF~/ e. drug metabolism~ > E.~ ~\
1
steic acid-, taurine
iii. (Iodine) 12 + 2GSH _. 2HI + GS-SG
The reaction is catalyzed by lutathione-S-trans-
@ wa~~).
,,....., -
Chapter 18: Aliphatic Amino Acids 275

Box 18.1: Summary of MeUCys metabolism 2 ATP+ SO4- ---+ PAPS+ ADP+ PPi


Methionine 0 0
S-adenosyl Met--... Transmethylation
II
Ho- s -
II
o - P- O-H C Adenine
Creatine
Methyl :::.-+ Epinephrine UV\Cfc1 bH , ~
~ Choline (.I
Melatonin
S-adenosyl homocysteine


Serinex Homocysteine
O-Po;

Homoserine ~Cysteine
\Pyrtvate Glucose

CoA

Taurine Deficiency in Transport Cystath ionine Cysta-


system synthase t hionase
Glutathione Mental retardation +++ +++
Tissue deposition
Ectopia lentis +
Thrombosis +
Renal insufficiency Late
Renal calculi and +
crystalluria
Aminoaciduria Cystine Homocysteine Cysta-
thionine
Amino acid Methionine, Cysta-
increased in blood Homocysteine thionine
Nitroprusside test ++ +++
Supplement Fluid and Cysteine Cysteine
alkali Pyridoxine
Restrict Methionine Met

a. inorganic sulfates,
b. organic or ethereal sulfates, and
c. neutral sulfur.

® mation of Active Sulfate (PAP~)


Active sulfate or Phos hoadenosine hospho-5'-sulfate
nervous system. (PAPS) is formed by the reac1ion between_AIP ancLS0 4
A summary of the methionine and cysteine metabo- and the sulfate is attached to the ribose-5'-phosphate
lism is shown in Box 18.1. (Fig.18.20). PAPS is used for various sulfuration reac-
tions, e.g. synthesis of sulfatides, glycosaminoglycans,

rr,- ,~.
Me olism of Sulfur etc.
The ~ p o n e n t o~oteins
ill!.::W.Jllli1~1Y--i~LLIU~jds) or as pai't"\)f swfatides
1cvsTINURIA -1
...,cans (GAG). n~anlc sul
~---~~c: is Cystinuria is one of the inborn errors of metabolism
derived from the sulfur-containing amino~acids. Th S included in the\Garrod's tetra.ii) It is an autosomal rece-
derived from c s ine--may be oxidized to sulfites ssive condition. The disorder is attributed to the
thiosulfat and further oxidized to sulfateB c,iency in Jransport,nf a~ n acjds (Table 18.2). Signs
tory forms of sulfur in urine are: and symptoms include:
~ ; 1..r "' ~ zr ,r-a ~CL ~ ~v.9.i°'
C4.tu&ne- G'f~ W - \ ~ ~
276 Section B: General MetabolisrH O.N.~a-~ N \11<0 ~ '& ~ t'

Fig. 18.21 : Cystine crystals in urine Fig. 18.22: Cystine stone in the bladder

i. Abnormal excrz:tion of cystine and to a Box 18.2. Ho111ocyste1r1e dnd t1ec1rt ,1tt:icks
extent lysine, orWi?hine and arginine. Hence the
An increase of 5 micromol/L of homocysteine in serum elevates
condition is also called Cystine-lysinuria. the risk of coronary artery disease by as much as cholesterol inc-
ii. ~ @ g _ caU~uh fbrrt!a~,1. In acidic pH , cys- rease of 20 mg/di. Homocysteine interacts with lysyl residues of
tine crystals are formed in urine (Fig. 18.21 ). collagen and bind to fibrillin producing endothelial dysfunction.
iii. Obstructive urogruhy, which may lead to renal insu- Providing adequate quantity of pyridoxine, vitamin B12 and folic
fficiency (Fig. 18.22). acid will keep homocysteine in blood at normal levels.

iv. Treatment is to increase urinary volume by increa- ...._ ?) !>~ N 1o


sing fluid intake. Solubility of cystine is increased Normal h9~ ysteine level in blood is 5-1 mic-

-
by®kalanizatjon of urini}y giving sodium bicarbo-
nate.

@ Cyanide-Nitroprusside Test
romol/L. In diseases, it may be increased~o@:0 to 1Q£
oderate .increase .Is seen .In agg(dt!__Persons,
\.,,
tamin B42 ?tr B6 deficiency, tot:>a co smokers, alee olics
1
.
vi-

and in,.hyp01flyroidism. Substantial increase is noticed in


It is a screening test. Urine is made alkaline with ammo- congenital diseases. ' N".l. .J(I...... hUf 1'
nium hydroxide and sodium cyanide is added. Cystine, if Large amounts of homocystine are excreted in
present, is reduced to cysteine. Then add sodium nitro- urine. In plasma, homocysteine (with -SH group) and
prusside to get a magenta-red colored complex. Specific homocystine (disulfide, -S-S- group) exist. Both of them
aminoaciduria may be confirmed by chromatography. are absent in normal urine; but if present, it will be the
" - I Na(. N ~-inr-e. C-1:JCt.e\ C homocystine (disulfide) form.
8 Cystinosis ~ · SLtr If homocysteine level in blood is increased, there
~";.-,~~ -~
It is a familial disorder characterized by tfiew laespreacf is increased risk for coronary artery dise.as . Clinical
deposition of cystine crystals in the lysosomes. ~ tilJ3 importance is shown in Box 18.2.
\),\~<t) sgleen, and bone marrow (Table The following are the causes of congenital homo-
_}-. ~¥" 18.2). There is an abnormality in transport of cys- cystinurias:
'(:l,Y tine which is responsible for the accumulation. It is an
autosomal recessive condition. Microscopy of blood Cystathionine Beta t ~ )
shows cystine crystals in WBCs. Treatment policies are Synthase Deficiency
to give adequate fluid so as to increase urine output,
alkalinization of urine b sodium bicarbonate, as well as It causes elevated plasma levels of methionine and
administration of D-penici amine. homocysteine. There is increased excretion of methio-
nine and homocystine in urine. Plasma cysteine is mark-
I HOMOCYSTINURIAS
- -----
edly reduced. General symptoms are mental retardation
and Charley Chaplin gait. Skeletal deformities are
First described in 1962, these are the latest in the se~s also seen. In eyes, ectopla lentis (subluxation of lens),
of inborn errors of metabolism. All of them are autosomal myopia and glaucoma may be observed. Homocysteine
recessive conditions. Incidence is 1 in 200,000 births. causes activation of H~ ~l ] ractor. This may lead
Chapter 18: Aliphatic Amino Acids 277

to increased plat Ladbeshl~s and lif.e _ :tbreatenjng aminotransferase


intravascula t rombosi . Amino acid + _ __ _ _,. Alpha keto acid
Cyanide-nitr'oprusside test will be positive in urine. Alpha keto ~Ilutarate + Glutamic acid
Urinary excretion of homocystine is more than 300 2. Glutamic aciid is also formed during the metabolism
mg/24 h. Plasma homocysteine and methionine levels of histidine, praline and arginine.
are increased. Treatment is a diet low in methionine 3. Oxidative cleamination: Glutamic acid is deami-
and rich in cysteine. Sometimes the affinity of apo- nated to form alpha keto glutarate by the enzyme
enzyme to the coenzyme is reduced. In such cases, glutamate dlehydrogenase with the help of NAO+
pyridoxal phospha~~ the coenzyme ql.'t:&'ldD).ar_g;,guan- (see Fig. 17.7)@1 ( ~ 9)
tities (500 mg) will correct the defect. Glutamic acid - - - Alpha ketoglutarate + NH 3
4. Gl ucogenic:: lutamic acid enters the TCA cycle
Cystathioninuria a§ a ketoglu!al:a.te, gets converted to oxaloacetate
It is due to cystathionase deficTenc . tis an a~so- and enters the glucogenic pathway.
7
mal rec ?sive condition. Mental reta ation, a~?nia, 5. N-acetyl ghutamate (NAG) is a'f)ositive modifier of
thrombo~ openia, and eh~ crjnopathies acc~y carbamoyl phosphate synthetase-I in the mito5k.- Q[l-
this condition . Acquired Cystathioninuria may be due to dria. .L.. \s' °'II r.-;
py~ ~ige de.ficirn_ Diagnosis rests on cyanide-nitro- Glutamic acid + Acetyl-CoA -. NAG + CoASH
prusside test (negative) and detection of cystathionine
6. Glutamine: (See below)
in-urine. ~ e quantities of pvrjdoxine (200-400 mg)
7. Gamma carboxy glutamic acid (GCGA) is pre-
may be beneficial. ¼ '31.,
sent in protlrirombin. The gamma carboxyl group is
Acquired Hyperhomocysteinemias added as a post-translational modification, which
B1'l.- needs vitamin K (see Chapter 32).
a. ujfitional' eficiency of vitamins, such as cobala- 8. Excitatory neurotransmitter. Neurons contain
min, folic ac!faand pyri~ine. NMDA (N-methyl-D-aspartate) receptor. Stimulation
b. {Metaboli~ Chronic renal diseases, hypothyroidism. of NMDA receptors by glutamate opens calcium
c. ( Orugimj~:Q) Folate ~nists, vitamin~nta-
channel, leading to stimulation of NOS (nitric oxide
gonists; py~ine antagonists; esWgen antago-
synthase). This in turn, results in transient production
nists, nitric')()xide antagonists.
of NO" (nitric oxide). This raises the cellular level
Table 18.2 shows the salient features of these
of cyclic GMP and neurons are excited.
amino acidurias.
9. Glutathion,e: Glutamate is a constituent of the
tripeptide £Ilutathione (see Fig. 18.18). Glutamic
ACIDIC AMINO ACIDS AND acid metabolism is summarized in Figure 18.26.
THEIR AMIDES '"- rb- 10. Glutamic acid is decarboxylated to GABA.
The acidic amino acids are glutamic acid and aspar-
tic acid. Glutamine and asparagine are the c0rres- Gamma-Amino Butyric Acid (GABA)
pond~ med bi_ addition of ammonia to the Metabolism : Glutamic acid G, decarboxylatiorl gives
ex&a'_ca~ form an amide. All the four amino rise to gcW1ma--amino 9J.!!Y.rjc acid (GABA). (Step 1,
acidsa re used for protein synthesis and serve important Fig.18.24). Part of the glutamate in the brain can be
metabolic functions. Since glutamic acid and aspartic shunted through the GABA pathway and catabolized
acid remain ionized at the physiological pH, they are often to succinate (St1:ips 2 and 3, Fig. 18.24).
referred to as glutamate and aspartate. _ AaA-ts-;e-4Rhtbito neurotransmitter because

I GLUTAMIC ACID (GLU) (E)


it ~ens the cbllorjde channels in p ~ ~ J J l -
bcanesJn-Ct'.IIS. Pu>
It plays a central role in the metabolism of amino acids. .f¥,cidaxat p1h o = ~ : and~
bolism of GABA re uir s rid al os ate as co-
Glutamate is generated during transamination reactions.
1. Transamination reactions : Most amino acids enzyme (Steps ·1 and 2, Fig. 18.24 ). Therefore, in pyrido-
transfer their amino group to alpha keto glutaric xine deficiency, GABA formation is reduced. Since GABA
acid to form glutamic acid (see Fig. 17.6). is an inhibitory transmitter, a low level of GABA or
278 Section B: General Metabolism

Glutamic acid
COO °o NH3 c.l COO COO
j (PLP)
I
CH-NH +
3 f i lCH-NH+3
2 I
CH-NH+
3
I I .., I CO2 / ! 1 ,:,. ...._{.
CH2 CH2 CH2
Gamma-amino butyric
6H2 ATP ADP+Pi 6H2 l NH
3
6H2
; c ~ABA)
I I I
COO CO-NH2 COO 2

Glutamic acid Glutamine Glutamic acid NH3~ 2 "' 'JBP) 0

Succlnic semialdehyde
Fig. 18.23:~ glutamine synthetasei 2 = lutaminase
NAO+~
3 i)
Gf0\ J'P.- · - incy of pyridoxal phosphate would lead to convul- NADH + H+
11 }'q ,lt«1 Sodium valproate which~ nhibits GABA oxidase is Succinate -+ TCA cycle
used in the treatment of epilepsy.
= =
1 glutamate decarboxylase; 2 GABA oxidase;
Congenital deficiencies of GABA aminotransferase 3 = succinate semialdehyde dehydrogenase I
and succinic semialdehyde dehydrogenase (leading

-~ - --
Fig. 18.24: GABA metabolism
to~~~tyric acidurla} are reported, but are very
rare.
Glutamate Transporters
IGLUTAMINE (GLN~) ~<Q~ - - ,..L -Glutamate (~
'--'-- l.
is the major excitato~eurotranwiit-
-
Glutamine was isolated as free amino acidffi.om beeti:ooi) ter. Different transportations are available in the brain.
by Schulze and Bosshard in 18 33 _Glutamine was iden- Dysfunction of transportation mechanism of gultamate
tified as a member of all proteins by Charles Chibnall cause neurodegenerative conditions such as Alzheimer's
and Manayath Damodaran in 1932. disease.

IASPARTIC ACID (ASP)


It is a atucogenic amino acid. It is synthesized from
glutamic acid (Fig.18.23). The amidation of glutamic acid D
to glutamine is catalyzed by glutamine synthetase. It is a non-essential , glucogenic amino acid. Aspartate,
Glutamic acid can react with a ~ecule of NH3 in pre- on transamination gives rise to oxaloacetate which initi-
sence of ATP (Fig.18.23). This M¥ction is i9lfanWn ates the TCA cycle. Aspartate amino transferase (AST)
ammonia trapping in brain as well as for tr port of transfers the amino group of aspartate to alpha ketoglu-
ammonia in a nontoxic form (see Chapter 17). Glutamine tarate to form oxaloacetate.
is hydrolyzed to glutamate and NH 3 by the enzyme glu-
,J;Jr-- r•b Nr4jdc
taminase. This reaction is seen in the renal tubular cells.
\ This ammonia reacts with H+ to form NH~ to excre; \
(AST) (PLP)
Aspartate + - -- - -- • Oxal~ e!a e
lalpha ketoglutarate + Glutamate ./
I
l hydrogen ions in urine (see Chapter.2I)______ )
• Major fate of glutamine is to be hydrolyzed to gluta- The clinical significance of AST is described in
mate and NH3 (Fig. 18.23). Glutamic acid is then deami- Chapter 6. AST i · creased in cardiacJ s ~ a and in
nated to alpha ketoglutarate and enters TCA cycle for h~atic diseases. alate asf)artate shuttle transfers
further catabolism. The N atoms 3 and 9 of purines are
- - --=
the cy.1Qp1a__§@C._NADH into mitgch2nd.!:@ for oxidation
derived from glutamine (see Chapter 38). Glutamine in the electron transport chain (s~ Fig. 21 .5). Aspar-
is the source of 3rd N of pyrimidine. Glutamine is the tic acid is an im_Qortant ~ _yci.e. (see Fig.
source of NH2 group of guanine and cytosine (see 17.10). It directly contributes its _!!Q1l'a aminQ,. ro.Jm, to
} Chapter 38). Glutamine is a conjugating agent, e.g. tlie-.,IJ.(fil3 molecyj.e. The carbon skeleton of aspartate can
l production of phenyl acetyl glutamine (see phenylke- also enter the glucogenic pathway as fumarate.
tonuria, Chapter 19). Glutamine donates the amino itrulline + As artate Ar inine + Fumarate)
group of amino sugars and amide group of nicotinamide. Aspartate is used for the synthesis of purines (1st
Glutamine metabolism is summarized in Figure 18.26. nitrogen and 6th NH 2 group) (see Fig. 38.8).
Chapter 18: Aliphatic Amino Acids 279

molecule of aspartate is incorporated into the pyrimidine Lysine (Lys) (K)


ring (1.fil_.!lj_tr_q_gen and carbons 4, 5, 6) (see Fig. 38.16).
LysiQ,e is an essential basic amino acid. It is deficient
Aspartic acid metabolism is summarized in Figure t ,· ' ;,?tfh · · L .
in cerea1s. It does not undergo transam1nat1on. ysme
18.26.
is predo~inantly ketogenic. In the catabolic pathway,
lysine is converted to saccharopine, which finally enters
IASPARAGINE ASN N)
into the pathway of odd numbered fatty acids{ Hype~
It is so named because it is originally i§9lated from as~- lysinemias result from con enital deficienc /bf any of
ragg_s. Vauquelin and Robiquet in 1906 isolated aspara- the enz~rqes of the above pathway. I retardation
gine. It was shown to be a member of all proteins by and cQdfc:al degeneratjon are seen in these conditions.
Manayath Damodaran in 1932. Aspartate reacts with
Lysine serves the following functions:
ammonia to form asparagine (Fig. 18.25). This is a reac-
a. Lysine and hydroxylysine residues of crul.a~
tion similar to formation of glutamige . Asparagine can be
and elastirl are important in ccasslioking (see
hydrolyzed to aspartate and NH 3 by asparaginasecr=-
Chapter 49).
asparaginas ·s an anticancer drug a ainst leukemias
b. The epsilon amino group of lysine can form Schiff
andjy-I11QbO!J:!eS, becaus ose cells cannot s nthesize
bases, thus linking to proteins, e.g. EYri~I
asparagine; the enzyme will destroy the, available asp'!-
phosphate with transamioases.
ragLne..in tb'~"liJ®d;_ can~er c_ells will die. Aspara-
c. Lysine is the p,.c..ecurso~arnitine.
gine is a glucogemc amino acid (Fig. 18.26).
d. ac~utri 1 1 (.decar b ~on) of lysine in
the intestinegives ris b cadaverine. }

IARGININE (ARG) (R)


Arginine contains g nidinium . It is a highly basic,
Aspartic acid Asparagine semi-essential ammo a . Arginine is (g!ucogen];>
Fig. 18.25: Asparagine synthesis and breakdown. 1 = a~paragine The catabolic pathway of arginine is shown in Figure
synthetase; 2 = asparaginase. Compare these reactions with
those of Figure 17 .1
18.27. In the urea cycle, where arginase splits arginine

Incorporation into proteins


Glutathione
GABA (gamma-amino butyric acid)
Transdeamination: ammonia from all amino acids - Urea

Histidine - - + Glutamic
Arginine - - + Alpha ketoglutarate -+ TCA cycle-+ Glucose
acid (E) ,\;,party!
Praline - - +
trans-

(AST) Fumarate- - Glucose


PLP Aspartic Urea cycle
Protein synthesis Oxalo- acid (D)
acetate Pyrimidine (N1 ; C4, 5, 6)
Trapping of ammonia in brain
Urinary ammonia Purines (1st N and NH2 at 6)
Purine (N 3, 9) Incorporation into proteins
Pyrimidine (N 3)
Guanine (NH2)
Glutamine
11
Asparagine- - - - Protein biosynthesis
Cytosine (NH2) - - - - - (Q) (N) Ammonia trapping in brain
Detoxification
Hexosamines

Fig. 18.26: summary metabolisms of aspartic acid, asparagine, glutamic acid, and glutamine
280 Section B: General Metabolism

Arginine =+ Nitric oxide

l
Creat!ne - Creatine phosphate
~ Proteins
Urea


-t- Ornithine --+ Putrescine --+ Spermidine

Glutamic semialdehyde Proline - Protein

!
Glutamate - - - Glucose
Nitric oxide
Fig. 18.27: Metabolism of arginine and ornilhine

into urea and ornithine (see Fig. 17.10). Ornithine is


then transaminated to glutamate semi-aldehyde and
then to glutamate. <c:ongenital de rclency of o, , ritl ,i, 1e
~mrnotransfera® causcvated plasma and urinary
Relaxes
Q.Cllllbioe; this leads to ~ - Arginine is smooth
n ~ r y for the synthesis of creatine (see Fig.18.5). muscle
Arginine is the precursor of nitric oxide which is an

.:..~~ .:.I
important signal molecule in the body.
Pre1vents platelet
~ ITRIC OXIDE (No · aggregation
'
It is a toxic pollutant of air and automobile exhausts.
But now it is shown to possess more potential biological
functions than any other known molecule. In 1977, Ferid Functions as a
.·.··1:.\~.~
neurotransmitter
Murad showed that the vasodilatory effect of nitroglyce- in brain
rine is due to the release of No·. In 1980, Robert Furch-
Mediates bactericidal
gott showed that "endothelium derived relaxing factor" actions of macrophages
(EDRF) is required for arterial dilatation. In 1987, Louis
F ig . 18.28: Nitric ,:,xide synthase (NOS) reaction. The enzyme
lgnarro showed that EDRF is chemically NO". They contains heme, FAD, FMN and tetrahydro biopterine. It utilizes
were awarded Nobel Prize in 1998. NADPH

Chemistry anion (o·; ), nitric oxide (NO-) is converted to a highly re-


Nitric oxide is an uncharged molecule having an un- active free radical, peroxynitrite (OONO"), which caus-
paired electron, so it is a highly reactive "free radical". es lipid peroxida1tion, cell injury and cell death.
So it is correctly written with a superscript dot (NO") (see
Chapter 30). Half-life is very short, only about 0.1 second. lsoenzymes of NOS

Synthesis of Nitric Oxide There are 3 isof,orms of NOS, these are products of 3
different genes. J\11 forms are seen in almost all tissues.
Nitric oxide is formed from arginine by the enzyme nitric Neuronal NOS: NOS-1 or nNOS or neuronal NOS
oxide synthase (NOS) (Fig. 18.28). It contains heme, is seen in central and peripheral neurons. Nitrogenic
FAD, FMN and tetrahydrobiopterine. The enzyme utilizes neurons are seen especially in cerebellum and gastro-
NADPH and molecular oxygen. Calmodulin is required to intestinal tract.
modulate its activity. The guanidino nitrogen of arginine is MacrophagH NOS: NOS-2 or iNOS or inducible
incorporated into No·.
NOS or macrophage NOS is mainly seen in macropha-
Metabolic Fate ges and neutrophils. It is induced by cytokines (interleu-
kin-1 and tumor necrosis factor).
No· combines with oxygen to form NO2 . These nitrites Endothelial NOS: NOS-3 or eNOS or endothelial
are excreted through urine. On exposure to superoxide NOS is seen in endothelial cells, platelets, endocardium
Chapter 18: Aliphatic Amino Acids 281

Pulmonary hypertension: Inhalation of No· is use-


ful in the treatment of pulmonary hypertension and high
altitude pulmonary edema. NO" produces pulmonary
vasodilatation, without lowering systemic blood pressure.
Impotence: NO" relaxes smooth muscles in the corpus
cavernosum and increases blood flow into the penis and
Ferid Murad Robert Furchgott Louis lgnarro makes it erect. Sildenafil citrate (Viagra) selectively in-
NP 1998 NP 1998 NP 1998 hibits the specific phosphodiesterase type 5 (PDE-5);
b. 1936 1916-2009 b. 1941
thus inhibiting hydrolysis of cGMP, and increasing the
and myocardium . In these sites, the NO" is constantly concentration of cGMP in corpus cavernosum.
produced and released, so as to have arterial relaxation.
I POLYAMINES
Mechanism of Action of Nitric Oxide Polyamines are putrescine, spermidine and sper-
NO" diffuses to the adjacent smooth muscle and acti- mine. They are synthesized from Ornithine with the
vates guanylate cyclase. Increased level of cyclic help of Methionine.
GMP activates protein kinase in smooth muscles, kinase DFMO {difluromethyl ornithine) is a powerful inhibi-
in smooth muscles, leading to relaxation leading to relaxa- tor of polyamine synthesis. It is an example of suicide
tion of muscles. Thus NO" is a vasodilator. inhibition (see Chapter 5). African sleeping sickness
and Indian Kala-azar are produced by parasites, trypa-
Physiological Actions of Nitric Oxide nosomes. In these parasites, the half-life of ODC is many
Blood vessels: NO" is a potent vasodilator. The normal hours. DFMO inhibits polyamine synthesis, so parasites
blood pressure is maintained by the NO" liberated by cannot divide, and the immune system of the host can
endothelial NOS (NO Se). No· causes cerebral, coronary, kill them. The half-life of ODC in man is only 5 minutes.
renal and muscle arteries to dilate. A deficiency of No· So, enzyme molecules are constantly synthesized, and
is associated with hypertension. Excessive production hence the drug will not affect human beings. DFMO is
of NO" results in refractory hypotension, which may be also useful against pneumocystis carinii parasite infec-
tion, which is common in AIDS.
seen in patients with septicemic shock.
Central nervous system: In CNS, NOSn isoform is Biochemical Functions of Polyamines
present. NO" stimulates the releasing hormones (CRH,
Polyamines are required for protein biosynthesis. Several
GHRH and LHRH).
roles are suggested for polyamines, e.g. cell prolifera-
Macrophages: Macrophages contain the isoform NOSi tion, synthesis of DNA and RNA, etc. Polyamine concen-
(i stands for inducible). This enzyme produces No· and tration is increased in cancer tissues. Polyamines are
peroxy nitrite; which are lethal to micro-organisms. growth factors in cell culture systems.
NO" production in macrophage is induced by interleukin
and tumor necrosis factor. Biogenic Amines
Platelets : No· inhibits adhesion of platelets and so They are generally synthesized by decarboxylation
depresses platelet functions. of amino acids. A list of biogenic amines are shown in
Intestinal system: NO" is a non-adrenergic and non- Table 18.3. They are basic in nature. They have diverse
cholinergic (NANC) neurotransmitter, especially in gastro- biological functions, which are described in appropriate
intestinal tract and urogenital tract. It relaxes smooth chapters.
muscles and leads to reduced gastrointestinal motility
and relaxation of sphincters.

Nitric Oxide in Diseases and Treatment


Angina pectoris: Nitroprusside can directly release
I BRANCHED CHAIN
AMINO ACIDS (BCAA)
Valine (Val) (V) is glucogenic; Leucine (Leu) (L) is keto-
NO". Nitroglycerin (glyceryl trinitrite) requires glutathione genic while lsoleucine (lie) (1) is both ketogenic and glu-
to produce NO". These will dilate coronary arteries, and cogenic. All the three are essential amino acids. Leu-
are beneficial in treating angina pectoris. cine is the major ketogenic amino acid. These amino
282 Section B: General Metabolism

TABLE 18.3 : B1ogenic amines Maple s yrup urine disease MSUD


Substrate Decarboxylated product, amine Chapter No. Normal MSUD

~ [I] ~ [I]
Serine Ethanolamine Choline 18
Tyrosine Tyramine 19
DOPA Dopamine 19
Tryptophan Tryptamine 19 Protein Protein Protein Protein
5-0H-tryptophan Serotonin 19 from food from muscles from food from muscles
Histidine
Ornithine
Histamine
Putrescine
19
18
I
Branched-chain
I
Branched-chain
Lysine Cadaverine 18 Amino acids (BCAAs) Amino acids (BCAAs)
Cysteine Taurine 18

acids serve as an alternate source of fuel for the brain


BCKAD
enzymes @
especially under conditions of starvation. I
Energy
l
Build up of BCAAs and
All the three amino acids undergo similar sequenc
other substances
of reactions. These are summarized in Table 18.4
Figure 18.29. In this pathway, the second enz e is
d
I
Growth
l
Acidosis and mental retardation
ranc a a e o ac, e ro t is
a complex of decarboxylase, transacylase and dihydro- Fig.18.29: Pathology of maple syrup urine disease
lipoyl dehydrogenase. So, this resembles pyruvate
d:~og?nase (see Chapter 10). f\Aah~ Su Kl.,\. in early childhood. The characteristic offensive odor of
~ ' t W,JClSL l.)Ae wet\1:.- urine is the first sign of the abnormal excretion of this
Maple Syrup Urine Disease (MS~D) metabolite. Very high amounts of abnormal metabolites
are excreted in urine. The defect lies in reaction No. 3
' It is also called~anched chain ketonuri~ it;e incidence
of Table 18.4 .
"' is 1 per 1 lakh births. The name originates from the
) characteristic smell of u~imilar to burnt sugar or 0
l maple sugar) due to ~ e~ b caocbed cbaio keto •ii- Clinical Case Study 18.1
acids . The basic biochemical defect is deficient dee.Jr- A 36-year-old woman reported with a dull pain in the
~ylation of branched chain keto acid~ (BKA) (reac- left flank which was radiating towards left leg. She
tion 2 in Table 18.4 ). reports fever and inability to pass urine for the last few
Clinical findings: Dis~e tarts in the days. Similar history of illness was reported in the last
/ life. It is char¾\erized k,(, 'ffi'nvul~ns, se 6 months. She was anemic and abdomen was tender.
retardation, v~iting, ~dosis, coma and Routine urinalysis revealed presence of RBC, pus cells,
{ the fir.§t year of life. WBC casts, characteristic hexagonal crystals and amino
Laboratory fi ndin~ UrinYnYontains b~ched acids. What is the probable cause? What is the patho-
chain keto acids, v~ e. leu~e and isoTeucine. genesis of the condition?
I Rothera 's test is positive, but unlike in cases of ketoaci-
0
dosis, even boiled and cooled urine will give the test.
• Clinical Case Study 18.2
( Diagnosis depends on enzyme analysis in cells. Diag-
nosis should be done prior to 1 week after birth. An adolescent girl presents with subluxation of lens and
mental retardation. On examination, she is tall and thin
\ Treatme · Giving a diet low in branched chain ami-
with elongated limbs. Mild scoliosis was present with
~o acids. ·1d vari t is d intermittent branched
pectus excavatum and genu valgum. One of her sisters
chah\__ketonuria. is w1 respond to high doses of thia-
had similar complaints. What is the likely disorder? What
mine. Tois is because the decarboxylation of the BKA
requireittiiamine. ( B 1)
is the basis of the disease?

lsovaleric Aciduria •ii- Clinical Case Study 18.3


Here~ucine catabolisrn is affecle~ Severe metabolic Patient 1 presented at 3 months of age with excessive
acidosis and neurological deficit are seen. It is often fatal irritability, abnormal posturing since birth and delayed
Chapter 18: Aliphatic Amino Acids 283

Valine Leucine Jsoleucine


Transamination to produce Alpha keto isovaleric acid Alpha keto isocaproic acid Alpha keto beta methyl
branched chain r,-keto acid valeric acid
2. Oxidative decarboxylation with the help of lsobutyryl-CoA lsovaleryl-CoA Alpha methyl
CoA, NAD+ and branched chain alpha keto butyryl-CoA
acid dehydrogenase (lacking in maple syrup
urine disease)
3. FAD dependent dehydrogenation Methyl acrylyl-CoA 13-methyl crotonyl-CoA Tiglyl-CoA
4. Individual reactions + H,O; remove CoA to form + CO2 with the help of + H,O to form alpha
beta-hydroxy isobutyrate bioti n to form beta methyl methyl beta hydroxy
glutaconyl-CoA butyryl-CoA
5. Individual reactions NAD dependent dehydro- Hydrolysis; beta NAD dependent dehy-
genase; to form Methyl hydroxy beta methyl drogenation; alpha
malonyl-CoA glutaryl-CoA(HMG-CoA) methyl acetoacetyl-CoA
6. End - products 812-Coenzyme to form HMG-CoA lyase to form Cleavage to form acetyl-
succinyl -CoA acetoacetate and acetyl-CoA CoA and propionyl-CoA

developmental milestones . There is history of sibling The disease is inherited as autosomal recessive.
death at Day 15 of life. The clinician reported abnor- Diagnostic tests are sodium nitroprusside test; cystine
mal urine odor. Laboratory analysis revealed ketonuria stones are detected by X-rays as they are radiopaque
and metabolic acidosis. HPLC analysis of amino acid or by CT of kidneys. Traditionally intravenous pyelogram
showed elevated leucine, isoleucine and valine. Child (IVP) was used. Microscopy reveals flat hexagonal crys-
died immediately afterward. tals of cystine.
Patient 2 presented at Day 12 with metabolic aci- Treatment is aimed at eliminating stones and pre-
dosis, abnormal urine odor, ketonuria and hepatosple- venting new stones. Dietary sodium and protein restric-
nomegaly. Blood and urine studies revealed elevated tion increases cystine excretion. Urine alkalinization
levels of valine, isoleucine and leucine. Aggressive improves cystine solubility. Severe cases need surgical
treatment was started including branched chain amino intervention.
acid restricted diet and supplementation. Patient has In the condition cystinosis, excess cystine crystals
survived until 3 years of age, without any episode of accumulate in eyes and kidneys. This is a lysosomal
exacerbation afterward. Levels of leucine, isoleucine storage disorder and is also inherited as autosomal
and valine came down to normal level. recessive condition. There is mutation in the gene
What is the diagnosis? What is the significant diffe- CTNS, which codes for the protein cystinosin, the lyso-
somal cystine transporter. Symptoms include excessive
rence between the two patients?
urination, followed by poor growth, photophobia and
renal failure by age 6. Cystinosis is the common cause of
¥ Clinical Case Study 18.1 Answer renal Fanconi syndrome. There is loss of large amounts
of salt and other minerals in urine. Definitive diagnosis
The likely cause is cystinuria. Cystinuria is characterized is by measurement of WBC cystine levels. The drug
by build-up of cystine stones or crystals in kidneys and cysteamine is used to clear cystine. Replenishment of
bladder. Patients with cystinuria cannot properly reab- lost fluid and electrolytes as well as high doses of vita-
sorb cystine into their bloodstream and the amino acid min D and phosphorus is needed.
accumulates in their urine. As urine becomes more con-
centrated, excess cystine forms crystals that can lodge
in bladder or kidneys. Cystine can also combine with • · Clinical Case Study 16.2 Answer
calcium to form larger stones. They can block passage Likely cause is homocystinuria. Classical homocysti-
of urine and this can also lead to infections. nuria is due to the deficiency of enzyme, CBS. It is
Symptoms are recurrent nephrolithiasis and obs- inherited as an autosomal recessive condition. Com-
truction of urine flow. This can produce severe, sudden mon clinical features are failure to thrive, developmen-
onset of flank pain, blood in urine, infection which can tal delay, ectopia lentis (subluxation of lens), severe
produce fever, WBC in urine and in advanced cases, myopia, cataracts, retinal detachment and optic atrophy
renal failure. during infancy and early childhood. Progressive mental
284 Section B: General Metabolism

retardation may be there, but in some individuals intel- 8. Glutathione helps in amino acid transport, acts as a
ligence may be normal. Skeletal abnormalities may re- coenzyme for reduction reactions, maintains RSC
semble Marfan syndrome. Generalized osteoporosis membrane integrity and helps in detoxification.
may be present. Thromboembolic episodes involving 9. Cystinuria is an inborn metabolic disorder result-
large and small vessels, especially those of the brain, ing from deficiency of amino acid transporter.
are common and may be seen at any age. Elevated Symptoms include crystalluria, uropathy and secon-
homocysteine level (hyperhomocysteinemia) is an inde- dary infections.
pendent risk factor for coronary artery disease, cerebra- 10. Cystathionuria is due to cystathionase deficiency.
vascular diseases, peripheral arterial disorders as well Mental retardation, anemia and endocrinopathies
as deep vein thrombosis. During pregnancy, it can lead accompany this condition.
to neural tube defects in embryo and pre-eclampsia in 11 . Arginine is tlhe precursor of Nitric oxide (NO ' ) by the
pregnant mother. action of NO synthase.
Diagnosis is by amino acid screen in urine and blood 12. NO" synthase has three isoforms: neuronal, indu-
(homocysteine and methionine are elevated, cystine cible and erndothelial.
will be low), liver biopsy and enzyme assay, skeletal 13. NO" is a potent vasodilator and participates in
X-ray, skin fibroblast enzyme assay, standard ophthal- macrophagB mediated antimicrobial activity. No· is
mic testing and genetic testing. Treatment with high
used in treatment of pulmonary hypertension and
doses of pyridoxine, folic acid, 8 12 and betaine reduces
erectile dysfunction.
homocysteine levels.
14. Glycine cleavage system, a multienzyme complex
$2. can break cjown glycine to CO2, NH 3 and one-
¥ Clinical Case Study 18.3 Answer carbon unit (N5- N10 methylene THFA).
15. Glycine condenses with succinyl-CoA in the first
The diagnosis is maple syrup urine disease. In the case
step of heme synthesis.
of the first patient (Patient 1), diagnosis was delayed
16. Whole mole·cule of glycine is incorporated into the
and hence treatment could not be instituted and the
purine ring (C4,C5 and N7)
baby died. But in the second case (Patient 2), diagnosis
17. The tripeptidle glutathione, an antioxidant, is gamma-
and treatment was started early in life and outcome was
glutamyl cysteinyl glycine.
better. These two case studies indicate the importance
18. Glycine is used for conjugating bile acids to form
of early diagnosis and treatment in MSUD.
glycocholic ;acid.
19. Glycine is a neurotransmitter which opens chloride
l!:_EARNING POINTS, CHAPTER 18 specific channels.
1. Glycine may be formed from serine, threonine and 20. As a constituent of protein, glycine plays a crucial
by the action of the enzyme glycine synthase. role in maintaining the triple helical structure of col-
2. Glycine is used in the biosynthesis of heme, crea- lagen.
tine, purine and glutathione. 21 . Deficiency of glycine cleavage system can cause
3. Stored creatine phosphate acts as an immediate nonketotic hyperglycinemia.
source of energy in the muscle. ATP is generated 22. Primary hyperoxaluria, characterized by repeated
by its hydrolysis by Lohmann's reaction. occurrence of urin ary calculi, is due to a protein tar-
4. Serine occurs in the active sites of many proteolytic geting defect. The absence of alanine glyoxylate
enzymes, such as Trypsin and Chymotrypsin. Such aminotransf,erase in peroxisomes increases the
enzymes are called 'serine proteases'. glyoxylate pool which can be oxidized to oxalates.
5. Major metabolic role of alanine is to provide sub- 23. Addition of a one carbon group to glycine produces
strate for gluconeogenesis. serine.
6. Methionine is activated to S-adenosyl methionine 24. Serine by cleamination can be converted to Pyru-
(SAM). This participates in transmethylation reac- vate, a glucogenic intermediate.
tions. 25. Serine on clecarboxylation gives rise to ethanol-
7. Cysteine is a component of the tripeptide gluta- amine. Both serine and ethanolamine are constitu-
thione. ents of phospholipids.
Chapter 18: Aliphatic Amino Acids 285

26. Methylation of ethanolamine can give choline, used 28. The selenoproteins contain the 21 st amino acid
for acetylcholine and phospholipid synthesis. selenocysteine formed by replacement of the "O"
27. The hydroxyl groups of serine and threonine can by"Se".
be linked in O-glycosidic linkage to carbohydrate 29. Glucose-alanine cycle is one of the metabolic
residues of glycoproteins. adaptations during starvation.

PART-1: ESSAY AND SHORT NOTE QUESTIONS

18-1 . What are the important substances derived from glycine? Describe the steps by which creatine is synthesized.
18-2. Write the reaction s by which glycine is synthesized and catabolized. Name six important compounds derived
from glycine and indicate their functions.
18-3. Describe the metabolism of methionine. Explain the term t ransmethylation with suitable examples.
18-4. Describe the fate and functions of methionine and cysteine.
18-5. Describe the pathway of methionine metabolism.
18-6. What is the biochemical basis of homocystinuria? What test you will do to diagnose homocystinuria?

SHORT NOTE QUESTIONS


18-7. Name the important compounds formed from gly- 18-12. Homocystinuria.
cine. 18-13. Cystathionuria.
18-8. Creatine synthesis. 18-14. Cystinuria.
18-9. Creatinuria. 18-15. Give 4 examples of transmethylation reactions.
18-10. Biological action of Glutathione. 18-16. S-adenosyl methionine (active methionine).
18-11 . Gamma-amino butyric acid (GABA). 18-17. Maple syrup urine disease.

PART-2: MULTIPLE CHOICE QUESTIONS



18 1 Glycine is used for synthesis of all the following 18-7. Serine is a precursor of all the following, except:
compounds, except: ]!.. Methionine B. Glycine
P A. Serine ~ - Creatinine C. Choline D. Cysteine
..e. Pyrimidine ring x D. Heme 18-8. Acetyl choline is derived from which amino acid?
18- · In the body, glycine is used for synthesis of all the A. Tyrosine B. Tryptophan
following substances, except: C. Glutamic acid D. Serine
A. Purine ring B. Glutamine 18-9. The methyl donor in methyl transfer reactions is:
C. Glutathione D. Creatine A. Methyl cobalamin
18 Normal serum creatinine level is: B. Methyl malonyl-CoA
A. 0.2 - 0.4 mg I di C. S-adenosyl methionine
B. 0.3 - 0.6 mg / di D. S-adenosyl homocysteine
C. 0.7-1.4mg / dl 18-10. All the following are substrates for transmethyla-
D. 1.4 - 2.8 mg / di tion reactions, except:
18-4. Creatine is synthesized from the following amino A. Guanido acetic acid
acids, except: B. Choline
A. Arginine B. Aspartic acid C. Nor epinephrine
C. Glycine D. Methionine D. N-acetyl serotonin
18-5. The sources of oxalic acid in urine are: 18-11. Name the defective enzyme in cystathionuria:
A. Ornithine and Citrulline A. Cystathionase
B. Oxalosuccinate and formic acid B. Phenyl alanine hydroxylase
C. Oxaloacetate and aspartic acid C. Homogentisic acid oxidase
D. Ascorbic acid and glycine D. Para hydroxy phenyl pyruvateoxidase
18-6. Serine when decarboxylated will produce: 18-12. Cysteine on transamination gives rise to:
A. Ethanol amine A. Pyruvic acid
B. Beta mecarpto ethanol amine B. Para hydroxy phenylpyruvate
C. Histamine C. Phenyl pyruvate
D. Putrescine D. Beta mercapto pyruvate
286 Section B: General Metabolism

18-13. Glutathioneisusedinthebodyforallthefollowing 18-24. Which amino acid will give rise to an inhibitory
functions , except: neurotransmitter?
A. For ammonia trapping A. Histidine B. Glutamic acid
B. For reduction of methemoglobin to normal hemo- C. Ornithine D. Tyrosine
globin 18-25. Gamma-annino butyric acid (GABA) is derived from
C. To carry amino acids across membranes which amiino acid?
D. For removal of peroxidases A. Tyrosine B. Tryptophan
18-14. One salient clinical manifestation in homocystinuria C. Glutamic acid D. Serine
is: 18-26. Transamination of glutamic acid leads to:
A. Agitation, hyperkinesia A. Glutamine
B. Renal stones B. Alpha k:etoglutaric acid
C. Ectopialentis (subluxation of lens) C. Beta h~rdroxy beta methyl glutaric acid
D. Nystagmus D. Gamma amino butyric acid
18-15. The incidence of homocystinuria in the population 18-27. Transamination of aspartic acid gives rise to:
is about: A. Asparagine B. Malate
A. 1 in 1,500 B. 1 in 50,000 C. Oxalos1uccinate D. Oxaloacetate
C. 1 in 1,00,000 D. 1 in 2,00,000 18-28. Lysine is:
18-16. Example for a transport defect is :
A. Deficient in cereals (rice, wheat )
A. Cystinuria
B. Deficient in pulses (dal, bengalgram)
B. Homocystinuria C. Mainly ,glucogenic
C. Crigler-Najjar syndrome
D. A non-essential amino acid
D. Sulfituria
18-29. Glutaminase which hydrolyses glutamine to glu-
18-17. The urine of a patient with homocystinuria will be
tamic acid is used for:
positive for:
A. Ammonia trapping in brain
A. Benedict's test .:
B. Excretion of ammonium ions in renal tubules
B. Ferric chloride test
C. Purine :synthesis
C. Rothera's test
D. Transamination
D. Cyanide nitroprusside test •
18-30. Which amino acid does not undergo transamination?
18-18. Urine of a 12 years old boy gave a positive cyanide
A. Leucine B. lsoleucine
nitroprusside test. He had renal stones. He is likely
C. Lysine D. Alanine
to have:
18-31 . Lysine is required for the synthesis of all the fol-
A. Homocystinuria B. Cystinosis
lowing, exc;ept:
C. Hartnup disease D. Renal glycosuria
18-19. Name defective enzyme in homocystinuria:
A. Collage,n B. Cadaverine
A. Propionyl-CoA carboxylase C. Carnitine D. Carnosine
B. Methyl malonyl racemase 18-32. Alpha amino group of aspartic acid is incorporated
C. Tyrosinase into all the following compounds, except:
D. Cystathionine synthase A. Adenint3 B. Cytosine
18-20. Following constituents of urine listed below contain C. Urea D. Creati ne
Nitrogen derived from glycine, except: 18-33. Aspartic areid is used for the synthesis of which
A. Creatinine B. Uric acid compound?
C. lndican D. Urobilinogen A. Porphyrin B. Sphingomyelin
18-21. Alanine on transamination gives rise to: C. Pyrimidine D. Folic acid
A. Pyruvic acid 18-34. All are monoamines, except:
B. Para hydroxy phenylpyruvate A . Histamine B. Serotonin
C. Phenyl pyruvate C. Dopamiine D. Glutamine
D. Hydroxy pyruvate 18-35. Ornithine when decarboxylated will produce what?
18-22. 5-adenosyl methionine transfers methyl groups to A. Ethanol amine
which compound? B. Beta m13carpto ethanolarnlne
A. Homocysteine B. Methionine C. Histamine
C. Homoserine D. Nor epinephrine D. Putrescine
18-23. All the following are excretory products of metabo- 18-36. Branched c;hain keto acids are excreted in urine in
lism of methionine and cysteine, except: large quantities in:
A. Ethereal sulphates B. Inorganic sulphates A. Phenylketonuria B. Maple syrup urine disease
C. Sulphites D. Phosphates C. Tyrosinosis D. Hartnup disease
Chapter 18: Aliphatic Amino Acids 287

18-37. Which test will be positive with the urine of patient 18.38. Name the defective enzyme in maple syrup urine
with maple syrup urine disease? disease (branched chain ketonuria):
A. Dinitro phenyl hydrazine test A. Cystathionase
B. Benedict's test B. Phenyl alanine hydroxylase
,_ C. Ferric chloride test C. Homogentisic acid oxidase
D. Cyanide nitroprusside test D. Oxidative decarboxylase

ANSWERS OF MULTIPLE CHOICE QUESTIONS

18-1. C 18-2. B 18-3. C 18-4. B 18-5. D 18-6. A 18-7. A


18-8. D 18-9. C 18-10. B 18-11 . A 18-12. D 18-13. A 18-14. C
18-15. D 18-16. A 18-17. D 18-18. B 18-19. D 18-20. C 18-21 . A
18-22. D 18-23. D 18-24. B 18-25. C 18-26. B 18-27. D 18-28. A
18-29. B 18-30. C 18-31. D 18-32. D 18-33. C 18-34. D 18-35. D
18-36. B 18-37. A 18-38. D

PART-3: VIVA VOCE QUESTIONS AND ANSWERS

18-1. Glycine is used for synthesis of what compounds? 18-10. Give examples of transmethylation reactions?
(Name biologically important substances derived Guanido acetic acid to creatine, Serine to choline,
from glycine). Nor-epinephrine to epinephrine, N-acetyl serotonin to
Serine, Creatine, Purines. Heme. Glutathione, Bile salts. melatonine
18-2. For creatine synthesis, which amino acids are used? 18-1 1. What is the methyl donor in transmethylatlon reac-
Glycine, arginine, methionine. tion? (What is active methionine)?
18-3. What is the significance of creatinine? S-adenosyl methionine
Excretion is increased in muscle dystrophy. 18-12. What is glutathione?
18-4. How creatinine is produced in the body? Gamma-glutamylcysteinyl glycine.
By spontaneous degradation of creatine phosphate. 18-13. What are the functions of glutathione?
18-5. What are the sources of oxalic acid in urine? Keeping RBC membrane integrity; carrying amino acids
Ascorbic acid and glycine.
across membranes; detoxification of peroxidases.
18-6. What is the cause of hyper oxaluria?
18-14. What is the test for homocystinuria?
Protein targeting defect.
Cyanide nitroprusside test will be positive in urine.
18-7. Choline is derived from which amino acid?
18-15. What is defective enzyme in homocystinuria?
Serine
Cystathionine synthase
18-8. Alanine is transaminated to what?
18-16. What are the characteristic features of homocys-
Pyruvic acid.
18-9. What is significance of glucose-alanine cycle? tinuria?
During starvation, alanine is released from muscle and Mental retardation, subluxation of lens, thrombosis.
is taken up by liver. In liver alanine is transaminatec '.- 18-17. What is defective enzyme in Cystathionuria?
pyruvate, and pyruvate under goes gluconeogenes1s. Cystathionase.
_ __ _ _Chapter 19
Aromatic Amino Acids
(Phenylalanine, Tyrosine, Tryptophan,
Histidine, Praline and Aminoacidurias)

Chapter at a Glance
The learner will be able to answer questions on the following topics:
(!5) Phenylalanine (Phe) (F) 0 Hypertyrosinemias
tO Tyrosine (Tyr) (T) 0 Tryptophan (Trp) (W)
I{) Phenylketonuria (PKU) 0 Histidine (His) (H)
.(} Alkaptonuria Praline (Pro) (P) and Hydroxyproline
Albinism Aminoacidurias

I PHENYLALANINE (PHE) (F~


Phenylalanine is an essential, aromatic amino acid.
The need for phenylalanine becomes minimal, if ade-
__:_~_,,
r -i.n..,,e_is supplied in the food. This is call~he
a t f tyrosi n phenylalanine. It is ~ y
~~~~"lld..n dpa_rt=I~ ===::::;' M.On~~~ ~ ·

Step 1: Phenylalanine to Tyrosine


Phenylalanine --.'.- \__32
-T---'--\'----•. Tyrosine

'J·,~ .,.._ Tetr~~Qi.Q.Q.terin Olhydrobioptenn


\(o-"I'
/ <ntBr) 1-A
Ct>H'oV
opterjne as coenzymes. As this is a frreversib e reac-
tion, m_Q§J_ne cannot reQ.leni sh pheny ~e. Hence,
phenylalanine is essential in food. It is a rni.li-edJ uoction 1~
oxidase (mono-oxygenase), which means one atom of 1__t!AbP
1
J NADPH
oxygen is incorporated to the product.
Fig. 19.1: Phenylalanine catabolism; 1 = Phenylalanine hx_droxy-

ITYROSINE (TYR) (T) lase; 1-A = NADPH dependent reductase ( Den J

Tyrosine is an aromatic amino acid. It is synthesized


from phenylalanine, and so is a non-essential amino
acid. The need for phenylalanine becomes minimal, if
,,.f -'l t-)~w)(;y'lohJr-
1b .;,, 1)e, \,v:f~,osi :J~c~'l'.r"
• ') ?<0
-,
t>Qi"''("ll;
r
--
l)\~

. ..-
-=---J~ Chapter 19: Aromatic Amino Acids 289
\...
\-\ a..._€ \4 0 tf\< b,... ) 0,:"\ C1 \. \ b<:l.al
1 = Phenylalanrne Jiydroxylase .:..--- Step 3: Production of Homogentisic acid
2 = Tyrosine
3 = Parahydroxyphenylpyruvate hydroxylase The n~ ~t .~ ~~ 3, Fig.19.2) is catalyzed b para-
4 = Homogentisic acid oxidase
5 = Maley! acetoacetate isomerase
hydroxyp!w~'lf>§hWate . It is a c pper
6 = Fumaryl acetoacetate liyarolase • containing enzyme. Interestingly, the'"fea9tion involves
, 'l'\.._'t ~\ ot,0-\-.tt
shifting of the sjde chajp posi-
u - tion. Ascorbic acid is helpful in this reaction.
Phenylalanine

l,~. Step 4: Cleavage of Aromatic Ring


11 Hom~ ~~ oxidase ~ pens th~iijg)step 4, Fig.
H0- 0 - CH-yH~C06 2 Tyrosine 19.2). The product i{'<f-maleyl acetoacetate.
\'l\e-<, ~enl-h . hi
NH 3 a-ketoglutarate
2
Step 5: lsomerization
Glutamic acid (PLP)
It then undergoes@ s to traa; )somerization to form
:J fumaryl acetoacetate by an isomerase (step 5, Fig.19.2).
HO CH -C-COO I v-\«- Para hydroxy•
- 0 - 2;g ~:_ ph~n
~!Jlpyruvlc acid
,.;---- - 02
Step 6 : Hydrolysis
:'\())-
\) Ascorbic acid 3
CO2
J..bo.o-' -l
HO ? b~ --. Dihydroxy-
r,Gi . phenylacetic acid
- Cleavage s,te (homogentislc acid)
CH2 -CO0

(+}02~ J
4

O0C-CH2-C0-CH2-CO-CH=CH-C0: 1 Important Specialized Products from

1,
+Maleyl acetoacetic acid
Tyrosine
1. Melanin

r,~tx
2. Catecholamines (epinephrine)
Fumarylacetoacetic acid
3. Thyroxine
(+)H20 A6
(' Synthesis of Melanin
-OOC-CH2-C0-CH3 -OOC-CH=CH-COO
Melanin pigment gives the black color to the skin and
Acetoacetic acid Fumaric acid hair (Greek word Melan means black). There is only one
(Ketogenic pathway) (Glucogenic pathway)
enzyme involved.w hich catalyzes the first two steps. The
Fig. 19.2: Catabolism of phenyl~ ct.tyrosjoe..,,--,....--_.;; remaining reactions are non-enzymatic and occur spon-
¼::_J ~ , 4 - 21,., O> 'l""'.) ,taneously.
Catabolism of Tyrosine i. Formation of DOPA: The first step is the !)¥drogta-
(and Phenylalanine) ~?J~ ti.on of tyrosine by tyrosinase. It is a mono-oxyge- 0 $
. t· 4- !}, ;.;;
na:;:.:s~e.;.c=on.::
ta;:in.;in
~g~ ~
o i!!!ieiiirlL(.::
st~e;;p.,;1;.
, Fig. 19. 3) to form
Step 2 : Transamma 10n <i---0 ~·- ,. .
~ydroxypheny a arnne or DOP _ (Box 19.1 ).
Degradative pathway of phenylalanine and tyrosine ii. Formation of DOPA quinone: Tyrosinase again
are the same. Tyrosine is transa i ate to give para- acts on DOPA to form dopaquinone (step 2,
hydroxyphenylpyruvic acid by rosme transaminase Fig.19.3).
(step 2, Fig.19.2). It is pyridoxal phosphate dependent. iii. Formation of indolequinone: It is converted to
It is induced by glucocorticoids. indolequinone through a series of reactions. The

~1 ~.Ro'°'-]\o.ttm l t f} IV\ ()-JU ~,e)) 1


·~ ()_~Pf,'\~~~

on Ct')?}' CR E._ ~oR.G>'\ C.. Ac\ o


- "~
J r. l ~st

l
BOX 19.1: Tyrosinr1se and tyrosine hydroxylase
B~tb v,6e
enzymes ~II a,dd hyd.roxybg_r~ to-tyros!Ae to pro- HO- OCIH i-yH~COO- Tyrosine \ fl~
duce dihydroxyphenylalanine (D'?,p"l- "':::, NH3 (•)O, 1
Tyrosinase is present in melanoblasts. The enzyme produces '<;
DOPA, which is used for melanin synthesis. HO \.o.g._
DOPA CL. '!'JI)
Tyrosine hydroxylase is present in adrenal medulla and the DOPA
thus generated is used for catecholamine synthesis. Thus even in HOOCH2-9H~COO- (Dihydroxy- 1
phenylalanine) r

~-
tyrosinase deficient person (albinism) synthesis of the catechola- .. NH 3 \.
mines is normal. '6 0~ •
0 .t'I

ti]
0 O -cH 2-CH-C00- DOPA-qulnone
- I+
Q NH 3
elan~ in the deeper layers of epidermis synthesize
mefa~in in granular form in nosomes ~ 9r of
;,.o 0 -a::: I
. ,ar,
1
the ~kin depends the di the f>PJ ( crle ._g,_ (00 0 .-,:: N
concentration of melanin an I s state oicidation. The extra- lndolequinone
Melanin
cellular granules are later dispersed under the influence of
melanocyte st imulating hormone (MSH). Fig. 19.3: Melanin synthesis pathway; 1 and 2 steps ave the
Copper deficiency: Since tyrosinase is a copper containing same enzyme, tyros,inase :rncr ~\
enzyme, there may b _s turban~ i!'.1 pigm~~on during \,Lor€
n I C i symhesized at
• thf tim
of deficiency tyrosinase involved in melanin synthesis which
c ~ may be depigmented. If copper deficiency · ntermittent, al-
catalyzes a similar reaction (Fig. 19.3 and Box 19.1).
(- ternate black and white regions may be s in the hair (flag-
type of hair). The tyrosirne hydroxylase reguires tetrahydro-
may multiply to give bj_opterine and NADPH (similar to phenylalanine
rise to malignant melano I&.' hydroxylase ). -
~ ~ o n d l i MD@uch urine if kept in a test ii. OPA-dec r x.ylase : DOPA is decarboxylated to
~ • ~~ GE Refl et rbe lllbe bernrne5 b!ar;~ due to oxi- form Dopamine y DOPA-decarboxylase, a pyridox,JI
dation loelanin. c:..c,_,....vJ_ .. .-.00,
phosphate cjependent enzyme (step 2, Fig.19.4).
iv. Leukoderma: W~~ osinase or meram-/l'for~ s or
bot h are ab~ from _;}derrlJls, leukoderT~hi~es) It is a catecholamine.
results. .
v. Graying of hair is also due to the disa@ earance 9f....m~ano-
from thf nair root.ft
vi. Albinism: Albinism and leukoderma are different. In albi- tract.
nism, tyrosinase is absent in melanocytes all over the body.
In ParkJ nsonism , the dopamine content in
See later in this chapter.
QJ:ain.ls_red~d- As dopam~ ot enter into
indolequinoneisg£1_y~ zegJofQrrn..rnrua,nin.Melanin the brain cells, the orecurso~-l.90.P
is used as a
is a group of polymers of random structure formed drug in Parkinsonism. ,..
- =,s
from indolequinone. Melanin when reduced chang- ,Alpha 111ethyldop will inhibit d_opa _decar:boxyl-
es from black to a tan color. Clinical applications of ase and prevent production of epinephrine; so it is
,,
melanin are shown in Box 19.2. an antihyge :t~ e drug.~

Synthesis of Catecholamines C~frvc.~h~-Nt.


r --=---,
Catecholamines are derived from tyrosine. They are so
named because oq l\e presen~ of catechol nuc~ s.
They include eQ~i~. nor-eTRnei:mrJne and dot a-
mine. They are produced by the adrenal edulla and
~ mpa;,Petic ganglia.(J'I'"'~ Y°'~\.
i. Tyrosine hydroxylase: Tyrosine is first hydroxy- Arvid Paul EricR Ulfvon
Carlsson Greengard Kandel Euler
lated to dihydroxyphenylalanine (DOPA) by tyrosine NP2000 NP2000 NP2000 NP 1970
hydroxylase (step 1, Fig.19.4 ). It is different from b. 1923~ b. 19~ b.1929 1905-1983
~09-."-'~ \ I~~\\~
~~e,- ttO.fV'"""
p
0 0-.mr)T') e -> 0, er' nCf>V\% V\e_ <p;ne, pI,\ 151 rte_,
Chapter 19: Aromatic Amino Acids 291

Tyrosine

021
Epinephrine and adrenaline are two names for the same hor-
(+) mone. John Jacob Abel discovered "epinephrine• in 1901 . In
NADPH and 1 the same year, Japanese-born chemist Jokichi Takamine, work-
tetrahydrobiopterin
ing independently, isolated the same hormone. which he called
~ . , ; , DOPA f "adrenalin. It was first marketted as Adrenaline for therapeutic
Odr (d1hydroxy- use. Hence the word adrenaline is more u~ed in clinical practice,
~v- ph::~la:lne) while the term epinephrine is more favored in academic circles

CO2 ~
~ . . . , , Dopamine
t
HO
"lv~ Cu++
Vil. C
1 {j 3

HOo ~¥HrNH, ....o,opi,ephrioo


Julius
Takamine Axelrod
~ SAM ~ ~ 1854-1922 NP 1970
1912-2004
HOO HOH-CH,-NH-CH, ; : .,,•••: .. pe~~
.t.V--..,_·:
-j
iv. Epinephrine: In the next step, norepinephrine is
A~ SAM methylated by the enzyme N-methyltransferase
(NMT to epinephrine or adrenaline (step 4, Fig. 19.4).
{i3c- o)==\ - ()o...f'o.... SAH 5
S-adenosyl methionine (SAM) is the methyl donor. It
H0 ~ H 0 H-CH 2-NH-CH3 / etanephrin:
is mainly produced.QY_;i~ ll and_;id~ er-
(\\.e.\tw. 16 gic nerve endin s. Epinephrine and Adrenaline are
H3C - 0 k;,o._~~ the two names for the same hormone (Box 19.3).
HO -)==\ -HOH-COO A ~~:yl- , .,,_ vn.p~ <:...
1 •• •
Actions of Epinephrine \ •
' , J o J ' ~ mandeiicacid)
L Epinephrine and norepinephrine increase the blood
1 = Tyrosine hydroxylase
2 = Dopa decarboxylase
y;s>- pressure. C. f '/ 0.9.Q:__{)n'-,t:fi chrrJ
3= . ii. Adrenaline also increases the rate and farce of
'°"dl..9-~
myocardial cont~a(;tion. C
~e
4 = Jt!j~~n MT)
5= a ethyltransferase (C0MT)
6 = Monoamine oxidase (MAO) Iii, Epinephrine causes rP.loxatjpp pf smooth

Fig. 19.4 : Metabolism of catecholamines


C
of branch!:, u,J"\~) r
B TQT'\(\>..0&: ,
iv. Adrenaline is an!i-ios1Ilia in nature, it i~ -
co e ol sis and i es i ol sis~ f ~ )
Arvid Carlsson established dQpamine as anew:o-
transmitter and developed L-dopa as a_do..ig for
the treatment of Parkinson's disease. Paul Green-
v. Adrenaline is released from adrenal medulla in
response to flight, fight, fri ht exercise and hyP-o-
.
g Iycerw a.
1L,.,
l
gard demonstrated that dopamine regulates neu-
ronal excitability. Eric Kandel showed dopamine
is required for short-term and long-term memory.
Degradation of Adrenaline C.€fnet>
All the three scientists were awarded Nobel Prize The half-life of epinephrine is very short, onl 2-5 inutes.
in 2000. Norepinephrine was identified as a neuro- EpineRb · e is catabolized in tissues, ,py catec bQl-O
'----7
transmitter in 1946 by Ulf von Euler, who was meth I trans~ ras (COMT) (step 5, Fig.19.4) to meta-
. r::::. --- ~ - -xi- ase
--
awarded Nobel prize in 1970. nephrine. It is then acted upon by-11,onoamine...._.,,,,._
iii. Norepinephrine: Dopamine is further hydroxy- (MAO) (step 6). MAO will oxidatively deaminate com-
lated to norepinephri~~ '{ldrenaline (step 3, pounds having the amino group attached to the terminal
Fig. 19.4 ). The term fn' ~11Q.91es that the molecule carbon atom. The major end product is 3-h drox -4-meth-
does not contain the R" or methyl group. ox mandelic acid or vanillylmandelic ac~ a l
292 Section B: General Metabolism

Transaminase Reduction
Phenyl- 7' , • Phenyl- 7" , • Phenyl- Guthrie test was developed in 1961 by Robert Guthrie (1916-
+ +
alanine
a-KG
r
Glutamate l
pyruvate f +
NADH+H NAO
+ lactate
1995). It is a rapid screening test. Cert ain strains of ~ II~

!'---. co
~ ~ ine as an essential growth factor. Bac-
Decarboxylation teria cannot row in a medium devoid= hen.xJal~jJle. Bacte-
2
rial growth is proporuQ~I tp..tl)e.p e anine content in the
Phenylacetate patient's blood.

Conjugation lr-- Glutamine Guthrie was the father of a mentally challenged child and had a
niece with PKU. He developed a rapid diagnostic screening test
~ H20 forPKU.
Phenylacetyl glutamine

Fig. 19.5: Alternate pathways in phenylketonuria is elevate~aernate mmor pathwa~ are ~ned
4,~ ~-t~o,q..
level of excretion of VMA is =6 mg/24"fi. It is ln'i:reased in
G (Fig.19.5).
lactate and fi
I ketone (phenylpyruvate), ~ n I-
!acetate are ex~r ted in urine.
R e chrom I toma (epinephrine excess) and in !]SY.-
stoma (norepinephrine excess). Clinical Manifestations '1
The classical PKU child is mental!
© Synthesis of Thyroid
Lo~
Hormones
Tyrosine residues are 1-monoiodo-
~c,,\~IQ of 50. About ~ ~ mate::s:::;;=p~s=yc
may PKU. AgiSfon,
; ::1~
a r~1c hospitals
h . a~ty, t_r~ors and
tyrosine (MIT) and 3,5-~dotyrosina (DIT). These are conv'M"s1ons are often manifested. This may be
coupled to gi~~ ~.3'-triiodothyronine (T3) and becaus~ phenylala~ine interferes with neurotransmitter
3,5,3',5'-t~dothyron ne or thyroxine (T4) (see Chap- synthesis. The child often has o 1gmen a 10
ter 45). explained by the i_ghigJ,tion of ~sJ~e. Phen ctlactic
Tyrami U'rru).._'l'tx, \ \<,.Gll\~\.-o.-0..P sweat ~ ay lead lo..JJlousy body odor.
~We,,; ~ i > ~&l\ , Laboratory Diagnosis
Tyrosine i ca cboxylated to tyramin'e by intestinal
bacteria. Tyramine is present in chocolate, cocoa, wine, A. Blood phenylalanine: Normal level is 1 mgldl In
dried fish, processed meat, buttermilk, cheese, yeast, PKU, the level is >20 mg/dl. This may be demon-
beans, peas, papaya and peanut. These may precipitate strated by chromatography. Tandem mass spec-
an attack of mi raine in susceptible individuals. troscopy is the most reliable test; but is costly.
B. Guthrie test is a rapid screening test. See Box

I PHENYLKETONURIA (PKU)- - --
19.4 and Figure 19.6.
C. Ferric chloride test: Urine of the patient contains
Deficiency of phenylalanine hydroxylase (Fig.19.1) is p~enylketones about 500-3,000 mg/day. This could
the cause for this disease. The gene~mutation be be detected by adding a drop of ferric chloride to
such that either the enzyme is not syht?iesized ~ - the urine. A ransient blue-gteen color is a positive
f4:1Actiooal eQZ.ynle is synthesized. It is a recessiv condi- test. But this is a less reliable test.
tion. Frequency of PKU was considered to be 1 in 10,000 D. DNA probes are now available to diagnose the

..I births; but recent introduction of better diagnostic facili-


ties showed that the incidence is as high as 1 in 1,500
births (World Health Organization, 2003). Incidence of
defects in phenylalanine hydroxylase.

Treatment
'.KU in lnd'.a is lesser th~t_,!W-n c ~ ies; only 1 Early detection is ve!Y importa11t. About 5 units of IQ
1n 25,000 births. There ar types of Pkll described, are lost for each 10-week delay in starting the treatment.
depending on which enzyme is de 1cient. 1.Y e 1 is the The treatment is to provide a diet containing IQ.w ghe-
classical one, due to deficiengy of t}ydroxylase enzyme. (10-20 mg/kg bo~ weight per day). Food
ioca (cassav ~ill have low phenylala-
Biochemical Abnormalities
nine conten . 1s special diet is to be continued during
Phenylalanine cannot be converted to tyrosine. So, phe- the first decade of life; after which the child can have a
~ lanin~cumulates. Phenylalanine level in blood normal diet.
f:G . - r""<"
\ I 1"'1 I Qxe.~\v-£~0j -
,C <)
C \.r b,_-~ a)
1

Chapter 19: Aromatic Amino Acids 293

The specimen
on the left, which
has been After two
standing for hours, the
fifteen minutes,
shows some
darkening at the
surface. due tq. Vertebral disc~.-r.::.;_,.
the oxidation showing blackish
. of nomogentisic d1
·scolorat1·on
F 19. 19.6: Guthrie test. Robert Guthrie is shown on right side
, •
=-~
.,c,u. (alkaptone
,,. :\J1lw1f? C.. :"":
I ALKAPTONURIA
bodies

--- - 9 v -~n;.Qe.. r
\(,\.£1' l=ig.19.7: Left side, urine of alkaptonuria; right side alkatone bod-
ies deposited in vertebral disc of patient
The term alkaptonuria arises from the Arabic word alka-
t f
p on. ?r
" lk 1·" d G k
1 an
lie
d " t:r,t.lC,. +- \?oll.lm-. ~Cl! .
ree wor to s_ucK up flxyg~n In Diagnosis of Alkaptonuria ..:,,, "ILJ,,,'.
,..,..,,.\ n
n . \00--'__..
v..n""~
_t1..,. -~
alkali. This Is based on the observation that the~ rioe ~\~\.J
becomes b lack on standing when it be_oomes_alka- 1. Urine becomes black on standing due to con-
line. Sir Archibald Garrod in 1902 reported that patie~ _ tact with oxygen or when it becomes alkaline.
comp.lai.D.Jhat..llleiumde.rweacs ace getting...bla~~ Blackening is accelerated on exposure to sunlig~t
Garrod concluded that the disease is i(lherited and ii and oxygen. The urine when kept in a test tube will
is due to the d~ ofi&ie enZ'@fl re91,urngJor fur- start to blacken from the top layer (Fig. 19.7).
ther meta oli ~-·---·- · · · lea tonurl and 2. Ferric chloride test will be positive for urine.
are two inborn errors included in Garrod's 3. Benedict's test is positive with a ~~!=!::::~
tetrad; the other two bein pentosurla and cysti ~ ipitq~ Therefore, alkaptonuria comes under
' Garrod introduced the term "inborn errors of meta• &differential diagnosis of reducing substances in
bolism" in 1908. The condition had been vividly des- urine.
cribed by Zactus Luxtanus in 1649. Egyptian mummies 4. Dectection of homogentisic acid by GC/M~ (gas
dating back 2000 BC had pigmented cartilages due to chromatography/mass spectrometry).

I ALBINISM
alkaptonuria.

Biochemical Defect -=> mo.we-· ino means white. Albinism is an


Alkaptonuria is an autosomal recessjve condition with a · ith an~ ~ j
an incidence of 1J n 250,0QO bjdhs. It is more common in '.!---2] :00Qopulation (Step 4, Fig.19._8).~!i}
India. The metabolic defect is the deficiency of homogen- Is completely absent, leading to defective synthesis of
tisate oxidase (step 3, Fig. 19.2 and item 2, Fig.19.8). melanin. The o r is Jiwoigmented ari iris
This results in e re · n of h m entisic acid i urine. may . e gra or red. There w~{ qe as_sociated _phot<!;_ YpvN)
Patient will hav i · U10Jmallif The abnormali~ _ehob1a, n mus and de,geas'ed visual ~u1ty. The Ll
is the bla£kem g of urine when come into contact with skin has low pigmentation, and s~ skin is sens1tiZ"e to-...UV
atmospheric oxygen. In the body, the homogentisic acid rays. The skin may show resence of n and '!lei
is oxidized and then polymerized to black colored mas. Hair is also white. Manifestations are less severe
alkaptone bodies. in tyrosinase positive type, where the abnormality is in
3rd or 4th decade of life, patient may develop
r ~ SiSi {deposition of 81.!@PlPPf baciie5 in interver-
tebral discs, cart'trages Of nose, pinna of ear) (Fig.19.7).
----
the uptake of tyrosine by melanocytes.

I HYPERTYROSINEMIAS
Black pigments are deposited over the conne~ v~
tis~s_iocluding_ joio.t crutities to produce F~ Hepat~ren~I Tyrosmem1a - -.- - -.- - -- -- - -

No specific treatment is required. But minimal protein (Tyrosmemia Type I) R.


intake with phenylalanine less than 500 mg/day is
recommended.
It is also called as i~
sive condition with an tnci
ef It is an autosomal reces-
ce of 1.5 per 1,000 births. It
..
294 Section B: General Metabolism

1 6
Phenylalanine - - - + Tyrosine - - • Hydroxyphenylpyruvic acid - - - -- ---+ Homogentisic acid

2t
DOPA ~ Dt A - - - - - - Thyroxine Maleyl acetoacer e

l
Dopamine
l
Dopaquinone Fumaryl acetoacetate

!
Norepinephrine l F,ma,ale (Gl•«>geolc~
+ Melanin

Epinephnne VMA Acetoacetate (Ketogenic)

1 = Phenylketonuna; absence of phenylalanine hydroxylase. rrnOIJJ!.'1 od.ou.,--c'.)


2 = Alkaptonuria: absence of homogentisic acid oxidase.
3 = Hypertyrosinem1a (Tyrosinemia type I); absence offumaryl acetoacetate hydroxylase.
4 = Albinism: absence of tyrosinase.
5 = Tyrosine hydroxylase, key enzyme of epmephnne synthesis.
6 = Tyros1nem1a type II; absence of tyrosine transaminase

Fig. 19.8: Summary of tyrosine metabolism


\6>-\. 1,• 1+ o -t 4 ½- (3)
is due to a deficiency of enzyme fumaryl acetoacetate of tryptophan are catabolized as formyl group (1 C which
hydrolase (step no. 5 in Fig.19.2 and item 3, Fig.19.8). enters the one carbon pool), lanme (3C, entering the
he fir JiJ~nd glucose pathway) and ciceloacetate (4C, going to keto-
age like odor and ~ l y- genic pathway). So, tryptophan is both glucogenic and
cemia and eventual ver failur~ are seen. There may ketogenic. The remaining 3 carbons are removed as
be mild me,ital retardation. Urine contains tyrosine and 3 CO2 mo ecules.
serum shows tyrosine. In the major pathway, kynureninase is an enzyme
A summary of phenylalanine and tyrosine metabo- d ende t on gyridoxal I) os te enzyme 2,Fig.
lism is shown in Figure.19.8. 19.9). Therefore, in vita · B8 ~ ency, the pathway
is blocked at this lev~ . This eads tGacin defici~
ITRYPTOPH~ N (TRP (W~ .---- - and manifestations of pellagra. The a~ mulated k mu.:.
reni n ~ o xan fiurenic ac@) So,in pyridoxal
Substances Produced 2fe'ficiency iJc c (Fig. 19.9) is excreted in
from Tryptophan are urine.
1. Alanine (glucogenic) Nicotinic Acid Pathway 9!--!ryptophan
Se.t.~ 2. Acetoacetyl-CoA (ketogenic) } N
Formyl group (One-carbon unit) About 97% molecules of tryptop~ r ~ metabolized in
~ 3.
the major pathway. About 3% molecules are diverted
~v,.\fJ.J 4. Niacin and NAO•
~0..-.,;\(J.>:- 5. Serotonin
at the level of 3-hydroxyanthranilic acid, to form NAO•
6. Melatonin l,~~I) ·
\t,'ti' 7 ' Hydroxyindoleacetic acid (excretory product)
'fo.9-. \1-v.'
~\C, ' 8. lndican (excretory product).
1 (Fig.19.10). The enzyme, QPRT (quinolinate phospho-
ribosyl transferase ) (step 1, Fig.19.10) is the rate-limit-
ing step.

~o;:o- Major Catabolic Pathw of Tryptopha·n


The development of pellagra-
like symptoms (see Chapter 33, r Niacin) in the
~(", Tryptophan is an a~ I amino acid with maize ea 1n o u tryptopti deficiency
an indole rin9. In the major pathway, tryptophan is oxi- in maize. Hydroxyanthranilate pro uc 10n is dependent
dized ~,tagh.,8!1 ii)'rro1gse (step 1, Fig. 19.9). It is on pyridoxa~phosp_hate (step no. 2, Fig. 19.9)".'"Hence in
a hemgpm~n. The enzyme is in ducible by cortico- @famin 8 8 deficien~otinamide deficiency is also
s ero1 s..,, In this major pathway, the total 11 carbon atoms manifested .
?~~'((}..
. ..
Chapter 19: Aromatic Amino Acids 295

J.. N DOl- :--------------------J~r-:i~


~ COOH ~ COOH

~ 77
~ C H 2 - f H :coo- "rNH 2 llNJ-cooH
NH 3
Ring opens
OH
3-hydroxy-
anthranillc acid l
Quinollnic acid

(-CO,)

~ COOH
N

~ COOH
ll.N
I
Ribose-5-phosphate

r " r - cooH Fig. 19.10 : Synthesis of niacin from t to han; PRPP= phosphori-
bosy\"yrophospha1e. = qu1nolinate phosphoribosyl transferase
l,eu o"e -:;ll"~)
CHO .J - NH2
HOOC ,.
Amino IJLUConat_e aldehyde Functions Serotonin
(- ~ t\
/' "'3. ~,, .) Serotonin is an important neurotransmitter in brain. 5-HT
Keto adipic acid ...,is an anti-depressant.
..J (-) 2C0212..-..@ When or Ina ro eIns are a all amino acids
1 are available in blood. This cause raffic jam in the
Ketogenic +-- CH3 -CO-CH 2-COOH Acetoacetyl-CoA ,,... amino acid transport systems in brain cells. Try12._tophan,
the b;:!lk~l,amino acid.is therefore t~ e i v slc>wiy. '/J'~
~ -- - - -- - -- - - - - -- ---t7in:~ lbcM However, when carbohydrate-rich diet is taken, •: ~o;
Fig. 19.9: MetabolisA1 ?ftryptophan l,.Q,r)\c:ui insulin secretion is increased, which will lower the amin~
l.-~ acid concentration in blood. So tryptophan easily enters
Serotonin@ ~: 'i~Ya.~iV\.L = ,.. 1011 Jre brain cells. When tryptophan is available in brain in
'])e,C.O,.,\>JO~~ iq excess quantity, serotonin may be generated to induce .~,::"
Serotonin (5-hydroxytryptamine) is produced in the sleep.(!ii,i:liiifu·drates wm induce sleep, w bl(e p.r.o-
brain, mast cells, platelets and gastrointestinal tract tein-nch foods will cause alert~
•mucosa. Tryptophan is first hydroxylated by !!YEto han ... Seroton~~ IE!vel is found to be low in patients~ith
l!Yj roxyl~ (step 1, Fig.19.11 ). This reaction is very p~ ~ p sycho~:-{,erotonin is involved in rl'Rlod,
much similar to phenylalanine hydroxylase (Fig. 19.1). ep, appetite an temf5'erature regulation. It increases
The coenzyme is tetrahydrobiopterine. The tetrahydro- gastrointestinal motility.
biopterine is regenerated using NAOPH. In the next step,
5-hydroxytryptophan is decar-boxylated to 5-hydroxy- Catabolism of Serotonin
tryptamine (SHT) (serotonin). The enzyme, decarboxy_. Monoamine OJ1cidase (MAO) converts serotonin to
lase requires pyridoxal phosp'hc:lft( (step 2, Fig. 19.11).'~ 5-hydroxyindolleacetic acid (HIAA) (step 3, Fig. 19.11 ).
296 Section B: General Metabolism

BOX 19.5: Mcld11,n <11HI 1nPl<1tonin rl,ff.,,.,,,t

CCT
dff'
CH2-yH~COO
Tryptophan
NH3 Melanin is the pigroem pf baic and 5kil)· it is synthesized from
N Tyrosi ne (Fig. 19.3).
NADP:xTetrahydrobiopterin Melatonin is a neurotransmitter synthesized from tryptophan
0
(Fig.19.11).
NADPH+H Dihydrobiopterin

~ , . -~>40 mg/dL). (Sometimes, oat cell carcinoma of lungs


may a{s~ _shpw j1.f[ee~epa5'ei~ secretion). The
5-hydroxytryptophan ( l patie~ ~pf~1n~TTiuslimg, sweating, intermittent diar-
/Q ~L rhea and often has fluctuating hypertension. Normally,
t?I 02 about 1% tryptophan molecules are channelled to sero-
HO~ CH2-C H2-NH2 tonin synthesis. But in ~ojd sypdro~, ~ o
V~) 5-hydroxytryptamlne is . d . i ~Qill Therefore ·acin defic1en~
N
3
2
~S , '1
(Serotonin)
olt~e~ -~llagra)may~eseenio ~ mme.

'(!5) NH3 ~ ~ , , Qei!d::::~=,--- Melatonin~


HO~ CH2-COO- Acetyl-CoA
VN) @ Serotonin is acetylated (step 4, Fig. 19.11 ). Further
5-hydroxy lndole methylated with the help of S-adenosyl methionine
acetic acid (5-HIAA) (SAM) (step 5, Fig.19.11 and Box 19.5). Pineal gland

SAH

Intestinal bacterial putre ction o t pt_oph§l results in


1 = tryptophan hydroxylase. the production of several · dole tiifcfs. These are
2 = PLP dependent aromatic amino acid decarboxylase. mainly excreted in the feces a atoxy ~ e part is
3 = monoamlno oxidase (MAO).
absorbed, detoxified an e 1n urine~ indoxyl
= meth I trans erase and indican.® dic~ s the potassium salt of i ~
Fig. 19.11: Serotonin and melatonin S,Ynlhesis sulfate. The foul smell of feces and the natural color
ge~ef"l;n __;;t -rno cd! • of urine is due to these compounds. The major urinary
'"'\} K)"1 te-e-P E... --n.r>~ '7 ,, excretory products of tryptophan are 5-hydroxy indole
This is similar to degradation of epinephrine (step 6, acetate and indole-3-acetate.
Fig.19.4). MAO inhibitors (e.g. iproniazid) will cause
mood elevation. Hartnup Disease@
Carcinoid Tumors @ The name originates after the first family in whom the
disea~ ~~ described. It is an inherited autosomal
'S,,Jl,llli:::::~.-:::::::,oo:;::e~lls of the gastro- recessive diseasP. Absorption of aromatic amino acids
intestinal tract and is necess GIT~ otility. These from intestine a's well as reabsorption from renal tubules
cells may grow into locally ma ignant argentaffinomas , a~ ctive. So amino acids are excreted in urine. The
4 •
otherwise known as carcino1K tumors. These tumors (eel~~ like symptoms are due to the deficiency of
develop in smqir intestine or in the a,911endix. Serum ~ i n derived fr tryptophan. The common manifes-
level of serotonin is increased in carcinoid tumors tations are ermatitis and diagnosis is based
y A
-v-~~
Chapter 19: Aromatic Amino Acids 297

Tryptophan - - - ~ N-formyl kynurenine --+ Formyl THFA Histidine


,,....:::::::::T'"
1-
i i i CH2-CHNH2-COOH

5-0H-tryptopha n

!
Kynurenine

+
1-C Unit N~NH

.-,~~.::::.,...,- CH=CH-COOH
NH,@1 1
3-hydroxykynurenine
Urocanic acid C\
Serotonin
+ +

J~
N NH

!
Alanine + 3-0H anthran11ic acid
(+) H20l 2
Glulgenic
CD

P
CH2-CH2-COOH
Acetoacetate 1.,
5-HIAA l H tmadazolone
Ketogenic
ni: acid \.N
{: ~:l: l

lnd1can Site of cleavage


.
Fig. 19.12: Summary of tryptophan metabolism COOH- CH-CH2-CH 2-COOH
I Formimino glutamlc
NH-CH=NH / @ Id (FIGLU)

,_,==.:::_....,I,-- CH2-CHNH2-COOH Histidine


C,.LU / (~ )THFAl 4
N~NH COOH-CHNH2-CH 2-CH 2-COOH
1 Glutamic acid

,. .====-.. .,- CH2-CH2NH2


co,~ ""
L ex- KG!tt T
N'-Formimino THFA
Pl
Histamine 1= hislidase. 2= urocan e. 3= imidazolone
N VNH propionate hydrolase; 4= THFA-form,mino transferase.

Fig. 19.13: Metabolism of histidine

,-I==--=---.-1- CH2-COOH formimino glutamic acid (FIGLU) is formed (step 3,


NADH
Fig. 19.13). Figlu is cleaved into N5-formimino-THFA
NVNH H202
and glutamic acid (step 4, Fig.19.13). Glutamic acid is
lmldazole acetic acid
1 = Histidine decarboxylase. ha keto lutar
2 = Histaminase {mono amino oxidase).

Fig. 19.14: Metabolis m of histamine

on aminoaciduria and increased excretion of i


compounds. Patient imp o when put on a high pro-
tein diet with suppl~71en ion o ia,fn and minimum
exRosure to s~tlf-The (Q§iiro s chiatric manifesta) Histamine
tions of Hartnup disease are said to be resp sible for
Histamine is formed from histidine by decarboxylation,
the sadistic and bizarre behavior of empero ·ke Nero
and Caligula. [ 'n~T'\ r-t\f'JO > t-\1.J, ~J catalyzed by histidine decarboxylase (step 1, Fig. 19.14).
A summary of tryptophan metabolism is shown in The effects of histamine are smooth muscle contraction,
Figure 19.12. enhanced vascular permeability, increased acid secre- "\
tion. So, histamine causes fall in blood pressure.CV'l..llodl\olo'tJ

I HISTIDINE (HIS) (H) The major cells producing histamine are platelets,7 S. tf€

Histidine has an imidazole ring.~ a semi-essential


basic amino acid. Its pK value i ~ and hence in pro-
mast cells and basophils. ~e.o.s,....gic1:J...9sJ
p~ n~ illin will elicit lgE antibodies that ar~ fixed on_the
mast cells. When the next dose of penicillin is injected, it
*
teins, his 1 • • responsible for the maximum buffering reacts with the antibodies; and degranulation of mast
action. cells takes place. Histamine and slow reacting sub-
Histidine is first non-oxidatively deaminated by histi- stance (SRS) are released. This leads to peripheral
dase to form urocanic acid (step 1, Fig. 19.13). Then vasodilatation, fall in blood pressure and anaphylaxis.
298 Section B: General Metabolism

Antihistamines are drugs which block histamine


Glucogenic Amino acids / Ketogenic
receptors. They are used to control: llecqic a aa- Glucose . 'd
Alanine - + Pyruvate ammo ac1 s
pl!z'lactic reactions. The stimulant effect of histamine Glycine
Serine
I / •
.
Leucine
on gastric acid secretion is by acting on H2 receptors. Threonine Lysine
c steine Oxalo lsoleuclne
Hence, H2 r~ r ! trui_onists are used in the treat-
ment of acid P.eptic u cers of stomach.
TY
I
ryp op
h
8
/4 acetic + Acetyl-CoA
acid ~ ~
Pherryr
Tryptophan
Aspartate / Citrat{
A) Histidinemia
Fumarate
' Ketone bodies
It is an au osomahecessi\1e disease. The deficiency
oL.bistidase leads to accumulation of histidine in blood
t\ Citric acid cycle
PhefTyr
and body fluids and increased e~on o i idazole
.Q ic cid in urine. The clinical features include men- succinyl-CoA Alpha keto---+ Glutamate
tal retardation and de,!gyed speech develoP.ment. A low f ,._____ glutarate t
histidine diet may have some effect. " Methionine, lsoleucine, Valine Arginine, Histidine, Proline

Fig. 19.15: Metabolic fates of amino acids


~) Urocanic Aciduria
It is due to the de · rocaoase. Urocanic acid On the other hand, those amino acids which produce
and histidine are excreted in urin .,,Clin~ manifesta- acetyl-CoA are called ketogenic amino acids. Acetyl-

I
tions are minimal'(C CoA entering into the TCA cycle, is completely oxidized.

PROLINE (PRO) (P) AND


Therefore, there is no net synthesis of glucose from
acetyl-CoA. So, acetyl-CoA is not entering into the glu- ,
coneogenesis pathway. Acetyl-CoA, however, can give
HYDROXYPROLINE
rise to ketone bodies. Thus, amino acids entering as
acetyl-CoA are known as ketogenic amino acids.
..
These amino acids are shown in Figure 19.15.
But some amino acids are shown in both the lists.
Phenylalanine, tyrosine, tryptophan and isoleucine
oxidation. are both glucogenic and ketogenic. This is because,
during their metabolism, part of the carbon skeleton will
enter into some of the TCA cycle intermediates; while
the other part will generate acetyl-CoA (Fig . 19.15).

I AMINOACIDURIAS
is reduced, causing decreased strength of fibers, lead-
Aminoacidurias are clinically very important. It is esti-
ing to scurvy. The excretion of hydroxyproline in urine
mated that about 20 to 25% of the inmates of psychiatric
is increased in tumors infiltrating bones and in diabetic
hospjtals. ~ g fr'om one of the ai;!inoecid• Irias.
patients due to enhanced rate of protein catabolism. Most of them man~ s mental retardation. It is
important to remember that thei ntal retardation could
Fate of Carbon Skeletons be prevented, if the condition is · gnosed immediately
of Amino Acids after the birth, and adequate tre nt is started. Delay
During catabolism of carbon skeletons, amino acids may in diagnosis for each week will a preciably reduce the
enter into the TCA cycle and they can be converted to intelligence quotient.
glucose. In other words, those amino acids, which give
rise to citric acid cycle intermediates can be converted
:ji.. Clinical Case Study 19.1
to glucose. Hence, those amino acids entering into TCA
cycle, or at pyruvic acid level are called glucogenic A 50-year-old man with 2 years history of refractory
amino acids. This is shown in Figure 19.15. hypertension and occasional panic attacks reported to
Chapter 19: Aromatic Amino Acids 299

the clinic with sudden episode of pounding headache.


There was excessive sweating. He had similar attacks 4. Tyrosinase requires copper, while tyrosine hydroxy-
earlier. Family history is positive for hypertension. On lase requires tetrahydrobiopterine as co-factor.
examination, BP was 170/90 mm Hg and pulse was 5. L-Dopa, an inhibitor of DOPA decarboxylase is
72/min. Weight was 80 kg. He was taking beta block- used in the treatment of Parkinson's disease.
ers. Other examination findings were unremarkable. 6. The end product of epinephrine metabolism is vani-
24 hours VMA was elevated (12.0 mg/day). What is the llylmandelic acid (VMA). Its excretion is increased
probable diagnosis? in pheochromocytoma and neuroblastoma.
7. Phenylketonuria (PKU) is an inborn error of metab-
olism, caused by defective Phenylalanine hydroxy-
8· Clinical Case Study 19.1 Answer
lase enzyme. The disease manifests with profound
Patient is suffering from pheochromocytoma, a catechola- mental retardation, convulsions and hypopigmen-
mine-producing tumor of sympathetic or parasympathetic ted skin.
nervous system. Catecholamines are degraded by two 8. Alkaptonuria is due to deficiency of homogentisate
enzymes, COMT and MAO and the final product is VMA, oxidase. There is excretion of urine that darkens on
vanillylmandelic acid. This is increased in pheochromocy- standing.
toma (and paraganglioma, another similar tumor). 9. Absence of tyrosinase leads to 'albinism'.
Pheochromocytoma usually is seen around the age 10. Tryptophan produces biologically important prod-
of 40, though it can set in at any time. The "rule of ten" for
ucts, viz Nicotinic acid, Serotonin and Melatonin.
pheochromocytoma states that 10% are bilateral, 10%
11 . Serotonin is an important neurotransmitter. It is
are extra-adrenal, and 10% are malignant. Up to 25%
catabolized to 5 HIAA, it is increased in carcinoid
may be familial. It may be part of a syndrome known as
tumors.
multiple endocrine neoplasia (MEN). Common laboratory
12. Melatonin produced in the pineal gland, regulates
investigations are 24 hours urinary VMA, metanephrine
the biological rhythms and sleep-wake cycle.
(total and fractionated), and plasma metanephrines.
Clinical features are varied, but the classic triad of 13. Decarboxylation of histidine produces histamine. It
symptoms is episodes of palpitations, headaches and is a stimulator of gastric secretion, both acid and
profuse sweating. Both benign and malignant pheochro- enzymes.
mocytomas can recur (after surgery) and hence long- 14. Glucogenic amino acids are: Glycine, Alanine,
term follow-up is important. Recurrence rate is also 10%. Valine, Serine, Threonine, Aspartic acid, Glutamic
acid, Asparagine, Glutamine, Methionine, Cysteine,
I LEARNING POINTS, CHAPTER 19 Arginine, Histidine, Proline.
15. Ketogenic amino acids are: Leucine, Lysine.
1. Phenylalanine is an essential amino acid. It has a
16. Partially glucogenic and partially ketogenic amino
sparing action on Tyrosine.
acids are: Phenyl alanine, Tyrosine, Tryptophan, lso-
2. Tetrahydrobiopterine is an important co-factor for
leucine.
phenylalanine hydroxylase, deficiency of which can
17. Aminoacidurias are inborn metabolic disorders,
lead to inborn metabolic defect.
which lead to mental retardation in most cases. Early
3. Biologically important products from tyrosine are
detection and prompt management helps prevent
melanin, catecholamines and thyroid hormones.
complications.

PART-1: ESSAY AND SHORT NOTE QUESTIONS


19-1 . Describe the steps of catabolism of phenylalanine and tyrosi ne. Ind icate the inborn errors of metabo lism
associated with this pathway.
19-2. Describe t he steps by which catecholamines are sy nthesized. What is t he fi nal excretory product of catechol-
amines?

- - - - -- - ---
302 Section 8: General Metabolism
19-15. Which amino acid will give rise to a vitamin? 19-22. Try~~tn is excreted in large quantities in which
Tryptophan gives rise to pyridoxal. condition?
19-16. Serotonin is derived from which amino acid? Hartnup disease.
Tryptophan. 19-23. What is physiological importance of histidine?
19-17. What is serotonin? Histidine has a pK value of 6.8; and so it has the maxi-
It is derived from tryptophan. It is a vasoconstrictor. mum buffering capacity at physiological pH.
19-18. What is MAO (Monoamino oxidase)? 19-24. What is the precursor of histamine?
It deaminates monoamines. MAO inhibitors are used Histidine.
to treat hypertension and depression. 19-25. What is decarboxylation product of histidine?
19-19. Tryptophan is deficient in which food stuff? Histamine.
Maize and corn. 19-26. What is the clinical significance of histamine?
19-20. Pellagra is manifested in which conditions? It is a powerful vasodilator and mediator of anaphylaxis.
Niacin deficiency; Pyridoxal deficiency; Tryptophan 19-27. What are partially ketogenic and partially gluco-
deficiency; Carcinoid syndrome. genic amino acids?
19-21. What is Hartnup disease? Phenylalanine, tyrosine, tryptophan, isoleucine.
~J...a[omatic amino acids from in ~ 19-28. What are important aminoacidurias which cause
w ~~tsciotiqp fcoro regal t11b11les are~~./ mental retarrf;,~1-=n?
So tryptophan deficiency, and pellagra like symptoms Phenylketonuria, homocystinuria, maple syrup urine
are seen. disease.
.,______ __ _ Chapter 20
• Citric Acid Cycle

Chapter at a Glance
The learner will be able to answer questions on t he following topics:
Citric acid cycle Regulation of citric acid cycle
Significance of citric acid cycle Integration of metabol ism

Historical Perspectives
l
Before 1937, Car Martias, Fray Knoop and Albert Szent-Gyorgyi had elucidated most of the reactions described in this chapter.
The complete cycle was proposed by Sir Hans Krebs in 1937 (Nobel prize, 1953). The cycle is therefore named after him.
.. Please note that the name is Krebs cycle (there is no apostrophe). Krebs proposed the original name as TCA (tricarboxylic
acid) cycle, because he was not sure whether citric acid is a member of the cycle. Later, Ogston (1948) showed that the tricar-
boxylic acid in question is indeed citric acid, and so the name Citric Acid cycle was given later. Scheele in 1780 had isolated
citric acid from citrus fruits.

r v-.\ • ( ' " 11-,-W~


Functions of the Citric Acid Cycle· ......-G-tu_i _se
- - __-_-:-
.----- - - ~- --.o'""x~id-aoo
..:i·c:::
n ,.,____,
f\ \ p<!i q"' --hl\_QU;; "f\, \t\ex(_ + -
1. The final common' oxidative pathway that oxidizes ~ate Fatty acid
acetyl-CoA to CO 2 .1 ~J c?. r C'N).SU"-j ----::::: Acetyl-Co.A ~ Cholesterol
2. The source of reduced coenzy!!)eS that provide the Fatty acids?
substrate for the r~pira~~a~ steroid hormones
3. The link betweer '~tabolic and anabolic pathways Ketogenic
(amphibolic role). ' cc ~~) amino acids Ketone bodies
4 . Provides precursors for synthesis of amino acids Fig. 20.1: Sources and utilization of acetyl-CoA
and nucleotides. -• \ 41 ''?'> 0
5. Components of the cycle have a direct or indirect con- {The acetyl-CoA is also derived from beta-oxidation of
.--==--~
trolllng_e.ffects on key enzymes of other pathways. fatty acids. All the enzymes of citric acid cycle are loca-
- - h.e.::d.. &, ~"C\t'.......,._ ted inside the mitochondri~
Reactions of the Cycle Preparatory Steps
Acetyl-CoA enters the cycle, and is completely oxidized. First Step: Formation of Citric Acid
During this process, energy is trapped. The sources of The 4 carbon, oxaloacetate condenses with 2 carbon,
acetyl-CoA are shown in Figure 20.1.{f>yruvate derived acetyl-CoA to form 6 carbon compound, the citrate
from glycolysis is oxidatively decarboxylated to acetyl- (tricarboxylic acid). The enzyme is citrate synthase
CoA by the pyruvate dehydrogenase (see Fig. 10.19). (Step 1, Fig. 20.2). The hydrolysis of the thioester bond
This is the link between the TCA cycle and glycolysi~ in acetyl-CoA drives the reaction forward. This is an
1\/\ :¼-t- ATf'
?~~u.'<ah- 9 1 l'JM:t) ff\i) ., (;'TP '!lo
(.3C) Jilln~ ,'PD\-\ ,tl\lU~ ""\i co.,.__
304 Section B: General Metabolism

Fig. 20.2). A molecule of GDP is phosphorylated to GTP


and succinate is formed. The GTP can be converted to
ATP by reacting with an ADP molecule:
GTP +ADP -. GDP + ATP

Citrate is isomerized to isocitrate by aconitase (Step 2,


Fig. 20.2). This reaction takes place in ste~. with Sixth Step: Formation of Fumarate
cis-aconitate.as the intermediary. Succinate is dehydrogenated to fumarate, an unsatu- ..
rated dicarboxylic acid, by succinate dehydrogenase
Third Step: Formation ofAlpha Ketoglutarate (Step 6, Fig. 20.2). The hydrogen atoms are accepted
by FAD. The FADH 2 then enters into ETC to generate
This reaction is catalyzed by the enzyme, isocitrate
ATPs. The ~cinate dehydrogenase is competitively
dehydrogenase (Step 3, Fig. 20.2). First isocitrate is
inhibited by malonate (see Fig. 5.19).
i d~ enated to form oxalosuccinate. It undergoes
J, spontaneous decarboxylation to form alpha ketogluta-
Seventh Step: Formation of Ma/ate
.+ rate. The NADH generated in this step is later oxidized
in electron transport chain (ETC) to generate ATPs. The formation of m~l::tte from fumarate is catalyzed by
lsocitrate (6 carbons) undergoes oxidative decarboxy- fumarase (Step 7, Fig. 20.2). The reaction involves the
lation to form alpha ketoglutarate (5 carbons). In this addition of a water molecule.
reaction, one molecule of CO2 is liberated.
Eighth Step: Regeneration
Fourth Step: Formation of Succinyl-CoA of Oxaloacetate
Next, alpha ketoglutarate is oxidatively decarboxy- Finally malate is oxidized to oxaloacetate by malate
lated to form succinyl-CoA by the enzyme alpha keto- dehydrogenase (Step 8, Fig. 20.2). The coenzyme is
glutarate dehydrogenase (Step 4, Fig. 20.2). The NAO•. The NADH is generated in this step, which enters
NADH thus generated enters into ETC to generate the electron transport chain, when ATPs are produced.
ATPs. Another molecule of CO2 is removed in this step. The oxaloacetate can further condense with another
This is the only irreversible step in the whole reaction acetyl-CoA molecule and the cycle continues (Fig. 20.2).
cycle. The enzyme alpha ketoglutarate dehydrogenase
is a multienzyme complex having 3 enzyme proteins Oxaloacetate as a Junction P~.t uWO-.
and 5 coenzymes. This is similar to the pyruvate dehy- :1\,>.4't--w.e, mn,,,"\1'1£.9'..-iilc.tc
Oxaloacetate may be viewecl as a catalyst, which ehftWs
drogenase reaction (Compare Fig. 20.3 with Fig. 10.19).
into the reaction , caug!s com_Elete o x ~n of acetyl-
CoA and comes out of it without any change. Oxalo-
Fifth Step: Generation of Succinate acetate is an important junction point in metabolisms. Var-
The next reaction involves a substrate level phospho- ious reactions of oxaloacetate are shown in Figure 20.4.
rylation whereby a high energy phosphate is generated Significance of citric acid cycle is shown in Box 20.1.
from the energy trapped in the thioester bond of suocinyl- \ & N \ F \ Ci'I" (£ Of T c..A ·
CoA. The enzyme is succinate thiokinase (Step 5, Complete Oxidation of Acetyl-CoA
_ _ _--"",.....
~,,._ 0 2 Removal Steps
During the citric acid cycle, two carbon dioxide mole-
cules are removed in the following reactions:
A. Step 3, oxalosuccinate to alpha ketoglutarate
B. Step 4, alpha ketoglutarate to succinyl-CoA (Fig.
20.5).
Carl Albert Alexander Hans Adolf Acetyl-CoA contains 2 carbon atoms. These two
Wilhelm Szent- George Krebs carbon atoms are now removed as CO2 in Steps 3 and 4.
Scheele Gyorgyi Ogston NP 1953
Net result is that acetyl-CoA is completely oxidized
1742- 1789 NP 1937 1911-1996 1900-1981
1893-1986 during one turn of cycle.
Chapter 20: Citric Acid Cycle 305

)'
H3C- C - 5--CoA
Acetyl-Co A CoA-SH

CHC.COO
I -
Ho- y-eoo
o=y-coo- 1 Citrate synthase
CHc-C~ O- Oxaloacetate (Irreversible) c;~~• ~
NADH+H+
ate dehydrogenase 2
HO- CK-COO- NAO+ • ~- ~ ~---~ CK-C00-

l f~;~H
2 Aconlt.aae 11 _
CH -COO Malate
2 Cis•Aconitate y-eoo _
• H ~~

2 Aconltase HO- yK-COO-


lsocitrate yK-COO
H- C-COO- Cfii-COO
- I - Fumarate
OOC- C-H

FADH 2
CITRIC
6 Succlnate dehydrogenaae
ACID
CYCLE
O=C-COO-
Oxalosuccinate tH-C00-
3 lsocltnte tH 2-COO-
dehydrogenaae

5 Succinate thloldnase

Alpha ketoglutarate o=y-eoo-


CH2
CH2-COO-

1
NAO+
NADH•~• TPP / __,.,..-
Succinyl-CoA

4 Alpha ketoglutarate dehydrouenaae


(Irreversible)

Acetyt-CoA (2 carbon), enters the cyde. These are released as CO2 in steps 3 and 4. SoAcetyl-CoA is completely oxidized by
the time cycle reaches alpha ketoglutarate.
All reacllOns are reversible· extllpt 1st end 4th steps

Fig. 20.2: Krebs cycle or citric acid cycle or lricarboxylic acid cycle
g
Generating Steps in TCA Cycle marked in Figun3 20.5 and in Table 20.1 . The summary
is shown in Tablt3 20.2.
There are 3 NADH molecules generated during one
Note: Recent wiork shows that in the electron transport
cycle, each of them will give rise to 2½ ATPs on oxida-
chain, NADH produces only 2.5 ATPs and FADH only
tion by electron transport chain (ETC}; so altogether
they will give 3 x 2½ = 7½ (7.5) high energy phos- 1.5 ATPs. The old values are also given for comparison
phates. The FADH2 will generate 1½ molecules of ATP. in Table 20.1.
In addition, one molecule of GTP (equivalent to one Alpha ketoglutarate dehydrogenase reaction is the
molecule of ATP} is formed by substrate level phos- only one irrever.sible step in the cycle. The free energy
phorylation . Hence, per turn of the cycle, 10 high changes of the reactions of the cycle are such that the
energy phosphates are produced. These steps are cycle will operate spontaneously in the clockwise direction.
306 Section 8: General Metabolism

Only about 33% of liberated energy is trapped as Figure 20.9, all the major ingredients of food stuffs are
ATP. The rest is used to keep the body temperature at a finally oxidlzed through the TCA cycle.
higher level than the environment. Almost all the biochemical processes use ATP
for meeting energy needs-muscle contraction, active
~ Fina Common Oxidative Pathway
Citric acid cycle may be considered as the final com-
mon oxidative pathway of all foodstuffs. As shown in

a KG-dehydrogenase
Alpha ketoglutarate ? "\ " Hydroxy succinyl-TPP
+ Acetyl-CoA

i
TPP CO2
Trans-succinylase
Hydroxy succinyl-TPP 7 "\ " Succinyl lipoamide Aspartate Malate Citrate

Lipoamide TPP Fig. 20.4: Reactions of oxaloacetate


Trans-succinylase
Succinyl lipoamide 7 '"\ " Succinyl-CoA cance of c1tnc acid cycle

CoA Dihydrolipoamide
(i., Complete oxidation of acetyl-CoA
2. ATP generatioin
DHL-Dehydrogenase v.3. Final common oxidative pathway
Dihydrolipoamide 7 " Lipoamide A.
.-5.
Integration of major metabolic pathways
Fat Is burned on the wick of carbohydrates
FAD FADH2
6. Excess carbohydrates are converted as neutral fat
FADH2 ---?....,._,.'\---+" FAD (z_ No net synthe:sis of carbohydrates from fat
(!!, Carbon skeletQns of amino acids finally enter the citric acid
NAO+ NADH + H+ cycle
Fig. 20.3: Alpha ketoglutarate dehydrogenase reaction; compare 9. Amphibolic p21thway
It with Figure 10.19 JO. Anaplerotic role.

Oxalo- -.,!cetyl-CoA ATP inhibits

4
Malate \
H
acetate ........,~
1 .X#

Citrate
(physiological
regulation)

i
/ \SATP 2
Fluoroacetate (toxic)
Aconita
Fumarate

Malonate (toxic)
FADH2 1.SATP

,
')
lsocitrate

1 = Citrate synthase
2 = Aconitase
3 = lsocitrate dehydrogenase
Succi: ate
A //
ATP
2 5,tl~NADH NADH inhibits;
ADP activates
4 = Alpha ketoglutarate (physiological)
~ ::. 25ATP
Oxalosuccinate

;{
dehydrogenase
5 = Succinic thioklnase
6 = Succinate dehydrogenase
7 = Fumarase 1)(__ 4 ~ ADH Alpha
8 = Malate dehydrogenase keto- ... CO
Arsenite (toxic)~ o 2
glutarate
2

Fig. 20.5: Summary of Krebs citric acid cycle. Enzymes are numbered. Reactions number 3 and 4 are carbon dioxide elimination
steps. Physiological regulatory steps are: Step No.1(citrate synthase) is physiologically inhibited by ATP. Step No.3 (ICDH) is inhibited
by NADH and activated by A DP. Steps where energy is trapped are marked with the coe,nzyme and the number of ATP generated dur-
ing that reaction. A total of 10 ATPs are generated during one cycle. Recent work shows that in the electron transport chain, NADH may
produce only 2 ½ ATPs and FADH only 1 ½ ATPs
Chapter 20: Citric Acid Cycle 307

1ometry of the TCA cycle


Acetyl-CoA 2 CO2 +coA-SH
Oxaloacetate Oxaloacetate
3 lsocitrate NADH 3 2.5
alpha ketoglutarate FAD FADH2

4 Alpha ketoglutarate NADH 3 2.5 3NAD• 3 NADH


succinyl-CoA GDP+Pi GTP
l 5 Succinyl-CoA GTP
Succinate
6 Fumarate FADH 2 1.5
8 Malate Oxalo NAOH 3 2.5 Carbohydrates ' Fat
acetate
Total 12 10 Fat (acetyl-CoA) X Carbohydrates

Fig. 20.7: Fat cannot be converted to glucose

One passage of cycle oxidizes acetyl-CoA into two CO2


molecules. Here oxaloacetate acts as a true catalyst; it
enters the cycle aind is regenerated at the end. The major
" source of oxaloacetate is pyruvate (carbohydrate).
Hence, carb9hydrates are absolutely required for oxida-
tion of fats, o fa1ts are burned in the fire of carboh -

• Fig. 20.6: Flame needs a wick; oxidation of fat needs carbohydrate


xcess Carbohydrates are
transport, biosynthetic reactions, etc. A thermodynami- Converted as Triacyl Glycerol
cally unfavorable reaction when coupled with hydrolysis
Excess calories are deposited as fat in adipose tissue.
of becomes favorable.
1) The pathway is glucose to pyruvate to acetyl-CoA to fatty
Integration of Major Metabolic Pathways acid. However, fat cannot be converted to glucose
i. Carbohydrates are metabolized through glycolytic because pyruvate dehydrogenase reaction (pyruvate
pathway to pyruvate, then converted to acetyl-CoA, to acetyl-CoA) is an absolutely irreversible step (Fig.
which enters the citric acid cycle.
ii. Fatty acids through beta-oxidation, are broken
down to acetyl-CoA and then enters this cycle. ~ et Syntlhesis of
iii. Glucogenic amino acids after transamination Carbohydra1tes from Fat
enter at some or other points in this cycle (Fig.
20.9). Ketogenic amino acids are converted into Acetyl-CoA entering in the cycle is completely oxidized
acetyl-CoA. to CO 2 by the time the cycle reaches succiny-CoA (see
iv. The integration of metabolism is achieved at junc- Fig. 20.2). So, ac:etyl-CoA is completely broken down in
tion points by key metabolites (Fig. 20.9). Several the cycle. Thus ;acetyl-CoA cannot be used for gluco-
pathways can converge at this point with the result neogenesis. Therefore, there is no net synthesis of
that carbon atoms from one source can be used for carbohydrates from fat (Fig. 20.7).
synthesis of another. Important intermediates are
P. '.[Uvate, acetyl-CoA and oxaloacetate . A~ no Acids Finally Enter the TCA Cycle
Fat is Burned on the Wick Some amino acids, such as leucine, catabolized to
acetyl-CoA; are not converted to glucose, because
of Carbohydrates
pyruvate to acetyl-CoA reaction is irreversible. The
The oil in a lamp by itself cannot be lighted; the flame acetyl-CoA mole,cules either enter the TCA cycle and
needs a wick (Fig. 20.6). Similarly in the body, oxida- are completely oxidized, or are channeled to ketone
tion of fat (acetyl-CoA) needs the help of oxaloacetate. body formation. Hence, they are called as ketogenic
308 Section B: General Metabolism

Glucose ----+ Pyruvate

l "'
Acetyl-CoA - - + Fatty acid synthesis

Tryptophan-+Jj,lanine /
W l'I
Aspartate Oxaloacetate Citrate Acetyl-CoA

I
Aspartate Oxaloacelate

Gl,oo,,.._
genesis ~
Jp0,2,
alate

Phenyl-
I( c'"\
/ J~j .
% alanine; - Fumarate o.-ketoglutarate
Tyrosine
Heme +-- Succinyl-CoA
Alpha keto-
glutaric acid
Ketolysis
!
Glutamic acid
Succinyl-
CoA
f 't',~ His~id_ine

!
GABA
,Vl<-l.'tlr4'
I'
Propionyl-CoA
f
Arginine
Praline
Odd chain
Valine. lsoleu~!ne, Methionine fatty acids

Fig. 20.8: Efflux of TCA cycle intermediates Fig. 20.9: Influx o'f TCA cycle intermediates

amino acids (Fig. 20.9). Glucogenic amino acids get are essential. Tllis is called anaplerotic role of TCA cycle
converted to intermediates of TCA cycle. (Greek word, ana = up; plerotikos = to fill). Anaplerotic
reactions are "fiilling up" reactions or "influx" reactions
TT-
r' Amphibolic Pathway or "replenishin!Q" reactions which supply 4-carbon units
All other pathways, such as beta-oxidation of fat or gly- to the TCA cycle (Fig. 20.9). The important anaplerotic
reactions are:
cogen synthesis are either catabolic or anabolic. But
a. Pyruvate to oxaloacetate by pyruvate carboxylase
TCA cycle is truly amphibolic (both catabolic and ana-
enzyme (sHe Fig. 10.21 ). It needs ATP.
bolic) in nature. (Greek, amphi = both). There is a con- b. Glutamate is transaminated to alpha ketoglutarate:
tinuous influx (pouring into) (Fig. 20.9) and a continuous and aspartate to oxaloacetate. Other important
efflux (removal) of 4-carbon units from the TCA cycle amino acids entering the TCA cycle are shown in
(Fig. 20.8). In a traffic circle, many roads converge and Figure 20.9.
traffic flows towards one direction. Since various com- c. Pyruvate can be carboxylated to malate by NADP•
pounds enter into or leave from TCA cycle, it is some- dependent malic enzyme.
times called as "metabolic traffic circle". Important
anabolic reactions related with citric acid cycle are: IREGULATION OF fa
a. Oxaloacetate is the precursor of aspartate ~ IC ACID CYCLE ~ c.o.l_
b. Alpha ketogutarate can be made into glutamate
Citrate and Citrate Synthase~ =
c. Succinyl-CoA is used for synthesis of heme
The formation a,f citrate from oxalo acetate and acetyl-
d. Mitochondrial citrate is transported to cytoplasm,
CoA is an important part of control (Step 1, Fig. 20.5).
where it is cleaved into acetyl-CoA, which then is the ATP acts as an allosteric inhibitor of citrate syn-
starting point of fatty acid synthesis (see Fig. 13.11 ). thase. Citrate inhibits PFK (key enzyme of glycolysis):

@ Anaplerotic Role of TCA Cycle stimulates fructose-1,6-bisphosphatase, (key enzyme


of gluconeogenesis) and activates acetyl-CoA car-
The citric acid cycle acts as a source of precursors of boxylase (key ernzyme of fatty acid synthesis).
biosynthetic pathways, e.g. heme is synthesized from
succinyl-CoA and aspartate from oxaloacetate. To coun- Availability and Cellular Need of ATP
terbalance such losses, and to keep the concentrations When the eneq~y charge of the cell is low, the cycle
u- ~ -•-.·.__...____~ - - - . . , . - - - - - - - - - - - ~ - - - - - - -
Chapter 20: Citric Acid Cycle 311

20-16. The amino acids entering TCA cycle as succinyl- C. Operates in the mitochondria close to ETC
CoA are all except: 0 . It is a cyclical process
A. Valine 20-19. Which of the intermediates listed is a dicarboxylic
B. Methionine hydroxy acid?
C. Tyrosine A. Succinate B. Fumarate
0 . lsoleucine C. Malato 0 . Oxaloacetate
20-17. Which of the reactions listed replenishes a TCA 20-20. Which amino acid can enter the TCA cycle as fuma-
cycle intermediate? rate and oxaloacetate?
A. Heme synthesis A. Aspartate B. Glutamate
B. Transamination of oxaloacetate C. Arginiine 0. Serine.
C. Carboxylation of pyruvate 20-21. From the pairs of inhibitors and enzymes, pick out
0 . Reutilization of ketone bodies. the mismatched pair.
20-18. The TCA cycle is the final common oxidative path- A. Alpha ketoglutarate dehydrogenase and arsenite
way beca use B. Malonate and succinate dehydrogenase
A. It provides large a fraction of energy
C. Fluorc,acetate and aconitase
B. Acetyl-CoA derived from all sources can be oxidi-
0 . lodoacetate and malate dehydrogenase
zed

ANSWERS OF MULTIPLE CHOICE UESTIONS


20-1 . B 20-2. 0 20-3. A 20-4. C 20-5. C 20-6. C 20-7. 0
20-8. A 20-9. 0 20-10. A 20-11. B 20-12. C 20-13. 0 20-14. 0
20-15. B 20-16. C 20-17. C 20-18. B 20-19. C 20-20. A 20-21 . 0

PART-3: VIVA VOCE QUESTIONS AN ANSWERS

20-1. What are the steps in which carbon dioxide is 20-6. How maniy ATPs are generated per one rotation of
liberated, during oxidation of glucose? the citric acid cycle?
Pyruvate dehydrogenase; lsocitrate dehydrogenase; 10ATP .
Alpha ketoglutarate dehydrogenase 20-7. What is tihe net yield of ATP from one molecule of
20-2. Acetyl-CoA is produced from what substrates? glucose in anaerobic glycolysis?
2ATP.
Pyruvate; Fatty acids; ketogenic amino acids
20-8. What is the net yield of ATP from one molecule of
20-3. Acetyl-CoA is used for what purposes?
glucose in aerobic glycolysis?
Oxidation in TCA cycle; fatty acid synthesis; choles-
?ATP.
terol synthesis; ketone body formation. 20-9. During cc>mplete oxidation, what is the net yield of
20-4. Give examples of substrate level phosphorylation. ATP from one glucose molecule?
1,3-bisphospho glycerate kinase; Pyruvate kinase; 32ATP.
Succinate thiokinase. 20-10. What is a1naplerosis?
20-5. Which is the substrate level phosphorylation step The reactions which involve replenishment of TCA
in the TCA cycle? cycle intermediates. For example, pyruvate gives rise
Succinate thiokinase to oxaloacetate.

Ir
_ _ _ _ _Chapter 2J
Biological Oxidation and
Electron Transport Chain

Chapter at a Glance
The learner will be able to answ er questions on the follow ing topics:
A Redox potentials E Organization of electro n t ransport chain
S Biological oxidation F Chemiosmotic t heory
C Enzymes and coenzymes 'i ATP synthase
J) High energy compound H Inhibit ors of )~TP synthesis

ehj {) \-(l-.TQQ... )1') Qtt\

(D Stages of Oxidation of Foodstuffs !.heJinal c,.QIT1moo}xi<1fil~th~ y,

®First Stage ----~ 'l'C"'


c acid c c . In this process, NADH and FAD.1:t.z are
generated. This is called

Digestion in the gastrointestinal tract converts the ~ - & it.
molecule~ small units. For example, proteins are
-- -
------
digested to amino acids.This is called primary meta-@rhird Stage (. .. .....
21
bolism (Fig. · 1). These reduced e·quivalents (NAD~ .,~~d FADH
2
) enter
Second Stage ~~into the electr~ 'i>JM' o ~~iH"{tlC), or Respira-
. . b b d b ,~ '(\
d tory chain, where energy is released. This is the
The products of d1gest1on are a so e , cata o 1z.e to .... or cellular respiration
!.. .._•_.t
,>,a.. ; .. •

sm~ components, and


(Fig. 2 1.1). This energy is then used for synthetic
The@ ducing equ1vale~ are mainly generated i
,, purpose in the body (Fig. 21.2).
Primary Secondary Tertiary
Principles of bio-energetics and thermodynamics

. ..Chapter- 1.
metabolism (intermediary) metabolism are descri~
(digestion) metabolism

Intestines TCACycle ETC REDOX(9C TIALS



Carbohydrates Glucose Acetyl-CoA Redox potential ,of a system is the electron transfer
Lipid ---+ Fatty acid-- -, )
CO2
potential EO· Oxidation is defined as the loss of elec-
trons and reducltion as the gain in electrons. When
Proteins --+ Amino acids ---+ a substance exists both in the reduced t te and in
the oxidized.state, the,patns-i::a rS-dox couple.
The ~ ~ of this couple is estimated by
Fig. 21 .1: Oxidation of foodstuffs in three stages measurin~~ctromotjye force (EM[) of a sample
~Af)H = IR:,Jm~
I

Chapter 21: Biological Oxidation and Electron Transport Chain 313

Metabolism of - - - - A-H

X ,: A+CO +H 0 BOX 21 .1: Summary of b1oenerget1cs


1. Free energy is a measure of the energy available to perform

N>
useful work
2. llG can predict the direction of a chemical reaction
3. Chemical reactionscan be coupled, which allowsan energeti-
H,0 0
<ADH•H o, cally unfavorable reaction to conclusion
4. llG measured under physiological conditions may be diffe-
ATP ADP + Pi rent from that at a standard state.
?~__..; r"
~ - b. Pyruvate kinase (see ~ cf\
Energy is used for: Muscle contraction Active
transport Biosynthesis
c. Succinate thiokinase(see Fig. 20.2).
----===----:-..
f"i:C--?
~o,.,._..,-__::!Y.=--
~ - - -- - - - - - - - - - -- - - - - - - - ' TP__ge eratlon is couI)led w ith a more exergonic
Fig. 21.2: ATP generation. Food is catabolized; energy from f~ metabolic reaction. ~n,6-0,'f'O-~
is trapped as ATP; it is then used for anabolic reactio~ ~ f-t-R _ _ ,·b, V\
t;,c cg."·,J...~ IOLQGICAL OXIDATION ~ mo~ t-o
The transfer of electron fr~.JIDhe, ~c~ ~ e~es T
through the respiratory chai o ox n is known as bio- t>.:>--
logical oxidation. Energy released during this process
is trapped as ATP. This coupling of oxidatiol'I with phos-

Negative and Positive


.--a, f' L

When a substance has ........~ - than


hydrogen, it has alllllllllfi e redox potential. If the sub-
stance has a positive redox potential, ~ - a higher
affinity for electrons than hydrogen. Thus~ · a strong
reducing agent, has a n.,e.gati~ edo21-J2otenlial (- 0.JL,V),
whereas a strong oxjdant like oxygernras;::!8 .Q.9~
re_gox--PQ.tential (+0.82 V). Table 21 .1 gives the redox
potentials of some of the important redox couples of the
biological system. A summary is shown in Box 21 .1.

Substrate Level Phosphorylation


Here energy from a high energy compound is directly

-
transferred to nucleoside diphosphate to form a tri-phos-
phate ithout the nelp of electron transport.chain, e.g. oxidation belong to the major class of xido~d~s.
a.
- ---
Bisphosphoglycerate kinase (see Fig. 10.11 ). They can be classified into the following 5 heai:rrngs:
L~,?E-tPr)
314 Section B: General Metabolism

@ oxidases
~ CO-NH 2
These enzym~talyze the removal of hydrogen from
substrates, but Cob~
0 x)(geo cao act as accept~of ijj
hydrogen, so that water is fo~ ed. R
AH + zS 0 RQTI\~ . ,,. ,_A+
2 2 IH
This group includes Cytochrome oxidase (termi- NADH

nal component of ETC), tyrosioase, polyphenol ~xida~. Fig. 21 .3: NAD· accepts H2
catechol ~1 <!§se and monoamine oxidase. f' ,
rn\ (MPtO)- r)~t tui~\e- vi. Pyruvate dehydrogenase P')'rl
~eJB.RLc Dehydrogenases ·~ ~dlj ~ vii. Alpha ketoglutarate dehydrogenase.o( 1';C7
These enzymes ca.tajyze the removal of hydrogen from b. NADP• linked dehydrogenases: NADPH cannot
a substrate, but oxygen can act as the acceptor. These be oxidized with concomitant production of energy.
enz mes are flavoproteins and the product is usually NADPH is used in reductive biosynthetic reac-
~~Alz, lions like f~tty acid syntheaj§ a~ ,,cholesterol syn~
- - - - --A+ H ~ ) l ' P sis, e.g. HMG-CoA ~ :..l ~ot~er example of
These flavoproteins contain eithe .f ~ r [1p
as Clt~ADPH linked dehydrogenase i~ _the glucose-
prosthetic group. Examples are L-a~ ~d d ox~ e 6-phosphate_de_h~ t ogenase (see Fig. 10.38:.
which catalyzes the o~ e deamination ~f L-amirJ.o c. FAD-linked dehydrogenases: When FAD 1s the
acids (see Chapter 17) and Xanthine oxidase (see coenzyme (unlike NAO•), both the hydrogen atoms
Chapter 38).. P,.0-'flVlr-."- ?;iu ">r\.£> , ' ·,j_ are attached t8 tQ~ ~avin~g. Examples:
.Qp.g~- i. Succinatl aehyd~~nase (see st~~~El~-J..9.:2)
@)
~,?GU\'\-\
Anaerobic Dehydrogenases ii. Fatty acyl-CoA ctehya16g~ s ,(see ~ - 8 )
~ mes catalyze the removal of hydrogen from iii. Glycerolphosphate dehydrogenase.CISPl))
a substrate but O!l_gen cannot act as the hydrogen d. Cytochromes: All the cytochromes, except cyto-
acceptor. They therefore require coenzy'ines as accep- chrome oxidase, are anaerobic dehydrogenases.
(Cytochrome oxidase is an oxidase, see above).
tors of the hydrogen atoms. When the substrate is
All cytochromes are h ~ m · s having iron atom.
oxidized, the coenzyme is reduced. - l'f"'I
a. NAO• linked dehydrogenases: NAD• is derived Cyto_c hro~~ c~ochr~m c and cyto romr( c J
. . . ·ct b f th ·t . B are In m1tochondna while cytol rom P-45 ana
from n1cotm1c ac1 , a mem er o e v1 amm com-
\&OJ
in "
plex (see Chapter 33). When the NAD• accepts the cytochrom~ l l Q Pplas ic reJLcu
two hydrogen atoms, one of the hydrogen atom~ HIGH ENERGY COMPOUNDS 0-1:t~e-v
is removed from the substrate as such. The other
4-H~trP These compounds when hydrolyzed will release a large
hydrogen atom is split into one hydrogen ion and
one electron. The electron is also accepted by the quantity of energy. High energy compounds are listed
NAO• so as to neutralize the positive charge on the in Table 21 .2.
/'~lO'(l'}
coenzyme molecule. The remaining hydrogen ion i denosine Triphosphate (ATP) ~c;r 0
release int in medium Fi . 21.3). COf"fi f'fl')l~
i. ATP is the universal currency of energy within the
H - H +H• +e- •
f,of"
~n
\a-"''~ L ~
;H2 + NAO·-+ A+ NADH + H' ..
II.
living cells. Structure of ATP is shown in Fi~~ ~-3.
The hydrolysis of ATP to ADP releases - ,U Tccal/
g' tOl" The NAO• linked dehydrogenases are: mol. The energy in the ATP is used to drive all
~~.,.;~ y eeraldehyde-3-phosphate dehydrogeriase g.pi;r e.Q..derg~ ic {bi~ yntlJ,etic) reactions. The energy
_ . ii. lsocitrate dehydrogenase '1 c: ?"' ( tJ\,e) } Re c1.U. efficiency of the cell is comparable to any machine
'bier-

=
•;
iii . Malate dehydrogenase {l.fl,::>1-) l~':J0-1") ""'faf! so far invented. ATP captures the chemical energy
~ iv. Glutamate dehydrogenase ('""''t) \ 1t~&. '" released by the combustion of nutrients and trans-
ca\,l.O.\o... v. Beta hydroxyacyl-CoA dehydrogenase oo"jJ fers it to synthetic reactions that require energy.
r~ ·fl\--;~
Pf>~ = ftT\>
fl>,~ ·
9kld1<:\ , U.,'6~U'1\\y ~e,
?Et' O.I'\..¢- c.,µ.,"'§b d,.~N'-.( Chapter 21: Biological Oxidation and Electron Transport Chain 315

TABLE 21.2: High energy compounds


Energy-rich compound /lG"' in kcal/ mo/
Phosphate Compounds
1. Nucleotides: (ATP, GTP, UTP, UDP-glucose) Inner
t ATP t o AMP+ PPi - 10.7 kcal membrane
ATPtoADP + Pi - 7.3
2. Creatine phosphate -10.5
3. Arginine p hosphate
4. 1,3-bisphosphoglycerate -10.1
5. Phosphoenolpyruvate -14.8
6. Inorganic pyrophosphate - 7.3 Fig. 21.4: Mitochondria
7. Carbamoyl phosphate -12.3
8. Aminoacyl adenylate (aminoacyl AMP) Yel/apragada Subbarao (1895-1948). His arti-
Sulfur Compounds cle is the 4th most cited paper in the world lite-
9. CoA derivatives: mture. Born in Andhra Pradesh, he studied
medicine in Madras, and conducted research
Acetyl-CoA
at USA. He discovered ATP, assayed phos-
Succinyl-CoA phates and isolated tetracyclins and many
Fatty acyl-CoA other dru_lJ§.
HMG-CoA
R ~ pt ATT"
10. -7.0
~ 1osphate -=J> 6.erf\A e\
11. Creatine phosphate or CP (phosphocreatine) provides a
t]Jgb re.si§:f.Ypir g{ AID to regenerate ATP rapidly
b the Lohmann's reactjop catalwed b}(.c~inase.
ATP + Creatine 1~ P h o s ~ +ADP+ ~G0 '
-10.5 kcal{mol "P~oL
- Energy transfer to the heart's myofibrils is b/grea.=...
~ ~er~ huttle CP is a smaller molecule
than Al P, therefore, GP can rapidly diffuse from the
Karl Fritz Lipmann Alexander myocardium to the myofibrils.
Lohmann NP 1953 Todd fitr,1_J._g~v,.t
1898-1978 1899-1986 NP 1957 © Structure o1f Mitochondrion~
1907-1997 e.lr-,.~~l"'-t.t..
The electcorl:' tr,~port chain is functioning inside the
mitochondria. Tlhe mitochondrion is a subcellular orga-
med. Other energy requiring nelle having the outer and inner membranes enc1o~~~ -
processes are, b~ nthesis of macromolecules,
the matrix (Fig. 21.4). The ~ membrane is yWN~
S£1ective in its permeability, containing Sl)ecific trans- TIC"'~
muscle contractio ~ eUular motion.
ROrt proteins. Certain en~es are specifically 1& ar=-;~ r
iv. ATP is continually being hydrolyzed and regene-
ized i mitochondria. The inrler membrane contains th
rated . An average person at rest consumes and
re<,piratory chain and transldf{ating systems. The k~ b ~i--
regenerates ATP at a rate ot._agf,roximately 3 mole-
i protrusions represent the ATP synthase system
cules per second, i.e. about'i .S1<g/day!
(Hg. 21.4).
Cyrus Fiske and Yellapragada Subbarao discov-
ered ATP in 1929; Karl Lohmann showed its importance ORGANl~~TION OF ELECTRON
in muscle contraction in 1929. In 1941, Fritz Lipmann
(Nobel Prize, 1953) showed that ATP is the universal
TRANSPIDRT CHAIN

--
carrier of chemical energy in the cell and coined tl'le In the,,E!ectron ,tran ort chain1 or esQiratory chain,
m()w..o, "'-lLVJ \,t 0-. uaJ~ ll'I
expression "e.oergy rich phosphate bmlds". Alexander the electrons are transferr ci from ADH to a chain
Todd (Nobel Prize 1957) elucidated its structure. of electron carriers. The electrons flow from the more
316 Section 8 : General Metabolism

I'

NADH Succinate

Complex I = NADH-Co Q reductase (NADH dehydrogenase


complex).
Complex II = Succinate-Q-reductase.
Complex Ill= Cytochrome reductase (Cytochrome b-c1 complex).
Complex IV= Cytochrome ox1dase.

Fig. 21 .6: Summary of electron flow

therefore to be transported from cytoplasm to mitochon-


ig. 21 .5: Mitochondrial transport of NADH by malate-aspartate dria for oxidation. This is achieved by malate-aspa,oote
t shuttle MOH =
malate dehydrogenase; AST aspartate~ = sh~le or ma~e shuttle, which operates mainly in Wer,
transferase; Glu = glutamic acid; AKG = alpha ketoglutaric acid ;
= =
f malate transporter; 2 glutamate aspartate tr'<fnsporter kiaff'ey and h~ rt. The cycle is operated with the help
of en~ )mal'!!_e_2ehydr<?jenas!_jMDH) and ~ ;
t!!_e ammotran~ (Fig. 21.5). From one molecule
---------- ---- ----------- .. -.............. ·---.--.. -........... -........................ -.. ------..... .. i nthe mitochondria,@:ATP molecul~
Summary: Electron flow NADH-+ FMN -+ Fe-S -+ Q -+
of NADH re ..
NADH'!f.FMN yReducedFeS~ Q
:i ~
FMNt½
K c.e,_klized£:eS
generated .

TC Complex I
-,ane.~
_ ""' c.,,. w
NAO+
.,,
'~Hz.;
It is also called N'..;;A=D=H
=-=C3o~ ~ ===:::-
Fig. 21.7: Complex I or NADH-CoQ reductase (NADH
genase complex) (!} drogenase complex. It is

electronegative components to the more electroposi-


tive components.~ Utle--eomponents,~ electron trans-
port chain {ETC) are located in the · r membrane of
mitochondria. There are four distinct multi-protein com-
plexes; these are named as complex-I, II and IV.
These onnected by two mobile carrier coenzyme 0 rail function of this com lex is to collect the pair
~nd
represen e
-__,..~~-- m 1gure 21 .6. The sequence of reaction is
of electrons from NADH and p ss them to CoQ. The
reac_........._..._ shown in Figure 2t.7. he energy released
depicted in Box 21 .2. is kcal/mol _This is utilized to driv protog1 out of

@ NADH Generation
The NADH is generated during intermediary metaboli
2. l
~ the m oc
omplex II
ndna.
,,,-'/
.
A-1,::: ~1,4...C~}o~~ ,

(Fig. 21 .11 ). ~ - , The reaction in Complex II is represented in Figure 21.8.

\l!) Malate As a ate Shuttle-"' ~"""


."'
1"
c}i~· The electrons from FADH 2 enter the ETC at the level
of coenzyme Q. This step does not liberate enough
Mitochonctrial membrane is impermeable to NADH . e.oergv to act as a(prblIDi.p So substrates oxidized
The NADH equivalents generated in glycolysis are by FAD-linked enzymes bypass Complex I.
Chapter 21: Biological Oxiidation and Electron ranspo ain 317

X " '" "


!!
Succinatex FAD FeSX Q

FADH, O,~i,edFeS OH,

Summary: Succinate -+ FAD-+ Fe-S -+ Co Q -+

'l
---------Cc, --------------
Summary:
--------
0-+ Fe-S-+ cyt. b-+ cyt. c1 -+ cyt. c

Fig. 21.9: Complex Ill or cytochrome reductase (cytochrome


b-c1 complex) of mspiratory chain r!:}
accepted from cytochrome c, and passed on to molecu-
lar oxygen. J..tc£:;

Summary: Cyt. c -+ cyt a-a3 -+ 0 2


Fig. 21 .10: Complex IV (cytochrome oxidase) of respiratory inter-membrane space.
chain (0 Cytochromei oxidase contains ~ e groups and
two copper ions. The two heme groups are denoted as
The three major enzyme systems that transfer their
cytochrome-a and cytochrome a-3. The ~ it

l
electrons to Complex II are:
of the enzy~s a single protein, ~ referred to as
Succinate dehydrogenase (see step 6, Fig. 20.2)
cytochrom~~ ,-
Fatty acyl-CoAdehydrogenase (see step 1, Fig. 13.8)
The sequential arrangement of members of electron
Mitochondrial glycerol plibsphate dehydrogenase.
1::-A-P ~n\;u:l ~e.v
>,:J (_r:,s:>~") transport chain iis shown in Box 21 .2 and Figure 21.11 .
Coenz me Q PTP ¼ ~ d - f=TTPf'IS~ \ <3'i\d"
The~ uinone Q is reduced successively t o ~ Current Concept, Energetics of IT'\ f'U0'-\-~
Q! · ) an mally to quinol (QH2 ) . It accepts a ATP Syntheisis
pair of electrons from NADH or'f-ADH2 through Complex I
o~ lex II respectively (Figs. 21 .6 and 21.11 ). The energy of electron transfer is used to drive protons
~ atrix by the ~ ~ s , I, Ill and IV __that
C lex Ill or Cytochrom ctase are]o \~e) The proton gradient thus created is
This contai s e b and c ochro e c1 , both maintained across the inner mitochondrial membrane till
contain heme prosthetic group. The sequ effectrons are..tr.,aruuecced to._px~oJoJ:m-water. The
tion inside the Complex Ill is shown in Fi electrochemical potential of this gradient is used to syn-
free energy change is - 10 kcal/mol; an rotons are
~
p~.gut. JU_ C.
esis of o A.~T n - . ..... 1.
ecuJe_js driven by the
flow o 3 pr on•~ through the TP s_ynthase (see below). ,
Cyto~ When NADH is ,oxidized 10 h dro en i s (proto [ '++'-4- 4111
It contains(one heme rosthetic group. The term cyto- pumped out (Fig . 21.11 ). According to recent findings, +-lj I
chrome is derived from Greek, meaning cellular colors. one NADH may generate only 2.5 ATP; and one FADH2
Axel Theorell (Nobel Prize, 1955) isolated it. Cytochrome may generate only 1.5 ATP. So, one molecule of glucose
c_collects electro1:ns from c _,mplex Il l and delivers them will generate only 32 ATPs. The traditional values and
to~ lex IV. ..... \-tl = @ ~the new values are compared in Table 21 .3.

C~ lex IV or C chrome Ox se Sites of ATP Synthesis


It contains cytochrome a and cytochrome a3. The In olden days the sites of ATP synthesis were marked ,
reaction is depicted in Figure 21 .10. W::eJectrons ar~ as site 1, 2 and 3, as shown in Figure 21 .11 . But now it
I
2 ~ ~\el<!'?-.
2 C,~ \ey !L
318 Section B: General Metabolism

Site 2 Site 3
4 protons pumped out 2 protons pumped out

MRcf\ ~) _1 c_o>,. 1
Complex I Complex Ill J Complex
Co Q --+ FeS -+ Cyt b -+ Cyt c1 Cyt c --+ IV 2 H+
nner
.membrane ------f
FMN--+ FeS
t
Complex II
- - ~1- ~ Cyt0 3, 1' l
Fe-S ti\ ~\l"\ r • f •
r
2. lsocitrate f t 8. Succinate
1/2 0 2 H20

3. Malate
4. Glutamate Fp (FAD)
f
FAo) 9. Ac yl-CoAl fo..tly )
10. Glycerol-3-phosphate
~ -
· -=: haveh
e,otn\\z.,,t.
h>
5. Beta hydroxyacyl-CoA
Lipoate
Co-~ o0 - v;:.t~<JJ\);
t
6. Pyruvate
7. Alpha ketoglutarate

Fig. 21 .11: Components and sequence of reactions of electron transport chain


~rt~'~
,"" ~9'

.. generation. old and new values

/p ~.lk( \ ATP generation by oxldation of Old value Presently accepted

(! :°f \ NADH 3 2.5


F,
i II ii 'I
.
I FADH
Glucose 38
2 1.5
32
: I
Acetyl-CoA 12 10
129 106 >

Proton Pump and ATP Synthesis


The proton pumps (Complexes I, Ill and IV) expe~1jt
from inside to outside of the mitochondrial inner me~~
brane. So, there is high W ~ n,s.e_flt ~on outside the.inner
membrane. Th~.;:.uses H'f-'toenrefihto mitochon ria
Fig. 21 .12: ATP synthase. Protons from outside pass through the
pore of Fo into the matrix, when ATP is synthesized thr~ugh the cna'i1nel~ {fo ; i f)ton influx causes
ATP synthesis by ATP synthase. A summary is shown in
Figure 21.13~ ' mo~

718\e
is known that ATP synthesis actually occurs when the
· 0 G i; • - !IIOMJOblffi
proton gradient is dissipated, and not when the protons
Further, outside is- pos'ltive
are pumped out (Fig. 21 .13).
~ lative to the insiide (+0.14 V) (Fig. 21 .13).

(11 CHEMIOSMOTIC THEORY _ _ _ ATP Synthas;e (C= lex V)


The coupling of oxidation with phosphorylation is ter-
It is a protein ass,embly in the inner mitochondrial mem-
med oxidative phosphorylation. Peter Mitchell in 19 6 1
brane. It is sometimes referred to as the 5th Complex
(Nobel Prize, 1978) proposed this theory to explain the
(Figs. 21 .12 and .21.13). Proton ~ ~ in ATP synthase
.1s a ~ Ill ft..~ l'-'o
• • • • t has two nct1ona . I units,
.
named Sill-? d' t.o\t~c.~ W.et-.t
O~ t\ .....,
F6 unit: The "o" is pro ed as "oh"; and not as
"zero". The "o" stands for gd~ s Fo isj!}!Jibited
b}'. oligomyc,icl. Fo unit SRa r mitochondrial mem-
brane. It serves as a proton channel, through which pro-
- x (Fig. 21.12). tons enter into mitochondria (Fig. 21 .12). _ _\
w~ \( e\ ~w :
,o -tn.e. ~e
\'("\'c. o- me. 'M.01 ruw~ ~'?o..a 6tJ <N"I~
rf\o'~~
,, w'°',c.
,Y\9,.td.e. ~ ' ti)( =>
:-\"
ll\\}w o\s _}1.1
Al P proo-o'.~-
Chapter 21: Biological Oxidation and Electron Transport Chain 319

, Mecco..
/ e,u-qH~
t
•iijY
High pH
LowH Matrix
concentration

AV::: O ,., Inner membrane


; Af'H, ..1.4-

F ig . 21 .13: Summary of ATP synthesis. One mitochondrion is depicted, with inner and outer memberanes. ETC complexes will push
hydrogen ions from matrix into the intermembrane space. So, intermediate space has more H' (highly acidic) than matrix. So, hydrogen
ions tend to leak into matrix through Fo. Then ATPs are synthesized. I, 11, 111, IV = cqmponents of ETC \ · , .~ r..-mAlt•cll'tQ
we.. o..u..nt" \ l"4 TI\()~ v-.y I..)'-'"" i; \v, "'--""""'. J'
i-1\. ~ll' V\.O\d.l~ €;'.~,~ ~~~"'fl\i .
reg11irajfp'ftbe roofor-
F1 Unit: It projects~ m~ t ~ talyzes
synthesis (Fig. 21.12). ATP synthesis requires
TP
15nal
syn~nesis\6f ATP, but enerfi;'. is
changes.

Regul . ATP Syn1,~t!.!!h~:a---.·. ~

----=~--~
c.o,"\~
Th egulates the process. When
synthesis (oxi- . ATP level is low and P level is high, oxidative phos-

i.50.
dative phosphorylation) is through the. intecactjop of f 1

C
Zr 1/2 «,.. c, ~,..
cawes ¥~
phorylation proceeds at a rapid rate. This is called respl-
ratory control or acceptor control. The major source
of NADH and FADH 2 is the citric acid cycle, th, 3!
mding Change Mechanism ~ ~,'(\~ ,af>,N
4' 7 Under resting conditions, ' §ii Sdiiklllffl,
The fwpgjga c;han9e mechanip proposed by ~ I
(Nobel Prize, 1997) explains the synthesis of ATP So, flow of protons back into the mitochondria through
by the proton gradient. The ATP synthase is a •~ u- ATP synthase is minimal. The energy charge, or more
la~a..fbine". CQO'.lpacable to a "water-driven hammer, precisely ADP concentration, normally determines the
minting coins". S IL, · of electron transport.

The F1 has - ~ibitors


of ATP Synthesis
3 conformation states for the alpha-beta functional unit:~ i t e Specific Inhibitors
r:::--P-:~ ~ ~~40."'"" ~
Ostat~Doesootbindsubstrateorproducts (Table 21.4; Fig. 21.14)
L state-Loose binding of substrate and proa1fcts~ -,e.1. )-
T state-Ti ht bindin of substrate and products .;.~ ~ ~nhibitors of Oxidative Phosphorylation
As protons translocate to the matrix, the tree en~ rgy i. Q.tractylosid!Vinhibits th.ajranslocase whereas(i;;:
is released and this is harnessed to interconvert these myc1 inhibits the Fo (Table 21.4 ). ·
~, - -=---
3 states{ [ he bond is s~hesized io the I statyiact::AIP ii. IQnophores are lle!9-soluble com~nds that increase
is released in the O s~ the permeab· · lipi bila ers to certain ions.
For the complete rotation of F1 ~d through the The~ r o · "[eJQo_carri~ @
3 states, 10 protons are tra~ he energy surplus .(~.g~ i mycin) and channel formers (e.g. ami:~
produced by the proton gradient is stored as chemical G.Ld.iol- Valinomycin c!l!ow~ pgJas_sium to permeate
energy in ATP. The energy requiring step is not at the mito-chondria and dissipate the proton gradient.
320 Section B: General Metabolism

Succinate +I
Malonate

Complex II I BAL
AntJmycin

--+-- - - - Carboxin

NADH - - - - Complex I CoQ Complex Ill i--- - + Cyt c - -- + Complex IV 1----,1--+ 0 2


L--- ~--'

t
DNP Amobarb1tal DNP DNP
CCCP Rotenone CCCP CCCP

~ Ol19omycm
ADP+Pi ATP
Fig. 21 .14: Inhibitors of electron transport chain and oxidative phosphorylation. Abbreviations are shown in Table 21.4.

TABLE 21 .4: Compounds which affect electron transport


chain and oxidative phosphorylalion
Uncoupler ow_ oxidation to proceed, butt
gy instead of being trapped by phosphorylation is , - . .
i. Alkylguanides (guanethide), hypotensive drug •
ii. Rotenone, insecticide and fish poison
0
)~ v~ ,lfl . p~@S @§ heat This is achieved by removal of the proton

? ..l~.-.-.-.-....=. ".--.-.--------------- -l
iii. Barbiturates (amobarbital), sedative {f'<r · gradient (Table 21 .4; Fig. 21.14).
iv. Cb!,o rpromazine, tranquilizer ~/
v. Piericidin, antibiotic
n animals and
=
.=
<:=:.
. .
i. Carboxin ____h_u_m_a~ infants, the liberation of heat energy
is required to maintain body temperature. In brown
i. BAL (British anti-lewisite), antidote of war gas adipose tissue, thermogenesis is achieved by this '
ii. Naphthoguinone
iii. AntimycilJ
a protein present in the mitochondria
, , 11 • I , • t
of is also
' ·' i.
ii.
~ onoxid~, inhibits cellular respiration
Cyanide (CNj known to act as
iii. Azide (N3-J
iv. H_ydrogen~sulfide (H2S}
·••·--• How Does Shivering Produce Heat?
i. Carbox(n, inhibits transfer of ions from FADH2 Shivering is due to muscle contraction, it increases
Ii. Malonate, competitive inhibitor of succinate DH ATP hydrolysis. During proton entry for ATP synthesis,
>1) electron transport chain is stimulated.
I. Atractylosid~, inhibits translocase
a : sa a ::c:: &&61t1diil;;elJI nas-
ii. Oligomycin, inhibits flow of protons t hrough Fo
lono hores, e.g. Valinomycin , ~-l5Q•t,.;,cj&;'('\ - --··-··- .. ..------··· __,,.
~,.,e,, er.akf.
t" W:!;> I'(\ ~kt,g,(i
Bro~ Adipose Tissue and
c'!,\~~i~ - - - --
The uncoupling of proton flow releases the en rgy f the
electrochemical proton gradient as heat. Thi

-------
es

-- ..
Thermogenin in brown adipose tissue is a normal physjo!ogjcal function of brown
tissue. Brown adipose tissue gets its color fro
iii. f · di o e lls.
The mitochondria in brown fat contain thermo~enin.
It acts as a channel in the inner mitochondrial membrane
~ hapter 21: Biological Oxidation and Electron Transport Chain ~21

:'f~
.,.. H
•~- free radicals or any other form of stress. As the mem- (:{)u
brane permeability increases, there is opening of a
release of the mitochondrial permeability transition pore (MPTP). It
enerf as . results in dissipation of mitochondrial proton gradient,
,, . ~,. t!!fll-
&
ATP depletion and release of cytochrome c. Thisrc a.Q-
ecific Inhibitors of.ETC chrome C acts_as._a tngger QI apfiptosis by forming an
an drug, but is cardiotoxic. It ; poptosomr c~ pl e~~h other pro-apoptotic factors.
inactivates oclfrorl~ Mase, affects ion pumps and The lm!@tor ~s then activated leading to activa-
inhibits ATP synthase. l(lficreases free radicals leadlng tion of effec_tou~spases, and finall)l--the cell death.
to mitochondrial membrane damage. End effect can be (i!> - ·c .---=== ' ------ - --
co-~ mein hfitochond1rial Dysfunction and
Insulin Resistance D~

- - • .L• ..------. • .l - ~- ;

~v. J.J.
© assav contains cyanogens, and improper ero-
in animal studies. There is also a decrease in insulin-
stimulated glucose uptake by skeletal muscle in elderl
cess1ng of cassava can result in toxicity. HCN is released
from cx.aoogellS..J>_y 11.
glucosida~ present in plants
people. Cumulative damage of mrtocilo dria b OS ?}
may be the reason. Therefore
, Cyanoglycosid~ mygdalin may also be found in
edible plants, such as al~ onds, ap~icots, pefches, plJ'ms,
chdrries, sorghum, soybeans, and bamboo shoots.
• i)! insulin resistance may o
chondrial biogenesis and
Diseases Associated with Mitochondria
Mitochondrial DNA is inherited cytoplasmically and is 0 'Dt ~c..y\. G.~c.W'\,
therefore transmitted maternally. OXPHOS (oxidative ·• Clinical Case Study 21 .1
A 68-year-old female in a hypertensive crisis is being
. treated in the intensive care unit (ICU) with intravenous

r o
blin ness i o males. It is caused by a single base nitroprusside for 48 hours. The patient's blood pressure
io tJA QH Coenzyme Q reductase. (C'omp~'i) was brought baick down to normal levels; however, she
in'tochondrial Permeability was complaining of a burning sensation in her throat
T ·t· p (@JP) and mouth followed by nausea and vomiting, excessive
rans, 10n ore ) sweating. (d.1apt1ores1s
. ) , ag1 -
-1a110n, and dyspnea. An arte-
C ochrome also the ru.e:§3tor of apapto$is (pro- rial blood gas revealed a significant metabolic acidosis. A
gramme cell death}. This can happen in response to serum test sugg1ests a metabolite of nitroprusside, thiocy-
anate, is at toxic levels.
1. What is tho likely cause of her symptoms?
2. What is the biochemical mechanism of this prob-
lem?
3. What is tho treatment for this condition?

0
Axel HT PeterD PaulD John E •• Clinical Case Study 21 .2
Theore/1 Mitchell Boyer Walker
NP 1955 NP 1978 NP 1997 NP 1997 A 55-year-old man was treated in the ICU with intrave-
1903-1982 1920-1992 b. 1918 b. 1941 nous nitropruss;ide for hypertensive crisis for 48 hours.
322 Section B: General Metabolism
BP was restored, but he had a burning sensation in his mining, electroplating, jewelry manufacture and X-ray
throat and mouth, followed by nausea and vomiting, film recovery. It can occur during fumigation of ships,
excessive sweating, agitation and dyspnea. There was warehouses, etc. and are also used commonly as sui-
a sweet almond smell in his breath and arterial blood cidal agents, espeicially by terrorists and healthcare and
gas analysis revealed severe metabolic acidosis. What laboratory workers. Cyanide affects all body tissues •
is the likely condition? How is it treated? What is the and attaches to many metalloenzymes, rendering them
pathogenesis? inactive.
Treatment includes administration of amyl nitrite,
sodium nitrite and sodium thiosulfate, increasing oxygen
8 · Clinical Case Study 21.1 Answer concentration in inspired air and sodium bicarbonate
Diagnosis: Cyanide poisoning from toxic dose of nitro- therapy. Amyl and sodium nitrites induce methemo-
prusside. globin formation , it combines with cyanide and reduces
its toxicity. Sodium thiosulfate converts cyanide to thio-
Biochemical mechanism: Cyanide inhibits mitochon-
cyanate and whicl1 is excreted in urine. Hydroxycobala-
drial cytochrome oxidase, blocking electron transport
min combines with cyanide to form cyanocobalamin
and preventing oxygen utilization. Lactic acidosis results
which is excreted! through urine. Sodium bicarbonate
secondary to anaerobic metabolism.
reduces lactic acidosis.
Treatment: Supportive therapy, oxygen, and antidotal
therapy with sodium nitrite, and sodium thiosulfate. (hEARNINGi POINTS, CHAPTER 21
Clinical correlation: Malignant hypertension is diag-
1. Oxidation of food stuff occurs in 3 stages-primary
nosed when there is elevated blood pressure (systolic
metabolism where macromolecules are converted
levels of 220 mm Hg and/or diastolic blood pressures
to smaller units, secondary metabolism where
exceeding 120 mm Hg). The symptoms may include
reducing equivalents are formed and tertiary meta-
severe headache, neurological deficits, chest pain, or
bolism where energy is released.
heart failure. Hypertensive emergencies require imme-
diate lowering of the blood pressure to lower levels.
2. Oxidation is loss of electrons and reduction is gain "
One hazard of abruptly lowering the blood pressure of electrons. A pair that exists in both oxidized and
is causing hypotension and subsequent ischemia to the reduced state is a redox couple.
brain or heart. Sodium nitroprusside induces a smooth 3. In substrate level phosphorylation, energy from
fall in blood pressure. One side effect of sodium nitro- high-energy compound is directly transferred to
prusside is that its metabolite is thiocyanate, and with NOP to form INTP without the help of electron trans-
prolonged use, cyanide poisoning may result, which port chain.
inhibits the electron transport chain. Thus, in clinical 4. Transfer of electrons from reduced coenzymes
practice, short-term nitroprusside is used. through respiratory chain to 0 2 is known as bio-
logical oxidation.
5. The energy released is trapped as ATP. This cou-
• Clinical Case Study 21.2 Answer pling of oxidation with phosphorylation is called
Patient is most probably suffering from cyanide poison- Oxidative phosphorylation. All enzymes of biologi-
ing. Cyanide inhibits mitochondrial cytochrome oxidase, cal oxidation are oxidoreductases. ..
blocks the electron transport chain and prevents oxy- 6. Electron flow occurs through successive dehydro-
gen utilization. Lactic acidosis is secondary to anaero- genase enzymes (located in the inner mitochondrial
bic metabolism. Cellular oxygen metabolism is impaired membrane), together known as Electron Transport
and can produce death within minutes. Nltroprusside Chain; the el,ectrons are transferred from higher to
therapy, which is the drug of choice for hypertensive lower potenti;al.
emergency, on prolonged usage can produce cyanide 7. NADH is imp,ermeable to mitochondrial membrane.
poisoning. Hence, in clinical practice, nitroprusside is Hence it is transferred via malate-aspartate shuttle
used only for short term. in liver, kidneiy and heart as NADH reducing equi-
Causes for cyanide poisoning include smoke inha- valents and in skeletal muscles as FADH 2 through
lation from residential or industrial fires, metal trades, glycerol 3-phosphate shuttle.
Chapter 21: Biological Oxidation and Electron Transport Chain 3~!3

8. The ETC has 4 distinct multiprotein complexes-- 9. Inhibitors of oxidative phosphorylation include atracty-
viz; Complex I, 11 , Ill and IV connected by two loside and oligomycin. Cyanide inhibits terminal cyto-
mobile carriers to Co Q and cytochrome c. chrome and brings cellular respiration to stand still.

• PART-1: ESSAY AND SHORT NOTE QUESTIONS


21-1 . Write the members of the Electron transport chain, in the order of redox potentials, and show the steps
where ATP is synthesized.
21-2. Define oxidative phosphorylation. Explain the Chemiosmotic theory.

SHORT NOTE QUESTIONS

21-3. Energy rich compounds 21-7. Chemiosmotic theory.


21-4. Inhibitors of ETC. 21-8. Cytochromes .
21-5. Oxidative phosphorylation. 21-9. Cytochrome oxidase.
21-6. Uncouplers of oxidative phosphorylation. 21-10. ATP synthase.

PART-2: MULTIPLE CHOICE QUESTIONS

21-1 . All contain high energy bond, except: 21 -8. Which is the inborn error due to a mutation in
A. ATP 8 . Glucose-6-phospahte NADH-0 reductase?
C. Acetyl-CoA D. Phosphoenolpyruvate A. Amyotrophic lateral sclerosis
21-2. All the following phosphate esters are high energy 8 . Leber's oplil; myopathy
compounds, except: C. Ragged red fiber disease
A. Cretine phosphate D. Duchenne muscular dystrophy
8 . Carbamoyl phosphate 21-9. Valinomycin inhibits oxidative phosphorylation,
C. 2,3-bisphosphoglycerate because:
D. 1,3-bisphosphoglycerate A. It inhibits ATP synthase
21-3. Coenzyme Q catalizes electron transport between 8 . It inhibits cytochrome oxidase
A. FADH and cytochrome b C. It forms a complex with NADH
D. It makes mitochondria permeable to potassium
8. It is the last member in the electron transport chain
21-10. Which is true with regard to oligomycin?
C. NADH and ubiquinone
A. It inhibits oxidative phosphorylation
D. Cytochrome Q and cytochrome C
8 . It is an uncoupler
21-4. Which contains copper?
C. It inhibits translocase
A. Cytochrome oxidase
D. It acts as an ionophore
8 . Cytochrome b5
21-11 . Which is true with cytochrome reductase?
C. Coenzyme Q
A. It catalyzes the electron transport between FADH
D. Cytochrome P-450 and cytochrome b
21-5. Death due to cyanide poisoning is a result of: 8 . It is the last member in the electron transport chain
I
A. Cyanide hemoglobin complex formation C. It catalyzes the electron transport between NADH
8 . Cyanide inhibiting complex I of respiratory chain and ubiquinone
C. Cyanide inhibiting cytochrome oxidase
..
D. It catalyzes the electron transport between Co Q
D. Cyanide blocking oxygen transport in blood and cytochrome C
21-6. All are true with ATP synthase, except: 21-12. Which is an NAO• linked dehydrogenase?
A. It has two subunits, F1 and Fo A. Succinate dehydrogenase
8 . F1 subunit has catalytic activity 8 . Fatty acyl-CoA dehydrogenase
C. F0 subunit serves as a proton channel C. Malate dehydrogenase
D. Proton flow is from F1 to Fo D. Glucose-6-phosphate dehydrogenase
21-7. Oxidative phosphorylation is inhibited by the 21-13. Which of the following compound does not con-
following, except: tain a high energy bond?
A. Oligomycin 8. Carbon monoxide A. Fructose-1 ,6-bisphosphate
C. Hydrogen cyanide D. Pyrophosphate 8 . 1,3-bisphosphoglycerate
324 Section B: General Metabolism
C. Succinyl-CoA C. Peroxiclases
D. Creatine phosphate D. Reducatses
21-14. Allare true with regard to mitochondria, except: 21-18. Which of tlhe following is NOT a high energy com-
A. Glycolytic enzymes are in mitochondrial matrix pound?
B. Inner mitochondrial membrane contains enzymes A. creatinB phosphate
B. 1,3-bisphosphoglycerate a
of electron transport chai n
C. Fluid matrix contains enzymes of TCA cycle C. Phosphoenolpuruvate
D. Acyl-CoA synthetase is localized in the outer D. Glucos,e-6-phosphate
21-19. The high energy phosphate with highest free r
mitochondrial membrane
energy of hydrolysis is:
21-15. ETC is located in the:
A . PEP B. ATP
A. Outer mitochondrial membrane
C. GTP D. Succinyl-CoA
B. Inner mitochondrial membrane
21-20. Wh ich of the electron carriers is soluble and
C. Mitochondrial matrix
mobile?
D. Nucleus
A . CoQ B. Cytochrome c
21-16. When electron transport occurs:
C . Cytochrome a D. Cytochrome b
A. Electrons are transferred from more electro- 21-21. Which of t he following is NOT true regarding ATP
negative to electropositive complexes synthesis'il
B. Electrons get tightly bound to the complexes A . ATP is formed by phosphorylation of ADP
C. Energy is required for the transport of electrons B. Proton gradient is dissipated
D. Electrons flow freely from a high redox potential to C . Oxidati,on is coupled to phosphorylation
low redox potential D. Inner membrane pores are opened to release ATP
21-17. The enzymes of ETC belong to the following 21-22. Which of lthe hormones listed can uncouple oxi-
classes, except: dative phosphorylation in high concentrations?
A. Oxidases A. Insulin B. Cortisol
B. Dehydrogenases C. Thyroxiine D. Glucagon

ANSWERS OF MULTIPLE CHOICE Q ESTIONS .,


21-1 . B 21-2. C 21 -3. A 21-4. A 21-5. C 21 -6. D 21-7. D
21-8. B 21-9. D 21 -1 0. A 21 -11 . D 21-12. C 21 -13. A 21-14. A
21-15. B 21 -16. A 21-17. C 21 -18. D 21-19 . A 21-20. A 21-21 . D
21-22. C

PART-3: VIVA VOCE QUESTIONS AN ANSWERS

21 -1. Give examples of high energy compounds. 21-6. What are FAD linked dehydrogenases?
ATP, GTP, Creatine phosphate, 1,3-bisphosphoglyce- Succinate dehydrogenase; acyl-CoA dehydrogenase.
rate, Phosphoenolpyruvate, Acetyl-CoA, Succinyl-CoA. 21-7. Which cytC)chromes contain copper?
Cytochrome oxidase.
21-2. What are the activities taking place inside mito-
21 -8. Wh ich is complex V of respiratory chain?
chondria?
ATP synthase.
Citric acid cycle; Electron transport chain; Beta oxi- 21-9. What is Valinomycin ?
dation fatty acid. It acts as an ionophore; dissipates the proton gradient:
21 -3. Where are enzymes of citric acid cycle located? and so inhilbits ATP synthesis.
Fluid matrix. 21 -10. What is Atractylocide? •
21-4. Cytochrome oxidase is present in which complex? It inhibits tr:anslocase; and inhibits ATP synthesis.
21-11 . What is Oligomycin?
Complex IV.
It inhibits o:(idative phosphorylation.
21-5. What are NAO• linked dehydrogenases?
21 .12. Name inhilbitors of oxidative phosphorylation.
Glyceraldehyde-3-phosphate dehydrogenase; Pyru- Oligomycin; Carbon monoxide; Cyanide.
vale dehydrogenase; alpha ketoglutarate dehydroge- 21-13. What is the cause for death due to cyanide poi-
nase; lsocitrate dehydrogenase; Malate dehydroge- soning?
nase; Beta hydroxyacyl-CoA dehydrogenase. Cyanide inhibits cytochrome oxidase.
_ _ _ _ _Chapter 22
Heme Synthesis and
I "

Breakdow1n

Chapter at a Glance
The learner w ill be able to answer questions on t he following topics:
St ructure of heme Bilirubin metabolism
Biosynthesis of heme Plasma biliriubin
Porphyrias Jaundice


Red blood cells (RBCs) are biconcave discs, with a
diameter of about 7 microns. RBCs live for about
ISTRUCTURE OF HEME
Heme is usually pronounced as "heem".
120 days in peripheral circulation, during which time
Heme is a derivative of the porphyrin. Por-
they traverse about 160 km! In a 70 Kg person, there will
phyrins are cyclic compounds formed
be about 25 x 1012 RBCs and 750 g of hemoglobin (Hb).
by fusion of 4 pyrrole rings linked by
100 ml blood contains about 14.5 g of Hb. Mature RBC
methenyl (=CHI-) bridges (Fig. 22.1).
is non-nucleated; have no mitochondria and does not Since an atom o,f iron is present, heme is
contain TCA cycle enzymes. However, the glycolytic Hans Fischer
a ferroprotoporphyrin . The pyrrole rings NP 1930
pathway is active which provides energy Erythropoietin are named as I, 11 , 111, IV and the bridges 1881-1945
is the major stimulator of erythropoiesis. It is synthesized as alpha, beta, gamma and delta. The
in kidney. RBC formation in the bone marrow requires possible areas of substitution are denoted as 1 to 8 (Fig.
amino acids, iron, copper, folic acid, vitamin 8 12 , vita- 22.2 and Table 22.1 ). Type Ill is the most predominant
min C, pyridoxal phosphate, and pantothenic acid; they in biological systems. Hans Fischer synthesized heme
are used as hematinics in clinical practice. in laboratory in 1920 (Nobel Prize, 1930). The usual
Heme is present in: substitutions am:
a. Hemoglobin a. Propionyl (- CH2- CH2- COOH) group
b. Myoglobin b. Acetyl (--CH2- COOH) group
c. Cytochromes C. Methyl 1 :-c H3} group
d. Peroxidase d. Vinyl (- C H=CH2 ) group.
e. Catalase Complete structure of heme is shown in Figure 22.3.
f. Tryptophan pyrrolase
g. Nitric oxide synthase. IBIOSYNTHESIS OF HEME
Heme is produced by the combination of iron with Heme can be synthesized by almost all the tissues in the
a porphyrin ring. Chlorophyll, the photosynthetic green body. Heme is synthesized in the normoblasts, but not
pigment in plants is magnesium-porphyrin complex. by the matured erythrocytes.
326 Section B: General Metabolism

0 N
H
2

3
Fig. 22.1 : Pyrrole ring

4
7
Ill
6 5

The pyrrole rings are numbered I to IV; the bridges named as alpha
to delta and the possible sites of substitutions are denoted from
1 to 8. (For brevity, the bridges and double bonds are sometimes
omitted, as shown on the right).

Fig. 22.2: Porphyrin ring


= Methyl

V = Vinyl Succinyl-CoA
= Proplonyl coo- ALA coo- ALA coo-
Fig. 22.3: Structure of heme
I synthase
(+) PLP
I synthase I
CH2 CH2 (+) PLP CH2

TABLE 22.1 : Porphynns of b1olog1cal importance See also


I
CH2
I
\ •
CoA-SH
I
CH2
I \ • CO2
I
CH2
I
Figure 22 3 for the structure of heme C=O C=O C=O
I
Name ofporphyrin Order of substituentsfrom 1st to 8th positions I CH-NH;
I
CH -NH
+
S-CoA
Uroporphyrin I A,P, A,P, A,P, A,P ... I -
2 3

Uroporphyrin Ill A,P, A,P, A,P, P,A


CH2-NH;
coo
Coproporphyrin I M,P, M,P, M,P, M,P I -
coo Alpha-amino Delta-amino
Coproporphyrin Ill M,P, M,P, M,P, P,M
beta-keto levullnic acid
Protoporphyrin Ill M,V, M,V, M,P, P,M Glycine adipic acid (ALA)
(A = acetyl; P = prop ionyl; M = methyl; V = vinyl) Fig. 22.4: Step 1 in heme synthesis

Step 1: ALA Synthesis The uroporphyrinogen I is converted to uroporphyrino-


gen Ill by the enzyme, uroporphyrinogen Ill synthase.
The synthesis starts with the condensation of succinyl-
Only the Ill series are further utilized. The pyrrole rings
CoA and glycine in the presence of pyridoxal phos-
phate to form delta amino levulinic acid (ALA) (Fig. are joined together by methylene bridges (-CH2- ) , which
22.4). Hence anemia may be manifested in pyridoxal are derived from the alpha carbon of glycine (Fig. 22.5,
deficiency. The enzyme ALA synthase is the rate- Step 3). Porphyrinogens are colorless, but are readily
limiting enzyme of the pathway. oxidized to porphyrins, which are colored compounds.

Step 2: Formation of PBG Step 4: Synthesis of CPG


Two molecules of ALA are condensed to form porpho- The UPG-111 is next converted to coproporphyrinogen
bilinogen (PBG). The condensation involves removal (CPG-111 ) by uroporphyrinogen decarboxylase. The
of 2 molecules of water and the enzyme is ALA dehy- acetate groups (CH2-COOH) are decarboxylated to
dratase (Fig. 22.5, Step 2). The enzyme contains zinc methyl (CH3 ) groups (Fig. 22.5, Step 4).
and is inhibited by lead.
Step 5: Synthesis of PPG
Step 3: Formation of UPG
Further CPG is oxidized to protoporphyrinogen (PPG-
Condensation of 4 molecules of the PBG, results in 111) by coproporphyrinogen oxidase. This enzyme spe-
the formation of uroporphyrinogen (UPG). It is a linear cifically acts only on type Ill series, and not on type I
tetrapyrrole. The enzyme catalyzing this reaction is PBG- series. Two propionic acid side chains are oxidatively
deaminase (otherwise called Uroporphyrin I synthase). decarboxylated to vinyl groups (Step 5, Fig . 22.5).
Chapter .22: Heme Synthesis and Breakdown 327

- I
coo_
6
PPG-111 - -.. PP __:_. Heme 1 !
Glycine + Succinyl-CoA

COO CH
I I 2
Mito-
chondria t fi ALA
... CH2 CH2 CPG-111

' D
C HN
NH+
I 2
Cytoplasm~CPG-II I
CO2 ~I N\3
l
ALA

2
3
2 ~ 3
ALA+ALA Porphobilinogen UPG-111 •4-----c==---- PBG
(PBG)
Fig. 22.6: Summary of heme biosynthesis. The numbers denote
4 x Porphobilinogen the enzymes. Pant of synthesis is in mitochondria, and the rest in
c ytoplasm
A p

Step 6: Generation of PP
A J LA
The protoporphyrinogen-III is oxidized by the enzyme
p~p Uroporphyrinogen Ill

PA l
(UPG-111)
Acetyl
protoporphyrino!gen oxidase to protoporphyrin-I11 (PP-Ill).
The methylene bridges (-CH2 ) are oxidized to methenyl

1'--
4 bridges (-CH= ) and colored protoporphyrin-9 is thus
Mp 4CO, Jhyl formed.

,
M
JLM
p~ p
Coproporphyrinogen Ill
(CPG-111)
NADP
Step 7: Generation of Heme
The last step ,s the attachment of ferrous iron to the
NADPH+H+ protoporphyrin to form the heme. The enzyme is heme
PM synthase or fe,rrochelatase.
Propionyl

i
Vinyl
Iron atom is coordinately linked with 5 nitrogen
atoms (4 nitrog,en of pyrrole rings of protoporphyrin and
M V 1st nitrogen atom of a histidine residue of globin). The

MJLM
P ~V
Protoporphyrinogen Ill
(PPG-Ill)
remaining valency of iron atom is satisfied with water or
oxygen atom (Fig. 22.7). A summary of the pathway is
shown in Figure 22.6.
When the f,errous iron (Fe++) in heme gets oxidized
p M 6~ 4H
to ferric (Fe•++) form, hematin is formed , which loses the
Protoporphyrin 111 property of carrying the oxygen. Heme is red in color, but
hematin is dark brown.

Regulation of Heme Synthesis


... 1. ALA synthase is regulated by repression mecha-
2 = ALA dehydratase nism. Heme inhibits the synthesis of ALA synthase
3 = PBG deamlnase and UPG-111 co-synthase
by acting as a co-repressor (see Fig. 42.7).
4 = Uroporphyrinogen decarboxylase
5 = Coproporphyrinogen oxidase 2. ALA synthase is also allosterically inhibited by
6 = Protoporphyrinogen oxidase hematin. When there is excess of free heme,
7 = Heme synthase or Ferrochelatase
the Fe•• is oxidized to Fe-• (ferric), thus forming
= = = =
A acetyl ; P propionyl; M methyl; V vinyl.
hematin.
3. The compartmentalization of the enzymes (Fig.
Fig. 22.5: Steps 2 to 7 of heme synthesis
22.6) of heme synthesis makes the regulation
328 Section B: General Metabolism

easier. The rate-limiting enzyme is in the mitochon- c. Porphyrias with both erythropoietic and hepatic
dria. The steps 1, 5, 6, and 7 are taking place inside abnormalities.
mitochondria, while the steps 2, 3 and 4 are in This classification is based on the major site, where
cytoplasm. the enzyme deficiency is manifested. The clinical
4. Drugs like barbiturates induce heme synthesis. manifestations vary. Porphyrias in general, are not •
Barbiturates require the heme containing cyto- associated with anemia.
chrome P450 for their metabolism. Out of the
total heme synthesized , two thirds are used for
Acute Interm ittent Porphyria (AIP) '
cytochrome P450 production. It is inherited as an autosomal dominant trait. PBG-
5. The steps catalyzed by ferrochelatase and ALA deaminase (uroporphyrinogen-I-synthase) is deficient
dehydratase are inhibited by lead. So lead toxicity (Table 22.2 and Fig. 22.8). This leads to a secondary
causes anemia. increase in activiity of ALA synthase, since the end-
product inhibition is not effective. The levels of ALA
and PBG are elevated in blood and urine. As they are
DISORDERS OF HEME SYNTHESIS
colorless compoU1nds, urine is colorless when voided,
Porphyrias are a group of inborn errors of metabolism but the color is increased on standing due to photo-
associated with the biosynthesis of heme. (Greek oxidation of PBG to porphobilin. Hence, urine samples
'porphyria' means purple). These are characterized by for PBG estimatiion should be freshly collected and
increased production and excretion of porphyrins and or transported in dark bottles. Porphyrins are not excreted
their precursors (ALA + PBG). Most of the porphyrias or elevated in blood; so there is no photosensitivity.
are inherited as autosomal dominant traits (Table 22.2). As the name indicates, the symptoms appear inter-
Porphyrias may be broadly grouped into 3 types: mittently and they are quite vague. Hence, it is at times
a. Hepatic porphyrias called the "little imitator". Most commonly, patients pre-
b. Erythropoietic porphyrias sent with acute abdominal pain . The patients often land
up with the surgeon as a case of acute abdomen and on
several instances exploratory laparotomies are done.
Pyrrole-N N-Pyrrole An attack is precipitated by starvation and symptoms
are alleviated by a high carbohydrate diet. Drugs like
barbiturates, whic:h are known to induce ALA synthase,
can precipitate an1attack.
Pyrrole-N N-Pyrrole Another group may have neurological manifesta-
tions like sensory ;and motor disturbances, confusion and
N-Histidine (proximal)
agitation. Some patients may present with psychiatric
Fig. 22.7: In the he me molecule, iron atom is coordinately linked problems and may be treated accordingly. It is said that
with nitroge n atoms King George Ill ('1760- 1820) was suffering from mania

TABLE 22.2: Features of important types of porphynas


Type Enzyme defect Inheritance Excretion in urine Other salient features
Acut e intermittent PBG-deaminase Autosomal Precursors, ALA and Most common porphyria (1 in 10,000). Hepatic
porphyria (AIP) (UPG-1 synthase) dominant PBG. No color on voiding p orphyria. Abdominal and neurological
(enzyme 3) manifestations. No photosensitivity
Congenital erythro - UPG-cosynthase Autosomal UP and CP; Portwine Marked photosensitivity. Erythrodontia.
poietic porphyria (enzyme 3b) recessive appearance Incidence, rare
Porphyria UPG-decarboxylase Autosomal Uroporphyrins. Second most common; incidence
cut anea tarda (enz4) dominant Urine colored 1 in 25,000. Photosensitivity (Fig. 22.9)
Hereditary CPG-111-oxidase Autosomal UP and CP excreted in urine Symptoms similar to AIP; but milder.
coproporphyria (enzymes) dominant and feces. Colored urine Photosensitivity is also seen
Hereditary Heme synthase Autosomal Neither porphyrins Protoporphyrin increased in plasma,
proto-porphyria or Ferrochelatase dominant nor precursors are HBCs and feces. RBCs show fluorescence
(enzym e 7) excreted in urine

(PBG = Porphobilinogen; CP = Coproporphyrin; ALA = delta amino levulinic acid; UP = uroporph,yrins). (Enzyme numbers are given as shown in
Fig ure 22.8. See also Figure 22.9 for appearance of porphyria cutanea tarda)
Chapter 22: Heme Synthesis and Breakdown 329

l G)
Glycine + Succinyl-CoA
ALA-synthase

6-Amino levulinic acid (ALA)

• l® ALA-dehydratase

Porphobilinogen (PBG)

• PBG-deaminase @ Acute intermittent


porphyria
@ UPG-lll-<:o-synthase --+ Congenital eryt~ropoietic
porphyna

t
Uroporphyrinogen (UPG)

Porphyria
© UPG-decarboxylase - - - -·I~ -cutanea
- --tarda
---

t
Coproporphyrinogen (CPG)

® CPG-oxidase
·I
Hereditary
coproporphyria

Protoporphyrinogen (PPG)

t@
j_
PPG-oxidase
·I Porphyria variegata

t
Protoporphyrin

0 Heme synthase - - - - - - . i..__ _


Hereditary
__ _ _ __,
• Herne
protoporphyria

• Fig. 22.8: Enzyme d eficiencies in porphyrias

due to porphyria. Many of his obstinate decisions, inclu- manifestations relate to the skin due to the photosensi-
ding the ones which led to war of American independence, tization by the presence of porphyrins in the capillaries.
were made when he had acute attacks of intermittent Reactive oxygen species (free radicals) are the cause
porphyria. for cell destruction (see Chapter 30). Repeated attacks
of dermatitis and scarring lead to a typical facial
ALA Synthase (ALAS) Deficiency deformity often r,eferred to as 'monkey face'. Repeated
It is the key enzyme of the pathway. ( See enzyme ulceration and scarring may cause mutilation of nose,
No.1 in Fig. 22.8) There is excessive production of ear and cartilagB. This may mimic leprosy. When UV
heme intermediaries causing neurological porphyrias. light is reflected ,on to teeth a red fluorescence is seen;
Porphyrin precursors and porphyrins in urine and feces this is called erythrodontia.
are normal. Definitive diagnosis is by demonstration of
mutation in erythroid ALA synthase. Porphyrias in General
' Congenital Erythropoietic Porphyria
Accumulation of porphyrin precursors, ALA and por-
phobilinogen leads to neurovisceral manifestations.
It is inherited as an autosomal recessive trait. (Table Accumulated uroporphyrin and coproporphyrin cause
22.2). (See enzyme No. 3b in Fig . 22.8) Normally the delayed bullous lesions. On the other hand, being more
type Ill isomer is produced in larger amounts, but in this lipophilic, protoporphyrin associates with cell membranes
condition, type I isomer is formed considerably. They and causes burning sensation and inflammatory reac-
are converted to porphyrins type I. This would lead to tion in skin exposed to sun. Hence biochemical diagnosis
ineffective feedback inhibition which further increases of porphyria will be considered based on the major
the rate of formation of type I porphyrins. clinical features, namely; a) neurovisceral and b)
Their level in blood increases leading to photo- cutaneous manifestations. The salient features of diffe-
sensitivity. Their excretion in urine makes the urine rent types of porphyrias are given in Table 22.2. See
dark red in color (portwine appearance). The major also Figure 22.9.
330 Section B: General Metabolism

Conditions/agents Enzyme inhibited


Ethanol, lead, PBG synthase
some malignancies (ALA accumulated)
Some malignancies Hydroxymethyl bilane synthase
(PBG and ALA accumulated)
Chronic renal failure, UPG decarboxylase
some malignancies
Diet, liver disease,
(UP accumulated)
CPGoxidase

chronic renal failure, (CP accumulated)
some malignancies,
hexochlorobezene,
lead, mercury, arsenic
Fig. 22.9: Porphyria cutanea tarda
Iron deficiency anemia, Ferrochelatase
lead, aluminium (PP accumulated)
TABLE 22.4 : Urinary excretion of porphyrins (PBG = porphobilinoqen; ALA = amino levulinic acid; PBG = porphobi-
Upper limit ofnormal linogen; UPG = uroporphyrinogen; UP = uroporphyrin; CPG = copro-
excretion value Increased in porphyrinogen; CP = coproporphyrin; PP = protoporphyrin)

ALA
Urine µg/24 hr
4000
Fecesµg/g
Nil AIP BOX 22.1: Bile . .. . .. .. .
Bile pigments arie Bilirubin and Biliverdin. They are the b reak-
PBG 1500 Nil AIP
down products of heme; they are useless excretory products.
CP 200 so CEP, HCP
Bile salts are the sodium salts of bile acids (glycocholate and
UP 25 so Acquired porphyria, CEP
taurocholate). ThEiy are produced from cholesterol; they help in
PP Nil 100 Acquired porphyrias t he absorption of fat.
(AIP = acute interm ittent porphyria; CEP = congenital erythropoietic Both bile pigments and bile salts are p resent in the bile.
porphyria; HCP= hereditary coproporphyria)

ring is broken down in reticuloendothelial (RE) cells of •


Acquired Porphyrias liver, spleen and bone marrow to bile pigments, mainly
Porphyria can result from lead poisoning. Most of the bilirubin (Fig. 22. 10).
paints contain lead more than the permitted levels. Microsomal hEtme oxygenase system : Heme is
Children suck painted toys; and they get the poison. degraded primarily by microsomal heme oxygenase. It
Causes of acquired porphyrias are listed in Table 22.3. requires NADPH and cytochrome c. The enzyme is
A characteristic difference from congenital porphyrias is induced by hemie. The oxygenase enzyme cleaves the
that there is associated anemia in the acquired variety. alpha methenyl bridge, which is linking the pyrrole
rings I and II. The methenyl group liberated as carbon
Diagnosis of Porphyrias monoxide (Fig. 22.11 ). The Fe++ liberated is oxidized to
To demonstrate porphyrins, UV fl uorescence is the best Fe++• and taken up by transferrin.
technique. The presence of porphyrin precursor in urine The linear t13trapyrrole formed is biliverdin which
is detected by Ehrlich's reagent. When urine is observed is green in color. In mammals it is further reduced to
under ultraviolet light; porphyrins if present, will emit bilirubin, a red-yellow pigment, by an NADPH depen-
strong red fluorescence. The diagnostic importance of dent biliverdin reductase (Fig. 22.11 ).
the heme precursors is shown in Table 22.4.

I CATABOLISM OF HEME _ _ __ Transport tc, Liver '


The liver disposes bilirubin (Figs. 22.12 and 22.13).
Generation of Bilirubin The bilirubin formed in the reticuloendothelial cells is
The end-products of heme catabolism are bile pigments insoluble in water. The lipophilic bilirubin is therefore
(Box 22.1). Bilirubin has no function in the body and transported in plasma bound to albumin. Albumin
is excreted through bile. The aged RBCs breakdown binds bilirubin in loose combination. So when present in
liberating the hemoglobin. From hemoglobin, the globin excess, bilirubin can easily dissociate from albumin. The
chains are separated, and broken down. The iron libe- binding sites for bilirubin on albumin can be occupied
rated from heme is re-utilized (Fig. 22.10). The porphyrin by aspirin, penicillin, etc. Such drugs can , therefore,
Chapter 22: Heme Synthesis and Breakdown 3:31

Hemoglobin Heme
(H me oxygenase system:
} - . Globin -+ amino acid pool
Cytochrome C and NADPH)
Heme Ring Carbon monoxide released
5 opens

!. . .__,.
l~:;~,i~tennase (NADPH; Cytochrome c) Iron liberated
Iron -+ Iron re-utilized
M V MP PM M V
• Biliverdin

i
Bilirubin
IV I I

OH
Fig. 22.10: Catabolic pathway of hemoglobin
Biliverdln
NADPH + H+
UDP-glucuronyl-transferase verdin reductase
Bilirubin / -........._ • Bilirubin NADP+
r monoglucuronide
Bilirubin
UDP-glucuronic UDP
M V M p p M M V
acid

Bilirubin UDP-glucuronyl-transferase
mono- • Bilirubin
glucuronide / dlglucuronide
UDP-glucuronic UDP
acid
Fig. 22.12: Production of bilirubin diglucuronide Fig. 22.11: Breakdown of heme

displace bilirubin from albumin. Hence, care should be colorless tetrapyrrole urobilinogen (UBG). It is further
taken while administering such drugs to newborn babies reduced to stercobilinogen (SBG). The SBG is mostly
• to avoid kernicterus. Liver takes up bilirubin by a carrier excreted through feces (250-300 mg/day) (Fig. 22.13).
mediated active process.
Enterohepatic Circulation
Conjugation in Liver
About 20% of the USG is reabsorbed from the intes-
Inside the liver cell, the bilirubin is conjugated with glucu- tine and returned to the liver by portal blood. The UBG
ronic acid, to rnake it water-soluble. (Fig. 22.12). About is again re-excreted (enterohepatic circulation) (Fig.
80% molecules are in the diglucuronide form, while 22.13). Since the USG is passed through blood, a small
20% are monoglucuronides. Drugs like primaquine, fraction is excreted in urine (less than 4 mg/day).
novobiocin , chloramphenicol, androgens and pregnane-
diol may interfere in this conjugation process and may Final Excretion
cause jaundice.
USG and SBG are both colorless compounds but are
Excretion of Bilirubin to Bile oxidized to colored products, urobilin or stercobilin res-

' The water-soluble conjugated bilirubin is excreted into


pectively by atmospheric oxidation. Both urobilin and
stercobilin are present in urine as well as in feces. The
the bile by an active process and this occurs against normal color of feces is due to these compounds. Black
a concentration gradient. This is the rate-limiting step color is seen in constipation. If intestinal flora is
in the catabolism of heme. Excretion of conjugated decreased by prolonged administration of antibiotics,
bilirubin into bile is mediated by multispecific organic bilirubin is not reduced to bilinogens, and in the large
anion transporter (MOAT). gut, it is re-oxidized by 0 2 to form biliverdin. Then green
tinged feces is seen, especially in children.
Fate of Conjugated Bilirubin in Intestine
The conjugated bilirubin reaches the intestine through Plasma Bilirubin
the bile . Intestinal bacteria deconjugate the conjugated Normal plasma bilirubin level ranges from 0.2-1.0 mg/dl.
bilirubin. This free bilirubin is further reduced to a The unconjugated bilirubin is about 0.2- 0.7 mg/dl,
332 Section 8 : General Metabolism

RBC lysis
... rt1es of conjugated and free b1llrub1n
Hemoglobin released Free bilirubin Conjugated bi/irubin
In water Insoluble Soluble
Heme
In alcohol Soluble Soluble

Reticuloendothelial
Normal plasma level
In b ile
0.2-0.7 mg/ dl
Absent
0.1 -0.4 mg/dl
Present

system
In urine
Absorption from GIT
Always absent
Absorbed
Normally absent
Not absorbed

Diffusion into tissue Diffuses Does not diffuse
Bilirubin diglucuro nide van den Berg's test Indirect positive Direct positive

purple in color. Conjugated bilirubin, being water solu-


Liver
ble gives the color immediately; hence called direct
Urobilinogen
in blood
reaction. Free bilirubin is water insoluble. It has to be
extracted first witlh alcohol, when the reaction becomes
Entero- positive; hence called indirect reaction.
Small hepatic
Intestine circulatio
Tests for Bih~ Pigments
! Bilinogens (UBG and SBG) react with Ehrlich's aldehyde

1,1e'::::r:J -+ reagent (para dimethylaminobenzaldehyde) to form


red color. Oxidiziing agents readily oxidize bilirubin to

!
Urobilinogen biliverdin to form a green color. In Fouchet's test, urine is

Stercobilinogen ! heated with barium sulfate which adsorbs bilirubin. Ferric


chloride oxidizes bilirubin to produce a green color. In
!
Urobilin (<4 mg/d) f
Gmelin's test, nitric acid is used as the oxidizing agent.
Stercobilin (300 mg/d)
The bilinogens (UB and SB) form complexes with zinc
Fig. 22.1 3: Production and excretion of bilirubin ions which exhibit brilliant green fluorescence. This is the
basis for Schlesinger's reaction. It is negative in normal
TABLE 22.6 : Tests for bile pigments urine. These findings are summarized in Table 22.6.

I HYPERBIL.IRUBINEMIAS
Fouchet's; Gmelin's; Ehrlich's Schlesinger's
Bile pigments van den Bergh test test
Bilirubin +ve - ve - ve
Depending on the nature of the bilirubin elevated , the
Bilinogens (UBG) - ve +ve -ve
condition may be grouped into conjugated or uncon-
Bilins (UB + SB) - ve - ve +ve
jugated hyperbilirubinemia. Based on the cause it may
also be classified into congenital and acquired.
while conjugated bilirubin is only 0.1-0.4 mg/dl. If the
plasma bilirubin level exceeds 1 mg/dl , the condition Congenital Hyperbilirubinemias
is called hyperbilirubinemia. Levels between 1 and
They result from abnormal uptake, conjugation or exc-
2 mg/dl are indicative of latent jaundice. When the
retion of bilirubin due to inherited defects.
bilirubin level exceeds 2 mg/dl , it diffuses into ti ssues
producing yellowish discoloration of sclera, conjunctiva, Crigler-Najjar Syndrome
skin and mucous membrane resulting in jaundice.
Here the defect is in conjugation. In type 1 (congenital
lcterus is the Greek term for jaundice.
nonhemolytic jauindice), there is severe deficiency of
UDP-glucuronyl transferase. The disease is ofte n fatal
Van den Bergh Test for Bilirubin
and the children dlie before the age of 2. Jaundice usually
Properties of bilirubin are shown in Table 22.5. Bilirubin appears within the first 24 hours of life. Unconjugated
reacts with diazo reagent (diazotized sulfanilic acid) to bilirubin level increases to more than 20 mg/dl , and
produce colored azo pigment. At pH 5, the pigment is hence kernicterus results.
Chapter 22: Heme Synthesis and Breakdown 333

The type 2 disease is a milder form; only the second second pregnancy, the Rh antibodies will pass from
stage of conjugation is deficient. When barbiturates are mother to the fetus. They would start destroying the
given, some response is seen and jaundice improves. fetal red cells even before birth. Sometimes the child is
born with severe hemolytic disease, often referred to as
Gilbert's Disease erythroblastosis fetalis . When blood level is more than
It is inherited as an autosomal dominant trait. The defect 20 mg/dl, the capacity of albumin to bind bilirubin is
(; ,, is in the uptake of bilirubin by the liver. Bilirubin level exceeded. In young children before the age of 1 year, the
is usually around 3 mg/dl, and patient is asymptomatic, blood-brain barrier is not fu lly matured, and therefore free
except for the presence of mild jaundice. bilirubin enters the brain (Kernicterus ). It is deposited
in brain , leading to mental retardation , fits, toxic ence-
Dubin-Johnson Syndrome phalitis and spasticity. If the child develops hemolytic
It is an autosomal recessive trait leading to defective disease, child may be given exchange transfusion along
excretion of conjugated bilirubin; so conjugated bilirubin with phototherapy and barbiturates. Phototherapy with
in blood is increased. There is defect in transport of blue light (440 nm wavelength) isomerizes insoluble bili-
conjugated bilirubin into bile. The bilirubin gets deposited rubin to more soluble isomers. These can be excreted
in the liver and the liver appears black. The condition is through urine without conjugation.
referred to as Black liver jaundice.
Hemolytic Diseases of Adults
Rotor Syndrome This condition is seen in increased rate of hemolysis. It
Bilirubin excretion is defective, but there is no staining of usually occurs in adults. The characteristic features are
the liver. It is an autosomal recessive condition. increase in unconj ugated bilirubin in blood, absence
of bilirubinuria and excessive excretion of UBG in urine
Acquired Hyperbilirubinemias and SBG in feces (Table 22.6). Common causes are:
i. Congenital spherocytosis
Physiological Jaundice ii. GPO deficiency
It is also called as neonatal hyperbilirubinemia . In all iii. Autoimmune hemolytic anemias
new born infants after the 2nd day of life, mild jaundice iv. Toxins like carbon tetrachloride.
appears. This transient hyperbilirubinemia is due to
an accelerated rate of destruction of RBCs and also Hepatocellular Jaundice
because of the immature hepatic system of conjugation The most common cause is viral hepatitis, caused by
of bilirubin. In such cases, bilirubin does not increase Hepatitis Viruses A, B, C, D or E. In pure hepatocellular
above 5 mg/dl. It disappears by the second week of life. disease, conjugation in liver is decreased and hence
free bilirubin is increased in circulation. However,
Breast Milk Jaundice inflammatory edema of cell often compresses intra-
cellular canaliculi at the site of bile formation and this
In some breast-fed infants, prolongation of the jaundice
produces an element of obstruction. When conjugated
has been attributed to high level of an estrogen derivative
bilirubin level also increases, mixed type of jaundice
in maternal blood, which is excreted through the milk. results. Bilirubinuria also occurs. The UBG level in urine
This would inhibit the glucuronyl transferase system. may be normal or decreased in hepatocellular jaundice
Sulfa and such other drugs may release bilirubin from (Table 22.7).
albumin, and may cause jaundice in newborn.
Obstructive Jaundice
Hemolytic Jaundice Conjugated bilirubin is increased in blood, and it is
Hemolytic Disease of the Newborn excreted in urine. If there is complete obstruction, UBG
will be decreased in urine or even absent (see Fig. 22.13
This condition results from incompatibility between and Tables 22.7). In total obstruction of biliary tree, the
maternal and fetal blood groups. Rh positive fetus may bile does not enter the intestine. Since no pigments are
produce antibodies in Rh negative mother. In Rh incom- entering into the gut, the feces become clay colored.
patibility, the first child often escapes. But in the The common causes of obstructive jaundice are:
334 Section B: General Metabolism

What is the lilkely diagnosis?


What is the underlying biochemical problem?
jaundice jaundice e
Blood, free bilirubin Increased Increased Normal 8 Clinical Case Study 22.2
Blood, conj. bilirubin Normal Increased Increased
The following am the biochemical values in a patient.
Blood, ALP Normal Increased Very high
What is your probable diagnosis?
Urine, bile salts Nil Nil Present
Serum bilirubin-13.0 mg%
Urine, conj. bilirubin Nil Present Present
Conjugated bilirubin-6.0 mg%
Urine, bilinogens Increased Present Nil Unconjugatecj bilirubin-7.0 mg%
Fecal urobilinogen Increased Decreased Absent
Serum alkaline phosphatase-280 IU/L
Normal values: unconjugated (free) (indirect) bilirubin 0.2-0.7 mg/ dl
AST-250 U/L
and conjugated (direct) bilirubin 0.1-0.4 mg/ dl. A rise in serum
bilirubin above l mg/dl is abnormal (latent jaundice); but jaundice ALT-370 U/L
appears only if the level goes above 2 mg/dl. Urine bile salts- Negative, Bile pigments-Positive
( +++ ), Urobilinogen- Positive
a. lntrahepatic cholestasis. This may be due to: Feces stercobilinogen-Positive
i. Chronic active hepatitis
ii. Obstructive stage of viral hepatitis.
e
b. Extrahepatic obstruction. This may be due to:
• · Clinical Case Study 22.3
i. Stones in the gallbladder or biliary tract Comment on the 'following laboratory results obtained in
ii. Carcinoma of head of pancreas a patient and give your probable diagnosis.
iii. Enlarged lymph glands in the porta hepatis Serum bilirubin- 9.0 mg%
More details on different types of jaundice are given Conjugated bilirubin-7.5 mg%
in Chapter 24. Unconjugated bilirubin-1.5 mg%

Bilirubin Toxicity AST-80 U/L


ALT-90 U/L
Bilirubin will uncouple oxidative phosphorylation, and
ALP-440 IU/L
inhibit ATPase activity in brain mitochondria. All of
Urine bile salts-Positive (++), Bile pigments-
these toxic effects of bilirubin are reversed by binding
Positive (++), Uroibilinogen-Negative
to albumin. In fact, albumin transports bilirubin from its
Feces stercobilinogen-Negative
sites of production (bone marrow and spleen) to its site
of excretion which is the liver. e
e 8 · Clinical Case Study 22.4
•• Clinical Case Study 22.1 Comment on the following laboratory results and give
your provisional dliagnosis.
A 21-year-old healthy male college student went to
celebrate his birthday with some friends at a bar. His Serum bilirubin- 9.0 mg%
Conjugated bilirubin- 0.5 mg%
friends convinced him to have his first beer since he
just turned 21 . After consuming the beer, he began to Unconjugated bilirubin-8.5 mg%
experience intense, worsening abdominal pain that AST- 26 U/L
was nonspecific in location and described as cramping. ALT-30 U/L
Nausea and vomiting then ensued and he was taken to ALP-56 IU/L
the ER. Urine bile salts- Negative, Bile pigments-Negative,
Upon arrival to the ER, he was found to be very Urobilinogen- Po1sitive (+++)
anxious with hallucinations. He was noted to be hyper- Feces stercobilinogen- Positive (+++)
tensive, tachycardic, and diaphoretic. Peripheral neuro- e
pathy wa s also noticed on examination. Initial laboratory •ii• Clinical Case Study 22.5
test revealed a normal CBC, drug screen, and EtOH A 42-year-old obese lady presented with intolerance to
level. Serum and urine aminolevulinic acid (ALA) and fatty food and pain in the right abdominal region. On
porphogilinogen (PBG) were both found to be elevated. examination, her eyes were yellowish and stools had
Chapter 22: Heme Synthesis and Breakdown 3~15

clay colored appearance. The doctor ordered some ,.(1


laboratory tests. • • Clinical Case Study 22.4 Answer
The results obtained were as follows:
Hemolytic jaundice. Causes are discussed in the text.
Serum bilirubin-25.0 mg%
AST- 35 U/L 0
ALT-40 U/L • • Clinical Case Study 22.5 Answer
ALP-400 IU/L Obstructive jaundice due to gallstones.
What is the probable diagnosis?
0 0
;fi. Clinical Case Study 22.6 ;fi. Clinical Case Study 22.6 Answer
A newborn baby presented with yellowish discoloration Hemolytic disease of the newborn. See text for descrip-
of skin and conjunctiva after 3 days of birth. The tion on the congenital types of jaundice.
neonatologist advised phototherapy. The child became
normal. What is the type of jaundice? What are the tests LEARNING POINTS, CHAPTER 22
to be done in this child? Why did phototherapy benefit 1. Hemoglobin- the oxygen transporter is a 67 Kd
the child? What are the other types of jaundice that may tetrameric conjugated protein with heme as pros-
be seen in a newborn baby? thetic group and globin polypeptide (2a and 213
0 chains).
•ii'• Clinical Case Study 22.1 Answer 2. Heme is a derivative of porphyrin, which is a cyclic
Diagnosis: Porphyria (likely acute intermittent porphyria). compound formed by the fusion of 4 pyrrole rings,
linked by methenyl bridges and has an iron atom at
Biochemical problem: Enzymatic deficiency in heme
its center.
biosynthetic pathway.
3. Regulation of heme synthesis is by repression of ALA
Clinical correlation: Patients often are asymptomatic synthase by heme. Glucose prevents ALA synthase
unless exposed to factors that increase production induction. Barbiturates induce the enzyme. ALA syn-
of porphyrias (drugs, alcohol, sunlight). Erythropoetic thase is also allosterically inhibited by hematin.
varieties primarily present with photosensitivity. 4. Porphyrias are the class of metabolic disorders
Hepatic porphyrias present with primarily neurovisce- associated with heme synthesis. AIP occurs due to
ral symptoms such as: abdominal pain, nausea and vomit- deficiency of PBG deaminase. Congenital erythro-
ing, tachycardia and hypertension, peripheral neuropathy, poietic porphyria is due to imbalance in activities
and mental symptoms (hallucinations, anxiety, seizures). of uroporphyrinogen I synthase and co-synthase.
Diagnosis is confirmed with elevated levels of ALA Acquired porphyrias result from lead poisoning.
and PBG in the urine and serum. Treatment is supportive 5. Normal plasma bilirubin levels range from 0.2- 1.0
with avoidance of triggers in the future. mg/dl and can be detected by van den Bergh test.
6. Congenital hyperbilirubinemias include, Gilbert's
(?
•ii'• Clinical Case Study 22.2 Answer disease where bilirubin uptake is defective, Criggler-
Najjar syndrome where conjugation is defective
Hepatocellular jaundice. Dubin-Johnson syndrome where defect is in excre-
tion of conjugated bilirubin.
0
•ii'• Clinical Case Study 22.3 Answer 7. Acquired hyperbilirubinemias or Jaundice may be
physiological jaundice, hemolytic jaundice, hepato-
Obstructive jaundice. Causes are discussed in the text. cellular jaundice and obstructive jaundice.

PART-1 : ESSAY AND SHORT NOTE QUESTIONS

22-1 . Give an account of heme synthesis.


22-2. What is porphyria? Classify different types of porphyrias. Give an account of acute intermittent porphyria.
22-3. Describe the catabolism of heme in the body.
22-4. How bile pigments are formed? Give the significance of their presence in blood and urine. How are they detected
in blood and urine?
336 Section B: General Metabolism

22-5. Classify jaundice. How do you investigate a case of jaundice?


22-6. Discuss the biochemical alterations seen in blood and urine in different types of jaundice.
22-7. How is bilirubin formed in the body? Describe how it is excreted. Describe the biochemical changes in
hepatocellular jaundice and obstructive jaundice.

SHORT NOTE QUESTION

22-8. ALA synthase. 22-14. Crigler-Nctjjar syndrome.


22-9. Acute intermittent porphyria. 22-15. van den B:ergh reaction.
22-10. Rate limiting step of hemesynthesis. 22-16. Hemolytic: jaundice.
22-11 . Regulation of heme synthesis. 22-17. Hepatocelllular jaundice.
22-12. Formation and fate of bilirubin. 22-18. Obstructi1ire jaundice.
22-13. Catabolism of heme 22-19. Urobilino!~en.

PART-2: MULTIPLE CHOICE QU STIONS

22-1 . All the following oxygen handling proteins contain 22-8. Urine of a patient with acute intermittent porphyria
heme as a prosthetic group, except: is likely tc, contain:
A. Cytochromes B. Peroxidase A. Porphobilinogen
C. Superoxide dismutase B. Uroporphyrinogen
D. Tryptophan pyrrolase C. Protop10rphyrinogen
2.2-2. The methenyl bridge of protoporphyrin is derived D. Bilirubin
from : 22-9. Degradati,on of heme to bilirubin releases one mole-
A. Alpha carbon atom of glycine cule of:
B. Alpha carbon atom of succinyl-CoA A. Carbon dioxide B. Water
C. Carboxyl group of succinyl-CoA C. Ammonia D. Carbon monoxide
D. Carboxyl group of glycine 22-10. Heme Is converted to bilirubin in:
22-3. Rate limiting enzyme in heme synthesis is: A. Mitoch1ondria B. Microsomes
A. Heme synthase C. Golgi bodies D. Plasma membrane
B. ALA dehydrase 22-11 . Bilirubin in serum can be measured by:
C. Uroporphyrinogen synthase A. Van den Bergh reaction
D. ALA synthase B. Ehrlich's reaction
22-4. All the following are precursors of heme, except C. Schlesinger's reaction
A. Porphobilinogen B. Urobilinogen D. Fouchet's reaction
C. Uroporphyrinogen D. Protoporphyrinogen 22-12. Which disease is associated with conjugated
22-5. Which enzyme of heme blosynthesis is inhibited hyperbilirubinemia?
by lead? A. Gilbert's disease
A. ALA dehydratase B. Crigler-Najjar syndrome
B. UBG I synthase C. Dubin--Johnson's syndrome
C. UBG decarboxylase D. Conge,nital spherocytosis
D. PPG oxidase 22-13. All the following have a regulatory effect on heme
22-6. Acute intermittent porphyria (AIP) is: synthesis, except:
A. Characterized by photodermatitis A. Cellular heme pool
B. An autosomal dominant trait B. Low calorie diet
C. More common in men than women C. Compartmentalization of enzymes
D. Hemolytic anemia is associated D. Hemat:ocrit value
22-7. Congenital erythropoietic porphyria Is caused by 22-14. Which int,e rmediate of heme synthetic pathway is
defi ciency of: not a tetrc1Ipyrrole?
A. ALA synthase A. Uroporphyrinogen
B. Uroporphyrinogen Ill cosynthase B. Coproporphyrinogen
C. Coproporphyrinogen oxidase C. Porphobilinogen
D. Uroporphyrinogen decarboxylase D. Protoporphyrinogen
Chapter 22: Heme Synthesis and Breakdown 337

22-15. Which of the following statements is NOT true? 22•27. Bilirubin is formed in all the tissues, except
A. ALA synthase requires energy for its activity A. Liver B. Spleen
B. ALA dehydratase is inhibited by lead C. Kidney D. Bone marrow
C. ALA synthase requires PLP for its activity 22-28. Which is inot involved in catabolism of heme?
D. Two molecules of ALA condense to form PBG A. Cytochrome a B. NADPH

-
22-16. Porphyrins are:
A. Monopyrroles
B. Colorless components
C. Showing fluorescence under UV light
C. Molecular oxygen D. Cytochrome c
22-29. Which enIzyme is not involved in catabolism of
heme?
A. Hemeoxygenase
D. Destroyed by sunlight B. UDPiilucuronyltransferase
22-17. In obstructive jaundice, prothrombin time: C. Biliverdin reductase
A. Remains normal B. Decreases D. Ferroc:helatase
C. Becomes normal when vitamin K is administered 22-30. All the following are produced from heme during
D. Increases when vitamin K is administered catabolisirn, except.
22-18. Which does not contain heme? A. Urobilnnogen B. Porphobilinogen
A. Myoglobin B. Cytochrome C. Bilirubin D. Biliverdin
C. Ferroxidase D. Peroxidase 22-31 . Normal color of feces is due to:
22-19. All are correct with regard to heme, except A. Bilirubin B. Stercobilin
A. Ferro protoporphyrin C. Porphobilinogen D. Biliverdin
B. Prosthetic group of cytochromes 22-32. All the following compounds undergo entero•
C. Synthesized by using glycine hepatic ciirculation, except:
D. Linear tetrapyrrole A. Urobil1inogen B. Iron
22-20. Which is required for synthesis of porphyrins? C. Cholic: acid D. Cholesterol
A. Ammonia B. Methionine 22-33. Increased level of urobilinogen In urine is observed

(
C. Glycine D. Carbon dioxide in all the !following conditions, except
22-21 . All are substituent groups present in heme (proto• A. Congemital spherocytosis
porphyrin), except: B. Mismatched transfusion
A. Methyl B. Vinyl C. Glucose-6-phosphate dehydrogenase deficiency
C. Propionyl D. Ethyl D. Lead poisoning
22-22. All are correct regarding heme synthesis, except 22-34. Congenital hyperbilirubinemia with a bilirubin level
A. Compartmentalized of 2.5 mg,/dl is suggestive of:
B. Stimulated by lead A. Criggler-Najjar syndrome
C. Regulated by end product inhibition B. Gilbert's syndrome
D. Induced by barbiturates C. Dubin-Jhonson's syndrome
22-23. All affect heme synthesis, except D. Rotor syndrome
A. Barbiturates B. Estrogens 22-35. Heme contains:
C. Glucose D. Oncogenes A. Magn,::!sium B. Calcium
22-24. A patient with acute intermittent porphyria should C. Iron D. Sodium
be given: 22-36. Heme is not found in:
A. High carbohydrate diet A. Cytochrome c
B. Barbiturates B. Cytochrome a
C. Aspirin C. Iron s1Ulphur proteins
D. Saline infusion D. Nitric oxide synthase
22-25. Congenital erythropoietic porphyria has all the 22·37. Bilirubin is:
features, except: A. Conjugated with glucuronic acid
A. Sensitivity to sunlight B. Transported free in plasma
B. Reddish discoloration of urine C. Normally excreted through urine
C. Erythrocyte porphyrin level elevated D. Bound to haptoglobin
D. Red fluorescence of teeth 22-38. UPG ;s cc>nverted to CPG by:
22-26. Presence of porphyrins in urine can be detected by: A. Cond13nsation B. Decarboxylation
A. UV fluorescence C. Dehydrogenation D. Reduction
B. Absorption band at 565 nm 22-39. Which of the following is colored?
C. Fouchet's test A. PBG B. UPG
D. Schlesinger's test C. CPG D. pp
338 Section B: General Metabolism

22-40. Heme synthesis requires all the following except:


B. Porphyria cutanea tarda
A. Glycine B. Succinyl-CoA
C. Variegate porphyria
C. PLP D. Cytochrome P450
22-41 . All are features of acute intermittent porphyria D. Congenital erythropoetic porphyria.
except 22-43. Which of the following is NOT true regarding biliru-
A. Abdominal colic B. Hypertension bin?
C. Anemia D. Psychotic behavior A. It is a lnnear tetra pyrole
22-42. Which porphyria is inherited as an autosomal B. Contaiins ferric iron
recessive trait? C. End pmduct of heme degradation
A. Hereditary coproporphyria D. Essential for fat digestion and absorption

ANSWERS OF MULTIPLE CHOICE Q UESTIONS


22-1. C 22-2. A 22-3. D 22-4. B 22-5,. A 22-6. B 22-7. B
22-8. A 22-9. D 22-10. B 22-11. A 22-121. C 22-13. D 22-14. C
22-15. A 22-16. C 22-17. C 22-18. C 22-191. D 22-20. C 22-21 . D
22-22. B 22-23. D 22-24. A 22-25. C 22-26. A 22-27. C 22-28. A
22-29. D 22-30. B 22-31 . B 22-32. B 22-33,. D 22-34. B 22-35. C
22-36. C 22-37. A 22-38. B 22-39. D 22-401. D 22-41 . C 22-42. D
22-43. B

PART-3: VIVA VOCE QUESTIONS AN ANSWERS


22-1 . What is the rate limiting enzyme of heme synthesis? 22-16. What is jaundice?
ALA synthase. When plasma bilirubin is more than 2 mg/di, it diffuses
22-2. Metheny! bridge of protoporphyrin is derived from into tissues, producing yellowish discoloration of tis-
what? sues.
Alpha carbon of glycine. 22-17. Enterohepatic circulation is seen in which sub-
22-3. How is heme synthesis regulated?
stances?
ALA synthase is repressed by heme.
Urobilinoge-n and bile salts.
22-4. What is the action of barbiturates on heme syn-
22-18. What is th•~ defect in Criggler-Najjar syndrome?
thesis?
Defect in oonjugation of bilirubin due to deficiency of
Barbiturates will induce heme synthesis.
22-5. What is the defect in acute intermittent porphyria? UDP-glucuronyltransferase.
PBG-deaminase. 22-19. Bilirubin in serum is estimated by what test?
22-6. How porphyrins are detected? Van den Be!rgh reaction .
They show fluorescence under ultra violet light. 22-20. What is Diirect van den Bergh reaction?
22-7. In acute intermittent porphyria, urine contains? The color is developed immediately when blood is
ALA and PBG. added.
22-8. Degradation of heme needs which enzyme? 22-21. What is th19 inference, when direct van den Bergh
Hemeoxygenase system, with NADPH test is posijtive?
22-9. Degradation of heme releases what gas?
Blood contains conjugated bilirubin; it is water soluble.
Carbon monoxide. 22-22. What is indirect van den Bergh test?
22-10. Heme is converted to bilirubin in which site?
Microsomes of reticuloendothelial cells.
22-11. Bilirubin in blood is carried by what?
Albumin.
When blood is added to the solution. there is no color:
but when alcohol is added, color is developed.
22-23. What is kemicterus?
-~
22-12. Where is the conjugation taking place? In young chiildren, when plasma bilirubin is more than
In liver. 20 mg/di, it diffuses into brain, causing permanent I
22-13. What is the enzyme needed for conjugation? damage to brain cells.
UDP-glucuronyltransferase 22-24. In obstructive jaundice, blood contains what?
22-14. What is the normal level of plasma bilirubin? Conjugated bilirubin in excess quantity.
0.2- 1 mg/dl. 22-25. Bile salts .ind bile pigments are excreted in urine
22-15. What is latent jaundice? in which ccmdition?
When plasma bilirubin is between 1 to 2 mg/di. Obstructive jaundice.
......,___ _ __ Chapter 23
Hemoglobin
{Struct ure, Oxygen and Carbon Dioxide
Transport, Abnormal Hennoglobins)

Chapter at a Glance
The learner wi ll be able to answer questions on the following topics:
Structure of hemoglobin O Hemoglobino pathies
0 Oxygen dissociation curve O Sickle cell anemia
Transport of carbon dioxide Thalassemia s
0 Fetal hemoglobin O Myoglo bin
Carboxy hemoglobin Anemias
M et-hemoglobin

Historical Perspectives
Marcello Malpighi described the RBCs in 1665. Felix Hoppe-Seyler in 1862 isolated pure hemoglobin. Christian Bohr in 1904
discovered that hemoglobin is the transporter of oxygen. In 1912, Kuster established the structure of hemoglobin. Hans Fischer
synthesized heme in laboratory in 1920 (Nobel Prize, 1930). Perutz (Nobel Prize, 1962) studied the three-dimensional structure
of hemoglobin.

ISTRUCTURE OF HEMOGLOBIN Attachment of Heme with Globin Chain


Normal level of Hemoglobin (Hb) in blood in males is There are 4 herne residues per Hb molecule, one for
and in females, 13-15 g/dl. Hb is globular each subunit in Hb. The heme is located in a hydrophobic
in shape. The adult Hb (HbA) has 2 alpha chains and
cleft of globin chain (Fig. 23.1). The iron atom of heme
2 beta chains. Molecular weight of HbA is
occupies the central position of the porphyrin ring. The
Daltons (66,684 to be exact). Hb F (fetal Hb) is made up
reduced state is called ferrous (Fe... ) and the oxidized
of 2 alpha and 2 gamma chains. Hb,¾ has 2 alpha and
2 delta chains. Normal adult blood contains 97% HbA, state is ferric (Fei....). The ferrous iron has
I • about 2% Hb,¾ and about 1% HbF. and ferric has 5 valencies. In hemoglobin,
Each alpha chain has 141 amino acids. The beta,
gamma and delta chains have 146 amino acids. The
N-terminal and C-terminal end amino acids are shown
in Table 23.1. The 58th residue in alpha chain is called
, because it is far away from the iron
atom. The 87th residue in alpha chain is called
, because it lies nearer to the iron atom (Fig.
23.1 ). The alpha and beta subunits are connected by
Marcello Felix Max Perutz Donald
relatively weak noncovalent bonds like van der Waals Ma/pighi Hotppe-Seyler NP 1962 van Slyke
forces, hydrogen bonds and electrostatic forces. 1628-1694 1825-1895 1914--2002 1883-1971
340 Section B: General Metabolism

TABLE 23.1: Amino acid sequence of globin


Hbchain Amino acid sequence number
1 58 63 87 92 141 145 146
Alpha Val Distal His - Proximal His - Last Arg Nil Nil
Beta Val - Distal His - Proximal His - Tyr Last His
Gamma Gly - Distal His Proximal His - Tyr Last His
Delta Val - Distal His - Proximal His - Tyr Last His

BOX 23.1: Oxygenation and ox1dat1on


I
N-Histidine (distal) When hemoglobin carries oxygen, the Hb is oxygenated. The
I iron atom In Hb is still in the ferrous state. Oxidized hemoglobin

"t /
is called Met-Hb; then iron is in ferric state and the oxygen
0 ca rrying capacity is lost.
Pyrrole-N N-Pyrrole

l"'
Fe by the oxygen dissociation curve (ODC) (Fig. 23.3).
Hemoglobin is oxygenated and not oxidized (Box
Pynole-N / N-P- - - -Arginine - 23.1). At the oxygen tension in the pulmonary alveoli,
the Hb is 97% saturated with oxygen. Normal blood
1 N-Histidine (proximal) with 15 g/dl of Hb can carry 1.34 x 15 = 20 ml of O/ dL
Arginine of blood. In the tissue capillaries, where the pO2 is
I only 40 mm Hg, theoretically, Hb saturation is 75%.
Thus under NTP conditions, blood can release only
Fig. 23.1: Linkage of heme with globin. Pink circle represents 22% (Fig. 23.3A). But actually in tissue capillaries,
the g lobin chain. Blue rectangle represents the protoporphyrin ring where pO2 is 40 mm Hg, the Hb is about

(Box 23.1 .- "


The iron is linked to the pyrrole nitrogen by 4 coordinate
. £ [l:,o physiologically, 40% of oxygen is released
(Fig. 23.3C).
The pO2 in inspired air is 158 mm Hg; in alveolar
valency bonds and a fifth one to the imidazole nitrogen air 100 mm Hg; in the blood in lungs, pO2 is 90 mm Hg;
of the (Fig. 23. 1). In oxy-Hb, the 6th and in capillary bed, it is 40 mm Hg. In tissues, oxygen
valency of iron binds the 0 2. In deoxy-Hb, a water mole- is liberated from hemoglobin. In lung capillaries, oxygen
cule is present between the iron and distal histidine is taken up by the hemoglobin. Oxygen carriage of
(Fig. 23.1). Figure 23.2 shows a diagramatic representa- hemoglobin is schematically depicted in Figure 23.4.
tion of the subunit interaction in hemoglobin. The following factors will affect the oxygen dissociation
curve.

I TRANSPORT OF OXYGEN BY
HEMOGLOBIN
Hemoglobin has all the requirements of an ideal respira-
Heme-Heme Interaction and Cooperativity
The sigmoid shape of the oxygen dissociation curve
(ODC) is due to the A teric effect, or cooperativity.
tory pigment (Barcroft): The equilibr~ m of Hb with oxygen is expressed by the
a. It can transport large quantities of oxygen Hill equation7 AV Hill, Nobel Prize, 1922). The binding
b. It has great solubility of oxygen to one heme residue increases the affinity
c. It can take up and release oxygen at appropriate of remaining heme residues for oxygen
partial pressures interaction) (Fig. 23.38 ). This is calle<11•••
coopera-
d. It is a powerful buffer. tivity.
Affinity Affinity Affinity Affinity
Oxygen Dissociation Curve (ODC) 1 time 2 times 4 times 8 times
The ability of hemoglobin to load and unload oxygen at Hb - HbO2 - HbO4 HbO6 HbO8
physiological pO2 (partial pressure of oxygen) is shown (+)0 2 (+)02 (+)02 (+)02

\
Chapter 23: Hemoglobin 341

(+)02 (+)02:

Deocy-Hb; T conformatioin
B IIHb-02
2

Hb-0 4
2

Fig. 23.2: Diagramatic representation of the subunit interaction in hemoglobin. Pinik rec tangles represent Hb monomers. Black
Hb-0 8; R form

connection lines represent salt bridges. As oxygen is added, salt bridges are successively broken and finally 2,3-BPG is expelled.
Simultaneously the T (taught) confonnation of deoxy-Hb is changed into R(relaxed) c:onformation of oxy-Hb. Blue circle represents
2,3-bisphosphoglycerate (BPG)

Deoxy-Hb
+ 402 ---+ Oxy-Hb

••
Lungs

•rt_e_ry_..J ~ ~- _ .
~--

Venous
circulation

Archibald
Vivian Hill
NP 1922 Deoxy-Hb + 402 +-- Oxy-Hb
1886-1977
Fig . 23.4: In tissues Oxy-Hb releases oxygen
20 40 60 80
p02 in mm of Hg
A-Theoretical curve as per mass action.
B- Sigmoid curve , due to heme-heme
interaction (Hill effect). C-Further shift to right
due to carbon dioxide (Bohr effect) and BPG.
This curve represents the pattern under normal
conditions. D- Further shift to right when
Christian Bohr
temperature is increased to 42°C.
1855-1911 Joseph Joseph Lavoisier John
Fig. 23.3: Oxygen dissociation curve (ODC)
Priestley Black 1743-1794 Haldane
1733-1804 '1 728-1799 1860-1936

Thus each successive addition of 0 2, increases the


and pH is low due to the formation of metabolic acids
affinity of Hb to oxygen synergistically. Similarly, binding
like lactate. Th,en the affinity of hemoglobin for 0 2 is
of 2,3-BPG at a site other than the oxygen binding site,
decreased (the ODC is shifted to the right) (Fig. 23.3C). In
lowers the affinity for oxygen
The quaternary structure of oxy-H 1s escn ed as the lungs, the opposite reaction is fou nd, where the pC0 2
f I ) form: and that of deoxy-Hb is is low, pH is high and p02 is significantly elevated. More
Thus, on oxygenation, the hemoglobin molecule can 0 2 binds to hemoglobin and the ODC is shifted to the left.
form two similar dimers.
The Bohr E/f'fect
(2 x alpha) + (2 x beta) - - -~ 2 x (alpha-beta)
The influence of pH and pC02 to facilitate oxygenation
(deoxy-Hb) (oxy-Hb)
of Hb in the lungs and deoxygenation at the tissues is
known as the Bohr effect (1904 ). Binding of CO2 forces
Effect of pH and pCO2
the release of 0 2 . When the pC02 is high, CO2 diffuses
When the pC02 is elevated, the W concentration into the red blood cells. The carbonic anhydrase in the
increases and pH falls. In the tissues, the pC02 is high red cells favors the formation of carbonic acid (H2 C0 3) .
342 Section B: General Metabolism

Erythrocyte in tissue capillary Erythrocyte in lung capillary

Chloride Chloride
enters into RBC comes out of RBC

Fig. 23.5: Chloride shift; reactions in tissues Fig. 23.6: Chloride shift; reactions in lungs

(see Fig. 10.20). The 2,3-BPG, preferentially binds to


deoxy-Hb (see Fig. 23.2). The high ~ gen affinity of
fetal blood (HbF) is due to the inability of gamma chains
to bind 2,3-BPG.

Adaptation to High Altitude


The Chloride Shift
1. Increase in the number of RBCs
When CO2 is taken up, the HCO3 concentration within 2. Increase in concentration of Hb inside RBCs
the cell increases. This would diffuse out into the plasma . 3. Increase in BPG.
Simultaneously, chloride ions from the plasma would
enter in the cell to establish electrical neutrality. This is
TRANSPORT OF CARBON DIOXIDE
called • • • • •• • • • • • • (Fig. 23.5).
Thus on venous side, RBCs are slightly bulged due to At rest, about 200 ml of CO2 is produced per minute in
the higher chloride ion concentration. tissues. The CO2 is carried by the following 3 ways:
When the blood reaches the lungs, the reverse rea-
ction takes place. The deoxyhemoglobin liberates pro- Dissolved Form
tons. These would combine with HCO3- to form H2CO3
About 10% of CO2 is transported as dissolved form.
which is dissociated to CO2 and H2 O by the carbonic
anhydrase. The CO2 is expelled. As HCO3- binds W, CO2 + H2 O -+ H2 CO3 -+ HCO 3- + W
more HCQ3- from plasma enters the cell and c1- gets The hydrogen ions thus generated, are buffered by
out (reversal of chloride shift) (Fig. 23.6). the buffer systems of plasma.

Effect of Temperature lsohydric Transport of Carbon Dioxide


Metabolic demand is low when there is relative hypo- lsohydric transport constitutes about 75% of CO2 • It
thermia. Shift in ODC to left at low temperature results means that there is minimum change in pH during the
in release of less 0 2 to the tissues. On the other hand, transport. The H+ ions are buffered by the deoxy-Hb and
under febrile conditions, the increased needs of 0 2 are this is called th
met by a shift in ODC to right (Fig. 23.30).
y : Inside tissues, pCO2 is high and carbonic
Effect of 2,3-BPG acid is formed. It ionizes to H+ and HCO3- inside the
RBCs. The H+ ions are buffered by deoxy-Hb and the
The 2,3-BPG concentration is higher in young children HCO3- diffuses out into the plasma. In order to maintain
compared to the elderly. The 2,3-BPG is produced ionic equilibrium, chloride ions are taken into RBC
, ---,
from 1,3-BPG, an intermediate of glyc~ pathway I ...
, Fig. 23.5). Thus, the CO 2 is transported
• '
Chapter 23: Hemoglobin 343

from tissues to lungs, as plasma bicarbonate, without


significant lowering of pH. The W are bound by
: . al appllcat1ons of oxygen d1ssoc1at1on curve
1. In all hypoxic states, the 0 2 affinity is decreased w it h a
N-terminal NH 2 groups and also by the imidazole groups shift in 0DC to the right and an increase in 2,3-BPG inside
of RBC. The adaptation to the high altitude where p02 is low,
includes increased pulmonary ventilation, polycythemia
and increase in 2,3-BPG level with a shift in 0DC to right.
The isoelectric point of oxyhemoglobin is 6.6, while that
2. In anemia, where the total concentration of Hb is
of deoxy-Hb is 6.8. Thus oxy Hb is more negatively reduced, inc1reased oxygen unloading alone will ensure
charged than deoxy Hb. The reaction in tissues may be proper oxygEmation of tissues.
written as 3. In many casEis 2,3-BPG level varies inversely as the hemo-
OxyHbs + H+ __. HHb- + 0 2 globin conce•ntration.
4. In chronic pulmonary diseases and cyanotic cardiac
Therefore, some cation is required to remove the
diseases also the 2,3-BPG level is increased ensuring
extra negative charge of Oxy-Hb. So W are trapped maximum unloading of 0 2 to tissues.
(Fig. 23.5). Hemoglobin binds 1 proton for every 2 5. The red cell 2,3-BPG level is decreased in acidosis and
oxygen molecules released. Or, 1 millimol of deoxy Hb increased in alkalosis. Hence, the expected shift in 0DC
can take up 0.6 mEq of H+, produced from 0.6 mmol of
••••Din
to the right or left is not observed.
carbonic acid. lung capillaries, where 6. Transfusion ,of large volumes of stored blood, w hich has
the pO2 is high, oxygenation of hemoglobin occurs. a low level of 2,3-BPG can lead to sudden hypoxia, since it
can cause a left-shifted 0DC.
When 4 molecules of 0 2 are bound and one molecule
of hemoglobin is fully oxygenated, hydrogen ions are
released. facilitating trans:placental oxygen transport. The major
H- Hb + 402 - - - • Hb(O),. + W reason is the diminished binding of 2,3-BPG to HbF.
The protons released in the RBC combine with When pO2 is 20 mm Hg, the HbF is 50% saturated.
HCO3- forming H2CO3 which would dissociate to CO2 , The synthesis of HbF starts by 7th week of gestation;
that is expelled through pulmonary capillaries. it becomes the predominant Hb by 28th week. At birth,
80% of Hb is HbF. During the first 6 months of life, it
Carriage as Carbamino-Hemoglobin decreases to about 5% of total. There is a rapid post-
natal rise in 2,:3-BPG content of RBC. However, HbF
The rest 15% of CO 2 is carried as carbamino-hemoglobin,
level may remaiin elevated in children with anemia and
without much change in pH. A fraction of CO2 that enters
beta thalassemia, as a compensatory measure.
into the red cell is bound to Hb as a carbamino complex.
R-NH 2 +CO2 - - - R -NH-COOH Hemoglobin A2
The N-terminal amino group (valine) of each globin
It is a normal hemoglobin; it is about 2% of total Hb.
chain forms carbamino complex with carbon dioxide.
It has 2 alpha chains and 2 delta chains. The delta
Deoxyhemoglobin binds CO2 in this manner more readily
chain has sequ,ance homology with beta chain. In beta
than oxyhemoglobin.
thalassemia, as a compensation, Hb~ is increased. The
Clinical applications are shown in Box 23.2.
isoelectric pH o1' HbA2 is 7.4, while HbA has the pl value
Fetal Hemoglobin (HbF) of 6.85. So, HbJ\ is slower moving on electrophoresis.

HbF has 2 alpha chains and 2 gamma chains. Gamma


chain has 146 amino acids. The differences in physico-
~ EMOGLOBIN DERIVATIVES
chemical properties when compared 'vfflh HbA are: Hemoglobin derivatives are formed by the combination
a. Increased solubility of deoxy HbF of different ligands with the heme part, or change in the
b. Slower electrophoretic mobility for HbF oxidation state of iron. Oxy-Hb is dark red, deoxy-Hb
c. Increased resistance of HbF to alkali denaturation is purple, met-Hb is dark brown, CO-Hb is cherry red
d. HbF has decreased interaction with 2,3-BPG and sulph-Hb is green in color. Normally concentration
The ODC offetus and newborn are shifted to left. This of deoxy-Hb is lless than 5% of the total Hb. If the level
increase in 0 2 affinity is physiologically advantageous i~ increase occurs.
X
344 Section B: General Metabolism

Carboxy-Hemoglobin (Carbon
Monoxy Hb) (CO-Hb)
Hb-Fe+++ y,,,- Cyto.b5-Fe.. NAO+

Hemoglobin binds with carbon monoxide (CO) to form


carboxy-Hb. The affinity of CO to Hb is 200 times more Hb-Fe++.,_A...._. Cyto.bS-Fe+++ NADH+H•

than that of oxygen. It is then unsuitable for oxygen Fig. 23.7: Met-hemoglobin reductase system
transport. When one molecule of CO binds to one
monomer of the hemoglobin molecule, it increases the . In
affinity of others to 0 2 ; so that the 0 2 bound to these persons with this enzyme deficiency, the condition may
monomers are not released. This would further decre- be manifested even with small doses of drugs. In such
ase the availability of oxygen to the tissues. persons, NADPHI is not available in the RBC. Therefore
CO is a colorless, in such individuals, disease is manifested easily (see
odorless, tasteless gas generated by incomplete Chapter 6).
combustion. CO poisoning is a major occupational
hazard for workers in mines. Breathing the automobile Hemin Crystals
exhaust in closed space is the commonest cause for When iron is oxidized to Fe++•, it has a net positive
CO poisoning. The carboxy-Hb level in normal people is charge. It can combine with negatively charged chloride,
0 .16%. An average smoker has an additional 4% of CO- to form hemin or hematin chloride. Hemin crystals can
Hb. One cigarette liberates 10-20 ml carbon monoxide be prepared from even very old blood stains in medico-
into the lungs. legal cases. Blood or eluted blood stains are heated with
Clinical symptoms manifest Nippe's fluid (1% solution of KCI, KBr and Kl in glacial
when carboxy-Hb levels exceed 20%. Symptoms are acetic acid) over a glass slide, when dark brown rhom-
breathlessness, headache, nausea, vomiting, and pain bic crystals are seen under the microscope. The test is
in chest. At 40-60% saturation, death can result. Admini- sensitive, but is answered by the heme part of blood of
stration of 0 2 is the treatment. In severe cases, oxygen all species.
under high pressure (hyperbaric oxygen) is helpful.
Glycated Hemoglobin
Met-Hemoglobin (Met-Hb) Described in detail in Chapter 11 .
When the ferrous (Fe·•) iron is oxidized to ferric (Fe•••)
state, met-Hb is formed. Small quantities of met-Hb HEMOGLOBIN (GLOBIN CHAIN)
formed in the RBCs are readily reduced back to the VARIANTS
ferrous state by met-Hb reductase enzyme systems. - -- - -
About 75% of the reducing activity is due to enzyme Hemoglobin1opathies
system using - and cytochrome b5 (Fig. 23.7). These are a group of heterogeneous disorders affecting
Another 20% of the reducing activity is due to globin chain synthesis. Common symptoms like anemia,
dependent system. met-Hb- splenomegaly and vase-occlusive episodes in sickling
reductase accounts for the rest 5% activity. disorders overlap in these conditions.
Qualitative defects in globin chain synthesis, i.e.
Met-Hemoglobinemias abnormal globin chains with altered amino acid sequence
Normal blood has only less than 1% of met-hemoglobin. (abnormal hemoi~lobins or hemoglobin variants) result
It has markedly decreased capacity for oxygen binding in hemoglobinopathies. These are structural variants of
and transport. An increase in met-hemoglobin in blood, hemoglobins. Huindreds of such hemoglobin variants
(met-hemoglobinemia) is manifested as have been discovered (Box 23.3). The variants may be
Causes may be congenital or acquired . either alpha chain variants or beta chain variants (Table
23.2).
Acquired or Toxic Met-Hemoglobinemia Quantitative defects in globin chain synthesis, i.e.
Drugs which produce met-hemoglobinemia are-aceta- normal globin chains in abnormal amounts res~ t in
minophen, phenacetin, sulfanilamide, amylnitrite, and thalassemias; alp1ha or beta thalassemia depending on
sodium nitroprusside. the deficiency of the chain.
Chapter 23: Hemoglobin 345

BOX 23.3: He111ogloh1nopathy and ll1.-1l.-1ssP-m1a TABLE 23.2: Important hemoglobinopath1cs


Abnormalities in the primary seq uence of globin chains lead to Point mutation Aminoadd Codon and base
hemoglobinopathies, e.g. HbS. Hemoglobin position substitution substitution
Abno rmalities in t he rate of synthesis would result in thalas- HbS Beta 6
semias. In other words, normal g lo bin chains in abno rmal con- Beta6 lys
HbC
centrations result in thalassemias e.g. beta thalassemia.
HbE Beta 26 lys AAG
HbD (Punjab) Beta 121
A third group of disorders result from defects in
HbM Proximal or dist al UAC
regulation of globin chain synthesis; gamma to beta histid ine in a or (3
chain switch, characterized by hereditary persistence of chains
fetal hemoglobin (HPFH).
The hemoglobin variants may be classified into 5 Hemoglobin S or sickle cell hemoglobin is the most
major types, based on their clinical manifestations. common monogenic disorder. The sickle cell disease
was first described by Dr Herrick in 1910.
Sickle Syndromes
A. Sickle-cell trait (AS) Sickle Cell Disease
B. Sickle-cell disease with SS, SC, SD, SO varieties
The glutamic acid in the
and S beta thalassemia.
of HbA is changed to valine in HbS (Fig. 23.8C). This
single amino acid substitution leads to polymerization
Unstable Hemoglobins
of hemoglobin molecules inside RBCs. This causes
Congenital Heinz body anemia; Hb Zurich. a distortion of cell into sickle shape (Fig. 23.8A). The
Unstable Hb variants have an increased tendency to substitution of hydrophilic glutamic acid by hydrophobic
denature and hence tend to form molecular aggregates
valine causes a localized stickiness on the surface of the
within cells which lead to increased hemolysis. Heinz
molecule (Fig. 23 .9). The deoxygenated HbS may be
bodies are stained purple with cresyl violet. These are
depicted with a protrusion on one side and a cavity on
due to oxidative stress. Patients will have moderate to
the other side, so that many molecules can adhere and
severe anemia.
polymerize. HbS can bind and transport oxygen. The
Hemoglobins with Abnormal Oxygen Affinity sickling occurs under deoxygenated state. The sickled
cells form small plugs in capillaries. Occlusion of major
A. High affinity-Polycythemia (familial): Hb Chesa-
vessels can lead to infarction in organs like spleen (Fig.
peake
23.8B). Death usually occurs in the second decade of life.
B. Low affinity-Cyanosis (familial): HbM.
The heterozygous state is very common in Central
Structural Variations Leading to and West Africa as well as in East and Central parts of
India. Tribals all over India show an increased incidence
Thalassemia
of SS and AS. The slave trade has played an important
A. Alpha thalassemia-Hb Constant spring, delta beta
role in spreading the gene from Africa to different parts
thalassemia, Hb Lepore
of America.
B. Beta thalassemia: Hb Quong sze (Hb QS).

Nonsymptomatic Hemoglobin Variants Sickle Cell Trait


HbP, Q, N, J, etc. In heterozygous (AS) condition, 50% of Hb in the RBC is
normal. Therefore, the sickle cell trait as such does not
Hemoglobin S (HbS) produce clinical symptoms. Such persons can have a
(Sickle Cell Hemoglobin) normal lifespan. At higher altitudes,• • •1 may cause
Of the hemoglobin variants, HbS constitutes the - manifestation of the disease. Chronic
variety worldwide. In 1949 Linus Pauling may also produce hypoxia-induced sickling in HbS trait.
(Nobel Prize, 1954) established that a hemoglobin with On electrophoresis, the abnormal HbS can be detected
abnormal electrophoretic mobility is responsible for the along with normal Hb in persons with HbS trait (Fig .
sickling disease. 23.11).
346 Section B: General Metabolism

Normal red blood cells

RBCs flow freely


withing blood vessel

Fig. 23.SA: Left side; normal RBCs. Right side, sickle cells

Abnormal, sickled, red


Molecular differences of HbA, HbS and HbC blood cells (sickle cells)

HbA
Val-His-Leu-Thr-Pro-Glu-Glu-Lys
12345 6 78

HbS
Val-His-Leu-Thr-Pro-Val-Glu-Lys
12345 6 78 QOIIHIM8M_of___
HbC
Sticky sickle cells

r~ ~~;ands
::..
Val-His-Leu-Thr-Pro-Lys-Glu-Lys
12345 6 78

Fig. 23.SC: Molecular changes in HbS and HbC


that cause
stickle shape
: The high
incidence of the sickle cell gene in population coincides Fig. 23.8B: Mechanism of vasc ular occlusion in sickle cell anemia
with the area endemic for malaria. HbS affords protection
against Plasmodium falciparum infection (Fig . 23.10). Therapy with hydroxyurea increases the concentra-
Hence the abnormal gene was found to offer a biologic tion of HbF. This would prevent polymerization of Hb S
advantage. molecules. So sickling will not occur. Short chain fatty
acids also have a similar effect on Hb F levels by dere-
Electrophoresis pression of gamma chai~ ynthesis.
Electrophoresis at alkaline pH shows a slower moving
band than HbA. At pH 8.6, carboxyl group of glutamic Hemoglobin E
acid is negatively charged. Lack of this charge on
It is the hemoglobin variant. It
HbS makes it less negatively charged, and decreases
is due to the replacement <:Ji.beta 26 glutamic acid by
the electrophoretic mobility towards positive pole (Fig .
Lysine (see Table 23.2). It is primarily seen in orientals of
23.11). At acidic pH, HbS moves faster than HbA. In
South-East Asia (Thailand, Myanmar, Bangladesh, etc.).
sickle cell trait, both the bands of HbA and HbS can be
noticed (Fig. 23.11 ). The variant is very prevalent in West Bengal in India.
Heterozygotes are completely asymptomatic. HbE has
Sickling Test similar mobility as~ on electrophoresis.
A blood smear is prepared. Add a reducing agent such
Hemoglobin C
as sodium dithionite. Blood smear examined under the
microscope shows sickled RBCs (see Fig. 23.8A). In normal HbA, 6th amino acid in beta chain is glutamic
acid; it is replaced by lysine in HbC (see Table 23.2 and
Management of Sickle Cell Disease Fig. ~ C). The presence of HbC is seen mostly in
Repeated blood transfusions may be required in severe the black race. The HbC is slower moving than HbA on
anemia. But this can lead to iron overload and cirrhosis. electrophoresis at alkaline pH.
Chapter 23: Hemoglobin 347

I 5 1-"§.
Normal RBC Result
0
--+ 008 Malaria:
0 00 000
Oxy-Hb Deoxy-Hb Oxy-HbS Deoxy-HbS Two deoxy-HbS stick dJ 0
RBC tysis
Fig. 23.9: Sticky patches on HbS molecule Malarial parasite enters and multiplies

RBC with sickle cell trait


Point of application No malaria:

+ve pole
!D - ve pole
--+ No RBC lysis
Sickle
cell trait

' • Sand A
Parasite enters, but cannot multiply Normal health


Fig. 23.10: Sickle cell trait protects from malaria
Sickle
cell anemia D s

I
Linus
Pauling Parents
Normal D A NP 1954
hemoglobin 1901- 1994

Fig. 23.11: Electrophoresis at pH 8.6

Offspring
Hemoglobin D Normal
Sickle HbC HbS-C
It does not produce sickling. HbD Punjab results from trait trait disease

replacement of beta 121 glutamic acid by glutamine I= Normal gene Q=Sickle gene
(HbD Punjab) (see Table 23.2). HbD migrates similar
to HbS on electrophoresis. HbSD disease is a severe
I= HbC gene

condition . HbD Punjab is the commonest Hb variant Fig. 23.12: Inheritance of HbS-C disease
seen in Punjabi population.
and are less likely to produce impairment of red cell
M-Hemoglobins (Hb M) function (codominant inheritance). Thus beta chain
These are a group of variants, where the substitution disease is more common and more severe than alpha
occurs in the proximal or distal histidine residues of alpha chain disease.
or beta chains. As a result, the heme has a tendency to Globin chain genes are located on two different
get oxidized to hemin, forming • • • • • •· Oxygen chromosomes: alpha and similar chains on chromo-
binding is decreased. This would result in some 16; beta and similar chains on chromosome 11 .
Genetic defects in hemoglobinopathies are mainly point
Inheritance of Hemoglobin Variants mutations resulting in substitution of a single amino acid.
They are inherited as ... -~.i . l
The abnormal phenotype is decided by the type and
One beta chain gene is inherited from each parent. If location of the substituted amino acid. If the properties of
both parents are heterozygous for S, (the abnormal the substituted amino acid are different from the original
gene), there is 50% chance that the child will be AS, one, clinical manifestations can occur even in heterozy-
25% chance for AA and 25% chance for SS genotype gous state.
(inheritance is described in Chapter 42). If the genotype
is SS , all the Hb molecules produced are abnormal, and THALASSEMIAS
hence disease is manifested .
If one parent is heterozygous for HbS and another The name is derived from the Greek word, "thalassa" ,
for C or beta thalassemia , 25% chances are that the which means "sea". Greeks identified this disease pre-
child will be a double heterozygote (Fig . 23.12). An sent around Mediterranean sea. It is the most common
individual inherits only 2 beta chain genes; but genetic disease and has a carrier frequency of about
j a dre inherited. So, the alpha chain variants 7% across the globe. The disease is also prevalent
constitute only 25% of the total hemoglobin in circulation in lndia.Thalassemia may be defined as the normal
348 Section B: General Metabolism

Fig. 23.13: Thalassemia. Nucleated RBC, target cells, sphero-


cytosis, poikilocytosis are seen

Fig. 23.14: Myoglobin chain


hemoglobins in abnormal proportions (Box 23.3). The
gene function is abnormal, but there is no abnormality
homozygous staIte, clinical manifestations are severe,
in the polypeptide chains. Reduction in alpha chain
and hence calleid Thalassemia major. There will be
synthesis is called alpha thalassemia, while deficient
nucleated RBCs in peripheral circulation (Fig. 23.13).
beta chain synthesis is beta thalassemia.
In heterozygous conditions, the clinical signs and symp-
Genetic defects may be (a) RNA splicing defects, (b)
toms are minimal; they are called Thalassemia minor.
promoter mutants, (c) poly-adenylation signal defects,
or (d) partial gene deletion. Homozygous bet.a thalassemia is characterized by severe
anemia, hypersplenism and hepatosplenomegaly.
Beta Thalassemia The marrow in the skull bones expand producing the
"hair-on-end appearance" described in X-ray. Repea-
Beta thalassemia is more common than alpha variety. ted transfusion is. the only available treatment. This may
Beta type is characterized by a decrease or absence of lead to iron overload.
synthesis of beta chains. As a compensation, gamma or Hydroxyurea and short chain fatty acids are useful
delta chain synthesis is increased. in ameliorating the symptoms.
Symptoms manifest in beta thalassemias only when
gamma chain synthesis is repressed. I MYOGL01_31_N_ _ __
Alpha Thalassemias It is seen in muscles. Myoglobin (Mb) content of skeletal
muscle is 2.5 g/I00 g; of cardiac muscle is 1.4 g% and
Alpha thalassemia is rarer because alpha chain defi-
of smooth musclBs 0.3 g%. Mb is a single polypeptide
ciency is incompatible with life.
chain (Fig. 23.14). Human Mb contains 152 amino acids
Most often the genetic defect is gene deletion. A silent
with a molecular weight of 17,500 Daltons. One mole-
carrier state can result from a single gene defect when
cule of Mb can combine with 1 molecule of oxygen. The
synthesis of alpha chains is 75%. Alpha thalassemia
Hb carries oxygen from lungs to tissue capillaries, from
trait occurs when two genes are deleted. A three gene
where oxygen dilffuses into tissues. In the muscles, the
deletion will lead to HbH disease with moderately severe
oxygen is taken up by Mb for the sake of tissue respi-
symptoms. Deletion of all the four genes result in death
ration. Mb has hi~1her affinity for oxygen than that of Hb.
in utero (hydrops fetalis), since synthesis of embryonal
The pO2 in tissue is about 30 mm of Hg, when Mb is 90%
Hb and HbF require alpha chains.
saturated. At this pO2 , Hb saturation will be only 50%.
Thalassemia Syndromes
Clinical Significance of Myoglobin
These syndromes are mainly seen in people of Asian, Severe crush injury causes release of myoglobin from
African and Mediterranean origin. All cases of thalas- the damaged muscles. Being a small molecular weight
semias are characterized by deficiency of HbA syn- protein , Mb is excreted through urine (myoglobinuria).
thesis. Hypochromic microcytic anemia is seen. In Urine color becomes dark red.
Chapter 23: Hemoglobin 349

BOX 23.4 : Causes of anemias • Lethargy, Fatigue • Shortness


of breath
1. Hemolysis due to Impaired Production of RBCs
• Angina
a. Defect In heme synthesis: Nutritional deficiency of • Exertional
iron, copper, pyridoxal phosphate, follc acid, vitamin B12 • Impaired immune system chest pain
or vitamin C. Lead will inhibit heme synthesis.
• Anorexia • Impaired
b. Defect in regulators: Erythropoietin synthesis is
concentration
reduced in chronic renal failure. • Endocrine/
c. Defect in stem cells: Aplastic anemia due to drugs (e.g. metabolic • Impaired libido/
Chloramphenicol), infections or malignant infiltrations. abnormalities impotence
2. Hemolysis due to lntracorpuscular Defect • Cardiorespiratory • Insomnia
a. Hemoglobinopathies such as HbS, HbC, HbM disturbances
b. Thalassemias-major and minor • Headache
• Gastrointestinal
c. Abnormal shape: Spherocytosis and elliptocytosis disturbances
• Pallor
d. Enzyme deficiencies: Deficiency of glucose-6-phos-
phate dehydrogenase (see Chapter 6). • Reduced exercise
• Neuromuscular
tolerance
3. Hemolysis due to Extracorpuscular Causes disturbances
a. Infections: Malarial parasites
b. Autoimmune hemolysis: Antibodies are seen against
RBC membrane components Fig. 23.15: Common symptoms of anemia
c. lsoimmune hemolysis: Rh incompatibility
d. Hemolysis due to drug sensitization: Many drugs (e.g.
alpha-methyldopa, quinine) may fix on RBC membrane, noticed that they were both in a confused state. The
and produce antibodies against the altered membrane. couple had been in good health. The previous night was
4. Hemorrhage bitterly cold, and so the elderly couple kept the coal fur-
Hemat uria, hematemesis, hemoptysis, peptic ulcer, metror-
nace in the bedroom throughout the night. In the morn-
rhagia and hemorrhoides are the usual causes for hemor-
rhage. Hem ophilia (absence of AHG) and thrombocyto- ing, they both complained that they had severe head-
penia are other major causes for bleeding tendencies. ache, confusion, fatigue and nausea.
On arrival at the emergency department, both
patients were afebrile with normal vital signs. Their lips
Myoglobin will be released from myocardium during
appeared to be very red. Both patients were slightly
myocardial infarction (Ml), and is seen in serum. Serum
confused. The physical examinations were within normal
myoglobin estimation is one of the biomarkers for Ml
limits. Carboxyhemoglobin levels were elevated.
(see Chapter 6).
1. What is the most likely cause of these patients'

IANEMIAS symptoms?
2. What is the biochemical rationale for 100% 0 2
In India, anemia is the most common medical problem. being the treatment of choice?
Perhaps about 75% of patients attending a primary
health center may have signs and symptoms directly or 0
indirectly related to anemia. Anemia results whe,, the •• · Clinical Case Study 23.2
Hb concentration in blood is reduced. Normal value for A 15-year-old Indian aboriginal girl presents to the
Hb in normal male is 14 to 16 g/dl and in female 13 to emergency room with complaints of bilateral thigh and
15 g/dl. If the Hb level is below 10 g/dl, it is a severe hip pain. The pain has been present for 1 day and is
condition. steadily increasing in severity. Acetaminophen and
The most common cause for anemia in India, is iron ibuprofen have not relieved her symptoms. She denies
deficiency which is described in Chapter 34. A list of any recent trauma or excessive exercise.
other causes is given in Box 23.4. Common symptoms She reported fatigue and burning sensation during
of anemia are shown in Figure 23.15. urination. She reports having similar pain episodes in
the past, sometimes requiring hospitalization. On exami-
0 nation, she is afebrile. Her conjunctiva and mucosa!
• • Clinical Case Study 23.1
membranes are slightly pale in color. She has non-
During a winter day morning, an elderly couple were specific bilateral anterior thigh pain with no abnormali-
taken to the emergency department, when their son ties appreciated. The remainder of her examination is
350 Section 8 : General Metabolism
completely normal. Her white blood cell count is ele- autosomal recessive fas hion. The sickled red blood cells
vated at 17,000/mm3 , and her hemoglobin (Hb) level is cause infarction of bone, lung, kidney, and other tissue
decreased to 7.1 g/dl. The urinalysis demonstrated an from vaso-occlus.ion.
abnormal number of numerous bacteria. The patient's description of pain is typical of a
1. What is the most likely diagnosis? sickle cell pain crisis. Often infection is a trigger, most
2. What is the molecular genetics behind this disorder? commonly pneumonia or a urinary tract infection. Here,
3. What is the pathophysiologic mechanism of her symptoms are consistent with a urinary tract infection,
symptoms? indicated by burning with urination (dysuria). Her white
blood cell count is elevated in response to the infection.
0 The low hemoglobin level is consistent with sickle cell
• • Clinical Case Study 23.3 anemia. Since she is homozygous (both genes coding
A 25-year-old female presents to her obstetrician at for sickle hemoglobin), both her parents have sickle cell
12 weeks gestation. This is her first pregnancy, and she trait (heterozygo1us) and thus do not have symptoms.
is concerned about her baby and the risk of inheriting a The diagnosis can be established with hemoglobin
"blood" disease like others in her family. Her brother had electrophoresis.
died at age 10 due to severe anemia. 0
Her ultrasound confirmed an intrauterine pregnancy •• Clinical Case Study 23.3 Answer
at 12 weeks gestation. The patient's hemoglobin level
Molecular genetics: Impaired production of 13-globin
shows a hypochromic, microcytic anemia. Hemoglobin
peptide chain.
electrophoresis demonstrated increased fetal hemoglobin
level, a pattern consistent with a-thalassemia minor. Likely test: Oligonucleotide probe. After chorionic villus
sampling is performed, a radioactive probe can be used
1. What is the molecular genetics behind this disorder?
and hybridized with specific genetic mutations in the
2. What was the likely test and what is the biochemical
fetus' deoxyribonucleic acid (DNA), allowing for prompt
basis?
detection and prematal diagnosis.
0 The most common cause of anemia is iron defi-
•• · Clinical Case Study 23.1 Answer ciency. Another common cause of microcytic anemia
is thalassemia. Certain ethnic groups have higher inci-
Most likely cause: Carbon monoxide poisoning
dence of thalassBmia .
(increase carboxyhemoglobin level).
The microcyltic (small red blood cell size) anemia
Rationale for treatment: Administration of 100% 0 2 with elevated hemoglobin Az and F is consistent with
displaces CO from hemoglobin. 13-thalassemia miinor. Patients with 13-thalassemia major
Carbon monoxide binds very avidly to hemoglobin. (Cooley anemia) typically have severe anemia requiring
It is a colorless and odorless gas and may arise from frequent transfusions and shortened life expectancy.
internal combustion engines, fossil-fueled home appli- Infants will appear healthy after birth, but as the hemo-
ances (heaters, furnaces, stoves). It does not give globin F levels fall, the infant becomes severely anemic.
warning signs. The patient generally has confusion and
symptoms of 0 2 deprivation, but not the symptoms of LEARNING POINTS, CHAPTER 23
dyspnea , since the hemoglobin is saturated. The lips
1. Hemoglobin (Hb) is a globular protein containing
are a distinct red color as a result of the hemoglobin
2 alpha 2 b13ta (HbA), 2 alpha 2 gamma (HbF), 2
being "oxygenated." However, because CO binds so
alpha 2 delta (HbA2). Alpha chain has 141 amino
avidly to the hemoglobin, no transfer of 0 2 occurs in the
acids while beta, gamma and delta have 146 amino
peripheral tissue. Treatment is thus 100% 0 2 to displace
acids.
the CO from the hemoglobin.
2. The ability cit Hb to load and unload 0 2 at physio-
0 logical pO2 iis shown by the 0 2 dissociation curve
• Clinical Case Study 23.2 Answer and its sigmoid shape is due to the allosteric effect
(co-ope rativiity).
Most likely diagnosis: Sickle cell disease (pain crisis 3. The influence of pH and pCO2 to facilitate oxygena-
Biochemical mechanism of disease: Single amino acid tion of Hb in lungs and deoxygenation at the tissue
substitution on hemoglobin beta chain , inherited in an is known as Bohr effect.
Chapter 23: Hemoglobin 351

4. Entry of Cl from plasma to cells to establish 11 . The positive co-operative effect is exhibited by
neutrality for the HCO3- out to the plasma is called different subunits where by binding of 0 2 by one
Chloride shift. subunit favors binding of 0 2 to other subunits. In a
5. HbF moves slower than HbA on electrophoresis, similar manner 0 2 release also shows a co-operative
is resistant to alkali denaturation, has decreased interaction referred to as heme-heme interaction.
interaction with 2,3-BPG. , 12. Transport of CO2 by is called isohydric transport,
6. Hb S is sickle cell Hb resulting from a Glu-Val which helps in buffering and the elimination of vola-
substitution at the 6th position on the beta chain. tile acid, carbonic acid. In the tissues where pCO2
HbS gives protection against malaria. is high, CO2 diffuses into RBCs and combines with
7. Thalassemias are caused due to functional abnor- Hp to form carbonic acid catalyzed by carbonic
mality of alpha or beta chains of Hb. Homozygous anhydrase. The H+ formed by dissociation of H2 CO3 ,
states exhibit Thalassemia major while heterozy- is buffered by HHb, which can take up H+when oxy-
gous state exhibit Thalassemia minor. gen dissociates.
8. Alpha chain gene is on chromosome 16 and beta, 13. A fraction of CO2 is transported as carbamino Hb.
(gamma, delta) chain genes on chromosome 11 . 14. Fetal hemoglobin has a higher affinity for 0 2 than
9. The factors which affect oxygen binding and Hb A, since it does not bind 2,3-BPG. This is bene-
release are pO2 , pCO2 , pH and 2,3-BPG concentra- ficial in oxygen transport and delivery to tissues in
tion in RBCs. foetal life.
10. Arterial blood is 97% saturated at pO2 of 100 mms 15. The fetal Hb level falls after birth and reaches adult
of Hg and can carry about 40 mms of Hg. In venous levels by 2 years of age (<2%).
blood, where pO 2 is about 40 mms of Hg, Hb is only 16. In anemic children and children with beta chain
60% saturated. Net effect is a release of 40% 0 2 to defects, Hb F level remains high as a compensatory
tissues. mechanism.

PART-1: ESSAY AND SHORT NOTE QUESTIONS


23-1. What is the difference between hemoglobinopathies and thalassemias? Describe any one of the hemoglobino-
pathies in detail.
23-2. Give biochemical explanation for the finding that geographical distribution of glucose-6-phosphate dehydro-
genase deficiency correlates well with malarial incidence.

SHORT NOTE QUESTIONS


23-3. Chloride shift. 23-8. Sickle cell anemia.
23-4. lsohydric transport of carbon dioxide. 23-9. Hemoglobin S.
23-5. Oxygen dissociation curve. 23-10. Hemoglobin F.
23-6. Abnormal hemoglobins. 23-11 . Thalassemia.
23-7. Hemoglobinopathies.

PART-2: MULTIPLE CHOICE QUESTIONS


23-1 . HbA differs from HbF in that: 23-3. All the following are true with regard to carboxy
A. HbA has only alpha chains hemoglobin, except
B. HbF cannot bind to 2,3-BPG A. CO binds to ferrous iron to form carboxyhemoglobin
C. HbF can bind only 2 molecules of oxygen B. CO has more affinity for Hb than oxygen
D. HbA is alkali resistant C. Oxygen dissociation curve is shifted to right
23-2. Methemoglobinemia is found in all the following
D. Cannot be converted back to deoxy Hb
conditions, except
23-4. One deciliter (1 00 ml) of blood with 15 grams of
A. Ingestion of nitrites
hemoglobin can carry how much oxygen?
B. Carbon monoxide poisoning
C. Presence of HbM A. 7.5 ml of oxygen B. 15 ml of oxygen
D. Poisoning by aniline dyes C. 20 ml of oxygen D. 30 ml of oxygen
352 Section B: General Metabolism

23-5. Which factor causes a shift in the ODC (oxygen 23-15. The common molecular defects in beta thalassemia
dissociation curve) to left? include the following , except
A. High pO2 B. Low pH A. Frame shift mutations
C. Low pO2 D. High temperature B. Nonsense mutations
23-6. All the following manifestations are seen in sickle C. Suppressor mutations
cell anemia (HbS disease), except D. Improper splicing of hnRNA
A. Pain and swellings in joints 23-16. Unstable hemoglobins will produce:
B. Hemolytic anemia A. Erythrocytosis B. Heinz body anemia
C. Sickled cells in peripheral circulation C. Polycythemia D. Sickling
D. Inclusion bodies in RBCs 23-17. Presence of an abnormal hemoglobin is suspected
23-7. A person with HbS trait is likely to have: if a patient has any of the signs, except
A. About 90% of hemoglobin as HbS A. Nonspherocytic hemolytic anemia
B. A single band on electrophoresis B. Hypochromic microcytic anemia
C. Sickling at high altitudes C . Polycythemia
D. Irreversibly sickled cells in peripheral smear D. Cyanosis without cardiopulmonary dysfunction
23-8. All are features of isohydric transport, except: 23-18. Hemoglobin will not copolymerize with HbS?
A. CO2 is transported in plasma as bicarbonate A . HbC B. HbA
B. Carbonic anhydrase is the major enzyme involved C. HbD D. HbE
C. Chloride shift is a result of CO2 transport 23-19. Which phenomenon is not related to functional
D. CO2 is bound to amino terminal group properties of hemoglobin?
23-9. Methemoglobinemia may be caused by all the A. Bohr effect
following, except B. Haldane effect
A. Methemoglobin reductase deficiency C. Pasteur effect
B. Presence of HbM D. Donnan effect
C. Ingestion of nitrites 23-20. In homozygous beta thalassemia:
D. Deficiency of ascorbic acid A. HbH is the major hemoglobin
23-10. Carbon dioxide is carried in blood with all the B. Symptoms appear al birth
forms, except C. Persistence of HbF is observed
A. Dissolved form D. Patients have a normal life span
B. lsohydric transport 23-21 . Beta thalassemia can result from, except
C. Carboxyhemoglobin A. Terminator codon mutations
D. Carbaminohemoglobin B. Abnormal splicing of mRNA
23-11. All are true with regard to HbF, except C. Defect in post-translational processing
A. Poorly binds 2,3-BPG D. Mutation in suppressor genes
B. Copolymerises with HbS 23-22. Myoglobinuria is observed in all the following con-
C. Affinity for oxygen is more ditions, except:
D. Is a tetramer of 2 alpha and 2 gamma chains A. Severe muscular exercise
23-12. Regarding myoglobin, all are true, except: B. Crush injuries of limbs
A. It has 4 polypeptide chains C. Myasthenia gravis
B. Binds oxygen D. Muscle wasting diseases
C. Oxygen binding affinity is higher than that of hemo- 23-23. Which hemoglobin derivative is normally found in
globin blood?
D. Contains one heme residue per molecule A. Deoxyhemoglobin
23-13. Regarding HbS and HbA, all are true, except: B. Sulfhemoglobin
A. The chain length of HbS is smaller than HbA C. Acid hemalin
B. Solubility of deoxy HbS is lower than deoxy HbA D. Cyanmethemoglobin
C. HbS is slower moving on electrophoresis than HbA 23-24. Methemoglobinemia may be caused by all the
D. Structural difference is with regard to beta chain following , except:
23-14. Anemia can result from deficiency of all the A. Methemoglobin reductase deficiency
following, except B. Presence of HbM
A. Iron B. Cobalamin C. Ingestion of nitrites
C. Cholecalciferol D. Pteroylglutamic acid D. Deficiency of ascorbic acid
Chapter 23: Hemoglobin 353

23-25. All are the characteristics of HbS, except it: 23-32. All are true with regard to hemoglobin M variants,
A. Is an abnormal hemoglobin except:

. B. Is less soluble in deoxy state


C. Tends to polymerize when pO2 is low
A. T hey have increased oxygen carrying capacity
B. Are oxidized to methemoglobin
D. Cannot bind oxygen C. Binds only 2 molecules of oxygen
23-26. A sudden shift in oxygen dissociation curve to the D. Responds to administration of methylene blue
left can occur due to: 23-33. During oxygen transport by hemoglobin:
A. Diabetes mellitus A. One hemoglobin molecule carries 4 oxygen atoms
B. Acute renal failure B. The graph of percentage of oxygen saturation
C. Metabolic alkalosis versus oxygen pressure is sigmoid in shape
D. Ascending to high altitudes C. Oxygen binding causes the iron to change from
23-27. All are characteristics of thalassemia major, except ferrous to ferric state
A. It is manifested when the affected genes are in D. Hemoglobin carries 1.34 ml of oxygen per 100 ml
homozygous state of blood
B. It is the result of gene deletion 23-34. Regarding the CO2 transport in blood:
C. It is a common molecular disease in India A. OxyHb is more negatively charged (stronger acid)
than deoxy Hb
D. It is incompatible with life
B. In normal blood, bicarbonate to carbonic acid ratio
23-28. Which factor decreases the affinity of hemoglobin
is 1 :20
for oxygen binding?
C. One millimole deoxy hemoglobin can take up 10
A. An increase in 2,3-BPG level in blood
mEq of hydrogen ions
B. A rise in pH
D. Carboxyl end group of Hb chain takes up carbon
C. Oxidation of iron to ferric state in one subunit
dioxide
D. Vomiting 23.35. In HbA, the 6th amino acid is glutamic acid. In HbS,
23-29. A patient with heterozygous alpha thalassemia this is replaced by valine. The mobility of HbA and
will have: HbS during electrophoresis will be
A. Anemia A. HbS and HbA have same mobility
B. HbH B. HbS moves faster than HbA to anode
C. HbF C. HbS moves faster than HbA to cathode
D. Hematuria D. HbS moves slower than HbA to anode
23-30. Patients having hemoglobin variants with increa- 23-36. A person with sickle cell trait can follow any of the
sed oxygen affinity will have: following occupations, except:
A. Increased level of HbF A. Software development
B. Shifting of ODC (oxygen dissociation curve) to left B. Biotechnologist
C. Target cells in circulation C. Fighter pilot
D. Binds 2 ,3-BPG with more affinity D. Air traffic controller
23-31 . All are correct with HbS trait, except: 23-37. Toxic Methemoglobinemia is characterized by all,
A. It confers a biological advantage against malaria except:
B. It is compatible with life A. Severe central cyanosis
C. It is manifested at high altitudes as vase-occlusive B. Cherry red color of blood

. phenomenon
D. Chronic hemolytic anemia is the usual manifestation
C. Blood gives a strong absorption band at 630 nm
D. Cyanosis improves on giving 1/V methylene blue

ANSWERS OF MULTIPLE CHOICE QUESTIONS


23-1. B 23-2. B 23-3. C 23-4. C 23-5. C 23-6. D 23-7. C
23-8. D 23-9. D 23-10. C 23-11. B 23-12. A 23-13. A 23-14. C
23-15. C 23-16. B 23-17. B 23-18. B 23-19. C 23-20. C 23-21 . D
23-22. C 23-23. A 23-24. D 23-25. D 23-26. C 23-27. D 23-28. A
23-29. A 23-30. B 23-31. D 23-32. A 23-33. 8 23-34. A 23-35. D
23.36. C 23-37. C
354 Section B: General Metabolism

PART-3: VIVA VOCE QUESTIONS AND ANSWERS


23-1 . Hemoglobin molecule Is made up of what? 23-11 . What is the difference between HbA and HbF?
Hb has 4 subunits, 2 alpha and 2 beta units. It contains HbA has 2 alpha and 2 beta chains, but HbF has 2
4 iron atoms. alpha and 2 gamma chains.
23-2. How many molecules of oxygen can bind with 23-12. What is the physiological significance of HbF?
hemoglobin? Oxygen affinity is more for HbF than HbA. HbF is seen
Hb can bind 4 molecules of oxygen. in foetal circulation .
23-13. What is the clinical significance of HbF?
23-3. 100 ml of blood can carry how much oxygen?
It is seen in adults in hemoglobinopathies and thalas-
20 ml.
semias.
23-4. What is Bohr effect?
23-14. Why carbon monoxide becomes a poison?
The influence of pH and pCO2 to facilitate oxygenation
Hb has more affinity to carbon monoxide than oxygen.
of Hb in the lungs and deoxygenation at the tissues is 23-15. What is met-hemoglobin?
known as the Bohr effect. Hemoglobin in which iron is in ferric state or in oxidized
23-5. What is chloride shift? form.
When CO2 is taken up, chloride ions from the plasma 23-16. What is Hemoglobin S (HbS)?
would enter. T his is called chloride shift or Hamburger The glutamic acid in the 6th position of beta chain of
effect. HbA is changed to valine in HbS.
23-6. How hemoglobin carries CO2 ? 23-17. What is the cause for sickle cell anemia?
75% as isohydric transport and 15% as carbamino- Solubility of deoxy HbS is lower than deoxy HbA; so
hemoglobin. HbS is precipitated intracellularly, leading to sickle
23-7. What are the forms in which carbon dioxide is shape of RSC.
23-18. How HbS is identified?
transported? •
HbS is slower moving on electrophoresis than HbA.
Dissolved form; lsohydric transport; and Carbamino-
23-19. What is sickle cell trait?
hemoglobin.
Heterozygous (AS) condition; One allele is normal; the
23-8. What is isohydric transport of carbon dioxide?
other is abnormal; so half of Hb molecules are normal;
There is minimum change in pH during the transport. and half abnormal.
23-9. How is this effected? 23-20. What is its clinical significance?
The W ions are buffered by the deoxy-Hb. Persons with sickle cell trait will not have any disease
23-10. What is the function of 2,3 BPG? manifestations. But, at higher altitudes, hypoxia may
It will decrease the affinity of hemoglobin for oxygen. cause manifestation of the disease.
"

SECTION
Clinical and Applied
Biochemistry

Chapter 24 Liver and Gastric Function Tests


Chapter 25 Kidney Function Tests
Chapter 26 Plasma Proteins
Chapter 27 Acid-Base Balance and pH
Chapter 28 Electrolyte and Water Balance
Chapter 29 Body Fluids (Milk, CSF and Amniotic Fluid)
Chapter 30 Free Radicals and Antioxidants
Chapter 31 LaboratoryTechniques,Quality Control and Metalbolic Diseases
"

i
_____Chapter 24
Liver and Gastric
Function Tests

Chapter at a Glance
The learner will be able to answer questions on the following topics:
D Serum and urine bilirubin Gastric function and HCI secretion
D Tests based on synthetic function D Gastric juice analysis
D Enzymes indicating hepatocellular damage Pancreatic function tests

Biochemical tests are of immense value in diagnosis


and monitoring of liver diseases. These tests are usually
IFUNCTIONS OF LIVER
--------
referred to as "liver function tests" (LFT). LFTs are the most Synthetic Function
widely performed biochemical tests in the laboratory.
The plasma proteins albumin, alpha and beta globu-
Important liver functions are listed in Table 24.1. Often
abnormal liver function will lead to jaundice (Fig. 24.1). lins, clotting factors, carrier proteins, hormonal factors,
A long list of tests was formerly included under this growth factors, bile acids, cholesterol and phospholi-
group and were classified based on the major functions pids are the major biomolecules synthesized by the liver
of liver. But nowadays, only clinically useful tests are (Box 24 .1).
being done. These liver function tests are broadly
classified as: Carbohydrate Metabolism
1. Tests to detect hepatic injury:
Glucose may be metabolized to yield energy. If not,
a. To detect the disease, whether mild or severe;
glucose can be stored as glycogen within the liver or
whether acute or chronic.
it can be converted into more stable storage form as
b. To assess the nature of liver injury; hepatocellular
triglycerides. Homeostasis of blood glucose is described
or cholestasis.
in Chapter 11 .
2. Tests to assess hepatic function.

Amino Acid Metabolism


lf_ROBLEMS IN INTERPRETATION
a. Normal LFT values need not indicate absence of Some of the proteins are produced only in the liver,
liver disease, because liver has very large reserve examples are albumin, alpha and beta globulins and
capacity. coagulation factors I, 11, V, VII, IX and X (see Box
b. Asymptomatic people may have abnormal LFT 24.1). Several proteins of acute phase reactants are
results. So interpretation should be based on produced in the liver, e.g . C-reactive protein (infectious
clinical picture. diseases).
358 Section C: Clinical and Applied Biochemistry

A. Classification based on laboratory findings


Group I (tests of hepatic excretory funct ion) Yellow color of sclera is seen in
Jaundice. Normal serum bilirubin
i. Serum-bilirubin; total, conjugated, and unconjugated value is 1 mg/dl. When 1I exceeds
ii. Urine-bile pigments, bile salts and urobilinogen 2 mg/dL, bilirubin deposits in tissues.

Group II: Liver enzyme panel (see Chapter 6) (These are markers of
liver injury and/or cholestasis)
i. Alanine ami not ransferase (ALT) Fig. 24.1: Jaundice
ii. Aspartate aminot ransferase (AST)
iii. Alkaline phosphatase (ALP)
Detoxification Functions
iv. Gamma-glutamyltransferase (GGT)
Group Ill: Plasma proteins (see Chapter 26) (Tests for synthetic func- Exogenous substances : Toxic substances entering
tion of liver)
from gut and parenteral route are mainly detoxified in the
i. Total prot eins
liver by different reactions. The cytochrome P450 enzyme
ii. Serum albumin, globulins, NG ratio
system of hepatocyte is mainly concerned with drug me-
iii. Prothrombin time
Group IV: Special tests
tabolism (Chapter 36).
i. Ceruloplasmin (see Chapters 26 and 34) Endogenous s1...bstanc9s: Disposal of bilirubin is
ii. Ferritin (see Chapter 34) already discussed. Ammonia produced from amino acid
Iii. Alpha-1-antit rypsin (AAT) catabolism is detoxified by the liver to form less toxic
iv. Alpha-fetoprotein (AFP) (see Chapter 48) urea. The key urea cycle enzymes are located entirely
B. Classification based on clinical aspects
in liver.
Group I: Markers of liver d ysfunction
i. Serum bilirubin, total, conjugated
ii. Urine: BIie pigments, bile salts and UBG
Excretory Functions
iii. Total protein, serum albumin and N G ratio Substances detoxified by the liver are excreted through
iv. Prothrombln ti me
bile. About 3 liters of bile is produced daily and out of this
v. Blo od ammonia, w hen indicated
1 Lis excreted and the rest is reabsorbed and circulated
Group II: Markers of hepatocellular injury
i. Alanine amino transferase (ALT)
in the enterohepatic circulation. The bile contains bile
ii. Aspartate aminotransferase (AST) salts, conjugated bilirubin, phospholipids and cholesterol.
Group Ill: Markers of cholestasis Major functions of the liver are summarized in Box 24.1.
i. Alkaline phosphatase

I CLINICAL MANIFESTATIONS
ii. Gamma-glutamyltransferase

OF LIVER DYSFUNCTION
----
Lipid Metabolism Jaundice
Fatty acids will be catabolized to release acetyl-CoA, Jaundice is the yellowish discoloration of sclera , skin and
and further to produce energy. Acetyl-CoA is also used mucous membrane. It is characteristic of liver disease
for fatty acid synthesis or cholesterol production. Dietary but it will occur when rate of hemolysis is increased
lipids are repackaged and secreted into the systemic leading to elevation of serum bilirubin.
circulation as lipoproteins. The protein parts of the lipo-
proteins, apoproteins are synthesized by the liver only. Portal Hypertension
Lipoproteins are described in Chapter 14.
The entire venous drainage of gastrointestinal tract, the
spleen, the pancreas and the gallbladder constitutes
Bilirubin Metabolism
portal circulation with a pressure of 5 mm of Hg. Any
Heme is catabolized to bilirubin, which is transported as obstruction in the course of portal circulation will cause
bilirubin-albumin complex. In the liver, bilirubin is conju- portal hypertension.
gated with glucuronic acid, and fi nally excreted through The major cause of portal hypertension is cirrho-
bile (Chapter 22). sis. Portosystemic shunting leads to deterioration of the
Chapter 24: Liver and Gastric Function Tests 359

BOX 24.1: MaJor functions of liver Scarring of liver


1. Synthetic function
a. Synthesis of plasma proteins (albumin, coagulation fac-
!
Increased resistance to blood flow

l
tors, many globulins)
b. Synthesis of cholesterol
c. Synthesis of triacyl glycerol Hypoalbuminemia, leading to decreased colloid intravascular
pressure, oozing of protein rich fluid into the intraperitoneal
d. Lipoprotein synthesis
space, increased lntraperitoneal colloid pressure
2. Metabolic function
a. Carbohydrates: Glycolysis; glycogen synthesis; glyco-
gen breakdown; gluconeogenesis
l
Decreased central blood pressure
b. Ketogenesis; fatty acid synthesis and breakdown
c. Protein catabolism
!
Decreased renal perfusion, leading to
d. Citric acid cycle, production of ATP activation of renin angiotensin system,
sodium retention, water retenbon
3. Detoxification and excretion
a. Ammonia to urea i
Further fluid escapes into the peritoneal cavity
b. Bilirubin (bile pigment)
c. Cholesterol Fig. 24.2: Pathogenesis of ascites
d. Drug metabolites
4. Homeostasis: Blood glucose regulation
BOX 24.2: lnd1cat1ons for liver function tests
5. Storage function: Vitamin A, D, K, B12
1. Jaundice
6. Production of Bile salts; help in digestion
2. Suspected liver metastasis
3. Alcoholic liver disease
metabolic functions of the liver. Failure of detoxification
4. Any undiagnosed chronic illness
of ammonia by urea synthesis leads to hyperammone-
5. Annual checkup of diabetic patients
mia and hepatic encephalopathy. Decrease in albumin
6. Coagulation disorders
synthesis leads to hypoalbuminemia.
7. Therapy with statins to check hepatotoxicity

Ascites
complex) (free bilirubin) (indirect bilirubin) varies from
It is due to effusion of serous fluid into the abdominal
0.2-0.7 mg/dl and conjugated bilirubin (direct bilirubin)
cavity. It is a common presenting feature of cirrhosis.
Ascites may be due to different causes. If the ratio of
0.1- 0.4 mg/dl. A rise in serum bilirubin above 1 mg/dL
serum albumin : ascitic fluid albumin is > 1.1 , it is diag- is abnormal (latent jaundice); but jaundice appears only
nostic of portal hypertension as the cause. Figure 24.2 if the level goes above 2 mg/dl.
explains the pathogenesis of ascites. Box 24.2 gives the The bilirubin is estimated by van den Bergh reac-
indications of liver function tests. A detailed classification tion, where diazotized sulfanilic acid (sulfanilic acid in
of the liver function tests (LFT) is shown in Table 24.1 . HCI and sodium nitrite) reacts with bilirubin to form a
Important liver function tests are described below: purple colored complex, azobilirubin. When bilirubin is
conj ugated , the purple color is produced immediately
Markers of Hepatic Dysfunction on mixing with the reagent, the response is said to be
van den Bergh direct positive. When the bilirubin is
Measurement of Bilirubin
unconjugated, the color is obtained only when alcohol
(Test of Excretory Function of Liver) is added, and this response is known as indirect
Bilirubin is the excretory product formed by the catabo- positive. If both conjugated and unconjugated bilirubin
lism of heme. It is conjugated by the liver to form bilirubin are present in increased amounts, a purple color is pro-
diglucuronide and excreted through bile (see Chapter duced immediately and the color is intensified on adding
22). Measurements of bilirubin is an important liver func- alcohol. Then the reaction is called biphasic.
tion test. In Hemolytic jaundice, unconjugated bilirubin is
Normal serum bilirubin level varies from 0.2 to 1.0 increased. Hence van den Bergh test is indirect positive.
mg/dl. The unconjugated bilirubin (bilirubin-albumin In obstructive jaundice, conjugated bilirubin is elevated,
360 Section C: Clinical and Applied Biochemistry

TABLE 24.2: Urinary findings in Jaundice TABLE 24.3: Class1f1cat1on of Jaundice


Type ofjaundice Bile pigment Bile salt Urobilinogen Type of billrubln Class ofjaundice Causes
Prehepatic (hemolytic) Nil Nil ++ Unconj ugated Prehepatic or Abnormal red cells;
Hepatocellular ++ + Normal · hemolytic antibodies; drugs and
toxins; t halassemia;
Posthepatic (obstructive) +++ ++ Nil "
hemoglobinopathies. Gilbert's
syndrome; Crig ler-Najjar
syndrome
BOX 24.3: Causes of cholestat1c liver disease
1. Extrahepatlc cholestasis Unconjugated Hepatic or Viral hepatitis; toxic hepatitis;
and conjugated hepatocellular intra hepatic cholestasis
Cholelithiasis (stone in gallbladder)
Carcinoma head of pancreas Conjugated or Posthepatic Ext rahepatic cholestasis;
Portal lymphadenopathy obstructive gallstones; tumors of bile
duct; carcinoma of pancreas;
Chronic pancreatitis
lymph node enlargement in
Biliary stricture
porta hepatls
Parasites (liver flukes) (rare in India)
2. lntrahepatic cholestasis
Alcoholic cirrhosis stage sets in and reappears when obstruction is cleared.
Primary biliary cirrhosis
Urobilinogen is ab~e:-,t in urine, when there is obstruc-
Non-alcoholic steatohepatitis (NASH)
Viral hepatitis (cholestatic phase) tion to bile flow. The first indication of recovery is the
Protoporphyria reappearance of urobilinogen in urine. In hemolytic
Dubin-Johnson syndrome anemias, urobilinogen is increased. Bilirubin is detected
3. Drugs
by Fouchet's test and urobilinogen by Ehrlich's test. The
Androgens, Chlo rpromazine
Chlorpropamide, Nitrofurant oin findings in urine in different types of jaundice are shown
Erythromycin, Phenytoin in Table 24.2. Table 24.3 shows the classification and
C closporine, Captopril causes for jaundice. Table 24.4 gives the tests to
distinguish different types of jaundice. Bilirubin synthe-
and van den Bergh test is direct positive. In hepatocel- sis, excretion and jaundice are described in detail in
lular jaundice, a biphasic reaction is observed, beca- Chapter 22.
use both conjugated and unconjugated bilirubins are
increased (see Chapter 22).
Urine Bile Salts
Normally bile salts (sodium salts of taurocholic acid
Urinary Bilirubin and glycocholic acid) are present in the bile; but are not
In all cases of jaundice, urine should be examined seen in urine. Bile salts in urine are detected by Hay's
for the presence of bile pigments (bilirubin), bile salts test. Positive Hay's test indicates the obstruction in the
and urobilinogen. Only conjugated bilirubin is soluble biliary passages causing regurgitation of bile salts into
in water and is excreted in urine. Hence in prehepatic the systemic circulation leading to its excretion in urine.
jaundice, when the unconjugated bilirubin is increased Obstruction can occur in obstructive jaundice and also
in blood, it does not appear in urine. But in obstructive in hepatic jaundice due to obstruction of micro biliary
jaundice, conjugation of bilirubin is taking place, which channels caused by inflammation.
cannot be excreted through the normal passage, and "
so it is regurgitated back into bloodstream; this is then Causes of Jaundice
excreted through urine. So in obstructive jaundice, urine Causes of jaundice are enumerated in Boxes 24.3 and
contains bilirubin. 24.4. The most common cause for hepatocellular jaun-
dice is infection with hepatitis viruses (viral hepatitis).
Urinary Urobilinogen
It may be due to hepatitis A virus (HAV), which is trans-
In cases of obstruction, bile is not reaching the intestine mitted by the intake of contaminated food and water.
and so urobilinogen may be decreased or absent in urine. Type A disease is usually self-limiting.
In hepatocellular jaundice, urobilinogen is initially eleva- Infection by hepatitis B virus (HBV) is transmitted
ted, then decreases or disappears when the obstructive mainly through parenteral contamination by infected
Chapter 2'4: Liver and Gastric Function Tests 361

TABLE 24,4: Tests useful to d1stingu1sh different types of jaundice


Prehepatic or hemolytic Posthepatic or obstructive
Specimen Test or retention jaundice Hepatocellularjaundice or regurgitation jaundice
Blood Unconjugated bilirubin ++ ++ Normal
(van den Bergh indirect test)
Conjugated bilirubin Normal Excretion is rate-limiting. It is the first ++
(van den Bergh direct test) impaired activity. In early phase, it is
increased
Alkaline phosphatase (40- 125 U/L) Normal 2-3 times increased 10-12 times
Urine Bile salt (Hay's test) Absent Absent Present
Conjugated bilirubin (Fouchet's) Absent Present Present
Urobilinogens (Ehrlich test) +++ Increased in early cholestatic phase; Absent
Absent later as production is low.
Earliest manifestaltion of recovery is
presence of bilinogen in urine
Feces Urobilins ++ Normal o r decreased Clay colored

BOX 24.4: Causes of hepatocellular damage


1. Viruses: HAV, HBV, HCV, Herpes, Adenovirus
: .
Hepatitis A
- . ogy to define viral hepatitis type
Anti-HAV lgM
2. Alcohol Hepatitis B acute HBsAg, anti-HBc lgM
3. Toxins: Carbon tetracholoride, Chloroform, Mushroom, Afla-
Hepatitis B chroni,c HBsAg, HBeAg, HBV DNA
toxin, Arsenic
4. Immunological: Autoimmune hepatitis, NASH Hepatitis C Anti-HCV and HCV RNA

5. General diseases: Wilson's disease, Hemochromatosis, AAT Hepatitis D (delta) HBsAg and anti-HOV
deficiency, Porphyrias, Sarcoidosis, Amyloidosis Hepatitis E Anti-HEV
6. Neoplasm: Hepatocellular carcinoma, Metastatic liver d isease,
Lymphoma
7. Bacterial infections:TB, Leptospirosis, Brucella, Abscesses In a small fraction of such cases, development of
8. Parasites: Helminths, Amebiasis, Plasmodia, Leishmania hepatocellular caircinoma is also noticed. Thus HBV is an
9. Drugs: Salicylate, Tetracyclines, Methotrexat e, lsoniazid,
Rifampicin, Halothane, Methyld opa, Valproate
oncogenic virus. Medical personnel, including medical
students, doctors, nurses and technicians are advised to
take Hepatitis B vaccination. Hepatitis viruses type A, B,
C, D, E and Gare identified. Box 24.5 gives the serology
to define the type of viral hepatitis.

Tests Basedl on Synthetic


Function of Liver
van den Baruch Barry J Robin
Bergh Blumberg Marshall Warren Serum Albumin Level
1869-1943 NP 1976 NP 2005 NP2005
1925-2011 b. 1951 b. 1937 Almost all the plasma proteins except immunoglobulins
are synthesized by the liver. Serum albumin (see
blood or blood products. The virus is highly contagious Chapter 26) is quantitatively the most important protein
and can be destroyed only by boiling for 20 minutes. It synthesized by the liver, and reflects the extent of
is a DNA virus, which destroys the hepatic cells. The functioning liver cell mass. Since albumin has a fairly
surface antigen (HBs) is seen in the circulation of long half-life of 20 days, in all chronic diseases of the
patients. For his contributions in hepatitis prevention, liver, the albumin level is decreased (see Chapter 26).
Baruch Blumberg was awarded Nobel Prize in 1976. Normal albumin level in blood is 3.5 to 5 g/dl ; and
About 5% of world populations are carriers of HBV. In globulin level is 2.5 to 3.5 g/dl . The turnover rates of
about 2-5% cases, the disease passes on to a chronic haptoglobin andl transferrin are lesser than albumin;
carrier state. About 1% of cases progress to chronic hence they are useful to identify recent changes in liver
cirrhosis and eventual hepatic failure. functions.
362 Section C: Clinical and Applied Biochemistry

Serum Globulins Enzymes Indicating Hepatocellular Damage


lmmunoglobulins are produced by B lymphocytes. But Liver enzyme panel is described in detail in Chapter 6.
alpha and beta globulins are synthesized mainly by Normal serum ALT (alanine aminotransferase) is 10-35
hepatocytes. Gamma globulins in the serum are increa- IU/L. The levels of aminotransferases (ALT and AST)
sed in chronic liver diseases (chronic active hepatitis, in serum are elevated in all liver diseases (Box 24.7).
cirrhosis). Further details of immunoglobulins are given Very high levels (more than 1000 units) are seen in
in Chapter 46. acute hepatitis (viral and toxic). The degree of elevation
Prothrombin Time (PT) may reflect the extent of hepatocellular necrosis. In
most cases the lowering of the level of transaminases
Since prothrombin is synthesized by the liver, it is a useful
indicates recovery, but a sudden fall from a very high
indicator of liver function. The half life of prothrombin is
level may indicate poor prognosis. Elevation of ALT is
6 hours only; therefore PT indicates the present function
more in cases of hepatic disease compared to AST. A
of the liver. PT is prolonged only when liver loses more
ratio of AST/ALT more than two is quite suggestive of
than 80% of its reserve capacity. Vitamin K deficiency
alcoholic liver disease.
is also a cause for prolonged prothrombin time. In case
of liver disease, the PT remains prolonged even after Moderate elevation of amino transferases often
parenteral administration of vitamin K. between 100 and 300 U/L is seen in alcoholic hepatitis
and non-alcoholic chronic hepatitis (Box 24.7). Minor ele-
Alpha-Fetoprotein (AFP) vation less than 100 U/L is seen in chronic viral hepatitis,
It is a normal component of fetal blood. It disappears fatty liver and in non-alcoholic steatohepatitis (NASH). A
after birth within a few weeks. It is a tumor marker normal value need not rule out minor liver diseases.
(Chapter 48). Mild elevation is suggestive of chronic
hepatitis or cirrhosis; drastic increase is seen in hepato- Markers of Obstructive Liver Disease
cellular carcinoma, germ cell tumors and teratoma of
ovary. Elevated AFP in the maternal serum is seen in
Alkaline Phosphatase (ALP)
cases of fetal open neural tube defects. Very high levels of alkaline phosphatase (ALP) are
Other important serum proteins, synthesized by the noticed in patients with cholestasis or hepatic carci-
liver, are ceruloplasmin, transthyretin, alpha-1-antitrypsin noma. Bile duct obstruction induces the synthesis of the
and haptoglobin. These are described in Chapter 26. enzyme by biliary tract epithelial cells (see Chapter 6).
In parenchymal diseases of the liver, mild elevation
Serum Electrophoresis of ALP is noticed. But in hepatitis, inflammatory edema
Abnormal electrophoretic patterns are shown in Figure produces an obstructive phase, during which ALP level
26.1. Pre-albumin is reduced in acute hepatitis. Albumin is elevated. (Table 24.4). Very high levels of ALP (10-1 2
is decreased in cirrhosis. Alpha-1 globulins fraction is times of upper limit) may be noticed in extrahepatic
decreased in hepatocellular disease almost parallel to obstruction (obstructive jaundice) caused by gallstones
albumin. It is increased in febrile illnesses and malig- or by pressure on bile duct by carcinoma of head of
nancies. Alpha-2 globulins and beta globulins, when pancreas.
increased suggest biliary obstruction. Gamma globulins lntrahepaticcholestasis may be due to virus (infective
are increased in cirrhosis. The rise in gamma globulin will hepatitis) or by drugs (chlorpromazine). ALP is produced
have wide base, suggestive of polyclonal gammopathy. by epithelial cells of biliary canaliculi and obstruction of
bile with consequent irritation of epithelial cells leads
Tests Based on Serum to secretion of ALP into serum. Drastically high levels
Enzymes (Liver Enzyme Panel) of ALP (10-25 times of upper limit) are seen in bone
The enzymes used in the assessment of hepatobiliary diseases where osteoblastic activity is enhanced. For
disease may be divided into two groups: (a) Those indi- example, Paget's disease (osteitis deformans), rickets,
cating hepatocellular damage, and (b) Those indicating osteomalacia, osteoblastoma, metastatic carcinoma of
cholestasis (obstruction). bone and hyperparathyroidism.
Chapter 24: Liver and Gastric Function Tests 363

BOX 24.6: Clinical features of acute liver failure


liver
: . al s1gnif1cance of AST/ALT ratio
Normal AST: ALT ratio is 0.8. A ratio >2 is seen in
Loss of metabolic function
Alcoholic hepatitis
Decreased gluconeogenesis leading to hypoglycemia
Hepatitis with cirrhosis
Decreased lactate clearance leading to lactic acidosis
Decreased ammonia clearance leading to hyperammonemia Non-alcoholic i;teatohepatitis (NASH)
Decreased synthetic capacity leading to coagulopathy Liver metastases
Portal hypertension Myocardial infarction
l ungs
Erythromycin t 1reatment
Adult respiratory distress syndrome
A low ratio (ALT is higher) is seen in
Adrenal gland
Inadequate glucocorticoid production contributing to hypoten- Acute hepatoc1~llular injury
sion Toxic exposure
Bone marrow
Extra hepatic obstruction (cholestasis)
Frequent suppression, especially in viral diseases
Circulating leukocytes
Impaired function contributing to sepsis
Brain
Hepatic encephalopathy Serum bilirubin
Cerebral edema
Albumin Normal Decreased
lntracranial hypertension
Globulin < 2.59% > 2.Sg%
Heart
ALP Normal Increased
Subcllnical myocardial injury
Transaminases < 100 U/ L 100-1000 U/L
Kidney
PT Prolonged Prolonged
Frequent dysfunction or failure
Autoantibodies AINA (Anti nuclear ANA, smooth muscle
antibody) antibody
Gamma-Glutamyltransferase (GGT)
GGT is clinically important because of its sensitivity The clinical features of acute liver failure are shown
to detect alcohol abuse. GGT level in alcoholic liver in Box 24.6.
disease roughly parallels the alcohol intake (see Chap-
ter 6). Elevated levels of GGT are observed in chronic Immunological Tests in Liver Disease
alcoholism. In liver diseases, GGT elevation parallels
lgG level is increased in chronic hepatitis, alcoholic and
that of ALP and is very sensitive of biliary tract disease.
autoimmune hepatitis. It shows a slow and sustained
increase in viral hepatitis. lgM shows marked increase
Tests for Metabolic Capacity of Liver
in primary biliary cirrhosis and moderate increase in
Blood Ammonia viral hepatitis anij cirrhosis. lgA is increased in alcoholic
cirrhosis and primary biliary cirrhosis (Table 24.5).
It is an index of urea synthesis by liver. It is a useful test
in hepatic encephalopathy. It is produced by action of
intestinal bacteria. The ammonia is later converted to
Selection of' Tests
urea by the liver, but this activity is considerably dec- Liver function tests are the most common group of bio-
reased in hepatic cell damage. In neonates suspected chemical tests done to diagnose and monitor the course
to have urea cycle disorders (and in organic acidurias), of liver disease. The increased incidence of infectious
ammonia estimation is indicated . diseases like viral hepatitis and leptospirosis requires
Precautions: Fasting plasma/serum is used for ammonia these tests to bo done in all patients with unexplained
estimation. Stringent precautions are to be maintained. illness.
Vacutainers must be used and the blood withdrawn until Several different tests are to be done for overall
it is full. Partial filling allows entry of air. Sample should assessment of tlhe liver function. Table 24.6 gives the
be immediately placed in ice and carry out the assay as different tests and the alterations in different types of
soon as possible. liver diseases.
364 Section C: Clinical and Applied Biochemistry

TABLE 24.6: Overview of liver function tests BOX 24.8: Liver regeneration
Parameter Remarks Prometheus was punished for stealing fire from the Gods. As

Serum albumin punishment he was chained to rocks and his liver was pecked by
Increased in chronic liver d isease
vultures everyday. However, he survived. How?
Serum globulins Increase in chronic hepatitis
About 90% of liver can be removed and still the remaining liver
PT Prolonged in liver disease
cells divide replacing the lost cells within weeks. The story tells
PT + vitamin K Prolonged in hepatocellular us that this ability of liver cells to regenerate was known even
If PT normal, cholestasis
from ancient days.
Al pha-fetoprotei n Increase in carcinoma
Ceruloplasmin Decrease in Wilson's
disease, Menke's disease Mechanism of HCI Secretion
Transthyreti n To assess nutritional stat us
The daily volume of gastric secretion is around 2000 ml.
Alpha-1-antitrypsin Decrease in neonatal
cholestasis, progressive juvenile The HCI secreted by the parietal cells is of high con-
cirrhosis, micronodular cirrhosis
centration (0.1 5 M) with a pH as low as 0.8. The parietal
Haptoglo bin, Transferrin Severe hepatocellular disease, cirrhosis.
cells transport protons against a concentration gradient. It
Amino acids Increased aromatic amino acids
+ branched chain aminoacids is an energy requiring process. The K• activated ATPase
in hepatic coma; both increased is necessary for the production of HCI (Fig. 24.3). The H+
in cirrhosis
ions are generated within the cell by ionization of carbonic
Serum bilirubin See Table 24.4
acid. The carbonic anhydrase is active in the parietal cells.
Urine bilirubin See Table 24.4
One molecule of ATP is hydrolyzed for every molecule
Urine urobilinogen See Table 24.4
of W secreted. The hydrolysis of ATP is coupled with
Plasma bile acids Post-prandial rise in hepatic
dysfunction; increased fasting an exchange of K• for W. The hydrogen ions are then
level in portosystemic shunting secreted into gastric lumen.
Urine bile salts Positive in posthepatic jaundice and
Side by side with the H+ to K• exchange, bicarbonate
hepatic jaundice
Ammonia Increase in cirrhosis, portocaval
to chloride exchange is also taking place (Fig. 24.3).
anastamosis When the bicarbonate level within the cell increases
Transamlnases (formed from H2C03) , it is reabsorbed into blood stream,
Viral hepatitis ALT and AST i ncreased in exchange for Cl-. The chloride is then secreted into the
Chronic active hepatitis N or slight increase
lumen to form HCI. This would account for the alkaline
Cholestasis Slight increase
tide of plasma and urine, following hydrochloric acid
Alcoholic hepatit is ALT/ AST ratio reversed
secretion, immediately after meals.
ALP Increase in cholestasis
GGT Increase in cholestasis
Regulation of Acid Secretion
Most commonly employed liver function tests in clini- i. Gastrin, the gastrointestinal peptide hormone
cal practice are serum bilirubin (total and direct), albu- secreted by G cells, stimulates secretion of HCI.
min, ALP, ALT, AST and GGT. Cholesterol level in blood The secretion of gastrin is cut off by acidic pH by a
is also increased in obstructive jaundice due to defective feedback regulatory control.
excretion through bile. In general, ALT and ALP distin- ii. The most potent stimulus for acid secretion is
guishes the pattern of liver disease. Albumin determines hi~tamine, which acts through specific H2 receptors
the chronicity and prothrombin time determines the on the gastric mucosa.
severity of liver dysfunction (Box 24.8).

Gastric Function Tests Assessment of Gastric Function


The gastric mucosa has different types of cells: (a) The In Fractional test meal or FTM, the fasting stomach
mucous secreting surface epithelial cells, (b) the oxyntic contents are aspirated and the secretion is stimulated
or parietal cells which secrete acid, and (c) the chief by giving test meals. Different samples are collected and
cells or peptic cells that secrete enzymes. the acidity (free and total) of each sample is measured .
Chapter 24: Liver and Gastric Function Tests 365

HCI mal hydrochloric acid secretion


,,,,.. lt.
Acid output in mmol/hour
-.....--
Lumen of Men Women
stomach
Lower limit Upper limit Lower limit Upper limit
Basal acid output - 10 - 5.5
Maximal acid 7 45 5 30
Oxyntic output
cell
Peak acid output 12 60 8 40

Basal acid output (BAO) is the acid output in


millimol per hour, in the basal secretion (in the absence
of all intentional ;and avoidable stimulation).
Maximal acid output (MAO) is the acid output in
Oxyntic millimol per hour, given by the sum of the acid output of
cell the four 15-minute samples after the stimulation.
Peak acid output (PAO) is the acid output in millimol
Blood capillary per hour, given by the sum of the acid output of the
Fig. 24.3: Hydrochloric acid secretion 2 consecutive 15-minute samples having the highest
acid content. Noirmal values are given in Table 24.7.
BOX 24.9: Causes for hyper and hypoacidity
Hyperacidity is seen in lnterpretaticms of Gastric Juice Analysis
i. Duodenal ulcer
ii. Gastric cell hyperplasia
Zollinger-Ellison syndrome. This condition results from
iii. Carcinoid tumors a gastrin secreting tumor (gastrinoma of the pancreas).
iv. Zollinger-Ellison syndrome There is no feedback regulation of gastrin secretion.
v. MEN (multiple endocrine neoplasia) There is very higIh level of gastric acid output along with
vi. Excessive histamine production as in systemic mastocytosis elevated serum ,gastrin levels. In this condition, BAO is
and basophilic leukemia. >15 mmol/h (may be as high as 150 mmol/h) and BAO/
Hypoacidity is seen in PAO ratio is 0.6 or higher.
Gastritis
In chronic duodenal ulcer, BAO, MAO and PAO
ii. Gastric carcinoma
are significantly elevated. BAO may vary from 4-6
iii. Partial gastrectomy
mmol/h and a B..11..O/PAO ratio of more than 0.3 indicates
iv. Pernicious anemia
increased basal secretory drive. Causes for hyperacidity
v. Chronic iron deficiency anemia.
and hypoacidity are shown in Box 24.9.
Gastric ulcers are perpetuated by the infection
The FTM is not done nowadays: but the modified
of Helicobacte1r pylori. Marshall and Warren were
version, though rare, is still in use. It is described below.
awarded Nobel Prize in 2005 for their discovery of the
bacterium H. pylori and its role in acid peptic disease.
Pentagastrin Stimulation Test The bacteria produce ammonia, with the help of bacterial
In the fasting condition, the gastric juice is aspirated urease. So, the o rganism can escape the attack of acidic
through a Ryle's tube (Residual juice). The gastric juice gastric juice. H. pylori infection is identified by presence
secreted for the next 1 hour is collected as a single of urease enzyme in gastric biopsies and detecting
sample (Basal secretion). The gastric secretion is now H. pylori antibodies in serum.
stimulated by giving pentagastrin. It is a synthetic peptide
having the biologically active sequence of gastrin. The
Pancreatic !Function Tests
gastric secretion is collected every 15 minutes for the The exocrine pancreas secretes about 1000-2500 ml
next 1 hour. of juice in 24 hours. The fluid is alkaline and contains
366 Section C: Clinical and Applied Biochemistry

bicarbonate and enzymes. This secretion is under the (vi) Crohn's disease or (vii) Whipple's disease. Pancreatic
control of the hormones, Secretin and Cholecystokinin. disease can leacj to defective digestion. The following
Secretin is produced under the stimulation of gastric HCI. tests are useful to assess the malabsorption states.
Secretin produces a secretion with high bicarbonate Fat balance studies: The estimation of fat in stool is
content. Gastrin stimulates production of cholecystokinin done. When feces contains split fatty acids, it points to a
(CCK), which in turn produces pancreatic secretion rich normal pancreatic function, but defective absorption. On
in enzymes. The major enzymes present in pancreatic the other hand, if the fat excreted is neutral fat, it is due
juice are amylase, lipase and proteolytic enzymes to defective digestion, and is more in favor of pancreatic
(trypsin, chymotrypsin, carboxypeptidase, elastase) as disease.
their zymogens (see Chapter 17).
0
•• Clinical Case Study 24.1
Assessment of Pancreatic Function
A 45-year-old maile with history of cirrhosis of the liver is
Measurement of pancreatic enzymes: Amylase or
brought to the emergency center by family members for
alpha-1,4-glucosidase is the major enzyme which
acute mental derangements, disorientation, alterations in
digests starch. Normal amylase level in serum is 50-120 personality and confusion over the last few days. Patient
units. The level rises within 5 hours of the onset of acute is vomiting blood. On examination, he is disoriented with
pancreatitis and the level reaches a peak within 12 evidence of icteric sclera. His abdomen is distended
hours. But the level need not parallel the severity of the and fluid shift si~1n is noted. His urine drug screen and
disease. Within 2-4 days of the attack, the level returns ethyl alcohol (EtOH) screen are both negative. A blood
to normal. ammonia level was elevated, and all other tests have
Amylase level in blood is mildly increased in cases been normal.
of cholecystitis. No significant change or only mild What is the most likely cause of the patient's symp-
elevation is noticed in chronic pancreatitis . toms? What was the likely precipitating factor of the
Serum lipase is the major lipolytic enzyme which patient's symptoms? What is the cause for fluid in abdo-
hydrolyzes glycerol esters of long chain fatty acids. The men?
level in blood is highly elevated in acute pancreatitis 0
and th is persists for 7-14 days. Thus lipase remains :ji. Clinical Case Study 24.2
elevated longer than amylase. Moreover, lipase is
A 45-year-old female presents to the clinic with mid-
not increased in salivary diseases. Therefore, lipase
epigastric pain and nausea/vomiting after eating "greasy
estimation has advantage over amylase.
meals". The symptoms gradually disappear, and return
Lundh test: The test meal is composed of milk powder, after some days. She denies any hematemesis. She
vegetable oil and glucose to make 6% fat, 5% protein had elevated cholesterol levels. On examination, she
and 15% carbohydrate. After aspirating the duodenal is afebrile with normal vital signs. Her physical exami-
contents, 500 ml of fl uid meal is given. Then duodenal nation is completely normal with no evidence of abdo-
secretions are collected at 30 minutes intervals for minal pain. An abdominal ultrasound is performed and
2 hours. The tryptic activity of duodenal aspirates are revealed a few gallstones in the gallbladder. What
measured. In chronic pancreatitis, the tryptic activity is factors would you need to consider to assess the need
decreased, but not in carcinoma of pancreas. for cholecystectomy? What are gallstones made of?
Can gallstones be seen on abdominal X-ray?
l.§_TUDIES ON MALABSORPTION 0
Malabsorption may result from defective digestion or • • Clinical Case Study 24.3
fau lty absorption or from both. Reduction of absorptive A 26-year-old female at 35 weeks gestation presents
surface may result from (i) Celiac disease; (ii) Gluten to the clinic with complaints of generalized itching (pru-
sensitive enteropathy; (iii) Tropical sprue; iv) ldeopathic ritis ). She denies any change in clothing detergent,
steatorrhea; (v) Extensive surgical removal of ileum; soaps, or perfumes. She denies nausea and vomiting.
Chapter 24: Liver and Gastric Function Tests 367

On physical examination, there are no rashes apparent fat. Gallstones form when the solutes in the gallbladder
on her skin. Blood test reveals slightly elevated serum precipitate. Cholesterol stones are usually yellow-green
transaminase and bilirubin levels. What is the patient's in appearance and account for approximately 80% of
likely diagnosis? What are treatment options? What is gallstones. Stones made of bilirubin appear dark in
the cause of the patient's generalized itching? color. Patients may have pain from the gallstones, usu-
ally after a fatty meal. The pain is typically epigastric
0
or right upper quadrant and perhaps radiating to the
• • Clinical Case Study 24.1 Answer
right shoulder. If the gallbladder becomes inflamed or
The likely cause is Cirrhosis. infected, cholecystitis can result. The stones can also
Patient presents with cirrhosis, most probably, travel through the bile duct and obstruct biliary flow
secondary to hepatitis virus infection, with acute mental leading to jaundice, or irritate the pancreas and cause
status change coinciding with recent onset of hemat- pancreatitis.
13mesis. Patient has an elevated serum ammonia level 0
and otherwise negative workup. Hepatic encephalopathy 6 Clinical Case Study 24.3 Answer
{disorientation, etc.) is secondary to elevated ammonia
Cholestasis of Pregnancy
levels. The precipitating factor is increased nitrogen load
from upper gastrointestinal bleeding. Etiology: Cholestasis of pregnancy is a condition in
Cirrhosis is a chronic condition of the liver with diff- which the normal flow of bile from the gallbladder is
use parenchymal injury and regeneration leading to dis- impeded , leading to accumulation of bile salts in the body.
tortion of the liver architecture and increased resistance Generalized itching and, possibly, jaundice may result. It
of blood flow through the liver. The patient usually is speculated that the hormones such as estrogen and
manifests malaise, lethargy, palmar erythema, ascites, progesterone, which are elevated in pregnancy, cause
jaundice, and hepatic encephalopathy in the late sta- a slowing of the gallbladder function, leading to this dis-
order. Uncomplicated cholestasis is usually diagnosed
11es. Toxins accumulating in the bloodstream affect the
clinically by generalized itching in a pregnant woman,
patient's mental status. The most common etiologies of
usually in the third trimester without a rash. Elevated
cirrhosis are toxins such as alcohol, viral infections such
serum bile salts, bilirubin and transaminase may also be
as hepatitis B or C infection, or metabolic diseases in
seen. The usual treatment includes antihistamine medi-
children (Wilson disease, hemochromatosis, or alpha-
cations for the itching. More severe cases may require
·1 -antitrypsin deficiency). As liver functions are reduced,
bile salt binders such as cholestyramine or corticosteroids.
albumin synthesis is lowered, which leads to ascites
(fluid in abdomen).
lh_EARNING POINTS, CHAPTER 24
0
,8 Clinical Case Study 24.2 Answer 1. Bilirubin is estimated by van den Bergh reaction.
Normal serum does not give a positive van den
Gallstones Bergh reaction.
Hurgical removal of gallbladder (cholecystectomy) 2. When bilirubin is conjugated, the purple color is
is done when there is frequent and severe attacks. produced immediately on mixing with the reagent,
Components of gallstones are cholesterol, calcium the response is said to be van den Bergh direct
bilirubinate, and bile salts. positive. When the bilirubin is unconjugated, the
Mixed stones are much easier to see on plain film color is obtained only when alcohol is added, and
secondary to calcifications, comprising approximately this response is known as indirect positive.
- 0% of gallstones. This individual fits the "classic" 3. The most common cause for hepatocellular jaundice
patient with gallbladder disease- female, middle-aged, is the infection with hepatitis viruses (viral hepatitis).
overweight. The gallbladder stores bile salts produced 4. Elevated levels of Gamma-glutamyltransferase
by the liver. The gallbladder is stimulated to contract (GGT) are observed in chronic alcoholism, pan-
when food enters the small intestine; the bile salts then creatic disease, myocardial infarction , renal failure,
travel through the bile duct to the duodenum. The bile chronic obstructive pulmonary disease and in dia-
salts act to emulsify fats, helping with the digestion of betes mellitus.
368 Section C: Clinical and Applied Biochemistry

Abnormal Liver Enzyme Profile

ALT or AST> 3 times normal value and ALT or AST < 3 times normal value and

..
ALP < 2 times value

Hepatocellular disease
ALP > 2 times
..
Cholestatic disease
I I
+
Albumin normal
+
Albumin low
+
Albumin 1normal
+
Albumin low
... ...
Acute cholestasis Chronic cholestasis
Acute hepatitis Chronic hepatitis

High GGT Mild increase in GGT

lntrahepatic cholestasis Extrahepatic cholestasis


Fig. 24.4: Algorithm for diagnosis of liver diseases using enzyme profile.
5. High levels of alkaline phosphatase (ALP} are 6. Clinical diai~nosis based on abnormal liver enzyme
noticed in patients with cholestasis or hepatic car- profile is shown in Figure 24.4.
cinoma.

PART-1 : ESSAY AND SHORT NOTE UESTIONS

24-1 . Enumerate liver function tests and describe in detail any two of them ,with clinical significance.
24-2. Classify jaundice. Give an account of the biochemical tests which will help in differentiat ing the types of
jaundice.
24-3. Name the bile pigments. Give the significance of their presence in blood and urine. How are they detected in
blood and urine?
24-4. Classify jaundice. How do you investigate a case of jaundice?
24-5. Discuss the biochemical alterations seen in blood and urine in differeint types of jaundice.

SHORT NOTE QUESTION

24-6. Enzymes used as liver function tests. 24-9. Clinical significance of serum b ilirubin level.
24-7. Albumin globulin ratio. 24-10. Fractional test meal.
24-8. van den Bergh test. 24-11 . Hyperchk>rhydria.

PART-2: MULTIPLE CHOICE QU STIONS

24•1. AH enzymes are elevated in obstructive liver dis- A. Hemolytic jaundice


ease, except: B. Obstructive jaundice
A. Gamma-glutamyltransferase (GGT} C. Defect in intestinal absorption
8 . 5' nucleotidase (NTP) D. Glommulonephritis
C. Alkaline phosphatase (ALP) 24-5. Conjugat1ed hyperbilirubinemia with raised alkaline
D. Lactate dehydrogenase (LOH) phosphatase levels are characteristic of:
24-2. All are features of obstructive jaundice, except: A. Obstructive jaundice
A. Increased level of conjugated bilirubin in blood
8 . Hemolytic jaundice
8 . Clay colored stools
C. Vira l h1epatitis
C. Presence of bile salts in urine
D. Physiological jaundice
D. Increased excretion of urobilinogen in urine
24-6. Gastric add secretion is stimulated by all the fol-
24-3. A patient w ith infective hepatitis is likely to have all
the following findings, except lowing, eJ(cept
A. Hyperbilirubinemia A. Hyperglycemia B. Gastrin
8. 8ilirubinuria C. Vagus D. Histamine
C. Absence of bile salts in urine 24-7. Hypoacidity is found in all the following conditions
D. Elevated AST except:
24-4. An increase in serum unconjugated bilirubin A. Pernicious anemia
occurs in : 8 . Carcinoma of stomach
Chapter 24: Liver and Gastric Function Tests 369

C . lnsulinoma 24-12. Which hormone has no effect on gastric acid sec-


D. Atrophic gastritis retion?
24-8. All the following biochemical parameters are indi- A. Cholecystokinin B. Secretin
ces of liver function , except: C. Gastrin D. Somatomedin
A. Bilirubin B. Cholesterol 24-13. Which of the following tests is not indicated in a
C. Albumin D. Creatinine patient with generalized edema?
24-9. All the following biochemical parameters are indi- A. Liver function tests
ces of liver function, except: B. Renal function tests
A. Bilirubin B. Cholesterol C. Thyroid function tests
C. Albumin D. Creatinine D. Pancreatic function tests
24-10. A patient with infective hepatitis is likely to have all 24-14. Which of the following biochemical findings does
the following findings, except: not agree with acute hepatic failure?
A. Hyperbilirubinemia A. Respiratory alkalosis
B. Bilirubinuria B. Hyperammonemia
C. Absence of bile salts in urine C. Lactic acidosis
D. Elevated AST D. Uremia
24.11. Which enzyme test is more specific for parenchymal 24-15. The laboratory data that is against a diagnosis of
(hepatocellular) liver disease? obstructive jaundice is:
A. Acid phosphatase (ACP) A. High alkaline phosphatse level
B. Alanine aminotransferase (ALT) B. Increased excretion of urobilinogen in urine
C. Lactate dehydrogenase (LDH) C. Elevated serum cholesterol level
D. Amylase D. Direct positive van den Bergh reaction

ANSWERS OF MULTIPLE CHOICE QUESTIONS


24-1. D 24-2. D 24-3. B 24-3. A 24-5. A 24-6. A 24-7. C
24-8. D 24.9. D 24.10. B 24.11. B 24-12. D 24-13. D 24-14. D
24-15. B

PART-3: VIVA VOCE QUESTIONS AND ANSWERS

24-1 . What is the normal serum bilirubin level? 24-10. What are important liver function tests?
0.2 to 1.0 mg/dl. Serum bilirubin, albumin, alkaline phosphatase; Urine
24-2. What is normal level of unconjugated bilirubin? bile salts, bile pigments and urobilinogen.
0.2-0.7 mg/dl. 24-11 . What are the features of obstructive jaundice?
24-3. What is normal level of conjugated bilirubin? Elevated conjugated bilirubin in plasma; presence of
Less than 0.4 mg/dL. bile salts and bile pigments in urine; increased alkaline
24-4. What is latent jaundice? phosphatase level in blood; increased cholesterol in
Serum bilirubin between 1 mg/dL and 2 mg/dl. blood.
24-5. Jaundice appears at what level of bilirubin? 24-12. Which tests will be positive in a urine of a patient
Jaundice appears if the serum bilirubin goes above with obstructive jaundice?
2 mg/dl. Gmelin's test, Hay's test, Fouchet's test.
24-6. How jaundice is classified? 24-13. What are the enzymes useful in diagnosis of liver
(i) Prehepatic or Hemolytic. (ii) Hepatocellular. (iii)
diseases?
Posthepatic or Obstructive jaundice.
Alanine aminotransferase; Alkaline phosphatase; Gam-
24-7. What type of bilirubin is present in hemolytic jaun-
ma-glutamyltransferase.
dice?
24-14. What is the characteristic laboratory finding in
Unconjugated bilirubin is increased in serum . Hence
chronic alcoholism?
van den Bergh test is indirect positive.
Elevation of gamma-glutamyltransferase level in serum.
24-8. What happens in obstructive jaundice?
In obstructive jaundice, conjugated bilirubin in serum is 24-15. What is alkaline tide?
elevated, and van den Bergh test is direct positive. When HCI is produced in stomach, bicarbonate level
24-9. What is the picture in hepatocellular jaundice? within the cell increases (formed from H2C03 ) , it is
In hepatocellular jaundice, a biphasic reaction is reabsorbed into blood stream. This would account for
observed, because both conjugated and unconjugated the alkaline tide of plasma and urine, immediately after
bilirubins are increased in serum. meals.
_ _ _ _ _Chapter 25
Kidney Function Tests

Chapter at a Glance
The learner will be able to answer questions on the follow ing topics:
0 Glomerular functions Clearance tests: lnulin, creatinine and urea
Tubular functions Proteinuria
Abnormal constituents of urine Tests for tubular functions

The major functions of the kidneys are to excrete meta- Glomerular Function
bolic waste products as well as to maintain water, pH, elec-
trolyte balance, production of calcitriol and erythropoietin When the blood is perfused through the Bowman's cap-
sule, an ultrafiltrate of the blood is produced in glome-
(Box 25.1 ). A decrease in kidney function is due to a reduc-
rulu s, while the cells and proteins are retained in the
tion in the performance of nephrons. The functional unit
blood. The sieves of the glomeruli are such that hemo-
of the kidney is the nephron, which is composed of the
globin (mol wt 67,000 D) is passed through to be excre-
Bowman's capsule with the glomerular tuft of capillaries,
ted in urine, while albumin (mol wt 69,000 D) is retained
the proximal convoluted tubule (PCT), loop of Henle,
in the blood. Therefore, the earliest manifestation of the
distal convoluted tubule (OCT) and collecting tubules.
abnormal fu nction of the glomeruli is the appearance of
IRENAL FUNCTION TESTS albumin in urine.

The classification of renal functional tests are shown in


Box 25.2. BOX 25.2: Class1f1cat1on of renal function tests
I. To screen for kidney disease
Complete urine analysis
BOX 25.1: Functions of kidney at a glance
Plasma urea and creatinine
1. Excretion of urea and other waste products, such as creati- Plasma electrolytes
nine, uric acid and metabolites of xenobiotics II. To assess renal function:
2. Maintaining water balance a. To assess glomerular function
3. Excretion of sodium (effect on BP) Glomerular filtration rate
4. Excretion of potassium (effect on heart) Clearance tests
5. Excretion of hydrogen ions (maintenance of pH) Glomerular permeability
6. Activation of vitamin D (effect on bone} Proteinuria
7. Production of erythropoietin (effect on RBCs} b. To assess tubular function
8. Filtration: 180 liters/day of water with all sodium, chloride, Reabsorption studies
sugar and amino acids Secretion tests
9. Reabsorption: 178.5 liters reabsorbed; all glucose and amino Concentration and dilution tests
acids reabsorbed; most of sodium and chloride reabsorbed. Renal acidification
Chapter 25: Kidney Function Tests 371

TABLE 25. 1: Handling of solutes by the renal tubules TABLE 25.2: Threshold value of some common substances
excreted through urine
Relative
Compound Mode of handling by tubules concentration Substance Threshold value plasma level
Creatinine Not reabsorbed; secreted in GF = Urine 1. Glucose 180 mg/dL
small amounts 2. Lactate 60 mg/dL
Uric acid 90% is first absorbed in PCT; GF ;:: Urine 3. Bicarbo nate 28 mEq/ L
but later secreted in OCT
4. Calcium 10 mg/dL
Urea About 40% rea bsorbed in GF > Urine
PCT
Sodium Partially reabsorbed GF > Urine Renal Threshold and Tubular Maximum
Glucose Completely reabsorbed GF >> Urine
Compounds whose excretion in urine are dependent
Amino acid Completely reabsorbed GF >> Urine
on blood level are known as threshold substances.
(PCT: Proximal convoluted tubules; OCT: Distal convo luted tubules)
At normal or low plasma levels, they are completely
reabsorbed and are not excreted in urine. But when the
blood level is elevated, the tubular reabsorptive capa-
TABLE 25.3: Main functions of kidney tubules
city is saturated, so that the excess will be excreted in
Segment ofnephron Reabsorption of Secretion of
urine (Table 25.2). The renal threshold of a substance is
Proximal Sodium (85%), H+,
convoluted Chloride (85%), Acids
the plasma level above which the compound is excreted
tubule (PCT) Bicarbonate (85%), and bases, in urine. For glucose, the renal threshold is 180 mg/dl.
Glucose (100%), NH;, In other words, glucose starts to appear in urine when
Amino acids (100%), Diodrast,
Uric acid, water (obligatory). PAH blood level is more than 180 mg/dl. Table 25.2 gives a
Loop of Henle Na+, Cl·, Ca++, Mg++
list of threshold substances. In abnormal conditions, the
renal threshold may be lowered so that even at lower
Distal convoluted Na',CI·, H+, K+, NH;,
tubule (OCT) Water (facultative) Uric acid blood levels, compounds are excreted in urine, e.g.
renal glucosuria (glucose); and renal tubular acidosis
(bicarbonate).
Glomerular Filtration Rate (GFR)
GFR is decreased when BP is below 80 mm of mercury. Reabsorption of Water
The GFR is reduced when there is obstruction to the The osmolality of urine can vary between 60- 1200
renal flow (calculi, enlarged prostate, etc.). It also mosmol/kg (specific gravity = 1.003 to 1.032), depend-
decreases with age. The renal blood flow is about 700 ml ing on the water intake and state of hydration. The GFR
of plasma or 1200 ml of blood per minute. The glome- is about 125 ml/min. In the proximal convoluted tubules,
rular filtration rate (GFR) is 120-125 ml per minute in a most of this is reabsorbed. Since Na+, c1- and HCO3-
person with 70 kg body weight. Glomerular filtrate formed ions are absorbed, water has to move along with the
is about 170 to 180 liters per day, out of which only solutes to maintain the osmolality. Hence, this is called
1.5 liters are excreted as urine. This means that most of obligatory reabsorption of water. By the time it reaches
the water content of glomerular fi ltrate is reabsorbed. the loop of Henle, the filtrate is only 25 ml/min. Here
sodium is again reabsorbed, but water absorption is less
Functions of the Tubules so that, urine, is hypotonic at this level.
By the time urine reaches distal tubules, the flow
When the glomerular filtrate is formed, it contains almost rate is reduced to 16 mUmin. Here again water is reab-
all the crystalloids of plasma. In the proximal convoluted sorbed, but it is under the influence of ADH. Therefore,
tubules, about 70% water, Na• and Cl- as well as 100% this is called facultative reabsorption of water. ADH
glucose, amino acids and K• are reabsorbed . Urea, phos- secretion, in turn, is controlled by hypothalamic osmo-
phate and calcium are partially absorbed. (Table 25.1 ). receptors. The osmolality of plasma is the stimulus for
Table 25.3 shows the functions of different parts of the modulating ADH secretion. Thus, when urine reaches
renal tubules. the collecting ducts, the flow rate is only about 1 ml/min,
372 Section C: Clinical and Applied Biochemistry

BOX 25.3: Clinical applications of d1ureltcs e appearance


1. Osmotic diuretics act by interferi ng with reabsorption of sol- Appearance ,!Significance
II
ute so that more water is obligatorily excreted along with the
solute. Osmotic diuretics mainly act at t he proximal convolut- 1. Clear Normal urine is straw colored
ed tubules, e.g. mannitol. 2. Cloudy/ Urine turns cloudy on standing due to
2. When carbonic anhydrase is inhibited, the dissociation Opalascent precipitation of phosphates on refrigeration;
of H2C03 to H,O and CO2 is not taking place. Net effect is Presence of pus causes cloudiness
decreased reabsorption of bicarbonate, sodium and water.
3. High color Concent rated urine, Oxidation of urobllinogen
Thus acetazolamide, a carbonic anhydrase inhibitor, will
to urobilin
cause diuresis.
3. The thiazide group of diuret ics act on distal convoluted 4. Yellow Bilirubinuria in jaundice; B-complex intake
tubules, inhibiting sodium reabsorption and therefore more
5. Smoky red Presence of blood
water is excreted obligatorily.
4. Frusemide acts on the ascending lim b of loop of Henle, 6. Brownish red Hemoglobinuria
inhibiting chloride reabsorption along with Na- and water. So, 7. Orange High levels of bilirubin; Rifampicin
chances of K• depletion are present.
5. Aldosterone antagon ist s (Spiranolactone) inhibit s sodium 8. Red Porphyria; Ingestion of red beet
reabsorption. 9. Black urine Alkaptonuria; Formic acid poisoning
6. In congestive cardiac failure, diuretics form an important
10. Milky urine Chyluria
part of therapy.

TABLE 25.S: Abnormal1t1es detected 1n d1pst1ck may refer the practical textbook. The routine analysis of
Test and normal range Interpretations urine is the most popular test in hospital practice. The
1. Specific gravity Low SG in renal tubular dysfunction; following param1:iters are usually checked when report-
1.005-1.025 diabetes insipidus; polydypsia. High ing on a urine sample.
SG in inadequate water intake; volume
depletion
Physical Characteristics of Urine •
2. pH 5.5-6.5 Low pH in high protein diet and acidosis,
recent meal-alkaline tide, high pH in low
protein diet
i. Volume: Th1:i average output of urine is about 1.5 liters
3. Blood Menstruation, t raumatic catheterizat ion,
per day. Urine volume may be increased in excess
glomerulonephritis, stones, tumor and water intake, diuretic therapy, diabetes mellitus and
trauma of urinary t ract, hemoglobinuria,
hemolysis
in chronic irenal diseases. Urine volume may be
4. Protein Fever, exercise, orthostatic proteinuria; decreased in excess sweating, dehydration, edema
<150 mg/day glomerulonephritis, urinary tract infection, of any etiolo~JY, kidney damage. Urine volume 1.5 U24 h;
tubu lar diseases
typical in health, oliguria < 400 ml, anuria < 100 ml,
5. Glucose Diabetes mellitus, renal glucosuria;
Fanconi's syndrome polyuria > 3,000 ml
6. Ketone bodies Diabetes mel litus, starvation ii. Specific gravity: Described under tests for tubu-
7. Bil irubin Hepatitis, obstructive jaundice lar function . A summary of the findings is listed in
B. Urobilinogen <4 Concentrated urine; hepatitis; intravascular Tables 25.4 and 25.5.
mg/day hemolysis; low in obstructive jaundice
9. Bile salt Obstructive jaundice Chemical Clharacteristi cs of Urine
10. Nitrite Urinary tract infection Reaction to Litmus
11. Leukocyte esterase Urinary tract infection, fever
The pH of urine varies from 5.5 to 7.5. If diet is rich in
proteins, sulfuric and phosphoric acids are produced
and the urine is hypertonic. Clinical applications of diu-
from amino acids, and the urine becomes acidic. If the
retics are shown in Box 25.3.
diet is rich in vegetables, urine is alkaline because the
organic acids (citric and tartari c) present in vegetables
ABNORMAL CONSTITUENTS OF
are converted to bicarbonate in the body.
URINE
In clinical biochemistry, urine is tested and report is Proteins
given on a urine sample. The procedure is called uri ne Proteinuria is an important index of renal diseases. In
analysis or urinalysis. For details of the tests, the student normal urine, protein concentration is very low, which
Chapter 25: Kidney Function Tests 373

TABLE 2S.6: Common tests to assess kidney function GFR


mUmln
Blood level or Urine Facrors affecting urinary
Constituent excretion excretion
Urea B = 15- 40 mg/dl Dietary proteins, Glomeruh
U =15-30 g/ day protein catabolism, Renal Proximal
blood flow convoluted
Creatinine B =0.7-1.4 mg/dl(M) GFR, tubular secretion, age, tubule
B = 0.4- 1.3 mg/ dl(F) U sex, muscle mass
=1- 2 g/dav Loop of Henle

Uric acid B = 3-7 mg/dl (M) Purine catabolism, tubular Distal


B = 2-5 mg/ dl (Fl excretion convoluted
U = 0.5- 0.8 g/day tubule
Sodium B = 135-142 mmol/L State of hydration, dietary
sodium, renal function Clearance lnulin Glucose Urea
mUmin 125 0 75
Potassium B = 3.5- 5 mmol/ L Dietary potassium, acid base
balance, renal function lnulin is neither absorbed nor secreted. Glucose is absorbed
com lately. Urea Is partially absorbed. Dlodrast Is active! secreted.
Calcium B = 9- 11 mg/dl Dietary calcium, PTH,
calcitonin, renal function Fig. 25.1: Tubules handle substances differently

cannot be detected by the usual tests. These proteins Bile Pigments


are secreted by the tubular epithelial cells. The proteinuria Bilirubin appears in urine during obstructive jaundice
is commonly assessed by the heat and acetic acid (see Chapters 22 and 24). It is detected by Fouchet's
test. Now dipstick test is replacing the old methods. test.
Microalbuminuria and proteinuria are described in detail
under markers of glomerular permeability. Urobilinogen
In hepatocellular jaundice, urobilinogen is absent in
Blood
urine. The earliest sign of recovery is the re-appearance
Hematuria is seen in nephritis and postrenal hemor- of urobilinogen in urine (see Chapter 24). It is identified
rhage. Hemoglobinuria is due to abnormal amount of by Ehrlich test In hemolytic jaundice excretion of uro-
hemolysis. Occultest tablets and Hemastix strips are bilinogen is increased and these tests will be frankly
available for rapid testing of blood in urine. positive.

Reducing Sugars (Glycosuria) Non-Protein Nitrogen (NPN)


Benedict's test may be used as a semiquantitative These include urea, creatinine and uric acid. These com-
method for sugar estimation in urine. Dipstick is now pounds are excreted through urine. In kidney dysfunc-
replacing the old Benedict's test for detection of glucose tion, the levels of these compounds are elevated in
in urine. plasma. Of the three, creatinine estimation is the
most specific and sensitive index of renal function.
Ketone Bodies Other minor components of NPN are urobilinogen,
They are acetoacetic acid , beta hydroxybutyric acid and indican, ammonia and amino acids.
acetone. Ketonuria is seen in diabetes mellitus, starva- Normal blood and urine levels of some of the impor-
tion, persistent vomiting, von Gierke's disease and in tant parameters are shown in Table 25.6.
alkalosis. Ketone bodies are analyzed by Rothera's test.
Nowadays, ketostix strips are available for rapid test MARKERS OF GLOMERULAR
for ketone bodies. I FILTRATION RATE
Bile Salts Clearance Tests
Bile salts are present in urine during the early phase of Measurement of the clearance is predominantly a test
obstructive jaundice (see Chapter 24). Their presence of glomerular filtration rate (GFR) (Fig. 25.1). Measure-
is identified by Hay's test. ment of glomerular filtration rate (GFR) provides the
374 Section C: Clinical and Applied Biochemistry

TABLE 25.7: Relationship of GFR with c learance


Example
BOX 25.4: Adva
nine clearance I
... • I • I ..
.
Mechanism Result
Substances filtered; GFR = clearance lnulin, Creatinine Advantages
neither reabsorbed nor 1. Extrarenal factors will rarely interfere.
excreted 2. Conversion of creatine phosphate to creatinlne is
Substance filtered; Clearance < GFR Urea spontaneous, non-enzymatic.
reabsorbed and excreted 3. As the production is continuous, the blood level will not
Substances filtered; Clearance> GFR Diodrast, PAH fluctuate. Blood may be collected at any time.
partially reabsorbed 4. It is not affected by diet or exercise.
Disadvantages
most useful general index for the assessment of the 1. Creatinine is filtered by glomeruli, and actively excreted by
the tubules. Of the total excretion, about 10% is by t ubular
severity of renal damage. A decrease in the renal func-
component. When the GFR is reduced, the secretion compo-
tion is due to the loss of functional nephrons, rather than nent is increas.ed, and will viciate the results. The creatinine
a decrease in the function of individual nephron. The clearance is said to overestimate GFR by about 10- 20 mUmin.
2. When GFR is severely reduced, extrarenal excretion increases.
relation between clearance value and GFR is shown in
Then the major route becomes the degradation by intestinal
Table 25. 7 and Figure 25.1. Substantial kidney damage bacterial flora.
occurs before GFR is decreased. 3. Very early stages of decrease in GFR may not be identified by
Normal GFR for young adults is 120-130 ml/min/ creatinine clearance (creatinine blind area).
4 . Other prerencil, renal and postrenal causes will influence
1.73 m2 • A decline with age is significant and more than creatinine clearance (see under urea clearance).
25% of people older than 70 years may have a GFR less
than 60 ml/min.
Creatinine Clearance Test
Definition Importance of Creatinine Clearance
Clearance is defined as the volume of blood or plasma
Creatinine is a waste product, formed from creatine
completely cleared of a substance per unit time. It is phosphate (see Chapter 18). This conversion is spon-
expressed as milliliter of plasma per minute (not as
taneous, non-e,nzymatic, and is dependent on total
g or mg). Clearance estimates the amount of plasma
muscle mass of the body. It is not affected by diet, age
that must have passed through the glomeruli per minute
or exercise. Women and children excrete less creatinine
with complete removal of that substance to account for
than men, because of their smaller muscle mass. About
the substance actually appearing in the urine.
98% of creatine pool is in muscle. Since the production
mg of substance excreted per minute is continuous, the blood level will not fluctuate much,
Cl earance = - - - ' ' - - - - - - - - - - - - ' - - - - -
mg of substance per ml of plasma making creatinine an ideal substance for clearance test.
Box 25.4 shows the advantages and disadvantages of
It is calculated by using the formula:
creatinine clearaince test.
uxv Since creati1nine excretion is a constant in a parti-
C = -p-
cular person , thH urine creatinine is sometimes used to
where U = concentration of the substance in urine; P = check whether the 24 hours urine sample does actually
concentration of the substance in plasma or serum and contain total urine volume or not. This is important when
V = the ml of urine excreted per minute. The value is urine is collected from children and mentally challenged
expressed as ml/minute. persons. In order to circumvent the difficulty of urine
If the substance is freely filtered across the capil- collection, nowadays it is customary to express urinary
lary wall, and neither secreted nor reabsorbed, then its concentration of other substances per gram of creati-
clearance is equal to glomerular filtration rate. A sub- nine rather than per 24 hours urine.
stance which meets these requirements is an ideal
filtration marker. If the substance is also secreted by the
Reference Values of Creatinine
tubules, the clearance exceeds GFR. For those which Adult males, 0. 7 - 1.4 mg/dl
are reabsorbed by tubules, clearance is less th an GFR Adult females, 0.6 - 1.3 mg/dl
(Fig. 25.1 ). Children, 0.4 - 1.2 mg/dl.
Chapter 25: Kidney Function Tests 375

TABLE 25.8: Normal reference values TABLE 25.9: Factors affecting serum creatinine
Serum crearinine GFR Factors reducing serum crearinine Factors increasing serum crearinine
Adult male 0.7 - 1.4 mg/ dL 95 - 115 ml/min Low muscle mass Old age
Adult female 0.6 - 1.3 mg/dL 85 - 110 ml/min Females Males
Children 0.5 - 1.2 mg/dl Malnutrition Renal diseases
Medicines Glomerulonephrltis
Thiazide Pyelonephritis
The kidney reserve is such that about 50% kidney
Vancomycin Renal failure
function must be lost before creatinine level in blood is
Urinary obstruction
raised. Serum level usually parallels the severity of the
Congestive cardiac failure
disease.
Dehydration. shock
Creatinine level more than 1.5 mgldL indicates
Medicines
impairment of renal function. Creatinine is quantitated by
Amphotericin B
Jaffe's test (alkaline picrate). Test kit based on specific
Cephalosporins
enzymatic reaction is also available.
Kanamycin
Reference values for creatinine level in blood and
the creatinine clearance are shown in Table 25.8. When
corrected for surface area, the creatinine clearance in healthy individuals, nor in children and obese people.
value will become comparable between males, females The accuracy of plasma creatinine estimation is a major
and children, which is about 100 mUmin/1.73 sq meter. deciding factor in both equations.

Interpretation of Creatinine Clearance Creatinine Coefficient


It is the urinary creatinine expressed in mg/kg body
A decreased creatinine clearance is a very sensitive
weight. The value is elevated in muscular dystrophy.
indicator of reduced glomerular filtration rate. Factors
Normal range is 20-28 mg/kg for males and 15--21 mg/
which will alter the creatinine level are indicated in Table
kg for females.
25.9. Clearance value up to 75% of the average normal
value may indicate adequate rena l function. In older
Cystatin C as a Filtration Marker
people, the clearance is decreased. The test is very
It is a marker which has advantages over serum creati-
helpful in long term monitoring of patients with renal
nine. Normal blood level of cystatin is 0.8 to 1.2 mg/L. It
insufficiency.
is expressed in virtually all organs of the body. It is the
most abundant extracellular Cysteine protease inhibitors.
Estimated GFR (eGFR)
Creatinine is the most widely used biomarker of
A simpler technique of estimating creatinine clearance kidney function. But sometimes, it is inaccurate in
and thereby GFR is by using serum creatinine level. detecting mild renal impairment. The tubular secre-
This would eliminate the need for timed urine collectior.::. tion contributes approximately 20% of the total creati-
A commonly used formula is Cockcroft-Gault equation. nine excretion by the kidney, and this contribution can
Ccr = (1 40 - age in years) x weight in Kg (0.85 in increase as GFR decreases. Serum creatinine does not
females)/72 x Per in mg/dl increase until the GFR has moderately decreased. This
The factor 0.85 is used in females assuming that insensitivity to moderate decreases in GFR is called
they have 15% less muscle mass. The issue of overes- creatinine blind GFR area (40- 70 mUmin/1.73 m 2 ).
timation cannot be eliminated by this calculation. So, serum creatinine may not be a good parameter for
A more recent equation used in the MORD (Modi- determination of GFR.
fication of Diet in Rena l Disease) study is more accu- On the contrary, Cystatin C is an excellent GFR
rate. This equation directly estimates GFR. marker. The blood levels are not depended on age, sex,
eGFR can be used for staging of patients with muscle mass or inflammatory processes. It is sensitive
chronic kidney disease. eGFR can only be used accu- to changes in the so-called creatinine blind area of GFR
rately in patients with chronic kidney disease and neither (40- 70 mUmin/ 1.73 m2) . So, serum level of cystatin is
376 Section C: Clinical and Applied Biochemistry

TABLE 25.1 0: Grading of chronic kidney disease TABLE 25.11 : K1dnP-y failure early symptoms
State Grade GFR mUmt/1.73 m' Polyu ria (passing more urine)
Minimal damage with normal GFR 1 >90 Nocturia (passing more urine during night) I
Mild damage with slightly low GFR 2 60- 89 Pedal edema, puffiness of face
Moderately low GFR 3 30-59 High blood pressure I
Severely low GFR 4 15-29 Unexplained anemia I
Kidney failure s < 15 Fatigue, lassitude and tiredness I

Microalbuminuria
a better test for kidney function (GFR) than serum Mild elevation of serum creatinine I

creatinine levels. Since, there is no tubular secretion of


Cystatin C, it is extremely sensitive to minor changes in minute. This is called maximum urea clearance and the
GFR in the earliest stages of chronic kidney diseases. normal value is found to be 75 ml/min.

Chronic Kidney Disease (CKD)


Standard Urea Clearance
GFR calculated from serum creatinine and MORD equa-
tion are used to grade CKD (Table 25.10). But the clearance value is decreased when V, the
CKD is a silent killer. CKD is a growing problem that volume of urine. is less than 2 ml/min. Then, it is called
affects approximately 12% of the adult population. Major standard urea clearance, where the normal value is
risk factors of CKD are diabetes mellitus, hypertension, found to be 54 ml/min, and is calculated as:
glomerular nephritis, urinary tract infection, autoimmune
diseases, kidney stones and toxic effects of some drugs. p
Markers for CKD are serum creatinine, eGFR, microal-
bumin and Cystatin C (Table 25.11 ). Interpretation of Urea Clearance Value
Acute Kidney Injury (AKI) If the value is below 75% of the normal, it is considered
to be abnormal. The values fall progressively with fail-
AKI is characterised by a rapid rise of serum creatinine
ing renal function. The clearance value may be abnor-
with low urine output. Onset of AKI can be swift and
mal even though the plasma urea values are within
often deadly. In acute kidney injury, serum creatinine
normal limits. The plasma urea values will start to rise
(sCr), can take days to show an increased level. Novel
only when the clearance value falls below 50% of the
biomarkers of tubular injury, such as urinary neutro-
phil gelatinase-associated lipocalin (uNGAl), may en- normal. Urea is normally reabsorbed from renal tubules
able the early detection of acute kidney injury before a and , therefore, tubular function also affects urea cle-
change in GFR is observed. The uNGAl is very useful arance. Hence, creatinine clearance test or Cystatin C
when sCr level is not increased . estimation are more preferred. Urea is freely filtered by
the glomerulus and passively reabsorbed in both PCT
Urea Clearance Test and OCT. Urea clearance is less than GFR (Fig. 25.1 ).
Importance of Urea Clearance
Blood Urea Level
The urea clearance is less than GFR, because urea is
partially reabsorbed (Fig. 25.1 ). Urea clearance is the Normal serum urea value is 20-40 mg/dl.
number of ml of blood, which contains the urea excre-
ted in a minute by kidneys. Interpretation of Blood Urea Value
Urea is the end-product of protein metabolism (see
Maximum Urea Clearance
Chapter 17). The serum concentration of urea generally
The urea clearance is calculated by the formula increases as the age advances. The lower range is usu-
u X V/P ally seen in young adults and the upper limit is normal
where U = mg of urea per ml of urine; P = mg of urea for elderly people. Therefore, a value of 40 mg/dl in a
per ml of plasma and V = ml of urine excreted per patient of 25 years may be considered as suspicious,
Chapter 25: Kidney Function Tests 377

BOX 25.5: Causes for increased blood urea involves administration of an extraneous compound, this
procedure is not used routinely.
1. Prerenal conditions:
Dehydration: Severe vomiting, intestinal obstruction, diarrhea lnulin clearance (GFR) = 125 mUmin and urea
Diabetic coma and severe burns clearance = 75 mUmin .
Fever and severe infections
inulin clearance - urea clearance 125-75 = 0 4
2. Renal diseases: 125 .
glomerular filtration rate
Acute glomerulonephritis
Nephrosis In other words, 40% of urea present in the glomeru-
Malignant hypertension lar filtrate is reabsorbed in the tubules.
Chronic pyelonephrltls
3. Postrenal causes:
Stones in the urinary tract
Diodrast Clearance
Enlarged prostate Diodrast is a contrast medium usually used in taking X-
Tumors of bladder
ray of urinary tract. Diodrast and PAH (para-amino hip-
4. Medications:
ACE inhibitors purate) are filtered and excreted, so that these substan-
Diuretics ces are removed by one passage of the blood through
NSAIDs kidney. PAH clearance is a measure of renal plasma
flow . It is about 700 ml of plasma or 1200 ml of blood
,vhile the same value in a person of 60 years can be per minute and is about 114th of the total cardiac output.
:onsidered as perfectly normal. Causes for increased Since, renal plasma flow is 700 mUmin and the
blood urea are enumerated in Box 25.5. GFR is 125 mUmin , it is obvious that about 115th of the
plasma brought to the glomeruli becomes the glomeru-
Renal Diseases lar filtrate. This is called the filtration fraction .

Serum urea is increased in all forms of kidney diseases.


In acute glomerulonephritis values may be as high as
300 mg/dl. In early stages of nephrosis, serum urea
may be normal, but in late stages serum urea increas-
I MARKERS OF GLOMERULAR
PERMEABILITY
The glomerulus acts as a selective filter of the blood
es along with decreasing renal function. In malignant
passing through its capillaries. Passage of macro-
hypertension and in chronic pyelonephritis, the values
molecules is restricted based on their charge, size and
may reach very high levels. Prerenal and postrenal
shape. Molecules smaller than 5 kD, such as urea, glu-
causes are shown in Box 25.5.
cose, creatinine and electrolytes are freely filtered by
the glomerulus. The glomerular membrane is such that
Decreased Blood Urea
albumin (mol wt 69 kD) is retained in the blood , but free
Urea concentration in serum may be low in late preg- hemoglobin (mol wt 67 kD) is filtered and excreted in
nancy, in starvation, in diet grossly deficient in proteins urine.
and in hepatic failure.

Proteinuria
Azotemia
It may be of the following types:
Increase in the blood levels of NPN is referred to as
azotemia and is the hallmark of kidney failure.
Glomerular Proteinuria
/nu/in Clearance Plasma proteins are absent in normal urine. When glo-
lnulin is a polysaccharide of fructose. It is not appre- meruli are damaged or diseased, they become more
ciably metabolized by the body. It is neither absorbed nor permeable and plasma proteins may appear in urine.
secreted by the tubules. Therefore, inulin clearance is a The smaller molecules of albumin pass through dam-
measure of GFR. The test needs continuous infusion of aged glomeruli more readily than the heavier globulins.
inulin so as to keep the plasma level adequate. Since it Albuminuria is always pathological.
378 Section C: Clinical and Applied Biochemistry

BOX 25.6: lnd1cat1ons for quant,tal1011 of proteinuna through normal glomeruli and, therefore, if it exists in
1. Diagnosis of nephrotic syndrome- Nephrotic syndrome is a free form (as in hemolytic conditions), hemoglobin can
triad of edema, hypoalbuminemia and proteinuria > 3g/day. appear in urine (hemoglobinuria). Similarly, myoglobi-
24 hours urine protein, creatinine clearance and sodium
should be measured for planning appropriate treatment. nuria is seen following crush injury of muscles.
2. Prognosis of progressive renal disease-It is a marker for asses- Yet another example is the Bence-Jones proteinu-
sing the progressive loss of renal function in renal disease;
diabet ic nephropathy, chronic glomerulonephritis, reflux ria. In about 20% cases of multiple myeloma (plasmacy-
nephropathy. Treatments that reduce proteinuria (like anti hy- toma), the light chains of immunoglobulins are produced
pertensive drugs) decrease rate of progression.
abnormally. Being of smaller molecular weight, they are
3. Diagnosis of early diabetic nephropathy-Early stages of dia-
betic nephropathy are characterized by increase in GFR, mi- excreted in urine. These are called Bence-Jones Pro-
croalbuminuria and hypertension. teins (monoclonal light chains produced by plasmacy-
tomas) (Chapter 46). When the urine is heated, at 45°C
Overnight first voided sample (early morning urine- they start precipitating, at 60°C there is maximum preci-
EMU) may be used for the measurement of protein. pitation , at 80°C these proteins start redissolving , and
Detection limit with Dipstick is 200- 300 mg/L. will form a clear solution at 100°C. The precipitate reforms
300 mg/day = Benign proteinuria on cooling. It is also detected by immunoprecipitation.
300 mg - 1000 mg = Pathological proteinuria
> 1000 mg/day = Glomerular proteinuria
Tubular Proteinuria
Indications for quantitation are shown in Box 25.6. The tubular reabsorption mechanism is impaired, so
low molecular weight proteins appear in urine. They are
Microalbuminuria retinol binding protein (RBP) and alpha-1-microglobulin.
The condition is also called minimal albuminuria or
Urogenic Proteinuria
paucialbuminuria. It is identified, when small quantity of
albumin (30-300 mg/day) is seen in urine. The test is This is due to inflammation of lower urinary tract, when
not indicated in patients with overt proteinuria (+ve dip- proteins are secreted into the tract.
stick). Early morning midstream sample is preferred.
Microalbuminuria is an early indication of nephropa- ITESTS FOR TUBULAR FUNCTION
thy in patients with diabetes mellitus and hypertension.
Specific Gravity of Urine
Hence, all patients who are known diabetics and hyper-
tensive should be screened for microalbuminuria. It is an Normal specific gravity of urine is 1.015-1 .025. Theo-
early indicator of onset of nephropathy. The test should retical extremes are 1.003 to 1.032. The specific gravity
be done atleast once in an year. will be decreased in excessive water intake, in chronic
nephritis and in diabetes insipidus. It is increased in
It is expressed as albumin-creatinine ratio; normal
diabetes mellitus, in nephrosis and in excessive per-
ratio being,
spiration . In chronic renal failure, the specific gravity of
Males < 23 mg/g of creatinine
urine is fixed at 1010. The earliest manifestation of renal
Females < 32 mg/g of creatinine
damage may be the inability to produce concentrated
Patients showing hig her values on more than one urine.
occasion are considered to have microalbuminuria. The simplest test of tubular function is the measure-
Confirmed by overnight urine collection and calculation ment of the specific gravity (SG) of urine. This is an indi-
of albumin excretion rate. A value more than 20 microg/ cation of osmolality.
min confirms microalbuminuria. Specific gravity depends on the concentration of
solutes, whereas osmolality depends on the number of
Overflow Proteinuria osmotically active particles. Hence, in cases of protein-
uria, the specific gravity is elevated considerably, but
When small molecular weight proteins are increased osmolality is only mildly elevated.
in blood, they overflow into urine. For example, hemo- The inability to excrete the waste products may
globin having a molecular weight of 67,000 can pass be counterbalanced by large urine output. Thus the
Chapter 25: Kidney Function Tests 379

earliest manifestation of renal disease may be diffi- TABLE 25.12: Summary of renal function tests
culty in concentrating the urine. Glomerular dysfunction Tubular dysfunction
Serum urea t Urine concentration ..i.
Measurement of Osmolality Serum creatinine t Dilution test Abnormal
The osmolality of urine samples vary widely from 60 lnulin clearance .!, Uric acid excretion ..i.
mosmol/kg to 1200 mosmol/kg. A random urine sample Creatinine clearance ..i. Blood uric acid t
may have an osmolality around 600 mosmols/kg and Urea clearance ..i.
it increases to 850 after 12 hours fluid restriction. The PAH clearance ..i. Acidification of ..i.
urine
normal value of plasma osmolality is 285- 300 mOsm/kg.
Proteinuria Present Aminoaciduria Present

Concentration Test Urine volume ..i. Urine volum e t


Specific gravity t Specific gravity ..i.
Specific gravity is measured after a 12 hr fast. If the
specific gravity is more than 1.022, the patient has A summary of renal function tests is shown in Table
adequate renal function. In normal persons, the SG may 25.12.
be as high as 1.032. As the disease progresses the
urine specific gravity is fixed at and around 1.010 (300 e
mOsmol/kg)). It is then called isosthenuria. The mea-
.fi. Clinical Case Study 25.1
surement of the volume of urine excreted during the day A 55-year-old hypertensive complaints of dry, painful
and the night is another simple index of tubular function. eyes and xerostomia for a few weeks. On examination,
Normally, night volume is only half of the day volume. he was afebrile, BP 140/94 mm Hg, pulse 80/min. Facial
But an increased excretion of urine at night or nocturia and lower limb edema was present. Investigation results
is an early indication of tubular dysfunction. are - Fasting blood sugar 280 mg/dl, urine Benedict's
test red precipitate, urine proteins +++, serum albumin
Dilution Tests 2.0 g/dl, serum cholesterol 280 mg/dl, serum creatinine
Bladder is emptied at 7 AM and a water load is given 2.0 mg/dl, blood urea 120 mg/dl. Serum immunoglobu-
(1200 ml over the next 30 minutes). Hourly urine sam- lins were normal, tests for hepatitis antigens were nega-
ples are collected for the next 4 hours separately. Volume, tive, culture for bacteria and fungi revealed no growth.
specific gravity and osmolality of each sample are mea- Serum electrophoresis showed increased a-2 band.
sured. A normal person will excrete almost all the water What is the possible diagnosis?
load within 4 hours and the specific gravity of at least
one sample should fall to 1.003. The test is more sensi- e
tive and less harmful than concentration test. -i i- Clinical Case Study 25.2
A 50-year-old patient was admitted for treatment of sore
Urinary Acidification throat and pneumonia. He had poorly controlled diabe-
The most useful test is acid loading test. It is indicated tes mellitus and on admission blood urea was 140 mg/dl
in unexplained hyperchloremic metabolic acidosis. Give and serum creatinine was 2.8 mg/dl. He received 2.0 l
ammonium chloride at a dose of 0.1 g/kg body weight. fluid, but blood urea rose to 160 mg/dl and serum creati-
The ammonium chloride (NH 4CI) is dissociated into NH/ nine to 3.0 mg/dl. Urine output which was initially good
and c1-. In the liver, the NH/ is immediately converted dropped to 500 ml over a 24 hours period. Next day,
into urea. Therefore, c1- ions are counter balanced by he developed shortness of breath and lower extremity
W to produce HCI, a powerful acid . It is then excreted edema. Blood urea rose to 300 mg/dl and serum creati-
through urine so as to produce acidification. Urine is nine to 6.3 mg/dl. What is the probable diagnosis?
collected hourly, from 2 to 8 hours after ingestion. The
e
pH and acid excretion of each sample is noted. At least
one sample should have a pH of 5.3 or less. In renal
•ii- Clinical Case Study 25.3
tubular acidosis, the pH 5.3 is not achieved. Liver disease A 50-year-old man was admitted with loss of appetite,
is a contraindication to perform this test. nausea, vomiting, difficulty in breathing and fatigue. History
380 Section C: Clinical and Applied Biochemistry

revealed that he had similar symptoms 5 years back and disturbances are clinical features of ARF. Causes for
was diagnosed with hypertension and kidney failure. On ARF are usually classified as prerenal, instrinsic and pos-
examination, temperature was 36.8°C, respiratory rate trenal causes. Risk factors for ARF include hypertension,
was 22/min , pulse rate 64/min, BP was 170/100 mm Hg, congestive heart failure, diabetes, multiple myeloma,
marked pallor was present, chest and lungs showed bi- chronic infection and myeloproliferative diseases.
lateral basal rales, abdomen was soft, flat and tender. Laboratory investigations include blood urea (or
No other abnormality was detected. Patient was an BUN, blood urea nitrogen), serum creatinine, myo-
occasional alcoholic, and a chronic smoker. globin, free hemoglobin, uric acid, serum electrolytes
Laboratory investigations showed-Blood urea (hyponatremia, hyperkalemia, hyperphosphatemia ,
65 mg/dl, serum creatinine 2.4 mg/dl, serum calcium 6.4 hypocalcemia, hyper-magnesemia and metabolic acido-
mg/dl, serum potassium 4.9 mg/dl, and serum sodium
sis are seen in ARF), serological tests for antinuclear anti-
139 mmol/L. Urine examination results were - Color straw
bodies (ANA}, hepatitis, antistreptolysin-O (ASO), cross
colored, pH 5.0, specific gravity 1.020, appearance turbid,
reacting antibody produced by streptococci and comple-
volume 900 mU24 h, albumin 3+, sugar negative, pus
ments, urine analysis (casts, myoglobin, hemoglobin,
cells 1-3/HPF, RBC - 1-2/HPF, and epithelial cells rare.
proteins, RBC and WBC or their casts, eosinophils, uric
What is the probable diagnosis?
acid, calcium oxalate crystals) and urine electrolytes.
Renal biopsy is indicated in patients in whom prerenal
•ii• Clinical Case Study 25.1 Answer and postrenal causes have been ruled out and cause for
Proteinuria, hypoalbuminemia and hypercholesterolemia instrinsic ARF is unclear.
along with renal failure and edema are the classi-
cal presentation of nephrotic syndrome (nephrosis). •• Clinical Case Study 25.3 Answer
Patient has diabetes mellitus and hypertension, and these
The patient is suflfering from chronic renal failure (CRF),
may have caused nephrotic syndrome. Confirmation of
also known as chronic kidney disease (CKD). The history
diagnosis can be done by renal biopsy. Hypercoagula-
of hypertension and kidney failure goes against the
ble state, hypovitaminosis D and immunodeficiency are diagnosis of ARF.
associated features seen.
Decreased n:inal function interferes with kidneys'
Causes of nephrotic syndrome are; (1) Primary - ability to maintain fluid and electrolyte balance. The abi-
minimal change nephropathy, focal glomerulosclerosis, lity to concentrate urine declines early and is followed
membranous nephropathy, hereditary nephropathies,
by inability to excrete phosphate, acids and potassium.
and (2) Secondary - Diabetes mellitus, lupus erythema- In advanced stagies, urine cannot be diluted and urine
tosus, amyloidosis, paraproteinemia viral infections, like osmolality is "fixeid". Plasma levels of urea and creati-
hepatitis B, C, HIV, etc., pre-eclampsia, vasculitis, drugs. nine are elevated rapidly, and abnormalities of calcium,
Treatment is based on treating protein loss, edema, phosphate, PTH, vitamin D, and renal osteodystrophy
hyperlipidemia, hypercoagulable state, associated are seen. Moderate acidosis is seen. Anemia is
nutritional deficiencies and protection from infections. normochromic-normocytic in nature. CKD is rarely rever-
Prognosis depends on cause, person's age and type and sible and leads to progressive decline in renal function.
degree of renal damage caused at the time of initiating Diagnostic tests are electrolytes, blood urea nitrogen
treatment. (BUN) or blood urea, creatinine, phosphate, calcium,
CBC and urinalysis. The definitive diagnostic tool is
• · Clinical Case Study 25.2 Answer renal biopsy, but 1is not recommended when ultrasound
The patient is suffering from acute renal failure (ARF), indicates small, fibrotic kidneys.
also referred to as acute kidney injury (AKI). A brief Treatment includes: (1) Control of underlying dis-
history of bacterial infection, rising urea and creatinine eases, (2) Restriction of dietary protein, phosphate and
values, oliguria, edema and failed response to diuretics potassium, (3) Vitamin D supplements, (4) Treatment
all point towards with diagnosis. Retention of nitroge- of anemia and heart failure, and (5) dialysis for severe
nous waste products, oliguria, electrolyte and acid-base decrease in renal function.
Chapter 25: Kidney Function Tests 381

[ LEARNING POINTS, CHAPTER 2L 7. Non-protein Nitrogen includes urea, creatinine and


uric acid. Minor components of NPN are urobilino-
1. The GFR of a person with 70 kg body weight is gen, indican, ammonia and amino acids.
120-125 ml per minute. 8. Clearance is defined as the quantity of blood or
2. Acetazolamide, a carbonic anhydrase inhibitor, plasma completely cleared of a substance per unit
will cause diuresis by decreasing reabsorption of time and is expressed as milliliter per minute.
bicarbonate, sodium and water. 9. lnulin is neither absorbed nor secreted by the tubules.
3. The glomeruli of kidney are not permeable to sub- Therefore, inulin clearance is a measure of GFR.
stances with molecular weight more than 69,000. 10. Creatinine coefficient is the urinary creatinine expres-
4. Large quantities (upto a few gram per day) of albu- sed in mg/kg body weight. The value is elevated in
min are lost in urine in nephrosis. muscular dystrophy. Normal range is 20-28 mg/kg
5. Microalbuminuria is seen as a complication of for males and 15-21 mg/kg for females.
diabetes mellitus and hypertension. 11. Maximum urea clearance is found to be 75 ml/min
6. Ketonuria may be detected by Rothera's test. in normals.

PART-1 : ESSAY AND SHORT NOTE QUESTIONS


25-1. Name the renal clearance tests. Give details of any one of them.
25-2. How creatinine clearance test is done? What is its diagnostic significance?
25-3. How urea clearance test is done? What is its clinical significance?

SHORT NOTE QUESTIONS

25-4. Glomerular filtration rate. 25-8. Creatinine clearance test.


25-5. Urine analysis 25-9. Creatinine coefficient.
25-6. Proteinuria 25-10. Urea clearance test.
25-7. lnulin clearance test.

PART-2: MULTIPLE CHOICE QUESTIONS

25-1. Normal specific gravity of urine is: 25-6. Renal plasma flow is measured by:
A. 1.003 to 1.010 B. 1.010 to 1.015 A. Creatinine clearance
C. 1.015to1 .025 D. 1.025 to 1.035 B. lnulin clearance
25-2. Polyuria is a characteristic feature of all the follow- C. Para amino hippurate clearance
ing, except D. Urine output
25-7. A patient with chronic renal failure will excrete:
A. Diabetes mellitus B. Glomerulo nephritis
A. Acid urine
C. Diabetes insipidus D. Chronic renal failure
B. Hypertonic urine
25-3. Specific gravity of urine increases in:
C. Urine with specific gravity more than 1018
A. Chronic glomerulonephritis
D. Urine with low creatinine concentation
B. Diabetes mellitus
25-8. Which indicates an abnormal renal function?
C. Liver diseases
A. Blood urea 30 mg/di
D. Intake of vegetables =
B. GFR 125 ml/min
25-4. Proteinuria is seen in all the following conditions, C. Serum creatinine 8 mg/di
except: =
D. Urine pH 6.8
A. Acute glomerulo nephritis 25-9. Urea clearance is lowered in:
B. Diabetes insipidus A. Acute nephritis
C. Pyelonephritis B. Chronic liver failure
D. Multiple myeloma C. Chronic cardiac failure
25-5. Which of the following compounds have a clea- D. Benign hypertension
ranee value higher than GFR? 25-10. Normal value of blood urea is:
A. Uric acid B. Diodrast A. 3-4 mg/100 ml B. 4-8 mg/100 ml
C. Urea D. lnulin C. 8-16 mg/100 ml D. 20-40 mg/100 ml
382 Section C: Clinical and Applied Biochemistry
25-11 . Failure of concentrating capacity of urine is as- A. Ehrlich's test
sessed by: B. Rothera's test
A. Urea clearance C. Jaffe's test
B. Water deprivation test D. Schlesinger's test
C. Specific gravity of urine after giving water 25-23. Which of the following constituents of urine does
D. Creatinine level in urine not contain nitrogen?
25-12. Urine of a patient with obstructive jaundice will A. Urobilinogen B. Uric acid
give a positive test for all the following, except C. Aceto acetic acid D. Urea
A. Gmelin's test B. Hay's test 25-24. To calculate the creatinine clearance, all the fol-
C. Fouchet's test D. Benzidine test lowing data are required, except
25-13. Urine will have abnormal color in all the following A. Plasma creatinine level
conditions, except B. Patient's height and weight
A. Melanuria B. Porphyria C. Daily urine output
C. Creatinuria D. Alkaptonuria D. Specific gravity of urine
25-14. All the following statements regarding glomerular 25-25. Urea level in blood may be elevated in all the fol-
filtration is correct, except: lowing conditions, except:
A. GFR is approximately equal to endogenous creati- A. Dehydration B. Renal failure
nine clearance C. Hepatic failure D. Hypertension
B. Measurement of GFR is a test of renal tubular 25-26. Renal stones may contain all the following organ-
function ic constituents , except:
C. Glomerular filtrate has a specific gravity of 101 O A. Cystine B. Uric acid
D. Glomerular filtrate is formed by ultrafiltration of C. Cholesterol D. Xanthine
plasma 25-27. All the following laboratory data are suggestive of
25-15. Clearance values of all the following compounds acute renal failure, except
are used to test renal function , except: A. Plasma sodium - 150 mmol/L
A. Creatinine B. lnulin B. Plasma potassium - 5.6 mmol/L
C. Urea - 220 mg/di
C. Glucose D. Urea
D. Creatinine - 3.2 mg/di
25-16. When 15N (radio-active nitrogen) labelled glycine
25-28. Which ion is decreased in renal failure :
is given, the label will appear in all the following
A. Phosphate
compounds in urine, excep t
B. Potassium
A. Urobilinogen B. Urea
C. Bicarbonate
C. Urocanic acid D. Uric acid
D. Hydrogen ions
25-17. Normal urine contains all following, except
25-29. Renal tubular function can be assessed by:
A. Glucose B. Ethereal sulfate
A. Urea clearance
C. Bicarbonate D. Creatinine
B. Plasma electrolytes
25-18. Opalescent urine is seen in?
C. Specific gravity of early morning sample of urine
A. Porphyria B. Alkaptonuria
D. Plasma urea
C. Chyluria D. Creatinuria
25-30. Plasma urea is increased but creatinine is normal
25-19. Which anions are not normally present in urine?
in all the following, except:
A. Phosphate B. Sulfate
A. High protein intake
C. Chloride D. Proteinate B. Gastrointestinal bleeding
25-20. All are functions of the kidney, except C. Starvation
A. Excretion of hydrogen ions D. Muscle wasting
B . Detoxification of alcohol
25-31 . Which of the following cannot pass through the
C. Stimulation of erythropoiesis normal glomerular membrane?
D. Formation of 1,25 DHCC A. Creatinine B. Albumin
25-21 . A patient with chronic renal failure may have all the C. Myoglobin D. Uric acid
following features , except: 25-32. Which substance is not normally present in urine?
A. Urine with fixed specific gravity of 101 0 A. Creatinine B. Glucose
B. Protein creatinine ratio more than 3 C. Uric acid D. Urobilinogen
C. Positive heat and acetic acid test 25-33. Excretion of which substance is not under control
D. Nocturnal polyuria of hormones?
25-22. All the following tests are answered by normal A. Calcium
urine, except: B. Potassium
Chapter 25: Kidney Function Tests 383

C. Bicarbonate A . Albumin B. Transferrin


D. Sodium C. Hemo~1lobin D. Bence-Jones proteins
25-34. The solute present in highest concentration in 25-36. The level of creatinine in urine is influenced by:
urine is: A. Protein content of diet
A. Sodium B. Chloride B. Rate of cellular turn over
C. Creatinine D. Urea C. Muscle mass
25-35. The first protein to be excreted in urine in glome- D. Patency of urinary tract
rular damage is:

ANSWERS OF MULTIPLE CHOICE UESTIONS

25-1. C 25-2. B 25-3. B 25-4. B 25-S. B 25-6. C 25-7. D


25-8. C 25-9. A 25-10. D 25-11 . B 25-1,!. D 25-13. C 25-14. B
25-15. C 25-16. C 25-17. A 25-18. C 25-Hl. D 25-20. B 25-21 . C
25-22. B 25-23. C 25-24. D 25-25. C 25-21>. C 25-27. A 25-28. C
25-29. C 25-30. D 25-31 . B 25-32. B 25-31J. C 25-34. D 25-35. A
25-36. C

PART-3: VIVA VOCE QUESTIONS AN ANSWERS

25-1. What is glomerular filtration rate? expressed as milliliter per minute. It is the ml of plasma
120-125 ml per minute. which contains the amount of that substance excreted
25-2. What is the function of glomerulus? by the kidney within a minute.
An ultrafiltrate of the blod is produced in glomerulus, 25-13. What is the best method for assessing glomerular
while the cells and proteins are retained in the blood. filtration rate?
25-3. What is the normal specific gravity of urine? Creatinine clearance test.
1.015 to 1.025. 25-14. What is the advantage of creatinine test?
25-4. What is the minimum and maximum specific gra- Creatinine is formed spontaneously (non-enzymatic),
vity of urine? so the blood level and excretion rate of creatinine is a
1.003 to 1.032. constant.
25-5. What is the specific gravity of urine in chronic 25-15. What is the significance of creatinine clearance?
renal failure? A decreased creatinine clearance is a very sensitive
Fixed to 1.010. indicator of a red uced glomerular filtration rate.
25-6. Specific gravity of urine increased when? 25-1 6. What is the normal creatinine level in blood?
Diabetes mellitus and acute glomerulonephritis. For adult males, 0.7-1.4 mg/di; for adult females,
25-7. Polyuria is seen in which conditions?
0.6-1.3 mg/di.
Diabetes mellitus, diabetes insipidus, chronic renal failure.
25-17. What is creatinine coefficient?
25-8. Proteinuria is seen in which conditions?
It is the utrinary creatinine expressed in mg/kg body
Acute glomerulonephritis, nephrotic syndrome, pyelo-
weight.
nephritis, multiple myeloma.
25-18. Urea clearance is lowered in which condition?
25-9. What is the test to detect urinary protein?
Chronic liver failure.
Heat and acetic acid test.
25-19. What is tlhe normal blood urea level?
25-10. What is microa lbuminuria?
When small quantities of albumin (30-300 mg/day) is 20-40 mg/di
seen in urine.
25-20. What is tlhe clinical significance of urea level?
25-1 1. What are Non-protein nitrogen (NPN)? Urea level is increased in renal failure.

Urea, uric acid, creatinine. In kidney diseases, the lev- 25-21 . How is tubular concentrating capacity measured?
els of these substances in blood are increased. By measutring urine specific gravity.
25-12. What is meant by renal clearance? 25-22. What are the tests to assess tubular function?
Clearance is defined as the quantity of blood or plasma Measurement of specific gravity, concentration test,
completely cleared of a substance per unit time and is dilution test. acidification test.
_ _ _ _ _Chapter 26
Plasma Proteins

Chapter at a Glance
The learner will be able to answer questions on the following topics:
0 Plasma proteins Transport proteins in blood
O Electrophoresis Acute phase proteins in blood
0 Albumin, functions, clinical significance Ceruloplasmin
0 Hypoalbuminemia O Alpha-1 antitrypsin
0 Globulins, alpha, beta, gamma Clotting factors

Total blood volume is about 4 .5 to 5 liters in adult human


being. If blood is mixed with an anticoagulant and centri-
LELECTROPHORESIS
In clinical laboratory, electrophoresis is employed regu-
fuged , the cell components (RSC and WBC) are precipi-
larly for separation of serum proteins. The term electro-
tated. The supernatant is called plasma. About 55-60%
phoresis refers to the movement of charged particles
of blood is made up of plasma.
through an electrolyte when subjected to an electric
If blood is withdrawn without anticoagulant and
field. The details are given in Chapter 31 . Normal and
allowed to clot, after about 2 hours liquid portion is sepa-
abnormal electrophoretic patterns are shown in Figures
rated from the clot. This defibrinated plasma is called
26.1A and B and 26.2.
serum, which lacks coagulation factors including pro-
thrombin and fibrinogen. Total protein content of nor- Normal Patterns and Interpretations
mal plasma is 6 to 8 g/100 ml. The plasma proteins
i. In agar gel electrophoresis, normal serum is sepa-
consist of albumin (3.5 to 5 g/dl), globulins (2.5-3.5
rated into 5 bands. Their relative concentrations are
g/dl) and fibrinogen (200-400 mg/dl). The albumin: given below:
globulin ratio is usually between 1.2: 1 and 1.5: 1. Almost
Albumin 55-65%
all plasma proteins, except immunoglobulins are synthe- Alpha-1 globulin 2-4%
sized in liver. Plasma proteins are generally synthesized Alpha-2 globulin 6-12%
on membrane-bound polyribosomes. Most plasma pro- Beta globulin 8-12%
teins are glycoproteins. In laboratory, separation can be Gamma globulin 12-22%
done by salts. Thus, fibrinogen is precipitated by 10% ii. Albumin has the maximum and gamma globulin
and globulins by 22% concentration of sodium sulfate. has the minimum mobility in the electrical field.
Ammonium sulfate will precipitate globulins at half satu- iii. Gamma globulins contain the antibodies (immuno-
ration and albumin at full saturation. Total proteins in globulins). Most of the alpha-1 fraction is made up
serum of patients are estimated by Biuret method of alpha-1 antitrypsin. Alpha-2 band is mainly made
(Chapter 4 ). Albumin is quantitated by Bromo cresol up by alpha-2 macroglobu lin. Beta fraction contains
green (BCG) method . low density lipoproteins.
Chapter 26: Plasma Proteins 385

Point of application
! 13

[JO 00 0 · ,
y a2 a1 Albumin -ve 2 3 4 5 6 7 8 9 10
Start point

Gamma

laJ~D oo o I· , Beta

Alpha-2

-I DO OO I I· ,
Alpha-1
Albumin
+ve

-IOo ID D I· ·
terminal
Lanes 2,4,8, 10 = Normal pattern. Lane 1 = Nephrotic syndrome;
hypoalbuminemia, prominent alpha-2 band. Lane 3 = Cirrhosi~;

o o oo o I· ,
hypoalbuminemia with beta-gamma bridging. Lane 5 = Chrome
infection, broad based increase in gamma region; general Increase In
alpha-1 and alpha-2 bands, comparative reduction of albumin. Lanes
6,7 = Multiple myeloma; monoclonal ba~d (M-band) betwee~ beta

-I and gamma. Lane 9 = Acute Innammat1on; reduced albumin and


increased alpha-2fraction.

I om 00 I I·
Fig. 26.1B: Serum electrophoretic patterns

6 6. Cirrhosis of liver: Albumin synthesis by liver is


decreased, with a compensatory excess synthe-
Fig. 26.1 A: Serum electrophoretic patterns. 1 = Normal pattern; sis of globulins by reticuloendothelial system. So
2 = Multiple myeloma (M-band) between p and y region; 3 = Chron- albumin band will be thin, with a wide beta fraction;
ic infection. broad based increase in y region; general increase in sometimes beta and gamma fractions are fused .
a 1 and a2 bands; 4 = Nephrolic syndrome; hypoalbuminemia;
7. Chronic lymphatic leukemia, gamma globulin
prominent a2 band; 5 = Cirrhosis of liver; decreased albumin; 6 =
Plasma showing fibrinogen (normal condition). This may be mis- fraction is reduced.
taken for paraproteins 8. Alpha-1 antitrypsin deficiency: The alpha-1 band
is thin or even missing.
Abnormal Patterns in Clinical Diseases
Various abnormalities can be identified in the electro- ALBUMIN
phoretic pattern (Figs. 26.1A and B).
The name is derived from the white precipitate formed
1. Chronic infections: The gamma globulins are
when egg is boiled (Latin, albus = white). Albumin con-
increased, but the increase is smooth and wide-
stitutes the major part of plasma proteins. It is synthe-
based.
sized by hepatocytes; therefore, estimation of albumin
2. Multiple myeloma: In paraproteinemias, a sharp
is a liver function test (see Chapter 24). Albumin is
spike is noted and is termed as M-band. This is due
synthesized as a precursor, and the signal peptide is
to monoclonal origin of immunoglobulins in multiple
removed as it passes through endoplasmic reticulum.
myeloma (Fig . 26.2).
Albumin can come out of vascular compartment. So
3. Fibrinogen : Instead of serum, if plasma is used for
albumin is present in CSF and interstitial flu id. Half-life of
electrophoresis, the fibrinogen will form a prominent
albumin is about 20 days. Liver produces about 12 g of
band in the gamma region, which may be confused
albumin per day, representing about 25% of total hepatic
with the M-band.
protein synthesis.
4. Primary immune deficiency: The gamma globulin
fraction is reduced. Functions of Albumin
5. Nephrotic syndrome: All proteins except very big
molecules are lost through urine, and so alpha-2
Colloid Osmotic Pressure of Plasma
fraction (containing macroglobulin) will be very pro- The total osmolality of serum is 278-305 mosmol/kg
minent. (about 5000 mm of Hg). But this is produced mainly
386 Section C: Clinical and Applied Biochemistry

Multiple myeloma
Normal pattern Inflammatory
Monoclonal band
I I I

Fig. 26.2: Normal and abnormal electrophoretic patterns

by salts, which can pass easily from intravascular to Extravascular space


extravascular space. Therefore, the osmotic pressure BP 35 mm Hg
Osmotic pressure
25 mm Hg
exerted by electrolytes inside and outside the vascular
compartments will cancel each other. But proteins can-
not easily escape out of blood vessels, and therefore,
proteins exert the "effective osmotic pressure". It is
about 25 mm Hg, and 80% of it is contributed by albu-
min. The maintenance of blood volume is dependent on
this effective osmotic pressure.
10 mm Hg
pressure out
10mmHg
pressure
sucked in
r
Arterial end
According to Starling's hypothesis, at the capillary of capillary Venous end
end, the blood pressure (BP) or hydrostatic pressure Fig. 26.3: Starling hypothesis
expels water out, and effective osmotic pressure (EOP)
takes water into the vascular compartment (Fig. 26.3). iii. Hormones: Steroid hormones, thyroxine.
At arterial end of the capillary, BP is 35 mm Hg and EOP iv. Metals: Albumin transports copper. Calcium and
is 25 mm; thus water is expelled by a pressure of 10 mm heavy metals are non-specifically carried by albu-
Hg. At the venous end of the capillary, EOP is 25 mm min. Only the unbound fraction of drugs is biologi-
and BP is 15 mm, and therefore water is imbibed with cally active.
a pressure of 10 mm. Thus, the number of water mole-
cules escaping out at arterial side will be exactly equal Buffering Action
to those returned at the venous side and therefore blood
All proteins have buffering capacity. Because of its high
volume remains the same.
concentration in blood, albumin has maximum buffering
If protein concentration in serum is reduced, the
capacity (see Chapter 27). Albumin has a total of 16 his-
EOP is correspondingly decreased. Then return of water
tidine residues which contribute to this buffering action.
into blood vessels is diminished, leading to accumula-
tion of water in tissues. This is called edema. Edema
Nutritional Function
is seen in conditions where albumin level in blood is less
than 2 g/dl (see hypoalbuminemia). Albumin may be considered as the transport form of
essential amino acids from liver to extrahepatic cells.
Transport Function
Albumin is the carrier of various hydrophobic substan- Clinical Applications
ces in the blood . Being a watery medium, blood cannot
Blood Brain Barrier
solubilize lipid components.
i. Bilirubin and non-esterified fatty acids are spe- Albumin-fatty acid complex cannot cross blood-brain
cifically transported by albumin. barrier and hence fatty acids cannot be taken up by
ii. Drugs (sulfa, aspirin, salicylate, dicoumarol, pheny- brain. The bilirubin from albumin may be competi-
toin). tively replaced by drugs like aspirin. Being lipophilic,
Chapter 26: Plasma Proteins 387

unconjugated bilirubin can cross the blood brain barrier Albumin-Gl,obulin Ratio
and get deposited in brain. The brain of young children
are susceptible; free bilirubin deposited in brain leads to In hypoalbuminemia, there will be a compensatory
kernicterus and mental retardation (see Chapter 22). increase in globulins which are synthesized by the reti-
culoendothelial system. Albumin-globulin ratio (NG
Drug Interactions ratio) is thus altered or even reversed. This again leads
to edema.
When two drugs having high affinity to albumin are
administered together, there may be competition for Hypoproteiinemia
the available sites, with consequent displacement of
Since albumin is the major protein present in the blood.
one drug. Such an effect may lead to clinically significant
any condition causing lowering of albumin will lead to
drug interactions, e.g. phenytoin-dicoumarol interaction.
reduced total proteins in blood (hypoproteinemia).
Protein-Bound Calcium
Hypergammaglobulinemias are Seen in
Calcium level in blood is lowered in hypoalbumine-
mia. Thus, even though total calcium level in blood is Low Albumin Level
lowered, ionized calcium level may be normal, and When albumin level is decreased, body tries to com-
so tetany may not occur (see Chapter 34). Calcium is pensate by incneasing the production of globulins from
lowered by 0.8 mg/dl for a fall of 1 g/dl of Albumin . reticuloendothelial system.

Edema Chronic Infections


Hypoalbuminemia will result in tissue edema (see Star- Gamma globulins are increased , but the increase is
ling's law). smooth and widie based (Fig. 26.1A).
a. Malnutrition , where albumin synthesis is depres-
sed (generalized edema) Multiple MyE,loma
b. Nephrotic syndrome, where albumin is lost through
Drastic increase in globulins are seen in paraproteine-
urine (facial edema)
mias, when a sharp spike is noted in electrophoresis.
c. Cirrhosis of liver (mainly ascites), where albumin
This is termed as M-band because of the monoclonal
synthesis is less and it escapes into ascitic fluid
origin of immunoglobulins (Figs. 26.1B and 26.2). The
d. Chronic congestive cardiac failure: Venous con-
monoclonal origin of immunoglobulins is seen in multiple
gestion will cause increased hydrostatic pressure
myeloma (see Chapter 46). Monoclonal gammopathies
and decreased return of water into capillaries and
are characterize,d by the presence of a monoclonal pro-
so pitting edema of feet may result.
tein, which can be detected by serum protein electro-
phoresis. The li~Iht chains are produced in excess which
Normal Value
is excreted in urine as Bence-Jones proteins (BJP) when
Normal level of Albumin is 3.5-5 g/dl. Lowered level of their serum lev,el increases. Multiple myeloma is the
albumin (hypoalbuminemia) has important clinical sig- most common type of monoclonal gammopathy. Free
nificance. light chain assay along with kappa and lambda ratio in
serum and urine is found to be very useful in early diag-
Hypoalbuminemia nosis, monitorin!g the response to treatment and predic-
a. Cirrhosis of liver: Synthesis is decreased. tion of prognosis.
b. Malnutrition: Availability of amino acids is reduced
and albumin synthesis is affected. ITRANSPORT PROTEINS
c. Nephrotic syndrome: Permeability of kidney glo- Blood is a wate1ry medium; so lipids and lipid soluble
merular membrane is defective, so that albumin is substances will not easily dissolve in the aqueous
excreted in large quantities. medium of blood. Hence such molecules are carried by
d. Albuminuria and microalbuminuria are described in specific carrier proteins. Their important featu res are
Chapter 25. summarized in Table 26.1.
388 Section C: Clinical and Applied Biochemistry

TABLE 26.1: Carner proteins or transport proteins of plasma


Name Plasma level Compound bound or transported Biological and clinical significance
Albumin 3.5- 5 g/dl Fatty acids, bilirubin, calcium, thyroxine, Bilirubin competes with aspirin for binding sites on
heavy metals, drugs, e.g. aspirin, sulfa albumin
Prealbumin 25- 30 mg/dl Steroid hormones, thyroxine, Rich in tryptophan. Half-life is 1 day. It is a negative
(Transthyretin) retinol acute phase protein
Transports T3 and T4 losely
Retlnol binding 3- 6mg/dl Retinol Synthesized by liver. RBP has a short half-life. Level
protein (RBP) (Vitamin A) indicates vitamin A status
Useful to assess the protein turn over rate
Thyroxine 1- 2 mg/ dL Thyroxine Assessment of the binding sites on TBG is important
binding globulin (TBG) in studying thyroid function
It is synthesized in liver
Transcortin; Cortisol 3- 3.5 mg/dl Cortisol and Corticosterone Synthesized by liver. Increased in pregnancy. Free
binding globulin (CBG) unbound fraction of hormone is biologically active
Haptoglobin (Hp) 40- 175 mg/ dL Hemoglobin Synthesized in liver. Low level indicates hemolysis.
Half-life of Hp is 5 days; but t hat of Hb-Hp is only 90
minutes. It is an acute phase protein (see Chapter 34)
Transferrin 200- 300 mg/ dl Iron 33% saturated Conserves iron by preventing iron loss through urine
(see Chapter 34)
Hemopexin 50- 100 mg/ dL Free heme Helps in preventing loss of heme (and so iron also)
from body (see Chapter 34)

1. Albumin: It is an important transport protein, which conditions. Such proteins are acute phase proteins.
carries bilirubin, free fatty acids, calcium and drugs Important acute phase proteins are described below:
(see above).
2. Pre-albumin or Transthyretin: It is so named C-Reactive Protein (CRP)
because of its faster mobility in electrophoresis than
So named because it reacts with C-polysaccharide of
albumin. It is more appropriately named as Trans-
capsule of pneumococci. It is synthesized in liver. It can
thyretin or Thyroxin binding pre-albumin (TBPA),
stimulate complement activity and macrophage phago-
because it carries thyroid hormones, thyroxin (T4)
cytosis. When the inflammation has subsided, CRP
and tri-iodo thyronine (T3). Its half-life in plasma is
quickly falls, followed by ESR (erythrocyte sedimenta-
only 1 day.
tion rate). CRP level, especially high sensitivity C-reac-
3. Retinal binding protein (RBP): It carries vitamin
tive protein level in blood has a positive correlation in
A (see Chapter 32). It is a negative acute phase
predicting the risk of coronary artery diseases (see
protein.
Chapter 15).
4. Thyroxine binding globulin (TBG): It is the spe-
cific carrier molecule for thyroxine and tri-iodo thy- Ceruloplasmin
ronine. TBG level is increased in pregnancy; but
Ceruloplasmin (cp) is blue in color (Latin, caeruleus =
decreased in nephrotic syndrome.
blue). It contains 6 to 8 copper atoms per molecule.
5. Transcortin: It is also known as Cortisol binding
Ceruloplasmin is mainly synthesized by the hepatic
globulin (CBG). It is the transport protein for cortisol
parenchymal cells. Ceruloplasmin is also called Ferroxi-
and corticosterone.
6. Haptoglobin: Haptoglobin (for hemoglobin), Hemo- dase, an enzyme which helps in the incorporation of
iron into transferrin (see Chapter 34 ). It is an important
pexin (for heme) and Transferrin (for iron) are
antioxidant in plasma. About 90% of copper content of
important to prevent loss of iron from body.
plasma isboundwithceruloplasmin, and 10o/owithalbumin.
Copper is bound with albumin loosely, and so easily
LACUTE PHASE PROTEINS exchanged with tissues. Hence, transport protein for
The level of certain proteins in blood may increase 50 to copper is Albumin. Lowered level of ceruloplasmin is
1000 folds in various inflammatory and neoplastic seen in Wilson's disease, malnutrition, nephrosis, and
Chapter 26: Plasma Proteins 389

cirrhosis. Ceruloplasmin is an acute phase protein. and is an important in vivo anti-coagulant. Its concen-
Increased plasma Cp levels are seen in hemochroma- tration is markedly increased in Nephrotic syndrome,
tosis, and obstructive biliary disease, pregnancy, estro- where other proteins are lost through urine.
gen therapy, inflammatory conditions, collagen disorders
and in malignancies. Drugs increasing the ceruloplas- Negative Acute Phase Proteins
min level are, estrogen and contraceptives. Reference During an inflammatory response, some proteins
blood level of normal adult male is 22-40 mg/dl.
are seen to be decreased in blood; those are called
Wilson's Disease negative acute phase proteins. Examples are albumin,
transthyretin (pre-albumin), retinal binding protein
Ceruloplasmin level is reduced to less than 20 mg/dl
in Wilson's hepatolenticular degeneration. It is an and transferrin. Transferrin is described in Chapter 34,
inherited autosomal recessive condition. Incidence of under iron metabolism.
the disease is 1 in 50,000. The basic defect is a mutation A comprehensive list of normal values for the sub-
in a gene encoding a copper binding ATPase in cells, stances present in blood is given in Appendix II.
which is required for excretion of copper from cells. So,
copper is not excreted through bile, and hence cop- CLOTTING FACTORS
per toxicity. Please also see Chapter 34, under copper
The word coagulation is derived from the Greek term,
metabolism. Increased copper content in hepatocyte
"coagulare" = to curdle. The biochemical mechanism of
inhibits the incorporation of copper to apoceruloplasmin.
clotting is a typical example of cascade activation.
So ceruloplasmin level in blood is decreased.
The coagulation factors are present in circulation as
inactive zymogen forms. They are converted to their
Clinical Features
active forms only when the clotting process is initiated.
Accumulation in liver leads to hepatocellular degenera- This would prevent unnecessary intravascular coagula-
tion and cirrhosis. Deposits in brain basal ganglia leads tion. Activation process leads to a cascade amplifica-
to lenticular degeneration and neurological symptoms. tion effect, in which one molecule of preceding factor
Copper deposits as green or golden pigmented ring activates 1000 molecules of the next factor. Thus within
around cornea; this is called Kayser-Fleischer ring. seconds, a large number of molecules of final factors
are activated. The clotting process is schematically rep-
Alpha-1 Antitrypsin (AAT) resented in Figure 26.4 and the characteristics of coagu-
It is otherwise called alpha-anti-proteinase or protease lation factors are shown in Table 26.2. Table 26.3 gives
inhibitor. It inhibits all serine proteases (proteolytic the assays of clotting factors.
enzymes having a serine at their active center), such as Several of these factors require calcium for their
plasmin, thrombin, trypsin, chymotrypsin, elastase, and activation. The calcium ions are chelated by the gamma
cathepsin. Serine protease inhibitors are abbreviated carboxyl group of glutamic acid residues of the factors,
as Serpins. prothrombin, VII, IX, X, XI and XII. The gamma car-
The AAT is synthesized in liver. It is a glycoprotein boxylation of glutamic acid residues is dependent on
with a molecular weight of 50 KO. It forms the bulk of vitamin K (see Chapter 32).
molecules in serum having alpha-1 mobility. The inci-
dence of AAT deficiency is 1 in 1000 in Europe, but
Prothrombin
uncommon in Asia. Bacterial infections in lung attract It has a molecular weight of 69,000 D. The plasma con-
macrophages which release elastase. In the AAT defi- centration is 10-15 mg/dl. The prothrombin is converted
ciency, unopposed action of elastase will cause dam- to thrombin by Factor Xa, by the removal of N-terminal
age to lung tissue, leading to emphysema. About 5% of fragment.
emphysema cases are due to AAT deficiency.
Fibrinogen
Alpha-2 Macroglobulin (AMG)
The conversion of fibrinogen to fibrin occurs by cleav-
It has a molecular weight of 725 KO. It is the major com- ing of Arg-Gly peptide bonds of fibrinogen. Fibrinogen
ponent of alpha-2 globulins. It is synthesized by hepato- has a molecular weight of 340,000 D and is synthe-
cytes and macrophages. AMG inactivates all proteases, sized by the liver. Normal fibrinogen level in blood is
390 Section C: Clinical and Applied Biochemistry

••Surface contact (aPTT)

a -L
Xlla Tissue activation (PT)

~ 1__
• -*.__-~
Xia

--- '- -~-- ! 1


IXa VIia

l VIiia calcium
platelet factor
Xa

Va calcium
platelet factor

iil:il·i::i·ii:I ---• Thrombin (TT)

liMii:i·H:M - -~•--
FibrinP

Fibrin clot

Hii:iiW+l-t--- ---i Fibrin degradation


Tissue plasm1nogen activators (TPA) products

Fig. 26.4: Cascade pathway of coagulation

TABLE 26.2: Factors involved 1n coagulation process


No. Name Activated by Function
I Fibrinogen Thrombin Forms the clot (fibrin)
II Prothrombin Factor Xa Activation of fibrinogen and factors XIII, VIII and V
IV Calcium - Activation of factor II, VII, IX, X,XI and XII
V Labile factor Thrombin Binding of prothrombin to platelet
VII Proconvertin; serum prothrombin convertin Thromb in Activation offactor X
antecedent (SPCA)
VIII Antihemophilic globulin (AHG) Thrombin Activation of factor X
IX Plasma thromboplastin- Factor Xia Activation of factor X
component (PTC);
Christmas factor
X Stuart Prower factor Factor IXa Activation of prothrombin
XI Plasma thromboplastin anticedent (PTA) Factor Xlla Activation of factor IX I
XII Hageman factor Kallikrein Activation of factor XI
XIII Fibrin stabilizing factor (Liki Lorand factor) Thrombin Stabilization of fibrin clot by forming cross links
Prekallikrein Activation of factor XII

200-400 mg/dl. The fibrin monomers formed are produce the active plasmin. Plasmin in turn, is inacti-
insoluble. They align themselves lengthwise, aggregate vated by alpha-2 antiplasmin.
and precipitate to form the clot. Fibrinogen is an acute Tissue plasminogen activator (TPA) is a serine
phase protein. protease present in vascular endothelium. TPA is relea-
sed during injury and then binds to fibrin clots. Then TPA
Fibrinolysis cleaves plasminogen to generate plasmin, which dis-
Unwanted fibrin clots are continuously dissolved in vivo solves the clots.
by Plasmin, a serine protease. Its inactive precursor Urokinase is another activator of plasminogen. Uro-
is plasminogen (90 kD). It is cleaved into two parts to kinase is so named because it was first isolated from
Chapter 26: Plasma Proteins 391

TABLE 26.3: Assays for clotting factors Father normal


Name Parameter measured
Prothromin time Time in seconds taken for t he patient's sample to
(PT) clot; thromboplastin reagent is added
Partial t hrom- A measure of how well patient's blood will clot
boplastin time
(PTT)
Activated partial Initiated by adding a negatively charged surface
thromboplastin like silica to the plasma and a phoispholipid
time (aPTT) extract that is free of tissue factor
Th rombin t ime Assess hemostasis. Measures ability of fibrinogen Daughter normal Son normal Daughter Son
(TT) to form fib rin strands in vitro. Exogenous carrier hemophlhc
thrombin is added to pre-wa rmed plasma
Fig. 26.5: Inheritance pattern of hemophilia
D-Dimer Assess hemostatic function. D-Dimer is formed
by degradation of fibrin clots by plasmin.
High level indicates increased risk of recurrent factor VIII (anti-hemophilic globulin) (AHG). It is the
thromboembolism. High negat ive predictive commonest of the inherited coagulation defects.
value of thrombosis
There will be prolongation of clotting time. Hence,
Activated Whole blood is mixed with a clot activator.
clotting time Normally takes 70-180 second s. Bedside even trivial wounds, such as tooth extraction will cause
monitoring of high dose heparin therapy excessive loss of blood. Patients are prone to internal
Anti-Xa test Exogenou s factor Xa and ant i-thrombin (AT) bleeding into joints and intestinal tract.
in excess and chromogenic substrate from
Until recently the treatment consisted of administra-
Xa. Heparin present complexes wit h AT and
inactivates factor Xa. Any excess Xa will release tion of AHG, prepared from pooled sera every 3 months.
the chromophore from the substrate. Adj ustment Since this was not generally available, the usual treat-
of patient's heparin level
ment was to transfuse blood periodically, which may
Coagulation Determi nes the level of various coagulation
factor assay factors. Factor deficient plasma is mixed with
lead to eventual iron overload, hemochromatosis (see
the specific factor being tested by adding Chapter 34). However, component separation of blood
patient's diluted citrated plasma. The patient's is now possible. As infusion of cell free plasma becomes
specimen supplies the missing factor and the
assay is completed by performing a standard PT
common, this type of hemochromatosis is becoming
Calibrated by using st andard reference plasma rare nowadays. Pure AHG is now being produced by
recombinant technology and is the treatment of choice.
urine. Urokinase is produced by macrophages, mono- (')
cytes and fibroblasts. Streptokinase, isolated from • • Clinical Case Study 26.1
streptococci is another fibrinolytic agent. A severe form of obstructive lung disease starting with
dyspnea and leading to emphysema was found in
Clinical significance
several members of the same family. Blood analysis of
Thrombosis in coronary artery is the major cause of the surviving members of the family revealed abnor-
myocardial infa rction (heart attack). If TPA, urokinase or mally low concentration of alpha-1 antitrypsin . What is
streptokinase is injected intravenously in the early phase the basis of this condition?
of thrombosis, the clot may be dissolved and recovery of
patient is possible. (')
~- Clinical Case Study 26.2
ABNORMALITIES IN A male child, born to a normal young couple, was found
COAGULATION to develop hemorrhagic tendency quite early in life. His-
tory revealed that the mother was the only daughter of
Hemophilia A (Classical Hemophilia) a family who did not have any male offspring during the
past 2 generations.
This is an inherited X-linked recessive disease affect- A. What are the possible causes?
ing males and transmitted by females. Male children B. How will you explain the nature of inheritance?
of hemophilia patients are not affected; but female chil- C. What is the advice to be given to the parents
dren will be carriers, who transmit the disease to their regarding bringing up the son and having another
male offspring (Fig . 26.5). This is due to the deficiency of child.
392 Section C: Clinical and Applied Biochemistry

0 I LEARNING POINTS, CHAPTER 26


;fi. Clinical Case Study 26.1 Answer 1. Total plasma protein content is 6-8 g/dl of which
Emphysema , a lung disease characterized by destruc- albumin is 3.5-5 g/dl and the rest is globulin .
tion of alveolar walls, has many causes including airway Almost all plasma proteins are synthesized in the
infections, cigarette smoking, air pollution and hereditary liver except immunoglobulins.
origin. Deficiency of alpha-1 antitrypsin leads to 2. On agar gel electrophoresis albumin has maxi-
mum mobility while gamma globulin has minimum
development of emphysema. alpha-1 antitrypsin makes
mobility.
up most of the proteins in alpha-1 globulin band during
3. In chronic infection, gamma globulins are increased
serum protein electrophoresis. smoothly, while in paraproteinemias, M-band is
Lungs contain a natural enzyme called neutrophil seen. The alpha-2 fraction is increased in nephrotic
elastase that digests damaged aging cells and bacteria syndrome while albumin is decreased in liver cirr-
and promotes healing of lung tissue. Being non-specific hosis, malnutrition, nephrotic syndrome.
it can attack lung tissue itself; but alpha-1 antitrypsin 4. Albumin contributes to colloid osmotic pressure of
protects against this process by destroying excess plasma, has buffering capacity and is a transport
amount of this enzyme. Absence of alpha-1 antitrypsin medium for various hydrophobic substances.
can lead to destruction of lung tissue and emphysema. 5. Hypergammaglcbulinemia is seen in conditions of
Clinical features of alpha-1 antitrypsin deficiency hypoalbuminemia, chronic infection and parapro-
include shortness of breath, reduced exercise tolerance, teinemias.
6. The transport proteins in blood are albumin, pre-
wheezing, recurrent respiratory infections and in advanced
albumin (transthyretin), RBP, TBG, transcortin and
cases, difficulty in breathing. Smoking exacerbates the
haptoglobin.
condition. About 10% of patients can have liver damage.
7. The levels of certain proteins in blood may increase
Diagnosis is by estimation of alpha-1 antitrypsin 50-100-fold in various inflammatory and neo-
levels, arterial blood gas analysis, chest X-ray, CT scan plastic conditions. Such proteins are called acute
of chest, pulmonary function tests and genetic testing. phase proteins. For example, CRP, ceruloplasmin,
Treatment involves supplementation of alpha-1 anti- haptoglobins, alpha-1 acid glycoprotein, alpha-1
trypsin and antioxidants. antitrypsin and fibrinogen.
8. Proteins that are decreased in blood during inflam-
0 matory response are called negative acute phase
•i i- Clinical Case Study 26.2 Answer proteins. For example, albumin, transthyretin, trans-
Hemophilia (see description in this chapter). ferrin.

PART-1 : ESSAY AND SHORT NOTE QUESTIONS

26-1 . Enumerate the major transport proteins of p lasma. Explain the t ransport of free fatty acids, bilirubin, iron and
calcium.
26-2. What are the important functions of albumin? Give the major causes and manifestations of hypoalbuminemia.

SHORT NOTE QUESTIONS


26-3. Albumin-globulin ratio 26-6. Haptoglobin.
26-4. Enumerate Transport proteins of blood. 26-7. Transferrln.
26-5. Ceruloplasmin. 26-8. Alpha-1 antitrypsin deficiency.

PART-2: MULTIPLE CHOICE QUESTIONS


26-1. Normal level of albumin in blood is: 26-2. Hypo albuminemia is seen in all the following condi-
A. 1.5-2.5 mg/dl tions, except:
B. 2.5-3.5 mg/dl
A. Cirrhosis of liver B. Nephrotic syndrome
C. 2.5-3.5 g/dl
D. 3.5-5.0 g/dl C. Malnutrition D. Acute infections
Chapter 26: Plasma Proteins 393

26-3. Hemopexin carries: 26-13. Albumin globulin ratio is reversed in all the follow-
A. Free hemoglobin B. Free heme ing conditions, except
C. Free bilirubin D. Free iron A. Cirrhosis liver
26-4. All the following are acute phase reactant proteins, B. Primary immune deficiency
except: C. Nephrotic syndrome
A. C-reactive protein (CRP) D. Multiple myeloma
B. HDL 26-14. It is not safe to give salicylate to infants having
C. Ceruloplasmin hemolytic disease, because:
D. Haptoglobin A. Bilirubin and salicylate compete each other for
26-5. Polymorphism is exhibited by all the following binding on albumin
B. Salicylate is toxic to children
proteins, except:
C. Hemolysis is produced by salicylate
A. Haptoglobin B. Transferrin
D. Both bilirubin and salicylate are metabolized by
C. Albumin D. Ceruloplasmin
the same enzyme systems in liver
26-6. Wilson's hepatolenticular degeneration is charac-
26-15. All are transport proteins of blood, except:
terized by:
A. Albumin B. Transcortin
A. Ceruloplasmin level in blood is increased C. Transferrin D. Ceruloplasmin
B. Copper is accumulated in liver to produce cirrhosis 26-16. One of the plasma proteins listed below Is not a
C Copper is deposited in skin to produce bronze transport protein (carrier protein):
color A. Transferrin B. Haptoglobin
D. Autosomal dominant inheritance C. Albumin D. Alpha-1-antitrypsin
26-7. All are true with regard to alpha-1 antitrypsin (AAT), 26-17. All are true with ceruloplasmin, except
except: A. It is a copper containing protein found in serum
A. It is a protease inhibitor B. It is an acute phase protein
B. It shows polymorphism C. Blood level Is increased in Wilson's disease
C. Deficiency leads to emphysema in lungs D. It has ferroxidase activity
D. Deficiency is associated with edema 26-18. All are correct with hemophilia, except:
26-8. Albumin level in blood is estimated by: A. Inherited as an x-linked recessive trait
A. Jaffe's picric acid reaction B. Inherited by son from father
B. Bromo cresol green reaction C. Daughters are likely to be carriers
C. Diacetyl monoxime method D. Prenatal diagnosis is important
D. Chromatography 26-19. Hemorrhagic tendency is observed in all the follow-
26-9. Which reaction takes place exclusively in liver: ing conditions, except:
A. Gluconeogenesis A. Hemophilia
B. Glycolysis B. Vitamin K deficiency
C. Glycogen synthesis C. Thrombocytopenia
D. Albumin synthesis D. Phenylketonuria
26-10. In blood, allarebound with albumin, except 26-20. Which does not show X-linked inheritance?
A. Glucose-6-P dehydrogenase deficiency
A. Non-esterified fatty acids
B. Hemophilia
B. Bilirubin
C. Von Willebrandt's disease
C. Iron
D. Christmas disease
D. Salicylate
26-21 . The anticoagulant found in the body is:
26-11 . Electrophoretic separation of proteins is of dia-
A. Potassium oxalate B. Sodium citrate
gnostic value in all conditions, except:
C. Heparin D. EDTA
A. Nephrotic syndrome 26-22. Which protein is not present in plasma?
B. Multiple myeloma A. Albumin B. Fibrinogen
C. Alpha-1 antitrypsin deficiency C. Hemoglobin D. Globulins
D. Acquired immunodeficiency syndrome 26-23. The protein present in highest concentration in
26-12. Edema due to hypoproteinemia may be seen in all plasma is:
the following clinical conditions, except A. Fibrinogen B. Gamma globulins
A. Rheumatoid arthritis C. Albumin D. Alpha globulins
B. Cirrhosis liver 26-24. Electrophoretic separation is based on:
C. Malnutrition A. Charge on the protein molecule
D. Nephrotic syndrome B. Molecular weight of the protein
394 Section C: Clinical and Applied Biochemistry

C. Charge and molecular weight A. Alpha fetoprotein B. Transcortin


D. Amino acid content C. Thyroxin binding globulin
26-25. Which statement is not true regarding albumin: D. High density lipoprotein
A. Synthesized by liver 26.29. From the pairs of proteins and diseases where
B. Has maximum buffering capacity they are altered pick out the mismatched pair:
C. Transports bilirubin A . Alpha-1 antitrypsin and emphysema
D. Excreted in urine B. Alpha fetoprotein and neural tube defects
26.26. Which of the following is not transported by C. Thyroxine binding globulin and hypothyroidism
albumin? D. Gamma globulins and nephrotic syndrome
A. Calcium B. Copper 26.30. Pick out the mismatched pair:
C. Sodium D. Aspirin A. Ceruloplasmin and iron absorption
26.27. Hypoalbuminemia is a feature of all except: B. Hemopexin and heme binding
A. Chronic liver disease C. Haptoglobin and hemolysis
B. Nephrotic syndrome D. Transferri n and copper binding
C. Hemoconcentration 26.31 . Which of the following is a negative acute phase
D. Protein energy malnutrition reactant protein?
26.28. Which of the following alpha globulins is a tumor A. Albumin B. Ceruloplasmin
marker? C. CRP D. Haptoglobin

ANSWERS OF MULTIPLE CHOICE QUESTIONS

26-1. D 26-2. D 26-3. B 26-4. B 26-5. C 26-6. B 26-7. D


26.8. B 26-9. D 26-10. C 26-11. D 26-12. A 26-13. B 26-14. A
26-15. D 26-16. D 26-17. C 26-18. B 26-19. D 26-20. C 26.21 . C
26.22. C 26.23. C 26.24. C 26.25. D 26.26. C 26.27. C 26.28. A
26.29. D 26.30. D 26.31. C

PART-3: VIVA VOCE QUESTIONS AND ANSWERS

26-1 . What is normal serum albumin level? Retinol binding protein, Thyroxine binding globulin,
3.5 to 5 g/dl. Transcortin (cortisol); Haptoglobin (hemoglobin); Trans-
26-2. What is normal value of total proteins in serum? ferrin (iron); Hemopexin (free heme).
26-10. Name some acute phase proteins.
6 to 8 g/100 ml.
C-reactive protein; Ceruloplasmin; Haptoglobin.
26-3. Edema due to hypoproteinemia is seen in which
26-11 . Polymorphism is exhibited by which proteins?
conditions?
Haptoglobin; Transferrin; Ceruloplasmin.
Cirrhosis liver; Malnutrition; Nephrotic syndrome. 26-12. What is the clinical manifestation of Alpha-1-anti-
26-4. Albumin carries what substances? trypsin deficiency?
Free fatty acids; Bilirubin; Salicylate; Calcium. Emphysema and chronic lung infections.
26-5. What are the functions of albumin? 26-13. What is ceruloplasmin?
It maintains colloidal osmotic pressure of plasma; it It is a copper containing enzyme (ferroxidase) seen in
transports non-esterified fatty acid and bilirubin. blood. It is an acute phase protein.
26-6. Albumin is synthesized in which organ? 26-14. What is the significance of ceruloplasmin?
Ceruloplasmin level in blood is decreased in Wilson's
Liver.
hepatolenticular degeneration.
26-7. Where are gamma globulins synthesized?
26-15. What is the carrier protein of copper?
By reticuloendothelial system (Spleen. lymph nodes).
Albumin.
26-8. Albumin globulin ratio is reversed in which condi- 26-16. Hemopexin carries what?
tions? Free heme.
Cirrhosis; Chronic infections; Nephrotic syndrome; 26-17. How Hemophilia is transmitted?
Multiple myeloma. It is inherited as an x-linked recessive trait; males are
26-9. Name some transport proteins. affected; females are carriers.
1----------Chapter 27
Acid-Base
Balance and pH

Chapter at a Glance
The learner will be able to answer questions on the following topics:

D Acids and bases D Renal regulation of pH


0 pH 0 Relation of pH and potassium
0 Buffers 0 Respiratory acidosis
0 Acid-base balance in the body 0 M etabolic acidosis
0 Bicarbonate buffer system 0 Respiratory alkalosis
D Respiratory regul ation of pH D Metabolic alkalosis

Hydrogen ions (H+) are present in all body compart- BOX 27.1: Terms explained
ments. Maintenance of appropriate concentration of Term Definition and explanations
hydrogen ion (H+) is critical to normal cellular function. pH Negative logarithm of hydrogen ion concentra-
The acid-base balance or pH of the body fluids is main- tion. Normal value 7.4 (range 7.38- 7.42)
Acids Proton donors; pH < 7
tained by a closely regulated mechanism. This involves
Bases Proton acceptors; pH > 7
the body buffers, the respiratory system and the kidney. Strong acids Acids which ionize completely; e.g. HCI
Some common definitions are given in Box 27 .1. Weak acids Acids which ionize incompletely, e.g. H,CO3
pKvalue pH at which the acid is half ionized; Salt:Acid
I ACIDS AND BAS~ = 1:1
Alkali reserve Bicarbonate available to neutralise acids;
Definition Normal 24 mmol/ L (range 22-26 mmol/L)
Buffers Solutions minimize changes in pH
The electrolyte theory of dissociation was proposed by
Arrhenius (Nobel Prize, 1903). According to the definition
proposed by Bronsted, acids are substances that
are capable of donating protons and bases are
those that accept protons. Acids are proton donors
and bases are proton acceptors. A few examples are
shown below:
Acids Bases
HA:;::::::::==W+A- NH 3 +H+:;::::::::==NH/
SPL Svante Arrhenius Johannes N
HCI :;::::::::== w + c1- Hco; + H+ :;::::::::== H2C03
Sorensen NP 1903 Bronsted
H2C03 :;::::::::== H+ + HC03 1868-1939 1859-1927 1879-1947
396 Section C: Clinical and Applied Biochemistry

Weak and Strong Acids TABLE 27. 1: Reli:lt1on between hydrogen ions hydroxyl ions
and pH of aqueous solutions Ionic: product of water~ [H·]
The extent of dissociation decides whether they are [OH]= 10 '
strong acids or weak acids . Strong acids dissociate
[OH"] [H'J log -log(H'J
completely in solution, while weak acids ionize incom- molsAiter molsAiter [H'] =pH pOH Inference
pletely, for example,
1 X 10 13 1 X 101 -1 1 13 Strong acid
HCI - - w+c1- (Complete)
1 X 1010 1 X 10_... -4 4 10 Acid
H2CO3 W + HCO3- (Partial)
1 X 107 1 X 10-7 -7 7 7 Neutral
In a solution of HCI , almost all the molecules disso- 1 X 10 4
1 X 10 10
-10 10 4 Alkali
ciate and exist as H+ and Cl- ions. Hence the concentra- 1 X 10 1 1 X 10 13 - 13 13 1 Strong alkali
tion of W is very high and it is a strong acid. But in the
case of a weak acid (e.g. acetic acid), it will ionize only
partially. So, the number of acid molecules existing in Thus the pH value is inversely proportional to the
the ionized state is much less, may be only 50%. acidity. Lower the pH, higher the acidity or hydrogen
ion concentration while higher the pH, the
Dissociation Constant
acidity is lower (Table 27.1 ). At a pH of 1,
Since the dissociation of an acid is a freely reversible the hydrogen ion concentration is 1o times
reaction, at equilibrium the ratio between dissociated that of a solution with a pH 2 and 100 times
and undissociated particle is a constant. The dissocia-
that of a solution with a pH of 3 and so on.
tion constant (Ka) of an acid is given by the formula,
The pH 7 indicates the neutral pH. KA Hasse/batch
[H•] [A] 1874-1962
K a = - -- - The Effect of Salt Upon the
[HA]
Dissociation
Where [W] is the concentration of hydrogen ions, [A-J =
the concentration of anions or conjugate base, and [HA] The relationship between pH, pKa, concentration of acid
and conjugate base (or salt) is expressed by the Hen-
is the concentration of undissociated molecules.
derson-Hasselbalch equation,
The pH at w hich the acid is half ionized is called
pKa of an acid which is constant at a particular tempera- [base] [salt]
ture and pressure. Strong acids will have a low pKa and pH = pKa + log [acid) or pH = pKa + log [acid]
weak acids have a higher pKa.
When (base]= [acid]; then pH = pKa
Acidity of a Solution and pH Therefore, when the concentration of base and
The acidity of a solution is measured by noting the acid are the same, then pH is equal to pKa. Thus,
hydrogen ion concentration in the solution and obtained when the acid is half ionized, pH and pKa have the same
by the equation. values.

_ K [acid] [HA]
[H+l - a - - or - -
[base] A- IBUFFER_S_ _ _ __ _
where Ka is the dissociation constant. Definition
To make it easier, Sorensen expres-
Buffers are solutions which can resist changes in pH
sed the W concentration as the negative
when acid or alkali is added.
of the logarithm (logarithm to the base 10)
of hydrogen ion concentration, and is
Composition of a Buffer
designated as the pH. Therefore, Lawrence J
Henderson Buffers are of two types:
pH = - log [W] = log _ 1_ 1878-1942 a. Mixtures of weak acids with their salt with a strong
[W] base or
Chapter 27: Acid-Base Balance and pH 397

b. Mixtures of weak bases with their salt with a strong The Henderson-Hasselbalch's equation, therefore
acid. A few examples are given below: has great practical application in clinical practice in
i. H2 CO/ NaHCO3 (Bicarbonate buffer) assessing the acid-base status.
(carbonic acid and sodium bicarbonate)
ii. CH3 COOH/CH 3COONa (Acetate buffer) Effective Range of a Buffer
(acetic acid and sodium acetate) A buffer is most effective when the concentrations of salt
iii. Na2 HPOj NaH2 PO4 (Phosphate buffer) and acid are equal or when pH = pKa. The effective
range of a buffer is 1 pH unit higher or lower than pKa.
Factors Affecting pH of a Buffer
Since the pKa values of most of the acids produced in
The pH of a buffer solution is determined by two factors: the body are well below the physiological pH, they
a. The value of pK: The lower the value of pK, the
immediately ionize and add H• to the medium. This would
lower is the pH of the solution. necessitate effective buffering. Phosphate buffer is
b. The ratio of salt to acid concentrations: Actual
effective at a wide range, because it has 3 pKa values.
concentrations of salt and acid in a buffer solution
may be varying widely, with no change in pH, so
~ CID-BASE BALANCE
_ __ _ _
long as the ratio of the concentrations remains the
same. Normal pH
Factors Affecting Buffer Capacity The pH of plasma is 7.4 (range 7.35-7.45). In normal
On the other hand, the buffer capacity is determined by life, the variation of plasma pH is very small. The pH of
the actual concentrations of salt and acid present, as plasma is maintained within a narrow range. The pH
well as by their ratio. of the interstitial fluid is generally 0.5 units below that of
Buffering capacity is the number of grams of strong the plasma.
acid or alkali which is necessary for a change in pH of
one unit of one litre of buffer solution. Acidosis
The buffering capacity of a buffer is defined as the If the pH is below 7.35, it is called acidosis. Life is threa-
ability of the buffer to resist changes in pH when an tened when the pH is lowered below 7.25. Acidosis
acid or base is added.
leads to CNS depression and coma. Death occurs when
pH is below 7.0.
How Do Buffers Act?
Buffer solutions consist of mixtures of a weak acid or Alkalosis
base and its salt.
To take an example, when hydrochloric acid is When the pH is more than 7.45, it is alkalosis. It is
added to the acetate buffer. the salt reacts with the acid very dangerous if pH is increased above 7.55. Alkalo-
forming the weak acid, acetic acid and its salt. Similarly sis induces neuromuscular hyperexcitability and tetany.
when a base is added, the acid reacts with it forming salt Death occurs when the pH is above 7.6.
and water. Thus changes in the pH are minimized.
CH3- COOH + NaOH -+ CH3-COONa + Hp Volatile and Fixed Acids
CH3-COONa + HCI -+ CH3-COOH + NaCl During the normal metabolism , the acids produced may
The buffer capacity is determined by the absolute be volatile acids like carbonic acid or nonvolatile (fixed)
concentration of the salt and acid. But the pH of the acids like lactate, keto acids. sulfuric acid and phosphor-
buffer is dependent on the relative proportion of the salt
ic acid. The metabolism produces nearly 20,000 milli
and acid (see the Henderson-Hasselbalch's equation).
equivalents (mEq) of carbonic acid and 60-80 mEq of
Application of the Equation fixed acids per day. The sulfoproteins yield sulfuric acid
and phosphoproteins and nucleoproteins produce phos-
The pH of a buffer on addition of a known quantity
of acid and alkali can therefore be predicted by the phoric acid. On an average about 3 g of phosphoric acid
equation. and about 3 g sulfuric acid are produced per day. The
398 Section C: Clinical and Applied Biochemistry

BOX 27.2: Mechanisms o f regulation of pH


First line of defense : Blood buffers
including plasma
Second line of defense : Respiratory regulation
Third li ne of defense : Renal regu lation 1. NaHC(\ K2 HP04 K ' Hb
H2C03 KH2PO. WHb
carbonic acid, being volatile, is eliminated as CO2 by the (bicarbonate) (phosphate) (hemoglobin)
lungs. The fixed acids are buffered and later on the H+ 2. Na2HPO, K' Protein K 2 HPO,
are excreted by the kidney. NaH2PO, H' Protein KH?O,
(phosphate) (protein buffer) (phosphate)
Mechanisms of Regulation of pH
3. Na.Albumin KHCO, KHC03
These mechanisms are interrelated. See Box 27.2. H' Albumin H, co, Hp 0 3

BUFFERS OF THE BODY FLUIDS


a. Presence of bicarbonate in relatively high concen-
Buffers are the first line of defense against acid load.
trations.
These buffer systems are enumerated in Table 27 .2.
b. The components ;:ire under physiological control,
The buffers are effective as long as the acid load is not CO2 by lungs and bicarbonate by kidneys.
excessive, and the alkali reserve is not exhausted. Once
the base is utilized in this reaction, it is to be replenished Alkali Reserve
to meet further challenge. Bicarbonate represents the alkali reserve and it has to
be sufficiently high to meet the acid load. Under physio-
Bicarbonate Buffer System
logical circumstances, the ratio of 20 (a high alkali
The most important buffer system in the plasma is the reserve) ensures high buffering efficiency against acids.
bicarbonate-carbonic acid system (NaHCO/ H2 CO3). It
accounts for 65% of buffering capacity in plasma and Phosphate Buffer System
40% of buffering action in the whole body. The base It is mainly an intracellular buffer. Its concentration in
constituent, bicarbonate (HCO3· ) , is regulated by the plasma is very low. The pKa value is 6.8. So applying
kidney (metabolic component). W hile the acid part, the equation,
carbonic acid (H2 CO3) , is under respiratory regulation [salt]
pH (7.4)= pKa (6.8) + log [acid]
(respiratory component).
[salt]
The normal bicarbonate level of plasma is 24 or 0.6 = log [acid]
mmol/L. The normal pCO2 of arterial blood is 40 mm
Antilog of 0.6 = 4; hence the ratio is 4. This is found
of Hg. The normal carbonic acid concentration in blood
to be true under physiological condition.
is 1.2 mmol/L. The pKa for carbonic acid is 6.1 . Substi-
The phosphate buffer system is found to be effective
tuting these values in the Henderson-Hasselbalch's
at a wide pH range, because it has more than one ioniz-
equation,
able group and the pKa values are different for both.
[ Hco·3 1
pH = pKa + log pKa = 1.96 ) H+ + H PO -
[ ~C0 3] H3 Po4 2 4

24 H2 Po. - pKa = 6 -8 ) H+ + HPO4 = (Na2 HPO4/NaH 2 PO4 )


7.4 = 6.1 + log
1.2 HPO• = pKa = 12.4 H+ + PO/ -
= 6.1 + log 20 = 6.1 + 1.3
In the body, Na2 HPO/ NaH 2PO4 is an effective buffer
Hence, the ratio o f HCO3- to H2 CO 3 at pH 7.4 is 20 system, because its pKa value is nearest to physiologi-
under normal conditions. This is much higher than the cal pH.
theoretical value of 1 which ensures maximum effective-
ness. Protein Buffer System
The bicarbonate carbonic acid buffer system is the Buffering capacity of protein depends on the pKa value
most important for the following reasons: of ionizable side chains. The most effective group is
Chapter 27: Acid-Base Balance and pH 399

52% BOX 27.3: Summary of buffering against acid load


Buffer
Stages Features components
Tissue cells Normal Normal ratio = 20:1 HC03• (N)
Phosphate; Protein Normal pH = 7.4 H,CO.(N)
58% intracellular 42% extracellular First line of defense Acidosis; H enters HC03 (..,.J,J
buffering buffering
Plasma buffer system blood, bicarbonate is
Fig. 27.1: Intracellular buffers play a significant role to combat used up
3cid load of the body Second line defense Hyperventilation
Respiratory H2C03 H,O + CO2i H,co1 (.J.J
compensation
histidine imidazole group with a pKa value of 6.1. The
Third line of defense Excretion of H ; Reabsorp· HC03 (.J..!.)
role of the hemoglobin buffer is considered along with kidney mechanism tion of bicarbonate; Ratio
H2C03 (H)
the respiratory regulation of pH. and pH tend to restore

Relative Capacity of Buffer Systems


generate bicarbonate or alkali reserve by the activity of
In the body, 52% buffer activity is in tissue cells and 6%
the carbonic anhydrase system (see Chapter 23).
in RBCs. Rest 43% is by extracellular buffers. In plasma
Carbonic anhydrase
3nd extracellular space, about 40% buffering action is
CO2 + Hp H2CO3
by bicarbonate system; 1% by proteins and 1% by phos-
HCO3- + H+
phate buffer system (Fig. 27.1). H2 C03 ------
W + Hb- HHb

Buffers Act Quickly, but Not Permanently The reverse occurs in the lungs during oxygenation
and elimination of CO2 • When the blood reaches the lungs,
Buffers can respond immediately to addition of acid or the bicarbonate re-enters the erythrocytes by reversal
base, but they do not serve to eliminate the acid from of chloride shift. It combines with W liberated on oxyge-
the body. They are also unable to replenish the alkali nation of hemoglobin to form carbonic acid which disso-
reserve of the body. For the final elimination of acids, ciates into CO 2 and Hp. CO2 is thus eliminated by the
the respiratory and renal regulations are very essential. lungs.

I RESPIRATORY HHb + 0 2 - - -+ HbO2 + H'


HCO3- + H' H 2CO3
~ EGULATION OF pH H2CO3 Hp + CO2

The Second Line of Defense The activity of the carbonic anhydrase increases in
acidosis and decreases with decrease in H' concentration.
This is achieved by changing the pCO2 (or carbonic acid,
the denominator in the equation). The CO2 diffuses from
the cells into the extracellular fluid and reaches the lungs
I RENAL REGULATION OF pH
An important function of the kidney is to regulate the pH
through the blood. When there is a fall in pH of plasma
of the extracellular fluid. Normal urine has a pH around
(acidosis), the respiratory rate is stimulated resulting
6; this pH is lower than that of extracellular fluid (pH =
in hyperventilation. This would eliminate more CO2 ,
7.4). This is called acidification of urine. The pH of the
thus lowering the H2 CO3 level (Box 27.3). However, this
urine may vary from as low as 4.5 to as high as 9.8,
cannot continue for long . The respiratory system res- depending on the amount of acid excreted. The major
ponds to any change in pH immediately, but it cannot renal mechanisms for regulation of pH are:
proceed to completion. A . Excretion of H' (Fig. 27.2)
B. Reabsorption of bicarbonate (recovery of bicarbo-
Action of Hemoglobin nate) (Fig. 27.3)
The hemoglobin serves to transport the CO2 formed C. Excretion of titratable acid (net acid excretion)
in the tissues, with minimum change in pH (see iso- (Fig. 27.4)
hydric transport, Chapter 23). Side by side, it serves to D. Excretion of NH/ (ammonium ions) (Fig. 27.5).
400 Section C: Clinical and Applied Biochemistry

Plasma Proximal convoluted Tubular Plasma Tubular cell Tubular lumen


tubular cell lumen

- - ---=--- Na•
HCOr3 + H• - -----'- (-Hydrogen
_. H+ Ions

excratad)

Fig. 27.2: Excretion of hydrogen ions in the proximal tubules; CA Fig. 27.3: Reabsorption of bicarbonate from the tubular fluid ;
= Carbonic anhydrase CA= Carbonic anhydrase

Pasma Tubular cell Tubular lumen Plasma Tubular cell Tubular lumen
Na2HPO4 (pH 7.4)

. A
(+)H 20
Glutamir,~ Glutamic Acid

~i
(Glutaminase)
Na ' -- ----"- Na + NaHPO4

-J
NH3 . NH
3
+
Na Na•

Hr H'-+.;:,. I/ HCOi "'y=i H+

)
H2C03 NaH:zPC)4 NH"
f CA !(pH 5.4)
H2 C03
jcA (H.llb$Ped
'
andexc:Nted)
H O+CO Excreted CO2 + H20

Fig. 27.4 : Phosphate mechanism in tubules Fig. 27.5: Ammonia mechanism

Excretion of H•; Generation of the urine is normally bicarbonate free. The bicarbonate
Bicarbonate combines with H' in tubular fluid to form carbonic acid.
It dissociates into water and CO2• The CO2 diffuses into
This process occurs in the proximal convoluted tubu- the cell , which again combines with water to form car-
les (Fig. 27.2). The CO2 combines with water to form bonic acid .
carbonic acid, with the help of carbonic anhydrase. The In the cell, it again ionizes to H+ that is secreted into
H2CO3 then ionizes to H' and bicarbonate. The hydrogen lumen in exchange for Na•. The HCO3- is reabsorbed
ions are secreted into the tubular lumen; in exchange for into plasma along with Na•. Here, there is no net excre-
Na• reabsorbed. These Na• ions along with HCO3- will tion of W or generation of new bicarbonate. The net
be reabsorbed into the blood. There is net excretion of effect of these processes is the reabsorption of filtered
hydrogen ions, and net generation of bicarbonate. bicarbonate. But this mechanism prevents the loss of
So this mechanism serves to increase the alkali reserve. bicarbonate through urine.

Reabsorption of Bicarbonate Excretion of H• as Titratable Acid


This is mainly a mechanism to conserve base. There is In the distal convoluted tubules net acid excretion
no net excretion of H' (Fig. 27.3). When Na• enters the occurs. Hydrogen ions are secreted by the distal tubules
cell, hydrogen ions from the cell are secreted into the and collecting ducts by hydrogen ion-ATPase located
luminal fluid. The hydrogen ions are generated within in the apical cell membrane. The hydrogen ions are
the cell by the action of carbonic anhydrase. Bicar- generated in the tubular cell by a reaction catalyzed by
bonate is filtered by the glomerulus. This is completely carbonic anhydrase. The bicarbonate generated within
reabsorbed by the proximal convoluted tubule, so that the cell passes into plasma.
Chapter 27: Acid-Base Balance and pH 401

The term titratable acidity of urine refers to the num- ~ ELLULAR BUFF_E_
RS_ _ __
ber of milliliters of N/10 NaOH required to titrate 1 liter
of urine to pH 7.4. This is a measure of net acid excre- Cytoplasmic pH varies from 6.8 to 7.3. Intracellular pH
1ion by the kidney. The major titratable acid present in modulates a variety of cell functions:
1. The activity of several enzymes is sensitive to
the urine is sodium acid phosphate. As the tubular fluid
changes in pH.
passes down the renal tubules more and more W are
secreted into the luminal fluid so that its pH steadily falls. 2. Reduction in pH reduces the contractility of actin
The process starts in the proximal tubules, but continues and myosin in muscles.
up to the distal tubules. Due to the Na• to W exchange 3. The electrical properties of excitable cells are also
affected by changes in pH.
occurring at the renal tubular cell boarder, the Na2 HPO4
(basic phosphate) is converted to NaH2 PO4 (acid phos- Intracellular buffers are depicted in Figure 27.1.
The major tissues involved in cellular buffering are
phate) (Fig. 27.4). As a result, the pH of tubular fluid
bone and skeletal muscle. The buffering of acid is
falls. The acid and basic phosphate pair is considered
achieved by the exchange of W that enters into the
as the urinary buffer. The maximum limit of acidifica-
tion is pH 4.5. cells for Na• or K· ions.

!Excretion of Ammonium Ions Relationship of pH with K• Ion Balance


This predominantly occurs at the distal convoluted When there is increase in H+ in extracellular fluid (ECF),
tubules. This would help to excrete W and reabsorb there may be exchange of H+ with K• from within the
HCO3- (Fig. 27.5). This mechanism also helps to trap cells. Net effect is an apparent increase in ECF potas-
hydrogen ions in the urine, so that large quantity of acid sium level (hyperkalemia). In general, acute acidosis
could be excreted with minor changes in pH . The excre- is associated with hyperkalemia and acute alkalosis
tion of ammonia helps in the elimination of hydrogen ions with hypokalemia. Sudden hypokalemia may develop
without appreciable change in the pH of the urine. The during the correction of acidosis. K· may go back into
Glutaminase present in the tubular cells can hydrolyze the cells, suddenly lowering the plasma K•. Hence it is
glutamine to ammonia and glutamic acid. The NH3 important to maintain the K• balance during correction
{ammonia) diffuses into the luminal fluid and combines of alkalosis.
with H+ to form NH/ (ammonium ion).
The glutaminase activity is increased in acidosis. So DISTURBANCES IN
large quantity of H+ ions are excreted as NH/ in acido- ACID-BASE BALANCE
sis. Since it is a positively charged ion, it can accompany
negatively charged acid anions; so Na• and K· are con- Acidosis is the clinical state, where acids accumu-
served (Fig. 27.5). The enhanced activity of glutaminase late or bases are lost. A loss of acid or accumulation
and increased excretion of NH4• take about days of base leads to alkalosis (Table 27.3). The body cells
to set in under conditions of acidosis. But once estab- can tolerate only a narrow range of pH. The extreme
lished, it has high capacity to eliminate acid. The titrat- ranges of pH are between 7.0 and 7.6, beyond which life
able acidity plus the ammonia content will be a measure is not possible. Box 27.4 shows the conditions in which
.. of acid excreted from the body. Maximum urine acidity acid-base parameters are to be checked. Box 27 .5 shows
reached is 4.4. A summary of buffering of acid load in the the steps to the clinical assessment of acid base status.
body is shown in Box 27.3. Box 27.6 summarizes the abnormal findings.

TABLE 27.3: Types of acid-base disturbances


Disturbance pH Primary change Ratio Secondary cl,unge
Metabolic acidosis Decreased Deficit of bicarbonate <20 Decrease in Paco,
Metabolic alkalosis Increased Excess of bicarbonate >20 Increase in Paco,
I Respiratory acidosis Decreased Excess of carbonic acid <20 Increase in bicarbonate
I Respiratory alkalosis Increased Deficit of carbonic acid >20 Decrease in bicarbonate
402 Section C: Clinical and Applied Biochemistry

BOX 27.4 : Acid-base parameters are to be checked 1n BOX 27.5: Ste


patients with disturbances
l . Any serious illness 1. Assess pH (mormal 7.4); pH <7.35 is acidemia and > 7.45 is
2. Multi organ failure alkalemia
3. Respiratory failure 2. Serum bicarbonate level: see Box 27.6.
4. Cardiac failure 3. Assess arterial pCO2: see Box 27.6.
5. Uncontrolled diabetes mellitus 4. Check compEmsatory response: Compensation never over-
6. Poisoning by barbiturates and ethylene glycol compensates the pH. If pH is <7.4, acidosis is the primary
disorder. If pH is >7.4, alkalosis is primary.
5. Assess anion 9ap.
BOX 27.6 : Acid-base disturbances 6. Assess the change in serum anion gap/change in bicarbo-
pCO, > 45 mm Hg = Respiratory acidosis nate.
7. Assess if them is any underlying cause.
pCO, <35 mm Hg = Respiratory alkalosis
HCO, > 33mmol/ L = Metabolic alkalosis
HCO3 < 22 mmol/L = Metabolic acidosis
ages of compensation
H• > 45 nmol/L Acidosis
Stage pH HCO, Paco, Ratio
H• < 35 nmol/L = Alkalosis
Metabolic acidosis Low Low N <20
Uncompensated Low Low N <20
Classification of Acid-Base Disturbances Partially compernsated Low Low Low <20
Fully compensated N Low Low 20
Acidosis (Fall in pH) Metabolic alkalosis High High N >20
a. Respiratory acidosis: Primary excess of carbonic acid. Uncompensated High High N >20
b. Metabolic acidosis: Primary deficit of bicarbonate Fully compensat,ed N High High 20
(Box 27.6). Respiratory acidosis Low N High <20
Uncompensated Low N High <20
Alkalosis (Rise in pH) Fully compensat,ed N High High 20
a. Respiratory alkalosis: Primary deficit of carbonic Respiratory alkalosiis High N Low >20
acid. Uncompensated High N Low >20

b. Metabolic alkalosis: Primary excess of bicarbonate Fully compensat,ed N Low Low 20


(Box 27.6).
d. The compensatory change will try to restore the
Compensatory Responses pH to normal. However, the compensatory change
Each of the above disturbance will be followed by a cannot fully correct a disturbance.
secondary compensatory change in the counteracting e. Clinically, acid-base disturbance states may be di-
variable, e.g. a primary change in bicarbonate involves vided into:
an alteration in pCO2• Depending on the extent of the i. Uncompensated
compensatory change there are different stages (Table ii. Partially compensated
27.3). In actual clinical states, patients will have different iii. Fully compensated (Table 27.4).
states of compensation. The compensatory (adaptive)
responses are: Mixed Responses
a. A primary change in bicarbonate involves an altera- If the disturbance is pure, it is !lot difficult to accurately
tion in pCO2 • There is an attempt at restoring the assess the nature of the disturbance. In mixed distur-
ratio to 20 and pH to 7.4. bances, both HCQ3- and H2 CO3 levels are altered
b. Primary decrease in arterial bicarbonate involves (Fig. 27.6). The adaptive response always involves a
a reduction in arterial blood pCO2 by alveolar hyper- change in the couinteracting variable; e.g. a primary change
ventilation. in bicarbonate involves an alteration in pCO2 • Depend-
c. Similarly, a primary increase in arterial pCO2 ing on the extent of the compensatory change there are
involves an increase in arterial bicarbonate by an different stages. Looking at the parameters, the stage
increase in bicarbonate reabsorption by the kidney. of the compensation can be identified (Table 27.4).
Chapter 27: Acid-Base Balance and pH 403

Black Respiratory alkalosis Diabetic ketoaciidosis: Accumulation of acetoacetate


Blue = Respiratory acidosis
Brown = Metabolic acidosis and beta hydroxy butyrate (See Chapter 13).
Red = Metabohc alkalosis
White = Normal Lactic acidosis: Lactic acid in blood is increased in
100
tissue hypoxia, circulatory failure, and intake of biguanides
7.1 (Table 27.5). Lactic acid is compensated by bicarbonate.
So, bicarbonate is lowered and measured; but increased
pH lactate is not measured; thus anion gap is increased.
50
7.4
H+ 7.6
nM/L Normal Anion Gap Metabolic
Acidosis (NAGMA)
8.0
0 When there is a loss of both anions and cations, the
10 50 100
pCO2 mm Hg anion gap is normal, but acidosis may prevail. Causes

Fig. 27.6: Bicarbonate diagram


are described in Table 27.6 .
i. Diarrhea: Loss of intestinal secretions lead to aci-
dosis. Bicarbonate, sodium and potassium are lost.
Biochemistry of Acid-Base Disturbances
ii. Hyperchlommic acidosis may occur in renal tubu-
Metabolic Acidosis lar acidosis, acetazolamide (carbonic anhydrase
i. It is due to a primary deficit in the bicarbonate. inhibitor) therapy, and ureteric transplantation into
This may result from an accumulation of acid or large gut (done for bladder carcinoma).
depletion of bicarbonate.
ii. When there is excess acid production, the bicar- Decreased Anion Gap is Seen in
bonate is used up for buffering. Depending on the • Hypoalbumin,emia: As a general rule of thumb, the
cause, the anion gap is altered. normal anion gap is roughly three times the albumin
value. For example, for a patient with an albumin of
Anion Gap
4.0, the normal anion gap would be 12.
The sum of cations and anions in ECF is always equal , • Multiple myeloma (paraproteinemia)
so as to maintain the electrical neutrality. Sodium and • The gap may be apparently narrowed when cations
potassium together account for 95% of the cations are decreased (K+, Mg2• and Ca2·) or when there is
whereas chloride and bicarbonate account for only 86% hypoalbuminemia.
of the anions (Fig. 27.7). Only these electrolytes are
commonly measured. Compensated Metabolic Acidosis
Hence, there is always a difference between the Decrease in pH in metabolic acidosis stimulates the
measured cations and the anions. The unmeasured respiratory compensatory mechanism and produces
anions constitute the anion gap. This is due to the hyperventilation-l<ussmaul respiration to eliminate car-
presence of protein anions, sulfate, phosphate and bon dioxide leacling to hypocapnia (hypocarbia). The
organic acids. The anion gap is calculated as the differ- pCO2 falls and tlhis would attempt to restore the ratio
ence between (Na• + K• ) and (HCQ3- + Cl-). Normally toward 20 (partial compensation).
this is about 12 mmol/L. Renal compensation: Increased excretion of acid and
conservation of base occurs. Na-H exchange, NH/
High Anion Gap Metabolic excretion and bicarbonate reabsorption are increased.
Acidosis (HAGMA) As much as 500 mmol acid is excreted per day. The
A value between 15 and 20 is due to accumulation of reabsorption of more bicarbonate also helps to restore
acid anions in metabolic acidosis (HAGMA) (Table 27.5). the ratio to 20. Ftenal compensation sets in within 2 to
Renal failure: The excretion of H' as well as gene- 4 days. If the ratio is restored to 20, the condition is said
ration of bicarbonate are both deficient. The anion gap to be fully comp,ensated. But unless the cause is also
increases due to accumulation of other buffer anions. corrected, restoration of normalcy cannot occur.
404 Section C: Clinical and Applied Biochemistry

(+) H (+) H (+) (--) rmal anion gap metabolic ac1dos1s (NAGMA)
+ + +
I I,.. I I 1s)
K ,4- 12 AG K AG K AG
1- Cause Remarks
Hco,-
25 HC0 - Diarrhea, intestinal fistula Loss of bicarbonate and cations.
3 Hco,- Sodium or Potassiu m or both
Na+ 138 105 Cl- Na• Cl
- Na• Cl
- Renal Tubular Acidosis Defective acidification of urine
Normal Acidosis Hyperchlo- Due to inability t o reabsorb
remlc bicarbonate or due to inability to
acidosis excrete hydrogen ions
Compensatory increase in chloride
The figures are the concentration in mEq/L or mmol/L. The red (hyperchloremic acidosis)
shaded area denotes AG (anion gap). In acidosis, the bicarbonate
is reduced causing an increase in the anion gap. In hyperchloremic Carbonic anhydras,e Loss of bicarbonate, Na and K
acidosis, there is no change in the anion gap, but as a compensation, inhibitors Similar to proximal RTA
chloride ions are increased.
Ureterosig mioidosltomy Loss of bicarbonate and reabsorption
Fig. 27.7: Gamblegram showing cations on the left and anions of chloride. Hyperchloremic acidosis
on the right side. Such bar diagrams were first depicted by Gam-
Drugs Antacids containing magnesium,
ble, hence these are called Gamble grams
lithium, polymixin B

TABLE 27.5: High anion gap metabolic ac1dos1s (HAGMA) <10 mmol/L. In such conditions, there is depressed
(organic acidosis) myocardial contiractility.
Cause Remarks
Renal failure Sulfuric, phosphoric, organic anions. Decreased Treatment cif Metabolic Acidosis
ammonium ion formation. Na•/H+ exchange results Treatment is to stop the production of acid by giving IV
in decreased acid excretion
fluids and insulin. Oxygen is given to patients with lactic
Ketosis Acetoacetate; beta hydroxy butyrate anions. Seen
acidosis. In all cases, potassium status to be monitored
in diabetes mellitus or starvation
Lactic
closely and promptly corrected.
Lactate anion. It accumulates w hen the rate of
acidosis production exceeds the rate of consumption Bicarbonate re1quirement: The amount of bicarbonate
Salicylate Aspirin poisoning required to treat acidosis is calculated from the base
Amino Acidic metabolic intermediates deficit. In cases ,of acidosis, mEq of base needed= body
acidurias Accumulation due to block in the normal metabolic wt in Kg x 0.2- base excess in mEq/L.
pathway
Organic Organic acids (methyl malonic acid, propionic
Metabolic Alkalosis
acidurias acid, etc.) excreted
Drugs Corticosteroids, Furosemide, Methanol, Nitrates, Primary excess of bicarbonate is the characteris-
Salicylates, Thiazides tic feature. Alkalosis occurs when (a) excess base is
added, (b) base excretion is defective or (c) acid is lost.
Associated hyperkalemia is commonly seen due to All these will lead to an excess of bicarbonate, so that
a redistribution of K• and H+. The intracellular K• comes the ratio becomes more than 20. Important causes and
out in exchange for H+ moving into the cells. Hence, care findings are givein in Table 27.7. Loss of acid may result
should be taken while correcting acidosis which may from severe vomiting or gastric aspiration leading to loss
lead to sudden hypokalemia. This is more likely to hap- of chloride and acid. Therefore, hypochloremic alka-
pen in treating diabetic ketoacidosis by giving glucose losis results. H)rperaldosteron!sm causes retention of
and insulin together. sodium and loss of potassium.
Hypokalemia is closely related to metabolic alkalo-
Clinical Features of Metabolic Acidosis sis. In a!kalosis, there is an attempt to conserve hydro-
The respiratory response to metabolic acidosis is to gen ions by kidney in exchange for K•. This potassium
hyperventilate. So there is marked increase in respira- loss can lead to hypokalemia. Potassium from ECF will
tory rate and in depth of respiration; this is called as enter the cells in exchange for H+. So, in alkalosis, pH
Kussmaul respiration. The acidosis is said to be of urine remains, acidic; hence this is called paradoxic
dangerous when pH is < 7.2 and serum bicarbonate is acidosis.
Chapter 27: Acid-Base Balance and pH 405

TABLE 27.7: Metabolic alkalos1s


Type Causes Changes
Chloride responsive Prolonged vomiting, Urine chloride < l O mmol/L
alkalosis Nasogastric suction, Hypovolemia, increased loss of Cl, K, H ions
Contraction alkalosis Up per GI obstruction Increased reabsorption of Na with bicarlbonate
Loss of H+ and K•
Hypokalemia leads to alkalosis due to H+-K• exchange. Cl is reabsorbed along with Na
Hence urine chloride is low
Alkalosis responds to administration of !NaCl
Loop Blocks rea bsorption Aldosterone secretion occurs causing Na retention
diuretics of Na, K and Cl and wastage of K• and H+
Chloride Mineralocorticoid excess, Urine chloride > 20 mmol/ L
resistant Primary and secondary Defective renal c1- reabsorption
metabolic hyperaldosteronism,
alkalosis Glucocorticoid excess,
Cushing's, Adrenal tumor
Exogenous Intravenous bicarbonate, Excess base enters the body
base Anatacids,
Milk alkali syndrome

BOX 27.7 : Maximum limits of compensation findings 1n respiratory ac1dos1s


Metabolic acidosis, pCO2 15 mm of Hg pH pCO, HCO,-
Metabolic alkalosis, pCO, 50mm of Hg Acute respiratory acidosis H ii Nor i
Respiratory acidosis, bicarbonate 32 mmol/L
Chronic respiratory ;acidosis ..l. i ii
Respiratory alkalosis, bicarbonate 15 mmol/ L
(partially compensated)
N = normal; ..l. = decreased; i = increased
Clinical Features of Metabolic Alkalosis
The respiratory center is depressed by the high pH lead- be minimal. The findings in chronic and acute respira-
ing to hypoventilation. This would result in accumu- tory acidosis are summarized in Table 27 .8. Chronic
lation of CO 2 in an attempt to lower the HCO3-/H2CO3 cases will be well compensated unlike acute cases. In
ratio. However, the compensation is limited by the respiratory acidosis, bicarbonate level is increased (not
hypoxic stimulation of respiratory center, so that the decreased). Clinically, there is decreased respiratory
increase in PaCO 2 is not above 55 mm Hg (Box 27 .7). rate, hypotension and coma. Hypercapnia may lead to
However, complete correction of alkalosis will beef- peripheral vasoclilation, tachycardia and tremors. The
fective only if potassium is administered and the cause findings in chronic and acute respiratory acidosis are
is removed. summarized in li:ible 27.8.
Increased neuromuscular activity is seen when pH
is above 7.55. Alkalotic tetany results even in the pres- Respiratory Alkalosis
ence of normal serum calcium. A primary deficit of carbonic acid is described as res-
piratory alkalosis. Hyperventilation will result in washing
Respiratory Acidosis
out of CO2. So, bicarbonate:carbonic acid ratio is more
A primary excess of carbonic acid is the cardinal fea- than 20. Causes are hysterical hyperventilation, raised
ture. It is due to CO2 retention as a result of hypoventil- intracranial pressure and brain stem injury. Early stage
lation. The ratio of bicarbonate to carbonic acid will be of salicylate poisoning causes respiratory alkalosis due
less than 20. Acute respiratory acidosis may result from to stimulation of respiratory center. But later, it ends up
bronchopneumonia or status asthmaticus. Depression in metabolic acidosis due to accumulation of organic
of respiratory center due to overdose of sedatives or acids, lactic and keto acids. Other causes include lung
narcotics may also lead to hypercapnia. diseases (pneumonia).
Chronic obstructive lung disease will lead to pCO2 is low, pH is high and bicarbonate level nor-
chronic respiratory acidosis, where the fall in pH will mal. But bicarbonate level falls, when compensation
406 Section C: Clinical and Applied Biochemistry

BOX 27.8: Causes of acid-base disturbances BOX 27.9: Nor


values
. - . . ..... ..... .
Acidosis Alkalosis
A. Respiratory Acidosis A. Respiratory Alkalosis pH = 7.4
Pneumonia High altitude Bicarbonate = 22- 26 mmol/L
Bronchitis, asthma Hyperventilation Chloride 96-1 06 mmol/ L
COPD, pneumothorax Hysteria Potassium 3.5-5 mmol/L
Narcotics, sedatives Febrile conditions 136-145 mmol/L
Sodium
Paralysis of respiratory Septicemia
PO, = 95 (85-100) m m Hg
muscles Meningitis
PCO 40 (35- 45) mm Hg
CNS trauma, tumor Congestive cardiac
Ascites, peritonitis fai lure
Sleepapnea
B. Metabolic Acidosis B. Metabolic Alkalosis U
51 Clinical Case Study 27 .2
i. High anion gap Severe vomiti ng
Diabetic ketosis Cushing syndrome A patient was operated for intestinal obstruction and
Lactic acidosis Milk alkali syndrome
had continuous gastric aspiration for 3 days. Blood
Renal failure Diuretic therapy
ii. Normal anion gap (potassium loss) pH - 7.55, pCO2 - 50 mm Hg, plasma bicarbonate
Renal tubular acidosis - 30 mEq/L, serum sodium-130 mmol/L, serum potassium
(hyperchloremic) - 2.9 mmol/L, se rum chloride- 95 mmol/L. Comment on
CA inhibitors
Diarrhea
the obtained values. What is the significance of potas-
Addison's disease sium in acid base status assessment? Why is chloride
measured in this patient? Calculate and comment on the
occurs. Compensation occurs immediately in acute anion gap.
stages. In prolonged chronic cases renal compensation 0
sets in. Bicarbonate level is reduced by decreasing the • · Clinical Case Study 27.3
reclamation of filtered bicarbonate.
Interpret the data and give the type of acid-base distur-
Clinically, hyperventillation, muscle cramps, tingling
bance. Blood pH - 7.54, pCO2 - 20 mm Hg, plasma
and paresthesia are seen. Alkaline pH will favor increased
bicarbonate - 2fi mEq/L, H2 CO3 - 0.7 mEq/L. What are
binding of calcium to proteins, resulting in a decreased
the causes for the condition?
ionized calcium, leading to paresthesia. Causes of
acidosis and alkalosis are enumerated in Box 27.8. 0
• Clinical Case Study 27 .1 Answer
Normal Serum Electrolyte Values Respiratory acidosis.
See Box 27.9. Students should always remember these
0
values. Upper and lower limits are shown in Box 27.7. • · Clinical Case Study 27.2 Answer
The causes of acid-base disturbances are shown in
Box 27.8. Metabolic alkalo:sis.

Related Topics Identifying the primary process


pH
Renal mechanisms and renal function tests are descri-
bed in Chapter 25. Metabolisms of sodium, potassium
and chloride are described in Chapter 28.

No abnormality or
0 mixed acid-base
• · Clinical Case Study 27.1 disorder
High Low
Interpret the data and give the type of acid base distur- pC02 pC02
bance. Blood pH - 7.12, pCO2 - 80 mm Hg, Plasma
Bicarbonate- 26 mEq/L, H2CO3 - 20.7 mEq/L. What are Respiratory Respiratory Metabolic
acidosis acidosis acidosis
the causes for the condition?
Chapter 27: Acid-Base Balance and pH 407

e 11 . Respiratory system regulates pH by controlling


,a. Clinical Case Study 27.3 Answer pCO2. When there is a fall in pH (increase in W),
the respirallory centre is stimulated and CO2 is
Respiratory alkalosis.
washed out by hyperventilation. A rise in pH (fall
in W), will depress the respiratory center, causing
l!:_EARNING POINTS, CHAPTER 27
hypoventila1tion to retain more CO2 •
1. The pH of plasma is 7.4. The regulation is by buff- 12. Respiratory compensation acts immediately, but
ers, lungs and kidney. cannot proceed or persist after a certain limit.
2. Buffer systems of the body are bicarbonate, phos- 13. Kidney regl.llates pH by net excretion of acid so that
phate, Hb, proteins. urine will ha1ve a pH less than glomerular filtrate.
3. Bicarbonate buffer system is quantitatively the most 14. Renal regullation mainly involves reabsorption of
significant among body buffers. filtered bicarbonate (alkali reserve).
4. Anion gap is the unmeasured anions. Normal value 15. Excretion of W is achieved by exchanging W with
is about 12 + 5 mM/L.
Na• which is conserved.
5. Metabolic acidosis is due to primary deficit in
16. The W excreted into the tubular fluid is eliminated
bicarbonate while respiratory acidosis is due to a
as titratable acid or NH 4 • ions.
primary excess of carbonic acid.
17. The renal compensatory mechanism is effective
6. Metabolic alkalosis is due to primary excess of
than respiratory regulation and effect of buffers, but
bicarbonate, while respiratory alkalosis is due to
takes more time to set in.
primary deficit of carbonic acid .
18. Renal W ion concentration in exchange for Na• is
7. Metabolic acidosis is seen during renal tubular aci-
mediated by Na-W exchanger or W ATPase. Bicar-
dosis, diabetic ketosis and organic acidemias.
8. Metabolic alkalosis occurs in hyperaldosteronism. bonate is re1absorbed along with so that alkali
hypokalemia and Cushing's syndrome. reserve is oonserved/replenished.
9. Respiratory acidosis may result from broncho- 19. When W leivel decreases (alkalosis), more Na is
pneumonia and chronic obstructive lung disease. exchanged for K• leading to potassium loss and
10. Respiratory alkalosis results from hysteria, raised hypokalemia. An increase in W leads to exchange
intra cranial pressure and salicylate poisoning. of W for K• causing K• retention and hyperkalemia.

PART-1: ESSAY AND SHORT NOTE UESTIONS

27-1 . What is the normal pH of blood? Explain the role of plasma buffers and renal mechanisms in the maintenance
of acid base balance of the body.
27-2. Name the important buffer systems in the body. Describe the role of kidlney and lungs in the maintenance of acid
base balance.
27-3. What is titratable acidity of urine? What is the role of kidney in maintaii ning acid base balance?
27-4. What is metabolic acidosis? Enumerate its causes. What are the compensatory mechanisms?

SHORT NOTE QUESTION

27-5. Bicarbonate buffer system of blood. 27-11 . Anion gap.


27-6. Alkali reserve. 27-12. Metabolic: acidosis.
27-7. Renal acidification of urine. 27-13. Respirato,ry acidos is.
27-8. Renal production al)d excretion of ammonia. 27-14. Metabol ic: alkalosis.
27-9. Urinary ammonia. 27-15. Respirato,ry alkalosis.
27-10. Role of kid ney in t he regulation of pH.
408 Section C: Clinical and Applied Biochemistry

PART-2: MULTIPLE CHOICE QUE TIONS

27-1 . Causes of metabolic alkalosis include the follow- C. Poliomyelitis


ing, except D. Artificial ventilation
A. Prolonged vom iting 27-1 o. Nl)n-respi1ratory alkalosis can result from:
B. Continuous gastric aspiration A. Persist.en! vomiting
C. Hyper ventilation B. Salicyl:ate poisoning
D. Ingestion of antacids C. Chronic renal failure
27-2. Biochemical findings in diabetic ketoacidosis in- D. Cyanotic heart disease
clude the following, except 27-11 . Proximal renal tubular acidosis is characterized by
A. pCO2 is lowered or normal all except
B. Increased anion gap A. Metabolic acidosis
C. Lowered bicarbonate level B. Normal anion gap
D. Elevation of plasma chloride C. Urine pH of 4.5
27-3. In respiratory acidosis, patient will show which of D. Hyperchloremia
the following features? 27-12. ABG data:: pH -7.47, pCO2 -30 mm of Hg, Bicarbo-
A. A low pCO2 nate -19 mmol/L. This suggests what?
B. Lowered plasma bicarbonate A. Acute respiratory alkalosis
C. Reduced rate and depth of respiration
B. Partially compensated respiratory alkalosis
D. Decreased hydrogen ion concentration in plasma
C. Partially compensated metabolic acidosis
27-4. A patient treated with carbonic anhydrase inhibitor
D. Mixed respiratory and metabolic alkalosis
may develop:
27-13. In the blood (pH= 7.4), which of the following buf-
A. Metabolic acidosis
fer system is most effective?
B. Respiratory acidosis
A. Citric aicid buffer (pK = 3.0)
C. Respiratory alkalosis
B. Carbonic acid buffer (pK = 6.1)
D. Metabolic alkalosis
C. Phosphate buffer (pK = 6.8)
27-5. Administration of diuretics cause loss of potas-
D. Glutamate buffer (pK = 3.2)
sium, which may lead to:
27-14. Identify a<:id base disturbance in this patient: pH
A. Metabolic acidosis
7.56, pCO,t 54 mm of Hg, Bicarbonate -45 mmol/L:
B. Respiratory acidosis
A. Chroni,c respiratory acidosis
C. Respiratory alkalosis
D. Metabolic alkalosis B. Mixed respiratory acidosis and alkalosis
27-6. Patients with diarrhoea may develop the following C. Partially compensated metabolic alkalosis
complications, except D. Uncompensated respiratory acidosis
A. Metabolic alkalosis 27-15. Which of 1the following parameters indicate com-
B. Hypertonic contraction pensated !State in a case of respiratory alkalosis:
C. Dehydration A. High pCO2
D. Metabolic acidosis B. Low anion gap
27-7. Which of the following conditions will produce a C. Low bic::arbonate
high anion gap acidosis? D. Low potassium level
A. Diarrhea 27-16. All are caL1set~ of metabolic acidosis, except
B. Renal tubular acidosis A. Ureterosigmoidostomy
C. Renal failure B. Ingestion of antacids
D. Ureterosigmoidostomy C. Addison's disease
27-8. Metabolicalkalosis is characterized by: D. DiabetHs mellitus
A. High hydrogen ion concentration in plasma 27-1 7. When pH falls by 1 unit, the hydrogen ion concen-
B. Increased pCO2 tration:
C. Elevated chloride level in serum A. Decreases 10 times
D. Low potassium level B. Increases two fold
27-9. Respiratory acidosis can result from all the follow- C. ChangE~S by 7 times
ing, except: D. !increases 10 times
A . Bronchial asthma 27-18. A patient with COPD (chronic obstructive pulmo-
B. Narcotic poisoning nary disease) on loop diuretics may have:
Cha1pter 27: Acid-Base Balance and pH 409

A. Respiratory and metabolic acidosis C. Hyperkalemia


B. Respiratory and metabolic alkalosis D. Decrease in hydrogen ions
C. Respiratory acidosis and metabolic alkalosis 27-21. In compensated metabolic alkalosis:
D. Respiratory alkalosis and metabolic acidosis A. Rrespiratory center is stimulated
27-19. In a case of partially compensated respiratory aci- B. pC02 increases
dosis, the findings in plasma include:
C. p02 e'levated
A. Normal pC02 level
D. Urine becomes acidic
B. Elevated bicarbonate level
27-22. Rapid correction of acidosis in a patient with hy-
C. Increased pH
poxia leads to:
D. Bicarbonate to carbonic acid ratio is 20
A. Improvement of p02
27-20. Overcorrection of acidosis can cause all the fol-
lowing complications, except B. Decre·ased tissue oxygenation
A. Shift of oxygen dissociation curve (ODC) to left C. Decre·ased affinity of hemoglobin for oxygen
B. Sudden hypoxia D. Examierated 2,3 BPG effect

ANSWERS OF MULTIPLE CHOICE


-- --
UESTIONS
27-1 . C 27-2. D 27-3. C 27-4. A 27-5. D 27-6. A 27-7. C
27-8. D 27-9. D 27-10. A 27-11 . C 27-12. B 27-13. B 27-14. C
27-15. B 27-16. B 27-17. D 27-18. C 27-19. B 27-20. C 27-21 . B
27-22. B

PART-3: VIVA VOCE QUESTIONS AN ANSWERS

27-1. When pH falls by 1 unit, what is the change in the In acidosis, there is hyperventillation, so CO2 is low-
hydrogen ion concentration? ered, and H 2C03 is lowered.
Increases 10 times. 27-12. Glutaminase enzyme is used for what purpose?
27-2. Relationship between pH and pK is given by which For production of ammonia in kidney tubules Figure 27 .5.
equation? 27-13. What is metabolic acidosis?
Henderson-Hasselbalch's equation. Primary d eficit of bicarbonate.
27-3. What determines the pH of buffer? 27-14. What are the causes of metabolic acidosis?
By the ratio of salt to acid. Diabetic ketosis, chronic renal fail ure, diarrhea.
27-15. What are the features of diabetic ketoacidosis?
27-4. What determines the buffering capacity?
Lowered bicarbonate; elevated plasma chloride;
The absolute concentrations of the salt.
increased anion gap.
27-5. What is meant by buffering capacity?
27-16. How to calculate anion gap?
The buffering capacity of a buffer is defined as the abil-
(sodium+ potassium) minus (chloride + bicarbonate).
ity of the buffer to resist changes in pH when an acid or
27-17. What is tine cause for high anion gap acidosis?
base is added.
Diabetic ketoacidosis, chronic renal failure, renal tubu-
27-6. Buffer is most effective at which pH?
lar acidosis, lactic acidosis.
When the pH is nearer to the pK of the acid.
27-18. What is metabolic alkalosis?
27-7. In the blood, which buffer is most effective? Primary excess of bicarbonate.
Bicarbonate buffer. 27-19. What are the causes of metabolic alkalosis?
27-8. What are the mechanisms for maintaining the nor- Prolonged vomiting; gastric aspiration; and ingestion
mal pH of plasma? of antacids.
Buffers of plasma; Lung mechanism; Kidney mechanism. 27-20. What is r,espiratory acidosis?
27-9. What is the alkali reserve of the body? Primary excess o f carbonic acid.
Bicarbonate is the alkali reserve. 27-21 . What are the causes of respiratory acidosis?
27-10. What is the normal ratio of bicarbonate to carbonic Bronchial asthma; bronchopneumonia; narcotic poisoning.
acid in blood? 27-22. What is r,espiratory alkalosis?
Bicarbonate to carbonic acid ratio is 20. Primary deficit of carbonic acid.
27-11 . What is the respiratory mechanism of pH regu- 27-23. What is tlhe cause for respiratory alkalosis?
lation? Hyperventillation, as in hysteria, salicylate poisoning.
_ _ _ _ _Chapter 28
Electrolyte and
Water Balanice

Chapter at a Glance
The learner will be able to answer questions on the following topics:
0 Intake and output of water O lsotonic/hypotonic/hypertonic contraction, ECF
0 0smolality of extracellular fluid O lsotonic/hypotonic/hyperton ic expansion, ECF
0 Electrolyte composition of body fluids O Sodium metabolism
0 Regulation of sodium and water balance O Potassium metabolism
Renin-angiotensin system O Chloride meltabolism

The maintenance of extracellular fluid volume and pH The thirst center is stimulated by an increase in the
are closely interrelated. The body water compartments osmolality of blood, leading to increased intake.
are shown in Box 28.1. Body is composed of about 60- The renal function is the major factor controlling the
70% water. Distribution of water in different body water
rate of output. Thie rate of loss through skin is influenced
compartments depends on the solute content of each
by the weather, the loss being more in hot climate (per-
compartment. Osmolality of the intra- and extracellular
spiration) and less in cold climate. Loss of water through
fluid is the same, but there is marked difference in the
solute content. skin is increased to 13% for each degree centigrade rise
in body temperature during fever.
IINTAKE AND OUTPUT OF WATER OSMOLALITY OF ECF
One gram carbohydrate, during oxidation produces
0.6 ml of water, 1 g protein releases 0.4 ml water and Osmolarity means osmotic pressure exerted by the
1 g fat generates 1.1 ml of water. Intake of 1000 kcal number of moles, per liter of solution. Osmolality is the
produces 125 ml water (Table 28.1 ). The major factors osmotic pressum exerted by the number of moles per
controlling the intake are thirst and the rate of meta- kg of solvent. Crystalloids and water can easily diffuse
bolism. across membranes, but an osmotic gradient is provided

BOX 28.1: The body water compartment er balance 1n the body

Total body water (42 l ) (60% of body weight) Intake pei· day Output per day

Water in food 1250 ml Urine 1500 ml


Intracellular (28 l ) Extracellular (14 l) Oxidation of 300ml Skin 500ml
(40% of body weight) (20% of body weight) food

Drinking water 1200ml Lungs 700ml

lntravascular Extravascular Feces 50ml


(4%) (2.8 l ) (16%) {11 .2 l )
2750ml 2750 ml
Chapter 28: Electrolyte and Water Balance 411

(+) (-) TABLE 28.2: Electrolyte concentration of body fluid compart-


ments
Pr
Mg Interstitial Intracellular fluid
(+) (- )
(+) (-) Solutes Plasma mEq I L fluid (mEq/L) (mEq/ L)
K+ 5 Cations:
K+ HC03-
HC03- Sodium 140 146 12
Potassium 4 5 160
Na• 146 K+ 160
- 5 3 -
Cl
- Cl 140 P0 4
Calcium
Magnesium 1.5 1 34
10 HC03-
Anions:
Na+ 12 Cl
Chloride 105 117 2
Plasma Interstitial fluid Intracellular fluid
Bicarbonate 24 27 10
Fig. 28.1: Gamblegrams showing compositio n of fluid compart-
m ents (See also Table 28.2)
Sulfate 1 1 -
Phosphate 2 2 140
Protein 15 7 54
TABLE 28.3: Osmolahty of plasma (In mmol/kg)
Other anions 13 1 -
Solute
92% Note - mEq/ L = mmol/ L x valency
Sodium with anions 270
Po tassium w ith anions 7
Calcium with anions 3 BOX 28.2: S ummary of ECF and ICF

Mag nesium with anions 1. At equilibrium, the osmolality of extracellular fluid (ECF) and
intracellular fluid (ICF) are identical
Urea 5 8%
2. Solute content of ICF is constant
Glucose 5 3. Sodium is retained only in the ECF
Protei ns 4 . Total body solute d ivided by t otal body water gives the body
292 fluid osmolality

by the non-diffusible colloidal (protein) particles. The col- reduced (dilutional hyponatremia). Hence, the plasma
loid osmotic pressure exerted by proteins is the major sodium is a reliable index of total and effective osmola-
factor which maintains the intracellular and intravascular lity in the normal and clinical situations. See summary in
fluid compartments. Albumin is mainly responsible in Box 28.2.
maintaining this osmotic balance (see Chapter 26). The
composition of each body fluid compartment is shown in Regulation of Sodium and Water Balance
Figure 28.1 and Table 28.2.
Since osmolality is dependent on the number of The major regulatory factors are the hormones (aldo-
solute particles, the major determinant factor is the sterone, ADH) and the renin-angiotensin system.
sodium. Therefore, sodium and water balance are Aldosterone secreted by the zona glomerulosa of
dependent on each other and cannot be considered the adrenal cortex regulates the Na• --> K• exchange and
separately. The osmolality of plasma varies from 285 Na• --> H· exchange at the renal tubules. The net effect
to 295 mosm/kg (Table 28.3). It is maintained by the kid- is sodium retention .
ney, which excretes either water or solute as the case
may be. Antidiuretic Hormone (ADH)
When osmolality of the plasma rises, the osmoreceptors
Effective Osmolality of hypothalamus are stimulated, resulting in ADH secre-
Permeable solutes, such as urea and alcohol enter into tion. ADH will increase the water reabsorption by the renal
the cell and achieve osmotic equilibrium. On the other tubules. Therefore , proportionate amounts of sodium
hand , if impermeable solutes like glucose, mannitol, and water are retained to maintain the osmolality.
etc. are present in ECF, water shifts from ICF to ECF When osmolality decreases, ADH secretion is inhi-
and extracellular osmolality is increased. So, for every bited. When ECF volume expands, the aldosterone
100 mg/dl increase in glucose, 1.5 mmol/L of sodium is secretion is cut off.
412 Section C: Clinical and Applied Biochemistry

BOX 28.3: Rernn and Rennin are different


Kidney secretes Renin; it is involved in fluid balance and hyper-
: . ay of ang1otens1n production
Renin
tension. Angiotensinogen - - -• Angiotensin-1
Rennin is a proteolytic enzyme seen in gastric juice, especially (453 amino acids) (10 amino acids)
in children.
Angiotensin-converting enzyme
Angiotensin-1- - -• Angiotensin-11 (8 a.a.)

Drop in BP tJalllr Liver Amino peptidase


Angiotens,in-11 - - -• Angiotensin-111 (7 a.a.)
{ Kidney
Kidney ,i ~ Angiotensinogen
•-- Angiotensinase
~ Renin - ! Angiotensin-11and Ill - -- • Degradation p roducts
Angiotensin I

!~
Angiotensin II ACE
Factor
trol of sodium and water
Acting through Effect

Hypo-
thalamus
l i cortex
Adrenal
Extracellular
osmolality
Thirst and ADH Water intake;
Rea bsorpt ion of
water from kidney
Hypovolemia Stimulation of Retention of w ater

L
thirst and ADH
-do- Stimulates aldosterone Retention of sodium
Thirst and ADH Vasoconstriction Aldosterone
Expansion of ECF Inhibits Reabsorption of

Increase in
ECFvolume
1 Kidney Hypo-osmolality
aldosterone
Inhibits ADH
secretion
sodium
Reabsorption of water

Elevated BP - - Salt and water retention

Fig. 28.2: Renin-angiotensin-aldosterone Autoregulation


Renin-Angiotensin System Angiotensin-1I increases blood pressure by causing
vasoconstriction of the arterioles. It also inhibits renin
When there is a fall in ECF volume, renal plasma flow release from the juxtaglomerular cells. The events thus
decreases and this would result in the release of renin leading to maintenance of sodium and water balance as
by the juxtaglomerular cells (Box 28.3). The factors
well as ECF volume are summarized in Figure 28.2.
which stimulate renin release are:
Table 28.4 gives the physiological stimuli involved in
a. Decreased blood pressure
the control of sodium and water balance.
b. Salt depletion
c. Prostaglandins.
The inhibitors of renin release are:
Disturbanceis in Fluid and
a. Increased blood pressure Electrolyte !Balance
b. Salt intake Assessment of fluid and electrolyte balance is sum-
c. Prostaglandin inhibitors marized in Box 28.5. When the effective osmolality is
d. Angiotensin-11. Renin is the enzyme acting on the increased, the body fluid is called hypertonic and when
angiotensinogen (an alpha-2 globulin , made in liver) osmolality is decreased the body fluid is called hypo-
(Box 28.4). tonic. A classification is given in Table 28.5.
Clinical effects of increased effective osmolality are
Clinical Significance due to dehydration of cells. A sudden reduction of effec-
Angiotensin-converting enzyme (ACE) is a glycoprotein tive osmolality rnay cause brain cells to swell leading to
present in the lung. ACE-inhibitors are useful in treating headache, vomiting and medullary herniation.
edema and chronic congestive cardiac failure. Various Some impo,rtant clinical features of electrolyte
peptide analogs of Angiotensin-11 (Saralasin) and anta- imbalance are shown in Box 28.6. Different types of
gonists of the converting enzyme (Captopril) are useful abnormalities due to disturbances in fluid and electrolyte
to treat renin-dependent hypertension. balance are given below:
Chapter 28: Electrolyte and Water Balance 413

BOX 28.5: Assessment of sodium and water balance • urbances of fluid volume

1. Maintenance of intake-output chart, in cases of patients on Abnormality Biochemical features Osmolality


IV fluids. Expansion of ECF
2. Measurement of serum electrolytes (sodium, potassium, Isotonic Retention o f water Normal
chloride and bicarbonate). This will give an idea of the ex- Hypotonic Relative water excess Decreased
cess, depletion. Hypertonic Relative sodium excess Increased
3. Measurement of hematocrit value to see if there is hemo- Contraction of ECI'
concentration or dilution Isotonic Loss of Na• and water Normal
4. Measurement of urinary excretion of electrolytes, especially Hypotonic Relative loss of Na• Decreased
sodium and chloride. Hypertonic Relative loss of water Increased

BOX 28.6: Clinical features of electrolyte imbalance b. Vomiting an1d excessive sweating can also cause a
1. Hypo-osmolatiy and hyponatremia go hand in hand similar situation.
2. Hypo-osmolality causes swelling of cells and hyper-
c. The increase in osmolality will stimulate thirst and
osmolality causes dehydration of cells
3. Hyponatremia of ECF causes symptoms only when asso-
increase in the water intake. ADH secretion occurs
ciated with hyperkalemia and urine volume decreases.
4. Fatigue and muscle cramps are the common symptoms of
electrolyte depletion Isotonic Expansion
5. Hypo-osmolality of gastrointestinal cells causes nausea,
vomitin and paralytic ileus. Water and sodium retention is often manifested as
edema and occurs secondary to hypertension or car-
diac failure. Heimodilution is the characteristic finding .
Isotonic Contraction
Secondary hyp1eraldosteronism often results from
This results from the loss of fluid that is isotonic with
hypoalbuminemia (edema in nephrotic syndrome, protein
plasma. The most common cause is loss of gastroin-
malnutrition , etc.). In these cases, the water retention
testinal fluid. Since equivalent amounts of sodium and
causes ADH secretion. The intravascular volume can-
water are lost, the plasma sodium is often normal.
Hemoconcentration is seen. In severe cases, hypoten- not be restored since the low colloid osmotic pressure
sion may occur. Hypovolemia may be seen. Com- tends to drive the fluid out into the extravascular space,
pensatory mechanisms will try to restore the volume. aggravating the edema.
Renin-aldosterone system is activated, and selective
sodium reabsorption occurs. ADH secretion leads to Hypotonic Expansion
reabsorption of equivalent amounts of water. There is water retention either due to glomerular dys-
function or ADH excess. The water excess will lower the
Hypotonic Contraction
osmolality. Hyponatremia persists due to the inhibition
There is predominant sodium depletion. Most important of aldosterone secretion by the expanded ECF volume.
cause is infusion of fluids with low sodium content like
dextrose. The hypo-osmolality will inhibit ADH secretion Hypertonic Expansion
resulting in water loss. Since only the excess fluid is
lost, the plasma sodium tends to return to normal. Thus, It can occur in cases of Conn's syndrome and Cush-
osmolality is restored, but at the expense of the volume. ing's syndrome. The excess mineralocorticoid would
Therefore in postoperative cases, care should be pro-duce sodium retention. There is associated hypoka-
taken to adequately replace sodium by giving sufficient lemia which often leads to metabolic alkalosis. Extracel-
quantity of normal saline. lular hypertonicity may lead to brain cell dehydration,
leading to coma and death.
Hypertonic Contraction
It is predominantly water depletion.
a. The commonest cause is diarrhea, where the fluid Sodium level is intimately associated with water balance
lost has only half of the sodium concentration of the in the body. Sodium regulates the extracellular fluid
plasma. volume. Total body sodium is about 4000 mEq. About
414 Section C: Clinical and Applied Biochemistry

BOX 28.7: Causes of hypernatrem1a


1. Cushing's disease
: .
1. Vomiting
es of hyponatrem1a

2. Prolonged cortisone therapy 2. Diarrhea


3. In pregnancy, steroid hormones cause sodium retention in 3. Addison's disease (adrenal insufficiency)
the body 4. Renal tubular acidosis (tubular reabsorption of sodium is de-
4. In dehydration, when water is predominantly lost, blood fect ive)
volume is decreased with apparent increased concentration 5. Chronic renal failure, nephrotic syndrome
of sodium 6. Congestive cardiac failure
5. Exchange transfusion with stored blood 7. Hyperglycemia and ketoacidosis

50% of it is in bones, 40% in extracellular fluid and 10% retention of wat,er. This vicious cycle is broken when
in soft tissues. Sodium is the major cation of extra- aldosterone anta1gonists are administered as drugs.
cellular fluid.
Sodium pump is operating in all the cells, so as Hypernatrern ia
to keep sodium extracellular. This mechanism is ATP Increased sodium in blood is known as hypernatremia.
dependent (see Chapter 2). Sodium (as sodium bicar- Symptoms of hypernatremia include dry mucous mem-
bonate) is also important in the regulation of acid-base brane, fever, thirst and restlessness. Causes of hyper-
balance (see Chapter 27). natremia are Cushing's disease, prolonged cortisone
Normal level of Na+ in plasma 136-145 mEq/L and therapy and prennancy, where steroid hormones cause
in cells 12 mEq/L. sodium retention in the body. Other causes are enume-
Normal diet contains about 5-10 g of sodium, mainly rated in Box 28.7.
as sodium chloride. The same amount of sodium is daily
excreted through urine. However, body can conserve Hyponatremia
sodium to such an extent that on a sodium-free diet Decreased sodiuim level in blood is called hypona tremia.
urine does not contain sodium. Ideally dietary sodium Clinical signs and symptoms of hyponatremia include
intake should be lower than potassium, but processed dehydration, drop in blood pressure, lethargy, confusion,
foods have increased sodium intake. tremors and corna. Causes of hyponatremia are shown
Normally kidneys are primed to conserve sodium in Box 28.8, most important causes being vomiting, diar-
and excrete potassium. Sodium excretion is regulated at rh ea, and adrenal insufficiency.
the distal tubules. Aldosterone increases sodium reab- Hyponatremia due to water retention is the com-
sorption in distal tubules. Antidiuretic hormone (ADH) monest biochemical abnormality observed in clinical
increases reabsorption of water from tubules. practice. This has to be treated with care, using diuretics
Different mechanisms are: (a) Sodium hydrogen and restricted flu id administration. Isotonic fluids have
exchanger located in the proximal convoluted tubules the same concentration of solutes as cells. So, cells will
and ascending limb; (b) Sodium chloride cotransporter remain viable in isotonic solutions. Hypertonic fluids
in the distal tubules (ascending limb); and (c) Sodium have a higher concentration of solutes (hyperosmolality)
potassium exchanger in the distal tubule. These are than is found inside the cells. This causes cells to shrink.
explained in Chapter 27, under renal regulation of pH. Hypotonic fluids have a lower concentration of solutes
(hypo-osmolality) than is found inside the cells, which
Edema causes fluid to flow into cells and out of the extracellular
In edema, along with water, sodium content of the body spaces. This causes cells to swell and possibly burst.
is also increased. When diuretic drugs are administered, Cellular dehydration and swelling are harmful, as both
they increase sodium excretion. Along with sodium, affect the brain cells.
water is also eliminated.
In the early phases of congestive cardiac failure, Treatment of Hyponatremia
hydrostatic pressure on venous side is increased; so Effects of administered sodium should be closely moni-
water is primarily retained in the body. This causes dilu- tored, but only after allowing sufficient time for distribu-
tion of sodium concentration, which triggers aldosterone tion of sodium, a minimum of 4 to 6 hours. Water res-
secretion . This is known as secondary hyperaldostero- triction, increased salt intake, fu rosemide and anti-ADH
nism. Thus sodium is retained, along with further drugs are the basis of treatment for hyponatremia.
Chapteir 28: Electrolyte and Water Balance 415

The correction of hypernatremia is to be done with


care to prevent sudden overhydration and water intoxi-
: . ses of hypokalem1a
1. Increased mnal excretion
cation. Serum concentration of sodium is generally - Cushing's syndrome
- Hyperaldosteronism
measured directly by ion selective electrodes. - Renal tubular acidosis

IPOTASSIUM (K+) - Adrenoqenital syndrome


2. Shift or red1istribution of potassium
- Alkalosis
Total body potassium is about 3500 mEq, out of which
- Insulin therapy
75% is in skeletal muscle. Potassium is the major 3. Gastrointes,tinal loss
intracellular cation, and maintains intracellular osmotic - Diarrhea, vomiting, aspiration
pressure. - Deficient intake or low potassium diet
The depolarization and contraction of heart require 4. lntravenou:s saline infusion in excess
5. Drugs
potassium. During transmission of nerve impulses,
- Insulin
there is sodium influx and potassium efflux; with depo- - Osmotic diuretics
larization. After the nerve transmission, these changes - Corticoi;teroids
are reversed.
The intracellular concentration gradient is main-
Signs and symptoms: Hypokalemia is manifested as
tained by the Na· -K· ATPase pump.
muscular weakness, fatigue, muscle cramps, hypoten-
Requirement sion, decreased reflexes, palpitation, cardiac arrythmias
and cardiac arrest. ECG waves are flattened, T wave
Potassium requirement is 3-4 g per day.
is inverted, ST segment is lowered with AV block. This
Sources may be corrected by oral feeding of orange juice. Potas-
sium administration has a beneficial effect in hyperten-
Sources rich in potassium, but low in sodium are
sion.
banana, orange, apple, pineapple, almond, dates, beans,
Redistribution of potassium can occur following
yam and potato. Tender coconut water is a very good
source of potassium. insulin therapy. For diabetic coma, the standard treatment
is to give glucose and insulin. This causes entry of
Normal Level glucose and potassium into the cell and hypokalemia may
be induced. K· should be supplemented in such cases.
Plasma potassium level is 3.5--5.2 mmol/L. The cells
Redistribution is also seen in alkalosis, where the
contain 160 mEq/L; so precautions should be taken to
potassium moves into the cell in exchange for H'.
prevent hemolysis when taking blood for potassium esti-
Diuretics used for congestive cardiac failure may
mation. The K· in serum is estimated directly by using an
cause K• excretion; hence potassium supplementation
ion selective electrode. Excretion of potassium is mainly
is the standard treatment along with diuretics.
through urine. Aldosterone and corticosteroids increase
the excretion of K•. On the other hand, K• depletion will
inhibit aldosterone secretion . Treatment of Hypoka/emia
Supplement adequate potassium (200 to 400 mmol for
Potassium Excretion every 1 mmol fall in serum potassium). Relatively large
The exchange of potassium for sodium at the renal doses can be given orally. Potassium solutions are
tubules is a mechanism to conserve sodium and excrete irritant to peripheral veins; it is preferable to give through
potassium. This is controlled by aldosterone. In chronic a central line. Serum potassium should be checked
renal failure, hyperkalemia is seen since the failing every hour throughout the therapy.
kidney is unable to handle the potassium load.

Hypokalemia Hyperkalernia
This term denotes that plasma potassium level is below Plasma potassium level above 5.5 mmol/L is known
3 mmol/L. A value less than 3.5 mmol/L is to be viewed as hyperkalemia. Since the normal level of K• is kept
with caution. Mortality and morbidity are high. Box 28.9 at a very narrow margin , even minor increase is life-
shows the causes of hypokalemia. threatening.
416 Section C: Clinical and Applied Biochemistry

BOX 28.10: Causes of hyperkalem1a -.. tnbut,on of serum potassium

1. Decreased renal excretion of potassium Increases K• entry Impairs K• entry into cells
into cells leading or exit of K• from cells;
- Renal failure
to hypokalemia hyperkalemia
- Severe volume depletion (heart failure)
Insulin Glucagon
2. Entry of potassium to extracellular space Alkalosis Acidosis
- Increased hemolysis Increased osmolality

.. ..
- Tumor lysis after chemotherapy
- Excess potassium supplementation
BOX 28.11 : Lab ' e I I • .
3. Redistribution of potassium to extracellular space mahties
- Metabolic acidosis
1. Serum potassium estimation
- Tissue hypoxia 2. Urine potassium: Low value (< 20 mmol/L) is seen in poor
4. Transmembrane shift intake, GIT loss or transmembrane shift. High (> 40 mmol/L)
5. Pseudohyperkalemia is seen in renal diseases
- Factitious (K• leaches out when blood is kept for a long 3. Sodium and Osmolality of spot urine: Low sodium (< 20
time before separation) mmol/ L) and high potassium indicate secondary hyper-
aldosteronism.
- Improper blood collection (hemolysis)
4. ECG in all cases
6. Hyperkalemic periodic paralysis
7. Drugs
-
-
Spironolactone
ACE inhibitors
LcHLoR,oE (._c _,_
n _ __
- Beta blockers Intake, output and metabolism of sodium and chloride
run in parallel. The homeostasis of Na•, K• and c 1- are
interrelated. Chloride is important in the formation of
In hyperkalemia, there is increased membrane hydrochloric acid in gastric juice (see Chapter 24).
excitability, which leads to ventricular arrythmia and Chloride ions are also involved in chloride shift (see
ventricular fibrillation, bradycardia and cardiac arrest. Chapter 23).
ECG shows elevated T wave, widening of QRS complex Chloride concentration in plasma is 96-106 mEq/L
and lengthening of PR interval. and in CSF, it is about 125 mEq/L. Chloride concentra-
Causes of hyperkalemia are shown in Box 28.10. tion in CSF is higher than any other body fluids. Since
True potassium excess results from decreased urinary CSF protein content is low, Cl- is increased to maintain
output, increased hemolysis and tissue necrosis. Donnan membrane equilibrium.
Redistribution occurs in metabolic acidosis, insulin Excretion of Cl- is through urine, and is parallel
deficiency and tissue hypoxia (Table 28.6). to Na•.
Pseudohyperkalemia is seen in hemolysis, throm-
bocytosis, leukocytosis or polycythemia; in these condi- Hyperchloremia is seen in
tions, potassium from within the cells will leak out into 1. Dehydration
plasma when the sample is collected. 2. Cushing's syndrome. Mineralocorticoids cause
Box 28.11 shows the conditions in which potassium increased reabsorption from kidney tubules
estimations are required. 3. Severe diarrhea leads to loss of bicarbonate and
compensatory retention of chloride
Treatment of Hyperkalemia
4. Renal tubular acidosis.
If serum potassium is > 6.5 mmol/L, emergency treat-
ment as intravenous glucose and insulin , should be Causes for Hlypochloremia
given. This stimulates glycogen synthesis. When 1 g 1. Excessive vomiting. HCI is lost, so plasma Cl- is
of glycogen is stored , 0.3 mM of K• is simultaneously lowered. ThE!re will be compensatory increase in
trapped intracellularly. So the serum K• is rapidly plasma bicarbonate. This is called hypochloremic
decreased. Continuous ECG monitoring should be done, alkalosis.
as sudden hypokalemia can occur. If patient is acidotic, 2. Excessive sweating.
give sodium bicarbonate. This will correct acidosis and 3. In Addison's disease, aldosterone is diminished,
help in shifting K• into the cells; but volume overload is renal tubular reabsorption of Cl- is decreased, and
to be monitored. more Cl- is e:(creted.
Chapter 28: Electrolyte and Water Balance 417

TABLE 28.7: Regulation mechanisms of electrolytes e


Sodium (Na•) Aldosterone, Antidiuretic hormone ,a. Clinical Case Study 28.3
(ADH) -water regulation
Atrial natriuret ic peptide (ANP)
A 70-year-old man with depression and weakness was
Renal reabsorption admitted to the Emergency Department. He was clini-
Renal excretion cally dehydrate1d, skin was lax and lips and tongue
Potassium (K·) Intestinal absorption were dry and shriveled looking . Pulse 104/min , BP
Aldosterone 95/65 mm Hg. Serum sodium 162 mmol/L, Potas-
Renal reabsorption
sium 3.7 mmol/ L, Chloride 132 mmol/L, Bicarbonate
Calcium (Ca-) Parathyroid hormone
17 mmol/L, blood urea 90 mg/dL, and serum creatinine
Calcitonin
Renal reabsorption 1.8 mg/dl. Interpret the findings.
Magnesium (Mg,.) Intestinal absorption
e
Chloride (Cl·)
Renal reabsorption
,a. Clinical Case Study 28.4
Intestinal absorption
Renal reabsorption A 55-year-old man was brought to the emergency with
severe multiple injuries in a road traffic accident and
crush injuries, fractures of the legs and scalp lacera-
Different mechanisms for electrolyte regulation are tions. He was conscious and breathing spontaneously.
summarized in Table 28.7. Pulse 130/min, BP 60/40 mm Hg, serum sodium 142
e mmol/L, potassium 7.9 mmol/L, chloride 110 mmol/L,
Blood urea 40 mg/dL, and serum creatinine 1.2 mg/dL.
• Clinical Case Study 28.1
Interpret the laboratory data?
A 3-year-old boy was brought to the clinic for chronic What is the basis of the changes?

e
productive cough not responding to antibiotics. There
was no history of fever, but there was abdominal dis-
tension, difficulty to pass stool and vomiting . History
,a. Clinical Case Study 28.5
revealed that the child frequently passed bulky, foul- A 65-year-old female complaining of vomiting and diarrhea
smelling stools. No diarrhea was present. He had many over the past few days presented to the clinic. Her physi-
relatives with chronic lung and "stomach" problems and cal examination revealed dry mucous membranes, pos-
some had died at a young age. tural hypotension was present. Pulse 140/min, serum
On examination, child was ill looking , slender and in sodium 132 mmol/L, potassium 2.7 mmol/L, chloride
moderate distress. Lung examination and chest X-ray 90 mmol/L, pCO2 - 31 mm Hg, Blood urea and serum
revealed poor air movement in base of lungs, bilateral creatinine were normal. Interpret the findings.
and coarse rhonchi throughout lungs and bronchopneu- e
monia. A quantitative pilocarpine iontophoresis sweat ,a. Clinical Case Study 28.1 Answer
test was done and serum chloride was 70 mEq/L. Repeat The most probable diagnosis is cystic fibrosis, a disease
testing after a few days yielded same results. with defective cl1loride ion channels of exocrine glands
What is the diagnosis? in acinar cells of pancreas, sweat glands and mucous
What is the mechanism involved? glands of respiratory, digestive and reproductive tracts.
There is mutation in the CFTR gene; more than 1400
e mutations have been identified and 230 mutations are
• Clinical Case Study 28.2
associated with clinical features. CFTR ~508 mutation
A 70-year-old woman was admitted with anorexia, accounts for 70% of cases. Cystic fibrosis is comparati-
weight loss and anemia and diagnosed to have carci- vely rare in India, but more common in western countries.
noma of the colon. Biochemical results were Serum Clinical findings are (1) Lungs- Thickening of
sodium 123 mmol/L, Potassium 3.8 mmol/L, Chloride mucus and depletion of periciliary liquid leading to adhe-
88 mmol/L, Bicarbonate 21 mmol/L. Serum osmolality sion of mucus to airway surface; infections of airways,
was 247 mOsm/kg and urine osmolality was 176 mOsm/ (2) GI tract- Damage to exocrine pancreas and des-
kg. Urea and creatinine were normal. truction of pancreas, desiccated intestinal intraluminal
What is the probable diagnosis? contents, obstruction of small and large intestines,
418 Section C: Clinical and Applied Biochemistry

thickened biliary secretions, focal biliary cirrhosis, bile Clinical features of hypokalemia are muscle weakness,
duct proliferation, chronic cholecystitis, cholelithaisis, polyuria and cardiac hyperexcitability. Hypokalemia is
(3) Sweat gland - normal volumes of sweat with defec- also common among hospitalized patients. Causes are
tive chloride content is hallmark of CF. shown in Box 28 .9.
Laboratory findings are; (1 ) Hypoxemia and in
advanced cases, chronic compensated respiratory aci- ILEARNING POINTS, CHAPTER 28
dosis. Pulmonary function shows mixed obstructive and 1. Osmolarity means osmotic pressure exerted by the
restrictive pattern. (2) Elevated chloride in sweat on number of moles per liter of solution.
two tests on different days is diagnostic. Normal sweat 2. Osmolality is the osmotic pressure exerted by the
chloride level does not rule out diagnosis of CF. (3) DNA number of moles per kg of solvent.
analysis (PC R test). 3. Major determinant factor of osmolality is sodium.
4. Major regulatory factors of sodium and water
•• · Clinical Case Study 28.2 Answer balance am aldosterone, ADH and renin-angio-
tensin system.
The patient has dilutional hyponatremia. Normal urea
5. The body water contributes about 60-70% of body
and creatinine exclude significant sodium depletion
weight (42 L). Intracellular compartment has 40%
and absence of edema exclude increase in total body
of body We'ight (28 L) and extracellular compart-
sodium. The results are classical of "syndrome of inap-
ment has 20% (14 L). Extracellular water is distrib-
propriate ADH secretion" (SIADH), due to secretion of
uted between intravascular (4%) and extracellular
AVP in response to nonosmotic stimuli. Hyponatremia
compartment (20%).
is the most common electrolyte disturbance, and there
6. Osmotic balance is maintained between the extra
is marked presence of hyponatremia in hospitalized pa-
and intracellular fluid compartments even though
tients (30% of patients in ICUs may have hyponatremia).
there is a dilfference in solute content and composi-
Causes are described in Box 28.8.
tion.
7. Kidney is tlhe major organ regulating water and
• Clinical Case Study 28.3 Answer electrolyte balance.
8. Osmolality of plasma varies from 285 to 295 milli-
The patient possibly has prerenal uremia and severe
moles/kg.
hypernatremia. Patient might be suffering from water
9. Colloid osmotic pressure of plasma proteins keeps
deprivation. Serum potassium is normal. It is important
the water wi1thin the vascular compartment.
to exclude nonketotic diabetic coma and blood glu-
10. Regulation of sodium and water balance is by ren-
cose and ketones should be estimated for this purpose.
nin angiotensin system.
Causes are described in Box 28.7.
11 . Any fall in ECF volume stimulates the juxtaglomerular
cells to secrete renin. Renin in turn cleaves angio-
• Clinical Case Study 28.4 Answer tensinogen to angiotensin I. Angiotensin I convert-
ing enzyme cleaves angiotensin I to angiotensin II .
Patient has severe hyperkalemia due to release of
Angiotensin II will stimulate the adrenal cortex to
potassium from the damaged tissues. Clinical features
secret aldosterone. Retention of sodium and water
are neuromuscular; muscle weakness, cardiac toxicity,
leads to restoration of ECF volume.
and may produce ventricular fibrillation and asystole.
12. ADH secretion and re nin production are under
Causes are shown in Box 28.10.
feedback control by changes in osmolality.
13. Gain or loss of water and sodium and relative
&'· Clinical Case Study 28.5 Answer amounts of each decides the tonicity of ECF.
There is hypokalemia due to severe diarrhea. Diarrhea ECF volume Herriatocrit Nature ofECF Plasma sodium
has also produced loss of fluid and sodium chloride. Contraction High Isotonic Normal
Main cause of hypokalemia in this patient is extracellular Contraction High Hypertonic High
volume depletion (ECVD). Contraction High Hypotonic Low
Hypokalemia is caused by deficit of potassium Expansion Low Isotonic Normal
stores or abnormal movement into cells. Common Expansion Low Hypertonic High
causes are excess losses from kidneys and GI tract. Expansion Low Hypotonic Low
Chapter 28: Electrolyte and Water Balance 419

PART-1 : ESSAY AND SHORT NOTE QUESTIONS


28-1 . Discuss the hormonal control of fluid and electrolyte homeostasis.

SHORT NOTE QUESTIONS

28-2 Renin-angiotensn system. 28-5 Hypertonic contraction of ECF.


28-3 Isotonic contraction of ECF. 28-6 Isotonic expansion of ECF.
28-4 Hypotonic contraction of ECF.

PART-2: MULTIPLE CHOICE QUESTIONS


28-1 . The intercellular cation present in maximum con- 28-9. From pairs of diseases and associated abnor-
centration is: malities, pick out the mismatched pair:
A. Potassium B. Magnesium A. Maple syrup urine disease and metabolic acidosis
C. Sodium D. Calcium B. Conn's syndrome and metabolic alkalosis
28-2. The extracellular cation present in maximum con- C. SIADH and hypertonic expansion
centration is: D. Waldenstrom's macroglobulinemia and hyper-
A. Potassium B. Magnesium viscosity
C. Sodium D. Calciu m 28-10. The major difference between intracellular fluid
28-3. When there is deficiency of ADH (anti-diuretic hor- (ICF) and extracellular fluid (ECF) is that:
mone): A. ECF has a lower protein concentration than ICF
A. ECF volume expands B. Chloride level is higher in ECF
B. Plasma osmolarity increases C. Sodium concentration is lower in ECF
C. Sodium depletion occurs D. Phosphates form the major anion of ECF
D. Thirst sensation is suppressed 28-11 . Secretion of ADH is stimulated by all, except:
28-4. Hypotonic expansion of extracellular fluid occurs: A. Increased sodium in serum
A. Hyperaldosteronism B. High osmolality of plasma
B. Inappropriate secretion of ADH C. Inability to retain sodium
C. Cushing's syndrome D. Contraction of ECF volume
28-12. A patient with diarrhea may have all the following
D. Intravenous infusion with normal saline
abnormalities, except:
28-5. All the following conditions produce isotonic
A. Metabolic acidosis
expansion of extracellular fluid volume, except:
B. Isotonic contraction of ECF
A. Congestive cardiac failure
C. Hypertonic contraction of ECF
B. Infusion with normal saline
D. Urine with a high specific gravity
C. Hyperaldosteronism
28-13. The incorrect statement regarding osmolality of
D. Pulmonary edema
ECF is:
28-6. Which of the following parameters are important in
A. Mainly contributed by proteins
the assessment of fluid balance, except:
B. Regulated by kidney
A. Hematocrit value C. Dependent on sodium level
B. Osmolality of urine D. Sensed by thirst center
C. Specific gravity of urine 28-14. Which of the following has NO effect on ECF vol-
D. Urinary pH ume?
28-7. All the following hormones affect fluid and electro- A. ADH
lyte balance, except: B. Aldosterone
A. Aldosterone B. Anti-diuretic hormone C. Calcitriol
C. Cortisone D. Thyroxine D. Renin
28-8. The type of fluid electrolyte imbalance seen in a 28-15. Which of the following is the major intracellular
case of diabetes insipidus is: cation?
A. Isotonic expansion A. Magnesium
B. Hypotonic contraction B. Potassium
C. Hypertonic contraction C. Sodium
D. Hypertonic expansion D. Calcium
420 Section C: Clinical and Applied Biochemistry

28-16. Which of the following causes hypokalemia? 28-17. Hyponatrennia is seen in:
A. Hemolysis A. Addison's disease
B. Polycythemia B. Conn 's syndrome
C. Leukemia C. Diabetes insipidus
D. Alkalosis D. Compulsive water drinking

ANSWERS OF MULTIPLE CHOICE Q ESTIONS


28-1 . A 28-2. C 28-3. C 28-4. B 28-5. D 28-6. A 28-7. D
28-8. C 28-9. C 28-10. A 28-11 . C 28-12. B 28-13. A 28-14. C
28-15. B 28-16. D 28-17. A

PART-3: VIVA VOCE QUESTIONS AND ANSWERS


28-1 . What is the major intracellular cation? 28-7. What are the causes for isotonic expansion of
Potassium extracellular fluid?
28-2. What is the major extracellular cation? Congestive cardiac failure and hyperaldosteronism.
Sodium 28-8. What is the cause for hypotonic expansion of
28-3. What are the factors regulating fluid and electro- extracellular fluid?
lyte balance? Inappropriate secretion of ADH.
Aldosterone, ADH, renin angiotensin, thirst. 28-9. What is the• important cause for hypertonic expan-
28-4. What is major cause for isotonic contraction? sion of ECF?
Small intestinal obstruction. Cushing's syndrome.
28-5. What are the causes for hypotonic contraction? 28-10. What are the metabolic imbalances seen in diarr-
Infusion of dextrose (without saline), Addison's disease. hea?
28-6. What are the causes for hypertonic contraction? Metabolic acidosis; hypertonic contraction of ECF; urine
Diarrhea, vomiting. with high specific gravity; urine output reduced.
___ __Chapter 29
Body Fluids
(Milk, CSF and Amniotic Fluid)

Chapter at a Glance
The learner will be able to answer questions on the following topics:
Milk D Cerebrospinal flu id
D Colostrum D Amniotic fluid

I MILK Lactase deficiency is described in Chapter 12; see


also Box 29.1.
Milk is the only food for the growth of young ones of all Net energy content (kcal / 100 ml) of milk of different
mammals. The milk is secreted by the mammary glands. species is as follows: Human 71 , Cow's 69, Buffalo's
Milk holds a unique place as an almost complete 117, and Goat's 84.
natural food from the point of view of nutrition. The major Human milk has higher carbohydrate content than
nutrients lacking in milk are iron, copper and cow's milk while protein content is less.
vitamin C. The composition of milk is given in Table 29.1.
Lipids in Milk
Lactose Synthesis
The white color of milk is due to the emulsified fat and
Synthesis of lactose in mammary gland is catalyzed by the calcium caseinate. The lipids of milk are dispersed
lactose synthase. A galactose unit is transferred from as small globul1es. The fatty acids are mainly saturated,
UDP-galactose to glucose. but 50% of them are medium chain fatty acids (mainly
Epimerase Lactose synthase lauric and myristic acids). Medium chain fatty acids
UDP glucose---. UDP galactose--- Lactose are easily digested , absorbed and metabolized (see
+ Glucose Chapter 16).

Proteins in Milk
TABLE 29.1: Composition of milk
A comparison of protein content in milk of different
Constituent Human Cow Buffalo Goat
species is shown in Table 29.1. The protein content is

. .
Water (%) 87.5 87.2 83.6 87.5
Total solids(%) 12.5 12.8 16.4 12.5
: .. tase deficiency leads to lactose intolerance
Proteins (g/ dl) 1.1 3.3 4.3 3.7
Many infants develop diarrhea and skin manifestations due to
Lipids (g/d l ) 3.8 3.8 6.0 3.5 lactose intoleranoe. (It may also be due to allergy to milk proteins).
Carbohydrate (g/dl) 7.5 4.4 5.3 4.7 These children arei to be fed with lactose free formulae or soybean
Calcium (mg/dl) 34 150 160 170 proteins.
422 Section C: Clinical and Applied Biochemistry

generally proportional to the requirement for growth.


For example, the time for doubling the body weight of a
TABLE 29.2: Min - . . -
Human milk Cow s milk Buffalo's milk
newborn human being is 180 days, but in the rabbit, it Mineral (mg/'1 00 ml ) (mg/ 100 mL) (mg/ lOOmL)
is only 6 days. As the growth rate is more in the young
Calcium 34 150 160
rabbit, the rabbit milk also has higher percentage of
protein content. Magnesium 2.2 13 10
About 80% protein of cow's milk is casein. It is a Phosphorus 16 100 100
phosphoprotein. The phosphate groups are added to Sodium 15 58 58
the hydroxyl groups of serine or threonine residues. If Potassium 55 138 130
milk is acidified and pH lowered to 4.6, the casein is pre-
Ch loride 43 100 60
cipitated (isoelectric precipitation).
Iron Negligible Negligible Negligible
The supernatant whey contains the rest of proteins.
The proteins in the whey are lactalbumin, lactoglobulin
and lysozyme. lgA (140 mg/dl) has the highest con- Biochemical Analysis of CSF
centration among the immunoglobulins. lgM and lgG are
The protein concentration is usually 10-30 mg/dl, out of
also present in small amounts.
which about 20 rng/dL is albumin, and globulin is about
Minerals in Milk 5-10 mg/dl.
In bacterial iinfections of the meninges, the pro-
Milk has a high content of calcium, phosphorus, sodium tein concentration is increased. But in such cases, the
and potassium; but is poor in iron and copper. Hence, neutrophil cell count is also increased.
young infants fed exclusively on milk may develop iron In viral infections, the protein concentration is not
deficiency anemia. Semisolid diet should be started
significantly increased, but mononuclear cells are abun-
in children after 3 months of age, so that anemia may
dant.
be prevented. A comparison of the mineral content of
In brain tumors, albumin level is raised, but cell
human and bovine milk is shown in Table 29.2.
count is normal; this is called albuminocytological dis-
sociation.
Colostrum (Colostral Milk)
It is secreted during the first few days after parturition. Glucose Level in CSF
Colostrum coagulates on heating. This coagulum forms Normal level of gl1Ucose in CSF is 50- 70 mg/dl , which is
a surface film containing casein and calcium salts. Colos-
lower than the plasma level. Hence, estimation of plasma
trum is mildly laxative, which helps to remove meconium
glucose along witl1 CSF glucose is always done to avoid
from the intestinal tract of the infant. The change from
misinterpretation due to a change in the plasma glucose.
colostrum to milk occurs within a few days after the Elevated levels are seen in diabetes mellitus. In bacterial
initiation of lactation. meningitis, however, the glucose level is far lower when
The proteins present in colostrum are predominantly compared to the plasma, because it is metabolized by
immunoglobulins. In the case of cow, these immuno- bacteria. The diagnostic findings in different diseases
globulins are readily absorbed by the calf, and give are given in Table, 29.3.
protection to the young animal.

ICEREBROSPINAL FLUID (CSF) I AMNIOTIC FLUID


Amniocentesis is the process by which amniotic fluid is
The total volume of fluid is about 125 milliliter. It is a
collected for analysis. Examination of amniotic fluid is of
transudate or ultrafiltrate of plasma. The composition
importance in pre,natal diagnosis. The normal composi-
of the fluid is given in Table 29.3. CSF has the chlo-
tion of amniotic fluid is given in Table 29.4.
ride concentration higher than the plasma. This is in
accordance with the Gibbs-Donnan equilibrium (see Lung Maturity
Chapter 1 ). Because the concentrations of non-diffusible
anions like proteins are lower in CSF than in the plasma, The lung maturity is assessed by measuring the lecithin-
as a compensation, the chloride ions are increased. sphingomyelin (US) ratio, which is an index of the
Chapter 29: Body Fluids (Milk, CSF and Amniotic Fluid) 423

Clear and colorless 10-30 mg/ dl 50- 70 mg/dl Not seen


Bact erial meningitis Opalescent or turbid Markedly increased Marked increase Marked decrease May clot on
(purulent menln itis) due to high cell content polymorphs standin
Tuberculous meningit is May be opalescent Lymphocytes and Increased Low but not very Cobweb t ype
mononuclear cells much decreased coa ulation
Viral infection Clear and colorless Increased Increased Normal Nil
Brain tumor Clear and colorless Within normal range Increased Low Solidifies
Su barachnoid Blood stained in fresh RBCs and WBCs Increased Not significant Nil
hemorrhage hemorrhage

TABLE 29.4: Normal composition of amniotic fluid 7. The best source of calcium and phosphorus for the
Early growing child is milk.
gestation Pre-term 8. CSF is produced as an ultrafiltrate of plasma by the
Volume 450- 1200 ml 500- 1400 ml choroid plexus.
Creati nine 0.8-1.1 mg/ 1.8-4.0 mg/ dl 9. Major constituents of CSF are proteins, glucose
dl
and chloride.
Lecithin-sphingomyelin (US) ratio <1:1 >2:1
10. The protein content is 10-30 mg/di of which albumin
Prot ein 0.6-0.24 g/ dl 0.26- 0.19 g/ dl
contributes more than 80%.
11. A small quantity of globulin is also present which
surfactant (surface tension lowering complex) concen- becomes significant in diseases like multiple
tration in amniotic fluid. In late pregnancy, the cells lining sclerosis.
the fetal alveoli start synthesizing d ipalmitoyl-lecithin so 12. The glucose level in CSF is about 75% of the level
that the concentration of lecithin increases, whereas that in blood .
of sphingomyelin remains constant. As a result, as the 13. Chloride content of CSF is higher than plasma to
fetal lung matures, the lecithin-sphingomyelin (US) ratio maintain Gibb's-Donnan equilibrium due to low
protein co ntent.
rises. An US ratio of 2 is taken usually as a critical value.
14. In bacterial meningitis, the protein content increases,

ILEARNING POINTS, CHAPTER 29 glucose falls and cell count is high.


15. V iral meningitis does not show significant diffe-
rence in protein content in CSF fro m the normal,
1. Milk is produced by the lactating mother under the
but mononuclear cells are present.
influence of the hormone prolactin.
16. Brain tumors are characterized by high protein
2. Lactose, the milk sugar is synthesised from UDP
content in the CSF; with normal cell count.
galactose and glucose .
17. Amniotic flu id is produced by the amnion and is the
3. Major proteins present in milk are casein and fluid which nourishes the fetus.
lactalbumin.
18. The lecithin/sphingomyelin ratio is an index of fetal
4. Milk is deficient in iron and copper. maturity.
5. Human milk has a higher lactose content and lower 19. Amniotic fluid cells can be collected by
protein content than cow's milk. amniocentesis and DNA analysis can be done.
6. The milk secreted by the lactating women during 20. At present most of the intrauterine assessment of
the first 2 days of lactation is called colostrum. It is fetal well-being is done by ultrasonographic studies;
rich in immunoglobulins (lgA) and can impart immu- amniocentesis is seldom done since it is an invasive
nity to the newborn. procedure .

PART-1 : SHORT NOTE QUESTIONS


29-1 . Proteins in milk.
29-2. Minerals in milk.
29-3. Biochemical analysis of CSF.
424 Section C: Clinical and Applied Biochemistry

PART-2: MULTIPLE CHOICE QUESTIONS


29-1 . Which of the following is NOT true regarding milk 29-4. Which of the following is NOT true regarding CSF?
sugar? A. It is an ultrafiltrate of plasma
A. Made up of glucose and galactose B. Chloride level higher than plasma
B. Linkage is 1, 4-beta galactosidic C. Glucose content same as plasma
C. Requires UDP-galactose for synthesis
D. Protein content is very low compared to plasma
D. Present in lower concentration in human milk
29-5. Albuminocytological dissociation is a feature of:
29-2. Which of the following nutrients is lacking in milk?
A. Bacterial meningitis
A. Proteins B. Iron
B. Viral meningitis
C. Calcium D. Phosphate
29-3. Which of the following proteins is absent in milk? C. Multiple sclerosis
A. lgA B. Casein D. Brain tumors
C. Lactalbumin D. lgM

ANSWERS OF MULTIPLE CHOICE QUESTIONS


29-1. D 29-2. B 29-3. D 29-4. C 29-5. D

PART-3: VIVA VOCE QUESTIONS AND ANSWERS


29-1 . How lactose is synthesized? 29-4. What is the advantage of Medium chain fatty
The galactose unit is transferred from UDP-galactose acids?
to glucose. This is catalyzed by lactose synthase. They are easily digested, absorbed and metabolized.
29-2. What is the major difference between human and 29-5. What is the major protein in cow's milk?
cow's milk? 80% protein of cow's milk is casein.
Human milk has higher carbohydrate content than
29-6. What type of protein is casein?
cow's milk while protein content is less.
II is a phosphoprotein.
29-3. Milk contains which type of fatty acids?
The fatty acids are mainly saturated, but 50% of them 29-7. How the phosphate group is attached to protein?
are medium chain fatty acids (Laurie and Myristic The phosphate groups are added to the hydroxyl
acids). groups of serine or th reonine residues.
________Chapter 30
Free Radicals and
Antioxidants

Chapter at a Glance
The learner will be able to answer questions on the following topics:
Reactive oxygen species Clinical significance
Generation of free radicals Lipid peroxidation
Damage produced by free radicals Antioxidants
Free radical scavenger enzyme systems

The outermost orbital in an atom or molecule contains Reactive oxygen species or ROS . The following are
two electrons, each spinning in opposite directions. The members of this group: (See Fig. 30.2)
chemical covalent bond consists of a pair of electrons, i. Superoxide anion radical (02- )

each component of the bond donating one electron each. ii. Hydroperoxyl radical (HOO·)
iii. Hydrogen peroxide (HP)
Definition iv. Hydroxyl radical (OH')
A free radical is a molecule or molecular fragment that v. Lipid peroxide radical (ROO-)
contains one or more unpaired electrons in its outer vi. Singlet oxygen (102 )
orbital (Fig. 30.1 ). Free radical is generally represented vii. Nitric oxide (NO')
by a superscript dot, (Rl viii. Peroxynitrite (ONOO-·).
Oxidation reactions ensure that molecular oxygen Out of this, hydrogen peroxide and singlet oxygen
is completely reduced to water. The products of partial are not free radicals (they do not have superscript dot).

•• .. -
reduction of oxygen are highly reactive and create
havoc in the living systems. Hence, they are also called
•o: o:
•• ••
•o: o:
•• • •
•• • •
H
••
: O : H

0 0 Su peroxide Hydroperoxyl Hydroxyl


anion (02•) radical (Hoo·) radical (OH")

e e
e
::,

• e
: ••
o ••
: 0 •
•• •• • H
Hydrogen
peroxide (H20 2)
••
o ::
•• ••
••
o
Oxygen
(02)
••
: 0 : R
••
Hydroxyl
ion (OH- )
Fig. 30.1: Left side = normal oxygen atom with all paired elec- Fig. 30.2: Some free radicals. Please compare hydroxyl radical
trons; one electron Is in the process of jumping out. Right side = (free radical) with hydroxyl ion, which is not a free radical. Also
free radical, with an unpaired electron compare oxygen with superoxide anion
426 Section C: Clinical and Applied Biochemistry

H
;:r pathwai
•••
UV Radiation Cigarette
Macrophage and PMN
(Respiratory burst)

i OH• • o-•
2

NADPH~ 02 O H.

IWMR:C-tftiEMJ NO.

NADP+ O{ EimJ H20 2 -===...•


6
+Cl-
HCIO

I
.___ _ _ _ _ __.
. Bactena
killed
Mitochondria 1
OH•

03
Fig. 30.3: Generation of ROS in m acrophages. (GPD: Glucose-
White Air
6-phosphate dehyrogenase; SOD: Superoxide dismutase; MPO:
blood 10 pollution
Myeloperoxidasee) cell 2

However, because of their extreme reactivity, they are


Fig. 30.4: Formation of free radicals
included in the group of reactive oxygen species.
The sequential univalent reduction steps of oxygen macrophages, the consumption of oxygen by the cell is
may be represented as: increased drastically; this is called respiratory burst.
(+)e- (+)e-,2w (+)e-,W (+)e-,W In chronic granulomatous disease (CGD), the
0 2 --+ Ot - - - H 0 2 2--- OH.-+- Hp NADPH oxidase is absent in macrophages and neu-
trophils. In this condition, macrophages ingest bacteria
(- )HP
normally, but cannot destroy them. Hence, recurrent
Important characteristics of the ROS are: pyogenic infection by staphylococci are common in
a. Extreme reactivity chronic granulomatous disease.
b. Short lifespan Macrophages also produce NO· from arginine by the
c. Generation of new ROS by chain reaction enzyme nitric oxide synthase (see Chapter 18). This
d. Damage to various tissues. is also an important antibacterial mechanism. Ionizing
radiation damages tissues by producing hydroxyl radi-
Generation of Free Radicals cals, hydrogen peroxide and superoxide anion. The
They are constantly produced during the normal oxida- capacity to produce tissue damage by Hp2 is minimal
tion of foodstuffs, due to leaks in the electron transport when compared to other free radicals (by definition,
chain in mitochondria. About 1-4% of oxygen taken up H2 0 2 is not a free radical). But in presence of free iron ,
in the body is converted to free radicals. Mitochondria Hp 2 can generate OH· (hydroxyl radical) which is highly
are major sites for production of superoxide ions from reactive (Fig. 30.5). Cigarette smoke contains high con-
the interaction between Coenzyme Q and oxygen in the centrations of various free radicals. These facts are
electron transport chain . Hence a high content of SOD summarized in Figure 30.4.
is needed. Mitochondria also have a high content of glu-
tathione and glutathione peroxidase for preventing lipid Free Radical Scavenger Systems
peroxidation. Superoxide Dismutase (SOD)
NADPH oxidase in the inflammatory cells (neu- The reaction is depicted in Figure 30.6. A defect in SOD
trophils, eosinophils, monocytes and macrophages) gene is seen in patients with amyotrophic lateral scle-
produces superoxide anion by a process of respira- rosis (Lou Gehrig's disease; named after the American
tory burst during phagocytosis (Fig. 30.3). The super- baseball captain who succumbed to the illness).
oxide is converted to hydrogen peroxide and then to
hypochlorous acid (HCIO) with the help of superoxide G/utathione Peroxidase
dlsmutase (SOD) and myeloperoxidase (MPO). The
superoxide and hypochlorous ions are the final effectors In the next step, the Hp 2 is removed by glutathione per-
of bactericidal action. This is a deliberate production of oxidase (POD) (Fig. 30.6). It is a selenium dependent
free radicals by the body. Along with the activation of enzyme.
Chapter 30: Free Radicals and Antioxidants 427

Iron 0

0 2 + H 20 2 - -. . 0 2 +0H + OH- (Haber-Weiss reaction) HzOz - - - - - - ~ -


PO
_D_ ..3\
- - - --+~ 2 HzO

Fig. 30.5: Iron produces free radicals 2GSH GS-SG

Glutathione Reductase
The oxidized glutathione, in turn, is reduced by the
glutathione reductase (GR), in presence of NADPH (Fig.
30.6). This NADPH is generated with the help of glu-
NADP•
R NADPH ..
GPD
~ - -- Glucose

Fig. 30.6: Free radical scavenging enzymes. SOD = superoxide


cose-6-phosphate dehydrogenase (GPD) in HMP shunt = = =
dismutase; POD peroxidase; GSH glutathione; GR glutathi-
one reductase; GPO = glucose-6-phosphate dehydrogenase
pathway. Therefore, in GPD deficiency, the RBCs are
liable to lysis, especially when oxidizing agents are
Almost all biological macromolecules are damaged
administered (drug induced hemolytic anemia). by the free radicals (Fig . 30.7). Thus, oxidation of sulf-
hydryl group containing enzymes, loss of function and
Catalase fragmentation of proteins are noticed. Polysaccharides
undergo degradation.
When Hp2 is generated in large quantities, the enzyme
DNA is damaged by strand breaks. The DNA
catalase is also used for its removal.
damage may directly cause inhibition of protein and
Catalase enzyme synthesis and indirectly cause cell death or
mutation and carcinogenesis (Fig. 30.7).

Polyphenols l_fLINICAL SIGNIFICANCE


Dietary polyphenols represent a wide variety of com-
Chronic Inflammation
pounds that occur in fruits, vegetables, wine, tea and
chocolate. They contain flavones, isoflavones, flavonols, Chronic inflammatory diseases such as rheumatoid
catechins and phenolic acids. They act as agents having arthritis, chronic ulcerative colitis, and chronic
antioxidant, antiapoptotic, antiaging, anticarcinogenic, glomerulonephritis are self-perpetuated by the free
antiinflammatory and anti-atherosclerotic effects. They radicals released by neutrophils.
are protective against cardiovascular diseases.
Acute Inflammation
Damage P:-oduced by Re:1ctive At the inflammatory site, activated macrophages pro-
Oxygen Species duce free radicals.

Free radicals are extremely reactive. Their mean effec- Respiratory Diseases
tive radius of action is only 30 A. Their half-life is only a
Breathing of 100% oxygen for more than 24 hours
few milliseconds.
produces destruction of endothelium and lung edema.
When a free radical reacts with a normal compound,
This is due to the release of free radicals by activated
other free radicals are generated. This chain reaction
neutrophils.
leads to thousands of events (see propagation phase
In premature newborn infants, prolonged exposure
below). to high oxygen concentration is responsible for bron-
Peroxidation of PUFA (poly unsaturated fatty cho-pulmonary dysplasia.
acids) in plasma membrane leads to loss of membrane Adult respiratory distress syndrome (ARDS) is
functions. Lipid peroxidation and consequent degrada- characterized by pulmonary edema. It is produced when
tion products such as malondialdehyde (-CHO-CH2- neutrophils are recruited to lungs which subsequently
CHO-) are seen in biological fluids. release free radicals.
428 Section C: Clinical and Applied Biochemistry

Xanthlne oxidase
ROS Protein damage;
loss of function
ATP - - - • Hypo,aa~;a, ( ) ,~ • x,.~;o,
! 0 0 000

~
Lipid peroxidation
membrane damage 11'0D !
ffJ_____ Mitochondrial II Block HzOz____. OH"
permeability transition
Fig. 30.8: Explanation for reperfusion injury
- - - - - ' - - - DNA damage; cell death;
mutation; cancer
Ageing Proce,ss
Reactive oxyge1n metabolites (ROM) play a pivotal
Fig. 30.7: Damages by reactive oxygen species role in degenerative brain disorders such as Parkin-
sonism, AlzheimBr's dementia and multiple sclerosis.
Cigarette smoke contains free radicals. Soot Cumulative effects of free radical injury cause gradual
attracts neutrophils to the site which releases more free deterioration in ,ageing process. A summary of free
rad icals, leading to lung damage. radica l toxicity is shown in Figure 30.9.

Diseases of the Eye Lipid Peroxidation


Retrolental fibroplasia (retinopathy of prematurity) is Initiation Phase
a condition seen in premature infants treated with pure
Polyunsaturated fatty acids (PUFA) present in cell mem-
oxygen for a long time. It is caused by free radicals,
branes are easily destroyed by peroxidation. During
causing thromboxane release, sustained vascular con-
the initiation phase, the primary event is the production
tracture and cellular injury.
of R (carbon centered radical) (PUFA radical) or ROO·
Cataract formation is related with aging process.
(lipid peroxide radical) by the interaction of a PUFA
Cataract is partly due to photochemical generation of
molecule with fre,e radicals generated by other means
free radicals. Tissues of the eye, including the lens, have
(Fig. 30.10).
high concentration of free radical scavenging enzymes.
RH + OH· - ---> R· + Hp (reaction 1-A)
Reperfusion Injury Metal ion
ROOH ------. ROO· + W (reaction 1-B)
Reperfusion injury after myocardial ischemia is caused
The R" and HOO·, in turn , are degraded to malon-
by free radicals. During ischemia, the activity of xan-
dialdehyde (3 carbon). It is estimated as an indicator of
thine oxidase is increased. When reperfused, this
fatty acid breakdown by free radicals.
causes conversion of hypoxanthine to xanthine and
superoxide anion. At the same time, the availability of
Propagation Phase
scavenging enzymes is decreased, leading to aggrava-
tion of myocardial injury (Fig. 30.8). The carbon cente,red radical (R") rapidly reacts with mo-
Allopurinol, a xanthine oxidase inhibitor, reduces lecular oxygen forming a peroxyl radical (ROO·) which
the severity of reperfusion injury. can attack anothm polyunsaturated lipid molecule.
R" + 0 2 --+ ROO· (reaction 2)
Atherosclerosis and Rao· + RH ROOH + R" (reaction 3)
Myocardial Infarction The net result of reactions 2 and 3 is the conversion
Low density lipoproteins (LDL) are deposited under of R' to ROOH (a hydroperoxide). But there is simul-
the endothelial cells, which undergo oxidation by free taneous conversiion of a carbon centered radical to a
radicals. This attracts macrophages. Macrophages are peroxyl radical , HOO·. This would lead to continuous
then converted into foam cells initiating atherosclerotic production of hyclroperoxide with consumption of equi-
plaque formation (see Chapter 15). Antioxidants offer molecular quantities of PUFA. One free rad ical gene-
some protective effect. rates another free radical in the neighbouring molecule;
Chapter 30: Free Radicals and Antioxidants 429

Eye - - - - - - - -- ------.
Diabetic retinopathy of prematurity, - - - - - - Brain
cataracts, age-related macular degeneration Parkinson's disease, Alzheimer's disease, amyotrophic lateral
sclerosis

Air tube - - - - -- - - - - - - ,
Bronchial asthma, inhalation injury

- - - - - Chest
Arrhythmia, cardiac infarction, high blood pressure

Abdomen- - - - - - - - - - - H ~
Gastric ulcer, ischemic colitis, fatty liver Age spots
wrinkles, sagging
- - -- Lower abdomen
Kidney failure, uremia

Body
Aging, diabetes, allergy, rheumatic
disease, cancer, arterial sclerosis

Fig. 30.9: Free radical toxicity

glutathione peroxidase and ethylenediaminetetra-


F\F"\
(+) OH'
---+ acetate (EDTA).
COOH • COOH b. Chain brea1king antioxidants: They can inhibit pro-
PUFA (R) PUFA radical (R. ) pagative phase. They include superoxide dismutase,
uric acid and vitamin E. Alpha-tocopherol (T-OH)
Fig. 30.1 0: Peroxidation of polyu nsaturated fatty a cids
(vitamin E) would intercept the peroxyl free radical
a "chain reaction" or "propagation" is initiated. This is and inactivate it before a PUFA can be attacked.
T-OH + Rao· ........ To· + ROOH (reaction 5)
sometimes called "death kiss" by free radicals. Accumu-
lation of such lipid damages lead to the destruction of The phenolic hydrogen of the alpha tocopherol
fine architecture, and integrity of the membranes. reacts with the peroxyl radical, converting it to a hydro-
peroxide product The tocoperoxyl radical thus formed is
Termination Phase stable and will not propagate the cycle any further.
The tocoperoxyl radical can react with another per-
The reaction would proceed unchecked till a peroxyl oxyl radical getting converted to inactive products.
radical reacts with another peroxyl radical to fo rm inac- TO· + Rao· ........ inactive products (reaction 6)
tive products. Vitamin E (Alpha tocopherol) acts as the most
ROO" + Rao· ........ RO-OR + 0 2 (reaction 4-A) effective naturally occurring chain breaking antioxidant
R·+ R" -+ R-R (reaction 4-B) in tissues. Vitamin Eis described in Chapter 32. It is seen
ROO· + R· ........ RO- OR (reaction 4-C) from reaction 5 and 6 that while acting as antioxidant,
alpha tocophenol is consumed . Hence, it has to be
Role of Antioxidants replenished by daily dietary supply.
Apart from the scavenging enzymes described earlier,
Antioxidants
there are two types of antioxidants:
a. Preventive antioxidants: They will inhibit the ini- 1. Vitamin E i s the lipid phase antioxidant.
tial production of free radicals. They are catalase, 2. Vitamin C is the aqueous phase antioxidant.
430 Section C: Clinical and Applied Biochemistry

3. Ceruloplasmin can act as an antioxidant in extra- peroxyl, lipid peroxide, nitric oxide and peroxy
cellular fluid (see Chapter 34). nitrite radicals.
4. Cysteine, glutathione, carotenoids, flavonoids and 5. Enzymatic antioxidants are SOD, peroxidase, GSH
vitamin A are minor antioxidants. reductase, catalase and peroxidase most of which
5. Food items containing good quantity of antioxidants are preventive antioxidants as well.
are: (a) Spices used in ordinary Indian cooking con- 6. Peroxidation of PUFA leads to damage to mem-
tain highest quantity of antioxidants. (b) Curcumin. branes compromising membrane integrity and
(c) Fruits and vegetables such as berries, broccoli, function.
spinach, asparagus and green tea, which contain 7. DNA can also be damaged by free radicals and
flavonoids, flavones, isoflavones and anthocya- unless repaired promptly, the change may be
nins. (d) Resveratrol present in grapes. perpetuated.
8. Cataract formation and reperfusion injury (after
1

LEARNING POINTS, CHAPTER 30 myocardial ischemia) are caused by free radicals.


9. LDL gets oxidized and the modified LDL particle is
1. A free radical is a molecule or molecular fragment
highly atherogenic and gets deposited beneath the
that has one or more unpaired electrons in its outer endothelium initiating atheromatous plaque.
orbital. 10. Degenerative diseases of old age like Alzheimer's,
2. Free radicals are generated in the body during Parkinson's etc are due to cumulative effects of
oxidation processes. free radical injury.
3. The stepwise reduction of oxygen to water by elec- 11. The carbon centred radical or lipid peroxide mole-
trons in Electron transport chain produces free cule is formed when a free radical reacts with PUFA.
radicals. 12. Vitamin E is the major biological antioxidant which
4. These include superoxide, peroxide, hydroxyl and will directly react with the peroxyl radicals to inacti-
singlet oxygen, together known as reactive oxygen vate them. Other antioxidants in this group are
species (ROS). Other free radicals include hydro- vitamin C, Ceruloplasmin, beta carotene etc.

PART-1: SHORT NOTE QUESTIONS

30-1. Detoxification of hydrogen peroxide.


30-2. Superoxide dismutase.
30-3. Antioxidants.
30-4. Free radical scavenger mechanisms.
30-5. Reactive oxygen species (Free radicals).

PART-2: MULTIPLE CHOICE QUESTIONS

30-1. Which of the following is NOT a property of free 30-4. Which of the following processes make use of free
radicals: radical effects?
A. Produced in a well controlled manner A. Cell adhesion B. Phagocytosis
B. Have a very short half life C. Contact inhibition D. Transcytosis
30-5. Which of the following diseases are NOT attributed
C. Highly reactive molecules
to free radical injury?
D. Can trigger the production of other free radicals
A. Hypothyroidism
30-2. All cells produce free radicals except
B. Cataract
A. Macrophages B. Neutrophils C. Reperfusion injury
C. Erythrocytes D. Basophils D. Retrolental fibroplasia
30-3. The organelle In the body mainly concerned with 30-6. Which of the following enzymes produce a free
free radical scavenging is: radical in macrophages?
A. Lysosomes B. Golgi bodies A. SOD B. Catalase
C. Nucleolus D. Peroxisomes C. GSH peroxidase D. NOS
Chapter 30: Free Radicals and Antioxidants 431

30-7. Which of the vitamins listed has NO antioxidant C. DNAbases


property? D. Low density lipoproteins
A. Vitamin C B. Vitamin E 30-9. Which of the following diseases result from defi-
C. VitaminA D. Vitamin K cient production of free rad icals?
30-8. Which of the following biomolecules is least prone A. Alzheimer's disease
to free radical injury? B. Cancer
A. PUFA in membranes C. Chronic granulomatous disease
B. Plasma albumin D. Amyotrophic lateral sclerosis

ANSWERS OF MULTIPLE CHOICE QUESTIONS


30-1. A 30-2. C 30-3. D 30-4. B 30-5. A 30-6. A 30-7. D
30-8. B 30-9. C

PART-3: VIVA VOCE QUESTIONS AND ANSWERS


30-1 . What are the enzymes generating ROS in macro- 30-3. Name some diseases related with ROS?
phages? Chronic inflammation, rheumatoid arthritis, broncho-
NADPH oxidase, superoxide dismutase, myeloperoxi- pulmonary dysplasia, respiratory distress syndrome,
dase. retrolental fibroplasia, cataract, reperfusion injury, ath-
erosclerosis, peptic ulcer.
30-2. What are the free radical scavenging enzymes?
30-4. Name important antioxidants.
Superoxide dismutase, glutathione peroxidase, gluta- Alpha tocopherol (vitamin E), ascorbic acid (vitamin C),
thione reductase, and catalase. vitamin A, beta carotene.
_ _ _ _ _Chapter 3~
Laboratory Techniques,
Quality Control and
Metabolic Diseases

Chapter at a Glance
The learner will be able to answer questions on the following topics:
D Electrophoresis D Anticoagulants and preservatives
D Adsorption chromatography D Quality Control in the laboratory
D Partition chromatography D Accuracy, precision, specificity and sensitivity
D Ion exchange chromatography D Quality control charts
D Gel filtration chromatography D External quality assurance
D Enzyme Linked lmmunosorbent Assay (ELISA) D Prenatal diagnosis
D Colorimeter 0 AFP, hCG, uE3, DIA, PAPP-A
D Spectrophotometer D Newborn screening
D Mass spectrometry D Investigations for metabolic disorders
D Preanalytical variables

1..§_LECTROPHORESIS Electrophoresis Apparatus


The term refers to the movement of charged particles The electrophoresis system basically consists of the
through an electrolyte when subjected to an electric electrophoresis tank to hold the buffer and fitted w ith the
field. The positively charged particles (cations) move to electrodes, as well as a power pack to supply electricity
cathode and negatively charged ones (anions) to anode. at constant current and voltage (Fig. 31.1 ).
Since proteins exist as charged particles, this method is Serum proteins are separated at a pH of 8.6 using
widely used for the separation of proteins in biological barbitone buffer. At this pH all serum proteins will have a
flu ids. The technique was invented by Tiselius (Nobel net negative charge and will migrate towards the anode.
Prize 1948).
Support Medium for Electrophoresis
Factors Affecting Electrophoresis Filter Paper
The rate of migration (separation of particles) during If the support medium is a filter paper, the electrophoresis
electrophoresis will depend on the following factors: is carried out for 16-18 hours at a low voltage. This long
1. Net charge on the particles (pl of proteins) time interval and diffusion of particles leading to blurring
2. Mass and shape of the particles of margins are the disadvantages of paper.
3. The pH of the medium
4. Strength of electrical field
Agar or Agarose
5. Properties of the supporting medium These are heterogeneous polysaccharides. They are
6. Temperature. viscous liquids when hot but solidify to a gel on cooling.
Chapter 31: Laboratory Techniques, Quality Control and Metabolic Diseases 433

Electric connection

Agarose slide

Filter paper connector

Tray with buffer

Fig. 31 .1 : Electrophoresis apparatus


- . . ~,. ~">. . ;
~ : .· Fig. 31.2: Polyacrylamide gel electrophoresis

' I~ . Albumin
Ame Wilhelm Mikhail Archer Richard
Tiselius Semenovich John Martin L Synge Albumin a 1 a2 P Y
NP 1948 Tswett

I· I
NP 1952 NP 1952
1902- 1971 1872-1919 1910-2002

The gel is prepared in the buffer and spread over


1914-1994

Point of application i
·I
microscopic slides and allowed to cool. A small sample
(few microliters) of serum or biological fluid is applied by
cutting into the gel with a sharp edge. The electrophoretic
run takes about 90 minutes. This technique is modified
for immunoelectrophoresis which is described later. Fig. 31.3: Electrophoresis of normal serum sample
Agarose gel is used as support to separate different
types of protein mixtures as well as nucleic acids. lmmunoelectrophoresis
Polyacrylamide Gel Electrophoresis (PA GE) Here electrophoretic separation is followed by an anti-
gen-antibody reaction. The electrophoresis is carried out
It has a molecular sieving effect that makes separation first by applying the patient's serum into the wells cut out
very efficient. In agar gel electrophoresis, serum com-
in the agar or agarose gel. The proteins are now sepa-
::>onents are separated into 5 fractions; while in PAGE
rated. To visualize them, a specific antibody is placed in
;;erum will show more than 20 different bands. The
a trough cut into the gel and incubated. The precipitation
amount of cross linking and thereby the pore size can
arcs are formed where the antigen and antibody mol-
oe controlled (Fig. 31 .2).
ecules are at 1:1 ratio (Fig. 31.4). So it is much more
sensitive and specific than ordinary electrophoresis.
Visualization of Protein Bands
After the electrophoretic run is completed, the proteins Ultracentrifugation
are fixed to the solid support using a fixative such as This technique was developed by Svedberg (Nobel
acetone or methanol. Then it is stained by using dyes Prize, 1926). Large molecules can be sedimented at
(Amido Schwartz, naphthalene black, Ponceau S or high centrifugal forces whereas small molecules cannot.
Coomassie Blue) and then destained by using dilute The rate of sedimentation is a function of the size and
acetic acid. The electrophoretogram can be scanned shape of the molecule and is a constant for a particular
using a densitometer and each band quantitated. In the molecule. It also depends on the difference in the
densitometer, light is passed through the agar gel plate; density of solute particles and the solvent. When solute
the absorption of light will be proportional to the quantity particles are lighter than the solvent, they would float
of protein present on a band. The electrophoretic pattern and vice versa. Sedimentation constant is expressed in
of serum proteins on agar gel are shown in Figure 31.3. Svedberg (5) units. Substances with different S units
Abnormal patterns are shown in Figures 26.1A and B will separate from one another as separate bands. See
and 26.2. Chapter 14 for ultracentrifugation of lipoproteins.
434 Section C: Clinical and Applied Biochemistry

Well containing serum


1. Sample applied and
ot Electrophoresis
solvent added

separation of 2. Molecules separated;


proteins; but they they move as two
are not visible different bands
3. Red molecules are
eluted out
-ve
+Ve
0 Antibody added to
! !
4. Later blue molecules
are eluted

*
the trough and
incubated; antigen-
antibody reaction
I takes place
+ Fig. 31 .5: Adsorption chromatography
Trough containing antiserum

Precipitate
depending on tt,e partition coefficient (solubility) of the
pattern is now particular substances. This results in separation of the
clearly visible
components of the mixture.
Albumin lgM lgA lgG

Fig. 31.4: lmmunoelectrophoresis pattern Paper Chromatography

ICHROMATOGRAPHY The stationary' phase is water held on a solid support


of filter paper (cellulose). The mobile phase is a
The term is derived from the Greek word "chroma", mixture of immiscible solvents which are mixtures of
meaning color. The method was first employed by Tswett, water, a nonpolar solvent and an acid or base, e.g.
a botanist in 1903, for the separation of plant pigments Butanol-acetic acid-water, Phenol-water-ammonia. A
using a column of alumina. Nowadays HPLC is used few microliters of the mixture is applied as a spot at
to separate almost all biological substances, including one corner of the paper. The paper is placed in a glass
proteins, carbohydrates, lipids and nucleic acids. trough containiing the solvent which ascends up the
solid support medium. The components of the mixture
Adsorption Chromatography to be separated are carried up with the solvent. The
distance to which each compound moves depends on
In this technique the separation is based on differences its partition coefficient.
in adsorption. The common adsorbing substances used
are alumina, silicates or silica gel. These are packed into Thin Layer Chromatography (TLC)
columns and the mixture is applied in a solvent on the This is another version of liquid-liquid chromatography.
top of the column. The components get adsorbed on the A thin layer of silica gel (Kieselguhr) is spread on a glass
column of adsorbent with different affinity. The fractions plate; biological sample is applied as a small spot; the
slowly move down; the most weakly held fraction moves plate is placed in a trough containing the solvent. The
fastest; followed by others, according to the order of stationary water phase is held on the silica gel and
tightness in adsorption. The eluent from the column is mobile phase of non-polar solvent moves up. In the
collected as small equal fractions and the concentration case of paper chromatography, it takes 14-16 hours for
of each is measured, in each fraction (Fig. 31.5). separation of components to be separated. But in the _..
case of TLC it takes only 2-4 hours. That is a distinct
Partition Chromatography advantage for TLC (Figs. 31.6A to C).

This technique was developed by Martin and Synge in Visualization of Chromatography


1941 (Nobel Prize, 1952). This is commonly used for After the chromatographic run is over, the paper or plate
the separation of mixtures of amino acids and peptides. has dried, it is sprayed with a location reagent. The
There is a stationary phase which may be either solid components of the mixture would appear as discrete
or liquid over which a liquid or gaseous mobile phase spots. Some common location reagents used are:
moves. By this process, the components of the mixture Ninhydrin for amino acids and proteins, sulfuric acid for
to be separated are partitioned between the two phases phospholipids; diphenylamine for sugars.
Chapter 31: Laboratory Techniques, Quality Control and Metabolic Diseases 435

Plate

Sample apply

B •• V

l
Solvent


••
Capillary

t
Two hours later turn plate column
90 degree: different
solvent

El • ,- Helium Incubator
Fig. 31. 7: Gas liquid chromatography

gel particles are porous in nature. These pores will

• • •H
allow small molecules to enter into the gel. But larger
molecules could not enter into pores of the gel and so
Two hours

• -
•---~-
later are excluded. The small molecules can enter the gel
particles, then come out, re-enter into another particle.
Bl Thus small molecule has to travel a long distance inside
the gels. Small molecules are held back. But the large
Figs. 31.6A to C: Thin layer c hromatography (two-dimensional)
molecules cannot enter the pores and sidetrack the gel
particles; so they move in the column rapidly (Figs. 31.8A
Importance of Rf Value to C). In short, larger molecules will come out first,
The spots may be identified by the Rf value of the while smaller molecules are retained in the column.
unknown substance and comparing with those of pure
standards. (Rf= ratio of fronts). The Rf value is the ratio High Pressure Liquid
of the distance travelled by the substance (solute) Chromatography (HPLC)
to the distance travelled by the solvent. The Rf value
Incompressible silica or alumina microbeads are used
is a constant for a particular solvent system at a given
as the stationary phase. This allows high flow rates.
temperature.
The liquid phase passes through this column under
Gas-Liquid Chromatography (GLC) high pressure (1000 times atmospheric pressure).
The method is therefore based on the same principle
Here the stationary phase is a liquid and the mobile
as for those types already described, but separation is
phase is gas. The stationary liquid phase is supported
by a column of inert material such as silica in a long achieved with better resolution and high speed (within
narrow column. The mixture is made volatile by an inert minutes) (Fig. 31 .9). HPLC can resolve mixtures of lipids
carrier gas like nitrogen. The fractions emerging from whose solubility differ only slightly. In reversed phase
the column are detected and quantitated by a detecting HPLC, hydrophobic polymers are used as stationary
device. This is more suitable for compounds (e.g. lipids) phase; this is generally used to separate peptides.
which resist degradation at high temperature (Fig. 31 .7).
Ion Exchange Chromatography
Gel Filtration (Size Exclusion) In this method, the separation is based on electrostatic
Chromatography attraction between charged biological molecules to
It is also called molecular sieving. Hydrophilic cross oppositely charged groups on the ion exchange resins.
linked gels are used for separation of molecules based These resins are cross linked polymers containing ionic
on their size. The gel is packed in a column. The groups as part of their structure.
436 Section C: Clinical and Applied Biochemistry

•••••
1. Gly 6. Glu 11 . Val

00••• 00
2. Ser/Asn
3. Asp
4 . Gin
7.
8.
9.
Cys/Lys
His
Pro
12.
13.
14.
Met
Tyr
lie
5
00 00 4
5. Ala/Thr 10. Arg 15.
16.
Leu
Phe

00 00 3
11
17. Trp

00 C,G 7 13

0-C~
12 14
00 15 16 17
00 00
0 0
00 gfj
m • 0 5 10 15

Fig. 31 .9: Amino acid profile in HPLC


20 25Min.

1. Gly = glycine; 2. Ser/Asn = serine/asparagine; 3. Asp= aspartic


Figs. 31 .8A to C: Sephadex (gel filtration) chromatography. (A) acid; 4. Gin = glutamine; 5. Ala/Thr = alanine/threonine; 6. Glu
protein solution is added on the top of the column. (8) small pro- =glutamic acid; 7. Cys/Lys =cysteine/lysine; 8. His =histidine;
teins get inside the beads, and so takes a longer time to reach 9. Pro = praline; 10. Arg = arginine; 11 . Val = Valine; 12. Met =
the bottom. (C) larger molecules cannot enter into the beads, so methionine; 13. Tyr = tyrosine; 14. lie = isoleucine; 15. Leu =
travels quickly, and reaches the bottom faster leucine; 16. Phe =phenylalanine; 17. Trp =tryptophan

Agarose or cellulose bead


with positive charges on surface lmmunoglobulins fixed on
glass column
\
A

~ dmixture

.__B
Finally ant.igens are eluted

+-

n Ant.igens retain in column


Fig. 31 .10: Ion-exchange chromatography unwanted molecules go out
A = negatively cha rged molecules attach with the beads and
Fig . 31 .11 : Affinity chromatography
so move slowly; B = positively charged molecules repel with
the beads, so move faster in the column

The cation exchange particles carry acid groups,


e.g. coo-. When cations (C•) are passed through the
I RADIOIMMUNOASSAY (R_IA~) _ _
column, c• particles are adhered in the column, while The technique of RIA was developed by Rosalyn Yalow
negatively charged particles are eluted out easily (Fig. (Nobel Prize, 1977). Insulin was the first substance thus
31 .10). quantitated by RIA. Nowadays, any other biological
substances could be quantitated accurately by the RIA
Affinity Chromatography
method. The specificity of antibody and the sensitivity of
The technique is based on the high affinity of specific pro- radioactivity are combined in this technique.
teins for specific chemical groups. By using antibodies, The antigen-antibody reaction is allowed to take
antigens could be easily separated (Fig. 31.11). Con- place. At the end of the incubation period, the tube will
versely, antibodies can be purified by passing through a contain free and bound antigen (labeled or unlabeled),
column containing the antigen. as shown in Box 31 .1. The bound and free forms are
Chapter 31: Laboratory Techniques, Quality Control and Metabolic Diseases 437

BOX 31 .1: Principle of rad101mmunoassay


Ag +Ag*+ [Ag-Ab) + [Ag*-Abl + Ag + Ag*
(bound (free
radio- radio-
Antigen Antibody in Enzyme Substrate added.
activity) activity)
coated patient's linked Color proportional to
well serum binds antibody antibody in patient's
to antigen binds to serum
separated by protein precipitating agents. The radio- specific antibody
activity of the bound form in the precipitate is measured.
F ig . 31 .12: Indirect ELISA to detect antibody
However, since the method is using harmful radioacti-
vity, the method is rarely used nowadays. (T4 in this example} present in the serum is fixed on the
antibody. Excess antigen and other unwanted proteins
I EUSA TEST
---- are washed out.
ELISA is the abbreviation for enzyme-linked immuno- Then, specific antibody (antibody against T4, tag-
sorbent assay. The ELISA techniques are widely ged with enzyme (HRP) is added. If the antigen is
used not only for hormone measurements but also for already fixed, the antibody-HRP-conjugate will be fixed
detecting any other growth factors, tumor markers, in the well. Then a color reagent, containing hydrogen
bacterial or viral antigens, antibodies against microbes peroxide (Hp 2 ) and a chromogen is poured over. HRP
and any other antigens or antibodies in biological flu ids. reacts with hydrogen peroxide to produce nascent oxy-
gen. This oxidises a non-colored chromogen to colored
Antibody Detection by ELISA substance.
This is known as "sandwich" ELISA. Development
A good example is the test for detection of HIV antibody.
of a brown color indicates that the antigen is originally
In patients with AIDS, the human immunodeficiency
present in the patient's serum. This is diagrammatically
virus (HIV) produces specific antibody. To detect the HIV
represented in Figure 31 .13. Color developed is pro-
antibody, the following method is used.
portional to the antigen in the serum. Therefore,
Antigen from HIV is coated in the wells of a multiwell
intensity of the color may be measured, from which the
plate. Patient's serum is added, and incubated. If it
concentration of the antigen is calculated.
contains the antibody, it is fixed. The wells are washed.
This is to remove excess antibodies in serum. I COLORIMETER_
Next a second antibody (antibody against human
Colored solutions have the property of absorbing light of
immunoglobulin) conjugated with HRP (horse radish
definite wavelengths. The amount of light absorbed or
peroxidase) is added . Then a color reagent, containing
transmitted by a colored solution is in accordance with the
hydrogen peroxide and a chromogen is poured over.
Beer-Lambert law. As per the Beer's law, the intensity of
HRP reacts with hydrogen peroxide to produce nascent
the color is directly proportional to the concentration of
oxygen. This oxidises a non-colored chromogen to
the colored particles in the solution. The Lambert's law
colored substance. If color develops, it means that the
states that the amount of light absorbed by a colored
antibody was originally present in the patient's serum
solution depends on the length of the column or the
(Fig. 31.12). Here the color developed is proportional
depth of the liquid through which light passes. The Beer-
to the antibody concentration. Therefore, from the
Lambert law combines these two laws.
color intensity, the concentration of the antibody can be
In the colorimeter, the length of the column through
calculated. HIV antibody is an example, any antibody
which the light passed is kept constant, by using test
could be detected by using the specific antigen .
tubes or cuvettes of the same diameter for both test and
standard, so that the only variable is the concentration .
Antigen Detection by ELISA Method The ratio of intensity of emergent light to intensity of
At first, specific antibody is fixed to the well of a microtiter incident light (E/i) is termed as transmittance (T). The
plate. A good example is the assay of thyroid hormone, absorbance is expressed as - log T. The Optical Density
T4. The patient's serum is added in the well, and is calculated as - log T. The plot of the concentration
incubated for 30 minutes at 37°C. By this time, antigen versus transmittance is not linear, but a graph of the
438 Section C: Clinical and Applied Biochemistry

Enzyme TABLE 31 .1: Color of filter and color of solution are comple-


4
mentary
0 Color
5 Color offilter
3 Enzyme Wavelength Color ofsolution
* reaction
linked Violet 420 Brown
antibody
Blue 470 Yellowish brown
Green 520 Pink
Yellow 580 Purple
Hormone 2
Red 680 Green/ blue

Anti T• antibody Incident light

Well in the
microplate

Fig. 31.13: Antigen detection by ELISA; 5 sequential steps


explained
Light Filter Photoelectric
concentration against absorbance (OD) is linear. Since Sample in cuvette cell
it is in logarithmic scale, values too low or too high are Fig. 31.14: Photoelectric colorimeter
not acceptable for accurate results (sensitive range is
between 0.1 to 0.6). 4. Detector (photocell)
Most of the clinical chemistry estimations are done
5. Display as a digital meter (Fig. 31 .14).
by colorimetric methods. A colored derivative of the com-
The solution absorbs part of the light and the
pound to be measured is prepared and its absorbance
remaining transmitted light is allowed to fall on the photo-
or OD is measured using a photoelectric colorimeter.
cells. The electrical impulse thus generated is measured
This value is compared with that of a standard of known
and is suitably displayed.
concentration. The basic components of a photoelectric
In clinical laboratory, serum sample and reagents
colorimeter are:
are mixed and incubated at 37°C for a fixed time, say
1. Light source
2. Filter, used for selecting the monochromatic light 10 minutes, to develop the color optimally. After the
(mono = single; chrome = color). Filters will absorb incubation period, the OD is ascertained and the con-
light of unwanted wavelength and allow only mono- centration of the substances is calculated. This is called
chromatic light to pass through. This light will have end point analysis.
maximum absorbance when passed through a par- On the other hand, the serum and reagents are
ticular colored solution. The color of filter should incubated, and readings are taken at 2 and 3 minutes
be complementary to the color of the solution. exactly; and from the difference in OD between the two
Table 31.1 gives the color of filters to be used for values, the concentration is calculated. This is the kinetic
the colors of solutions analysis. Here the optimum color is not developed; but
3. Sample holder, called "cuvette", made up of glass is quicker and hence is often used in autoanalyzers.
tubes
Spectrophotometer
A spectrophotometer has all the basic components
of photoelectric colorimeter with more sophistication.
Wavelengths in the ultraviolet region are also utilized in
the spectrophotometer. Light is separated into a continu-
ous spectrum of wavelengths and passed through the
solution. [In colorimeter, wavelengths of one color are
grouped together].
Johann Lambert August Beer Walther Nemst,
1770 1852 NP 1920 The advantage of the spectrophotometer over
1728-1777 1825-1863 1864-1941 the colorimeter, is that the former is 1000 times more
Chapter 31: Laboratory Techniques, Quality Control and Metabolic Diseases 439
Ron3.orPi o.:c:cm
i;ensitive. Therefore, even minute quantities of the Discrete Selective Analyzers
substance (very dilute solution) can be assessed in the
They have the capability of wmlyzing sirn111taoeo11$ly
spectrophotometer.
many parameters in a single sample. It is therefore called
The characteristic absorption maximum (in wave-
sample oriented.
length) of some important substances are; protein,
peptide linkage (220 nm); protein, tryptophan units (280 Dry Chemistry Systems )(
nm); nucleic acids (260 nm): NADH (340 nm).
Here all the reagents necessary for the reaction are
Ion-selective Electrodes embedded on a plastic matrix in their dry state. The reac-
tion is initiated by the addition of the blood/serum over
Ion-selective electrodes are available to detect sodium,
the matrix and the color that is produced by the reaction
potassium, calcium and lithium. Glass electrode, made
is measured by reflectance spectrophotometry. The
up of very thin glass membrane, allows ions to permeate
method is very useful for emergency practice of critical
through. The potential difference across the glass mem-
care medicine or point of care tests.
brane of the electrode is quantitated by the instrument.
Glucometers are working on this dry chemistry
principle. Glucometers are usually used by the diabetic
The pH Meter
patients for blood glucose analysis at their home. Sev-
The hydrogen ion concentration and pH are described in eral assays have now been adapted for Point of Care
Chapter 27. pH= - log [W). pH is measured accurately tests (POCT).These tests can be done by the attending
by potentiometric methods. physicians at the bedside for quick results.
The Glass electrode is made of a very thin glass
membrane, which allows passage of hydrogen ions.
Inside the bulb of the glass electrode, a solution of
I MASSSPECTROM_ET_R
_Y_ _ __
known hydrogen ion concentration is kept and a silver- Mass spectrometry (MS) is a very sensitive quantitative
silver chloride electrode connects the solution to the analytical technique used to measure a wide range of
input part of the instrument. The reference electrode or clinically relevant analytes. MS identifies molecules
calomel electrode (mercury with mercurous chloride) is based on their mass or molecular size. When inter-
filled with KCI solution. The potential difference across faced to HPLC or GLC, the MS functions as a powerful
the glass membrane (inside and outside) is calculated detector.
by Nernst (Nobel Prize, 1920). A sample is vaporized such that positive charge is
produced on the molecule. Electrical field is applied,
I AUTOANALYZ_E_R_ _ so that particles move. At the same time, a magnetic
field is applied, so the cations are deflected and hit
By means of this modern equipment, hundreds of blood on the detector. For molecules of identical charge, the
samples could be analyzed within a short tim? . time required for the molecule to reach the detector
"$~PA) is inversely proportional to the mass of the molecule.
Semiautomated Photometric Analyzers Conventional MS identifies molecules of 4000 D or
These are semiautomated analyzers. Some of the steps less. For higher molecular size materials, time-of-flight
involving pipetting of the sample, reagent mixing and mass spectrometers are useful (Fig. 31 .15).
incubation are manij611y done, while measyremeot, c_gJ;
culation and d.isp,!By or printing of the results are a~ - Tandem Mass Spectrometry (MS-MS)
matically done. The instrument can analyze Two mass spectrometers are arranged sequentially. The
param~ ter at a time (e.g. urea).l)'l>wro.~) first MS separates big peptides, based on their mass.
. From this observation, a single peptide can be directed
Automatic C~ g:~f nalyze~ ~CAty the second MS (called par~~t ion or precursor ion).
Here all samples are analyzed for one~ uent only The parent 10n enters the collision cell of the second
and analysis is parameter oriented. Once samples are MS. Here the ions collide with argon gas molecules
loaded, all processes are performed automatically with and are broken into smaller ions. These daughter ions
no manual intervention. (product ions) are detected and quantified by their
440 Section C: Clinical and Applied Biochemistry

Principle of tandem mass spectrometry

-- -----------------
Ion beam
Digest
' Ionization of
Source molecules
Protein
vaporized
biomolecules

.....
..... Identification - ----
-.N
v-.. l
Energy
. .. -IV
)
Smaller - ii l
molecule rh i l spectrum
urh \-
U[:J
Bigger
molecule Fig. 31 .16: Tandem mass spectrometry

Ion detectors
BOX 31.2: Types of preanalyt1cal variables
Fig. 31.1 5: Mass spectrometry
Test conducted: The appropriate test should be requested and
performed.
mass spectrum in the second instrument (Fig. 31.16). Patient identification: The labeling of specimen may be imp-
MS-MS is used for identification and quantification of roper. Corrected by bar coding.
proteins, drug screening, pesticides and pollutants and Turnaround time (TAT): The time required from the specimen
screening of inborn errors of metabolism, especially reaching the laboratory and the result being dispatched should
be kept minimum. Time of arrival, complet ion of test and
organic acidurias. dispatch should be noted.
Laboratory logs: Entry of patient and test details in laboratory
I REFERENCE VALUE_S_ registers and computers.
Transcription errors: Substantial number at different levels
A patient's results have to be compared with the ref- especially if sample number is high. Electronic identification and
tracking of specimens. Check digits, units and test correlation.
erence values to either confirm a clinical diagnosis or
Patient preparation: Improper standardization of the collection
exclude a particular disease. Reference values and time and manner of collection.
<
normal values are not the same. The term "normal" Specimen collection: Container, anticoagulant time taken to
was used arbitrarily in the past, but was found to be send specimen to laboratory, corrected by using vacutainer
ambiguous when used in a statistical sense. As per tubes and collection of samples by laboratory personnel.

IFCC (International Federation of Clinical Chemistry


and Laboratory medicine) recommendations, reference on the period of gestation. So, the reference values for
values are based on that of a reference individual. these parameters must be available in laboratories on
The reference values are established in healthy a weekly basis. The reference values are compared
individuals. Refere·nce values are based on applica- with the obseNed values and expressed as Multiple of
tion to values generated in the Median (MoM) values. The de~ er
laboratories. Ideally, the group of reference individuals to continue the pregnancy or terminate, is based on the
should be a random sample (equal chance of being · MoM values for a particular subject. This is obtained by
selected). dividing each test result by the median for the relevant
gestational week.
Reference Values and Observed Values
PREANALYTICAL VARIABLES _,
In actual clinical practice, often we have to compare
several obseNed values with their reference inteNals. Preanalytical variability is defined as errors which occur
For analytes which are under hormon;;ir,.ir.-.metabolic when non-analytical factors change the concentrations
control, there may be subject based ~ iati ; iolra- of analytes, so that the results do not reflect the condi-
indiv· · -individual e.g. p~ e levels in tion of the patient. Preanalytical variability may be due to
pre nant women). a) pre-collection causes, or b) blood collection causes.
The variations obseNed in the level of alpha-fetopro- Important types of preanalytical variables are shown in
tein, beta hCG and estradiol in maternal serum depend Box 31 .2. Clinicians should know about these variables;
Chapter 31 : Laboratory Techniques, Quality Control and Metabolic Diseases 441

then only a correct interpretation is possible. Preanalyti- The possibility of enzyme inhibition especially creatine
cal variables may be controllable so that their effect can kinase, ALP, ACP, amylase and LOH are observed with
be minimized.The terminologies, preanalytical as well as several of these anticoagulants. Oxalates are unsuitable
analytical variables are recently re-designated as pre- for estimation of sodium and potassium also.
examination or examination variables. Pre-analytical Heparin is the most widely used anticoagulant for
variables include time of collection , posture, prolonged clinical chemical analysis. It interferes the least with test
bed rest, exercise, circadian variations and diet. procedures.
Ethylenediarnine tetraacetic acid (EDTA) is a chelating
Noncontrollable Variables agent, and is particularly useful for hematological exami-
Factors like age, sex, race, pregnancy, lactation, men- nation because it preserves cellular components of the
strual cycle, altitude, temperature, stress, obesity, fever, blood. It may affeict some of the clinical chemistry tests.
trauma and transfusion can all cause significant varia- Sodium fluc,ride is usually used as a preservative for
tion in several specific parameters. Separate reference blood glucose b}I inhibiting the enzyme systems involved
standards are maintained for children. adult and aged. in glycolysis. Without an anliglycolytic agent, the blood
glucose concentration decreases about 10 mg/dl per
~ PECIMEN COLLECTION hour at 25°C andl false results may be obtained. However,
such serum should not be used for enzymatic assays.
Plasma values are higher than serum values in the
Citrate is widely used for coagulation studies. Oxa-
case of calcium and chloride. Plasma values are less late inhibits blood coagulation by forming insoluble
than serum values for albumin, ALP, AST, bicarbonate, complexes with calcium ions. Potassium oxalate may
creatine kinase, urea, uric acid, sodium, glucose (5%), be used at a concentration of 1-2 mg/ml; at higher con-
phosphate (7%) and potassium (8.4%). centrations, oxalate may cause hemolysis.
Arterial blood is used for ABG (arterial blood gas)
analysis. Most of the other investigations use venous Storage and Preservation of Specimens
blood. However capillary blood gives higher values Plasma or serum should be separated from the cells
than venous for glucose and potassium, lower values for as soon as possible, and certainly within 2 hours. If the
bilirubin, calcium, chloride and sodium, but no difference specimen cannot be analyzed at once, the separated
for urea and phosphate. serum should be stored in capped tubes at 4°C. When a
sample is to be :stored beyond a week, it should be kept
Hemolysjs at -20°C in a freiezer.
Vigorous suction by the syringe during blood collection,
or forceful transfer from the syringe to the container may Care of lnfe,cted Specimens
cause hemolysis of blood. Even minimal hemolysis will Samples from patients, who are positive for HBsAg
alter the values of potassium. The presence of visually (hepatitis B su1face antigen) or HIV, should be kept
appreciable hemolysis is seen when the hemoglobin separately, in protected bags. The specimen is disposed
level is more than 200 mg/L. Enzymes like aldolase, LOH, off properly. An), serum specimen may be from patients
ACP, ICD and electrolytes potassium, magnesium and of these disea:ses. Therefore, all serum or blood
phosphate are elevated by the presence of hemolysis. samples should be handled as if they are infected
Hemolyzed samples give falsely low values for bilirubin and adequate precautions should be taken.
and special care is to be taken when blood is collected
for neonatal bilirubin estimation . Urine Colleiction
An early morn1ing fasting specimen is generally the
Anticoagulants most concentrated specimen. Therefore, it is preferred
for microscopic examination and for the detection of
Serum from coagulated blood is the specimen of choice
proteins and beta chorionic gonadotropin.
for many assay systems. Commonly used anticoagu-
lants are heparin , EDTA, oxalates, citrate and fluoride.
Timed Urin1e Specimen
Of these, lithium heparin is best suited for most of the
biochemical estimations. All other anticoagulants chelate Usually, urine sample is collected for a 24-hour period.
calcium and hence unsuitable for calcium estimation. This will minimize the influence of short-term biological
442 Section C: Clinical and Applied Biochemistry

variations and diurnal rhythms. Generally, collection of


urine samples are done from 6 AM to next 6 AM. The
bladder should be emptied when the collection is started
(6 AM), and this urine is discarded. Thereafter, all the
urine should be collected. The next day urine is voided
at 6 AM, and this sample is also collected.

Figs. 31 .17A to C:: (A) Imprecise; (B) Precise but inaccurate:


Urine Preservatives (C) Precise and accurate
The preservatives are used (1) to reduce bacterial action,
(2) to minimize chemical decomposition, and (3) to
Accuracy
decrease atmospheric oxidation of unstable compounds. It is the closeneiss of a result to the true value. For
The most satisfactory form of preservation of urine spe- example, if one !technician performs a test on a serum
cimen is to refrigerate it during the collection. Forma- which is known to contain 100 mg/dl glucose and obtains
lin, thymol, chloroform, toluene, concentrated HCI and a result of 99 mg/ dl. A second technician does the same
glacial acetic acid are the commonly used urine preser- test on the same sample, and gets the result of 95 mg/dl.
vatives. Then the value of the first technician is considered as
accurate. Values, farther away from the true value are

IQUALITY CONTROL (QC) less accurate tha1n those closer.

Quality control should become an integral part of the Precision


function of a clinical chemistry laboratory. The purpose This refers to the reproducibility of the result. If one tech-
of quality control is to ensure the reliability of each nician performs ,glucose analysis on the same sample
measurement performed on a sample. Quality is defined on three different occasions and obtains 100 mg/dl,
as conformance to satisfying the needs and expectations 99 mg/dl and 101 mg/dl respectively, then the results
of the customers. have been reproduced very well, and the precision is
The term error should be discriminated from mistake very good. Precision depends on the technique, the rea-
in laboratory practice. A mistake is an avoidable error. gents, as well as on the technical ability of the technician
(Figs. 31 .17A to C).
Therefore, quality management programs aim at totally
avoiding mistakes. However, in spite of all efforts we can
Precision is how close repeated measures of the
same sample lit:l; accuracy is how close the value
only minimize errors.
reported is to the true value and bias describes variables
A central reference laboratory sends a serum
which may affect precision and accuracy and lead to
sample containing known quantity of a substance: this is
over or under reporting.
analyzed in a peripheral (small) laboratory. If the result
obtained in the peripheral laboratory is the same as that of Specificity
the reference laboratory, then the arrangements available
Specificity of a reaction denotes that only one substance
in the peripheral laboratory is said to be reliable. This is
will answer that particular test. For example, in the case
called external quality control. This type of checking
of glucose oxidase method, only glucose molecules are
is usually done once or twice in a month. Moreover, the assayed. So it is a very specific method. But if the reduc-
peripheral laboratory itself makes a reference standard ing property of thle glucose is utilized for the assay pur-
serum sample, and checks the results on a daily basis; pose (e.g., Nelson Somogyi method), then other reducing
this is called internal quality control. agents in the blood will interfere in the reaction, and
Control material is a device, solution, lyophilized hence specificity is lowered. Specificity is determined by
material or pooled specimen or artificially derived mate- the method of thEi analysis.
rial to be used in quality control process. The use of
reference materials, calibrators and controls will help a Sensitivity
routine method to be correlated to a reference method It indicates whelther the method could be utilized to
when evaluated. test a very dilute solution. For example, biuret method
Chapter 31: Laboratory Techniques, Quality Control and Metabolic Diseases 443

is used for solutions having a few gram of protein/dl. BOX 31.3: Common medical indications for a referral to a
Spectrophotometric method is useful to detect a few genetic counselor
milligram of protein/dl , while ELISA method is employed 1. Advanced maternal age (greater than 35 years)
f the solution has only microgram of protein/dl. Thus 2. Positive maternal serum screening
3. Patient or family member with a known Mendelian disorder
::LISA method is most sensitive.
4. Prior pregnancy with a chromosomal disorder
The sensitivity of an assay is the fraction of those
5. Family history of mental retardation or birth defect
with a disease that the assay correctly predicts. Speci-
6. Fetal anomalies or markers by sonogram
4city is the fraction of those without the disease that the 7. Recurrent pregnancy loss/stillbirth
assay correctly predicts. A test should be both sensitive 8. lnfertilit
and specific. Generally speaking, as the sensitivity is
increased, specificity is decreased.
Genetic Counseling
Quality Control Charts This process involves an attempt by trained persons to
These are used to compare the observed control values help the individual or family to:
with the control limits and provide a visual display which 1. Comprehend the medical facts including the dia-
can be quickly reviewed. A daily QC chart should be gnosis, probable course of the disorder, and the
available in the laboratory. The control chart belps to available management,
detect accuracy problems shift in mean and recision 2. Appreciate the way heredity contributes to the
problem shift in SD). The values will indicate if the disorder and the risk of recurrence in specified
relatives,
analytical run is in control (acceptable) or out of control
(unacceptable). If any of these changes are noticed, 3. Choose a course of action which seems to them
prompt action is warranted. Commonly employed charts appropriate in view of their risk, their family goals,
and their ethical and religious standards and act in
in the laboratory practice are Levey-Jennings chart,
accordance with that decision, and
Westgard multirule chart and Cumulative sum chart.
4. To make the best possible adjustment to the disorder
in an affected fam ily member and/or to the risk of
External Quality Assurances (EQAs)
recurrence of that disorder. Indications for referring
Typically, a national organization will send the same a patient to a genetic counselor are shown in
sample to different laboratories; the laboratories will Box 31 .3.
send the results to the organization, which will measure Ultrasound is the main diagnostic tool for prenatal
l1ow similar the results are. Internal QC maintains diagnosis of congenital disorders. Ultrasound screening
the accuracy and precision of the analytical method, is offered routinely to all pregnant women. It is usually
whereas EQAs is necessary for maintaining long-term performed at 18- 23 weeks of pregnancy.
accuracy of analytical methods.
Amniocentesis
lf._RENATAL DIAGN_
O_S_IS_ _ __ Prenatal diagnosis of inborn errors of metabolism can
About 2% of live births are associated with a genetic be made by enzymatic assays of cultured amniocytes.
defect. In addition, genetic disorders are also a major
cause of pregnancy loss as well as perinatal mortality Chorionic Villi Sampling (CVS)
and morbidity. Taking a detailed family history is very
The most common indications for CVS are advanced
important in prenatal genetic evaluation, permitting the
maternal age, or a biochemical or genetic disorder indi-
counselor or physician to identify problems for which a
cated by molecular markers. The genetic makeup of the
couple may be at risk. One of the most important of these
placenta is identical to that of the fetus.
1s a three-generation family history analysis (pedigree
analysis). Details to be obtained include miscarriage,
Cordocentesis
neonatal or early life death, consanguinity as well as
-specific information of mental retardation, anemia and Fetal blood sampling (cordocentesis) can be performed
congenital anomalies. at 20 weeks gestation.
444 Section C: Clinical and Applied Biochemistry

Cytogenetics and Molecular BOX 31 .4: Suggested protocol for screening


1. Measurements of NT, PAPP-A and free HCG are made in the
Cytogenetics first trimester, but not interpreted or acted upon until the
Samples include amniocentesis, transabdominal and second trimester.
transcervical chorionic villus sampling (CVS), fetal blood 2. In the second trimester a second serum sample is drawn and
quadruple test performed.
sampling and fetal skin biopsy.
3. Results for all the six tests, NT, PAPP-A, AFP, uE3, hCG and DIA
are combined into a single risk estimate for interpretation in
Biochemical Screening the second trimester.
They are cheap, easy, quick and reliable. But they do 4. 85% detection rate for DS with only 1% false positive is
achieved.
not give definitive answer. On the other hand diagnostic
(NT = Nuchal translucency; PAPP-A = Pregnancy associated
tests are performed only on "risk" population, they are = =
plasma protein-A; AFP Alpha-fetoprotein; uE3 unconju-
generally expensive; but will give definitive answer. =
gated estriol; hCG human chorionic gonadotropin; DIA =
Dlmeric inhibin A)
Maternal Serum Screening
Prenatal screening has become standard obstetric Unconjugated estriol (uE3): It is an estrogen with
practice in all pregnancies at risk. Four analytes-alpha- 3 hydroxyl groups and 3 organs (fetal adrenal, fetal
fetoprotein (AFP), human chorionic gonadotropin (hCG), liver and maternal liver) are involved in the synthesis.
unconjugated estriol (uE3), and inhibin-are estimated. Maternal serum uE3 levels rise by 8 weeks of gestation
The triple screen (AFP, hCG, uE3) is done during the and continue to increase through out pregnancy. A 25%
second trimester between 14 and 18 weeks. Neural tube reduction in uE3 levels was found when the fetus had
defects, trisomy 21 and trisomy 18 are detected. chromosomal aneuploidy.
Alpha-fetoprotein (AFP) is the major serum protein The triple screen has a high detection rate , 80%
of the fetus synthesized by the fetal liver and yolk sac. for neural tube defects and 55-60% for chromosomal
There is a steady increase in AFP level in maternal aneuploidy and a false positive less than 5%.
serum from 10th week of gestation and reaches a peak
by 25 weeks of gestation in unaffected pregnancy. Then The Quadruple Test (Quad Screen)
the maternal serum alpha-fetoprotein (MSAFP) steadily This includes AFP, uE3, hCG and an additional marker
declines until term. In fetal serum and amniotic fluid, the lnhibin-A. Dimeric lnhibin A (DIA) is a glycoprotein
AFP level reaches a peak by 9th week of gestation and produced by the placenta. It is a dimer, but with dissimilar
then slowly falls till term. In neural tube defect (NTD), subunits alpha and beta. lnhibin A is measurable in
the AFP is increased but in chromosomal aneuploidy it maternal serum and has a feed back effect on FSH
is decreased. In diabetes mellitus, AFP was found to be secretion. The level increases in the first trimester until
40% lower than non diabetics. In twin pregnancy, AFP 10 weeks and then remains stable up to 25 weeks of
was higher than those having single fetus. gestation. Thereafter, it increases to reach a peak by term.
Human chorionic gonadotropin (hCG) is a glyco- The DIA levels are increased in OS and remains elevated
protein hormone produced during normal pregnancy through out the second trimester unlike AFP and uE3
by the trophoblast and placenta. hCG appears in that increase and hCG that decreases during the testing
maternal serum by 6 to 8 weeks and reaches a peak period. Reference value is 0.7-2.5 mg/L in unaffected
by 10 weeks. By the second trimester it falls to a con- pregnancy at second trimester. DIA is an independent
stant level by 18 to 20 weeks. hCG is a heterodimer variable having no correlation with maternal age, race or
having alpha and beta subunits of which the beta sub- diabetes mellitus. There was no correlation with AFP and
unit is specific for hCG. A marked increase of about uE3, but significant correlation was found with hCG. The
twice the normal value was found in pregnancies with suggested protocol for screening is given in Box 31.4.
trisomy 21 during the second trimester. Free beta hCG Nowadays, a double marker test is also done in the
was increased during the 1st trimester in Down's syn- first trimester between 1Qth and 12th week of gestation.
drome, even though total hCG (alpha and beta subunits Free HCG and Pregnancy associated plasma protein-A
combined) remained normal. Both were increased dur- (PAPP-A) are measured. A low PAPP-A and elevated
ing the second trimester in Trisomy 21 . In trisomy 18, the free HCG are suggestive of high risk. Hence a sequential
hCG levels remain lower than normal. screening is done in the second trimester.
Chapter 31: Laboratory Techniques, Quality Control and Metabolic Diseases 445

Biochemical Genetics individuals with S gene was found to be 15.1 %. The inci-
dence of GPO deficiency was reported as 28% in males
Biochemical tests for diagnosis of inherited metabolic
and 1% in females.
disorders consist of identification of abnormal metabo-
Screening newborn infants for phenylketonuria
ites or abnormal levels of metabolites or the defective
(PKU) was the first, large-scale genetic screening initia-
or deficient gene product. Fetal tissues (chorionic villi
tive to be widely adopted. High-risk individuals should
and fetal liver biopsy) or cells (trophoblasts, amniotic
be detected by a simple, inexpensive test with high sen-
fluid cells, fetal erythrocytes, and leukocytes) are used
sitivity (the proportion of affected infants with a positive
·'or analysis of the enzyme or other protein primarily
screening test), specificity (the proportion of unaffected
nvolved. Testing of parents, the index case and unaffec-
infants with a negative test), and predictive efficiency
'!ed siblings can provide valuable information.
(ratio of true-positive to false-positive tests). For screen-
ing tests for PKU , see Chapter 19, under phenylketon-
Enzyme Assays and Molecular uria.
Techniques
Direct demonstration of abnormality or deficiency of the Screening Technology
gene (molecular techniques) or gene product (biochemical The commonly employed screening tests are: 1 . Test
techniques) is the preferred diagnostic approach . These for PKU (phenylketonuria), 2. for TSH (congenital hypo-
are carried out in trophoblast or amniotic fluid cell cul- thyroidism); 3. 17-alpha-hydroxy progesterone (for
tures. congenital adrenal hyperplasia), and 4. Tandem mass
Molecular genetics techniques like qRT-PCR, spectrometry (useful for most other disorders).
Southern blotting, linkage analysis as well as mutation The sensitivity of tandem MS-MS testing in screen-
analysis and a variety of PCR-based techniques have ing for PKU is greater than the sensitivity of any other
been used. tests. Newborn screening programs tend to focus on
Molecular biology techniques are described in three groups of metabolites: amino acids, fatty acid
Chapter 44. oxidation intermediates, and short chain organic acids.

NEWBORN SCREENING LABORATORY INVESTIGATIONS


Newborn screening aims at the earliest possible recog-
nition of disorders to prevent the most serious con- L TO DIAGNOSE METABOLIC
DISORDERS
sequences by timely intervention. Screening is not a They include routine biochemical tests like measure-
confirmatory diagnosis and requires further investiga- ments of arterial blood gases, plasma electrolytes,
tions. Newborn screening may be done within the first glucose, urea, creatinine, liver function tests, routine
week after birth, because metabolic errors, if recog- hematologic tests, and various endocrinological tests,
nized later, contribute to significant morbidity. Deve- such as thyroxine, triiodothyronine, thyroid stimulating
loped countries are using tandem mass spectrometry to hormone. Studies also include measurements of lactate,
screen for a wide range of disorders. This technique is pyruvate, amino acids, 3-hydroxybutyrate, acetoace-
available only in a few centers in India. In Indian stud- tate, and free fatty acids in plasma; analyzes of urinary
ies, organic acidurias, homocysteinemia, hyperglycine- organic and amino acids; tests for mucopolysaccharides
mia, MSUD, PKU , congenital hypothyroidism (C H), and oligosaccharides in urine; and measurements of
congenital adrenal hyperplasia (CAH), GPO deficiency, certain trace elements, such as copper. The ultimate
biotinidase deficiency and galactosemia were found to specific diagnosis of inherited metabolic disease gene-
be the common errors. Even though individually rare, rally requires the demonstration of a primary biochemi-
collectively a very high prevalence of inborn errors of cal abnormality, such as a specific enzyme deficiency, or
metabolism to the extent of 1 in every thousand new- mutations that have been shown to cause disease.
borns was observed in Indian studies. The incidence of The onset of signs of disease will give an important
CH is 1 in 2500 births. In India, the carrier frequency clue to the nature of the underlying disorder. Diseases
of beta thalassemia is about 3.3%. The frequency of presenting with a sudden onset are generally more likely
446 Section C: Clinical and Applied Biochemistry

BOX 31 .5: Class1f1cat1on of disorders of amino acid BOX 31 .6: Disorders of organic acid metabolism
metabolism 1. Alkaptonuria
1. Hyperphenylalaninemias 2. Branched chain organic acidurias (e.g. MSUD)
2. Hypertyrosinemias 3. Propionic acidurla, methylmalonic acidu ria
3. Disorders of histidine metabolism
4. Defect in lysine oxidation: 2-keto adipic acidemia and glu-
4. Disorders of proline and hydroxyproline
taric acidemia
S. Hyperornithinemias
S. Gamma-glutamyl cycle disorders
6. Urea cycle disorders
7. Errors of lysine metabolism 6. Lactic acidemias
8. Disorders of branched chain amino acids and keto acids 7. Mitochondrial fatty acid oxidation disorders
9. Disorders of trans-sulfuration 8. Oxidative phosphorylation disorders
10. Nonketotic hyperglycinemia 9. Glutanc acidemia type II (respiratory chain).
11 . Ot her disorders

abnormalities. Enzyme-replacement therapy is available


to be inherited defects of small molecule metabolism. for Gaucher disease, Fabry disease, MPS I, MPS 11 ,
Definitive diagnosis generally requires further in vitro MPS VI , and Pompe disease.
metabolic studies, usually specific enzyme assay.
Mucopolysaccharide Screening
Lactic Acidemia
It is performed for the diagnosis of different types of
Deficiency of pyruvate dehydrogenase complex is the
mucopolysaccharidoses (Table 31.2).
most common cause of lactic acidemia. In severe cas-
es, death occurs at neonatal period. In moderate cases,
Urine Screening Tests
profound mental retardation is observed. In mild cases,
developmental delay is noticed. Useful tests are Rothera's test (for ketone bodies),
Pyruvate carboxylase (PC) deficiency may also pro- Cynide nitroprusside test (for homocystine ), Ferric
duce moderate or severe lactic acidemia. chloride test (for amino acids), Benedict's test (reducing
sugars), Dinitro phenyl hydrazine test (for Maple syrup
Plasma and Urinary Amino Acids urine disease), Cetavlon test (for Hurler's syndrome)
They are useful in the diagnosis of specific and gene- and Ninhydrin test (for amino acidurias).
ralized aminoacidurias. Disorders of amino acid meta- Treatment policy of genetic disease is shown in
bolism are classified in Box 31.5. Box 31.7.

Organic Acidurias Other Clinical Biochemistry Topics


Box 31 .6 shows the organic acidurias. GC-MS is emplo-
The-basic principles of methods for analyzing different
yed for their diagnosis.
constituents in clinical laboratory are given under
Lysosomal Storage Disorders (LSDs) appropriate headings, e.g. serum proteins (Chapter 4 ),
serum enzymes (Chapter 6), blood sugar estimation
There are more than 40 known LSDs, out of which the
(Chapter 11 ), serum cholesterol (Chapter 15), serum
most common are: Gaucher disease, Pompe disease,
creatinine (Chapter 25), blood urea (Chapter 25), urine
Fabry disease, Niemann-Pick disease, mucopolysac-
analysis (Chapter 25), renal clearance tests (Chapter
charidosis I, and Krabbe disease. Almost all LSDs are
25) and molecular biology techniques (Chapter 44 ).
inherited as autosomal recessive traits, except for the
Normal reference values are given in the Appendix II.
X-linked Fabry and Hunter (mucopolysaccharidosis
type II [MPS II]) diseases. Individually, the incidence 0
of these inherited diseases is rare, ranging between •ii• Clinical Case Study 31 .1
1 in 50 000 and 1 in 1:4 x 106 . However, collectively,
LSDs are far more common, approximately 1 in 7000 to A 29 year-Old pregnant woman, while waiting in the
8000 birth. Clinical features suggestive of LSD include clinic for results of glucose tolerance test, complained
developmental delay, ataxia, seizures, weakness, and of severe abdominal pain. A doctor was called to the
dementia. An LSD diagnosis should also be considered laboratory, to see the patient. Doctor suspected acute
in the presence of coarse facial features, and bone pancreatitis, and requested the laboratory to assay
Chapter 31: Laboratory Techniques, Quality Control and Metabolic Diseases 447

-
TABLE 31.2: Urinary mucopolysaccharides in different muco- BOX 31.7: General pnnc1ples of genetic disease
polysacchandoses (MPS ) management
Heparan Keratan 1. Enhancing anabolism, depressing catabolism
Disease sulfate I sulfate 2. Correcting primary imbalance In metabolic relationships
Hurler's 3. Enhancing excretion of accumulated products
++++ + +
4. Providing alternate metabolic pathways
Hunter's +++ + +
5. Using metabolic inhibitors
Sanfilippo's - +++ + 6. Supplying products of blocked secondary pathways
Morquio's ++ + 7. Stabilizing altered enzyme proteins
Maroteaux- +++ ± + 8. Replacing deficient coenzymes
Lamy's 9. Artificially inducing enzyme production
Sly's ++ ± ++ 10. Replacing enzymes
Normal ± ± ± + 11. Transplanting organs.
12. Correcting underlying DNA defect.

serum amylase and lipase. The technician performed


the tests, using the plasma remaining from her glucose What is the defect in this patient?
tolerance test. The serum amylase value was 180 U/L What is the molecular basis of the condition?
(normal 50-120); but her lipase value was normal. In the 0
meanwhile, patient felt comfortable. But, the discrepancy • · Clinical Case Study 31 .1 Answer
in the laboratory results caused confusion in diagnosis. In the first analysis, serum amylase was tested on serum
So doctor requested a repeat analysis. A fresh blood samples taken for blood sugar estimation. For blood
sample was taken for the tests. This time, her serum
sugar estimation, fluoride is added. Fluoride will activate
amylase and lipase were found to be normal.
amylase. Preanalytical errors are the commonest cause
A. Explain the discrepancy in the amylase results.
of wrong laboratory results.
B. Do you think serum amylase value will come down
to normal levels within a few hours? 0
C. What are conditions in which serum amylase is • · Clinical Case Study 31 .2 Answer
increased? Likely diagnosis: Phenylketonuria (PKU).
D. What are the advantages of lipase estimation?
Biochemical basis of hypopigmentation: Phenyl-
0 alanine is competitive inhibitor of tyrosinase (key enzyme
• • Clinical Case Study 31 .2 in melanin synthesis).
A 1-year-old girl is brought to the hospital OP and mother Clinical correlation: The most common deficiency is
reports that the baby was not achieving the normal mile- in phenylalanine hydroxylase (autosomal recessive)
stones for a baby of her age. She also reports an unusual
resulting in the classic picture of PKU . Deficiency of
odor to her urine and some areas of hypopigmentation
dihydropteridine reductase and 6-pyruvoyl-tetrahydrop-
on her skin and hair. On examination, the girl is noted
terin synthase, enzymes necessary for the biosynthesis
to have some muscle hypotonia and microcephaly. The
of tetrahydrobiopterin will also cause PKU. If unrecog-
urine collected is found to have a "mousy" odor.
nized, the child will develop profound mental retardation
W hat is the most likely diagnosis?
What is the biochemical basis of the hypopigmented and impairment of cerebral function.
Treatment consists of dietary modifications with
skin and hair?
limitation of phenylalanine intake and supplementation
0 of tyrosine. The diagnosis of PKU and initiation of diet
• · Clinical Case Study 31 .3 modification needs to be implemented prior to 3 weeks
A 2-month-old baby was brought by her parents to the of age to prevent mental retardation.
pediatrician. She had pale skin and blonde hair. The
baby was otherwise healthy, was feeding well but was 0
unable to fix the gaze. Ophthalmic examination showed
8 Clinical Case Study 31 .3 Answer
absence of pigment in the retina. Two siblings had Oculocutaneous albinism has autosomal recessive
complete albinism, but parents were normal. inheritance. There is defect in synthesis of melanin,
448 Section C: Clinical and Applied Biochemistry

and results in pale skin, blonde hair and pink iris as 3. Chromatography is the technique used to separate
seen in this patient. Visual impairment is also typical of all classes of biomolecules.
this condition. Other symptoms include photophobia, 4. HPLC and GiC form two powerful chromatographic
nystagmus and functional blindness. The disease does techniques that can resolve mixtures with high sen-
not affect lifespan. sitivity and specificity.
Albinism can be tyrosinase positive or negative. 5. Affinity chromatography relies upon the biological
In tyrosinase positive type, enzyme is present but the property of the molecule to be separated. E.g. Affinity
melanocytes are unable to produce melanin due to a of an enzymH to its substrate, ligand to its receptor.
variety of reasons. In tyrosinase negative type, enzyme 6. Enzyme linked immunosorbent assay (ELISA) is
is absent or non-functional. In oculocutaneous type, skin, used in clinical diagnostics. It has the advantage
eyes and hair lack melanin, whereas in ocular albinism, of not being radio hazardous and having a longer
only eyes lack melanin. Ocular albinism may have shelf life.
X-linked inheritance and hence may be more common 7. Colorimeters work in accordance with the Beer-
in male offspring. The other type has equal chances in Lambert law.. As per the Beer's law, the intensity of
both male and female. the color is directly proportional to the concentration
Genetic testing confirms diagnosis. Treatment invol- of the colored particles in the solution .
ves protecting skin and eyes from direct sunlight. 8. Preanalyticall variability may be due to (a) pre-
collection caiuses, or (b) blood collection causes.
lh_EARNING POINTS, CHAPTER 31 9. Accuracy is tile closeness of a result to the true value.
1. The term 'Electrophoresis' refers to the movement 10. Precision refers to the reproducibility of the result.
of charged particles, through an electrolyte when 11 . Specificity of a reaction denotes that only one
subjected to an electric field. substance will answer that particular test.
2. PAGE is useful in determining relative molecular 12. Sensitivity indicates that whether the method could
weights of proteins. be utilized to test a very dilute solution.

PART-1: ESSAY AND SHORT NOTE Q UESTIONS

31-1 . Give the salient features of electrophoresis. What are the abnormalities that you could detect in serum
electrophoresis?
31-2. Describe the principle and applications of ELISA.

SHORT NOTE QUESTIONS


31-3. Electrophoresis. 31-8. Spectrophotometer
31-4. Paper chromatography. 31-9. Dry chemistry systems
31-5. Thin layer chromatography. 31-10. Prenatal di,a gnosis
31 -6. Affinity chromatography. 31-11 . Newborn s,c reening
31 -7. ELISA (enzyme linked immunosorbent 31-12. Preanalytic:al variables
assay). 31-13. Quality control

PART-2: MULTIPLE CHOICE QUE TIONS

31-1 . Which is the anticoagulant of choice for biochemi- 31 -3. Which of thte following is not an emergency investi-
cal estimations in general? gation?
A. EDTA A. PotassilJm B. Fasting glucose
B. Oxalate-fluoride mixture C. Creatinine D. Calcium
C. Lithium heparin D. Citrate 31-4. Which of the following parameters is not affected
31-2. Which of the following is not a point of care test even if the, assay is carried out after 6 hours of
(POCT)? collection?
A. ABG analysis B. Plasma electrolytes A. Glucose B. Potassium
C. Plasma glucose D. Plasma proteins C. Carbondioxide D. Albumin
Chapter 31: Laboratory Techniques, Quality Control and Metabolic Diseases 449

31-5. Which of the following parameters is not affected 31 -9. The quick.est method for separation of protein:
by using a tourniquet while collecting blood? A. Electrophoresis
A. Calcium B. Ammonia B. High performance liquid chromatography
C. Glucose D. Lactate C. Ion ex,change chromatography
31-6. Mobility of particles during electrophoresis is D. Thin layer chromatography
based on all the following factors, except: 31-10. Ultracentrifugation is useful for all the following ,
A. Current voltage and ampere except:
B. Quality of supporting medium A. Isolation of mitochondria
C. pH of buffer medium B. Preparation of blood group antigens
D. Solubility of particular protein C. ldentifi,cation of lipoproteins
31-7. A polysaccharide made up of D-galactose and D. Separation of proteins
anhydrogalactose units, which are used as matrix 31-11 . Which of the following has maximum absorption at
for electrophoresis is? 340 nm?
A. Cellobiose B. Dextran
A. Heme B. Proteins
C. Polystyrene D. Agarose
C. Nuclei,c acid D. NADH
31-8. In a protein molecule, normal glutamic acid is sub-
31-12. All the techniques listed below measures proteins,
stituted to valine. Both normal and mutated proteins
except
are electrophoresed at pH 8.6. The mobility will be:
A. Nephe lometry B. Flame photometry
A. Mutated protein moves faster than normal protein
C. ELI SA D. Radial immunodiffusion
towards positive pole 31-13. lmmunosC)rbent assays will use antibody conju-
B. Mutated protein moves slower than normal protein gated to a II the following reagents, except:
towards positive pole A. Alkaline phosphatase (ALP)
C. Mutated protein remains at the point of application B. Horse radish peroxidase (HRP)
D. Mutated protein moves to negative pole and nor- C. Riboflavin
mal protein to positive pole D. Biotin

ANSWERS OF MULTIPLE CHOICE UESTIONS


31-1 . C 31-2. D 31-3. B 31-4. D 31-!i. C 31-6. D 31-7. D
31-8. B 31-9. B 31-10. B 31-11 . D 31-1:~. B 31-13. C

PART-3: VIVA VOCE QUESTIONS AN ANSWERS

31-1 . What is electrophoresis? Paper chromatography and thin layer chromatography.


The term refers to the movement of charged particles 31-6. What is the basic principle of ion-exchange chro-
through an electrolyte when subjected to an electric field. matography?
31-2. Electrophoresis is commonly employed for what The separation is based on electrostatic attraction
purpose in laboratory? between charged molecules to oppositely charged
For serum electrophoresis. To see abnormalities in groups on the ion exchange resins (Fig. 31.10).
serum protein concentrations. 31-7. What is trhe quickest method for separation of
31-3. What is the principle of adsorption chromato- proteins?
graphy?
HPLC (Hi£1h performance liquid chromatography).
Separation is based on differences in adsorption at the
31-8. What is tllie advantage of radloimmunoassay?
surface of a solid stationary medium.
Very small quantities of substances could be accurately
31-4. What is partition chromatography?
measured ..
The components of the mixture to be separated are
31-9. What are the disadvantages of RIA, when com-
partitioned between the two phases depending on
the partition coefficient (solubility) of the particular pared to ELISA?
substances. 1. Since radioisotopes are used, only approved labo-
31-5. What are the common types of partition chromato- ratories could take up the assay.
graphy? 2. The shelf life of the reagent is short.
Nutrition1

Chapter 32 Fat Soluble Vitamins (A, D, E and K)


Chapter 33 Water Soluble Vitamins
Chapter 34 Mineral Metabolism and Abnormalities
Chapter 35 Energy Metabolism and Nutrition
Chapter 36 Detoxification and Biotransformation of Xenobiotics
Chapter 37 Environmental Pollution and Heavy Metal Poisons
t-------- - - Chapter 32
Fat Soluble Vitamins
(A, D, E and K)

Chapter at a Glance

The learner will be able to answer questions on the followini t opics:

I
D Vitam in A D Deficiency of vitamin D
D Wald's visual cycle D Vitamin E
D Deficiency of vit amin A D Vitamin K
fD Vitamin D

Vitamins may be defined as organic compounds occur- TABLE 32.1: Comparison of two types of vitamins
ring in small quantities in different natural foods and Fat soluble vitamins Water soluble vitamins
necessary for growth and maintenance of good health in Solubility in fat ~ uble Not soluble
human beings. Vitamins are essential food factors, which Water solubility Not soluble ~ luble

are required for the proper utilization of the proximate Absorption Along with lipids ~ sorption simple
Requires bile salts
principles of food like carbohydrates, lipids and proteins.
Carrier proteins l,fesent *No carrier proteins
"A vitamin is a substance that makes you ill if you don't Storage l~ ored in liver *No storage
eat it" (Albert Szent-Gy6rgyi , Nobel Prize winner, 1937). Excretion Not excreted .i<creted
Discovery of vitamins started from observation of Deficiency <CA(nifests only when *Manifests rapidly as
stores are depleted there is no storage
deficiency manifestations, e.g. scurvy, rickets, beriberi,
Toxicity ~ ervitaminosis may ¥nlikely, since excess is
etc. The vitamin theory was suggested by Hopkins in result excreted
1912 (Nobel Prize, 1929). The term "vitamine" was Treatment of ~ gle large doses may Regular dietary supply
coined from the words vital + amine, since the earlier deficiency prevent deficiency is required

identified ones had amino groups. Later work showed Major vitamins A, D, Eand K Band C
*Vitamin 8 12 is an exception.
that most of them did not contain amino groups, so the
last letter 'e' was dropped in the modern term of vita~
The vitamins are ~ai~ly classified into two:
1. The fat soluble vItamIns are A, D, E and K
& .
VITA_M_I_N
_ A_____________
Mccollum, Simmonds and Kennedy isolated vitamin A
2. Water soluble vitamin s are named as B complex in 1913. Richard Kuhn (Nobel Prize, 1938) identified
and C. The major differences between these two carotenes. Paul Karrer in 1931 elucidated the structure
groups of vitamins are given in Table 32.1. of vitamin A, (Nobel Prize, 1937).
454 Section D: Nutrition

H3C C H3 CH3 Reductase


?

!
Retinol (alcohol) • Retinal (Aldehyde) (-CHO)
CHO
(-CH20H) I
NAO+ NADH+H•
Retinoic acid (COOH)
All trans-retinal
Fig. 32.2: lnterconv1ersion of vitamin A molecules
H3C CH3

(
12 lntestino cell
Intestinal Liver
lumen

11-cis retinal OHC

Ir Beta-
l. ionone
ring

RE = Retlnol ester
Beta-carotene R = Retinol
RA = Retinoic acid Target cell
Fig. 32.1: Structure of vitamin A Chy = Chylomicrons
RBP = Retinol bincling protein
A) Chemistry Fig. 32.3: Vitamin A metabolism
Vitamin A is fat soluble. The active form is present only
in animal tissues. The provitamin, beta-carotene is pre•c)Transport froim Liver to Tissues
sent in plant tissues. Beta-carotene has two beta ionone
rings (Fig. 32.1). All the compounds with vitamin A activity The vitamin A from liver is transported to peripheral
. . . tissues as trans-r,etinol by the retinol binding protein
lT
are referred to as retmo1ds. They have beta-1onone
or
RBP h f . . Ad fi •
. In t e cas1e o vitamin
h RBP
e 1c1ency, t e 1eve 1
(cyclohexenyl) ring sys '.~ hree different compounds in blood falls.
with vitamin A ac~ y ar etinol (vitamin A alcohol),
)J
retinal (vitamin A aldehyde) and retinoic acid (vitamin At> ptake by Tissues
acid) (Fig. 32 .1 ). Th~ AV-t'IP.-f~~@_R]I The retinol-RBP complex binds to specific receptors on
by r · 1 readily reversible.
r
the retina, skin, gonads and other tissues. Vitamin binds
®_tinaj_is ox1a1zed to retinoic acid, which cannot ·be to cellul ar retingjc acjd binding._pwtejn.-4CR2P) and
converted back to the other forms (Fig. 32.2). The side finally to hormo~~responsive elewews-(1::iB.E.l-oLQ!::!A.
chain contains alternate double bonds, and hence many Thus genes are activated (Fig. 32.3).
isomers are possible. The all-trans variety of retinal ,E s·10chem1ca
. I RoIe of v ·t . A
I amm
also called vitamin A 1 is most common (Fig. 32.1 ). Vita-
min A2 is found in fish oils and has an e~ double bond l Wald's Visual Cycle
in the ring. Biologically important compound is 11-cis- '(§_emmrtlon of Ne,ve JmpuTs'e: Wald was awarded Nobel
retinal. Prize in 1967, fo r identifying the role of vitamin A in

S) Absorption of Vitamin A
Beta carotene is cleaved by a di-oxyge0gse, to form reti-
nal. The retinal is reduced to retinol by retinal reductase
present in the intestinal mucosa. Intestine is the major
site of absorption (Fig. 32 .3). The absorption is along
with other fats and requires bile salts. Vitamin is incor- Frederick Richard Paul Otto P Kurt
G Hopkins Kuhn Karrer Diels Alder
porated into chylomicrons and transported to liver. In the NP 1929 NP W38 NP 1937 NP 1950 NP 1950
liver, vitamin is stored a~ ~tinol palmitati.,) 1861- 1947 1900--1 967 1889-1971 1876-1954 1902- 1958
Chapter 32: r=at Soluble Vitamins (A, D, E and K) 455

Rods Cones
Rho, tpsin (Visuaxl pigment) ? Light
Pigment epitheli1um
Photo-
receptor
,;;lr;'
,§' Photoisomerization;
,z, ·~ Visual excitation els-retinal
Opsin
I\
l
11-cis-retinal All-trans-retinal OPSIN

Retinal
epithelium
l Retinal isomerase
in retina
els-retinal + - - - - - trans-retinal trans-retinal

I Blood J
7
OPSIN

Incident light

Fig. 32.5: Structure of retina showing rods and cones. The inset on
cis-retinal trans-retinal right side shows the structural alteration during photoisomerization
Liver J

George Wald to regenerate rhodopsin. Alternatively, all-trans-retinal


NP 1967 NADH is transported to liver and then reduced to all-trans-
1906-1997 ADH in liver
-retinal by alsgj!loLdehycicb\~~se (ADl;;L), an NADH
Retinol isomerase NAO+
in liver dependent enzyme. The all-trans-retinal is isomerized to
cis-retinol + - - - - - trans-retinol 11-cis-retinol and then oxidized to 11 -cis-retinal in liver.
Fig. 32.4: Wald's visual cycle. Blue color represents reactions in This is then transported to retina. This completes the
photoreceptor matrix. Green background represents reactions in
Wald's visual cycle (Fig. 32.4).
retinal pigment epithelium. Red depicts blood. Brown shows reac-
tions in liver
Dark Adaptation Mechanism(DA M)
vision hodopsi s a membrane protein found in the
For their work on information processing in visual path-
photorecep or cells of the retina. Rhodopsin is made up
ways, Torsten Wiesel and David Hubel were awarded
of the protein opfil_n and 11 -cis-retioal . When light falls
on the retina, the 11-cis-retinal isomerizes to all-trans- Nobel Prize in 1981 . Rods and Cons are diagrammati-
retinal (Fig. 32.4 ). The photon produces immediate cally represented in Figure 32.5.
conformational change so as to produce opsin + all-trans-
retinal. The all-trans-retinal is then released from the Therefore, when a person shifts suddenly from bright
protein. light to a dimly lit area, there is difficulty in seeing, for
Visual pigments are G-prote,ip..coufalle.d- reGepiGts. example, entering a cinema theater. After a few minutes,
The 11-cis retinal locks the receptor protein (opsin) rhodopsin is res~ · e vision is improved. This
in its inactive form Fi . 32.5). The isomerization and period is called(dark adaptation ti
photo-excitation ea ac 1va 10n of G-prote1 and v itam in A defi.ci.sW.
gen'®!!QQ_of cyclic-§JOl'E> Transducin is the G-protein I ilk, so doctors use tinted glasses, during fluoro-
in retina. The Cyclic GMP generates a nerve impulse in scopic X-ray examination of the patients.
the retina which is transmitted to visual centers in the
brain.

Regeneration of 11-cis-retinal: After dissociation, opsin


remains in retina; but transretjnal enters the blood
circulation (Fig. 32.4). Later cis-retinal is generated ,
reaches retina. The all-trans-retinal is isomerized to 11-cis-
Torsten Wiesel David Hubel

---
retinal in the retina itself iny.aar~ by the enzyme, retinal NP 11981 NP 1981
isomer.a.~. The 11-cis-retinal can recombine withopsin b. 1924 1926-2013
456 Section D: Nutrition

Jy: Rods are for Vision in Dim Light


In the retina, there are two types of photosensitive cells,
the rods and the cones. Rods are responsible for per-

.s.
Treatment of
ception in dim light. Rhodopsin present in rods is made i:1romy_elQc~
up of 11-cis-retinal + opsin. • • • • • • • • • will 1'1!!'ukem1a

lead to

Cones are for Color Vision


Cones are responsible for vision in brjaht light as wfl!!
q,a_G9lor v.i§..i.a.n. They contain the photosensitive protein,
There are 3 types of cones, sensitive to bJt1e, ~muentiation at
fTJ.aiC.l.lW._aac~
gr~n o lors. Maintenance Promotion epil!)effal.tiss~e, Maintenance of
of vision of !Jrowth gene ress1on reproduction
. Reduction in number of cones or
Fig. 32.6: Summary of actions of vitamin A
the cone proteins, will lead to color blindness.

F Colors have Profound Influence in Life Xerophthalm.ia


About 70% of information inputs to the brain are visual. The conjunctiva becomes dry, thick and wrinkled. The
The optimists view the world through "rose-colored" eyes. conjunctiva gets keratjniz.ed and loses its normal trans-
When sad, a person is in a "blue" mood. SrJf{rqn color ha.,5 parency. Dryness spreads to cornea. It becomes glazy
trangujlizing effect.esw cjaUy in acwategPi)[§QQS. Even a and lusterless due to keratinization of corneal epithe-

-
brief display of saffron color pepduceS1Deasytabl~ elaxa-
lium . Infections niaµ.up.er:5.f:de.
tiw] jn mus,c~. ~ s:wa _on.e,.pe~ ul
-----

a~ ont~ s th~ uscle tone.


, while in(9Iffltci1igf)t -~"'~,__,,,..,e
Bitot's Spots
. The effects of colors on human moods have These are seen as grayish-white triangular 12!£_qu~s
applications in psychiatry, interior decoration and Yoga. fi~berent to the.,,onjunctiva. This is due to incre-

l Other Biochemical Functions of Vitamin A ased thickness of conjunctiva in certain areas.


~ r chan_ru)s mentioned so far are completely rever-
i. Retinoic acid has a role in the r,eg,u!atjon of ge(le sible when vitamin is supplemented.
express.ion and d.!!f.erentiatig{I of tissues. Retinoic
acid acts like sterojd hormones.
Keratomalacia
ii. Vitamin A is required for normal reproduction. In
vitamin A deficiency, '!]iscarrjaaes are noticed in When the xerophthalmia persists for a long time, it
female rats. progresses to keratomalacia (softening of the cornea).
iii. The ~ticanceucti¥,j!y has been attributed to the There is degeneration of corneal epithelium w hich may
naturaia'ntioxidant property of carotenoids. Fresh get vascularized. Later, corneal opacities develop (Fig.
vegetables containing c,arotenoids were shown to 32.7). Bacterial infection leads to corneal ulceration,
r~ ~ _ f r . a o c er. -
perforation of cornea and total blindness.
iv. Vitamin A is necessary for the maintenance of nor-
mal epithelium and skin (see Fig. 32.6).
Preventable Blindness
Ci/ Deficiency Manifestations of Vitamin A
The deficiency of vitamin A is the most common cause
i- Night Blindness or Nyctalopia of blindness in Indian children below the age of 5. -
Visual acuity is diminished in dim light. The patient can-
not read or drive a car in poor light. The dark adaptation About 40% of blindness is preventable. Vitamin A defi-
time is increased. cie~¥,.is a majru:-f)l,!bljs-Realta ~eJll. A§i.Q_gle ~e of
Chapter 32: Fat Soluble Vitamins (A, D, E and K) 457

Fig. 32.8: A parody of the old proverb is "One carrot a day will
keep the Ophthalmologist away". Papaya, carrot and mango are
good sources of Vitamin A precursors
Fig. 32.7: Keratomalacia
Da•~Y Requirement of Vitamin A
vitamin A is given, as a prophylactic measwe, to children
The recommended daily allowance (RDA) for
below 1 year age.
i.Children =400-600mcg/day (C' , L\-.,., · 6~/c\.01j)
Skin and Mucous Membrane Lesions ii.Men = 750-1000 mcg/day
iii.Women =750mcg/day c_n.tt'"~/cl~)
t.=
Pregnancy= 1000 mcg/day ( 1m.it i:la.i.l)
i. - _, - 1 ; ~. • • • ffiSU~
iv.
from hyperkeratinization of the epithelium lining One international unit = 0.3 mcg of ret fnol. One
the follicles. The skin becomes rough. Keratinizing retinal equivalent = 1 microgram of retinal or 6 microgram
metaplasia of the epithelium of the respiratory, of beta carotene.
gastrointestinal and genitourinary tracts have been
observed. Dietary Sources of Vitamin A
ii. The alterations in skin may cause increased
Animal sources include roitt<, ~er, cre~m. cheese, egg
occurrence of generalized Therefore
in old literature, vitamin A is referred to as anti- y;,1fkand li\le'r.
. Vegetable sources
@fl9JII01atorµtam[!y
iii. ~ e,,. a synthetic variant of vitamin A is known contain the yellow pigment beta carotene. Carrot
to reduce the sebaceous secretions, hence it is used contains significant quantity of beta carotene (Fig. 32.8).
to preven during adolescence. Papaya, mango, pumpkins, green leafy vegetables
(spinach, amaranth
A activity
Causes for Vitamin A Deficiency
i. Decreased intake.
...
Therapeutic Use of Vitamin A
ii. Obstructive jaundice causing defective absorption.
Therapeutic dose is generally 20~r:mLs...higber..t.baQ_
-..;;. --=---;;
iii. Cirrhosis of liver leading to reduced synthesis of RBP
iv. Severe malnutrition, where amino acids are not
available for RBP synthesis. Hypervitaminosis A or Toxicity
v. Chronic nephrosis, where RBP is excreted through
urine. Excessive intake can lead to(foxicify since the vitamin is
stored . It has been reported in children where parents
have been overzealous in supplementing the vitamins.
Assessment of Deficiency
Eskimos refrain from eating the liver of polar bear due
a. Dark adaptation test- It is the time required to to its high vitamin A content. Symptoms of toxicity
adapt the eye to see objects in dim light. It is inc- include anorexia, irritability, headache, peeling of skin,
reased in vitamin A deficiency. drowsiness and vomiting.
b. RBP (retinal binding protein) level in serum is Hypercarotenemia can result from persistent exces-
decreased. (see Table 26.1 ). sive consumption of foods rich in carotenoEs. The
c. Vitamin A in serum is decreased. ~ . but n...9 staining of s ~ as in ja!J.Ddice
d. Normal blood level of vitamin A is 25 to 50 mg/dl. is observed.
458 Section D: Nutrition

22
26

27 27 - - +
UV rays in skin
Photolysis

HO
Cholecalciferol (D3}(trans)

Kidney

25-hydroxylase 1-alpha-hydroxylase

&pl:Fig. 32.10: eneration of calcitriol [Su.N- SHINf -


@ I VITAMIN D (CHOLa:;ALCIFEROL) BOX 32.1: Calc1tnol and ca lc1tornn are different
Calcitriol is the physiological active form of vitamin D. It increases
Experimental rickets induced by dietary deficiency was the blood calcium level.
produced in rats by McCollum in 1919. Angus and cowor- Calcitonin is the peptide hormone released from thyroid gland. It
kers isolated vitamin O in 1931 and named it as calci- decre ses the blood calcium. •

ferol, which was later identified as Vitamin 0 3. The struc-


tural elucidation was done independently by Otto Commercially the vitamin is derived from the fungus,

---
and Kurt Alder. Both were awarded Nobel Prize in 1950. l:i-'g3l. The ergosterol when treated with ultraviolet light,
ergocalciferol or vitamin 0 2 is produced. Ergocalciferol
@ Formation of Vitamin D differs in having an unsaturation in the side chain and an
extra methyl group (C28).
Vitamin Dis derived either from 7-dehydrocholesterol or
ergosterol by the action of ultraviolet radiations. 7-dehy Q ctivation of Vitamin D
drocholesterol, a derivative of cholesterol, is present in Vitamin o is a prohormone. The cholecalciferol is first
epidermis. In the skin, ultraviolet light breaks the bond transported to liver, where hydroxylation at 25th position
between position a.and 10 .of tbe steroid ring. So, the occurs, to form 25-hydroxy cholecalciferol (25-HCC)
ring B is opened, to form the provitamm, secosterol (Fig. 32.10). 25-HCC is the major storage form. In
(Fig. 32.9). The cis double bond between 5th and 6th plasma, 25-HCC is bound to "vitamin O binding protein"
carbon atoms, is then isomerized to a trans double bond (VOBP). In the kidney, it is further hydroxylated at th_e 1st
(rotation on the 6th carbon atom) to give rise to vitamin position. Thus 1, 25-dihydroxy cholecalciferol (DHCC) is
' (Fig . 32.9). So, vitamin O is called generated. Since it contains three hydroxyl groups at 1, 3
0 3 or cholecalciferol
the "sun-shine vitamin". and 25 positions, it is also called Calcitriol (Fig. 32.10).
The production of vitamin O in the skin is directly The calcitriol thus formed is the active form of vitamin; it
proportional to the e~ osyr.e to ~ 1!9,bt and inversejy is a hormone (Box 32.1 ).
proportional to the _Qigmenta..!!9n of skin. Vitamin PTH is released in response to low serum calcium
deficiency is seen in winter. and induces the production of calcitriol. The ..,alcitriol
Chapter 32: Fat Soluble Vitamins (A, D, E and K) 459

Intestinal cell Lumen ffect of Vitamin D on Renal Tubules


CR(VW.+~) Calcitriol increases the reabsorption of calcium and
phosphorus by renal tubules, therefore both minerals
are conserved. (PTH conserves only calcium) (see
Chapter 34 ).

~ egulation of Calcitriol
1\1\I\I\I\I\I\
(C~ Formation by Feedback Control
The hormonal level of calcitriol is maintained by the
c a++ feedback control. The rate of production is modulated
by serum levels of calcium, phosphorus, PTH and cal-
Fig. 32.11: Calcitriol increases calcium absorption
= =
C Calcitriol; R R~¼i~ ~if S,alc{~ receptor mplex; CB citriol itself.
UL 'reb"\
"' Calbindin. (.
m,nv-- /
\._.V ' •
~ eficiency of Vitamin D
binds to its highly specific nuclear rece tor V The deficiency diseases are rickets in children a.nd
binds to VORE (Vitamin D response elements osteomalacia in adult§; Hence vitamin D is known as
and modulates the expression of more than 50 antirachitic vitamin.
T~ ctivation takes place within minutes. The optimal concentration of 25-hydroxy 0 3 is > 30
ng/ml, where as 20:29 ng/ml is considered insufficient ~h
Biochemical Effects of Vitamin D and 10-19 ng/ml is deficient. A level below 10 ng/ml '-''~

May be divided into calcemic and extracalce c ects.


indicates severe deficiency. (Concentrations more than rt !
150 ng/ml is toxic).
Calcemic effects are exerted at-
Various studies have shown that 50-80% of elderly

~
a. Intestinal villous cells
b. Bone osteoblasts
idney distal tubular cells.
J and 20-50% of children have hypovitaminosis D. Even in
affluent countries, vitamin D deficiency is very common.
Vitamin D deficiency was associated with poor bone
health, low calcium concentration, higher systolic blood
~ min D and Absorption of Calcium pressure and lower HDL cholesterol values; all of which
Calcitriol promotes the absorption of calcium and phos- are risk factors for heart disease.
phorus from the intestine. On the brush-border surface~
calcium is absorbed passively. From the intestinal ce~ auses for Vitamin D Deficiency
to blood, absorption of calcium needs energy. Catcitriot i. Deficiency of vitamin D can occur in people who are
acts like a steroid hormone. Transcriptional activation not exposed to sunlight properly, e.g . inhabitants of
of specific genes that code for calbindin occurs (Fig. northern latitudes, in winter months, in people who
32.11 ). Due to the increased availability of calcium bind- are bedridden for long periods, or those who cover
in~ tein, the absorption of calcium is increased. the whole body (purdah).
ii. Nutritional deficiency of calcium or phosphate may
E~ t of Vitamin D on Bone also produce similar clinical picture.
iii. Malabsorption of vitamin (obstructive jaundice and
Mineralization of the bone is increased by increasing steatorrhea). High phytate content in diet may also
'the activity of osteoblasts (see Chapter 34). Calcitriol reduce the absorption of vitamin.
coordinates the remodeling of bone and increases bone iv. Abnormality of vitamin o activation. Liver and renal
mineral density. diseases may retard the hydroxylation reactions.
Intracellular production of calcitriol by osteoblasts v. Deficient renal absorption of phosphates.
promotes their differentiation and mineralizati~
Osteoclastic bone re~rption and alkaline phosphatasWClinical Features of Rickets
activity of osteoblasts provide adequate calcium and Rickets is seen in children. There is insufficient minera-
phosphorus to promote mineralization . lization of bone. Bones become soft and pliable. The
460 Section D: Nutrition
bone growth is markedly affected. The classical features
of rickets are bone deformities. Weight bearing bones H3C
are bent. Continued action of muscles also cause bone
malformations. The clinical manifestations include bow
legs, knock-knee, rickety rosary, bossing offrontal bones,
and pigeon chest. An enlargement of the epiphysis
Fig . 32.1 2: Alpha tocopherol
at the lower end of ribs and costochondral junction leads
to beading of ribs or rickety rosary. Harrison's sulcus iii. Pregnancy, lactation = 1o mcg/day
is a transverse depression passing outwards from the iv. Above the age of 60 = 600 IU per day.
costal cartilage to axilla. This is due to the indentatio
of lo~ r ribs at the sj - c ; 1i aphragcn. <) ources of Vitamin D
ft:) . . . Exposure to sunligbLproduces cl"lo~ cal~ I. More-
\.Y Ch mca I Features of Osteomalac1a over, fi~ ver olffisn· and egg· yolk are good sources
The term is derived from Greek "9steon" = ~ ; and of the vitamin. Milk contains moderate quantity of the
"malakia" = softness. The bones are softened due to vitamin. The current recommendation is to fortify dairy
insufficient mineralization and increased osteoporosis. products with vitamin D and adequate exposure to~ -
Patients are more prone to ractures. It may be noted light without sunscreen before 10 am aod after 3 pm at
that vita · nc eve roduces se,yere.,byp_g- le ast 15..minutes a day (safe sun).
c;alc_e.mia. Tetany will not be anifested. Serum alkaline
phosphatase, bone isoenzyme, is markedly increased. ypervitaminosis D

(9 Different Types of Rickets Doses above 10,000 units per day for long periods
may cause toxicity. Symptoms include weakness,
1. The classical vitamin D deficiency rickets can be polyuria, intense thirst, hypertension and weight loss.
cured by giving vitamin D in the diet. Hypercalcemia leads to calcification of soft tissues,
2. Vitamin D resistant rickets is found to be associ- (metastatic calcification , otherwise called calcino-
ated withl Fanconi s yndrom;J where the renal sis ), especially in vascular and renal tissues. Although
tubular reabsorption of bicarbonate, phosphate,
vitamin D is toxic in higher doses, excessive expo-
glucose: and amino_acids ~re also deficie~t. . . sure to sunlight does not r esult in vitamin D toxicity,
3. Renal rickets: In kidney diseases, ~ en1f wtarrun because excess O is destroyed by sunlight itself.
3
q_ js ayaila~le, ~iPIJ.Q § ~- These
cases wi'.I respond to.admi~istration of calcitriol. i)._y.lTAMIN E /~
@ Other Actions of V1tamm D The active vitamin was isolated from wheat germ,..Q,!_I
1,25-DHCC has been found to have a modulatory effect and namec{fgcopherol \ta{Ws =,_c!Jild,JJidh: Qlleros.,= to
on immune-hematopoietic system. Therapeutic doses ~ : $J}. = alcohol). Initial studies of induced vitamin E
given to children with rickets have been found to correct deficiency in laboratory animals resulted in infertility and
the anemia. It has also been found to reduce the risk therefore the vitamin came to be known as a..Q.t i-infert[fily
of cancer and coronary vascular disease. A negative vitamin. Now vitamin E is known as the most potent bi~-
correlation between lgE and vitamin D concentration;
and a positive relation between vitamin D and lung
function were shown in children with asthma. Vitamin
D insufficiency is associated with metabolic syndrome,
----
loQical antioxid~ t.

Chemical Nature
A chromane ring (tocol) system, with an isoprenoid
,'":'\ insuljn resistwce and glucose iotGleraoce. side chain is present in all the eight naturally occurring
tocopherols. Of these, alpha tocopherol (5, 7,
\t:j/ Requirement of Vitamin D 8-trimethyl tocol) has greatest biological activity (Fig.
i. Children= 10 mcg (400 IU)/day 32.12). The structure of vitamin E was elucidated by Paul
ii. Adults= 5 to 10 mcg (200 IU)/day Karrer, who was awarded Nobel Prize in 1937.
Chapter 32: Fat Soluble Vitamins (A, D, E and K) 461

M-CH,
to poor nerve conducf n. These include neurR!fluscular
p~lems such as s ocerebellar ataxia, relrnopathy,
Renpheral neuropa\hy and m pathies. Deficiency can
~R
also cause anemia, due to oxidative damage to red
0
blood cells.

-------.~
R = 20C in (Phylloquinone) in K 1
R = 30C in (Menaquinone) in K2 Recommended Daily Allowance
R = H in Menadione

Fig. 32.13 : Vitamin K Males: 10 mg per day


Females: 8 mg/day ::.& -~lV
Biochemical Role of Vitamin E Pregnancy: 10 mg/day
Lactation: 12 mg/day.
i. Vitamin E is the most powerful natural antioxi-
15 mg of vitamin E is equivalent to 33 international units.
dant (see Chapter 30). Free radicals are continu-
The requirement increases with higher intake of PUFA.
ously being generated in living systems. Their
Pharmacological dose is 200-400 IU per day.
prompt inactivation is of great importance. Vitamin E
is a known biological antioxidant able to quench the
lipid peroxidation chain.
Sources of Vitamin E
ii. The free radicals would attack bio-membranes. Vegetable oils are rich sources of vitamin E; e.g. wheat
Vitamin E protects RBC from hemolysis. By pre- germ oil, sunflower oil, safflower oil, cotton seed oil, etc.
venting the peroxidation, it keeps the structural and Fish liver oils are devoid of vitamin E.
functional integrity of all cells.
iii. Gradual deterioration of aging process is due to Hypervitaminosis E
the cumulative effects of free radicals. Vitamin E
At doses above 1000 IU per day, it may cause tendency
also boosts immune response.
to hemorrhage, as it is a mild anticoagulant.
iv. It reduces the risk of atherosclerosis by reducing

oxidation of LDL. @ l_,_


V_IT_A_M
_ IN_ K_ _ _ _ _ _ __ _ _ __
Inter-Relationship with Selenium Chemistry of Vitamin K
Selenium is present in glutathione peroxidase; an
The letter "K" is the abbreviation of the
important enzyme that oxidizes and destroys the free
German word "koagulation vitamin". They
radicals (see Chapter 30). Selenium has been found to
are naphthoquinone derivatives, with a
decrease the requirement of vitamin E and vice versa.
long isoprenoid side chain. The length of
They act synergistically to minimize lipid peroxidation.
Selenium is described in Chapter 34. side chain will differ. Vitamin K1 has 20C Henrik Dam
side chain (phylloquino.n.e) (Fig. 32.13). NP 1943
Deficiency Manifestations of Vitamin E Vitamin ~ has a 30C side chain. Yet another ,...1_8_9_
5 -_1_9_
76~

In rats, inability to produce healthy ovul.Jl and loss of structurally similar synthetic compound
motility of spermatozoa, hemolysis of red ceUs, acute having vitamin K activity is Menadione. It
hepatic necrosis and muscular dystrophy are observed. is water soluble synthetic vitamin, widely
In a normal adult, the body vitamin E stores can meet used in clinical practice. Henrik Dam
the requirement for several months. isolated vitamin K1 in 1929, while Edward Ed rd .
wa O01sy
No major disease states have been found to be Doisy isolated vitamin K2 in 1939. Both of NP 1943
associated with vitamin E deficiency due to adequate them were awarded Nobel Prize in 1943. 1893-1986
levels in the average diet. Vitamin E deficiency is seen
Biochemical Role of Vitamin K
in persons (a) who cannot absorb dietary fat, and (b) in
premature infants (birthweight less than 1500 grams). Vitamin K is necessary for coagulation. Factors
Vitamin E deficiency causes neurological problems due dependent on vitamin K are Factor II (prothrombin);
462 Section D: Nutrition

Clinical Manifestations of Deficiency


Glutamic acid Gamma carboxy
glutamate (9 G(i) Hemorrhagic disease of the newborn is attributed to vita-
min K deficiency. The newborns, especially the premature
infants have relative vitamin K deficiency. This is due
to lack of hepatic stores, limited oral intake (breast milk
has very low levels) and absence of intestinal bacterial
flora. It is often advised that ~term infants be given.
e_rgphylactic doses of vitamin K (1 mg Menadio_ne).
In children a1nd adults, vitarrii~ K deficiency may be
manifested as bruising tendency, echymotic patches,
mucous membrane hemorrhage, post-traumatic bleed-
ing and internal bleeding. Prolongation of prothrombin
NADPH
time and delayed clotting time are characteristic of
vitamin K deficiency.
Measurement of prothrombin time (PT) is taken
as an index of liver function. W hen liver function is con-
siderably lowened, prolongation of PT occurs due to
deficient synthesis of the coagulation factors. In such
cases, administration of vitamin fails to restore PT to
normal levels. Hence before undertaking any surgery
on jaundiced patients, PT before and after administra-
Fig. 32.14: '41amin K as co-factor in GCG synthesis
-= tion of vitamin K shou ld be dcne .
Factor VII (SPCA); Factor IX (Christmas facto r); Factor X Warfarin and dicoumarol will competitively inhi-
(Stuart Prower factor). All these factors are synthesized bit the gamma icarboxylation system due to structural
similarity with vitamin K. Hence they are widely used as
by the liver as inactive zymogens. They undergo post-
-. anticoagulants for therapeutic purposes.
translational modification; gamma carboxylat~ of
glutamic acid residues. These are the binding sites for Daily Requi1rement of Vitamin K
--- -~
calcium ions. The gamma carboxyglutamic acid (GCG)
Recommended daily allowance is 50-100 micro gram/
synthesis requires vitamin Kasa cofactor (Fig. 32.14).
day. This is usually available in a normal diet.

Vitamin K Dependent Carboxylase Sources of Vitamin K


It require s ~. @ ) ~and ~ Green leafy VE~getables are good dietary sources.
This enzyme is competitively ill!libite..2....!2>'. wa~ rin Even if the diet does not contain the vitamin, intestinal
and dicoumarol. bacterial synthesis will meet the daily requirements,
as long as absorption is normal.
Causes for Deficiency of Vitamin K
Hypervitamiinosis K
In normal adults dietary deficiency will not occur since
Hemolysis, hyp1erbilirubinemia, kernicterus and brain
the intestinal bacterial synthesis is sufficient to meet the
damage are the manifestations of toxicity.
needs of the body. However, deficiency can occur in
conditions of malabsorption of lipids. This can result 0
from obstructive jaundice, chronic pancreatitis, sprue, ••· Clinical Case Study 32.1
etc. Prolonged antibiotic therapy and gastrointestinai A 6-year-old child was brought to the hospital with com-
infections with diarrhea will destroy the bacterial flora plaints of slow !~rowth and pain in bones. On exami-
and can also lead to vitamin K deficiency. nation, he was anemic, had frontal bossing, bowing of
Chapter 32: Fat Soluble Vitamins (A, D, E and K) 463

legs and swelling of costochondral junc~Laboratory 2. Retinals are polyisoprenoid compounds with vitamin
results were: ~ m calcium-8~2
g/d~rum phos- A activity, having the b ionone ring system.
phorus-2.8 ~ L and serum 720 U/L. What is 3. Active forms of the vitamin A include; Retinal,
the likely diagnosis? Retinal, Retinoic acid. The two important isomers
are all trans-retinal and 11-cis-retinal.
4. Vitamin A is transported with the help of Retinal
• • Clinical Case Study 32.1 Answer Binding Protein and this retinal-RBP complex has
specific receptors in various tissues.
Interpretation: Rickets. 5. Rhodopsin is a membrane protein made up of
Tests: Vitamin D, Calcium. opsin plus 11-cis-retinal and it is important in the
Vitamin D functions as both a vitamin and a prohormone. visual cycle.
Low levels of vitamin D are associated with increased 6. Rods are for dim light vision and cones for color vision.
mortality; excess as well as deficiency of vitamin D cau- 7. Decrease in number of cones/cone proteins lead
ses premature aging. Low vitamin D levels are associated C
to color blindness - -
8. Vitamin D is derived from 7 dehydrocholesterol by
with osteomalacia, rickets, falls and low bone mineral the action of UV rays.
density. Lower vitamin D levels also seem to be correlated 9. Vitamin D deficiency results in rickets and osteo-
with some cancers, bronchial asthma, heart palpitations, malacia. Different types of rickets are; vitamin D
multiple sclerosis, infections and neurodegenerative dis- resistant, and renal rickets.
eases. Serum ~hydroxy vitamin D l§V~s also have to 1a.Vitamin E is tocopheror. ,::.t:: ,s=a::;b::s==o==.rb:==e=-d-:---
a:= wm;-
lo-=-
n-=-
g
be maintained for bone and overall health. fats with the help of bile salts. It is transported as
chylomicrons and stored in adipose tissue.

I LEARNING POINTS, CHAPTER 32


11 . Vitamin E is the most important antioxidant in tissues.
'u.Vitamin K is absorbed in intestine along with chylo-
micron~ They are also synthesized by intestinal flora.
1. Vitamin A is a fat~sbluble vitamin whose active form 13. Vitamin K is involved in blood coagulation. Vitamin
is present only in animal tissues, but provitamin A K is required for post-translational modification of
(beta carotene) is present in plant tissues. coagulation factors.

PART-1: ESSAY AND SHORT NOTE QUESTIONS

32-1 . Describe sources, biochemical functions, requirement and deficiency manifestations of Vitamin A.
32-2. Describe the sources, biochemical functions, normal requirement and deficiency manifestations of Vitamin D.

SHORT NOTE QUESTIONS

32-3. Sources and daily requirement of 32-9. Functions of Vitam in D.


Vitamin A. 32-10. Activation of vitamin D.
32-4. Functions of Vitamin A. 32-11. Vitamin D deficiency.
32-5. Visual cycle. 32-12. Tocopherol.
32-6. Hypervitaminosis A. 32-13. Biological role of Vitamin K.
32-7. Provitamins. 32-14. Anticoagulants.
32-8. Anti-vitamins. 32-15. Gamma carboxylation reaction.

PART-2: MULTIPLE CHOICE QUESTIONS

32-1 . Daily requirement of vitamin A for a normal adult is: 32-3. Nyctalopia is due to the deficiency of:
A. 1 microgram B. 5 microgram A. Vitamin K B. Vitamin E
C. 100 microgram D. 750 microgram C . Vitamin 812 D. VitaminA
32-2. Deficiency of vitamin A leads to: 32-4. When kidney diseases are present, oral doses of
A. Night blindness B. Rickets vitamin D may not be effective in curing rickets,
C. Macrocytic anemia D. Microcytic anemia because:
464 Section D: Nutrition
A. Hydroxylation reaction is taking place in kidney 32-15. The structure of vitamin A contains:
which activates vitamin A. Chromane ring B. Beta ionone ring
B. Dehydrogenation of vitamin D is taking place in C. Thiazole ring D. Naphthoquinone ring
kidney 32-16. The structure of vitamin E contains
C. Hydroxylation of vitamin D is taking place in kidney A. Chromane ring B. Beta ionone ring
which destroys vitamin C. Thiazole ring D. Naphthoquinone ri ng
D. Vitamin D is stored in liver 32-17. All are good sources of vitamin A, except:
32-5.· Daily requirement of vitamin D is: A. Pumpkin B. Carrot
A. 1 microgram B. 10 microgram C. Mangoes D. Oranges
C . 100 microgram D. 750 microgram 32-18. Rickets may occur in all conditions, except:
32-6. Bleeding tendency is common in deficiency of all A. Chronic renal failure
the following, except: B. Liver diseases
A. Vitamin K B. Vitamin B 12 C. Under exposure to sun light
C. Vitamin C D. Platelets D. Prolonged antibiotic therapy
32-7. Cholecalciferol is synthesized in: 32-19. Richest source of vitamin Dis:
A. Liver B. Skin A. Fresh leafy vegetables
C. Kidney D. Intestinal mucosa B. Fish liver 011
32-8. The most important biological role for vitamin Eis: C. Egg yolk
A. To produce clotting factors D. Vegetable oils
8 . Antidote of selenium poisoning 32-20. Which is the enzyme specially involved in ossifica-
C. Anticoagulant tion of bone?
D. Antioxidant A. Alkaline phosphatase
32-9. Large doses of vitamin K in neonates may cause: B. Acid phosphatase
A. Rebound bleeding C. Hexokinase
B. Porphyria D. ATPase
C. Jaundice 32-21 . When kidney diseases are present, oral doses of
D. Cyanosis vitamin D may not be effective in curing rickets,
32-10. Deficiency of vitamin K can occur in the following because vitamin D is:
conditions, except: A. Stored in kidneys
A. Following gastrectomy B. Dehydrogenated in kidneys
8. Prolonged antibiotic therapy C. Destroyed in kidneys
C. Obstructive jaundice D. Activated in kidney
D. Admin istration of dicoumarol 32-22. If vitamin K is given to a patient with hemophilia:
32-11 . Biochemical function of vitamin K is for: A. Bleeding time prolonged, clotting time remains
A. Converting proline to hydroxyproline normal
8 . Conversion of prothrombin to thrombin B. Blotting time prolonged, bleeding time remains normal
C. Gamma carboxylation of clotting factors C. Both bleeding and clotting times prolonged
D. Inhibition of lipid peroxidation in biomembranes D. Both bleeding and clotting times decreased
32-12. Vitamin K is inhibited by: 32-23. All water soluble vitamins are excreted through
A. lsoniazid (INH) B. Methotrexate urine and are not stored in the body, except:
C. Dicoumarol D. Avidin A. Vitamin 81 2
32-13. Skin and mucous membrane are affected in defi- B. Vitamin 8 6 (pyridoxal)
ciency of all the vitamins listed below, except: C. Vitamin C
A. Retinol 8 . Niacin D. Niacin
C. Riboflavin D. Calcitriol 32-24. All these vitamins are required for normal bone
32-14. The active form of vitamin D is known as: formation, except:
A. Cholecalciferol A. Vitamin C
B. Cyclopentano phenanthrene 8 . Vitamin D
C. Calcitriol C. VitaminA
D. Lanosterol D. Vitamin E
Chapter 32: Fat Soluble Vitamins (A, D, E and K) 465

ANSWERS OF MULTIPLE CHOICE QUESTIONS

32-1. D 32-2. A 32-3. D 32-4. A 32-5. B 32-6. B 32-7. B


32-8. D 32-9 . C 32-10. A 32-11 . C 32-12. C 32-13. D 32-14. C
32-15. B 32-16. A 32-17. D 32-18. D 32-19. B 32-20. A 32-21 . D
32-22. D 32-23. A 32-24. D

PART-3: VIVA VOCE QUESTIONS AND ANSWERS

32-1. What is the major function of vitamin A 7 32-12. What is the function of vitamin 07
Important in vision, used in the Wald's visual cycle. It increases absorption of calcium from intestine; it also
32-2. What causes the nerve impulse in retina? increases mineralization of bone.
Photoisomerization of 11-cis-retinal to all trans retinal. 32-13. What are deficiency manifestations of vitamin D?
32-3. How is all-trans retinal regenerated? Rickets in children and osteomalacia in adults.
Trans retina l is taken to liver, where it Is made to trans- 32-14. In renal disease, oral doses of vitamin D may not
retinol. then isomerised to cis retinol and then to cis be effective, why?
retinal. Hydroxylation and activation of vitamin is taking place
32-4. What enzymes are required for regeneration? in kidney.
Alcohol dehydrogenase and retinol isomerase 32-15. What is the daily requirement of vitamin 07
(Fig. 32.4 ). 10-15 microgram.
32-5. What are the deficiency manifestations of vitamin 32-16. What is the function of vitamin E?
It is the most powerful antioxidant.
A?
32-17. How selenium and v itamin E are related?
Night blindness, xerophthalmia, keratomalacia, kerati-
They act synergistically as antioxidants.
nisation of epithelium.
32-18. What is the source of vitamin E?
32-6. What are the sources of vitamin A?
Vegetable oils e.g. wheat germ oil, sunflower oil, saf-
Carrot, mangoes, papaya, green leafy vegetables, fish
flower oil, cotton seed oil, palm oil.
oil.
32-19. What is the daily requirement of vitamin E?
32-7. What is the dally requirement of vitamin A?
15 mg or 33 international units.
750 to 1000 microgram.
32-20. What is the function of vitamin K?
32--8. How is cholecalciferolsynthesized?
Gamma carboxylation of clotting factors such as pro-
From 7-dehydro cholesterol in the Malpighian layer of
thrombin
epidermis, by the action of ultra violet rays. 32-21 . Deficiency of vitamin K can occur in which condi-
32-9. How is vitamin D activated?
tions?
Cholecalciferol from skin reaches liver. There it is Obstructive jaundice; antibiotic therapy; administration
hydroxylated to form 25-hydroxy cholecalciferol (25- of dicoumarol.
HCC ). It then reaches kidney, where further hydroxyla- 32-22. What is the mechanism of action of dicoumarol?
tion takes place to form 1,25-dihydroxy cholecalciferol It competitively inhibits vitamin K epoxide reductase.
(DHCC). 32-23. Dicoumarol is used for what purpose?
32-10. What is calcitriol? To prevent intravascular thrombosis.
1,25-dihydroxy cholecalciferol, or active vitamin D, 32-24. Excess dose of vitamin K in neonates may lead to
contains three hydroxyl groups at 1, 3 and 25 positions which condition?
(Fig. 32.9). Hemolysis and jaundice.
32-11 . Which vitamin acts as a pro-hormone? 32-25. Bleeding tendency is common in deficiency of?
Vitamin D is converted to calcitriol. Vitamin K; Vitamin C; Platelets; prothrombln.
_ _ _ _ _Chapter 33
Water
9
Soluble Vitan)!ns
(Thia 11line, Ribgflavin, Ni:t:in, Pyridoxine,
Pantot hen1c . ac1.d, e·1ot1n,
s~ . Fo 1·1cB.9ac1.d,
. 812
V 1tam1n• B an d Ascor C b.1c ac1.d)
12

Chapter at a Glance
The learner will be able to answer questions on the following to pics:
1D Thiam ine (Vitamin 81 ) GD Biotin
2D Riboflavin (Vitamin B2) an~ D TD Folic acid
3 D Niacin.@AD· anc(t}Ao p• ¥D Vitamin B12
D Pyridoxine (Vitamin B6) {.9 D Ascorbic acid (Vitamin C)
D Pant othenic acid and coenzyme A

ATP AMP

These vitamins are chemically not related to one..aoQthfilr


Thiamine '::::::--,,, d
TPP - transferase
Thiamine pyrophosphate (TPP)

They are grouped together because all of them function


in the cells as~ ••H. -~ ~ ~ o -· ..
(}) l!HIAMINE (VITAMIN B )
Thiamine is also called as vitamin 8 1 (Box 33.1). In old I~ I ii I o- er
literature, it is designated as Aneurine (it can relieve
Substitute! pyrimidine Substituted thiazole~


neuritis) or antiberberi factor. In 1900, Christian Eijkman I I
produced beriberi in chicken by feeding polished rice Thiamine Pyrophosphate
(Nobel Prize, 1929). Adolf Windaus (Nobel Prize, 1928)
elucidated the structure of the vitamin. Thiamine pyrophosphate (TPP)

Fig. 33.1 : Structure of thiamine pyrophosphate


ources
Aleurone layer of cereals (food grains) is a rich source polished, aleurone layer is usually removed. Yeast is
of thiamine. Therefore, \"!hole wheat flour and LU:lPOlished also a very good source. Thiamine is partially destroyed
hand-pounded rice have better nutritive value than by heat.
completely polished refined foods. When the grains ar tructure of Thiamine
Thiamine contains a substituted pyrimidine ring. The
vitamin is then converted to its active coenzyme form
by addition of two phosphate groups, with the help of
THIAMINE is vitamin B,.
ATP (Fig. 33.1 ).
Chapter 33: Water Soluble Vitamins 467

,ological Role of Thiamine Thiamine deficiency in alcoholism may cause imp-


airment of conversion of pyruvate to acetyl-CoA.
i. Pyruvate dehydrogenase: The coenzr me form
The result is increased plasma concentration of
is thiamine pyrophosphate (1PP), ft is used in
pyruvate and lactate, leading to lactic acidosis.
oxidative decarboxylation ef afpba keto acips, e.g.
pyruvate dehydrogenase catalyze~ the breakdo'!Yn
b
ochemical Parameters
oi..Qyruvate... ~o acetyl-Co,t\. and c.arbon dioxide
(see Fig. 10.19). In thiamine deficiency, blood thiamine is reduced, but
ii. Alpha-ketoglutarate dehydrogenase: An analo- pyruvate, alpha ketoglutarate and lactate are increased.
gous biochemical reac~ o th,t requires TPP is the Erythrocyte transketolase activity is reduced; this is
oxidative decarboxylatIon of '!!f_h,!:~~J_uta~e the earliest manifestation seen evep before clinical
t ~ d_C~ (see citric acid ~ . Fig. disturbances.
20.2).
iii. Transketolase : The second woup of enzymes R~ mmended Daily
that use TPP as coenzyrne are the t[ansketolases, Allowance of Thiamine
in the h ose o o.sphate sbunt patbwsy of
It depends on
gll~Se metabOl§Rl see Fig. 10.40).
Requirement is iamine is useful in the
iv. The main role of thiamine (TPP) is in carbohydrate
treatment
- - - _;;;,. _--=_,,
of beriberi, alco olic polyneuritis, neuritis of
metabolism. So, the requirement of thiamirie-is
il).Gf~ sed along with h~ her intake ~ rboh~tes. -
pregnancy and neuritis of old age.

De~ cy Manifestations of Thiamine I RIBOFLAVIN (VITAMIN B )


A. Beriberi: Deficiency of thiamine leads to beriberi. This vitamin is synthesized by green plants and micro-
It is a Singhalese wor~ eanin~ weaknes~ The organisms. Warburg (Nobel Prize 1931 ), isolated the

and weai<ness.
4 7
early sy9Wtoms are anorexia, dy~ psia, heat rness "yellow enzyme'' of cellular respiration. Later Axel
¾°~"~ A\>~ ~ T h e o r e l l (Nobel Prize, 1955) isolated riboflavin. In 1935,
B. Wet beriberi: Here rdiovascula~station;, Paul Karrer (Nobel Prize, 1937) determined the structure.
are prominent. Edema of l~s. fas:e trun~ anp serous
cavit~s_are the main featu res. Palpitation,breathless- Structure of Riboflavin
ness and distended neck veins are observed. Death
._.__,;,;..iiiii.iii:::i=\Fig. 33.2). Riboflavin
occurs due to heart failure.
is converted to its ~'11£.§~ ~~w,g.JQlms (FMN and
C. Dry beriberi : In this condition, CNS manifestations
FA..Ql with the help of ATP (Fig. 33.4). Riboflavin is
are the major features. Walking becomes difficult.
Peripheral neuritis With sensory disturbance leads
to complete paralysis.
D. Infantile beriberi: It occurs in infants born to mothers
Coenzyme Activity of Riboflavin
suffering from thiamine deficiency. &_§_tlessoess Riboflavin exists in tissues tightly bound (but not
and sJeeple.ssness are observed. covalently) with enzymes. Enzymes containing riboflavin
E. koff syndrome: It is also called
as cerebral beribe i. Clinical features are those
of ejcephalopathy (opbthalmoplegia, rJ_Ystagmus,
cerebellar alaxia) plus psychosis. It is seen only
when the nutritional status is severely affected.
F. Polyneuritis: It is common in chronic alcoholics.
Alcohol utilization needs large doses of thiamine.
Christiaan Adolf Otto Axe/Hugo Otto
Alcohol inhibits intestinal absorption of thiamine,
Eijkman Windaus Theore/1 Warburg
leading to thiamine deficiency. Polyneuritis may NP 1929 NP 1928 NP 1955 NP 1931
also be associated with pregnancy and old age. 1858-1930 1876-1959 1903-1982 1883-1970
468 Section D: Nutrition
b

~':n:{~o
Dimethyl lsoalloxazlne

3 I Paul Karrer
CHr{CHOHlJ-CHrOH NP 1937
1889-1971 Ring of FAD
D-Ribitol I
Fig . 33.2: Riboflavin structure Fig . 33.3: Acceptance of hydrogen by FAD

lsoalloxazine - Ribitol - P - P - Ribose-Adenine BOX 33.2: FAD-dependent enzymes


I I l__.......J 1. llffll> !£_,fum~ by s u ~ deeydrn~se (see Fig.
i
Riboflavin
+
Adenosine
20.2, step 6).
t -beta..J:!!!S~~acyl-~ by a<)'I-Co~
deil¥-drogecase (see Fig. 13.8, step 1).
Flavin mono- Adenosine mono- C: )3.'JinlNDf t;a \/fil acid. by xanthine oxidase (see Chapter 38).
nucleotide (FMN) phosphate (AMP)
(. 4. ~c.e.cyj;.CQA by Bi'.fUvate_fle_by_progena~e (see
I I
r
Fig. 10.19).
'j 5. to su ·nyl-C~by aJpha ketqglutarate
Flavin adenine d.ebydrogeQl!wee ig. 20.2, step 4).
dinucleotlde (FAD)

\ . \.
Flavokinase FMN FAD synthase
Riboflavin
( ( FAD
Riboflavin Deficiency
ATP ADP ATP PPi Causes: Natural deficiency of riboflavin in man is
uncommon, because riboflavin is sy_nthesized b.y: the
Fig. 33.4 : Coenzymes FMN and FAD
inies*iRBl::flsra.
Manifestations: Symptoms are confined to skin and
are called flavoproteins. The two coenzymes are FMN
mucous membranes.
(flavin mono nucleotide) and FAD (flavin adenine
dinucleotide) (Fig. 33.4 ).
i. ~!s>tU ' s.,LC?jeek, glossa = tongue)
ii. ' Mal
enta coio'red tongue
FAD Accepts Hydrogen iii. Cheilosis (Greek, cheilos = lip)
iv. Angular stomatitis (inflammation at the corners of
During the oxidation process
mouth)
atoms from substrate. In turn,
v. Circumcorneal vascularization
The
vi. Proliferation of the bulbar conjunctiva! capillaries is
(Fig. 33.3).
the earliest sign of riboflavin deficiency.
FMN-Dependent Enzymes
Dietary Sources of Riboflavin
,,.
i. During the amino acid oxidation, FMN is reduced
(see Fig.17.8).
Good sources are Usb,
ii. In the resp.Jri ory chain, the NADH dehydrogenase
green leajy vegetables.
contains FMN. The electrons are transported in the
following manner (see Chapter 21 ):
NAO• _ _ ..,.. FMN - -- CoQ
Daily Requirement
Riboflavin is concerned maioJ_y with the metabolism of
FAD-Dependent Enzymes
carbohydrates and r;_~ uirerr1:1nt is related to calorie
These are _enumerated in Box 33.2. FADH2 when intake. Adults on sederlt~ • Ork require about 1.5
oxidized in the electron transport chain will generate 1.5 mg per day. During pregnancy, lactation and old aJie,
ATP molecules (see Chapter 21 ). additional 0.2 to 0.4 mg/day are required.

---
Chapter 33: Water Soluble Vitamins 469
N I<.O ' a.m{,l,...

'O'
2
UCOOH UCONH,
4

6
f
I I
&N I! r~-
,,,. NH2

I P-
~.'1 ~.t

o-'... ' I N.~


o -CO-NH 2'
/
N N N , ~ N "" I I I
1
.........,N 0 2-O-P-O-P-0-H
1111 2~
0

-I lcv
Pyridine Niacinamlde
Niacin
r , o o
Fig. 33.5 : Structure of niacin

NAD+ + H2 --- NADH + H•


I@ HO ow HO o~ I
Adenine mononu~ otide Nicotinamide mono~ leotide

H H
- CH (YA )-\

6-CQ.NH, 6-CO-NH
, Alexander
N I N RTodd
NP 1957
I I 1907-1997
Ribose-5-P Ribose-5-P
Fig. 33.7: Acceptance of hydrogen BOX 33.3: NAO· dependent enzymes
1. Lactate dehydrogenase (lactate pyruvate) (see Fig. 10.14)
2. Glyceraldehyde-3-phosphate dehydrogenase (glyceraldehyde-
~ IACIN @ N!COT">NIC. AC.1D_ 3-phosphate 1,3-bisphosphoglycerate) (see Fig. 10.10)
3. Pyruvate dehydrogenase (pyruvate acetyl-CoA) (see Fig.
Niacin and Nicotinic acid are synonyms . It is also called
10.19)
as Pertagra preventing factor of Goldberger. The term 4. Alpha-ketoglutarate dehydrogenase (alpha-ketoglutarate
nicotinic acid should not be confused with nicotine. succinyl-CoA) (see Fig. 20.2)
Nicotinic a~ ~ n icotine is the otent S. Beta hydroxyacyl-CoA dehydrogenase (beta hydroxyacyl-
poison tforn
tobacco~cinamict is the active form of CoA beta ketoacyl-CoA (see Step 3, Fig.13.8)
6. Glutamate dehydrogenase (Glutamate alpha-ketogluta-
the vitamin, present in tissu s. Warburg (Nobel Prize,
rate (see Fig.17.7)
1931) elucidated the structure of NAO• and Alexander
Todd (Nobel Prize 1957) demonstrated its function . C
ne Hydrogen Atom and One Electron
1stry of Niacin
i. In the ~ nitmge,ri of the nicotinamide
Niacin is pyridine-3-carboxylic..-acid. Niacinamide is the residue--ha~ ~ ~ 4 \ Hence the oxidized
acid amide (Fig. 33.5). In NA[Yo,NAOP+, the reactive form of coenzyme is usually wifflen as ~ .
site is the carbon alo.!Jl and the nitrogen atom of the ii. In the process of reduction, NAO• accepts one
nicotinamide ring (see numbering in Fig . 33.5). The
hydrogen atom fully. The other hydrogen is ionized.
CQenz@ is bound to the appepzyme. Only the electron is accepted (Fig. 33. 7). See the
positive sign in the molecule is removed.
Coenzyme Forms of Niacin
2H - - - • + H• + -
Niacin is converted to its coenz me forms, viz, Nicoti-
Thus NAO• accepts one H atom and one e· (elec-
namide adenine dinucleotide Ao• and Nicotinamide
tron), to form NAOH. The hydrogen ion (H•) is released
adenine dinucleotide phosphate AD ). The niacin is
into the surrounding medium. During the oxidation of
attached to a ribose phosphate to form a mononucleotide.
NAOH, the reaction is reversed .
It is then attached to AMP, to form the dinucleotide
. (Fig . 33.6). ®
NAO• Dependent Enzymes@) N PtD~
In the case of NAOP·, one more phosphoric acid is
attached to the ribose of the AMP (see the asterisk in They are so many, that an exhaustive ·1isting is no~
Fig. 33.6). attempted. A few examples are given in Box 33.3.
BOX 33.5: NADPH utilizing reactions
1. ~ a t e dehydrogenase in the hexose mono- 1. {Beta-ketoacyl-ACP beta
phosphate shunt pathway (Glucose-6-phosphate hydroxyacyl-ACP) {see Step 4, Fig. 13. 13)
6-phospho-glucono lactone) (see Fig. 10.38)
2. Alpha, b e 1 a ~-ACP >acyl-ACP (see Step 6, Fig.
2. ~ t e dehydrogenase in the shunt pathway 13.13) ( ~ ~ \ ~ \ CMc..)
(6-phosphogluconate 3-keto-6-phosphogluconate) (see
3. AMG-CoA reductase {HMG-CoA mevalonate (see Fig. 14.2)
Fig. 10.38).
hemoglobin
3. Cytoplasmic lsocftnttedeh)1frogenase
Folate reductase (Folate dihydrofolat e tetrahydrofolate)
4. t,iatrc (malate to pyruvate).
(see Fig. 33.15)
6. Phenylalanine ~ytme (Phenylalanine tyrosine) {see
Fig. 19.1 ).

iacin is Synthesized from Tryptophan


For details see under tryptophan metabolism in Chapter
19. About 60 mg of tryptophan is equivalent to 1 mg of
Figs. 33.BA and B: Niacin deficiency; (A) shows early signs; (B) niacin. ··. 1l.OO m.9 fu b~ c.c,,- ·~t, -J ,j~
depicts advan~ ·n lesions _ zrp)
@ Causes for Niacin Deficiency ,
One NAD!:1 m,.,,, ,_ _ is oxidized in the respiratory
--= C i. Dietary deficiency of tryptophan : Pellagra is
chain to general ½ATP see Chapter 21 for details).
seen among people whose staple diet is maize
But NADPH is use
thiNfiimfiatl,;'jr;;;;t~.
eactlon cp9 ·3 \1+) (South and Central America). In w.aiz niacin 1s
eresent; but it is in a f>ound rm, andls unavailable.
® NADPH Dependent Reactions Pellagra is also seen when staple diet is sorghum
The NADPH generating reactions are shown in as in Central and Western India. Sorghum, contairfs
Box 33.4. A few examples of NADPH utilizing enzymes leucine in high quantities. Leucine inhibits the
are shown in Box 33.5. Some enzymes can use conversion of niacin to (L~ucine pellagra).
either NAO• or NADP• as coenzyme, e.g. glutamate ii. Deficient synthesis: Kynureni~ase, an important
dehydrogenase. enzyme in the pathway of tryptophan, is pyridoxal

® Niacin Deficiency
© Pellagra
Pellagra is an Italian word, meaning "rough skin". c.,
Pellagra is caused by the deficiency of Tryptophan as ietary Sources of Niacin
well as Niacin. The symptoms of pellagra are:
The richest natural sources of niacin are an eas ,
i. Dirmitltl._: In early stages, bright red erythema
rice J)OliShing, liver, peanut, who e cereals, legumes,
occurs, especially in the feet, ankles and face (Figs.
meat and fish. About half of the requirement is met by
33.8A and B). Increased pigmentation around the
the conversion of tryptophan to niacin. About 60 mg of
neck is known as CalaL'a:: . The dermatitis
to han w~ d ~of..wapin.
is precipitated by exposure to sunlight (Figs. 33.8A
and B). ecommended Daily Allowance {RDA)

-----
ii. Diarrhea: This may lead to weig~ oss. Nausea
Normal requirement is 20 mg/day. During lactation,
and vomiting may also be present.
additional 5 mg is required.
iii. Dementia: It is frequently seen in ron cases.
1
De~ um is common in acute pellagra. I · b'li ,
i~ li~ co centf.e_te am:l~ J ; Dory are more
common in mild cases. At~ia,_§ln__Q_ Sfil!Sticity are
~.,u..:L.W.~
also seen. ·
Chapter 33: Water Soluble Vitamins 471

CHO

HO-rl-~: -0
0
II
P- OH

HC-ll..)
I
OH
3
N

~ver·
Pyridine Pyridoxine Pyridoxamine Pyridoxal Pyridoxal phosphate (PLP)

i. Gl1:Jtamate __.,. GABA (gamma aminobutyric acid) (see


Fig. 18.24). GABA is an inhibitory neurotransmitter,

Pyrldoxal phosphate is involved i etabolism. and hence in B6 deficiency, especially in children ,


convulsions may occur.
ii. Histidine __.,. Histamine, which is the mediator of
hence serum cholesterol is lowered. In high doses
allergy and anaphylaxis (see Fig.19.14).
niacin is useful to reduce Lp(a) levels.
iii. 5-hydroxytryptophan __.,. Serotooi.rl (see Fig.19.11 ).

Sulfur Containing Amino Acids


Nicotinic acid when given orally or parenterally produces
a tr · ~tian of the cutaneous vessels and The PLP plays an important role in methionine and
hista jne e!ej.se. The reaction is accompanied by cysteine metabolism.
itching,. burni1 19.,fill,~ling. a. Homocysteine + Serine __.,. Cystathionine (Enzyme
Cy:stathionine synthase) (see Fig.18.14)
I VITAMIN B6 b. Cystathionine __.,. Homoserine + Cysteine (Enzyme
--- Cystathionase).
Coenzyme Form Both these reactions require PLP. Hence in vitamin
Vitamin B6 is the term applied to a family of 3 related B6 deficiency homocysteine in blood is inc ased.
pyridine derivatives; pyridoxine (alcohol), pyridoxal o ·S'<GOUa~ted-w
(aldehyde) and pyridoxamine (Fig. 33.9). Active form
of pyridoxine is pyridoxal phosphate (PLP).

Functions of Pyridoxal Phosphate


The pyridoxal phosphate (PLP) acts as coenzyme for Heme Synthesis
many reactions in amino acid metabolism (Box 33.6).
The ALA synthase is a PLP dependent enzyme. This
is the rate limiting step in heme biosynthesis (see Fig.
Transamination 22.4)...So, in B6 deficiency, anem ay be'seen.
These reactions are catalyzed by amino transferases
(transaminases) which employ PLP as the coenzyme Procluction of Niacin
(see Fig. 17.6). For example: Thus pyridoxal phosphate is required for the synthesis
Alanine + Alpha ketoglutarate __.,. Pyruvate + of niacin from tryptophan (one vitamin is necessary
Glutamic acid (Enzyme Alanine transaminase) for synthesis of another vitamin) (see Fig. 19.10).
The clinical significance of blood levels of Kynureninase is a PLP dependent enzyme. Hence
transaminases is given in Chapter 6. in vitamin B6 deficiency niacin production is less.

Decar/joxylation G/ycogenolysis
All decarboxylation reactions of amino acids require Phosphorylase enzyme (glycogen to glucose-1-
PLP as coenzyme. A few examples are given below: phosphate) requires PLP. In fact, more than 70% of total
472 Section D: Nutrition

CH3 OH Pantothenic acid +


I I Adenine+ l}-mercaptoethanol-

9
HO - CH - C - C- CO- NH - CH - CH-COOH
2 I I 2 2 amine + P

I CH3 H
Pantoic acid
11
+ Beta alanine
I
3· ,s·-Adenoslne- +
r
Phospho-

I I phosphate pantotheine
Fritz A

r
Pantolhenic acid
I
CoenzymeA
I Lipmann
NP 1953
1899-1986
Fig. 33.10: Structure of pantothenic acid Fig. 33.11 : Structure of Coenzyme A (CoA)

PLP content of the body is in muscles, where it is a part reduces the formation of PLP and causes vitamin
of the pfiosphorylase enzyme. B6 deficiency.
/ ii. Cycloserine: Acts as ~ ntag~ ist.
Deficiency Manifestations iii. Ethanol: B6 deficiency oe• 1ritis is quite common in
of Pyridoxine alcoholics.
@ Neurological Manifestations
Dietary Sources of Vitamin 8 6
enzymes
ffine, Rich sources are yeast, rice polismng, wheat germs,
cereals, legumes (pulses), oil seeds, - - illtKl:-meat
are not produced in adequate amounts. Neurological fish and
symptoms are therefore quite common in B6 deficiency.
In children, B 6 deficiency leads to convulsions due to Requirement of 8 6
decreased formation of lW~I\. PLP is involved in the
synthesis of ; so B6 deficiency leads to
Vitamin B6 requirements are rslated to protein int,!ke
and noJ ,tg c.,aJ,grie lD,!gls_e (Box 33.6). It is recommended

--
demyelination of nerves and conse uent peripheral
that adults need 1 .!£ 2 rngW§';Y· During pregnancy and
neuritis. This is reversible with high doses of B 6 •
lactation, the requirement is increased to 2.5 mg/day.
@ Dermatological Manifestations
Deficiency of B6 will also affect tryptophan metabolism.
I PANTOTHENIC-ACID
-------.----,---
Since niacin is produced from tryptophan, B6 deficienc~ tructure
in turn leads to nigcin deficiency, which is manifested as
Pantothenic acid (Fig. 33.10) and beta mercaptoethanol-
pellagra.
amine are parts of coenzyme A (CoA) (Fig. 33.11 ). The
HSCoA is a nucleotide derivative.
@;ematological Manifestations
In adults, hypochromic microcytic anemia may occu oenzyme Activity of Pantothenic Acid
due to the inhibition of heme biosynthesis. Impaired anti- The beta mercaptoethanol amine contains one thiol or
body formation is also reported. sulfhydryl (-SH) group. It is the active site where acyl
The metabolic disorders which respond to vitamin
groups are carried. Therefore, coenzyme A is sometimes
B6 therapy are xanthurenic aciduria and llomocysti-
abbreviated as CoA-SH to denote this active site. The
nuria. ')) XOM-~v..€.
thio ester bond in acyl-CoA is a high energy bond.
t)~ d.t..wo~\ These acyl groups are transferred to other acceptors,
Effect of Drugs on Vitamin 8 6 for example:
i. INH: lsonicotinic acid hydrazide (isoniazid) is an Acetyl-CoA + Choline _, Acetylcholine + CoA
antituberculosis drug. It inhibits pt oxal kinase; (enzyme is acetylcholine synthase)
\I , t .Bb QJ/1,ted/ ~l
Chapter 33: Water Soluble Vitamins 473

Fatty acid - - - - - . - - - - CA cycle 0


Acetyl - _ _ . Fatty acid
synthesis
Pyruvate - - -• CoA CO2 carriage-+ NJ lN Apoen_zyme
___. Cholesterol

Pool
Steroids
lmidazole ring -+ ~
Thlophene ring-+ 1
ysine
Vincent du
Vigneaud
Ketone bodies S COOH NP 1955
1901-1 978
Acetylcholine
F ig. 33.13: Structure of biotin
Detoxification

Alpha keto- / .fCA cycle Dr C Gopalan (b. 1918), Founder Director of


glutarate '---...._ the National Institute of Nutrition, Hyderabad,
/ Gluconeogenes1s and former Director General of Indian Council
Odd chain - - _ . Succinyl- / of Medical Research, New Delhi, reported
fall acids CoA __.. Porphyrins the Burning Foot Syndrome, among the war
refugees, in 1946.
Pool - . Activation of
acetoacetate
ources of Pantothenic Acid
~etoxification
It is widely distributed in plants and animals. Moreover, it

Acyl groups are also accepted by CoA molecule during


-
is S}'.!2!,hesize2.,by !b,e ~!:.ma! bacterial flora in intestines.
Therefore, deficiency is very rare. Yeast, liver and eggs
are good sources.
the metabolism of other substrates, for example:
Pyruvate + CoA + NAO• -+ equirement of Pantothenic Acid
Acetyl-CoA + CO2 +' NADH l H+-
RDA is assumed to be about 10 mg/day.
(Enzyme is pyruvate dehydrogenase)
The important CoA derivatives are:
a. Acetyl-CoA
IBIOTIN_ _ - - - - - - -
b. Succinyl-CoA In old literature Biotin was known as anti-egg white injury
c. HMG-CoA factor. Biotin was isolated in 1942 by Vincent du Vig-
d. Acyl-CoA. neaud, who was awarded Nobel Prize in 1955.
The reactions of acetyl-CoA and succinyl-CoA are
summarized in Figure 33.12. Phosphopantothene is an
Structure of Biotin
important component of fatty acjq,.,svmhase complex. It consists of an imidazole ring fused with a thiophene
The ACP (acyl carrier protein) also contains pantothenic ring (Fig. 33.13).
acid ©
Coenzyme Activity of Biotin
Deficiency of Pantothenic Acid
Biotin acts as coenzyme for carboxylation reactions.
Gopalan's Burning Foot Syndrome is manifested as Biotin captures a molecule of CO2 which is attached to
paresthesia (burning, lightning pain) in lower extremities, nitrogen of the biotin molecule (Fig. 33.13). The energy
staggering gait due to impaired coordination and sleep required for this reaction is provided by ATP. Then the
disturbances. activated carboxyl group is transferred to the substrate.
These 9eficiency manifestations are rare in human
beings. The syndrome is seen during ~ e. in prison
camps, in alcoholics and in some renal dialysis
patients.
In experimental animals, deficiency has resulted in This enzyme adds CO2 to acetyl-CoA to forn:i malonyl-
anemia (due to reduced heme synthesis from succinyl- CoA. This is the rate limiting reaction in biosynthesis of
CoA), and reduced steroidogenesis (due to lack of fatty acids (see Step 1, Fig. 13.13).
acetyl-CoA). Acetyl-CoA + CO2 + ATP-+ Malonyl-CoA + ADP + Pi .
474 Section 0: Nutrition
'('_ t.9d ~c ci.c
OH ._ - Deficiency of Biotin

l.-J
@ A
Prolonged use of antibacterial drugs may lead to biotin
N ~ C ~:rN~ ~ CO- NH
deficiency. = ---
H2N~N -.cl~ Biotin deficiency symptoms include dermatitis, atro-
1 @ CH phic glossitis, hyperesthesia, muscle pain, anorexia and
I 2
hallucinations. Injection of biotin 100-300 mg will bring
IH2
I
__P_te_ri_di_ne,--_ _ _ _ _P_A_BA
____.I about rapid cure of these symptoms.
I c~o- 1
IPteroic acid Glutamic acid I Requirement of Biotin
+
Pteroylglutamic acid or folic acid About 200-300 mg will meet the daily requirements.
Fig. 33.14: Structure of folic acid
Sources of Biotin
Propionyl-CoA Carboxy/ase Normal bacterial flora of the u will provide adequate
Propionyl-CoA +CO2 + ATP-+ quantities of biotin. Moreover, it is distributed ubiquitously
Methylmalonyl-C,,q + ADP + Pi in plant and animal tissues. L~ yeast, peanut,
(see Step 1, Fig. 13.10) tnvcc CJ>'?' soybean, ~g yol~ are rich sources.

Pyruvate Carboxylase
1£.0LIC A_C_ID_ _ _ _ _ __
Pyruvate + CO2 + ATP -> Oxaloacetate + ADP + Pi
(see Fig. 10.21 ). This is important in two aspects. One, Chemistry of Folic Acid
it provides the oxaloacetate, which is the sparking
The Latin word folium means leaf of vegetable. Folic acid
molecule for TCA cycle. Second , it is an important
is abundant in vegetables. It is composed of three consti-
enzyme in the gluconeogenic pathway.
tuents. The pteridine group link~d with para amino-
Biotin-Independent benzoic acid (PABA) is called pt~roic acid. It is then
Carboxylation Reactions attached to glutamic acid to form· pteroylglutamic acid
or folic acid (Fig . 33.1 4). Folic acid is soluble in water.
i. Carbamoyl phosphate synthetase, which is the
When expo~ rapidly destroyed.
stepping stone for 4rea_and:::Qy:cimidio~ synthesis
(see Step 1, Fig. 17.11).
Absorption of Folic Acid
ii. Addition of CO2 to form C6 in purine.Jing
iii. Malic enzyme, converting pyruvate to malate. Folic acid is readily absorbed by the upper part of
. =- "'=--~
jejunum.
Biotin Antagonists
Avidin, a protein present in egg white has great affinity Coenzyme Functions of Folic Acid

-
to biotin. Hence intake of raw (unboiled) egg may cause
A. The folic acid is first reduced to 7J.8-dihydrofolic
biotin deficiency. Biotin was originally named as anti-
acid and further reduced to 5,6,7,8-tetrahydro
egg-white-injury-factor.
Avidin is heat labile, and boiling of egg will neutralize folic acid (THFA) (Fig. 33.15). Both reactions are
the inhibitory activity. One molecule 9f avid in can combine catalyzed by ~ e n t folate reductase.
with four molecules of biotin. Egg white contains avidin B. The THFA is the carrier of one-carbon groups.
and egg yolk contains biotin. One carbon compound is an organic molecule that
The affinity of avidin to biotin is greater than most contains only a single carbon atom. The following
of the usual antigen-antibody reactions. Therefore, groups are one-carbon compounds:
avidin-biotin system is commonly utilized for detection i. Formyl (-CHO)
of pathogens by ELISA. ii. Formimino (-CH=NH)
Chapter 33: Water Soluble Vitamins 475

Folic acid

NADPH + H.

7,8-dihydrofolic acid
Fig. 33.16: NS, N 10-methenyl THFA. One carbon unit (red ring) is
NADPH + H• attached to N5 and N 10 groups (blue rings) of tetrahydrofolic acid

One carbon pool __.


Gertrude Melhyl-THFA~HFA
Elion

""K
5,6,7,8-tetrahydrofolic acid (THFA) NP 1988
1918-1999
Vi~mio Melhyl~balamioo
Fig. 33.15: Folate reductase

iii. Metheny! (-CH=) Methionine Homocysteine

iv. Methylene (-CH 2- )


ATP Adenosine
v. Hydroxymethyl (-CHpH)
vi. Methyl (-CH 3 ). :; er ,
yl ( \
-"""'~~!
1t-i ...... "'1r
C. These one carbon compounds are attached either

-y
PPi + Pi
to the 5th or to the 10th or to both 5th and 10th
nitrogen atoms of THFA (Fig. 33.16). The one-carbon 3
5-adenosyl S-adenosyl
metabolism and their interconversions are given methionine homocysteine

-----...
in detail in Figure 17 .1 4.
D. Methyl group in N5-methyl THFA s used for synthesis
C 3

of active methio es part in transme- L __ _ _ _ M_'rlh


_- .;.
Yl'-~-
c~;_
e_p1_or_-_- +
__M
_e_th_y~las..ite..,,d_p_r_,
oi_u_c__,_=i-
' ....:.J
,- \-\.\A
thylation reactions (Fig. 33.17). Such transmethyla- Fig. 33.17: Transmelhylation reactions. (2) = Homocysteine
tion reactions, where the methyl group do,nor is methyl transferase ; (3) = methyl transferase

SAM, are required for synthesis of ch'oline, e ine h-


rine, cseatine, etc. (Table 18.1). iv. Hemolytic anemias: As requirement of folic acid '- ,.,, l-o....,
becomes more, deficiency is manifested.
Causes for Folate Deficiency - v ,/
v. Dietary deficiency: A~ of vegetables in food
Folic acid deficiency is very common in India, and is fQ!. ~ ods may lead to deficiency.
'perhaps the most commonly seen vitamin deficiency. vi. Folate trap: The only way for generation of free
i. Pregnancy: Folate deficiency is commonly seen in THFAis step No.1 in Figure 33.17. When B12 is defi-
pregnancy, where requirement is increased. cient, this reaction cannot take place, 1 ading to
7 ...__
1
ii. Defective absorption: In sprue, celiac disease folate deficiency (see under vitamin B 12) . I t-.\Q\-1-Un-r Y~e
etc., absorption is defective.
~¼iN:
iii. Drugs: In the diet, ~i"ns:are mai ly_jQ p.ol}'91L®- Deficienty Manifestations
\.... \J.ff. .r:-U'(" ~v ~~''°' \rll-p-,...~f"'\~·
rus-te _torm. Gastrointestinal enzymes in the gut ~o-'('.
Reduced DNA synthesis
remove the glutamate residues and only the mono-
glutamate for~ ~li~ ~ ~ s2 r~ .~ n- In folate deficiency, THFA is reduced and thymidylate
vulsant drugs (dilantin, phenobarbllone) will inhiEit synthase enzyme is inhibited. Hence dUMP is not con-
the intestinal digestive enzyme, so that fQlate absorp- verted to dTMP. So dTTP is not available for DNA synthe-
tion is reduced. sis. Thus cell division is arrested. Very rapidly dividing
476 Section 0: Nutrition

Assessment of Folate Deficiency


i. Blood level: Normal folic acid level in serum is
~bo~ L and a~ L of packed
cells. The level is measured by radiojmrnun.oassay.
ii. Peripheral blood picture .
....-"> ch C>-P>e "'-:, do..o..l
me.. ,
Sources of Folic Acid . 0-l'Glc..Cbo,\t& ) £,
pu:c\•'IX' J a ru.,
Rich sources of folate are yeast, green leafy vegetables.
Fig. 33.18: (Left) Common manifestation of folic acid deficiency is ~ oderate sources are cereals, pulses, oil seeds and egg.
macrocytic anemia. (Right) Megaloblastic anemia with nucleated ~ ilk is a poor source for folic acW
RBCs in vitamin B12 deficiency

cells in bon/riiarrow and intes!n"a1 mucosa are therefore Recommended Daily Allowance (RDA)
, most seriously affected. The requirement o f ~ mcg/day. In Q _
(r Macrocytic Anemia 0 nancy the re uirement is increased to 1QO mcq/dhd
durin actati to 00 mcg/day.

J:i It is the most characteristic feature of folate deficiency


(Fig. 33.18). During RBC generation, DNA synthesis is
Folic Acid Therapy
delayed, but protein synthesis is continued. Thus hemo- Therapeutic dose is 1 mg of folic acid per day orally. Felic
globin accumulates in RBC precursors. This asynchrony acid alone should not be given in macrocytic anemia,
or dissociation between the maturity of nucleus and because it may aggrava e the neurological manifestation
cytoplasm is manifested as immature looking nucleus of B 12 deficiency. So, fglic acid and vitamjn 2 are givena,
in cpmhinatjon to patients. Regular supplementation I.
and mature eosinophilic cytoplasm in the bone marrow
cells. lteticulocytosis is often seen. These abnormal of folic acid may reduce the incidence of birth defects,
RBCs are rapidly destroyed in spleen. Hemolysis leads cardiovascular diseases and cancers.
to the reduction of lifespan of RBC. Reduced generation .
and increased destruction of RBCs result in anemia . ~ olate Antagonists
The peripher I ood picture in folate deficiency is'fsulfonamides
described as macrocyti and in cobalamin deficiency
They have structural similarity with PABA (Fig. 33.19).
as egal~blasic n 12 deficiency, there are additional
Therefore, they ~ tiveLy...inR-ibit-the enzymerespon-
neurologi symptoms.
sib1eiort1Te 1ncornfuatirin_o.f.PAS). tfifcfdih.'id,[Qptero~ ,
Hyperhomocysteinemia \fJJ~...,, a.. the immediate precursor of folic acid (see competitive
~'c- inhibition, Chapter 5). Bacteria can synthesize folic acid
Felic acid deficiency may cause increased homocysteine from the components, pteridine, PABA and glutamate.
levels in blood since remeth~ f I homocyste}it_ When su!!9namid are._.~eo. such rntcroor nisms
is affected. Plasma homocysteine levels (above 15 c a ~ c . i Q _ a n d hence their growth is
mrriol7t)s known to increase the ri~ of corona!}'. artery inhibited. As man cannot synthesize folic acid, the entire
diseases. Providing adequate doses of pyridoxine, B12 molecule has to be supplied in the diet. The preformed
§na=foiic acid may lower the homocysteine levels. folic acid cannot enter into bacteria, but only into
mammalian cells. Thus sulphonamides are very good
Birth Defects $ antibacterial agents, which do not affect the human
Felic acid deficiency luring pregnancy may lead t~ cells. Gerhard Domagk discovered the antibacterial effect
r1[.D 109 detects io the fetusJso, intake of folic acid is a of Prontosil (a sulfonamide) in 1932 for which he was
must from early pregnancy. awarded Nobel Prize in 1939.
Chapter 33: Water Soluble Vitamins 477

QNH, N '-::::-----r
0 N
Pyrrole ring lmidazole ring 1--1
N~NH

H
II /4 N-pyrrole-substituted
H2NAN Cl
Substituted pyrrole-N - + C o ~ N-pyrrole-substituted
NH2
Substituted pyrrole-N __... f - . . . . . __ N-benz-1midazole
Sulfanilamide Pyrimethamine
I substituted
Fig. 33.1 9: Folate antagonists R

R = -CN, - OH, -adenosyl or -methyl groups


Py~ thamine
Fig. 33.20: Simplified structure of vitamin B12
This antifolate agent is used against plasmodial infec-
tions (ant~ la_fi...al d_!.ug) (Fig. 33.19). Gertrude Elion
cobalamin. The 6th valency of tbe cobalt is satisfied by
synthesized pyrimethamine, who got Nobel Prize in 1988.
any qf the following groups: c~ nide, h_ycfro\cyl, adenosyl
',
or methyl (Fig. 33.20).
Amin~ terin and A~opterin
FoR.nn& ·
Aminopterin (4-amino folic acid) and amethopterin Cyanocobalamin
(methotrexate) (4-amino, 10-methyl folic acid) are 6 ~0.J
powerful competitive inhibitors of,olate reductasID and When cyanide is added at the R position, the molecule
THFA generation. Thus these drugs ~_@aseJ_he DNA is called cyanocobalamin. The CN group has no physi-
f o r m a ~ . They are widely used as anti- ological function, it is only a laboratory artefact. Oral
cancer drugs (antimetabolites), especially for leukemias preparations are in this form.
and choriocarcinomas.Tolinic acid (citrovorum factor) is
..
given to rescue the patient from toxicity of methotrexate.
Methotrexate is also used in treatment of rheumatqid
Hydroxycobalamin
When hydroxyl group is attached at the R position, it
artl),r_itis q_S an immunosuppressant. is called hydroxycobalamin or vitamin 2
Elf
--, Injectable
preparations are in this form.
I VITAMIN B12
Adenosylcobalamin (Ado-B 1 )
Synonyms are co alamio, extrinrrc factor (EF) of Castle
and anti-p&roicia11s anemia factor. In 1849, Thomas Inside the cells, deoxy adenosylcobalamin or Ado-B 12 is
Addison described pernicious anemia. William Murphy formed. This is the major
So-
storage fg!;Dl, seen io liver.
and George Minot showed that liver therapy is very
effective to treat pernicious anemia. For this work, they
Methylcobalamin
were awarded Nobel Prize in 1934. Dorothy HoC.:~kin This is the major fonn s,eo in bloop circulation. The
suggested the structure by X-ray diffraction studies Ado-B12 and methyl B12 are the functional coenzymes
(Nobel Prize, 1964 ). Later Robert Woodward synthesi- in the body.
zed B 12 and proved the structure (Nobel Prize, 1965).
Absorption of Vitamin 8 12
Chemistry
Absorption of vitamin B 12 requires the intrinsic factor
Vitamin B12 is water soluble, heat stable and red in color. (IF) of Castle (Fig. 33.21 ). The B12 is otherwise known
It contains 4.35% cobalt by weight. It contains one cobalt as extrinsic factor (EF), that is, the factor derived from
atom. Four pyrrole rings coordinated with a cobalt atom external sources. Intrinsic factor is secreted by the
is called a Corrin ring. Note the similarity between the gastric parietal cells~°"'-1 <0 J._.... i po r LC.~
Corrin ring and the po hyrin ring system (see Chapter 22). One molecule of IF can combine with 2 molecules
covalently linked to of B 12• This IF-812 complex is attached with specific
a substituted - ~ - ..\.-~,------ . This is then called receptors on mucosa! cells. The whole IF-B 12 complex is
478 Section D: Nutrition

Food-Cbl The metabolism of oQQ_chafnt fatty acids, ~ e,


isole(dcine, me i;fonine and tllr:[o;;ine lead~e produc-
tion of methylmalonyl-CoA. f½, 1 M \/ T
'J II ~e.JJ_ ~ v e ~
William Thomas
R-Cbl
'B- omocysteine Methyl Transferase
)
Castle Addison
(1897- 1990) (1793-1860)

Duodenum
< The production of methyl THFA is an irreversible step
.__.. _. ........ ---+ IF-Cbl
Ileum
i (see Fig. 17.14). Therefore, the only way for regeneration
of free THFA is step No. 1 in Figure 33.17. When 8 12 is
deficient, this reactio canoot ake place. This is called
f'.'.">T'1""a. 1"1\~r i : ~... r t
the meJ.byl folate trap. This le ds to he asso"tiated folic
\.;
acid scarcity in B12 deficiency. \ "' \ "l'e.t'
,f
~I
't> fr\ - r' o~eNi"
t>
'D
Causes of B12 Deficiency
~\,,,..o.,....'
Fig. 33.21 : Absorption and storage of vitamin B 12 • R = cobalo-
philin; Cbl = cobalamin; IF = intrinsic factor; TC = trans cobalamin Nutritional vitamin~ di ienc~ ·s v~ COJTimon in India,
. VK~~ 'ft\ ot . .
especially a.wang y o ~no"1!c
group. The only source for 8 12 in vegetarian diet is
is absorbed from ileum, curd/milk, and lower income group may not be able to
afford it.
Transport and Storage .
In the blood, methyl B12 form .1s .
predominant. Trans-® Decrease in Absorption--
cobalamin, a glycoprotein, is the specific carrier (Fig. Absorptive surface is reduced by gastrectomy, re§.ection
33.21 ). It is stored in the liver cells, as ado-B12 form, ~ m anc;l malabsorption syn<;l.IQmes.
in combination with transcorrin. Generally, 8 complex . r ~H · -\P-.L~'l O.• O\: rr w"11 \i If'\
vitamins are not stored in the body, is an excep io ddtSe[1lan Permctous AQ!;,;;:_f!J" > J. -~
Whole liver contains about 2 mg of 8 12 , which is sufficient It is~jv rare inIndra. but common in European countries.
for the requirement for 2-3 years. So, B12 deficiency is When it was described in 1849 by Thomas Addison, it
seen only years after gastrectomy. was pernicious (fatal), without any known remedy. It is
- :~ ~ \ e. manifested usually in persons over 40 years. It is an
Function I Role of B1 eo.}-h,.,, m autoimmune disease with a strong fa milial background.
\...> Q.14 u.& ~ ~ ~ ~ ~ntibodies are generated against IF. So, IF becomes
® Me\hylmalonyl-CoA
.,./CA.Cf\ 'lf' -l ~\..
lsomerase deficient, leading to defective absorption of 8 (Figs.
12

~'f,...0:},1ethylmalonyl-_9oA is formed in the body from ~ 22A to D).


?~ '- PIQPiQDY!-CoA. It is then converted to L form by a t . At h _,, , $ ~
I? racemase and then isomerized by methylmalonyl-CoA as rte rap Y
mutase (containing Ado-812 ) to sucb'in 1-CoA, which Although true Addisonian pernicious anemia is rare,
enters into citric acid cycle (Fig . 13.10). I~ , similar atrophy of gastric epithelium leadin to deficiency
methylmalony!-CoA accumulates and methyl malonic of IF and decreased 8 12 absorption is mon in India.
- .as;idurja.J;1.CCULS In chronic iron deficiency anemia, there is generalized
- w,\,lo\\~ . ~ - &~c.lID~ ~ a . ,
'\9U, e,U-e '-30\)._ ~){ 3fr
"\J (\-, g l'l- 6L.I)' CA~ .( 9-: GnolUS,(.\{)14 )
'-:tl"-Rc.") Chapter 33: Water Soluble Vitamins 479

IC___,____ Vrtamin 8
12
ii.( Megaloblas ic anemia: In the peripheral blood,
megaloblasts and immature RBCs are observed
Intrinsic factor from stomach (Fig. 33.18).
iii. 'Abnorm~ homocY.steine le~~ In vitamin B,2
Ileum epithelium deficiency, step No. 2 (Fig. 33.17) is blocked, so
that homocysteine is accumulated , leading to horn-
=
E-• 4

,-1;:,~ .. ~-:... "-:~


.. - :. -=-
,..-:~;i- IF and 8
12
combines
iv. e!'1 elination : J n vitamin B12 deficiency, step 3
(Fig . 33.17) is also suppressed due to the non-avail-
n""\'~ RsH
~,!)
ability of active methionine. 1h~
; -- - - - - - 8 12 absorbed
of ~J~hatid lethanola in~ @l~ospbatlc1ylmaiinF
is not adequate. This leads ~ ent formation of

1: 1- Antibody against IF
myelirts-heaths of nerves, dem~ rlnation and rtfuiro-
logical lesions.
v. Subacute combined degeneration : Damage to
nervous system is seen in 8 12 deficiency (but not in
folate def)Riency). There · em elinatlo affect-
ing~y reb~ ? cort a;,>Vell as dorsl:ll ~ orumn and
Antibody combines
with IF; IF and 8 12 p~ midal tract of s~inal cord. ~ _ senSQIY.,and
could not combine;
8 12 not absorbed
m...Q!~ct~are_2ffected, it is named as combi~ d
degenerati,on. s i mmetric_gj_paraesthesia of extre-
mities, alterations oU endon..and daep..seoses and
Figs. 33.22A to D: (A) Intrinsic factor secreted from stomach reflexes, l9ss ~ ~se, uosteadiness. in
reaches intestine; (B) Vitamin B,2 absorbed with the help of intrinsic gait, positive Romb.e(g.'s sign {falling when eyes
factor; (C) In pernicious anemia , antibody against IF is produced ;
-;; closed) and ROSiti~e Babinski's sign (extensorf

---
(D) In presence of antibody, absorption is not taking place
plantar reflex) are seen.
mucosal atrophy. In about 40% cases of iron deficiency vi. chlorh dria: Absence of acid in gastric juice is
anemia, superadded gastric atrophy is seen. associated with vitamin B 12 deficiency.

Pregnancy - P~b~ Assessment of B12 Deficiency


lncre~ ed c~Qui~ement of vitamin in pregnancy is another i. Serum B12 : It is quantitated by radioimmunoassay
common cause for vitamin B 12 deficiency in India. or by ELISA.
. Po~ ii. Peripheral smear: Peripheral blood and bone
Fish Tapeworm ..,.;,, f 'l-.., :\,i f -X' marrow morphology shows megaloblastic anemia
Al thoughG t seen in ln&a, the fish tapeworm 'mghi/-~1',,., ~ Fig. 33.18).
lobothrium ./;HJmJ. infection is common in Scandinavianr- ~'fii.
Homocystinuria: (see Chapter 18, under Methio-
countries where eating live fish is a delicacy. This tape- nine).
worm has a special affinity to B12 causing reduction in
available vitamin. Treatment
If megaloblastic anemia is treated with folic acid alone,
Deficiency Manifestations the anemia may improve, but associated neurological
iCToJate-tr, p: Vitamin B12 deficiency causes simul- symptoms are aggravated. Hence all macrocytic
taneous folate deficiency due to the folate trap. anemias are generally treated with folate and vitamin
Therefore all the manifestations of folate deficiency 8 , 2• Therapeutic dose of B,2 is 100 to 1000 microgram
are also seen (for details, see under folic acid). by intramuscular injections.

~ ~ \ - on -..1\t -B,~
NOC -V L c...€ a.. i n 1',,\~"ev-,.Y'E - ~~~'I"'>~ 'l.'C'i~ ¾
480 Section D: Nutrition

OH
Figs. 33.23A and B: (A) Choline, left side and (B) Myoinositol,
right side Henry Dale Otto Loewi Bernard Katz
NP 1936 NP 1936 NP 1970
1875-1968 1873-1961 1911-2003
Requirement of Vitamin 8 12
Normal daily re uirement is(1-2 mcgtciay.'Ouring preg- Acetylcholine esterase

ac atio , this is increas , 2~ ~y. Acetylcholine - -+ Choline + Acetate


Those who ta e o ic'aci , should also take vitamin 8 12 . Acetylcholine is an important neurotransmitter.
©derly peopi&are advised to take 8 12 supplementation.

Dietary Sources
IINOSITOL_ - -- --
Myo-inositol is seen in tissues (Fig. 33.23B). Inositol
Vitamin 8 12 is not ~nt in ve etables@¥er js tha deficiency is unknown to man.
riEnesf sou.@;curd is a because ~
b[ c]llus can synthesize B 12 • Biochemical Functions of Inositol
ICHOLINE_ --- 1. Inositol is a constituent of certain phospholipids.
2. Phosphatidylinositol is an important constituent
Choline is synthesized in the body (see Fig. 18.11) and
of cell membranes. In response to extracellular
therefore it is not a vitamin. But in view of its importance
signals, inositol triphosphate (IP3 ) is released from
in nutrition, conventionally, it is included as a member
phosphatidylinositol. IP3 is a second messenger (see
of vitamin B complex. Rice polishings, vegetables,
milk, egg and liver are good sources. It is synthesized Chapter 45) leads to cellular metabolic activation.
from serine. Structure of Choline is shown in Figure 3. It is a lipotropic factor and prevents fatty liver.
33.23A.

Biochemical Functions
Historical Perspectives
i. Myelin sheath: It is made by phospholipids. Demye-
A description of scurvy was found in the Ebers papyrus
lination causes severe diseases of nervous written in 1500 BC in Egypt. During the voyage of Vasco
system. da Gama, around the cape of Good Hope to India in 1498,
ii. Fatty liver: In choline deficiency, neutral fat and he lost two-thirds of the crew due to scurvy. The French
explorer, Jacques Cartier, in 1536, during the voyages to
cholesterol esters accumulate in liver, leading to fatty discover eastern parts of Canada, was laid up with scurvy.
liver. Choline is able to prevent fatty liver and cirrhosis A friendly native gave an extract from the leaves of spruce
(see Chapter 13). tree, which produced remarkable cure to scurvy (Fig. 33.24).
James Lind published "Treatise on Scurvy", in 1753. These
iii. Transmethylation reactions : Choline can donate observations led to compulsory rationing of lime or lemon
three methyl groups to the one-carbon pool (see juice to all the crew of the British Royal Navy from 1795
one-carbon metabolism in Chapter 17). Finally, onwards. So the British sailors were nicknamed as "Limeys".
However, it helped to eliminate scurvy from the British Navy,
these methyl groups are transferred to homo- I
while opponents continued to suffer. No wonder, in course
cysteine to produce methionine, which is used for of time, Britain had the colonies in which the sun never set.
transmethylation reactions (see Chapter 18). In 1907, Holst and Trochlich produced scurvy in guineapigs.
The factor was isolated in 1930 and named as "Hexuronic
iv. Acetylcholine (ACh) synthesis: acid" by Albert Szent-Gyorgi (Nobel Prize, 1937). In 1933,
Choline acetylase Haworth established the molecular structure. He renamed it
as ascorbic acid (Nobel Prize, 1937).
Choline + Acetyl-CoA-- - Acetylcholine + CoA
Chapter 33: Water Soluble Vitamins 481

William George DorothyC Robert


PMurphy R Minot Hodgkin Woodward
NP 1934 NP 1934 NP 1964 NP 1965
1892-1987 188fr1950 1910-1994 1917- 1979

Fig. 33.24: Red Indian gives the leaves to Cartier

Chemistry of Vitamin C
Biochemical Functions of Vitamin C
It is V{at~ b l e and is e2§ily destroyed by heat. alkali
and~e. In the process of ~ki~g, 70% of vitamin Reversible Oxidation-Reduction
C is lost. It can change between ascorbic acid and dehydro-
The structural formula of ascorbic acid closely ascorbic acid. Most of the physiological properties of the
resembles that of carbok9drates (Fig. 33.26). The strong vitamin could be explained by this redox system.
red~Ci!).IJ 12,[0!WrtY qW£i.1an1i~J1epends on the double-
bonded (enediol) carbons. Hydroxylation of Praline and Lysine
Ascorbic acid is necessary for the post-translational
antiscorbutic activity. -ascorb acid has no activity. hydroxylation of praline and lysine residues. Hydroxy-
- - ~~\e, proline and hydroxylysine are essential for the formation
Biosynthesis of Ascorbic of cross links in the collagen, which gives the tensile
Acid in Animals strength to the fibers. This process is absolutely neces-
sary for the normal production of supporting tissues such
Most animals and plants can s~tbesize ascorbic acjd
as osteoid, collagen and intercellular cement substance
from~ The pathway is described in Figure 10.44. of capillaries.
Man, higher primates, guinea pigs and bats are the
only species which caQJl.Ot syolbesize ascorbic acid Tryptophan Metabolism
(block in g ~ ~ i E , , - . ~- ~y lack the
Ascorbic acid is necessary for the hydroxylation of
genes responsible fol~~ rs nzyme. The tryptophan to 5-hydroxytryptophan . This is required for
vitamin, therefore, should be su plied in the diet of these the formation of serotonin (see Fig.19.11 ).
species.
Tyrosine Metabolism
Metabolism of Ascorbic Acid Vitamin C helps in the oxidation of parahydroxyphenyl
Ascorbic acid is readily absorbed from gastrointestinal pyruvate to homogentisic acid (see Fig. 19.2).
tract. The vitamin is excreted in urine. Since vitamin C is a
strong r~ t . the Benedict's test will be posi-
Iron Metabolism
tive in the urine sample after the vitamin administration. Ascorbic acid enhances the iron absorption from the
Oxidation of ascorbic acid yields dehydro ascorbic intestine (see Chapter 34). Ascorbic acid reduces ferric
acid, which is oxidized further to oxalic acid through iron to ferrous state, which is preferentially absorbed.
diketo-L-gulonic acid (Fig . 33.26). Ascorbic acid is partly
excreted unchanged and partly as oxalic acid. Most of Hemoglobin Metabolism
the oxalates in urine are derived from ascorbic acid, and It is useful for re-conversion of met-hemoglobin to
the rest from glycine metabolism. hemoglobin.
482 Section D: Nutrition

Hemorrhagic Tendency
In ascorbic acid deficiency, collagen is abnormal
and the intercellular cement substance is brittle. So
capillaries are fragile, leading to the tendency to bleed
even under minor pressure. Subcutaneous hemorrhage
may be manifested as petechiae in mild deficiency
and as ecchymoses or even hematoma in severe
conditions.

Internal Hemorrhage
In severe cases, hemorrhage may occur in the
conjunctiva and retina. Internal bleeding may be seen
Figs. 33.25A to C: (A) Gingivitis and bleeding gum in vitamin as epistaxis, hematuria or melena.
C deficiency; (B) Lime and (C) Gooseberry are good sources
of vitamin C Oral Cavity
In severe cases of scurvy, the gum becomes painful ,
Folic Acid Metabolism swollen , and spongy (Fig. 33.25A) and finally teeth are
lost. Wound healing may be delayed .
Ascorbic acid is helping the enzyme folate reductase
to reduce folic acid to tetrahydrofolic acid (Fig. 33.15). Bones
Thus it helps in the maturation of RBC .
In the bones, the deficiency results in the failure of the
osteoblasts to form the intercellular substance, osteoid.
Steroid Synthesis
Without the normal ground substance, the deposition of
Large quantities of vitamin C are present in adrenal bone is arrested. The resulting scorbutic bone is weak
cortex. The ascorbic acid is depleted by ACTH and fractures easily. There may be hemorrhage into joint
stimulation. So the vitamin has some role in adrenal cavities. Painful swelling of joints may prevent locomo-
steroidogenesis. Vitamin C helps in the synthesis of bile
tion of the patient.
acids from cholesterol.
Anemia
Antioxidant Property
In vitamin C deficiency, microcytic, hypochromic anemia
As an antioxidant (see Chapter 30). Daily intake of
is seen. The reasons for anemia may be:
vitamin C reduces this risk for cancer.
a. Loss of blood by hemorrhage
Cataract b. Decreased iron absorption
Vitamin C is concentrated in the lens of eye. Regular
Dietary Sources of Vitamin C
intake of ascorbic acid reduces the risk of cataract
formation. Rich sources are amla (Indian gooseberry) (700 mg/
100 g), guava (300 mg/100 g), lime, lemon and green
Deficiency Manifestations of Vitamin C leafy vegetables (Figs. 33.25B and C).

Scurvy Requirement of Vitamin C


Gross deficiency of vitamin C results in scurvy. Recommended daily allowance is 75 mg/day (equal to
50 ml orange juice). During pregnancy, lactation, and
Infantile Scurvy (Barlow's Disease) in aged people requirement may be 100 mg/day.
In infants between 6 and 12 months of age, (period in
which weaning from breast milk), the diet should be
Therapeutic Use of Vitamin C
supplemented with vitamin C sources. Otherwise, i. Vitamin C is used as an adjuvant in infections.
deficiency of vitamin is seen. Clinical dose is 500 mg per day.
Chapter 33: Water Soluble Vitamins 483

O=C O=C-H
0=1 7
I
OH-! 7 O=C O=C
11 o I o I
-2H

H-C
1
OH- _ J O=, _ J
H-C
O=C
I
H-C-OH
Walter N Albert
I I I Haworth Szent-Gyorgyi
OH-C-H OH-C-H HO-C-H - - + COOH NP 1937 NP 1937
I
CH2OH
I I I 1883- 1950 1893-1 986
CH2OH CH - OH
2 COOH
L-ascorbic Dehydro- Diketo-l- Oxalic acid
acid ascorbic acid gulonic acid

Fig . 33.26: V itamin C ; structure and catabolism

TABLE 33.1: Summary of water soluble v1tam1ns discussed in this chapter


Name Coenzyme form RDA Main reacrion using the coenzyme Deficiency disease
Thiamine Thiamine pyrophosp hate {TPP) 1- 1.Smg Oxidative decarboxylation of alpha Beriberi
keto acids
Riboflavin Flavin adenine dinucleotide 1.5 mg Dehydrogenation, oxidized in ETC Glossitis, angular stomatitis
{FAD) (1.5 ATP)
Niacin Nicotinamlde adenine 20 mg Dehydrogenation, oxidized in ETC Pellagra
dinucleotide {NAD and NADP) (2.5 ATP), reductive biosynthetic
reactions and hydroxylation

Pyridoxine Pyridoxal phosphate (PLP) 1- 2 mg Transamination, decarboxylation of Seizures, anemia, homocystinuria


amino acid s
Pantothenic acid Coenzyme A, ACP 10mg CoA derivatives, acyl carrier proteins Burning foot syndrome
Biotin Biotin 200-300 mg Carboxylation No specific disease
Folic acid Tetrahydrofolic acid {THFA) 200 microg One-Carbon group carrier Macrocytic anemia
Vitamin B12 Adenosyl B12, methylcobalamin 1- 2 microg lsomerization of methylmalonyl-CoA, Megaloblastic anemia, Subacute
remethylation of homocysteine to comb ined degeneration, methyl
methionine malonic aciduria
Ascorb ic acid No specific form 75 mg ~riti.9xldant pco~rty. h_ydroxylation Scurvy
ofcollaa~ri
Note:The requirements are significantly higher in pregnancy and lactation.

ii. Because of its power to heal wounds, vitamin C gaze is noted bilaterally. His gait is very unsteady. The
has been recommended for treatment of ulcer, urine drug screen was negative and he had a positive
trauma, and burns. Except in scurvy, the therapeutic blood alcohol level. The emergency room physician
use of vitamin is not specific. administers thiamine. What is the most likely diagnosis?
Summary of all water soluble vitamins are shown in
Table 33.1.
!t
•• · Clinical Case Study 33.2
Sl
u:, Clinical Case Study 33.1 A 65-year-old chronic smoker and alcoholic suffered
from nonspecific symptoms like painful swallowing, inso-
A 59-year-old male is brought to the emergency depart- mnia, epigastric discomfort and recurrent diarrhea. On
ment after a family member found him extremely confused examination , he had disorientation, stomatitis, glossitis,
and disoriented, with an unsteady gait. The patient has esophagitis and exfoliative dermatitis. Laboratory
been known in the past to be a heavy drinker. He has no measurements revealed leukocytosis, elevated ALT,
known medical problems. On examination, he is afebrile AST and GGT and there was ultrasound evidence of
with a normal blood pressure. He is extremely disoriented fatty liver. Treatment with 500 mg daily nicotinamide
and agitated. Horizontal rapid eye movement on lateral was started when rashes improved and other symptoms
484 Section D: Nutrition
were reduced. He restricted alcohol and smoking and demonstrated megaloblastic anemia. What is the most
increased consumption of other B complex vitamins and likely diagnosis? What is the most likely underlying
food sources of B complex vitamins. What is the likely problem for this patient? What are the two most common
condition? What is the pathophysiology of the findings? causes of megaloblastic anemia and how would this
patient's history and examination differentiate the two?
0 0
• • Clinical Case Study 33.3 • • Clinical Case Study 33.6
A 32-year-old female is being treated with methotrexate A 45-year-old man presented with loss of appetite,
for a recently diagnosed choriocarcinoma of the ovary, fatigue, muscle weakness and emotional disturbances.
and presents with complaints of oral mucosal ulcers. Physical examination showed enlarged liver that was
About 5 weeks ago the affected ovary was surgically firm and nodular, mild jaundice and smell of alcohol in
removed. The patient has been taking methotrexate breath . Hematological examination showed macrocytic
for 2 weeks. On examination, patient was afebrile but anemia, and bone marrow showed presence of
appeared ill. Several mucosal ulcers were seen in her megaloblasts. Serum folate was reduced and vitamin
mouth. The patient also had some upper abdominal B 12 and iron were normal. What is the cause of
tenderness. Her platelet count was decreased at megaloblastic anemia in this patient?
60,000/mm3 (normal 150,000 to 450,000/mm3 ) . What is 0
the most likely etiology of her symptoms? What is the •'ii• Clinical Case Study 33.1 Answer
biochemical explanation of her symptoms? What part of
Wernicke-Korsakoff syndrome (thiamine deficiency)
the cell cycle does me th0trexate act on? often associated with chronic alcoholics.

0 Importance of thiamine: It is used as a co-factor in


• • Clinical Case Study 33.4 enzymatic reactions involving the transfer of an aldehyde
group. Without thiamine, individuals can develop
A 47-year-old female is brought to the emergency dementia, macrocytic anemia (folate deficiency), liver
department with complaints of malaise, vomiting and disease, depression, cardiomyopathy, and pancreatitis.
fatigue. The patient reveals alcohol abuse for the last 10 Thiamine deficiency is uncommon except in alco-
years. She has been to rehab on several occasions for holics as a result of nutritional deficiencies and mala-
alcoholism but has not been able to stop drinking. She bsorption . The classic clinical triad of dementia, ataxia
denies cough, fever, chills, upper respiratory symptoms. (difficulty with walking), and eye findings may be seen,
She reports feeling hungry. On physical examination, she but more commonly, only forgetfulness is noted . Some-
appears malnourished but in no distress. Her physical times, thiamine deficiency can lead to vague symptoms
examination is normal. Her blood count revealed a such as leg numbness or tingling . Other manifestations
normal white blood cell count but also showed anemia include beriberi, leading to a high cardiac output, heart
with large red blood cells. Other tests were normal. failure and vasodilation. Affected patients often feel
What is the most likely cause of her anemia? What is warm and flushed.
the molecular basis for the large erythrocytes?
0
• Clinical Case Study 33.2 Answer
0
••· Clinical Case Study 33.5 Diarrhea, dementia and dermatitis (3Os) are typical
features of niacin deficiency, pellagra. Chronic alcoholics
A 38-year-old vegetarian female presented to her doctor
are prone to multiple vitamin deficiencies, which are seen
with fatigue and tingling/numbness in her extremities
in this patient. Fatty liver is also due to chronic alcoholism.
(bilateral). The symptoms were gradually getting worse
Treatment with multivitamins resulted in amelioration of
over the last year. She reported frequent episodes of
diarrhea and weight loss. On examination. she was the symptoms. A balanced diet including all B complex
pale with tachycardia. Her tongue was beefy red. vitamins is needed.
Neurological examination revealed numbness in all Niacin in large doses can also lower LDL cholesterol
extremities with decreased vibration senses. The CBC and triglycerides, and increase HDL cholesterol level.
Chapter 33: Water Solubfe Vitamins 485

e tongue, weight loss and diarrhea which are common for


• • Clinical Case Study 33.3 Answer both folate and 8 12 deficiencies. In addition, neurological
Likely cause of symptoms: Side effects of methotrexate symptoms are common for 8 12 deficiency, these include
(antimetabolite chemotherapy) affecting rapidly dividing numbness, paresthesia, weakness, ataxia, abnormal
cells such as oral mucosa. Folate antagonists inhibit renexes and diminished vibratory sensation. Treatment
includes supplementation of folate and vitamin 8 12 .
dihydrofolate reductase (tetrahydrofolate needed for
purine synthesis).
I LEARNING POINTS, CHAPTER 33
e 1. The coenzyme form of Thiamine is thiamine pyro-
• • Clinical Case Study 33.4 Answer
phosphate (TPP). ,..,, (>'i'IJU.VO.\ c..
Cause of anemia: Folic acid deficiency. 2. TPP is essential for © DH, transketolase, alpha-
Molecular basis of macrocytosis: Abnormal prolifera- ketoglutarate dehydrogenase.
tion of erythroid precursors in the bone marrow, since 3. Deficiency of thiamine leads to beriberi.
folate deficiency encumbers the maturation of these cells 4. Coenzyme forms of riboflavin are FMN and FAD.
by inhibition of deoxyribonucleic acid (DNA) synthesis. 5. Examples of FAD-dependent enzymes are succinate
dehydrogenase and aeyl-CoA dehydrogenase.
e 6. Examples of NAD· dependent enzymes are
• · Clinical Case Study 33.5 Answer lactate deh~drogen~e. glyceraldehyde-3-Q.t,o.fil)bate ..,-Pl"'\n\
Diagnosis: Cobalamin (vitamin 8 12 ) deficiency. dehydrogenase and pyruvate de!}ydrogena.se. ~J>H
Underlying problem: Lack of cobalamin intake. 7. Niacin is synthesized from tryptophan. P.l>-t
Patients with folate deficiency have similar hematologic 8. In Hartnup disease, tryptophan absorption from
and GI findings but do not have the neurologic symptoms intestine is limited. This leads to deficiency of
as with cobalamin deficiency. Treatment consists of tryptophan and consequently of nicotinamide.
identifying/treating the underlying cause of deficiency 9. Active form of pyridoxine is pyridoxal phosphate
and replacement of cobalamin and folate. (PLP).
1O. PLP is essential for transaminatLQ_Q and ..decai:b..Q-
e ~on reactions of amino acids.
• • Clinical Case Study 33.6 Answer 11 . ~ ..§Yliffilise 111.h eme bios · n ~sis is also a PLP
Megaloblastic anemia can be due to different causes, dependent enzyme. Hence anemia is common in
like vitamin 8 12 deficiency, folate deficiency, genetic 8 6 deficiency.
defects in utilization of these vitamins or defects in DNA 12. lsonicotinic acid hydrazide (INH) (isoniazid) used
synthesis. Alcoholics are at particular risk for folate defi- as an antituberculosis drug can produce pyridoxine
ciency because of poor nutrition and defective absorption. deficiency.
Folate is needed for one carbon metabolism, purine 13. Coenzyme A contains pantothenic acid.
and pyrimidine synthesis, DNA and RNA replication, 14. lmportant-CoA derivatives are, a~I-CoA, ~ -(ASH~
methionine synthesis, conversion of serine to glycine, ~of.. HMG-Co~ and 8.C.\d::.C,oA
and in various other transmethylation reactions. 15. Biotin acts as coenzyme for carboxylation reac-
Vegetarian diet without milk and lack of supple- tions. Eg. acetyl-CoA carboxylase, propionyl-CoA
mentation of cobalamin are the common causes for carboxylase, pyruvate carboxylase.
vitamin 8 12 deficiency. Absence of intrinsic factor, perni- 16. Avidin, a protein present in egg white has great
cious anemia, gastrectomy, pancreatic insufficiency, affinity to biotin. Hence intake of raw (unboiled) egg
Helicobacterpylori, fish tapeworm infestation, decreased may cause biotin deficiency.
ileal absorption , Crohn's disease and surgical resection 17. THFA (tetrahydro folic acid) is the carrier of one-
can also produce 8 12 deficiency. carbon groups.
Symptoms are megaloblastic anemia, fatigue, weak- 18. Macrocytic anemia is the most characteristic feature
ness, palpitations, vertigo, tachycardia, sore, beefy-red of folate deficiency.
486 Section D: Nutrition
19. Folate antagonists are Sulphonamides, Trimethoprim, 22. 8 12 deficiency leads to pernicious anemia.
Pyrimethamine, Aminopterin and Amethopterin. 23. Man, higher primates, guineapigs and bats are the
20. Absorption of vitamin B12 requires the intrinsic fa ctor only species which cannot synthesize ascorbic acid
(IF) of Castle. Transcobalamin-11 , a glycoprotein, is (vitamin C).
the specific carrier of vi tamin B 12 • 24. Scurvy is characterized by abnormal collagen,
21 . B12 containing enzymes in the human body are ecchymoses, hemorrhage and anemia.
methylmalonyl-CoA isomerase and homocysteine
methyl transferase.

PART-1 : ESSAY AND SHORT NOTE QUESTIONS

33-1. Describe the source biochemical functions normal requirement and deficiency manifestations of thiam ine.
33-2. Describe the sources, biochemical functions, normal requirement and deficiency manifestat ions of pyridoxal
phosphate.
33-3. Describe sources, biochemical functions, requirement and deficiency manifestations of folic acid.
33-4. Describe sources, biochemical functions, requirement and deficiency manifestations of vitam in B 12.
33-5. Describe sources, biochemical functions, requirement and deficiency manifestations of vitamin C.

SHORT NOTE QUESTIONS

33-6. Functions of thiamine pyrophosphate. 33-15. Avidin.


33-7. Deficiency of thiamine. 33-16. Deficiency of follc acid.
33-8. Beriberi. 33-17. Folate trap.
33-9. Metabolic role of riboflavin . 33-18. Folate antagonists.
33-10. Coenzyme function of niacin. 33-19. Deficiency of vitamin B 12•
33-1 1. Pellagra. 33-20. Biological role of vitamin B 12•
33-12. Functions of pyridoxal phosphate. 33-21. Absorption of vitamin B 12.
33-13. Biotin requiring reactions. 33-22. Intrinsic factor of Castle.
33-14. Coenzyme functions of biotin. 33-23. Methylmalonyl aciduria.

PART-2: MULTIPLE CHOICE QUESTIONS

33-1 . In thiamine deficiency, serum levels of the following 33-5. All the following vitamins are produced from
compound is increased precursors in the human body, except:
A. Glycine B. Pyruvic acid A. Niacin B. Ascorbic acid
C. Glutamic acid D. Transketolase C. Vitamin D D. Vitamin A
33-2. Thiamine deficiency is first manifested as 33-6. Neuritis is a manifestation of the deficiency of all
A. Reduced transketolase activity in RBC the following vitamins, except:
B. Increased glucose-6-phosphate dehydrogenase A. Thiamine B. Vitamin B 12
activity C. Pantolhenic acid D. Vitamin C
C. Absence of aldolase in RBC 33-7. Which vitamin is required for oxi dative decarboxy-
D. Altered lactate dehydrogenase isoenzymes in RBC lation?
33-3. Beriberi is due to the deficiency of A. Pyridoxal phosphate
A. Niacin B. Thiamine B. Thiamine
C. Ribonavin D. Vitamin B,2 C. Biotin
33-4. Daily requirement of vitamin B1 (thiamine) for a D. RiboOavin
normal healthy adult is 33-8. Glossitis is due to the deficiency of
A. 1 microgram B. 5 microgram A. Niacin
C. 100 microgram D. 1 milligram B. Thiamine
Chapter 33: Water Soluble Vitamins 487

C. Riboflavin 33-20. Riboflavin is a constituent of


D. Vitamin 8 12 A. Co-carboxylase 8 . Co-decarboxylase
33-9. All the following are riboflavin (FAD) dependent C. NAO· D. FAD
reactions, except: 33-21. Vitamin B6 is required for
A. Succinate dehydrogenase A. Conversion of glycine to serine
B. Acyl-CoA dehydrogenase B. Production of gamma aminobutyric acid
C. Xanthine oxidase (GABA)
D. Lactate dehydrogenase C. Production of xanthurenic acid
33-10. The water soluble vitamin synthesized in human D. Esterification of palmitic acid
body is 33-22. Pantothenic acid is the coenzyme for
A Ascorbic acid B. Pyridoxine A. Decarboxylation of glutamic acid
C. Niacin D. Pantothenic acid B. Formation of fumarate
33-11 . Pellagra is due to the deficiency of C. Oxidation of pyruvic acid
A. Niacin B. Thiamine D. Methylation of homocysteine
C. Riboflavin D. Vitamin B, 2 33-23. Xanthurenic acid is excreted in urine in the
deficiency of
33-12. All the following conditions can lead to symptoms
A. Felic acid B. Vitamin 8 12
of pellagra, except:
C. Niacin D. Pyridoxal phosphate
A. Low tryptophan content in diet
33-24. Chronic alcoholism or consumption of milled rice
B. Anti-tuberculosis therapy by isoniazid
may result in deficiency of
C. Excessive intake of alcohol
A Vitamin 8 6 B. Vitamin 8 12
D. High leucine content of diet
C. Vitamin K D. Vitamin B,
33-13. Deficiency of vitamin 8 6 Is manifested as
33-25. All are features of pellagra, except:
A. Wet beriberi B. Scurvy A. Skin lesions B. Break down of scars
C. Rickets D. Infantile convulsions C. Diarrhea D. Decreased mental agility
33-14. Daily requirement of pyridoxine for an adult is 33-26. Pyridoxine deficiency will affect the formation of
A. 2 mg 8 . 10 mg all the following , except:
C. 20 mg D. 70 mg A. Gamma aminobutyric acid
33-15. Pyridoxal phosphate is required for all the following B. Glutamine
reactions, except: C. Dopamine
A. Transamination D. Serotonin
B. Deamination 33-27. Transamination reactions require
C. Decarboxylation of amino acids A. Pyridoxal phosphate
D. Darboxylation B. Vitamin 8 12
33-16. Deficiency of pantothenic acid leads to C. Thiamine
A. Night blindness D. Vitamin C
B. Rickets 33-28. All are NAO• dependent enzymes, except:
A. Lactate dehydrogenase
C. Macrocytic anemia
B. Glyceraldehyde-3-phosphate dehydrogenase
D. Burning foot syndrome
C. Pyruvate dehydrogenase
33-17. Biotin is inhibited by
D. Glucose-6-phosphate dehydrogenase
A. lsoniazid (INH) B. Methotrexate
33-29. All are NADP• dependent enzymes, except:
C. Dicoumarol D. Avidin
A. Glucose-6-phosphate dehydrogenase
33-18. Biotin is a coenzyme for all the following carbon B. Cytoplasmic isocitrate dehydrogenase
dioxide fixation reactions , except: C. 6-phosphogluconate de hydrogenase
A. Pyruvate carboxylase D. Glycerophosphate dehydrogenase
B. Acetyl-CoA ca rboxylase 33-30. Which of the following vitamins is required for
C. Propionate carboxylase oxidative decarboxylation?
D. Carbamoyl phosphate synthetase A. Pyridoxal phosphate
33-19. Allcompounds act as antivitamin, except: B. Thiamine pyrophosphate
A. Avid in B. Menadione C. Biotin
C. INH D. Methotrexate D. Ribofl avin
488 Section D: Nutrition

33-31. Folic acid is inhibited by 33-44. Megaloblastic anemia is due to deficiency of


A. lsoniazid (INH) B. Methotrexate A. Niacin B. Thiamine
C . Dicoumarol D. Avidin C. Riboflavin D. Vitamin 8 12
33-32. Daily requirement of fol ic acid is 33-45. Which acts as a strong reducing agent?
A. 1 microgram B. 5 microgram A. Folic acid B. Tocopherol
C. 100 microgram D. 750 microgram C. Ascorbic acid D. Pantothenic acid
33-33. Deficiency of folic acid leads to 33-46. A patient with vitamin C deficiency will show all the
A. Night blindness B. Rickets following cli nical features, except
C. Macrocytic anemia D. Microcytic anemia A . Bone pains
33-34. In vitamin B12 deficiency, which compound is 8 . Delayed wound healing
excreted in urine? C. Subcutaneous hemorrhages
A. FIGLU (formiminoglutamic acid) D. Megaloblastic anemia
B. Homocysteine 33-47. A patient who has undergone gastrectomy is likely
C. Xanthurenic acid to develop deficiency of
D. Methyl malonic acid A. Vitamin A B. Vitamin B 12
33-35. Formiminoglutamic acid (FIGLU) is excreted in C. Vitamin B, D. Vitamin K
urine in the deficiency of 33-48. Absorption of vitamin B12 takes place from
A. Folic acid B. Vitamin 8 , 2 A. Stomach B. Duodenum
C. Niacin D. Pyridoxal phosphate C. Jejunum D. Ileum
33-36. Daily requirement of vitamin B 12 for a normal 33-49. In human body, highest concentration of ascorbic
healthy adult is acid is found i n
A. 1 microgram B. 5 microgram A . Liver
C. 100 microgram D. 750 microgram B. Adrenal cortex
33-37. The metal present in vitamin B12 is C. Posterior pituitary gland
A. Copper B. Cobalt D. Muscle
C. Chromium D. Manganese 33-50. Pernicious anemia is characterized by all the follo-
33-38. Daily requirement of ascorbic acid for an adult wing, except
A. 2 mg 8 . 10 mg A . Histamine-fast achlorhydria
C. 20 mg D. 70 mg 8 . Megaloblastic anemia
33-39. Scurvy is due to the deficiency of C. Subacute combined degeneration of spinal cord
A. Niacin 8. Thiamine D. Response to oral administration of folic acid
C. Riboflavin D. Vitamin C 33-51 . Riboflavin is needed as a coenzyme (FAD) in the
33-40. Pernicious anemia is caused by reaction :
A. Inadequate intake of vitamin B 1 A. Glyceraldehyde-3-phosphate dehydrogenase
B. Absence of hydrochloric acid in gastric juice B. Glucose-6-phosphate dehydrogenase
C. Absence of intrinsic factor of the gastric juice C. Fatty acyl-CoA dehydrogenase
D. Over production of extrinsic factor D. Malate dehydrogenase
33-41 . One of the features listed below is not true
33-52. The vitamin totally absent in plant source is
regarding pernicious anemia
A. Thiamine
A. Neurological manifestations
B. Vitamin 8 12
B. Megaloblastic anemia
C. Riboflavin
C. Increased gastrin secretion
D. Vitamin K
D. Decreased basal acid output
33-53. All substances contain sulphur, except
33-42. Methotrexate is acting by inhibition of
A. Thiamine B. Tocopherol
A. Cellular oxidation 8 . RNA synthesis
C. Biotin D. Taurine
C. Cyanocobalamine D. Folate reductase
33-43. The structure of vitamin B12 contains 33-54. Folic acid is required for synthesis of
A. Pyridine ring B. Corrin ring A. ATP B. TTP
C. lmidazole ring D. Pteridine ring C. GTP D. CTP
Chapter 33: Water Soluble Vitamins 489

33-55. All manifestations are seen in Vitam in 8 (pyrido- 33-56. Cystathionuria may be seen in the deficiency of all
6
xal phosphate) deficiency, except: the following, except:
A. Skin rashes as in pellagra A. Vitamin B 12
·- B. Macrocytic anemia B. Folic acid
C. Peripheral neuritis C. Pyridoxal phosphate
D. Convulsions in children D. Vitamin K

ANSWERS OF MULTIPLE CHOICE QUESTIONS


33-1 . B 33-2. A 33-3. B 33-4. D 33-5. B 33-6. D 33-7. B
33-8. C 33-9. D 33-10. C 33-11 . A 33-12. C 33-13. D 33-14. A
33-15. D 33-16. D 33-17. D 33-18. D 33-19. B 33-20. D 33-21 . B
33-22. C 33-23. D 33-24. D 33-25. B 33-26. B 33-27. A 33-28. D
33-29. D 33-30. B 33-31 . B 33-32. C 33-33. C 33-34. D 33-35. A
33-36. A 33-37. B 33-38. D 33-39. D 33-40. C 33-41 . C 33-42. D
33-43. B 33-44. D 33-45. C 33-46. D 33-47. B 33-48. D 33-49. B
33-50. D 33-51. C 33-52. B 33-53. B 33-54. D 33-55. B 33-56. D

PART-3: VIVA VOCE QUESTIONS AND ANSWERS

33-1 . What is the source of thiamine? 33-11 . What are the dietary sources of riboflavin?
Aleurone layer of cereals (food grains) is a rich source Rich sources are liver, d ried yeast, egg, and milk.
of thiamine. Whole wheat flour and unpolished rice 33-12. What is the daily requirement of riboflavin?
and yeast are other good sources. 1.5 mg per day.
33-2. Thiam ine pyrophosphate is required for? 33-13. Which vitamin is synthesized in the body?
Transketolase; Pyruvate dehydrogenase; Alpha ketog- Niacin.
lutarate dehydrogenase. 33-14. What are the coenzyme function of niacin?
33-3. Which vitamin is required for oxidative decarbo- NAO• and NADP•
xylation? 33-15. Name some NAO• dependent enzymes.
Thiamine pyrophosphate. Lactate dehydrogenase; Glyceraldehyde-3-phosphate
dehydrogenase; Pyruvate dehydrogenase; Beta
33-4. In thiamine deficiency, what alterations are seen in
hydroxyacyl-CoA dehydrogenase; Mitochondrial
the blood?
isocitrate dehydrogenase.
Increased pyruvic acid level and increased trans-
33-16. Name the NADPH generating reactions.
ketolase.
Glucose-6-phosphate dehydrogenase; 6-phospho-
33-5. What are the c linical manifestations of thiamine
gluconate dehydrogenase.
deficiency?
33-17. What are important NADPH utilizing reactions?
Beriberi, Wernick syndrome. polyneuritis.
Beta keto acyl ACP dehydrogenase; Alpha, beta unsa-
33-6. Beriberi is due to deficiency of which vitam in?
turated ACP dehydrogenase; HMG-CoA reductase;
Thiamine.
Met-hemoglobin reductase; Dihydrofolate reductase
33-7. What is the daily requirement of thiamine? (Box 33.5).
1 to 1.5 milligram. 33 -1 8. Pellagra is seen in deficiency of which vitam in?
33-8. What is the coenzyme function of riboflavin? Niacin.
FMN and FAD. 33-19. What are the salient features of pellagra ?
33-9. Name some FAD dependent enzymes. Dermatitis, diarrhea, dementia.
Succinate dehydrogenase; Acyl-CoA dehydrogenase; 33-20. What is the precursor of niacin?
Xanthine oxidase; Glutathione reductase; Pyruvate Tryptophan.
dehydrogenase; Alpha-ketoglutarate dehydrogenase. 33-21. Tryptophan will give rise to how much niacin?
33-10. What are the manifestations of riboflavin deficiency? About 60 mg of tryptophan will yield 1 mg of niacin.
Glossitis, cheilosis, angular stomatits, circumcorneal 33-22. Tryptophan is deficient in which food stuff?
vascularization. Maize and corn.
490 Section D: Nutrition

33-23. What are the conditions that will lead to symptoms 33-37. Deficiency of pantothenic acid leads to what?
of pellagra? Burning foot syndrome.
lsoniazid therapy; Low tryptophan content in diet; Niacin 33-38. What is the function of biotin?
deficiency; Hartnup disease; Carcinoid syndrome. Carboxylation reactions.
33-24. What is the dietary sources of niacin? 33-39. Name biotin dependent carboxylation reactions
Rice polishing, cereals, legumes, meat and fish Acetyl-CoA carboxylase; Propionyl-CoA carboxylase;
contain niacin. About half of the requirement is met by Pyruvate carboxylase.
the conversion of tryptophan to niacin. 33-40. What is the antagonist for Biotin?
33-25. What is the dally requirement of niacin? Avidin.
The R.D.A is 20 mg/day. 33-41 . What is the coenzyme form of folic acid?
33-26. Transamination reaction requires which vitamin? Tetrahydrofolic acid.
Pyridoxal phosphate. 33-42. What is the main function of folic acid?
33-27. Pyridoxal phosphate is required for what reactions? Tetrahydrofolic acid is the carrier of one-carbon units.
Transamination, decarboxylation of amino acids, ALA 33-43. What are the causes of folate deficiency?
synthase, glycogen phosphorylase. Pregnancy; defective absorption; anticonvulsant drugs
33-28. What is clinical significance of transaminase?
(dilantin, phenobarbitone); hemolytic anemias, dietary
Blood level of ALT is increased in liver diseases.
deficiency.
33-29. Give examples of decarboxylation reactions.
33-44. What is the major manifestation of fol ic acid
Glutamate to GABA (gamma aminobutyric acid); Histi-
deficiency?
dine to histamine; 5-hydroxytryptophan to serotonin;
Macrocytic anemia.
Serine to ethanol amine.
33-45. What is the complication of folic acid deficiency in
33-30. What are the manifestations of pyridoxal deficiency?
pregnancy?
Infantile convulsions; peripheral neuritis; pellagra;
Felic acid deficiency during pregnancy may lead to
anemia.
neural tube defects in the fetus.
33-31 . What is the reason for peripheral neuritis in
33-46. What is the daily requirement of folic acid?
..
pyridoxal deficiency?
The RDA of folate is 200 mg/day. In pregnancy the
PLP is involved in the synthesis of sphingolipids; so
B6 deficiency leads to demyelination of nerves and requirement is increased to 400 mg/day.
consequent peripheral neuritis. 33-47. What is the mechanism of action of sulphonamides?
33-32. What is the reason for pellagra like disease in They have structural similarity with PABA. Therefore
pyridoxal deficiency? they competitively inhibit the enzyme responsible for
Niacin is produced from tryptophan. One enzyme of the incorporation of PABA into folic acid. So, they kill
this pathway is PLP dependent. So B6 deficiency in turn bacteria.
leads to niacin deficiency. It is manifested as pellagra. 33-48. What is mechanism of action of methotrexate?
33-33. Give an example of one vitamin deficiency leading It inhibits folate reductase, and is a powerful anticancer
to another vitamin deficiency? drug.
PLP deficiency in turn leads to niacin deficiency which 33-49. What are inhibitors of folic acid?
is manifested as pellagra. Methotrexate; Pyrimethamine; Sulphonamide.
33-34. What is the daily requirement of pyridoxal 33-50. Name a water soluble vitamin , which Is stored in
phosphate? the body.
1 to 2 mg/day. Vitamin B12.
33-35. What is coenzyme form of pantothenic acid? 33-51. What is the metal present in vitamin B12 ?
CoenzymeA. Cobalt.
33-36. What is the function of CoA? 33-52. How vitamin B 12 Is absorbed?
Oxidation of pyruvic acid and activation of acyl groups. With the help of Intrinsic factor of Castle.
i - - - - - - - -_ _ Chapter 34
'!..

Mineral Metabolism
and Abnormalities

Chapter at a Glance

The learner will be able to answer questions on the following topics:


0 Calcium, availability and functions O Copper
Factors regulating blood calcium level O Zinc
0 Calcium, clinical applications O Fluoride
0 Phosphorus Selenium
Magnesium Manganese
Sulfur O Lithium
0 Iron absorption, transport, deficiency

Minerals are essential for the normal growth and main- BOX 34.1: Important minerals
tenance of the body. If the daily requirement is more Major elements Trace elements
than 100 mg, they are called major elements or macro- 1. Calcium 1. Iron
minerals. They are listed in Box 34.1. 2. Magnesium 2. Iodine
If the requirement of certain minerals is less than 3. Phosphorus 3. Copper
100 mg/day, they are known as minor elements or micro- 4. Sodium 4. Manganese
minerals or t race elements. They are shown in see 5. Potassium 5. Zinc
Box 34.1, in order of their essentiality: 6. Chloride 6. Molybdenum

The following minerals are necessary for the body; 7. Sulfur 7. Selenium
8. Fluoride
but their exact functions are not known . They are chro-
mium, nickel, bromine, lithium and barium. The following
minerals are seen in tissues, but are nonessential and vegetables are medium sources for calcium. Cereals
are contaminants in foodstuffs. These are rubidium, silver, (wheat, rice) contain only small amount of calcium. But
gold, and bismuth. The following minerals are toxic cereals are the staple diet in India. Therefore, cereals
and should be avoided: aluminium, lead, cadmium and form the major source of calcium in Indian diet.
mercury. Heavy metal poisons are discussed in Chapter 37.

I CALCIUM (Ca Daily Requirement of Calcium


Total calcium in the human body is about 1-1 .5 kg, 99% An adult needs 500 mg per day and a child about
of which is seen in bone and 1% in extracellular fluid. 1200 mg/day. Requirement may be increased to
1500 mg/day during pregnancy and lactation. After
Sources of Calcium the age of 50, there is a general tendency for osteo-
Milk is a good source for calcium. Calcium content porosis, which may be prevented by increased calcium
of cow's milk is about 100 mg/100 ml. Egg, fish and (1500 mg/day) plus vitamin D (20 mg/day).
492 Section D: Nutrition

Blood Intestinal cell Lumen Ca++ ... Calmodulin

CR +
Ca-bound calmodulin

Kinase
+ .
--------. Active kinase

+
Enzyme ---------------• Phosphorylated enzyme

l +
Biological effect
""""""" CB protejn
Fig. 34.2: Mechanism of action of calmodulin

ratio of calcium to phosphorus which allows maxi-


Fig. 34.1: Calcitriol increases calcium absorption
mum absorption is 1 :2-2:1 as present in milk.
(C= Calcitriol; CR= Calcilriol receptor complex; CB= Calbindin).
Calcium in Cells
Absorption Calcium is present both in the extracellular and intracel-
lular compartments. However, it is mainly extracellular.
Mechanism of Absorption of Calcium
The cell membrane is generally impermeable to
Absorption is taking place from the first and second part calcium ions. Calcium influx into the cell is by Na•/Ca..
of duodenum. Calcium is absorbed against a concen- exchange mechanism. This mechanism is rapid, but has
tration gradient and requires energy. Absorption requires low affinity for calcium.
a carrier protein, helped by calcium-dependent ATPase. Entry of Ca.. into mitochondria is by a calcium uniport
Out of the 500 mg of calcium taken orally per day, system. But calcium ions exit by a Na•-ca.. antiport
400 mg is excreted in stool and 100 mg is excreted system, which in turn is dependent on the Na•-H+-
through urine. ATPase pump. For calcium channels, see Chapter 2.

Factors Causing Increased Absorption Functions of Calcium


i. Vitamin D: Calcitriol induces the synthesis of the Activation of Enzymes
carrier protein (Calbindin) in the intestinal epithe-
Calmodulin is a calcium binding regulatory protein.
lial cells, and so facilitates the absorption of calcium
Calmodulin can bind with 4 calcium ions. Calcium bind-
(Fig. 34.1 ).
ing leads to activation of enzymes. Calmodulin is part
ii. Parathyroid hormone: It increases calcium trans-
of various regulatory kinases. The mechanism of action
port from the intestinal cells by enhancing 1a hydro-
is summarized in Figure 34.2. Calmodulin dependent
xylase activity.
enzymes are listed in Box 34.2.
iii. Acidity favors calcium absorption.
Some other enzymes are activated directly by Ca..
iv. Amino acids : Lysine and arginine increase cal-
without the intervention of calmodulin; examples are
cium absorption.
pancreatic lipase; enzymes of coagulation pathway; and
rennin (milk clotting enzyme in stomach).
Factors Causing Decreased Absorption
i. Phytic acid : Hexaphosphate of inositol is present Muscles
in cereals. Fermentation and cooking reduce phy- Calcium mediates excitation and contraction of mus-
tate content. cle fibers. Upon getting the neural signal, calcium is
ii. Oxalates: They are present in some leafy vegeta- released from sarcoplasmic reticulum. Calcium activa-
bles, which cause formation of insoluble calcium tes ATPase; increases action of actin and myosin and
oxalates. faci litates excitation-contraction coupling. The trigger of
iii. Malabsorption syndromes: Fatty acid is not muscle contraction is the interaction of calcium with Tro-
absorbed, causing formation of insoluble calcium ponin C (see Chapter 49). The active transport system
salt of fatty acid. utilizing calcium binding protein is called calsequestrin.
iv. Phosphate: High phosphate content will cause Calcium decreases neuromuscular irritability. Calcium
precipitation as calcium phosphate. The optimum deficiency causes tetany.
Chapter 34: Mineral Metabolism and Abnormalities 493

BOX 34.2: Selected list of enzymes regulated by ca· · and


mediated by calmodulin
Ionized form 5 mg/dL
Adenylyl cyclase
c a- dependent protein kinases (PKC) Complexec:l with anions 1 mg/dL
ca~ -Mg++ -ATPase
Glycerol-3-phosphate dehydrogenase Bound with proteins 4 mg/dL
Glycogen synthase
Myosin kinase Fig . 34.3: Differenit forms of calcium in serum
Phospholipase C
Phosphorylase kinase
Pyruvate carboxylase
: . . .
1tomn and calc1tnol are different
Calcitonin is t he peptide hormone released from thyroid gland.
Pyruvate dehydrogenase It decreases blood calcium level.
Pyruvate kinase
Calcitriol ls the active form of vitamin D. It increases the blood
calcium.
Nerve Conduction
Calcium is necessary for transmission of nerve impulses body. Osteoblasts induce bone deposition and osteo-
from presynaptic to postsynaptic region. clasts produce demineralization.

Secretion of Hormones Calcium in Blood


Calcium mediates secretion of Insulin, parathyroid hor- i. Normal bl,ood level : Normal calcium level is 9-1 1
mone, calcitonin, vasopressin, etc. from the cells. mg/dl. (10 mg/dl of ca•· = 5 mEq/L).
ii. Ionized ca1lcium: About 5 mg/dl of calcium is in
Second Messenger ionized form and is metabolically active (Fig. 34.3).
Calcium and cyclic AMP are second messengers of Another 1 rng/dl is complexed with phosphate, bic-
different hormones (see Table 45. 1). One example is arbonate and citrate. These two forms are diffusible
glucagon. Calcium is used as second messenger in sys- from blood to tissues.
tems involving G proteins and inositol triphosphate. iii. Protein bo,und calcium: About 4 mg/dl of calcium
is bound to, proteins in blood and is nondiffusible.
Vascular Permeability
Factors Re!gulating Blood
Calcium decreases the passage of serum through capil-
laries. Thus, calcium is clinically used to reduce allergic Calcium Level
exudates. There are effective controls to maintain this narrow
range of blood calcium (9- 11 mg/dL).
Coagulation
Calcium is known as factor IV in blood coagulation cas- Vitamin D
cade. Prothrombin contains gamma-carboxyglutamate i. Causes for vitamin D deficiency and clinical featu-
residues which are chelated by Ca++ during the thrombin res of rickets are described in detail in Chapter 32.
formation (see Chapter 32, under vitamin K). Cholecalci'ferol is synthesized from 7-dehydro-
cholesterol in skin, under the influence of sunlight.
Myocardium
It is then hlydroxylated at 25th position in liver and
ca- prolongs systole. In hypercalcemia, cardiac arrest further hyclroxylated at the 1st position in kidney.
is seen in systole. This fact should be kept in m ind when The active derivative is called dihydroxycholecalcif-
calcium is administered intravenously. It should be given erol or calc::itriol. Calcitriol and calcitonin are differ-
very slowly. ent (Box 34.3).
ii. Vitamin D and absorption of calcium: Calcitriol
Bone and Teeth promotes 1the absorption of calcium and phospho-
The bulk quantity of calcium is used for bone and teeth rus from the intestine. Calcitriol enters the intesti-
formation. Bones also act as reservoir for calcium in the nal cell and binds to vitamin D receptor (VDR). The
494 Section D: Nutrition

Mechanism of Action of PTH


Parathyroid glands PTH binds with a receptor protein on the surface of
target cells. This activates adenyl cyclase with conse-
IIntestine I quent increase in enzyme systems. The PTH has three
Calcium
major independent sites of action; bone, kidney and
intestines. All the three actions of PTH increase serum
Bone calcium level.
resorptior}
PTH and bones: In the bone, PTH causes deminera-
Increased
lization · or decalcification (see Fig. 34.4). The number
calcium
Bone mineralization
of osteoclasts are also increased. Osteoclasts release
absorption

/
lactate into surrounding medium which solubilizes cal-
t cium. Parathyroid hormone (PTH) also causes secre-
Calbindin Decrease urinary Calcitriol
calcium tion of collagenase from osteoclasts. This causes loss
Increase urinary phosphate
of matrix and bone resorption.
Fig. 34.4: Calcium homeostasis When serum calcium is low, PTH PTH and kidney : In kidney, PTH causes decreased
is stimulated, resulting in increased calcium release from bone renal excretion of calcium and increased excretion
and decreased renal calcium excretion. PTH also stimulates of phosphates. The action is mainly through increase in
increased production of calcitriol, which acts to increase absorption
reabsorption of calcium from kidney tubules.
of calcium from intestine
PTH and intestines: Parathyroid hormone stimu-
hormone-receptor complex interacts with DNA and lates 1-hydroxylation of 25-hydroxycalciferol in kidney
causes transcription of specific genes that code to produce calcitriol. This indirectly increases calcium
absorption from intestine.
for Calbindin (see Fig. 34.1 ). Due to the increased
availability of calcium binding protein, the absorp- Calcitonin
tion of calcium is increased. Hence, blood calcium
level tends to be elevated. It is secreted by the thyroid parafollicular or clear cells.
Calcitonin and Calcitriol are different (Box 34.3). Calci-
iii. Vitamin D and bone: Vitamin D is acting indepen-
tonin is a polypeptide with 32-34 amino acids, depend-
dently on bone. Vitamin D increases the number and
ing on the species difference. Calcitonin secretion is
activity of osteoblasts, the bone forming cells. It also
stimulated by serum calcium. Calcitonin level is increa-
has a role in osteoclastogenesis. Calcitriol stimulates sed in medullary carcinoma of thyroid and therefore is
osteoblasts to secrete alkaline phosphatase. Due a tumor marker. Calcitonin decreases serum calcium
to this enzyme, the local concentration of phosphate level. It inhibits resorption of bone. It decreases the
is increased. The ionic product of calcium and phos- activity of osteoclasts and increases that of osteoblasts.
phorus increases, leading to mineralization and Calcitonin and PTH are directly antagonistic. The PTH
remodeling of bone (Fig. 34.4). and calcitonin together promote the bone growth and
iv. Vitamin D and renal tubules: Calcitriol increases remodeling (see Table 34. 1).
the reabsorption of calcium and phosphorus by renal
Calcitonin, Calcitriol and PTH Act Together
tubules, therefore both minerals are conserved.
(PTH conserves only calcium). When blood calcium tends to lower, PTH secretion is
stimulated and calcitonin is inhibited; bone deminerali-
Parathyroid Hormone (PTH) zation leads to entry of more calcium into blood. When
blood calcium is increased, PTH is inhibited and cal-
This hormone is secreted by the four parathyroid glands
citonin is secreted, causing more entry of calcium into
embedded in the thyroid tissue. The chief cells of the bone. These effects are summarized in Figure 34.4 and
gland secrete the PTH. Table 34.1 . Bone acts as the major reservoir of calcium.
PTH has 84 amino acids. The first 35 amino acids
of PTH are biologically active. Control of release of the Procalcitonin (PCT)
hormone is by negative feedback by the ionized calcium It is a relatively new marker that has been associated
in serum. The release of hormone is mediated by cyclic with inflammation and sepsis. It is the precursor to calci-
AMP. tonin. The levels have been shown to rise with severity
Chapter 34: Mineral Metabolism and Abnormalities 495

TABLE 34.1: Comparison of action of the three ma1or factors


affecting serum calcium
:
1.
. toms of hypercnlcem,a
Anorexia, nau•sea, vomiting
ViraminD PTH Calcironin
2. Polyuria and polydipsia (ADH antagonism)
Blood calcium Increased Drastically Decreased
3. Confusion, depression, psychosis
increased
4. Renal stones
Main action Absorption Demineralization Opposes
from gut demineralization 5. Ectopic calcification and pancreatitis
Calcium Increased Increased 6. Blood alkaline phosphatase is increased.
absorption (indirect)
from gut
Bone Decreased Increased Decreased is reduced with lowering of each g/dl of albumin. In such
resorption cases, the metabolically active ionized form is normal,
Deficiency Rickets Tetany and so there will be no deficiency manifestations.
manifestation
Use in Drug of Contraindicated Theoretically Alkalosis anci Acidosis
Rickets choice beneficial
Alkalosis favors binding of more calcium with proteins,
Effect of Hypercal- Hypercalcemia++ Hypocalcemia
excess cemia+
with consequent lowering of ionized calcium. Here total
calcium level is normal, but calcium deficiency may be
manifested. Acidosis favors ionization of calcium.
BOX 34.4: Causes of hypercalcem1a
1. Hyperparathyroidism
Hypercalcemia
2. Multiple myeloma
3. Paget's disease The term denotes that the blood calcium level is more
4. Metastatic carcinoma of bone than 11 mg/dl. Tt1e major cause is hyperparathyroidism.
5. Thyrotoxicosis, Addison's disease This may be due to a parathyroid adenoma or an
6. Prolonged immobilization ectopic PTH secreting tumor. Important causes of hyper-
7. Milk-alkali syndrome calcemia are enumerated in Box 34.4. There is osteo-
8. Drugs
porosis and X-ray shows punched out areas of bone
Excess vitamin Dor vitamin A
resorption. Pathological fracture of bone may result.
Excess calcium given IV
Box 34.5 gives major symptoms of hypercalcemia.
In the blood, ,calcium and alkaline phosphatase levels
of sepsis. PCT is markedly elevated in acute bacterial are increased, while phosphate level is lowered. In
infections; but not in viral infections. urine, calcium is excreted, which may cause inhibition of
elimination of chloride. This may lead to hyperchloremic
Phosphorus acidosis. Calcium may be precipitated in urine, leading
There is a reciprocal relationship of calcium with phos- to recurrent bilateral urinary calculi. Ectopic calcification
phorus. The ionic product of calcium and phosphorus in may be seen in renal tissue, pancreas (pancreatitis),
serum is kept as a constant. (In normal adults, calcium arterial walls, anid muscle tissues (myositis ossificans).
= 10 mg/dl x phosphorus 4 mg/dl; so ionic product is In multiple myelloma, Paget's disease and metastatic
40). In renal insufficiency, phosphorus excretion is dimi- carcinoma of bone, there will be bone resorption and
nished; phosphorus level in blood is increased and cal- hypercalcemia.
cium level is lowered. This may lead to tetany.
Hypocalcemia
Children
Tetany
In children , the calcium level tends to be near the upper
limit. In children, ionic product of calcium and phospho- Causes of hypocalcemia are enumerated in Box 34.6.
rus in blood is about 50 (instead of 40 in normal adults). When serum callcium level is less than 8.8 mg/dl, it is
hypocalcemia. If serum calcium level is less than 8.5
Serum Proteins mg/dl, there will be mild tremors. If it is lower than 7.5
In hypoalbuminemia (e.g. nephrosis, malnutrition), the mg/dl, tetany, a life-threatening condition will result.
total calcium is decreased. About 0.8 mg/dl of calcium Symptoms of hypocalcemia are enumerated in Box 34.7.
496 Section D: Nutrition

BOX 34.6 : Causes of hypocalcem1a


1. Deficiency of vitamin D
:
1.
.
Muscle cramps
toms of hypocalcem1a

Decreased exposure to sunlight 2. Paresthesia, especially in fingers


Malabsorption, dietary deficiency 3. Neuromuscular i rritability, m uscle twitchings
Hepatic diseases 4. Tetany (Chvostek's sign, Trousseau's sign)
Decreased renal synthesis of calcitriol 5. Seizures
2. Deficiency of parathyroid 6. Bradycardia
Hypoparathyroidism (primary, secondary)
7. Prolonged QT interval
3. Increased calcitonin
Medullary carcinoma of thyroid
4. Deficiency of calcium
Intestinal malabsorption
Alkalosis decreasing ionized calcium
5. Deficiency of magnesium
6. Increase in phosphorus level
Renal failure
7. Hypoalbuminemia

Tetany may be due to accidental surgical removal


of parathyroid glands or by autoimmune diseases. In Fig. 34.5: Carpopedal spasm in tetany
tetany, neuromuscular irritability is increased. Main mani-
festation is carpopedal spasm (Fig. 34.5). Laryngismus phosphate from substrates. So, the ionic concentration
and stridor are associated findings. Laryngeal spasm of [calcium x phosphate] is increased to supersaturation
may lead to death. Chvostek's sign (tapping over facial level. Calcium phosphate is deposited as hydroxyapa-
nerve causes facial contraction) will be positive.
tite crystals ovm the matrix of triple stranded quarter
Trousseau's sign (inflation of BP cuff for 3 minutes
staggered colla£Ien molecules.
causes carpopedal spasm) could be elicited. Increased
Q-T interval in ECG is seen. Serum calcium is lowered
Osteoporosis
with corresponding increase in phosphate level. Urinary
excretion of both calcium and phosphate are decreased. After the age of 40-45, calcium absorption is reduced
Treatment is to give intravenous injection of calcium and calcium e>ccretion is increased; so, there is a
salts. It should be emphasized that vitamin D deficiency net negative bailance for calcium. This is reflected in
will not cause tetany. The vitamin D deficiency causes demineralization. After the age of 60, osteoporosis
rickets, where serum calcium level is lowered marginally. is seen. Then there is reduced bone strength and an
increased risk of fractures. Decreased absorption of
Bone Mineralization and Demineralization vitamin D and reduced levels of androgens/estrogens
Bone is a specialized connective tissue made up of a in old age are the causative factors.
matrix with embedded fibers, cells and apatite crystals.
See Box 34.8 for the requirement for bone production . ~ HOSPHORUS
Mineralization of bone: It is the process by which Total body phosphate is about 1 kg; 80% of which is
inorganic calcium and phosphate are deposited on the seen in bone and teeth and 10% in muscles. Phosphate
organic matrix. The specialized matrix in bone is termed is mainly an intracellular ion and is seen in all cells. Func-
Osteoid. Osteocalcin is a unique protein seen in bone.
tions of phospha1te ions are enumerated in Box 34.9.
The osteoblasts synthesize and secrete organic matrix,
which is then mineralized. Osteoclasts are involved in
Requirement and Source
bone resorption. Combined activities of osteoblasts and
osteoclasts are important in bone remodeling . The Requirement is about 500 mg/day. Milk is a good source,
osteoblasts are under the effect of hormones PTH and which contains about 100 mg/dl phosphate. Cereals,
calcitriol. Secretion of alkaline phosphatase by osteo- nuts and meat aire moderate sources. Calcitriol increa-
blasts is increased by vitamin D. The enzyme liberates ses phosphate aibsorption.
Chapter 34: Mineral Metabolism and Abnormalities 497

BOX 34.8: Requirements for growth of bone BOX 34.9: Functions of phosphate ions
1. Calcium 1. Formation of bone and teeth
2. Phosphorus 2. Production of high energy phosphate. compounds, such as
3. Vitamin D and Calcitriol ATP, CTP, GTP, creatine phosphate, etc.
4. Parathyroid hormone 3. Synthesis of nucleoside coenzymes, such as NAD and NADP
5. Calcitonin 4. DNA and RNA synthesis, where phosphodiester linkages
6. Vitamin A (for ground substance) form the backbone of the structure
7. Vitamin C 5. Formation of phosphate esters, such as glucose-6-phos-
phate, phospholipids
8. Sex steroids
9. Amino acids. 6. Formation of phosphoproteins, e.g. casein
7. Activation of enzymes by phosphorylation
8. Phosphate buffer system in blood. The ratio of Na2 HPO4:
BOX 34.10: Causes of hyperphosphatem1a NaH2 PO4 in blood is4:1. This maintains the pH of blood at 7.4.
1. Increased absorption of phosphate
Excess vitamin D
BOX 34.11 : Causes of hypophosphatem1a
2. Increased cell lysis
1. Decreased absorption of phosphate
Chemotherapy for cancer
Malnutrition
Bone secondaries
Malabsorption
3. Decreased excretion of phosphorus
Chronic diarrhea
Renal impairment
Vitamin D deficiency
Hypoparathyroidism 2. Intracellular shift
4. Hypocalcemia Insulin therapy, glucose phosphorylation
5. Massive blood transfusions Respiratory alkalosis
6. Drugs 3. Increased urinary excretion of phosphate
Chlorothiazide, Nifedipine, Furosemide. Hyperparathyroidism
4. Hereditary hypophosphatemia
5. Hypercalcemia
Serum Level of Phosphorus 6. Chronic alcoholism
7. Drugs
Serum level of phosphate is 3--4 mg/dl in normal adults Antacids, diuretics, salicylate intoxication.
and is 5-6 mg/dl in children. Fasting levels are higher.
The postprandial decrease of phosphorus is due to the
utilization of phosphate for metabolism. Monovalent and I MAGNESIUM (Mg)
divalent phosphate ions are present in plasma at a ratio Magnesium is the fourth most abundant cation in the
of 1:4 at pH 7.4. body and second most prevalent intracellular cation.
The whole blood phosphate is 40 mg/dl. This is Total body magnesium is about 25 g, 60% of which is
because RBCs and WBCs contain a lot of phosphates. complexed with calcium in bone. Magnesium orally
Hemolysis should be prevented when blood is taken for produces diarrhea; but intravenously it produces CNS
phosphate estimation . depression.
Serum phosphate level is decreased in hyper-
parathyroidism and rickets. Causes of hyperphosphatemia Requirement
are given in Box 34.10, and those of hypophosphatemia
The requirement is about 400 mg/day for men and
inBox34.11 .
300 mg/day for women. Major sources are cereals,
Phosphorus holds an inverse relationship with cal-
beans, leafy vegetables and fish.
cium. An excess of serum calcium or phosphate results
in the excretion of the other by the kidney.
Normal Serum Level of Magnesium
Like calcium, phosphate level in blood is controlled
by the parathyroid hormone. Parathyroid hormone inc- Normal serum level Mg++ is 1.8-2.2 mg/dl. Homeostasis
reases calcium and phosphate release from the bone is maintained by intestinal absorption as well as by
and decreases loss of calcium and increases loss of excretion by kidney. Magnesium is reabsorbed from
phosphate in the urine. loop of Henle and not from proximal tubules.
498 Section D: Nutrition

BOX 34.12: C,1uses of hypornagneserrna BOX 34.13: Causes of l1yperrm1911esem1cJ


1. Increased urinary loss (tubular necrosis) 1. Excess intake orally or parenterally
2. Hyperaldosteronism, volume expansion 2. Renal failure
3. Familial hypomagnesemia 3. Hyperparathyroidism
4. Increased intestinal loss 4. Multiple myeloma
Diarrhea, laxatives, ulcerative colitis
Nasogastric suction, vomiting
5. Protein calorie malnutrition units together, e.g. insulin, immunoglobulins. Chondroi-
6. Hypoparathyroidism tin sulfates are seen in cartilage and bone. Keratin is rich
7. Drugs: Thiazide diuretics in sulfur, and is present in hair and nail. Many enzymes
and peptides contain -SH groups at the active site, e.g.
Functions of Magnesium glutathione. Thiamine, biotin, lipoic acid, CoA are also
1. MgH is the activator of many enzymes requiring -SH containing molecules. If sulfate is to be introduced
ATP. Alkaline phosphatase, hexokinase, fructo- in glycosaminoglycans or in phenols for detoxification,
kinase, phosphofructokinase, adenylyl cyclase, cAMP it can be done only by phosphoadenosine phosphosul-
dependent kinases, etc. need magnesium. fate (PAPS). Sulfates are also important in detoxification
2. Neuromuscular irritability is lowered by magnesium. mechanisms, e.g. production of indoxyl sulfate.

Hypomagnesemia Excretion
It is commonly seen in hospitalized patients. When serum All the sulfur groups are ultimately oxidized in liver to
magnesium level falls below 1.7 mg/dl, it is called sulfate (SO4 ) group and excreted in urine. The total
hypomagnesemia. Causes are shown in Box 34.12.
quantity of sulfur in urine is about 1g/day. This contains
Deficiency of magnesium leads to neuromuscular 3 categories.
hyper-irritability and cardiac arrhythemias. The mag-
i. Inorganic sulfates: It is about 80% of the total
nesium deficiency symptoms are similar to those of
excretion. This is proportional to the protein intake.
calcium deficiency; but symptoms will be relieved only
ii. Organic sulfate or ethereal sulfate: It is also
when magnesium is given. Acute symptomatic defi-
called as conjugated sulfate. It constitutes 10% of
ciency is treated by giving parenteral magnesium. Oral
urinary sulfates. Tryptophan is converted to phenol
therapy may lead to diarrhea, hence intravenous mag-
nesium sulfate is given. and indoxyl by intestinal bacteria. These are absor-
bed and conjugated with sulfates and excreted
Hypermagnesemia through urine. Therefore, this represents the putre-
factive activity in intestine.
Causes of hypermagnesemia are listed in Box 34.13.
Magnesium intoxication causes depression of neuro- iii. Neutral sulfur or unoxidized sulfur: This fraction
muscular system, causing lethargy, hypotension, respira- constitutes 10% of total sulfates. Sulfur containing
tory depression, bradycardia and weak tendon reflexes. organic compounds, such as amino acids. It is also
Hypermagnesemia induces decrease in serum calcium increased in amino acidurias.
by inhibiting PTH secretion .
IIRON (Fe)
I SULFUR (S) Distribution of Iron
Source of sulfates is mainly amino acids cysteine and Total body iron content is 3-5 g; 75% of which is in
methionine. Proteins contain about 1% sulfur by weight. blood, the rest is in liver, bone marrow and muscles. Iron
Inorganic sulfates of Na+, K• and Mg•+, though available is present in almost all cells. Heme containing proteins
in food, are not utilized. are shown in Table 34.2.
Blood contains 14.5 g of Hb per 100 ml. About 75%
Functions of Sulfur of total iron is in hemoglobin, and 5% is in myoglobin
Sulfur containing amino acids are important constituents and 15% in ferritin. Normal iron kinetics is shown in
of body proteins. The disulfide bridges keep polypeptide Figure 34.6.
Chapter 34: Mineral Metabolism and Abnormalities 499

Requirement of Iron (ICMR) ii. Children between 13-15 years need 20-30 mg/
day.
i. Daily allowance of iron for an adult Indian is 20 mg
iii. Pregnant women need 40 mg/day. Transfer of iron
of iron, out of which about 1- 2 mg is absorbed. In
and calcium from mother to fetus occurs mainly in
Western countries, requirement is less (15 mg/day)
the last trimester of pregnancy. Therefore, during
because the diet does not contain inhibitory sub-
this period, mother's food should contain surplus
stances.
quantities of iron and calcium.
iv. In the first 3 months of life, iron intake is negligible
because milk is a poor source of iron. During this
time, child is dependent on the iron reserve recei-
Heme containing proteins ved from mother during pregnancy. In premature
Hemoglobin 65,000 4 RBC babies, the transplacental transfer of iron might not
Myoglobin 17,000 Muscle have taken place. Hence such babies are at a risk
Cytochrome oxidase 180,000 2 Mito of iron deficiency. After 3 months of life, diet sup-
Cytochrome b 30,000 do plementation with cereals is essential for supplying
Cytochrome cl 37,000 do the iron requirement.
Cytochrome c 12,000 do
Cytochrome bS 15,000 ER Sources of Iron
Cytochrome P450 55,000 ER, Mito
i. Leafy vegetables (20 mg/100 g) are good sources.
Catalase 240,000 4 RBC
Pulses (10 mg/ 100 g) and cereals (5 mg/100 g)
Lactoperoxidase 93,000 Milk
contain lesser quantity of iron. In a typical Indian
Tryptophan pyrrolase 4 Cytosol
diet, the major quantity of iron is received from
Nitric oxide synthase Endothelium
cereals because of the bulk quantity taken, although
.. Iron-sulfur complexes
they contain iron only in moderate amounts.
Com plex Ill Fe-S 30,000 2 Mito
ii. Liver (5 mg/100 g) and meat (2 mg/100 g)
Succinate DH 27,000 4 M ito
Liver
iii. Jaggery is a good source for iron.
Xanthine oxidase 275,000 8
iv. Cooking in iron utensils will improve the iron con-
Nonheme iron containing proteins
Aconitase 66,000 2 TCAcycle tent of the diet.
110,000 2 Liver
v. Milk is a very poor source of iron , containing less
Phehydroxylase
Transferrin 77,000 2 Plasma than 0.1 mg/100 ml.
Ferritin 450,000 4,000 Tissues
Factors Influencing Absorption of Iron
Hemosiderin Many Liver
(Mito = Mitochondria; ER = Endo plasmic reticulum; DH = Dehydroge- Iron is absorbed by upper part of duodenum. The follow-
nase; Phe= Phenyl alanine) ing factors affect absorption of iron:

2 Feces (19 mg)

!
3
Food (Fe ·) (20mg) - -- -- -- -- - Upper GI tract {Fe •)

(1 mg)

ECF Plasma Hematopoietic


(4 mg) system {RBC)
Hemoglobin
(2600 mg)
2Fe3+-Transferrrin
Enzymes in
(4 mg) Muscle myoglobin
cells (2 mg)
(200 mg)

Storage ferritin
+
Loss through skin Losa through
(600 mg) (10 µg) bleeding

F ig. 34 .6 : Normal iron kinetics


500 Section D: Nutrition

Reduced Form of Iron


Only Fe.. (ferrous) form (reduced form ) is absorbed.
11M 0 Fe'
0
(ferric iron) in food

Intestinal lumen
Fe<++ (ferric) form is not absorbed.
Divalent metal transporter
Ascorbic Acid Brush borde \V with Fe...
nnnnn nnnnnnnnnnnnnnn
M@
.@
Ferric ions are reduced with the help of gastric HCI,
Fe++ (ferrous) Intestine
ascorbic acid , cysteine and -SH groups of proteins. mucosal cell

-
Therefore, these will favor iron absorption. About
50-75 mg of ascorbic acid per day will be sufficient for Fe..'(ferric)
normal iron absorption.

Interfering Substances
Fe....Ferritin F0
Iron absorption is decreased by phytic acid (in cereals) Fe" crosses membrane Ill
and oxalic acid (in leafy vegetables) by forming insolu-
ble iron salts. An average Indian diet contains more than
20 mg of iron. But the phytates and oxalates in the diet
Fe...Transferrin
in bloodstream T
11111
0
reduce the absorption, and only about 1 mg of iron is 1= Fe- (ferric iron) in food. 2= Iron is reduced to Fe" (ferrous)
absorbed. In Western diet, even though iron content is state, and attaches to divalent metal transporter on the mucosal
surface. 3= Ferrous iron is internalized . 4= Iron is oxidized to ferric
about 10 mg, about 2 mg is absorbed. state. 5= Ferric iron binds with ferritin for temporary storage.
6=Ferric iron released, reduced to ferrous state crosses the cell
membrane, re-oxidized to ferric state by ceruloplasmin. In the
Other Minerals bloodstream, ferric iron is bound with transferrin,

Calcium, copper, lead and phosphates will inhibit iron Fig. 34.7: Absorption of iron from intestine
absorption. One atom of lead will inhibit absorption of
1000 atoms of iron .
When iron is in excess, absorption is reduced: this is the
Mucosa/ Block Theory basis of "mucosal block" (Fig. 34.7).
Duodenum and jejunum are the sites of absorption. Iron This mechanism of iron absorption from intestinal
metabolism is unique because homeostasis is main- lumen to the mucosal cell is different from the iron
tained by regulation at the level of absorption and not by release from intestinal cell to the bloodstream (see Fig .
excretion. No other nutrient is regulated in this manner. 34.7). Iron in the ferritin is released, then crosses the
In other words, iron is a one-way element. When iron mucosal cell with the help of a transport protein called,
stores in the body are depleted, absorption is enhanced. ferroportin. But this can happen only when there is free
When adequate quantity of iron is stored, absorption is transferrin in plasma to bind the iron. Iron crosses cell
decreased. This is referred to as "mucosa! block" of membrane in ferrous form. In blood it is re-oxidized to
regulation of absorption of iron. ferric state, and transported by transferrin.
Only ferrous (and not ferric) form of iron is absorbed. Absorption of iron needs divalent metal ion trans-
Ferrous iron in the intestinal lumen binds to mucosal porter and ferroportin. Synthesis of both these proteins
cell protein, called divalent metal transporter-1 (DMT-1). is downregulated by hepcidin, a peptide secreted by the
This bound iron is then transported into the mucosal cell. liver when body iron reserves are adequate. If there is
The rest of the unabsorbed iron is excreted. Inside the hypoxia or anemia, the synthesis of hepcidin is reduced;
mucosal cell, iron is oxidized to ferric state, and is com- so ferroportin synthesis will increase.
plexed with apoferritin to form ferritin . It is kept tempo-
rarily in the mucosal cell. If there is anemia, the iron is Iron Transport in Blood and
further absorbed into the bloodstream. If transferrin is Uptake by Cells
saturated with iron , any iron accumulated in the mucosal
cell is lost when the cell is desquamated. The fraction of Transport form of iron is transferrin. It is synthesized in
iron absorbed and retained is decided by the iron status. liver. Normal plasma level of transferrin is 250 mg/100 ml.
Chapter 34: Mineral Metabolism and Abnormalities 501

TABLE 34.3: Parameters of iron status RBC lysis


Hemolytic t
Transferrin
Normal
250
Pregnancy anemia
300 200
Hemosiderosis
1000
l
Hb Hb-Haptoglob1n complex taken
up by liver Kupffer·s cells - - - • Iron re-utilized
(mg/dl)
!'---+
r~::::=...-
----'---+
Globin removed
Serum iron 120 so 200 250
(µg/dl)
TIBC (µg /dl) 400 500 300 300 Iron re-utilized
120
l
RBC. lifespan, 120 75 75
days
'-+ Porphyrin ring opened - + Bilirubin
Ferritin (ng/ ml) 20-250 Reduced <10 > 400

Iron -+ lron-Translerrln--+ S ~ ----+ Iron re-utilized


In iron deficiency, this level is increased. One molecule
of transferrin can transport 2 ferric atoms. Total iron Fig. 34.8: Conservation of iron in the body
binding capacity (TIBC) in plasma is 400 mg/ 100 ml;
this is provided by the transferrin. One-third of this capac- Excretion of Iron
ity is saturated with iron . This protein bound iron (serum
iron) is about 120 mg/dl. Abnormalities in these para- Iron is a one-way element. That is, very little of it is excreted.
meters are shown in Table 34.3. In iron deficiency The regulation of homeostasis is done at the absorption
anemia, TIBC is increased (transferrin level is increased); level. Any type of bleeding will cause loss of iron from
but serum iron level is reduced. Transferrin has a half life the body. Menstrual flow is the major cause for loss of
of 7-10 days, and is a useful index of nutritional status. iron in women. Women up to menopause will lose iron
One molecule of transferrin can bind two ferric ions. at a rate of about 1 mg/day. The loss in male is less than
In blood, ceruloplasmin is the ferroxidase, which 0.5 mg/day. Almost no iron is excreted through urine.
oxidizes ferrous to ferric state (see Fig. 34. 10). Feces contain unabsorbed iron as well as iron trapped
Ferroxidase in the intestinal cells, which are then desquamated.
Apo-transferrin - -- •Transferrin combined About 30% of cells in the intestinal lining is replenished
+ 2 Fe++ with 2 Fe••• everyday, and so this loss is considerable. All the cells
Transferrin receptors (TfR) are present on most of in skin contain iron. The upper layers of skin cells are
the body cells, especially on cells, which synthesize heme. constantly being lost, and this is another route for iron
The iron-transferrin complex is taken up by the body cells loss from the body.
by the receptor mechanism.
Iron Deficiency Anemia
Storage of Iron About 30% of world population anemic. About 70% of
The storage form is ferritin. It is seen in intestinal muco- Indians have iron deficiency and 85% of pregnant women
sa! cells, liver, spleen and bone marrow. The apoferritin suffer from iron deficiency anemia. Anemia often leads
can take up to 4,000 iron atoms per molecule. Ferritin to irreversible impairment of child's learning ability. In
contains about 23% iron. Hemosiderin is formed by adults, anemia results in impaired work capacity. Causes
partial degradation of ferritin. It contains more iron and of iron deficiency are given in Box 34. 14. Anemia is clas-
is found in tissues like liver, spleen and bone marrow. sified in Box 34. 15.

Iron is Conserved Microscopic Appearance


When RBC is lysed, hemoglobin enters into circulation.
Iron deficiency is characterized by microcytic hypo-
Being a small molecular weight substance, Hb will be
chromic anemia (Fig. 34.9A). Anemia results when
lost through urine. To prevent this loss, Hb is immediately
hemoglobin level is less than 12 g/dl.
taken up by haptoglobin (Hp) (Fig. 34.8). When the
globin part is removed from Hb, the heme is produced,
Clinical Manifestations
and is released into circulation. In order to prevent its
excretion through urine, heme is bound with hemopexin When the level is lower than 10 g, body cells lack oxy-
(see Fig. 34.8). gen and patient becomes uninterested in surroundings
502 Section D: Nutrition

BOX 34.14: Causes of iron deficiency BOX 34.15: Class1f1cat1on of anemias


1. Nutritional deficiency of iron 1. Impaired Production of RBCs
2. Lack of absorption: Subtotal gastrectomy and hypoch lor- a. Defect in heme synthesis: Deficiency of iron, copper,
hydria pyridoxal phosphate, folic acid, vitamin B12 or vitamin C.
3. Hookworm infection: One hookworm will cause the loss of Lead will inhibit heme synthesis
about 0.3 ml of blood per day. Calculation shows that about b. Defect in regulators: Lack of erythropoietin, due to
300 worms can produce a loss of 1%oftotal body iron per day chronic renal failure
4. Repeated pregnancies: About 1g of iron is lost from the c. Defect in stem cells: Aplastic anemia due to drugs (e.g.
mother during one delivery Chloramphenicol), infections and malignant infiltrations
5. Chronic blood loss: Hemorrhoids (piles), peptic ulcer, may lead to anemia
menorrhagia 2. lntracorpuscular Defects
6. Nephrosis: Haptoglobin, hemopexin and transferrin are lost
a. Hemoglobinopathies: HbS, HbC, HbM
in urine, along w ith loss of iron
b. Thalassemlas: Major and minor
7. Lead poisoning: Iron absorption and hemoglobin synthesis
c. Abnormal shape: Spherocytosis
are reduced. In turn, iron deficiency causes more lead
d. Deficiency of glucose-6-phosphate dehydrogenase (see
absorption. It is a vicious cycle.
Chapter 6)
3. Extracorpuscular Causes
(apathy}. All metabolic processes become sluggish. a. Infections: Malaria, Streptococcus
Prolonged iron deficiency causes atrophy of gastric b. Autoimmune hemolysis
c. lsoimmune hemolysis: Rh incompatibility
epithelium leading to achlorhydria, which in turn causes
d. Hemolysis due to drug sensitization: Alpha-methyldopa,
lesser absorption of iron , aggravating the anemia. Simi- quinine, etc.
lar atrophy of epithelium in oral cavity and esophagus 4. Hemorrhage
causes dysphagia termed Plummer-Wilson syndrome, Hematuria, hematemesis, hemoptysis, peptic ulcer, menor-
which is a known precancerous condition. Very chronic rhagia, hemorrhoids, hemophilia (absence of AHG), thrombo-
cytopenia.
iron deficiency anemia will lead to impaired attention,
irritability, lowered memory and poor scholastic perfor-
mance. Anemia and apathy go hand in hand (Fig. 34.98 ).

Laboratory Findings
Laboratory investigations generally used to diagnose
anemias are listed in Table 34.3.
i. Serum iron level: It is depressed in iron deficiency, Figs 34.9 A and B: (A) Peripheral blood smear. Iron deficiency
acute and chronic infections, hypothyroidism and manifests as microcytic hypochromic anemia; (B) Tired girl.
Apathy and poor scholastic performance are very characteristic of
Kwashiorkor. iron deficiency anemia
ii. Total iron binding capacity (TIBC): It is elevated
in hypochromic anemias, acute hepatitis and preg- granules, seen in spleen and liver. Prussian blue
nancy. reaction is positive for these pigments. Hemo-
siderosis occurs in persons receiving repeated
Treatment of Iron Deficiency
blood transfusions. Here the regulation at the
Oral iron supplementation is the treatment of choice. level of intestine is circumvented leading to iron
100 mg of iron + 500 mg of folic acid are given to overload . Hemophilic children require blood trans-
pregnant women, and 20 mg of iron + 100 mg folic fusion every 3 months. If whole blood is given every
acid to children. Iron tablets are usually given along time, by about 20 years of age, the patient will have
with vitamin C, to convert it into ferrous form, for easy hemosiderosis. This is the commonest cause for
absorption. Unabsorbed iron may generate free radicals hemosiderosis in India. Nowadays, component
and so, it is advisable to give vitamin E (to prevent separated plasma is available for transfusion,
free radical generation) along with iron. which eliminates the risk of hemosiderosis. Modern
treatment of hemophilia is to give cell free plasma;
Iron Toxicity this will avoid future hemosiderosis.
i. Hemosiderosis: Iron excess is called hemo- ii. Primary hemosiderosis: It is also called hereditary
siderosis.Hemosiderin pigments are golden brown hemochromatosis. It is an autosomal recessive
Chapter 34: Mineral Metabolism and Abnormalities 503

Ferroxidaze
Functions of Copper
T
Fe" (Transferrin) \ • Fe' '"' (Transferrin) 1. It is necessary for iron absorption and incorporation
cu- cu• of iron into hemoglobin.
(Oxidized ceruloplasmin) (Reduced ceruloplasmin) 2. It is necessary for tyrosinase activity.
Fig. 34.1 0: Function of ceruloplasmin 3. It is a co-factor for vitamin C requiring hydroxylations.

Abnormal Metabolism of Copper


condition. The gene is named as HFE-1. Due to
a mutation in HFE-1 , iron absorption is increased, i. Wilson's disease: Ceruloplasmin level in blood
leading to iron overload. is drastically reduced in Wilson's hepatolenticular
iii. Iron vessels: Cooking in iron vessels increases degeneration. The incidence of Wilson's disease is
the availability of iron. 1 in 50,000. The basic defect is in a gene encoding
iv. Bantu siderosis: Bantu tribe in Africa is prone to a copper binding ATPase in cells. This is required
hemosiderosis because the staple diet, corn, is low for normal excretion of copper from liver cells; in its
in phosphate content. absence, copper is accumulated in cells, leading to
v. Hemochromatosis: In the liver, hemosiderin de- copper deposits in liver and brain (see Chapter 26).
posit leads to death of cells and cirrhosis. Pancre- ii. Copper deficiency anemia: Copper deficiency affe-
atic cell death leads to diabetes. Deposits under cts hemoglobin formation. Copper containing cerulo-
the skin cause yellow-brown discoloration, which plasmin helps in iron transport. Copper is an integral
is called hemochromatosis. The triad of cirrhosis, part of ALA synthase, which is the key enzyme in
hemochromatosis and diabetes are referred to as heme synthesis. Copper deficiency thus results in
bronze diabetes. microcytic normochromic anemia. If there is added
iron deficiency, hypochromic anemia results.
ICOPPER (Cu) iii. Cardiovascular diseases: Copper is a constituent
of lysyl oxidase. It makes cross linkages in elastin .
Total body copper is about 100 mg. It is seen in muscles, In copper deficiency, elastin becomes abnormal,
liver, bone marrow, brain, kidney, heart and in hair. leading to weakening of walls of major blood ves-
Copper containing enzymes are ceruloplasmin, cyto- sels. This favors aneurysm and fatal rupture of the
chrome oxidase, cytochrome c, tyrosinase, lysyl oxi- wall of aorta.
dase, ALA synthase, monoamine oxidase, superoxide iv. Menke's kinky hair syndrome: It is an X-linked
dismutase and phenol oxidase. defect {affects only male child). Dietary copper is
Copper requirement for an adult is 1.5 -3 mg per day. absorbed. But it is not transported to blood due
Major dietary sources are cereals, meat, liver, nuts and to absence of an intracellular copper binding
green leafy vegetables. Milk is very poor in copper content. AT Pase The copper that has entered into the cell is not
Excretion is mainly through bile. Urine does not con- able to get out of the cells, and so it is accumulated.
tain copper in normal circumstances. v. Melanin: Copper is present in tyrosinase, which is
Whole blood contains about 100 mg/dl of copper. necessary for melanin formation (see Chapter 19).
Out of this, 95% is in RBC as colorless erythrocuprein. Copper deficiency thus leads to hypopigmentation
In plasma ceruloplasmin is an important copper con- and in extreme cases, gray color of hair. The period
taining protein (see Chapter 26). Normal serum level of of copper deficiency may be marked on hair as
ceruloplasmin is 25-50 mg/dl. It is also called serum alternate white patches; sometimes called flag type
ferroxidase. It promotes oxidation of ferrous ion to ferric of hair growth.
form, which is incorporated into transferrin (Fig. 34.10).
The copper atoms are tightly bound with ceruloplasmin. Copper Toxicity
So copper from ceruloplasmin cannot be released easily. Excess copper intake may lead to toxic manifestations.
About 10% of copper in plasma is loosely bound Chronic toxicity is manifested as diarrhea and blue-green
with albumin , which constitutes the transport form of discoloration of saliva. Copper poisoning may result in
copper. hemolysis, hemoglobinuria, proteinuria and renal failure.
504 Section D: Nutrition

I IODINE (I) _ _ Zinc Toxicity


Three major micronutrient deficiencies seen in India are Toxic manifestations are seen when intake is more than
those of iodine, iron and vitamin A. In India, 235 districts 1000 mg/day. Toxicity of zinc is usually seen in welders
are endemic for iodine deficiency. Iodine level in blood due to inhalation of zinc oxide fumes.
is 5-10 mg/dl. Daily requirement is 150-200 mg and it
is now provided by fortification of table salt with iodine. IFLUORIDE
Iodine is necessary for production of thyroid hormones; Fluoride is known to prevent caries. Caries is a Latin
these are described in Chapter 45. term, meaning "decay". In the pits and fissures of pre-
molar and molar teeth, bacterial fermentation of residual
I ZINC (Zn) food leads to acid production. The acid removes enamel
and dentine to expose the pulp, leading to inflammation
Total zinc content of body is about 2 g, out of which 60%
and toothache.
is in skeletal muscles and 30% in bones. Highest con- The safe limit of fluorine is about 1 ppm in water.
centration of zinc is seen in hippocampus area of brain (ppm = parts per million; 1 ppm = 1 gram of fluoride in
and prostatic secretion. million grams of water; this is equal to 1 mg per 1000 ml).
Rich dietary sources are grains, beans, nuts, cheese, Fluoride ions enter the hydration shell surrounding
meat and shellfish. Zinc and copper will competitively the apatite crystals and may become incorporated into
inhibit each other's absorption. So, zinc is therapeutically the crystal surface. The fluorapatite makes the tooth sur-
useful to reduce copper absorption in Wilson's disease. face more resistant to plaque bacterial attack.
More than 300 enzymes are zinc dependent. Some
important ones are carboxypeptidase, carbonic anhy- Fluorosis is more Dangerous than Caries
drase, alkaline phosphatase, and lactate dehydrogenase. Fluoride level more than 2 ppm will cause chronic inte-
Extracellular superoxide dismutase is zinc dependent stinal upset, gastroenteritis, loss of appetite and loss
and so, zinc is an anti-oxidant. of weight. Level more than 5 ppm causes mottling of
Insulin when stored in the beta cells of pancreas enamel , stratification and discoloration of teeth.
contains zinc, which stabilizes the hormone molecule. A level more than 20 ppm is toxic, leading to alter-
But the insulin released into the blood does not contain nate areas of osteoporosis and osteosclerosis, with brittle
zinc. The commercially available preparation, protamin- bones. This is called fluorosis. Genu valgum is the cha-
zinc-insulinate (PZI) also contains zinc. racteristic feature.
Nellore, Nalgonda and Prakasam districts of Andhra
Zinc Deficiency Manifestations Pradesh and Patiala district of Punjab are badly affected.
25 million people in India are suffering from fluorosis,
Poor wound healing, lesions of skin, impaired spermato-
spread in 15 states of India. In the vicinity of irrigation
genesis, hyperkeratosis, dermatitis and alopecia are
dams, the water level in wells will come up, along with
deficiency manifestations of zinc.
salts including fluoride. This has resulted in widespread
fluorosis in Punjab, Rajasthan, UP, Delhi, Andhra
Acrodermatitis Enteropathica Pradesh, Karnataka and Tamil Nadu. Certain salts used
It is a recessive condition where zinc absorption is in paan supari also have large content of fluoride. Fluori-
defective and is characterized by acrodermatitis (inflam- nated toothpaste contains 3,000 ppm of fluoride. Even
mation around mouth, nose, fingers, etc.), diarrhea, and ordinary toothpaste contains fluoride about 700 ppm.
alopecia (loss of hair in discrete areas).
l SELENIU~Se)
Requirement of Zinc Requirement is 50-1 00 mg/day. Normal serum level is
For adults is 10 mg/day; children 10 mg/day; in preg- 50-100 mg/dl.
nancy and lactation 15- 20 mg/day. Since iron inhibits In mammals, glutathione peroxidase (GP) is the
absorption of zinc, when iron is supplemented, zinc is important selenium containing enzyme. RBC contains
also given to prevent any deficiency. good quantity of glutathione peroxidase.
Chapter 34: Mineral Metabolism and Abnormalities 505

Thyroxin is converted to T3 by 5'-de-iodinase, which TABLE 34.4: Summary of mineral metabolism


is a selenium containing enzyme. In Se deficiency, this Requirement for adult male/day Blood/eve/
enzyme becomes inactive, leading to hypothyroidism. Calcium 500 mg 9- 11 mg/dl
It is necessary for normal development of sperma- Phosphorus 500mg 3-4mg /dl
tozoa. Selenocysteine may be considered as the 21st Magnesium 400mg 1.8-2.2 mg/ dl
amino acid (see Box 3.2). Sodium 5- 109 136-145 mEq/L
Selenium acts as a nonspecific intracellular antioxi- Potassium 3- 49 3.5-5 mEq/ L
dant. This action of Se is complementary to vitamin E. 2.0-2.3 g 96-106 mEq/ L
Chloride
Availability of vitamin E reduces the selenium require- 20mg 120 mcg/dl
Iron (plasma)
ment. In Se deficiency, tissue vitamin E content is depleted. 100 mcg/ dL
Copper 1.5- 3 mg
In Keshan province in China , the soil is deficient in
Iodine 200 mcg l0mcg/dL
selenium . This leads to prevalence of Keshan disease.
Zinc 10mg 100 mcg/dl
It is characterized by multifocal myocardial necrosis,
Chromium 50 mcg 25 mcg/ dl
cardiac arrythmias and cardiac enlargement.
Selenium 100 mcg 100 mcg/ dl
Selenium toxicity is called Selenosis. Selenium is
present in metal polishes and anti-rust compounds. The
toxicity symptoms include hair loss, falling of nails, diar- Related Topics
rhea , and weight loss. Sodium, potassium and chloride are very important

IMANGANESE (Mn) _ __
electrolytes, having much clinical applications. These
are described in chapter 28. A summary of the mineral
Requirement of manganese is 5 mg/day. metabolism is shown in Table 34.4. Lead, Cadmium,
Sources : Nuts are good sources and tea leaves are ex- Phosphorus and mercury are toxic minerals; these are
ceptionally rich in manganese. described in Chapter 37. Iodine metabolism is described
.. Functions: The following enzymes either contain or are in Chapter 45.
activated by manganese: Hexokinase, phosphogluco- 0
mutase, pyruvate carboxylase, isocitrate dehydrogenase, 8· Clinical Case Study 34.1
succinate dehydrogenase, arginase, glutamine synthe-
A 54-year-old grossly overweight woman presented
tase and Mn..-dependent superoxide dismutase. Man-
with complaints of cramps and spasms of both hands.
ganese is an integral part of glycosyltransferases,
She was depressed and had positive Trosseau's and
responsible for synthesis of glycoproteins and chon- Chvostek signs. Past medical history revealed history
droitin sulfate. Manganese is also required for RNA
of thyroidectomy for Grave's disease. The laboratory
polymerase activity. results obtained were as follows:
Deficiency leads to impaired growth and skeletal Serum Creatinine - 1.0 mg%, serum Calcium - 4.1
deformities. mg%, serum phosphate - 5.9 mg%, ALP - 60 IU/L,

ILITHIUM (Li)
serum albumin - 4.0 g%.
Comment on the laboratory results and give your
Lithium is used in treating manic depressive psycho- likely diagnosis. What additional tests are to be done to
sis (bipolar disorders), the dose being 25-500 mg/day. come to a conclusive diagnosis?
Lithium will counteract both mania and depression. 0
Therapeutically optimum concentration of u• in plasma • • Clinical Case Study 34.2
is 7- 10 mg/ml, while 12 micrograms is toxic. Since A 10-year-old girl presented with excessive tiredness,
margin of safety is narrow, the treatment requires constant poor appetite, inability to concentrate and tingling sen-
monitoring of blood level. Lithium elevates serotonin sations. On examination , there was pallor. Laboratory
levels and reduces catecholamines in brain tissue. examination revealed decrease in hemoglobin, ferritin
Lithium toxicity leads to hypothyroidism, hyperpara- and MCV. Total iron binding capacity (TIBC), transferrin
thyroidism and kidney damage. and ROW were increased. What is the likely diagnosis?
506 Section D: Nutrition

e free iron is highly toxic. Ferritin and hemosiderin are


•i i- Clinical Case Study 34.3 storage forms of iron.
Laboratory indices of iron deficiency are lowered
A 42-year-old female presented to the clinic with com-
serum iron, low ferritin level, and high total iron bind-
plaints of vague abdominal discomfort, weakness and
ing capacity (TIBC). Other tests include complete blood
fatigue, and bone pain. She had frequent urinary tract
count (CBC) where MCV and MCHC are reduced, peri-
infections and had several episodes of kidney stones.
pheral smear, soluble transferrin receptor protein (TRP),
Her physical examination was within normal limits. The
stool for hemoglobin , hemoglobin electrophoresis and
patient had a normal complete blood count (CBC), but
urine for hemoglobin and hemosiderin.
electrolytes revealed a significantly elevated calcium
Treatment includes iron (oral or parenteral) and red
level and low phosphorus level. What is the most likely
cell transfusion, as necessary.
diagnosis?
e
•i i• Clinical Case Study 34.4 ae Clinical Case Study 34.3 Answer
Diagnosis: Hyperparathyroidism, leading to hypercalce-
A 9-year-old girl presented with muscle pain and
mia and hyperphosphatemia.
cramps, tingling of hands and feet, stiffness, recurrent
Biochemical mechanism: Elevated parathyroid hor-
carpopedal spams, and titanic posturing of both hands
mone level acts by binding its receptor to activate the
and feet. She was a strict vegetarian consuming no milk
adenylate cyclase/protein kinase signaling system. Pri-
or milk products. Other siblings are normal.
mary hyperparathyroidism, usually because of a solitary
On examination, no symptoms of rickets or short
parathyroid adenoma, is the most likely cause when
stature, BP was normal and general examination was
hypercalcemia is discovered in an otherwise asympto-
normal. Trousseau's sign was positive. Calcium level
matic patient. Hypercalcemia may be the presentation
was 6.5 mg/dl. What is the diagnosis? What are the
of intoxication with vitamin D, or calcium-containing
causes for the disease? What other investigations are
antacids, genetic conditions like familial hypocalciuric
needed to come to a confirmatory diagnosis?
hypercalcemia and hyperparathyroidism as part of a
(') multiple endocrine neoplasia syndrome are uncommon.
•i i• Clinical Case Study 34.5 Most patients have no symptoms with mild hypercal-
A 36-year-old female was on treatment with 131 1 (radio- cemia below 12.0 mg/dL, except polyuria. With levels
active iodine) for hyperthyroidism. Two months later, above 13 mg/dl, patients begin developing neurological
she developed severe carpopedal spasm and plasma symptoms (lethargy, stupor, coma, psychosis),
calcium was 6.0 mg/dl. Laboratory results also revealed gastrointestinal symptoms (anorexia, nausea, peptic ulcer
normal fT3, fT4 and TSH and low PTH. What is the disease), kidney problems (polyuria, nephrolithiasis),
possible diagnosis? What is the cause of hypocalcemia and musculoskeletal complaints (myalgias, weakness)
in this patient? The symptoms of hyperparathyroidism can be summari-

ae
zed as: stones (kidney), moans (abdominal pain),
Clinical Case Study 34.1 Answer groans (myalgias), bones (bone pain), and psychiatric
Hypocalcemia due to accidental removal of parathyroid overtones. Diagnosis can be established by finding
glands. Please see details under Clinical Case Study hypercalcemia and hypophosphatemia, with elevated
34.4 also. PTH levels. Symptomatic patients can be treated with

e parathyroidectomy.
.:ti-- Clinical Case Study 34.2 Answer (')
The symptoms are suggestive of iron deficiency ane-
.:ti• Clinical Case Study 34.4 Answer
mia, the commonest deficiency disease prevalent world- Muscle pain and cramps, stiffness, tingling of hands and
wide. Iron requirements are high during infancy, child- feet, carpopedal spasm and positive Trousseau's sign
hood, adolescence and last two trimesters of pregnancy. are indicative of hypocalcemia. Two common causes for
Iron is a critical element in the functioning of cells, but hypocalcemic tetany are hypoparathyroidism or chronic
Chapter 34: Mineral Metabolism and Abnormalities 507

renal failure. Abnormal renal function tests, arterial blood ( LEARNING POINTS, CHAPTER 3L
gases, hemogram and urine calcium/creatinine ratio
1. Factors favorably influencing calcium absorption
would point toward a renal cause for hypocalcemia.
are-calcitriol, PTH , acid pH, and arginine.
High serum alkaline phosphatase and low parathyroid
2. Factors decreasing calcium absorption are-phytic
hormones would indicate hypoparathyroidism. Antimicro-
acid, high phosphate content and malabsorption syn-
somal antibody may be elevated in the latter state.
dromes.
Hypoparathyroidism may be due to parathyroid aplasia
3. Calcium activates enzymes, such as protein kina-
or hypoplasia, parathyroid hormone gene mutations, ses, glycogen synthase, pyruvate carboxylase,
autoimmune parathyroiditis, hemosiderosis, Wilson's pyruvate dehydrogenase.
disease, or accidental removal during thyroid surgery. 4. Calcium mediates contraction of muscle fibers.
Additional tests include electrolytes (magnesium, 5. Calcium mediates secretion of insulin. PTH, calci-
phosphate and other electrolytes), renal function tests tonin, ADH and can act as a second messenger for
(urea and creatinine), arterial blood gases, liver function some hormones, e.g. Glucagon.
tests (albumin, coagulation studies and other liver func- 6. Calcium is known as Factor IV in blood coagulation.
tion tests), PTH and Vitamin D levels. Cardiac assess- 7. Hypercalcemia may occur in hyperparathyroidism,
ment may be necessary to rule out cardiac effects of multiple myeloma, bone cancer, and Paget's disease.
hypocalcemia. 8. Hypocalcemia leads to tetany as seen in renal tubu-
(") lar acidosis.
•ii• Clinical Case Study 34.5 Answer 9. Hypernatremia is seen in Cushing's syndrome and
after prolonged cortisone therapy. Hyponatremia is
The patient is suffering from hypoparathyroidism, due to seen in Addison's disease.
decreased activity of PTH . Treatment with 131 1decreased 10. Renal loss of potassium is seen in renal tubular
thyroid cells, but also killed neighboring parathyroid cells acidosis, tubular necrosis , metabolic alkalosis.
leading to persistent hypoparathyroidism in this patient. 11 . Hyperchloremia is seen in dehydration, Cush-
Carpopedal spasm is due to hypocalcemia and can be ing's syndrome, respiratory acidosis, renal tubular
treated by calcium and vitamin D supplementation. acidosis.
Causes of hypoparathyoidism are: (1) Primary- 12. Hypochloremia is seen in excessive vomiting,
which is permanent and irreversible, could be congenital Addison's disease, and respiratory alkalosis.
or acquired, (2) Iatrogenic-Excision of all 4 parathyroid 13. Iron homeostasis is maintained by regulation of
glands, as during surgery of thyroid or other neck surge- absorption and not by excretion.
ries, extensive irradiation of face, neck or mediastinum, 14. Transport form of iron transferrin. Storage form is
parathyroidectomy, (3) Autoimmune hypoparathyrodism. ferritin. Iron deficiency may be caused by hookworm
(4) Congenital, (5) Metal overload conditions like hemo- infection, nephrosis, lead toxicity, and leads to micro-
chromatosis and thalassemia, (6) Wilson's disease, cytic hypochromic anemia. Iron toxicity leads to hem-
(7) Hypomagnesemia, (8) Neonatal hypoparathyroidism, osiderosis.
and (9) Pseudohypoparathyrodism-due to resistance 15. Copper in plasma is bound to ceruloplasmin. It
to PTH . functions as co-factor for Vitamin C mediated
Clinical features are paresthesias, irritability, fatigue, hydroxylations.
anxiety, mood swings, personality disturbances, seizu- 16. Copper is essential for formation of Hb, deficiency
res, hoarseness of voice, wheezing and dyspnea, muscle leading to microcytic normochromic anemia.
cramps, biliary colic, electrolyte disturbances (hypomag- 17. Copper level in blood is lowered in Wilson's hepato-
' nesemia, hypokalemia and alkalosis) and tetany with posi- cellular degeneration.
tive Chvostek's sign and Trousseau sign. Hypocalcemia 18. Menkes Kinky hair syndrome, an X-linked disease
can produce a variety of associated features like extrapy- is caused due to decreased copper transport in the
ramidal choreoathetotic syndromes, spastic paraplegia, blood.
ataxia, papilledema, emotional disturbances, cataracts, 19. Zinc is stored in combination with metallothionein.
abnormal dentition, dry, puffy skin and effects on heart More than 300 enzymes are known to be zinc
including congestive heart failure. dependent.
508 Section D: Nutrition

PART-1 : ESSAY AND SHORT NOTE QUESTIONS


34-1 . What is the normal blood level of calcium? What are the mechanisms by which calcium homeostasis is main-
tained?
34-2. Describe the sources, daily requirement, absorption, biochemical functions and deficiency manifestations of
iron.
34-3. Name any four trace elements and mention the biological functions of each of them.

SHORT NOTE QUESTIONS


34-4. Factors influencing calcium absorption. 34-9. Causes and deficiency manifestations of iron.
34-5. Homeostasis of blood calcium level. 34-10. Copper metabolism.
34-6. Hemosiderosis. 34-11. Ceruloplasmin.
34-7. Absorption of iron 34-12. Wilson's Hepatolenticular degeneration.
34-8. Transferrin. 34-13. Fluorosis.

PART-2 : MULTIPLE CHOICE QUESTIONS


34-1. Absorption of calcium from intestine is influenced 34-10. Daily requirement of iron is:
by all the following, except A. 10-20 microgram B. 0.1--0.5 gram
A. 1,25-dihydroxycholecalciferol C. 0.1- 0.5 milligram D. 10- 20 milligram
B. Calcitonin 34-11 . Iron is deficient in:
C. Parathyroid hormone A. Cereals (rice, wheat)
D. Phosphate content of diet B. Maize and com
34-2. Plasma calcium level is influenced by the follow- C. Pulses (bengal gram etc.)
ing, except D. Milk
A. Calcitriol B. Parathyroid hormone 34-12. Which enzyme do not contain copper?
C. Calcitonin D. Thyroxin A. Cytochrome oxidase
34-3. Normal level of calcium in blood is: B. Superoxide dismutase
A. 3-4 mg/dl B. 9-11 mg/dl C. Xanthine oxidase
C. 4-5 mEq/L D. 96-106 mEq/L D. Tyrosinase
34-4. Daily requirement of calcium for an adult is: 34-13. Daily requirement of zinc for an adult is:
A. 2 mg B. 15 mg A. 2 mg B. 10 mg
C. 70 mg D. 500 mg C. 40 mg D. 70 mg
34-5. Calcium is required for the following, except 34-14. Zinc is present in all enzymes, except
A. Coagulation A. Alkaline phosphatase
B. Neuromuscular transmission B. Amylase
C. Absorption of iron C. Carbonic anhydrase
D. Intracellular messenger D. Carboxypeplidase
34-6. Sodium level in serum is affected by all the follow- 34-15. Tetany can result from all, except
ing factors, except A. Hypocalcemia
A. Aldosterone B. Cortisone B. Vitamin C deficiency
C. ADH D. Cholecalciferol C. Alkalosis
34-7. Normal level of sodium in plasma is: D. Hypomagnesemia
J
A. 3-4 mg/d l B. 9-1 1 mg/dl 34-16. Which is not true regarding iron?
C. 4- 5 mEq/L D. 136-145 mEq/L A. Body level is regulated by increasing excretion
34-8. Normal level of potassium in plasma is: B. Transported as ferric iron bound to transferrin
A. 3-4 mg/dl B. 9-11 mg/dl C. Absorbed as ferrous form only
C. 4-5 mEq/L D. 96-106 mEq/L D. Main storage is ferritin in liver
34-9. Normal level of chloride in plasma is: 34-17. Daily requirement of iodine for an adult is:
A 3-4 mg/dl B. 9-11 mg/dl A. 15-20 microgram B. 150-200 microgram
C. 4-5 mEq/L D. 96-106 mEq/L C. 800-900 microgram D. 1-2 milligram
Chapter 34: Mineral Metabolism and Abnormalities 509

34-18. The micromineral present in teeth is: 34-25. Hyperparathyroidism is characterized by all the
A. Calcium B. Iodine following, except
C. Fluorine D. Manganese A. Osteoporosis 8. Hypercalcuria
34-19. Zinc is present in all the enzymes, except: C. Alkalosis D. Recurrent urinary calculi
A. Al kaline phosphatase 34-26. Sodium level in serum is affected by all the follow-
8 . Amylase ing factors , except:
• C. Carbonic anhydrase A. Aldosterone
C. ADH
B. Cortisone
D. Cholecalciferol
D. RNA polymerase
34-20. Which of the following trace elements has an anti- 34-27. Which contains iron?
oxidant role? A. Ceruloplasmin
A. Chromium B. Zinc B. Xanthine oxidase
C. Selenium D. Nickel C. Albumin
34-21 . Accumulation of iron (hemosiderosis) leads to all D. Superoxide dismutase
the following clinical features, except: 34-28. All are true with selenium, except
A. Diabetes mellitus A. Action of selenium is complementary to vitamin E
B. Cirrhosis of liver B. Glutathione peroxidase contains selenium
C. Defective coagulation C. Selenium may cause hepatic necrosis
D. Hyper pigmentation D. Requirement is 50-100 microgram/day
34-22. Which of the following is not a feature of Wilson's 34-29. In a patient the serum calcium level was 12 mg/dL,
hepato lenticular degeneration? and serum inorganic phosphate was 2 mg/dL. The
A. Decreased excretion of copper in urine above findings are characteristic of:
B. Low ceruloplasmin level A. Excess calcitonin secretion
C. Keyser-Fleischer ring in cornea 8 . Vitamin D deficiency
t
D. Cirrhosis of liver C. Hyper parathyroidism
34-23. Which of the following is not true regarding the D. Hypervitaminosis D
absorption of calcium? 34-30. Fasting samples give lower values than post pran-
A. Absorption is increased by vitamin D dial samples for all the parameters, except:
B. Acidity favors calcium absorption A. Glucose
C. Deficiency of bile favours absorption B. Phosphate
D. Basic amino acids increase absorption C. Triglycerides
34-24. Parathyroid hormone regulates calcium homeo- D. Cholesterol
stasis by all the mechanisms, except: 34-31 . Hypoklaemia occurs in all conditions, except:
A. Bone resorption A. Rapid correction of acidosis
B. Secretion of calcitonin B. Infusion of insulin and glucose
C. Increased absorption of calcium from intestine C. Use of loop diuretics
D. Increased reabsorption from renal tubules D. Polycythemia

ANSWERS OF MULTIPLE CHOICE QUESTIONS


34-1 . B 34-2. D 34-3. B 34-4. D 34-5. C 34-6. D 34-7. D
34-8. C 34-9. D 34-10. D 34-11 . D 34-12. C 34-13. B 34-14. B
34-15. B 34-16. A 34-17. B 34-18. C 34-19. B 34-20. C 34-21 . D
34-22. A 34-23. C 34-24. 8 34-25. C 34-26. D 34-27. B 34-28. C
34-29. C 34-30. B 34-31 . D

PART-3: VIVA VOCE QUESTIONS AND ANSWERS

34-1 . What influences absorption of calcium from intes- 34-3. What are the sources of calcium?
tine?
Milk, egg, fish, vegetables.
Vitamin D; Calcitriol; Parathyroid hormone; phytic acid;
34-4. What is the function of calcium?
oxalate.
34-2. What is the daily requirement of calcium? Coagulation; Neuromuscular activity; Intracellular mes-
500 mg per day. senger; Activation of enzymes; Bone formation.
510 Section D: Nutrition
34-5. What is the normal level of calcium in blood? Hepcidin is secreted by liver, when body iron reserves
9-11 mg /dl . are adequate. In anemia, the synthesis of hepcidin is
34-6. Which will influence serum calcium level? reduced.
Calcitriol; Calcitonin; Parathyroid hormone. 34-23. What is the carrier protein in iron in blood?
34-7. How parathyroid hormone regulates calcium level Transferrin.
in blood? 34-24. What is the storage form of iron?
Bone resorption; increased absorption of calcium from Ferritin.
intestines; increased absorption of calcium from renal 34-25. What is hemosiderin?
tubules. Excess iron is loaded as hemosiderin.
34-8. Hypocalcemia results in what condition? 34-26. What is haptoglobin?
Tetany. It is the carrier of free hemoglobin.
34-9. What is the cause for tetany? 34-27. What is hemopexin?
Hypoparalhyroidism. It is the carrier of free heme.
34-10. What are the features of hyperparathyroidism? 34-28. Anemia is resulted in the deficiency of?
Osteoporosis; hypercalciuria; urinary calculi. Iron, copper, vitamin C, folic acid, vitamin 8 12, pyridoxal
34-11. What is the normal level of phosphorus in blood phosphate.
3-4 mg/dl. 34-29. What are the causes of iron deficiency?
34-12. What is the normal level of sodium in blood? Nutritional. hookworm infection, repeated pregnancy,
136- 145 m Eq/L. chronic blood loss, nephrosis.
34-13. What is the daily requirement of iron for a normal 34-30. How iron deficiency manifests?
adult male? Microcytic hypochromic anemia.
10-20 milligram per day. 34-31 . What are the features of hemosiderosis?
34-14. What are the dietary sources of iron? Cirrhosis of liver: diabetes mellitus; yellow color of skin
Green leafy vegetables; jaggery. 34-32. What is the major cause for hemosiderosis?
34-15. Which trace element is deficient in milk? Repeated transfusion of whole blood.
Iron. 34-33. Name some copper containing enzymes.
34-16. What are important iron containing proteins? Ceruloplasmin, cytochrome oxidase, cytochrome C,
Hemoglobin, myoglobin, cytochromes, catalase, trypto- tyrosinase, lysyl oxidase, super oxide dismutase.
phan pyrrolase, xanthine oxidase, transferrin, ferritin. 34-34. What are the dietary sources of copper?
34-17. Which will increase iron absorption from intestines? Cereals, meat, liver.
Gastric HCI, ascorbic acid, cysteine. 34-35. What is ceruloplasmin?
34-18. What are the factors which will retard iron absorp- It is ferroxidase; it promotes oxidation of ferrous ion to
tion? ferric form.
Phytic acid in cereals; oxalic acid in leafy vegetables; 34-36. What are the characteristic features of Wilson's
calcium, zinc, lead and phosphates. hepatolenticular degeneration?
34-19. How is iron absorbed? Ceruloplasmin level in blood is decreased; copper
Iron in the ferrous state is binding to divalent metal excretion is reduced; copper is accumulated in liver to
transporter (DMT-1) present in mucosa I surface of produce cirrhosis.
intestinal cell; thus iron is transported into the mucosa! 34-37. What is the daily requirement of iodine?
cell. 150-200 microgram.
34-20. What is mucosal block? 34-38. What is the dally requirement of zinc?
Iron homeostasis is maintained by regulation at the 15-20 milligram.
level of absorption and not by excretion. When iron 34-39. Which enzymes contain zinc?
stores in the body are depleted, absorption is enhanced. Alkaline phosphatase; Amylase; Carbonic anhy-
When adequate quantity of iron is stored, absorption is drase; RNA polymerase. Zinc is required for insulin
decreased. This is referred to as "mucosa! block". secretion.
34-21. How is this mucosa! regulation takes place? 34-40. What is the importance of selenium?
Absorption of iron needs DMT and ferroportin. Synthe- It is an antioxidant.
sis of both these proteins is downregulated by hepcidin. 34-41 . Name the selenium containing enzyme.
34-22. What is hepcidin? Glutathione peroxidase.
_ _ _ _ _Chapter 35
.. Energy Metabolism
and Nutrition

Chapter at a Glance
The learner will be able to answer questions on the following topics:
Calorific value, respiratory quotient Nutritional values of proteins
Basal metabolic rate (BMR) Limiting amino acids, supplementation
Specific dynamic action (SDA) Kwashiorkor and marasmus
0 Proximate principles Prescription of the diet
Nitrogen balance Glycemic index

.. A sound knowledge of the principles of nutrition is of para- TABLE 35.1: Calorific value of nutrients
mount importance. Dietetics is the science of food and Energy yield 0 1 consumed co,
nutrients, their action, interaction and balance in health Nutrient kcaVg Ug producedUg RQ

and disease. The main purpose of the food is to provide Carbohydrates 4 0.829 0.829 1

energy for muscular activity and also to supply basic Fats 9 2.016 1.427 0.71

body building materials such as essential amino acids. Proteins 4.2 0.966 0.782 0.81
Alcohol 7 1.429 0.966 0.66

ICALORIFIC VALUE
The energy content of food materials is measured in ii. RQ of carbohydrates is 1; RQ of fats is 0.7; that of
calories. One calorie is the heat required to raise ~lie proteins is 0.8 (Table 35.1 ).
temperature of 1 g of water through 1°C. Since it is a iii. For a mixed diet it is between 0.7 and 1, often
very small unit, in medical practice, the energy content around 0.82-0.85. When the rate of utilization of
is usually expressed in kilocalorie (kcal) which is equal fat increases in relation to carbohydrates, RQ falls.
to 1000 calories. [One kilo calorie is equal to 4.2 kilo- This happens in diabetes mellitus, when utilization
joules (kJ)]. The calorific value of nutrients otherwise of carbohydrate is reduced. The RQ is lowest when
known as "energy density" (energy yield per unit weight ketolysis is very active.
of food) is given in Table 35.1.
Energy Requirements
Respiratory Quotient of a Normal Person
i. Respiratory quotient (RQ) is defined as the ratio of i. Maintenance of basal metabolic rate (BMR)
volume of CO2 produced in Ug to the oxygen con- ii. Specific dynamic action or thermogenic effect of food
sumed in Ug. iii. Extra energy expenditure for physical activities.
512 Section D: Nutrition

vi. Thyroid hormones: Since thyroid hormones have


a general stimulant effect on rate of metabolism and
heat production, BMR is raised in hyperthyroidism
and lowered in hypothyroidism. All other factors
(No. 1 to 6) are taken into account in the definition
of BMR. Thus, thyroid function determines the chan-
Wilbur Francis
Atwater Benedict ges in BMR.
(1844--1907) (1870-1957)
Normal Value for BMR
Basal Metabolic Rate
Since BMR is affected by body surface area, it is usually
Definition expressed in kilocalories per hour/square meter of body
The basal metabolic rate (BMR) is the energy required surface. Nomograms showing body surface area from
by an awake individual during physical, emotional and height and weight are also available. The BMR is then
digestive rest. It is the minimum amount of energy requi- calculated from the values of oxygen consumption,
red to maintain life or sustain vital functions like the calorific value and surface area. For adult men normal
working of the heart, circulation, brain function, respira- value for BMR is 34-37 kcal/m 2/hour, and for women,
tion, etc. The metabolic rate during sleep is less than 30-35 kcal/m2 /hour. For easier calculations, BMR for
BMR. BMR is measured directly by the heat evolved, or an adult is fixed as 24 kcal/kg body weight/day.
indirectly by the volume of oxygen consumed and car-
bon dioxide evolved per unit time. Specific Dynamic Action (SDA)
This refers to the increased heat production or increased
Measurement of BMR
metabolic rate following the intake of food (thermoge-
Procedure : Atwater-Benedict-Roth basal metabolism nic effect of food) (diet-induced thermogenesis). Part
apparatus (closed circuit method) is used. The subject of this is due to the expenditure of energy for digestion;
breathes in oxygen from a metal cylinder. The CO2 pro- absorption and active transport of products of the diges-
duced is absorbed in soda lime. The volume of oxygen tion. Another reason for this expenditure of energy is
consumed is recorded. that reserve materials, such as glycogen, triacylglycerol,
Calculation: The BMR is calculated from oxygen con- protein, etc. are synthesized from small molecules avail-
sumption, calorific value and surface area. Calorific able after digestion. This energy is trapped from previ-
value of oxygen is 4.8, that is, when 1 liter of oxygen is ously available energy, so that the actual energy from
utilized, 4 .8 kilocalories are generated. From the value the food is lesser than that of theoretical calculation .
of oxygen consumed, the heat generated in kilocalories SDA can be considered as the activation energy needed
can be calculated. for a chemical reaction. This activation energy is to be
supplied initially.
Factors Affecting BMR Suppose a person takes 250 g of carbohydrates;
i. Age: During the period of active growth, BMR is this should produce 250 x 4 = 1000 kcal. But before this
high. It reaches a maximum by 5 years of age. In energy is trapped, about 10% energy (= 100 kcal) is
old age BMR is lowered. drawn from the reserves of the body. Thus the next
ii. Sex: Males have a higher BMR than females. generation of energy is only 1000 minus 100 = 900 kcal. I

iii. Temperature: BMR increases in cold climate as a If the person wants to get 1000 kcal, he should take food
compensatory mechanism to maintain body tempe- worth 11 00 kcal. Thus additional calories, equivalent
rature. Eskimos have a higher BMR. to SDA has to be added in diet.
iv. Exercise: The increase in BMR during exercise is The values of SDA are: for proteins 30%, for lipids
due to increased cardiac output. 15%, and for carbohydrates 5%. This means that out
v. Fever: About 12% increase in BMR is noticed per of every 100 grams of proteins consumed, the energy
degree centigrade rise in temperature. available for doing useful work is 30% less than the
Chapter 35: Energy Metabolism and Nutrition 513

TABLE 35.2: Energy requirement and occupation TABLE 35.3: Calculation for energy requirement ror a 55 kg
person. doing moderate work
Type ofactivity Occupation
Light Office workers, Shopworkers
For BMR = 24x ss kg = 1320 kcal
+ For activity = 40% ofBMR = 528 kcal
Moderate Industry workers, Farm workers
Very active Agricultural workers, Unskilled laborers, Athletes
Subtotal = 1320 + 528 = 1848 kcal
= 1848 X 10% = 184 kcal
• Heavy work Construction workers
+ Need for SDA
Total =1848+ 184 =2032 kcal
Rounded to nearest multiple of SO = 2050 kcal
TABLE 35.4: Carbohydrates in common foods
1. Cane sugar 100% 2. Cassava (Tapioca) 85%
3. Rice 80% 4. Honey 80% vitamins are also to be provided. Further, additional
5. Wheat 70- 80% 6. Cakes 55-65%
requirements for growth, pregnancy, lactation and conva-
7. Bread
lescence are to be provided in the food.
50-60% 8. Potatoes 25%

calculated value. Hence for a mixed diet, an extra 10%


IIMPORTANCE OF CARBOHYDRATES
calories should be provided to account for the loss of The dietary carbohydrates provide a major fraction of
energy as SDA. It is a common experience that during the body's energy needs. Ideally carbohydrates may
hot weather following the consumption of a protein-rich provide about 60-65% of total calories. In addition to
meal, one feels hot and humid for a while. calories, the carbohydrates also provide dietary fiber.

Physical Activity Dietary Carbohydrates


i. The activity level may be divided into 3 groups- i. Available carbohydrates: These can be metabo-
sedentary, moderate and heavy. Additional calories lized by the body to give energy, e.g. starch and
... are to be added for each category. sugars.
ii. For sedentary work, +30% of BMR; for moderate ii. Unavailable carbohydrates: These cannot be
work, +40% of BMR; and for heavy work, +50% of assimilated and constitute only the dietary fi ber.
BMR should be added (Table 35.2). These are cellulose, lignin, pectin, etc.
iii. Requirement for energy during pregnancy is + 300 iii. The major dietary polysaccharide is starch. It is
kcal/day, and during lactation is + 500 kcal/day, in digested by amylase to maltose and then hydro-
addition to the basic requirements. lyzed to glucose. This glucose is the major source
iv. The energy requirement of a 55 kg male doing of fuel for most organs and tissues. Excess is con-
moderate work, may be calculated as shown in verted to fat and stored.
Table 35.3. iv. On cooking, starch is made more soluble and
accessible to digestive enzymes. Cereals, pulses
Requirements of Dietary Nutrients and tubers are the major sources of starch in the
diet. Table 35.4 gives a list of common food items
The recommended dietary allowance (RDA) provides
with their starch content.
extra provisions to prevent the development of deficie-
ncy. The RDA has been prescribed for all the essential
Sucrose
nutrients as per the stipulations of the WHO and FAO.
' The Indian Council of Medical Research (ICMR) has Cane sugar is mainly used as a sweetening agent. In
suitably modified these for Indian conditions (see also young children high intake of sucrose and sucrose-ri ch
Appendix V). food items predispose to the development of dental
caries. Sucrose is easily fermented by the bacteria
present in dental plaque, which would damage the
Proximate Principles
enamel and lead to caries (tooth decay).
In the diet proximate principles are carbohydrates, fat and While prescribing diets for diabetic patients and for
proteins. Moreover, required amounts of minerals and weight reduction, sucrose should be strictly avoided.
514 Section D: Nutrition

TABLE 3S.S: Cholesterol content of food items TABLE 3S.6: Fatty acids 1n oils
Food item Cholesterol content mg/ 100 g Mono- Poly-
Hen's egg, whole 300 Fotor oil Saturated (%) unsaturated(%) unsaturated(%)
Egg yolk 1330 Butter/ghee(*) 75 20 5
Liver 300-600 Safflower oil 9 12 79
2000 26 19
Brain
280
Cotton seed oil 65

Butter Coconut oil(') 86 12 2
Ghee 310 Ground nut oil 18 46 36
Meat and fish 40-200 (") Butter/ ghee contains short chain fatty acids and coconut oil
Milk 10 contains medium chain fatty acids, which are absorbed and meta-
bolized quickly

Jaggery, an alternative source of sucrose is beneficial, Cholesterol and Heart Diseases


since it is a good source of iron.
The atherogenic effect of cholesterol and the risk of coro-
Dietary Fiber nary artery disease in people with hypercholesterolemia
are described in Chapter 15. Food items known to be
The unavailable or indigestible carbohydrate in the diet
rich in cholesterol (egg yolk, liver, brain, kidney) are to
is called dietary fiber. Green leafy vegetables are good
be consumed in limited amounts. Table 35.5 gives a list
sources of fiber. Diet rich in fiber improves bowel motility,
of food items with their cholesterol content. Vegetables,
prevents constipation, decreases reabsorption of bile acids
cereals and pulses do not contain any cholesterol. On
thus lowering cholesterol level and improves glucose
the other hand, vegetable sterols will inhibit cholesterol
tolerance. Dietary fiber has hypoglycemic and hypolipi-
absorption.
demic effect. The inclusion of fiber rich food in weight
Saturated fats raise serum cholesterol; while unsatu-
reducing diets is found to be helpful, since it provides
rated fats lower it. Therefore unsaturated fat (vegetable
a feeling of fullness without consumption of excess
oils and fish oils) are to be preferred. The polyunsatu-
calories.
rated fatty acids (PUFA) are present in vegetable oils
and fish oils (see Chapter 8). They belong to essential
UTRITIONAL IMPORTANCE
fatty acids. They are precursors of prostaglandins and
F LIPIDS ----- leukotrienes. PUFAs are required for esterification and
excretion of cholesterol. They reduce the cholesterol
Neutral Fats or Triacylglycerols
level in blood and are antiatherogenic (see Chapter 15).
Fats provide a concentrated source of energy. In develo- The omega-3 fatty acids from fish oils decrease the
ped countries, the percentage of calories derived from plasma lipoproteins (VLDL and LDL), and thereby dec-
fats may be as high as 40%, but in the developing coun- rease the risk of coronary artery disease. The contents
tries it is much less, around 10%. A minimum intake of of PUFA in oils are given in Table 35.6. High fiber content
lipids is essential since the requirements of fat soluble also reduces serum cholesterol, lowers LDL fraction and
vitamins and essential fatty acids are to be met. Fats raises HDL fraction . Leafy vegetables and fruits contain
increase the taste and palatability of food. They are the good quantity of fiber.
favored cooking medium. Trans fatty acids (TFA) are atherogenic. They lower
Visible fat or fat consumed as such, e.g. butter, HDL level and elevate LDL level. TFA content is high in
ghee, oils. Recommended daily intake of visible fat is processed foods and bakery products, where hydroge-
10% of calories or 20 g/day. Invisible fat or fat present nated vegetable oils are used for cooking.
as part of other food items, e.g. egg, fish , meat, cereals,
nuts and oil seeds. Even cereals contain 1 g of fat per Recommended Daily Intake of Fat
100 g. More than half of essential fatty acid in Indian diet The ideal fat intake is about 15-20% of total calories,
is in the form of invisible fat. out of which about 25-30% may be PUFA. This will be a
Chapter 35: Energy Metabolism and Nutrition 515

i
TABLE 3S.7: Recommended protein allowances Body proteins

Infants 2.4 g/kg body wt/day Anabolism j Catabolism

Children up to l Oyears 1.75 g/kg body wt/day -+ Loss through skin


Amino acid (S), hair, sweat
Adult (Men and women) 0.8 g/kg body wt/day Absorption
pool or
Diet (I)
Pregnancy 2 g/kg body wt/day nitrogen pool -+ Urinary nitrogen (U)
.. Lactation 2.5 g/kg body wt/day
(urea, creatmlne)
......._. Feces (F)

F ig . 35.1: Nitrogen balance


total of about 20- 25 g of oils and about 3 g of PUFA for
a normal person.
l=U+F+S
Excess of PUFA: Anything in excess is deleterious.
When the excretion exceeds intake, it is negative
Excess PUFA may lead to production of free radicals
nitrogen balance. When the intake exceeds excretion, it
that may be injurious to the cell. PUFA should not be
is a state of positive nitrogen balance.
more than 30% of total fat.
Moreover, the fat content should be such that satura- Factors Affecting Nitrogen Balance
ted fatty acid (SFA) : monounsaturated fatty acid (MUFA): i. Growth: During the period of active growth, a state
polyunsaturated fatty acid (PUFA) may be in 1:1:1 ratio. of positive nitrogen balance exists. On an average
Further, cholesterol intake should be less than 250 mg/ when a person gains 5 kg , about 1 kg proteins are
day.
added to the body. For this, about 160 g of nitrogen

I IMPORTANCE OF PROTEINS has to be retained, so he/she has to be in positive


nitrogen balance.
Essential Amino Acids ii. Hormones: Growth hormone, insulin and andro-
Proteins form the building blocks for body tissues. Only gens promote positive nitrogen balance, while cor-
10- 15% of the total energy is derived from proteins. ticosteroids cause a negative nitrogen balance.
When enough carbohydrates are present in the diet, iii. Pregnancy: A pregnant woman will be in a state of
the amino acids are not used for yielding energy. This is positive nitrogen balance due to the growth of fetus.
iv. Convalescence: A person convalescing after
known as the protein sparing effect of carbohydrates.
an illness or surgery will be in positive nitrogen
During starvation , amino acids may act as energy sour-
balance, due to active regeneration of tissues.
ces. Proteins are the only source of essential amino
v. Acute illness: Negative nitrogen balance is seen
acids (see Box 3.1). The requirement of protein is
in subjects immediately after surgery, trauma and
shown in Table 35.7. As per the WHO/FAO recommen-
burns.
dation, the safe levels of protein intake for an adult is vi. Chronic illness: Malignancy, uncontrolled diabetes
0.75-0.8 g/kg/day. For the synthesis of body proteins, mellitus and other debilitating diseases show nega-
all the essential amino acids should be supplied in ade- tive nitrogen balance.
quate quantities at the same time. vii. Protein deficiency: The deficiency of even a single
The non-essential amino acids can be synthesized, essential amino acid can cause negative nitrogen
provided there is enough supply of proteins in total. Only balance. Prolonged starvation is another important
3 amino acids (alanine, aspartate and glutamate) are cause.
truly dispensable, as they can be synthesized from pyru-
vate, oxaloacetate and alpha-ketoglutarate respectively; Maintenance of Nitrogen Balance
and these precursors are generally available in plenty. Obligatory nitrogen loss is 3.5 g of Niday for a 65 kg
person (urinary, fecal and cutaneous loss). This could be
Nitrogen Balance equivalent to 22 g of protein.
A normal healthy adult is said to be in nitrogen balance Requirement for protein turnover. The minimum daily
(Fig. 35.1). because the dietary intake (I) equals the requirements to compensate for the above two categories
daily loss through urine (U), feces (F) and skin (S). are 0.75-0.8 g/kg wt of good quality protein.
516 Section D: Nutrition

Protein requirements for growth. This is applicable Zein + Tryptophan + Lysine


in the case of infants, children, adolescents, pregnancy,
lactation and convalescence. As growth stops, protein
requirement also decreases. Weight of animal

Zein + Tryptophan

Nutritional Values (Nutritional Indices) Zein + Lysine


Zein alone
Assessment of Nutritional Values
Whipple (Nobel Prize, 1934) introduced Days
plasmapheresis as a means to assess Fig. 35.2: Identifying the limiting amino acid
the nutritional value of proteins. Albumin
from blood is removed from the animal.
The time for regeneration of original level Lactalbumin Cereal+
George H + Zein pulse
of albumin in plasma was taken as a
Whipple
parameter of the quality of the food given NP 1934 Weight
Lactalbumin Pulses alone
1878-1976 of
to the animal. animal alone
The modern, easier way, to assess the nutritional
value of a protein, is to give that protein as the only Zem alone Cereal alone

source of nitrogen to an animal, and assess the weight Days Days


gain (Fig . 35.2). The protein content of various food
Fig. 35.3 : Two second class proteins, when combined, are equiv-
items are shown in Appendix VI. The following indices alent of the first class protein
are used to assess the nutritional value of proteins.
rat as the only source of protein, it fails to grow. This
Biological Value (BV) of Protein amino acid is said to be the limiting amino acid, Limit-
It is the ratio between the amount of nitrogen retained ing amino acid is that which limits the weight gain when
and nitrogen absorbed during a specific interval. a protein is supplied to an animal (Fig. 35.2).
BV = Retained nitrogen x
Absorbed nitrogen
100 Mutual Supplementation
Suppose 127 mg of protein "A" was consumed by a rat This problem may be overcome by taking a mixture of pro-
in a day and 4 mg is recovered in feces and 24 mg is teins in the diet. Mutual supplementation of proteins
seen in urine. Then is thus achieved (Table 35.8). For example, pulses are
Amount ingested = 127 mg deficient in methionine, but rich in lysine.
Amount absorbed = 127 - 4 = 123 mg On the other hand, cereals are deficient in lysine,
Amount retained = 123 - 24 = 99 mg but rich in methionine.
Therefore BV : 99/123 X 100: 81% Therefore a combination of pulses plus cereal (e.g.
chapafi + dal) will cancel each other's deficiency and be-
Net Protein Utilization (NPU) come equivalent to first class protein. The supplemen-
NPU = Retained nitrogen/intake of nitrogen x 100 tation effect of proteins may be seen in weight gain in
In the above example, animals (Fig. 35.3).
NPU of protein "A" = 99/127 x 100 = 78%
and for protein "B", it is 4.5/100 x 100 = 4.5%. IE_ROTEIN ENERGY MALNUTRITION
Thus NPU is a better index than BV to denote nutritional
It is the most widespread nutritional problem in develop-
quality and availability of a protein. ing countries; predominantly affecting children. At one
end of the spectrum of malnutrition is marasmus (Greek
Limiting Amino Acids word, "to waste"), which results from a continued severe
Certain proteins are deficient in one or more essential deficiency of both dietary energy and proteins (primary
amino acids. If this particular protein is fed to a young calorie inadequacy and secondary protein deficiency).
Chapter 35: Energy Metabolism and Nutrition 517

TABLE 3S.8: L1m1llng amino acids in proteins TABLE 3S.9 : Comparison between the salient features of
Kwash1orkor and marasmus
Protein supplemented to
Protein Limiting amino acid rectify deficiency Marasmus Kwashiorkor
Rice Lys, Thr Pulse proteins Age of onset Below 1 year One to 5 years

Wheat Lys, Thr Deficiency of Calorie Protein


Pulse proteins
Cause Early weaning and Starchy diet after weaning,
Gelatin
• Tryptophan Milk proteins
repeated infection precipitated by an acute
Zein Trp, Lys Meat proteins infection
Tapioca Phe, Tyr Fish proteins Growth Marked Present
Bengal gram retardation
Cys, Met Cereals
Attitude Irritable and fretful Lethargic and apathetic
Appearance Shrunken with Looks plump due to
At the other end of the spectrum is Kwashiorkor, where skin and bones edema on face and
isolated deficiency of proteins along with adequate only. Dehydrated lower lim bs
Appetite Normal Anorexia
calorie intake is seen. Kwashiorkor means "sickness
Ski n Dry and atrophic •crazy pavement
the older child gets, when the next child is born", a term
dermatitis" due to pealing,
from the local language of Ga tribe of Ghana. cracking and denudation
Hair No characteristic Sparse, soft and thin
Biochemical Alterations in PEM change hair; curls may be lost
Associated Other nutritional Angular stomatitis and
Table 35.9 gives a comparison between Kwashiorkor
features deficiencies; cheilosis are common;
and marasmus. Watery diarrhea; Watery diarrhea;
The hallmarks of Kwashiorkor are hypoalbumi- Muscles are weak Muscles undergo
and atrophic wasting
nemia, poor wound healing and edema .
Serum albumin 2to3g/dL < 2 g/dl
Hypoalbuminemia: Albumin values less than 2 g/dl
Serum cortisol Increased Decreased
is a biochemical marker in cases of Kwashiorkor. In
marasmus, this may not be so low. lgG increases due
to associated infections. the intellectual capacity in later life. There may not be
Fatty liver is seen in some cases of Kwashiorkor, any sequelae where the moderate and mild forms are
but not in marasmus. Fatty liver is due to decreased lipo- corrected in time.
protein synthesis.
Glucose tolerance is often normal , but hypoglyce- IOBESITY
mia may be seen in marasmic children.
Malnutrition may be of two types; undernutrition or over-
Hypomagnesemia is a usual finding.
nutrition. The latter is otherwise called obesity. Obesity
is the most prevalent nutritional disorder in prosperous
Treatment of Protein-Energy
developed countries. All over the world, obesity is preva-
Malnutrition
lent in affl uent people. This is because human race is
Optimal response is observed with diets providing accustomed to poverty and malnutrition from time imme-
150-200 kcal/kg body weight and 3-4 g of protein/kg morial, and the body is designed to store energy when-
body weight. A mixture of three parts of vegetable pro- ever available. This is the fi rst generation in history,
teins (Bengal gram or peanuts) and one part of milk where foodstuffs are in plenty. So, by habit people eat
protein is found to be very effective. It is monitored by
t more and get obese. Obesity is the condition in which
disappearance of edema, rise in serum albumin level
excess fat has accumulated. This is due to the increased
and gain in weight.
energy intake and decreased energy expenditure. The
obesity index (body mass index, BMI), is calculated as
Sequelae of Protein Calorie Malnutrition
W/H2 (where W =weight in kg and H =height in meters);
Severe malnutrition in early life can lead to permanent it is used to assess the obesity. A person is obese when
and irreversible physical and functional deficits. Severe BMI exceeds 27.8 kg/m2 in men and 27.3 kg/m2 in women
persistent malnutrition may have deleterious effects on (excess of 120% of desirable body weight).
518 Section D: Nutrition

Obesity can occur only as a result of ingestion of high levels of leptin in plasma. There is an "adipo-
food in excess of the body's needs. insular axis", with insulin promoting leptin secretion
and leptin inhibiting insulin release.
Diseases Related to Obesity 2. Neuropeptide-Y, a hypothalamic polypeptide, stimu-
Sensitivity of peripheral tissues to insulin is decreased. lates desire for carbohydrates. The action is to inhi-
The number of insulin receptors are decreased in adi- bit insulin secretion. ..
pose tissue cells. Plasma insulin levels may be elevated. 3. Ghrelin is secreted mainly by adipocytes. It stimulates
Obesity is associated with substantially increased cardio- hunger and appetite by acting on the hypothalamus.
vascular risk. Increased waist to hip ratio (abdominal Plasma level of Ghrelin is increased in fasting state,
obesity) is a greater risk. In metabolic syndrome, insulin which produces hunger signals.
resistance, hyperglycemia, hyperlipidemia (increased 4. Non-esterified fatty acids (NEFAs) are primarily
LDL and decreased HDL) and obesity are seen. released from adipose tissue during fasting . Circu-
The major ill effects of obesity are increased risk of lating NEFAs reduce glucose uptake, to promote
coronary artery disease, diabetes mellitus, hyper- lipid burning as a fuel source in most tissues, while
tension, metabolic syndrome and a reduced lifespan. sparing carbohydrate for neurons.
Calorie-fat-restricted diet may retard aging process and
extend the lifespan. l fRESCRIPTION OF DIET _
General Principles
Treatment of Obesity
Recommended dietary allowances (RDA) of nutrients
Lifestyle modification is the best suitable remedy. The are given in detail in Appendix V.
goal is to reduce the intake of calories and fat. Frequent i. The ideal body weight: The underweight person
small meals with lots of vegetables will make the food should be given more nutrients and the overweight
palatable and give a feeling of satiety. Controlled exer-
cise is very useful.
person should reduce calorie intake. The ideal body
weight is calculated from height.

ii. Protein requirement: 1 g per kg body weight for
Regulators of Appetite adults and 2 g per kg for children. During preg-
Hypothalamus has the central control of appetite. Psy- nancy and lactation, protein requirement is about
chologic, genetic, neural and humoral factors are invol- 2.5 g per kg.
ved in the control. Polypeptides that increase appetite iii. Calorie requirement: This depends on age, sex,
height, weight, health status and above all on the
are: Neuropeptide Y (NPY), Ghrelin, Polypeptide YY
physical activity and occupation. As a rule of thumb,
(PYY), Insulin and Cortisol. Appetite decreasing fac-
the calorie requirement for a person is taken as
tors are: leptin, melanocyte stimulating hormone (MSH),
30-35 kcal per kg of ideal body weight. For seden-
and Serotonin.
tary habits, lower values (30 kcal) and for moderate
muscular activities, higher values (35 kcal) are pre-
Adipose Tissue Talks to
scribed.
Brain through Factors
Considering the average weight of adult Indians
1. Leptin (Greek leptos = thin) is a hormone secre- between 55 and 65 kg, average calorie requirement
ted by adipocytes. It is mainly produced by white will be between 1700 and 2300 kcal per day.
adipose tissue. It functions as a satiety signal. It is An extra allowance of 300 kcal is necessary dur-
an index of the energy reserve in the body. When ing pregnancy and lactation periods.
the energy reserve is adequate, leptin levels are iv. Specific dynamic action : Extra energy is to be
increased and this would suppress further food supplied to account for SDA.
intake. Leptin inhibits neuropeptide-Y secretion, and v. Ratio: Balanced diet should contain calories from
so when fat depots are full, appetite is decreased. carbohydrate, proteins and fat in the ratio of
Obesity is associated with leptin resistance and 60:20:20.
Chapter 35: Energy Metabolism and Nutrition 519

TABLE 35.10: First step in the prescription of diet TABLE 35.11: Prescription of diet. 2nd step
Energy required + SDA 2000kcal Proteins : 60g
Protein : 60g Fats : 35 g
Calcium : 400mg Carbohydrates : 350g
Iron : 25 mg Calories : 2000 kcal
:
• Calcium
Iron :
400 mg
25 mg
TABLE 35.12: A diet for a 60 kg sedentary man
Item Quantity vegetarian Quantity non-vegetarian
Cereals 350g 350 g Fourth Step: Food Items
Pulses 75 g 60g
A judicious combination of cereals and pulses provide
Vegetable oil 40 ml 25 ml
all the essential amino acids (pulses are deficient in
Milk 250ml 150 ml
methionine, while cereals lack in lysine). (See mutual
Leafy vegetable 200g 200g
supplementation, described previously). An accepted
Sugar 25 g 25 g
formula is that the food should contain pulses and cereals
Fish/ meat - 60g
in the ratio of 1 :5 to provide good quality proteins.
First Step: Calorie Requirement Supplementing the cereals with animal proteins is
certainly good to improve the quality, but at a higher cost.
For a 60 kg sedentary man , the energy requirement is Fruits and vegetables will provide vitamins and minerals.
60 x 30 = 1800 kcal plus additional allowance for specific Vitamin A is available in ripe mango, papaya, carrot, beet
dynamic action (1800 x 10% = 180). Therefore the total root, and other leafy vegetables. B complex vitamins
requirement is roughly 2000 kcal. The recommended may be supplied by cereals (parboiled rice, whole wheat
dietary intake for a 60 kg sedentary man, based on the powder) and green leafy vegetables. Major sources of
above principles is given in Table 35.10. calcium are milk, cereals, pulses and green leafy vegeta-
t bles. Iron is mainly supplied by pulses, cereals, vegetab-
Second Step: Proximate Principles les, meat and liver. Jaggery is a good source for iron.
He requires 60 g proteins. This will give 60 x 4 = 240 kcal Jaggery is therefore superior to refined sugar, especially
of energy. His total requirement is 2000 kcal. Therefore for children. Milk is a very poor source for iron.
carbohydrates plus fats should provide (2000 - 240) =
1760 kcal. As a general rule, about 20% of total calo- Fifth Step: Prescribing a Normal Diet
ries are supplied by fat. Therefore, fats should supply The requirements shown in Table 35.11 may be trans-
1760 x 20% = 350 kcal which is provided by (350/9) = lated into Table 35.12. This will satisfy the requirements
about 35 g of fats. (About 30% of the total fat may be regarding protein (60 g); fats (45 g), calories (2000 kcal),
supplied as poly unsaturated fatty acids). calcium (400 mg) and iron (25 mg). See that cereals-
The rest 1400 kcal are supplied by 350 g of carbohy- pulses ratio is maintained at 5:1. When calories alone
drates. These calculations are based on the fact that 1 g are to be increased, as in the case of a person having
carbohydrate provides 4 kcal, 1 g fat supplies 9 kcal and severe muscular exercise, tubers and roots will serve
1 g protein gives rise to 4 kcal. Thus the requirements this purpose.
calculated in Table 35.10 may be rewritten as in Table The basic principles are summarized in Box 35.1 .
35.11 .
Diet for Patients with Diabetes
Third Step: General
Management of diabetes gives great emphasis for die-
Composition of Food tary control and exercise. The main aim is to keep the
The third step is to calculate how these proximate blood HbA 1c level to nearly normal values. Glucose level
principles are supplied as common foodstuffs. For this as near the normal values, for as many hours, as possi-
exercise we should familiarise with the nutritive value ble in a day. The calorie requirement is distributed for
of foodstuffs. Detailed values are shown in Appendix a diabetic patient: Carbohydrate 60-70%, fat 15-25%
VI of this book. and protein 15-20%.
520 Section D: Nutrition

BOX 35.1: Import,mt prnnts for prescribing a diet


175
1. It should be a balanced, well planned diet containing all essen-
t ial nutrients. 150
2. The diet should be simple, locally available, palatable and
digestible.
125
Plasma
3. Adequate protein content with essential amino acids should be glucose 100
supplied. This is achieved by a cereal-pulse mixture with addi- mgldl
75
tional animal proteins, if necessary.
4. Calorie intake should be correct and should balance energy 50
expenditure.
25
5. Special cctre should be taken to see that adequate quantity of I
calcium and iron are obtained from the diet. The absorption of Meal 1 hr 2 hr 3 hr
these minerals is reduced by other factors in Indian diet.
6. Should have variety and should not differ very much from the Fig. 35.4 : Glycemic index curve
habitual diet of the person.
7. Should provide adequate rou hage. will not increase the blood glucose to the same extent.
because digestion and absorption are slow. So the
TABLE 35.13: Glycem1c index of food items glycemic index of complex carbohydrate is lesser than
ltemoffood G/ycemic index
cane sugar (refined sugar).
Potato chips 80-90
As a general rule, the glycemic index of carbohy-
Bread 70- 79
drate is lowered if it is combined with protein, fat or fiber,
White rice (polished) 70- 79
60-69 preferably at least two of the three. The glycemic index
Parboiled (brown) rice
Bananas 60- 69 of some of the foods are shown in Table 35.13.
Beans, peas 40-49 Although ice cream contains sugar, it has a low gly- •
Legumes, peanuts 35-40 cemic index, because it contains lot of fat which pre-
Milk 35- 40 vents absorption. Thus ice cream may be occasionally I
Ice cream 35-40
consumed by diabetic patients.

0
a) Sugar, sweet and refined carbohydrates are avoided. • · Clinical Case Study 35.1
b) Leafy vegetables are increased, and tubers are res-
A 2-year-old boy was brought to the hospital. He was
tricted.
c) Frequent small meals are prescribed, with distribu- eating poorly for the last one month, had intermittent
tion of calories such that, breakfast 15%, mid-morn- diarrhea, and had become irritable and apathetic. On
ing snack 5%, lunch 30%, evening tea 10%, dinner examination, he was underweight for height and small
35%, bedtime snack 5%. for age. He was pale, weak, skin was flaky, hair was
brittle, abdomen was distended, liver was moderately
Glycemic Index enlarged and generalized edema was present. Labo-
It is assessed by the glycemic response after the particu- ratory tests were as follow: Hemoglobin-6.5 g/dL, Total
lar diet and comparing it with a reference meal. The refer- protein-4.0 g/dL, Albumin- 1.8 g/dL. What is the prob-
ence meal is always taken as 50 g of glucose (Fig. 35.4). able diagnosis?

Glycemic Index=
Area under glucose tolerance
a0 Clinical Case Study 35.2 I

curve after 50 g test meal


X 100 An 8-month-old girl was brought to the clinic in an irri-
Area under curve after 50 g of
table state. Weight was much lower than expected,
reference meal (glucose)
mid-arm circumference and triceps thickness were very
Simple carbohydrates such as glucose or sugar will low for age. Creatinine-height index was low and serum
have a high glycemic index. But the same quantity of albumin was normal. The mother tells that she had
complex carbohydrates (such as starch or dietary fiber) stopped breastfeeding at the age of 6 months and was
Chapter 35: Energy Metabolism and Nutrition 521

now giving only formula milk (which was diluted). What that increase metabolic demands like infections, hyper-
is the probable diagnosis? thyroidism, burns, trauma, etc. Edematous PEM is
0 Kwashiorkor and non-edematous PEM is marasmus.
• • · Clinical Case Study 35.3 Another type is known as marasmic-Kwashiorkor PEM .
All organs are affected in malnutrition. Dietary pro-
A 35-year-old female presented with complaints of alter-
teins, energy and micronutrients become deficient.
nating diarrhea and constipation. She reported abdominal
Weight loss, immunodeficiency, neurological changes,
discomfort and bloating that were relieved by bowel move-
fatty degeneration and other effects are seen. Labora-
ment. The episodes were worse in times of stress. She
tory tests include CBC, measures of protein nutritional
denies any blood in the stools, weight loss and anorexia.
status assessment like serum albumin, retinal-binding
No history of fever or drug intake. Physical examination
protein, prealbumin, transferrin, creatinine and BUN
was within normal limits. She was prescribed a cellulose
levels, serum electrolytes, blood glucose, urinalysis and
containing dietary supplementation to increase the bulk of
culture and arterial blood gases (often metabolic acidosis
stools. What is the likely diagnosis? What is the effect of is present). If child has history of abnormal stools, stool
the treatment modality prescribed? specimens should be checked for ova and parasites.
0 Best treatment is oral balanced diet, if child can take
•• Clinical Case Study 35.1 Answer oral feeds. Severe PEM or prolonged starvation requires
The child is suffering from protein energy malnutrition treatment in a hospital with controlled diet.
(Kwashiorkor). This is a severe form of childhood mal- 0
nutrition characterized by edema, irritability, anorexia, •i i• Clinical Case Study 35.3 Answer
ulcerating dermatitis, enlarged liver with fatty infiltrates The patient is likely suffering from irritable bowel syn-
and when well advanced, there may be inadequate drome, a gastrointestin al disorder of unknown etiology.
growth, lack of stamina, loss of muscle tissue, increased Common symptoms include abdominal cramps or pain,
susceptibility of infections, vomiting, diarrhea, anorexia bloating and flatulence. There might be alternating peri-
and edema. Eventually there is stupor, coma and death. ods of diarrhea and constipation and passing mucus in
The condition may develop when mother weans the the stools. Vomiting, blood in stools, pain that interrupts
child from breast milk and replaces the diet with food sleep, fever and weight loss are rare.
rich in carbohydrates, and deficient in proteins. Along Diagnosis is made by excluding other causes. Stool
with protein deficiency, micronutrient and antioxidant bulking agents, antispasmodics, antidiarrheal agents,
deficiencies, like iron, folate, iodine, selenium, vitamin antidepressant drugs, serotonin receptor agonists and
C, glutathione, albumin, vitamin E and PUFA, may play antagonists and chloride channel activators are used for
an important role. Treatment includes treatment of dehy- therapy. Dietary fibers are the indigestible part of plant
dration, antibiotics, a diet providing adequate proteins, foods that make stool soft and enables smooth bowel
vitamins, minerals and electrolytes. movements, prevent constipation hemorrhoids and diver-
ticulosis. Soluble fibers lower total and LDL cholesterol
a0 Clinical Case Study 35.2 Answer and help to prevent ischemic heart disease and stroke.
Soluble fibers in excess can cause abdominal bloat-
• The child is suffering from nutritional marasmus, a type ing and flatulence, dehydration and pectins can reduce
of protein energy malnutrition (PEM). There is inade- absorption of cholesterol-lowering drugs like lovastatin.
t quate intake of proteins and calories leading to emacia- Insoluble fibe rs present in food include cellulose,
tion. Low body weight, diminished skin fold thickness, hemicelluloses, and lignins. Excess of insoluble fibers
and reduced arm muscle circumference reflect loss of can result in excessive gas , constipation and intesti-
fat and catabolism of proteins from the body. All avail- nal obstruction, diarrhea in sensitive patients, and can
able fat stores have been exhausted due to starvation. reduce absorption of calcium, iron, copper and zinc,
PEM might be (1) primary (due to lack of food) or (2) especially in children. Insoluble fibers consumed on
secondary, due to disorders affecting GI function, wasting empty stomach can aggravate symptoms of irritable
diseases like AIDS, cancer, renal failure, or conditions bowel syndrome.
522 Section D: Nutrition

I LEARNING POINTS, CHAPTER 4. Increased heat production following intake of food


is referred to as specific dynamic action. Values
1. One calorie is the heat required to raise the tempera-
of SDA for proteins, lipids and carbohydrates are
ture of 1 g of water through 1°C. 1 kcal = 4.2 kJ.
30%, 15%, and 5%, respectively.
2. Respiratory quotient is defined as the ratio of the 5. Dietary fibers are essential to maintain normal
volume of CO2 produced to the 0 2 consumed. Car-
motility of GI tract, prevent constipation, decrease
bohydrates, fats, proteins have RQ's of 1, 0. 7, and
cholesterol levels and to improve glucose tolerance.
0.8, respectively.
6. Biological value of a protein is the ratio of the
3. BMR is defined as energy required by an awake indi-
amount of nitrogen retained to that absorbed dur-
vidual during physical, emotional and digestive rest. ing a specific interval.
BMR can be measured using Benedict's Roth appa-
7. Marasmus and Kwashiorkor are two conditions of
ratus. Normal value for men is 34-37 kcal/m2/hour protein energy malnutrition.
and for women is 30-35 kcal/m2/hour.

PART-1: ESSAY AND SHORT NOTE QUESTIONS

35-1 . Define BMR. What are the factors that affect BMR?
35-2. What is a balance diet? How do you prepare a diet for a normal young adult male of sedentary habits?
35-3. What is the nutritional importance of dietary proteins? Explain how the dietary deficiency of proteins will affect
growing children.
35-4. An adult male is consuming 350 g carbohydrate, 70 g protein and 60 g lipid per day. Calculate the total caloric
content of his diet. Calculate the percentage of calories obtained from carbohydrates lipids and proteins. Are
the percentages of calories that have obtained from these macronutrients conforming to the requirements of a
balanced diet?

SHORT NOTE QUESTIONS

35-5. Basal metabolic rate. 35-11 . Essential amino acids.


35-6. Specific dynamic action. 35-12. Essential fatty acids.
35-7. Biological value of proteins. 35-13. Protein sparing effect of carbohyd rates.
35-8. Protein efficiency ratio. 35-14. Protein cal orie malnutrition.
35-9. Nitrogen balance. 35-15. Kwashiorkor.
35-10. Dietary fiber.

PART-2 : MULTIPLE CHOICE QUESTIONS

35-1 . Negative nitrogen balance is observed in 35-4. Methionine is deficient in


A. Pregnancy B. Chronic fever A. Cereals (rice, wheat)
C. Convalescence D. Growth period B. Maize and corn
35-2. Positive nitrogen balance observed in all the follo- C. Pulses (Bengal gram etc.)
wing conditions, except: D. Tapioca (cassava) •
A. Pregnant woman of 25 years 35-5. Which has the highest calorific value?
B. Obese diabetic of 60 years A. Glucose B. Palmitic acid
C. Young athlete of 15 years C. Albumin D. Ethanol •
D. 25-year-old man during convalescence after 35.6. The respiratory quotient (RQ) of a person oxidizing
surgery mainly fat is
35-3. Lysine is deficient in A. 0.5 B. 0.7
A. Cereals (rice, wheat) C. 0.9 D. 1.0
B. Maize and corn 35-7. Calories generated per gram of fat is
C. Pulses (Bengal gram etc.) A. 4 kcal B. 5 kcal
D. Tapioca (cassava) C. 8 kcal D. 9 kcal
Chapter 35: Energy Metabolism and Nutrition 523
35-8. Basal metabolic rate is increased by all the follow- A. Respiratory quotient
ing, except B. Specific dynamic action
A. Fever B. Thyroxine C. Nature of work
C. Starvation D. Cold climate D. Basal metabolic rate
35-9. The beneficial effects of dietary fiber include all 35-20. Nutritional status of a person can be assessed by
the following, except. all, except:
A. Increased motility of intestine
' B. Helping in the digestion process
A. Serum protein estimation
B. Hemoglobin level
C. Decreased absorption of cholesterol C. Gain or loss in weight
D. Increased glucose tolerance D. Mid-arm circumference
35-10. A balanced diet should have calories for carbohy- 35-21 . Consumption of refined sugar can lead to all the
drate, proteins and fats in the following ratio
following effects, except.
A. 50:15:35 B. 40:30:30
A. Sudden hyperglycemia
C. 60:20:20 D. 70:10:10
B. Hypertriglyceridemia
35-11 . Calorie requirement of a 60 kg person doing
C. Glycogen accumulation
sedentary work is approximately
D. Dental caries
A. 1000 kcal B. 1500 kcal
35-22. Which of the following pairs of food items will not
C. 2000 kcal D. 2500 kcal
provide all essential amino acids?
35-12. The nutritional quality of a protein is dependent on
A. Fish and tapioca B. Chapathy and dal
the following, except.
C. Rice and curds D. Bread and jam
A. Content of essential amino acids
35-23. Which of the following is the best index of the nutri-
B. Digestibility
tive value of a protein?
C. Relative proportion of amino acids
A. Biological value
D. Energy yield from the protein
• B. Protein energy ratio
35-13. The daily nitrogen excretion in a normal healthy
C. Net protein utilization
adult consuming 100 gm of protein will be
D. Net dietary protein value
• A. 8gm B. 10gm
35-24. Deficiency of proteins in the diet can lead to the
C. 16 gm D. 20 gm
following, except.
35-14. The minimum daily requirement of proteins in food
A. Positive nitrogen balance
for a normal healthy adult is
B. Edema
A. 0.2 g/kg body weight
C. Recurrent infection
B. 0.4 g/kg
D. Loss of appetite
C. 0.7 g/kg
35-25. Negative nitrogen balance may occur due to all the
D. 0.9 g/kg
following conditions, except:
35-15. Phenylalanine is deficient in
A. Vegetarian diet B. Cytotoxic therapy
A. Cereals (rice, wheat)
C. Extensive burns D. Traumatic hemorrhage
B. Maize and corn
35-26. All the following diseases may be associated with
C. Pulses (Bengal gram etc.)
obesity, except:
D. Tapioca (cassava)
A. Grave's disease
35-16. Which of is not an essential nutrient?
B. Cushing's syndrome
A. Sucrose B. Linoleic acid
C. Tryptophan D. Ascorbic acid C. Depressive psychosis
" 35-17. The following carbohydrate is undigestible D. Diabetes mellitus
A. Lactose B. Glycogen 35-27. All the following abnormalities are observed in
obese people, except:
C. Fructose D. Pectin
35-18. A person with 60 kg weight, was given food con- A. Decreased glucose tolerance
taining 65 g of proteins per day for two weeks. After B. High insulin levels
the experiment, his weight was found to be 60 kg C. Hypoalbuminemia
itself. His nitrogen excretion is said to be D. Hyper lipidemia
A. Minus 5 g B. Minus 2.5 g 35-28. A child with Kwashiorkor may have all the follow-
C. 0 g D. Plus 2.5 g ing clinical features, except:
35-19. Which has no role in calculating calorie require- A. Hypoalbuminemia B. Hypercaicemia
ments? C. Hypomagnesemia D. Normoglycemia
524 Section D: Nutrition

ANSWERS OF MULTIPLE CHOICE QUESTIONS

35-1.
35-8.
B
C
35-2.
35-9.
B
B
35-3.
35-10.
A
C
35-4.
35-11 .
C
C
35.5.
35-12.
B
D
35.6.
35-13.
B
C
35.7.
35-14.
D
C
..
35-15. D 35-16. A 35-17. D 35-18. C 35-19. A 35-20. D 35-21. C
35-22. D 35-23. C 35-24. A 35-25. A 35-26. A 35-27. C 35-28. B

PART-3: VIVA VOCE QUESTIONS AND ANSWERS

35-1. What is the calorific value of carbohydrates? 35-15. What is the recommended daily allowance of pro-
4 kcal per gram tein for a normal adult?
35-2. How much calories are generated from fat ? 1 g/kg body weght.
9 kcal per gram of fat 35-16. Negative nitrogen balance is observed in which
35-3. What is the respiratory quotient? conditions?
It is the ratio of carbon dioxide produced to the oxygen Chronic infection; old age, malnutrition.
consumed. 35-17. Positive nitrogen balance is observed in which
35-4. What is the respiratory quotient of carbohydrates? conditions?
One. Pregnancy; convalescence, growth period.
35-5. The respiratory quotient for a mixed diet? 35-18. Lysine is deficient in which foodstuff?
About 0.82. Pulses.
35-6. What is specific dynamic action (SDA)? 35-19. Phenylalanine is deficient in which food stuff?
Increased heat production after intake of food. It is the Tapioca.
extra heat produced by the body, over and above the 35-20. Methionine is deficient in which food stuff?
calorie value of the given food . It is also called thermo- Cereals. •
genie action of food. 35-21 . How all amino acids are made available in a mixed
35-7. Wh ich foodstuff has maximum specific dynamic diet?
action? Mutual supplementation by combining cereals and
Proteins. pulses.
35-8. BMR (basal metabolic rate) is increased in which 35-22. What is protein calorie malnutrition (PCM)?
conditions? Deficiency of protein causes Kwashiorkor. Deficiency
Fever; Starvation; Cold climate; Increased thyroid hor- of calorie produces marasmus. Both are two spectra
mones. of PCM.
35-9. Increased basal metabolic rate is observed in which 35-23. What are the salient features of Kwashiorkor?
clinical condition? Hypoalbuminemia, hypomagnesemia, growth retarda-
Grave's disease (Hyperthyroidism). tion, lethargy, loss of appetite.
35-10. What are the beneficial effects of dietary fiber? 35-24. How much energy is required for a 60 kg person,
Increased motil ity of intestine; prevents constipation; with sedentary work?
decreased absorption of cholesterol; increased glu- 2000 kcal.
cose tolerance. 35-25. A balanced diet should have calories for carbohy-
35-11 . Which food stuffs contain polyunsaturated fatty drate, proteins and fats in which ratio?
acids (PUFA)? 60:20:20.
Vegetable oils, such as sunflower oil, groundnut oil. 35-26. What is the optimum ratio for cereals an d pulses in
35-12. Name polyunsaturated fatty acids. a balanced diet?
Linoleic acid; Linolenic acid; Arachidonic acid. Cereals:pulses ratio= 5: 1.
35-13. What is the dietary advice for a patient with hyper- 35-27. What is the recommended ratio of saturated, mono-
cholesterolemia? unsaturated and polyunsaturated fatty acids in a
They should reduce intake of food containing cho- balanced diet?
lesterol and increase intake of polyunsaturated fatty SFA:MUFA:PUFA = 1:1 :1 ratio.
acids. 35-28. What is the recommended fat intake?
35-14. What foodstuffs contain high cholesterol? The ideal fat intake is about 20% of total calories, out
Egg, butter, animal fat, meat (Table 35.5). of which about 30% be of PUFA.
_ _ _ _ _Chapter 36
Detoxification and
Biotransformation of
Xenobiotics
Chapter at a Glance

The learner will be able to answer questions on the following topics:


0 Detoxification mechanisms O Phase two reactions
0 Phase one reactions O Glucuronic acid, sulfate methylation
0 Oxidation, reduction, hydrolysis O Phase three reactions

• Biotransformation is the process whereby a substance more toxic than the parent substance. This is known
is changed from one chemical to another by a chemi- as bioactivation or toxication. An example is the bio-
cal reaction within the body. For some drugs, it is not transformation of vinyl chloride to vinyl chloride epoxide,
the absorbed drug, but a metabolite that has therapeutic which covalently binds to DNA and RNA, a step leading
action . For example, phenoxybenzamine, a drug given to cancer of the liver. The compounds that are detoxified
to relieve hypertension , is biotransformed into a metabo- include:
lite, which is the active agent. a. Compounds accidentally ingested like preservati-
Biotransformation also serves as an important ves, food additives and adulterants.
defense mechanism in that toxic xenobiotics and meta- b. Drugs taken for therapeutic purposes.
bolites are converted into less harmful substances that c. Compounds produced in the body which are to be
can be excreted from the body. eliminated, e.g. bilirubin and steroids. Bilirubin is
In general, biotransformation reactions generate toxic to the brain. Biotransformation of the lipophilic
more polar metabolites, that are readily excreted from bilirubin molecule in the liver results in the pro-
the body. The liver plays the most important role in the duction of water-soluble (hydrophilic) metabolites
biotransformation reactions. excreted into bile.
The biochemical processes whereby the noxious d. Compounds produced by bacterial metabolism,
substances are rendered less harmful and more water e.g. amines produced by decarboxylation of amino
soluble, are known as detoxification. Lipophilic toxi- acids:
cants are hard for the body to eliminate and can accu- Histidine -> Histamine
mulate to hazardous levels. Lysine -> Cadaverine
Xenobiotics are compounds which may be acciden- Ornithine --> Putrescine
tally ingested or taken as drugs or compounds produced Tyrosine --> Tyramine
in the body by bacterial metabolism (Greek, xenos = Tryptophan -> Tryptamine
strange). Molecules to be eliminated/detoxified are called The transformation of a specific xenobiotic can be
xenobiotics. either beneficial or harmful, and perhaps both depending
Biotransformation is not exactly synonymous with on the dose. A good example is the biotransformation of
detoxification, since in many cases, the metabolites are acetaminophen, a commonly used drug to reduce pain
526 Section D: Nutrition l

OH

O cH
,T) O cH,-oH + 02

0 - P
_ha_se 1_
6
Toluene NADPH NADP• (Benzyl alcohol)
Benzene Phenol Phenyl sulfate

Fig. 36.1: Phase one; oxidative reaction Fig. 36.2: Sometimes both phase one and two reactions are
needed to detoxify a compound

and fever. It normally undergoes rapid biotransformation (detoxification). Sometimes this may result in increased
with the metabolites quickly eliminated in the urine and toxicity (entoxification), e.g. methanol to formic acid.
feces; hence no toxicity is observed . The phase one reactions include hydroxylation, oxida-
But at high doses, the normal level of enzymes tion, reduction , hydrolysis, dealkylation, epoxidation, etc.
may be saturated . The excess acetaminophen under- The products of metabolic transformations are
goes additional biosynthetic pathway, which produces a either excreted directly or undergo further meta-
metabolite that is toxic to the liver. bolism by phase two reactions. They involve con-
jugation with a conjugating agent, thus converting
I CYTOCHROME P450 lipophilic compounds into water soluble, easily excre-
table forms. Phase two reactions are sulfation , acetyla-
The cytochrome P450 enzymes are involved in the bio-
tion, methylation and conjugation with glucuronic acid ,
transformation reactions. They are heme-containing
glutathione or glycine.
..
enzymes, localized in the endoplasmic reticulum of
In some instances, products of phase two reactions
liver.
may further be metabolized by phase three reactions.
NADPH (and not NADH) is the coenzyme for all the
P450 enzymes. One atom of oxygen is inserted into the
substrate. IPHASE ONE REACTIONS
Almost all common drugs are metabolized by the
Oxidative Reactions
P450 system. They are inducible enzymes. Phenobar-
bital causes increased activity of P450. The anticoagu- It may be either aromatic or aliphatic hydroxylation.
lant Warfarin is metabolized by CYP2C9. If the same The reactions also include sulfoxidation, N-oxidation
patient is taking phenobarbitone, the enzyme is induced, and epoxidation. For example, toluene is hydroxylated
the level of the enzyme is increased, warfarin is broken to benzyl alcohol by mixed function oxidase system
down quickly, and the dose of warfarin becomes inade- (Fig. 36.1). Sometimes both phases one and two reactions
quate. are necessary. The biotransformation of benzene req-
Some of the isoforms of the enzyme exhibit low uires both phase one and phase two reactions. Ben-
catalytic activity (polymorphism). This explains the vari- zene is biotransformed initially to phenol by a phase one
ation in drug responses among different persons. For reaction (oxidation). Phenol has the functional hydroxyl
example, there are 3 alleles for the enzyme which cata- group that is then conjugated by a phase two reaction
lyze nicotine of tobacco. The person with the inactive (sulfation) to phenyl sulfate (Fig. 36.2).
allele is protected against addiction to tobacco. The oxidation and detoxification of alcohol is also
an important function of the liver. Two enzymes are
Phases of Detoxification Processes involved in this process: alcohol dehydrogenase oxidizes
Biotransformation reactions are usually classified as alcohol to aldehyde; and aldehyde dehydrogenase oxi-
phase one and phase two reactions. dizes aldehyde to acid (Fig. 36.3).
Phase one is the alteration of the foreign mole- Oxidation of some compounds may result in produc-
cule, so as to add a functional group, which can be tion of substances which are more toxic, e.g.
conjugated in phase two. Phase one reactions result in Methanol ---+ Formic acid
the formation of compounds with decreased toxicity Ethylene glycol _. Oxalic acid
Chapter 36: Detoxification and Biotransformation of Xenobiotics 527

0 6
CH3-CH2-0H - ---T"-:-=
""""'----+• CH3-CHO
Ethyl alcohol ( + Acetaldehyde

NAO+ NAOH+H+


CH 3- CHO
Acetaldehyde n
- - -------::-=------+•

NAO+ NADH+H+
CH3-COOH
Acetic acid
Nitrobenzene Amlnobenzene (Aniline)

Fig . 36.4: Phase one; reductive reaction

e.g. aspirin, acetanilide, procaine, xylocaine, aliphatic


esters, diisopropyl fluorophosphate (DFP), etc. (Fig. 36.5).
O CHO /\. O COOH Aspirin is the drug most widely used in clinical prac-
tice . It has analgesic, antipyretic and anti-atherogenic
Benzaldehyde NAO+ NADH+H• Benzolc acid activities.
Fig. 36.3: Oxidation of alcohol groups
I PHASE TWO REACTIONS;

(5i
CONJUGATIONS
Acetanlllde Aniline + acetic acid
H 3C-CO A xenobiotic that has undergone a phase one reac-

6
I CH,-COOH tion is now a new metabolite that contains a reactive
chemical group, e .g . hydroxyl (-OH), amino (-NH 2),
and carboxyl (-COOH ). These metabolites must un-

,. dergo additional biotransformation as a phase two


reaction. Phase two reactions are conjugation rea c-
tions, that is, a molecule normally present in the body
~ COOH ~ COOH is added to the reactive site of the phase one me-
V - o--e0-cH3 tabolite. In most cases, the conjugation will make the
V - oH
compounds nontoxic and easily excretable. Conjugat-
Aspirin (acetyl salicylic acid) Sallcyllc acid + acetic acid
ing agents and their active forms are shown in Table
36.1. Glycine and glutamine can also act as conjugat-
Diisopropyl fluoro- HF + dialkyl phosphate
phosphate (DFP) ing agents.

Fig. 36.5: Phase one. Examples of hydrolysis


Glucuronic Acid
Reduction Reactions Glucuronide conjugation is the most common phase
The major group of compounds which are reduced and two reactions. Bilirubin is a good example for a com-
detoxified by the liver are nitro compounds. These are pound conjugated and excreted as its glucuronide.
Glucuronic acid can conjugate with hydroxyls (both
reduced to their amines, while aldehydes or ketones
phenolic and alcoholic), carbonyl, sulfhydryl and amino
are reduced to alcohols. An example is the reduction of
compounds.
.. nitrobenzene to aniline (Fig. 36.4). Other examples are:
Picric acid
The glucuronic acid is added to xenobiotics by UDP-
-> picramic acid
glucuronyl-transferases, present in the endoplasmic
Para nitrophenol -> para aminophenol.
reticulum .
• Hydrolysis UDP-glucuronyl-transferase
Hydrolysis is a chemical reaction in which the addition UDP glucuronic acid - -- -~ R-glucuronide
of water splits the toxicant into two fragments or smaller + R- OH + UDP
molecules. The hydroxyl group (OH-) is incorporated into The drug metabolizing systems are induced by the
one fragment and the hydrogen atom is incorporated into drug, e.g. barbiturates induce glucuronyl transferase
the other. Esters, amines, hydrazines, amides, and glyco- and heme synthesis. A list of important types of conjuga-
sidic bonds are generally biotransformed by hydrolysis. tion with glucuronic acid is given in Table 36.2.
528 Section D: Nutrition

TABLE 36.1: Phase two con1ugat1ng agents TABLE 36.2: Con1ugat1on with glucuronic acid

Conjugating agent Active form Compounds Types ofbond Products


Glucuronic acid UDP-glucuronic acid Phenol Glucosidic (Ether) Phenyl glucuronide
(0-glucuronide)
Sulfate PAPS (phosphoadenosine phosphosulfate)
Benzoic acid Ester Glucuronic acid monobenzoate
Cysteine Glutathlone
Bilirubin Ester with p ropionic Glucuronic acid (see Chapter
Acetic acid Acetyl-CoA

Steroids
acid side chain
Ester with OH group
22)
Glucuronide of steroid
'
GSH transferase Amines Amide N-glucuronides
R-X + Glutathione - - -- - - G-S· R + X-H
GS-R + Acceptor GGT Cysteinyl glycine-R +
gamma glutamyl acceptor
lsoniazld - N-acetyl
- +---=------
transferase
Acetylated isoniazld

6NH-NH, 6H-NH-<:O-CH,
Peptidase acetyl-CoA
Cysteinyt glycine-R _ _ _ _.,. R-cysteine + Glycine

N-acetyl transferase
R-cysteine + - - ~ -- - - Mercapturic acid
Acetyl-CoA +CoA
N N
Fig. 36.6: Phase two; glutathione as detoxifying agent
(GGT = Gamma glutamyl transferase)
Sulfanilamide
N-acetyl transferase

o~CH,
Acetyl sulfanllamide

6~oA
+ acetyl-CoA

0
Benzoyl-CoA + Glycine --+ Hippuric acid + CoA
CO-NH

( )H,-COOH
Fig. 36.7: Phase three reaction; acetylalion
CH2-COOH !Hz

6 ?H- COOH

CH 2
I
CH2
I
Cysteine and Glutathione
The cysteine is derived from glutathione, which is the
active conjugating agent. The reaction is given in detail
in Figure 36.6. Alkyl or aryl halides, and epoxides are
CO-NH
Phenylacetic- + Glutamine - -- -+ Phenylacetyl•
detoxified in this manner.
acid glutamine

Fig. 36.8: Conjugation w ith amino acids


Acetylation
Conjugation with acetic acid is taking place with drugs
Sulfate Conjugation like sulfanilamide, isoniazid and PAS (para amino sali-
In general, sulfation decreases the toxicity of xenobiotics. cylic acid) (Fig. 36.7).
..
The highly polar sulfate conjugates are readily excreted
through urine. Conjugation with Glycine
Phenolic and alcoholic compounds are conjugated Benzoic acid is conjugated with glycine to form hip-
with sulfate. The enzyme is sulfo-transferase and the puric acid (benzoylglycine), which is excreted in urine
sulfate group is transferred from PAPS (phosphoadeno- (Fig. 36.8).
sine phosphosulfate, see Chapter 18).
Phenol + PAPS - - - - - Phenyl sulfate + PAP Conjugation with Glutamine
lndole + lndoxyl sulfate + PAP
Important compounds excreted as their sulfates Similarly, phenylacetic acid is conjugated with glutamine
include steroids and indole compounds. to form phenylacetylglutamine (Fig. 36.8).
_ _ _ _ _Chapter 37
1

Environmental Pollution
'
and Heavy Metal Poisons

Chapter at a Glance
The learner will be able to answer questions on the following topics:
D Corrosives D Pesticides and insecticides
D Irritants D Occupational or industrial hazardous agents
D Heavy metal poisons D Air pollutants
D Lead, Mercury, Aluminium D Toxic substances in foodstuffs


Man has tried to change the environment in different oxidase. Poisoning may be due to suicidal attempts.
ways from the days when he was a cave dweller. Any Industrial exposure may occur in the persons working
substance present in the environment, which may pro- with hydrocyanic (prussic) acid or with potassium cyanide.
duce abnormality in metabolism or alter the wellbeing of Ingestion of amygdalin, present in kernels of certain
an organism, is called an environmental pollutant. fruits (apricots, almonds, peaches) is also a common
A poison is a substance which causes death or cause.
harm if introduced in the living body or brought into con- Dicobalt edetate (kelocyanor) is the antidote, which
tact with parts of the body. chelates the cyanide. Another method is to give sodium

ICORROSIVES nitrite and sodium thiosulfate intravenously. The nitrite


converts hemoglobin to methemoglobin. Ferric ion of
These are strong acids (sulfuric acid, hydrochloric acid); MetHb takes up cyanide as cyanmethemoglobin so that
strong alkalis (sodium hydroxide, ammonia); and salts cytochrome oxidase is now free of cyanide. Later, thio-
(zinc chloride, potassium chromate). sulfate detoxifies the cyanide by forming thiocyanate,
They remove water from the tissues, coagulate which is excreted. But in practice, death is instanta-
the cellular proteins and convert hemoglobin into acid neous and time may not be available for the treatment.
hematin.
The cause of death is circulatory collapse and spasm IIRRITANTS
• of glottis. Ammonia, potassium hydroxide, sodium hydro-
xide, calcium hydroxide and ammonium carbonate are Important chemical irritants are phosphorus, chlo-
the common alkalis encountered. They remove water rine, bromine and iodine. Metallic irritants are arsenic,
from the tissues and precipitate proteins. Ammonia is antimony, mercury, copper, lead, z:nc and silver. Organic
highly neurotoxic (see Chapter 17). irritants from plants include castor, croton, and calotropis.

Cyanide Poisoning Phosphorus


Cyanide causes tissue anoxia by chelating the ferric It is a poison affecting cellular oxidation. Accidental poi-
ions of the intracellular res!")ir::itory enzyme. cytor.hrome soning in children may occur due to chewing of fireworks
532 Section D: Nutrition

or rat poisons. Symptoms resemble acute liver disease. vi. One pack of cigarette contains 15 micrograms of
The poison is oxidized in the body. lead and chronic smokers have higher blood levels
of lead.
Neurotoxins vii. Lead chromate is commonly used as adulterant in
curcumin.
These may act at cerebral level, e.g. opium, alcohol, ether,
chloroform. datura, belladona, cannabis, etc. Those act-
ing at spinal level are aconite, quinine and oleander.
viii. Battery repair, radiator repair, soldering, painting
and printing are occupations prone to get lead poi- •
soning.

~ EAVY METAL POISONS Signs and Symptoms of Lead Poisoning


Lead Poisoning i. Lead is a cumulative poison and is accumulated
A 61-year-old male was admitted in a medical college in tissues over the years. It is not biodegradable.
hospital in Bangalore with typical symptoms of lead 90% of lead is seen in bones, 9% in blood and 1%
poisoning (encephalopathy, nephropathy and anemia). in brain and kidneys.
The lead level in blood was over 100 mg/dl. On detailed ii. There is no "safe" level in blood; about 10 mg/dL
questioning, the patient told that he was very particu- can be tolerated. More than 10 mg/dL in children
lar to take only "pure food" and he was preparing fresh and more than 25 mg/dL in adults leads to toxic
"atta" (wheat flour). He was powdering wheat on a small manifestations.
hand-mill at his house. The machine had a crusher which iii. Lead can pass through placenta and milk. Miscar-
was fixed on the pivot by soldering with lead. So for the riage, stillbirth, and premature birth are reported in
last 30 years, he was getting daily doses of lead along lead poisoning of mothers. •
with his "pure atta". The company had already supplied iv. Developing brains are more susceptible to lead.
thousands of such instruments throughout the country. Permanent neurological sequelae, cerebral palsy
All those families might be getting harmful doses of lead. and optic atrophy may be seen.
v. In children, mental retardation, learning disabili-
Sources of Lead Poison
ties, behavioral problems, hyperexcitability and sei-
i. Lead is the most common environmental poison in zures are seen. Even 10 mg/dL of lead in blood for
India. It is dispersed into air, food, soil and water. a long time will reduce IQ to 8 points.
Lead poisoning is also included in the class of "sum- vi. Anemia is very common.
mer disease", as increased temperature brings out vii. If the blood level is more than 70 mg/dL, acute
the dust, and lead particles will also be in the air.
toxicity is manifested, as encephalopathy, convul-
ii. Paint is the major source for exposure, especially
sions, mania, neuropathy, abdominal colic, severe
in children, as they bite painted toys. Paints should
anemia and kidney damage. Discoloration and blue
not contain more than 0.06% of lead but Indian
line along the gums are characteristic features of
paints, especially cheaper ones may contain up to
acute lead poisoning.
30% lead salts. Paint is peeled off as small flakes
from walls of living rooms. viii. Lead inhibits heme synthesis. Basophilic stippl-
iii. Increased content of lead is seen in air, water and ing of red cells are seen in the peripheral blood
vegetables in cities and near highways. This is due and bone marrow smears. Lead particularly inhibits
to the tetraethyl lead derived from the exhaust delta aminolevulinic acid (ALA) synthase and ALA-
dehydratase (see Chapter 22). Lifespan of RBC is
'
of vehicles. Statutory use of lead-free petrol has
reduced this type of contamination. shortened. Anemia enhances lead absorption, lead
iv. Lead pipes are important sources for contamination. in turn produces more anemia; thus a vicious cycle
v. Newspapers and xerox copies contain lead, which is operating. Lead will inhibit absorption of iron and
is adsorbed to fingertips, and later contaminate calcium. 1 molecule of lead will inhibit absorption of
foodstuff taken by hands. 1000 molecules of calcium.
Chapter 37: Environmental Pollution and Heavy Metal Poisons 533

Treatment of Lead Poisoning 1- 10 mg of aluminium per day, but only part of it is


absorbed. Tolerable upper limit of absorption is 1 mg/
Calcium dodecyl edetate (Calcium disodium versenate),
day. Only up to 100 mg/day can be eliminated through
penicillamine and dimercaprol (BAL) are used as anti-
urine. It prevents absorption of calcium, phosphorus and
dotes. Dimercaptosuccinic acid is a better but costly
iron. It also interferes with heme synthesis.
antidote.
Aluminium precipitates Alzheimer precursor proteins
and may lead to Alzheimer's disease. Aluminium is
Mercury Poisoning
implicated in degeneration of dendrites. It is also invol-
It is the most common industrial poison. The source for ved in Parkinson's disease.
poisoning may be elemental, inorganic or organic mer-
cury. Arsenic Toxicity
The oxides of arsenic are commonly used as fruit
Elemental Mercury
sprays, pesticides, rat poisons, etc. It acts on sulfhydryl
Hazard may come from inhalation of mercury vapor from enzymes and interferes with cell metabolism. It may also
broken thermometers, sphygmomanometers or from cause intravascular hemolysis, which leads to hemoglo-
dental amalgam. In acute poisoning, pulmonary edema binuria. The symptoms are anaphylactic reactions or
and encephalopathy may result. later development of agranulocytosis, hepatitis, jaun-
A classical triad of (a) oral lesions (gingivitis, sali- dice and encephalitis. The presence of arsenic in hair
vation and stomatitis), (b) tremor and (c) psychologi- and nails can be detected even after long intervals after
cal changes (insomnia, shyness, emotional instability, the poisoning has occurred. There are certain regions
,. memory loss) are the hallmark of chronic elemental in India, where arsenic is seen in water, which leads to
mercury poisoning. This is called erethism. chronic toxicity.
I
.!
Organic Mercury I PESTICIDES AND INSECTICIDES
Poisoning may occur from paints, fungicides and cos- Dichlorodiphenyltrichloroethane
metics. From mercury salt wastes, the bacteria synthe-
It is fat soluble and deposited in the adipose tissue.
size methyl mercury (CH3-Hg•). This then enters into
It is not excreted. Thus concentration inside the body
the fish. Eating of such fish is the most common cause
goes on increasing. The dichlorodiphenyltrichloroethane
for organic mercury poisoning.
(DDT) used in North America as pesticide during 1970s
In 1953, in Minamata bay, Japan, industrial effluent
has reached Antarctica and reduced the thickness of
caused methyl mercury poisoning, an epidemic lasting
shell of eggs of penguins. Even though DDT is banned
for several years. Organic mercury poisoning is there-
in many countries, it is still available in India. Many
after called Minamata disease.
antifungal agents sprayed on fruits are having long-term
The classical triad of methyl mercury exposure is
effects on depressed spermatogenesis and fertility.
dysarthria, ataxia and visual field constriction. In severe
cases, toxic encephalopathy, sensory neuropathy, Organophosphorus Compounds
intention tremor, hearing loss and spasticity may also
Organophosphorus (ORP) and organocarbamates
be seen. Dimercaprol derivatives, D-penicillamine, and
(ORC) are the common pesticides and organosulfur
N-acetylcysteine can increase the excretion of mercury
compounds (dithiocarbamates) are fungicides. Organo-
and are useful in treatment.
phosphorus compounds, parathion and malathion are
powerful neurotoxic agents. They inhibit acetylcholine
Aluminium Toxicity esterase through phosphorylation of the active center
Exposure is from packing and building materials, paint of the enzyme. Hence, acetylcholine accumulates in
pigments, insulating materials, cosmetics, antacids, and the nerve endings. Thus, the transfer of nerve impulse
aluminum cooking vessels. An average Indian consumes across synapses and at the nerve-muscle junction is
534 Section D: Nutrition

TABLE 37.1: Common industrial pollutants


Agents Causative industry Acute manifestation Chronic manifestation
Acid fumes Fertilizers, chlorinated organic Mucous membrane irritation followed by Chronic bro nchitis
(H,so., HNO,) compounds, dyes, explosives, plastics chemical pneumonitis
Cyanides Electrop lating, extractio n of gold o r
silver, manufacture offumigants
Increased respiratory rate; respiratory - I I
arrest; lactic acidosis
Formaldehyde Resins, rubber; labo ratory works; Same as for acid fumes Cancers in animals. l
urethane foam
Halides (Cl, Br, F) Bleaching in pulp, paper or textile Mucous membrane irritation, pulmonary Bronchitis, epistaxis, dental
industry; synthetic rubber, plastics edema fluorosis
lsocyanates Polyurethane foams, plastics, adhesives, Muco us mem brane irritation, dyspnea, Upper respiratory tract
surface coatings pulmonary edem a irritation, cough, asthma
Nitrogen dioxide Metal etching, explosives, welding, Cough, d yspnea, p ulmonary edema, Emphysema, chronic
byproduct of burning fossil fuels bronchiolitis obliterans bronchitis
Sulfur dioxide Coating of nonferrous metals, food Mucous membrane irritant, epistaxis Asthma, chronic bronchitis
processing, burning of fossil fuels

prevented. Diagnosis depends on the estimation of cho- The poisonous mixture of smoke, fog, air and other
line esterase in serum and RBC. The antidote is atro- chemicals is called smog. The chemically reducing
pine sulfate and cholinesterase reactivators (diacetyl smog is derived from the combustion of coal and oil,
monoxime or pralidoxime ). and contains sulfur dioxide (SO2 ), sulfur trioxide (SO3 ),
mixed with soot. SO2 and SO3 in the presence of atmos-

I
OCCUPATIONAL AND pheric water vapor, become sulfurous and sulfuric acids,
INDUSTRIAL HAZARDS respectively. This is the precursor of acid rain that may •
be carried by wind to long distances.
Methanol Chronic respiratory symptoms are associated '
.1,

It is the organic solvent widely used in paints and anti- with sulfur oxide or particulates in air. Exacerbations of
freezes. It may be consumed in place of ethanol as a bronchitis were associated with high concentrations of
substitute. Alcohol dehydrogenase converts methanol to smoke and sulfur oxide. Children living in polluted areas
formaldehyde and then formic acid. It is more toxic show diminished ventillatory function when compared
than ethanol. Optic neuritis and blindness is the cha- with their counterparts living in less polluted areas.
racteristic toxicity. The treatment is to give large doses Heart diseases are also related to pollutants such
of ethanol, which is preferentially oxidized in the body so as ozone, sulfur dioxide, sulfates and cadmium in the air.
that formaldehyde generation is reduced.
Industrial Pollution
I AIR POLLUTANTS Dramatic and disastrous episodes of air pollution have
The atmosphere contains mostly nitrogen (78.09%) and been documented in many industrialized centers in the
oxygen (20.94%), carbon dioxide (0.03%), and water world. An example was the London Fog of 1952, in which
vapor. The permissible level of total suspended particles approximately 4,000 deaths occurred over a period of
(TSPs) is 230 mg/cu.m. 2 weeks, following 5 days of severe cold and dehse fog.
A chemical other than those conventionally accep- Another such event was the Bhopal gas tragedy in
ted in the composition of clean air is called a Contami- December 1984 which claimed thousands of lives due
nant. A contaminant that occurs in the atmosphere in to methyl isocyanate (MIC) poisoning.
sufficiently high concentrations to cause an adverse
effect, is called a Pollutant.
Passive Cigarette Smoking
The major artificial sources of pollution arise due to The particulate load in a household is directly propor-
emissions from automobiles, industry and power plants. tional to the number of cigarette smokers living at home.
These are carbon dioxide, carbon monoxide, hydro- Increased preval ence of respiratory illnesses and redu-
carbons, oxides of nitrogen, oxides of sulfur and lead. ced levels of pulmonary function measurements have
Chapter 37: Environmental Pollution and Heavy Metal Poisons 535

been found in children of smoking parents. Studies have ii. Ergot (C/aviceps purpurea): It is the fungus that
also concluded that lung cancer risk is higher in non- usually grows in moist food grains (rye, millet,
smokers, who live under the same roof with smokers. wheat, barley, bajra). Ergotamine, ergotoxin and
ergometrine are present in this fungus (Ergometrin
Other Obnoxious Indoor Agents is clinically used to prevent postpartum hemor-
Common industrial pollutants and their effects are listed rhage). The toxins may produce peripheral vascular
in Table 37.1. contraction, causing painful cramps, gangrene in
extremities and convulsions. The disease is called
Toxic Substances in Foodstuffs ergotism.
These may be considered under the following headings:
Adulterants
Toxins Normally Present in Plants
i. Lathyrism: It is characterized by paralysis of lower
i. Goitrogens: They prevent iodine uptake or utiliza- limbs. It is seen in persons consuming large quanti-
tion by thyroid gland. Thio-oxazolidone is present in ties of Lathyrus sativus (Khesari dal). Khesari dal
cabbage, radish, turnip and brussel sprouts. Thio- is widely used to adulterate ordinary dal, and hence
cyanates and isothiocyanates are seen in mustard the disease may be seen sporadically all over India.
and other oilseeds. Polyphenolic glycosides are Neurotoxins present in lathyrus cause damage of
present in the red skins of groundnut and almonds. upper motor neurons. There is exaggerated knee
All these compounds have goitrogenic effect. jerk, ankle clonus, scissor gait and spastic paraly-
ii. Antivitamins: Orange peel, used in making orange
sis. The toxic principle from lathyrus sativus is
marmalade, contains citral, which inhibits vitamin A
• activity. Linseed oil, which contains linetin, inter-
identified as beta oxalyl amino alanine (BOAA).
Thorough cooking and decanting the supernatant
feres with pyridoxine utilization. Black berries and
! two or three times will remove these toxins (leach-
"- red cabbage contain thiaminase, which destroys
ing out the toxin by hot water). Ironically, the protein
vitamin 8 1 . Raw eggs containing avidin can dec-
content in khesari dal is of very good quality.
rease available biotin.
ii. Argemone oil: Mustard oil may be adulterated with
iii. Cyanogenic glycosides: Legumes (lima beans)
argemone oil. This is from a wild plant, Argemone
and tubers (tapioca or cassava) contain cyano-
mexicana. Argemone seeds are similar to mustard
genic compounds, which on hydrolysis produce
seeds, and oil from both seeds are similar in con-
hydrocyanic acid. Hence, they are highly toxic
sistency. Hence, adulteration is easy. Argemone oil
when taken raw. Cattle and sheep eating tapioca
contains the alkaloid, Sanguinarine which causes
leaves often get acute fatty degeneration of liver
vomiting, diarrhea, congestive cardiac failure and
with fatal outcome. The toxins can be removed by
cooking and decanting the supernatant water. edema. It is then called epidemic dropsy.
iv. Favism: Ingestion of uncooked broad bean (vicia
fava) may cause hemolytic anemia in susceptible Toxins Entering during Food
persons with glucose-6-phosphate dehydrogenase Preparation
(GPO) deficiency (see Chapter 6). The toxic glyco- Mono sodium glutamate (Aginomoto): It is a com-
side is known as vicin. Cooking and decanting will
mon food additive. Packets of mono sodium glutamate
remove the toxins.
carry the statutory warning that it is unsuitable for child-
Storage Contamination ren below the age of 5. It produces transient symptoms
like numbness and palpitation. It may deteriorate mental
i. Fungal infections: During postharvest storage,
alertness in children.
contamination with fungus is very common. Asper-
gil/us ffavus produces aflatoxins, which are hepato-
Related Topics
toxic and carcinogenic. The fungus grows in moist
conditions in groundnut, coconut, rice, maize, Ethanol metabolism (see Chapter 12); Carbon mono-
wheat, etc. xide (see Chapter 23); Detoxification and xenobiotics
536 Section D: NutrWon

(see Chapter 36); Free radicals (see Chapter 30); 3. Corrosive substances like strong acids and alka-
Chemical carcinogens (see Chapter 48). lis have been long recognized as poisons, mainly
found in laboratories and chemical industries.
0 4. Cyanide, the most potent poison, will arrest cellular
• · Clinical Case Study 37.1
respiration by its effect on cytochrome oxidase.
A 3-year-old girl was brought to the emergency depart- 5. Organic irritants include camphor, castor, croton
ment. She was cold and clammy and was breathing and calotropis.
rapidly. She was obviously confused and lethargic. Her 6. Neurotoxins are opium, alcohol, ether, chloroform,
mother had indicated that she had accidentally ingested datura, cannabis, etc., which are ingested acciden-
automobile antifreeze while playing in the garage. Fol- tally or willfully or by substance abuse.
lowing GI lavage and activated charcoal administration, 7. The most common environmental poison in India is
a nasogastric tube for ethanol was administered. What lead. In spite of banning, lead in paints and ink still
is the likely cause? occurs.
8. Lead inhibits heme synthesis and anemia is a very
0 common finding. Permanent neurological damage
• · Clinical Case Study 37.1 Answer
like lowering of intelligence and optic atrophy may
This is a case of ethylene glycol poisoning. Ethylene be seen.
glycol is the major ingredient of radiator fluid products. It 9. Chelation of lead using penicillamine, dimercaprol,
is used to increase boiling point and decrease freezing dimercaptosuccinic acid are the lines of manage-
point of radiator fluid. Thus, overheating and freezing of ment.
the fluid is reduced, depending upon season. 10. Entry of organic mercury into fish from industrial
effluent can cause indirect poisoning as in Mina-
Ethylene glycol is sequentially converted to glyco-
aldehyde, glycolate, glyoxylic acid and finally to oxalate. mata disease. •
Glycolate can produce acidosis and hyperventilation. Cal- 11. Widespread use of aluminium in all fields of life,
cium oxalate crystals can form and accumulate in blood including cooking has posed the problem of poison-
and other tissues and hypocalcemia can occur. Thus, ing.
symptoms include severe anion gap metabolic acidosis, 12. Arsenic used in rat poison, pesticides and insect
tachypnea, confusion, convulsions, and coma, oxalate repellents can cause blood diseases.
crystalluria and renal failure. 13. Organophosphorus (ORP) compounds used as
Laboratory investigations are (1) Ethylene glycol pesticides and fungicides are potent poisons. ORP
levels in blood, (2) Serum osmolality, (3) Serum electro- Inhibits acetylcholine esterase.
lytes, (4) Serum calcium, (5)Arterial blood gas analysis, 14. Contamination of ethyl alcohol by methanol (illicit
and (6) Urinalysis for calcium oxalate crystals. liquor) has claimed a large number of lives. Metha-
nol gets oxidized to formic acid, a potent poison

I LEARNING POINTS, CHAPTER 37 which can cause blindness and even death,
15. Lathyrism and epidemic dropsy are due to intake of
1. Cyanide poisoning causes tissue anoxia by chelat- adulterated dal and mustard oil respectively.
ing the ferric ions of the enzyme cytochrome oxi- 16. The following also cause environmental pollution
dase. Death is instantaneous. (a) new building materials and internal carpet-
2. Lead is a cumulative poison. Acute toxicity can ing practices, (b) newly built/remodeled build-
cause encephalopathy, convulsion, mania, anemia ings are known to cause "sick building syndrome",
and renal damage. Blue line along the gums is a
characteristic feature of acute lead poisoning.
(c) releasing of "off gas" toxicants and (d) carpets
are notorious for trapping toxicants.
'
PART-1 : SHORT NOTE QUESTIONS

37-1 . Lead poisoning. 37-3. Aluminium toxicity.


37-2. Mercury poisoning.
Chapter 37: Environmental Pollution and Heavy Metal Poisons 537

PART-2: VIVA VOCE QUESTIONS AND ANSWERS

37-1. What is most common environmental poison? 37-4. What are the manifestations of chronic lead poi-
Lead. soning?
37-2. What are common causes of lead poisoning? In children, mental retardation, learning disabilities,
,. Paint, lead containing petrol, lead pipes, newspapers behavioral problems are seen. Anemia and loss of
appetite are very common.
and cigarette smoke are important contaminants.
37-5. What is the cause of anemia in lead poisoning?
37-3. What are the occupations in which persons are
Lead inhibits delta amino levulinic acid (ALA) synthase,
prone to get lead poisoning?
ALA-dehydratase. So. heme synthesis is blocked.
Battery repair, radiator repair, soldering, painting and 37-6. What ls Mlnamata diseases?
printing. Organic mercury poisoning .


r
,
<.

'
SECTION E

Molecular Biology

Chapter 38 Nucleotides: Chemistry and Metabolism


Chapter 39 Deoxyribonucleic Acid: Structure and Replication
Chapter 40 Transcription
Chapter 41 Genetic Code and Translation
Chapter 42 Control of Gene Expression
Chapter 43 Recombinant DNA Technology and Gene Therapy
Chapter 44 Molecular Diagnostics and Genetic Techniques
1 - - - - -_ _ _ _ Chapter 38
Nucleotides:
Chemistry and Metabolism

Chapter at a Glance
Th e learner wi ll be able to answer questions on the following topics:
A Purines and pyrimidines
E Uric acid and gout
B Nucleosides and nucleotides
0 De novo synthesis of pyrimidines
C De novo synthesis of purine nucleotides
"' 0 Disord ers of pyrimidine metabolism
j) Degradation of purine nucleotides

• ~·
,'
" i·._.
t- )

.,,'l.-, .., ·;·


~4
Kassel described the 4..bases in nucleic
·J
Friedrich Albrecht Adolph Frederick
Miescher Kosse/ Strecker Hopkins
a s eticJnfo:m:mnon. The universal currency of 1844-1895 NP 1910 1822-1871 NP 1929
energy, namely AIE. is ~t1ele~e derivative. Nucleo- 1853-1927 1861-1947

tides are also components of impornmt~nzymes like


generated. The attachment of a 3rd phosplJ.a1e group
~ · and fA_rt.-aA metab · u at !IS su~h-as..cAMP results in the formation of a nucleo;i°e triphosphate. The
and cgMJ?. Wha.::~'1:::::.K=?--~~_;;~~Z
~win; - it:m.b,~g)"YI~ nucleic acids (DNA and RNA) are polymers of nucleoside
monophosphates.
COMPOSITION OF NUCLEOTIDES
A nucleotide is made up of 3 components: Bases Present in the Nucleic Acids
a. Nitrogenous base (a purine or a pyrimidine) Two types of nitrogenous bases; the (Purines and
b. Pentose su92,r, either ribose or deoxyribose pyrimidines are present in nucleic acids.
c. Phosphate groups e~ d to the sugar.
When a base combines with a pentose sugar, a urine Bases
n s formed. The purine bases present in RNA and DNA are the
When the nucleoside is esterified to a phosphate group, same; aden ine and guanine$Adeiilol.is 6-amjno purine
it is called ~ nucleotide or nucleoside mono-phosphate. and}guanin~ is 2-amino, 6-oxopurine. The numbering of
When a e d hos hate gets eslerified to the the purine ring with the structure of adenine and guanine
existing ph sphate group, a nucleoside diphosphate is are shown in Figure 38.1 .
d,.rut) H
- c'n°
.--.stt l"\,L{_~ )'"ca, 9r $E
/'."''\ ~ ~~

Hypoxanthine Xanthine

N
9 N N H2N
Purine nng

f
Adenine

Fig. 38.1 : Structure of purines '(f\~


® --N

3No~5 4
N~
¾
_j [ _

-
HO N r---
11 Pyrimidi ne Lactam or keto form of uric acid
La or enol form of unfaod
2 ".,..:;::: 6
N
Fig. 38.2: Minor bases seen in nucleic acids
2.~
~WlcP.~ ~

+o."'....,,
Uracil ,, hymine THYMINE is the bas_g present in DNA.
I
elN
(~O~ THIAMINE is a emb r \dtamin13 complex.

Fig. 38.3: Common pyrimidines

- Pyrimidine Bases @ Nucleotides


The pyrimidine bases present in nucleic acids
These are phosphate esters of nucleosides. Base plus
cytosine, thymine and uracil. £ .i~ ~seat.J o pentose sugar plus phos horic acid is a nucleotide. The
DNA and RNA. is resent in ON and s ion..occUis-at tl':i&-eth-er-3ro-nydroxyl.-g£m1P of
ilJ...BNA Structures are shown in Figure 38.3. See also the pentose sugar. Most of the nucleoside phosphates
Box 38.1. involved in biological function are 5'-phosphates (Table
A few other modified pyrimidine bases like 38.2). Since '-nucleoti e , they are
and Q • · + .3re also found rarely in simply written with efix. For exampl~, 5'-AMP is
some types of RNA.
abbreviated as AMP; but ~~Ya,J,iett is a~ _s~ itten as
~ - Moreover, a base can combine with either ribose '
~ ucleosides or deoxy ribose, which in turn can be phosphorylated
Nucleosides are formed when bases are attached to at 3' or 5' positions an coenz ~ r!vatives
the pentose sugar, D-ribose or 2-deoxy-D-ribose (Fig. of ~ s· ophosphate. Examples
38.4). All the bases are attached to the corresponding NADP, FA~ nd coenzyrfe A. .
NAD = N,U>~no.m,ck AdU)ine J)inu.de.t>\:-t~
Chapter 38: Nucleotides: Chemistry and Metabolism 543

Adenosine Deoxythymidine
0 0
NH2 0

'"~).
2
O
~N)
aN ~ C H ,

I
~ - -~ N 6
OH OH
9
D-ribose 2-deoxy-D-ribose
s· 0 s· I
O
H-";Vo":•·
Fig. 38.4: Sugar groups in nucleic acids 0
2
OH-H C
~
4' 1·
TABLE 38.1: Base + sugar are nucleosides
Ribonuc/eosides
IHJ 3' 2· H
OH OH OH H
Adenine + Ribose Adenosine
Guanine+ Ribose Guanosine Fig. 38.5 : Numbering in base and sugar groups. Atoms in sugar
are denoted with prime numbers
Uracil + Ribose Uridine
Cytosine + Ribose Cytidine
- TABLE 38.2: Base + sugar + phosphate = nucleotide
Hypoxanthine + Ribose lnosine Ribonuc/eotides
Xant hine + Ribose Xanthosine Adenosine + Pi monophosphate
Deoxyribonucleosides (AMP) (Adenylic acid)
Guanosine + Pi monophosphate
Adeni ne + Deoxy ribose Deoxy adenosine (d-adenosine)
(GMP) (Guanyiic acid)
Guanine + Deoxy ribose d -guanosine
Cytidine + Pi monophosphate
Cytosine + Deoxy ribose d-cytidine (CMP) (Cytidylic acid)

Thymine+ Deoxy ribose d-thymidine Uridine + Pi monophosphate


(UMP) (Uridylic acid)
+ Pi monophosphate (IMP)
"t ~ osine
TABLE 38.3 .. Nucleoside triphosphates·• (lnosinic acid)

-
Deoxyribonucleotides
Nucleoside d-ad enosine + Pi (d-adenylic acid)
Nuc/eoside monophosphote
d-guanosine + Pi (d-guanyllc acid)
Ribonuc/eoside phosphates
d -cytidine + Pl (d-cytid yiic acid)
Adenosine Adenosine Adenoslne Adenosine
d-thymidine + Pi (d-thymidylic acid)

.;;--)
mono phosphate diphosphate triphosp hate
(AMP) (ADP) (ATP)
Guanosine GMP GDP GTP
lnosine IMP IDP ITP
Cytidine CMP CDP CTP
Uridine UMP UDP UTP
Deoxyribonucleoside phosphates ' 0 0 0
~ -~ N
I
I
&
N
d-adenosine d-AMP d -ADP d-ATP II II o
d-guanosine d-GMP d-GDP d -GTP HO- i~O-, -O-, -O-H2C
~ • O
d -cytidine d-CMP d-CDP d-CTP
OH OH OH
d-thymidine d -TMP d-TDP ·d-TTP
H
OH OH
@ Nucle Tri phosphates
Fig. 38.6 : Adenosine triphosphate (ATP)
Corresponding nucleoside di- and tri-phosphates are
formed by esterification of further phosphate groups to diphosphate contains one high energy bond and
the existing ones. In general, any nucleoside triphospt)ate triphosphates have 2 high energy bonds. ATP is the
is abbreviated as t:JTP or d-NTP (Table 38.3). Nucleoside universal energy currency (Fig. 38.6). It is formed
Cr)
5'
N N

o- - -
1

HO-P=O

I
(1~3 '~ "",;;;-57
Fig. 38.8: The assembly of purine ring is from various sources.

Fig. 38.7: 3',5'-cyclicAMP or cAMP


Q HFA (FH4! • lelralJY!'romne:> . '1J

However, the dietary purines and pyrimidines are


0
II not incorporated into nucleic acids. They are directly

catabolized.
OH- i-O-H 2C O OH O
O
OH II II BIOSYNTHESIS OF PURINE
O-P-0-P-OH
I I NUCLEOTIDES (Bae.~- ~ " 5 ~ )
OH OH OH OH
The purine nucleotides are synthesized by most of the
Fig. 38.9: Phosphoribosyl pyrophosphate CPR1'f>) tissues. However the mqjpc.site is the This pathway
operate · . T~- major pathway is denoted
a e ov nthes , because the purine ring is synthe-
i

-
sized from j Since the ~ ma~ ..i,
being can synthesize the~ rine and yrimidine ba~ s~ J

-
nova, they are said to be
;,;;:.:.;.;..;;.;;;.;.;..,;;;.;;,;.;.;;.;;...;.;.;:..-~difl~
he contribution of
~ for the formation of
the purine ring is shown in the Figure 38.8.
During de nova' synthesis, purine ring is built up on
a ,.......,_.... molecule. He.n ce~ teotide-s-are-
mediating the action of several hormones. th~ products""tlf the de-.ro~ t~ . There are
Deoxy ribonucleotides are used for synthesis of . . in the de novo synthesis pathway. The enzymes
DNA and ribonucleotides for RNA. In pse11daurjdyjic a£!? catalyzing these reactions are existin as a mu ·e me
(found in tRNA) uridine is attached to ribose phosphate
in a bond instead of C-N bond in UMP. High energy
compound"'s are listed in Table 21.2. Please note that

---.-. etc. are higher


mo~ phosp~ te.
'~;::;:;;.::;::::;::::::,-;s,c~::::::!:::::-
ene;g? oo ·
~t.)
m ...
tep O(Preparatory Step), PRPP Synthesis
Phosphoribosyl pyrophosphate (PRPP) is the donor of
' ribose-5-phosphate for de n ~~nthesis. The reaction is:
@ Digestion of Nucleic Acids Ribose-5-phosphate + ATP--::=. l?
ADP + Phosphoribosyl pyrophosphate (PRPP) '<:
The nucleic acids in the diet are ~y_drolyzed to a mixture The structure of PRPP is shown in Figure 38.9. ?
of n ~ s yr[ onuc ease ao . e x rtbom:ieteas The purine ring is later on assembled on the ribose-
present in pancr tic and intes I al secretions. Then h,~_h o_s~hate. PR~P is • for the synthesis of
(d_ucleotidase_,filliberate the p~e_Jrom nucleotides. W~ e..nude_ru_ides, 1~.,P--- c oe~ymes and also
The resulting nucleosid hydrolyzed by c eo- for th~ ~atb':C!'~ ~ nee the synthesis of PRPP
M ase~ forming f,:.:
re~e::::b~ ~~~~~::_::.:~ is not c~ sider~d as a step in the de nova synthesis

s I
~1o®.1

I • ¥ N
©JI ClN:rru
~ tt,yi c.~
-4 -
Formyl
c) . ,1 ADP
,.
ATP
ormyl glycinamide
methenyl-
THFA THFA
0
I /
)

5'-.,-----~
Q N
glycinamide
ribonucleotide fA'I
ribonucleotide
(FGADI
-------,........
If:\ R p
I (FGAI'\) 0; IV 01 5

RsP ATP ~ @ G1~•;/', /f~t"e/ ('cHo ® 't

O' ef'N I r s~~oP~~"'1-""'


ADP+ Pi (+) H20
c, ........__ /
'- R 5P / cP~

J
~"'

JCI
ATP ADP+ Pi N-succlnyl-5-amino-i:::-'"
N~ Amino imidazole (+) \ 1m1dazole
ribonuteotide (AIR) as \
2
. . 5-amin~carboxtamino- ->-----~-+ carboxamide
1 \.V 1m1daz
~ole nbonucleollde (ACAIR) - (?)
H2 N :i- ~ 0 - N> (+)~ C,
s rtic acid O N
R 5P £ 't cw"\:tu:a t ~ '\
\'v.P>
, 1""'1 cu-f
\,-.J.M £._
H2 N I Asp/
N
I / @
- ~- R5P N
H2N I
').{t'-1~\e. ref_
(§1\\
¥
R5P
C - - ...H c.a8 °-formyl-
v Fumaric acid
GDP + Pl GTP .:y THFA
Adenylic ~cid
(Adenosine
lnosinic acid
(lnosine
N-f?r'.1'1yl amino J
5-amino-imidazole-
1m1dazole +-~~.,e;.___ 4-carboxamlde
N>
monophosphate) • monophosphate) @' 4-carboxamide
4
®
ribonucle.otide tr~
(AMP) @ (IMP) 1-1 rlbonucleotide (AICAR) 1\;.f CU..,.O,..,.,

I
Fumaric Aspartic H20 (FAICAR) AICAR

o 0

@)[)
ac~ cid 0

R,P I ~ ) "'")I)
'lNJ-, @D---N I R5 P R5P
H2N I
R5P
Fig. 38.10: First eight steps of purine synthesis
~ .,0 h\~-p @ ~ ~Q . y _ -4 h"TQ."t 2...btl'lc.~e.s?.'~ 'S ~esrt~~c:a9..')
Ll 8 ::l:1"'1"1---.\bt-ro~
~be · C>"'~
TABLE 38.4: Summary of steps of purine synthesis of purine nucleotides; it is called a Jprelirninary or pre-
Added atom • ........ 1111. paratory stea .....,
1 Glutamine N9 (Rate limiting) PRA
2 Glycine C4,s'®ATP required) GAR F ormation of AMP
3 Methenyl-THFA ca FGAR
Steps 1 to 10 are summarized in Table 38.4. Flow dia-
4 Glutamine @ ATP required) FGAM
grams of these steps are shown in Figures 38.10 and
5 Ring closure (ATP) AIR
6 Carbondioxide C6 ACAIR
38.11.
7 Aspartic acid ®ATP required) SAICAR .. ('";!Fr;:ir:::-
s.-t s::.t:::-
e=-p"is~ ra:r
te=---r.
tir,,;,n·;.
;t...
,~- Step 4 is inhibited by aza~
8 Fumarate removed AICAR serine, an anticancer drug. 6-mercaptopurine inhibits
9 Formyl-THFA C2 FAICAR amination of IMP to AMf, and therefore it is used as an
10 - Ring closure IMP anticancer drug. C1.'1.. "-~I
546 Section E: Molecular Biology

nova synthesis. Hence these two pathways are closely


AICAR inter-related. The free purines are salvaged by two
10
N -formyl-THFA;:;; J~~nt enzymes; adenine phosph~ I transferase
~ 'r>RTasaj_a goxantbine gyanjne ohosphorJbosyl
THFA ~e!) transferase (HGPRTaseh
Salvage pathway is summarized below:
N-formyl amino imidazole
4-carboxamide APRTase
ribonucleotide (FAICAR) ,;:: Adenine + PRPP AMP+ PPi
1 HBPRTase
Guanine + PRPP GMP + PPi
Hypoxanthine + PRPP HGPRTase > IMP + PPi

Regulation of Purine Synthesis


lnosinic acid
(lnosine monophosphate)
(IMP)

Adenylic acid (Adenosine factor. The activity


monophosphate) (AMP)
o PRPP synthetase is regulated by negative modifiers;
purine and pyrimidine nucleotides. ~ of ope.cation
Fig. 38.11 : Last steps in purine synthesis. Enzyme 1 =ospho of aIva e_Qfilhway has a ~ J ; ;t on de nova
ribosyl amido transferase; Enzyme 2 = GAR synthetase; Enzyme synthesis since the availability of PRPP is modulated
3 = GAR transformylase; Enzyme 4 = FGAR amido transferase; I
Enzyme 5 = Cyclase; Enzyme 6 = AIR-carboxylase; Enzyme 7 and inhibitory nucleotides are provided.
= =
SAICAR synthetase; Enzyme 8 SAICAR lyase; Enzyme 9 =
AICAR transformylase; Enzyme 10 = IMP synthase; Enzyme 11. =
Adenylosuccinate s~etase and adenylosu · ase C/ti.S~ tructural Analogs as Purine
'J D _ v_ I"' p s Gt" ., Synthesis Inhibitors
Conversion of IMP to GMP cJN Structural analogues act as c
-{,"I'm\ r..,.~
n.e.
· ive i ito of the
The conversion of IMP to GMP involves two steps, first naturally occurring rtucleotides. They may be iocorpor.at.eq__
oxidation of iMP to xanthylic (xantb,osine mono- iQ!.Q..0.l:¼.Qlllin1'
.ohibiti.og u!1@r eloogatfo'n<or rendering
phosphate) (XMP) by an NAO• depen8ent dehydro- such DNA functi~ally inactiv~l \ hereby cell division is
genase. Then an amidotransferase transfers the NH 2 arrested. So they ar useful as anticancer dru . A few
group from glutamine to XMP to rm GMP. 18Il?-is ,....n ,.
hydrolyzed to MP level in this reagj_o (Fig. 38.12 . a. inhibits the conversion of IMP to ·,
Both AMP and GMP can be converted to their di- and
triphosphates. Synthesis of one molecule of purine . - b. Other synthetic nucleotide analogues used as anti
nucleotide re<l#!res 6 high energy phosphates. cancer agents are 6-thio guaaw~ and 8-~:i~~uanine_
o. ,a, tt, ,- , t\
Otherj·
0 Salvage Pathway
This pathway @ sures the rec clin of purines formed
by degradation of nucleotides. Nucleosides and deoxy-
nugwide_s__cae1.Rlso be salvag_ed. PRPP is the starting
material in this pathway; it is also a substrate for de

IMP dehydrogenase
Chapter 38: Nucleotides: Chemistry and Metabolism 547

Amidotransferase
IMP • XM P • GMP
(inosinic acid) { \ (xanthylic acid) ( '\ { \ (guanylic acid)

11~..)LNN
>9 NAO NADH •HN:x~;m,oe GMamate
0 + + 0 .
ATP
AMP.PP
;N~N>
N I
@AN N
I
~-)LNI
N
.'.3 Ribose-5-phosphate Ribose-5-phosphate RsP

Fig. 38.12: Conversion of IMP to GMP. R-5-P = ribose-5-phosphate

Gout attack locations

Most common

Common

Less common

First toe affected i.!J. 90o/0-9!


in.d.illklJHlls ~ QQ!!.

Fig. 38.14: Location of gout attack

degradation is tal<-tAQ place.mainl¥.ifutbe iv.er. The steps


are shown in Figure 38.13.
urine Nucleoside Di- and Tri phosphates The fxanth in0-0x~dase) is a meta" oflavoP.rotein
containing FAD, molybdenum and iron.
These reactions are catalyzed by(spec1fic kioase~nd . .
p~osphate group is donated by ATP. One example i( fc~ :>( )(::
X Q.f'\~n''~- =
g1ve~ below. ..., ..,.
GM + ATP (~ phosphate kinase) > GDP+ ADP Caffeine is t~ im h · · anthine. Coffee
GDP + A1 P ~ hosphate kinase) > G p + A!5P

® oegradation of Purine Nucleotides


The end product of purine nucleotide catabolism is uric hrine on
acid (urate). The structure is shown in Figure 38 .2. This
u.9tco.uo\. \)e'\ mal = - 'o f'Oi/ clL
~--,,_~et"'I = s-~-1-00,_-
548 Section E: Molecular Biology

'Abnormal PRPP Synthetase C~~-o)


Uric acid, the end product of purine catabolism is a - This leads to increased production of PRPP.
I J JI.:. Its role as an antioxidant is particularly significant
in t heSi j ] ilsv,hlh ~! YI and pe~ ~I radicals l\i eficiency of Enzymes of C,i~~)
can be removed. ½ Salvage Pathway

URICACID @_ ~
.,~
...-,<.;iJjlllll":...

· acid ranges from 2- 5 mgld~lucose-6-phosphatase Deficiency (He-() P)


in males. The daily excretion
-
Reactions which consime PRPP and produce more
nucleotides w ~ . e.g. salvage pathway.

This condition is known a von Gierke's diseas (gly-


cogen storage disease, typ I, s e Chapter 10). When
this enzyme is deficient, l ~ n o t
be converted to luco . So more glucose is channeled
Disorders of Purine Metabolism into the pentose- sphate shunt pathway, resulting in
. -::.:::.,..,,,...........=-'. t•!!li• "1iilf
~!c increased ability of ribose-5-phosphate. This would
ac1 ev m oo , .1t1s lead to· c~a_s__.e~J_prmation of PRPP.
~'fe, defined as serum uric acid concentration eeding ~c=--R--®©
<:- PPP <- l-1 N\?
~ (irngtdL io male and 6 mg/dL in fem;;Tu. may or may ~econdaiy~peruricemia
.l, not be associated with increased exccetioo at 11cic acj~I d p d t· f U · A 'd
t - .,LL .
m_.unne, v.J,,;
r' . can"'1"
..c1, 1s 11111
-"arl • .
The man,·testat·ions\JJ ncrease ro uc ,on o nc c,
e,'11.~-a ~ to the low soll.lbility of uric acid in water. It may be due to e~u,mo\ler rate.QfJll.!.CleiC acids
as seen in: (Pi)
i. Ra~ly growing malionant tissugs, e.g . eul<ElW.Las,
lym~ o ~ QQ)ySJi;m ia.
ii. A1 · cancer patients n
OT =lll"'-......,~.....___...._..~~t:!.lerapy (tumor lysis synd-
.i caw Thus tophi are seen i .
ii-'~ Increased excretion of uric acid may cause deposition o<:iReduced Excretion Rate
f uric acid crystals in the urinary tract leading.!!
9I formation with renal d a m ~ G a be
oeither
ut i. Renal failure.
rimary or secondary. The~m ~ c_:.__ · ii. Treatment with th- ia-=z==
":::,c'
id===~==d=iu::::::~::::-Jics which inhibit tubu-
patients with gout is II lar secretion of uric acid.
@- and excess production of nucleotides resulting
innyperuricemia.
. .
(igClm1cal
F" . fG
mdmgs o out
f'F::\ ~J e Galen has describ~ the cla~cal image ~he gouty
Pn out [I su~ts as the re~aced, gaS°d living, hao/cl' drinking,
About 0% of cases of pritjjrry gout are idiopathic. coLJITTry squire. Many geniuses, includin(isaac Newt®'
Primary gout may show a familial incidence. Incidence of .-Gibbon and Johnson were suffering from gout.
primary gout is about 1:500 in total population. Causes. " Often the patients have a few alcoholic drinks, go to
of primary gout are: sleep symptomless, but are awakened during the early

CDAbnormal 5-phosphoribosyl IA-'\ hours of morning by excruciating joint pains.


Tb§~pical gouty arthritis affects the first metatarso-
Amidotransferase Ce0'X.- 1-) W CM,P
phalang al joint (big toe), but other joints may also be
Enzyme is not inhibited by n~cleotides. This would lead affected. The joints are extremely painful. Synovial fluid
to over production of purine nucleotides. will show birefringent urate crystals (see Fig. 38.14).
PRPP synthetase, HGPRT, Hyperuricemia
f~ glucose 6 phosphatase
Lesch Nyhan HGPRT Lack of enzyme
syndrome
lmmuno- Lack of enzyme
deficiency
ant inuria Xanthine oxidase hypouricemia,
1-----t--- Allopu/41 ,? · r nal lithiasis
blocks formation of uric acid

---
Uric acid
1--- - - 4 - - Uricase Lesch-N~n Syndrome
breaks down uric acid
Probe. + ~ ( It is an X-linked inherited disorder of purine metabolism.
1-+- - + - - - Probenecid
Increases excretion of
uric acid in urine
I Incidence is 1:10,000 males. There is deficiency of
~PRTase. So, the rate of salvage pathway is decreased
Crystals deposit
in tissue ( ~n ~ t~ ) resulting in accumulation of PRPP and decreased level,,
of inhibitory puf.l!l,e nucleotides. e disease : ~::~:; &l'\dl ,1
i-+- - - i - - - NSAIDs, colchicine, terized by self tff6r tion, m t rdation, . moIW
corticosteroids
reduce inflammation ~ d and· nep litbiasis Gout develops in later life.
The neurological manifestations suggest that the brain is
dependent on the salvage pathway for the requirements
of IMP and GMP. -
Fig. 38.15: Action of medicines in gout o
• •
Jt Hypour1cem1a
n ,otil ~Y\~~-
r· ( pq ,9(\-)
Adenosine Deaminase (ADA) Deficiencr,,
2:n.o~,"e_ __. KlobPS11'.;!-0uni• ,-_;Q
It is associated with severe immunodeficien[y where~ V

both T and B cells are deficient. It is an inherjted


a• •tosoroal recessive disease ADA deficiency leads to
accumulation of adenosine and dATP; this would inhibit
further production of precursors for DNA synthesis
Treatment Policies in Gout especially dCTP. Lymphocytes usually contain high
i. Reduce dietary ~ e intake and restrict ~ ol. levels of ADA. Therefore, ADA deficiency is mainly
ii. Increase reoal exc•elign of hte by Gi,icosuric manifested as reduced lymphocytes. Hypouricemia is
drugs) which decrease the reabsorption of uric due to defective breakdown of purine nucleotides. The
acid from kidney tubules, e.g. proben_ecid-. first successful gene replacement therapy has been tried
iii. Reduce urate production by allopurinol. an aoalqg in ADA deficiency (see Chapter 43). ADA is sometimes
of hypoxanthine. Allopurinol is a competitive inhi- used for the rapid diagnosis of tuberculosis.
bitor of xanthine oxidase thereby decreasing the See Table 38.5 for summary of disorders of purine
formation of uric acid. Xanthine and hypoxanthine nucleotides.
are more soluble and so are excreted more easil .
Xanthine oxidase converts allopurinol to alloxa
DE NOVO SYNTHESIS OF
thine. It is a more effective inhibitor of xanthine oxi- PYRIMIDINE
dase. This is a good example of 'suicide inhibition' The pyrimidine ring (unlike the purine) is synthesized
(see Chapter 5). as free pyrimidine and then it is incorporated into the
iv. Colcfflclne, an anti-inflammatory agent is very nucleotide. The origin of atoms of pyrimidine nucleus is
useful to arrest the arthritis in gout (Fig. 38.15). indicated in Figure 38.16.
550 Section E: Molecular Biology

iCD.9. ~ e . .
4
C HC0 3- A •~' \
( .>t( +
1
r '\+•
'
NH2-C0-0-P03
~\001

Carbamoyl
,J
~C 5 Glutamine GM amate 2ATP 2ADP phosphate
From
From
carbamoyl
phosphate
~
I
C6
aspartic
acid
3 2
NH2-CO-NH-CH--COOH
1 6

I
7
2
Aspartate

CH:2- COOH ~ Pi
N 5 4
1 0

Fig. 38.16: Sources of C and N atoms of pyrimidine

Detailed steps of pyrimidine synthesis are shown in


Figure 38.17. Step 1 is catalysed by the enzyme carba-
'6
0 2 N
1

0
COOH
7

"2 :;"t:
/'
Dlhydro orotic acid (OHO)

NAO+
moyl phosphate synthetase II (CPS-II). The differences
4
between CPS-I and CPS-II are described in Table 17.2. N d;6,1'.°'4 NADH
Uridine monophosphate (UMP) is the first pyrimidine I
f......, 'D~ Orotic acid (OA)
that is synthesised (step 6). T zyme' is OMP-
0 N COOH
decarboxylase (OMPOC). 6-aza-uridin inhibits this 0 <!)O"M'.&.

N~ - 'Vt-""""'"'-' PPi O
Regulation of Pyrimidine S
O~ N ) lCOOH Oroti dylic acid (Orotidine
i. In eukaryotes the ~ ~ e zy , viz, CPS lL, I ~, _A\...., ,.?lmonophosphate) (OMP)
ATC and OHOase are pre~ ~ myltjenzvme R5P IV\ ~

c~ ;;ind referred to a ' AD' taking the first o ~o~mP -~


letters of the 3 enz mes.
ii. Th~ enzy , OPRTase and OMP dec,ar- N I
.... 6

boxylase are also present as a s.ua_le functjg~I _____ Uridylic acid (U ridine

tp
0 N ..,,-" monophosphate) (UMP)
complex. ecause of this clustering of enzymes,
C)~l..4 f'-'~ ATP ~
the synthesis is well co-ordinated. Both complexes \-\me. Ot\"'- 7
are cytosolic
ADP

~ iii. Uridine dlphosphate (UDP)

ATP ~
iv. ~1~ ,c:k
@
8
d,.::,r,<J1-:..J-- - - - - - , ADP
NH.,

(, J
~ - 7\Uridine triphosphate (UTP)

V.
<l\~ ATP * Glutamine

O N i,,,A.!ia..J\'-" ADP lu amate


The first 3and last 2 enzymes are regulated by I
R5P- P- P Cytidine triphosphate (CTP)

'-T'• =~~i~~~~=
-~-;.--:.
- •-
- - l.~2
carbamoylase (ATC ; . . ••r = asparty1 trans-
&AWi • t. - dihydro
orotate dehydrogen:ase {Dt10DH); = orota e p osplio ribosyl
transferase enzyme. transferase (OPRTase); 6 = OMP-<lecarboxylase (OMPDC); 7 =
Cytosine + PRPP CMP UMP kinase; 8 =nucleoside diphosphate kinase; 9 =CTP synthetase

Thus, both purines and pyrimidines can be salvaged. Fig. 38.17: Synthesis of pyrimidine nucleotides
-~ t& o.a \1"''h\~m
Chapter 38: Nucleotides: Chemistry and Metabolism 551

DISORDERS OF PYRIMIDINE TABLE 38.6: Disorders of pyrimidine metabolism

METABOLISM OROTIC ACIDURIA Disorder Defective Enzyme

• The condition results from absence of either or both of Orotic aciduria OPRTase, OMP decarboxylase

the enzymes, OP~[ase and OMP decarboxylase. It Orotic aciduria Ornithine Transcarbamoylase
is an autosmti~~cessiv disease. There is r~larded Drug induced Orotic OMP decarboxyla se
growth and The rapidly growing aciduria
cells are more affected and hence the anemia. J3 amino isobutyric Transamina,sJ, affects urea cycle
are _excreted in urioe which may ca~se o c~ ry tea!? aciduria function, deamination of a amino acids
to a keto acids
obstruction. Due to lack of feedback inhiWon orotic
acid production is excessive. The.t q_~ ition can be
successfully treateo by feeding~ ihe or aine. They Cytosine
may be c ~ UIP which £aD acl_j.s..feedback
~ r. See Table 38.6 for summary of disorders of
1•0 Nf' 1

pyrimidine nucleotides. Uracil Thymine

Deoxyribonucleotide Formation ,
t NADP H+
t:ADPH;H'
C..'PP d c..'"DP Cl Tf NA! + ADP+
Deoxyribonucleotides (both P. nes and pyrimidine
Dihydrouracil Dihydrothymine
uctiort at the( g: carbon
. hospha ':9 (NDP l l
ribonucleotide reductase Beta ureido
propionate
Beta ureidoc ~B)
1sobutyrate I
'T·~:ae)
complex, which contains thioredoxin. Thus, UDP is

1~0,
(carbamoyl (carbamoyl
first converted to dUDP, and then to dUTP. beta amino isobutyrate)
)
-1\1

Beta alanine Beta amino 1sobuty;ie

ig. 38.18: Catabolism of pyrimidines

ucleotides

0
• · Clinical Case Study 38.1
Anticancer Agents Acting on Pyrimidines A40-year-old male presented with severe pain, redness
and swelling of the base of the first metatarsophalangeal
joint in the night after a bout of alcohol consumption.
The patient was in usual state of health until early in
the morning when he woke up with severe pain in his
right big toe. The patient denies any trauma to the toe
and no previous history of such pain in other joints. On
examination, he had mild fever 38.2°C. The right big toe
was swollen , warm, red, and exquisitely tender. Serum
uric acid was 9.7 mg/dl. What is the likely diagnosis?
What is the pathogenesis of the condition? How would
you make a definite diagnosis?
552 Section E: Molecular Biology

0 to vitamin B12 and folic acid. Lack of CTP, UTP, and


e Clinical Case Study 38.2 TMP as a result of enzyme inhibition decreases nucleic
acid synthesis and decreases RBC production leading to
A 4-year-old boy presented with hypotonia, developmental
megaloblastic anemia, physical and mental retardation. •
delay, irritability and self mutilating behavior. On exami-
Diagnosis is made by severe megaloblastic anemia
nation, there was testicular atrophy and hematuria. The
serum uric acid level was 10.0 mg/dl. What is the likely
with normal B 12 and folate levels and no evidence of ..
diagnosis? What is the biochemical basis of the condition? transcobalamine II deficiency. Elevated orotic acid
and RBC enzyme assay (transferase) confirms the
0 diagnosis. Treatment is to administer uridine, which will
• · Clinical Case Study 38.3 synthesize UTP, CTP and TMP reducing the symptoms
A 3-year-old girl presented with megaloblastic anemia and also produces feedback inhibition of erotic acid
and failure to thrive. Obstetric history was uneventful. production.
Anemia was present, which did not improve despite
blood transfusions. There was no response to B12 , folate LEARNING POINTS, CHAPTER 38
and pyridoxine therapy. Urinalysis revealed presence of
1. A nucleotide is composed of a nitrogenous base, a
a crystalline sediment, which was identified to be orotic
pentose sugar and phosphate groups esterified to
acid. Very high levels of orotate (above 1.0 g/day, normal
the sugar.
being< 1.4 mg/day) were excreted. Enzyme assays were
done and showed deficiency of orotate phosphoribosyl
transferase (OPRTase). What is the likely condition?
What is the pathogenesis of the findings?
3. ••••••••ts
2. Purine bases in both DNA and RNA are the same.
mediated by two enzymes:
Adenine phosphoribosyl transferase (APRTase)
and Hypoxanthine guanine phosphoribosyl trans-
0 ferase (HGPRTase).
• · Clinical Case Study 38.1 Answer 4. Committed step of de novo purine synthesis is the
Diagnosis: Gouty arthritis. reaction catalyzed by 9 midotransferase.
Pathophysiology: Elevated levels of uric acid are 5. Xanthine oxidase is a metalloflavoprotein containing
detectable in the blood and urine, resulting in precipitation FAD, molybdenum and iron. Allopurinol inhibits it,
of urate crystals in the joints. Synovial fluid will reveal which is a good example of 'suicide inhibition'.
needle-shaped crystals of monosodium urate that are 6. End product of purine catabolism is uric acid. Its
negatively birefringent under polarizing microscopy will normal serum level)~ 3-:7 mg/dl. LeYE::! is increased
confirm the diagnosis. Formation of urate crystals in the in gout. e.iu
big toe, is thought to be associated with the decreased 7. Uric acid crystals deposited in the cooler areas of
temperature of the extremities that aids in urate crystal the body caus
formation when levels exceed solubility. 8. is a X-linked inherited dis-
order of purine metabolism, due to deficiency of
0
•i 'i· Clinical Case Study 38.2 Answer HGPRTase. The condition is characterized by self-
mutilation tendency.
Diagnosis: Lesch-Nyhan syndrome. 9. deficiency has been treated
See text for details about biochemical basis. by gene replacement therapy.
10. In mammalian cells, regulation of pyrimidine syn-
0
e Clinical Case Study 38.3 Answer
thesis occurs at the level of Carbamoyl phosphate
synthase 11, which is inhibited by UTP.
The likely condition is erotic aciduria. There is excessive 11 . Deficien~ of OMP decarboxylase and OPRTase
excretion of orotic acid. The enzyme deficient may be leads to
OPRTase or orotidylate decarboxylase. It can also be 12. Disorders o purine metabolism are summarized in
secondary to OTC deficiency. Clinical findings appear in Table 38.5.
the first year of life and include growth failure, develop- 13. Disorders of pyrimidine metabolism are summarized
mental retardation and megaloblastic anemia, refractory in Table 38.6.
Chapter 38: Nucleotides: Chemistry and Metabolism 553

14. De nova synthesis of purine nucleotides uses 21. Lesch Nyhan syndrome is an X-linked trait cha-
from amino acids and racterized by nephrolithiasis, mental retardation ,
aggressive and self-mutilation.
15. The first purine nu~leotide formed is- which is 22. De nova synthesis of pyrimidine nucleotides use
then converted to AMP and GMP. amide group of glutamine and aspartic acid.
16. Requirement of GTP for the formation of AMP from 23. There is a reciprocal relation between the synthesis
IMP and ATP for the formation of GMP ensures of purine and pyrimidine nucleotide synthesis which
balanced synthesis of both the nucleotides. ensures a balanced supply of both.
17. Amidotransferase is subject to feedback inhibition 24. dUMP can be converted to dTMP by thymidylate
by the products AMP and GMP. synthase enzyme using one-carbon group carried
18. Catabolism of purine nucleotides produces uric acid. byTHFA.
19. Normal serum uric acid level varies from 2-5 mg/di 25. Inhibition of nucleic acid synthesis by nucleotide
in women and 3-7 mg/di in men. and nucleoside analogues is used in cancer
20. A high level of uric acid {hyperuricemia) can lead chemotherapy (6-mercapto purine, 6-thioguanine,
to deposition of urate crystals in joints producing 5-flourouracil etc). Methotrexate which inhibits thy-
arthritis. Treatment of hyperuricemia is by giving allo- midylate synthase is also an anticancer drug.
purinol a competitive inhibitor of Xanthine oxidase. 26. Some antiviral drugs are also nucleotide analogues.

PART-1: ESSAY AND SHORT NOTE QUESTIONS


38-1 . Give the sources of carbon and nitrogen atoms of purine and pyrimidine rings . How is the de novo synthesis
regulated? Indicate the clinical uses of inhibitors of purine nucleotide synthesis.
38-2. Describe the inborn errors of metabolism associated with degradation pathways of purines and pyrimidines.
38-3. How are purine nucleotides degraded? Add a note on abnormalities due to excessive purine catabolism.

SHORT NOTE QUESTIONS


38-4. Sources of carbon and nitrogen atoms in purine ring. 38-10. Gout.
38-5. Phosphoribosyl pyrophosphate (PRPP). 38-11. Allopurinol.
38-6. Salvage pathway of purine synthesis. 38-12. Secondary hyperuricemias.
38-7. Xanthine oxidase. 38-13. Lesch-Nyhan syndrome.
38-8. Formation of uric acid. 38-14. Orotic aciduria.
38-9. Purine catabolism. 38-15. • Adenosine deam inase deficiency.

PART-2: MULTIPLE CHOICE QUESTIONS

38-1 . All the following amino acids are used for synthesis C. Hypoxanthine
of purine nucleotides, except: D. Beta aminoisobutyric acid
.. A. Glycine
C . Aspartate
8 . Alanine
D. Glutamine
38-5. Hyperuricemia is observed in all the following con-
ditions, except:
38-2. N-3 of purine ring is donated by:
A. Lesch-Nyhan syndrome
A. Ammonia 8 . Glutamine
B. von Gierke's disease
C. Aspartate D. Glycine
C. Glutathione reductase deficiency
38-3. Wh ich amino acid is required for both purine and
pyrimidine synthesis? D. Xanthine oxidase deficiency
A. Glycine B. Aspartate 38-6. Hyperuricemla can result from deficiency of all the
C. Alanine D. Glutamate following enzymes, except
38-4. The chief product of catabolism of purines in A. HGPRTase
human beings is: B. APRTase
A. Urea C. Glucose-6-phosphatase
B. Uric acid D. Adenosine deaminase
554 Section E: Molecular Biology

38-7. The reduction of nucleotide to deoxyribonucleotide: C. Methylation is taking place on dUMP and not on
A. Takes place at the level of nucleoside diphosphate UMP
B. Involves the cleavage of glycosidic bond D. 1st nitrogen is donated by glutamine
C. d-ATP will enhance the reaction 38-16. Regarding gout, all are true, except:
D. Needs hydrolysis of ATP A. It is seen in HGPRT deficiency
38-8. Orotic aciduria is a feature of deficiency of all the B. There is deposition of urate in jjoints
following enzymes except: C. Can be treated using allopurinol
A. OMP decarboxylase D. It is seen in PRPP synthetase ,deficiency
B. Ornithine transcarbamoylase 38-17. If a cell is unable to synthesise PRPP, synthesis of
C. OPRTase which is likely to be directly affected?
D. Aspartate transcarbamoylase A Pyridoxal B. OMP
38-9. Pyrimidine nucleotide synthesis and purine nucle- C. CoASH D. FAD
otide synthesis are similar in which aspects? 38-18. During de novo synthesis of Ipyrimidine, which
A. Enzymes are present as multienzyme complexes nucleotide is first formed?
B. Ring is assembled and ribose-5-phosphate added A. TMP B. OMP
C. Ribose-5-phosphate is donated by PRPP C. UMP D. CMP
D. Aspartic acid is fully incorporated into the ring 38-19. The drug of choice for primary gIout is:
38-10. All the following molecules have regulatory effect A. Allopurinol B. Aspirin
on synthesis of purine nucleotides, except C. Colchicine D. Probenecid
A. PRPP 38-20. All manifestations are seen i n ILesch-Nyhan syn-
drome, except:
B. AMP
A. Self mutilation B. lmmun,odeficiency
C. UMP
C. Hyperuricemia D. X-linke,d inheritance
D. GTP
38-21. All are true regarding pyrimidine nucleotide syn-
38-11. Which property of uric acid is responsible for the
thesis, except:
manifestations of gout?
A. The reactions occur in the cytoplasm
A. Ketoenol tautomerism :.
B. CTP is a negative modifier
B. Acidic nature
C. Glutamine and aspartic acid are used
C. Reducing action
D. Pyrimidine ring is built up on a ribose-5-phosphate
D. Solubility constant
38-22. Formation of dTMP requires:
38-12. Which compound is not derived from purine nuc-
leotides? A. S-adenosylmethionine
A. Uric acid B. Beta aminoisobutyric acid B. NADPH
C. Allantoin D. Xanthic acid C. Adrenodoxin
38-13. Uric acid is the end product of breakdown of all the D. Molybdenum
following, except 38-23. Orotic aciduria is characterized by all the findings,
A. AMP B. IMP except
C. Xanthosine E. CMP A. Megaloblastic anemia
38-14. Secondary hyperuricemia is seen in all the follow- B. Response to oral uridine
ing conditions, except: C. Failure to thrive
A. Leukemia B. Psoriasis D. Deficiency of HGPRTase
C. Hepatitis D. Cytotoxic drug therapy 38-24. Which of the following conditions is not associated
38-15. Which is correct regarding thymine synthesis? with hyperuricemia?
A. 4th carbon is donated by carbamoyl phosphate A. Multiple Myeloma B. Xanthi1nuria
B. 3rd nitrogen is donated by aspartate C. Psoriasis D. Polycythemia vera

ANSWERS OF MULTIPLE CHOICE QUESTIONS

38-1. B 38-2. B 38-3. B 38-4. B 38-5. D 38-6. D 38-7. A


38-8. D 38-9. C 38-10. C 38-11 . D 38-12. B 38-13. D 38-14. C
38-15. C 38-16. A 38-17. B 38-18. B 38-19. A 38-20. B 38-21 . D
38-22. B 38-23. D 38-24. B
Chapter 38: Nucleotides: Chemistry and Metabolism 555

PART-3: VIVA VOCE QUESTIONS AND ANSWERS

38-1. What is a nucleotide? 38-20. What is the normal uric acid level in blood?
Nitrogenous base+ sugar+ phosphate. 2-5 mg/dl in females and 3-7 mg/dl in males.
38-2. What is a nucleoside? 38-21 . What is gout?
Nitrogenous base + sugar. It is a disease associated with hyperuricemia, deposit
38-3. What are the sugars in nucleic acids? of urate crystals in joints and excruciating joint pain.
Ribose in RNA and deoxyribose in DNA. 38-22. Which property of uric acid is responsible for the
38-4. What are the bases present in nucleotides? manifestations of gout?
Purines and pyrimidines. Uric acid is sparing soluble in water.
38-5. Name the common purines. 38-23. Increased uric acid level is seen when?
Adenine and guanine. Gout, pre-eclampsia, Hodgkin's lymphoma, leukemia.
38-6. Name the common pyrimidines. 38-24. Hyperuricemia is observed in which conditions?
Cytosine, uracil, thymine. Gout, Lesch Nyhan syndrome, van Gierke's disease
38-7. What are the bases present in DNA? 38-25. What is the mechanism of Allopurinol?
Adenine, guanine, cytosine, thymine. It is an analogue of hypoxanthine. It inhibits xanthine
38-8. Which base is absent in DNA? oxidase. thereby decreasing the formation of uric acid.
Uracil. 38-26. It is what type of inhibition?
38-9. Which base is found exclusively in DNA and not in Xanthine oxidase converts allopurinol to alloxanthine.
RNA?
It is a more effective inhibitor of xanthine oxidase. This
Thymine.
is a good example of 'suicide inhibition' .
38-10. Which base is found exclusively in RNA?
38-27. What is the reason for severe combined immuno-
Uracil.
deficiency?
38-11 . Which amino acid is required for both purine and
Adenosine deaminase deficiency.
pyrimidine synthesis?
Aspartic acid and Glutamine. 38-28. What are starting materials for pyrimidine synthe-
38-12. NJ of purine ring is donated by what? sis?
Glutamine. Carbamoyl phosphate and aspartic acid.
38-13. Glycine donates what part of the purine ring? 38-29. What is the rate limiting step in pyrimidine synthe-
C4, CS, N7 atoms. sis?
38-14. What is the key enzyme of de novo synthesis Aspartate transcarbamoylase.
pathway of purines? 38-30. How is pyrimidine synthesis pathway regulated in
Phosphoribosyl amidotransferase (step 1). mammals?
38-15. How is de novo synthesis of purine regulated? CPS II is inhibited by CTP.
Amidotransferase enzyme is inhibited by AMP and 38-31 . Orotic aciduria is a feature of deficiency of which
GMP. enzymes?
38-16. Give a few examples of purinP. analogues, used as OMP decarboxylase; OPRTase; Ornithine transcarba-
anticancer drugs. moylase.
6-mercapto-purine; Cytosine arabinoside; Methotre- 38-32. What are the characteristic features of Orotic aci-
xate; Azaserine. duria?
38-17. What is the end product of catabolism of purines Megaloblastic anemia.
in human beings? 38-33. What is the mechanism of Methotrexate?
Uric acid. II inhibits dihydrofolate reductase and thereby reduces
38-18. What is xanthine oxidase? the regeneration of THFA; it is a powerful anticancer
It is the enzyme catalysing the reactions, hypoxanthine agent.
to xanthine and xanthine to uric acid. 38-34. What is the mechanism of 5-fluorouracil?
38-19. Name an enzyme containing molybdenum. It inhibits conversion of dUMP to dTTP; and acts as an
Xanthine oxidase contains FAD, molybdenum and iron. antimetabolite.
_ _ _ _ _ Chapter 39
Deoxyribonucleic Acid:
Structure and Replication

Chapter at a Glance
The learner will be able t o answer questions on the following topics:
D Watson-Crick model of DNA stru cture D DNA polymerase
D Chromosomes D Okazaki pieces
D Replication of DNA D DNA repair mechanisms

Thomas Morgan (1866-1945), the founder of modern genetics, showed that chromosomes contain genes in a sequential
manner in Drosophila (Nobel Prize, 1933). In 1931 , Barbara McClintock showed the rearrangement of genes or mobile genes
in chromosomes in corn (Nobel Prize, 1983). George Beadle, working with mutant strains of Neurospora suggested •one
enzyme one gene" hypothesis in 1941 (Nobel Prize, 1958). Avery in 1944 demonstrated that DNA is the genetic material. In
1952, Hershey (Nobel Prize, 1946) showed that only DNA of virus and not the proteins will enter into the host before infection.
Erwin Chargaff elicited the base pairing rule of DNA in 1950. X-ray crystallographic studies on DNA by Maurice Wilkins (Nobel
Prize, 1962) showed the details of structure of DNA. Rosalind Franklin worked out the helical structure of DNA. (She died of
ovarian cancer at the age of 37, probably due to irradiation during her work on X-ray crystallography). Based on these data,
James Watson and Francis Crick in 1953 deduced the double helical structure of DNA (Nobel Prize, 1962).

ISTRUCTURE OF DNA In the DNA, the base sequence is of paramount


importance . The genetic information is encoded in the
Deoxyribonucleic acid (DNA) is composed of four deoxy-
specific sequence of bases; if the base is altered , the
ribonucleotides, i.e. deoxyadenylate (A), deoxyguany-
information is also altered.
late (G), deoxycytidylate (C), and thymidylate (T ).
These units are linked by 3' to 5' phosphodiester
bonds to form a long polypeptide chain. T he nucleotide
is formed by a combination of base + sugar+ phosphoric
acid. The 3'-hydroxyl of one sugar is linked to the
5'-hydroxyl of another sugar through a phosphate group
(Fig. 39.1). In this particular example, the thymidine Thomas Oswald Alfred George Barbara
Morgan Avery Hershey Beadle McClintock
is attached to cytidine and then cytidine to adenosine NP 1933 1877-1955 NP 1969 NP 1958 NP 1983
through phosphodiester linkages {Fig . 39.1). 1866-1945 190/J-1997 1903-1989 1902-1992
Chapter 39: Deoxyribonucleic Acid: Structure and Replication 557

OH
I 5' phosphate end S'end
Phosphate bonds
O=P-0- form the rail of
1 staircase
O-H2C Thymine
0 Base pair

0 H

0
I
I
0 HC Cytosine
2
0

Francis Crick
NP 1962
3' end 5' end
1916-2004
H
Fig. 39.2: Watson-Crick model of double helical structure of DNA.
0 Adjacent bases are separated by 0.34 nm. The diameter or width
of the helix is 2 nanometers
Adenine
0

Common abbreviations
in DNA chemistry:
A= Adenine, G = Guanine
C = Cytosi ne, T = Thymine
OH H
Red boxes show Erwin Chargaff
Phosphodiester linkages 3' OH end
1905-2002

Fig. 39.1: Poiynucleotide

The deoxyribose and phosphodiester linkages are


the same in all the repeating nucleotides. Therefore, the
message will be conveyed, even if the base sequences Rosalind Franklin
alone are mentioned as shown: 1921-1958
5'P--Thymine-Cytosine-Adenine-3'0 H
Or, 5'-----T--C--A---3'
This would convey all the salient features of the
polynucleotide shown in Figure 39.1.

Maurice Wilkins
Polarity of DNA Molecule Nucleotide NP 1962, 1916-2004
In the case of DNA, the base sequence is always written F ig . 39.3: Base pairing rule. Base pairing of A with T and G with
from the 5' end to the 3' end. This is called the polarity C. Hydrogen bonds between bases
of the DNA chain.
Right Handed Double Helix
Watson-Crick Model of DNA Structure DNA consists of two polydeoxyribonucleotide chains
The salient features of Watson-Crick model of DNA are twisted around one another in a right handed double helix
given in Figures 39.2 and 39.3. similar to a spiral stair case. The sugar and phosphate
558 Section E: Molecular Biology

BOX 39.1: DNA 1s the storehouse of genetic information


The major differences between DNA and RNA are the presence of DNA strand
2-deoxyribose and absence of uracil in DNA. RNA is more prone to
spontaneous hydrolysis of the P-0 -P bonds. On the other hand the H1 histone
DNA chain is much more stable and the half-life for spontaneous
hydrolysis is about 200 million years. Hence, DNA becomes the more
suitable storehouse of genetic information. Core histones
as octamer

Fig . 39.4: DNA wraps twice around histone octamer to form one
groups comprise the handrail and the bases jutting nucleosome
inside represent the steps of the staircase. The bases
are located perpendicular to the helix axis, whereas the Denaturation of DNA Strands
sugars are nearly at right angles to the axis.
The double stranded DNA may be denatured and
Base Pairing Rule separated by heat. This is called as melting of DNA. At
lower temperature, the melted strands are re-associated;
Always the two strands are complementary to each
this is called annealing.
other. So, the adenine of one strand will pair with
thymine of the opposite strand, while guanine will pair
Higher Organization of DNA
with cytosine. The base pairing (A with T; G with C) is
called Chargaff's rule, which states that the number of In higher organisms, DNA is organized inside the
purines is equal to the number of pyrimidines. nucleus. Double stranded DNA is wound round histones
to form nucleosomes (Fig. 39.4). Chromatin is a loose
Hydrogen Bonding term employed for a long stretch of DNA in association
with histones. Chromatin is then further and furthe r
The DNA strands are held together mainly by hydrogen
condensed to form chromosomes (Fig. 39.5).
bonds between the purine and pyrimidine bases. There
are two hydrogen bonds between A and T while there are
Histones
three hydrogen bonds between C and G. The GC bond
is therefore stronger than the AT bond. A mispairing will These are proteins containing unusually higher con-
disturb the stable double helical structure. centration of basic amino acids. Histones synthesized
in the cytoplasm migrate to the nucleus. Acetylation of
Antiparallel histones leads to activation of transcription, whereas
deacetylation causes depression of transcription. Meth-
The two strands in a DNA molecule run antiparallel,
ylation generally occurs when the gene is repressed.
which means that one strand runs in the 5' to 3' direction,
while the other is in the 3' to 5' direction (Fig. 39.2). This
DNA is a Very Long Molecule
is similar to a road divided into two, each half carrying
traffic in the opposite direction. Human diploid genome consists of about 7 x 109 base
pairs. So when placed end to end it will be about 2 meters
Other Features long, If one nucleotide is added per second, it will take
In the DNA, each strand acts as a template for the 250 years to synthesize the whole DNA of a human cell.
synthesis of the opposite strand during replication The length of a DNA molecule is compressed to 8,000
process. The spiral has a pitch of 3.4 nanometers per to 10,000 fold to generate the chromosomes (Fig. 39.5).
turn. Within a single turn, 10 base pairs are seen. Thus,
Inactivation of DNA during Differentiation
adjacent bases are separated by 0.34 nm. The diameter
or width of the helix is 1.9 to 2.0 nm. A major groove All human cells are derived from a single cell, the zygote.
(1.2 nm) and a minor groove (0.6 nm) wind along the Therefore, all cells contain the same genetic information.
molecule, parallel to the phosphodiester backbone. In But, a cell from the gastrointestinal epithelium is different
these grooves, proteins interact with the exposed bases. from a cell of central nervous system, by structure and
DNA is the storehouse of genetic information (Box 39.1). function. How such a differentiation is made possible?
Chapter 39: Deoxyribonucleic Acid: Structure and Replication 559

repetitive sequences, 5 to 500 base pairs repeated many


DNA double helix
2nm times. One such sequence, the Alu family is repeated

i
Nucleosomes
about 5,00,000 times, and accounts for about 5% of total
human DNA.
10 nm

i I REPLICATION OF DNA
During cell division, each daughter cell gets an exact
Chromatin fibril
30 nm copy of the genetic information of the mother cell.

l This process of copying the DNA is known as DNA


replication.
Extended In the daughter cell , one strand is derived from the
chromosome
300 nm
mother cell; while the other strand is newly synthesized.

!
This is called semi-conservative type of DNA replication.
,'I, I , I ,
Each strand serves as a template or mold, over which a
I ' • I
Chromosome new complementary strand is synthesized (Fig. 39.7).
I 11 ,• \ .. ·, ' 1 , l 1400 nm

Meselson-Stahl Experiment (1958)


Fig. 39.5: DNA condenses repeatedly to form chromosome
Bacteria were grown in a medium containing the heavy
In a cell, about 90% DNA are permanently inactive. isotope of nitrogen 15N, when all the DNA was labeled
Histones and other specific proteins help in this inactiva- with heavy nitrogen. These cells were allowed to divide
tion process and consequent differentiation. in a medium containing normal nitrogen, 14 N. In the
first generation, all DNA molecules were half labeled.
lntrons, Exons, Cistron In the second generation half labeled and completely
unlabeled molecules were present in equal numbers.
With the progress of evolution, DNA content has also From this experiment (Fig. 39.6), it was proved that DNA
increased. For example, bacteria have 1 x 106 base replication is semiconservative in vivo.
pairs per cell; while human cells contain 6 x 109 base The base pairing rule is always maintained. The
pairs. new strand is joined to the old strand by hydrogen bonds
The segments of the gene coding for proteins are between base pairs (A with T and G with C) (Fig. 39.8).
called exons (expressed regions). They are interspaced The replication is summarized in Figure 39.9. The whole
in the DNA with stretches of silent areas, called introns process may be studied under the following steps:
(intervening areas). Intrans are not translated,
A Cistron is the unit of genetic expression. It is the Origin of Replication
biochemical counterpart of a "gene" of classical genetics.
The DNA replication starts with the recognition of the site
One cistron will code for one polypeptide chain. If a
of origin of replication (ORI). Many origins of replication,
protein contains 4 subunits, these are produced under called autonomous replicating sequences (ARS)
the direction of 4 cistrons ("one cistron-one polypeptide" have been identified in mammals.
concept).
Components of DNA Replisome
Repeat Sequences of DNA
DNA replication needs the participation of more than 20
Only about 1- 2% of the human DNA contain genes; the enzymes and proteins, collectively called DNA replicase
rest are silent areas. About 1% of DNA is present inside system or replisomes. DNA helicases (topoisomerases,
mitochondria. There are only about 25,000 to 30,000 gyrases and helicases) unwind the DNA, Helicase moves
protein-coding regions in the human DNA. About 90% in both directions, separating the strands in advance for
of DNA is made of noncoding intervening sequences, replication. This forms a replication bubble (Fig. 39.10)
called introns. About 30% of the genome consists of with two replication forks .
560 Section E: Molecular Biology

(\ Absorbance
I \ at 260 nm

I
N1•

Matthew Meselson
(b .1930)
and Franklin W Stahl
(b.1929)

Conservative replication (theoretical; Semiconservative


but actually not taking place) replication (actual)

Fig. 39.7: Semiconservative replication (Anew complementary


strand is synthesized over the old template)

Template strand

--G
- T - C - C - G - A - C -T- C -

P
jpJJP
JJO
H OH
S'end New strand 3' end
Synthesis from 5' to 3' direction
Parent cell

II DNA (15N + 15N)

First generation
(15N + 14N) x 2
Fig. 39.8: New strand is synthesized from 5' lo 3' direction. Base
pairing rule is always maintained

II 11 Second
generation
Old

(15N + 14N) x 2
11 1111 11 (14N + 14N) x 2 Old strands

Fig. 39.6: Meselson-Stahl experiment; GO = parent cell; G1 = first


generation; G2 = second generation
Arthur Kornberg
NP 1959
1918-2007
DNA Polymerase
This enzyme synthesizes a new complementary strand New strand is
made, based on
of DNA, by incorporating dNMP sequentially in 5' to 3' base pair rule
direction, making use of single stranded DNA as template.
Old strand acts
Bacterial DNA Polymerases as a template
Old New New Old

In bacteria there are 3 DNA polymerases.. Arthur


Fig. 39.9: Both strands are replicated
Kornberg (Nobel Prize, 1959) isolated the DNA
polymerase I (Kornberg's enzyme) from Escherichia
coli. It is a repair enzyme. It has both 3' to 5' and 5' to 3' Mammalian DNA Polymerases
exonuclease activities. Bacterial DNA polymerase Ill is In mammalian cells (eukaryotic), there are 5 DNAPs,
the main replication enzyme in bacteria. named as a, 13, y, o and£. Alpha polymerase polymerizes
Chapter 39: Deoxyribonucleic Acid: Structure and Replication 561

about 100 nucleotides per second . (Bacterial enzyme and repair enzyme, where as DNAP gamma is concerned
has 10 times more speed). It is the major enzyme which with mitochondrial DNA replication.
synthesizes Okazaki fragments. DNAP delta completes
lagging strand synthesis and DNAP epsilon is used for RNA Primer is Required for DNA Synthesis
leading strand synthesis. DNAP beta is a proofreading
An RNA primer, about 100-200 nucleotides long, is
synthesized by the RNA primase. Then the RNA
Origin of replication primer is removed by DNAP, using exonuclease activity
r7

l
and is replaced with deoxyribonucleotides by DNAP
:::::::·:::::::::~:::~::!·n·:::::::l::im::::::::::. (Fig. 39. 11).
Opening of

,. , ., )~..,.... .................
l ...
DNA helix
JI''"'' Elongation of DNA Strand
........... ..............
.,.
ftfft HtHtlftUOtttlO ..,,,- - , ,.. ...... H tmu 1ottttH

Under the influence of DNA polymerase, the 3' hydroxyl


RN.A. primer
synthesis group of the end nucleotide combines with the 5'
_.,111mtufft
phosphate group of the new deoxynucleotide. The

l
YllOflWf{llfllffflWl..,pli' "'""""""""
IIINHINll llffllllUIJ~-IIIJt= ---~
pyrophosphate is released from the deoxynucleoside
Synthesis of triphosphate (Figs. 39.8 and 39.9). This newly added
leading strand
nucleotide would now polymerise with another one,
1"'1"""""'"''""41c.
"'llWll'"""'""
.Mhllllll1h-..,
,=:Ioo,,
, ....,_tllllllJII
~ IIUlllllQWIIIIM

Additional new chains


! forming the next phosphodiester bond. If "A" is present
on the template, "T" enters in that place in the newly
synthesized DNA strand. The base pairing rule is
Lagging ~
strand of ,,-~nm:,~:1::: ;•~:r,u~~'-!~- -
" •-un 11
Leading
strand always observed. The DNA polymerase carries out the
~1 ~, cl~2
NIIHI'>- • hffiWNiMfiNfNfWril ~114 sequential addition of each nucleotide complementary
' •y w MMI NMtttll.HUtW l lWJatll ,
to the one in the template strand.
Leading strand of Fork 1 Lagging strand of Fork 2
Polymerization: Polymerization of the new strand
of DNA is taking place from 5' to 3' direction. This means
Fig. 39.1 0: Replication bubble (Replication fork)

Primase synthesize an RNA primer over the


parent DNA

Primase

RNA primer

"~
! - - - --+ DNAP synthesizes the new strand of DNA
5'

DNA polymerase

ONA polymerase

3' A new DNA polymerase removes RNA and


replaces with DNA

5' ·•••111...•••11111...
llaal!• 3'
. , -,;•
Then ligase joins the ends
Daugher strand is now complete
3' ~
Fig. 39.11: RNA primer is needed for the DNA synthesis; blue is the parent strand ; green is the new strand ; red is the primer.
562 Section E: Molecular Biology

strands simultaneously; but it is in pieces of about 100

Direction of replication l t - - - - Topoisomerase


to 250 nucleotides in length. This "discontinuous DNA
synthesis" produces replication forks or replication
bubbles (Fig. 39.10). Both strands are simultaneously
replicated.
The small DNA molecules attached to its own
primer RNA are called Okazaki fragments. Several
DNA polymerase Okazaki pieces are produced. The synthesis along the
lagging strand is in 5' to 3' direction. As it moves, the
primase synthesizes short RNA primer, to which deoxy-
ribonucleotides are added by DNA polymerase.
polymerase DNA synthesis continues until the primer and pre-
viously added Okazaki fragment is encountered. Then
the RNA primer is removed by DNAP, using exonuclease
activity and is replaced with deoxyribonucleotides by
5'
3' DNAP. The remaining nick is sealed by the DNA ligase.
Newly synthesized DNA is rapidly arranged into
Lagging strand Leading strand nucleosomes. A summary of DNA replication is given in
with Okazaki pieces
Box 39.2. Differences between bacterial and mammalian
replication process is summarized in Table 39.1 .

Modification after Replication


DNA methylation at CS of cytosine catalyzed by DNA
Reljl Okazaki (1930-1975) and Tsuneko Okazaki (b. 1933) discovered methyl transferase is commonly associated with gene
Okazaki fragments in 1968. He died or leukemia; he had been heavily
irradiated In Hiroshima when the first atomic bomb exploded.
silencing. One of the effects of ROS (reactive oxygen
species, free radicals) is through hypermethylation.
Fig. 39.12: Lagging strand and Okazaki pieces
Aberrant methylation is also observed in cellular senes-
cence and may affect age-related diseases, such as
that the template is read in the 3' to 5' direction (see Fig. type 2 diabetes mellitus.
39.8). Thus, the 3' end of the last nucleotide is free.
Of the two double stranded molecules of DNA I DNA REPAIR MECHANISMS
produced, one goes to one daughter nucleus and the The replication process should be carried out with high
other to the second daughter nucleus. But each daughter fidelity , otherwise the genetic information is altered.
cell gets only one strand of the parent DNA molecule. Hence, there should be a foolproof mechanism to correct
Old DNA strand is not degraded, but is conserved in the the mistakes. While printing a page, the typesetter sets
daughter cell, hence this is semiconservative synthesis the types; an impression is taken, proof reading is done
(see Fig. 39.7). Synthesis of new DNA requires all the to correct mistakes if any, and then the final printing is
four dNTPs and divalent metal ions, magnesium. done. A similar follow-up mechanism operates after DNA
synthesis. Nobel Prize in 2015 was awarded to Tomas
Discontinuous Synthesis and
Lindahl, Paul Modrich and Aziz Sancar "for studies
Okazaki Pieces
of DNA repair". Various physical and chemical agents
DNA synthesis is always in the 5' to 3' direction in both produce base alterations; these are to be appropriately
strands. The strand which is discontinuously synthesized corrected immediately. There are different types of
is referred to as the "lagging strand" and the one DNA repair mechanisms; all of them follow the general
continuously polymerized as the "leading strand" mechanism outlined above, but details may vary. These
(Fig. 39. 12). Replication of DNA is taking place on both mechanisms are briefly described below:
Chapter 39: Deoxyribonucleic Acid: Structure and Replication 563

Exonucleolytic Proofreading
The DNA polymerase has 3' to 5' exonuclease activity.
Hence any mispaired nucleotide added is immediately
removed (Fig. 39. 13).

Mismatch Repair
The original template DNA contains methylated residues
(N6-methyladenine and 5-methylcytosine). The newly
Aziz Sancar, Paul Modrich, Tomas Lindahl,
NP 2015, NP2015, NP 2015, synthesized strand will not have methylated bases.
b 1946 b 1946 b 1938 So enzymes can recognize the original (correct) DNA
strand. The mismatched base is identified and removed
BOX 39.2 : Summary of DNA rephcalion
along with a few bases around that area. The wrong
1. Origin of replication is identified. Then unwinding of parental
base is removed by the endonuclease activity. A small
DNA to form a replication fork. segment of DNA with correct base sequence is then
2. RNA primer complementary to the DNA template is synthesized by DNA polymerase beta. Then the gap or
synthesized by RNA primase. nick is sealed by DNA ligase (Fig. 39.14).
3. DNA synthesis is continuous in the leading strand (toward
replication fork) by DNA polymerase.
4. DNA synthesis is discont inuous in t he lagging strand (away
Nucleotide Excision Repair (NER)
from t he fork), as Okazaki fragments. This mechanism repairs damage of DNA and replaces
5. Elongation: In both stran ds, the synthesis is from 5' to 3'
a segment up to 30 nucleotides in length. Pyrimidine
direction.
6. Then the RNA pieces are removed; the gaps filled by
dimers caused by UV light, benzopyrene (guanine
deoxynucleotides by DNAP and the pieces are ligated by adducts formed by smoking), other modifications
DNA ligase. caused by chemical agents, cancer chemotherapy etc.
7. Proofreading is done by the DNA polymerase. are thus repaired. Initially two phosphodiester bonds on
8. Finally organized into chromatin.
the damaged strand are hydrolysed by endonucleolytic
9. Main enzym es involved in replication are: DNA polymerases;
helicases; topoisomerases; DNA primase; single strand cleavage and the activity is referred to as excinuclease.
binding proteins; and DNA ligase. A fragment of DNA, 25-30 nucleotides in length is

TABLE 39.1: Comparison of features of repllcat1on 1n prokaryot1c and eukaryot1c cells

DNA Circular DNA Linear

Origin Origin of replication Replication at multi ple sites

Single strand binding Co-operative binding to the SS DNA 55 DNA binding pro teins bind at the replication fork.
proteins

RNA primer Required for synthesis of both strands DNAP alpha has primase activity and initiates synthesis of both lagging
and leading strands

DNA polymerases Major polymerizing enzyme is DNAP- 111 Major polymerizing enzymes are DNAP delta and DNAP epsilon.
The DNAP delta completes polymerization of the Okazaki fragments
(lagging st rand) and DNAP epsilon com pletes polymerization of
leading strand

Proofreading DNAP Ill al so has 3' to 5' exonuclease DNAP delta and epsilon both have 3'-> 5' exonuclease activity and
Activity so that any wrong base is removed therefore serve the proofreading function
and the correct one added

Gapfilling The RNA primer is removed and gap filled RNA primer is removed by RN Ase H and FEN1. Polymerase beta is
byDNAP involved in gap filling and DNA repair. Mitochondrial DNA replicat ion
is by DNAP gamma

Inhibitors Ciprofloxacin and Novobiocin inhibit Etoposide, Adriamycin and Camptothedn inhibitTopoisomerase
topoisomerase (DNA gyrase)
564 Section E: Molecular Biology

CH3 Old template strand


TGC
contains methylated
bases; so wrong
base could be

1 = DNA polymerase starts


working, by adding new
l identified

CH3 CH3 A portion around


bases one by one.
: ; JGC • that area is cut
i and removed by
an endonuclease

2 = In the new strand,


instead of T, one C is Gap Is filled with
correct base

Remo,~ :"+!Q
incorrectly added; this
blocks further elongation. sequences by DNA
polymerase

3 = 3'-5'-exonuclease nucleotides
activity of DNA polymerase
removes the unwanted base. CH3 TGC CH 3 DNA ligase seals
the nicks

Fig. 39.14: Mismatch repair


4 = DNA polymerase
continues to add the
correct base. Diseases Associated with DNA Repair
Fig. 39.13: Exonucleolytic proofreading of DNA polymerase
Xeroderma Pigmentosum
It is derived from the Greek terms xeres = dry and derma
removed . It is replaced by DNAP delta/epsilon by = skin. It is an autosomal recessive condition. Defect lies
polymerising deoxyribonucleotides. The ends are joined in the NER (nucleotide excision repair) mechanism.
by DNA ligase. Patients with XP have a 1000-fold greater chance of
developing skin cancer than do normal persons. Death
Base Excision Repair (BER) usually occurs in the second decade of life due to
Depurination is a common spontaneous damage occur- squamous cell carcinoma of skin.
ring in DNA. Deamination of cytosine to uracil may also
occur. These abnormal bases are removed and correct
Ataxia Telangiectasia
bases are added by BER. N-glycosylase recognizes and It is a common autosomal recessive disease. Sensitivity
removes the wrong base. An endonuclease will excise to UV, cerebellar ataxia, telangiectasia in eyes and
the remaining sugar. The correct base is added by a lymphoreticular neoplasms are common. Ataxia
DNA polymerase and joined by ligase. telangiectasia mutated (ATM) gene is present in 1% of
total population. The disease is manifested in 1:40,000
Mutations persons. Other such defects are summarized in Box 39.3.
Even after proofreading, some mistakes may be retained
in the print. Similarly a few defects may remain in the Telomere and Telomerase
DNA. These are called mutations. Mutations are due The replication always takes place from 5' to 3' direction
to a change in the base sequence of DNA. These may in the new strand. The DNA polymerase enzyme is not
result from faulty replication or repair of DNA. Mutation able to synthesize the new strand at the 5' end of the
rate is about 6 nucleotide changes per year in the germ new strand. In other words, a small portion (about 300
cell line cells of an individual. Moreover, out of every nucleotides) at the 3' end of the parent strands could
106 cell divisions, one somatic mutation is taking not be replicated (Fig. 39.15). This end piece of the
place. Mutations and mutagens are further explained in chromosome is called telomere. Unless there is some
Chapter 42. mechanism to replicate telomeres, the length of the
Chapter 39: Deoxyribonucleic Acid: Structure and Replication 565

BOX 39.3: Diseases associated with DNA repair mechanisms


Xeroderma Pigmentosum (XP): Defective NER mechanism;
sensitivity to UV light; skin cancers 3·
Ataxia Telangiectasia (AT): Defective ATM gene; sensitivity to UV
light; lymphoreticular neoplasms
Newly synthesized

!
Fanconi's Anemia: Defect in DNA cross link repair; increased occur- lagging strand
Telomerase
rence of cancer


Bloom's Syndrome: Defect in DNA ligase or helicase; lymphoreticu- New telomere repeat
lar malignancies
Cockayne Syndrome: Defect in NER mechanism; transcription factor
coupled repair, transcription factor II H is defective; stunted growth
and mental retardation.
Hereditary Polyposis Colon Cancer (Lynch syndrome): Mismatch
repair is defective.
! RNA template
inside telomerase

chromosomes will go on reducing at each cell division.


The stability of the chromosome is thus lost. The
shortening of telomere end is prevented by an enzyme
telomerase. It contains an RNA component, which DNA polymerase completes RNA produced
lagging strand by telomerase
provides the template for telomeric repeat synthesis. acts as primer
Telomerase acts like a reverse transcriptase. Telomerase
recognizes 3' end of telomere, and then a small DNA
strand is synthesized. In old age, the telomerase activity
is lost; leading to chromosome instability and cell death.
As cancer cells have increased and persistent activity
of telomerase, the cancer cells become immortal.
Elizabeth Blackburn, Carol Greider and Jack Szostak Elizabeth H Carol W JackW
Blackbum Greider Szostak
discovered the telomeres and telomerase, all the three NP 2009 NP 2009 NP 2009
were awarded Nobel Prize in 2009. b. 1948 b. 1961 b. 1952
Fig. 39.15: During replication, there will be a gap at the 5' end of
new strand. Telomerase is required to fill this gap
Hydrolysis of DNA
Those enzymes which hydrolyze only from the end of
Box 39.4: lnh1b1tors of DNA replication
the DNA molecule are called exonucleases. Those
Drug Action (inhibition of)
enzymes which hydrolyze the internal phosphodiester Antibacterial agents
bonds are called endonucleases. Certain endonucle- Ciprofloxacin Bacterial DNA gyrase
Nalidixic acid do
ases cut at specific sequences of DNA; these "molecular Novobiocin do
scissors" are called restriction endonucleases (RE) Anticancer agents
Etoposide Human topoisomerase
(see Chapter 43).
Adriamycin do
Doxorubicin do
Inhibitors of DNA Replication 6-mercaptopurine Human DNA polymerase
5-fluorouracll Thymidylate synthase
Certain compounds will inhibit bacterial en;::ymes, but
will not affect human cells; such drugs are useful as anti-
bacterial agents.
I LEARNING POINTS, CHAPTER 39
Some other compounds will inhibit human enzymes, 1. DNA sequence is afways written from the 5' end to
they will arrest new DNA synthesis, and arrest the cell 3' end. This is called polarity of the DNA chain.
division. Those drugs are therefore useful as anticancer 2. Chargaff's rule states that the number of purines is
agents. A list of such drugs is given in Box 39.4. equal to the number of pyrimidines.
566 Section E: Molecular Biology

3. The two strands run antiparallel to each other. 11 . DNA synthesis is continuous on the leading strand
4. Supercoiling of DNA is mediated by the enzymes and discontinuous on the lagging strand. This
topoisomerases and gyrases. results in the formation of the replication fork.
5. DNA is assembled into nucleosomes. 12. The small DNA molecules attached to their own
6. Histones are unusually rich in basic amino acids. RNA primers are called Okazaki fragments.
7. Transcriptionally active chromatin is called "euchro-
13. Xeroderma pigmentosum is an autosomal reces-
matin" and stains less densely as compared to
"heterochromatin" which is the inactive region of sive condition caused due to a defective nucleotide
the chromatin. excision repair mechanism.
8. DNA replication in vivo is semiconservative. 14. In every replication the 3' end of the parent strands
9. In mammals, the DNA polymerase is called alpha cannot be replicated. This end piece of the chromo-
polymerase. There are five DNA polymerases. some is called telomere. This shortening is prevented
(alpha, beta, gamma, delta and epsilon) by telomerases.
10. The DNA polymerase requires a RNA primer syn- 15. Telomerases have been implicated in aging pro-
thesized by RNA polymerase for initiation of its
cess and cancers.
activity. This forms the first step in DNA synthesis.

PART-1: ESSAY AND SHORT NOTE QUESTIONS

39-1. Describe the structure of DNA. What are the differences between DNA and RNA?
39-2. What are the salient features of Watson-Crick model of DNA?
39-3. Describe the process of DNA replication. Name two inhibitors of replication.

SHORT NOTE QUESTIONS


39-4. Watson and Crick model of DNA. 39-7. Okazaki pieces.
39-5. Base pairing rule. 39-8. Inhibitors of replication.
39-6. DNA polymerase.

PART-2: MULTIPLE CHOICE QUESTIONS


39-1 . Which nucleoside is found In DNA? C. Ataxia telangiectasia
A . Pseudouridine B. Dihydrouridine D. Cystic fibrosis
C . Deoxythymidine D. lnosine 39-6. Which enzyme protects DNA from ageing?
39-2. Nucleosome is: A. DNA polymerase B. Topoisomerase
A . Synonym of nucleolus C. Deoxyribonuclease D. Telomerase
B. DNA-RNA complex present in nucleus 39-7. Which is not true in Xeroderma pigmentosum?
C. mRNA attached with snRNA A. It is an autosomal recessive condition
D. DNA wrapped around histones B. Defect is in the repair mechanism of DNA, espe-
39-3. Fidelity of replication is ensured by: cially thymine dimers
A. Complementary base pairing C. Sensitivity to UV light is the major manifestation
B. Specificity of DNA polymerase D. Cells are unable to synthesize vitamin D even in
C. Recognition of specific nucleotide sequence presence of sunlight
D. Formation of phosphodiester bonds 39-8. In prokaryotes, DNA replication is inhibited by:
39-4. During DNA replication , 5'-GTCAAT-3' strand pro- A. Rifampicin B. Ciprofloxacine
duces a new complementary DNA strand with the C. Erythromycin. D. Streptomycin
following structure: 39-9. DNA synthesis in eukaryotes is affected by the
A. 5'-CAGTTA-3' B. 5'-ATTGAC-3' following, except:
C. 5'-ACACCG-3' D. 5'-GCCACA-3' A. Cytosine arabinoside
39-5. All are due to defective DNA repair, except: B. 5-fluorouracil
A. Xeroderma pigmentosum C. Rifampicin
B. Fanconi's anemia D. 6-mercaptopurine
Chapter 39: Deoxyribonucleic Acid: Structure and Replication 567

39-10. All the following statements are true with regard to C . Reverse transcriptase
Watson-Crick model of DNA, except D. Deoxyribonuclease
A. Double stranded helix 39-17. All cause fidelity in replication, except
B. DNA strands running in opposite directions A. There is an inherent repair mechanism
C. Adenine bonds with thymine B. DNA polymerase beta has an editing function
D. Guanine bonds with uracil C. DNA polymerase also has exonuclease activity
39-11 . All are true with regard to histones, except D. Replication stops if a wrong base is added
A. They are basic proteins 39-1 8. Regarding DNA structure which is not correct?
B. Nucleosomes contain histones and DNA A. Two strands are covalently bonded
C. Phosphorylation increases transcriptional activity
B. Two strands are antiparallel
D. Histone synthesis is coupled with DNA synthesis
C. Strands have polarity
39-12. Histones are modified by all, except:
D. Strands are complementary to each other
A. ADP ribosylation B. Acetylation
39-19. Which of the following is not a feature of mitochon•
C. Glycosylation D. Phosphorylation
drial myopathies?
39-13. Transcriptionally active chromatin is:
A . Cytoplasmic inheritance
A. Resistant to digestion by deoxyribonuclease
B. Heteroplasmy
B. Methylated at cytosine residues
C. Genomic imprinting
C. Densely packed
D. Low energy threshold
D. Called euchromatin
39-20. All are regulatory sequences on DNA, except
39-14. DNA replication requires all, except
A. Attenuator sequences
A. Elongation factor
B. Deoxynucleotide phosphates B. Enhancersequences
C. DNA polymerase C. Consensus sequences
D. RNA primer D. Promoter sequences
39-15. Which enzyme is not required for replication? 39-21. Denaturation of DNA is accompanied by all the
A. DNA ligase B. Topoisomerase changes,except
C. Helicase D. Reverse Transcriptase A. Increase in viscosity
39-16. All are involved in synthesis of DNA, except B. Hyperchromicity
A. DNA polymerase C. Unstacking of bases
B. Topoisomerase D. Destabilization of DNA helix

ANSWERS OF MULTIPLE CHOICE QUESTIONS

39-1 . C 39-2. D 39-3. A 39-4. B 39-5. D 39-6. D 39-7. D


39-8. B 39-9. C 39-1 0. D 39-11 . C 39-12. C 39-13. D 39-14. A
39-15. D 39-16. D 39-17. D 39-18. A 39-19. A 39-20. C 39-21 . A

PART-3: VIVA VOCE QUESTIONS AND ANSWERS

39-1. What are the salient features of Watson-Crick 39-3. What are histones?
model of DNA? They are nuclear proteins found in nucleus. They bind
(i) Right-handed double helix; (ii) Each turn of helix has with DNA and stabilizes DNA structure.
10 base pairs; (iii) Hydrogen bonds between purines 39-4. What is a nucleosome?
and pyrimidines; (iv) DNA strands running in opposite DNA wrapped around histones.
directions (anti parallel). 39-5. What is meant by DNA replication?
39-2. What is base pairing rule (Chargaff's rule)? During cell division, each daughter cell gets an exact
Pairing (hydrogen bonding) of adenine with thymine copy of the genetic information of the mother cell.
and guanine with cytosine. Purine is paired with This process of copying the DNA is known as DNA
pyrimidine. A+ T = G+C. replication.
568 Section E: Molecular Biology

39-6. What is semi-conservative replication? 39-13. What is meant by the lagging strand?
Each one of the newly synthesised DNA has one-half The strand which is discontinuously synthesized is
parental strand (old) and one-half new DNA strand. referred to as the "lagging strand" and the one conti-
Since half of the original DNA is conserved in daughter nuously polymerized as the "leading strand"
DNA, it is called semi-conservative replication. 39-14. What are Okazaki fragments?
39-7. What are the enzymes required for ONA replication?
The small DNA molecules attached to its own primer
DNA polymerase, Topoisomerase, DNA ligase. RNA in the lagging strand are called Okazaki
39-8. What is a replisome? fragments.
DNA replication needs more than 20 enzymes and 39-15. Xeroderma pigmentosum is due to deficiency of
proteins, collectively called DNA replicase system or what process?
replisomes. Defect in DNA repair mechanism (nucleotide excision
39-9. What is a replication bubble? repair).
Helicases move on both directions, separating the 39-16. In prokaryotes, DNA replication is inhibited by
strands in advance of the replication. This forms a what drugs?
replication bubble. Ciprofloxacine, nalidix acid, novobiocin.
39-10. Replication is in which direction? 39-17. DNA replication in eukaryotes is inhibited by what
Polymerization of the new strand of DNA is taking place drugs?
from 5' to 3' direction. This means that the template is 5-fluorouracil; 6-mercaptopurine; Cytosinearabinoside;
read in the 3' to 5' direction. Etoposide.
39-11. How replication starts? 39-18. What is telomere?
An RNA primer, about 100-200 nucleotides long, is The end piece of the chromosome is called telomere.
synthesised by the RNA primase. 39-19. What is telomerase?
39-12. What is meant by semi discontinuous nature of It replicates the end piece of chromosomes.
replication? 39-20. What is the clinical significance of telomerase?
DNA synthesis is always in the 5' to 3' direction in both In old age, the telomerase activity is lost; leading to
strands. In one strand, the replication is taking place chromosome instability and cell death. As cancer cells
continuously; but in the other strand replication is in have increased and persistent activity of telomerase,
small pieces. the cancer cells become immortal.
t - - -_ __ _ Chapter 40
Transcription

Chapter at a Glance
The learner will be able to answer questions on the following topics:
0 Ribonucleic acid 0 Post-transcriptional processing
0 Messenger RNA 0 Reverse transcriptase
0 Transcription

I RIBONUCLEIC ACID
Ribonucleic acid (RNA) is also a polymer of purine
and pyrimidine nucleotides linked by phosphodiester
bonds. However, RNA differs from DNA as shown in Table DNA
40.1 and Figure 40.1. Cellular RNAs are of five types:
a. Messenger RNA (mRNA): The genetic information
present in DNA is transcribed into mRNA, which is
the coding RNA . They have a short half life and are
generally degraded quickly. mRNA

b. Ribosomal RNA (rRNA): 28S, 18S and 5S are the


Fig . 40.1: DNA is double-stranded, while RNA is single-stranded
major varieties. They are involved in protein biosyn-
thesis and are very stable.
c. Transfer RNA (tRNA): There are about 60 different
species present. They are very stable.
d. Small RNA: There are about 30 different vari e-
RNA DNA ties. They are very stable. Small Nuclear RNAs
1. Mainly seen in cytoplasm M ostly inside nucleus (SnRNAs) are a subgroup of small RNA. They are
2. Usually 100-5000 bases Millions of base pairs involved in mRNA splicing.
3. Generally single stranded Double stranded e. Micro RNA (miRNA): They regulate genetic expres-
4. Sugar is ribose Sugar is deoxyribose sion by altering the function of mRNA.
5. Purines: Adenine, guanine Ad enine, guanine
pyrimidines: cytosine, uracil cytosine, thymi ne
Central Dogma of Molecular Biology
6. Guanine content is not Guanine is equal to cytosine
equal to cytosine and and adenine is equal to As shown in Figure 40.2, the information available in
adenine is not equal to uracil thymine the DNA is passed to messenger RNA, which is then
7. Easily destroyed by alkali Alkali resistant used for synthesis of a particular protein.
570 Section E: Molecular Biology

Coding strand S' G-T-C-A-A-T-C-C-G-3'

Template strand 3' C-A-G-T-T-A-G-G-C-5'

mRNA transcript 5' G-U-C-A-A-U-C-C-G-3'

Fig. 40.3: Transcription. The mRNA base sequence is comple-


mentary to that of the template strand and identical to that of the
coding strand. In mRNA, U replaces T

sequence of mRNA is called coding strand (Fig. 40.3).


'7 New protein
As it is complementary to the template strand, it is also
_ synthesis
'-"" called antitemplate strand.

Messenger RNA or mRNA


Fig. 40.2: Central dogma of molecular biology
The mRNA acts as a messenger transporting the infor-
mation in the gene in DNA to the protein synthesizing
machinery in cytoplasm. It carries the message to be
translated to a protein. The template strand of DNA is
DNA transcribed into a single stranded mRNA. This is accom-
plished by the DNA dependent RNA polymerase. The
mRNA is a complementary copy of the template strand
5' mRNA 3' 5· mRNA3'
of the DNA (Fig. 40.3). However, thymine is not present
Fig. 40.4: Genes may be on any strand of DNA. Transcription is in RNA; instead uracil will be incorporated. In the DNA,
in 3' to 5' direction of the template
one strand will harbor certain genes, while others are
borne by the other strand (Fig. 40.4).
Replication, Transcription
and Translation I TRANSCRIPTION PROCESS
DNA replication is like printing a copy of all the pages of
Roger Kornberg got Nobel Prize in 2006 for his studies
a book. The replication process occurs only at the time on molecular basis of transcription. He is the son of Ar-
of cell division. Transcription can take place at any time thur Kornberg (Nobel Prize 1959) who worked on DNA
but only certain selected regions of the DNA are copied. replication. Transcription includes initiation, elongation,
This is like taking xerox copy of particular page of the and termination steps followed by post-transcriptional
book. So, the genetic information in DNA is transcribed processing. A summary is shown in Figure 40.5.
(copied) to the messenger RNA (mRNA). During trans-
cription , the message from the DNA is copied in the Mammalian RNA Polymerases
language of nucleotides (4 letter language). The mRNA
There are three different DNA dependent RNA polymera-
then reaches the cytoplasm where it is translated into
ses (RNAP) in higher organisms.
functional proteins (Fig. 40.2). During translation, the
i. RNAP type II or B is the main enzyme synthesizing
nucleotide sequence is translated to the language of
mRNAs. It is inhibited by alpha amanitin (a toxin
amino acid sequence (20 letter language).
from the mushroom Amanita phalfoides).
ii. RNAP type I or A is responsible for synthesis of
Template and Coding Strands
rRNA (ribosomal); it is not inhibited by amanitin.
The template strand is transcribed to give rise to mRNA. iii. RNAP type Ill or C is responsible for production of
The template strand has the complementary sequence tRNA; it is moderately sensitive to amanitin.
of mRNA. The opposite strand has the same sequence Bacterial RNA polymerase is different from mamma-
as the mRNA. The DNA strand having the same lian RNAP.
Chapter 40: Transcription 571

! l
RNA polymerase attaches to DNA RNAP enzyme

l. Start site

!
Promoter site
~ .lqV""'/&.."'-'JJ.:J~ , DNA
Template 3' C A T T GA T G
DNA - - -- - - - - - - -- - - 5'
strand 5' G U A 3'
New RNA strand - 11111111"'"""
Roger Kornberg, F ig . 40.6 : Initiation of transcription
NP 2006
(b. 1947)
the starting point is numbered as -1 . Further upstream,
these negative numbers are increased. The DNA helix
partially unwinds, and the RNAP binds with the promoter
site on DNA with the help of sigma factor (Fig. 40.7).
When it reaches the appropriate site on the gene, the
first nucleotide of the mRNA attaches to the initiation site
Fig. 40.5: Transcription process on the beta subunit of RNAP. This becomes the 5' end of
the mRNA. It will be complementary to the base present
Signals for Initiation of Transcription in the DNA at that site. This is the initiation of transcrip-
tion. The next nucleotide attaches to the RNAP. A phos-
Promoters
phodiester bond is formed. Then the enzyme moves to
There are certain consensus sequences on DNA which the next base on the template DNA (Fig. 40.6).
act as start signals which may be located upstream or
downstream from the start site. The RNAP attaches at Mammalian System
the promoter site on the template DNA strand. In eukaryotes, the situation is more complex. There are
atleast 7 transcription factors, collectively called as
TATA Box and Golberg-Hogness Box Tf-11. First, the TATA box is recognized by TSP (TATA
In the case of bacteria, about 10 bp upstream, there is binding protein). In humans about 105 transcription ini-
sequence 5'-TATAAT-3'. This is referred to as TATA box tiation sites are available.
or Pribnow box. The TATA box is not on template strand,
but on coding strand. In mammals, the exact sequence Elongation Process of Transcription
in TATA box is slightly different {TATAAA) and is known The RNAP moves along the DNA template. New nucleo-
as Goldberg-Hogness box. This signal sequence located tides are incorporated in the nascent mRNA, one by
at -25 to -30 position indicates the start site. one, according to the base pairing rule {Fig. 40.8). Thus,
A in DNA is transcribed to U in mRNA; T to A; G to C and C
Other Regulatory Signals to G. The synthesis of mRNA is from 5' to 3' end. That means
Enhancers increase the rate of transcription and silencers the reading of template DNA is from 3' to 5' (Fig. 40.3).
decrease the rate. Other regulatory signals for transcrip- This is analogous to the polarity in DNA synthesis.
tion are hormone response elements (HRE) {see Chap- As the RNAP moves along the DNA template, rewin-
ter 45), repressors, inducers and derepressors {see ding of the double helix occurs at the upstream areas. RNAP
Chapter 42). has DNA unwinding property. Topo-isomerase will also
help in this unwinding process. A transcription bubble
Initiation of Transcription containing RNAP, DNA and nascent RNA is formed
{Fig. 40.7). This bubble is about 20 bp length. RNAP
Bacterial System has no nuclease activity; so there is no proof reading.
The starting point of transcription corresponds to the 5' Hence , fidelity is less; mistake rate in mRNA transcrip-
nucleotide of the DNA, designated as +1. Then num- tion is 104 or 105 times more than DNA replication. But it
bering is done 2, 3, 4, etc. to the downstream region of is less serious, since these mistakes are not transmitted
the DNA. The nucleotide adjacent to downstream of to the daughter cells or to the next generation .
572 Section E: Molecular Biology

l l
RNAP enzyme
Start site
Pro=""''
Template
DNA
strand
3
, l C A T TG A TG

5' G U A A C U A C 3'
5'

DNA double helix New RNA strand _ _ . ,.,.,.,.,.,.,.,.,.,.,.,.,.,.,.,.


unwinds partially

Fig. 40.7: DNA unwinds for transcription process Fi g . 40.8: Elongation process of transcription

-
Exons lntrons

DNA I

:, <> em 8 ... a I

- -
e Cl I Iii CC- :::J
Rho factor
identifies stop Primary RNA transcript ! Transcription
signal II I CS 0 AAAAA


I Adding 5' cap and
5' cap poly A tail
c::j CSI I Cll Q AAAAA

Termination Removal of introns i RNA splicing


Mature mRA AAAAA
RNA released Export I from nucleus to cytoplasm
Fig. 40.9: Termination of transcription Fig. 40.10: Splicing process; removal of introns

Termination of Transcription a. Poly-A tailing: The 3' terminus is polyadenylated


in the nucleoplasm (Fig. 40.10). This poly-A tail
The specific signals are recognized by a termination pro-
may be 20 to 250 nucleotides long. This tail pro-
tein , the Rho factor (abbreviated with Greek letter, "p").
tects mRNA from attack by 3' exonuclease.
When the Rho factor attaches to the DNA, the RNAP
b. 5' capping at the 5' terminus by ?-methyl guano-
cannot move further. So, the enzyme dissociates from
DNA and consequently newly formed mRNA is released sine triphosphate.
(Fig. 40.9). c. Methylation of N6 of adenine residue and
In humans, the termination signals exist far down- 2'-hydroxyl group of ribose are common.
stream of the coding sequence, usually 1000 to 2000 d. Removal of introns
bases away. The differences between mammalian and e. Splicing of exons (connect together). These pro-
bacterial transcription are summarized in Table 40.2. cesses occur mainly in the nucleoplasm.

Post-transcriptional Processing Removal of lntrons


i. In bacteria, mRNA is not changed; and translation The primary transcripts are very long; they have molecu-
of mRNA starts even before completion of trans- lar weights more than 107 • Molecular weight of mature
cription. However, in mammalian cells, the mRNA mRNA is about 1- 2 x 106• This means large portions of
formed and released from the DNA template is hnRNA are cleaved off. The primary transcript contains
known as the primary transcript. It is also known coding regions (exons) interspersed with non-coding
as heteronuclear mRNA or hnRNA. regions (introns). These intron sequences are cleaved
ii. In mammalian system, it undergoes extensive pro- and the exons are spliced (combined together) to form
cessing to become the mature mRNA. These modi- the mature mRNA molecule. This processing is done in
fications are: nucleus (Fig. 40.10).
Chapter 40: Transcription 573

TABLE 40.2: Differences between prokaryot1c and eukaryollc transcription

Feature Prokaryotes Eukaryotes


Site Nucleus Nucleus
Enzym, DNA dependent RNA polymerase RNAP II for mRNA transcription, RNAPl for rRNA and RNAP3 for tRNA
lnitiatic n Sigma subunit binds to The recognition of promoter requires transcription factors, TATA binding protein
promoter site
Start si nals TATA box (Pribnow box} Goldberg-Hogness Box
Termi n ~tion Termination may be Rho Termination may be Rho factor dependent o r independent. A lariat
factor dependent or Rho structure is formed and the RNAP dissociates.
independent
Post-tr nscriptional mRNA transcribed is mRNA undergoes several post-transriptional modifications like 5 methyl
modifi ations immediately translated capping, poly A tailing, splicing etc. in the nucleoplasm .
without any modification
Action ~ f antibiotics Inhibited by actinomycin Inhibited by actinomycin and alpha amanitin
and rifampicin

Sma I Nuclear RNAs (snRNAs) a ribozyme. The amino acid binding sequence CCA is
Their ,ize ranges from 90-300 nucleotides. They take added to the 3' end. Modification of bases is another
part ir the formation of spliceosomes. All of them are change during processing.
locate j in the nucleus. They complex with specific pro-
teins, to form small nuclear ribonucleoprotein particles Ribozymes
(SnRI\ PsJ. It is pronounced as "Snurps". Production Enzymes made up of RNA are called ribozymes. Ribo-
of aut ::>antibodies against "Snurps" is implicated in the zymes or RNA enzymes are catalytic RNA molecules
patho ienesis of systemic lupus erythematosis (SLE), with sequence specific cleavage activity. Thomas Cech
a fata auto-immune disease. and Sidney Altman discovered the ribozymes, for which
they were awarded Nobel Prize in 1989. Spliceosomes
Splii~eosomes contain ribozymes as well as protein components which
SnRN 0 s associated witn hnRNA at the exon-intron junc- serve to stabilize the structure of ribozymes. RNAse-P is
tion fc rm spliceosomes. This is taking place inside the another ribozyme, which generates the ends of tRNAs.
nucle JS . Cuts are made at both ends of intron; it is Peptidyl transferase present in ribosomes (used for
remO\ ed and exon-exon ends are Iigated at G-G residues. protein biosynthesis is another exam ple of ribozy me.
For elpcidation of spliceosome activity, Richard Roberts
and Rhillip Sharp were awarded Nobel Prize in 1993. Reverse Transcriptase
essing of rRNA Generally speaking, the genes are made up of DNA.
mal RNA is transcribed by RNAP-I and different Usually, DNA dependent RNA polymerase transfers the
rRN molecules are transcribed as single precursor. information of DNA to mRNA. However genetic material
Then it is cleaved into three different molecules; 28s of some animal and plant viruses is made up of RNA.
RNA, 5.8s RNA & 18s RNA. The rRNA genes are
locat d in the nucleolar region .

Pro essing of tRNA


RNA -Ill transcribes tRNA. Similarly, pre-tRNA (pre-
curs r of transfer RNA) is also synthesized as a longer
mole ule. The precursor is trimmed at both 5' and 3' ends Sidney Thomas R Richard J Phillip A
to for the mature tRNA, so as to give rise the functional Altman Cech Roberts Sharp
NP 1989 NP 1989 NP 1993 NP 1993
tRN . The 5' end is trimmed by RNAse-P which is also b. 1939 b. 1947 b. 1943 b. 1944
574 Section E: Molecular Biology

TABLE 40.3: lnh1b1tors of RNA synthesis

1
Virus RNA Inhibitor Source Mode ofaction
Primer RNA 5' AAAAA 3' Polymerization by Actino- Antibiotics from Insertion of phenoxa-zone
reverse transcriptase mycin-D streptomyces ring between two G-C bp
of DNA
3' AI\I\.I\/\AI\I\I\I\I\I\I\AAI\I\I\AAl\lt 5•
Rifampicin Synthetic derivative of Binds to beta subunit of
RNA-DNA hybrid 5' AMA- - - - - - 3'

l
rifamycin RNA polymerase which is
inactivated
RNAse H hydrolyzes RNA

!
Alpha Toxin from mushroom Inactivates RNA
DNA copy of RNA amanitin polymerase II
DNA polymerase Ill
3' -deoxy Synthetic analog Incorrect entry into chain
Double stranded DNA adenosine causing chain termination

Fig. 40.11: Action of reverse transcriptase Interfering RNA or RNAi or siRNA


Andrew Fire and Craig Mello found that when double-
Retrovirus is a subgroup of RNA viruses. The human
stranded RNA was given to round worms, it would sile-
immunodeficiency virus (HIV) causing AIDS is a retro- nce the gene corresponding
virus. Here, the RNA acts as a template. Based on this
to that RNA. They were
RNA, the enzyme, RNA dependent DNA polymerase or awarded Nobel Prize in 2006.
reverse transcriptase will make a new DNA strand. Temin
RNA interference is a protec-
and Baltimore isolated this enzyme in 1970 and they tive mechanism against vi-
were awarded the Nobel Prize in 1975. ruses, which sometimes ere- .___ _ ___,
From the RNA-DNA hybrid, the RNA part is hydro- Andrew Craig Mello
ate double-stranded RNA Fire
lysed by a specific RNAse-H . The remaining DNA acts NP 2006
when they replicate. Short NP 2006 b. 1960
as a template to produce double stranded DNA. Thus double-strand RNA, again b. 1959
genetic information is transferred from RNA to DNA about 21 to 25 bases, would silence the corresponding
(Fig. 40.11 ). Some of the tumor viruses were also shown gene. RNAi is a faster way to turn off genes.
to possess reverse transcriptase. The presence of the RNA interference (RNAi) is a naturally occurring
enzyme may be taken as an indication of a retrovirus biological pathway in which small, double-stranded RNA
infection. molecules suppress gene expression in an exquisitely
sequence-specific manner. Exogenous manipulation of
Inhibitors of RNA Synthesis RNAi is being explored as a powerful method of silen-
cing disease-causing genes in incurable neurological
Actinomycin D and Mitomycin intercalate with DNA strands,
disorders. Preclinical trials in animals are being con-
thus blocking transcription. They are used as antican-
ducted.
cer drugs. Rifampicin is widely used in the treatment of
tuberculosis and leprosy (natural antibiotic rifamycin is Antisense Therapy
produced from streptomyces). Other inhibitors of RNA The mRNA contains a message or "sense" to be trans-
synthesis are shown in Table 40.3. lated into protein. If a nucleotide having complementary
sequence to an mRNA is made, it is said to be "anti-
sense". When antisense oligonucleotide (either RNA
or DNA) is added , it will trap the normal mRNA and so
protein biosynthesis can be stopped. Clinical trials on
HIV and cancer are being conducted using antisense
molecules.

Howard Temin
NP 1975
David Baltimore
NP 1975
Robert Holley
NP 1968 ILEARNING POINTS, CHAPTER 40
1934-1994 b. 1938 1922- 1993
1. Template strand of the DNA is transcribed to mRNA.
Chapter 40: Transcription 575

2. R AP type II or B is the enzyme synthesizing RNA 11 . Both rRNA and tRNA are non coding RNAs, but play
in mammals. It is susceptible to amanitin. crucial roles in translation.
3. A transcription bubble contains RNA polymerase, 12. The enzyme which brings about transcription is
D A and nascent RNA. DNA dependent RNA polymerase.
4. li rmination of transcription can be Rho factor 13. Bacterial RNA polymerases are different from mam-
d pendent or independent. malian enzymes; so antibiotics which inhibit bacte-
5. P st-transcriptional processing of the primary RNA rial RNAP will not affect human beings.
tr nscript includes RNAse-P tailing at 3' end, cap- 14. The reverse transcription process is catalyzed by
pi g at 5' end , methylation and intron splicing. RNA-dependent DNA polymerase (RT) which was
6. R bozymes are enzymes made up of RNA. Exam- first reported in tumor viruses.
p s are RNAse-P, peptidyl transferase and spliceo- 15. The HIV is the most clinically relevant retrovirus
s mes. now.
7. R verse transcriptases are RNA dependent DNA 16. The inhibitor of RNA synthesis (in both eukaryotes
p lymerases. They synthesize a DNA strand using and prokaryotes) is actinomycin D and mitomycin
A as their template. Retroviruses, such as HIV which intercalate with DNA, blocking transcription.
a d tumor viruses, possess this enzyme. 17. The widely used anti-tuberculous drug rifampicin
8. D A replicates before cell division, so that each (Rifamycin) inhibits prokaryotic RNA polymerase.
d ughter DNA molecule gets an exact copy of the 18. The small interfering RNA (si-RNA) protects the
p rent cell. genome from bacteriophages and viruses in lower
9. T e genetic information present in the mRNA is
organisms.
tr nscribed to mRNA and then translated into the
19. In eukaryotic cells, small dsRNA molecules will
a ino acid sequence of the polypeptide chain, as
silence specific genes, and this is a normal regula-
t e gene product.
10. T e strand of DNA which is transcribed is called tory mechanism.
t e templet strand, which is complementary to the 20. Use of synthetic antisense strands (RNA mole-
RNA. The anti-template strand has the same cules having complementary sequence of cellular
!
s quence as the mRNA transcript, and is called the mRNA) to manipulate genes are now being used as
ding strand. therapeutic tools.

PART-1: ESSAY AND SHORT NOTE QUESTIONS


40-1 . Describe different types of RNA.
40-2. Give a detailed account of the transcription process. How is it regulated? Name inhibitors of transcription.

SHORT NOTE QUESTIONS

40-3. RNA polymerase. 40-6. lntrons and exons.


40-4. Reverse transcriptase. 40-7. Inhibitors of transcription.
40-5. Post-transcriptional modifications.

PART-2 : MULTIPLE CHOICE QUESTIONS

4 -1. All the bases are found in mRNA, except B. A subunit of RNA polymerase
A. Adenine B. Guanine C. A subunit of 50 S ribosome
C. Uracil D. Thymine D. Responsible for initiation of replication
4 -2. The nitrogenous base absent in DNA Is: 40-4. The processing of mRNA involves:
A. Uracil B. Cytosine A. Capping at 5' terminus
C. Thymine D. Guanine B. Poly A tail attachment
4 -3. Sigma factor is: C. Removal of intrans
A. A subunit of DNA polymerase D. Translocation of mRNA into nucleus
576 Section E: Molecular Biology

40•5. lntron is the portion of: 40-1 6. In an organism, only DNA, but not RNA is
A. DNA that is cleaved off during replication labelled when grown in a medium co ntaining
B. mRNA that is removed after transcription rad ioactive nucleotide of:
C. tRNA that is added on after its synthesis A. Adenine B. Guanine
D. Protein that is removed after translation C. Thymine D. Cytosine
40-6. Which is an inhibitor of transcription in 40-17. The RNA present in largest amounts in a cell at
eukaryotes? any time is
A. Rifampicin 8 . Chloramphenicol A. mRNA B. tRNA
C. Alpha amanitin D. Streptomycin C. rRNA D. hnRNA
40-7. Mechanism of action of mitomycin is: 40-1 8. All the bases are found in mRNA, except
A. Inactivation of EF-2; blocks elongation of proteins A. Adenine B. Guanine
C. Uracil D Thymine
B. Intercalation with DNA strands; blocks transcription
40-19. The nitrogenous base absent in DNA is:
C. Abrupt termination of protein synthesis
A. Uracil B. Cytosine
D. It is a structural analogue of amino acyl tRNA
C. Thymine D. Guanine
40-8. Reverse transcriptase differs from DNAP in
40-20. The TATA box:
that
A. Present on the template strand
A. Synthesizes DNA in 5' to 3' direction
B. Present about 70 bp away from transcription start
B. Has 3' to 5 ' exonuclease activity
site
C. Follows the base pairing rule
C. Serves as the signal for attachment of RNAP-11
D. Can insert IMP in the growing strand
D. Acts as a silencer of the gene
40-9. RNAP is inhibited by
40-21 . The processing of mRNA involves
A. Streptomycin B. Chloramphenicol
A. Capping at 5' terminus
C. Ciprofloxacin D. Rifampicin
B. Poly A tail attachment
40-10. Which of the following nucleotides is not
C. Removal of intrans
present in mRNA?
D. Translation of mRNA into nucleus
A. GMP B. TMP
40-22. All are post-transcriptional modifications in
C. CMP D. UMP
mRNA, except
40-11 . Which of the enzymes listed below is involved
A. Modification of bases
in processing of RNA?
B. Splicing of intrans
A. RNA polymerase II
C. Trimming of 3' end to CCA sequence
B. Ligase
D. Attachment of poly A tail.
C. Helicase 40-23. Transcription is terminated when:
D. Ribonuclease-P A. The codon on the mRNA is AUG
40-12. All the following are inhibitors of eukaryotic B. Rho factor binds to mRNA
transcription, except: C. The codon on the mRNA is UGA
A. Rifampicin B. Puromycin D. The ribosome reaches the poly-A tail on mRNA
C. Actinomycin D. Amanitin 40-24. Which is caused by autoantibodies to Snurps?
40-13. The coding unit of the DNA is called as: A. Xeroderma pigmentosum
A. lntron B. Exon B. Mushroom toxicity
C. Cistron D. Prion C. Systemic lupus erythernatosus
40-14. Which is not a transcription start signal? D. Thalassernia
A. CAAT box B. Hogness box 40-25. lntron is the portion of:
C. Pribnow box D. Homeo box A. DNA that is cleaved off during replication
40-15. All are true regarding transcription, except: B. rnRNA that is removed after transcription
A. RNA polymerase has proofreading capacity C. tRNA that is added on after its synthesis
B. RNA strand is complementary to template strand D. Protein that is removed after translation
of DNA 40-26. All are correct with retroviruses, except:
C. Is inhibited by rifampicin A. Contains a DNA dependent RNA polymerase
D. RNA is synthesized in the 5' to 3' direction B. Transcription occurs in the reverse direction
Chapter 40: Transcription 577

C. Viral RNA is copied into host cell DNA 8. Intercalation of DNA strands and blocks trans-
D. New DNA is incorporated into host cell genome. cription
40- 7. Mechanism of action of mitomycin is: C. Abrupt termination of protein synthesis
A. Inactivation of EF-2, and so blocks elongation D. It is a structural analogue of amino acyl tRNA

ANSWERS OF MULTIPLE CHOICE QUESTIONS

40 1. D 40-2. A 40-3. 8 40-4. 8 40-5. 8 40-6. C 40-7. 8


40 8. D 40-9. 8 40-10. 8 40-11. D 40-12. A 40-13. C 40-14. D
40-15. A 40-16. C 40-17. C 40-18. D 40-19. A 40-20. C 40-21 . 8
40- 2. C 40-23. 8 40-24. C 40-25. 8 40-26. A 40-27. 8

PART-3: VIVA VOCE QUESTIONS AND ANSWERS

40-1 . What is transcription? 40-10. What is a spliceosome?


The process of making a complementary mRNA copy Small nuclear ribonucleoprotein particles (SnRNPs)
of DNA. associated with hnRNA at the exon-intron junction
40-2. The mRNA is a complementary copy of which form spliceosomes.
strand of DNA? 40-11. What are ribozymes?
Template strand. Enzymes made up of RNA are called ribozymes.
40-3. What is the coding strand? 40-12. Give some examples of ribozymes.
The opposite strand of template strand. Coding strand Spliceosomes, RNAse-P (which generates the ends
has the same sequence as of the mRNA. of tRNAs) and Peptidyl transferase (present in ribo-
40-4. What is the enzyme necessary for transcription? somes) are examples of ribozymes.
DNA dependent RNA polymerase or RNAP. 40-13. Name inhibitors of RNA synthesis
40-5. What is the direction of transcription? Rifampicin , Actinomycin, Mitomycin and Amanitin.
It takes place in the 5' to 3' direction. 40-14. What is the mechanism of action of mitomycin?
40-6. What is TATA Box? Actinomycin D and Mitomycin intercalate with DNA
It is a signal for initiation of transcription in prokaryotes. strands, thus blocking transcription. They are used as
It is about 10 bp upstream of starting of mRNA anticancer drugs.
synthesis. 40-15. What is the mechanism of action of Rifampicin?
40-7. What is the corresponding signal in mammals? It binds to beta subunit of RNA polymerase, and inac-
Godberg-Hogness Box. tivates it. Rifampicin is widely used in the treatment of
40-8. What e~e post-transcriptional modifications? tuberculosis and leprosy.
Removal of introns; addition of a cap at 5' end: adding 40-16. What is reverse transcriptase?
poly A tail at 3' end; and methylation. It will make a new DNA strand based on the RNA temp-
40-9. What is intron? late. Thus genetic information is transferred from RNA
Part of mRNA that is removed is called intron. to DNA.
_ _ _ _ _Chapter 41

Genetic Code and Translation

Chapter at a Glance
The learne r will be able to answer question s on the following topics:

D Transfer RNA 0 Post-translational processing


0 Ribosomal RNA 0 Inhibitors of protein synthesis
0 Genetic code 0 M itochondrial DNA
0 Protein biosynthesis

l fROTEIN BIOSYNTHESIS in the cytoplasm. Each molecule is only 73-93 nucleotides


in length; much shorter than mRNA molecules.
Synthesis of proteins are based on the genetic informa-
tion present on the DNA. The DNA is first transcribed Structure of tRNA Molecule
into the mRNA nucleotide sequence and then trans- The transfer RNAs show extensive internal base pairing
lated to the polypeptide chain . The mRNA encodes the and acquire clover leaf like structure (Fig. 41 .2). They
information. The tRNA transports the amino acid. Ribo- contain a significant proportion of unusual bases. These
somes provide the platform for the protein synthesizing include dihydrouracil (DHU), pseudouridine ('I'), and
machinery. This is summarized in Figure 41 .1. hypoxanthine. Moreover, many bases are methylated.

Transfer RNA (tRNA)


3'end
They transfer amino acids from cytoplasm to the ribo- Acceptor arm -amino acid
carrying amino acid c
somal protein synthesizing machinery; hence the name C

transfer RNA. RNA Polymerase 11 1 transcribes tRNA.


Since, they are easily soluble, they are also referred to as
Clover leaf
soluble RNA or sRNA. They are RNA molecules present
Pseudo-
uridine
~: 11 I I I I I I I I I I I 3' DNA coding strand arm

l 5' Template strand


Transcription
arm
'f' = pseudouridine
(pronounced as "psi")

5' ,,.,.,,.,.IV\,..,.," "'""""""""',._,.,, 3


l
1
mRNA extra arm

Translation

NH2ooooooooooooooCOOH Polypeptide chain Anticodon arm


Fig. 41 .1: Expression of a gene into a protein Fig. 41.2: Transfer RNA-general structure
Chapter 41: Genetic Code and Translation 579

DHU arm serves as the recognition site for the enzyme


3• ! Ser)
which adds the amino acid.
-5'

tRNA Pseudouridine Arm of tRNA


The opposite arm is called pseudouridine arm, as it
contains a pseudouridine. It is generally denoted with
the Greek alphabet \v" which is pronounced as "psi"
(Fig. 41 .2). It is involved in binding tRNA to ribosomes.
U C A Anticodon About 75% of tRNA molecules possess a short extra
5' -----...ii.---
A G U codon
3' mRNA
arm, about 3 to 21 base pairs in length.

Fig. 4 .3: Transfer RNA carrying serine


Processing of tRNA
The tRNA is synthesized as a long precursor and is
Ace ptor Arm is at the 3' End
trimmed by Ribonuclease P, a ribozyme. The CCA se-
It car es the amino acid (Fig. 41 .2). The end sequence quence is added at 3' end and bases are modified.
is CC -3'. The 3' end hydroxyl group is forming an ester
ith the carboxyl end of amino acids. Ribosomal RNA (rRNA)

Anti odon Arm of tRNA Ribosomes provide necessary infrastructure for the
mRNA, tRNA and amino acids to interact with each other
At th opposite side of the acceptor arm is the anticodon for the translation process. Thus ribosomal assembly is
arm ( ig. 41 .3). It recognizes the triplet nucleotide codon the protein synthesizing machinery. Nucleolus is the
prese t in mRNA. The specificity of tRNA resides in area where rRNA is synthesized and ribosomal assem-
the a ticodon site, which has base sequences comple- bly is produced. For studies on structure and function
ment ry to that of mRNA codon. of ribosomes, Nobel Prize was awarded in 2009 to
F r example, if the mRNA has a codon with the Venkataraman Ramakrishnan (born in India), Ada Yonath
sequ nee UUU, the anticodon sequence of the tRNA and Thomas Steitz. Ribosomal RNA is synthesized by
will b AAA, by which it base pairs with mRNA codon. RNAPI .
So, t e specific tRNA can bind correctly to the mRNA
codo s. In this case, the UUU codon is translated as
Components of rRNA
phen !alanine. Recognition of codon by the tRNA anti-
codo is illus~~ated in see Figure 41 .3. The mammalian ribosome has a sedimentation constant
T e tRNA molecule will show specificity in both of 80S unit. It has a larger 60S subunit and another
smaller 40S subunit. They contain different rRNAs and
aspe ts; in recognizing the mRNA codon as well as in
specific proteins. Ribosomal RNA has catalytic activity.
acce ting the specific amino acid coded by that codon.
Peptidyl transferase activity is carried out by 28 S RNA
The t NAs act as adapter molecules between mRNA
which acts as a ribozyme.
and t e amino acids coded by it. Methionine-tRNA was
isolat d by Paul Berg in 1956 (Nobel Prize, 1980). The
compete sequencing of alanine-tRNA was done by
Robe Holley in 1963 (Nobel Prize, 1968). Khorana
(lndi n born US scientist) got Nobel Prize in 1968, for
synth sizing the gene for alanine tRNA.

Paul Har Gobind Marshall W Gunter


Berg Khorana Nirenberg Blobel
The arm or DHU region is so named due to the pre- NP 1980 NP 1968 NP 1968 NP 1999
senc of a dihydrouridine in that area (Fig. 41 .2). The b. 1926 1922-2011 1927-2010 b. 1936
580 Section E: Molecular Biology

BOX 41.1 : Salient features of genetic code Nonpunctuated


1. Triplet base sequence There is no punctuation between the codons. It is con-
2. Nonoverlapping
3. Nonpunctuated
secutive or continuous.
4. Degenerate
5. Unambiguous Degenerate
6. Universal
Table 41 .1 shows that 61 codons stand for the 20 amino
acids . So, one amino acid has more than one codon.
Bacterial Ribosomes are Different For example, Serine has 6 codons; while glycine has
Bacterial ribosomes are smaller than mammalian ones. 4 codons. This is called degeneracy of the code. In
Bacteria has 70S ribosomes; with 30S and 50S sub- general , if the amino acid has more than one codon, the
units. So, many antibiotics will inhibit bacterial protein first two bases in the codon will be the same, only the third
synthesis, but will do no harm to human cells. See end one is different. This reduces the effect of mutations.
of this Chapter.
Unambiguous
Genetic Code
Though the codons are degenerate, they are unam-
A triplet sequence of nucleotides on the mRNA is biguous; or without any doubtful meaning. That is, one
the codon for each amino acid. Since, there are four codon stands only for one amino acid.
different bases, they can generate 64 (43 ) different
codons or code words by permutations and combina- Universal
tions. For example, the codon for phenylalanine is UUU.
The codons are the same for the same amino acid in all
Nirenberg was awarded the Nobel Prize in 1968 for deci-
species; the same for "Elephant and E. coif. The genetic
phering the genetic code. There are 31 tRNA species,
codon has been highly preserved during evolution.
carrying 20 amino acids, which translate 61 codons.
Important features of genetic code are shown in Box 41 .1. Wobbling Phenomenon
Salient Features of the Genetic Code The reduced stringency between the third base of the
codon and the complementary nucleotide in the
Triplet Codons
anticodon is called wobbling. The pairing of codon and
The codons are on the mRNA. Each codon is a con- anticodon can wobble at the third letter. For example,
secutive sequence of three bases on the mRNA, e.g. GGU, GGC and GGAare the codons for glycine; all three
UUU codes for phenylalanine (Table 41.1 ). will pair with the anticodon CCI ( I = lnosinic acid) of gly-
cine-tRNA. The degeneracy of genetic code and wobbling
Nonoverlapping phenomenon together will reduce the possibility of
The codons are consecutive. Therefore, the starting mutations.
point is extremely important. The codons are read one
after another in a continuous manner, e.g. AUG, CAU, Terminator Codons
GAU, GCA, etc. There are three codons which do not code for any parti-
cular amino acid. They are "nonsense codons", more cor-
rectly termed as punctuator codons or terminator codons.
They mark end of protein synthesis. These three codons
are UAA, UAG, and UGA.

Initiator Codon
Thomas A Steitz Ada E Yonath Venkatraman
NP2009 NP 2009 Ramakrishnan In most of the cases, AUG acts as the initiator codon.
b. 1940 b. 1939 NP2009 AUG also acts as the codon for methionine. In a few
b. 1952 proteins, GUG may be the initiator codon.
Chapter 41 : Genetic Code and Translation 581

TABLE 41 .1: Triplet codons and corresponding amino acids


Firt
nucle >tide
S'e ,d u C A G
u uuu uuc UUA UUG cuu CUC CUA CUG AUU AUC AUA AUG GUU GUC GUA GUG
Phe Phe Leu Leu Leu Leu Leu Leu lie lie lie Met Val Val Val Val
C ucu ucc UCA UCG CCU CCC CCA CCG ACU ACC ACA ACG GCA GCC GCA GCG
Ser Ser Ser Ser Pro Pro Pro Pro Thr Thr Thr Thr Ala Ala Ala Ala
Sec nd A UAU UAC UAA UAG CAU CAC CAA CAG AAU AAC AAA AAG GAU GAC GAA GAG
nude :itide Try Try Stop Stop His His Gin Gin Asn Asn Lys Lys Asp Asp Glu Glu
G UGU UGC UGA UGG CGU CGC CGA CGG AGU AGC AGA AGG GGU GGC GGA GGG
Cys Cys Stop Trp Arg Arg Arg Arg Ser Ser Arg Arg Gly Gly Gly Gly
Th rd u C A G u C A G u C A G u C A G
nudeptide
3' e hd
(UAA,lIAG and UGA and Stop or nonsense codons; AUG acts as the init iator codon).

Mite chondria have Different Codons ATP. Then the carboxyl group of the amino acid is esteri-
fied with 3' hydroxyl group of tRNA.
The Iirotein synthesizing machinery of mitochondria is
Amino acyl tRNA
differ mt from that in the cytoplasm. There are only about Amino acid synthetase ) Aminoacyl tRNA
22 tF NAs in mitochondria; but there are 31 tRNA spe- +tRNA+ ATP +AMP
cies in cytoplasm. Therefore, some of the mitochondrial
codo ~s are different from nuclear codons. In this reaction, ATP is hydrolyzed to AMP level, and
so two high energy phosphate bonds are consumed.
I TI 'ANSLATION PROCESS
Initiation of Protein Synthesis
Tran• lation is a cytoplasmic process. The mRNA is
Initiation can be studied as 4 steps: 1. Recognition steps.
trans ated from 5' to 3' end. In the polypeptide chain
2. Formation of preinitiation complex. 3. Binding of
synt~ esized, the first amino acid is the amino terminal
mRNA to the preinitiation complex. 4. Formation of
one I see Fig. 41 .1). The chain growth is from amino ter-
ribosomal complex.
mina to carboxyl terminal. The process of translation
can I e conveniently divided into the phases of:
Recognition Steps
A. !\ctivation of amino acid
B. nitiation In eukaryotes, the first amino acid incorporated is methio-
C. =longation nine (AUG codon). But in prokaryotes, the same codon
D. rermination stands for N-formyl methionine, which is the first amino
E. ~Ost-translational processing. acid.
The first AUG triplet after the marker sequence is
Act vation of Amino Acid identified by the ribosome as the start codon . In mam-
(Charging Reaction) malian cells, the marker is the "Kozak" sequence; in
bacteria, it is the "Shine-Dalgarno" sequence. For the
The enzymes aminoacyl tRNA synthetases activate
process, initiation factors (elF) are required. Poly-A
the c mino acids. The enzyme is highly selective in the
tail of the mRNA is also important for the recruitment of
reco 1nition of both the amino acid and the transfer RNA
40S ribosomal subunit to the mRNA.
acceptor. There is at least one tRNA for each of the
20 amino acids. The D arm of tRNA is very important
Formation of Preinitiation Complex
for ti e recognition by the enzyme. The CCA 3' terminus
of t~e acceptor arm carries amino acid (see Figs. 41 .2 GTP, IF-2, met-tRNA (tRNA carrying methionine)
and 41.3). Amino acid is first activated with the help of and 40S ribosomal subunit are complexed to form
582 Section E: Molecular Biology

Met-tRNA
+GTP + IF-5 + IF-2
+ elF-2 + factor 4

(\
ATP ADP+Pi
{+)605 ' \
GDP+Pi
UAC AC
+ elF-3
AUG ACC
+ elF-1 /\AAA/\ AAAAAAAAA AA

43S Pre-initiation BOS Ribosome


Fig. 41 .4: Initiation steps; UAC= anticodon on met-tRNA; AUG= start signal; P= peptidyl site; A= amino acyl site

5' AA/\/\/\AAA/\/\/\/\/\/\A/\/\/\/\/\/\/\/\/\1\/\ 3' 5 /\/\AAAAAAl\1'AAM/\/\AAI\AA/\AA/\"3' 5 /\/\AA/\/\I\/\AAA/\M/\/\/\/\/\/\AA/\A/\/\3 5'


1 1 1
/\/\/\MAAAA/\/\AAA/\I\A/\1\1\/\/\/\/\/\A 3'

Attachment of new Peptide bond Old tRNA Translocation Attachment of new


amino acid lo A site formed detached to next codon amino acid to A site
Fig. 41 .5: Elongation phase. P= peptidyl site; A= amino acyl site

pre-initiation complex (Fig. 41.4 ). Met-tRNA has the The anti-codon of met-tRNA is correctly base pairing
anticodon UAC. with the AUG codon on mRNA (Figs. 41.4 ahd 41 .5).

Binding of mRNA Elongation Process of Translation


Elongation has 3 steps: 1. Binding of amino acyl tRNA to
This pre-initiation complex binds with mRNA; the 5'
the A site. 2. Peptide bond formation and 3. Transloca-
methylated cap of mRNA facilitates this binding. It also
tion of the ribosome on the mRNA.
requires energy from hydrolysis of ATP. This forms ini-
tiation complex (Fig. 41.4 ). This is the rate limiting step Binding of New Amino Acy/ tRNA
in the whole of translation process.
A new aminoacyl tRNA comes to the "A" site. The next
codon in mRNA determines the incoming amino acid.
Formation of BOS Ribosomal Assembly The GTP is hydrolyzed to GDP, and the tRNA binds to
The initiation complex now binds with 60S ribosomal unit the "A" site (Fig. 41 .5).
to form the full assembly of 80S ribosome. This needs
hydrolysis of GTP (Fig. 41.4). Peptide Bond Formation
P and A Sites of Ribosomal Assembly: The whole The alpha amino group at the incoming amino acid in
ribosome contains two receptor sites for tRNA mole- the "A" site forms a peptide bond (CO-NH) with carboxyl
cules. The "P" site or peptidyl site carries the peptidyl- group of the peptidyl tRNA occupying the "P" site. This
tRNA. It carries the growing peptide chain. The "A" site reaction is catalyzed by the enzyme peptidyl trans-
or aminoacyl site carries the new incoming tRNA with ferase . This is an example of ribozyme; RNA acts as
the amino acid to be added next. When the 80S ribo- the enzyme. Since, the amino acid brought in by the
some is assembled, the tRNA-Met is now at the P site. tRNA is already activated, there is no need for further
Chapter 4 1: Genetic Code and Translation 583

-----• l~:.: :
.... •····· •• free protein
Protein synthesis
Polypeptide half nearing to completion
length synthesized NH2

I
Synthesis started NH 2
NH 2
I I
RF+GTP
AMAMMMA free mRNA

Free40S 3' 1\/\.l\l+.I\ \ /\/\/\1\/\/\I\AI\M/\A/\1 A /\A /\l\1\/\/\./\./\/\/\/\/\I\

mRNA

Fig. 41 .6: Termination of translation. (RF: Release factor) Fig. 41 .7 : Polyribosomes

energy supply for the purpose of peptide bond forma- peptide chain from the tRNA at the P site. This needs
tion. Now, the growing peptide chain is occupying the hydrolysis of GTP to GDP. The completed peptide chain
"A" site (Fig. 41.5). is now released.Finally, SOS ribosome dissociates into
its component units of 60S and 40S (Fig. 41 .6). The
Translocation Process differences between mammalian and bacterial transla-
At this time, the tRNA fixed at the "P" site does not carry tion are given in Table 41.3.
any amino acid and is therefore released from the ribo-
some. Then the whole ribosome moves over the mRNA Polyribosomes
through the distance of one codon (3 bases). The pep- One eukaryotic ribosome can synthesize 5 to 6 peptide
tidyl tRNA is translocated to the "P" site. Now, the "A" bonds per second . Many ribosomes can work on the
site is ready to receive another aminoacyl tRNA bear- same mRNA molecule simultaneously and these aggre-
ing the appropriate anticodon. The new aminoacyl tRNA gates are called polyribosomes or polysomes (Fig. 41 .7).
is fixed to the "A" site, by base pairing with the mRNA Polyribosomes may be attached on the walls of the
codon. Translocation requires hydrolysis of GTP to GDP endoplasmic reticulum to form the rough ER (see
(Fig. 41 .5). The elongation reactions are repeated till Chapter 2). The proteins are then transported through
the polypeptide chain synthesis is completed. cisternal space to Golgi apparatus, where they are tem-
porarily stored. Cytoplasmic proteins are synthesized by
Energy Requirements ribosomes that exist free in cytoplasm.
For each peptide bond formation, 4 high energy phos-
phate bonds are used; two for the initial activation and Protein Targeting
one for binding of amino acyl tRNA to A site (GTP to Proteins for External Secretion
GDP) and one for translocation step (GTP to GDP).
The process is also called as "protein sorting" or "protein
Actual peptide bond formation (peptidyl transferase
localization". The secreted proteins, plasma membrane
step) does not require any energy, because the amino integral proteins, lysosomal enzymes and membrane
acids are already activated. Further, 1 ATP is used for proteins of ER are synthesized on rough endoplasmic
initiation complex formation ; 1 GTP for SOS ribosome reticulum by membrane bound polyribosomes. The
formation and 1 GTP for termination. newly synthesized protein is then delivered to the des-
tined compartment. Blobel and Sabatini proposed the
Termination Process of Translation
signal hypothesis to explain the different destination
After successive addition of amino acids, ribosome rea- of proteins. Guenter Blobel was awardec Nobel Prize in
ches the terminator codon sequence (UM, UAG or UGA) 1999.
on the mRNA. Since, there is no tRNA bearing the The nascent protein is passed through the mem-
corresponding anticodon sequence, the "A" site remains brane into the channels of ER. Then protein synthesis
free. The release factor (RF) enters this site. The RF binds is completed, and the protein molecule is now inside
with RF3 and GTP. This complex along hydrolyzes the the endoplasmic membrane. As the nascent protein is
584 Section E: Molecular Biology

TABLE 41 .2: Post-translaltonal mod1ficat1ons TABLE 41 .3: Comparison of translation In eukaryotes and
prokaryotes. M1tochondna are similar to prokaryotes
Reversible Irreversible
Eukaryotes
Disulfide bridge Proteolysis
(mammalian Prokaryotes
Glycosylation Ubiquitination Feature cells) (bacteria) Mitochondria
Phosphorylation Lysine hydroxylation DNA Open Circular Circular
Acylation Proline hydroxylation Ribosomes 80S 70S 70S
N-acetylation Methyl at ion tRNA (No.) 31 22 22
Initiating Methionine Formyl Formyl
amino acid methionine methionine
BOX 41 .2: Protein targeltng Inhibited
Effect of tetracycline Not affected Inhibited
Zellweger syndrome is due to defective oxidation of very long chain
fatty acids (VLCFA). Here the correct "address• is not printed on
the protein packet; so that it could not be delivered to the correct
location. Peroxisomal enzymes are produced; but their entry into
peroxisome is denied. This leads to insufficient oxidation of VLCFA. NH 2 V---
------
-----
- ----•~-
Pre-proinsulin
====~ - - - • • • COOH
Accumulation of VLCFA in CNS causes neurological impairment and
death in childhood.
Another example is primary hyperoxaluria, which causes kidney
stones at an early age. The defect is due to protein targeting defect Proinsulin
and the enzyme alanine glyoxylate aminotransferase (see Fig. 18.7)
is seen in mitochondria, inst ead of its normal peroxisomal location.
Inclusion cell disease is due to non-entry of normal enzymes into
lysosomes. Mannose-6-phosphate is the marker to target enzymes
to lysosomes; this is absent.
Mature insulin

traversing the inner membrane of ER, carbohydrate


moieties are added at particular regions by specific Fig. 41 .8: Post-translational processing of insulin by proteolytic
enzymes; this is called co-translational glycosylation. cleavage

Correct Address of Destination is Labeled iv. Co-translational glycosylation: Carbohydrates are


The proteins carry an "address" that is specific for its attached to serine or threonine residues through
correct destination inside the cell. This is present in the O-glycosidic linkages and to asparagine or gluta-
mine residues through N-glycosidic linkages
carboxy terminal end of proteins. Diseases due to
(Table 41 .2).
defective protein targeting are shown in Box 41 .2.
v. Acetylation and methylation of lysine residues of
Post-translational Processing histones modulate transcriptional activity of chro-
matin. Acetylation increases transcription (genes
These processes are categorized in Table 41 .2. are switched on) and deacetylation represses trans-
cription (genes are switched off).
Proteolytic Cleavage
Subunit Aggregation
Conversion of proinsulin to insulin (Fig. 41 .8).
Examples are immunoglobulin, hemoglobin and matura-
Modifications of Amino Acids tion of collagen. Failure of post-translational modification
i. Gamma carboxylation of glutamic acid residues affects the normal function of many proteins. For exam-
of prothrombin, under the influence of vitamin K ple, poor cross linking of collagen in scurvy, since ascor-
(see Chapter 32). bic acid is required for the hydroxylation of proline and
ii. Hydroxylation of proline and lysine in collagen lysine (see Chapter 33, under Vitamin C).
with the help of vitamin C (see Chapter 49).
iii. Phosphorylation of hydroxyl groups of serine, Inhibitors of Protein Synthesis
threonine or tyrosine by kinases, e.g. glycogen The modern medical practice is heavily dependent
phosphorylase on the use of antibiotics. They generally act only on
Chapter 41: Genetic Code and Translation 585

TABLE 41 .4: Differences between prokaryotic and eukaryobc translation


Feature Prokaryotes Eukaryotes
Ribosomes 305 small subunit and SOS large subunit associate t o form 405 small subunit and 605 large subunit together form
705 ribosome the 80S ribosome
Initiation Translation starts before transcription is completed. No Transcription and translation are well separated and the
m RNA p rocessing processed mRNA is t ranslated.
t Consensus Initiating codon AUG is recognized by the Shine Dalagarno Initiating codon AUG is recognized by Kozak seq uence
sequences sequence
Initiator tRNA The initiating tRNA carries formylated methio nine The initiating tRNA carries methionine
Formation of Initiation factors and GTP hydrolysis are req uired for the Several IF sand GTP hydro lysis are required for the
initiation complex bindingof the small ribosomal subunit t o the m RNA formation of the initiatio n complex.
Peptide bond The peptidyl t ransferase ribozyme is In the 235 rRNA Peptidyl transferase activit y in the 285 rRNA of the large
formation subunit .
Posttranslational Mainly co-translational modifications Mainly Post-translational modification, sorting, export
events and localization.
Inhibition Antibiotics can inhibit d ifferent stages of translation, which Toxins like diphtheria , ricin, puromycin and
may be reversible or non reversible cyclo heximide

bacteria and are nontoxic to human beings. This is Mitochondrial DNA and RNA
because mammalian cells have 80S ribosomes, while
1. There is a dichotomy on the mitochondrial metabo-
bacteria have 70S ribosomes.
lism. Some of the mitochondrial protein synthe-
Reversible Inhibitors in Bacteria sis is under the control of mitochondrial DNA; but
important proteins of the outer membrane of the
These antibiotics are bacteriostatic. Tetracyclines
mitochondria are synthesized under the influence of
bind to the 30S subunit of bacterial ribosome and so
inhibit attachment of aminoacyl tRNA to the A site of nuclear DNA. Table 41.4 shows that mitochondria
ribosomes. Chloramphenicol inhibits the peptidyl trans- are similar to bacteria more than mammalian cells.
ferase activity of bacterial ribosomes. Erythromycin This fact supports the theory that mitochondria are
(macrolides) and clindamycin prevent the translocation derived from prokaryotes symbiotically adapted to
process. multicellular organisms.
2. Maternal inheritance: Since, the mitochondria are
Irreversible Inhibitors in Bacteria inherited cytoplasmically, the mtDNA is inherited
These antibiotics are bactericidal. Streptomycin and from the mother. Mother transmits mtDNA through
all other aminoglycoside antibiotics bind to 30S subunit oocyte.
of bacterial ribosomes. They cause misreading of mRNA 3. There are hundreds of copies of mtDNA in each
and at high concentrations, they completely inhibit the cell (nuclear DNA has only 2 copies). During cell
initiation complex formation and totally inhibit protein division, mtDNA replicates and they segregate to
synthesis. the daughter cells. If a mutation occurs in mtDNA,
the daughter cells may inherit the mutant or normal
Inhibitors of Protein Synthesis in Mammals mtDNA. Heteroplasmy is defined as the presence
They are not suitable for clinical use; but they are used of normal and mutant mtDNA in different propor-
as research tools. Puromycin is structurally similar to tions in different cells.
tyrosine-tRNA and gets attached to the "A" site of the 4. Defects in mitochondrial genome will lead to mito-
ribosome. So, the incomplete peptide is released. It acts chondrial myopathies. Leber's hereditary optic
both in bacterial and mammalian cells. Cycloheximide neuropathy is caused by a single base mutation
inhibits peptidyl transferase in 60S subunit. It acts only which alters one arginine to histidine in the NADH
on eukaryotic cells. Inhibitors of transcription (Chapter 40) Coenzyme Q reductase. OXPHOS (oxidative phos-
will also in turn inhibit translation process. phorylation) diseases are shown in Table 41 .5.
586 Section E: Molecular Biology

TABLE 41 .5: OXPHOS diseases iv. Proteomics: It directly addresses the protein com-
Syndrome Features
plement of the genome. The study of all proteins by
a cell type or an organism is called 'proteomics'.
Leber's Hereditary Complex I defect; blindness, cardiac
Neuropathy (LHON)
Myoclonic epilepsy ragged
conduction defects
Myoclonic epilepsy, myopathy,
I LEARNING POINTS, CHAPTER 41
red fiber disease (MERRF) dementia
1. Transfer RNA (tRNA) or soluble RNA (sRNA) is the
Leigh's syndrome Complex I defect; movement adapter molecule between transcription and trans-
disorders
lation. Each amino acid has a specific tRNA.
2. The triplet sequence on the anticodon arm of the
Genomics and Proteomics tRNA is complementary to the codon triplet on the
mRNA.
i. Genome means all the DNA contained in an orga- 3. Six important characteristics of the genetic code
nism or a cell, which includes both the chromosomes are that it is triplet, universal, degenerate, non-over-
within the nucleus and the DNA in mitochondria. lapping, nonpunctuated and exhibit wobbling.
Thus the genome of an organism is the totality of 4. Three terminator codons are UAA, UAG and UGA.
5. Four high energy phosphate bonds are required
genes making up its hereditary constitution.
for the formation of one peptide bond, two for initial
ii. Genomics is the study of the genome and its activation, one for EF-1 step and one for EF-2 step.
actions. 6. Post-translational processing of proteins includes
iii. Proteome is the sum of all proteins expressed removal of the signal sequences, gamma carboxy-
by the genome of an organism, thus involving the lations, methylations, acylation, subunit aggrega-
identification of the proteins in the body and deter- tion and phosphorylations.
mination of their role in physiological and pathologi- 7. Clinically useful protein synthesis inhibitors are
streptomycin, chloramphenicol, tetracyclines, erythro-
cal functions. While the genome remains largely
mycin.
unchanged, the proteins of a particular cell change 8. Chaperones are proteins that govern protein fold-
dramatically as genes are turned on and off in ing. Improper protein folding may lead to Prion
response to the environment. diseases.

PART-1: ESSAY AND SHORT NOTE QUESTIONS

41-1 . What is a codon? Describe the salient features of genetic code.


41-2. Describe the phases of activation, initiation, elongation and termination of biosynthesis of protein.
41-3. Describe the steps of protein synthesis.

SHORT NOTE QUESTIONS

41-4. Function of mRNA. 41-9. Genetic code.


41-5. Ribosomes. 41-10. Degeneracy of codons.
41-6. Initiation of translation. 41 -11 . Anti codon.
41 -7. Translocatlon. 41 -12. Post-translational modifications.
41-8. Structure and function of tRNA. 41-13. Inhibitors of protein biosynthesis.

PART-2: MULTIPLE CHOICE QUESTIONS

41-1 . Degeneracy of genetic code denotes: A. Occasional error in translation


A. Single codon stands for multiple amino acids B. Low fidelity in mRNA synthesis
B. One amino acid is represented by multiple codons C. The ability of anticodons to pair with codons that
C. Overlapping of codons is observed at times differ at the third base
D. Some codons contain 4 bases instead of 3 bases D. Post-transcriptional modification of excision of
41-2. The term wobbling means: introns
Chapter 41: Genetic Code and Translation 587

41-3. Which is not true regarding the genetic code: A. Amino terminal modification
A. Degenerate 8 . Ambiguous 8 . Protein folding
C. Nonoverlapping D. Universal C. Phosphorylation of serine residues
41-4. During protein biosynthesis, high energy bonds D. Glycosylation of asparagine
are utilized in all the following steps, except 41-15. In eukaryotes, the initiation of translation requires
A. Formation of amino acyl tRNA all the following, except:
8. Binding of amino acyl tRNA to A site of mRNA A. Methionyl tRNA
ribosome complex 8 . 70 S ribosome
C. Formation of peptide bond (peptidyl transferase C. Kozaksequence
step) D. mRNA codon AUG
D. Translocation 41-16. Elongation of polypeptide chain requires all the
41 -5. Post-translational modifications include all the fol- following proteins, except:
lowing, except A. Peptidyl transferase 8 . Rho factor
A. Glycosylation 8 . Hydroxylation C. Translocase D. GTPase
C. Decarboxylation D. Phosphorylation 41-17. Which complex is involved in translation?
41 -6. All are diseases resulting from failure of post- A. Polysome 8. Nucleosome
translational modifications, except C. Spliceosome D. Lysosome
A. Lathyrism 41-18. Signal peptide (SP) region is necessary for all the
8. Ehler-Danlos syndrome following, except
C. Osteogenesis imperfecta A. Attachment of ribosomes to mRNA
D. Scurvy 8 . Anchorage of ribosomes to endoplasmic reticulum
41-7. Failure of post-translational modification occurs in C. Release of newly synthesized proteins from
deficiency of which vitamin? ribosomes
A. Ascorbic acid 8. Retinol D. Digestion of signal recognition particle (SRP)
C. Vitamin 812 D. Vitamin D 41-19. Which is not a post-translational modification?
41-8. The mechanism of action of tetracycline is: A. Gamma carboxylation of prothrombin
A. Inhibits tRNA binding to ribosome 8. Hydroxylation of proline in collagen
8. Decreases binding of ribosomes to mRNA C. Methylation of histones
C. Causes misreading of codes D. Hydroxylation of benzene ring of phenyl alanine
D. Inhibits translocation 41-20. Which is an inhibitor of translation in eukaryotic
41-9. All are true regarding tRNA, except: cells?
A. Amino acid is attached to 3' end. A. Erythromycin 8 . Puromycin
8. Anticodon sequence is at 5' end. C. Chloramphenicol D. Tetracycline
C. Contains several modified bases. 41-21 . Mechanism of action of streptomycin is that it:
D. Has a clover leaf pattern due to hydrogen bonding. A. Inhibits tRNA binding to ribosome
41-10. Unusual nucleotide bases are found in significant 8 . Decreases binding of ribosomes to mRNA
quantities in: C. Causes misreading of codes
A. mRNA B. tRNA D. Inhibits translocation
C. rRNA D. snRNA 41-22. All the following statements are true with regard to
41-11 . All are inhibitors of RNA synthesis, except: mitochondrial DNA, except
A. Rifampicin B. Puromycin A. All mitochondrial proteins are encoded by mito-
C. Amanitin D. Actinomycin D chondrial DNA
41-12. All are structural features of tRNA, except: 8 . Mitochondrial DNA is inherited by cytoplasmic
A. Has clover leaf appearance inheritance.
8 . Has amino acid binding site at 5'end C. Inheritance is predominantly from mother
C. Contains unusual bases D. DNA in mitochondria is circular
D. Has a pseudouridine arm 41 -23. The disease resulting from mutation in mitochon-
41-13. All the following are inhibitors of translation in drial DNA is:
prokaryotic cells, except: A. Zellweger syndrome
A. Clindamycin 8. Streptomycin 8 . Jakob-Creutzfeldt disease
C. Mitomycin D. Erythromycin C. Xeroderma pigmentosum
46-14. Which of the following post-translational events is D. Leber's hereditary optic neuropathy
defective in mad cow disease?
588 Section E: Molecular Biology

41-24. Which is due to defective protein targeting?


A. Xeroderma pigmentosum C. Primary oxaluria
B. Thalassemia D. Ataxia telangiectasia

ANSWERS OF MULTIPLE CHOICE QUESTIONS


41-1. B 41-2. C 41-3. B 41-4. C 41-5. C 41-6. C 41-7. A
41-8. A 41-9. B 41-10. B 41-11 . B 41-12. B 41-13. C 41-14. B
41-15. B 41 -16. B 41-17. A 41-18. D 41-19. D 41-20. B 41-21. C
41-22. A 41 -23. D 41-24. B

PART-3: VIVA VOCE QUESTIONS AND ANSWERS

41-1 . What are structural features of tRNA molecule? 41 -8. What is meant by the term wobbling?
It has clover shape appearance. Amino acid binding is Anticodons pair with codons that differ at the third
at 3' end. The opposite part has anticodon arm. base.
41-2. What is a codon? 41-9. Where is protein biosynthesis is taking place?
Codon is a triplet of bases, present in mRNA. Collec- Ribosomal assembly either attached to endoplasmic
lion of these codons make up the dictionary of genetic reticulum, or in cytoplasm.
code. 41-10. Give examples for post-translational modifications.
41-3. During replication, DNA is synthesised in wh ich Gamma carboxylation of prothrombin; Hydroxylation
direction? of praline in collagen; Methylation of histones; Glyco-
From 5' to 3' direction. sylation of proteins.
41-4. During transcription, mRNA is synthesized in 41-11 . Give examples of inhibitors of translation in
which direction? eukaryotic cells?
From 5' to 3' direction. Puromycin; Cycloheximide; Diphtheria toxin; Ricin.
41-5. During translation, protein is synthesized in which 41 -12. Give examples of reversible inhibitors of protein
direction? synthesis in bacteria.
From amino terminal end to carboxy terminal end. These antibiotics are bacteriostatic. Tetracyclines;
41-6. What are the salient features of genetic code? Chloramphenicol; Erythromycin and clindamycin are
i. They are consecutive three bases pairs in mRNA. examples.
ii. Codons are nonoverlapping; iii. degenerate; iv. but 41-13. Give an example of irreversible inhibitor of protein
unambiguous and v) universal. synthesis in bacteria.
41-7. What is meant by degeneracy of genetic code? These antibiotics are bactericidal. Streptomycin is an
One amino acid is represented by multiple codons. example.
-----------Chapter 42
Control of Gene Expression

Chapter at a Glance
The learner will be able to answer questions on the following topics:
0 Principles of heredity 0 0peron concept
0 Dominant and recessive inheritance 0 Repression and derepression
0 Mutations 0 Transcription regulation in eukaryotes

..
It is estimated that more than 6% of all infants born alive suffer from genetic diseases and 1% from chromosomal aberrs -
tions. The former conditions involve minor alterations in DNA make up, e.g. phenylketonuria. The latter onies are due to major
changes in chromosomes; e.g. Down's syndrome, Turner's syndrome. Gregor Johann Mendel (1822-H4}, who was Abbot
I
of Bron, described the principles of heredity in 1866. As it was printed in an obscure journal, it remained unnoticed for many
years. In 1900, Hugo de Vries and C Correns, rediscovered and confirmed Mendel's theory. Walther Flemming demonstrated
chromosomes in 1882. In 1902, Walter Sutton showed that chr mosomes are in pairs and are the carriers of Mendel's Unit of
heredity. The word 'Gene' was coined by Wilhelm Johannsen in 1909.

( BASIC PRINCIPLES OF HEREDITY Dominant Inheritance


1. Heredity is transmitted from parent to offspring as It is characterized by the phenotypic expression of the
individual characters. disease, even if only one allele is abnormal or in hetero-
2. The genes are linearly distributed on chromosomes zygous state. In the example, shown in Figure 42.1, the
at fixed positions (loci). father has the defective gene, marked as D. The pos-
3. Genes that may replace one another at the same sible permutations of gametes are shown in the Figure
locus are called allelomorphic genes or alleles. Alleles 42.1. When an affected heterozygot,e parent (Dd) has
are genes responsible for alternate or contrasting a normal spouse (dd), half of the progeny will have the
characters. Usually, one allele is inherited from fa- disease. Examples of diseases with autosomal dominant
ther and the other from mother. inheritance are achondroplasia (dwarfism) and some
4. When both alleles carry the same defect, it is said types of porphyrias.
to be homozygous.
5. When one allele is normal, and the counterpart is Autosomal Recessive Inheritance
defective; it is called heterozygous. If the disease is manifested only in homozygous state
6. The observed character expressed by the gene is (not expressed in heterozygous condition), it is known
called phenotype. as recessive transmission. For example, in a person
7. The genotype represents the set pattern of genes suffering from sickle cell anemia, both the alleles for
present in the cell. beta-globin gene have mutated. Hen ce, all beta globin
590 Section E: Molecular Biology

Affected Both parents Gametes


heterozygote Normal Gametes Carrier heterozygous
0 0 9

mother

-
father
® d
9 d N n


D Dd Dd N NN Nn

Dd dd
Nn ~ Nn
cf
INN Nn Nn nn l

I
n
d dd dd
® 0 0 Nn nn

•I
lod dd Dd ddi
®
1111
affected normal
I
affected
®
II]
normal
D = abnormal allele
(dominant)
d= normal allele
[I
normal carrier
Result: offsprings
25% are affected
[]
carrier affected
n = abnormal allele
(recessive)
N = normal allele
Result: 50% offsprings 50% are carriers
are affected 25% normal
Fig. 42.1: Autosomal dominant inheritance Fig. 42.2: Autosomal recessive inheritance

Gregor Johann Hugo Walther Walter Wilhelm Godfrey Wilhelm


Mendel de Vries Flemming Sutton Johannsen Hardy Weinberg
1822-1984 1848-1935 1843-1905 1877-1916 1857-1927 1877- 1947 1862- 1937

chains will be abnormal. He is homozygous for sickle X chromosome carries the abnormal gene (Fig. 42.3).
cell disease. In certain cases, the carrier state may be When a normal male marries a carrier female (unaffected
identified biochemically, then it is referred to as the trait parent), the children can be affected male (25%), female
of the disease. For example, in sickle cell trait, one beta- carrier (25%), normal male (25%) and normal female
globin gene (allele) is normal; while the other one is (25%). All male children of an affected male and normal
abnormal (carrier state). Such an individual is hetero- female will be normal; but all female children will be car-
zygous for that character. Therefore, normal gene pro- riers since they inherit the abnormal X from their father.
duces normal Hb and abnormal gene produces HbS. There is no male-to-male transmission, but male-to-
Thus inside the RBC, 50% of hemoglobin molecules are female and female-to-male transmission of the affected
X can occur. X-linked traits are expressed in males
abnormal. This can be identified by electrophoresis.
who are hemizygous (XY) for the condition, but not in
When both father and mother are carriers, one-
females who may be heterozygous (XX).Hemophilia, glu-
quarter of siblings express the disease (both alleles
cose-6-phosphate dehydrogenase deficiency, pseudo-
abnormal), another one-quarter of siblings are normal, and
hypertrophic muscular dystrophy (Duchenne type), and
half of the children are carriers (Fig. 42.2). This chance
red-green color blindness are examples of sex-linked
factor is acting on each progeny. If only one parent is
recessive inheritance.
carrier and the other is normal; then there will be no
affected child, but 50% children are carriers. Most of the Marriages with Close-Cousins
inborn errors of metabolism are recessively transmitted. are Inadvisable
A few examples are phenylketonuria, albinism, galacto-
The probability of two carriers getting married is increa-
semia and sickle cell anemia.
sed in consanguineous marriages (Latin, con = with;
sanguis = blood). So, there is increased frequency of
Sex-linked (X-linked)
genetic diseases in their children. For example, phenyl-
Recessive Inheritance
ketonuria has an incidence of 1 in 25,000 in general
In the autosomal conditions, the disease occurs in both population; but it is 13/ 25,000 in children of first cousin
sexes with equal frequency. But in sex-linked conditions, marriages.
Chapter 42: Control of Gene Expression 591

Affected male gametes Normal male gametes Affected male gametes

X V Carrier
female
gametes
X V Carrier
female
gametes
X V

Normal
X xx xv
Carrier Normal X xx xv
Carrier Affected X xx xv
Affected Affected

xx xv xx xv xx xv
daughter son daughter son daughter son
female
gametes

X Carrier Normal X Normal Normal X Carrier Normal


daughter son daughter son daughter son

Fig. 42.3: Sex-linked recessive inhe rita nce

TABLE 42.1 : Genes in human chromosomes C. Mitochondrial abnormalities.


Chromosome No. Gene D. Multifactorial disorders.
1 Alkaline phosphatase (liver, bone, kidney)
2 Alkaline phosphatase (placental) Treatment Policies of Genetic Diseases
3 Transferrin 1. Replace the end-product of the missing enzyme,
4 Fibrinogen e.g. administer thyroxine in familial goiter.
5 HMG-CoA reductase
2. Limit the substrate of the missing enzyme. In phenyl-
6 MHC (Major histocompatibility) locus
ketonuria, reduce phenylalanine in diet. In galacto-
7 Urea cycle enzymes; cyst ic fibrosis (disease)
semia remove lactose from diet.
8 Carbonic anhydrase
9 Interferon
3. Replace missing protein, e.g. administer AHG (anti-
11 Hemoglobin 13, y and o chains hemophilic globulin) in hemophilia.
12 Glyceraldehyde-3-P-dehydrogenase 4. Activity of abnormal enzyme is enhanced, e.g. large
13 Breast Cancer (BRCA2) quantities of vitamin B12 is useful in methyl malonic
14 lmmunoglobulin heavy chains aciduria.
15 Cytochrome P-450 5. Induction of enzyme; in Crigler-Najjar syndrome,
16 Hem oglobin a-chain glucuronyl transferase enzyme can be induced by
17 Breast cancer (BRCA 1), growth hormone phenobarbitone.
18 Prealbumin
6. Gene therapy is still in experimental stages, but
19 13 chain of hCG
may become common in the next generation.
20 Adenosine deaminase
21
22
Superoxide d ismutase
lmmunoglobulin, lambda chain
I MUTATIONS
X Glucose-6-P-dehydrogenase; Antihemophilic A mutation is defined as a change in nucleotide seq-
globulin; HGPRTase; Duchenne type muscular uence of DNA. This may be either gross, so that large
dystrophy
areas of chromosome are changed, or may be subtle
y SRY (sex determining gene)
with a change in one or a few nucleotides. Mutation may
also be defined as an abrupt spontaneous origin of new
Genetic Disorders character. Statistically, out of every 106 cell divisions,
The locations of many genes have been identified on one mutation takes place.
specific genes. A small selected list is given in Table 42.1.
Genetic disorders are of different types: ( CLASSIFICATION OF MUTATIONS
A. Chromosomal disorders: These are identified by A point mutation is defined as a change in a single
Karyotyping, e.g. Trisomy 21 . nucleotide. This change may have missense or non-
B. Single gene defect, sometimes identified by bio- sense effects. Deletion or insertion of a single nucleotide
chemical methods, e.g. Phenylketonuria. leads to a frameshift effect.
592 Section E: Molecular Biology

Substitution becomes alanine; this is called Hb Sydney. This variant


is functionally normal. Hence, this type of mutation is
Replacement of a purine by another purine (A to G or referred to as conserved substitution.
G to A) or pyrimidine by pyrimidine (T to C or C to T} is
called transition mutation. If a purine is changed to a Missense; Partially Acceptable Mutation
pyrimidine (e.g. A to C) or a pyrimidine to a purine (e.g.
T to G), it is called a transversion. The point mutation HbS or sickle-cell hemoglobin is produced by a mutation
present in DNA is transcribed and translated, so that the of the beta-chain in which the 6th position is changed
defective gene produces an abnormal protein. to valine, instead of the normal glutamate. Here, the
normal codon GAG is changed to GUG (transversion).
Deletion HbS has abnormal electrophoretic mobility and subnor-
Deletions may be subclassified into mal function, leading to sickle-cell anemia. Details are
given in Chapter 23.
i. Large gene deletions, e.g. alpha-thalassemia
(entire gene) or hemophilia (partial)
Missense; Unacceptable Mutation
ii. Deletion of a codon, e.g. cystic fibrosis (one amino
acid) 508th phenylalanine is missing in the CFTR The single amino acid substitution alters the properties
protein. of the protein to such an extent that it becomes non-
iii. Deletion of a single base which will give rise to functional and the condition is incompatible with normal
frameshift effect. life. For example, several variants collectively called
Hemoglobins M result from histidine-to-tyrosine substi-
Insertion tution (CAU to UAU) of the distal or proximal histidine
residues of alpha- or beta chains. Since, these resi-
Insertions or additions or expansions are subclassified
dues are directly linked to heme, the Ferrous iron gets
into:
oxidized to ferric state and there is methemoglobinemia.
i. Single base additions, leading to frameshift effect.
Methemoglobin cannot transport oxygen and there is
ii. Trinucleotide expansions. In Huntington's chorea,
severe cyanosis.
CAG trinucleotides are repeated 30 to 300 times.
This leads to a polyglutamine repeat in the protein.
Nonsense; Terminator Codon Mutation
The severity of the disease is increased as the
numbers of repeats are more. A tyrosine (codon, UAC) may be mutated to a termina-
iii. Duplications. Gene duplication results from unequal tion codon (UM or UAG). This leads to premature termi-
crossing over of chromosomes during meiosis and nation of the protein, and so functional activity may be
plays an important role in evolution. destroyed, e.g. beta-thalassemia. Or, a terminator codon
is altered into a coding codon (UM to CAA). This
Effects of Mutations results in elongation of the protein to produce run on
polypeptide (Hb Constant spring) (see Chapter 23).
Silent Mutation
A point mutation may change the codon for one amino Frameshift Mutation
acid to a synonym for the same amino acid. For exam- This is due to addition or deletion of bases. From that
ple, CUA is mutated to CUC; both code for leucine, so point onwards, the reading frame shifts. A "garbled"
this mutation has no effect. (completely irrelevant) protein, with altered amino acid
sequence is produced. An example,
Missense but Acceptable Mutation Normal mRNA AUG ucu UGC AAA ......
A change in amino acid may be produced in the protein; Normal protein Met Ser Cys Lys.......
but with no functional consequences. For example, in DeletedU mRNA AUG cuu GCA AA .........
the normal hemoglobin A molecule, the 67th amino acid Garbled protein Met Leu A@ ........... ..
in beta-chain is valine. The codon in mRNA is GUU. In this hypothetical example, deletion of one uracil
If a point mutation changes it to GCU , the amino acid changes all the triplet codons thereafter. Therefore, a
Chapter 42: Control of Gene Expression 593

useless protein is produced. Frameshift mutations can a mutation of a single gene resulted only in a single
also lead to thalassemia, premature chain termination chemical reaction. which gave evidence to the concept
and run-on-polypeptide. of "one gene, one enzyme·.
Frameshift mutations can result from aberrant splic-
ing of primary transcript due to mutations at splice sites.
Lethal Mutations
Alternative splicing can give rise to different isoforms of The alteration is incompatible with life of the cell or the
the same protein in different tissues e.g. Tropomyosin organism. For example, a mutation which does not pro-
in skeletal and cardiac tissue. mRNA editing can also pro- duce alpha chains (4 gene deletion) will result in intra-
duce two different proteins: e.g. Apo-B-100 and Apo-B-48. uterine death of embryo.

Conditional Mutations Silent Mutations


Most of the spontaneous mutations are conditional; they Alteration at an insignificant region of a protein may not
are manifested only when circumstances are appro- have any metabolic effect.
priate. Bacteria acquire resistance, if treated with anti-
biotics for a long time. This is explained by spontaneous Beneficial Mutations
conditional mutations. In the normal circumstances, wild Although rare, beneficial spontaneous mutations are
bacilli will grow. In the medium containing antibiotic, the the basis of evolution. Such beneficial mutants are arti-
resistant bacilli are selected. In a tuberculosis patient, ficially selected in agriculture. Normal maize is deficient
a lung cavity may harbor about 10 12 bacilli. This may in tryptophan. Tryptophan-rich maize varieties are now
contain about 106 mutations, out of which a few could available for cultivation. Microorganisms often have anti-
be streptomycin resistant. Therefore, if only one drug genic mutation. These are beneficial to microorganisms
is given, there will be overgrowth of drug resistant bacilli. (but of course, bad to human beings).
To avoid this, combinations of two antituberculous drugs
are given. So, drug-1-resistant mutants are killed by Carcinogenic Effect
drug-2 and drug-2-resistant mutants are removed by
The mutation may not be lethal, but may alter the regula-
drug-1. The statistical probability of a single bacillus
tory mechanisms. Such a mutation in a somatic cell may
acquiring resistance against both drugs is negligible.
result in uncontrolled cell division leading to cancer. Any
Mutagens and Mutagenesis substance causing increased rate of mutation can also
increase the probability of cancer. Thus all carcinogens
Any agent which will increase DNA damage or cell pro-
are mutagens.
liferation can cause increased rate of mutations also.
Such substances are called mutagens. X-ray, gamma- Site-directed Mutagenesis
ray, UV ray, acridine orange, etc. are well-known muta-
Michael Smith (Nobel Prize, 1993) described this tech-
gens. MOiier (Nobel Prize, 1946) showed that the rate
nique. An oligodeoxyribonucleotide is synthesized,
of mutation was proportional to the dose of irradiation.
whose sequence is complementary to a part of a known
Beadle (Nobel Prize, 1958) showed that the effect of
gene. A specific deletion/insertion is produced in the
X-irradiation on metabolism was due to mutations of
oligo. It is then extended by DNAP. After replication, one
genes. Tatum (Nobel Prize, 1958) further showed that
strand is normal and the other strand contains the muta-
tion at the specific site. This allows study on the effect of
that particular mutation.

Old Age Genetics


The concept is that "genes determine how one ages".
Environment determines 70-80% of how we age, and
Hermann George Edward Michael
Muller Beadle Tatum Smith
genetics contributes remaining 20-30% of the same.
NP 1946 NP 1958 NP 1958 NP 1993 Environmental factors are lifestyle habits like diet, exer-
1890-1967 190:J-1989 1909-1975 1932- 2000 cise, smoking, alcohol etc. Not only how long one lives,
594 Section E: Molecular Biology

but also how healthy one is during one's lifetime is de- 5 different cyclin dependent kinases (CDK 1, 2, 4, 5
termined at least partially be genes. Molecular variations and 6) control the cycle. Cyclins are so named because
or single nucleotide polymorphisms (SNPs) in numerous they are synthesized throughout the cell cycle, and are
genes are responsible for this phenomenon. abruptly destroyed during mitosis.
There are 3 gene systems which determine lifespan,
namely. , insulin signaling, free radicals and antioxidants Retinoblastoma (RB)
as well as DNA repair mechanisms. Oncosuppressor Protein
Insulin greatly influences the body's ability to grow
RB protein is the product of an oncosuppressor gene (see
and utilize energy. Elderly people can live longer if they
Chapter 48). It is so named, because it was isolated from
consume fewer calories.
patients of retinoblastoma (cancer arising from retina).
Antioxidants mitigate the dangerous cell-killing
Rb inhibits cell cycle at G1 phase. But in controlled cell
activities of free radicals and might prolong life.
cycles, cyclin D levels rise in the late G 1 phase. The
DNA repair systems help the body's fight against
cyclin D inactivates Rb, which is separated from E2F.
harmful agents in the environment and help to sustain This is the normal mechanism to overcome the G1 arrest
healthy lifespan. by Rb. Certain tumor antigens derived from viruses such
Specific variations in certain genes are also found to as SV40, HSV, HPV may combine with Rb. Then, Rb
influence healthy lifespan. Some of these genes include cannot inhibit cell cycle, leading to continuous cell divi-
telomerase, Foxo3, inflammatory cytokines, and mito- sion and cancer (Fig. 42.5).
chondrial DNA.
The p53 Oncosuppressor Protein
So named because it is a protein with 53kD size, hav-
LCELLCYCLE_ _ __ ing 393 amino acids. The half-life is only 5-10 minutes.
The term 'cell cycle' refers to the events occurring p53 inhibits cell division, allowing time for any damage
during the period between two mitotic divisions. It is to DNA to be repaired. If damage is extensive and repair
divided into G1 (gap-1 ), S (synthesis), G2 (gap-2), and is not possible, the p53 directs the cell to apoptosis. In
M (mitosis) phases. The cell division is taking place in most cancer cells, the p53 is mutated or nonfunctional.
M phase. It is the shortest phase, lasting about 1 hour.
The daughter cells then either enter into GO (undivid-
ing or dormant) phase or re-enter the cell cycle when UV
there is necessity for growth and repair. In a normal cell
population, most of the cells are in GO phase. Inter- ! released
ATM
phase is the period between the end of M phase and
the beginning of the next mitosis. In G1 phase, protein !
and RNA increase. Duration of G1 phase is about 12 CHK 1 kinase activated
hours. In the S phase, DNA is synthesized, but only !
CDC25 inactivated
once. DNA content doubles, nucleus becomes tetra-
ploid (4n). The entire diploid genome is replicated into !
Cyclin CDK2 active
a tetraploid genome. S phase lasts about 6 to 8 hours.
!
In the G2 phase, there is cytoplasmic enlargement. G1-S checkpoint released
DNA repair is also taking place in the G2 phase. It !
lasts for about 4 to 5 hours. The total cell cycle is about
20-22 hours duration in mammalian cells (Fig. 42.4 ). G1 f:: s
Cell Cycle
Cell Cycle Controls or Checkpoints
Hartwell, Hunt and Nurse were awarded Nobel Prize in
2001 for their contributions in elucidating the cell cycle
regulation. Four types of cyclins (A, B, D and E) and Fig. 42.4: Cell cycle phases (total 20-22 hours)
Chapter 42: Control of Gene Expression 595

·- ·
Rb is removed bl( tumor antigen;
G2 checkpoint ---+ No inhibition

•••
••••
Rb maximum phosphorylated
Rb phosphorylated

•-
.' No inhibition

Rb inhibits G1 checkpoint
G1 to Sentry

GO (resting)

Fig. 42.5: Cell cycle controls or Checkpoints. Retinoblastoma protein inhibits cell cycle at G1 checkpoint. Body ,circumvents this block
by phosphorylation of Rb protein. This is done normally by cyclin D-CDK. Tumor antigens will attach with Rb protein, so Rb inhibition is
lost; there will be uncontrolled cell division, leading to cancer

Apoptosis (Programmed Cell Death)


Differentiation and growth needs reshaping of organs;
this could be done only when old cells are removed
from the area. Removal of superfluous, aged or partially
damaged cell is done by apoptosis. The term literally
means "dropping off', similar to the old leaf falling from Leland Timothy Paul Nurse
Hartwell Hunt NP2001
the tree. Nuclear shrinkage, chromatin condensation NP2001 NP 2001 b. 1949
and membrane blebbing are characteristic features of b.1939 b.1943
apoptosis. Apoptosis-mediating genes (suicidal genes) The same gene may be alternatively opened and
(oncosuppressor genes) are c-fos, p53, Rb. Apoptosis- shut, as per the need of the metabolism. Such regula-
protecting genes are bcl-2 and other oncogenes. Bren- tion is done by induction and repression mechanisms.
ner, Horvitz and Sulston were awarded Nobel Prize in Induction is the phenomenon of increased synthesis of
2002 for their contributions to apoptosis mechanisms. protein or enzyme in response to certain signal. Such
enzymes are said to be inducible; and the signals are

I REGULATION OF
GENE EXPRESSION
Induction and Repression
called inducers. Induction is turning "on" the switch of the
gene. Repression is turning "off" the f)ene expression.

Operon Concept of Gene Regulation


Pioneer in genetic studies, Ochoa was awarded Nobel Francois Jacob and Jacques Monad put forward the
Prize in 1959. Synthesis of proteins under the influence operon concept in 1961, for which they were awarded No-
of gene is called gene expression. All genes of the cell bel Prize in 1965. Andre Lwoff who worked on derepres-
are not expressed at all the time. For example, the insu- sion also received the Nobel Prize in 1965. Jacob and
lin gene is expressed only in the beta cells of pancreas; Monad theory was based on the observations on lactose
but not in other tissues. In other words, insulin gene is metabolism in Escherichia coli (bacteria).Cells grown
in the state of repression in all other cells. Some genes in glucose medium do not contain lbeta-galactosidase
are expressed almost always in all cells. For example, (lactase). But when cells are transferred to a medium
enzymes of glycolysis are synthesized by all cells. Such containing only lactose, then the enzyme level in the cell
genes are called constitutive genes or housekeeping increases several thousand fold. Thus, lactose metabo-
genes. lism is regulated by an induction or derepression process.
596 Section E: Molecular Biology

Francois Jacques
Sydney Robert JohnE BruceN
Jacob Monad
Brenner Horvitz Sulston Ames
NP 1965 NP 1965
1920-2013 1910-1976 NP2002 NP2002 NP 2002 b. 1928
b. 1927 b. 1947 b. 1942
The Lac Operon
operator region. So there is no repressor molecule at
Operon is a unit of gene expression; it includes struc- the operator site. Now, RNAP can transcribe the struc-
tural genes, control elements, regulator/inhibitor gene, tural genes, which are then translated (Fig. 42.6B). Thus
promoter and operator areas. In the bacterial cell, the lactose switches the genes "on". Lactose induces the
z gene encodes beta-galactosidase, the enzyme which synthesis of lactose utilizing enzymes. Hence, lactose is
hydrolyzes lactose to galactose and glucose. The Y
an inducer of these genes and the mechanism is said
gene is responsible for production of a permease which
to be derepression of the gene.
transports lactose and galactose into the cell. The A
gene codes for thiogalactoside transacetylase. Since Z, Clinical Applications
y and A code for the structure of the proteins, they are
Lactase in human intestine is an inducible enzyme.
called structural genes. These three genes are present
Clinical manifestations of lactase deficiency and lactose
as contiguous segments of DNA (Figs 42.6). The trans-
intolerance are described in Chapter 10. In humans,
cription of these genes start from a common promoter
(P), located close to the Z gene. The RNA polymerase examples of derepression include induction of trypto-
binds to the promoter and transcribes these three struc- phan pyrrolase, and transaminases by glucocorticoids;
tural genes as a single mRNA. as well as ALA synthase by barbiturates.

Transcription is Normally Repressed Regulation of Genes by Repression


Transcription of the structural gene is under the control of Repression is the mechanism by which the presence
another regulator or the "i" (Inhibitor) gene. It is far away of excess product of a pathway shuts off the synthe-
from the structural genes. Regulatory gene produces a sis of the key enzyme of that pathway. Heme synthesis
repressor molecule. The lac repressor has strong affi-
is an example. It is regulated by repression of ALA syn-
nity to the operator site. The operator region is 27 bp thase, the key enzyme of the pathway (see Chapter 22).
long, to which the repressor tightly binds. The operator
Transcription of structural gene for ALA synthase is
site is between the promoter and structural genes (Fig.
controlled by a regulatory gene. It produces the apo-
42.6A). When RNAP identifies the promoter sequence
repressor, which binds with heme and becomes the
and moves towards the structural genes, it is stopped
active holo-repressor. Here heme acts as the co-repressor
by the hindrance produced by repressor molecule. This
(Fig. 42.7).
is like the action of a zip. If a thread is placed across
The holo-repressor binds to the operator and stops
its way, the zip cannot move further. Similarly, when
repressor is attached to the operator, RNAP cannot transcription of the gene. Upstream to the structural
move further. So, structural genes are not transcribed. genes lies the promoter site, where the RNA polymerase
Thus, when lactose is not available, the lactose utilizing (RNAP) attaches and starts mRNA synthesis. The
enzymes are not synthesized (Fig. 42.6A). operator site is in between promoter and structural genes.
So, when RNAP reaches operator site, it cannot move
Derepression of Lac Operon further (Fig. 42.7). So enzyme synthesis stops, and
When lactose is introduced into the medium, lactose heme synthesis slows down.
binds to the repressor protein (Fig. 42.6B). Repressor- On the other hand, when heme is not available,
lactose complex is inactive, which does not bind to the co-repressor is not available, therefore, repression is not
Chapter 42: Control of Gene Expression 597

Glucose present; Lactose absent Glucose absent; Lactose present


Operon shut off Operon on

Operon
Inducer •
•• • •

=t=
Inducer + repressor

!
Promoter Operator Structural (Lactose) now repressor
cannot bind
I Iz y A
RNAP binds -+

- I
' "ii, !
RNAP
Repressor bound
to operator site RNAP moves on

a Structural genes
Not transcribed
Structural genes
now transcribed

Figs. 42.6A and B: (A) Repression of Lac operon. When lactose is absent, repressor molecules bind to the operator site. So RNAP
cannot work, and genes are in "off' position. (B) Induction or Derepression of Laci operon. Lactose attaches to repressor; so repressor
cannot bind to operator site which is free; genes are in ·on" position; protein is synthesized

Regulator Promoter Operator Structural gene


DNA

l
Ai.A synthase

l
~:::~/ \ • • Co-repressor (Heme)

Fig. 42.7: Repression by heme on the enzymes responsible for


heme synthesis

effective and enzyme synthesis starts. Thus, the synthesis


of heme is autoregulated by the repression mechanism.

Hormone Response Elements (HRE)


In higher organisms, hormones or their second messen-
gers fu nction as inducers. Many hormones, particularly Fig. 42.8: Steroid hormone binds to the HRE (hormone response
steroid hormones, elicit physiological response through element) region of DNA, leading to gene activation. 1 = Steroid
hormone reaches cytoplasm; 2 = Hormone binds with cytoplasmic
controlling gene expression (Fig. 42.8). Glucocorticoids receptor. 3 = Hormone binds HRE

attach to a cytoplasmic receptor; then the receptor-


hormone complex translocates to the nucleus. It finally
attaches on the HRE in the DNA. The receptor binds at
the enhancer region, which activates the promoter, so
that transcription is accelerated. Examples are recep-
Stanley Alfred D Salvador Rita tors for glucocorticoid, mineralocorticoid, progesterone,
Cohen Hershey E Luria Levi- androgen, estrogen, thyroxine, vitamin D and retinoic
NP 1986 NP 1969 NP 1969 Montalcini
1908-1997 1912- 1991 NP 1986 acid. Several cytokines also exert their effect by regulat-
b. 1922
1909-2012 ing gene expression.
598 Section E: Molecular Biology

VIRUSES
•U:- Clinical Case Study 42.1
Viruses are absolute parasites on living cells. They con-
tain only the bare minimum of genetic information for A healthy 10-year-old boy interested in sports started
survival and replication. Salvador Luria in 1942 showed complaining of muscle cramps and weakness of lower

that viruses are visible only by electron microscopy. limbs after the sports. On examination, he had mild
Alfred Hershey proved that nucleic acids, but not pro- wasting of the lower limb muscles. History revealed that
teins, are the genetic material in viruses. Both got Nobel his mother had a younger brother, who had a similar ill-
Prize in 1969. Viruses generally bind to specific recep- ness to which he succumbed around the age of 20.
tors on the host cell surface. For example, binding of HIV A. What is the most probable cause?
to CD4 receptor occurs in lipid raft areas through GP120 B. What are the investigations to be done?
(glycoprotein of the virus). Influenza virus binds NANA C. How does history give a clue in this case?
(N-acetyl neuraminic acid) residues on glycoprotein D. What is the prognosis?
receptors on cell surface. After entry into the host cell, E. Explain the nature of inheritance in this case.
the viruses utilize the host cell machinery for growth and 0
replication. • · Clinical Case Study 42.1 Answer
X-linked muscular dystrophy. Creatinine in urine and CK
Epigenetic Regulation in blood will be increased.
Yet another mechanism for gene regulation occurring in
both prokaryotes and eukaryotes is by the phenomenon
(b_EARNING POINTS, CHAPTER 42
known as epigenetic regulation. The important epige- 1. Mutations can be acceptable (Hb Bristol), partially
netic mechanisms are DNA methylation and histone acceptable (HbS) or unacceptable (HbM).
modification. External or environmental factors causes 2. Cell cycles have 5 phases: GO, M, G1 , Sand G2.
genes to switch on or off. Such changes may or may 3. Operon is a unit of gene expression. It includes
not be heritable. This is especially important during the structural genes, control elements, regulator/inhibitor
phenomenon of cellular differentiation. gene, promoter and operator areas, e.g. Lac operon.

PART-1: ESSAY AND SHORT NOTE QUESTIONS


42-1. Explain with suitable examples, how mutations result in abnormal proteins.
42-2. What is mutation? What are mutagens? Describe point mutation and frameshift mutation.

SHORT NOTE QUESTIONS


42-3. Induction. 42-7. Mutagens.
42-4. Repression. 42-8. Point mutation.
42-5. Operon concept. 42-9. Frame-shift mutation.
42-6. Mutations. 42-10. Conditional mutation.

PART-2: MULTIPLE CHOICE QUESTIONS

42-1 . All the following traits are inherited through the X B. 25 % of children will be carriers
chromosome, except: C. 25 % of children will be normal
A. Sickle cell anemia D. All the children will be genotypically abnormal
B. Hemophilia 42-3. All the following lead to cancer, except
C. Glucose-6-phosphate dehydrogenase deficiency A. Promoter insertion
D. Duchenne type muscular dystrophy B. Chromosomal translocation
42-2. In autosomal dominant inheritance, if only one C. Point mutation
parent is affected: D. Trinucleotide expansion
A. 50% of children will be sufferers 42-4. Point mutations may be silent because:
Chapter 42: Control of Gene Expression 599

A. A change in third base of the codon may produce 42-11. In the process of transduction:
a synonym A. Bacteria takes up DNA from the surrounding
B. Change in base sequence is corrected during medium
transcription 8 . New genes are produced by accumulation of
C. Non-specific base pairing between mRNA and DNA mutations
D. High fidelity of transcription of mRNA C. Normal cell is transformed into a malignant cell
42-5. HbS (Sickle cell anemia) is a result of D. Virus carries some part of the genetic information
A. Transversion mutation from one host to another host
B. Transition mutation 42-12. All are correct with retroviruses, except
C. Nonsense mutation A. Can use their RNA as template
D. Frameshift mutation B. Have reverse transcriptase
42-6. Mutagenicity of a compound is checked by: C. Can multiply in host cells
A. Hay's test B. Guthrie test D. Are inactivated after the first multiplication cycle
C. Ames' test D. FISH test
42-13. Frameshift mutation results from
42-7. In which phase of the cell cycle synthesis of deoxy-
A. Substitution of a single base
ribonucleotide triphosphate is highest?
8 . Deletion of a single base
A. GO B. G1
C. Addition of a codon
C. S D. M
D. Deletion of a codon
42-8. Which hormone does not act at the level of trans-
42-14. Abnormal genotype and normal phenotype is seen
cription?
in mutation of
A. Cortisol B. Calcitriol
A. Splice site B. Intervening sequences
C. Aldosterone D. Calcitonin
42-9. Viruses may have all the following fates within the C. Terminator codon D. Transcription factors
host cells, except 42-15. When GAC is replaced by GAA, the result is:
A. Virus particles may multiply within host cells A. Back mutation B. Deletion mutation
B. Viral DNA may be incorporated into host cell DNA C. Silent mutation D. Conservative mutation
C. Viral DNA may be transcribed 42-16. The cell cycle is regulated by
D. Viruses may suppress oncogenes A. Availability of nucleotides
42-10. All the following are antiviral drugs, except B. Presence of inducers in cells
A. Cytosine arabinoside B. Acyclovir C. Cyclin dependent kinase
C. Ribavirin D. Azidothymidine D. DNA dependent kinases

ANSWERS OF MULTIPLE CHOICE QUESTIONS


42-1 . A. 42-2. A. 42-3. D. 42-4. A. 42-5. A. 42-6. C. 42-7. C.
42-8. D. 42-9. D. 42-10. A. 42-11 . D. 42-12. D. 42-13. B. 42-14. B.
42-15. C. 42-16. C.

PART-3: VIVA VOCE QUESTIONS AND ANSWERS

42-1. Give examples of X chromosome linked trans- 42-6. What is cell cycle?
mission. Cell cycle refers to the sequence of events that occur
Hemophilia; Glucose-6-phsophate dehydrogenase during the growth and cell division.
deficiency; Duchenne type muscular dystrophy. 42-7. In which phase of the cell cycle, DNA synthesis is
42-2. What is a mutation? maximum?
An alteration in the genetic material results in a muta- During S phase.
tion. 42-8. What is the p53?
42-3. Give an example. It is an oncosuppressor gene product.
HbS or sickle-cell hemoglobin is produced by a mu- 42-9. What is an operon?
tation of the beta chain in which the 6th position is One or more structural genes, together with their
changed to valine, instead of the normal glutamate. operator gene.
42-4. Give examples for mutagens. 42-10. Give examples of enzyme induction.
Acridine orange: X-rays; gamma rays; Methyl cholan- Beta galactosidase by lactose; Tryptophan pyrrolase
threne by glucocorticoid and ALA synthase by barbiturates.
42-5. What is apoptosis? 42-11 . Give an example of repression.
Prog rammed cell death. ALA synthase by heme.
_ _ _ _ _ Chapter 43
Recombinant
DNA Technology and
Gene Therapy
Chapter at a Glance
The learner will be able to answer questions on the following topics:
Recombinant DNA technology Molecular cloning
Restriction endonucleases Gene therapy
Vectors

C ECOMBINANT DNA
ECHNOLOGY
-----------
8 io tech noIog y may be defined as "the method by which
Risk of Contamination is Eliminated
It is now possible to produce a biological substance with-
out any contamination. Hepatitis, caused by the hepa-
titis B virus (HBV), is highly contagious. A vaccine is
a living organism or its parts are used to change or to
prepared from the hepatitis B virus surface proteins,
incorporate a particular character to another living
which will give protection from infection. Originally the
organism". Biotechnology involves the application of
virus was isolated from pooled blood of patients, and the
scientific principles to the processing of materials by
specific protein was isolated. However, blood of hepatitis
biological agents. The use of new varieties of micro-
patients is known to be highly infective. It is absolutely
organisms to breakdown pollutants in soil or water to harm-
essential to make sure that the preparations of vaccines
less end products is known as bioremediation. Genetic
or clotting factors are free from contaminants such as
recombination is the exchange of information between
hepatitis B particles. Recombinant DNA technology pro-
two DNA segments. This is a common occurrence within
the same species. But by artificial means, when a gene vides the answer to produce safe antigens for vaccine
of one species is transferred to another living organism, production.
it is called recombinant DNA technology. In common
Specific Probes for Diagnosis of Diseases
parlance, this is known as genetic engineering.
Specific probes are useful for:
Applications of i. Antenatal diagnosis of genetic diseases. For example,
Recombinant Technology many of the single gene defects (e.g. cystic fibrosis,
phenylketonuria, etc.) could be identified by taking
Quantitative Preparation of Biomolecules
cell samples from fetus.
If molecules are isolated from higher organisms, the ii. To identify viral particles or bacterial DNA in sus-
availability will be greatly limited. For example, to get pected blood and tissue samples.
1 unit of growth hormone, more than 1000 pituitaries iii. To demonstrate virus integration in transformed
from cadavers are required . By means of recombinant cells.
technology, large scale availability is now assured. iv. To detect activation of oncogenes in cancer.
Chapter 43: Recombinant DNA Technology and Gene Therapy 601

TABLE 43. 1: Spec1fic1ty of restriction enzymes (The arrows


show the site or cut by the enzyme) - C-T-C-G-A-T G-A- A- T- T C A- C-C-
Specific sequence identified
Enzyme Source ofenzyme by the enzyme 1 I I I I I r+--+--+--1---h I I I
-G-A-G-C-T-A C-T-T-A-A..:.C, T-G-G-
EcoRI Escherichia coli RY 13 Gt AATT l C

.Yi-rn-
C TTAA G
Hindlll Haemophilus influenzae Rd A 1 AGCT l T
T TCGA A
Taql Thermus aquaticus T~A
A GC T - C- T- C- G- A- T- G Sticky ends G- T- G- G-
GTT j AAC
Hpal Haemophilus
parainfluenzae CAA ! TTG 111 11 11 I
-G-A- G- C- T-A-ef T-T-A-AI

v. To pinpoint the location of a gene in a chromosome. Fig. 43.1 : EcoRI enzyme cuts the bonds marked with red arrow.
This results in the sticky ends
vi. To identify mutations in genes and for pedigree
analysis: point mutations, deletions, insertions and
rearrangements of DNA could be identified. Sickle
cell disease is an example of point mutation (see
Chapter 23). The substitution of T for A in the tem-
plate strand of DNA in the beta globin gene changes
the Mstl l restriction site. Thus normal, heterozygous
and homozygous individuals in the family could be Paul Hamilton Wemer Daniel
Berg Smith Arber Nathans
identified.
NP 1980 NP 1978 NP 1978 NP 1978
b. 1926 b. 1931 b. 1929 1928-1999
Gene Therapy
An important application of recombinant technology is Restriction Sites
in gene therapy. Normal genes could be introduced into
Restriction endonucleases have specific recognition
the patient so that genetic diseases can be cured. These
sites where they cut the DNA. (Table 43.1 ). There are
techniques are described later in this chapter.
more than 800 such enzymes now available commer-
Restriction Endonucleases (RE) cially. These enzymes recognize specific sequence with
In order to transfer a gene, it is to be fi rst selectively split palindrome arrangement. Palindrome in Greek means
from the parent DNA. This is usually achieved by restric- "to run backwards". It is similar to a word that reads back-
tion endonucleases which are referred to as "molecular wards or forwards similarly, e.g. "madam". These are
scissors". also called inverted repeat sequences, which means
Werner Arber showed that certain enzymes of bac- the nucleotide sequence in 5' to 3' direction is the same in
teria restrict the entry of phages into host bacteria. both strands. The resultant DNA fragments will generally
Hence, the name restriction endonucleases. Hamilton have overlapping sticky ends (Fig.43.1).
Smith in 1970 isolated the first restriction enzyme beta
Hind-I. Daniel Nathans in 1971 for the first time applied I VECTORS
the enzyme to cut the DNA. All the three got Nobel Prize
in 1978. Paul Berg (Nobel Prize 1980) developed the In order to introduce the human gene into bacteria, at
cutting technique for recombinant DNA. The restriction first, the gene is transferred to a carrier, known as a
endonucleases are named after the species and strains vector. Most commonly used vectors are plasmids.
of bacteria and the order of discovery. For example, the Plasmids are circular double-stranded DNA molecules
enzyme EcoRI is isolated from Escherichia coli RY13 seen inside bacteria. In nature, plasmids confer antibiotic
strain. The Roman numeral "one" indicates the order of resistance to host bacteria. This feature has profound
discovery of an enzyme from that species. Restriction significance in clinical practice because, antibiotic resis-
enzymes are isolated from bacteria. tance property is exchanged between bacteria. Plasmids
602 Section E: Molecular Biology

selective expression of a particular gene in certain tissue.


Total RNA isolated, containing mRNAs and rRNAs
For example, insulin mRNA will be abundant in the beta
+
Pass through a column with immobilized oligo-dT, when cells of pancreas. From the specific mRNA, the cDNA is
mRNA with poly A tail are retained, as T pairs with A
produced by using reverse transcriptase (RT) (Fig.43.2).
+
mRNAs eluted; these are of different sizes coding for making
different proteins Preparation of Chimeric DNA Molecules
+
Fractionate mRNA species; specific insulin-mRNA is identified Chimera is the Greek mythological monster with a lion's
and separated by passing through column head, goat's body and serpent's tail. Narasimha (lion's
+
DNA synthesis by reverse transcriptase + dNTPs;
head and human body) is another example from Indian
RNA- DNA hybrid is produced mythology. A vector carrying a foreign DNA is called chi-

l
meric DNA or hybrid DNA or Recombinant DNA. A sum-
5' dTTT• • • • • • • • • • • • • • • • • • - 3' DNA mary of the procedure is given in Figure 43.3.
3' AAAMAMMAMMMMMMMMM 5' RNA
A circular plasmid vector DNA is cut with a specific
Extend 3' end of DNA by terminal deoxy nucleotidyl
transferase + dCTP
restriction endonuclease (RE). If EcoRI is used, sticky

l
ends are produced with TTAA sequence on one DNA
5' dTTT· • • • • • • • • • • • • • • • - • CCCC 3' DNA
3' AAAAI\I\AI\I\I\I\AI\I\I\AAI\AA/+,I\I\AI\AAl\1\1\ 51 RNA
strand, and AATT sequence on the other strand (Table

l
43.1 and Fig. 43.3). The human DNA is also treated with
RNA hydrolyzed by RNAase; making single stranded DNA
the same RE, so that the same sequences are gene-
rated at the sticky ends of the cut piece. Then the vector
s· ,m-------------------•cccc ,, DNA
DNA and human cut-piece DNA are incubated together
DNA double strand is primed by a piece of oligo dG
so that annealing takes place. The sticky ends of both
vector and human DNA have complementary sequences,
5' dTTT • · • • • • • • • • • • • • • • • • • • CCCC 3' DNA and therefore they come into contact with each other.

1 GGGG 5'

DNA double strand is completed by the action of DNA


DNA ligase enzyme is added, which introduces phos-
phodiester linkages between the vector and the insert
polymerase I + dNTP; now cDNA is complete molecules. Thus the chimeric DNA is finally produced.

5' dTTT • • • • • • • • • • • • • • • • • • • • • CCCC 3' DNA Cloning of Chimeric DNA

1 3' AAA· • • • • • · • • • • • • • • • • • • • • GGGG 5' DNA

Sticky ends on cDNA are generated by terminal


The next step is cloning . A clone is a large population
of identical bacteria or cells that arise from a common
transferase + dCTP. ancestor molecule. Cloning allows the production of a
5' dTTT· • • • • • • • • • • • • • • • • • CCCCCCCCC 3' large number of identical DNA molecules. The hybrid
3' CCCCCCAAA· • • • • • • • • • • • • • • • • • GGGG 5' DNA
molecules are amplified by the cloning technique.
Fig. 43.2: Steps to produce cDNA from mRNA
Transfection of Vector into the Host
replicate independent of bacterial DNA. Foreign DNA The process by which plasmid is introduced into the host
could be incorporated into them by using specific RE. is called transfection. Host E. coli cells and plasmid vec-
tors are incubated in hypertonic medium containing cal-
Procedure of DNA Recombination
cium for a few minutes. Then calcium ion channels are
Preparation of Specific Human Gene opened, through which the plasmid is imbibed into the
Isolation of a specific gene from the human DNA is a host cell. Now the host cells are allowed to grow in agar
laborious process. The beta globin gene is only 0.00005% plates containing growth medium.
of total human genes. This is like searching for a needle
in a haystack. This problem is generally solved by
Expression Vectors
preparation of cDNA (complementary copy DNA). It is To produce the human proteins, E. coli carrying the vec-
easier to start the work with mRNA, because there is tor with the insert is allowed to grow, without any protein
Chapter 43: Recombinant DNA Technology and Gene Therapy 603

Human mRNA
!
cDNA copy (see Fig. 43.2)
!
Plasmid vector cleaved by
specific restriction
. • endonuclease;
y

Plasmid cut
by ECoRI
0 _'
, Donor DNA
cut by ECoRI
vector carrying human
gene (see Fig. 43.3)

~ I aod joio by ONA ligas, Transfectlon Into


host bacteria

i
Selection of bacteria carrying
1• 0 • 00

the human gene; by antibiotic


Recombinant sensitivity
DNA

Clone amplification
Fig. 43.3: Production of chimeric DNA molecule by using ECoRI
restriction endonuclease. Red arrows show the site of cut by
ECoRI

Isolation of proteins 000000 000000

Fig. 43.4: DNA-recombinant technology


inhibitors. Such a vector carrying the foreign gene, which
is translated into a protein, is called expression vector. BOX. 43.1: Proteins produced by recombinant DNA technology

The human proteins can be harvested from the bacterial 1. Recombinant human Insulin
2. Recombinant human growth hormone (HGH, somatotropin)
cu lture. A summary of the DNA recombinan t technology and other human hormones (e.g. FSH)
is shown in Figure. 43.4. Depending on the length of the 3. Recombinant blood clotting factor VIII and other clotting
factors (Factor IX, tPA, hirudin)
DNA insert, different vectors like plasmids, cosmids and 4. Recombinant hepatitis B vaccine, HPV vaccine, etc.
artificial chromosomes are used. 5. Cytokines and growth factors (interferon, interleukins, etc.)
6. Monoclonal antibodies and other related products (rituximab,
trastuzumab, etc.)
Human Recombinant Proteins 7. Recombinant enzymes (acid a glucosidase [myozyme),
alpha-L-iduronidase [aldurazymel)
Hundreds of human proteins are now being synthe- 8. Recombinant HIV protein for HIV ELISA testing
9. Herbicide and insect resistant crops
sized by the recombinant technology. A list of important 1O. Other products: Bone morphogenic protein (BMP), albumin,
proteins produced by recombinant technology is given fibrinolytic and thrombolytic agents, etc.
in Box 43.1.
set of human DNA contains about 3 billion base pairs
Human Genome Project (HGP) (one cell contains 2 sets) and about 10,000 genes. The
"Book of Human Life" contains "23 Chapters", as the
The US Department of Energy together w ith the US
23 chromosomes.
National Institutes of Health started this project in 1990.
The impact of HGP will be on all branches of medi-
James Watson (co-discoverer of the structure of DNA,
cine and related health sciences. It is now possible to
Nobel laureate, 1962) was the first head o f the project;
isolate any human g ene of interest. Many previously
later Francis Collins succeeded him. The project included unknown genes have been identified.
scientists from 16 centers all over the world, mostly By December 1998, human chromosome 5 (about
from USA, with coordination from laboratories of Britain , 6% of human genome) was sequenced completely.
France, Japan and Germany. So, the Project was The final version of the sequence of the entire human
named as International Human Genome Sequencing genome was completed in 2003.
Consortium. Pharmacogenomics is a recently emerged science
It is one of the greatest achievements of humanity. from the genome project; it is the use of genetic infor-
The ambitious project was to decode the whole human mation towards the development of new drugs and their
genome and to sequence the whole human DNA. One targets of action.
604 Section E: Molecular Biology

GENE THERAPY
Gene therapy was once considered a fantasy. However, Take cells
from patient
thousands of individuals have already undergone human
clinical trials. A great leap in medical science has taken
place on 14th September, 1990, when a girl suffering
from adenosine deaminase deficiency (severe immuno-
©
Patient·s~II
now cont ns
normal
deficiency) was treated by transferring the normal gene gene
for adenosine deaminase.

What is Gene Therapy?


It is intracellular delivery of genes to generate a thera- Normal gene is
defective gene
peutic effect by correcting an existing abnormality. Only introduced into
patient's cell by Retrovirus carrying
somatic gene therapy, by inserting the new gene into the virus normal gene
somatic cell of the patient is under trial. Germ cell gene
Fig. 43.5: Ex vivo gene therapy
therapy is considered as unethical.

Summary of the Procedure deleted from the wild type retrovirus, rendering it inca-
pable of replication inside human body. Then the human
1. Isolate the healthy gene along with the sequence
gene is inserted into the virus. This is introduced into a
controlling its expression.
culture containing packaging cells having gag, pol and
2. Incorporate this gene into a carrier or vector as an
env genes (Fig . 43.6). These cells provide the neces-
expression cassette.
3. Finally deliver the vector to the target cells. sary proteins to pack the virus. The replication-
deficient, but infective, retrovirus vector carrying the
How the Genes are Introduced? human gene, now comes out of the cultured cells. These
There are three ways of applying gene carrying vectors: are introduced into the patient. The virus enters into the
a. Ex vivo strategy: Where the patients' cells are cul- target cell via specific receptor. In the cytoplasm of
tured in the laboratory, the new genes are infused the human cells, the reverse transcriptase carried by
into the cells; and modified cells are administered the vector converts the RNA to proviral DNA, which is
back to the patient (Fig. 43.5). integrated into the target cell DNA. The normal human
b. In situ strategy: When the expression cassette is gene can now express (Fig. 43.6).
injected to the patient either intravenously or direc- Advantages of retroviruses: The virus is modified,
tly to the tissue. and replication deficient. So infection with viral particle
c. In vivo strategy: Where the vector is administered is limited to one cycle, and is very safe. They can infect
directly to the cell, e.g. CF (cystic fibrosis ) gene to a wide variety of human cells. This strategy is very suit-
the respiratory tract cells. able for treatment of all diseases produced by single
gene mutations.
The Vectors Adenoviruses, Liposomes and Gene gun are some
other commonly employed carriers for transfer of foreign
Different vector (carrier) systems used for gene delivery
genes to host cells.
are: retroviruses, adenoviruses, adeno associated viral
vectors and herpes simplex viruses. Nonvirus systems
Accomplishments
include liposomes, plasmids and physical methods.
Gene therapy is effective in inherited disorders caused
Retroviruses by single genes. Several clinical trials have been
Retroviruses are RNA viruses that replicate through conducted. Success stories are few. The best developed
a DNA intermediate. Moloney murine leukemia virus and most successful cell therapy is hematopoietic stem
(MMLV) is commonly used. The gag, pol, env genes are cell transplantation (HSCT). Another established gene
Chapter 43: Recombinant DNA Technology and Gene Therapy 605

TABLE 43.2: Success stones of gene therapy


Disease Gene transferred by
LTR 1, Severe combined Adenosine deaminase enzyme
imm unodeficiency in chromosome 13 and 20 into

l
(SCIO) lymphocytes; by retrovirus
Delete gag, pol, anv genes; In that place add
Cassette containing human gene 2, Duchenne muscular Oystrophin gene on short arm of
dystrophy (OMO) X chromosome; by retrovirus
r-"""-~
LTR I '
3. Cystic fibrosis (CF) CFTR gene on chromosome 7 to
bronchial epithelium; adenovirus
4. Familial hyper- LDL receptor gene on chrom 19 to
cholesterolemia hepatocytes; retrovirus
5, Hemophilia A_a nd 8 genes for factor VIII and IX into
Naked viral RNA has to get fibroblasts; retrovirus
protein coat to enable them to
enter into human host cell. 6. Cancer Activation of p53 (tumor suppressor
So, defective virus is put gene) by liposome
into packaging cells, which
provide the viral coat 7. Leber's hereditary Introducing the gene for the enzyme
optic neuropathy (isomerohydrolase) using an adenoviral
vector directly to the retina

Virus particles with protein coat.


These are infective (can enter
into human cells); but cannot therapy for OTC (Ornithine transcarbamoylase) defi-
replicate or produce disease
ciency was reported.

Receptor mediated
entry of retrovirus into
STEM CELLS
human host cell
Stem cells are defined as cells with the capacity for
self-renewal and having potential to differentiate into
Reverse transcriptase
progenitors of different lineages which ultimately give
of virus copies RNA rise to mature tissues. Mario R. Capecchi, Sir Martin J
into virus ONA
Evans and Oliver Smithies were awarded Nobel Prize
in 2007 for their discoveries of principles for introduc-
Integrates with host ing specific gene modifications in mice by the use of
DNA inside nucleus
embryonic stem cells. Stem cells have the ability to divide

Fig. 43.6: Gene transfer by retroviral vector

therapy application is adoptive immunotherapy, which


exploits ex vivo expanded T cells, in order to harness
the power of immune effector and regulatory cells
for use against malignancies, infections and auto- Shinya Sir John Gurdon
immune diseases. The most dazzling ones are shown Yamanaka NP 2012
NP 2012 b. 1933
in Table 43.2 .
b. 1962
Obstacles to Success
The potential of gene therapy is enormous. It is now
theoretically possible to cure all the genetic diseases.
However, it may take several years to get it available
for common clinical use. The following limitations are
Mario R Sir Martin J Oliver
encountered for gene therapy: (a) Inconsistent results
Capecchi Evans Smithies
(b) Lack of ideal vector (c) Lack of targetting ability in NP 2007 NP 2007 NP 2007
nonviral vectors (d) Death during the course of gene b. 1937 b. 1941 b. 1925
606 Section E: Molecular Biology

for an indefinite period. They can give rise to a variety gene of one species is transferred into another under
of specialized cell types. This phenomenon is known as laboratory conditions, the technique is called
developmental plasticity. Stem cells can be isolated from recombinant DNA technology or genetic engineering.
embryos, umbilical cord as well as from adult bone marrow. 2. Restriction endonucleases (RE), also known as
Plasticity is more for embryonic stem cells. Plasticity is 'molecular scissors' cut at sequences which are
defined as the ability of stem cells from one germinal palindromes. Each RE is characterized by a spe-
layer to give rise to tissues of another germinal layer. cific 'restriction site'.
Stem cells have the unique capacity to produce 3. Plasmids are commonly used vectors. They pro-
unaltered daughter cells (renewal) and also to generate vide antibiotic resistance to their host bacteria. This
specialized cells (potency). Stem cells may be capable property is used as a marker in genetic engineering.
of producing all types of cells of the organism (totipo- 4. A vector carrying a foreign DNA is called 'Chimeric
tent), or able to generate cells of the three germ layers DNA'.
(pleuripotent). Active research is being done to utilize
5. The process of introducing a plasmid into a host is
stem cells in the treatment of the following diseases:
called transfection.
stroke, brain injury, Alzheimer's disease, Parkinsonism,
6. Gene therapy involves the delivering genes to gen-
wound healing, myocardial infarction, muscular dystro-
erate a therapeutic effect by correcting an existing
phy, spinal cord injury, diabetes, cancers.
abnormality.
General belief was that only stem cells are plastic,
Introducing genes involves three ways of applying
and the differentiated matured cells will lose their plas-
gene carrying vectors, ex vivo, in situ and in vivo.
ticity. But Sir John Gurdon and Shinya Yamanaka have
7. Retroviruses, adenoviruses and herpes simplex
independently discovered that mature cells can be
reprogrammed to become pluripotent; they were awar- viruses have been used carrier systems in human
ded Nobel Prize in 2012. gene studies.
8. Diseases for which gene therapy has been attemp-
·LEARNING POINTS, CHAPTER 43 ted are severe combined immunodeficiency (SCI O),
Duchenne muscular dystrophy (DMD), cystic fibro-
1. Genetic recombination involves the exchange of
sis, hemophilia.
information between two segments of DNA. When a

PART-1 : ESSAY AND SHORT NOTE QUESTIONS


43-1 . Describe recombinant DNA technology. What are the important applications of the technique?
43-2. How the DNA molecule cloned? Write two applications of DNA cloning.

SHORT NOTE QUESTIONS


43-3. Restriction endonucleases. 43-4. Gene therapy.

PART-2: MULTIPLE CHOICE QUESTIONS

43-1 . Which enzyme is used for preparing a recombi- C. Inhibition of replication


nant DNA molecule? D. Gene therapy
A. Restriction endonuclease 43-3. With plasmids, all are correct, except
B. RNA polymerase A. Used as vectors in recombinant DNA technology
C. DNA polymerase B. Carriers of genes that confer drug resistance
D. Topoisomerase C. Extra chromosomal duplex DNA molecules
43-2. Application of recombinant DNA technology include D. Specialized regions of bacterial chromosomes
all the following, except: 43-4. W ith plasmids, all are correct, except:
A. Detection of oncogenes A. They are useful for ELISA technique
B. Synthesis of human proteins B. They are circular double stranded DNA inside bacteria
Chapter 43: Recombinant DNA Technology and Gene Therapy 607

C. They replicate independent of bacterial DNA replication 43-9. The c DNA is prepared by using the enzyme:
D. They confer antibiotic resistance to host bacteria A. RNA polymerase B. DNA polymerase
43-5. All the following are nucleic acids, except C. Reverse transcriptase D. Restriction endonuclease
A. Plasmids B. Prions 48-10. Which of the following is not used as a vector in
C. Cosmids D. Virions molecular cloning?
43-6. Synthesis of recombinant DNA (rDNA) requires all A. Plasmid B. Cosmid
the following, except: C. Artificial chromosome D. Hapten
A. Restriction endonuclease 43-11. Restriction endonucleases
B. RNA primer
A. Cleaves DNA at splice sites
C. Plasmid vector
B. Protects bacterial genome
D. DNA ligase
C. Hydrolyses DNA from 5' end
43-7. Which is not amenable to gene therapy?
D. Inhibits human DNAP
A. Hemophilia
43-12. The enzyme deficiency which was first corrected
B. Severe combined immunodeficiency
by gene therapy is:
C. Cystic fibrosis
A. Adenosine deaminase
D. Infective hepatitis
43-8. All are vectors for gene therapy, except: B. HGPRTase
A. Proteasomes B. Liposomes C. Glucose-6-phosphatase
C. Adenoviruses D. Retroviruses D. APRTase

ANSWERS OF MULTIPLE CHOICE QUESTIONS


43-1. A 43-2. C 43-3. D 43-4. A 43-5. B 43-6. B 43-7. D
43-8. A 43-9. C 48-10. D 43-11 . B 43-12. A

PART-3: VIVA VOCE QUESTIONS AND ANSWERS

43-1. What are restriction endonucleases? They are small circular, extra chromosomal DNA
They act as "molecular scissors". These enzymes recog- present in bacteria. They are used as vectors in DNA
nise specific sequences in the DNA, and then cleave recombinant technology.
at those sites. They are useful in recombinant DNA 43-4. What are the vectors used for gene therapy?
technology. Retrovirus; Adenovirus; Plasmid, liposome complex.
43-2. What are required for preparing a recombinant 43-5. Name some diseases in which gene therapy is
DNA molecule? used successfully?
Restriction endonuclease; Plasmid vector; DNA ligase. Severe combined immunodeficiency; Duchenne mus-
43-3. What are plasmids? cular dystrophy; cystic fibrosis; hemophilia
_ _ _ _ _Chapter 44
Molecular Diagnostics and
Genetic Techniques

Chapter at a Glance
The learner will be able to answer questions on the following topics:
D DNA hybridization techniques Restriction fragment length polymorphism
D Southern, Northern and Western blots D Polymerase chai n reaction (PCR)
D Animal cloning D Monoclonal antibodies
D Molecular cloning DNA sequencing

Molecular (DNA based) diagnostics is rapidly becoming Nick-translation. Non-radioactive probes are also
a standard laboratory procedure for a large number of available and fluorescent probes are easier to detect.
disorders. Many DNA based molecular techniques are
being used in clinical practice. In addition to the diag-
Southern Blot Technique
nosis of a particular patient, molecular diagnostics can It is based on the specific base pairing properties of
be used for diagnosis of index cases, prenatal diagnosis complementary nucleic acid strands. This technique is
and screening. In many cases, the diagnosis is based therefore based on DNA hybridization (Fig. 44. 1). The
on detecting mutations in genes involved, but in some blot technique was developed by EM Southern in 1975.
cases it may be to detect specific OMA sequences as This is used to detect a specific segment of DNA in the
in infectious diseases. In this chapter, a brief review of whole genome.
major molecular techniques employed in clinical diagno- DNA is isolated from the tissue. It is then fragmented
sis is given. by restriction endonucleases. The cut pieces are

I HYBRIDIZATION AND
b LOT TECHNIQUES 3'
Double stranded DNA

Probes DNA denatured and


strands separated by
A probe is defined as a single stranded piece of DNA, heat or alkali
labeled (either with radioisotope or with non-radioactive
label), the nucleotide sequence of which is complemen- Add probe (radio-
labeled DNA)
tary to the target DNA. The DNA of the specific gene
Parent DNA and probe
is used for the hybridization techniques. The DNA is
5' hybridized, if
nicked (a few breaks are made) and repaired. During 3' , complementary
3
this process, 32P-labeled dCTP is added. Therefore, sequences available

the radioactivity is tagged into the gene. This is called Fig. 44.1: DNA-DNA hybridization
Chapter 44: Molecular Diagnostics and Genetic Techniques 609

2 Southern Northern Western

--
1. DNA cut with
(for DNA) (for RNA) (for protein)

II
restriction enzymes;
DNA cutpieces or
Electrophoresis
--+ on agar gel
RNA or proteins
placed in the well
and electrophoresed
2. DNA fragments

3
blotted on nitrocellulose
membrane

3. DNA probe added to


-- Transfer to
nitrocellulose
membrane
membrane
I I

--
4. Bands visualized DNA* cDNA * Antibody* Radioactive ( • )
by autoradiography • • • probe added

Fig. 44.2: Southern blot technique Autoradiograph

electrophoresed on agarose gel. Treatment with Na OH


Fig. 44.3: Comparison of blot transfer techniques
denatures the DNA, so that the pieces become single-
stranded. This is then blotted (adsorbed) over to a
Western Blot Analysis for Proteins
nitrocellulose membrane. The single-stranded DNA is
adsorbed on the nitrocellulose membrane. An exact In this technique, proteins (not nucleic acids) are identi-
replica of the pattern in the gel is reproduced on the fied. The proteins are isolated from the tissue and electro-
membrane (Fig. 44.2). The DNA is then fixed on the phoresis is done. The separated proteins are then trans-
membrane by baking at 80°C. There will be many DNA ferred on to a nitrocellulose membrane. After fixation,
fragments on the membrane, but only one or two pieces it is probed with radioactive antibody and autoradio-
contain the target DNA. The radioactive DNA probe graphed. Alternately, the specific antibody is poured
is placed over the membrane. If the target genes are over, washed and a second antibody carrying horse
present in the host DNA, the probe will detect the com- radish peroxidase is added . Hydrogen peroxide and a
chromogen are layered (Figs 44.3). This technique is
plementary nucleotide sequence in the host DNA (Fig.
very useful to identify the specific protein in a tissue,
44.2). So the probe is hybridized to the particular pieces
thereby showing the expression of a particular gene.
of host DNA. The membrane is then thoroughly washed
Southern, Northern and Western blots are compared in
to remove excess probes. An X-ray plate is placed over
Figure 44.3.
the membrane in the dark for a few days. The radia-
tion from the fi xed probe will produce its mark on the
In Situ Hybridization
X-ray plate. This is called autoradiography. Mutant
genes such as HbS, cystic fibrosis, DMD, PKU as well It allows one to examine the tissue first by microscope.
as presence of viral DNA {hepatitis virus B and C) can It is a modified version of DNA-DNA hybridization. If a
be identified by this method. metaphase spread chromosome preparation is probed
with a gene, the location of the gene on a specific chro-
mosome can be identified. The principle may be applied
Northern Blotting for Identifying RNA
to histology slide also. In the tissue preparation, DNA
The Northern blot is used to demonstrate specific RNA. is denatured, the specific probe tagged with fluorescent
The total RNA is isolated from the cell, electrophoresed labels, incubated, washed and seen under a fluorescent
and then blotted on to a membrane. This is then probed (UV) microscope. The process is named as Fluores-
with radioactive cDNA. There will be RNA-DNA hybridi- cence in situ hybridization (FISH).
zation. This is used to detect the gene expression in a Examples of applications of ISH and FISH are:
tissue (Fig. 44.3). (a) assessment of gene rearrangements in leukemia
610 Section E: Molecular Biology

~--$
(b) diagnosis of B-cell lymphoma by demonstration of Prepare cDNA probe Prepare microarray chip
reduced light-chain mRNA (c) determination of amplifi- RNA from patient
cation of HER2/neu in breast cancer and (d) diagnosis of
various types of lymphomas.
l l te;~:ue
Microarray Technique
A microarray is a series of thousands of microscopic
spots of DNA oligonucleotides (probes) on a slide that
~ =--
are used to hybridize a particular cDNA or cRNA sample Label~
fluorescent
(target). Microarray generally contains 5,000 to 20,000
genes in a gene chip. Each gene (DNA probe) solution
dye +
Hybndae +----
is prepared, a nanoliter sized drop is added to one well
in the plate; the droplet dries in a few seconds, leav- Fig. 44.4: Microarray technology
ing the DNA in the well; thousands of such wells, each
containing a different gene is prepared. The DNA from A single adult cell contains all the genetic material
the clinical sample (or PCR amplified DNA) is tagged for making the entire animal. In the case of Dolly, it was
with a fluorescent dye. These tagged DNA fragments a cell from the udder (mammary cell).
are then incubated with the chip. After the DNA-DNA Next, the mammary cell and denucleated oocyte
hybridization is over, unbound DNA is washed away. were fused. The fused cell was then implanted into the
The surfa ce of the microarray is then scanned with a uterus of a surrogate mother, which delivered the baby
laser beam. Color intensity indicates the extent of hybridi- in course of time.
zation (Fig. 44.4). In a similar manner, specific proteins
can be identified using monoclonal antibody microarray. Applications of Cloning of
Microarray tests are useful in diagnosis of (a) Cancer; Animals and Plants
(b) Infectious diseases and (c) Allergy. 1. Animals with genetically desirable traits could be
bred more efficiently, e.g. cows yielding more milk.
Animal Cloning 2. Biopharmaceuticals: By November 1998, the first
goats were born , who were genetically engineered
The term cloning has two broad meanings. When a gene
to produce milk containing antithrombin Ill. Any
of higher organism is introduced into a bacterial DNA, it
human protein could be introduced into the make up
is called "cloning of the gene" or "molecular cloning";
of goat or cow and get the desired protein cheaply
details of which are described in Chapter 43. When a
through milk. Eggs have been genetically manipu-
cell from an animal is grown to an exact duplicate of
lated to produce interferon and insulin in the egg-
that animal, it is known as "cloning of an animal" or
white.
"somatic cloning".
3. Cloning is successfully employed in agriculture, to
It made big news when Ian Wilmut and Keith Camp-
propagate plants such as rubber, banana, orchids,
bell of Scotland cloned a sheep named "Dolly" in July
etc. If a good yielding rubber is available, it is cloned
1996. (Dolly died naturally in 2002, not due to any com-
so that, thousands of progenies of the same quality
plications of cloning). Today the sheep, tomorrow it could
could be produced within a short time.
be the shepherd. So this raised a number of moral, ethi-
cal and legal issues.
Disadvantages of Cloning
Cloning will never replace selective breeding. Cloning
Dolly and Bonnie. Roslln Institute, Scotland halts any further progress. Cloning can produce the ani-
reported that the first cloned sheep "Dolly" mals/plants with the same characteristics; new charac-
was born in 1996. Her first lamb, Bonnie,
teristics could not be developed. The cloned animal and
was born in 1999. Dolly died in 2002.
parent need not be exactly identical. First, mitochondrial
DNA invariably comes from the egg. Second, DNA in
Chapter 44: Molecular Diagnostics and Genetic Techniques 611

an adult cell differs from the DNA in a fetal cell by the


accumulated damages of a life-time. Thirdly, any animal
is not just the product of its genes, but also of its environ-
s·I 1.15 kbp I I 0.2
3• Two Mst-11 sites on
gene of HbA

One Mst-11 site is


ment, both in utero and after birth; this is especially so s·I 1.35 kbp
13• abolished in HbS
when higher organisms are concerned.
Southern blot

Applications of Molecular 1.35 kbp fragment

Cloning in Medicine
Diagnosis of Genetic Diseases .__ __________ _,1
1.15 kbp fragment

AA AS ss Genotype
Various genetic diseases can be identified by using Normal HbS trait HbS disease Phenotype
appropriate probes from defective genes. A point muta-
Fig. 44.5: Southern blot analysis of DNA from normal. HbS trait
tion may destroy or create a restriction enzyme cleavage and s ickle cell anemia disease
site. Then the fragment size produced from normal gene
and mutated gene will be different. This can be easily or individuals in which the disease has not yet been
identified by Southern blot. manifested.
For example, sickle cell anemia is caused by a point Such a test is possible for prenatal diagnosis also.
mutation. In the beta chain of the hemoglobin, the 6th DNA from cells collected from amniotic fluid can be used
amino acid normally is glutamate. In sickle-cell anemia, for Southern blot analysis.
this is altered to valine. In the DNA, this is seen as a Duchenne muscular dystrophy (DMD) is a degen-
change from normal T replaced by abnormal A nucleo- erative disease of muscle affecting only male children.
tide (T to A substitution). The normal gene for HbA has The gene is in the X chromosome. This gene produces
the following nucleotide sequence. a protein called dystrophin with 3700 amino acids. It
CC t TGA GG is one of the largest human genes known. In DMD pa-
Coding strand tients, the gene for dystrophin is mutated. This could be
GG AC(!) t CC identified by using a cDNA probe for dystrophin. In the
Template strand Southern blot analysis, the fragment corresponding to
The arrows indicate the cleaving site for the restric- this gene will be absent.
tion enzyme Mst-I1. In the sickle cell anemia gene, the
underlined T is replaced by A, and the DNA sequence of DNA Finger Printing in
the HbS gene is as follows: Forensic Medicine
CC TGT GG There are tandem repeats (TR) in chromosomes. These
Coding strand are short sequences of DNA, located at scattered sites.
GG AC@ CC The number of these repeat units varies from person to
Template strand person, but is unique for a particular person. Therefore,
This alteration in base sequence abolishes the recog- it serves as a molecular fingerprint. It is also known as
nition site for this RE. But other Mst-II cleavage sites are ONA profile. Probability of similarity between two per-
preserved. sons is only 1 in 3 x 1010 persons. The technique is used
T herefore, analysis of DNA from AA (normal HbA), to pinpoint the culprit of the crime, and also in disputes
heterozygous AS (Sickle-cell trait) and homozygous SS of parenthood. DNA can be isolated from stains on
(Sickle-cell disease) individuals fall into 3 different pat- clothing made of blood.
terns in Southern blot technique (Fig. 44.5). The muta-
Restriction Fragment Length
tion eliminates one restriction site for Mst 11 enzyme, and
hence a larger fragment is present in sickle cell anemia.
Polymorphism
This becomes useful as a diagnostic test for the The human genome contains hundreds of variations in
presence of the disease allele in heterozygotes (carriers) base sequences that do not affect the phenotype. The
612 Section E: Molecular Biology

property of the molecules to exist in more than one form is


known as polymorphism. ,.--;
Region to be amplified
DNA strand
Difference between mutation and DNA polymor-
Melt DNA! at 95' C and add primers
phism: If more than 1% of the population has a par-
cool to 50°C to Cycle 1 =
ticular alteration in the sequence, it is polymorphism. If Primer Ill- - . Primer anneal primers 4 strands
only a few individuals have it, then it is mutation. Poly-
morphism is normal variation, and generally having no l Add dNTPs and
Taq polymerase at 72•c

deleterious effect. Mutation is abnormal , and sometimes


will have defective function, e.g. phenylketonuria.
A polymorphic gene is one, in which the variant Melt DNA! at 95°C and add primers

alleles are common in more than 1% of the total popula- Old Strand

- -
ID
tion. The existence of two or more types of restriction c1::c:
1 ===!-~ ~> New Strand
New Strand
fragment patterns is called restriction fragment length < I I I
Old Strand
polymorphism (RFLP). This can be used as a genetic Cycle 2 =
I Add dNTPs and
marker. +Taq polymerase at n •c 8 strands

DNA is treated with restriction enzymes, which


cleave DNA into fragments of defined lengths. Then
electrophoresis is done in agarose gels, when the frag-
ments are separated. Finally, the DNA from the agarose
gel is transferred on to nitrocellulose paper (Southern
blotting) and hybridized with labeled probe sequences.
Genotypic changes can be recognized by the altered Melt DN a j idd µrmers and cool

i
restriction fragments. to '>O rt' nQ Ado primers dNTPs, Cycle 3 =
l m, I l, 16 strands

Clinical Applications of RFLP


1. RFLP is also useful in human population genetics,
geographical isolates and comparison of genetic
make up of related species.
Cycle 5 = Cycle 4 =
Kary B 32 strands +-- 16 strands
2. Genetic diseases will produce alteration in size distri- Mullis
NP 1993
bution of RE fragments, and show RFLP; examples b. 1944
Cycle 6 = +-- and so on
are shown in Figure 44.5. 64 strands

I POLYMERASE CHAIN REACTION Fig. 44.6: Polymerase chain reaction (PCR)

Karry Mullis invented this ingenious method in 1989, who Step 2: Priming (Annealing): The primers are
was awarded Nobel Prize in 1993. Polymerase chain re- annealed by cooling to 50°C for 0.5 to 2 minutes. The
action (PCR) is an in vitro DNA amplification procedure primers hybridize with their complementary single stran-
in which millions of copies of a particular sequence of ded DNA produced in the first step.
DNA can be produced within a few hours. It is like xerox Step 3: Extension: New DNA strands are synthe-
machine for gene copying. sized by Taq polymerase. This enzyme is derived from
The flanking sequences of the gene of interest bacteria Thermus aquaticus that are found in hot springs.
should be known. Two DNA primers of about 20- 30 Therefore the enzyme is not denatured at high tempera-
nucleotides with complementary sequence of the flank- ture. The polymerase reaction is allowed to take place
ing region can be synthesized. The reaction cycle has at ?2°C for 30 seconds in presence of dNTPs (all four
the following steps: deoxy ribonucleotide triphosphates). Both strands of
Step 1: Separation (Denaturation): DNA strands DNA are now duplicated (Fig. 44.6).
are separated (melted} by heating at 95°C for 15 sec- Step 4: The steps of 1, 2 and 3 are repeated. In each
onds to 2 minutes (Fig. 44.6). cycle, the DNA strands are doubled. Thus 20 cycles
Chapter 44: Molecular Diagnostics and Genetic Techniques 613

provide for 1 million times amplifications. These cycles BOX 44.1: Apphcat1ons of PCR
are generally repeated by automated instrument, called Detection of infectious diseases
Thermal cycler. AIDS, tuberculosis, CMV, H1 N1 , hepatitis C, etc.
Step 5: After the amplification procedure, DNA Detect 3 sexually transmitted diseases in one swab- herpes,
papilloma virus, Chlamydia.
hybridization technique or Southern blot analysis with a
PCR can diagnosis even one bacteria or virus present in the
suitable probe, shows the presence of the DNA in the specimen.
sample tissue. Latent viruses can also be diagnosed.
Detection of variations and mutations in genes
Detects people with inherited disorders and carriers
. Clinical Applications of PCR Track presence o r absence of DNA abnormalities characteristic of
cancer
1. Diagnosis of bacterial and viral diseases: In early
Prenatal diagnosis of genetic disorders.
phases of tuberculosis, the sputum may contain PCR combined with RE cleavage and Southern blot t ing is used for
only very few tubercle bacilli, so that usual acid mutation detection.
fast staining may be negative. But PCR can detect PCR and the law
DNA fingerprinting-can multiply small amounts of DNA found in
even one bacillus present in the specimen. Any other blood samples, hair, semen, and other body fluids
bacterial infection can also be detected similarly.
The specific nucleotide sequences of the bacilli are
cancer (oncogenes and oncosuppressor genes are
amplified by PCR and then detected by Southern
described in Chapter 48). See also Box 44.1.
blot analysis. If reverse PCR is done, living organ-
isms can be detected. This technique is widely used Reverse Transcriptase PCR (RT-PCR)
in the diagnosis of viral infections like hepatitis C, It is the method used to amplify, isolate or identify a
cytomegalovirus and HIV. known sequence from a cell or tissue RNA library. The
2. Medicolegal cases: PCR allows the DNA from PCR is preceded by reverse transcription (to convert the
a hair follicle or a blood cell to be analyzed.The RNA to cDNA). This is widely used to determine when
restriction analysis of DNA from the hair follicle from and where certain genes are expressed. Instead of Taq
the crime scene is studied after PCR amplification. polymerase described above, Tth polymerase from Ther-
This pattern is then compared with the restriction mus thermophilus may be used. This enzyme has both
analysis of DNA samples obtained from various DNA polymerase and reverse transcriptase activities at
suspects; the culprit's sample will perfectly match high temperature. This allows both cDNA synthesis from
with that of PCR amplified sample. The restriction mRNA followed by PCR amplification . In ordinary PCR,
analysis pattern of DNA of one individual will be very DNA is detected; that DNA could be from a living or non-
specific (DNA fingerprinting); but the pattern will be living organism. But in reverse PCR, mRNA is detected;
different from person to person. This is highly useful that means, it is derived from a living organism. Pres-
in forensic medicine to identify the criminal. ence of HIV RNA in blood can be detected as early as 4
3. Diagnosis of genetic disorders: The PCR techno- weeks after infection.
logy has been widely used to amplify the gene seg-
ments that contain known mutations for diagnosis Real-time PCR
of inherited diseases such as sickle cell anemia, By this method , quantitation of the number of virus pre-
beta thalassemia, cystic fibrosis, etc. sent in a sample can be calculated, e.g. viral load in HIV
4. PCR is especially useful for prenatal diagnosis of or HBV. So, the treatment modalities can be planned
inherited diseases, where cells obtained from fetus and th e response to treatment can be assessed.
by amniocentesis are very few.
5. Cancer detection: PCR is widely used to monitor Multiplex-PCR
residual abnormal cells present in treated patients. By targeting multiple genes at once, additional informa-
Similarly identification of mutations in oncosup- tion may be elicited from a single test run that otherwise
pressor genes such as p53, retinoblastoma gene, would require several times the reagents and time to
etc. can help to identify individuals at high risk of perform.
614 Section E: Molecular Biology

Spleen cells

Aot q"
HGPRT +ve
HAT resistant
lg secretion +ve • Myeloma cell
HGPRT-ve
1• • 1 1 • 1 HAT sensitive

....
lg secretion -ve
/ Proliferation +ve

Immune spleen cells


, i ,
Fusion by PEG
Hybridoma produced

Only fused cells will


Antigen with 3 different In hybridoma technique, only Test supernatant grow in HAT medium
epitopes; when injected, animal one type of antibody is for antibody
produces 3 different types of produced (monoclonal)
antibodies (polyclonal)
0 0 0 0
0 0 0
Fig. 44. 7: Polyclonal and monoclonal antibodies Expand positive clones
lee• ••I
in vitro /p ~ t
Hybridoma Technology and V
c. ••..)-
Propaga e • - - • •
Monoclonal Antibodies
Differences between Monoclonal
and Polyclonal Antibodies

Fig. 44.8: Principle of production of monoclonal antibodies by
hybridoma technique. Myeloma cells propagated in mice ascites.
If an antigen is injected into an animal, the animal pro- Immunize normal mice with specific antigen. Immune spleen cells
duces different types of antibodies against various and myeloma cells are fused with the help of PEG-1500. The cells
are incubated in HAT medium. Unfused lymphocytes will die as
epitopes of the antigen. The antibodies thus generated they do not have the proliferation capacity. Unfused myeloma cells
are polyclonal in nature. Different molecules will have lack in HGPRT gene, so alternate pathway for ONA synthesis is
different specificities and affinities (Fig. 44.7). In all micro- not available. The main pathway for DNA synthesis is blocked
by aminopterine in the HAT medium. So, unfused myeloma cells
bial infections, body responds with polyclonal antibody die. Only fused cells can grow in HAT medium. Some of the wells
production . contain the hybridoma cells secreting the specified antibody. The
However, in nature, monoclonal antibodies are pro- supernatant in each well is tested for the presence of antibody.
Those wells containing the antibody, and selected further. The
duced in multiple myeloma where only one clone secretes useful hybridoma cells can be cultured in vitro in culture flasks
a particular type of antibody (see Chapter 26). But in or in vivo as mice ascitic fluid. Indefinite amount of monoclonal
multiple myeloma, the antibody thus generated is antibodies could be harvested

unwanted.
In the laboratory, monoclonal antibody can be myeloma cells are defective in the enzyme HGPRTase
generated. In this case, only a particular type of antibody and so they lack the salvage pathway for DNA synthesis.
against a specific epitope of the antigen is produced In the culture, HAT medium is used containing
(Fig. 44.7). Monoclonal antibodies were fi rst produced hypoxanthine, aminopterin and thymidine. The amino-
by Georges Kohler and Cesar Milstein in 1975. They pterin, a folic acid antagonist, will inhibit the de novo
were awarded Nobel Prize in 1984. synthesis of DNA. Since both pathways are blocked, the
non-fused myeloma cells also die in the special medium
Production of Hybridoma provided. The only cells that survive are the cells where
The antigen is injected into mice. Spleen cells from the fusion has taken place between normal spleen cells with
immunized mice are fused with mice myeloma cells, so myeloma cells. In this case, normal cells provide the
as to produce a hybrid cell. The hybrid cells now con- HGPRTase enzyme and so DNA synthesis is possible
tain the gene of normal mice as well as the myeloma from the hypoxanthine and thymidine provided in the
cells. (Fig. 44.8). However, hybridization might have medium. The normal cellular genes also provide the
occurred between two normal cells. Normal cells lack the information for specific antibody synthesis. The myeloma
multiplication potential. So all the hybridized normal cells cancer genes provide the endless multiplication drive,
die in the usual culture conditions within 5-6 days. The so that hybrid cells are immortalized (Fig. 44.8).
.:t.
,,.. ,.~ , Chapter 44: Molecular Diagnostics and Genetic Techniques 615

'ti
·-r Table 44.1: Common therapeutic uses of monoclonal antibodies

. , ·.· ,·- ·,; . A


•~ • ·, . I
I

• • 1
,a,-
-

•~
~ Indication
Transplantation rejection
Non-Hodgkin's lymphoma
Target
CD3
CD20

Georges Cesar
&.~
Walter
- · -
Frederick
Rheumatoid arthritis
Metastatic breast cancer
TNF -a
HER-2
Kohler Milstein Gilbert Sanger
NP 1984 NP 1984 NP 1980 NP1958 and
When growth hormone gene is introduced, the ani-
1946-1995 1927-2002 b.1932 1980
1918-2013 mals became twice as large as their normal counterparts.
The potential applications of these findings are many.
Applications of Monoclonal Antibody Attempts are already under way to increase milk pro-
Monoclonal antibodies are used for immunosuppres- duction in cows by transgenic method . Production of
sion (Basiliximab and Daclizumab), for treatment of biopharmaceuticals, described under the heading appli-
autoimmune diseases (Etanercept, lnfliximab, Adali- cation of cloning of animals, is another example of
mumab), and as anticancer drugs. These are shown transgenesis.
in Chapters 46 and 48. Common therapeutic uses of mono-
DNA Sequencing
clonal antibodies are shown in Table 44.1.
In 1977, two sequencing techniques have been develo-
Advantages of Monoclonal Antibody ped. Sanger's technique is called "controlled termination
In a monoclonal preparation, all the antibody molecules of synthesis"; it uses chain terminating agents. Gilbert
are specific against a particular antigen. Therefore, developed the base-specific chemical cleavage method.
more and more monoclonal antibodies are now com- Both were awarded Nobel Prize in 1980.
mercially produced. These are highly useful to detect (a) Sanger's technique is summarized below: Suppose
serum proteins, (b) enzymes, (c) hormones, (e) drugs, the sequence of the polynucleotide is 3'AACTCGAG-
(f) bacterial antigens, (g) viral antigens, (h) cell surface TA5'. This DNA sample is taken in 4 different test tubes
receptors, (i) HLA antigens and U) cancer antigens. (Fig. 44.9). In all tubes, the Kienow enzyme (DNA poly-
merase without exonuclease activity) and radiolabeled
Single Nucleotide Polymorphism (SNP) TT as the primer are added. In all tubes radioactive
SNP is a change in a single nucleotide, which can pro- dNTPs (all the 4 nucleotides which are labeled with 32P)
are added. Synthesis of a new strand of DNA having a
duce change in one amino acid in a protein by altering
sequence of 5' TTGAGCTCAT3' is started. But in the
single codon. The SNP may not produce clinical symp-
first test tube, ddTTP (2',3'-di deoxy TTP) is also added.
toms. The person will show only an abnormal genotype
The ddTTP will add the T, but it cannot form the next
which is detected by DNA analysis. SNPs are detected
phosphodiester link, and so further chain lengthening is
by single strand conformation polymorphism.
stopped. In other words, ddTTP will stop chains at T.
There are many techniques now available for detec-
Instead of ddTTP, the polymerase might add a normal
tion of mutations. Some of the most commonly employed
dTTP, in which case chain growth will continue till the
methods are Single Strand Conformation Polymor-
next T. Thus in first test tube, TTGAGCT (7 nucleotides)
phism and Conformation Sensitive Gel Electrophoresis and TTGAGCTCAT (10 nucleotides) are produced.
(Table 44.1 ).
In the second test tube, ddATP is added, so chain
length is stopped at A. So in the 2nd tube, DNA strands
Transgenesis
having sequences of TTGA (4 nucleotides) and TT-
It is a form of germ cell gene therapy. A recombinant GAGCTCA (9 nucleotides) are produced. Similarly, in
DNA segment, containing the desired gene from another the 3rd test tube, ddCTP is added, which will contain
species, is introduced into the fertilized ova. The embryos TTGAGC (6 nucleotides) and TTGAGCTC (8 nucleo-
are allowed to develop in the uterus of another tides). In the 4th tube containing ddGTP will have TTG
animal. The animals born are called transgenic animals. (3 nucleotide) and TTGAG (5 nucleotides).
616 Section E: Molecular Biology

-- -
1st tube 2nd tube 3rd tube 4th tube Length Sequence of Table 44.2: Techniques and their apphcat1ons
ddTTP ddATP ddCTP ddGTP of new new strand
strand Technique Applications
10 Southern blot Specific DNA segment (sequence) detection

----
ti)

~g
9
8 li1 a ro Northern blot Detection of a specific gene expression

7 -g. g: Western blot Identificatio n of a specific protein


&I Q:::,

-
m ~--
6 ti) 0 In situ hybridi- Location of a gene on a specific chromosome
5 zation

m
4 DNA microarray Presence and expression of various genes
3
Polymerase chain In vitro amplicat ion of a DNA sequence; PCR
Fig. 44.9: DNA sequencing reaction can detect even a single bacterium or virus
Single strand Detection of mutations in small fragment s
conformation
Then the contents of each tube are simultaneously
polymorphism
examined on polyacrylamide electrophoresis. The posi-
Conformation Detection of single base mutations in
tion of these fragments will correspond to the chain sensitive gel longer fragments
length; e.g. chain with 10 nucleotides will move least, electrophoresis I
while with 3 nucleotide will move maximum and other
nucleotides will be arranged in their order of molecular the extracellular matrix. Dystrophin and its isoforms are
size. The gel is then autoradiographed. The radiation found in skeletal muscles, smooth muscles and brain.
from 32P labelled primer will be available in all pieces Patients usually present in the 3rd- 5th year of life
and will be seen as dark bands in the X-ray plate (Fig. with motor delay or abnormal gait, difficulty in running,
44.9). From the picture, it can be inferred that 7th and getting up from ground, frequent falls, etc. There may be
10th bases are T. Similarly the positions of other bases pseudohypertrophy of calf muscles (fibro-fatty replace-
can also be deciphered. Thus the sequence of the newly ment of degenerating muscle tissue). Cardiac defects
synthesized strand is known. The complementary include cardiomyopathy and rhythm changes. Respira-
sequence will be present in the original unknown DNA. tory involvement by 16-18 years include scoliosis (which
Applications of the above mentioned techniques are impairs pulmonary functions), respiratory failure (due to
summarized in Table 44.2. progressive failure of intercostals muscles), hypercap-
nia and severe respiratory infections.
·• · Clinical Case Study 44.1 Laboratory analysis reveals elevated serum CK
(10-100 times above normal), AST, ALT and LOH. Muscle
A 4-year-old boy was brought to the hospital. His mother
biopsy (shows deficient dystrophin), electromyography
was concerned that he was walking in an awkward
and PCR can be used for confirmatory diagnosis.
manner, fell over frequently, and had difficulty in climb-
ing stairs. Family history revealed a maternal uncle who
died due to muscular dystrophy at the age of 19. Clini-
ILEARNING POINTS, CHAPTER 4_!_
cal examination showed muscle weakness in shoulder 1. A probe is a single stranded piece of DNA, labeled
with a radioactive or a non-radioactive label, the
and pelvic girdles and enlargement of calf muscles.
sequence of which of complementary to the target
A tentative diagnosis of Duchenne muscular dystrophy
DNA.
was made. What is the defect in this condition?
2. Southern blot is a technique used to detect a speci-
fic segment of a DNA. It has diagnostic and forensic
•i i• Clinical Case Study 44.1 Answer applications.
3. Northern blot is used to detect a specific RNA.
Duchenne muscular dystrophy (DMD) is due to muta-
4. Western blot is used to detect a specific protein .
tions in DMD gene. The gene encodes a protein known
5. Molecular cloning in medicine can be used for diag-
as dystrophin. All muscular dystrophies are progressive nosis of genetic diseases such as Duchenne Mus-
in nature and many are genetically inherited. Dystrophin cular Dystrophy (OMO), Sickle cell anemia, etc.
acts as a link between actin in the cytoskeleton and the 6. Polymerase chain reaction (PCR) is an in vitro DNA
extracellular matrix and helps anchor the muscle fiber to amplification procedure in which millions of copies
Chapter 44: Molecular Diagnostics and Genetic Techniques 617

of a particular sequence of DNA can be produced 8. Hybridoma technique is used to generate mono-
within hours. clonal antibodies. They are used in immunological
7. The enzyme used in PCR is Taq polymerase Appli- research, ELISA, nephelometry, etc.
cations of PCR include diagnosis of bacterial and 9. DNA microarray is used for identification of the
viral diseases, genetic disorders, cancers, solving presence and expression of various genes.
medicolegal cases. 10. Polymerase chain reaction is used for in vitro ampli-
fication of a DNA sequence.

PART-1: SHORT NOTE QUESTIONS


44-1. Southern blotting.
44-2. Western blot technique.
44-3. Northern blotting.
44-4. In-situ hybridization.
44.5. DNA finger printing.
44-6. Restriction fragment length polymorphism (RFLP).
44-7. Polymerase chain reaction.
44-8. Monoclonal antibodies.
44-9. Hybridoma technology.

PART-2: MULTIPLE CHOICE QUESTIONS

44-1. Specific DNA can be identified by all the following 44-7. All the following are requisites for PCR, except.
techniques, except A. DNA primer

A. Southern blotting B. In situ hybridization B. Taq polymerase


C. Restriction endonuclease
C. Western blotting D. Northern blotting
44-2. Southern blot technique involves all, except D. Deoxynucleotide triphosphates
44-8. Gene amplification can be achieved by:
A. Cleavage of DNA by restriction endonuclease
A. Southern blotting
B. Addition of radiolabelled antibody for detection
B. DNA finger printing
C. Electrophoresis of fragments
C. Polymerase chain reaction
D. Blot on nitro-cellulose sheet D. Gene cloning
44-3. A radiolabelled DNA probe is usually employed for 44-9. Which of the following techniques is NOT useful in
all the following processes, except prenatal diagnosis?
A. Southern blotting B. Northern blotting A. DNA probing B. Linkage analysis
C. DNA hybridization D. Western blotting C. DNA finger printing D. DNA looping
44-4. RFLP (Restriction fragment length polymorphism) 44-10. Which is involved in renaturation of DNA?
is used to: A. Excision B. Splicing
A. Identify a specific gene in bacteria C. Annealing D. Priming
44-11 . PCR is used for the following except
B. Locate mutations in DNA
A. Diagnosis of tuberculosis
C. Study the rate of transcription
B. Establishment of biological parentage
D. To amplify genes
C. Detection of oncogenes
44-5. Which exhibits RFLP?
D. Detection of anti nuclear antibodies
A. Haptoglobin B. Insulin 44-12. Which cannot detect specific DNA sequences?
C. Beta globin chain D. Myoglobin A. FISH
44-6. Which technique has medicolegal application? B. ELISA
A_ DNA hybridization B. DNA fingerprinting C. Dot blot hybrdization
C. DNA denaturation D. DNA transposition D. Autoradiography
618 Section E: Molecular Biology

ANSWERS OF MULTIPLE CHOICE QUESTIONS


44-1 . C 44-2. B 44-3. D 44-4. B 44-5. A 44-6. B 44-7. C

44-8. C 44-9. D 44-10. C 44-11 . D 44-12. B

PART-3: VIVA VOCE QUESTIONS AND ANSWERS T

44-1 . What is the use of Southern blotting? 44-6. What is the application of DNA fingerprinting?
To identify abnormal genes, to demonstrate virus inte- It has medicolegal application.
gration. A lso used for prenatal d iagnosis. 44-7. Which enzyme is required for PCR (polymerase
44-2. What Is Northern blotting? chain reaction)?
The Northern blot is used to demonstrate specific RNA . Taq polymerase.
44-3. What is Western blotting? 44-8. What is the use of PCR?
This is to identify proteins (not nucleic acids). (Fig. 44.3).
Diagnosis of bacterial and viral diseases; Medicolegal
44-4. What Is meant by DNA probes?
cases; Diagnosis of genetic disorders, especially pre-
They are single stranded DNA , labeled with radio-
natal diagnosis. Only very minute quantity of sample is
activity. They are used to identify specific bases seq-
required.
uences (genes, virus etc) in the cellular DNA material.
44-9. How are monoclonal antibodies produced in the
44-5. RFLP (restriction fragment length polymorphism)
laboratory?
is used for what?
Locating mutations in DNA. By means of hybridoma technology.
SECTION F

Advanced Biochemistry

Chapter 45 Hormones and Growth Factors


Chapter 46 lmmunochemistry
Chapter 47 Biochemistry of AIDS and HIV
Chapter.48 Biochemistry of Cancer
Chapter 49 Tissue Proteins in Health and Disease
Chapter 50 Applications of Isotopes in Medicine
e - - - - - - - -_ _ _ _ Cha pter 45
Hormones and
Growth Factors

Chapter at a Glance
The learner will be able to answer questions on the following topics:
Signal transduction Biological effects of glucocorticoids
Cyclic AMP and G-proteins Adrenal hyper and hypofunction
Cyclic GMP Ovarian hormones
Hormone response element Testicular hormones
Antidiuretic hormone Synthesis of thyroxine
Oxytocin Metabolic effect of thyroid hormones
Hypothalamic releasing factors Assessment of thyroid function
Growth hormone Hyperthyroidism
Adrenocorticotropic hormone Hypothyroidism
Thyroid stimulating hormone Gut hormones
Gonadotropins Growth factors
Synthesis of steroid hormones

...r.,..). -,.cu.C\J"y s~
The nervous system and endocrioe___system are the
major control mechanisms that integrate the functions of
the tissues in the body. The nervous system transmits
electrochemical signals between the brain and periphe-
ral tissues for coordinatin the diverse body functions.
The endocrine system releases chemical
-.::
me.Qialor.s_or
Earl Alfred Martin
hormones into the circulation. However, b th these sys-
Sutherland Gilman Rodbe/1
t; ms n ~errjf. t~am u~ 10n of endocrine NP 1971 NP 1994 NP 1994
glands is arfecterl. oreover, neurotransmitters have 1915-1974 b. 1941 1925-1998
RvJ'I r'1
sev~: r~;es in ~common 1th hormones.
· e signaling occurs when ame cell acts as mesengers secreted into circulation . Once they reach
sender and reci ie t, e.g. gro~ /1, diffe~ ntiation, immune the target cell, they bind to specific target cell receptors
and inflammatory response. P~rac ne signaling is effec- with high affinity.
ted by local mediators which have their effect o.e.ar.Jl!e The classical definition of a hormone is "substances
site of secretion without enterin the circulation. The released from ductless or endocrine glands directly to the
effect is rapid and transient. Pi signaling is blood". A more modem definition of a hormone is that it is
between cells which are located at a distance from each s nthesized by on(1Jxp__e_of...CEII an transported through
other and the signal may be hormones or chemical @ o act on angth.er fy.Jl!l. of ce 1s. There are hundreds
622 Section F: Advanced Biochemistry

Mechanism of action
Hormones that bind to
intracellular receptors
,
(:,
Robert Brian Kobilka
IIA Hormones bind with cell CTH, ADH, FSH, HCG, LH, Lefkowitz NP 2012
SH, PTH, CRH, Glucagon, NP 2012 b. 1955
Calcitonin, Catecholamines b. 1943

II B Hormones having cell ANF (atrial natriuretic


surface receptors; cGMP fact or), NO (nitric oxide)
as second messenger ':Sf'
IIC Hormones having cell TRH, GnRH, catecholamines,
surface receptors'; second CCK, Gastrin, Vasopressin,
messenger is Qkium or OJ,Ytocin, PDGF
'd~ to result in the i f l ~ ~ffe-et. Different types of G-proteins
phosphatidylinositol PW2l
11D Hormones having cell
are present in the cells.

@ surface receptors and


mediated through ~osi
The extracellular messenger, the hormone (H) com-
bines with the specific receptor (R) on the plasma mem-
kin~
II E Hormones having cell
braf)e (Fig. 45.3). Thelli-R compl~ ctivates the G-pro-
surface receptors, but tein or nucleotide regulatory protein. G-proteir,s are so
intracellular messenger
named, because they can bind GTP and GDP. The G-pro-
is a kinase or y.tlli~
hosp_batase case cle tein is a trimeric membrane protein consisting of alpha,
beta and gamma subunits (Fig. 45.3). Alfred Gilman
ofqJQjy eptid_es that come under this category. These are and Martin Rodbell were awarded Nobel Prize in 1994
~ cal hormones, or si nal molecld!e educed at the local for their work on G-protein. Robert Lefkowitz and Brian
sites, and therefore are generally called a actor~"- Kobilka were awarded Nobel Prize in 2012 for their studies
Based on mechanism of action, the hormonesmay be on G:e_rotein-cou~2,.@.cepto1s (GPCRs). Most of the
classified into two (Table 45.1): hormone/ligand signals are transduced through 0'CR.
Jr Hormones with cell surface receptors ( ~ ,2,1 c_,t>,9 €_ound 50% of medicines are acting through GPCR'.
X Hormones with intracellular receptors.
G-Protein Activates Adenyl Cyclase
~ 1 HORMONES ACTING .?1'1 \ ¢ ~ When the hormone receptor complex is fo rmed, the
THROUGH CYCLIC AMP ~ ~<=1· activated receptor stimulates the G-protein, which carries

~-i\,
the excitation signal to adenylate cyclase (Fig. 45.4-2).
,~ yclic AMP (cAMP) was first discovered by Earl Suther-
The hormone is not assed throu h the Qliilllb.(ane;
Tand in 1961, who was awarded Nobel Prize in 1971. Signal
but only the si nal is Q.g_S_,§ed; hence this mechanism
transduction pathways are like a river flowing in one
is called signal transduction. The a5tepy.Lcycla§e is
directiop on~ ; ~ll1P.?b1~..ntscloser to the receptor are
emged~ in thr; plasma membrane (Fig. 45.4).-
~ ~called "ue~ ',.,anBcioser to the response are called
"(;' ~ "dmID.§tream" ~"6\".'.
Subunit Activation of G-Protein
Signal Transduction through G-Protein The inactive G-protein is a trimer with alpha, beta and
Action is through ~ protein coupled receptors (GPCRi)) gamma subunits. When activated, GTP binds and the
Binding of different types of signal molecules to G-pro- beta-gamma subunits dissociate from the alpha subunit.
tein coupled receptors is a ~ b , w i s m atsignijJ Adenylate cyclase is activated by G-alpha-GTP (Fig.
t~ansductiQ.!1 Action of several hormones is effected 45.4-2). The ~n_9:!.ng__of brumoneJo.Jru rece tor 1_[ggers
through this mechanism (Table 45.1). a confi U@tionalcilang · h - rotein which induces
Chapter 45: Hormones and Growth Factors 623

Adenyl cyclase Phosp odiesterase


GDP-a-py _ _ _ _(_+_) G _ _ _-+ a-GTP + py
_T_P
ATP --"\.,----+• 3',5'-cyclicAMP 5'-AMP
• (cAMP) (+) H20 (inactive (active
PPi G-protein) GTPase G-protein)

Fig. 45.1 : Synthesis, degradation of cyclic AMP Fig. 45.2: Inactive and active forms of, G-protein

Hormone Outside eel


(1st messenger) membrane

- i

Protein kinase Protein kinase

Adenyl cyclase C C
produces cAMP (2nd messenger)
R R
Fig. 45.3: Hormone binding activates G;_P_rotein R R
3. 4.
4 ,Jk....~~~
Protein kinase
..
the rel~ase o ~ s GJE....tQ..jjjnd.
The hormone has an amplified response, since several

t,.
molecules of G-alpha-GTP are formed.

Inactivation_.,
The ac~iv G-alpha-GTP is imme~ ~ ated by R= receptor: G= G-protein with a, 13. y subunits : AC= adenyl
- r ,, ·~ cyclase: H= hormone; C= catalytic unit; R= regulatory unit;
(2TPas . he G-alpha-GDP form is ive (Fig. 45.2).
cAMP = cyclic AMP.
Theacti'vation is switched off when the GTP is hydrolyzed
1= Receptor is attached to G-protein, which has a , 13, y subunits. It
to GDP by the GTPase activity of the alpha subunit (Fig. is bound with GDP, and is inactive. These are membrane bound.
45.2). This is a built-in mechanism for ~ tivation. Thus 2= When hormone attaches, er. subunit detaches, GTP is bound :
@ TPase acts as a molecular switch) 12S:.) Ga-GTP activates adenyl cyclase , cAMP is generated.
3= Protein kinase contains two catalytic units; but these are
Cyclic AMP attached to two regulatory units, and are inactive.
4= cAMP binds with regulatory units; now catalytic units are free;
Adenyl cyclase or adenylate cyclase converts ATP to kinase is now active.
cAMP (3' ,5'-cyclic AMP), and phosphodiesterase hydro- 5= Active protein kinase phosphorylates enzyme proteins.

lyzes cAMP to 5' AMP (Fig. 45.1 ). Structure of cAMP


Fig. 45.4: Action of hormone through G-protein
is shown in Figure 38.7. Table 45.1 contains the list of
hormones mediated through cyclic AMP. Box 45.1 shows
(_gyclic AMP binds to the regu a ory subunit of PKA so that
the hormones which stimulate or inhibit the adenyl
the caJ.§.!yt
.J.g_.s_uhuolKba'lin.g__kioas_a_acfu(il¥ can p=
bo=is=, v
cyclase.(C1_chc AM P Is a second messenger produced rylate proteins. The cascade amplification effect is seen in
injhe cell in response to activation of adenylate cyclas7 this series of activation reactions. This PKA is a tetrameric
by active G-protein. ( ~-ti-(::,;T? ~ n ) molecule having two regulatory_(R) and two catalytic (C)
subunits (R2 C2) (Fig. 45.4-3).( T,his complex has no acJ1-
Second Messenger Activates PKA ~- But cAMP binds to the r ulatozy subunit _and .d~~
The cAMP (second messenger), in turn, activates the c ~ e r into regulatory and catalytic subunits
enzyme, P~ (CY,9Iic AMP d pende,nt protein kinase). (Fig. 45.4-4 ). The Walyt1c subunit is now freetoact.
\.. .?I Cf ~ \f!1"'
624 Section F: Advanced Biochemistry

BOX 45.1 : Hormones acting through adenyl cyclase ATP - - - cAMP


Hormones stimulate adenyl cyclase: ACTH, ADH, Calcitonin,
Protein kinase - - - Protein kinase
CRH, FSH, Glucagon, epinephrine, hCG, LH, LPH, MSH, PTH and TSH. (inactive) (R2 C2) (active) (R2 + C2)
Hormones inhibit adenyl cyclase: Acetylcholine, angiotensin II
and somatostatin. e.nL:,"l-r, "'\ fc'lr-- - - - Phosphorylase kinase
(active; phosphorylated)

- - - + Glycogen phosphorylase
(active; phosphorylated)
(tetramer)
Cholera toxin is present inside the bacteria Vibrio cholerae. The
enterotoxin contains A and B subunits. The B subunit binds to - - - - Glycogen (n- 1 units)
a ganglioside GMt. The A subunit then enters into the mucosal + Glucose-1-phosphate
cell membrane, the alpha subunit of Gs protein is activated.
This results in the irreversible activation of G protein. Therefore, Fig. 45.5: cAMP mediated cascade
t.mr~-e,ag C9VT > t..l\S'

I
adenyl cyclase remains continuously active and keeps cyclic
AMP levels high. This prevents absorption of salts from intestine CALCIUM-BASED SIGNAL
leading to watery diarrhea and loss of water from body. Hyper-
activity of these channels w ill result in loss of sodium chloride TRANSDUCTION '> c~~~~i%qitjJw
w ith watery diarrhea (liquid stools), that may have fatal resu lts.
The patient may lose as much as 1 L of water per hour. Calcium is an impoJSnt intracellular reg1~ ~tor of cell
Pertussis toxin ribosylates the alpha subunit of Gi-protein and
function like contr ;t1'on of muscles secrmlon o or-
prevents the Gi-GDP complex from interacting with the activa- manes and neurot-=-=--~nsmitters, cell J~fsi~n and re ula J~n
ted receptor. Hence, the action of hormones acting through Gi of=9ene regulation. Rapid but transient increase in cyto-
is inhibited.
solic calcium result from either ~ ng_<?.!,_~um_gian-
~ ~n~ nels.lrr the
Kinase Phosphorylates the Enzymes sfi. The released calcium can be rapidly taken-up by
The catalytic subunit then transfers a phosphate group ER to term)-nate the response.
from ATP to different enzyme proteins (Fig. 45.4-5). T~ h i tra ltGlar calcium coi'i"Cel ,tr-ation is1k,w (10·7)
Phosphorylation u uall akes place on the OH groups wherea r llular calcium concentration is very high
of serine, threoni e or rosine residues of the sub- (10·3 ), aintaining a 10,000 fold calcium gradient across
strates. Hence, ese kinases are called Ser/Thr kinases the membrane.
The calcium transporting ATPase transporter accu-
The enzymes may be ~ted o~ ctiyated this
mulates calcium within the lumen of ER (sarcoplasmic
p~ j ~ This is an example of covalent modifi-
reticulum) in muscle.
cation. A summary of the cascade activation of enzymes
Hormones can increase the C>!tosolic calcit~m level
by the hormone is shown in Figure 45.5. Gty_cogmi.
by the following mechanisrnJ . ~
~ hory~~ e (see Fig . 10.36) and t1on::nooase0sif. 1
A. By altering the permea 6llit e membrane.
Ii ase (see Fig.13.14) are @lr'olle~
hormones that are acting through cyclic AMP are
~ he
B. The action of ~ 2+- +_A ¥f which
extrudes calcium in exchange forW.
enumerated in Table 45.1. 1..bt., c\.)u :f,t ... C. By rel e . intracellular calcium stores.
There are Many G-proteins D. ~ lmnH_· the calcium dependent .cegulstQ....ry
p~ II has fo:W calcium bjnding
About 30 different G-proteins are identified, each being
sites. When calcium binds there Is a conformational
used for different signal transduction pathways. The
change ~ -J .~,~ ~lmodulin, which has a role in
G-protein, adenyl cyclase, is called
regulating various kinases. Examples of enzymes
Gs (G-stimuY5ito ry) opposite group is called Gi
or functional proteins regulated by calmodulin are:
(G-inhibitQ['{)~-pc~ also involved in toxic
Adenyl cyclase, calcium-dependent protein kinases,
manifestations of ~ raao.c:Lper.tussis (see Box 45.2). nitric oxide synthase and phosphorylase kinase.
There are Many Protein Kinases
HORMONES ACTING
More than thousand protein kinases are now known. All [ THROUGH PIP2 CASCADE
the known effects of cAMP in euka otic cells result from

---------
activation of rotein kinases, which are serine/threonine
kinases.
The major player in this type of signal transduction is
phospholipase C that hydrolyses phosphatidyl inositol in
Chapter 45: Hormones and Growth Factors 625

Hormone Inactive receptor Steroid hormone attached (SR)


Outside cell

""
(tetramer) (R) dimer form is active
PKC
s s
- - - WW + -,c;,
-

---+
Inside cell
(e-, ; p) GTP
IP3 .
\
flfl
R R
.l.. ' RNAP
lcl~ RNAP

Plasma IP3 opens


membrane Ca channel SRE P Genes SRE P Genes

i= !
No proteins No proteins

S = steroid hormone; R = receptor; SR= steroid receptor complex:


SRE = steroid hormone response element in DNA; P = promoter site;
RNAP = RNA polymerase. (Left side) = In normal conditions,
promoter site is repressed so that proteins cannot be synthesized.
(Right side)= When steroid receptor complex is binding to the HRE,
Fig. 45.6: PIP2 and DAG acting as second messengers. PIP2 the P site is open, when RNAP fixes and transcription starts: new
= phosphatidylinositol bisphosphate. IP3 = inositol triphosphate. protein is synthesized
DAG= diacylglycerol. PLC = phospholipase C. PKC = protein
kinase C Fig. 45.7: Mechanism of action of steroid hormones

membrane lipids to : ! . , 4 . ~ ) a~d Cyclic GMP activates cGMP-dependent protein


~ ~ ol (DAG) that act as second me~ r s . kinase G (PKG), which phosphorylates important effector
The phospholipase C may be activated either by G-pro- proteins that can regulate calcium dependent contraction
teins or calcium io n ~ n also be generated by the or motility by modulating calcium influx. An example

1
action of(iWospbol~that produce hos.ehatidic is smooth muscle myosin, leading to relaxation and
acid which is h~rolY?ed to DAG. 1 ( ..._.- • vasodilatation. Cyclic GMP is also involved in the rho-
The binding of hormones like serotonin to cell sur- dopsin cycle. The role of cGMP in the light sensing cells
face receptor triggers the activation of the enz me hos- of retina and its interaction with the G-protein transducin
, J)holi ~se..:,.C which hydrolyzes the phosphatidyl inositol is described under visual cycle, Chapter 32.
to diacylglycerol. IP3 can release ca+• from intracellular NO (Nitric oxide) is the major activator of guanylate
stores, such as from endoplasmic reticulum and from cyclase. NO in turn is produced by the action of NOS
(Nitric oxide synthase) in tissues like vascular endothe-
sarcoplasmic reticulum (Fig. 45.6).(RAG activates PKf .
lial cells (Chapter 18). Increased level of cyclic GMP in
The elevated intracellular calcium then triggers proces-
ses like smooth m~cle contraction, glyco~i break- smooth muscle triggers rapid and sustained relaxation
of the smooth muscles. The vasodilatation resulting from
down and exocytosis. -
NO induced increase in cGMP has great physiological
and pharmacological significance. The drugs that act via
._ Role of Cyclic GMP
NO release are nitroprusside, nitrites (used in angina as
Cyclic GMP (cGMP) is another important second mes- coronary vasodilators) and sildenaphil citrate (Viagra).
senger involved in contractile function of smooth mus-

__
cles, visual signal transduction and maintenance of Hormones with Intracellular Receptors
blood volume. Cyclic GMP degradation is catalyzed by The hormones in this group include the steroid hormo-
membrane bound PD Es ..- t Q.....,~~ ..,.,,.,__
-----___./ ¼ ' f, -~ nes and thyroid hormones. They diffuse through the
It is formed from GTP by the action of(guan~I c clas plasma membrane and bind to the receptors in the cyto-
Several compounds have been found to increase the con- plasm. The hormone receptor (HR) complex is formed
centration of cGMP by activating guanyl cyclase. These in the cytoplasm. The complex is then translocated to
include drugs like nitroprusside, nitroglycerin , sodium the nucleus. Steroid hormone receptor monomer binds
nitrite. All these compounds act as potent vasodilators, to a single steroid molecule at a hydrophobic site, but on
by inhibiting the phosphodiesterase. binding to genes they dimerize (Fig. 45.7).
626 Section F: Advanced Biochemistry

In the nucleus, the HR binds to the hormone res- are long polypeptides. They are synthesized in hypotha-
ponse elements (HRE) or steroid response elements lamus. It is cleaved into the active molecule, and then
(SRE). The SRE acts as an enhancer element and when transported to posterior pituitary and stored there. They
stimulated by the hormone, would increase the transcrip- are released to bloodstream by exocytosis.
tional activity. The newly formed mRNA is translated
to specific protein, which brings about the metabolic IANTIDIURETIC HORMONE
effects. This would lead to induction of protein synthesis. It is also called vasopressin. It has 9 amino acids. If
Best examples of the effect of hormones on genes are: arginine is replaced by lysine, it is called lysine vaso-
a. The induction of synthesis of amino transferases by pressin (LVP). Its main action is to prevent diuresis. So
glucocorticoids. it reduces the urine output. The ADH acts on the distal
b. Synthesis of calcium binding protein by calcitriol convoluted tubules of the kidney, producing reabsorp-
(see Fig. 32.11 ). tion of water. The ADH binds to membrane receptor
and activates adenylate cyclase. The cyclic AMP thus
Insulin-Signaling Pathway produced will activate the protein kinase. This, in turn,
phosphorylates proteins of the microtubules and micro-
filaments. The net effect is the reabsorption of water.
The regulation of ADH secretion is through the osmolal-
ity of blood. Lowering of the osmolality (hemodilution)
suppresses ADH secretion. Conversely, an increase in
osmolality (hemoconcentration or dehydration) leads to
stimulation of the secretion of ADH.
Horylates insulin receptor sub- Deficiency of ADH results in diabetes insipidus. It
different IRS molecules, named is characterized by excretion of large volumes of dilute
urine. Hypernatremia and hypertonic contraction of extra-
Activation of IRS2 results in activation of the Pl-3 cellular fluid volume are also seen. It is a very rare
kinase, which eventually activates various protein kina- condition. Excess secretion of ADH often results from
ses, PKB, PKC, SGK (serum glucocorticoid regulated ectopic production of ADH by malignant tumors else-
kinase), etc. This leads to transcription of specific genes where, referred to as the "syndrome of inappropriate
for key enzymes of glycolysis, such as glucokinase. secretion of ADH" or SIADH. Here ADH is continuously
There are more than 100 enzymes influenced by insulin. secreted and is not subjected to any control mechanisms.

I HYPOTHALAMIC AND
PITUITARY HORMONES
Generally, there is hypotonic expansion of extracellular
volume, water intoxication (headache, confusion, ano-
rexia, nausea, vomiting, coma and convulsions).

The hypothalamus produces two types of endocrine Oxytocin


factors; (a) the hypothalamic neuropeptides and (b) the
The term means "to stimulate birth". Oxytocin acts on
hypothalamic releasing factors. The releasing factors an estrogen-primed uterus. The synthetic derivative of
are neurosecretions synthesized in the hypothalamus oxytocin, pitocin, is used to induce labor.
and released through the hypothalamic pituitary portal Oxytocin has an effect on the mammary glands.
circulation. They have their effect on the secretion of Suckling generates a neurogenic reflex, which stimu-
pituitary tropic hormones. lates the production of oxytocin. It causes contraction of
the myoepithelial cells expelling the milk into milk ducts
IHYPOTHALAMIC NEUROPEPTIDES from the acini.

The hypothalamic neuropeptides are produced by the Hypothalamic-Releasing Factors


supraoptic and paraventricular nuclei of the hypothala- Andrew Schally in 1971 isolated TRH. Roger Guillemin
mus. These neurohormones are antidiuretic hormone isolated LHRH (now termed GnRH) and somatostatin in
(ADH) and oxytocin. The precursors of ADH and oxytocin 1973; both of them were awarded Nobel Prize in 1977.
Chapter 45: Hormones and Growth Factors 627

TABLE 45.2: MaJor hypothalamic-releasing factors


Name Chemical nature Biological actions
TRH; thyrotropin releasing hormone Tripeptide; (pyro-Glu-His-Pro-NH2) Induces secretion ofTSH and PRL; neuromodulat or
GnRH; gonadotropin releasing hormone Decapeptide Releases LH and FSH; induces spermatogenesis,
ovulation and testosterone
GHRH; growth hormo ne releasing hormone 37-44 amino acid Stimulates growth ho rmone secretion
CRF; corticotropin releasing factor Peptide with 41 amino acids Release o f ACTH. Inhibited by cortisol
Somatostatin; g rowth hormone inhibitory factor Cyclic peptide w ith 14 amino acids Inhibits secretion of GH and TSH. Inhibit s gut
hormones, pancreatic and gastric secret ion
PIF; prolactin inhibitory factor Dopamine Inhibits PRL release

The secretion of hormones by adenohypophysis or ante-


Hypothalamus
rior pituitary is under the control of peptides secreted by
hypothalamus. Table 45.2 shows a list of these factors.
The secretion of the hypothalamic peptides are also Regulatory hormones
(CRH, TRH. GnRH, GRJH, PRIH) Short
under the feedback control of anterior pituitary tropic
hormones (short loop feedback) as well as the target
l loop
Long
loop

gland hormones (long loop feedback) (Fig. 45.8). Anterior pituitary

!
The paraventricular nucleus (PVN), located in the
anterior medial area of hypothalamus, is involved in
Tropic hormones
oxytocin and vasopressin release from the pituitary. The (ACTH, TSH, LH, FSH, GH, Prolactin)
arcuate nucleus (ARC) is involved in secretion of vari-
ous pituitary releasing hormones. The overall function of
!
Target organs
the hypothalamus is to link the central nervous system (Thyroid, adrenal glands,
_. gonads, liver)
to the endocrine system via the pituitary gland (also
termed the hypophysis).
l
Target organ hormones
(T3, T 4, cortisol, estrogen,

U HORMONES OF ANTERIOR progestins, androgens, somatomedins)

PITUITARY ------ Fig. 45.8: Long and short loops of feedback

The anterior pituitary hormones are tropic in nature,


Hypoglycemia stimulates GH secretion, and hyperglyce-
stimulating the secretion of hormones from target
mia suppresses it. The hypothalamic growth hormone
organs. Secretions of all these hormones are under the
releasing hormone (GHRH) stimulates GH synthesis
control of hypothalamic releasing or inhibitory factors.
and release. Ghrelin, a peptide derived from stomach
Table 45.3 lists them.
induces GHRH and directly stimulates the release of

l GROWTH HORMONE GH. Somatostatin synthesized in the hypothalamus inhi-


bits the GH secretion .
It is also called somatotropin. It is a single polypeptide The metabolic effect of GH is partly mediated by
chain with 191 amino acids. Growth hormone (GH) is somatomedin, also known as insulin-like growth
synthesized by acidophils (somatotropic cells) of ante- factor-1 (IGF-1). The growth of long bones is stimulated
rior pituitary. Plasma concentration of GH is less during by this factor. IGF-1 , the peripheral target hormone of
day time, with secretary peak appearing 3 hours after GH exerts feedback inhibition. GH increases the uptake
meals. Maximum level of GH is seen during deep sleep. of amino acids by cells; enhances protein synthesis,
For measurement of serum GH, the samples are col- and produces positive nitrogen balance. The antiinsulin
lected during sleep and also during waking hours to effect of GH causes lipolysis and hyperglycemia. The
assess the circadian rhythm. GH secretion is regulated overall effect of GH is to stimulate growth of soft tissues,
by the balance between GHRH and GHIH (somatostatin). cartilage and bone. It is anabolic.
628 Section F: Advanced Biochemistry

TABLE 45.3: Hormones of anterior pItuItary pattern of secretion is reflected in cortisol also. Factors
Acronym Full name Chemical nature Amino acids that increase ACTH secretion include stresses such as
GH Growth hormone Polypeptide 191 pain, cold exposure, acute hypoglycemia, trauma, dep-
ACTH Adrenocorticotropic Polypeptide 39 ression, and surgery.
hormone
Luteinizing hormone
ACTH binds to specific receptors on the adrenal
LH Glycoprotein; a, ~ a=89
chains ~ = 115 gland, then activates adenylate cyclase and so, cAMP
F5H Follicle stimulating Glycoprotein; a, a = 89 level is raised. ACTH induces adrenocortical steroido-
hormone chains = 11 5 genesis through the melanocortin-2 receptor. Steroid
TSH Thyroid stimulating Glycoprotein; a, a = 96
hormone chains = 115
hormones in turn cause feedback inhibition of HPA
MSH Melanocyte Polypeptide a = 13 (hypothalamopituitary adrenal axis).
stimulating hormone ~ = 18 ACTH secreting tumors of pituitary will cause Cush-
v = 12
ing's disease. Deficiency of ACTH secretion may occur
PRL Prolactin Polypeptide 198
Polypeptides 31 as a part of panhypopituitarism.
Cosyntropin stimulation (Rapid ACTH) test: With
Excess secretion by GH secreting tumor, leads to rapid administration of ACTH , blood cortisol level will
gigantism in children and acromegaly in adults. Defi- be raised. If there is adrenal insufficiency, adrenal gland
ciency of GH secretion in early childhood results in pitui- cannot be stimulated by the administered ACTH and
tary dwarfism. Dwarfism may also result from congenital subnormal or low response of cortisol occurs.
deficiency of GH due to end organ resistance. It is Dexamethasone suppression test: Dexamethasone,
treated by giving GH produced by recombinant technol- an analog of cortisol suppresses ACTH hormone and
ogy. Inhibition of GH secretion by hyperglycemia (GTT) cortisol production in normal subjects; but not in patients
and stimulation by hypoglycemia (insulin infusion) are with Cushing's syndrome.
used to check the status of GH secretion in hyper as well
as hypopituitarism. Endorphins

ADRENOCORTICOTROPIC Small peptides formed from POMC have endogenous


morphine-like or opiate-like activity. They are responsi-
HORMONE ble for increasing the threshold of pain, especially under
It is secreted as a large precursor molecule, known as conditions of stress. Morphine binds to the receptors for
pro-opiomelanocortin (POMC), with a molecular weight endorphins, by which morphine induces the pain relief.
of 32 kD. It is cleaved to give about 30 different endor-
phins and several hormones. The major products are Glycoprotein Hormones
ACTH , MSH (melanocyte stimulating hormone) and
beta endorphin. The active adrenocorticotropic hormone TSH, FSH and LH are the glycoprotein hormones of pit-
(ACTH) is a polypeptide with 39 amino acids. uitary. All the three are made up of two subunits. The
The secretion of POMC is under the control of CRF. alpha unit is common to all the three, while the beta
ACTH is released from the pituitary in a pulsatile manner, subunit is specific for each of them.
with a definite diurnal rhythm, the secretion being high- The alpha unit is also common to human chorionic
est in the early morning, and minimum at midnight. This gonadotropin (hCG); so beta hCG is estimated as an
index of pregnancy. Beta hCG is a tumor marker for
choriocarcinoma (see Chaplet 48).

THYROID-STIMULATING
HORMONE JHYROTROPIN _ .____

Thyroid-stimulating hormone (TSH) increases the secre-


Roger Guillemin Andrew Schally
NP 1977 tion of thyroid hormones by stimulating all the steps of
NP 1977
b. 1924 b. 1926 production of synthesis of thyroxine. It acts through
Chapter 45: Hormones and Growth Factors 629

cAMP by binding with a receptor on thyroid cell surface. High FSH levels are an indication of subfertility and/
TSH secretion is stimulated by TRH. The TRH also sti- or infertility. Diminished secretion of FSH can result in
mulates prolactin secretion. The TSH secretion is also hypogonadism. This condition is typically manifested
controlled by the level of thyroid hormones. High levels in males as failure in production of normal numbers of
of TSH may occur due to primary hypothyroidism and sperm. In females, cessation of reproductive cycles is
lack of feedback control. commonly observed.

'
.., Increased serum TSH levels are seen in primary
hypothyroidism (3-100 times normal), Hashimoto's thy-
Serum level of FSH is raised in primary gonadal
failure, ovarian or testicular agenesis, castration, Kline-
roiditis, ectopic TSH secretion by tumors (lung, breast). felter's syndrome, and gonadotropin secreting pituitary
The TSH is elevated in euthyroid patients during treat- tumors. Serum level of FSH is decreased in anterior
.ment of hyperthyroidism but TSH is low for 4-6 weeks pituitary hypofunction, hypothalamic disorders, preg-
after achieving euthyroid state in treated hyperthyroid nancy, anorexia nervosa, polycystic ovary disease and
patients. in hyperprolactinemia.
Decreased levels are observed in primary hyper- Serum level of LH is raised in primary gonadal dys-
thyroidism, secondary hypothyroidism (pituitary origin), function, polycystic ovary syndrome, postmenopausal
tertiary hypothyroidism (hypothalamic), subclinical women and in pituitary adenoma. Serum level of LH is
hyperthyroidism (e.g . toxic multinodular goiter, exo- decreased in pituitary hypothalamic impairment, ano-
genous thyroid hormone administration). rexia nervosa and severe illness.

I GONADOTROPINS Human Chorionic Gonadotropin


They are LH (Luteinizing hormone) and FSH (Follicle It is produced only during pregnancy. The human chori-
stimulating hormone) from pituitary. The placenta also onic gonadotropin (hCG) binds to the receptor, LHCGR,
produces human chorionic gonadotropin (hCG). FSH in the luteal cells of the ovary. Initially the developing
and LH are secreted under the effect of gonadotropin embryo synthesizes and secretes hCG. Following
releasing hormone (GnRH). Puberty does not set in implantation of the embryo, the cells of the placenta
secrete hCG. Its appearance in the plasma and urine is
until the pulsatile secretion of LHRH is started by hypo-
one of the earliest signals of pregnancy and the basis of
thalamus. The fundamental ~hange during puberty is a
many pregnancy tests. The role of hCG during pregnan-
reduction in hypothalamic inhibition of LHRH release.
cy is to prevent disintegration of the corpus luteum so as
FSH stimulates growth of ovarian follicles in females
to maintain the synthesis of progesterone by this tissue.
and spermatogenesis (Sertoli cells) in males. Testoste-
rone in males (secreted by Leydig interstitial cells) and
Prolactin (Somatomammotropin)
progesterone in females (secreted by corpus luteum),
are increased under the influence of LH. Prolactin (PRL) secreted by lactotropic cells of adeno-
FSH secretion rises during the follicular phase of the hypophysis is under the control of hypothalamus. Lacto-
menstrual cycle, reaches a peak by the 14th day and trops hyperplasia is induced by estrogen (during last
starts falling when ovulation occurs. Ovulation occurs as two trimesters of pregnancy). Its secretion is primarily
a result of positive feedback effect of estrogen produc- controlled by inhibitory effect of dopamine and GABA.
ing the preovulatory LH surge. The level of FSH and LH Its release is stimulated by TRH and vasoactive intesti-
falls during the postovulatory phase. unless fertilization nal peptide. It controls the initiation and maintenance of
and implantation occur. The gonadotropin production lactation. The PRL secretion is pulsatile: highest levels
is under the feedback control by the sex hormones. during rapid eye movement sleep and peak serum level
High levels of FSH and LH are seen in postmenopausal occurs between 04:00 and 06:00 hours. Prolactin stimu-
women due to lack of this feedback. FSH is synthesized lates lactation on estrogen primed breast. It increases
and secreted by gonadotrophs of the anterior pituitary synthesis of milk protein and fat. Hyperprolactinemia is a
gland. FSH regulates the development, growth, pubertal cause of infertility in females. Secretion of PRL is stimu-
maturation, and reproductive processes of the body. lated by TRH and inhibited by PIF.
630 Section F: Advanced Biochemistry

~~;;:~:oer•~ 21

I
10 I
CH3

C=O20

l
17.
(Progesterone)
21
CH3

HO I
C=O 20
17-hydroxylase
(NADPH) HO
A 21
CH3
10 I I
17.

l
C=O 20
D 17-hydroxy progesterone - - - - -. 10 I
17.- 0H

HO 1~:tr~i""'
Pregnenolone CHpH

Fig. 45.9: Synthesis of pregnenolone


(11-deoxy-cortisol)
0
IC=O

l
1a I
11-beta- n:- OH
Prolactin stimulation test by chlorpromazine and
hydroxylase
Prolactin suppression test by L-dopa are used to assess (NADPH)
the function.

Human Placental Lactogen "A" ring in prednisolone- - - - - - i

The amino acid composition of human placental lac-


togen (hPL) is similar to human growth hormone. The Fig. 45.10: Cortisol synthesis
amount of hPL secreted is proportional to the size of the
placenta. Low levels of hPL during pregnancy are a sign
r YNTHESIS OF STEROID
of placental insufficiency. The biological actions of hPL
are similar to those of growth hormone.
HORMONES

I ADRENAL CORTICAL
HORMONES
Cholesterol is first acted upon by desmolase and a 6-car-
bon unit is cleaved off, formi ng the 21 carbon steroid,
pregnenolone (Fig. 45.9). It is a common precursor for
all the steroid hormones. Adrenocorticotropic hormone
The adrenal cortex has three different zones each res- (ACTH) stimulates this step. This is the rate limiting step
ponsible for production of different classes of steroid hor- for synthesis of all steroid hormones.
mones (C21, C19 and C18). The smallest and outermost Progesterone is the fi rst steroid hormone formed
zona glomerulosa produces the C21 steroids, mine- from pregnenolone in two steps. Progesterone is fur-
ralocorticoids. They have effects on water and electro- ther converted into glucocorticoids (Fig. 45.10), miner-
lyte balance. The middle zone of the adrenal cortex, the alocorticoids and sex steroids (Fig. 45.11 ). The major
zona fascicularis produces the glucocorticoids mainly; adrenal glucocorticoids are cortisol, 11-deoxycortisol
and adrenal androgens and estrogens to a lesser extent. and corticosterone in that order. The major mineralo-
The innermost zona reticularis produces the androgens corticoid is aldosterone, but 11-deoxycorticosterone and
(C19) and estrogens (C18). Cortisone was isolated by corticosterone also have significant mineralocorticoid
Tadeus Reichstein; the structure was identified by activity. These reactions are effected by hydroxylation.
Edward Kendall, while Philip Hench in 1948 showed its These specific hydroxylases are NADPH dependent.
efficacy in rheumatoid arthritis. All the three were awar- These hydroxylation reactions are summarized in Figure
ded Nobel Prize in 1950. 45.12. The ACTH stimulates the synthesis of all steroid
Chapter 45: Hormones and Growth Factors 631

Cholesterol (C21)

!
Pregnenolone (C21 ) - - - - - - - - Progesterone (C21)
Progesterone (C21)

l l
~r~• T
17-hydroxylase 17-hydroxy

17-hydroxy-
0
17-hydroxyprogesterone Androstene-
dione (C19)

1
Cortisol Dehydro-
(C21) epiandro- Corticosterone
sterone
(DHEA)
(C19)
Testosterone
(C19)
!
l
l
Aldosterone (C21)
Androstenedione - . Estradiol

l
(C19) (C18)
0
!
17-reductase 17-hydroxy DHEA 17-keto-
(NADPH) steroids sulfates steroids
in urine in urine in urine

Testosterone Fig. 45.12: Summary of major pathways for production of gluco-


(C19)
corticoids, mineralocorticoids and sex steroids. Precursors in re d

l
box; intermediaries in grey box; hormones in b lue box; excretory
Aromatase products in brown box
(NADPH)

.: feedback effect on ACTH secretion. ACTH also increases


Estradiol-17-beta - -• the secretion of aldosterone. The level of aldosterone
(C18) is also affected by posture, highest values in upright pos-
HO
ture and lowest while lying down. All steroid hormones
Fig. 45.11 : Synthesis of sex hormones
act through intracellular messengers and increase the
rate of transcription.
hormones by activating desmolase so that the availabil-
ity of pregnenolone is increased. Transport and Metabolism
Approximately 70% of cortisol in blood is bound to cor-
Secretion of Adrenal Hormones
tisol binding globulin (CBG) or transcortin. About
The diurnal variation of secretion of cortisol (highest val- 20% is bound to albumin and the rest is free, which is
ues early in the morning and minimum at night) paral- the biologically active fraction. The half-life of cortisol is
lels the pulsatile release of ACTH from anterior pituitary about 2 hours. The steroid hormones are metabolized
under the influence of CRF. Cortisol exerts the negative and inactivated by the liver. The major processes are
reduction and conjugation. The C21 steroids are redu-
ced to their tetrahydro derivatives, which are excreted
as their glucuronides or sulfates in urine.

Uri nary Steroids


The urinary steroids are referred to as 17-ketosteroids
Tadeus EdwardC Philip S AdolfFJ and 17-hydroxy steroids. The 17-ketosteroids may be
Reichstein Kendall Hench Butenandt
derived from both adrenal steroids and androgens from
NP 1950 NP 1950 NP 1950 NP 1939
1897-1996 1886-1972 1896-1965 1903- 1995 the gonads. However, 11-hydroxylatioi i occurs only in
632 Section F: Advanced Biochemistry

TABLE 45.4 : Effects of glucocort1co1ds


System Effect
Carbohydrates Activity of transaminases and gluconeogenic enzymes (PC, PEPCK, F-1,6-BPase and GPase) are stimulated,
i ncreasing gluconeogenesis. Glycolytic enzymes (GK, PFK and PK) are suppressed. Decreased glucose uptake
by peripheral t issues. All these lead to hyperglycemia
Lipids Increase lip id mobilization; facilitate lipo lytic ho rmones leading t o increase in free fatty acids
Proteins and nucleic acids Catabollsm of proteins and nucleic acids increased. Increased urea production
Fluid and electrolytes Promotes water excretion by increase in GFR and inhibition of ADH secret ion
Bone and calcium Decrease serum calcium by inhibiting osteoblast function, leading to osteoporosis
Secretory action Stimulates secretion of gastric acid and enzyme. Induces acid peptic disease
Connective tissue Impaired collagen formation. Poor wound healing
Imm une system lmmunosuppressant. Lysis of lymphocytes. Antiinflammatory and antiallergic

adrenal cortex. The term 17-ketogenic steroids is used immunoassay (CUA). Maximum activity is at 9 AM and
to include all the compounds having a keto or hydroxyl minimum activity is at 9 PM. A loss of diurnal rhythm may
group at 17th carbon. be an early indication of disease.

Biological Effects of Glucocorticoids Estimation of Urinary Free Cortisol


The free cortisol in plasma is the biologically active
The glucocorticoids, as the name suggests, mainly affect
fraction. High levels are seen in hyperfunction and low
metabolism of glucose. The major biological effects of
levels in hypoactivity.
glucocorticoids are given in Table 45.4.
Plasma ACTH
Biological Effects of Mineralocorticoids Suppressed ACTH levels are seen in hyperadrenalism
Mineralocorticoids (mainly aldosterone) increases sodium and high ACTH levels in hypoadrenalism as well as in
reabsorption from renal tubules, leading to sodium Cushing's disease: In hyperadrenalism due to ectopic
retention and resultant water retention (see Chapter 28). ACTH secretion, ACTH levels are elevated.
The sodium potassium exchange at the renal tubules
Dexamethasone Suppression Test
lead to increased potassium excretion.
Dexamethasone produces a fall in cortisol secretion
I MECHANISM OF ACTION due to feedback suppression of ACTH. In normal people,
the overnight suppression with low dose (2 mg) causes
(.QF STEROID HORMONES
a 50% fall in the original value. But this dose may fail to
Steroid hormones in general have nuclear receptors and produce suppression in cases of adrenal hyperactivity.
act by regulation of gene expression. Steroid hormones
being lipid soluble can easily pass through plasma Urinary Steroids
membrane and enter the cytoplasm. Here they combine Estimation of 17-ketogenic steroids is indicated only in
with specific receptors and the HR complex translocates AG syndrome. Since the 24-hour excretion is measured,
to the nucleus. The steroid hormone response elements the diurnal variation is also taken care of.
on DNA (specific sequences bind specific hormones)
and regulate transcription of genes. The proteins pro- Stimulation Test
duced in response to the hormonal stimulation produce Infusion of synthetic ACTH (Synacthen or tetracosactrin)
metabolic effects, e.g. enzymes and transporters. is given. In the absence of reserve, stimulation tests fail
to evoke any response.
Assessment of Glucocorticoid Secretion
Basal Level of Cortisol Metyrapone Test
The plasma cortisol level is determined by enzyme-lin- Metyrapone inhibits the 11-hydroxylase enzyme. Hence,
ked immunosorbent assay (ELISA) or chemiluminiscent alternate androgenic pathways are enhanced and urinary
Chapter 45: Hormones and Growth Factors 633

TABLE 45.5: Findings m adrenal hyperfunctions


Cause Plasma cortisol Urinary free cortisol Plasma ACTH Dexamethasone suppression
Adrenal adenoma Increased; diurnal rhythm is lost Increased Decreased No suppression with low dose

Adrenal carcinoma Increased; diurnal rhythm is lost Increased Decreased No suppression even with high dose
Pituitary adenoma Increased; no diurnal rhythm Increased Increased Suppression with high dose
Ect opic ACTH production Increased; no diurnal rhythm Increased Increased No suppression
I

Cause ofadrenal insufficiency Plasma cortisol Urinary free cortisol Plasma ACTH ACTH stimulation CRH stimulation Na• and K' in blood
Primary Low Low Elevated No effect No effect Na• ; K•t

Secondary Low Low Low Normal/exaggerated No effect Na•.,I,; K•t

Tertiary Low Low Low Normal/exaggerated Exaggerated Na• .,I,; K•t

excretion of 17-ketosteroids increase. When it is given, d. Plasma pH is elevated (hypokalemic alkalosis)


cortisol is not formed. Then there is no feedback inhibi- e. Osmolality is elevated (hypertonic expansion).
tory effect. Hence, alternate pathways of sex steroids are
more operative and the urinary excretion of 17-ketoste- Aldosterone in Blood
roids tends to elevate. Secondary aldosteronism can result from indiscriminate
use of diuretics, congestive cardiac failure, following
CRH Test hypovolemia, etc. Decreased levels are seen in Addi-
The test is of importance in establishing the cause of son's disease.
adrenal hyperfunction (primary, secondary or tertiary).
Adrenogenital Syndrome (AG Syndrome)
Adrenal Hyperfunction There is congenital deficiency of steroid hydroxylases
Hyperactivity of adrenal cortex may be due to primary leading to deficient secretion of cortisol. Since cortisol,
defect in adrenal gland itself (Cushing's syndrome) or the major feedback effector is not present, ACTH secre-
secondarily by excessive production of ACTH from pitui- tion continues leading to adrenal hyperplasia (CAH).
tary (Cushing's disease) or ectopic ACTH production Depending on the enzyme defect the manifestations
by other malignant tumors (Table 45.5). also vary.
21-Hydroxylase deficiency: It is the most common
Adrenal Hypofunction type, where the production of cortisol is totally absent.
The most common cause of adrenal hypofunction is pri- The lack of feedback leads to increased androgen syn-
mary adrenal insufficiency or Addison's disease. It is cha- thesis. This would result in virilization of female children
racterized by tiredness, dehydration, hyponatremia and who develop ambiguous genitalia. Precocious puberty is
hyperpigmentation (due to high ACTH levels and its MSH seen in male children. Early diagnosis and supplementa-
activity). The diagnostic findings are given in Table 45.6. tion of cortisol is effective in children.
11-Hydroxylase deficiency: In this condition, the
Primary Hyperaldosteronism
symptoms are more serious. The hypertensive variety
(Conn's Syndrome) of the AG syndrome manifests and the child may not
This may result from an aldosterone secreting tumor. survive.
The condition may be diagnosed by:
a. Elevated plasma aldosterone levels and no change SEX HORMONES
with posture
b. Plasma renin activity is decreased due to feedback These are secreted by the gonads in response to pituitary
effect gonadotropins (LH and FSH). Adolf Butenandt isolated
c. Serum electrolytes show hypernatremia and hypo- estrogen (1929), progesterone (1934) and testosterone
kalemia (1935), for which he was awarded Nobel Prize in 1939.
634 Section F: Advanced Biochemistry

I OVARIAN HORMONES Certain breast cancers, especially in perimenopau-


sal women are estrogen-dependent. In such patients,
They are C18 estrogens, C19 androgens and C21
estrogen receptor antagonists (Tamoxifen ) will block
progesterone. These are produced by the ovarian fol-
the estrogen receptors, and cancer cells tend to die.
licles. The follicular thecal cells produce C 19 androgens.
These are converted to C 18 estrogens by granulosa
cells, by aromatization of ring A and loss of C19 methyl
l!_ESTICULAR HORMONES
group (see Fig. 45.11 ). Estradiol is the most important In humans, testosterone is the major male hormone,
estrogen. It is converted to estrone by liver. It is further while in animals, it is androstenedione. Androgen is
hydroxylated to estriol , which is inactive. Estradiol is derived from Greek word, ander, meaning male. The Leydig
bound to plasma SHBG (sex hormone binding globu- cells (interstitial cells), secrete the androgens, under the
lin). Estradiol has not only a critical impact on reproduc- influence of LH. LH is also called ICSH (interstitial cell
tive and sexual functioning, but also affects other organs stimulating hormone). The androgens exert feedback
including the bones. effect on LH secretion, through inhibition of GnRH. Thus,
Estradiol enters cells freely and interacts with a in patients with hypogonadism, LH level is high.
cytoplasmic target cell receptor; then, estradiol enters FSH binds to Sertoli cells (basement membrane
the nucleus of the target cell, and regulates gene trans- cells of seminiferous tubules) and promotes the synthe-
cription. sis of androgen binding protein (ABP). Thus high con-
centration of androgen is made available locally at the
Regulation of Ovarian Hormones seminiferous tubules, at the site of spermatogenesis. In
FSH influences follicles to ripen, which produces estro- patients with azoospermia, the FSH level is very high,
gen. Estrogen level gradually increases in the second due to lack of negative feedback.
week of the menstrual cycle. Estrogen level is maximum Androgens stimulate spermatogenesis, produce
24 hours before the LH peak. High doses of estrogen hypertrophy of prostate, seminal vesicles, muscle, bone
can suppress the LH release and, therefore effective as and kidney cells. It is anabolic. Dihydrotestosterone
contraceptive. Under the influence of estrogen, uterine (DHT) is the cause for the benign prostatic hypertro-
endometrium proliferates, glands in endometrium are phy, that affects more than 75% of men over the age
hypertrophied, ducts in mammary gland are proliferated of 60 years. The enzyme 5-alpha-reductase is needed
and progesterone receptors are synthesized. to convert testosterone to DHT. Finasteride can inhibit
LH level peaks 16 hours before the ovulation. The 5-alpha-reductase, and hence it is used as a treatment
surge of LH induces the ovulation. The corpus luteum for prostate hypertrophy.
then starts secreting progesterone. Under the influence Prostate cancers are androgen dependent, so
of progesterone, endometrium enters the secretory phase, androgen ablation is used for treatment of advanced
and prepares for implantation of the fertilized ovum. prostate cancer. Androgen receptor (AR) is believed to
LH is required for maintenance of corpus luteum. play critical roles in the development and progression of
If implantation occurs (day 22-24), the LH function is cancer.
taken over by the hCG, produced by the cytotrophoblast Action of steroid hormones is summarized in
cells of the early embryo. The hCG can be detected 5-7 Table 45.7.
days after missing a period. If implantation has not oc-
curred, hormone levels are decreased and the secretory
glands of the endometrium are denuded.
I
THYROID HORMONES
Clomiphene citrate competes with estrogen for re- Iodine Metabolism
ceptors in hypothalamus, thus removing the feedback Daily requirement of iodine is 150-200 mg/day. Its
inhibition. So GnRH level is increased, with consequent sources are drinking water, fish, cereals, vegetables and
high levels of LH and FSH, which may produce follicular iodinated salt. Total body contains 25-30 mg of iodine. All
stimulation and ovulation. Clomiphene is therefore used cells do contain iodine; but 80% of the total is stored in the
to produce ovulation in infertile females. thyroid gland. Iodine level in blood is 5-1 0 µg/dl. In most
Chapter 45: Hormones and Growth Factors 635

..
TABLE 45.7: Summary of steroid hormones
Hormone Tissue of origin Function
Estrogen (Estradiol) Ovary Maturation and function of female secondary sex organs
Estrogens Placenta Maintenance of pregnancy
Progestins (Progesterone) Ovary Implantation of ovum and maintenance of pregnancy
Progestins Placenta Mimic action of progesterone
Androgens (Testosterone) Testes Maturation and function of male secondary sex organs
Glucocorticoids (Cortisol and corticosterone) Adrenal cortex Diverse effects on inflammation and protein synthesis
Mineralocorticoids (Aldosterone) Adrenal cortex Maintenance of salt balance

•-----------~ .
...
Plasma Thyroid cell Acinar space
(Iodide) ( '-\ (active
2
,- iodine)

l
Iodide

!3 ( \ H2 0 2 H2 0

TgF
9
HMP shunt pathway - NADPH NADP•
DIT
F i g . 45.14: Step 2 of thyroxine synthesis

10
PBI
.,(f , r
T, ~ Tgb-Tc +--
6
Tgl T;

Stored colloid
Step 2: Oxidation of Iodine
The iodide taken up by the thyroid cell is oxidized to
active iodine (Step 2 in Fig. 45.13). The thyroid is the
Fig. 45.13: Metabolism of thyroid hormones only organ which can perform this oxidation step. This
is catalyzed by the enzyme thyroperoxidase. The reac-
parts of the world, iodine is a scarce component of the tion needs hydrogen peroxide, which is produced by an
soil. Upper regions of mountains generally contain less NADPH-dependent reaction (Fig. 45.14). The NADPH
_. is generated by the hexose monophosphate shunt path-
iodine. Such areas are called goitrous belts, e.g. Hima-
way. This second step is stimulated by TSH and inhibited
layan region.
by antithyroid drugs such as thiourea, thiouracil and
Commercial source of iodine is seaweeds. The pro-
methimazole. In patients with an inborn error of iodide
gram of iodination of common salt has resulted in inc-
oxidation defect, treatment is T4 administration.
reased availability of iodine. Ingredients in foodstuffs,
which prevent utilization of iodine are called goitrogens. Step 3: Iodination
Goitrogens are seen in cassava, maize, millet, bamboo
Then thyroglobulin (Tgb) is iodinated. Thyroglobulin is
shoots, sweet potatoes and beans. Cabbage and tapi-
synthesized by the thyroid follicular cells. Iodination of
oca contain thiocyanate, which inhibits iodine uptake
the tyrosine is taking place on the intact Tgb molecule
by thyroid. Mustard seed contains thiourea, which inhi-
in the follicular space. Thus 3-monoiodotyrosine (MIT)
bits iodination of thyroglobulin. and 3,5-diiodotyrosine (DIT) are produced (Fig. 45.15).
The thyroid hormones are thyroxine (T4) and triiodo-
thyronine (T3). Step 4: Coupling
Some of the tyrosine residues in the thyroglobulin are
Synthesis and Secretion of Thyroxine aligned opposite each other, and are coupled (Step 4,
Fig . 45.13). When two DIT molecules couple, one mol-
Step 1: Uptake of Iodine ecule of tetraiodothyronine (T4) is formed (Fig. 45.1 5).
Thyroid gland takes up and concentrates iodine (Step 1 Triiodothyronine (T3) may be formed by deiodination of
in Fig. 45.13). This step is inhibited by thiocyanate and outer ring of T4 by 5'-deiodinase. Under normal condi-
perchlorate, which compete for the carrier mechanism. tions, 99% of the hormone produced by the thyroid gland
This step is stimulated by TSH. The congenital defect, is T4. The T4 residues are now attached to the thyro-
iodine trapping defect, may be treated by large doses globulin molecule. The iodotyrosyl coupling defect, an
of iodine. inborn error, affects this 4th step. Treatment is to give T4.
636 Section F: Advanced Biochemistry

Step 8: Release
3-monoiodotyrosine (MIT)
The T4 thus generated is released into the bloodstream.
The T3 is produced by deiodination at 5' position, either
HO inside the thyroid cell or in the peripheral tissues.

3,5-diiodotyrosine (DIT) Step 9: Salvaging of Iodine ..


The MIT and DIT that are not utilized are deiodinized and
HO CH2-CHNH2-co o H salvaged for re-utilization inside the cell itself (Step 9 in
Fig. 45.13). Deiodinase defect is the inborn error of
metabolism affecting this step. In such cases, DIT and
MIT are seen in urine. Since iodine is excreted, iodine

-b-
3,5,3'-triiodothyronine (T3) deficiency is manifested. Treatment is to give iodine.
'
HO 0 Step 10: Transport of Thyroid Hormones
Thyroid hormones are transported in plasma by proteins
(Step 10 in Fig. 45.13). The bound form is biologically
inactive, but they can be rapidly released. Total protein
bound iodine (PBI ) is about 10 mg/dl . The thyroxine
binding globulin (TBG) (54 kD) carries about 80% of T4
and 60% of T3. The rest of thyroid hormones are loosely
HO 0 bound with Transthyretin (TTR}, prealbumin and albumin.

Step 11: Catabolism of Thyroid Hormones


3,5,3',5'-tetraiodothyronine or thyroxin (T4 )
T3 is biologically more active. T4 is a prohormone which
Fig. 45.15: Thyroid hormones and precursors is deiodinated to T3. In the peripheral tissues, deio-
dination takes place. This is done by a deiodinase, a
Step 5: Storage selenium containing enzyme. Part of the T3 and T4 are
conjugated with glucuronic acid and excreted through
The thyroid gland is unique, in that it is the only endo-
bile, and to a lesser extent, through urine.
crine gland to store appreciable amounts of the hormone
(Step 5 in Fig. 45.13). The thyroglobulin contains about
Mechanism of Action of
eight T4 residues per molecule. It is stored as colloid in
the thyroid acini.
Thyroid Hormone
The hormone attaches to specific nuclear receptors.
Step 6: Utilization Then the receptor-hormone complex binds to the speci-
When necessity arises, thyroglobulin is taken up from the fic region of DNA.
acinar colloid, into the cell by pinocytosis (Step 6, Fig. ~(')t>b!..t
45.13). METABOLIC EFFECTS ..x
[ OF THYRO!Q_HORMONES LN ·
Step 7: Hydrolysis
i. The hormone exerts action o n ~ I of the
The T4 is liberated by hydrolysis by specific proteases. body. Calorigenic effect o r ~rnog'e!1e~i9 is the
This activity is marke~ly enhanced by TSH. This hydroly- major effect of thyroid hormone. TI'ltsinermogenic
sis is depressed by iodide and therefore potassium effect is mediated by uncoupling of oxidatjye pbas.-
iodide (Kl) is used as an adjuvant in hyperthyroidism. In phorylation.
a genetic disorder, abnormal Tgb is synthesized, result- ii. Basal metabolic rate (BMR) is increased. Thyroxine
ing in deficient proteolysis and deficiency of thyroxine. increases cellular metabolism.
To = - \J B Chapter 45: Hormones and Growth Fae
-\-'ve- .,
iii. Earliest effect of T4 is stimulation of RNA synthe- TRH Response Test
sis and consequent increase in protein synthesis.
The{IRH administration will stimul~ the production of
Higher concentration of T3 causes protein catabo-
TSH. If the hvpothaiamo-pjtujtary-thyroid axis is normal,
lism and negative nitrogen balance.

\oiJ
the T 3 and T 4 secr_£tions will b~ ased~ normal
iv. Loss of body weight is a prominent feature of
\ response is observed in:
yroidism.-,C 1,-ll: .
J
hyp~ a. Hyp ro·dism: The negative feedback effecY-ni.. "n">u
v. GI eogenesis and car fl drate oxidation are of hi;;verpowers the stimulant effect of TRH~
increased. Glucose tolerance test shows rapid Here the thyroid hormone levels are elevated.
absorption. fji,\ (1;;'\ b. Hv.po~rism: The pituitary could not respond
vi. Fatty acid metabolis~ increased. Ch~ es~ to TRH.nthese cases the plasma thyroid hormon(I)
degradation is increased and hence cholesterol levels are subnormal.
level in blood is decreased, which is another hall- c. An exaggerated response is observed in primary
mark of hyperthyroidism. \-\~ , ()).)"'ooh, hypothyroidism since the n~ ve feed back effect
\'.o.\-, \7~'\:. >~'ct - oLT4 is reduced.
Thyroid Function Tests
c<ffe21esterol
.@say of Hormones
In hypothyroidism, cholesterol level in blood is incre-
The plasma levels of T 4 and T3 levels may be meas-
ased. It is not diagnostic, because hypercholesterolemia
ured by any of the techniques enzyme-linked immu-
is seen not only in hypothyroidism, but also in diabetes
nosorbent assay (~ . chemiluminescent immu- mellitus, hypertension, obstructive jaundice and nephro-
noassay (CLIA) orfiuorescent immunoassay (FIA) for tic syndrome. However, cholesterol level is a useful
the diagnosis of thyroid diseases. In hyperthyro1aism, index in monitoring the effectiveness of the therapy
thyroid hormone levels are increased. Both T3 and in thyroid conditions. Cholesterol level is increased in
T4 levels are increased, while TSH is reduced due to hypothyroidism, because cholesterol carrying lipopro-
feedback inhibition. In hypothyroidism, T3 and T4 are tein degradation is decreased.
reduced; but TSH levels are increased due to (\ick ~f
®
feed6ack effecU But when hypothyroidism is due to Detection of Thyroid Antibodies
hypothalamic or pituitary defect, then TSH, T3 and T4,
In Gcave'.s, disease, the resence of thyroid stimulat-
all are decreased .
ing Tmrminoglobuli TSlg) also known as long acting
® thyroid stimulator ( is seen cjrcu!atjan. The
Free T3 (fT3) and fT4 LATS can bind to TSH receptors on thyroid gland and
The free hormones are the really active molecules. produce stimulation which is not under feedback con-
Nowadays, very sensitive ELISA techniques are avail- trol. The TS lg is an antibody generated against the TSH
able to quantitate this free fraction. The free T4 consti- receptor.
tutes only 0.03% of the total T4, whereas free T3 forms The measurement of anti-TPn tibodies is useful
.a=:==~~
0.3% of total T3. in autoimmune thyroiditis (~ s). Anti-
© thyroglobulin antibodies are measured in thyroid cancer
Plasma TSH ~e.e..\o.Q'le. as well.
In primary hypothyroidism, TSH level is "elevated due
to lack of feedback. But in secondary hypothyroidism,
Abnormalities of Thyroid Function
TSH, T3 and T4 levels are low; this could point to a In 1835, Robert James Graves and in 1840 Carl
pituitary or hypothalamic cause. Hyperthyroidism due Adolph Basedow described hyperthyroidism (Graves-
to primary thyroid disease has high T3 and T4 levels, Basedow disease). In 1915, Kendall (Nobel Prize,
but suppressed TSH levels. Hyperthyroidism due to 1950), isolated thyroxine. Emil Kocher was the first
pituitary cause is indicated by high TSH, T3 and T4 surgeon to excise thyroid gland to treat goiter in 1883.
levels. For his contributions in thyroid pathology he was
38 Section F: Advanced Biochemistry

.
'
TABLE 45.8: Laboratory findings in hyperthyroidism TABLE 45.9: Laboratory findings 1n hypothyro1d1sm
.. , , , Plasma Response T, and T,in
r,4 TSH toTRH blood TSHin blood Response to TRH
Grave's disease Increase Primary Decreased Increased Exaggerated
High increase Decrease Nil
hy pothyroidism response
Toxic goiter Increase High increase Decrease Nil Secondary Decreased Decreased No response
T3 toxicosis T3 increase Increase Decrease Sluggish hypothyroidism
T4 normal "
Excess intake Increase Mild increase Decrease Sluggish tumors, pituitary surgery or irradiation, infiltration, Shee-
of thyroxin
han's syndrome and isolated TSH deficiency. Hypotha-
=
lamic diseases causing secondary hypothyroidism are
awarded Nobel Prize in 1909. Diseases of the thyroid tumors, trauma and infiltration.
are the most com mon afflictions involving endocrine Most common cause is primary thyroid disease,
system. The most common types of thyroid diseases often seen in autoimmune thyroiditis, leading to myxe-
are hyperthyroidism (excess secretion ), hypothyroi- dema in adults. Women are more affected than males.
dism (decreased secretion) and goiter (enlargement Symptoms are lethargy, tolerance to heat, cold intole-
of thyroid gland). Goiter may or may not be associated rance, slow heart rate, weight gain, dry coarse skin, slow
with abnormal function, e.g. euthyroid goiter (diffuse responses and sluggishness. Table 45.9 shows the lab
enlargement); nodular goiter which may lead to hyper- findings in hypothyroidsm.
function , or iodine deficiency goiter which may result In ch~n.~itPQlll!Yroidism reduces mental and
in hypothyroidism. physical retardation, known retinis . The TBG may
be elevated due to maternal hypefestrogenism and
HYPERTHYROIDISM therefore total T4 and T3 may be normal. The lack of
Patients have an increased rate of metabolism, weight feedback will give elevated TSH level also. Prompt diag-
loss, tachycardia, fine tremors, sweating, diarrhea, nosis and treatment are important c ~ since
emotional disturbances, anxiety and sensitivity to heat. aoy_,dela): in starting replacement may lead to irrever,Sible
damage. Maternal hypothyroidism may also cause con-
=--
Table 45.8 summarizes the laboratory findings in com-
mon types of hyperthyroidism. genital hypothyroidism in the newborn.

Secondary hyperthyroidism: It is due to diseases of


i uthyroid Goiter
pituitary or hypothalamus. It is seen in TSJ-i,-J-ecre_,ting
pituitary adenoma, thyroid hormone resi~an~ tyn- Iodine ,Mficiency may lead to euthyroid goiter. There
drome, chorionic gonadotropin secreting tumors and in is rais~ TSH level which would roduce conti~d
gestational thyrotoxicosis. stimulation of gland leading to h r lasi and
Primary hyperthyroidism : It is due to diseases of thy- Hormone levels are seen in the lower limits of the
roid gland. It is seen in Graves' disease toxic multino- normal values.
dular goiter, thyroid carcinoma, TSH receptor mutation , . . .
Struma ovarii (teratomas of ovary), iodine excess. @ Congenital Hypothyro1d1sm
Congenital hypothyroidism is seen in approximately 1 in
HYPOTHYROIDISM 4000 newborns. Thyroid hormones are very low or com-
Primary hypothyroidism: It is due to diseases of thy- pletely absent from birth. Symptoms include puffy face,
roid gland. It is seen in autoimmune hypothyroidism dullness, thick protruding tongue, which get worsened with
(e.g. Hashimoto's thyroid_)Jis), thyroidectomy and radia- time. Infants may have choking episodes, constipation ,
tion therapy. Congenital hypothyroidism is seen in iodine dry brittle hair, jaundice, floppiness, sluggishness and
deficiency and in TSH receptor mutation. sleepiness. Many hospitals routinely screen all newborn
Secondary hypothyroidism : It is due to diseases of for this disease. Treatment consists of oral administration
Pituitary or Hypothalamus. Hypopituita rism is caused by of thyroid hormgne. If untreated, child will have mental
Chapter 45: Hormones and Growth Factors 639

the genesis of obesity. The major p,eptides included in


this group are Ghrelin, glucose-dependent insulinotropic
polypeptide (GIP), Somatostatin (S:S), Neuropeptide Y
and glucagon-like peptide (GLP). Tlhese are described
in the alphabetical order. There is an intestinal lipid-
induced gut-brain neuronal axis to regulate energy
I Robert Carl Emil T Edward homeostasis. See also discussion 0111 regulators of appe-
James Adolph Kocher Kendall
Graves Basedow NP 1909 NP 19!',n tite, in Chapter 35.
1797-1853 1799-1854 1841- 1917 1886-1972
Adipose Tissue Derived Hormones
and growth retardation. Newborn screening helps to These include peptide hormones like· Leptin, Adiponectin
totally avoid this condition. and Resistin that can regulate the energy intake, storage
Non-thyroid illnesses may alter the levels of thyroid and metabolism. Initially these were called adipokines
hormones. Therefore , care should be taken before arriv- (mediators of endothelial function and inflammation
ing a diagnosis of thyroid dysfunction. involved in atherosclerosis), but presently they are more
often referred to as adipose tissue derived hormones.
GUT HORMONES
~ DIPONECTIN
- --
There are more than 30 peptides expressed within the
digestive tract, making the gut the largest endocrine It is an adipokine; a peptide with 2~~4 amino acids. It is
organ in the body. These are identified as regulatory pep- produced exclusively by adipose tissue. The circulating
adiponectin has pronounced effects, on the metabolism
tide hormones, peptide neurotransmitters and growth
of both carbohydrates and lipids in liver and muscle.
factors. Several of these peptides were first identified in
It promotes the uptake and oxidation of fatty acids by
the central nervous system; and they have subsequently
myocytes, but blocks the synthesis of fatty acids and
been found in endocrine cells of the gut. Important fac-
gluconeogenesis by hepatocytes. At the same time,
' tors among them are shown in Table 45.10.
uptake and metabolism of glucose by muscle and liver
are favored. The hormone plays a role in the suppression
Hormones Regulating Digestion of Food of the metabolic derangements that may result in type
The gastrointestinal peptide hormones are synthesized 2 diabetes, obesity, atherosclerosis and non-alcoholic
and secreted in the gastrointestinal tract. Their action fatty liver disease (NAFLD). The plasma level of adipo-
is mainly on gastrointestinal secretions, digestion and nectin parallels with the HDL level. Low levels of both
absorption of nutrients and food intake. These gut hor- are seen in metabolic syndrome and diabetes mellitus.
mones are gastrin group (Gastrin, Cholecytokinin); Decreased adiponectin is associated with polycystic
Secretin like peptides [Secretin, Vasoactive intestinal ovarian syndrome (PCOS).
polypeptide (VIP), Glucagon, GLP-1 , GLP-2, Glicentin,
Cholecystokinin (CCK)
GIP]; Pancreatic polypeptide group of hormones (Pan-
creatic polypeptide, Neuropeptide Y), Gaunylin, Seroto- Formerly called pancreozymin, this peptide hormone is
nin. They regulate the rate of secretion and composition secreted by C cells of duodenum and jejunum and has
33 amino acids. The major effect is o,n stimulation of gall-
of digestive juices. They are derived from a common
bladder contraction and secretion of bile. It decreases
precursor peptide and shows sequence homology espe-
appetite. The main stimulus is the ingestion of food con-
cially at the C terminal fragment.
taining lipids. CCK also stimulates the secretion of pan-
creatic enzymes.
Peptide Hormones
Regulating Food Intake Glucose-Dependent
The discovery of a group of peptides secreted by the GIT lnsulinotropic Polypeptid,e
which influences food intake has led to the understand- The old name was gastric inhibitory polypeptide.
ing of the relation between hunger and satiety signals in The active GIP has 42 amino acids. It is secreted by the
640 Section F: Advanced Biochemistry

TABLE 45.1O: Important peptide hormones of gastromtest,nal tract


Name Site of Origin Structure Function
Cholecystokinin Cells of duodenum, 33 amino acids Stimulates gallbladder contraction and bile flow, increases
(CCK) jejunum secretion of d igestive enzymes from pancreas
Enkephalins CNS, stomach, Pentapeptides (2 forms, met- They bind to opiate receptors, opiate-like actions
d uodenum, enkephalin and Leu-enkephalin
gall b ladder
Gastrin Gastric antrum, 17 amino acids Stimulat es acid and pepsin secretion, also stimulates
duodenum pancreatic secretions
Ghrelin Stomach and 28 amino acids; acylated on Ser3 Increases appetite, stimulates NPY release, energy
hypo thalamus with n-octanoic acid homeostasis, gastric secretion and emptying, insulin
secretion
Glucagon Alpha cells of pancreas 29 amino acids Increases lipid mobilization; glycogenolysis
Glucagon-like L cells in ileum and Two forms: 31 amino acids, GLP-1 Potentiates glucose-dependent insulin secretion, inhibits
peptide (GLP-1) colon (7-37) glucagon secretion, inhibits gastric emptying
Glucose- K cells of duodenum Polypeptide of 42 amino acids Inhibits secretion of gastric acid, enhances insulin secretion
dependent and proximal jejunum
insulinotropic
poly peptide (GIP)
Insulin Beta cells of pancreas A and B chains; 21 + 30 amino Increases glucose uptake and utilization, increases
acids, total 51 amino acids lipogenesis
Neuropeptide CNS, N cells of ileum, 36 amino acids Inhibits glucose stimulated insulin secretion; controls
Tyrosine, NPY adrenal, pancreas feeding behavior and energy homeostasis; inhibits gastric
motility and gastric acid secretion. Levels increase during
starvation to induce appetite
Secretin S cells of duodenum, 27 amino acids Stimulates pancreatic bicarbonate secretion so that gast ric
jejunum HCI is maintained and optimum pH is maintained for
the action of pancreatic enzymes. It also inh ibits gastric
secretion
Somatostatin (55) Delta cells of pancreas, 14 amino acids variety by Inhibits release of numerous gut pept ides, e.g. CCK, gastrin,
gut and hypothalamus hypothalamus; 28 amino acids by secretin, motilin, GIP; Insulin and glucagon
gastrointestinal tract
Vasoactive Pancreas, 28 amino acids Relaxes smooth muscles of GI, stimulates pancreatic
intestinal peptid e hypothalamus bicarbonate; inhibits acid and pepsin secretion;
(VIP) neurotransmitter in peripheral autonomic nervous system

neuroendocrine K cells of the duodenum and proximal Glucagon-like Peptides


jejunum. It increases glucose-mediated insulin release.
The hormone regulates glucose and lipid metabolism Insulin production is 70% greater when glucose is
by increasing the release of insulin through an incretin administered orally than when similar blood concen-
effect. Response to GIP is defective in type 2 diabetes trations were reached after intravenous administration
mellitus. of glucose; this increment is due to intestinal factors,
termed incretins. The most important incretin is GLP1 .
Gastrin GLP is a good target for treating diabetes mellitus. GLP1
increases the level of insulin resulting in significant
It is secreted by G cells of antral mucosa of the stomach reduction in circulating levels of glucose.
and proximal part of duodenum. The biologically active
portion of gastrin is the C terminal pentapeptide which is
Ghrelin
also available as a synthetic peptide, called pentagas-
trin. The major effect of gastrin is stimulation of HCI sec- Ghrelin stimulates the release of growth hormone. The
retion. Excessive gastrin secretion occurs when gastrin name ghrelin is derived from growth-hormone release.
producing tumors are present. This condition is referred It is secreted by oxyntic cells of stomach and duodenum.
to as Zollinger-Ellison syndrome, resulting in very high Ghrelin stimulates arcuate nucleus to release neuropep-
levels of acidity and chronic gastric ulcers. tide Y (NPY). The action of NPY is to enhance appetite.
Chapter 45: Hormones and Growth Factors 641

Production of ghrelin is increased by fasting, hypoglyce- EPIDERMAL GROWTH FACTOR


mia and leptin. Conversely, inhibition of ghrelin production
RECEPTOR {EGFR)
is exerted by food intake, hyperglycemia, and obesity.
Ghrelin increases appetite, and decreases insu- Also called ErbB-1 (HER1 in humans), it is the cell-sur-
lin secretion . The level is high before a meal but falls face receptor which binds extracellular protein ligands.
rapidly after taking food. Mutations of EGFR are implicated in about 30% of all
epithelial cancers. The identification of EGFR as an
Neuropeptide Y oncogene has led to the development of anticancer anti-
It is secreted by the N cells of ileum and also those pre- bodies, directed against EGFR, including gefitinib and
sent in entire nervous system , adrenal gland and pan- erlotinib for lung cancer, and cetuximab for colon cancer.
creas. The action is to inhibit glucose-stimulated insulin
Erythropoietin (EPO)
secretion.
Otherwise called hematopoietin or hemopoietin, it is
Secretin a glycoprotein hormone that controls erythropoiesis, or
Secretin stimulates secretion of pancreatic juice having red blood cell production. It is produced by the peritubu-
a high bicarbonate conten t, that maintains the optimum lar capillary endothelial cells in the kidney. It is used in
pH for the action of pancreatic enzymes by neutralizing treating anemia resulting from chronic kidney disease
gastric HCI. It acts synergistically with CCK. and myelosuppression induced by chemotherapy or
radiation.
Somatostatin
HER2/NEU
SS is the main inhibitory peptide of the GI tract. SS inhi-
bits the production of gastrin, CCK, secretin, VIP, insulin, Also known as ErbB-2, which stands for "Human Epi-
pancreatic enzymes and secretion of electrolytes into dermal growth factor Receptor 2" and is a protein giving
the intestine. higher aggressiveness in breast cancers. It is encoded
within the genome by HER2/neu, a known proto-onco-
Vasoactive Intestinal Polypeptide gene. Approximately 15-20% of breast cancers have an
amplification of the HER2/neu gene or overexpression of
A potent secretagogue of NaCl and water in the intestine
its protein product, leading to increased disease recur-
and pancreas. It is a neuropeptide released by enteric
neurons. Functions of VIP are (a) neurotransmitter in rence and worse prognosis. Trastuzumab (Herceptin) is
the central and autonomous nervous system , (b) vaso- the monoclonal antibody which will block the activity of
dilatation and relaxation of smooth muscles of the gut, HER2. This drug is used in treatment of breast cancers
(c) secretion of water and electrolytes from the pancreas showing over-expression of HER2.
and gut as well as (d) release of hormones from the
hypothalamus, pancreas and gut.
PLATELET DERIVED
[ GROWTH FACTOR (PDGF)
l_gROWTH FACTORS It regulates cell growth and division. In particular, it plays
The modern definition of a hormone is that, it is syn- a significant role in blood vessel formation (angiogen-
thesized by one type of cells and transported through esis), the growth of blood vessels from already exist-
blood to act on another type of cells. There are hun- ing blood vessel tissue. Uncontrolled angiogenesis is a
dreds of polypeptides that come under this category; an characteristic of cancer. PDGF has provided a target for
exhaustive survey of all of them is out of the objective protein receptor antagonists to treat disease.
of this textbook. These are local hormones, or signal
Vascular Endothelial
molecules produced at the local sites, and therefore are
generally called as "factors". Growth factors are gene-
Growth Factor (VEGF)
rally described in Chapter 48. Some important ones are The normal function of VEGF is to create new blood
described below. vessels during embryonic development, after injury, and
646 Section F: Advanced Biochemistry
45-14. What is the function of FSH? It increases uptake of iodine; enhances the oxidation
FSH stimulates growth of ovarian follicles in females of iodine to iodide; and favors the hydrolysis of thyro-
and spermatogenesis (Sertoli cells) in males. globulin to produce T4.
45-15. Which is the precursor of steroid hormones? 45-25. What are the functions of thyroid hormones?
Cholesterol. Calorigenic effect. BMR is increased. Basal meta-
45-16. What are the hormones produced from proge- bolic rate (BMR) is increased by increasing the cellular
sterone? metabolism.
Corticosterone; Aldosterone; Testosterone; Estrogens. 45-26. Deficiency of thyroxine results in?
45.17. Cholesterol has how many carbon atoms? Myxedema and cretinism.
27. 45-27. What are the thyroid function tests?
45-18. Cortisol has how many carbon atoms? Assay ofT3 and T4; free T3 and T4; plasma TSH; TRH
21. response test: thyroid antibodies.
45-19. Testosterone has how many carbon atoms? 45-28. What are salient features of hypothyroidism?
19. Decreased T3 level; increased TSH level; lethargy;
45-20. Estrogen has how many carbon atoms? hypercholesterolemia; weight gain; decreased BMR.
18. 45-29. What are the characteristic features of hyperthy-
45-21 . What is the immediate precursor of estrogens? roidism?
Testosterone. High T4 levels; increased rate of metabolism, weight
45-22. What is the precursor of thyroxin? loss, tachycardia, fine tremors, sweating, diarrhea,
Tyrosine. emotional disturbances, anxiety and sensitivity to heat.
45-23. What is the ratio of T4 and T3 in blood? 45-30. What is the cause for Grave's disease?
Blood T4 is 70 times more than T3. Auto-antibodies binds to TSH receptor, mimicking con-
45-24. What is the function of TSH? tinuous stimulation ofTSH.
!

lmmunochemistry

Chapter at a Glance

The learner will be able to answer questions on the following topics:


0 Immune response 0 M ultipl e myeloma
0 Effector mechanisms 0 Bence-Jones proteinuria
0 Structure of immunoglobulins 0 Primary and secondary immune response
0 lmmunoglobulins G, M, A, D and E 0 Lymphokines and interleukins

Historical Perspectives
Introduction of cowpox vaccination by Jenner in 1798 paved the foundation for immunization procedures. This does not mean
that he had discovered the procedure. For 10 years, he was in Punjab, India as a doctor, when he came to know about the
inoculation procedure available throughout India from time immemorial. Jenner has developed and simplified the procedure.
Rabies vaccine was discovered by Pasteur in 1881 . Emil von Behring (Nobel Prize, 1901) introduced therapy with antibodies,
especially against diphtheria. In 1908, Nobel Prize was awarded to Paul Ehrlich for explaining the antibody production and to
Metchnikov for discovery of phagocytosis. Life saving blood transfusion was made possible by the pioneering work on human
blood groups, the ABO and Rh system by Karl Landsteiner (Nobel Prize, 1930). By employing specific antibodies, nano-
gram or picogram quantities of biologically important substances can be estimated. This is termed radioimmunoassay. Radio-
immunoassay was first described by Rosalyn Yalow (Nobel Prize, 1977). Peter Doherty and Rolf Zinkernagel were awarded
Nobel Prize in 1996 for their work on the specificity of cell mediated immune response. Bruce Beutler and Jules Hoffman were
awarded Nobel Prize in 2011 for their discoveries concerning the activation of immune system. Nobel Prize was awarded to
Ralph Steinman in 2011 for his work on dendritic cells in immunity.

Edward Emil Adolf Paul Henry Karl Daniel Niels K Jerne Rosalyn
Jenner Behring Ehrlich HDale Landsteiner Bovet NP 1984 Yalow
1749-1823 NP 1901 NP 1908 NP 1936 NP 1930 NP 1957 1911- 1994 NP 1977
1854-1917 1854-1915 1875-1968 1868-1943 1907- 1992 1921- 2011
648 Section F: Advanced Biochemistry
mainly in the paracortical areas of lymph nodes and
periarteriolar sheaths in the spleen. In peripheral blood
80% lymphocytes are T-cells, and 15% are B-cells.
Certain other cells originated from bone marrow and
processed by the Bursa of Fabricius in avians, are called
B-cells. The Bursa equivalent organs in human beings
Peter Doherty Rolf Bruce Beutler Ralph
NP 1996 Zinkernagel NP 2011 Steinman are gut associated (including Peyer's patches) and lung
b .1940 NP 1996 b. 1957 NP 2011 associated lymphoid organs. lmmunoglobulins are
b. 1944 1943-2011 secreted by Plasma cells belonging to the 8-lympho-
cytes. The 8-cells govern the humoral immunity.
Immunology is one of the rapidly advancing branches of Clonal selection: lmmunoglobulins of different
medical science. Small pox has been completely eradi- specificity are available on the 8-cell surface. When an
cated from the world by 1985; this is a triumph of im- antigen is introduced, the antigen selects out that parti-
munology. Three salient features of immunological reac- cular cell carrying the specific antibody. This results in a
tions are: Recognition of self from nonself or foreign series of divisions of that cell and a clone of cells are pro-
substances; specificity of the reactions, and Memory
duced. These cells are fi nally differentiated into plasma
of the response.
cells. This is the antigen dependent clonal selection.
When injected with 100 diffe rent proteins, the ani-
A particular clone of cells secretes antibodies of the
mal will produce 100 different antibodies; this is called
same specificity.
specificity. If a person belongs to A group, antibodies
against B group are seen in circulation. There is an extra Effector Mechanisms
N-acetyl group in antigen A; this is the only molecular
The following are the immunological effector mecha-
difference between A and B antigens. Immune system nisms by which foreign cells are destroyed or particles
is exquisitely specific to recognize even this small dif- are removed:
ference at molecular level. If the same antigen is intro-
duced for a second time, body will react immediately; Cell Mediated Immunity
this memory is the basis of vaccination.
The following are the major activities of T-lymphocytes.
A. Immunity against infections: T-cells mediate effec-
Antigens
tive immunity against bacteria, such as mycobacte-
Certain components of the cell membranes act as ria, many viruses and almost all parasites.
specific antigens. They will be different from person to 8. Rejection of allograft: When an organ (heart, kidney)
person in chemical composition and three dimension- is transplanted from one person to another, it is
al structure. Hence, the immunocompetent cells could called al log raft. Body tries to reject such transplanted
recognize the self from nonself. Any substance which organs, mainly by T-cell mediated mechanism.
invokes an immunological response is an antigen or C. Tumor cell destruction: Although other mecha-
immunogen. Antibody response will usually be selective nisms are also involved in killing tumor cells, T-cell
against specific spatial configurations on the antigen, activity is the predominant one.
which are called antigenic determinant sites, known as D. Helper function: T helper (Th} cells are a sub-
epitopes. group of cells which carry CD4 determinants on the
cell surface (CD = cluster determinant). They are
Immune Response necessary for optimal antibody production by plasma
The lymphocytes generated from the bone marrow, cells and for generation of cytotoxic T-cells. They
passed through and processed by the thymus gland, are selectively destroyed in AIDS (see Chapter 47).
are then called T-lymphocytes. They can directly kill the E. Suppressor function: T suppressor (Ts) cells are
target cells and are the effector cells for the cell- CD8 positive cells. They downregulate the activities
mediated immunity (CMI). The T-lymphocytes are found of both T- and 8-cells.
Chapter 46: lmmunochemistry 649

Humoral Immunity Antigen binding Heavy chains

Antibodies are produced by plasma cells. These are


immunoglobulins, described in detail below.
2

Macrophages
A Phagocytosis is the nonspecific mechanism by which
body tries to eliminate invading organisms. Foreign mate-
rials are ingested by the phagocytes and later digested Light chain
intracellularly. The myeloperoxidase present inside the
phagocytes destroys the bacteria (see Chapter 30). When
Complement activation
a foreign particle enters the body, the macrophages
phagocytose it, and present the antigens to the lympho- Heavy chain Heavy chain
cytes.
Cell binding

ISTRUCTURE OF COOH COOH

IJ.M.M.UN OGLOBU LINS _ _ Fig. 46.1: lmmunoglobulin molecule.


(NH 2: Amino terminal end; COOH: Carboxy terminal end; Constant
lmmunoglobulin is abbreviated as lg. The terms gamma regions are shown as dark; VH: Variable heavy region; VL:
Variable light chain; CH: Constant heavy region; CL: Constant light
globulin and immunoglobulin are not synonymous. Gam- region. Chains are connected by disulfide bridges. shown as S-S-
ma globulin is the term describing its mobility in electri- linkages).
cal field. Most of the immunoglobulins have the gamma
mobility; but some abnormal lg molecules may move
terms for these regions on the light chain; while VH and
along with beta or even with alpha globulins. lmmuno-
CH specify variable and constant regions on the heavy
globulin is a functional term, while gamma globulin is
chain (see Fig. 46.1 ). The first 108 amino acids in light
• a physical term. The antibody reacts with antigen very
chains and first 118 amino acids in heavy chains consti-
specifically. In 1962, Rodney Porter and Gerald Edel-
tute the variable region. Here the amino acid sequence
man independently proposed the structure for immuno-
can vary in H and L chains, so that the body could
globulin molecule, for which both of them were awarded
synthesise enormous varieties of different antibodies.
Nobel Prize in 1972.

Different Classes of lmmunoglobulins


Heavy and Light Chains
lmmunoglobulin G (lgG)
The structure 0f lgG molecule is shown in Figure 46.1. It
is made up of 2 heavy (H) chains and 2 light (L) chains, lgG contains two heavy chains and two light chains;
combined through disulfide bridges. Heavy chains are heavy chains being of gamma type (Figs. 46.1, 46.2 and
2-3 times bigger than light chains. Depending on the Table 46.1). lgG is the major antibody; it constitutes
.. heavy chain make up, the immunoglobulins are differen- about 75-80% of total immunoglobulins in circulation .
It is the antibody seen in secondary immune response.
tiated into 5 major classes. These are lmmuoglobulin G
(lgG), lgM, lgA, lgD and lgE. lgG is made up of y (gamma) It can pass from vascular compartment to interstitial
heavy chain; lgM has µ (mu) heavy chain; lgA has space. It can cross placental barrier, and protects the
a (alpha) chain. The light chains are either K (kappa) or newborn child from infections. These maternal antibodies
A (lambda) in all the classes. are seen in neonatal circulation up to 2-4 months.

Variable and Constan t Regions /mmunog/obulin M (lgM)


Both the heavy and light chains contain relatively vari- lgM are macroglobulins. Five subunits, each having 4
able (V) and constant (C) regions with regard to their peptide chains (total 10 heavy chains and 10 light chains)
amino acid composition. VL and CL are the general are joined together by a J-chain polypeptide (Fig. 46.2).
650 Section F: Advanced Biochemistry

It can combine with 5 antigens simultaneously, and so in liver, reaches the intestinal muco:sal cells, where it
lgM is very effective for agglutinating bacteria. Being a combines with lgA dimer to form the siecretory lgA which
large molecule, it cannot come out of vascular space. is then released.
lgM are the predominant class of antibodies in primary
response. Natural antibodies are lgM in nature . Thus, lmmunog/obulin E (lgE)
a person having blood group A antigen will have anti-B
antibodies in his circulation (isohemagglutinins). These They are cytophilic antibodies. They mediate allergy
are produced without any known antigenic stimulation , (Greek, allo =other; ergon =work), hJtpersensitivity and
and hence called natural antibodies. These lgM anti- anaphylaxis. They have the property to fix on mast cells
bodies cannot cross placenta, and therefore the fetus, and basophils. When certain antigens such as penicillin
even though it carries an incompatible antigen, is protec- are injected a few times, lgE class antibodies are produced
ted from natural antibodies of the mother. which anchor on mast cells. When tlrle same chemical
is injected next time, the antigens fi:x on such antibo-
lmmunog/obulin A (lgA) dies, causing mast cell degranulatio n, and release of
lgA usually are dimers (total 4 heavy chains and 4 light histamine and slow reacting substance. This leads to
chains). The J chain connects the dimers (see Fig.46.2). vasodilatation, hypotension and bron chiolar constric-
They are the secretory antibodies seen in seromu- tion. This is the basis of penicillin anaphylaxis, hay fever
cous secretions of gastrointestinal tract, nasopharyn- caused by fungus, asthma by pollen and urticaria by
geal tract, urogenital tract, tears, saliva, sweat, etc. The absorbed food elements.

dimers are stabilized against proteolytic enzymes by The role of histamine in anaphyla:1Cis was elucidated
the secretory piece. The secretory piece is produced by Sir Henry Dale (Nobel Prize, 19~16). The first anti-
histaminic drug, mepyramine was prepared in 1944 by
Daniel Bovet who was awarded Nobel Prize in 1957.

fgG lgE lgD

IE_ARAPROTEINEMIAS •
Multiple Myeloma (Plasmac:ytoma)
lgM lgA When lg-secreting cells are transformed into malignant
cells, one clone alone is enormously proliferated. Thus,
lg molecules of the very same type are produced in
large quantities. This is seen on electrophoresis as the
myeloma band or monoclonal band or M band with a
sharp narrow spike (see Fig . 26.2). Multiple myeloma
is characterized by paraproteinemia , anemia, lytic bone
lesions and proteinuria. Bone marrow examination
Fig. 46.2: lgG, lgE and lgD have one basic unit each, lgM has
5 basic units and lgA has 2 basic units. Red circles represent J reveals large number of malignant plasma cells. Spon-
pieces. Green squares are secretory pieces taneous pathological fracture of weight bearing bones,

TABLE 46.1: Characterisltcs of different 1mmunoglobuhn classes


lgG lgA lgM lgD lgE
Nomenclature of heavy chain y a µ 6 E
Heavy chain domains 4 4 5 4 5
No. of basic 4-peptide units (2L + 2H) 1 2 5 1 1
Additional unit - Sand J J piece - -
Molecular weight (Dal tons) 1,46,000 3,85,000 9,70,000 1,85,000 1,90,000
Concentration in normal serum/ 100 ml 800-1200 mg 150-300mg 50-200 mg 1-10mg 1.5-4.5 µg
Chapter 46: lmmunochemistry 651

glomerulonephritis, and such autoimmune disorders


Secondary response;
mainly lgG antibodies where cryoglobulins may also be present. It is also seen
in paraproteinemias such as in multiple myeloma.

Primary
response;
Hypogammaglobulinemia
_g mainly
°E lgM antibody Decrease or absence of immunoglobulin levels may be
<(
seen in congenital or acquired conditions. Deficiency can
also occur due to loss of proteins as in nephrotic syn-
drome. A primary failure in production may occur as a con-
0 10 20 0 3 10 20 30 genital X-linked disorder (Bruton's disease). Decreased
Days after immunization production may also be secondary to diseases like
myeloma, leukemia or drug induced.
Fig. 46.3: Primary and secondary immune responses

Primary and Secondary


rib and vertebrae may occur. X-ray shows punched-out
Immune Responses
osteolytic lesions. Hypercalcemia and hypercalciuria are
therefore common features. When an antigen is injected, antibodies in blood appear
Total lg content may be very high; but useful anti- within about 10 days, reach a peak level within 20 days
bodies may be very low, so that general immunity is and response declines by about 30 days. The lgM mole-
depressed and recurrent infections are common. Mono- cules will be predominant in this primary response.
clonal gammopathies are characterized by the presence When the same antigen is reinjected into the same
of a monoclonal protein which can be detected by serum animal after a few months, the antibody response is
protein electrophoresis. The light chains are produced in quicker (within 3 days), stronger (100 to 1000 times more
f excess which is excreted in urine as Bence-Jones pro- quantity of antibody), more avid (lgG type) and more
teins (BJP). prolonged (response lasts for months). This is the secon-
dary immune response, which is due to the memory
Bence-Jones Proteinuria cells produced in the primary response (Fig. 46.3). This
Henry Bence-Jones described it in 1848. This disorder is the basis of immunization.
is seen in 20% of patients with multiple myeloma. Mono-
Molecular Structure of Antigens
clonal light chains are excreted in urine. This is due to
asynchronous production of H and L chains or due to Blood groups of RBCs express more than 160 antigens.
deletion of portions of L chains, so that they cannot com- The most important of them is called ABO system. The
bine with H chains. The Bence-Jones proteins have the RBC of the person may carry antigen A, B or AB; if none
special property of precipitation when heated between of these antigens are present, it belongs to blood group
45°C and 60°C; but redissolving at higher than 80°C 0 . Group A person's serum contains anti-B antibody.
and lower than 45°C. These proteins may block kidney Therefore, when RBCs of 8 group are introduced into
tubules, leading to renal failure. So, myeloma with the person having blood group A, there will be agglutina-
Bence-Jones proteinuria has poor prognosis. tion of the introduced cells. This is the basis of blood
transfusion.
Hypergammaglobulinem ia
Hypergammaglobulinemia are seen in chronic infections,
HLA Antigens
where antibody production is high. Examples are lep- When organs are transplanted, the donor and recipient
rosy, tuberculosis, syphilis, malaria, kala-azar, sub acute are matched for HLA (human leukocyte antigen) sys-
bacterial endocarditis. Also seen in aberrant immune tem. Joseph Murray did the first kidney transplantation
reactions such as rheumatoid arthritis, collagen disorders, in 1954 and Edward Thomas did the fi rst bone marrow
652 Section F: Advanced Biochemistry

TABLE 46.2: Lymphok,nes. interleukins and interferons


Name Secreted by Target cell/ Function
IL-lo and~ Macrophages and other antigen Induces IL-2 receptors; induces acute phase proteins I
presenting cells
IL-2 Activated T helper (Th 1) cells Maturation ofT and NK cells into LAK (lymphokine activated killer) cells
IL-4 T helper (Th2) cells B-cell proliferation, lgE expression on B cells
IL-6 Activated Th2 cells, hepatocytes Acute phase response, B cell proliferation, synergistic with IL-1 and TNF
and adipocytes on Tcells
IFN-alpha Macrophages, neutrophils Proliferation of macrophages, antiviral effects
IFN-beta Fibroblasts Antiviral effects
IFN-gamma Activated Th 1 and NK cells Antiviral effects, activates macrophages, neutrophils,
(-reactive protein (CRP) Hepatocytes, adipocytes Level of CRP parallels inflammatory process in the body. Level is
increased by IL-6. CRP enhances phagocytosis by macrophages;
TNF-alpha Macrophages Inflammation, fibrosis, pyrexia, acute phase proteins, necrosis
TNF-beta Tcells Inflammation
G-CSF Macrophages Stem cell stimulation of granulocytes
GM-CSF T cells and macrophages Stem cell stimulation of granulocytes and macrophages
MIF Tcells Activation and inhibition of mobility of macrophages

(IL: Interleukin; IFN: Interferon; TNF: Tumor necrosis factor; G-CSF: Granulocyte colony stimulating factor, GM-CSF: Granulocyte macrophage
colony stimulating factor; MIF: Macrophage migration inhibition factor).

transplantation in 1956; both of them received Nobel Prize on their function (Table 46.2). The six major types are
in 1990. But Christian Bernard, who did the first heart receptors for hematopoetins, interferons, interleukins,
transplantation in 1964, was omitted from the Nobel Prize chemokines, tumor growth factors and lg-stimulating
list. The genes of major histocompatibility complex (MHC) factors.
are involved in the recognition between self and non-self Cytokine is the generic name of a group of polypep-
antigens. In human beings, the MHC genes are present tides that may be called the hormones of the immune
on chromosome 6. There are A, C, B, D and DR loci. All system.
these loci together contain more than 150 alleles. Permu-
tation and combination of them could produce an astro- Interleukins (IL)
nomical number of variations. Hence, the antigenic Interleukins are growth factors targeted to cells of hema-
constitution of one person will be entirely different from topoietic origin. More than 30 interleukins are known, some
another one. These are main transplantation antigens, important ones are listed in Table 46.2. They are a group
responsible for rejection of allograft. Frank Macfarlane of cytokines (signaling molecules) that were first seen
Burnet and Peter Brian Medawar were awarded Nobel to be expressed by white blood cells (leukocytes).
Prize in 1960 for elucidation of basic rules in transplan- Recombinant interferons and interleukins are used
tation immunity and acquired immunological tolerance. in immunomodulatory therapy and in viral infections.

Soluble Factors
Cytokines are a diverse group of low molecular weight
¥ Clinical Case Study 46.1
proteins, that are soluble mediators of inflammatory and A 45-year-old man presented with severe back pain and
immune response. They instruct the receiving cells to weakness. He had lost 7 kg in the last 3 months. Loss
proliferate, differentiate and secrete additional cytokines of appetite is present. No history of fever. He reports
and communicate signals from one cell type to another extreme fatigue, body pain and complains that he is
to effectively regulate growth and differentiation. unable to do any work. X-ray of skull revealed punched
Since cytokines may belong to different classes out lesions. Bone marrow biopsy was done and it
based on their structure, they are classified depending showed plasma cells in excess. Serum electrophoresis
Chapter 46: lmmunochemistry 653

Alb o.1 o.2 f3 gamma

Fig. 46.4: Protein electrophoresis of concentrated urine sample; Fig. 46.5: Serum protein electrophoresis showed a spike in the
presence of a spike in the beta region beta region

was ordered on the basis of clinical features. It showed a narrow spike (Fig. 46.5). What is the most probable
an abnormal band between 13 globulin and y globulin. diagnosis? What are the potential causes of increased
Urine was positive for Bence-Jones Proteins (BJP). 13-globulins? What investigations shoiuld be performed to
What is the probable diagnosis? What is the significance characterize the protein responsible for the spike in the
of laboratory findings in this patient? 13 region of the urine?

¥· Clinical Case Study 46.2 · • Clinical Case Study 4fi.1 Answer


The patient is suffering from multiple myeloma.
A 70-year-old man with benign prostatic hyperplasia
The classic triad of multiple myeloma is marrow
was admitted for pain in left hip. He also complained
plasmacytosis (>10%), lytic bone l,esions and serum
of fatigue, weakness. Physical examination revealed
and/or urine M component. Diagnosis is confirmed by;
lymphadenopathy and hepatosplenomegaly. A hip X-ray
(1) Electrophoresis- Monoclonal antibodies seen as M
revealed a large osteolytic lesion in the left greater tro-
band (myeloma band) are seen as a sharp spike, (2)
chanter, and a bone scintigraphy evaluation showed
Bone marrow shows a large number of malignant plas-
increased activity in the same area. Metastasis from
ma cells, (3) X-ray skull shows punched out lesions, (4)
prostate carcinoma was suspected, and bone biopsies
Hypercalcemia and hypercalciuria ana seen , (5) Protein-
were performed. Ten days later, the patient was hospitali-
uria is seen as a rule and in 20% of patients it is seen as
zed in emergency department after a fall that fractured
Bence-Jones protein which is excreted in urine, (6) 132
the femoral neck. At admission, plasma sodium, chlo-
microglobulin is increased in patients with renal failure,
ride, and potassium concentrations were all within their
(7) Serum free light chains- These are superior to urine
respective reference intervals. The plasma concentra-
tests and are detected in multiple myeloma and related
tion of total protein was 5.0 g/dl , and that of albumin was
plasma cell dyscrasias.
1.5 g/dl. Hematology tests showed normocytic anemia,
In some patients with multiple myeloma, myeloma
low hemoglobin level and leukopenia. Urine protein was
• cells also secrete excessive amounts of light chains of
540 mg/24 h (reference interval, 0-30 mg/24 h). Urine
lg. The excess light chains are either Kor l. Sometimes
protein electrophoresis showed the presence of an
they are excreted as a dimer. In normal conditions, they
unusual spike in the 13 region (Fig. 46.4). lmmunoglobulin
are not excreted in urine. Free light chains can also be
quantification revealed an increased lgG value 2950
detected in serum. They are also present as K or }-.; their
mg/dl (reference interval, 700-1000 mg/dl ). lgA and ratio is also sometimes measured.
lgM were within their respective reference intervals.
Serum protein electrophoresis (SPEP) showed an
increased a1 region, along with decreased albumin and •i i• Clinical Case Study 46i.2 Answer
regions and an increased 13 region, along with a loss of Fibrinogen (found with or incomplete blood clotting) and
separation between the 131 and 132 regions because of hemoglobin (no hemolysis in the sample) are the most
654 Section F: Advanced Biochemistry

Gerald Rodney Donna/I Joseph E Frank M Peter Brian Jules


Edelman Porter Thomas Murray Burnet Medawar Hoffmann
NP 1972 NP 1972 NP 1990 NP 1990 NP 1960 NP 1960 NP 2011
b. 1929 1917-1985 1920-2012 1919-2012 1899-1985 191~1987 b. 1941

common confounding factors leading to apparent mono- bridges. Base on heavy chains, lgs are of 5 classes;
clonal peaks in the 13-globulin region of an electrophore- lgG, lgM, lgA, lgD, lgE.
sis analysis. Further, serum immunofixation electropho- 3. The light chains are either lambda or kappa.
resis revealed the presence of a monoclonal gamma 4. The antigen-binding region of the lg is the Fab por-
tion (2 in number). The rest of the molecule is the
band with no corresponding light chain. This finding is
Fe portion. lgG, the major antibody of secondary
consistent with the diagnosis of gamma-heavy chain
immune response forms about 75-80% of the total
disease (GHCD). HCD is a proliferative B-cell disorder
lgs. They can cross the placenta and is responsible
characterized by the production of monoclonal proteins for Rh isoimmunization.
with incomplete heavy chain components and without 5. lgM are macroglobulins; a pentameric structure
associated light chains. Fatigue, weakness, and lym- joined by J chains and are the predominant class
phadenopathy are the most frequent initial symptoms, of antibody in primary response. Natural antibodies
and hepatosplenomegaly are the most common physi- are of lgM type.
cal findings, which are often accompanied by anemia. 6. lgA are secretory dimeric antibodies seen in mucus
GHCD can also be associated with other lymphoprolif- secretions in the body.
erative diseases or autoimmune disorders. Patient's hip 7. lgE are cytophilic antibodies responsible for medi-
pain, subsequent fractured femur neck and associated ating allergic reaction, hypersensitivity and anaphy-
proteinuria and anemia suggested a gammopathy. laxis.
A monoclonal peak with 13 mobility was present in both 8. Malignancy of lg secreting plasma cells leads to
serum and urine electrophoresis. enormous proliferation of one clone and result in
multiple myeloma where monoclonal M band is

I LEARNING POINTS, CHAPTER 46 seen in electrophoresis. In Bence Jones proteinuria,


there is monoclonal light chain excretion in urine.
1. Immune response could be cell mediated (by T BJ proteins precipitates at 45-60°C and redissolve
cells), or humeral (by B cells/Plasma cells). Other at> 80°C. In heavy chain defective diseases, heavy
immunological effector mechanisms include anti- chains are excreted in urine.
body dependent cell mediated immunity, natural 9. Hypergammaglobulinemia occurs in chronic infec-
killer cells and macrophages. tions, autoimmune disorders and paraproteinemias,
2. lmmunoglobulins (lg) are made up of 2 heavy chains while, hypogammaglobulinemia may be congenital
and 2 light chains combined through disulphide X-linked disorder or secondary to other diseases.

PART-1: SHORT NOTE QUESTIONS


46-1 . lmmunoglobulins.
46-2. Bence Jones proteins.
46-3. Multiple myeloma.
46-4. lmmunoglobulin E and its clinical significance.
Chapter 46: lmmunoche mistry 655

PART-2: MULTIPLE CHOICE QUESTIO NS

46-1 . lmmunoglob ulin structure consists of: 46-6. M band is seen in which condition?
A. 2 light chains A. Cirrhosis
B. 2 heavy chain B. Chronic infections
C. 1 light chain and 1 heavy chai n C. Multiple myeloma
D. Heavy chain disease
D. 2 light chains and 2 heavy chains
46-7. Which has anti-viral effects?
46-2. Which is the largest immunoglob ulin?
A. TNF alpha B. lnterfeirons
A. lgM B. lgG C. IL-1 D. IL-2
C. lgD D. lgE 46-8. Which is involved in primary immune response?
46-3. Hinge region is seen in which segment? A. lgG B. lgM
A. VL C. lgA D. lgD
B. VH
46-9. Which is involved in secon d ary immune res-
C. CL D. CH
ponse?
46-4. Which immunoglob ulin has the highest serum
A. lgG B. lgM
concentration? C. lgA D. lgE
A. lgG B. lgM 46-10. Which is involved in anaphylaxis?
C. lgA D. lgD A. lgG B. lgE
46-5. Which immunoglobulln exists as a dimer? C. lgG D. lgD
A. lgG B. lgM
C. lgA D. lgE

46-1 . D 46-2. A 46-3. D 46-4. A 46-5. C 46-6. C 46-7. B


46-8. B 46-9. A 46-10. B
.::
_ _ _ _ _Chapter 47
Biochemistry of
AIDS and HIV

Chapter at a Glance
The learner will be able to answer questions on the following topics:
Acquired immunodeficiency syndrome (AIDS) 0 Immunology of AIDS
Human immunodeficiency virus (HIV) Laboratory diagnosis of AIDS

IEPIDEMIC OF AIDS at the National Institute of Health, USA and Luc Montagnier
at the Pasteur Institute, Paris. The latter was awarded
Toward the end of the 20th century, medical science
Nobel Prize in 2008. The virus was originally designated
was able to control and even conquer many of the pre-
as HTLV-11I (human T cell leukemia viirus). In 1986, it was
viously incurable diseases. But AIDS has now become
redesignated as HIV (human immunodeficiency virus). By
a disease of pandemic proportions.
retrospective analysis, it was inferred that the human pan-
In 1981, a cluster of 5 cases of Pneumocystis carinii
demic started in Africa by around 1970, and that a simian
pneumonia were reported in USA. These protozoa can
virus transformed into HIV by around HOO.
produce pneumonia only in immunodeficient individuals.
Based on the clinical manifestations, the disease was
Indian Scenario
named as acquired immunodeficiency syndrome with
acronym of AIDS. In 1983, Frarn;oise Barre-Sinoussi The virus entered in India in 1980. Fmm 1986 onwards,
(Nobel Prize, 2008) showed that the disease is due to a the Indian Council of Medical Research has started the
retrovirus (virus having reverse transcriptase enzyme). serosurveillance against HIV. The first seropositive indi-
The isolation of a virus from the lymphocytes of the AIDS viduals in India were identified in 1986. The Government
patients was done in 1984 independently by Robert Gallo of India has established the National AIDS Control
Organization (NACO) in July 1992 for the prevention
of AIDS. The adult HIV prevalence in India is now in a
declining phase.

Transmission
1. Eighty percent of the cases are sexually transmitted.
2. In about 15% of patients, the dise,asewas transmitted
Rhoab'elerts Luc Fram;oise through blood. The drug abusers usually use the
C , Montagnier Barre-Sinoussi
Gallo NP 2008 NP 2008 same needle without any sterilization for intrave-
_ _ _ _ _ _ _b_._1_9..;.
3 7_ _ _ _ _.....;..
b .....1..;.
9..;..
32 ::........,_ _ __,;,;,
b.;..
• .;..
19 ;..4;.;,7_ _ _ _ _..:.:
n..: o..:
u.:.
s...:.i:..::
nj:.::
e..:
ct=ioQ,,,,,,JJ;,-<u.~"' · 's..b.i...._.._...._ __
Chapter 47: Biochemistry of AIDS and H/V 657

stage. HIV infection can lead on to infection with other


Antibody in blood
organisms; fungi like Candida and Cryptococcus, other
viruses like cytomegalovirus and herpes simplex para-
C
,Q sites like Pneumocystis carinii, as well as bacteria like
'E
Mycobacterium and Salmonella. Cancers associated
C Virus capsid antigen with AIDS are Kaposi sarcoma and non-Hodgkin's lym-
0
u p24 in blood phoma (NHL). Cognitive impairment due to meningitis,
neuropathies and neuropsychiatric manifestations may
occur later.
II Ill
6 months 5 years 7 years

Fig. 47.1: Course of HIV infection; I =window period; II = sero-


Clinical Presentations
positive period; Ill = AIDS disease. Black line is antigen in blood;
Lymphadenopathy and fever may be seen by the end
Red dots indicate antibody response
of the second stage. The Al DS related symptoms (ARS)
are wide. Since the immunity is deficient, nonpathogenic
patients who receive blood transfusion many times.
microorganisms enter into the body and produce lesions
e.g. hemophilia patients.
in skin, gastrointestinal tract, lungs, urinary tract and
3. In the remaining 5% cases, virus may be transmit-
brain. Gastroenteritis and tuberculosis are the pre-
ted from mother to fetus through placenta. About
dominant pattern in India. In all the cases, there will be
30% of infants born to HIV positive mothers may
weight reduction.
get the infection.

Natural Course of the Disease Laboratory Diagnosis of AIDS


Window Period i. The antibodies in the blood are detected by the
When the virus enters the body, it is multiplied in the ELISA test (see Chapter 31). ELISA positive blood
body cells, but it cannot be detected easily. This is called is then confirmed with Western blot analysis (see
the window period (Fig. 47.1). The viral capsid antigen Chapter 44). In ELISA, antibody against only one
p24 can be detected in the blood during this time. antigen (gp 120) is being tested; so there is prob-
ability of false results.
Seropositive Stage ii. In Western blot analysis, antibodies against 6 dif-
After a few months, antibodies are seen in circulation. ferent components of the virus are analyzed; so it is
This is called seropositivity (Fig. 47.1 ). During this confirmatory.
period, the person is completely normal. However, this iii. T-helper count is lowered in blood. The normal level
person is a carrier of the disease, and can transmit the is more than 400 per cubic mm (cumm) of blood.
disease to others. About 10% of seropositive individuals In AIDS patients, the level is always below 300.
progress to the 3rd stage of AIDS within 5 years, 50% As the disease progresses, the helper cell count is
within 10 years and 90% within 15 years if no proper correspondingly lowered.
treatment is given. iv. By real time PCR (RTPCR) (see Chapter 44), the
For each AIDS patient, there are 100 seropositive number of HIV particles in blood can be estimated.
persons in the general population. A value of less than 5000 copies per ml of blood
has good prognosis, while a count more than 1 lakh
AIDS Disease per ml means very bad prognosis.
The third stage is when the clinical manifestations start. vi. Nucleic acid testing (NAT) involves amplifica-
By this time, the immune status of the individual is tion and detection of one or more specific target
depressed. Therefore, commensal microbes will start sequences located in specific HIV genes, GAG,
multiplication inside the body. Patient usually suc- POL and ENV. Detection limit is 200 to 400 copies
cumbs to death within about 2 years after entering this per ml. By this test, window period is shortened to
658 Section F: Advanced Biochemistry

with complementary sequence (minus strand) is pro-


MA (p17) duced. Then RNA strand is hydrolyzed by RNase-H.
Keeping the DNA minus strand as the template, a com-
plementary (plus strand) of DNA is synthesized. This
DNA double strand (proviral DNA) migrates into the
RNA two
copies
nucleus of the host cell . The viral DNA is then integrated ..,
into the host cell DNA by the action of the viral integrase
(p32). Retroviruses do not replicate in nondividing cells.
RT (p66/51 )
Similarly HIV also requires activation and division of T
cells for the viral particle synthesis. The viral genes are
IN (p32) PR (p10) transcribed and translated by the host cell mechanisms.
p = protein having molecular weight in kO, e.g. p17 = protein
Later the gag and pol genes produce gp120 and gp41 .
having molecular weight 17,000. gp = glycoprotein with
molecular weight in kO, e.g. gp120 = glycoprotein with mol. wt. These are then enveloped on the virus DNA.
120,000. RT = reverse transcriptase; IN = integrase: PR =
= =
protease. SA surface antigen; TM transmembrane protein;
HIV Genes
MA= membrane antigen; CA= capsid antigen.

Fig . 47.2: Structure of HIV There are 3 structural genes (gag, pol and env), 3 regu-
latory genes (tat, rev and nef) and 5 accessory genes
(vif, vpr, vpu, vpt and tev/tnv) in between long terminal
less than 2 weeks. Viral load is monitored during
repeats (LTR).
treatment by RT-PCR.
Immunology of AIDS
HUMAN IMMUNODEFICIENCY VIRUS The CD4 (T-helper) lymphocytes are decreased in num-
Structure ber, leading to immunodeficiency. The gp120 surface
unit could specifically attach with CD4 molecule present
HIV belongs to the retrovirus group. They are RNA-con- on the surface of T-helper cells. Therefore, HIV prefer-
taining viruses that replicate with the help of the reverse
entially enters into the T-helper cells and they are lysed.
transcriptase (RT) or RNA dependent DNA polymerase.
Since T-helper cells play a pivotal role in the immuno-
A schematic representation of the structure of the virus
logical system, their deficiency will lead to suppression
is shown in Figure 47.2. The virus has two copies of
of almost all the immunological effectors. T-helper (CD4)
single stranded genomic RNA. The protein components
count is less than 400 cumm of blood. T-killer cytotoxic
are named after its molecular weight. For example, pro-
activity is reduced. When all the effector mechanisms
tein having molecular weight 32,000 Daltons is called
p32. Other details are shown in Figure 47.2. of immunity are thus paralyzed, opportunistic pathogens
get entry into the body.
Virus Entry
Genetic Heterogeneity of Virus
The binding of HIV with target cell is through a receptor
Two types, HIV-1 and HIV-2 are characterized; HIV-1 is
mechanism. The gp 120 of the virus envelope will spe-
more virulent, infective and predominant. It is seen all
cifically bind with CD4 molecule on the surface of target
over the world. In India, both types are reported. There
cells. Thus CD4 acts as a receptor for the virus. The
are many subtypes of HIV-1 . Different subtypes may be
CD4 molecules are present on the surface of T-helper
seen in the same patient. Moreover, each type exhibits
cells and therefore helper cells receive the maximum
remarkable microheterogeneity. There is high mutation
attack of HIV.
rate in the virus. Such mutations accumulate to produce
the various types, strains and microvariations. About
Replication of HIV
15% of amino acids in the envelope gp120 are highly
After entry, the viral RNA is acted upon by the reverse variable. Therefore, the antigenicity of the virus also
transcriptase (RT). Based on the RNA, a DNA strand varies, and virus can escape from the immune attack.
Chapter 47: Biochemistry of AIDS and HIV659

~ NTI-HIV DRUGS last few months. On examination , he had multiple


enlarged lymph nodes. Several nodules were present
i. Reverse transcriptase (RT) inhibitors; nucleoside on the skin of chest and arms. Body temperature was
analogs: Dideoxy nucleosides are nucleosides, 40°C, respiratory rate was 40 breaths/min and respi-
where oxygen is absent from both 2' and 3' positions ration was shallow. Chest radiogra1ph showed diffuse
of the ribose group. The DNA chain is produced by pneumonia. What is the possible diagnosis? What are
,. 3' ,5'-phosphodiester linkages. In the dideoxynucleo- the laboratory tests to be done in this patient? What is
side, there is no hydroxyl group in the 3' position the biochemical basis?
and hence chain is terminated. So, they inhibit RT
of the HIV. The drugs used are AZT (dideoxythymi-
dine, azidothymidine, Zidovudine); ddl (dideoxyino- • · Clinical Case Study 47.1 Answer
sine , didanosine, Zidanosine, etc. Patient might be suffering from ac1quired immunodefi-
ii. RT inhibitors; non-nucleoside analogs: These agents ciency syndrome (AIDS); there is collapse of immune
make the enzyme inactive. Examples are Delavir- system leading to a series of viral, fungal and protozoa!
dine, Nevirapine. infection which are otherwise not seen. Kaposi's sar-
iii. Protease inhibitors: They are extremely selective coma and Pneumocystis carinii infection are typical
for HIV protease. So, they block final assembly and features of HIV infection, but are rare in India.
package of HIV particles. Examples include Saqui- Infection is transmitted when virus enters blood or
navir, Ritonavir, lndinavir and Nelfinavir. tissue and comes in contact with suitable host cells, clas-
iv. A combination of drugs (HAART, highly active anti- sically CD4 lymphocytes. Tissues and cells which are
retroviral therapy) will reduce the virus load and susceptible include B lymphocytes, monocytes, macro-
prolong the life of the patient for many years. phages, specialized macrophages like alveolar macro-
phages in lungs, Langerhans cells in dermis, gl ial cells,
Prevention microglial cells and follicular dendritic cells from tonsils.
! Damage caused to CD4+ lymphocytes include (1)
Public education and awareness are the only means to T4 cells decrease and T4: TB ratio reverses, (2) decrea-
limit the spread of HIV infection. Since the major method sed cytokines, (3) cell mediated immune system is
of transmission is through sexual contact, avoidance of dampened, (4) cell mediated and humoral immunity
extramarital relationships will decrease the chances of is affected, (5) AIDS patients are unable to respond
spread. All the blood samples should be tested for the to new antigens, (6) there is polyclonal activation of B
presence of HIV antibodies before transfusion. All sur- lymphocytes leading to hypergammaglobulinemia which
gical instruments should be properly sterilized. Dispos- is nonspecific, (7) monocyte macrophage system is dep-
able syringes and needles are to be used and destroyed ressed, (8) activity of NK cells and Tc (T cytotoxic) cells
immediately after use. Boiling for 10 minutes will inactivate are affected, (9) all these features lead to decreased
the virus. Ordinary autoclaving at 120°c for 20 minutes immunity and patient becomes susceptible to life-threa-
is effective to sterilize instruments and gloves. Blood tening opportunistic infections and malignancies.
Laboratory tests include; (1) Total leukocyte and lym-
spills can be decontaminated by washing with 1%
phocyte count-Decreased, (2) T ciell subset assays-
sodium hypochlorite solution, containing 10,000 ppm
Absolute CD4+ count < 200, T4:T8 count reversed, (3)
chlorine. Heat sensitive instruments may be decontami-
Platelet count-Decreased, (4) lgA and lgG- lncreased,
nated by immersing in 2% glutaraldehyde (cidex) for (5) Diminished cell mediated immunity as evidenced by
3 hours. Global strategy aims at reducing sexual trans- skin tests, (6) Lymph node biopsy, (7) Demonstration
mission by half and eliminating vertical transmission. of p24 antigen (p24 capture ELISA assay), (8) Screen-
ing tests-include ELISA, rapid tests (Dot blot assay,
particle agglutination tests and simple tests based on
•ii· Clinical Case Study 47.1 ELISA), (9) Supplemental tests-Western blot test, indi-
A 32-year-old man presented with complaints of chills rect immunofluorescence test, and (10) Demonstration
and breathing difficulty. He had lost 12 kg weight since of viral nucleic acid-by PCR and RT-PCR.
660 Section F: Advanced Biochemistry

LEARNING POINTS, CHAPTER 47 4. Laboratory investigation for the presence of HIV


are ELISA to detect presence of gp120, Western
1. Upon entry of the human immunodeficiency virus
blot to detect the presence of six components of
(HIV), it multiplies within the cells and cannot be
detected easily in this period. This is called "window the virus. RTPCR is used to estimate viral load.
period". 5. The CD4 present on the T helper cells acts as a
2. Seropositivity indicates presence of antibodies receptor for the virus.
against the viral antigen. 6. Hypervariability of gp120 makes it difficult to deve-
3. This may also be regarded as the carrier state of lop a vaccine against HIV.
the disease.

PART-1 : SHORT NOTE QUESTIONS


47-1. HIV transmission. 47.4. Anti-HIV drugs.
47-2. AIDS. 47-5. Prevention of AIDS.
47-3. Laboratory tests for detection of HIV infection. 47-6. Genetics of HIV virus.

PART-2: MULTIPLE CHOICE QUESTIONS

47-1 . AIDS is caused by what type of virus? 47-3. All the following are structural genes of HIV, except
A. Adenovirus B. Adeno associated virus A. ~t B. gag
C. Retrovirus D. Parvovirus C. pol D. env
47-2. What is the most definitive test for detecting HIV? 47-4. Which anti-HIV drugs is a protease inhibitor?
A. ELISA B. Western blot A. Zidovudine B. Nevirapine
C. RTPCR D. T-helper cell count C. lndinavir D. Didano:sine

ANSWERS OF MULTIPLE CHOICE QUESTIONS


47-1. C 47-2. C 47-3. A 47-4. C

PART-3: VIVA VOCE QUESTIONS AND ANSWERS

47.1 . What is the full form of AIDS? 47-6. What is the normal T-helper cell ,count?
Acquired immunodeficiency syndrome. More than 400 per cumm of blood.
47-2. What is the cause for AIDS? 47-7. Which molecule binds to CD4 re;ceptor on HIV?
Human immunodeficiency virus (HIV). gp120.
47-3. What type of virus is HIV? 47-8. What are the structural genes of HIV?
HIV is a retrovirus, an RNA virus. gag, pol and env.
47-4. A patient is Infected with HIV, but is.asymptomatic; 47-9. Which is the commonest treatmEmt for HIV?
what is this called Combination of drugs, highly active anti retroviral therapy
The window period. (HAART).
47-5. Is the window period dangerous? 47.10. How to handle blood spills?
Yes, he can transmit the disease as he is a carrier. Use 1% sodium hypochlorite solution.
t - -_ _ __ Chapter 48

Biochemistry of Cancer

Chapter at a Glance

The learner will be able to answer questions on the following topics:


Mutagens and carcinogens Tumor markers
Oncogenic viruses Anticancer drugs
Oncogenes and oncosuppressor genes Tumor immunology
0ncofetal antigens

The term "cancer" is derived from Latin word "cancrum" have occurred. Thanks to the surveillance by the immune

-
or Greek "karkinoma", that is equivalent to Sanskrit term ~ tern, these aberrant cells are usually destroyed. As
"karkitakam", which means "crab". The disease is so nces, the number of mutations accumulate,
called because of swollen veins around the area, resem- hence the st · tical probability of the incidence of can-
ble a crab's limbs. Indian Medical Science had identified cer is increased. wonder, cancer is a disease of old
cancer, gave the name "arbuda", which literally means age, especially after years.
the number 108 , identifying the extreme cellularjty of the 'l. Cancer is the second most common cause for death
cancer tissue. The International Union Against Cancer in developed countries, second only to cardio~ascular
(UICC; Union Internationale Cantre le Cancer) has defi- diseases. When the average life-expectancy is less, as
ned cancer as a disturbance of growth characterized by in the case of India, the death due to cancer is also low.
excessive proliferation of cells without apparent relation
Mutagens
to the physiological demands of the organs involved.
Oncology deals with the etiology, diagnosis, treatment, Any substance which increases the rate of mutation can
prevention and research aspects of cancer. also enhance ~ t.e of incidence oif cancer. Therefore,
all carcinogen's ar~ mutagens. Examples are X-rays,
ETIOLOGY OF CANCER gamma-rays, ultraviolet rays. Some human cancers are
caused by chemicals. These may be introduced into the
All cancers are multifactorial in origin . They include body by means of (a) occupation (aniline, asbestos),
genetic, hormonal, metabolic, physical, chemical and (b) diet (aflatoxins) or (c) lifestyle (smoking). Chemical
environmental factors. Most human cancers are ~ - carcinogens act cumulatively. Tobacco, food additives,
taneous. coloring agents, and aflatoxins are common carcino-
All cancers originate usually from one aberrant cell, gens in our environment.
which goes on to multiply and produce a tumor mass. Thousands of chemicals are known mutagens and
One mutation occurs out of 106 cell divisions. By the time carcinogens. A selected small list of chemical carcino-
a person reaches adulthood , about 1026 cell divisions gens is given in Table 48.1. Methyllcholanthrene is a
( 1t'4'1 ~ )
662 Section F: Advanced Biochemistry

TABLE 48.1: Some chemical carcinogens


Polycyclic aromatic hydrocarbons Benzopyrenes,
Cholanthrenes, Dimethyl-
benzanthracene (DMBA)
Fig. 48.1: Solid tumc1r in a mouse induced
Aromatic amines N-Methyl-4-aminoazobenzene by injection of methylcholanthrene (from
{MAB) _..._________,, author's laboratory)
Nitroso compounds Dimethylnitrosamine
~ ...J>SFIF~
Natural compounds Aflatoxins S-transferas~ s involved in thB detoxification of
various carcinogens, including cigarette smoke. About
powerful carcinogen. only nanograms are sufficient to 5% of population are lacking in GST. Smokers who are
produce a tumor in a mouse (Fig. 48.1 ). devoid of GST are more prone to deveilop lung cancer.

Aflatoxins Progression
They are a group of chemically related compounds The biological history of a tumor shows progression of
synthesized by the fungi, Aspergillus f/avus. The mould malignancy. Cells with faster rowth r:~te have a selec-
grows on r¥ , wh~t and grou9 dnut, when kept in tion advantage. Thus, c Is with increased malignant
damp conditions. The fungi may grow in cattlejodder, Ch3/.acti3 are pro ressive selected. Familial adeno-
which may enter into human body through the cow's matgifs polyposis is a typical exam[Ple for multistep
milk. Aflatoxins are powerful carcinogens, which pro- progression. Mutations in the APC gene are inherited
duce hepatomas. from parents. By the time the patient becomes adult,
there will be different dy_sglastic ~ 1rant crypts in the
Cigarette large intestine. Some of the cells will ~Jet somatic muta-
t_Lung canci, is associated with the habit of cigarette tions in the K-Ras gene; these will progress to form
smoking. Cigarette contains many carcinogens, the adenomas. Further mutation in TGF qene or p53 gene
or Bax gene will give the push for the development of 1
most important group being benzo(a)pyrenes. Other
~r-..e.. important deleterious substances in cigarette smoke malignancy.
Co are nicotine, carbon monoxide, nitrogen dioxide and
carbon soot. Statistically, it is estimated that one ciga- Action of Chemical Carcino,gens
N~,.
·c; rette reduces 10 minutes from the lifespan of the indi-
vidual. The incidence of lung cancer is increased to 15
Mechanisms of action of chemical carcinogens are: (a)
Carcinogens are generally elec!J:2Bllllits (molecules
times more in persons smoking 10 cfgifettes per day deficient in electrons); they r~adily attack nucleoghilic
and 40 times more when smoking 20 cig1~~es per day. (electron rich) groups of DNA, (b) Carcinogens may
Thus, WHO suggested the slogan "Cigarf e smoke is bind covalent!_y to cellular DNA. N2, 1\13, and N7 atoms
injurious to health". Moreover, non-smok~g spouse of a of guanine are highly prone to addition of carcinogen
heavy smoker will have 5 times more probability to get groups, (c) These changes will lead to DNA alterations,
lung cancer than a non-smoker. This effect of "passive in spite of DNA repair, with increased probability of
smoking" made the International Union against Cancer mutations.
(UICC) to change the slogan to "Your smoking is injuri-
ous to our health". Physical Carcinogens
loral cancer} s strongly associated with chewing of
X-ray, gamma-ray and UV-ray may cause: (a) formation
obacco Pr !3 of pyrimidine dimers, (b) af)urinic sites with consequent
\ Alcohol · take increases the risk of oral, pha n- break in DNA, and (c) formation of free radicals and
geal, esophageal and Ii.Per cancers. Di~ igh i otal fat superoxides which cause DNA break, leading to somatic
an)icholesterol, increases the risk of cnlbn, ast and mutations. Exposure of X-ray in fetal life will increase
p~ ate cancers. the risk of leukemia in childhood. In p,opulation studies,
An often asked question is, why only some smokers 1 @d per _y_ear will increase the cancer incidence by
are getting cancer and not all smokers? Glutathione 40/million peo,rle per year.
Chapter 48: Biochemistry of Cancer 663

Ludwik Denis Jozsef Peyton Renato Michael Harold E


Gross Burkitt Marek Rous Dulbi~cco Bishop Varmus
1904-1999 1911-1993 1868-1952 NP 1966 NP 1975 NP 1989 NP 1989
1879-1970 1914---2012 b. 1936 b. 1939

Anti mutagens
i. These are substances which will interfere with tumor
promotion. Vitamin A and carotenoids are shown Papova virus groulP

to reverse precancerous conditions. CCloe>&) SV-40 DNA Mouse Sarcoma

ii. Vitamin E acts as an antioxidant, preventing the Papilloma DNA Rabbit Papilloma

damage made by fr~ radicals and sup~ roxides. Marek DNA Chicken Lymphoma

iii. Vitamin C regularly given to persons ~ king with Retrovirus type C

prevented the production of new cancer Gross RNA Mouse Leukemia


cases. Rous RNA Avian Sarcoma
iv. Tubers, beans and leafy vegetables are shown to Retrovirus type B
interrupt tumor promotion. Bittner RNA Mouse Mammary tumor

• V. Curcumin, the yellow substance in Turmeric is


known to prevent mutations. argued that the filtrate could contain only viruses (and
vi. The beneficial effect of the fiber content of the diet not bacteria or cancer cells) which produce a tumor.
• is described in Chapter 35. Low protein, low fat, diet After a long time, at the ripe age of 87, Rous was awar-
decreases the risk of cancer in animal studies. ded Nobel Prize in 1966.
vii. Flavinoids are phytochemicals that possess anti- A list of important oncogenic viruses in animals is
mutagenic properties . Phenolic compounds found shown in Table 48.2. The list is only representative and
in fruits lil(egrapes, strawberries, walnuts, etc. are is far from exhaustive. In 1930s, an aggressive form of
found to ~)lntimutagenic. Green tea is shown to lymphoma (cancer arising from lymphocytes) was seen
be effective a~~m9ke1nd_y_ced mutg_tions. as an epidemic in chicken in North America, wiping out
almo'i_~o-thircls of the total poultry population. For the
I_QNCOGENIC VIRUSES first time, a c~ ~s ,ecce_Eted !£._be ~..!l,smissi~ e@
Another etiological factor of carcinogenesis is the inte- Soon, the Marelk virus was isolated which was proved to
gration of viral genes into the host DNA. Thus, the genes be the etiologic agent. By the end of 1940s. an effective
of the virus become part and parcel of the cellular DNA. vaccine was pnepared and the disease was controlled.
The drive for multiplication by the viral genome over- A list of possible oncogenic viruses in man is given in
rules the re ulato ch cks lances of the cellular Table 48.3.
me9heD,iSJTI. So, there is uncontrolled multiplication of the Burkitt in 1964 reported a type of lymphoma seen
cells. This is called transformation by oncogenic virus. mainly in African children. Epidemiology suggested
Rous in 1911 demonstrated that sarcoma in avians a strong possibility for a transmitting agent for the
can be transferred from one animal to another by inject- Burkitt's lymphoma (BL). In 1969, Epstein reported
ing the soluble fractions. In 1944, Gross finally proved that all the bio1Psies of BL when placed in tissue cul-
that viruses could be oncogenic. A homogenate of mice ture for some tiime, generated the viral particles which
tumor was prepared, passed through bacterial filter, could be seen under electron microscope (Barr was the
and the supernatant was injected into another mouse. A technician who first perfected this technique). The new
new tumor developed at the site of injection. Gross virus was name,d as Epstein-Barr (EB) virus.
664 Section F: Advanced Biochemistry

TABLE 48.3: Human oncogenic viruses


Virus Abbreviation Associated human cancer
Epstein-Barr virus EBV Burkitt's lymphoma (BL);
Nasopharyngeal carcinoma
(NPC)
Human papilloma HPV Uterine cervical carcinoma
Fig. 48.3: HPV and host interaction. Left slide, HPV infects one
virus
cell in the basal layer in human uterine cervix. Middle slide, within
Hepatitis B virus HBV Hepatoma few weeks, HPV spreads to most of the cells; replicates, and a
lesion is manifested. Some host cells will escape the in~ ction. In
99% cases, the lesion subsides within 6 months to 2 years time.
Right slide, In 1% cases, virus is integrated into the host DNA.
1 2 3 4 remains dormant. After 10-30 years, cancer is developed. Here
malignant cells are shown to break the basal layer and invade into
+ + - + surrounding tissues

BOX 48.1: Oncogens and oncogenes are different


1. Oncogen is the chemical which produces ca.!1cer.
2. Oncogens are the chemicals that produce cancer.
3. Oncogene is the genecausing cancer.
Fig. 48.2: zur Hausen showed that HPV is integrated into the host 4. Oncogenes are the genes causing cancer.
DNA of human cervical cancer cells. Host cell DNA is isolated, cut 5. Oncogenes are writt~ h small l,~tters, and antioncogenes
with restriction endo-nucleases, electrophoresed, and hybridized a-;;.,ritten'with capital letters.
with radioactive probes of HPV DNA. The DNA from patients are 6. The gene present in normal cell is named with prefix,..c:- (to
seen to hybridize with virus probes, as shown +ve in the slide. show that it is in the cell), whereas the corresponding gene pre-
Details of DNA hybridization are given in Chapter 44
refix v- (standing for virus).
\ -(\i \
e~~S.:·\T'he lymphoma progression is through 3 different
3-t" ~ vents. The first step is infection with EBV which speci- 95% protection from infection of HPV, thereby reduc ing
~ callv jnfects B lymphocytes. The B cells are now the chances of developing cervical cancer (Figs. 4 8.2 I

immortalized, that is, they can be cultured indefinitely in and 48.3).


artificial medium.
The second step is the chromosome transloca- lQNCOG~NE_S_ _ _ --c--_

~ i9n usually from chromosome 6 to 14 Th e chromo- Oncogenes are Normal


some 14 contains gene for immunoglo,bulin heavy cha in. Constituents of Cel Is
The transposing regio n in c hromosome 8 contains the
o ncogene c-myc. The third step is the activation of These a re genes capable of ca us ing cancer (Box 48.1).

~ -myc oncogene, with consequent malignancy. Michael Bishop and Harold Varmus, pioneers in the
HPV (Human Papilloma Virus) is the most com- oncogen~ rch were awarded Nobel Prize in 1989.
mon sexua lly tra nsmitted infectio n in adults. It has a ci r- A defin~i:2of for an oncogene was first d emonstrated
0 cular double stranded DNA. More tha n 100 HPV types
are known. H,r" ti'.pes 16 a nd..__W are associated with in avians but a
--- defjcje
in Rous sarcoma virus. The full viru~rod uces sarcoma
~~--.....__
ntJQ_a pa rticular
human ute rine cervical c r; they cause 70% of all gene, cou ld not cause the disease. He nce, this g ene
cervical cancers. a rald zur Ha uzen (Nobel Prize, 2008) was named as ~ e ne...abbreviated as Src. How-
showed the HPV DNA in the cancer cells. HPV infects ever, the same DNA sequences are avail~ le in normal
e_e_!the lial cells in the cervical mucosa; the virus avian cells also. This reveals th at no rma l cells do contain
plies a nd lyses the host cells , causing a lesion., $ ~ DNA sequences similar to vi ra l oncogenes. To distin-
of such cases healing occurs w ithin 6 months to~ years. g uish these two genes, they a re denoted as V-src (viral
But in about 1 % c~ses, the HPV DNA is integrated into gene) and C-src (cellular g e_!l.e ). The onco_g_enes...eresenf
some of th e host ce lls. After about 10- 30 ys)ars, these i~ s ar~ proto-;,;cog;,:;es.
cells develop inv_asive cancer. Vac~s against high risk A few important human proto-oncoge nes are abl, e rb ,
HPV 16 and 18 types a re 110w developed that provide myc, sis , etc.
Chapter 48: Biochemistry of Cancer665

Point mutation of proto-oncouene: The ras gene


produces a protein termed P21 (Mol. wt. 21 ,000), that
suppresses the activity of adenyl cyclase. Adenyl cyc-
lase has a key role in cellular response to hormones
(see Chapter 45). C-ras oncogene ~,Rduct is a mutated
version of P21. p.GTPase activit,\~ decreased lead-
4 5 ing to continuo~ ctivity of adenyl cyclas~
it1)~a5l ~()
Antioncogenes or -1.1-I
Oncosuppressor Genes CoJ·~~ -
These are the genes, which normally protect the indivi-
Oncogene activation or
deletion of oncosuppreseor gene dual from getting the cancer. When the gene is deleted or
J mutated, cancer results. Some important oncosuppres-
sor genes are RB (retinoblastoma), WT (Wilms tumor),
FAP (familial adenomatous polyposis), BRCA1 (familial
breast cancer) and p53. Antioncoge1nes are written with
capital letters, whereas oncogenes are represented by
Mallgnant transformation small letters.
The p53 is so named, because it is a protein with
Fig. 48.4: Unified concept of carcinogenesis. 1= ionizing radia-
tion; 2= chromosome translocation; 3= chemical carcinogens; 4= molecular weight 53 kDa. It is a tumor suppressor pro-
oncogenic viruses; 5= spontaneous mutations tein and prevents formation of cancer. So, p53 has been
described as "the g~ dian angel o-f the genome," and
the "master watchman ," referring to i1ts role in conserving
Proto-oncogenes are Regulatory Genes
stability by preventing genome mutation. It can activate
Products of many oncogenes are polypeptide growth DNA -re~ proteins when DNA has sustained damage.
factors , e.g.sls gene produces platelet derived growth It can initiate apo~ s (programmed cell death), if the
factor (PDGF). This factor is required for normal wound
DNA damage proves to be irreparable. People who
healing. Some of the products act as receptors for
growth factor, e.g. e_!:b-B produces receptor for EGF (epi-
ff
inherit only 6ne ctjonal cnp0,f thH p53 will most likely
develop tumors eady adulttJood. The gene can also
thelial growth factor). Some other oncogene products act
be da~ ged in cells by mutagens (chemicals, radiation ,
on key intracellular pathways involved in growth control,·
or ;;--u~ s)~asmgtheuncontrolled cell division.
e.g. srCJllQill!.Ct, a membrane-bound enzyme, phos,eho-
More than 50% of human tumors contain a mutation or
rylates a sp~cjfic tyrosine resi9u~ cascad,e
activatjon of cellular events. Receptors for EGF, insulin, deletion of the p53 gene.
PDGF, etc. are also activated by src-product protein. The retinoblastoma protein (Rb) Molecular weight:
Proto-oncogene activation has been demonstrated in 105,000. is a tumor suppressor pro1tein that is dysfunc-
different types of human tumors. tiooal in many types of cancer. The function of Rb is to
prevent excessive cell growth by ~ biting the cell cycle.
Many Factors Activate Oncogenes Only when both alleles of the RB gene are deleted
(homozygous), retin~ stoma results.
The oncogenes also provide an explanation for the
multifactorial origin of cancer. Thus viruses, chemical BRCA gene mutations: A blood test is done for
carcinogens, chromosome translocati~ amm;;:;ys, women who are likely to have BRCA mutations (those
spontaneous mutations. and all such factors may con- with a f~ily history of breast _c2 ncer). Mutations are
verge into one biochemical abnormality, the activation of seen in ~ of breast cancers and 10-15% p of oy~n
oncogenes. This would lead to malignancy. This unified cancers and more than 50~~l1je_nts with positive
theory is depicted in Figure 48.4. family hisJ,QJY...,o f J2reast or .£,_-!arian cancer.
666 Section F: Advanced Biochemistry

TABLE 48.4: Some important growth factors


Growth factor Abbreviation Function II
Epidermal growth factor EGF Stimulates epidermal and epithelial cells 1
Transforming growth factor-a TGF-a Similac ta EGE J
Transforming growth factor-(3 TGF-(3 Inhibition of fibroblasts
Platelet derived growth factor PDGF Accelerates wound healing
Nerve growth factor NGF Growth of sensory nwrons
Insulin-like growth factor IGF- 1 ~ulfation into cartijage
Erythropoietin EP Stimulates erythropoiesis
Granulocyte macrophage colony stimulating factor GMCSF Stimulates granulocytes, monocytes, megakaryocytes
Granulocyte colony stimulating factor GCSF Stimulates granulocytes
Monocyte colony stimulating factor MCSF Stimulates monocytes
Tumor necrosis factor- alpha TNF-a Necrosis of tumor cells, proliferation of leukocytes

Growth Factors Oncofetal AntiQens


~en"' C"'
Many of the oncogenes a ~ ~f During the fetal life, a particular gene is active, and the
The gro~ factors generally cause mito- product, a pr~in is therefore produced in the cell (Fig.
sis or differentiation of tt get cells. These may be con- 48.5). Duri~ differentiation process, this gene is
sidered as local hormones. There are more than 100 suppr~ssed and therefore the protein is not present in
growth factors; a few important ones are shown in Table adult c@However, along wit~ ~
48.4. Interleukins and interferons are growth factors formation, de-differentiation odtrs, tt1e gene is dere-
pressed and the protein is again available in the cell.
released by lymphocytes/macrophages (see Chapter 46).
Such products are classified as oncofetal proteins. The
best examples are the appearance of alpha-fetoprotein
Malignant Transformation
(AFP) in hepatomas and carcinoenrioryonic antigen
When a normal cell has acquired malignant character, (CEA) in colon cancers. They generally serve as tumor
it is said to be transformed. For his pioneering work in markers.
transformation studies in tissue culture, Renato Dulbe-
cco was awarded Nobel Prize in 1975. In the cell culture, l!_UMOR MARKERS_ _
this is seen as alterations in morphology as well as They are also called as tumor index siubstances. They
changes in the alignment among the cells. Normal cells are factors released from the tumor cells, which could be
form a monolayer, while cancer cells show multilayered detected in blood and therefore indicat,3 the presence of
appearance. the tumor in the body. They are useful for the following
purposes.
Tumor Immunology a. For follow-up of cancer and to monitor the effec-
tiveness of the therapy and also to detect the recur-
All forms of treatment of cancer (surgery, radiotherapy
rence of the tumor (Fig. 48.6).
and chemotherapy) leave some residual cancer cells in
b. To facilitate detection of cancer. The presence of
the body. These are annihilated by the body's immune
tumor marker suggests the diagnosis, but caution
mechanism. Burnet (Nobel Prize, 1960) had postulated
is to be taken to rule out other non--malignant condi-
that the major purpose of immunological system is the tions.
sl!!Yeillance agaiost spontaneously occurring cancer c. For prognosis. Serum level of the marker may indi-
cells. In the tumor bearing host, appreciable level of cate roughly the tumor load, which in turn indicates
immunological reaction against the cancer is detected. whether the disease is curable or 1not.
This is because of the presence of tumor associated d. Tumor markers are sometimes elevated in nonma-
antigens (TAA) on the surface of cancer cells. lignant conditions. Not every tumoir will cause a rise
Chapter 48: Biochemistry of Cancer 667

Antigen is not expressed

nctioning gene

! • •

Embryonic cell

Fig. 48.5: Oncofetal antigen


Normal adult cell Malignant cell

TABLE 48.5: Common tumor markers


Group Nome Serum level increased in
Oncofetal pro ducts Alpha feto protein (AFP) Hepatoma, germ cell cancers
Carcinoembryonic antigen (CEA) Colorectal, gastrointest inal, and lung cancer
Carbohyd rate antigens CA-125 Ovarian ca ncer of epit helial origin
nssue antigens n ssue polypeptide antigen General cancer load
Enzymes Alkaline phosphatase (ALP) Bone secondaries
Placental t ype ALP (Regan) Lung, seminoma
Prostatic acid phosphatase (PAP) Prostate cancer
Prostate specific antigen (PSA) Prostate cancer
Neuro ne specific eno lase (NSE) Neuroendocrine tumors
Hormones and their metabolites Beta-hCG Choriocarcinoma
Calcitonin Medullary t hyroid carcinoma
Vasoact ive intestinal polypeptide Apudomas (Amine precursor uptake decarboxylation-omas)
Vanillyl mandelic acid (VMA) Pheochromocytoma and neuroblast oma
Hydroxy indo le acetic acid Carclnoid syndro me
Serum proteins lmmunoglobulins (lg) Multiple myeloma, macroglobulinemia
Bence-Jones proteins (in urine) Multiple myelo ma

in the level of its associated marker, especially in Carcinoembryonic Antigen (CEA)


the early stages of some cancers.
The CEA level is markedly increased in colorectal can-
Clinically Important Tumor Markers cers. Its molecular weight is 180 kD (Table 48.5). In
Alpha-Fetoprotein (AFP) 1965, Gold and Freedman identified the CEA. Over 50%
of persons with breast, colon, lung, gastric, ovarian, pan-
In 1963, Abelev demonstrated AFP. It is fetal albumin
creatic, and uterine cancer have elevated levels of CEA.
and has similarities with adult albumin. AFP is increased
in the circulation of patients with hepatocellular carci-
Beta Chain of Chorionic Gonadotropin
noma, germ cell tumors, teratocarcinoma of ovary and
in pregnancy with fetal malformations of neural tube Beta-hCG is synthesized by normal syncytiotropho-
(Table 48.5). In adult males and nonpregnant females, blasts (cells of placental villi). The hCG is a glycopro-
the values are less than 15 ng/L. A value of AFP above tein; it has alpha and beta subunits. The alpha subunit
300 ng/L is often associated with cancer, although levels is identical with those of FSH, TSH and LH. The beta
in this range may be seen in nonmalignant liver diseases. subunit is specific for hCG. It is increased in hydatidi-
Levels above 1000 ng/L are almost always associated form mole, choriocarcinoma and germ cell tumors
with cancer (except in pregnancy). (Fig. 48.6).
668 Section F: Advanced Biochemistry

Progesterone Receptor (PR)


...J
E Tissue that does not express the PR receptors is less
0)
C
.!: likely to bind estrogen analogs used to treat the tumor.
C
.Q Persons who test negative for both ER and PR have less
c than a 5% chance of responding to eindocrine therapy.
C
Those who test positive for both mar~:ers have greater
0
(..) than a 60% chance of tumor shrinkaige when treated
Months after with hormone therapy.
initial treatment
Treatment Recurrence
HER2/neu (or erbB-2, or EGFR2)
Fig. 48.6: Monitoring of serum level of beta-HCG in chorionic
carcinoma (The level is decreased after treatment and goes up
It is a protein that stimulates breast cancer cells to grow.
when the disease recurs) Higher than normal levels can be found in some other
cancers, too. The HER2 level is usually found by testing
a sample of the cancer tissue itself, not in blood. HER2
Carbohydrate Antigen 125 (CA-125) is positive in about 1 in 5 breast cancers. These can-
CA-125 is a tumor maker for ovarian cancers. It is a cers tend to grow and spread more aggressively than
glycoprotein with a molecular weight of 10 million; one other breast cancers. All newly diagnosed breast can-
of the biggest molecules identified. Approximately 75% cers should be tested for HER2. HER:?-positive cancers
of persons with ovarian cancer will have elevated serum are more likely to respond to certain tneatments such as
trastuzumab (Herceptin) and lapatinil) (Tykerb), which
levels (50% of persons with stage I disease and 90%
work against the HER2 receptor on bn:last cancer cells.
with stage II). Elevated levels of CA-1 25 are also found
Paraproteinemias and multiple rnyeloma are des- •
in approximately 20% of persons with pancreatic and
cribed in Chapter 46. Oncofetal proteins and tumor
digestive tract cancers.
markers are listed in Table 48.5.
Prostate Specific Antigen (PSA)
IANTICANCER DRUGS
Chu isolated it in 1980. It is produced by secretory epi-
Surgery and radiotherapy are most effective to reduce
thelium of prostate gland. It is normally secreted into
the initial tumor load. These are the prime modalities of
seminal fluid, where it is necessary for the liquefaction of treatment in solid tumors. Chemotherapy is the sheet
seminal coagulum. PSA has been found to be elevated anchor of therapy in leukemias, advaInced lymphomas,
in 60-70% patients with cancer of the prostate. Most choriocarcinoma and other widely disseminated malig-
PSA is bound to antitrypsins in plasma but some PSA nancies. The effectiveness of cytotoxiic drugs is directly
circulates unbound to protein (free PSA). Normal blood proportional to the doubling time of the tumors, and is
level of total PSA is less than 4 ng/L. Persons with a inversely proportional to the number of cancer cells.
borderline total PSA (between 4 and 10 ng/L), but who Cytotoxic drugs affect all the cells which are in the divid-
have a low free PSA are more likely to have malignant ing phase. Rapidly dividing normal c1:llls (gastrointesti-
prostate disease. nal tract, hematopoietic system, hair follicles, gonads)
are also affected by chemotherapeul!ic drugs, leading
Estrogen Receptor (ER) to toxicity. In fact, pharmacological dose and toxic dose
usually overlap in the case of these drugs.
It is a protein found in the nucleus of breast and uterine
Some important anticancer drugs are listed in
tissues. The level of ER in the tissue is used to deter- Tables 48.6 and 48.7.
mine whether a person with breast cancer is likely to
respond to therapy with tamoxifen, which binds to the Methotrexate
receptors blocking the action of estrogen . Women who It inhibits dihydrofolate reductase. Methotrexate has
are ER-negative have a greater risk of recurrence than structural similarity to folic acid, and hence will com-
women who are ER-positive. petitively inhibit folate reductase. So in presence of
Chapter 48: Biochemistry of Cancer 669

TABLE 48.6: Common anticancer drugs


Name Type Mode ofaction

Methotrexate Follc acid analogue Competitive inhibitor of dihydrofolate reductase. THFA is n!q uired for nucleotide
synthesis

6-Mercapto purine Purine analogue Inhibits the conversion of IMP to AMP


6-thio guanine Purine analogue Inhibits the conversion of IMP to AMP
Cyclophosphamide Alkylating agent Crosslinking of bases of DNA; inhibition of strand separation
M itomycin C Antibiotic Cross bridqes are formed between DNA base pairs
Actinomycin D Antibiotic Intercalates with guanine bases of DNA; prevents transcription
Vincristine and Vinblastine Alkaloids from Vinca rosea Interferes with assembly of cytoskeleton and inhibits
Stathmokinesis (spindle movement)
Adriamycin Anthracyclins Topo-isomerase mediated breaks in DNA
Etoposide Podophyllot oxin Stabilises topo-isomerase-11-DNA cleavage complexes
Cisplatin Platinum compound Forms intrastrand DNA adducts
lmatinib Monoclonal antibody Tyrosine kinase inhi bit or
Fluorouracil (FU) Pyrimid ine analog Inhibits thymidylate synthase

TABLE 48.7: Some monoclonal ant1bod1es used as anticancer is described in Chapter 44. The mechanism of action of
agents monoclonals against cancer may be!:
Name Target Used against the cancer a. The antibody marks the cancer cell and makes it
Rituximab CD20on Bcells NHL, CLL, B cell leukemla easier for the immune system to attack. The drug
Trastuzu mab HER-2/neu Breast cancer
rituximab attaches to CD20 found only on B cells;
(Hercepti n) (EGFR2, Erb-B2)
makes the cells more visible to the immune system,
Bevacizumab VEGF Colorectal, solid tumors of
kidney, breast which can then attack.
Alemtuzumab CD52 on B cells CLL b. Block growth factors. Certain cancer cells make
Cetuximab KRas Colorectal, head and neck extra copies of the growth factor receptor. This
Panitumumab EGFR Colorectal makes them grow faster than the normal cells.
lmatinib Tyrosine kinase CML Monoclonal antibodies can block these receptors
and prevent the growth signal. For example, Cetuxi-
methotrexate, tetrahydrofolic acid is not produced, which mab attaches to epidermal growth factor receptors
is necessary for incorporation of C2 and C8 of purines (EGFR) on cancer cells. Blod;ing this signal from
and CS methyl group in thymidine. Thus, there is inhibi- reaching its target on the canoer cells may slow or
tion of DNA synthesis and consequently of cell division. stop the cancer from growing.
Methotrexate is commonly employed in the treatment of c. Stop new blood vessels from forming. To attract
choriocarcinoma, which is a curable cancer. It is also blood vessels, cancer cells send out growth signals.
useful in acute leukemia. Monoclonal antibodies that block these growth sig-
nals may help prevent a tumor from developing a
6-Mercaptopurine
blood supply, so that it remainis small. The mono-
It is a purine analogue which prevents amination of IMP clonal antibody bevacizumab intercepts vascular
to AMP, so that the availability of AMP is reduced (see
endothelial growth factor (VEGF) and stops them
Fig. 38.11 ). This leads to inhibition of synthesis of DNA, from connecting with their targets (Table 4 8. 7).
and in turn cell division. It is commonly employed in
treating acute lymphoblastic leukemia.
Related Topics
Monoclonal Antibody Anti-oxidants (see Chapter 30), telomerase (see Chap-
These drugs are a relatively new innovation in cancer ter 39), cell cycle (see Chapter 42),. and apoptosis (see
treatment. Technique for monoclonal antibody production Chapter 42).
670 Section F: Advanced Biochemistry

••
St Clinical Case Study 48.1
ovarian cancer. However, it should be emphasized that
the vast majority of breast cancers aire not genetically
A 40-year-old female presented to the clinic with lump based, but occurs sporadically. BRCA2 mutations are
in right breast. A mammogram performed revealed a also associated with ovarian cancers and male breast
breast mass measuring 3 cm with numerous microcal- cancers.
cifications suggestive of breast cancer. The family his-
tory revealed that she had a sister who was diagnosed LEARNING POINTS, CHAPTER 48
with breast cancer at the age of 35. She also recalls that 1. All cancers are multifactorial in ori gin. They include
one of her aunts had died long ago with great abdominal genetic, hormonal, metabolic, pl1ysical, chemical
distension (probably ovarian carcinoma). On physical and environmental factors. Examples of physical
examination, there was a fixed and nontender mass on carcinogens are X-ray, gamma-ray, ultraviolet ray.
right breast, measuring 3 cm. Axillary lymph nodes were Examples of chemical mutagens are Aflatoxins,
palpable on the right side. Skin was not involved . The Methylcholanthrene, Nicotine.
biopsy report was intraductal carcinoma. What cancer 2. Examples of antimutagens include Vitamin A, Vita-
gene might be associated with cancer breast? What is min E, Vitamin C and Curcumin.
the likely mechanism of activation of oncogene in this 3. Example of viruses producing turmors in animals
case? are Polyoma, SV 40, Gross, Rous, etc.

e 4. Viruses possibly oncogenic in man are EBV, HPV


•i'i• Clinical Case Study 48.1 Answer and Hepatitis B.
5. Genes capable of causing cancer are termed onco-
Most likely cancer gene: Breast cancer (BRCA) gene genes, e.g. myc, src, ras, abl, erb-·B, etc.
Likely mechanism: Inhibition of tumor-suppressor gene 6. Genes which normally protect the individual from
Discussion: This patient has developed breast cancer.
getting a cancer are called Anti-oncogenes or
Her two relatives had breast or ovarian cancer prior to
Oncosuppressors. e.g. p53, pRB.
menopause. This makes BRCA gene mutation likely.
7. Examples of oncofetal antigens are alpha-fetopro-
Proto-oncogenes are normal genes that are present
in normal cells and involved in normal growth; but if tein (AFP) in hepatomas and carcinoembryonic
mutated , they become oncogenes, leading to cancer. antigen (CEA) in colon cancers
The BRCA 1 gene encodes a protein which is necessary 8. Tumor markers are useful for diagnosis and fo llow-
for DNA repair. A woman with a BRCA1 mutation has up of cancer chemotherapy, e.g . CA-125, VMA,
70% risk of developing breast cancer, and 30% risk of Placental ALP, Prostate specific antigen.

PART-1 : SHORT NOTE QUESTIONS

48-1. Antimetabolites. 48-7. Name the t umor marker most appropriate for t he
48-2. Oncogenes. following: (a) prostate carcinoma; (b) c horio car-
48-3. Oncosuppressor genes. cinoma; (c) colon cancer; (d) hepatoma; (e) pheo-
48-4. Tumor markers. c hromocytoma; (f) carcinoid synd rome; (g) bone
48-5. A lpha-fetoprotein. metastasis.
48-6. Carcinoembryonic antigen.

PART-2: MULTIPLE CHOICE QUESTIONS

48-1 . Which is carcinogenic? 48-3. Which is an oncogene?


A. Curcumin B. Vitamin A A. abl B. BRCA
C. X-rays D. Fiber diet C. P53 D. Rb
48-2. Which causes nasopharyngeal carcinoma? 48-4. Which is a tumor suppressor genie?
A. HBV B. EBV A. myc B. ras
C. HCV D. HIV C. RB D. HER-2/neu
Chapter 48: Biochemistry of Cancer671

48-5. Which tumor marker belongs to the category of A. Telomerase


oncofetal protein? B. Alkaline phosphatase
A. CA-125 B. ALP C. Dihy-drofolate reductase
C. ACP D. AFP D. Thyrnidylate kinase
48-6. Which enzyme is elevated in prostate cancer? 48-8. Herceptin inhibits which of the following?
A. Beta hCG B. ALP A. Estrogen receptor B. Progesterone receptor
! C. PSA D. NSE C. Andrngen receptor D. HER-2/neu
48-7. Which of the following enzymes is inhibited by
methotrexate?

ANSWERS OF MULTIPLE CHOICE QUESTIONS


48-1 . C 48-2. B 48-3. A 48-4. C 4l3-5. D 48-6. C 48-7. C
48-8. D

PART-3: VIVA VOCE QUESTIONS A DANSWERS

48-1 . Name anti-mutagens and anti-carcinogens. 48-5. Name s,ome commonly used tumor markers.
Vitamin A, Vitamin C, Vitamin E, Curcumin. Alpha-fe'loprotein; Carcinoembryonic antigen; Prostate
48-2. What are Oncogenes? specific antigen
Genes capable of causing cancer. Oncogenes are spe- 48-6. Alpha-f1~toprotein (AFP) level is used as a tumor
cific sequences in DNA which when expressed may marker for which cancer?
Hepatoma.
produce cancer.
48-7. What is the marker for colorectal cancers?
48-3. What are tumor markers?
Carcinoembryonic antigen.
They are factors released from the tumor cells, which
48-8. What is the significance of beta chain of human
could be detected in blood and therefore indicate the
' presence of the tumor in the body.
chorionic gonadotropin?
It is a tumor marker for choriocarcinoma.
48-4. What is clinical application of tumor markers? 48-9. What is the mechanism of action of mitomycin?
They are useful (1) to follow-up cancer and to monitor Intercalation with DNA strands
the effectiveness of the therapy, (2) to detect the recur- 48-10. What is the mechanism of action of Methotrexate?
rence of the tumor. It is a folic acid antagonist.
_ _ _ _ _Chapter 49
Tissue Proteins in
Health and Disease

Chapter at a Glance
The learner will be able to answer questions on the following topics:
D Collagen, structure and synthesis D Muscle proteins
D Abnormal collagens D Myosin, actin, troponins
D Elastin D Lens proteins and cataract
D Keratins D Prions and Alzheimer's disease

COLLAGEN large proportions in collagen. The hydroxylated amino


acid residues are of special functional :significance.
The major structural protein found in connective tissue \
is the collagen . Collagen is a Greek word which means
Synthesis of Collagen
the substance to produce glue. It is the most abundant
protein in the body. About 25-30% of the total weight of The collagen is synthesized by fibroblasts intracellularly,
protein in the body is collagen. It serves to hold together as a large precursor, called procollagen. It is then secre-
the cells in the tissues. It is the major fibrous element ted. The extracellular procollagen is ch:aved by specific
of tissues like bone, teeth , tendons, cartilage and blood peptidases to form tropocollagen.
vessels.
Post-translational Modifications
Structure of Collagen The hydroxylation of proline and lysine residues of
The tropocollagen is made up of three polypeptide collagen is a post-translational modification taking place
chains. Depending on the amino acid variations, there intracellularly. The enzyme also contains ferrous iron at
are 27 types of collagens described. Type I is the most its active site and requires a reducing agent like ascor-
abundant form, seen in connective tissues in almost all bic acid to preserve the iron in the redwced ferrous state.
regions of the body. So, vitamin C deficiency leads to poor hiydroxylation. It is
The amino acid composition of collagen is quite the major biochemical defect in scurvy (see Chapter 33).
unique. About 33% of the amino acids are glycine, that is,
every third residue is glycine. The repetitive amino acid Glycosylation of Procollagen
sequence may be represented as Gly- X - Y- Gly - X - Y -; The hydroxylated polypeptides are n13xt glycosylated.
where X and Y are other amino acids, most commonly The common carbohydrate residues added are galac-
proline and hydroxyproline. Moreover, 4-hydroxyproline, tose and glucose. The glycosylation occurs only on the
3-hydroxyproline and 5-hydroxylysine are found in fairly hydroxylysine residues.
1---- ----·-------
Chapter 49: Tissue Proteins in HHalth and Disease 673

• w
Fig. 49.1 : Triple stranded collagen fiber
2
3

---===------
5 _ _ __

4
----
F ig. 49.2: Quarter staggered arrangement in collagen fiber; each
row moves one-fourth length over the last row; the 5th row repeats
the position of the first row

Triple Stranded Helix Functions of Collagen


The collagen is a rod-like structure. Each of the 3 poly- 1. To give support to organs.
peptide chains is held in a helical conformation by wind- 2. To provide alignment of cells, so, that cell anchoring
ing around each other. The three strands are hydrogen is possible. This in turn, helps in proliferation and
bonded to each other. Glycine, because of its small size differentiation of cells.
can fit into the crowded interior of the collagen triple helix 3. In blood vessels, if collagen is exposed, platelets
(Fig. 49.1 ). For the same reason , glycine also produces adhere and thrombus formation is initiated.
a shallow groove into which other polypeptide strands
are intertwined.
Degradation of Collagen
When a solution of collagen is boiled, the viscosity Collagenases are enzymes that can degrade collagen.
of the solution decreases. The native rod like structure is Collagen is a protein resistant to attack by ordinary enzy-
altered and a protein, wi th random coil structure results. mes. The collagenase is produced by Clostridium
It is then called gelatin. histolyticum, the pathogen responsibl,e for producing gas
gangrene. The bacterium can destroy connective tissue
Quarter Staggered Arrangement barriers and this accounts for its invasiveness. Adult
human tissues do not have any appreciable amount of
The tropocollagen molecules are arranged in a "quarler
collagenase activity.
staggered array" to form the collagen fibers {Molecules
Degradation of collagen is seen when there is bone
in each row separated by 400 A and adjacent rows by
and cartilage resorption, osteoporo:sis, tumor metas-
680 A). The structure repeats after each row (Fig . 49.2). tasis, during postpartum involution of uterus, Paget's
Thus, the collagen fiber has triple stranded, quarter disease, rickets, osteoarthritis, rheumatoid arthritis, and
staggered arrangement. This arrangement helps in scurvy.
mineralization.
Abnormalities in Collagen
Crosslinks in Collagen Fibers
Osteogenesis lmperfecta
The collagen fibers are strengthened by covalent cross It is inherited as a dominant trait. It is the result of a muta-
links between lysine and hydroxylysine residues. The tion which results in the replacement: of a single glycine
crosslinks are formed by lysyl oxidase which converts residue by cysteine in Type I collage1n. Hence the poly-
these amino acids into aldehydes. Lysyl oxidase is a peptide becomes excessively glycositlated and hydroxy-
copper containing enzyme, the copper ion being located lated. The result is brittle bones leadling to multiple frac-
at its active site. lures and skeletal deformities.
In copper deficiency, reduced crosslinking of collagen
results (see Chapter 34). The older the collagen, the Ehlers-Dan/as Syndrome (EDS)
more the extent of cross linkages. The process continues, It is due to defective Type Ill collagen formation due to
especially in old age, so that the skin. blood vessels and defective lysyl oxidase or lysyl hydro:xylase. It is charac-
other tissues become less elastic and more stiff, contri- terized by weakening of collagen, loose skin, hypermo-
buting a great extent to the medical problems of the old bile and lax joints. Hyperextensibility of skin and joints
people. are the hall mark of this condition.
674 Section F: Advanced Biochemistry

Marfan 's Syndrome I band A band I band

Inherited as an autosomal dominant trait. Arachnodactyly


'zli;;,
(long digits), ectopia lentis (dislocation of lens), hyper-
extension of joints, aortic aneurysm are manifestations.
A defect in the gene, coding for a connective tissue pro-
tein , fibrillin-1 , leads to this disease.
'
Menke 's Disease
Deficiency of copper results in defective function of lysyl
oxidase, and reduced crosslinkage of collagen (see
Chapter 34 ).

Deficiency of Ascorbic Acid Fig. 49.3: Sliding and shortening of actin and myosin is the basis
It is characterized by defective hydroxylation of collagen. of muscle contraction. Compare the distance between Z lines in
the upper and lower pictures
The collagen formed is weak, leading to frag ility of blood
vessels, poor wound healing, etc. (see Chapter 33).
Keratins
Hyperhomocysteinemia Keratins are proteins present in hair, s~<in and nails, horn,
The accumulated homocysteine in this condition, reacts hoof, etc. The fibers present are called alpha keratins
with lysyl aldehydes to block crosslinking. The skeletal and matrix as keratohyalin. They mainly have the alpha
deformities, vascular and ocular defects are thus pro- helical structure. Each fibril has 3 polypeptide chains
duced (see Chapter 18). and each bundle has about 10- 12 fiibrils. The kerato-
hyalin matrix has cysteine-rich polypeptide chains which
Lathyrism are held together by disulfide bonds. The more the num-
It is due to ingestion of lathyrus sativa or sweet pea. It is ber of disulfide bonds, the harder the !keratin is.
due to a toxic agent beta oxalyl amino alanine (see
Chapter 35). This compound has been found to inhibit [ MUSCLE PROTEINS
------------
lysyl oxidase. This would interfere with formation of lysyl Striated muscle is made up of multinucleated cells bound
crosslinking. by plasma membrane called sarcolemma. Sarcomere is

I ELASTIN
the functional unit of muscle. Each muscle cell contains
myofibrils about 1 mm in diameter. lrhe myofibrils are
Elastin is a protein found in connective tissue and is the immersed in a cytosol that is rich in glycogen, ATP,
major component of elastic fibers. The elastic fibers can creatine phosphate and glycolytic enzymes.
stretch and then resume their original length. They have The functional unit of a myofibril is a sarcomere.
high tensile strength. They are found in the ligaments as The dark A bands and light I bands alternate regularly
well as in the walls of the blood vessels, especially large (Fig. 49.3). The central H zone of A band is lighter, while
vessels like aorta. One-third of the residues are glycine. the dark M line is found in the middle of the H zone. The
Praline is present in large amounts, so also alanine. I band is bisected by a very dense narrow Z line.
Hydroxyproline is present in small amounts while hydroxy- These bands are formed by variable combination of
lysine and glycosylated hydroxylysine are absent. Triple thick and thin filaments. The thick filament is primarily
helix structure is also absent. When elastin matures, myosin and thin filament contains actin, tropomyosin,
desmosine cross links are formed from 4 lysine and troponin. The Z line contains 2 actin molecules and
residues. M protein is located in the M line (Fig. 49.3).
Pseudoxanthoma elasticum: It is an inherited Thick and thin fi laments slide past each other during
defect in the formation of elastin. Clinical manifestations the muscle contraction, so that the muscle shortens by
are similar to Ehlers-Danlos syndrome. as much as a third of its original len!;ith. However, the
Chapter 49: Tissue Proteins in Heialth and Disease 675

myocardial infarction (see Chapter 6). Its level in serum


is increased within 4 hours of myocardial infarction, and
remains high for about 7 days.
Troponin-T (TnT, 37 kD) bindls to tropomyosin.
Fig. 49.4: During muscle contraction, myosin moves over actin Two isoforms of cardiac TnT, called TnT1 and TnT2 are
I
filament present in adult human cardiac tissue. Serum levels of
TnT2 increases within 4 hours of m)rocardial infarction,
lengths of the thick and thin filaments do not change during and remains high for up to 14 days. The TnT2 is 100%
muscle contraction. The mechanism is explained in sensitive index for myocardial infarction.
Figure 49.4. In the resting muscle, only the high affinity sites are
occupied by Calcium, but when ca·· is released from
Myosin sarcoplasmic reticulum, low affinity sites are also occu-
Myosin molecules contain 6 polypeptide chains; 2 iden- pied by This results in a conformational change
tical heavy chains and 4 light chains. that is transmitted to tropomyosin. The events may be
depicted as
Actin Ca .. -. troponin -. tropomyosin - , actin -. myosin

It is the major protein of the thin filaments. It is a mono-


meric protein often referred to as G-actin due to its glo-
Transduction of Chemical
bular shape. It can polymerize into a fibrous form, called Energy to Mechanical Energy
F-actin , which is a helix of actin monomer. The amount of ATP in muscle is sufficient to sustain con-
• The muscle contraction results from interaction tractile activity for less than one second. The reservoir
of actin and myosin, to form actomyosin, with energy of high energy phosphate in skeletal muscle is creatine
provided by ATP. When the two thin filaments that bind phosphate. The reaction (Lohman's re,action) is catalyzed
the cross bridges of a thick filament are drawn toward by Creatine Kinase (CK).
each other, the distance between Z lines becomes
CK
shorter (Figs. 49.3 and 49.4). This could result in the Creatine phosphate +ADP ---+ ATP + Creatine
process of contraction of muscle fibers. This needs The creatine phosphate is therefore! able to provide a
energy from hydrolysis of ATP, effected by the ATPase high ATP concentration during muscle contraction . (In
activity of myosin. athletes, it is the major source of energy during the first
4 seconds of a short sprint).
Troponins The reduced energy charge of active muscle stimu-
The muscle contraction is modulated by troponin and lates glycogen breakdown, glycolysis, TCA cycle and
tropomyosin through Ca ... which is the physiological oxidative phosphorylation.
regulator of muscle contraction . In the resting muscle, The sequence of events in the muscle contraction
the is within the sarcoplasmic reticulum. The nerve and relaxation are summarized in Box 49.1.
impulse releases Ca·• from the sarcoplasmic stores and
increases its cytosolic concentration about 10 times (1 mM Inherited Diseases due to
• to 10 mM). The action of calcium is brought about Abnormality of Proteins
by 2 proteins, troponin complex and tropomyosin locat- Dystrophin is a structural protein, attached to muscle
ed in the thin filament. cell membrane. Dystrophin links the actin of cytoskeleton
The troponin complex has 3 different polypeptide of the cell into the extracellular matrix. Mutations in the
chains. Out of this, troponin-C (TnC, 18 kD) binds calcium. dystrophin gene cause Duchenne muscular dystrophy.
Troponin-1 (Tnl , 21 kD), otherwise called "actomyosin- Mutations in the genes for the glycosyl transferases
ATPase inhibitory elemenr, binds to actin and inhibits (which add the sugar groups to proteins) are also res-
binding of actin to myosin. Troponin I is a marker for ponsible for some types of muscle dystrophies.
676 Section F: Advanced Biochemistry

BOX 49 .1: Muscle contract1on-relax,it1on events heated and denatured. In diabetes mellitus, when the blood
1. Nerve impulse releases acetyl choline (ACh) at motor end-plate.
glucose level is increased, lysine resiclues of these pro-
This ACh binds with the receptors. teins are glycated. This leads to increased susceptibility
2. Sodium-potassium conductance in neuromuscular end-plate, for sulfhydryl oxidation and consequent aggregation of the
so that a potential is generated at the end plate. proteins, resulting in opalescence and cataract. Protein
3. This Is transmitted as the action potential to m uscle fibers.
aggregates with molecular weight more than 50 million
4. Depolarization; release of calcium ions from SR \
5. Binding of calcium ions to TnC. will produce scattering of light.
6. Crosslink formation between actin and myosin In lens, the enzyme aldose reductase reduces mono-
7. Sliding of thin filaments over thick filaments; muscle contracts. saccharides to corresponding sugar alcohols; glucose to
8. Calcium is pumped back into SR.
sorbitol and galactose to galactitol. Th1~se polyols do not
9. Release of calcium from troponin.
10. Actin and myosin are separated; muscle relaxed. readily cross cell membranes and hE~nce accumulate;
causing osmotic swelling, and cons1equent disruption
of cell architecture. Thus diabetes m,ellitus (increased
I LENS PROTEINS
__ glucose in blood) and galactosemia (hifJh galactose level)
cause cataract. Drugs that inhibit aldose reductase are
India has the maximum number of blind persons in the shown to retard cataract formation in diabetic rats.
world. Cataracts and opacities of cornea are the cause
for 70% of blindness. The eyes of older people and dia- lf_RIONS
betics are prone to cataract formation. Being avascular,
The central dogma in molecular biology postulated by
lens relies on the aqueous humor for the provision of
Watson and Crick in 1953 was that g,enetic information
oxygen and essential metabolites. Uppermost part of
passes from DNA to RNA and then to protein. In general,
the lens consists of a monolayer of epithelial cells, which
this rule still holds good. In 1970, Ternin and Baltimore •
divides and differentiates to form the long fibre cells, that
showed that DNA could be synthesized from RNA by
make up the lens. These normal lens cells (but not the old
cells) possess the usual protein synthesizing machinery. reverse transcriptase. This has partly shattered the
The proteolytic activity of the lens is quite low and is central dogma . But could proteins act as an information
due to the presence of endogenous protease inhibitors. molecule? Could proteins replicate 1themselves? This
Lens tissue has a very active HMP shunt pathway, and question was considered to be heretical till a few years
has the maximum concentration of NADPH. Lens also ago, but no longer so. There are a few diseases charac-
contains high quantity of ascorbic acid. They scavenge terized by very long incubation period of many years.
the free radicals and maintain the transparency of lens. These "slow disease agents" were originally thought to
be "unconventional viruses", but now they are proved as
Crystallins prion proteins.

Major lens proteins are alpha, beta and gamma crystal- Prion Proteins: Abnormal
lins. Small quantities of delta and epsilon varieties are Tertiary Structure
also described. They undergo no replacement through-
"Prions" is the acronym for "proteinaceous infective par-
out the life of the individual. There is no turnover of these
ticles". Stanley Prusiner has described prion proteins
proteins. The proteins at the center of the lens are as old
(PrP) in 1982, who was awarded NobeII Prize in 1997. PrP
as the individual. The orderly arrangements of the mole- is a normal protein of 253 amino acicls, found in leuko-
cules make the lens proteins transparent. Alpha crystallin cytes and nerve cells. The matured prion protein (PrP)
is present not only in lens, but is seen in almost all cells has 210 amino acids. It exists as a sialoglycoprotein,
of the body. anchored on the cell surface. PrP molecules can
undergo a change in structural conformation. The altered
Cataracts
molecule is resistant to heat and proteolytic enzymes.
When there is change in three dimensional structure of The abnormal protein is called PrPsc; "sc" stands for
lens proteins, the lens becomes opaque. This is similar scrapie, the disease in which it was first isolated. Thus,
to the clear albumin becoming white and opaque when prions are proteins with correct primary structure,
Chapter 49: Tissue Proteins in Heialth and Disease 677

dead person); Carleton Gajdusek is,olated the "uncon-


ventional virus" from the affected individuals; both of
them were awarded Nobel Prize in 1976.

PrPsc Prion Diseases in Animals


Classical example of the "slow disease," is scrapie disease
in sheep. It is characterized by constant scratching;
43' alpha hel x 30' alpha helix hence the name. The disease is manifested only about
B t;i sheel n,I 50"' beta sheet
10 years after the entry of the agent. In Bovine spongi-
Fig. 49.5: Prions have correct primary structure. but have altered form encephalopathy (BSE) (mad cow disease) the
tertiary structure
brainstem is affected. It has resulted in economic disas-
ter in Great Britain in 1996. Cattle fe1~d containing infec-
but with abnormal tertiary structure. The PrP is in
alpha helical form; but PrPsc is in beta pleated sheets ted meat from sheep suffering from Scrapie caused the
(Fig. 49.5). entry of prions into millions of cows. Thus, it is obvious
that prions had crossed the species barrier from sheep
Abnormal Proteins Can Be Infectious to cattle. Could it cross to human beings? Hundreds
The infectious prion induces the nearby normal protein of thousands of infected animals have been eaten by
molecules to unfold to abnormal form. It is similar to the Europeans.
conversion of the good "Dr Jekyll" to the criminal "Mr Alzheimer's Disease (AD)
Hyde" (same person with two personalities, described in
It was first reported by Aloysius Alzheimer in 1906.
the famous novel). These abnormal proteins now convert
About 5-1 0% of people above 60 years are affected by
fu rther normal proteins into abnormal varieties, produc-
AD. As the number of aged persons is increasing, the
ing a "chain reaction" that generates new infectious
condition is becoming a major health problem in India.
materials.
It is characterized by slow progressi,on of memory loss,
Pathogenesis of Prion Diseases confusion, dementia, hallucinations, personality changes,
and finally patient enters into a vege,tative state with no
The lysosomal enzymes could break down the normal
comprehension of the outside world. Shakespeare's
PrP; but PrPsc cannot be digested. Hence, the prions
King Lear; who is losing his memory and becoming dis-
are accumulated inside the cells, and eventually the cell
oriented is a well-known example. Death occurs about
dies. As a group, they are also called transmissi ble 1o years after the first onset of the symptoms. There is
spongiform encephalopathies (TSF), because the brain no defin ite treatment now available.
becomes riddled with small holes like a sponge. Neurons
degenerate, protein deposits may accumulate as pla- Molecular Defects in AD
ques and glial cells grow larger. The pathological hallmarks of AD are neurofibril tangles
in CNS, senile neuritic plaques and cerebral amyloid
Human Prion Diseases deposition. Inflammation within the brain plays a role in

Prion diseases in human beings are Kuru, Creutzfeldt-


Jakob disease (CJD), Gerstmann-Straussler-Scheinker
disease (GSSD) (familial CJD) and fatal familial insomnia
(FFI). Cerebral cortex becomes sponge like in CJD.
Thalamus is affected in FFI. In Kuru, cerebellum is affec-
ted. It is seen in Fore aborigines in Papua New Guinea.
Stanley Baruch Carleton
The term "Kuru" means "laughing death" in the tribal
Prusiner Blumberg Gajdusek
language. Baruch Blumberg showed that the disease NP 1997 NP 1976 NP 1976
was transmitted by ritual cannibalism (eating of brain of b. 1942 1925-2011 1923-2008
678 Section F: Advanced Biochemistry

the development of Alzheimer's disease; and long-term [ LEARNING POINTS, CHAPTER 49


use of anti-inflammatory drug was found to reduce the
1. Collagen has a triple helical structure, where the
incidence of this disease . The neurofibrillary tangles
are paired helical filaments made up of Tau protein.
3 strand s are hydrogen bonded to each other and
further stabilized by hydrogen bonds between OH
.
Normal Tau is soluble and catabolized easily; but groups and water molecules, thus giving high tensile
abnormal Tau is insoluble, cannot be degraded by tissue strength to the fiber.
cathepsins and are deposited around neurons. Tau is 2. Tropocollagen molecules are arranged in a quarter
required for stabilizing axonal microtubules, the commu- staggered airray.
nication channels in nerve fibers. 3. The strength of collagen fibers is due to covalent
crosslinks between lysine and hydroxylysine resi-
Familial AD and Apo-E4 gene dues. Thesei are formed by lysyl oxidase, which is a
Cu containing enzyme.
About 30% cases have genetic background; and these
4. Abnormalities of collagen structure are seen in
are called familial AD. The genes related with Alzheimer's
osteogenesis imperfecta, homocystinuria, Marfan's
Disease are presenilin 1&2, AD 3&4 and Apo-E4.
syndrome, ascorbic acid deficiency and copper
deficiency.
Protein Misfolding Diseases
5 . Myosin mollecules act as ATPase and binds to
About 18 different diseases, most of them fatal , are asso- actin.
ciated w ith extracellular deposition of normally soluble 6. Actin is the major protein of the thin filament. Muscle
proteins, which become insoluble as amyloid deposits. contraction is effected by the sliding of actin over
myosin.
These include Alzheimer's disease, Transmissible spon-
7. Prions are proteinaceous infective particles. Prion
giform encephalopathies and familial amyloidosis. All
particles {PrP) undergo change in the structural
these diseases are due to misfolding of proteins leading
conformation and resu lt in PrPsc. PrPsc is resistant
to a toxic conformation. A common feature in the altered
to heat and proteolysis.
conformation of the protein is the change from alpha 8. Human Pric,n diseases are Kuru, CJD, GSSD an
helical to beta sheet structure which makes the proteins FFI.
and aggregates resistant to normal proteolysis. 9 . In animals prions can cause scrapie, BSE, nvCJD.

PART-1: SHORT NOTE QUES IONS


49-1. Structure of collagen.
49-2. Collagen abnormalities.
49-3. Troponins.
49-4. Lens proteins.
49-5. Prion diseases.
49-6. Molecular pathology of Alzheimer's disease.

PART-2: MULTIPLE CHOICE QUE TIONS

49-1 . Hydoxylation occurs in which of the following ami- 49-3. Glycosyla11ion of collagen occurs i n which of the
no acids in collagen? following ,residues?
A. Glycine B. Lysine A. Hydrox:yproline B. Lysine
C. Tyrosine D. Threonine C. Proline D. Hyproxylysine
49-2. Wh ich is the most common amino acid present in 49-4. Random c:oil structure obtained by boiling colla-
collagen? gen is kno,w n as
A. Glycine A. Procollagen
B. Lysine B. Gelatin
C. Threonine C. Collagen degradation product
D. Proline D. Tropooollagen
Chapter 49: Tissue Proteins in He,alth and Disease 679

49-5. Which disease is due to the toxin beta oxalyl ami- 49-8. Which troponin binds to tropornyosin?
no alanine? A. Troponin I B. Troponin C
A. Ehlers-Danlos syndrome C. Troponin M D. Troponin T
B. Marfan's syndrome 49-9. Which is associated with Alzheimer's disease?
C. Menke's disease A. Tau protein B. Amylc1id protein
D. Lathyrism C. APO-E4 gene D. All of 11he above
49-6. Functional unit of myofibril is 49-10. Which statement regarding prions is incorrect?
A. Actin B. Myosin A. They are infectious particles
C. Sarcomere D. A band B. Normal prion proteins have important functions in
49-7. Which of the following is not a component of tro- the body
ponin complex? C. It is normally anchored on the, cell surface
A. Troponin I B. Troponin C D. They have normal primary structure, but abnormal
C. Troponin M D. Troponin T tertiary structure

ANSWERS OF MULTIPLE CHOICE QUESTIONS


46-1 . D 46-2. A 46-3. D 46-4. A 46-5. C 46-6. C 46-7. B
46-8. B 46-9. A 46-10. B

' PART-3: VIVA VOCE QUESTIONS AND ANSWERS

49-1 . What is the structure of collagen? It depends on vitamin C.


It has triple stranded quarter staggered arrangement. 49-4. In ascorbic acid deficiency, what happens?
One-third residues are glycine. About 1% residues are Poor hydroxylation; defective collagen and scurvy.
proline and hydroxy proline. 49-5. What is the enzyme for cross liinkage formation?
! 49-2. What are the post-translational modifications tak- Lysyl oxidase.
ing place in collagen? 49-6. What is its clinical application?
Hydroxylation of proline and lysine residues; hydroly- Lysyl oxidase contains copper. So, in copper defi-
sis of pro-collagen. ciency lysine cross link formation is deficient, leading
49-3. Hydroxylation of proline needs what? to defective collagen.
_ _ _ _ _Chapter 50
Applications of
Isotopes in Medicine

Chapter at a Glance
The learner will be able to answer questions on the following topics:
0 Isotopes 0 Diagnostic applicati ons
0 Radioactive decay and half-life 0 Treatment applications
0 Units of radioactivity 0 Biological effects of radiation
0 Resea rch applications

INTRODUCTION convention is that mass number is written on the upper


left side of the symbol letters to denote the particular
Lord Ernest Ru therford (Nobel Prize, 1908) put forward isotope. Atomic number may be shown on the left lower
the nuclear theory of atom. Accordingly, the atoms are corner of the symbol.
composed of a central dense positively charged nucleus, For example, 1 H is normal hyd ro9en with 1 proton.
around which negatively charged electrons revolve. Sub- It is present 99.985% of hydrogen ions in nature. 2H is
atomic stable particles are proton, neutron and electron. heavy hydrogen or Deuterium. It has 1 proton (p) + 1
neutron (n). It is present only 0.015% in nature. 3 H
Isotopes
is Tritium with 1p + 2n. It is not present in nature, but
Isotopes are the elements having the same atomic may be produced artificially. These three isotopes of
number (protons) but different mass number (vary- hydrogen will react similarly in chemical reactions,
ing number of neutrons). The Greek word "iso" means because all of them contain only one ialectron.
equal and "lope" means place; that is, isotopes occu- To take another example, 160 is normal oxygen, 17 0
py the same place in the periodic table . The accepted is unstable and 180 is stable isotope.

Ernest Wilhelm Antoine Willard Pierre Marie Frederic Irene Joliot


Rutherford Rontgen Becquerel Frank Libby Curie Sk/odowska Joliot Curie
NP 1908 NP 1901 NP 1903 NP 1960 NP 1903 Curie NP 1935 NP 1935
1871-1937 1845-1923 1852- 1908 1908-1980 1859-1906 NP 1903, 1911 1900-1958 1897- 1956
1867- 1934
Chapter 50: Applications of Isotopes in Medicine 681

TABLE 50.1: Different forms of radiation


Type ofradiation Composed of Mass Charge Stopped by Application
Alpha 2p + 2n 4 +2 Few sheets of pa per Radiation hazard
Beta e- Negligible -1 Few sheet s aluminum Research/diagnosis
Gamma Electromagnetic waves Nil 0 Few cm thick lead Diagnosis/ treatment
t

2
Isobars are atoms having same mass number, but : Radium - - 2~ Radon+ ~He (a particle)
are having different atomic numbers, e.g. 14C and 14N.
The nucleus of Radium. being unstable, emits 2 protons
and 2 neutrons (one helium nucleus) to become Radon-
Atomic Number and Atomic Weight
222. The alpha particles will carry 2 positive charges and
The number of protons (or electrons) in an atom will produce maximum ionization in their path. Thus, they
determine the mass number or its place in the periodic are most damaging to tissues. Alpha particles emitted
table. The presence of neutrons will add on the mass of have a high mass and therefore a high momentum. They
the atom. The atomic weight or mass number is equal
do not travel far and can be stopped by a few layers of
to the number of protons plus neutrons in the atom.
paper. However, they collide with other molecules and

I RADIOACTIVITY - - cause a lot of damage, hence considered to be most


hazardous. The alpha radiations are not useful in clinical
Isotopes may be stable or unstable (radioactive), and medicine. In fact, radium needles are covered so that
the latter may be naturally occurring or artificially made. alpha particles are absorbed, before applying to tissues.
In the above example, Deuterium is stable, which will not If not, the radiation in the vicinity of the needle will be
alter its nuclear composition during passage of time. But hazardously high (Table 50.1 ).
tritium is unstable and will transform by nuclear decay.
The spontaneous degradation of nucleus and trans- Beta Radiation
mutation of one element to another with consequent
When a neutron is split, one proton, one electron (beta
emission of rays or particles is known as radioactivity.
particle) and one neutrino are generated. The element
Chemical reactions are based on the activity of elec-
is changed to one having a higher number in periodic
trons, while radioactivity is due to subnuclear compo-
table.
nents. Elements capable of undergoing radioactive
1
decay are called radionuclides. ; N + e- + neutrino
Antoine Becqueral was the pioneer in demonstrat- One neutron from carbon is changed to a proton.
ing spontaneous radioactivity (Nobel Prize, 1903). Marie Therefore, mass remains the same, but the element is
Curie and Pierre Curie were awarded Nobel Prize in changed with one number more in atomic number. The
1903 in physics, for their study on radiation phenomena. electrons thus emitted become the beta rays. So they
Marie Curie was awarded the Nobel Prize again in 1911 , are negatively charged. Since their mass is negligible,
but this time in chemistry, for the isolation of radium and they can penetrate more distance. But they can be
polonium. In 1935, Nobel Prize was awarded to Frederic absorbed by metal sheets.
Joliot (son-in-law of Madam Curie) and Irene Joliot
(daughter of Curie) for artificial production of radioactive Gamma Radiation
phosphorus from aluminium.
While alpha and beta radiations are particles, gamma
Radioactive Decay radiation is in the form of electromagnetic waves.
Gamma ray has no mass and no charge, and therefore
Alpha Decay
penetration power is maximum. It is now widely used for
When the alpha particle (2p + 2n) is released , the ele- treatment of cancer cases. X-rays and gamma rays are
ment changes, the atomic number is reduced by 2 and similar electromagnetic waves (Table 50.1 ). The gamma
mass number is lowered by 4. For example, radiation is produced by:
682 Section F: Advanced Biochemistry

Percentage of
original radioactivity

8 16 24 days

Fig. 50.1: Radioactive decay and half-life of 131 1

TABLE 50.2: Commonly used radioisotopes


Element Isotope Approximate half-life Major radiation Important applications
Carbon "C 5600 years Beta Research in metabolism, carbon dating
Hydrogen 'H 12 years Beta Research in cell biology
Phosphorus llp 14 days Beta Nucleic acid research, treatment for poly,:ythemia
Iodine '''I 8 days Gamma Treating hyperthyroidism and thyroid cancer
Cesium "'Cs 30 years Gamma Teletherapy for cancer

1
'~ Iodine ;.!Xenon(metastable) Roentgen (R)
H3) (-v)
1
t Xenon It is the measurement of exposure dose. 1 R is the radia-
A comparison of d ifferent forms of radiation is given tion which will give rise to 2 x 109 ion pairs/cc of air.
in Table 50.1. One mCi source kept at a distance of 1 cm will produce
12.9 R/hour.
Half-life of Radioactivity
The radioactivity is halved within a fixed time. For exam-
Rad and Gray
ple, if 100 mCi of 131
1 is kept , after 8 days the activity is Rad is the absorbed dose by tissue. 1 Rad = 1.5 x 10 12 ion
seen to be 50 mCi. The half-life of 1311is 8 days (Fig. 50.1). pairs/g tissue. One Gray (Gy) = 1()7 ergs/ kg tissue
The half-life is the time taken for a radioactive = 100 rads.
isotope to become half of its original activity. The
rate of decay or the half-life is a constant for a particular Applications of Radioactivity
isotope. Commonly used isotopes are listed in Table 50.2. in Research
Isotopes of an element will have identical chemical reac-
UNITS OF RADIOACTIVITY tions. Hence, when a radiolabeled compound is admin-
Curie (Ci) istered, these molecules are metabolized by the body
similar to normal molecules. This is cailled Tracer tech-
One Curie, abbreviated as Ci, is equivalent to 3. 7 x 1010 nique. Almost all biochemical research will utilize such
disintegrations per second (dps) or 37 giga becquerals tracer methods. A few examples are given below.
(gBq). This unit is used to measure the radioactivity of i. Almost all the pathways described in earlier chap-
the source. ters were studied by using traceirs. For example,
14
C-labeled acetoacetic acid is st1own to be incor-
Becqueral (Bq)
porated into palmitic acid. Suppose labeled "A" is
Becqueral (Bq) is defined as decay per second, (dps). administered to an animal. After a few minutes, liver
1 Bq = 1 dps. It is often expressed as kilobecquerals (kBq). contains labeled "B" and after one, hour, labeled "C"
Chapter 50: Applications of Isotopes in Medicine 683

is seen in liver. Thus, we can say that the pathway emission of positrons and their combination with
is A to B to C. an electron resulting in the simultaneous emission
ii. 32 P is useful to trace the nucleic acid synthesis in of two gamma rays is detected by a PET camera.
t
vivo and in vitro. It is therefore employed in genetic Combination of PET and CT (PET/CT) improves
research. 3 H-labeled thymidine is incorporated the diagnostic accuracy. The abnormality may be
in the newly synthesized DNA and therefore used either lesser uptake of the isotope by the organ
in assessing cell division kinetics. (cold spot) or more uptake (hot spot). A series of
iii. The total body content of a particular substance images taken over a period of time will give specific
(also designated as the "pool" of the substance) patterns that indicate normal or malfunction of the
can be quantitated by the isotope dilution tech- organ concerned. An advantage over X-ray imaging
nique. To cite an example, 1 ml of 131 1-labeled is that both bone and soft tissues can be studied.
albumin is seen to have 1 million dps. This is injected
intravenously to a man. The radioactivity will be
Applications of
uniformly mixed in the total blood volume. After Radioactivity in Treatment
10-15 minutes, a blood sample is withdrawn. One ml Radioactivity is used for treatment of cancer. Radiations
of blood is shown to have a radioactivity with 200 dps. when absorbed by the tissues, produce ionization in the
The volume injected and removed is the same 1 ml, path. Nucleic acids in the cell are damaged, so that next
but the original count is now diluted 5,000 times. cell division is not possible. Radiotherapy mainly affects
Thus, the intravascular space is 5,000 times more cells in the divis.ion phase. Since cancer tissue contains
than the volume injected. Therefore, the blood more dividing ciells than normal tissue, cancer cells are
volume is 5,000 ml. preferentially affected by radiation. Radiotherapy may
iv. Carbon dating technique is an important tool in be classified as:
paleobiology, the techn ique was developed by Wil-
lard Libby who was awarded Nobel Prize in 1960. Unsealed Sources
These are radioactive substances kept in liquid form.
Applications of The beta rays are the main effective radiation in these
Radioactivity for Diagnosis sources. For thyroid cancer and secondaries of thy-
The branch of medicine that deals with the diagnostic roid cancer, 131 1 (dose 50-100 mCi) is administered.
applications of radioactivity is referred to as Nuclear Similarly, 32P is used to treat polycythemia vera.
Medicine. A quick and accurate diagnosis can be made
by radioimaging of organs like thyroid, liver bone, etc. Sealed Sources
Diagnostic uses of radioactive tracers are using gam- They utilize gamma irradiation. The source is applied on
ma ray emitters. These are short lived isotopes linked the cancer or sometimes implanted as a needle into the
to chemical compounds that permit the observation of tissue.
specific physiological processes. They may be given by Cs (cesium) with a half-life of 30 years, is the pre-
injection, inhalation or orally. ferred sealed source nowadays. Application of such
i. Thyroid uptake studies by 131 1 are used to detect sources directly on cancer tissue is called Brachy-
functional derangements of thyroid gland. About 15 therapy. lntracavitary applications (for cancer body of
mCi of 131 1 is given intravenously. After a few hours,
.. the patient is monitored at the neck region by a
uterus, cancer of cervix uteri, cancer of vagina) and
interstitial applications (buccal cancer, tongue cancer)
movable gamma-ray counter, which will pick up the
are common.
radiation emitted by the thyroid gland. The normal
values are about 25% uptake by thyroid within
Teletherap~,
2 hours and about 50% uptake within 24 hrs. In
hyperthyroidism there will be increased uptake and The term "tele" means distant (as in the case of tele-
hypothyroidism shows the reverse effect. scope, telephone, etc.). Here the source of radiation is
ii. Positron emission tomography (PET) scan is kept at a distance from the patient. Historically, telethe-
a more precise and sophisticated technique. The rapy started with deep X-ray. X-ray was discovered by
684 Section F: Advanced Biochemistry
Wilhelm Rontgen in 1895 (Nobel Prize, 1901 ). Due to There will be atrophy of skin , hypopigmentation, fibros is,
its inefficiency, deep X-ray is no more used for cancer loss of elasticity, etc.
treatment.
Linear accelerator (LINAC): Here electrons are Effects on Mucous Membran1e
accelerated to higher energy levels of 8-12 MV and The gastrointestinal mucosa is very sensitive to radi-
directed into the cancer tissue. It has more penetrating ation. These include nausea, vomitin!J, diarrhea and in
power and accurate beam focussing capabilities. As there severe cases ulceration and bleeding. Late sequelae
is no permanent radioactive source in the machine, the such as adhesions, fibrosis, stenosis and obstruction
radiation hazards are minimal. LINAC is used for exter- may appear many months after radiotherapy.
nal beam radiation treatments for patients with cancer.
Effects on Blood Cells
Radiosens itivity Bone marrow and lymphoid tissues are highly radio-
The effectiveness of radiotherapy varies with different sensitive because of the higher rate of cell division in
tumors. In general, lymphomas, Hodgkin's disease and these organs. Leukopenia and thrombocytopenia is an
neuroblastoma are highly radiosensitive. Epithelioma, accepted side effect of radiotherapy. If WBC count is
cancer of oral cavity, cancer cervix, cancer breast and below 2,000/cu mm and platelet count is below 80,000/cu
cancer lung are moderately radiosensitive. Poorly radio- mm, the therapy is temporarily stopped till recovery is
sensitive tumors are osteosarcoma, and malignant mel- effected .
anoma.
Effects on Reproductive Organs
I BIOLOGICAL EFFECTS OF Gonads (ovary and testis) are highly radiosensitive.
I..BAQJATION Complete sterility is effected at 1000 rads. Even low
doses of radiation, too low to have any obvious effect
Direct Effects on Cancer Tissues on mitosis, can still affect the genes, so as to produce
The radiation damages DNA molecules. No effects are genetic alterations in the offspring. This is especially
visible immediately. But the damage is observed during important when radiation is given in pelvic region.
the next mitosis. Since new DNA cannot be synthesized,
cells die at the attempt of the next division. Chromosome Carcinogenic Potential
breakage is often noticed. Radiation produces large
During the period 1900-1910, people were working with
quantities of free radicals in tissues. The catastrophic
X-rays without any precautions. This caused non-heal-
effects of free radicals on different biological compounds
ing ulcers in many of them. During 1910s and 20s, lip
(including DNA) are described in Chapter 30.
cancer was common among painters of watch dial with
Effects of Radiation on Normal Tissues radioactive stain. Gradually, along w ith the increasing
knowledge on radiation hazard, strin£1ent safeguard for
In 1904, Madam Curie went for a lecture-demonstration
radiation protection was introduced.
class, keeping a few mg of impure radium ore in her
breast pocket. Within 1 hour, this caused severe der-
matitis. That was the first indication of a health hazard
Radiation Protection
by radioactivity. Madam Curie succumbed to radiation- There is always some amount of background radiation,
induced leukemia in 1934. of about 150 m Rem/year. Out of this, ,about 50% is from
the cosmic rays, about 30% from terrestrial environ-
Effects on Skin ment and 20% from internal environment (e.g. decay
Radiation will produce epilation, however hair may grow of 4°K). Granite and brick walls will increase external
after 3 months. Sweat glands may be permanently dama- background. At higher elevation, cosmic rays are more.
ged. There may be erythema and sometimes blisters. At an altitude of 2000 m (e.g. Gangtok, Sikkim state),
This is called acute radiodermatitis. Chronic radio- the background irradiation is 20% more. In some coastal
dermatitis is seen after a few months of radiotherapy. areas (e.g. Kerala state) natural deposits of radioactive
Chapter 50: Applications of lsoiopes in Medicine 685

thorium is seen, where background is 20-30% high. 2. Spontaneous degradation of nucleus and trans-
One diagnostic X-ray exposure may cause 75 milliRem. mutation of one element to another with conse-
quent emission of rays or particles is known as
f
Maximum Permissible Dose
radioactivity.
The MPD of radiation for whole body among radiation
3. The total body content of a particular substance can
workers (doctors, technicians) is 5 mRem/year, and for
be quantitated by the isotope dilution technique.
general population is 0.5 mRem/year.
4. Applications of radioisotopes in medicine include
LEARNING POINT~ CHAPTER 50 thyroid uptake studies, radiotherapy in cancers.
5. Radiations affect mucous membrane, skin, blood
1. Isotopes are the same element having the same cells, reproductive organs and ,can damage DNA
atomic number (protons) but different mass number
preventing mitosis.
(varying number of neutrons).

PART-1: ESSAY AND SHORT NOTE QUESTIONS


50-1 . Give two examples of isotopes and Its applications in medicine.
50-2. What are the medical uses of Isotopes?

SHORT NOTE QUESTIONS


50-3. Applications of isotopes for diagnosis.
50-4. Biological effects of radiation on tissues.

,_ PART-2: MULTIPLE CHOICE QUESTIONS

50-1. Molecules having same atomic number but differ- C. 1 neutron


ent mass numbers are known as? D. 1 electron+ 1 proton
A. Isobars B. Isotopes 50-3. What is the radioisotope used for measuring nu-
C. Radiation D. Electroactive elements cleic acid synthesis?
50-2. What is Alpha particle? A. s•cr B. 131 1
A. 2 protons + 2 neutrons C. 32p D. " C
B. 1 electron

ANSWERS OF MULTIPLE CHOICE QUESTIONS


50-1 . B 50-2. A 50-3. C

. '
PART-3: VIVA VOCE QUESTIONS AND ANSWERS

50-1. What are the major types of radioactivity? 50-3. Which isotope is used for thyroid scanning?
Alpha particle, along with beta and gamma radiation . 131 1

50-2. Wh ich isotope is used for carbon dating? 50-4. What is meant by half-life of an isotope?
,.C The half-life is the time taken for a radioactive isotope
to become half of its original activiity.
;

You might also like